You are on page 1of 785

FIRST AID ™

Q&A FOR THE
USMLE
STEP 1
Third Edition
SENIOR EDITORS
Tao Le, MD, MHS James A. Feinstein, MD
Associate Clinical Professor Clinical Instructor, Section of General Pediatrics
Chief, Section of Allergy and Clinical Immunology The Children's Hospital Colorado
Department of Medicine Research Fellow
University of Louisville Primary Care Research Program
University of Colorado School of Medicine

EDITORS
Mark W. Ball, MD Kimberly Kallianos
Resident Harvard Medical School
The James Buchanan Brady Urological Institute Class of 2012
The Johns Hopkins Hospital
Cesar Raudel Padilla
Annie Dude, MD University of Rochester School of Medicine
Resident Class of 2012
Department of Obstetrics and Gynecology
Duke University Medical Center Lauren Rothkopf, MD
Masters in Public Health candidate
Rebecca L. Hoffman, MD Temple University College of Health Professions and
Resident Social Work
Department of General Surgery
Hospital of the University of Pennsylvania James Yeh, MD
Resident
Mark Robert Jensen Department of Medicine
University of Rochester School of Medicine Cambridge Hospital, Cambridge Health Alliance
Class of 2012

New York / Chicago / San Francisco / Lisbon / London / Madrid / Mexico City
Milan / New Delhi / San Juan / Seoul / Singapore / Sydney / Toronto
Copyright © 2012 by Tao Le. All rights reserved. Except as permitted under the United States Copyright Act of 1976, no part of this publication may be reproduced
or distributed in any form or by any means, or stored in a database or retrieval system, without the prior written permission of the publisher.

ISBN: 978-0-07-174596-3

MHID: 0-07-174596-3

The material in this eBook also appears in the print version of this title: ISBN: 978-0-07-174402-7,
MHID: 0-07-174402-9.

All trademarks are trademarks of their respective owners. Rather than put a trademark symbol after every occurrence of a trademarked name, we use names in an
editorial fashion only, and to the benefit of the trademark owner, with no intention of infringement of the trademark. Where such designations appear in this book,
they have been printed with initial caps.

McGraw-Hill eBooks are available at special quantity discounts to use as premiums and sales promotions, or for use in corporate training programs. To contact a
representative please e-mail us at bulksales@mcgraw-hill.com.

Notice

Medicine is an ever-changing science. As new research and clinical experience broaden our knowledge, changes in treatment and drug therapy are required. The
authors and the publisher of this work have checked with sources believed to be reliable in their efforts to provide information that is complete and generally in
accord with the standards accepted at the time of publication. However, in view of the possibility of human error or changes in medical sciences, neither the authors
nor the publisher nor any other party who has been involved in the preparation or publication of this work warrants that the information contained herein is in every
respect accurate or complete, and they disclaim all responsibility for any errors or omissions or for the results obtained from use of the information contained in
this work. Readers are encouraged to confirm the information contained herein with other sources. For example and in particular, readers are advised to check the
product information sheet included in the package of each drug they plan to administer to be certain that the information contained in this work is accurate and that
changes have not been made in the recommended dose or in the contraindications for administration. This recommendation is of particular
importance in connection with new or infrequently used drugs.

TERMS OF USE

This is a copyrighted work and The McGraw-Hill Companies, Inc. (“McGrawHill”) and its licensors reserve all rights in and to the work. Use of this work is subject
to these terms. Except as permitted under the Copyright Act of 1976 and the right to store and retrieve one copy of the work, you may not decompile, disassemble,
reverse engineer, reproduce, modify, create derivative works based upon, transmit, distribute, disseminate, sell, publish or sublicense the work or any part of it
without McGraw-Hill’s prior consent. You may use the work for your own noncommercial and personal use; any other use of the work is strictly prohibited. Your
right to use the work may be terminated if you fail to comply with these terms.

THE WORK IS PROVIDED “AS IS.” McGRAW-HILL AND ITS LICENSORS MAKE NO GUARANTEES OR WARRANTIES AS TO THE ACCURACY,
ADEQUACY OR COMPLETENESS OF OR RESULTS TO BE OBTAINED FROM USING THE WORK, INCLUDING ANY INFORMATION THAT CAN
BE ACCESSED THROUGH THE WORK VIA HYPERLINK OR OTHERWISE, AND EXPRESSLY DISCLAIM ANY WARRANTY, EXPRESS OR IMPLIED,
INCLUDING BUT NOT LIMITED TO IMPLIED WARRANTIES OF MERCHANTABILITY OR FITNESS FOR A PARTICULAR PURPOSE. McGraw-Hill
and its licensors do not warrant or guarantee that the functions contained in the work will meet your requirements or that its operation will be uninterrupted or
error free. Neither McGraw-Hill nor its licensors shall be liable to you or anyone else for any inaccuracy, error or omission, regardless of cause, in the work or for
any damages resulting therefrom. McGraw-Hill has no responsibility for the content of any information accessed through the work. Under no circumstances shall
McGraw-Hill and/or its licensors be liable for any indirect, incidental, special, punitive, consequential or similar damages that result from the use of or inability
to use the work, even if any of them has been advised of the possibility of such damages. This limitation of liability shall apply to any claim or cause whatsoever
whether such claim or cause arises in contract, tort or otherwise.
P O W E R E D BY usmte

FIRST AID
STEP 1 EXPRESS

FIRST AID

RESPIRATORY
PHYSIOLOGY
TAO LE, MD, MHS The official FIRST AID course
Assistant Clinical Professor of Medicine and Pediatrics
Chief, Section of Allergy and Immunology
Department of Medicine
by the FIRST AID authors
University of Louisville

50+ hours of high-yield online


lectures based on FIRST AID
600+ new color images & multimedia
expand key FIRST AID concepts
Exclusive FIRST AID color PDF
workbook reinforces your test prep
Watch as many times as you want
100% pass guarantee!
See website for Terms and Conditions.

www.usmlerx.com
D e d i c at i o n

To the contributors to this and future editions, who took time to share their
knowledge, insight, and humor for the benefit of students, residents, and
clinicians.

and

To our families, friends, and loved ones, who supported us in the task of
assembling this guide.
This page intentionally left blank
Contents

Authors vii

Preface ix

Acknowledgments xi

How to Contribute xiii

S EC T I O N I G e n e r al P r i n c i p le s 1

Chapter 1 Behavioral Science 3

Chapter 2 Biochemistry 17

Chapter 3 Embryology 53

Chapter 4 Microbiology 71

Chapter 5 Immunology 101

Chapter 6 Pathology 123

Chapter 7 Pharmacology 141

S EC T I O N I I O r g a n Syst e m s 15 9

Chapter 8 Cardiovascular 161

Chapter 9 Endocrine 197

Chapter 10 Gastrointestinal 233

Chapter 11 Hematology-Oncology 271

Chapter 12 Musculoskeletal 307

Chapter 13 Neurology 339

Chapter 14 Psychiatry 363

Chapter 15 Renal 379

Chapter 16 Reproductive 415

Chapter 17 Respiratory 451

v
S EC T I O N I I I F u ll-Le ng t h E x am i nat i o n s 487

Test Block 1 489

Test Block 2 527

Test Block 3 563

Test Block 4 599

Test Block 5 633

Test Block 6 669

Test Block 7 705

Appendix: Common Laboratory Values 741

Index 743

About the Authors 767

vi
AUTHORS

Kirsten Austad Katherine Latimer


Fellow Johns Hopkins University School of Medicine
Edmond J. Safra Center for Ethics Class of 2012
Harvard University
Joseph Liao
Eike Blohm Boston University School of Medicine
Johns Hopkins University School of Medicine Class of 2012
Class of 2012
Jerry Loo
Benjamin Caplan, MD University of Southern California Keck School of Medicine
Resident Class of 2012
Department of Family Medicine
Boston University Aya Michaels, MD
Resident
Po-Hao Chen Department of Radiology
Harvard Medical School Brigham and Women’s Hospital
Class of 2012
Somala Muhammed, MD
Lauren de Leon, MD Resident
Intern Department of General Surgery
Department of Internal Medicine
Baylor College of Medicine
The Alpert Medical School of Brown University
Behrouz Namdari, MD
Philip Eye Resident
Boston University School of Medicine
Department of Psychiatry
Class of 2012
Duke University Medical Center
Jim Griffin, MD
Resident
Tashera Perry, MD
Resident
Department of Surgery and Surgical Oncology
Department of Obstetrics and Gynecology
Johns Hopkins Hospital
The University of Illinois at Chicago School of Medicine
John Hegde
Harvard Medical School Christopher Roxbury
Class of 2012 Johns Hopkins University School of Medicine
Class of 2012
Emily Heikamp
Johns Hopkins University School of Medicine Neepa Shah
Class of 2014 Boston University
Class of 2012
Thomas Robert Hickey, MD
Resident Bethany Strong
Department of Anesthesiology Harvard Medical School
Brigham and Women’s Hospital Class of 2012

Henry R. Kramer, MD Seenu Susarla, MD, DMD, MPH


Resident Resident
Department of Medicine Department of Oral & Maxillofacial Surgery
Massachusetts General Hospital Massachusetts General Hospital

Thomas Lardaro Jeffrey Tosoian


Johns Hopkins University School of Medicine Johns Hopkins University School of Medicine
Class of 2012 Class of 2012

vii
Jackson Vane, MD Marc E. Walker
Resident Harvard Medical School
Department of Pediatrics Class of 2012
University of California, Irvine School of Medicine

Daniel J. Verdini, MD
Resident
Department of Internal Medicine
University of Nevada School of Medicine at Reno

viiiviii
Preface

With the third edition of First Aid Q&A for the USMLE Step 1, we continue
our commitment to providing students with the most useful and up-
to-date preparation guides for the USMLE Step 1. This new edition represents
an outstanding effort by a talented group of authors and includes the following:

  Almost 1000 high-yield USMLE-style questions based on the top-rated


USMLERx Qmax Step 1 Test Bank (www.usmlerx.com)
  Concise yet complete explanations to correct and incorrect answers
  Questions organized by general principles and organ systems
  Seven full-length test blocks simulate the actual exam experience
  High-yield images, diagrams, and tables complement the questions and
answers
  Organized as a perfect complement to First Aid for the USMLE Step 1

We invite you to share your thoughts and ideas to help us improve First Aid
Q&A for the USMLE Step 1. See How to Contribute, p. xiii.

Louisville Tao Le
Denver James A. Feinstein

ixix
ix
This page intentionally left blank
Acknowledgments

This has been a collaborative project from the start. We gratefully acknowl-
edge the thoughtful comments and advice of the medical students, interna-
tional medical graduates, and faculty who have supported the authors in the
continuing development of First Aid Q&A for the USMLE Step 1.

For support and encouragement throughout the project, we are grateful to


Thao Pham, Louise Petersen, Selina Franklin, Jonathan Kirsch, and Vikas
Bhushan. Thanks to our publisher, McGraw-Hill, for the valuable assistance
of their staff. For enthusiasm, support, and commitment to this challenging
project, thanks to our editor, Catherine Johnson. For outstanding editorial
work, we thank Mary Dispenza and Emma D. Underdown. A special thanks
to Rainbow Graphics for remarkable production work.

Louisville Tao Le
Denver James A. Feinstein

xi
This page intentionally left blank
How to Contribute

This edition of First Aid Q&A for the USMLE Step 1 incorporates hundreds of contributions and
changes suggested by faculty and student reviewers. We invite you to participate in this process. We also
offer paid internships in medical education and publishing ranging from three months to one year (see
next page for details). Please send us your suggestions for:

  Corrections or enhancements to existing questions and explanations


  New high-yield questions
  Low-yield questions to remove

For each entry incorporated into the next edition, you will receive a $10 gift certificate, as well as per-
sonal acknowledgment in the next edition. Diagrams, tables, partial entries, updates, corrections, and
study hints are also appreciated, and significant contributions will be compensated at the discretion of
the authors.

The preferred way to submit entries, suggestions, or corrections is via our blog:
www.firstaidteam.com

Alternatively, you can email us at: firstaidteam@yahoo.com. All entries become property of the authors
and are subject to editing and reviewing. Please verify all data and spellings carefully. In the event that
similar or duplicate entries are received, only the first entry received will be used. Include a reference to
a standard textbook to facilitate verification of the fact. Please follow the style, punctuation, and format of
this edition if possible.

I N TER N SHI P O P P ORTU N ITIES

The First Aid Team is pleased to offer part-time and full-time paid internships in medical education and
publishing to motivated medical students and physicians. Internships may range from three months (e.g.,
a summer) up to a full year. Participants will have an opportunity to author, edit, and earn academic
credit on a wide variety of projects, including the popular First Aid and USMLERx series. Writing/
editing experience, familiarity with Microsoft Word, and Internet access are desired. For more informa-
tion, e-mail a résumé or a short description of your experience along with a cover letter to firstaidteam@
yahoo.com.

xiii
This page intentionally left blank
Section I

General Principles

 Behavioral Science

 Biochemistry

 Embryology

 Microbiology

 Immunology

 Pathology

 Pharmacology

1
This page intentionally left blank
Chapter 1

Behavioral Science

3
High-Yield Principles
4 Section I: General Principles  •  Questions

Q u e st i o n s

1. Researchers investigating the development comes nervous before a test, he feels as if he


of the idiopathic inflammatory myopathies cannot move his legs. He admits that he has
(IIMs) such as polymyositis read that vitamin even fallen to the floor because of leg weak-
D may act as an immunomodulator that re- ness while laughing. Which of the following is
duces the development and severity of autoim- the best choice for treating this patient?
mune diseases. Given that many Americans
(A) Chloral hydrate
are vitamin D deficient, the researchers design
(B) Hydroxyzine
an observational study to assess the impact of
(C) Modafinil
vitamin D supplementation on IIM symptom
(D) Prochlorperazine maleate
severity. Subjects are surveyed at time 0 and af-
(E) Zolpidem
Behavioral Science

ter two years, and the results are listed in the


chart. Which equation represents the chance 3. A 52-year-old woman is being treated by a
of symptom improvement in subjects who took male psychiatrist for depression stemming
vitamin D supplements relative to subjects from her recent divorce. Recently, the patient
who did not take vitamin D supplements? has been coming to her appointments dressed
up and wearing expensive perfumes. She has
also started to flirt with the doctor. The pa-
Vitamin D Vitamin D
supplement supplement tient’s demeanor and appearance had initially
taken not taken reminded the psychiatrist of his aunt. He is
Symptoms improved 50 60 uncomfortable with the patient’s new behav-
ior patterns and tells her so. She becomes very
Symptoms not
300 400 angry and storms out of the office, canceling
improved
all remaining appointments on her way out.
Reproduced, with permission, from USMLERx.com.
Which of the following behaviors is an exam-
ple of negative transference?
(A) The doctor seeing the patient as his aunt
(A) (50 / 300) / (60 / 400) (B) The doctor telling the patient he is un-
(B) (50 / 350) / (60 / 460) comfortable
(C) (50 / 460) / (60 / 300) (C) The patient being angry with the doctor
(D) (60 / 400) / (50 / 300) (D) The patient dressing up for appointments
(E) (60 / 460) / (50 / 350) (E) The patient flirting with the doctor
2. A 16-year-old boy is brought to the pediatri- 4. A 28-year-old woman presents to her primary
cian by his mother because of excessive day- care physician because of depressed mood.
time sleepiness. She states that over the past She states that she has been depressed for as
six months she has received numerous phone long as she can remember and feels bad about
calls from the boy’s school informing her herself almost all of the time. She states that
that her son sleeps throughout all of his after- her only happy moments were during her hon-
noon classes and is often difficult to arouse at eymoon two years ago, and during a ski trip in
the end of class. The patient reports that oc- college when she felt “on top of the world.”
casionally when he wakes up in the morning She confides that for a couple weeks last
he cannot move for extended periods. He says month she felt life was no longer worth living.
that sometimes when he laughs at jokes or be- At that time, she was having extreme difficulty
High-Yield Principles
Chapter 1: Behavioral Science  •  Questions 5

sleeping, a complete loss of energy, and a lack 7. The figure below is a common representation
of appetite. A review of the patient’s history used in studying the characteristics of a test’s
shows that during the past two years she has results. Using the letters in the figure, which
seen a physician for complaints of stomach up- of the following accurately describes the preva-
set, fatigue, headaches, and an unintentional lence of the disease?
3.6-kg (8-lb) weight gain. Physical examina-
tion and results of laboratory tests are within
normal limits. Which of the following is an ad- Disease
verse effect the patient may experience during + –
the course of the treatment of this illness?
(A) Agranulocytosis + W X
(B) Anorgasmia

Test

Behavioral Science
(C) Arrhythmia
(D) Polyuria
(E) Stevens-Johnson syndrome – Y Z

5. A 6-year-old girl is brought to the pediatrician


by her mother because of fecal incontinence. Reproduced, with permission, from USMLERx.com.
The mother says this behavior usually occurs
at school. According to Freud, which stage of
psychosexual development has this child failed (A) (W+X) / (W+X+Y+Z)
to progress through? (B) (W+Y) / (W+X+Y+Z)
(A) Anal stage (C) W / (W+X+Y+Z)
(B) Genital stage (D) W / (X+Y+Z)
(C) Latency stage (E) W / (X+Z)
(D) Oral stage
8. A 75-year-old man is recovering in the hospi-
(E) Phallic stage
tal from a left-sided below-the-knee amputa-
6. A 20-year-old man became very agitated at a tion. Three days after the surgery, the patient
party, and as a result was brought to the emer- suddenly develops chest pain and shortness of
gency department. In the waiting room he is breath that last for 20 minutes. His pain medi-
belligerent and uncooperative. A physical ex- cation is increased, which improves the pain
amination reveals fever, tachycardia, horizon- but not the shortness of breath. X-ray of the
tal nystagmus, hyperacusis, and pupils that are chest is negative for a pulmonary embolus, so
3 mm in diameter bilaterally. Which of the the medical team decides to monitor him ex-
following substances is most likely causing the pectantly. The next day, a similar episode of
behavioral changes and physical findings ex- shortness of breath and chest pain occurs. The
hibited by this patient? patient then sustains cardiac arrest and dies.
Autopsy reveals multiple pulmonary emboli.
(A) Alcohol The family threatens to sue for malpractice for
(B) Amphetamines mismanaged postoperative care. Which of the
(C) Cocaine following is necessary to prove malpractice?
(D) Lysergic acid diethylamide
(E) Nicotine (A) A patient directly suffers harm
(F) Phencyclidine (B) A physician’s presence at the time of injury
(C) Intent to harm
(D) Proof beyond reasonable doubt
(E) Use of standard procedures
High-Yield Principles
6 Section I: General Principles  •  Questions

9. A 2-month-old boy is brought to the emer- (A) 8%


gency department with respiratory insuffi- (B) 20%
ciency and failure to thrive. The pregnancy (C) 80%
and perinatal course were uneventful. Gener- (D) 85%
alized hypotonia, tongue fasciculations, and (E) 93%
flaccid paralysis are noted on physical exami-
nation. His hospital stay is complicated by the 11. A 66-year-old man presents to his primary care
development of tracheobronchomalacia and physician with a complaint of erectile dysfunc-
respiratory insufficiency that necessitates me- tion. His past history is significant for hyper-
chanical ventilation. Despite these efforts, tension, type 2 diabetes mellitus, peripheral
the patient dies of respiratory complications. vascular disease, and coronary artery disease
Muscle biopsy shows denervation and panfas- status postmyocardial infarction. His current
cicular atrophy. A genetics consult yields the medications are propranolol, captopril, aspirin,
Behavioral Science

pedigree shown in the image. Which of the lovastatin, metformin, fluoxetine, and sublin-
following diseases is most consistent with this gual nitroglycerin. On further questioning, he
patient’s presentation and the pedigree shown admits to wanting a prescription for sildenafil.
in the image? Which of his medications is unsafe to take
with sildenafil?
(A) Aspirin
(B) Captopril
(C) Fluoxetine
(D) Lovastatin
(E) Metformin
(F) Nitroglycerin
(G) Propranolol

12. A 17-year-old girl presents to her primary care


Reproduced, with permission, from USMLERx.com. physician with a complaint of missed menses.
A urine pregnancy test confirms that she is
pregnant. She returns to the office two weeks
(A) Becker muscular dystrophy later asking for recommendations on obtaining
(B) Duchenne muscular dystrophy an abortion. She explains that she works, lives
(C) Kugelberg-Welander disease with her husband, and is not ready for a child.
(D) Spinal muscular atrophy, type II She decides that she does not want to notify
(E) Werdnig-Hoffmann disease anyone, and says she has chosen not to talk
with her parents for many months. Her doctor
10. A new screening test for the development of understands that he must abide by her wishes
mitral regurgitation in the setting of rheumatic because she is emancipated. Which of the fol-
fever is created. A study of 1000 patients with a lowing makes this patient emancipated?
history of Streptococcus pyogenes infection and
(A) Age 17 years is considered an adult
a diagnosis of rheumatic fever is performed
(B) Full-time work
using this test, which has 90% sensitivity and
(C) High school diploma
85% specificity. The prevalence of mitral re-
(D) Living separately from her parents
gurgitation in this population is estimated to
(E) Marriage
be 40%. What is the positive predictive value
of this test?
High-Yield Principles
Chapter 1: Behavioral Science  •  Questions 7

13. A group of scientists decides to conduct a study (D) Projection of the areola and papilla (with
addressing the long-term effects of maternal separate contours), and adult-type pubic
alcohol consumption on their infants after hair limited to the genital area
conception. Two hundred women, including (E) Small breast buds with elevation of breast
those who suffer from alcoholism and those papilla, and sparse, straight, downy hair on
who do not, are recruited into the study when the labial base
they present for their first primary care visit. A
medical history is taken on alcohol use, prena- 15. A 10-year-old Hispanic boy is admitted for
tal care, nutritional status, and smoking behav- bone marrow transplantation as treatment
iors; these are measured monthly during the for acute myelogenous leukemia. The doctor
pregnancy. The researchers follow the wom- wants to enroll the patient in a clinical trial
en’s pregnancies until term, after which they for a new pain medication, but both the par-
devote their attention to the health and behav- ents speak only Spanish. The consent form

Behavioral Science
iors of the offspring. Which of the following is in English, and the physician has a limited
is the most appropriate statistic the research- knowledge of Spanish. What is the physician’s
ers will be able to calculate as a result of their best option for obtaining consent from this pa-
study? tient?
(A) Attributable risk of offspring abnormalities (A) Explain the study to the whole family in
in mothers who smoke Spanish, to the best of the physician’s abil-
(B) Odds ratio of offspring abnormalities in ity
mothers who consume alcohol during (B) Have a Spanish-speaking employee of the
pregnancy hospital translate for the patient
(C) Prevalence of alcohol consumption during (C) Have the parents sign the English form af-
pregnancy ter discussing the study via an interpreter
(D) Proportion of all offspring abnormalities (D) Obtain a translated consent form and dis-
that are due to alcohol consumption dur- cuss the study via an interpreter
ing pregnancy (E) The boy speaks English, so the parents’
(E) Relative risk of offspring abnormalities in consent will not be required
mothers who consume alcohol during
pregnancy 16. A 70-year-old man comes into his doctor’s of-
fice for a routine check-up. His past medi-
14. An 11-year-old girl is brought to the pediatri- cal history is significant for a heart attack,
cian with complaints of back pain. On physical for which he takes a daily baby aspirin and a
examination, a right thoracic scoliotic curve is b-blocker. He practices safe habits and always
noted. The pediatrician mentions to the parent wears his seat belt while driving. His health
that the development of adolescent idiopathic has been “great” for the past few years, al-
scoliosis is due to the girl being at peak height though he is concerned about his wife because
growth velocity. Peak height velocity is associ- she recently suffered a mild stroke. He denies
ated with a certain Tanner stage; what other any visual loss or motor or sensory weakness.
physical attributes would one expect to occur The patient’s physical examination is unre-
in this girl at the same time? markable. Which of the following is the lead-
ing cause of death among people age 65 years
(A) Elevation of the breast papilla only, and no
or older?
pubic hair
(B) Enlargement of the breast and areola with (A) Heart disease
a single contour, and darker, coarse, curled (B) Malignancy
pubic hair (C) Motor vehicle crashes
(C) Mature breast, and adult quantity and pat- (D) Stroke
tern of pubic hair that extends to the (E) Suicide
thighs
High-Yield Principles
8 Section I: General Principles  •  Questions

17. A 54-year-old man with a history of poorly con- (A) The incidence rate of diabetes among
trolled hypertension complains of new-onset mothers with LGA babies is four times that
headaches. His mother passed away at an early of non-LGA mothers
age due to a stroke, and his father died of a (B) The incidence rate of LGA among women
myocardial infarction. When asked why he with diabetes is four times that of women
does not take better care of his blood pressure, without diabetes
he states that he is so busy with work and with (C) The incidence rate of LGA among women
the church that, by the end of the day, he often without diabetes is four times that of
forgets to take his pills. He states that he “feels women with diabetes
fine, anyway.” What ego defense mechanism is (D) The odds of diabetes among mothers with
this patient using? LGA babies is four times that of non-LGA
mothers
(A) Denial
(E) The odds of LGA among women with dia-
Behavioral Science

(B) Displacement
betes is four times that of women without
(C) Projection
diabetes
(D) Rationalization
(E) Repression
20. A 45-year-old man presents to a psychiatrist at
his wife’s prompting. He is an English profes-
18. A 3-year-old girl presents for her regular check-
sor at the University of Virginia and regularly
up. Her mother reports that she is fully toilet
wins accolades for his well-organized and ar-
trained, and that she can dress and undress
ticulate lectures. In the past three months, he
with minor assistance. She speaks in full sen-
has become convinced that his wife is having
tences, can name four colors, and can copy a
an affair with a co-worker, despite her protests
simple circle drawing. What other milestone
to the contrary. His wife recently discovered
would this child most likely have reached
that he hired a private investigator to track her
since her last visit one year ago?
whereabouts. He is very defensive when the
(A) Engages in cooperative play counselor questions his suspicions about his
(B) Has imaginary friend(s) wife. Mental status examination reveals a well-
(C) Hops on one foot dressed, middle-aged man without hallucina-
(D) Reads tions or other mood disturbances. His speech
(E) Rides a tricycle is normal and displays an appropriate affec-
(F) Stacks five blocks tive range. Which of the following is the most
likely diagnosis?
19. A retrospective cohort study is examining birth
complications in women with diabetes. The (A) Antisocial personality disorder
study determines that babies are more likely to (B) Avoidant personality disorder
be born large for gestational age (LGA) if the (C) Delusional disorder
mother has diabetes. The relative risk for the (D) Schizoid personality disorder
study is calculated to be 4. Which of the fol- (E) Schizophrenia
lowing accurately describes this relative risk? (F) Schizophreniform disorder
High-Yield Principles
Chapter 1: Behavioral Science  •  Answers 9

An s w e r s

1. The correct answer is B. This prospective, ob- 2. The correct answer is C. This patient exhib-
servational study is a cohort study. Therefore, its some of the classic symptoms of narcolepsy,
the likely unit of measure is the relative risk, including daytime sleepiness, cataplexy, and
which is the risk of a health outcome with a sleep paralysis. Cataplexy is defined as brief
given exposure versus the risk of a health out- episodes of bilateral weakness brought on by
come without the exposure. In this case, the strong emotions such as laughing or fear, with-
relative “risk” of a health outcome is really the out alteration in consciousness. Sleep paralysis
relative chance of improvement of symptoms. is an episode of partial or total paralysis that
To calculate the relative “risk,” first calculate occurs at the beginning or end of a sleep cycle.
the chance of improvement with vitamin D Patients are often aware that they are awake,

Behavioral Science
supplementation by dividing the number of but may suffer from frightening hallucina-
subjects receiving supplementation whose tions known as hypnagogic when they occur
symptoms improved (50) by the total number at sleep onset, and hypnopompic when they
of subjects taking vitamin D; this is 50/350. occur on awakening. Modafinil is an amphet-
Then calculate the chance of improvement amine derivative used to treat attention deficit/
without vitamin D supplementation by divid- hyperactivity disorder and narcolepsy. Patients
ing the number of subjects not receiving sup- suffering from cataplexy and sleep paralysis
plementation whose symptoms improved by may also benefit from the initiation of tricyclic
the total number of subjects not taking vitamin antidepressants or selective serotonin reuptake
D; this is 60 / 460. The ratio of these values is inhibitors.
the relative risk: (50 / 350) / (60 / 460).
Answer A is incorrect. Chloral hydrate is a
Answer A is incorrect. This value does not ap- nonbenzodiazepine hypnotic that is used for
propriately calculate the relative risk with vita- sedation and insomnia. This patient does not
min D supplementation. The value is equiva- need help sleeping.
lent to the odds ratio, which is the measure
Answer B is incorrect. Hydroxyzine is a non-
typically used to analyze a retrospective, case-
selective antihistamine that is used in the treat-
control study.
ment of anxiety, pruritus, nausea/vomiting, se-
Answer C is incorrect. This value does not ap- dation, and insomnia.
propriately calculate the relative risk with vita-
Answer D is incorrect. Prochlorperazine
min D supplementation. It does not represent
maleate is a typical antipsychotic used in the
any commonly used measure of analysis.
treatment of nausea, vomiting, anxiety, and
Answer D is incorrect. This value does not ap- psychosis.
propriately calculate the relative risk with vita-
Answer E is incorrect. Zolpidem is a nonben-
min D supplementation. Instead, the value is
zodiazepine hypnotic that is used in the treat-
essentially equivalent to the inverse of a calcu-
ment of insomnia.
lation for odds ratio, which is not a measure
used in data analysis. 3. The correct answer is C. Transference occurs
Answer E is incorrect. This value does not ap- when a patient projects feelings from his or her
propriately calculate the relative risk of vitamin personal life onto a doctor; countertransfer-
D supplementation. The value instead calcu- ence takes place when the doctor projects feel-
lates the chance of improvement without vita- ings onto the patient. These feelings can be
min D supplementation relative to the chance either positive or negative. The patient’s anger
of improvement with vitamin D supplementa- at the doctor when her sexual advances are re-
tion. This is the inverse of what the question buffed is an example of negative transference.
asked.
High-Yield Principles
10 Section I: General Principles  •  Answers

Answer A is incorrect. The doctor being re- est (anhedonia), Guilt, Energy loss, Concen-
minded of his aunt by this patient is an exam- tration changes, Appetite changes, Psychomo-
ple of countertransference. tor abnormalities, and Suicidal thoughts (SIG
E CAPS). One of the symptoms has to be de-
Answer B is incorrect. The doctor telling the
pressed mood or anhedonia. Major depressive
patient that he is uncomfortable is not an ex-
disorder is diagnosed after a major depressive
ample of countertransference or transference.
episode without a history of mania, hypoma-
Answer D is incorrect. The patient dressing nia, or mixed episodes (when criteria for both
up for appointments is positive transference. manic and major depressive episode are simul-
Answer E is incorrect. The patient flirting taneously present for at least one week), and
with the doctor is positive transference. In its is further specified by modifiers such as recur-
most extreme form, positive transference can rent, chronic, or postpartum onset.
Behavioral Science

take the form of sexual desire. Answer C is incorrect. Arrhythmias are a


well-known adverse effect of the tricyclic anti-
4. The correct answer is D. This patient has a depressants. This class of medications, which
history of at least one major depressive episode includes imipramine, clomipramine, and
and at least one hypomanic episode without amitriptyline, work to block the reuptake of
the presence of mixed or manic episodes, a his- norepinephrine and serotonin. They are com-
tory consistent with bipolar II disorder. Treat- monly used medications in the treatment of
ment for this disorder is a mood stabilizer, major depression, obsessive compulsive disor-
most commonly lithium. A common adverse der, and fibromyalgia. However, the patient in
effect of this therapy is nephrogenic diabetes this case has a clinical history most consistent
insipidus, in which the principal cells of the with bipolar II disorder, and thus a mood sta-
renal collecting duct are unable to respond to bilizer such as lithium should be used rather
ADH secreted by the posterior pituitary. As a than an antidepressant, which could lead to
result, the patient will be unable to concen- further manic episodes in this patient.
trate urine and will thus experience frequent
urination. Answer E is incorrect. Stevens-Johnson syn-
drome is a well-known adverse effect of carba-
Answer A is incorrect. Agranulocytosis is a tox- mazepine, an anti-epileptic drug that is some-
icity associated with clozapine, an atypical an- times used to treat bipolar disorder. Although
tipsychotic. This medication is generally used this patient has a medical history consistent
in treatment of schizophrenia, but may be with bipolar II disorder, first-line treatment is
used in cases of mania that are unresponsive to generally with a different mood stabilizer such
first-line drugs such as lithium. Although the as lithium, as this medication has been proven
patient did have a history of a hypomanic epi- effective, is cheaper, and has a relatively less
sode, there is no indication for clozapine as a severe adverse-effect profile.
first-line treatment for bipolar II disorder.
Answer B is incorrect. Anorgasmia is a com- 5. The correct answer is A. The proper stage se-
mon adverse effect of selective serotonin quence of Freud’s psychosexual theory is oral,
reuptake inhibitors. This class of medication, anal, phallic, latency, and genital. Freud’s the-
which includes fluoxetine, paroxetine, sertra- ories of psychosexual development associate
line, and citalopram, is commonly used in the pleasure with certain bodily functions. Freud
treatment of major depression. Although this believed that between the ages of 18 months
patient has a history of feeling depressed, she and 3 years, children are preoccupied with
also has a history notable for a hypomanic epi- anal functions. Encopresis is fecal inconti-
sode. A major depressive episode is diagnosed nence and can range from mild to severe.
if the patient has 5/9 symptoms for at least two Answer B is incorrect. Freud’s genital stage
weeks, including Sleep changes, loss of Inter- encompasses adolescents to adults and is char-
High-Yield Principles
Chapter 1: Behavioral Science  •  Answers 11

acterized by the desire to achieve sexual grati- Answer C is incorrect. Patients presenting
fication. with acute cocaine intoxication will show
symptoms of euphoria, psychomotor agitation,
Answer C is incorrect. Freud’s latency stage
impaired judgment, tachycardia, pupillary di-
encompasses the ages of 6-12 years and is char-
lation, hypertension, hallucinations, paranoid
acterized by a suppression of sexual desire.
ideations, angina, and sudden cardiac death.
Answer D is incorrect. Freud’s oral stage en- On withdrawal, they will show a post-use
compasses birth to the age of 18 months. “crash” that includes severe depression, hyper-
Freud believed that children of this age gain somnolence, fatigue, malaise, and severe psy-
satisfaction from oral functions. chological craving.
Answer E is incorrect. Freud’s phallic stage Answer D is incorrect. Patients presenting
encompasses the ages of 3-5 years and is most with acute lysergic acid diethylamide intoxica-
commonly known as the oedipal phase. Freud

Behavioral Science
tion will display marked anxiety or depression,
believed that at this stage children begin to de- delusions, visual hallucinations, flashbacks,
velop sexual fantasies. and pupillary dilation.

6. The correct answer is F. This patient has Answer E is incorrect. Patients presenting
taken phencyclidine, or PCP. Patients with with acute nicotine intoxication will show
PCP intoxication show signs of belligerence, symptoms of restlessness, insomnia, anxiety,
impulsiveness, fever, psychomotor agitation, and arrhythmias. On withdrawal, they will
vertical and horizontal nystagmus, tachycardia, have symptoms of irritability, headache, anxi-
ataxia, homicidality, psychosis, and delirium. ety, weight gain, craving, and tachycardia.
On withdrawal, patients may demonstrate a
7. The correct answer is B. The prevalence is
recurrence of intoxication when the PCP,
the number of individuals with a disease in a
which was trapped in an ionized form in the
given population at a given time. Prevalence is
acidic gastric lumen, is reabsorbed in the alka-
estimated by test results but is not a measure of
line duodenum. PCP users will have normal
a test’s validity. In the chart shown, the preva-
or small pupils. Death can result from a variety
lence can also be determined by calculating
of causes, including respiratory depression and
the number of true-positive plus false-negative
violent behavior.
results divided by the total number of patients.
Answer A is incorrect. Patients presenting with
Answer A is incorrect. This term represents
acute alcohol intoxication will show symptoms
the incidence of positive test results.
of disinhibition, emotional lability, slurred
speech, ataxia, coma, and blackouts. On with- Answer C is incorrect. This represents true-
drawal, they will demonstrate a tremor, tachy- positive results divided by the total number of
cardia, hypertension, malaise, nausea, seizures, patients. This would be the percent of true-
delirium tremens, tremulousness, agitation, positive results of all tested, but it is not used
and hallucinations. very often.
Answer B is incorrect. Patients presenting Answer D is incorrect. This represents true-
with amphetamine intoxication will display positive results divided by the total number of
psychomotor agitation, impaired judgment, patients tested less those with true-positive re-
pupillary dilation, hypertension, tachycardia, sults, and would not be a meaningful calcula-
euphoria, prolonged wakefulness and atten- tion.
tion, cardiac arrhythmias, delusions, hallucina-
Answer E is incorrect. This represents the
tions, and fever. On withdrawal, they will show
number of true-positive results over the to-
a post-use “crash” that includes depression,
tal number of patients without disease. This
lethargy, headache, stomach cramps, hunger,
would not be a meaningful calculation.
and hypersomnolence.
High-Yield Principles
12 Section I: General Principles  •  Answers

8. The correct answer is A. Malpractice suits fascicles (panfascicular atrophy). Unlike SMA
require that the patient prove dereliction, types II and III, this patient’s disease developed
damage, and direct harm by a physician with at an early age, so early milestones were not
whom there was an established relationship. achieved. This is not the case in the less severe
Direct harm is a concept that the injury is forms of SMA.
causally related to the actions of the physician.
Answer A is incorrect. BMD involves the
This is also known as proximal cause, and in
same genetic locus that is affected in DMD,
many cases is the most difficult aspect to prove,
but its occurrence is less common. It follows a
as a temporal relationship does not necessarily
more indolent course, with onset often occur-
imply a causal relationship.
ring in late childhood.
Answer B is incorrect. It is not necessary for
Answer B is incorrect. Duchenne muscular
the doctor to have been present at the time of
dystrophy (DMD) and Becker muscular dystro-
Behavioral Science

injury, but there must be an established rela-


phy (BMD) are both characterized by defects
tionship between the physician and patient.
in the 427-kDa protein dystrophin, encoded on
Answer C is incorrect. Intent is not a factor the Xp21 region. DMD is the most common
in malpractice proceedings. These proceed- form of muscular dystrophy, with an incidence
ings are civil lawsuits, not criminal. As such, of about one in 3500 live births. Onset typi-
when intent or gross misconduct is proven, cally occurs after infancy and before the age
additional punitive damages may be assessed of five. The clinical course is characterized by
against the physician. progressive muscle weakness and wasting that
lead to wheelchair dependence by 10-12 years
Answer D is incorrect. As malpractice suits
of age. Early motor milestones are met in pa-
are civil rather than criminal proceedings, the
tients with BMD and DMD.
plaintiff is required only to prove “more likely
than not” that the actions of the defendant led Answer C is incorrect. Type III spinal muscu-
to damages. lar atrophy, or Kugelberg-Welander disease, is
characterized by the onset of proximal muscle
Answer E is incorrect. Proof of malpractice re-
weakness after the age of 2 years, the ability
quires dereliction, or deviation from standard
to walk independently until the disease pro-
procedure, that leads to the injury in question.
gresses, and survival into adulthood.
9. The correct answer is E. Spinal muscular Answer D is incorrect. Type II spinal mus-
atrophy (SMA) is one of the most common cular atrophy is characterized by the onset of
autosomal-recessive diseases, affecting ap- proximal muscle weakness before 18 months
proximately one in 10,000 live births. It has a of age, the ability to sit but not to walk un-
carrier frequency of approximately one in 50 aided, and survival beyond 4 years of age.
and is characterized by symmetric proximal
muscle weakness due to the degeneration of 10. The correct answer is C. Positive predictive
the anterior horn cells of the spinal cord. SMA value (PPV) is the probability that a positive
is classically divided into three subtypes based test result is truly positive. It can be calculated
on age of onset and clinical severity. Type I by taking the number of true-positive results
SMA (Werdnig-Hoffmann disease), the most and dividing it by the total positive results.
severe, is characterized by the onset of signifi- Since the prevalence is 40%, the number of
cant muscle weakness and hypotonia in the positive patients will be 0.4 × 1000 = 400 and
first few months of life, and the inability to the number of negative patients will be 1000 -
sit or walk. Manifestations may even occur in 400 = 600. The number of true-positives can
utero with reduced fetal movement. Fatal res­ be found by multiplying the sensitivity by this
piratory failure usually occurs before the age total, giving us 0.9 × 400 = 360. The number
of 2 years. Muscle biopsy demonstrates large of false-positives can be found by multiplying
numbers of atrophic fibers that involve entire the specificity by the total negative patients
High-Yield Principles
Chapter 1: Behavioral Science  •  Answers 13

and subtracting that product from the total tension-related adverse effects. However, the
negatives, or 600 - (0.85 × 600) = 90. Then, captopril is probably contributing to his erec-
360 / (360 + 90) = 0.8, or 80%. Remember: tile dysfunction.
PPV and negative predictive value change
Answer C is incorrect. Fluoxetine and silden­
with prevalence in a population, so the esti-
afil have no known dangerous interactions.
mated prevalence must be taken into account
However, the fluoxetine is probably contribut-
when calculating the number of true-positives
ing to his erectile dysfunction.
and true-negatives.
Answer D is incorrect. Lovastatin and silden­
Answer A is incorrect. Dividing the number of
afil have no known dangerous interactions.
false-negative results by the total negative re-
sults would give an answer of 8%. Answer E is incorrect. Metformin and silden­
afil have no known dangerous interactions.
Answer B is incorrect. Dividing the number

Behavioral Science
of false-positive (rather than true-positive) find- Answer G is incorrect. Propranolol and
ings by the total positive results would give an silden­afil have no known dangerous interac-
answer of 20%. tions, though the combination may increase
the risk of hypotension related adverse effects.
Answer D is incorrect. Switching the values
However, the propranolol may be contributing
for sensitivity and specificity would give an an-
to his erectile dysfunction.
swer of 85%.
Answer E is incorrect. The negative predictive 12. The correct answer is E. Emancipation is a
value is 93%. legal definition through which minors become
independent of their parents and are free to
11. The correct answer is F. As is common for make medical decisions for themselves. A mi-
many older patients, this man is taking several nor, which is a legal condition defined by age,
prescription medications. Though a couple of can generally acquire emancipation through
his prescriptions should be used with caution court order or marriage. These situations usu-
with sildenafil (captopril and propranolol), the ally suggest that the minor will be financially
only one that might significantly interact with independent of his or her parents. This patient
sildenafil is sublingual nitroglycerin. Nitroglyc- is married and is therefore emancipated.
erin is used for prompt relief of an ongoing at-
Answer A is incorrect. While this patient has
tack of angina precipitated by exercise or emo-
many adult responsibilities, 18 years is the le-
tional stress. Nitrates relax vascular smooth
gal age of consent and adulthood.
muscle by their intracellular conversion to
nitrite ions and then to nitric oxide, which in Answer B is incorrect. Full-time work suggests
turn activates cGMP and increases the cell’s that the patient is financially independent, but
cGMP level. Elevated cGMP ultimately taken alone it is not proof of emancipation.
causes vascular smooth muscle relaxation. Answer C is incorrect. A high school diploma
This is the same mechanism sildenafil uses to does not provide emancipation. Even though a
cause smooth muscle relaxation and increased minor becomes the primary decision maker af-
blood flow into the corpus cavernosum at a ter high school graduation, he or she is not nec-
certain level of sexual stimulation. Using these essarily financially independent of the parents.
drugs together can lead to severe hypotension
and cardiovascular collapse. Answer D is incorrect. A teenager may state
he or she has separated from the parents, but
Answer A is incorrect. Aspirin and sildenafil unless the courts have approved a legal separa-
have no known dangerous interactions. tion, merely saying she is “separated” from her
Answer B is incorrect. Captopril and sildenafil parents is not enough; legally the parents are
have no known dangerous interactions, though still financially responsible for the child until
the combination may increase the risk of hypo- he or she turns 18.
High-Yield Principles
14 Section I: General Principles  •  Answers

13. The correct answer is E. The study described the prevalence of the risk factor in the popula-
here is a cohort study, because it includes a tion, which would not be known in this study.
group with and a group without a given risk
factor (fetal exposure to alcohol) and then 14. The correct answer is B. Tanner stage 3 is the
looks at whether the risk factor changes the stage when most girls experience peak height
chances of offspring getting the disease (abnor- velocity. Peak height velocity occurs approxi-
malities). The study is prospective, because the mately one year after the initiation of breast de-
group members are looked at before the dis- velopment. Also, pubic hair becomes dark and
ease (abnormality) develops in the offspring. curly during this stage.
Relative risk can be calculated from the results Answer A is incorrect. This description corre-
of a cohort study by comparing the rate of dis- sponds to Tanner stage 1.
ease in the group with the risk factor to the rate
of disease in the group without the risk factor. Answer C is incorrect. This description corre-
Behavioral Science

sponds to Tanner stage 5.


Answer A is incorrect. Attributable risk can be
calculated from the results of a cohort study Answer D is incorrect. This description corre-
and describes the proportion of disease that is sponds to Tanner stage 4.
due to the risk factor under study. Although Answer E is incorrect. This description corre-
smoking behavior of the women is being re- sponds to Tanner stage 2.
corded, the study is not designed to look at
the impact of this risk factor on fetal abnor- 15. The correct answer is D. Obtaining informed
mality; the rate of smoking in the two groups consent from the patient means that the pa-
of women is unknown, and thus we do not tient understands the risks, benefits, and al-
know whether there are sufficient numbers ternatives to the study, and that the doctor re-
of women in the “exposed” and “unexposed” lays to the patient pertinent matters about the
groups when it comes to tobacco. plan of care. For the non-English-speaking
Answer B is incorrect. An odds ratio is similar patient, the consent is translated into the ap-
to relative risk, but it is calculated from the re- propriate language and discussed with him/her
sults of a case-control study, not from a cohort through an interpreter. This allows the patient
study. Because birth abnormality is a relatively (or in this case, his parents) freedom to read
rare outcome, the odds ratio from a case-control and process the consent and to discuss it later.
study would likely closely approximate the ac- Whereas this option may not be possible for
tual relative risk. every language or reasonable for every study, it
is appropriate in this non-emergent situation.
Answer C is incorrect. Prevalence is a mea-
sure of how many cases of a given disease exist Answer A is incorrect. With limited knowl-
in a population that is at risk for that disease. edge of Spanish, the doctor will unlikely be
This is not the best answer in this case, be- able to address all the important issues delin-
cause there are no data to judge whether this eated in the consent form.
group of 200 women represents the true preva- Answer B is incorrect. Having someone other
lence of alcoholism during pregnancy in the than an interpreter translate will be invading
community. patient privacy, incomplete, and not perfectly
Answer D is incorrect. The statistic described accurate/reliable.
in this answer is the population attributable Answer C is incorrect. In a non-emergent set-
risk (PAR), which helps us understand, in a ting, the best approach is to allow the patient/
given population, how much less common a family to view a translated copy of the consent
disease (fetal abnormality) would be if a given and consider all their options in an unbiased
risk factor (alcohol consumption during preg- manner. The use of an interpreter, however,
nancy) were completely eliminated. To calcu- would be invaluable in an emergent setting.
late PAR, the attributable risk is multiplied by
High-Yield Principles
Chapter 1: Behavioral Science  •  Answers 15

Answer E is incorrect. The patient is too ently more or less logical explanation for an
young to give consent (<18 years). act or decision actually produced by uncon-
scious impulses. This patient’s assertion that he
16. The correct answer is A. Heart disease is the “feels fine, anyway” may be a form of rational-
leading cause of death among the elderly (65 ization, but the primary ego defense mecha-
years old and older), as well as the leading nism that he is using is denial.
cause of death if all ages are combined. The
Answer E is incorrect. Repression is the un-
patient is at a particularly high risk for subse-
conscious exclusion of a painful or anxiety-
quent cardiac events due to his previous history
provoking thought, impulse, or memory from
of myocardial infarction. Other major risk fac-
awareness.
tors for cardiac events are high blood pressure,
hypercholesterolemia, and diabetes mellitus.
18. The correct answer is E. This is a typical
Answer B is incorrect. Cancer is the second well-child visit. Other milestones reached at

Behavioral Science
most common cause of death. It ranks after approximately this age include stacking nine
heart disease, but before stroke. blocks, riding a tricycle, and beginning to en-
gage in group play. Riding a tricycle at 3 years
Answer C is incorrect. Motor vehicle crashes,
is easy to remember because a tricycle has
and accidents in general, are leading causes of
three wheels. The number of blocks stacked
death. However, they are the most common
between ages 2 and 4 years is about three times
cause of death among children (1-14 years
the child’s age in years; that is, a 2-year-old can
old) and adolescents (15-24 years old) and not
stack six blocks, whereas a 3-year-old can stack
among the elderly.
nine blocks.
Answer D is incorrect. Stroke is the third most
Answer A is incorrect. Engaging in coopera-
common cause of death among the elderly, be-
tive play is achieved between 4 and 5 years of
hind heart disease and then cancer.
age.
Answer E is incorrect. Suicide is not the lead-
Answer B is incorrect. Imaginary friends are
ing cause of death in the elderly. It is a com-
typically present between 4 and 5 years of age.
mon cause of death among adolescents.
Answer C is incorrect. Hopping on one foot is
17. The correct answer is A. The mechanism of typical of a 4-year-old child.
denial is when one fails to recognize the obvi-
Answer D is incorrect. Reading is most often
ous implications or consequences of a thought,
learned at 5-6 years of age and older.
act, or situation. This ego defense mechanism
is often seen in patients with recently diag- Answer F is incorrect. Children at 2 years of
nosed HIV or cancer. age can usually stack six blocks, whereas chil-
dren at 3 years can stack nine blocks.
Answer B is incorrect. Displacement is a de-
fense mechanism whereby ideas and feelings
19. The correct answer is B. A retrospective co-
that a patient wishes to avoid are transferred
hort study includes a group of subjects who
to another person or object; for example, a pa-
had a certain condition or received a certain
tient who yells at the nurse because he is angry
treatment at some time in the past and com-
at news he has just received from the doctor.
pares their outcomes to those of another group
Answer C is incorrect. Projection is the pro- (a control group) made up of subjects who did
cess of attributing one’s thoughts or impulses, not have this condition or receive the treat-
usually ones that are unacceptable or undesir- ment. In this study the risk factor is the pres-
able, to another person. ence of diabetes in the mothers and the out-
come is LGA babies. The incidence of LGA
Answer D is incorrect. Rationalization pro-
births in women with diabetes is four times
duces a more socially acceptable and appar-
that in women without diabetes. Relative risk is
High-Yield Principles
16 Section I: General Principles  •  Answers

defined as the incidence rate of some outcome Answer A is incorrect. Cluster B personality
in those exposed to a risk factor divided by the disorders include antisocial, borderline, histri-
incidence rate of those not exposed. This defi- onic, and narcissistic types. Patients with anti-
nition gives the factor at which the incidence social personality disorder show a disregard for
rate of LGA among women with diabetes is and often violate the rights of others. These in-
larger than the incidence rate of LGA among dividuals often have a criminal history. This is
women without diabetes. the only personality disorder with an age limit
(18 years). Minors with similar behavior are
Answer A is incorrect. This choice describes
classified as having conduct disorder.
the correct type of risk analysis but describes
the relationship in reverse. Answer B is incorrect. Cluster C personal-
ity disorders are characterized by anxiety and
Answer C is incorrect. This choice reverses
include avoidant, obsessive-compulsive, and
the findings of the study, which shows that
Behavioral Science

dependent personality types. Individuals with


the incidence of LGA is four times more in
avoidant personality disorder are sensitive to
women with diabetes.
rejection, are socially inhibited, and have over-
Answer D is incorrect. This choice incorrectly whelming feelings of inadequacy.
uses odds rather than incidence rates and also
Answer D is incorrect. Patients with schiz-
describes the relationship of the findings of the
oid personality disorder exhibit voluntary so-
study in reverse.
cial withdrawal (unlike avoidant patients) and
Answer E is incorrect. This choice describes have limited emotional expressions. They lack
an odds ratio for a case-control study. A case- strange beliefs and thoughts of the schizotypal
control study evaluates the presence of risk personality disorder.
factors in people with and without a disease.
Answer E is incorrect. Schizophrenia is a
Although this is the opposite of a cohort study,
chronic psychiatric condition. It affects 1% of
the results are still reported in terms of disease
the population and usually begins before age
presence with respect to risk factors; that is, the
25. The Diagnostic and Statistical Manual
presence or absence of disease is categorized
of Mental Disorders, Fourth Edition, specifies
in the group with risk factors and compared to
the active phase of the disease and requires
the group without risk factors. The difference,
that at least two of the following symptoms be
however, is that odds are used rather than inci-
present during a one-month period: delusions,
dence. The incidence rate is a percentage (eg,
hallucinations, disorganized speech, grossly
50 out of 100). Odds are calculated by dividing
disorganized or catatonic behavior, and nega-
those with disease by those without (50 to 50,
tive symptoms (eg, flat affect, lack of motiva-
or 1 to 1).
tion, or poverty of speech). Moreover, signs of
20. The correct answer is C. Delusional disor- the disturbance must be present for at least six
der is diagnosed following one month of non-­ months, such as one of the above symptoms in
bizarre delusions that are usually focused an attenuated form (eg, magical thinking, so-
around a particular topic, in this case, the fi- cial withdrawal, or other negative symptoms).
delity of the patient’s wife. The delusions are Answer F is incorrect. Schizophreniform dis-
not attributable to another psychiatric disorder order is similar to schizophrenia except that
such as schizophrenia. Delusional disorder its symptoms have lasted between one and six
does not markedly impair the person’s func- months. In contrast, patients with schizophre-
tioning in daily activities. The ramifications nia must have had symptoms for longer than
are limited to the delusional content. six months.
Chapter 2

Biochemistry

17
High-Yield Principles
18 Section I: General Principles  •  Questions

Q u e st i o n s

1. A 6-year-old boy presents to his pediatrician 3. A 54-year-old woman complains of fatigue,


with skin lesions all over his body. For several difficulty climbing stairs, and weight loss. Her
years he has been very sensitive to sunlight. medical history is notable for hypertension.
Neither the boy’s parents nor his siblings have She takes no medications. Her skin is moist
the same skin lesions or sun sensitivity. Biop- and she has a prominent stare. The patient’s vi-
sies of several of the boy’s lesions reveal squa- tal signs are:
mous cell carcinoma. Which mutation would
Heart rate: 112/min
one expect to see in this patient’s DNA?
Blood pressure: 143/90 mm/Hg
(A) Methylation of the gene Respiratory rate: 14/min
(B) Missense mutation in the gene Oxygen saturation: 98% on room air
(C) Nonsense mutation in the middle of the
Biochemistry

Laboratory tests reveal markedly elevated levels


gene
of a specific hormone. What is the first mole-
(D) Point mutation within the enhancer region
cule produced within a cell in response to this
(E) Point mutation within the operator region
hormone?
(F) Point mutation within the promoter region
(G) Thymidine dimers (A) Carbohydrate
(B) DNA
2. A metabolic process is pictured below. Which (C) Fatty acid
intermediate in this process inhibits the rate- (D) Protein
limiting enzyme of glycolysis and activates the (E) RNA
rate-limiting enzyme of fatty acid synthesis?
4. A 35-year-old man presents to the physician
Pyruvate
with arthritic pain in both knees along with
back pain. He states that the pain has been
Acetyl-CoA present for months. In an effort to obtain re-
lief, he has taken only aspirin, but this has
D B been of little benefit. The patient is afebrile,
NADH
cis-aconitate
and his slightly swollen knee joints are neither
hot nor tender to palpation; however, the pain
C Isocitrate
does restrict his motion. The cartilage of his
CO2 + NADH
Fumarate ears appears slightly darker than normal. No
FADH2
A tophi are present. A urine specimen is taken
CO2 + NADH for analysis of uric acid content and turns
Succinate
GTP black in the laboratory while standing. A de-
+
CoA E fect in which of the following is the most likely
underlying cause of the patient’s condition?
Reproduced, with permission, from USMLERx.com. (A) a-Ketoacid dehydrogenase
(B) Galactokinase
(C) Homogentisic acid oxidase
(A) A
(D) Orotate phosphoribosyl transferase
(B) B
(E) Phenylalanine hydroxylase
(C) C
(D) D
(E) E
High-Yield Principles
Chapter 2: Biochemistry  •  Questions 19

5. A patient who is a carrier of sickle cell trait 6. A 57-year-old woman visits her primary care
presents to the clinic. The single base-pair mu- physician. Laboratory studies reveal an LDL of
tation for sickle cell anemia destroys the MstII 194 mg/dL and HDL of 41 mg/dL. Her phy-
restriction enzyme recognition site represented sician begins therapy with a drug that inhibits
by an asterisk in the image. The restriction production of mevalonic acid. Which of the
enzyme-binding sites are shown as arrows on following is a common side effect of this ther-
the map. DNA from this patient is treated with apy?
MstII and run on an electrophoresis gel. The
(A) Hepatomegaly without elevations in aspar-
DNA is then hybridized with a labeled probe
tate aminotransferase or alanine amino-
that binds to the normal gene in the posi-
transferase
tion shown on the map. In the Southern blot
(B) Muscle injury clinically similar to myositis
shown in the image, which lane represents the
(C) Spontaneous abortion of a pregnancy
patient?
(D) Suicidality and homicidality in patients
with bipolar disorder

Biochemistry
βA Globin gene (E) Tonic-clonic seizures
MstII Probe MstII MstII
A B C D E F 7. A nucleic acid fragment is added to four dif-
1.15 Kb 0.2 Kb 1.35 Kb ferent tubes along with a polymerase, a radio-
MstII MstII 1.15 Kb labeled primer, and deoxynucleotides. Each
0.2 Kb
* tube also contains one of the four bases as
βS Globin gene dideoxynucleotides. The four tubes are then
run on electrophoresis gel and visualized by
Reproduced, with permission, from USMLERx.com.
autoradiography. For which of the following
purposes would the described laboratory tech-
(A) A nique be utilized?
(B) B (A) To amplify DNA fragments
(C) C (B) To create an allele-specific oligonucleotide
(D) D probe
(E) E (C) To decipher the order of nitrogenous bases
(F) F in the human genome
(D) To determine the base pairing of a seg-
ment of DNA with a DNA probe
(E) To determine the base pairing of a seg-
ment of RNA with a DNA probe
(F) To establish the presence of a given pro-
tein
(G) To show the presence of a specific anti-
body in plasma
High-Yield Principles
20 Section I: General Principles  •  Questions

8. A 45-year-old white woman presents to her regulated both across the cell membrane and
physician complaining of several months of within the cell via sequestration in the en-
worsening shortness of breath. Previously she doplasmic reticulum and mitochondria. In
was told she had asthma because she was hav- which of the following ways does increased
ing intermittent episodes of wheezing com- intracellular calcium concentration cause the
bined with a productive cough and difficulty most cell damage?
catching her breath. She used to run two miles
(A) Enzyme activation
every morning but can no longer walk more
(B) Free radical generation
than 10 city blocks without stopping. She has
(C) Increased membrane permeability
never smoked. On physical examination she is
(D) Inhibition of glycolysis
using her accessory muscles to assist with res-
(E) Inhibition of oxidative phosphorylation
piration. Pulmonary examination is notable
for an increased decreased FEV1/FVC ratio, 10. A scientist working in a research laboratory has
decreased air movement with each breath, and been examining different agonists of serotonin
Biochemistry

increased resonance upon percussion. X-ray of receptor 1B (5-HT1B), a G-protein-coupled re-


the chest is shown in the image. Which of the ceptor. Compound A has a much higher affin-
following is the most likely underlying cause ity for 5-HT1B than compound B. Both com-
for this patient’s disease? pounds have a higher affinity for the receptor
than serotonin. Which of the following de-
scribes the relationship between compound A
and compound B when considering the guan­
ine-nucleotide exchange activity of 5-HT1B?
(A) Km for the exchange reaction with com-
pound A is higher than that with com-
pound B
(B) Km for the exchange reaction with com-
pound A is lower than that with compound
B
(C) Km values with compounds A and B are
the same
(D) The maximum reaction rate with com-
pound A is greater than that with com-
Courtesy of Dr. James Heilman. pound B
(E) The maximum reaction rate with com-
pound B is greater than that with com-
(A) A genetic mutation resulting in deficient pound A
levels of a protease
(B) A genetic mutation resulting in deficient 11. A mother brings her 6-month-old son to the
levels of a protease inhibitor pediatrician. She has noticed that he seems
(C) A mutation in the p53 gene “afraid of light” and, after some Internet re-
(D) A mutation of the CFTR gene, which en- search, she is concerned that he might be an
codes a regulated chloride channel albino. Laboratory analysis reveals uropor-
(E) Airway inflammation, airflow obstruction, phyrin in his urine. The child most likely has
and bronchial hyperresponsiveness which of the following conditions?
(A) Deficiency of coproporphyrinogen oxidase
9. As increased intracellular calcium is detrimen-
(B) Deficiency of porphobilinogen deaminase
tal to the cell, calcium homeostasis is tightly
(C) Deficiency of uroporphyrinogen decarbox-
ylase
High-Yield Principles
Chapter 2: Biochemistry  •  Questions 21

(D) Inhibition of ferrochelatase and (A) 3-Hydroxy-3-methylglutaryl coenzyme A


δ-aminolevulinic acid dehydrase reductase
(E) Overexpression of porphobilinogen deami- (B) Adenosine deaminase
nase (C) Aldose reductase
(D) Galactose-1-phosphate uridyltransferase
12. A 48-year-old woman of Mediterranean de- (E) Hexokinase
scent presents because of fatigue, arthralgias, (F) Insulin-like growth factor
discomfort in her right upper abdominal quad-
rant, and polyuria. Laboratory tests are remark- 14. Acquired mutation in the p53 gene is the most
able for elevated glucose level, elevated biliru- common genetic alteration found in human
bin, low hemoglobin, elevated reticulocytes, cancer (> 50% of all cancers). A germline
and increased transferrin saturation. Cardiac mutation in p53 is the causative lesion of Li-
testing shows moderate restrictive cardiomyop- Fraumeni familial cancer syndrome. In many
athy. She frequently has required blood trans- tumors, one p53 allele on chromosome 17p is

Biochemistry
fusions throughout her life. Which hereditary deleted and the other is mutated. What type of
disorder does this patient most likely have? protein is encoded by the p53 gene?
(A) Absence of the hemoglobin a-chain (A) Caspase
(B) Absence of the hemoglobin b-chain (B) DNA repair enzyme
(C) Mutation resulting in increased absorption (C) Membrane cell adhesion molecule
of dietary iron (D) Serine phosphatase
(D) Mutations in the gene encoding ankyrin (E) Telomerase
(E) Mutations resulting in copper accumula- (F) Transcription factor
tion (G) Tyrosine kinase

13. A 52-year-old man with a 12-year history of 15. A segment of DNA is isolated and added to a
poorly controlled diabetes mellitus presents to mixture of four deoxynucleotides, two specific
his physician complaining of changes in his vi- oligonucleotide sequences, and heat-stable
sion. Physical examination reveals opacities on DNA polymerase. The mixture is then heated
the lens of the eye similar to those seen in this to denature the DNA, cooled, and reheated
image. Which enzyme most likely contributed in a number of cycles. Which of the following
to this complication? laboratory techniques does this describe?
(A) Enzyme-linked immunosorbent assay
(B) Gel electrophoresis
(C) Northern blot
(D) Polymerase chain reaction
(E) Sequencing
(F) Southern blot
(G) Western blot

Courtesy of Dr. Rakesh Ahuja.


High-Yield Principles
22 Section I: General Principles  •  Questions

16. A 32-year-old woman presents to her physician (C) Microorganism’s ability to produce b-lacta-
for the third time in six months. She has been mase
feeling very tired and depressed, and has come (D) Mutation in terminal amino acid of micro-
to talk about starting antidepressants. She also organism’s cell wall component
complains of a 4.5-kg (10-lb) weight gain over (E) Mutation in the microorganism’s penicil-
the past three months. During her physical lin-binding protein
examination the physician notices that she is
wearing a sweater and a coat, despite the room 18. A 2-year-old boy presents to the pediatrician
being at a warm temperature. Problems with with fever, facial tenderness, and a green, foul-
the thyroid are suspected, and a biopsy is per- smelling nasal discharge. The patient is diag-
formed (see image). This woman may have a nosed with sinusitis, and the physician notes
human leukocyte antigen subtype that also in- that he has a history of recurrent episodes of
creases her risk of which disease? sinusitis. X-ray of the chest is ordered because
of the fever; it reveals some dilated bronchi
Biochemistry

and shows the heart situated on the right side


of his body. A congenital disorder is diagnosed.
Which other finding would this patient be
most likely to have?
(A) Defective chloride transport
(B) Elevated blood sugar
(C) Infertility
(D) Reactive airway disease
(E) Tetralogy of Fallot

19. A 5-day-old boy is brought to the emergency


department after a tonic-clonic seizure at
home. The infant is the product of a full-term,
uneventful pregnancy, and was normal until
Reproduced, with permission, from USMLERx.com. two days prior to presentation. The mother re-
ports irritability and poor feeding at home, and
the infant was difficult to rouse this morning
(A) Multiple sclerosis
before suffering the seizure. On physical ex-
(B) Pernicious anemia
amination, the infant is tachypneic to 75/min,
(C) Psoriasis
has icteric sclerae, and has poor muscle tone
(D) Steroid-responsive nephrotic syndrome
throughout. Laboratory studies show the fol-
lowing levels: plasma ammonia, 300 µmol/L
17. A 65-year-old woman who has been in the hos-
(normal = 10-40 µmol/L); blood urea nitro-
pital for three weeks receiving cefotaxime to
gen, 1.5 mg/dL; and creatinine, 0.4 mg/dL.
treat Klebsiella pneumonia develops a urinary
A plasma amino acid analysis fails to detect
tract infection. Urine cultures are positive for
citrulline. Urine amino acids demonstrate el-
Enterococcus faecium. Treatment with vanco-
evated orotic acid levels. This patient suffers
mycin is attempted but is unsuccessful. Which
from a deficiency of which of the following en-
of the following aided in this microorganism’s
zymes?
ability to persist despite vancomycin treat-
ment? (A) a-Galactosidase A
(B) Aldose B
(A) Alteration of microorganism’s gyrase
(C) Galactose 1-phosphate uridylyltransferase
(B) Methylation of microorganism’s rRNA at a
(D) Lysosomal a-glucosidase
ribosome-binding site
(E) Ornithine transcarbamylase
High-Yield Principles
Chapter 2: Biochemistry  •  Questions 23

20. A 42-year-old woman presents to her physician gin. X-ray of the chest reveals cardiomegaly.
with generalized itching. Physical examination He has a difficult time sitting upright and
reveals scleral icterus. Laboratory tests show: cannot squeeze the physician’s fingers or the
ring of his pacifier with any noticeable force.
Total bilirubin: 2.7 mg/dL
Despite a number of interventions, the child’s
Conjugated bilirubin: 2.4 mg/dL
symptoms continue to worsen until his death
Alkaline phosphatase: 253 U/L
two weeks later. On autopsy, it is likely that this
Aspartate aminotransferase: 36 U/L
patient’s cells will contain an accumulation of
Alanine aminotransferase: 40 U/L
which of the following substances?
What is the most likely mechanism underlying
(A) Glucose
this patient’s jaundice?
(B) Glycogen
(A) Absence of UDP-glucuronyl transferase (C) Oxaloacetate
(B) Decreased levels of UDP-glucuronyl trans- (D) Pyruvate
ferase (E) Urea

Biochemistry
(C) Extravascular destruction of the patient’s
RBCs 23.
After consumption of a carbohydrate-rich
(D) Intrahepatic or extrahepatic biliary ob- meal, the liver continues to convert glucose to
struction glucose-6-phosphate. The liver’s ability to con-
(E) Intravascular destruction of the patient’s tinue this processing of high levels of glucose
RBCs is important in minimizing increases in blood
glucose after eating. What is the best explana-
21. A 5-year-old boy was playing outside during tion for the liver’s ability to continue this con-
recess when he began to experience difficulty version after eating a carbohydrate-rich meal?
breathing. He was brought to his physician,
(A) The hepatocyte cell membrane’s permea-
because his symptoms seemed to be getting
bility for glucose-6-phosphate
worse. On examination, the physician notes
(B) The high maximum reaction rate of gluco-
that the boy is struggling to breathe and hears
kinase
diffuse wheezing bilaterally. The boy’s heart
(C) The high maximum reaction rate of hexo-
rate is 98/min, respiratory rate is 24/min, and
kinase
oxygen saturation is 90%. His medical history
(D) The high Michaelis-Menten constant of
is significant only for seasonal allergies and
hexokinase
mild eczema. Which type of medication will
(E) The low Michaelis-Menten constant of
alleviate this patient’s respiratory symptoms?
glucokinase
(A) b1 Antagonist
(B) b1 Agonist 24. A 30-year-old man is diagnosed with type I fa-
(C) b2 Agonist milial dyslipidemia. Recent laboratory studies
(D) Histamine1 agonist show an elevated triglyceride level but normal
(E) Histamine2 agonist LDL and HDL cholesterol levels. Which of
the following explains the pathophysiology of
22. An 8-month-old boy is brought to the pediatri- this disease?
cian by his parents because he has recently lost
(A) Apolipoprotein E deficiency
the ability to crawl or hold his toys. On exami-
(B) LDL cholesterol receptor deficiency
nation the patient is tachypneic and breathing
(C) Lipoprotein lipase deficiency
with considerable effort; the liver is palpable
(D) VLDL cholesterol clearance deficiency
five finger widths below the right costal mar-
(E) VLDL cholesterol overproduction
High-Yield Principles
24 Section I: General Principles  •  Questions

25. A 59-year-old woman with history of morbid cheeks. He has arachnodactyly, pes cavus,
obesity, hypercholesterolemia, and diabetes and bilaterally dislocated lenses, and demon-
mellitus presents to the emergency depart- strates developmental delay with mild mental
ment with complaints of substernal chest pain retardation. His mother is told that her child
lasting two hours. An ECG reveals ST eleva- might benefit from folic acid supplementation.
tions in the lateral leads. The troponin level Which is the most appropriate test to confirm
at admission is extremely elevated, and a cre- the diagnosis?
atine kinase-myocardial bound test is pend-
(A) Enzymatic assay for the enzyme HGPRT
ing. Which of the following is a key cell me-
(B) Genetic studies demonstrating a mutation
diator in the pathogenesis of an atherosclerotic
in type I collagen
plaque?
(C) Genetic studies indicating >200 copies of
(A) γ-Interferon the CGG trinucleotide repeat on the X
(B) Complement chromosome
(C) Interleukin-6 (D) Nitroblue tetrazolium test
Biochemistry

(D) Natural killer cells (E) Nitroprusside cyanide test


(E) Platelet-derived growth factor
28. A 6-year-old boy is brought to his pediatrician’s
26. A 53-year-old man presents to his physician, office by his parents, who report that the child
because he has blood in his urine and some has been unusually thirsty for the past week.
low back pain. A gross specimen of kidneys He also has increased urinary frequency and
from a patient with the same condition is has wet the bed three times in the past two
shown in the image. Which of the following weeks. A random blood glucose level is 215
also is associated with this disorder? mg/dL. The pediatrician suspects that the
child has type 1 diabetes mellitus caused by
autoimmune destruction of insulin-producing
pancreatic b cells. Which of the following is
the transporter for glucose to enter pancreatic
b cells?
(A) GLUT 1
(B) GLUT 2
(C) GLUT 4
(D) Simple diffusion

29. Hemoglobin consists of four polypeptide sub-


units: two a subunits and two b subunits. The
arrangement of these subunits shifts between
a taut and relaxed conformation, resulting in
Reproduced, with permission, from USMLERx.com. changes in hemoglobin’s oxygen affinity. At a
given partial pressure of oxygen, which of the
following will decrease hemoglobin’s affinity
(A) Astrocytomas
for oxygen?
(B) Berry aneurysm
(C) Ectopic lens (A) Decreasing the partial pressure of carbon
(D) Optic nerve degeneration dioxide
(E) Squamous cell carcinoma (B) Increasing the amount of 2,3-bisphospho-
glycerate in RBCs
27. At a routine check-up, a 7-year-old boy is (C) Increasing the number of oxygen mole-
found to have osteoporosis. The patient is tall cules bound to a hemoglobin from one to
and thin with pale skin, fair hair, and flushed three
High-Yield Principles
Chapter 2: Biochemistry  •  Questions 25

(D) Increasing the pH by moving from periph- (A) Adenosine deaminase


eral tissue to lung (B) Homogentisic acid oxidase
(E) The presence of excess carbon monoxide (C) Lysosomal a-1,4-glucosidase
(D) Ornithine transcarbamylase
30. A 15-year-old boy presents with prolonged fa- (E) Porphobilinogen deaminase
tigue and mild jaundice following a serious
infection. Blood tests reveal hemoglobin of 33. A 2-year-old boy is brought by his parents to
10.5 g/dL and an elevated reticulocyte count. the emergency department after the discovery
A peripheral blood smear reveals Heinz bodies. of blood in a wet diaper. The physician pal-
Which of the following best describes the nor- pates an abdominal mass in the right flank. CT
mal action associated with this patient’s meta- of the abdomen reveals a large tumor invading
bolic defect? the right kidney; the gross specimen is shown
in the image. Cytogenetic analysis of the tu-
(A) To generate glucose-6-phosphate in all
mor cells reveals a deletion of chromosome
cells

Biochemistry
11p. Which of the following is the most likely
(B) To generate glucose-6-phosphate in RBCs
diagnosis?
only
(C) To generate mucopolysaccharides
(D) To regenerate reduced nicotinamide ade-
nine dinucleotide phosphate in all cells
(E) To regenerate reduced nicotinamide ade-
nine dinucleotide phosphate in RBCs only

31. Hyperparathyroidism is a common manifesta-


tion of several distinct genetic disorders that
predispose to endocrine gland neoplasia and
cause hormone excess syndromes. Which of
the following is a consequence of parathyroid
hormone?
(A) Increase calcium absorption in the small
intestine
(B) Inhibit the production of 1,25-dihydroxyvi-
tamin D
(C) Promote calcium excretion in the renal tu-
bules
(D) Stimulate further secretion of parathyroid
hormone
(E) Stimulate phosphate reabsorption in the
renal tubules

32. A 22-year-old woman presents to the hospital


with severe abdominal pain, abdominal dis- Reproduced, with permission, from USMLERx.com.
tention, and ileus, along with peripheral neu-
ropathy. Her boyfriend notes that she has been
(A) Adrenocortical adenoma
acting strange lately, and that she “seems like
(B) Neuroblastoma
a different person.” Which of the following en-
(C) Renal cell carcinoma
zymes is deficient in this patient?
(D) Transitional cell carcinoma
(E) Wilms tumor
High-Yield Principles
26 Section I: General Principles  •  Questions

34. A 6-year-old boy is diagnosed with a worsening 37. A term child is delivered by spontaneous vagi-
ataxic gait and a cardiac dysrhythmia. His un- nal delivery without complications. Upon
cle also has this condition, but his symptoms physical examination the child has bilateral
did not appear until he was 12 years of age. hip dislocations, restricted movement in shoul-
What is the molecular mechanism of this dis- der and elbow joints, and coarse facial features.
ease? Laboratory studies show that the activities of
b-hexosaminidase, iduronate sulfatase, and
(A) Unstable repeats affect protein folding
arylsulfatase A are deficient in cultured fibro-
(B) Unstable repeats affect protein splicing
blasts, but are 20 times normal in the patient’s
(C) Unstable repeats cause an amino acid sub-
serum. The primary abnormality in this disor-
stitution
der is associated with which of the following
(D) Unstable repeats impede protein transla-
organelles?
tion
(E) Unstable repeats result in a truncated pro- (A) Golgi apparatus
tein (B) Lysosomes
Biochemistry

(C) Ribosomes
35. A 3-month-old child is brought to his pediatri- (D) Rough endoplasmic reticulum
cian’s office for a check-up. On examination, (E) Smooth endoplasmic reticulum
the physician notices that he has a social smile,
but does not hold his head up on his own or 38. Patients with albinism appear white-pink (skin
make noises. The infant also has pale skin, ec- color), have white hair, and have nonpig-
zema, odd odor, and hyperreflexia. What is the mented or blue irises. In many cases, these
most appropriate treatment for the condition individuals may have melanocytes, but lack
the infant most likely has? melanin in their skin. What is the most use-
ful advice to give to a guardian of a child diag-
(A) A diet low in isoleucine and leucine
nosed with albinism?
(B) A diet low in phenylalanine
(C) A diet low in tyrosine (A) To avoid foods with lactose
(D) A high-protein diet (B) To avoid foods with phenylalanine
(E) Recombinant enzyme therapy (C) To avoid strenuous activity
(D) To give growth hormone to help the child
36. A 63-year-old man who is an alcoholic is grow to a normal height
brought into the emergency department by his (E) To wear clothing and sunscreen that pro-
daughter. She states that the patient’s memory tect from the sun when outside
has been very poor, and he constantly creates
elaborate yet untrue stories. Physical examina- 39. A 65-year-old African-American man presents
tion reveals ataxia and bilateral horizontal nys- to his physician because of jaundice. He says
tagmus. Wernicke-Korsakoff syndrome, caused that in the past few months he has not had
by a water-soluble vitamin deficiency, is sus- much of an appetite and has lost 13.6 kg (30
pected. Which of the following conditions is lb). Physical examination is notable for a gall-
also a result of a water-soluble vitamin defi- bladder that is palpable. What set of character-
ciency? istics is expected in this patient?
(A) Increased erythrocyte hemolysis
(B) Neonatal hemorrhage
(C) Night blindness
(D) Osteomalacia
(E) Pellagra
High-Yield Principles
Chapter 2: Biochemistry  •  Questions 27

Type of Urine Urine


Na+: 135 mEq/L
Choice
hyperbilirubinemia bilirubin urobilinogen K+: 4.5 mEq/L
conjugated normal
Cl-: 95 mEq/L
A
HCO3-: 9 mEq/L
conjugated
B
Glucose: 40 mEq/L
C unconjugated
Serum pH: 6.8
D unconjugated Lactate: 9.5 mmoL/L
E unconjugated
What metabolic process induced this patient’s
current condition?
Reproduced, with permission, from USMLERx.com.
(A) Decreased levels of glycerol 3-phosphate
(B) Elevated pyruvate levels
(A) A (C) Inappropriate induction of gluconeogene-
(B) B sis

Biochemistry
(C) C (D) Overproduction of reduced nicotinamide
(D) D adenine dinucleotide
(E) E (E) Thiamine deficiency

40. A 3-year-old boy recently developed weak- 42. A 78-year-old man with asthma presents to his
ness of his lower extremity and uses his arms primary care physician for an annual check-
to stand up even though his lower legs appear up. The physician performs a physical exami-
quite muscular. Laboratory tests reveal a cre- nation and orders routine blood work, which
atine kinase level of 20,000 U/L. A DNA test reveals a macrocytic anemia. Subsequent
confirms the working diagnosis. What is the laboratory tests show an elevated serum meth-
function of the altered gene product in this pa- ylmalonic acid level. A peripheral blood smear
tient? is shown in the image. If this patient’s vitamin
(A) Exocytosis of acetylcholine at the neuro- deficiency is not corrected, what neurological
muscular junction symptoms is he most likely to experience?
(B) Linking actin filaments to laminin
(C) Promoting actin-myosin cross-bridge cy-
cling
(D) Receptor for acetylcholine
(E) Release of calcium from the sarcoplasmic
reticulum

41. A 55-year-old man is found unresponsive and


breathing rapidly in his apartment. His daugh-
ter found him while stopping by to visit. She
stated that she was concerned after he told her
on the phone that he was “drowning his sor-
rows,” having been fired from his job earlier
that day. He has no significant medical history Reproduced, with permission, from USMLERx.com.
other than moderate hypertension, for which
he takes a b-blocker. Relevant laboratory find-
(A) Confusion and confabulation
ings are:
(B) Deficiency in this vitamin does not cause
neurological symptoms
(C) Dysarthria and diplopia
(D) Paresthesias and ataxia
(E) Syncope and lethargy
High-Yield Principles
28 Section I: General Principles  •  Questions

43. A young woman currently being treated for is notable only for the ocular findings shown
HIV is brought to the emergency department in the image. Deficiency of which enzyme is
because of a headache and cyanosis of her nail responsible for this disease?
beds and lips. She also reports feeling dizzy.
The resident on call immediately places her
on supplemental oxygen and draws blood for
arterial blood gas analysis. While drawing the
blood, he notes that the arterial blood has a
dark brown color. Blood gas analysis reveals a
pH of 7.39, partial oxygen pressure of 96 mm
Hg, partial carbon dioxide pressure of 35 mm
Hg, and oxygen saturation of 82% on room air.
What enzyme is primarily responsible for pre-
venting this condition in the normal adult?
Biochemistry

(A) ATPase
(B) Flavin adenine dinucleotide reductase
(C) GTPase Reproduced, with permission, from USMLERx.com.
(D) Lactase
(E) Nicotinamide adenine dinucleotide reduc-
tase (A) a-Galactosidase A
(F) Pyruvate kinase (B) Arylsulfatase A
(C) Hexosaminidase A
44. A mass is felt in the groin of an infant girl dur- (D) Iduronate sulfatase
ing a physical examination. Surgical resection (E) Lysyl hydrolase
shows that it is a testicle. The baby is diag-
nosed with testicular feminization syndrome. 46. A pediatrician examines a baby with a defi-
In this syndrome, androgens are produced but ciency in fructose metabolism. Upon adminis-
cells fail to respond to the steroid hormones tration of a fructose bolus, the child becomes
because they lack appropriate intracellular re- symptomatic and blood glucose levels begin to
ceptors. After binding intracellular receptors, decrease. Which of the following will also oc-
steroids regulate the rate of which of the fol- cur after the administration of a fructose bolus
lowing? in this patient?

(A) Initiation of protein synthesis (A) A fall in serum phosphate levels


(B) mRNA degradation (B) A rise in cellular ATP levels
(C) mRNA processing (C) A sustained rise in serum fructose levels
(D) Protein translation (D) An increase in the serum pH
(E) Transcription of genes (E) Large amounts of fructose in the urine

45. The wife in an Ashkenazi Jewish family brings


her 1-year-old daughter to the pediatrician.
Her previous pregnancy was uneventful and
resulted in a full-term healthy girl who is now
4 years old. Her younger daughter, however,
has demonstrated a progressive series of behav-
iors over the first year of life. Her motor skills
have diminished and she demonstrates an in-
creased startle reaction. Physical examination
High-Yield Principles
Chapter 2: Biochemistry  •  Questions 29

47. A 28-year-old African-American man is re- 50. A 28-year-old woman is trying to conceive a
ceiving primaquine therapy for treatment of child. She has a nephew with fragile X syn-
malaria, which he contracted while visiting drome (a genetic disorder characterized by
Asia. He presents to his physician after noting trinucleotide repeat expansion) and she would
blood in his urine. Physical examination is sig- like to assess her risk as a carrier for the disease.
nificant for scleral icterus, and urinalysis shows Blood is drawn, and DNA is extracted and cut
hemoglobinuria. A peripheral blood smear by restriction enzymes that flank the CGG
shows spherocytes, bite cells, and Heinz bod- repeat region. The DNA is then treated with
ies. Which of the following is the most likely a labeled probe that binds the affected region
diagnosis? of the gene. The woman is found to carry one
normal X chromosome and one X chromo-
(A) Alkaptonuria
some with some expansion of the CGG se-
(B) Cystinuria
quence. However, the number of CGG repeats
(C) Glucose-6-phosphate dehydrogenase defi-
in this X chromosome is not sufficient to alter
ciency

Biochemistry
phenotype. Which lane on the Southern blot
(D) Hereditary fructose intolerance
represents this woman’s genotype?
(E) Hereditary spherocytosis
(F) Lactase deficiency
Affected Unaffected
48. A neonate born at 28 weeks’ gestation is having nephew husband A B C D E
difficulty breathing. On physical examination, _
the neonate’s heart rate is 120/min, blood pres-
sure is 100/60 mm Hg, and respiratory rate is
55/min. He has nasal flaring and subcostal re-
tractions. Which of the following components
+
is deficient in this infant?
(A) Dipalmitoyl phosphatidylcholine Reproduced, with permission, from USMLERx.com.
(B) Elastase
(C) Functional cilia
(D) Phosphatidylglycerol (A) A
(E) Sphingomyelin (B) B
(C) C
49. A group of scientists at a pharmaceutical com- (D) D
pany are conducting in vitro experiments to in- (E) E
vestigate the effects of an antineoplastic drug.
Under the microscope, it appears that with
treatment, the majority of the cells are arrested
at a stage in which their chromosomes are
aligned in the vertical axis of the cells. Which
antineoplastic agent has a mechanism of ac-
tion similar to the one described?
(A) 5-Fluorouracil
(B) Cyclophosphamide
(C) Etoposide
(D) Methotrexate
(E) Vincristine
High-Yield Principles
30 Section I: General Principles  •  Answers

An s w e r s

1. The correct answer is G. This patient has xe- Answer E is incorrect. Succinyl-CoA down-
roderma pigmentosa, an autosomal recessive regulates its own synthesis by inhibiting the
disease characterized by a defect in excision enzyme responsible for dehydrogenation of
repair. This disease results in an inability to a-ketoglutarate.
repair thymidine dimers that can form in the
presence of ultraviolet light. This can lead to 3. The correct answer is E. Characteristic symp-
the development of skin cancer and photosen- toms of hyperthyroidism include tachycardia,
sitivity. heat intolerance, weight loss, weakness, tremu-
lousness, and diarrhea. This patient also displays
Answer A is incorrect. Methylation of a partic-
another symptom of elevated thyroid hormone
ular gene does not cause xeroderma pigmento-
levels, exophthalmos. Thyroid hormone enters
sum.
Biochemistry

target cells through carrier-mediated transport


Answer B is incorrect. A missense mutation or possibly diffusion, and binds to nuclear re-
does not cause xeroderma pigmentosum. ceptors. The hormone-receptor complex then
binds DNA and acts as a transcription factor,
Answer C is incorrect. A nonsense mutation
regulating the transcription of genes. Tran-
does not cause xeroderma pigmentosum.
scription results in the production of RNA,
Answer D is incorrect. A mutation in the en- specifically messenger RNA.
hancer region of a gene does not cause xero-
Answer A is incorrect. Carbohydrate is pro-
derma pigmentosum.
duced by the joining of sugar molecules dur-
Answer E is incorrect. A mutation in the oper- ing cellular metabolism; for example, glycogen
ator region of a gene does not cause xeroderma is produced from the linkage of glucose mol-
pigmentosum. ecules by glycogen synthase.
Answer F is incorrect. A mutation in the pro- Answer B is incorrect. DNA binding occurs
moter region of a gene does not cause xero- when the thyroid hormone molecule enters
derma pigmentosum. the nucleus and binds on nuclear receptors.
However, this leads to the transcription of
2. The correct answer is B. Citrate, formed from RNA from DNA, not production of new DNA
oxaloacetate and acetyl CoA by the enzyme or DNA replication.
citrate synthase, inhibits phosphofructokinase
and allosterically activates acetyl CoA carbox- Answer C is incorrect. Fatty acid is synthe-
ylase. Citrate synthase regenerates a molecule sized from acetyl-coenzyme A (CoA) in the
of CoA and is an important regulator of the tri- cell cytoplasm via the action of acetyl-CoA car-
carboxylic acid cycle. It is inhibited by adeno­ boxylase and other enzymes.
sine triphosphate. Answer D is incorrect. Protein may be pro-
Answer A is incorrect. a-Ketoglutarate is not duced from the RNA template but is not the
an important regulator of the tricarboxylic acid first molecule produced in response to thyroid
cycle, but it is an important intermediate in hormone.
protein metabolism.
4. The correct answer is C. The patient has al-
Answer C is incorrect. Malate is not an impor- kaptonuria, a condition corresponding to the
tant regulator of the tricarboxylic acid cycle, one described in the stem. A deficiency of the
but it is important in the malate shuttle. enzyme homogentisic acid oxidase leads to de-
Answer D is incorrect. Oxaloacetate is not an position of homogentisic acid in the joints and
important regulator of the tricarboxylic acid cartilage, giving them a dark color (ochronosis)
cycle, but it is important in glyconeogenesis. and resulting in degenerative changes. Clas-
High-Yield Principles
Chapter 2: Biochemistry  •  Answers 31

sically, the urine of these patients turns black sickle cell anemia. The b-A-globin gene re-
on contact with air or when the urine is made sults in a 1.15-kb fragment of DNA cut by the
alkaline. The associated defect is on chromo- MstII restriction enzyme. The b-S-globin gene
some 3. results in a 1.35-kb band because the single
base-pair mutation responsible for sickle cell
Answer A is incorrect. A deficiency in a-keto-
anemia eliminates an MstII restriction site. A
acid dehydrogenase causes maple syrup urine
heterozygote will have two bands indicating
disease, a metabolic disorder of autosomal re-
one normal allele with an intact MstII site
cessive inheritance that affects the metabo-
(two fragments), and a mutant allele with a
lism of branched-chain amino acids (leucine,
missing MstII site (one fragment).
isoleucine, and valine) and causes the urine
of affected patients to smell like maple syrup. Answer A is incorrect. The band in lane A is
The urine does not, however, turn black upon from a sickle cell anemia patient with two cop-
standing. The disease is not classically associ- ies of the b-S-globin gene. This gene results
ated with arthritis in middle-aged individuals. in a 1.35-kb band because the single base-pair

Biochemistry
mutation responsible for sickle cell anemia
Answer B is incorrect. A deficiency in galac-
eliminates an MstII restriction site.
tokinase causes galactosemia and galactosuria,
but is otherwise a fairly benign condition and Answer C is incorrect. The band in lane C
would not present with any of the symptoms is from an unaffected patient with two copies
seen in this patient. Other symptoms would of the b-A-globin gene. The gene results in a
be cataracts in affected children, owing to the 1.15-kb fragment of DNA cut by the MstII re-
accumulation of galactitol, a by-product of ga- striction enzyme.
lactose metabolism when galactokinase is not
Answer D is incorrect. The bands in lane D
present.
could not result from any patient. The labeled
Answer D is incorrect. Orotate phosphoribo­ DNA probe does not bind to the 0.2-kb DNA
syltransferase is an enzyme involved in pyrimi- fragment and therefore would not be visual-
dine synthesis. Deficiencies in this enzyme or ized on the Southern blot.
in orotidine 5′-monophosphate decarboxylase
Answer E is incorrect. The bands in lane E
(an enzyme involved in the same pathway
could not result from any patient. The labeled
and located on the same chromosome) cause
DNA probe does not bind to the 0.2-kb DNA
a very rare disorder called hereditary orotic ac-
fragment and therefore would not be visual-
iduria. Symptoms include poor growth, mega-
ized on the Southern blot.
loblastic anemia, and orate crystals in urine.
Treatment involves cystidine or uridine to by- Answer F is incorrect. The bands in lane F
pass this step in pyrimidine synthesis and also could not result from any patient. The labeled
to negatively downregulate orotic acid produc- DNA probe does not bind to the 0.2-kb DNA
tion. fragment and therefore would not be visual-
ized on the Southern blot. A heterozygote will
Answer E is incorrect. A congenital deficiency
have two bands indicating one normal allele
of phenylalanine hydroxylase causes phenyl-
with an intact MstII site (two fragments), and
ketonuria. This enzyme converts phenylala-
a mutant allele with a missing MstII site (one
nine to tyrosine, and a deficit of this enzyme
fragment).
leads to a deficiency of tyrosine and a build-up
of phenylketones in the urine. It is associated 6. The correct answer is B. This patient was
with mental retardation and with the presence started on an 3-hydroxy-3-methylglutaryl co-
of phenylketones in the urine (which do not enzyme A (HMG CoA) reductase inhibi-
classically turn black upon standing). tor (statin), which prevents the conversion of
HMG CoA to mevalonic acid, the rate-limiting
5. The correct answer is B. Lane B represents
step in cholesterol biogenesis. HMG CoA is
the Southern blot of a heterozygous carrier of
High-Yield Principles
32 Section I: General Principles  •  Answers

formed from three acetyl CoA molecules and the DNA more than a million times. Dideoxy-
is a precursor to sterols and ketone bodies. nucleotides are not used in PCR techniques.
Muscle pain or injury resembling myositis has
Answer B is incorrect. Allele-specific oligo-
been known to occur with statins. Although
nucleotide probes are short, labeled DNA
the mechanism is unknown, it may be related
sequences complementary to an allele of in-
to a decrease in muscle tissue synthesis of
terest. These probes can be used to detect the
ubiq­uinone, a coenzyme used in muscle cell
presence of disease-causing mutations.
metabolism.
Answer D is incorrect. In a Southern blot pro-
Answer A is incorrect. Another common side
cedure, DNA is separated with electrophoresis,
effect of statins is transient elevation of trans-
denatured, transferred to a filter, and hybrid-
aminases. Alanine aminotransferase and aspar-
ized with a labeled DNA probe. Regions on
tate aminotransferase elevations are usually
the filter that base-pair with the labeled DNA
seen within 12 weeks after the onset of therapy
probes can be identified when the filter is ex-
and may be persistent. No studies have demon-
Biochemistry

posed to film that is sensitive to the radiola-


strated an adverse affect of this transaminitis.
beled probe.
Hepatomegaly, however, is not a known side
effect. Answer E is incorrect. In a Northern blot
procedure, RNA is separated by electrophore-
Answer C is incorrect. The teratogenicity of
sis, denatured, and transferred to a filter. RNA
statins is unknown. Fetal toxicity has been
is hybridized to a labeled radioactive DNA
demonstrated at high enough concentrations
probe. The hybridized RNA/DNA strand is ra-
to adversely affect the mother, but the conse-
dioactive and visualized when the filter is ex-
quence of standard doses is unknown. Statins
posed to film.
have not been shown to induce spontaneous
abortions. Answer F is incorrect. In a Western blot pro-
cedure, protein is separated by electrophore-
Answer D is incorrect. Statins have also been
sis and labeled antibodies are used as a probe.
implicated as causes of irritability and depres-
This technique can be used to detect the exis-
sion, specifically in patients with major depres-
tence of an antibody to a particular protein.
sion, but this relationship has not been con-
firmed. Homicidality is not caused by statins. Answer G is incorrect. Enzyme-linked immu-
nosorbent assay (ELISA) is an immunologic
Answer E is incorrect. Seizures are not a com-
technique used to determine whether a par-
mon side effect of statin therapy.
ticular antibody is present in a patient’s blood.
Labeled antibodies are used to detect whether
7. The correct answer is C. Sequencing is a
the serum contains antibodies against a spe-
laboratory technique that uses dideoxynucleo-
cific antigen precoated on an ELISA plate.
tides to randomly terminate growing strands of
DNA. Gel electrophoresis is used to separate
8. The correct answer is B. This patient has
the varying lengths of DNA. The DNA se-
a1-antitrypsin deficiency, a genetic disease
quence can then be read based on the position
characterized by a deficiency in the serine
of the bands on the gel.
protease inhibitor a1-antitrypsin. This protein
Answer A is incorrect. Polymerase chain reac- normally functions to inhibit neutrophil elas-
tion (PCR) is a laboratory technique used to tase in the lung. When deficient, there is over-
produce many copies of a segment of DNA. In abundant activity of elastase, which destroys
the procedure, DNA is mixed with two specific elastin and collagen in the alveolar walls, pro-
primers, deoxynucleotides and a heat-stable gressing to emphysema. Most patients with
polymerase. The solution is heated to denature a1-antitrypsin deficiency are homozygous for
the DNA and then cooled to allow synthesis. the Z allele. Clinically, a1-antitrypsin defi-
Twenty cycles of heating and cooling amplify ciency can affect the lung, liver, and less com-
High-Yield Principles
Chapter 2: Biochemistry  •  Answers 33

monly the skin. In the lung, the most com- radiographic findings are more consistent with
mon manifestation is early onset panacinar emphysema.
emphysema, which is more prominent at the
lung bases than apices. Slowly worsening dysp­ 9. The correct answer is A. Calcium is main-
nea is the most common symptom, although tained in high concentrations outside of the
patients may initially complain of cough, spu- cell and in discrete compartments within the
tum production, or wheezing. As in this case, cell (eg, in mitochondria). Free intracellu-
patients who present early complaining of lar calcium can activate several enzymes the
episodes of wheezing and productive cough cumulative effect of which is to induce sig-
may be told they have asthma. Although treat- nificant cell injury. A few important enzyme
ment for asthma may initially improve symp- classes include ATPases, which decrease the
toms, it does not slow the progression of the ATP supply; phospholipases, which decrease
disease. Her x-ray of the chest shows a pattern membrane stability; endonucleases, which in-
typical for this disease; hyperinflated lungs, a duce DNA damage; and several proteases, re-

Biochemistry
flattened diaphragm, and hyperlucent lungs sponsible for protein breakdown.
due to decreased lung markings (it is difficult Answer B is incorrect. Free radical generation
to see at this resolution due to the overlying is a common mechanism of cell injury, but
breast tissue, but we expect that the lung mark- calcium excess does not induce free radical
ings would be especially absent at the bases). generation.
a1-Antitrypsin deficiency can also cause cir-
rhosis of the liver and panniculitis of the skin. Answer C is incorrect. Activation of proteases
and phospholipases induces the breakdown of
Answer A is incorrect. a1-Antitrypsin defi- necessary components of cell membranes.
ciency is characterized by low levels of a prote-
ase inhibitor. This leads to elevated activity of Answer D is incorrect. ATP depletion, result-
the protease elastase and increased destruction ing from the activation of ATPases, can con-
of elastin. tribute to the inhibition of glycolysis.
Answer C is incorrect. Mutations in the tu- Answer E is incorrect. Inhibition of oxidative
mor suppressor gene p53 lead to uncontrolled phosphorylation is an effect of ATP depletion
cellular proliferation. Such mutations are caused by enzyme activation. Although this
commonly seen in the lung cancers associated may contribute to cell damage, it is not the
with smoking (small cell and squamous cell), best answer. The enzyme activation resulting
and they have been found in many non-small from calcium excess is the root cause of the
cell types. This patient’s history and x-ray find- cell damage, and thus would be the primary
ings do not suggest cancer. insult responsible for the majority of cell dam-
age.
Answer D is incorrect. Mutations in the gene
encoding the cystic fibrosis transmembrane 10. The correct answer is B. G-protein coupled
conductance regulator (CFTR) on chromo- receptors exist in an equilibrium between their
some 7 lead to the disease cystic fibrosis (CF). active and inactive states that is dependent on
CF is a multisystem disease that affects the res­ whether ligand is present, and the affinity of the
piratory tract, digestive tract, sweat glands, and ligand for the receptor. When active, these re-
reproductive tract. This patient does not pre­ ceptors catalyze guanine-nucleotide exchange
sent with symptoms characteristic of CF. (GTP for guanosine diphosphate) of their asso-
Answer E is incorrect. Airway inflamma- ciated G proteins. The Michaelis-Menten con-
tion, airway obstruction, and bronchial hyper- stant (Km) for any enzyme-catalyzed reaction
responsiveness are characteristic of asthma. is inversely proportional to the affinity of the
While this patient’s presentation mimicked enzyme for its substrate. Therefore, the Km for
that of asthma, her history, physical exam, and compound A will be lower than that for com-
pound B because compound A has a higher
High-Yield Principles
34 Section I: General Principles  •  Answers

affinity for the receptor than compound B. porphobilinogen. Lack of this enzyme causes
The maximum rate of reaction (Vmax) will be porphobilinogen and δ-aminolevulinic acid to
reached at a lower concentration of A than it accumulate in the urine. Patients with acute
would for B, although the Vmax is unchanged. intermittent porphyria are not photosensitive,
but they do experience symptoms of painful
Answer A is incorrect. The Km of compound
abdomen, polyneuropathy, and psychological
A will be lower than that of compound B.
disturbances. They also have pink coloration of
Answer C is incorrect. Given that compounds their urine.
A and B have different affinities for the recep-
Answer D is incorrect. Ferrochelatase and
tor, their Michaelis-Menten constant values
δ-aminolevulinic acid (ALA) dehydrase are
cannot be the same.
sensitive to inhibition by lead. Thus, lead poi-
Answer D is incorrect. Vmax is directly propor- soning leads to an accumulation of copropor-
tional to the enzyme concentration, and is un- phyrin and ALA in the urine. Lead poisoning
affected by the concentration of substrates or is a problem seen in children who live in old
Biochemistry

competitive inhibitors. houses with chipped paint (lead was used in


Answer E is incorrect. Vmax is directly propor- paint manufacturing until the 1970s). Inges-
tional to the enzyme concentration, and is un- tion of large quantities of lead can cause lines
affected by the concentration of substrates or on the gingiva and epiphyses of long bones,
competitive inhibitors. encephalopathy, erythrocyte basophilic stip-
pling, abdominal colic, sideroblastic ane-
11. The correct answer is C. This individual suf- mia, and neuropathy leading to foot and wrist
fers from porphyria cutanea tarda. The por- drops. It is not associated with photosensitivity.
phyrias are diseases resulting from enzymatic The first line of treatment is dimercaprol and
deficiencies in heme biosynthesis, and por- EDTA to bind up the free lead in the serum.
phyria cutanea tarda is the most common Answer E is incorrect. Porphobilinogen deam-
form. This disorder is caused by deficiency of inase deficiency, not excess, results in acute in-
uroporphyrinogen decarboxylase, the hepatic termittent porphyria. One would expect to find
enzyme that catalyzes the formation of copro- δ-aminolevulinic acid and porphobilinogen in
porphyrinogen III from uroporphyrinogen III. the urine and no photosensitivity.
Lack of this enzyme results in uroporphyrin
accumulation in the urine (giving the urine 12. The correct answer is B. This woman suffers
a tea-colored appearance) and uroporphy- from b-thalassemia major, the most severe
rinogen accumulation systemically. This com- form of b-thalassemia, in which the b-chain is
pound absorbs light and releases heat, causing absent. Clinically b-thalassemia major mani-
extreme photosensitivity. fests as severe hemolysis and ineffective eryth-
Answer A is incorrect. Hereditary copropor- ropoiesis. These individuals are transfusion
phyria is a disease due to the deficiency of dependent and frequently develop iron over-
coproporphyrinogen oxidase, the enzyme that load. The consequences of iron overload due
catalyzes the formation of protoporphyrinogen to transfusion dependency or secondary hemo-
from coproporphyrinogen III. Coproporphy- chromatosis are described in the stem. These
rinogen III accumulates in the urine. Patients manifestations are due to iron deposition in
with this condition tend to be photosensitive. various tissues including the pancreas, heart,
and skin. b-Thalassemia is more common
Answer B is incorrect. Acute intermittent por- among Mediterranean populations, whereas
phyria is caused by a deficiency in porphobi- b-thalassemia is more common among Asian
linogen deaminase (also called uroporphyrino- and African populations.
gen 1 synthetase), the enzyme that catalyzes
the formation of pre-uroporphyrinogen from Answer A is incorrect. This answer describes
the most severe form of a-thalassemia, a disease
High-Yield Principles
Chapter 2: Biochemistry  •  Answers 35

in which a fetus is unable to make any func- drug is currently approved to inhibit aldose re-
tional hemoglobin aside from the γ4-tetramer ductase, but aldose reductase inhibitors such
(Hb Bart). Clinically a-thalassemia manifests as epalrestat and ranirestat are currently being
as congestive heart failure, anasarca, and intra­ tested.
uterine fetal death.
Answer A is incorrect. 3-Hydroxy-3-methyl-
Answer C is incorrect. This answer describes glutaryl coenzyme A (HMG CoA) reductase
hereditary hemochromatosis, a disease caused catalyzes the conversion of HMG CoA into
by iron overload due to an intrinsic defect in mevalonate and eventually into cholesterol. In-
the body’s ability to control the absorption of hibition of this enzyme is commonly affected
iron. Clinically this disease manifests in a man- by statin drugs to reduce cholesterol levels, but
ner similar to that of secondary hemochroma- it would not help prevent the development of
tosis. However, the laboratory picture in hered- cataracts.
itary hemochromatosis is not characterized by
Answer B is incorrect. Adenosine deaminase
hemolysis.

Biochemistry
inhibition would result in problems in the pu-
Answer D is incorrect. This answer describes rine salvage pathway. Disrupting this pathway
hereditary spherocytosis, a disease in which would result in excess ATP and dATP via feed-
mutations in either the ankyrin or spectrin back inhibition of ribonucleotide reductase.
gene contribute to instability of the RBC This excess ATP prevents DNA synthesis and
plasma membranes. This condition is charac- thus affects lymphocyte development. Congen-
terized by extravascular hemolysis. Clinically ital deficiency of this enzyme results in severe
this disease manifests as gallstones, anemia, combined immunodeficiency. Inhibition of
jaundice, and splenomegaly. The definitive this enzyme would not prevent the develop-
treatment is splenectomy, thus obviating any ment of cataracts.
need for chronic blood transfusion.
Answer D is incorrect. Galactose-1-phosphate
Answer E is incorrect. This answer describes (G-1-P) uridyltransferase is important in the
Wilson disease, a disease in which failure of breakdown of galactose; it catalyzes the for-
copper to enter the circulation in the form of mation of glucose-1-phosphate from G-1-P.
ceruloplasmin results in copper accumulation Hereditary deficiency of this enzyme leads to
in the liver, brain, and cornea. Clinically this hepatosplenomegaly, mental retardation, jaun-
disease manifests as parkinsonian symptoms, dice, and cataract formation. Inhibition of this
Kayser-Fleischer rings, asterixis, and dementia. enzyme in an adult would certainly not pre-
vent the development of cataracts.
13. The correct answer is C. Aldose reductase
Answer E is incorrect. Hexokinase is the en-
catalyzes the breakdown of glucose into sorbi-
zyme that catalyzes the first step in the catab-
tol. Sorbitol is then metabolized to fructose, a
olism of glucose, converting glucose to glu-
process that is relatively slow. In patients with
cose-6-phosphate. It is stimulated by insulin.
hyperglycemia, as would be present in this pa-
Inhibition of hexokinase would not prevent the
tient with poorly controlled diabetes, sorbitol
development of cataracts in this patient. Con-
accumulation with the cells of the lens leads
genital hexokinase deficiency is a rare autoso-
to a rise in intracellular osmolality, causing
mal recessive condition that results in severe
water movement into the cells. This results in
hemolysis. Inhibition of hexokinase would
cellular swelling and osmotic damage. It also
likely have a similar, albeit less severe, result.
leads to a decrease in intracellular myoinositol,
interfering with cellular metabolism. Swelling Answer F is incorrect. Insulin-like growth fac-
of lens fiber cells can lead to rupture and cata- tor (IGF) is a product synthesized in the liver
ract formation. Inhibition of aldose reductase that mediates many of the physiologic effects
could decrease sorbitol accumulation in the of growth hormone (GH). Its name refers to a
lens and thus prevent cataract formation. No high degree of structural similarity to insulin,
High-Yield Principles
36 Section I: General Principles  •  Answers

and it is even capable of binding to the insu- Answer G is incorrect. Tyrosine kinase prod-
lin receptor directly, although with lower affin- ucts play a role in cell signaling through phos-
ity than insulin. Its effects include increased phorylation. The p53 gene product is not a ty-
protein synthesis, and IGF levels are especially rosine kinase.
high during puberty. Inhibition of IGF would
not help prevent the development of cataracts. 15. The correct answer is D. This question de-
scribes the polymerase chain reaction (PCR).
14. The correct answer is F. The p53 gene protein PCR is a laboratory technique used to produce
product is a transcription factor that regulates many copies of a segment of DNA. In the pro-
apoptosis. It acts as a cell-cycle regulator, pre- cedure, DNA is mixed with two specific prim-
venting cells from undergoing division. Muta- ers, deoxynucleotides and a heat-stable poly-
tions in p53 cause uncontrolled cell division, merase. The solution is heated to denature
leading to various types of tumors. Another the DNA and then cooled to allow synthesis.
example of a cell-cycle regulator is the retino- Twenty cycles of heating and cooling amplify
Biochemistry

blastoma gene. the DNA over a million times.


Answer A is incorrect. The p53 gene product Answer A is incorrect. Enzyme-linked immu-
is involved in apoptosis induced by DNA dam- nosorbent assay (ELISA) is an immunologic
age and other stimuli, but it is not a caspase technique used to determine whether a par-
protein. ticular antibody is present in a patient’s blood.
Labeled antibodies are used to detect whether
Answer B is incorrect. p53 is involved in cell
the serum contains antibodies against a spe-
cycle regulation but not direct DNA repair ac-
cific antigen precoated on an ELISA plate.
tivity. DNA repair products are produced by
This is not the technique described above.
genes such as BRCA1 (chromosome 17) and
BRCA2 (chromosome 13), among others. Answer B is incorrect. Gel electrophoresis
uses an electric field to separate molecules
Answer C is incorrect. Membrane cell adhe-
based on their sizes.
sion products are produced by the APC gene
found on chromosome 5. Mutations in the Answer C is incorrect. Northern blots are sim-
APC gene lead to colon cancer. ilar to Southern blots except that in Northern
blotting, mRNA is separated by electrophoresis
Answer D is incorrect. p53 is not a G-protein
instead of DNA. This is not the technique de-
product. G proteins such as ras can be in-
scribed above.
volved in oncogenesis. Mutations in ras can
lead to cancer in the lungs, pancreas, and co- Answer E is incorrect. Sequencing is a labo-
lon as well as leukemia. ratory technique that utilizes dideoxynucleo-
tides to randomly terminate growing strands of
Answer E is incorrect. DNA polymerase is un-
DNA. Gel electrophoresis is used to separate
able to replicate at the end of chromosomes
the varying lengths of DNA. The DNA se-
(telomeres), resulting in the loss of DNA with
quence can then be read based on the position
each replication cycle. Telomerase is an en-
of the bands on the gel. This is not the tech-
zyme that adds repeats onto the 3′ends of chro-
nique described above.
mosomes to protect them from being recog-
nized as broken or damaged DNA. Although Answer F is incorrect. In a Southern blot pro-
most normal somatic cells do not express cedure, DNA is separated with electrophoresis,
enough telomerase to prevent telomerase attri- denatured, transferred to a filter, and hybrid-
tion with each cell division, telomerase is often ized with a labeled DNA probe. Regions on
reexpressed in cancer cells. However, p53 is the filter that base-pair with the labeled DNA
not involved in the process of adding nucleo- probes can be identified when the filter is
tides to telomeres. exposed to film that is sensitive to the radio-
High-Yield Principles
Chapter 2: Biochemistry  •  Answers 37

labeled probe. This is not the technique de- enzymes that convert the D-ala D-ala peptide
scribed above. bridge to D-ala D-lac, preventing vancomycin
from binding. Vancomycin resistance is much
Answer G is incorrect. In a Western blot pro-
more common with Enterococcus faecium than
cedure, protein is separated by electrophore-
with Enterococcus faecalis. High-dose ampicil-
sis and labeled antibodies are used as a probe.
lin, often in combination with gentamicin, is
This technique can be used to detect the exis-
generally first-line treatment in urinary tract
tence of an antibody to a particular protein.
infections due to vancomycin-resistant Entero-
16. The correct answer is B. This woman has coccus.
symptoms of Hashimoto thyroiditis, an auto- Answer A is incorrect. Microorganisms be-
immune disorder resulting in hypothyroidism come resistant to quinolones through the al-
(also known as myxedema), although there teration of their gyrase.
may be a transient hyperthyroidism at the very
Answer B is incorrect. Microorganisms be-
onset of disease when follicular rupture occurs.

Biochemistry
come resistant to macrolides through the
It is a type IV hypersensitivity associated with
methylation of its rRNA at a ribosome-binding
autoantibodies to thyroglobulin, thyroid per-
site.
oxidase, and the thyroid-stimulating hormone
receptor itself. The most common presenting Answer C is incorrect. b-Lactamases are en-
symptoms of Hashimoto thyroiditis are those zymes produced by microorganisms that cleave
seen in this patient, as well as constipation b-lactam antibiotics, deactivating them. To
and dry skin. Histologic characteristics include overcome resistance, b-lactams are usually
massive infiltrates of lymphocytes with germi- given with b-lactamase inhibitors such as cla-
nal cell formation. Hashimoto thyroiditis is as- vulanic acid, tazobactam, and sulbactam.
sociated with the DR5 human leukocyte anti-
Answer E is incorrect. b-Lactam antibiotics
gen subtype, as is pernicious anemia, a disease
bind to penicillin-binding proteins (enzymes
that leads to vitamin B12 deficiency caused by
that synthesize peptidoglycan, a major com-
atrophic gastritis and destruction of parietal
ponent of bacterial cell walls), preventing cell
cells.
wall synthesis by the microorganism. Microor-
Answer A is incorrect. Multiple sclerosis is as- ganisms such as methicillin-resistant Staphy-
sociated with the DR2 human leukocyte an- lococcus aureus and penicillin-resistant Strep-
tigen subtype. It is not associated with Hashi- tococcus pneumoniae have alterations in their
moto thyroiditis. penicillin-binding proteins that result in low
affinity and thus resistance to these b-lactams.
Answer C is incorrect. Psoriasis is associated
b-Lactam antibiotics include penicillins, ceph-
with the B27 human leukocyte antigen sub-
alosporins, monobactams, and carbapenems
type. It is not associated with Hashimoto thy-
(not vancomycin).
roiditis.
Answer D is incorrect. Steroid-responsive ne- 18. The correct answer is C. Kartagener syn-
phrotic syndrome is associated with the DR7 drome, or immotile cilia, is caused by a defect
human leukocyte antigen subtype. It is not as- in dynein that prevents effective movement of
sociated with Hashimoto thyroiditis. cilia. The full syndrome is characterized by si-
nusitis, bronchiectasis, situs inversus, and male
17. The correct answer is D. Vancomycin is a gly- infertility. Cilia play an important role in mov-
copeptide antibiotic that is effective in fighting ing mucus along the airway and clearing de-
only gram-positive bacteria. It binds tightly to bris; the absence of this function contributes to
a cell wall precursor that contains the termi- the pulmonary findings of the syndrome. Cilia
nal amino acid sequence D-ala D-ala and pre- are also very important for leukocyte move-
vents cell wall synthesis. Resistance to vanco- ment and phagocytosis. Infertility is present in
mycin is transferred via plasmids and encodes most patients due to immotile cilia.
High-Yield Principles
38 Section I: General Principles  •  Answers

Answer A is incorrect. Defective chloride opacities, acroparesthesias, and vascular dis-


transport is the cause of cystic fibrosis. Cystic ease of the kidney, heart, and brain. Labora-
fibrosis frequently causes bronchiectasis, but it tory results show diminished a-galactosidase A
is not associated with situs inversus. activity in plasma, leukocytes, or cultured fi-
broblasts. Enzyme replacement therapy is now
Answer B is incorrect. Patients with diabetes
available for patients, and renal transplant and
are predisposed to developing chronic fungal
long-term hemodialysis are mainstays of treat-
sinusitis. However, the bronchiectasis and situs
ment.
inversus are not consistent with diabetes.
Answer B is incorrect. Hereditary fructose in-
Answer D is incorrect. Mucus plugging in
tolerance is caused by the inability of aldose B
reactive airway disease can cause atelectasis
to split fructose 1-phosphate, resulting in its
at the lung bases. An x-ray film of the chest of
accumulation along with inhibition of glucose
a patient with reactive airway disease would
production. Patients are usually asymptom-
likely reveal hyperinflated lungs with areas of
atic until they begin ingesting food contain-
Biochemistry

atelectasis, not bronchiectasis.


ing fructose, sucrose, or sorbitol after weaning
Answer E is incorrect. Tetralogy of Fallot is a from breastfeeding. Symptoms include nausea,
congenital heart defect, but it is not associated vomiting, pallor, sweating, and trembling with
with infections or cardiac inversion. Patients fructose ingestion; continued ingestion can
with this condition develop early cyanosis be- lead to seizure and coma.
cause of the malformed right-to-left shunt. The
Answer C is incorrect. Absent function of ga-
four components of the tetralogy are (1) ven-
lactose-1-phosphate uridylyltransferase in galac-
tricular septal defect, (2) overriding aorta, (3)
tosemia results in the accumulation of galactose
infundibular pulmonary stenosis, and (4) right
and galactose-1-phosphate. Galactose-1-phos-
ventricular hypertrophy.
phate has direct toxic effects on renal, hepatic,
19. The correct answer is E. This child is suffer- and neuronal cells. The disorder is character-
ing from an inherited form of hyperammone- ized by onset of clinical symptoms within the
mia as a result of a defect in ornithine trans- first few days of life: vomiting, diarrhea, failure
carbamylase (OTC). In the urea cycle, OTC to thrive, and hypotonia. Patients who undergo
combines carbamoyl phosphate and ornithine early galactose restriction may still have devel-
to make citrulline. When OTC is deficient, opmental delays, ataxia, and apraxia. Labora-
excess carbamoyl phosphate enters the py- tory findings include an elevated blood galac-
rimidine synthesis pathway to cause increased tose level, low glucose, and galactosuria.
orotic acid in the blood, which distinguishes Answer D is incorrect. Pompe disease is
OTC deficiency from other urea cycle dis- caused by absent function of lysosomal
orders. A defect in OTC causes an excess of a-glucosidase, characterized by generalized
ammonia in circulation, which leads to men- hypotonia, muscle weakness, and hypertrophic
tal retardation, seizures, and ultimately death. cardiomegaly. Patients usually have cardiores­
Some patients with OTC deficiency also ex- piratory failure by 1 year of age with the early
hibit a very low blood urea nitrogen, but this onset form of the disorder. Laboratory findings
is not enough to make a conclusive diagnosis. include significantly elevated serum creatinine
Unlike the rest of the urea cycle disorders that kinase.
are autosomal recessive, deficiency of OTC is
X-linked. 20. The correct answer is D. This patient has ob-
structive jaundice, causing her pruritus and
Answer A is incorrect. Fabry disease is caused
scleral icterus. In this situation conjugated
by mutations in the a-galactosidase A gene,
bilirubin cannot be excreted, and its levels are
resulting in the accumulation of ceramide tri-
therefore elevated in the serum. The unconju-
hexoside. Patients classically have angiokera-
tomas, hypohidrosis, corneal and lenticular
High-Yield Principles
Chapter 2: Biochemistry  •  Answers 39

gated bilirubin level, however, is not elevated. heart contraction) and chronotropy (increased
Alkaline phosphatase is usually elevated in heart rate).
cases of obstructive jaundice.
Answer D is incorrect. Activation of hista-
Answer A is incorrect. This is seen in Crigler- mine1 receptors results in pruritus, broncho-
Najjar syndrome type I, which leads to kernic- constriction, and increased nasal and bron-
terus and is diagnosed during childhood. Ex- chial mucus production. Histamine1-receptor
tremely high levels of unconjugated bilirubin antagonists are primarily used in the treatment
would be expected in this case. of seasonal allergy symptoms.
Answer B is incorrect. This is seen in Gilbert Answer E is incorrect. Activation of hista-
syndrome, which does not have any clinical mine2 receptors leads to increased gastric acid
consequence. This syndrome is more common production. Therefore, histamine2-receptor
in men, the jaundice is typically associated antagonists such as cimetidine are used in the
with stress or exercise, and alkaline phospha- treatment of gastroesophageal reflux disease.

Biochemistry
tase levels are normal.
22. The correct answer is B. This patient has
Answer C is incorrect. Extravascular hemo-
Pompe disease, a glycogen storage disorder.
lysis (eg, hereditary spherocytosis) would lead
Pompe disease is an autosomal recessive dis-
to increased levels of unconjugated bilirubin.
ease that is characterized by a deficiency or
Hemoglobinuria would not be observed in
defect in lysosomal a-1,4-glucosidase. This
these patients.
enzyme is necessary for the dissolution of the
Answer E is incorrect. Intravascular hemo- polymer linkages in glycogen. In its absence,
lysis (eg, due to mechanical injury to RBCs, glycogen accumulates to toxic levels in both
defective cardiac valves, toxic injury to RBCs) the cytoplasm and lysosomes.
would lead to an increase in unconjugated
Answer A is incorrect. Glucose is stored as gly-
bilirubin, which is not the case in this patient.
cogen in the cells and is also present in blood.
Also, levels of alkaline phosphatase would not
However, hyperglycemia is not responsible for
be elevated in a patient with increased RBC
the symptoms observed in this patient.
destruction. Hemoglobinuria would be ex-
pected in these patients. Answer C is incorrect. Oxaloacetate is the first
intermediate in the Krebs cycle. It is regener-
21. The correct answer is C. This patient is expe- ated with each turn of the cycle but is not pres-
riencing an acute asthma attack. Treatment is ent in excessive amounts in the cell.
targeted at relieving bronchoconstriction and
Answer D is incorrect. Pyruvate is a compo-
inflammation. This patient should be treated
nent of the cellular respiration pathway and
with a b2 agonist such as albuterol. When ac-
an intermediate in gluconeogenesis. It is not
tivated with an agonist, the b2 receptor will
stored in cells in any significant quantity.
cause bronchodilation.
Answer E is incorrect. Disorders of the urea
Answer A is incorrect. b1 Blockers inhibit
cycle lead to nitrogen accumulation in the
sympathetic stimulation of b1 receptors by
body and result in progressive lethargy and
epinephrine. b1 Blockers reduce the effect of
coma. They do not cause the myopathy seen
excitement/physical exertion on heart rate and
in this patient.
force of contraction; they also add to the dila-
tion of blood vessels, some opening of bronchi, 23. The correct answer is B. Glucokinase catalyzes
and also reduction of tremor. the initial step of glycolysis, which is the phos-
Answer B is incorrect. b1 Agonists work on the phorylation of glucose to glucose-6-phosphate.
heart more than the lungs. Activation of b1- In most other tissues, this process is catalyzed by
receptor agonists leads to inotropy (increased hexokinase. Both enzymes are found in the liver.
Glucokinase has a higher Michaelis-Menten
High-Yield Principles
40 Section I: General Principles  •  Answers

constant and a higher Vmax than hexokinase; cholesterol levels rise, which causes an in-
it thus has a low affinity for glucose but large crease in plasma cholesterol; triglyceride levels
capacity of activity. Importantly, it is not inhib- remain normal.
ited by glucose-6-phosphate, as is hexokinase.
Answer D is incorrect. Mixed hypertriglyc-
Answer A is incorrect. The hepatocyte cell eridemia (type V) is a dyslipidemia character-
membrane is permeable to glucose, which ized by extremely high triglyceride levels and
is trapped in the cell after phosphorylation to visibly foamy plasma. Unlike type I, type V is
glucose-6-phosphate. characterized by elevated VLDL cholesterol
levels and is thought to be related to a VLDL
Answer C is incorrect. Hexokinase has a
cholesterol clearance problem.
high affinity (low Michaelis-Menten constant,
Km) for glucose and processes glucose to glu- Answer E is incorrect. VLDL cholesterol over-
cose-6-phosphate at lower levels of glucose. production is another characteristic of type V
At higher glucose levels, hexokinase is over- dyslipidemias, as well as type IIb combined hy-
Biochemistry

whelmed (low Vmax), and sufficient substrate is perlipidemia.


available for glucokinase to process the excess
glucose despite its higher Km. 25. The correct answer is E. Following endothe-
lial injury, the subendothelial space accumu-
Answer D is incorrect. Hexokinase has a rela-
lates lipoproteins. Next, chemical modifica-
tively low Michaelis-Menten constant.
tion (eg, glycation or oxidation) of lipoproteins
Answer E is incorrect. Glucokinase has a rela- occurs that recruits monocytes to the vessel
tively high Michaelis-Menten constant. wall. Monocytes are converted to macrophages
upon entry into the subendothelial space, and
24. The correct answer is C. Type I dyslipidemia unregulated macrocytosis of modified LDL
(or familial lipoprotein lipase deficiency) is cholesterol occurs, yielding foam cells. At this
caused by a deficiency of lipoprotein lipase. point, various cell mediators, most notably
This enzyme exists in capillary walls of adipose platelet-derived growth factor, tumor necrosis
and muscle tissue and cleaves triglycerides into factor, and interleukin-1, recruit platelets and
free fatty acids and glycerol. The enzyme is ac- smooth muscle to the intimal lining, where
tivated by apolipoprotein C-II, which is found proliferation and production of extracellular
on VLDL cholesterol and chylomicrons. matrix leads to the development of a fibrous
Type I dyslipidemia is characterized by an ac- plaque.
cumulation of triglyceride-rich lipoproteins in
Answer A is incorrect. γ-Interferon is secreted
the plasma. Deficiency in apolipoprotein C-II
by T-helper cells and stimulates macrophages.
produces a similar result.
This mediator does not play a prominent role
Answer A is incorrect. VLDL cholesterol in the pathogenesis of atherosclerotic plaque.
remnants are removed from the circula-
Answer B is incorrect. Complement defends
tion by apolipoprotein E receptors. Thus,
against gram-negative bacteria and is activated
apolipoprotein E deficiency (dysbetalipopro-
by IgG or IgM in the classic pathway, and acti-
teinemia) results in an elevated VLDL choles-
vated by molecules on the surface of microbes
terol, triglyceride, and cholesterol levels. Often
in the alternate pathway. Complement has not
this disorder manifests with other conditions
been shown to be an active participant in the
that cause hyperlipidemia such as diabetes.
pathogenesis of atherosclerotic plaque.
Xanthomas are often present.
Answer C is incorrect. Interleukin-6 (IL-6) is
Answer B is incorrect. LDL cholesterol recep-
secreted by T-helper cells and macrophages,
tor dysfunction is characteristic of familial hy-
and is responsible for stimulating the produc-
perbetalipoproteinemia, also known as type II
tion of acute-phase reactants and immuno-
hyperlipidemia. In these cases, plasma LDL
High-Yield Principles
Chapter 2: Biochemistry  •  Answers 41

globulins. IL-6 does not play a prominent role mocysteine, which is then excreted in urine.
in the pathogenesis of atherosclerotic plaque. In this condition, cysteine becomes an essen-
tial amino acid. Clinically, homocystinuria is
Answer D is incorrect. Natural killer cells are
manifested by mental retardation, osteoporo-
a form of cytotoxic lymphocytes and constitute
sis, tall stature, kyphosis, lens subluxation, and
a major component of the innate immune sys-
atherosclerosis (causing premature stroke and
tem. These cells play a major role in the host
myocardial infarction). Diagnosis is based on
rejection of both tumors and virally-infected
a positive nitroprusside cyanide test. Marfan
cells, and do not play a prominent role in the
syndrome is the primary differential diagno-
pathogenesis of atherosclerotic plaque.
sis. Clinical features of homocystinuria, such
26. The correct answer is B. This patient has as ectopia lentis, tall and thin body habitus,
adult polycystic kidney disease, an autosomal and chest and spinal deformities, are similar
dominant condition characterized by mas- to the features found in patients with Marfan
sive bilateral cysts in the kidneys, asymptom- syndrome. However, generalized osteoporosis,

Biochemistry
atic hepatic and pancreatic cysts, mitral valve arterial and venous thrombosis, premature ath-
prolapse, and berry aneurysms. All disease erosclerosis, changes in hair, and the disorders
manifestations are believed to be secondary to of mental development are absent in patients
abnormal epithelial cell differentiation, most with Marfan syndrome.
likely caused by a mutation in the polycystic Answer A is incorrect. Lesch-Nyhan syndrome
genes. The renal cysts eventually progress to is an X-linked recessive condition caused by
end-stage renal disease. Berry aneurysms tend a defect in the purine salvage pathway due to
to increase in size with age, thus increasing the the absence of HGPRT (hypoxanthine guan­
risk of rupture and intracranial hemorrhage. ine phosphoribosyl transferase), which con-
Answer A is incorrect. Astrocytomas are seen verts hypoxanthine to inosine monophosphate
in patients with tuberous sclerosis, an autoso- and guanine to guanosine monophosphate.
mal dominant disorder affecting the tuberin As a result, excess uric acid accumulates and
and hamartin proteins, which regulate cellular causes hyperuricemia and gout. Patients also
growth and differentiation. present with mental retardation, self-mutilating
behavior, aggression, and choreoathetosis.
Answer C is incorrect. Ectopic lens is seen
in Marfan syndrome, an autosomal dominant Answer B is incorrect. Osteogenesis im-
connective tissue disorder associated with slen- perfecta is an autosomal dominant disorder
der body habitus and aortic dissection. caused by a variety of gene defects that result
in abnormal synthesis of type I collagen. Clini-
Answer D is incorrect. Optic nerve degen- cally, it is characterized by multiple fractures
eration can be seen in Leber hereditary optic occurring with minimal trauma (“brittle bone
neuropathy, a condition in which patients de- disease”), blue sclerae due to the translucency
velop a rapid loss of central vision. of connective tissue over the choroid, hearing
Answer E is incorrect. Increased incidence of loss due to abnormal middle ear bones, and
squamous cell carcinoma is seen in patients dental imperfections due to lack of dentition.
with xeroderma pigmentosum, an autosomal Answer C is incorrect. Fragile X results from
recessive disease caused by a deficiency in an expansion of the CGG trinucleotide repeat
DNA repair of thymine dimers. on the FMR-1 locus of the X chromosome.
Methylation of these trinucleotide repeats will
27. The correct answer is E. Homocystinuria is result in nonproduction of the FMR-1 protein,
an autosomal recessive condition caused by resulting in a phenotype that includes mental
deficiencies of various enzymes involved in retardation, an elongated face, protruding ears,
the pathway that converts methionine to cys- macro-orchidism, and low muscle tone. While
teine. This results in the accumulation of ho- children suffering from fragile X might exhibit
High-Yield Principles
42 Section I: General Principles  •  Answers

the same developmental delays and mild re- lower affinity for oxygen. The taut form is sta-
tardation as the child in this vignette, they will bilized by the processes that result in increased
not have pes cavus, arachnodactyly, and dislo- oxygen unloading, including binding of car-
cated lenses, and folic acid supplementation bon dioxide, low pH, and increased levels of
will not help them. 2,3-BPG in RBCs. Therefore, by decreasing
the partial pressure of carbon dioxide, one in-
Answer D is incorrect. Chronic granuloma-
creases hemoglobin’s affinity for oxygen.
tous disease is an X-linked recessive condition
caused by a lack of reduced nicotinamide ad- Answer C is incorrect. Binding of oxygen mol-
enine dinucleotide phosphate oxidase activity ecules is the major cause of the shift of hemo-
within neutrophils. As a result, neutrophils can globin from its taut structure to the relaxed
ingest bacteria but cannot kill them due to a form. The oxygen molecule disrupts the weak
defective oxidative burst. Patients present with polar bonds and “opens up” the molecule for
a marked susceptibility to opportunistic infec- more oxygen to bind.
tions with bacteria, especially Staphylococcus
Biochemistry

Answer D is incorrect. The Bohr effect comes


aureus, Escherichia coli, and Aspergillus. Di-
from an increase in protons, which subse-
agnosis is confirmed with a negative nitroblue
quently stabilize the taut form of hemoglo-
tetrazolium test.
bin preferentially. In addition, an increase in
protons means an increase in carbon dioxide
28. The correct answer is B. Glucose enters pan-
because of the bicarbonate buffer present in
creatic b cells via GLUT 2 channels. GLUT
blood. Remember, though, that increasing the
2 transporters are insulin-dependent glucose
pH means a decrease in protons.
channels that are present on pancreatic b cells
when glucose levels are low. Answer E is incorrect. Carbon monoxide sta-
bilizes the relaxed form of hemoglobin so that
Answer A is incorrect. GLUT 1 transport-
the dissociation curve shifts dramatically to the
ers are found in RBCs and the brain. GLUT
left; thus oxygen cannot be unloaded.
2 transporters are found in pancreatic b cells,
liver, and kidney.
30. The correct answer is D. Glucose-6-phosphate
Answer C is incorrect. GLUT 4 transporters dehydrogenase (G6PD) deficiency often mani-
are found in adipose tissue and skeletal mus- fests in young adulthood or adolescents after
cle. a serious infection. Genetic mutation variants
have been described, including the common
Answer D is incorrect. Glucose cannot simply
G6PDA and G6PD Mediterranean, both of
diffuse across membranes; it requires transport-
which are X-linked. Normal G6PD generates
ers.
reduced nicotinamide adenine dinucleotide
29. The correct answer is B. 2,3-Bisphosphoglyc- phosphate (NADPH) from oxidized nico-
erate (BPG) promotes oxygen unloading by tinamide adenine dinucleotide phosphate
binding to a pocket formed by the two a sub- (NADP+), which is used to reduce glutathi-
units. It can bind only when they are close to- one. Reduced glutathione is used to detoxify
gether, such as in the taut form. It is essential oxidizing agents. Oxidizing agents are found
to add inosine to stored blood for transfusions at times of infection, as with certain drugs
to prevent the loss of 2,3-BPG. Any change such as primaquine, and with ingestion of fava
that enhances the taut form of hemoglobin beans (called favism when symptomatic).
will decrease hemoglobin’s affinity for oxygen. Answer A is incorrect. Glucose-6-phosphate
Answer A is incorrect. Hemoglobin carries is generated from glucose by hexokinase in
oxygen better when it is in the relaxed form, all cells. High levels of glucose are processed
which has a higher affinity for oxygen. Con- by glucokinase in liver cells and pancreatic B
versely, the taut form of hemoglobin has a cells.
High-Yield Principles
Chapter 2: Biochemistry  •  Answers 43

Answer B is incorrect. Glucose-6-phosphate Answer B is incorrect. PTH stimulates the


is generated from glucose by hexokinase in all production of 1,25-dihydroxyvitamin D, which
cells. in turn increases calcium and phosphate ab-
sorption from the gut. It does not inhibit
Answer C is incorrect. Hunter disease is an X-
1,25-dihydroxyvitamin D production.
linked mucopolysaccharidosis that results from
a deficiency of mucopolysaccharide break- Answer C is incorrect. PTH promotes cal-
down. It is not associated with this patient’s cium reabsorption in the renal tubules, not
presentation. calcium excretion.
Answer E is incorrect. The conversion of Answer D is incorrect. PTH inhibits the fur-
glucose-6-phosphate (G6P) to 6-phosphoglu- ther release of PTH as part of a negative feed-
conate (6PG) is the first step in the pentose- back loop; it does not induce further release of
phosphate pathway (also known as the hexose- PTH.
monophosphate shunt). This particular step
Answer E is incorrect. PTH inhibits phos-

Biochemistry
produces NADPH (reduced NADP+), which
phate reabsorption in the renal tubules and,
is used to maintain a reduced pool of gluta-
in fact, promotes phosphate excretion. Think
thione. It is also used to synthesize fatty acids
“PTH”: Phosphate Trashing Hormone.
and steroids, to generate the cytochrome P450
(CYP450) enzyme system, and in phagocytosis. 32. The correct answer is E. Deficiency of por-
6PG can be converted to ribose-5-phosphate phobilinogen deaminase (also known as uro-
for use in DNA synthesis. These processes oc- porphyrinogen I synthetase) causes acute
cur in all cells, but deficiency of G6P has the intermittent porphyria (AIP), a disease char-
greatest effect on RBCs, which have no alter- acterized by acute attacks of gastrointestinal,
nate means of reducing oxidizing toxins. neurologic/psychiatric, and cardiovascular
symptoms. A deficiency in porphobilinogen
31. The correct answer is A. Parathyroid hormone
deaminase leads to a deficiency in heme syn-
(PTH) plays a key role in the regulation of cal-
thesis, in turn causing a build-up of intermedi-
cium and phosphate balance. When the serum
ary products such as aminolevulinic acid and
calcium level falls, PTH is rapidly released from
porphobilinogen, which cause the symptoms
the parathyroid glands and acts to restore serum
of the disease. Age at onset is almost always af-
calcium levels to normal through the following
ter puberty, it is more common in women, and
mechanisms: (1) increasing calcium absorption
symptoms often are precipitated by drugs that
in the small intestine; (2) promoting calcium
induce heme-containing CYP450. Because of
reabsorption in the renal tubules; (3) inhibiting
the intermittent nature of attacks, AIP can be
phosphate reabsorption in the renal tubules; (4)
difficult to diagnose. These patients often will
inhibiting the further release of PTH as part of
present with urine that turns a dark color when
a negative feedback loop; and (5) stimulating
exposed to air.
the production of 1,25-dihydroxyvitamin D.
Primary hyperparathyroidism is one of the most Answer A is incorrect. A deficiency in adeno­
common forms of hypercalcemia, whereby a sine deaminase (ADA) can cause severe com-
parathyroid adenoma spontaneously produces bined immunodeficiency disorder. It presents
excess PTH, leading to hypercalcemia and hy- with recurrent viral, bacterial, fungal, and
pophosphatemia. Secondary PTH is caused by protozoal infections. ADA normally functions
excess PTH in response to renal failure or other to convert adenosine to inosine in the purine
situations like 1,25-dihydroxyvitamin D defi- salvage pathway. Deficiency of this enzyme
ciency (rickets or osteomalacia), in which low causes an excess of ATP and dATP that im-
calcium and high phosphate levels detected by balances nucleotide synthesis through nega-
the body lead to excess PTH production. tive feedback, thus preventing DNA synthesis
and decreasing lymphocytes. ADA deficiency
would not be associated with the symptoms
High-Yield Principles
44 Section I: General Principles  •  Answers

seen in this patient, nor would it become clini- deletion of WT1, a tumor suppressor gene lo-
cally apparent at the age of 22 years. cated on chromosome 11p.
Answer B is incorrect. Congenital deficiency Answer A is incorrect. Adrenocortical adeno-
of homogentisic acid oxidase, an enzyme in mas are usually asymptomatic and are often in-
the degradative pathway of tyrosine, results in cidentally found at autopsy or on CT. They are
a disease called alkaptonuria. This deficiency usually smaller tumors, nodular lesions <2.5
leads to the accumulation of homogentisate, cm in diameter. They rarely cause pathology.
forming polymers that cause urine to darken
Answer B is incorrect. Neuroblastoma is the
on standing. This disease is generally innocu-
most common extracranial solid tumor in chil-
ous but may cause arthritis with darkening of
dren. They can present retroperitoneally, often
the joints. It would not be associated with the
involving the adrenal medulla, as they arise
systemic symptoms described in this patient.
from the neural crest cells; they would not typ-
Answer C is incorrect. Lysosomal a-1,4-­ ically be of renal origin. Neuroblastoma is as-
Biochemistry

glucosidase deficiency causes a rare glycogen sociated with the N-myc oncogene, located on
storage disease called Pompe disease, which chromosome 2p.
is transmitted in an autosomal recessive man-
Answer C is incorrect. Renal cell carcinoma
ner. It presents with cardiomegaly along with
is the most common renal cancer in adults. It
weakness and hypotonia in the first six months
usually presents in older adults after the fifth
of life. It is often fatal by age 2 years. Pompe
decade of life and is uncommon in children.
disease would not be associated with the symp-
toms seen in this patient. Answer D is incorrect. Transitional cell carci-
noma can involve any part of the urothelium.
Answer D is incorrect. Ornithine transcarba-
In the kidney, it usually involves the renal pel-
mylase deficiency, an X-linked disorder, is the
vis. Transitional cell carcinoma accounts for
most common genetic deficiency of the five
5%-10% of primary renal tumors.
enzymes of the urea cycle. Failure to synthe-
size urea, and therefore break down ammo- 34. The correct answer is D. This child and his
nia, leads to hyperammonemia during the first uncle appear to have Friedreich ataxia, a tri-
weeks of life and can lead to mental retarda- nucleotide repeat disease. Like other trinucle-
tion. Treatment includes limiting protein in- otide repeat diseases, illness occurs because
take in the diet and administering compounds the unstable microsatellite regions on certain
like phenylbutyrate, which help bind to amino chromosomes have triplet codons that expand,
acids and promote their excretion. This disor- typically worsening from generation to genera-
der would not present with the symptoms seen tion (and often making the age of onset earlier
in this patient, nor would it normally present for each successive generation). These regions
at age 22 years. of massively expanded triplet repeats (most
commonly between 600 and 1200 in Fried-
33. The correct answer is E. The patient is a
reich ataxia) cause a decrease in the product of
young boy presenting with an abdominal mass
a gene, frataxin, at the translation stage.
involving the kidney. The most common re-
nal tumor in young children is Wilms tumor, Answer A is incorrect. The triplet repeats in
which accounts for approximately 6% of pe- Friedreich ataxia typically do not affect protein
diatric malignancies. Wilms tumor typically folding because the gene product does not usu-
presents with a palpable mass, and about 25% ally get to that stage.
of patients present with gross or microscopic Answer B is incorrect. The triplet repeats in
hematuria. It is an embryonal neoplasm that Friedreich ataxia typically do not affect protein
tends to occur between ages 1 and 3 years, splicing.
with most cases occurring before the age of 7
years. Wilms tumor is often associated with the
High-Yield Principles
Chapter 2: Biochemistry  •  Answers 45

Answer C is incorrect. The triplet repeats in glycogen storage diseases. In particular, high
Friedreich ataxia typically do not cause a sub- protein plus creatinine supplementation may
stitution, such as that which occurs in sickle be recommended in McArdle disease, a glyco-
cell disease. gen storage disease that primarily affects skele-
tal muscle. This disease causes painful muscle
Answer E is incorrect. The triplet repeats in
cramps and myoglobinuria with strenuous ex-
Friedreich ataxia typically do not contain a
ercise.
stop codon.
Answer E is incorrect. Recombinant enzyme
35. The correct answer is B. The clinical vignette therapy has not been developed for PKU. This
strongly suggests phenylketonuria (PKU). PKU technology has been used to develop a treat-
results most commonly from a deficiency of ment for the adult-onset form of Gaucher dis-
phenylalanine hydroxylase, the first enzyme ease, a lysosomal storage disorder. This disor-
in the breakdown pathway for phenylala- der is characterized by hepatosplenomegaly,
nine. The product of this reaction is tyrosine, aseptic necrosis of the femur, and bone crisis.

Biochemistry
a nonessential amino acid in healthy patients.
Infants with PKU have inappropriately high 36. The correct answer is E. Pellagra is caused by
blood phenylalanine levels. High phenylala- vitamin B3 (niacin) deficiency. Patients clas-
nine levels are neurotoxic, so this may lead sically present with diarrhea, dermatitis, and
to mental retardation if untreated. Other dementia (and beefy glossitis). Other water-
symptoms include hypopigmentation (due to soluble vitamins include riboflavin (B2), niacin
impaired melanin synthesis), musty odor, ec- (B3), biotin, folate, and cobalamin (B12).
zema, and hyperreflexia. Treatment includes
Answer A is incorrect. Increased erythrocyte
a diet low in phenylalanine with tyrosine sup-
hemolysis is the result of vitamin E deficiency,
plementation (since it is now essential). It is
which results in increased fragility of RBCs.
most effective if instituted before the child is 3
Vitamin E is an antioxidant that protects RBCs
weeks old. Most infants in North America are
from hemolysis. It is fat-soluble.
currently routinely screened for PKU.
Answer B is incorrect. Neonatal hemorrhage
Answer A is incorrect. A diet low in the
is due to vitamin K deficiency. Vitamin K cata-
branched-chain amino acids leucine and iso-
lyzes γ-carboxylation of glutamic acid residues
leucine is used to treat patients with maple
on various proteins concerned with blood clot-
syrup urine disease. These patients lack a
ting. It is a fat-soluble vitamin.
branched-chain dehydrogenase, and thus
have inhibited breakdown of leucine, isoleu- Answer C is incorrect. Night blindness is a
cine, and valine. Infants with this disorder consequence of vitamin A (retinol) deficiency.
would have an odd smell to their urine. They Vitamin A is a constituent of visual pigments
can have severe central nervous system de- and is a fat-soluble vitamin.
fects. They do not typically have pale skin, as Answer D is incorrect. Osteomalacia is due to
branched chain amino acids, unlike phenylala- vitamin D deficiency. Vitamin D is a fat-soluble
nine, are not involved in the melanin synthesis vitamin.
pathway.
Answer C is incorrect. A diet low in tyrosine 37. The correct answer is A. This patient suffers
would cause harm to a patient with PKU, who from I-cell disease, which is caused by a fail-
cannot make this amino acid. ure of addition of mannose-6-phosphate by
GlcNAc phosphotransferase on the Golgi ap-
Answer D is incorrect. A high-protein diet paratus. Without mannose-6-phosphate, lyso-
would not be recommended in a patient with somal enzymes cannot be properly directed for
PKU, as it would lead to high levels of amino inclusion into lysosomes and will instead be
acids including phenylalanine. High-protein excreted by the cell. Thus lysosomal enzymes,
diets may be used in therapy of some of the
High-Yield Principles
46 Section I: General Principles  •  Answers

including hexosaminidase, iduronate sulfa- are a few of the many ophthalmologic prob-
tase, and arylsulfatase A, will be found in the lems such patients can have.
extracellular, but not intracellular, space. I-cell
Answer A is incorrect. Lactose intolerance is
disease is characterized by skeletal abnormali-
not associated with albinism.
ties, restricted joint movement, coarse facial
features, and severe psychomotor impairment. Answer B is incorrect. Children with phenyl-
Death usually occurs in the first decade of life. ketonuria should avoid foods with phenylala-
nine.
Answer B is incorrect. Although lysosomes
may be abnormal in I-cell disease, that ab- Answer C is incorrect. Avoiding strenuous ac-
normality results from defective trafficking of tivity may be useful for patients with certain
intracellular proteins caused by an abnormal rare cardiac diseases, but is not necessary for
Golgi apparatus, not from an intrinsic lyso- children with albinism.
somal abnormality. This is proven by the find- Answer D is incorrect. Patients with Turner
ing that cultured cells from patients with I-cell
Biochemistry

syndrome will be very short at full growth (<5


disease are capable of incorporating lysosomal feet tall) unless GH is given in childhood.
enzymes if properly tagged with mannose- Children with albinism have normal growth
6-phosphate. and development.
Answer C is incorrect. Ribosomes are primar-
ily responsible for ensuring the proper match- 39. The correct answer is B. The patient most
ing of messenger RNA with transport RNA, likely has pancreatic adenocarcinoma that is
therefore ensuring protein synthesis. The ri- located at the head of the pancreas, leading to
bosomes of patients with I-cell disease are not obstruction of the common bile duct. Weight
abnormal. loss, painless jaundice, and a palpable gallblad-
der (Courvoisier sign) can occur in pancreatic
Answer D is incorrect. Rough endoplasmic cancer. The obstruction from the growing tu-
reticulum (RER) typically translates proteins mor results in conjugated hyperbilirubinemia,
meant for extracellular use. The RER in I-cell increased urine bilirubin levels, and decreased
disease is not abnormal. urine urobilinogen levels. The majority of
Answer E is incorrect. Smooth endoplasmic bilirubin results from the breakdown of heme
reticulum (SER) is the site of cell lipid synthesis groups in senescent RBCs. After cellular re-
and processing of protein. The SER of patients lease, bilirubin binds to albumin, which deliv-
with I-cell disorder is not abnormal and does ers the molecule to the liver. Hepatocellular
not play a role in the addition of 6-mannose- uptake and glucuronidation in the endoplas-
phosphate. mic reticulum generate conjugated bilirubin,
which is water soluble and excreted in the bile.
38. The correct answer is E. Patients with congen- Gut bacteria deconjugate the bilirubin and de-
ital albinism have problems producing mela- grade it to urobilinogens. The urobilinogens
nin. This can be due to a deficiency in the are excreted in the feces, with some reabsorp-
tyrosine (precursor to melanin) transporters or tion and excretion into urine. Based on this
a deficiency in the tyrosinase enzyme. Either metabolic schema, the laboratory values in ob-
way, patients will have generalized decreased structive liver disease become evident. Failed
pigmentation in the skin, eyes, and hair. With- excretion of conjugated bilirubin leads to a
out aggressive photoprotection, most albinism direct bilirubinemia. Failure of urobilinogen
patients will eventually develop skin cancer. production by gut flora leads to low levels of
The other major problems are ocular, as the urine urobilinogen. The urine bilirubin level
lack of melanin causes poor development of is elevated secondary to the increased plasma
the retinal pigment epithelium. The lack of concentration of direct bilirubin, which under-
pigment in the iris also causes problems. Nys- goes renal excretion.
tagmus, strabismus, and impaired visual acuity
High-Yield Principles
Chapter 2: Biochemistry  •  Answers 47

Answer A is incorrect. Conjugated hyperbili- including laminin. The legs look muscular,
rubinemia, increased urine bilirubin levels, but are actually pseudohypertrophic due to
and normal urine urobilinogen levels can massive interstitial fibrosis.
be seen in patients with hepatocellular jaun-
Answer A is incorrect. Dystrophin has no in-
dice. The urine urobilinogen level is normal
volvement with signaling at the neuromuscular
because, unlike in obstructive jaundice, gut
junction.
bacteria have the opportunity to synthesize
urobilinogen. The clinical presentation and Answer C is incorrect. Actin-myosin cross-
physical findings presented here supports a bridge cycling is promoted by calcium binding
diagnosis of pancreatic adenocarcinoma, not to troponin C, which causes a conformational
hep­atocellular jaundice. change that allows tropomyosin to move so
that actin-myosin cycling can occur.
Answer C is incorrect. Unconjugated hyper-
bilirubinemia, increased urine bilirubin lev- Answer D is incorrect. Dystrophin is not an
els, and decreased urine urobilinogen levels acetylcholine receptor.

Biochemistry
can occur with hepatocellular disease if there Answer E is incorrect. The ryanodine receptor
is also concurrent conjugated hyperbilirubine- and the dihydropyridine receptor are involved
mia. However, a pure obstructive condition is in the control of exocytosis of calcium from
not characterized by unconjugated hyperbili- the sarcoplasmic reticulum.
rubinemia.
Answer D is incorrect. Unconjugated hyper- 41. The correct answer is D. This patient pre­
bilirubinemia, decreased urine bilirubin lev- sents with altered mental status, hyperventila-
els, and increased urine urobilinogen levels is tion, and lactic acidosis with hypoglycemia.
a classic pattern seen in hemolytic jaundice. This presentation is consistent with ethanol-
In this case the bilirubin level rises due to in- induced hypoglycemia. Oxidation of ethanol
creased heme turnover and may overwhelm produces the reduced form of nicotinamide
the liver’s ability to conjugate it. These fea- adenine dinucleotide (NADH) in the liver via
tures would not be seen in obstructive liver two key enzymes: alcohol dehydrogenase and
disease. Patients with certain diseases that in- acetaldehyde dehydrogenase. A high ratio of
crease RBC turnover, such as sickle cell dis- NADH to oxidized nicotinamide adenine di-
ease, spherocytosis, and glucose-6-phoshate nucleotide (NAD) induces pyruvate metabo-
dehydrogenase deficiency, may have hemolytic lism to lactic acid, which leads to lactic aci-
jaundice. The clinical presentation and physi- dosis. Note the anion gap and the high lactate
cal findings here point to a diagnosis of pancre- level. This elevated NADH / NAD ratio limits
atic adenocarcinoma, not hemolytic jaundice. the supply of pyruvate needed for gluconeo-
genesis, hence hypoglycemia ensues.
Answer E is incorrect. Unconjugated hyper-
bilirubinemia, decreased urine bilirubin, and Answer A is incorrect. Glycerol 3-phosphate
decreased urine urobilinogen are very unlikely levels are increased (rather than decreased) as
to occur simultaneously in any given condi- a result of the low pyruvate levels. Incidentally,
tion. it is important to understand that this leads to
increased very low density lipoprotein levels
40. The correct answer is B. This boy suffers from and hyperlipidemia.
Duchenne muscular dystrophy, an X-linked Answer B is incorrect. Pyruvate is not overpro-
recessive disorder caused by a frameshift mu- duced; rather, it is metabolized to lactate.
tation that deletes the dystrophin gene and
causes accelerated muscle breakdown (in- Answer C is incorrect. Gluconeogenesis is in-
creases creatine kinase). Dystrophin normally hibited (note the low glucose level) during lac-
links actin filaments to a group of transmem- tic acidosis as a result of the pyruvate conver-
brane glycoproteins in the extracellular space, sion to lactic acid.
High-Yield Principles
48 Section I: General Principles  •  Answers

Answer E is incorrect. Thiamine (vitamin B1) Neither of these symptoms is typically associ-
deficiency can cause lactic acidosis, because ated with a vitamin deficiency.
it is a cofactor in the pyruvate dehydrogenase
complex; this causes pyruvate accumulation, 43. The correct answer is E. This patient is suf-
which induces lactic acid production. How- fering from methemoglobinemia, in which
ever, this patient has not been drinking for a symptoms typically develop as the percentage
long time, so he is less likely to be vitamin B1 of circulating hemoglobin that is instead met-
deficient. hemoglobin rises above 3%. Methemoglobin
contains Fe3+ (as opposed to the Fe2+ of circu-
42. The correct answer is D. Vitamin B12 is a nec- lating hemoglobin) and is formed by nonoxy-
essary cofactor in the regeneration of folate for gen oxidizing agents. One such agent known
methyl group donation in DNA synthesis. For to cause this is dapsone, which is used for toxo-
this reason, vitamin B12 and folate deficiencies plasmosis prophylaxis in HIV-positive patients.
each present with megaloblastic anemia and The Fe3+ of methemoglobin is unable to bind
Biochemistry

hypersegmented neutrophils, as all blood cell oxygen, and so cannot deliver it to the cells of
lines are affected by the defect in DNA syn- the body and the patient becomes cyanotic.
thesis. However, only vitamin B12 deficiency Nicotinamide adenine dinucleotide reductase
increases serum methylmalonic acid levels (NADH) converts Fe3+ into Fe2+ by simulta-
and impairs myelin synthesis. This myelin de- neously oxidizing NADH to the oxidized form
fect primarily impacts the posterior and lateral of nicotinamide adenine diphosphatase, and
spinal columns, causing paresthesias and im- so oxygen-carrying capacity is restored. This
paired proprioception. Thus, neurologic ab- system may be overwhelmed in the presence
normality in the context of megaloblastic ane- of oxidizing agents such as dapsone. Of note,
mia is a USMLE-favorite hint that vitamin B12, the arterial blood gas of patients with methe-
rather than folate, is deficient. moglobinemia will show a normal partial oxy-
gen pressure of because the level of dissolved
Answer A is incorrect. Confusion, confabula-
oxygen is normal; it is only the level of hemo-
tion, and ataxia are typical of Wernicke-Korsakoff
globin-bound oxygen that is reduced, hence,
syndrome, a disorder of thiamine (vitamin B1)
reduced oxygen saturation. The patient may
deficiency typical in alcoholics.
be acidotic secondary to lactic acidosis from an
Answer B is incorrect. Folate and vitamin oxygen deficit at the tissue level.
B12 deficiencies present similarly, but folate
Answer A is incorrect. ATPase, the enzyme re-
deficiency does not manifest with an elevated
sponsible for releasing a phosphate group from
methylmalonic acid level or neurologic prob-
ATP, is not responsible for converting met-
lems. In the absence of increased serum meth-
hemoglobin to hemoglobin. It is involved in
ylmalonate levels, therefore, the physician
many other reactions in the body and is often
would diagnose folate deficiency, and no neu-
used to provide energy for an energetically un-
rologic symptoms would be expected.
favorable but necessary reaction.
Answer C is incorrect. Concurrent onset of
Answer B is incorrect. Flavin adenine dinu-
dysarthria (defective articulation) and diplo-
cleotide (FADH2) reductase is not responsible
pia may indicate that a transient ischemic epi-
for converting methemoglobin to hemoglobin,
sode or cerebrovascular accident has occurred.
but instead returns FADH2 to its reduced form
These symptoms are not typical of any vitamin
so that it can accept electrons in the Krebs cy-
deficiency.
cle.
Answer E is incorrect. Syncope may be a sign
Answer C is incorrect. GTPase, the enzyme
of vasovagal stimulation, low blood pressure,
responsible for releasing a phosphate group
arrhythmia, and other cardiovascular disorders.
from GTP, is not responsible for converting
Lethargy is a sign of decreased consciousness.
methemoglobin to hemoglobin. It is used in
High-Yield Principles
Chapter 2: Biochemistry  •  Answers 49

many second messenger pathways and can trophy, a syndrome that consists of central and
help provide energy for an energetically unfa- peripheral demyelination, ataxia, and demen-
vorable but necessary reaction. tia.
Answer D is incorrect. Lactase, the enzyme Answer D is incorrect. Hunter syndrome is a
responsible for cleaving lactose into its mono- lysosomal storage disease that is characterized
saccharide constituents, is not responsible for by developmental delay, aggressive behav-
converting methemoglobin to hemoglobin. ior, and airway obstruction. It is due to a defi-
ciency of iduronate synthase.
Answer F is incorrect. Pyruvate kinase, the en-
zyme responsible for converting phosphoenol- Answer E is incorrect. Mutations in lysyl hy-
pyruvate to pyruvate at the end of glycolysis, is droxylase have been found in certain forms of
not responsible for converting methemoglobin Ehlers-Danlos syndrome. These mutations re-
to hemoglobin. duce the content of hydroxylysine in collagen
and reduce the content of collagen cross-links.

Biochemistry
44. The correct answer is E. Steroid hormones These defects are believed to decrease the ten-
enter cells and bind to receptor proteins. The sile strength of collagen and account for the
receptor-hormone complex binds to specific symptoms of Ehlers-Danlos syndrome.
response elements, or the regulatory region of
DNA, and activates gene transcription. 46. The correct answer is A. This patient has fruc-
tose intolerance caused by a deficiency in al-
Answer A is incorrect. Steroid hormones do
dolase B. Aldolase B catalyzes the reaction in
not regulate the initiation of protein synthesis.
which fructose-1-phosphate is metabolized to
Answer B is incorrect. Steroid hormones do dihydroxyacetone-phosphate and glyceralde-
not regulate mRNA degradation. hyde. If there is a deficiency in aldolase B, the
Answer C is incorrect. Steroid hormones do reactant fructose-1-phosphate accumulates in
not regulate mRNA processing. the liver. This depletes the liver’s stores of free
phosphate, which inhibits the production of
Answer D is incorrect. Steroid hormones reg- glucose through gluconeogenesis and causes a
ulate gene transcription, not translation. fall in cellular ATP levels. The common pre-
ceding event to the drop in glucose and ATP
45. The correct answer is C. Tay-Sachs disease levels is a decrease in free phosphate levels.
results from a deficiency of hexosaminidase A
activity. This disorder is inherited as an autoso- Answer B is incorrect. ATP will not be in-
mal recessive trait and has a carrier frequency creased, because much of the free phosphate
of 1:25 in the Ashkenazi Jewish population. needed to combine with adenosine diphos-
Findings include loss of motor skills, increased phate to form ATP is already bound to fruc-
startle reaction, macular pallor, and a cherry- tose.
red spot on the macula (as shown in the im- Answer C is incorrect. When a patient with
age). Diagnosis is confirmed by the quantifica- aldolase B deficiency is challenged with fruc-
tion of hexosaminidase level in isolated WBCs tose, glucose levels will fall and a lactic acido-
in the blood. Future at-risk pregnancies can be sis will ensue. Therefore the pH will decrease,
monitored via prenatal diagnosis by either am- not increase.
niocentesis or chorionic villus sampling.
Answer D is incorrect. Rough endoplasmic
Answer A is incorrect. Fabry disease occurs in reticulum (RER) typically translates proteins
a-galactosidase A deficiency. This disease is meant for extracellular use. The RER in I-cell
characterized by symptoms of peripheral neu- disease is not abnormal.
ropathy, cardiovascular, and renal disease.
Answer E is incorrect. In aldolase B defi-
Answer B is incorrect. Arylsulfatase A defi- ciency fructose can be converted to fructose-
ciency characterizes metachromatic leukodys- 1-phosphate and become trapped intracellu-
High-Yield Principles
50 Section I: General Principles  •  Answers

larly. Thus there should not be a significant phate leads to inhibition of gluconeogenesis
rise in urinary fructose excretion as occurs in and glycogenolysis. Patients present with jaun-
fructokinase deficiency. dice, cirrhosis, hypoglycemia, and scleral ic-
terus. Hepatosplenomegaly is often seen on
47. The correct answer is C. Glucose-6-phosphate physical examination. It is usually diagnosed
dehydrogenase (G6PD) deficiency is charac- in early childhood, when children are weaned
terized by acute episodes of hemolytic anemia from formula to regular table food.
following administration of certain medica-
Answer E is incorrect. Hereditary spherocyto-
tions, infection, or ingestion of fava beans.
sis is caused by a variety of molecular defects
G6PD is the first and rate-limiting enzyme of
in genes that code for spectrin, ankyrin, pro-
the pentose phosphate pathway, which pro-
tein 4.1, and other RBC membrane proteins.
duces two reducing equivalents that keep glu-
The structural defect makes the RBCs more
tathione in its reduced state. Reduced glutathi-
fragile and they take on a spherical shape,
one is necessary to detoxify peroxides and free
hence the name “spherocytes.” Spherocytes
Biochemistry

radicals that can accumulate within RBCs and


are recognized as abnormal and destroyed
cause damage to various cellular structures.
in the spleen, resulting in severe anemia and
Heinz bodies are small round inclusions seen
splenomegaly. Splenectomy curbs the anemia;
within RBCs that comprise hemoglobin and
however, the RBCs will continue to be spheri-
other protein precipitates. G6PD deficiency is
cal in shape.
most common in people who are black, people
of Mediterranean descent, and people from Answer F is incorrect. Lactase deficiency can
tropical Africa and Asia. be due to either an inherited intolerance or
an age-dependent acquired intolerance of the
Answer A is incorrect. Alkaptonuria is a rare
sugar lactose. Acquired lactase deficiency (de-
genetic disorder in which patients present
creased expression with increased age) is more
with urine that darkens on exposure to air.
common in Africans and Asians. Patients pre­
These patients cannot breakdown the amino
sent with osmotic diarrhea, abdominal cramps,
acid tyrosine, which results in the accumula-
and bloating.
tion of a by-product called homogentisic acid.
Homogentisic acid builds up in the skin and
48. The correct answer is A. At 28 weeks, this neo­
connective tissues and is excreted in the urine,
nate’s lungs have not had the opportunity to
causing the dark color. Patients typically have
fully develop, and are deficient in dipalmitoyl
a good prognosis, although many suffer from
phosphatidylcholine, also known as surfactant.
arthritis as well as heart disease and kidney/
Surfactant, synthesized by type II pneumo-
prostate stones.
cytes, is essential in stabilizing air-expanded
Answer B is incorrect. Cystinuria is caused alveoli by decreasing surface tension, thus pre-
by a defect in an amino acid transporter in the venting lung collapse during expiration. Sur-
renal tubules. This is an inherited defect that factant is usually made most abundantly after
affects the absorption of four amino acids: cys- the 35th week of gestation. The most com-
tine, ornithine, lysine, and arginine. Cystine mon way to determine lung maturity is by the
kidney stones can form as a result of excess lecithin:sphingomyelin ratio of the amniotic
cystine in the urine. This disorder is not associ- fluid. If this ratio is >2.0, then the risk of de-
ated with the jaundice or hematologic abnor- veloping neonatal respiratory distress syndrome
malities seen in this patient. (NRDS) is significantly decreased.
Answer D is incorrect. Hereditary fructose Answer B is incorrect. Elastase is an endog-
intolerance is an autosomal-recessive inher- enous proteolytic enzyme in the lung that is
ited disease due to a deficiency of aldolase B, normally broken down by antitrypsin. In pa-
which causes an accumulation of fructose- tients with a1-antitrypsin deficiency, there is an
1-phosphate. The decrease in available phos- increased level of elastase, which leads to lung
High-Yield Principles
Chapter 2: Biochemistry  •  Answers 51

tissue destruction and emphysema. Liver cir- nently cross-linking DNA strands, cyclophos-
rhosis also occurs as a result of the increased phamide causes breaks in double-stranded
level of elastase in the liver. DNA during cellular repair efforts, leading to
apoptosis.
Answer C is incorrect. Cilial dysfunction may
result in decreased mucus clearance from the Answer C is incorrect. Etoposide acts in the
lungs and may predispose patients to recurrent growth 2 (G2) and synthesis (S) phases of the
respiratory infections. In Kartagener syndrome, cell cycle, by inhibiting topoisomerase II and
cilia are immotile because of a dynein arm de- thereby inhibiting DNA synthesis. In the G2
fect. This results in infertility, bronchiectasis, and S phases, the chromosomes are not verti-
and recurrent sinusitis. cally aligned in the cell.
Answer D is incorrect. This neonate has signs Answer D is incorrect. Methotrexate acts
and symptoms consistent with NRDS second- in the synthesis (S) phase of the cell cycle. It
ary to his prematurity. NRDS is caused by a binds to dihyrofolate reductase and thereby

Biochemistry
deficiency of dipalmitoyl phosphatidylcholine, inhibits formation of purine nucleotides and
not phosphatidylglycerol. Phosphatidylglycerol thymidylate so that DNA synthesis cannot take
is a compound measured in amniotic fluid to place. In the S phase the chromosomes are not
determine fetal lung maturity, and is used in vertically aligned in the cell.
conjunction with the lecithin:sphingomyelin
ratio. 50. The correct answer is C. Fragile X syndrome
is a complex genetic disorder that most closely
Answer E is incorrect. Sphingomyelin is
follows an X-linked dominant pattern of inheri-
found primarily in the brain and nervous tis-
tance. It is caused by a trinucleotide repeat ex-
sue. Although it is measured to help in de-
pansion (CGG). The disease is characterized
termining fetal lung maturity, deficiency of
by mental retardation and physical features
sphingomyelin does not lead to NRDS.
such as macroorchidism (large testicles), long
face, large mandible, and large, everted ears.
49. The correct answer is E. The cells described
The number of CGG repeats in patients with
above are in metaphase of the mitosis (M)
the full mutation average around 230-4000. In
phase of the cell cycle, which is character-
the normal population, this sequence repeats
ized by chromosomes migrating and lining up
an average of 29 times. Patients with premu-
in the middle of the cell. In metaphase, the
tations have from 52 to 230 repeats and lack
nuclear envelope has disintegrated and the mi-
the disease phenotype. Differences in the size
totic spindle moves into the nuclear area. The
of CGG repeats can be shown on a Southern
chromosomes then become arranged in the
blot. Shorter fragments of DNA migrate more
plane of the spindle equator. Vincristine acts
rapidly through a gel toward the positive elec-
by binding to tubulin and blocking formation
trode. Therefore, patients with full mutations
of microtubules, which are required to form
(who have longer strands of DNA) will have a
the mitotic spindle.
“sluggish” band that stays toward the top of the
Answer A is incorrect. 5-Fluorouracil acts gel, the normal population will show a band at
in the synthesis (S) phase of the cell cycle. It the bottom of the gel, and patients with a pre-
works by complexing to folic acid, which in- mutation will have a band in the middle. Lane
hibits thymidylate synthesis and therefore C would best represent the Southern blot of a
DNA synthesis. In the S phase the chromo- woman who carries a premutation for fragile X
somes are not vertically aligned in the cell and one normal X.
Answer B is incorrect. Cyclophosphamide Answer A is incorrect. Lane A on the South-
is an alkylating agent, a class of cell cycle- ern blot shows two bands. The band at the top
nonspecific antineoplastic drugs. By perma- of the gel represents a large fragment of DNA
High-Yield Principles
52 Section I: General Principles  •  Answers

and an amplification of the CGG region to a Answer D is incorrect. The DNA band on
full-blown mutation. The band at the middle lane D represents a patient with no trinucleo-
of the gel represents a medium-sized fragment tide repeat expansion.
of DNA or a premutation.
Answer E is incorrect. Lane E on the South-
Answer B is incorrect. Lane B represents a ern blot shows two bands. The band at the bot-
patient with the full fragile X mutation. The tom of the gel represents a short fragment of
DNA band is located at the top of the gel and DNA and thus a normal-sized CGG repeat re-
represents a large fragment of DNA and an gion. The band at the top of the gel represents
amplification of the CGG region to a full- a large fragment of DNA and an amplification
blown mutation. of the CGG region to a full-blown mutation.
Biochemistry
Chapter 3

Embryology

53
High-Yield Principles
54 Section I: General Principles  •  Questions

Q u e st i o n s

1. A physician is asked to evaluate a 5-year-old (D) Incomplete fusion of urethral (urogenital)


girl who has developed a mass in her neck. folds
During the interview, he learns that the mass (E) Patency of the processus vaginalis
appeared within the past few months and has
been enlarging; however, it causes no pain or 4. Ensuring adequate maternal intake of a spe-
discomfort. The mass is in the midline of the cific nutrient is especially important in reduc-
neck just inferior to the hyoid bone. Labora- ing the incidence of the congenital anomaly
tory tests reveal a triiodothyronine level of 150 shown in the image. A deficiency of this nu-
ng/dL, a thyroxine level of 8.0 µg/dL, and a trient, when present in an adult, is associated
thyroid-stimulating hormone level of 1 µU/ with which of the following conditions?
mL. A CT scan of the neck is performed and
the doctor recommends surgery. Which of the
Embryology

following is the most likely diagnosis?


(A) Branchial cleft cyst
Subarachnoid
(B) Dermoid cyst space
(C) Ectopic thyroid gland
(D) Enlarged pyramidal lobe of the thyroid
(E) Lipoma
(F) Thyroglossal duct cyst

2. A 5-year-old girl is brought to her pediatri-


cian because her mother says she is frequently
bumping into stationary objects while playing.
Visual field examination shows bilateral pe-
ripheral vision defects. CT of the head reveals
Reproduced, with permission, from USMLERx.com.
calcifications in the pituitary fossa. Which of
the following is the most likely origin of this
child’s brain tumor? (A) Confabulation and anterograde amnesia
(A) Adenohypophyseal lactotrophs (B) Diarrhea, dermatitis, and dementia
(B) Fourth ventricle neuroectoderm (C) Megaloblastic anemia
(C) Rathke pouch (D) Megaloblastic anemia with neurologic
(D) Vascular endothelium symptoms
(E) Ventricular lining (E) Microcytic anemia
(F) Polyneuritis and cardiac pathology
3. A 6-week-old boy is brought to the clinic be- (G) Swollen and bleeding gums, easy bruising,
cause of fever. Urinalysis is positive for WBCs. and poor wound healing
Physical examination reveals the boy’s urethral
opening to be displaced to the ventral (infe-
rior) aspect of the penis. Which of the follow-
ing is the embryologic cause of this congenital
abnormality?
(A) Abnormal expansion of the ureteric bud
(B) Abnormal positioning of genital tubercle
(C) Failure of the mesonephric (wolffian) duct
to regress
High-Yield Principles
Chapter 3: Embryology  •  Questions 55

5. A 1-year-old girl presents to the pediatrician


because of increasing neuromuscular irritabil-
ity and tetany over the past few weeks. She has
also had recurrent viral and fungal infection
since birth, and was born full term. Results of
physical examination are within normal lim-
its. X-ray of the chest reveals the absence of a
thymic shadow. Thorough laboratory testing
reveals a T-lymphocyte deficiency. What other
findings are most likely in this child?
(A) Elevated parathyroid hormone levels
(B) Enlarged thymus gland
(C) Enlarged thyroid gland
(D) Low serum calcium levels Reproduced, with permission, from USMLERx.com.

Embryology
(E) Low serum phosphorus levels

6. A 13-month-old child is brought to the emer- (A) Cardiac tamponade


gency room after his parents found blood in (B) Coarctation of the aorta
his stool. They state that he did not appear dis- (C) Diaphragmatic eventration
tressed at the time, although he now displays (D) Mediastinal shift
some tenderness to abdominal pressure. Other (E) Pulmonary hypoplasia
than this tenderness, there are no significant
findings on physical examination. After per- 8. A 5-month-old girl is brought to the emer-
forming radionuclide imaging using 99mTc gency department by her parents because she
pertechnetate, the doctor makes a diagnosis is “turning blue.” She is cyanotic, weak, and
and recommends surgery to correct the prob- dyspneic. Her parents state that she has expe-
lem. What is the probable source of this child’s rienced similar episodes in the past, but never
condition? this severe. Physical examination reveals the
lungs are clear to auscultation, with no wheez-
(A) Blockage of the intestine due to folding of ing, rales, or rhonchi. Cardiac examination
the distal ileum into the proximal colon reveals a regular rate and rhythm, normal S1,
(B) Breakdown of the stomach mucosal bar- single S2, a grade III rough systolic murmur
rier, with erosion of the underlying mu- at the left sternal border in the third intercos-
cosa tal space, and a palpable right ventricular lift.
(C) Damage to the intestinal epithelium due Echocardiography demonstrates unusual posi-
to ingestion of coins tioning of the aorta, which overrides both the
(D) Ectopic gastric epithelium in a persistent left and right ventricles in the long axis view.
omphalomesenteric duct In this condition, the primary developmental
(E) Incomplete bowel rotation resulting in ob- defect occurs in which portion of the primitive
struction of the superior mesenteric artery heart?
7. The x-ray shown in the image is from a child (A) Bulbus cordis
with a congenital condition. Which of the fol- (B) Conal septum
lowing is the most common cause of death in (C) Left and right horns of the sinus venosus
infants with this condition? (D) Primitive atria
(E) Primitive ventricle
High-Yield Principles
56 Section I: General Principles  •  Questions

9. Spermatogenesis, the process of forming sper- (C) Failure of the midgut to return to the ab-
matozoa, occurs in the seminiferous tubules of dominal cavity after physiological midgut
the testes. As the cells proceed through the dif- herniation
ferent stages of spermatogenesis, they contain (D) Hypertrophy of the pyloric muscles
varying numbers of chromosomes and varying (E) Incomplete separation of the esophagus
amounts of DNA. A laboratory investigator is and laryngotracheal tube
observing, under the microscope, cells under-
going this process. He has focused on a group 12. A 16-year-old pregnant girl has not received
of cells that recently gained an acrosome and prenatal care. At 17 weeks, she noted some
tail. Which of the following best describes the painless vaginal spotting (bleeding) which
amount of DNA and the number of chromo- prompted her to seek medical attention. On
somes that exist in those cells? arrival at the hospital, pelvic examination
showed fetal parts in the cervical os and the
(A) 23, 1n
patient was told that a miscarriage was inevi-
(B) 23, 2n
Embryology

table. The products of conception as delivered


(C) 46, 1n
are shown in the image. What is the most
(D) 46, 2n
likely pathogenesis of this condition?
(E) 46, 4n

10. During embryonic development of the uri-


nary system, a portion of the bladder extends
into the umbilical cord. Failure of this vestigial
structure to degenerate may lead to which of
the following complications?
(A) Bladder exstrophy
(B) Bladder adenocarcinoma
(C) Meckel diverticulum
(D) Polycystic kidney disease
(E) Renal agenesis

11. A 4-year-old boy is brought to the pediatrician


by his mother, who says that he has been hav-
ing problems eating. She says he quickly be- Reproduced, with permission, from USMLERx.com.
comes full and often vomits following a meal.
When questioned, the boy says his stomach
hurts when he eats. The mother says that the (A) Ethanol toxicity
boy ate well up until about six months ago, (B) Malrotation
when the current problems began; he expe- (C) Maternal folate deficiency
rienced no abnormal vomiting as an infant. (D) Omphalomesenteric vessel occlusion
X-ray of the abdomen is remarkable for a dis- (E) ToRCHeS infection
tended, air-filled stomach that narrows at the
level of the proximal duodenum and then di- 13. Immediately following delivery, a newborn is
lates again in the distal duodenum. Which of observed to have multiple abnormalities, in-
the following developmental abnormalities is cluding a small lower jaw, abnormal feet, and
most likely responsible for this patient’s condi- hands that are clenched into fists. Treatment
tion? is started for a congenital heart condition. The
survival of a patient with this condition is most
(A) Abnormal rotation of the ventral pancre- similar to that of a person affected by which of
atic bud around the duodenum the following genetic abnormalities?
(B) Failure of duodenal recanalization
High-Yield Principles
Chapter 3: Embryology  •  Questions 57

(A) An F508 deletion in the CFTR gene (D) Migration of neural crest cells to the distal
(B) CAG tandem repeats colon
(C) CGG tandem repeats (E) Recanalization of the duodenum
(D) Trisomy of chromosome 13
(E) Trisomy of chromosome 21 17. A 34-year-old man complains of dry mouth
and difficulty swallowing. He has no other
14. A neonate is found to have a congenital heart complaints. Physical examination is begun in
defect. Physical examination reveals that the the head and neck region. The eye examina-
patient is not cyanotic, but a harsh holosystolic tion is grossly benign. The nasal sinus appears
murmur is heard at the left lower sternal bor- unremarkable. The patient’s oral cavity is then
der. No other murmurs are heard on ausculta- examined. To enable evaluation of the palatal
tion. From which of the following congenital elevation, the patient is asked to “say ah.” The
heart defects is this neonate most likely suffer- muscles used to perform palatal elevation are
ing? derived from which of the following embryo-
logic structures?

Embryology
(A) Atrial septal defect
(B) Tetralogy of Fallot (A) Branchial arches 1 and 2
(C) Transposition of the great arteries (B) Branchial arches 3 and 4
(D) Truncus arteriosus (C) Branchial arches 4 and 6
(E) Ventricular septal defect (D) Branchial clefts 1 and 2
(E) Branchial pouches 3 and 4
15. A scientist creates a model of fetal circulation
in order to study blood flow during this stage 18. A child has a unilateral notch of the upper lip
of development. During one experiment, he (see image). What is the most likely etiology of
measures the partial oxygen pressure in various this lip malformation?
fetal vessels. His results are as follows:
Vessel A: 20 mm Hg
Vessel B: 27 mm Hg
Vessel C: 35 mm Hg
Vessel D: 12 mm Hg
The vessel labeled C will develop into which
structure in the adult?
(A) Falciform ligament
(B) Ligamentum teres hepatis
(C) Ligamentum venosum
(D) Medial umbilical ligament
(E) Median umbilical ligament

16. A baby boy dies several hours after birth. He


was born with wrinkled skin, deformed limbs, Courtesy of Dr. James Heilman.
and abnormal facies. The mother’s pregnancy
was complicated by oligohydramnios. Which
(A) Abnormal fusion of the maxillary and me-
of the following embryologic processes most
dial nasal processes
likely failed in this child?
(B) Altered development of the third and
(A) Development of the ureteric buds fourth branchial pouches
(B) Closure of the rostral neural tube (C) Flaws in the mandibular and maxillary
(C) Formation of the tracheoesophageal sep- bones
tum (D) Incomplete development of the third pha-
ryngeal arch
High-Yield Principles
58 Section I: General Principles  •  Questions

19. A 3-week-old boy presents to his pediatrician 20. Over the course of embryologic development,
because his mother has noticed that he “looks the predominant location of hematopoiesis
yellow.” On questioning, she elaborates that changes several times. When the uterine fun-
the jaundice began several days after birth and dus is palpable above the umbilicus, where is
has been associated with dark urine and clay- the main location of hematopoiesis in the fe-
colored stools. Laboratory studies show a direct tus?
bilirubin level of 5.0 mg/dL and a total biliru-
(A) Bone marrow
bin level of 5.5 mg/dL. Which of the following
(B) Liver
is characteristic of the most likely diagnosis?
(C) Pancreas
(A) Caused by a genetic mutation in a pro- (D) Thymus
moter region (E) Yolk sac
(B) Caused by deficiency in uridine 5’-diphos-
phoglucuronosyltransferase
(C) Commonly treated with phototherapy
Embryology

(D) Inherited in an autosomal dominant pat-


tern
(E) Untreated, it leads to cirrhosis by six
months of age
High-Yield Principles
Chapter 3: Embryology  •  Answers 59

An s w e r s

1. The correct answer is F. The thyroid gland thyroid gland before surgery is performed on a
originates as the thyroid diverticulum on thyroglossal duct cyst.
the floor of the pharynx. It descends into the
Answer D is incorrect. Hypertrophy of the
neck during development, but remains con-
pyramidal lobe of the thyroid is not the most
nected to the tongue by the thyroglossal duct.
likely cause of midline neck swelling in a
The thyroglossal duct eventually disappears,
young child. Furthermore, hypertrophic thy-
leaving a small cavity (the foramen cecum)
roid tissue would most likely alter thyroid hor-
at the base of the tongue. The pyramidal lobe
mone and TSH levels.
of the thyroid can be thought of as the caudal
part of the duct. Occasionally, part of the duct Answer E is incorrect. Lipomas may cause
epithelium persists in the neck and may form neck swelling, but the location of this mass
cysts. Thyroglossal duct cysts are usually pain- and the age of the patient make a thyroglossal

Embryology
less or slightly tender and appear in the mid- duct cyst much more likely. Lipomas tend to
line of the neck. They often appear over or just be very superficial, with poorly defined edges.
below the hyoid, but may appear anywhere be-
tween the base of the tongue and the thyroid. 2. The correct answer is C. The visual-field de-
If a normal thyroid gland is present, surgery fect described is a bitemporal hemianopia, typ-
to remove the thyroglossal duct cyst is recom- ically caused by lesions in the sella turcica im-
mended to prevent infection. In this case, the pinging on the optic chiasm. In children the
presence of a normal thyroid is demonstrated most common tumor in this location is a cra-
by normal triiodothyronine, thyroxine, and niopharyngioma, derived from the remnants
thyroid-stimulating hormone (TSH) levels and of Rathke pouch. This embryologic structure
is confirmed by CT scan. buds from the roof of the mouth to form the
anterior pituitary. Bitemporal hemianopia is
Answer A is incorrect. Branchial cleft cysts typically accompanied by severe headaches
can also occur in the neck but are not always and poor pituitary function. Treatment in-
in the midline. Unlike thyroglossal duct cysts, cludes surgery, radiotherapy, or both.
they are often associated with fistulas or sinus
tracts. Answer A is incorrect. Pituitary adenomas,
derived from secretory cells of the adenohy-
Answer B is incorrect. Dermoid cysts are the pophysis, can cause bitemporal hemianopia
second most common cause of midline neck and headaches, as they are also sella turcica
masses, after thyroglossal duct cysts. They tend tumors. However, they are more common
to be more superficial than thyroglossal duct in older patients, and are unlikely in a child.
cysts and more mobile relative to underlying Also, pituitary adenomas tend to secrete pi-
structures. tuitary hormones. The three most common
Answer C is incorrect. Ectopic thyroid glands forms of pituitary adenoma are prolactino-
are often seen in the presence of a thyroglos- mas (which are derived from lactotrophs and
sal duct cyst. An ectopic thyroid gland occurs secrete prolactin, causing galactorrhea and
when the thyroid fails to descend during devel- amenorrhea), growth hormone-secreting tu-
opment; in contrast, ectopic thyroid tissue may mors (somatotrophs, causing acromegaly), and
occur along the path of the thyroglossal duct ACTH-producing tumors (corticotrophs, caus-
in the presence of a normal thyroid gland. Un- ing Cushing disease). In this case, the only
like this patient, who has normal thyroid lev- data making this an incorrect answer are the
els, about one third of patients with an ectopic patient’s age and the lack of secretory action.
gland are hypothyroid. A CT scan is usually Answer B is incorrect. Medulloblastoma arises
performed to confirm the presence of a normal from primitive neuroectoderm in the fourth
High-Yield Principles
60 Section I: General Principles  •  Answers

ventricle. It does not cause the visual symp- Answer B is incorrect. Abnormal positioning
toms seen in this patient, but the tumor may of the genital tubercle may result in epispadias,
compress the fourth ventricle to cause hydro- a condition in which the urethral opening is
cephalus and symptoms consistent with in- located on the dorsal (superior) surface of the
creased intracranial pressure such as morning penis. This condition is less common than hy-
headache and vomiting. An additional, specific pospadias and is associated with exstrophy of
sign of this posterior fossa tumor is a head tilt. the urinary bladder.
Treatment consists of surgery, chemotherapy,
Answer C is incorrect. The mesonephric
and radiation. When in doubt, remember that
(wolffian) duct develops into the seminal vesi-
medulloblastoma is the most common malig-
cles, epididymis, ejaculatory duct, and ductus
nant brain tumor in children.
deferens in the male. Regression of this duct
Answer D is incorrect. Hemangioblastomas might lead to an absence of these reproductive
are vascular tumors of the central nervous structures rather than hypospadias.
system that usually occur in the cerebellum
Embryology

Answer E is incorrect. Patency of the proces-


and spinal cord and thus would be unlikely
sus vaginalis allows fluid to flow from the peri-
to cause the visual field defects described in
toneum into the tunica vaginalis, resulting in a
this case. Symptoms include cerebellar ataxia,
hydrocele of the testes. This would not cause
motor weakness, and sensory dysfunction. He-
hypospadias, an abnormality in which the ure-
mangioblastomas can occur sporadically or
thra opens on the ventral surface of the penis.
in patients with von Hippel-Lindau disease,
which is an autosomal dominant disease in 4. The correct answer is C. The image shows a
which patients develop cerebellar and reti- meningomyelocele, a neural tube defect in
nal hemangioblastomas, pancreatic cysts, and which the meninges and spinal cord herniate
pheochromocytomas. through a defect in the spinal canal. Folate,
Answer E is incorrect. Ependymomas form if given to a mother early in pregnancy, low-
from the cells lining the ventricles and most ers the risk of developing neural tube defects
often occur in the fourth ventricle. Like me- (spina bifida occulta, meningocele, or menin-
dulloblastomas, ependymomas can block the gomyelocele). In the adult, folate deficiency
flow of cerebrospinal fluid and cause hydro- produces a megaloblastic anemia without neu-
cephalus. These patients, however, do not rologic symptoms. Folate deficiency is seen in
have the visual disturbances of the patient in alcoholics, pregnant women, patients with he-
this vignette. molytic anemia, and those taking drugs such
as methotrexate that inhibit folate metabolism.
3. The correct answer is D. This vignette de- Folate can be found in uncooked vegetables
scribes a urinary tract infection in an infant and fruits. A deficiency of vitamin B12 also
with hypospadias, a congenital abnormality in causes a megaloblastic anemia, but produces
which the urethra opens on the ventral (infe- neurologic symptoms as well. Such symptoms
rior) aspect of the penis. It occurs as a result include distal neuropathy and loss of position
of a failure of the urethral folds (also known as sense due to demyelination of the posterior
the urogenital folds) to fuse fully. Infants with and lateral columns of the spinal cord and of
hypospadias should undergo surgery to prevent the peripheral nerves.
urinary tract infections.
Answer A is incorrect. Confabulation and
Answer A is incorrect. The ureteric bud devel- anterograde amnesia, chronic neurologic se-
ops into the upper urinary system (collecting quelae of thiamine (vitamin B1) deficiency, are
duct, calices, renal pelvis, and ureters) and is seen in Wernicke-Korsakoff syndrome. This is
unrelated to the lower urinary system. Its ab- commonly seen in alcoholics.
normal expansion is not associated with known
urogenital anomalies.
High-Yield Principles
Chapter 3: Embryology  •  Answers 61

Answer B is incorrect. Diarrhea, dermatitis, Answer C is incorrect. DiGeorge syndrome


and dementia are characteristic of pellagra, can less frequently involve the clear cells of the
caused by a deficiency of niacin (vitamin B3). thyroid gland, but would not be expected to
cause an enlarged thyroid.
Answer D is incorrect. Megaloblastic anemia
with neurologic symptoms is a result of vita- Answer E is incorrect. With hypoparathyroid-
min B12 (cobalamin) deficiency. Typically, ism, an increased serum phosphorus level is
neurologic symptoms result from demyelin- anticipated.
ation of the posterior and lateral columns of
the spinal cord and of peripheral nerves. Sym- 6. The correct answer is D. The child has a
metric numbness or burning in the extremities Meckel diverticulum, which describes the
and loss of position sense are common. Unlike persistence after birth of part of the omphalo-
folate deficiency, this is not associated with mesenteric duct (vitelline duct or yolk stalk).
congenital neural tube defects. Cobalamin de- Meckel diverticulum is usually found in the
ficiency can manifest as dementia as well. mid to distal ileum, and may end blindly or

Embryology
connect to the umbilicus. It is described by
Answer E is incorrect. Microcytic anemia is
the “rule of 2s”: It is about 2 inches long, is
caused by iron deficiency as well as lead poi-
located 2 feet proximal to the ileocecal valve,
soning and the thalassemias.
occurs in about 2% of the population in a
Answer F is incorrect. Polyneuritis and car- 2:1 male:female ratio, often presents before 2
diac pathology are signs of beriberi, which re- years of age (60% of cases), and may contain
sults from thiamine (vitamin B1) deficiency. 2 types of epithelium (gastric or pancreatic).
Ectopic gastric epithelium is present in about
Answer G is incorrect. Scurvy is a disorder of
half of cases and can cause ulcers and pain-
swollen and bleeding gums, easy bruising, and
less bleeding, but does not generally cause se-
poor wound healing seen in vitamin C (ascor-
vere pain unless inflammation occurs. In the
bic acid) deficiency. Vitamin C is an essential
half of Meckel diverticulum patients with no
cofactor for the hydroxylation of proline and
ectopic gastric epithelium, there is no bleed-
lysine residues. Hydroxyproline and hydroxyly-
ing. 99mTechnetium (99mTc) pertechnetate is
sine allow crosslinking of collagen, giving con-
absorbed preferentially by gastric mucosa, and
nective tissue adequate tensile strength.
thus may be used to detect ectopic gastric mu-
5. The correct answer is D. This child has Di- cosa in the diverticulum.
George syndrome, which is associated with Answer A is incorrect. Intussusception is the
defective development of the third and fourth telescoping of the distal ileum into the proxi-
pharyngeal pouches. These structures typi- mal colon. It usually presents in the first two
cally give rise to the thymus and parathyroid years of life, and a Meckel diverticulum may
glands. Their absence explains the typical predispose to this condition. However, intus-
T-lymphocyte immunodeficiency and hypo- susception typically has an abrupt and severe
parathyroidism seen in these patients, and evi- presentation, with paroxysmal bouts of scream-
denced by the hypocalcemia in this case. Most ing, vomiting, diarrhea, and bloody bowel
patients also display congenital cardiac defects. movements occurring within 24 hours of on-
Answer A is incorrect. With hypoparathyroid- set. A sausage-shaped mass in the abdomen
ism from underdeveloped parathyroid glands, may be palpable on physical exam.
a decreased parathyroid hormone level would Answer B is incorrect. Breakdown of the mu-
be expected. cosal barrier of the stomach and erosion of
Answer B is incorrect. DiGeorge syndrome is the underlying mucosal epithelium describes
characterized by thymic hypoplasia, not hyper- the pathology of a peptic ulcer. Peptic ulcers
plasia. may present at any age, but are more com-
mon in patients 12-18 years than in very young
High-Yield Principles
62 Section I: General Principles  •  Answers

children. Additionally, it is not diagnosed by compromise breathing. Similarly to congeni-


99mTc-pertechnetate scanning, but would be tal diaphragmatic hernias, this can also cause
diagnosed by endoscopy. pulmonary hypoplasia, although usually not as
severe.
Answer C is incorrect. Ingestion of foreign
objects occurs frequently in young children Answer D is incorrect. Mediastinal shift does
and may cause mechanical damage to the in- occur in congenital diaphragmatic hernia, as
testinal lining. However, these tears are not de- the bowel invades the thorax and pushes the
tected with 99mTc-pertechnetate scanning. mediastinum to the right. However, this in it-
self is not a cause of death.
Answer E is incorrect. Abnormal or incom-
plete rotation of the intestine as it returns to
8. The correct answer is B. This patient has te-
the abdomen after physiologic herniation can
tralogy of Fallot (TOF). TOF is the most
trap and twist loops of bowel; twisting of these
common congenital cyanotic lesion and is
loops (volvulus) can result in obstruction of
characterized by four congenital cardiac ab-
Embryology

circulation and potentially lead to gangrene


normalities: (1) pulmonary stenosis, (2) ven-
of the affected segment of intestine. Most af-
tricular septal defect (VSD), (3) overriding
fected infants present within the first three
(right-arched) aorta, and (4) right ventricular
weeks of life with bile-containing vomit or
hypertrophy (due to outflow blockage). The
bowel obstruction. Bloody stool is not a princi-
severity of the condition is determined by the
pal sign of malrotation or volvulus.
degree of cyanosis, which is, in turn, most de-
pendent on the degree of pulmonary stenosis.
7. The correct answer is E. Pulmonary hypo-
In the most severe cases patients are cyanotic
plasia is the most common cause of death in
at birth, but most infants with TOF present
infants born with congenital diaphragmatic
at 4-6 months of age with recurrent bouts of
hernia. When the pleuroperitoneal folds fail
hypoxemia and cyanosis. The primary devel-
to fuse with the other components of the dia-
opmental defect is an abnormal anterior and
phragm during development, a hole is created
cephalad displacement of the conal septum,
that allows bowel into the thorax. The physical
resulting in an enlarged aorta and obstructed
compression of the bowels on the lung buds
pulmonary artery.
then prevents full development of the respira-
tory system (pulmonary hypoplasia). This leads Answer A is incorrect. The bulbus cordis be-
to a common presentation of dyspnea and cya- comes the smooth parts of the left and right
nosis and, unless it can be repaired surgically, ventricles, which are not primarily involved in
eventually leads to death. this lesion.
Answer A is incorrect. Cardiac tamponade is Answer C is incorrect. The left horn of the
most frequently associated with a pericardial sinus venosus gives rise to the coronary sinus.
effusion. This is not a common complication The right horn of the sinus venosus gives rise
of congenital diaphragmatic hernia. to the smooth part of the right atrium. Neither
of these structures is primarily involved in this
Answer B is incorrect. Cardiac abnormalities
lesion.
such as ventriculoseptal defects, vascular rings,
and coarctation of the aorta are associated with Answer D is incorrect. The primitive atria
congenital diaphragmatic hernias; however, become the trabeculated parts of the left and
they are not the most common cause of death. right atria, which are not primarily involved in
this lesion.
Answer C is incorrect. Eventration of the
diaphragm is a disorder in which all or part Answer E is incorrect. The primitive ventricle
of the diaphragmatic muscle is replaced by becomes the trabeculated parts of the left and
fibroelastic tissue. The weakened hemidia- right ventricles, which are not primarily in-
phragm is displaced into the thorax, which can volved in this lesion.
High-Yield Principles
Chapter 3: Embryology  •  Answers 63

9. The correct answer is A. Spermiogenesis is These cysts are potential sites of bladder ade-
the series of post-meiotic morphologic changes nocarcinomas.
that marks the final maturation of the sperm.
Answer A is incorrect. Bladder exstrophy is a
Spermatids are the 23, 1n cells that result from
congenital developmental anomaly resulting
secondary spermatocyte meiosis II comple-
from failure of the abdominal wall to close
tion. They undergo morphologic changes to
during embryogenesis. This causes the poste-
become mature sperm that include acrosome,
rior bladder wall to protrude through the lower
head, neck, and tail.
abdominal wall. It is not a complication of a
Answer B is incorrect. Secondary spermato- patent urachus.
cytes are 23, 2n cells that result from primary
Answer C is incorrect. Meckel diverticulum
spermatocytes completing meiosis I. Each pri-
is the most frequent congenital anomaly found
mary spermatocyte forms two secondary sper-
in the gastrointestinal tract, and it is caused by
matocytes, each going on to form two sperma-
persistence of the omphalomesenteric duct.
tids.
It is often asymptomatic but can present with

Embryology
Answer C is incorrect. At no point during rectal bleeding, volvulus, intussusception, and
male gametogenesis is there a haploid cell obstruction. Meckel diverticulum can be re-
with 46 chromosomes. membered by the “rule of 2s”: found in 2%
of the population, commonly presents around
Answer D is incorrect. Both primordial germ
age 2 years, measures 2 inches long, can be
cells in the testes, which are dormant until
found 2 feet from the ileocecal valve, and of-
puberty, and type A spermatogonia, which de-
ten contains 2 types of epithelia (gastric and
velop at puberty, are 46, 2n cell types. A type A
pancreatic).
spermatogonium perpetuates itself to provide a
constant supply of sperm cells; it also differen- Answer D is incorrect. Polycystic kidney dis-
tiates into type B spermatogonia. ease is a hereditary disease characterized by
bilateral development of multiple cysts in the
Answer E is incorrect. Primary spermatocytes
renal parenchyma, ultimately leading to re-
are 46, 4n cells that result from the DNA repli-
nal failure. There are two forms of polycystic
cation of type B spermatogonia.
kidney disease: adult autosomal dominant
10. The correct answer is B. The portion of the (ADPKD) and childhood autosomal reces-
bladder that extends into the umbilical cord is sive (ARPKD). They are caused by distinct
the allantois, an extraembryonic cavity within genes. ADPKD has been mapped to the PKD1
the body stalk that projects onto the cloaca and PKD2 genes, which code for the integral
(the future bladder), and regresses prior to membrane proteins polycystin-1 and poly-
birth as the bladder descends into the pelvis. cystin-2, respectively. ARPKD is caused by
It gives rise to the urachus, which later degen- mutations in the PKHD1 gene, which codes
erates into a fibrous structure running along for a large protein called fibrocystin. Neither
the anterior abdominal wall extending to the ADPKD nor ARPKD is a complication of a
umbilicus known as the median umbilical patent urachus.
ligament. Failure of the urachus to completely Answer E is incorrect. Ectopic kidney is a
degenerate (persistent urachus) results in a condition that arises from development of a
number of clinical complications, depending kidney at an unusual anatomic location. Most
on the degree of patency of the urachus. A to- ectopic kidneys are found either within the
tally patent urachus creates a fistulous urinary pelvis or just above the pelvic brim. Function-
tract between the bladder and the umbilicus. ally, these kidneys are normal; however, the
At other times, only the umbilical end, the ureter may kink, leading to urinary flow stagna-
bladder end, or the central urachal region re- tion that predisposes to recurrent bacterial in-
mains patent, which gives rise to urachal cysts. fections. Ectopic kidney is not a complication
of a patent urachus.
High-Yield Principles
64 Section I: General Principles  •  Answers

11. The correct answer is A. This patient most Answer E is incorrect. Incomplete separation
likely has an annular pancreas. Normally the of the esophagus and laryngotracheal tube re-
ventral bud of the pancreas rotates around the sults in a tracheoesophageal fistula. In its most
duodenum to fuse with the dorsal bud. The common form, the tracheoesophageal septum
ventral bud forms the uncinate process and is deviated posteriorly and the esophagus ends
part of the head of the pancreas, and the duct in a blind pouch connected to the trachea.
of the ventral bud becomes the main pancre- The fetus cannot swallow amniotic fluid and
atic duct. Rarely, a bifid ventral bud grows polyhydramnios may occur during pregnancy.
around the duodenum in both directions, Newborns appear healthy at first and swallow
forming a ring. Especially in the setting of in- normally, but quickly begin regurgitating fluid
flammation or malignancy, this ring can block through the nose and mouth and enter respira-
movement of food through the duodenum; tory distress. The defect can be repaired surgi-
such blockage causes epigastric pain, postpran- cally with high success rates.
dial fullness, nausea, and vomiting. Onset of
Embryology

symptoms can occur any time between infancy 12. The correct answer is B. The patient has an
and adulthood, or not at all. Blockage is also omphalocele, which results from failure of
associated with the “double-bubble sign” in closure of the anterior abdominal wall. In
the radiograph: the stomach is dilated proximal this midline abdominal wall defect, the herni-
to the blockage point and the duodenum is di- ated viscera are covered by a membrane con-
lated distal to it. sisting of the amniotic membranes, Wharton
jelly, and peritoneum. Between 50% and 70%
Answer B is incorrect. During normal devel-
of children with omphalocele have additional
opment, the lumen of the duodenum is ob-
congenital anomalies (including cardiac de-
structed by overgrowth of endothelial cells and
fects and genitourinary malformations such
then restored as these cells recede. Failure of
as bladder exstrophy), which are also thought
this process results in duodenal atresia. The
to be related to ventral closure defects. The
signs and symptoms of duodenal atresia are
pathogenesis of omphalocele is believed to be
very similar to those seen with an annular pan-
sporadic defective closure of the abdominal
creas, but they always present within hours af-
wall secondary to malrotation of the midgut
ter birth. Vomit containing bile and the radio-
derivatives during the 10th week of embryonic
graphic double bubble sign are indicative of
development. Alternative theories include ab-
duodenal atresia in an infant.
normal persistence of the primitive body stalk
Answer C is incorrect. Physiological midgut and the failure of body wall closure secondary
herniation occurs at the beginning of the sixth to incomplete lateral body wall migration. The
week of embryogenesis, when the midgut her- incidence of omphalocele is 1:5000 live births,
niates into the proximal umbilical cord. Nor- and it is most commonly associated with ex-
mally, it returns to the abdomen during the tremes of maternal age (<20 or >40 years of
tenth week. Failure to return to the abdomen age). It may be associated with a chromosomal
results in omphalocele, in which a portion of abnormality if there is only herniation of the
the newborn’s abdominal contents remain out- small bowel or the liver.
side the abdomen, covered by the umbilical
Answer A is incorrect. Ethanol toxicity is not
cord epithelium. This condition requires im-
associated with abdominal wall defects. Etha-
mediate surgical repair.
nol exposure during embryogenesis is associ-
Answer D is incorrect. Hypertrophy of the py- ated with fetal alcohol syndrome, which in-
loric muscles results in blockage of the diges- cludes mental retardation and a typical facies
tive pathway. Infants with congenital hyper- characterized by a smooth philtrum, thin up-
trophic pyloric stenosis generally present with per lip, and small palpebral fissures.
nonbilious projectile vomiting soon after birth.
Answer C is incorrect. Maternal folate defi-
ciency is not associated with abdominal wall
High-Yield Principles
Chapter 3: Embryology  •  Answers 65

defects. Folate deficiency has been associated Answer A is incorrect. F508 deletion in the
with a number of neural tube defects, includ- CFTR gene is the most common cause of cys-
ing anencephaly and spina bifida. A myelo- tic fibrosis. Patients with cystic fibrosis have
meningocele would be located at the posterior frequent pulmonary infections, impaired mu-
side (superior if using fetal terminology). cous clearance, and poor growth secondary to
pancreatic involvement. Survival can extend
Answer D is incorrect. Omphalomesenteric
into the fourth and fifth decades in these pa-
vessel occlusion and consequent ischemia is
tients.
believed to be the cause of gastroschisis, an-
other congenital abdominal wall defect. This Answer B is incorrect. CAG tandem repeats
condition is a full-thickness defect in the ab- are found in Huntington disease, among oth-
dominal wall, and there is no protective mem- ers. Huntington is asymptomatic until the
brane with a variable amount of herniation patient’s third or fourth decade and is caused
of the intestines and parts of other abdominal by degeneration of the caudate and putamen.
organs. Unlike omphalocele, gastroschisis her- Patients present with gradually worsening cho-

Embryology
niation occurs to the right of the umbilicus, reiform (dance-like) movements, but not the
and is rarely associated with other congenital birth defects found in this patient.
anomalies. Children with this defect should be
Answer C is incorrect. This is fragile X syn-
given a warm moist occlusive dressing and re-
drome. Associated with CGG tandem repeats,
ceive immediate surgical intervention.
this syndrome is characterized by mental retar-
Answer E is incorrect. The ToRCHeS infec- dation, a large jaw, and large testes. Children
tions include Toxoplasmosis, Rubella, Cyto- with fragile X syndrome typically survive be-
megalovirus (CMV), Herpesvirus/HIV, and yond one year of age.
Syphilis. Congenital infection with one of the
Answer E is incorrect. Down syndrome can
ToRCHeS viruses is not associated with ab-
cause mental retardation and characteris-
dominal wall defects. Instead, common mani-
tic physical findings that include microge-
festations include fevers, hepatosplenomeg-
nia (a small chin), macroglossia, epicanthal
aly, jaundice, poor feeding, and intrauterine
folds, and a round face, but clenched fists and
growth restriction. Other specific signs include
“rocker-bottom” feet are classic for trisomy 18.
chorioretinitis, cataracts, and neural deafness
The survival of a child affected by Down syn-
(rubella); intracranial calcifications (toxoplas-
drome is approximately 50 years.
mosis); rhinitis, “blueberry-muffin” skin le-
sions, and interstitial keratitis (syphilis); cere- 14. The correct answer is E. This individual is
bral calcifications (CMV); and vesicular skin most likely suffering from a VSD. Infants with
lesions (herpes simplex virus). VSDs are generally not cyanotic at birth, but
can become cyanotic if a large defect is left
13. The correct answer is D. This newborn has
untreated. The majority of small VSDs involv-
Edwards syndrome, or trisomy 18. Affected
ing the membranous portion of the septum
children are born with clenched fists, “rocker-
resolve on their own. With larger defects in-
bottom” feet, micrognathia (a small lower jaw),
volving the muscular portion of the septum,
congenital heart disease, and mental retar-
higher pressures in the left ventricle initially
dation. The survival rate of <1 year is similar
cause a shunt of blood from the left ventricle
to that of trisomy 13 (Patau syndrome), from
to the right ventricle during systole. Over time,
which it should be distinguished. Trisomy of
due to the volume and pressure overload on
chromosome 13, or Patau syndrome, is charac-
the right-sided circulation, the child will de-
terized by a constellation of findings including
velop increased resistance in the pulmonary
mental retardation, microphthalmia, micro-
circulation (termed pulmonary hypertension),
cephaly, cleft lip/palate, abnormal forebrain
which increases the pressure in the right ven-
structures, polydactyly, and congenital heart
tricle to a point at which the shunt reverses
disease.
High-Yield Principles
66 Section I: General Principles  •  Answers

and blood flows from the right ventricle to the of deoxygenated blood with oxygenated blood,
left through the VSD. This shunt reversal is causing cyanosis in the neonate. The cyanosis
termed Eisenmenger syndrome. As described is not as pronounced as in transposition of the
in the vignette, these children present early great arteries, and greatly depends on the anat-
with a harsh holosystolic murmur at the left omy of the individual’s defect.
lower sternal border. The intensity of the mur-
mur is inversely proportional to the size of the 15. The correct answer is B. The vessel labeled
defect. C is most likely the umbilical vein, which has
the highest oxygen saturation level in the fe-
Answer A is incorrect. Atrial septal defect
tus. It carries blood enriched in oxygen from
(ASD) is a communication between the right
the placenta to the fetus. Its oxygen satura-
and left atria. Patients may remain asymptom-
tion is typically between 30 and 35 mm Hg.
atic or manifest symptoms of right-sided heart
Soon after birth, the umbilical vein becomes
failure, depending on the size of the defect.
dysfunctional as the neonate makes the tran-
Cyanosis may occur if there is a patent fora-
Embryology

sition from fetal circulation to that found in


men ovale with right-to-left shunting, but is
adult anatomy. In place of this vein is a fibrous
not systematically seen. On exam, patients
structure, the ligamentum teres hepatis, or the
with ASD have an ejection systolic murmur
round ligament of the liver. It extends from the
with fixed splitting of S2.
umbilicus to the transverse fissure of the liver,
Answer B is incorrect. Tetralogy of Fallot is where it joins the ligamentum venosum, thus
characterized by pulmonary artery stenosis, effectively separating the liver into its right and
right ventricular hypertrophy, an overriding left lobes. Recanalization of this vein occurs
aorta, and a VSD. Infants with cardiac defects under the pathologic condition of portal hy-
are typically cyanotic at birth. The pulmo- pertension associated with liver cirrhosis.
nary artery stenosis reduces the caliber of the
Answer A is incorrect. The falciform ligament
outflow tract, causing the pressure in the right
is a developmental remnant of the ventral mes-
ventricle to be unusually high. As a result, de-
entery of the fetus, thus it is a peritoneal fold
oxygenated blood is shunted from the right to
enclosing the round ligament of the liver an-
the left side of the heart through the VSD.
teriorly and the ligamentum venosum posteri-
Answer C is incorrect. Transposition of the orly.
great arteries is a congenital defect in which
Answer C is incorrect. The ligamentum veno-
the pulmonic artery exits from the left ventri-
sum is a fibrous structure that is derived from
cle and the aorta exits from the right ventricle.
the ductus venosus in the fetal circulatory sys-
In this defect there are essentially two closed
tem. The ductus venosus is a shunt that con-
circuits with no way for oxygenated blood to
ducts oxygen-rich blood from the umbilical
reach the systemic circulation outside of the
vein into the inferior vena cava. It may be as-
patent ductus arteriosus (PDA). Efforts to keep
sociated with the round ligament of the liver,
the PDA open are necessary to keep the baby
coursing through the fissure that demarcates
alive so it can be taken to the operating room
the boundaries between the left and caudate
to have the defect corrected. Neonates with
lobes of the liver. Most often, it is found at-
this condition are typically cyanotic at birth,
tached to the left branch of the portal vein in
have shortness of breath, and feed poorly.
the porta hepatis.
Since the defect is so severe, it is typically dis-
covered in the first week of life. Answer D is incorrect. There are two medial
umbilical ligaments in the adult. They course
Answer D is incorrect. Truncus arteriosus is
longitudinally on the deep surface of the an-
a congenital defect in which there is a large
terior abdominal wall underneath the medial
VSD and a common trunk that eventually sep-
umbilical folds. The medial umbilical liga-
arates into the aorta and pulmonary arteries.
ments represent vestigial remnants of the fetal
Due to the anatomy there is significant mixing
High-Yield Principles
Chapter 3: Embryology  •  Answers 67

umbilical arteries. The paired umbilical arter- Answer D is incorrect. This describes the de-
ies have a very low oxygen saturation level (ves- fect in Hirschsprung disease, which manifests
sel D), as they carry blood depleted of oxygen as severe constipation and an inability to pass
from the fetus back to the placenta. meconium. Hirschsprung disease would not
account for the symptoms described in this
Answer E is incorrect. The median umbilical
scenario.
ligament is a single ligament that runs longi-
tudinally on the deep surface of the anterior Answer E is incorrect. A failure of duodenal
abdominal wall between the medial umbili- recanalization may give rise to duodenal atre-
cal ligaments in the adult, extending from the sia. Duodenal atresia is associated with Down
apex of the bladder to the umbilicus. It is a ves- syndrome, and it is often marked by a “double
tigial remnant of the embryonic urachus. bubble” sign on abdominal radiographs. Duo-
denal atresia would result in polyhydramnios
16. The correct answer is A. The presentation rather than oligohydramnios.
described here is consistent with Potter syn-

Embryology
drome, one cause of which is bilateral renal 17. The correct answer is B. The muscles that ele-
agenesis. The renal parenchyma (except for vate the palate are derived from branchial arch
the nephrons) is derived from the ureteric bud 3 (the stylopharyngeus) and branchial arch 4
(recall that the nephrons arise from mesoderm (the levator veli palatini). These are innervated
surrounding the ureteric bud). A failure of ure- by cranial nerves IX and X, respectively.
teric bud maturation would result in a fetus
Answer A is incorrect. The first branchial arch
without kidneys. An absence of kidneys would
generates “M” muscles: muscles of Mastica-
lead to oligohydramnios, as the fetus would
tion (teMporalis, Masseter, Medial and lateral
be unable to excrete urine into the amniotic
pterygoids) and the Mylohyoid. The second
sac. This, in turn, would lead to compression
arch gives rise to “S” muscles: Stapedius, Stylo-
of the fetus by the uterine wall, causing limb
hyoid, and facial expression muscles. None of
deformities, abnormal facies, and wrinkly skin.
these muscles is involved in palatal elevation.
Death would occur shortly after birth unless
an appropriate kidney donor could be found. Answer C is incorrect. Although branchial
arch 4 does give rise to the levator veli pala-
Answer B is incorrect. Anencephaly may re-
tini, branchial arch 6 gives rise to the intrinsic
sult from a failure of the rostral neural tube
muscles of the larynx (except the cricothyroid,
to close. Anencephalic infants are born with a
which is a fourth arch derivative). These mus-
marked reduction in fetal brain tissue and usu-
cles are not involved in elevating the palate.
ally an absence of the overlying skull. These
infants are unable to swallow amniotic fluid in Answer D is incorrect. The first branchial
utero, so their mothers’ pregnancies are usu- cleft gives rise to the external auditory meatus,
ally marked by polyhydramnios (too much am- and the second, third, and fourth clefts are
niotic fluid) rather than oligohydramnios (too obliterated during development. The clefts are
little). formed from ectoderm and could not give rise
to muscles, which are derived from mesoderm.
Answer C is incorrect. Failure of development
of the tracheoesophageal (TE) septum is the Answer E is incorrect. Branchial pouch 3
cause of a TE fistula. There are several variants gives rise to the thymus (ventral wings) and in-
of TE fistula, the most common of which is a ferior parathyroid glands (dorsal glands), and
blind upper esophagus with the lower esopha- the fourth branchial pouch gives rise to the su-
gus having an anomalous connection to the perior parathyroids. These are not involved in
trachea. TE fistulas commonly result in poly- palatal elevation. Remember that pouches give
hydramnios rather than oligohydramnios, and rise to endoderm-derived tissue, and arches
they are unlikely to cause the other findings in give rise to mesoderm-derived tissue such as
this infant. muscle. Use the mnemonic “CAP” to remem-
High-Yield Principles
68 Section I: General Principles  •  Answers

ber that Clefts, Arches, and Pouches give rise the promoter region of uridine 5′-diphospho-
to ecto-, meso-, and endoderm, respectively. glucuronosyltransferase, leading to diminished
expression of the gene. Patients with Gilbert
18. The correct answer is A. This child has a cleft syndrome develop a mild unconjugated hyper-
lip, which is most often caused by failure of bilirubinemia but usually are asymptomatic
the maxillary prominence to fuse with the me- and have a normal life expectancy.
dial nasal prominence. Cleft lip may occur
Answer B is incorrect. Crigler-Najjar syn-
unilaterally or bilaterally and represents the
drome type 1 is caused by a complete defi-
most common congenital malformation of the
ciency in uridine 5’-diphosphoglucuronosyl-
head and neck. This commonly occurs with a
transferase, the hepatic enzyme necessary to
cleft palate.
conjugate bilirubin. This disorder produces
Answer B is incorrect. Abnormal development a severe unconjugated (indirect) hyperbiliru-
of the third and fourth branchial pouches gives binemia that causes death within the first few
rise to DiGeorge syndrome, which results in years of life. The patient in this case, however,
Embryology

thymic aplasia and failure of parathyroid devel- has a conjugated hyperbilirubinemia, suggest-
opment. ing an obstructive cause and ruling out Cri-
Answer C is incorrect. The mandibular and gler-Najjar syndrome.
maxillary bones are typically normally devel- Answer C is incorrect. Physiologic jaundice
oped in a cleft lip. Abnormal development of refers to the mild unconjugated (indirect)
these bones typically causes various facial dys- hyperbilirubinemia that affects nearly all
ostoses. newborns because of the greater turnover of
Answer D is incorrect. The third pharyngeal neonatal RBCs and the decreased bilirubin
arch forms the hyoid bone, stylopharyngeus clearance in the first few weeks of life. The
muscle, and glossopharyngeal nerve, which are peak total serum bilirubin occurs between 72
not altered in a simple cleft lip. and 96 hours of age and resolves within the
first few weeks of life. Often phototherapy is
19. The correct answer is E. The patient is pre- used to hasten resolution. This patient has a
senting with congenital extrahepatic biliary severe conjugated hyperbilirubinemia that
atresia. Descriptions of a pure elevation in di- cannot be explained by normal neonatal physi-
rect (conjugated) bilirubin strongly suggest ologic jaundice.
an obstructive etiology, as the liver is able to Answer D is incorrect. Biliary atresia is a rare
effectively conjugate bilirubin but fails to ex- condition whose cause is not entirely known; it
crete it into the small intestine. The absence of is not inherited in an autosomal dominant pat-
bilirubin in the small bowel results in acholic tern. Hereditary spherocytosis is an example
stools, whereas increased renal excretion of of an autosomal dominant condition that can
conjugated bilirubin causes a darkening of the cause jaundice and hyperbilirubinemia sec-
urine. Congenital extrahepatic biliary atresia ondary to hemolytic anemia. This autosomal
occurs when the developing bile ducts close dominant condition is due to mutations in
completely and fail to recanalize. Surgical spectrin or ankyrin causing RBC membrane
therapy of biliary atresia involves anastomosis defects that make the cells more fragile to he-
of the small bowel directly to intrahepatic bile molysis. Peripheral blood smears show small
ducts, a maneuver known as Kasai’s procedure, RBCs without central pallor, and diagnosis can
which is appropriate for the 10% of patients be confirmed with the osmotic fragility test.
with limited disease. Liver transplantation con- Unlike this case, hereditary spherocytosis usu-
tinues to be the best chance of survival for the ally presents later in life, with a mixed hyper-
remaining patients. bilirubinemia and normal stools.
Answer A is incorrect. Gilbert syndrome is
a benign disorder caused by a mutation in
High-Yield Principles
Chapter 3: Embryology  •  Answers 69

20. The correct answer is B. The uterine fundus Answer C is incorrect. The pancreas produces
is palpable above the umbilicus at 20 weeks’ insulin, glucagon, and digestive enzymes. It
gestational age. Another landmark to keep plays no role in hematopoiesis.
in mind is that the uterine fundus is palpable
Answer D is incorrect. The thymus is the site
above the pubic symphysis at 12 weeks’ gesta-
of early development of the immune system; it
tional age. The liver is the predominant site of
is not a site of hematopoiesis.
hematopoiesis at weeks 6-30 of gestation.
Answer E is incorrect. The yolk sac is the pre-
Answer A is incorrect. The bone marrow is
dominant site of hematopoiesis between fetal
the predominant site of hematopoiesis begin-
weeks three and eight.
ning around week 28 and remains so through-
out adult life.

Embryology
This page intentionally left blank
Chapter 4

Microbiology

71
High-Yield Principles
72 Section I: General Principles  •  Questions

Q u e st i o n s

1. A 30-year-old sexually active woman presents (A)


Blastomyces dermatitidis
with a painful vesicle on her external genita- (B)
Coccidioides immitis
lia and bilateral inguinal lymphadenopathy. A (C)
Cryptococcus neoformans
Tzanck smear from the vesicle is negative, and (D)
Histoplasma capsulatum
a Venereal Disease Research Laboratory assay (E)
Paracoccidioides brasiliensis
is also negative. Which of the following medi-
cations would be most appropriate for this pa- 3. A 17-year-old boy visits his physician with com-
tient? plaints of recurrent bouts of dizziness, palpita-
tions, and joint pain. He went on a summer
(A) Acyclovir
hiking trip in eastern Massachusetts about six
(B) Ceftriaxone
months ago but does not recall getting a tick
(C) Foscarnet
Microbiology

bite and notes no rashes. The ECG is shown


(D) Ribavirin
in the image. What is the most likely diagnosis
(E) Vancomycin
of this patient’s symptoms?
2. A 51-year-old man living near St. Louis, Mis- (A) Brugada syndrome
souri presents to his primary care physician (B) Chagas disease
with recent-onset productive cough, pleuritic (C) Hypertrophic cardiomyopathy
chest pain, and a fever of 39.1°C (102.4°F). (D) Lyme disease
He recently returned from a business trip to (E) Third-degree heart block
Phoenix, Arizona. A sample of the man’s puru-
lent sputum is sent for analysis, which reveals 4. A 30-year-old woman complains of a nonpro-
yeast cells up to 15 µm in diameter. The pa- ductive cough that has developed over the past
thologist is able to identify several dividing 10 days. She reports feeling achy, and having
yeast organisms. Direct fluorescent antibody a sore throat and headaches. X-ray of the chest
staining (see image) is notable for large, broad- demonstrates patchy bilateral interstitial infil-
based budding from mother cells. What fungal trates. After work-up, the patient is diagnosed
species is responsible for this man’s illness? with Mycoplasma pneumoniae pneumonia.
Which of the following is associated with the
causative organism?
(A) Growth on buffered charcoal yeast extract
media
(B) IgM cold agglutinins
(C) Phyocyanin production
(D) Polysaccharide capsule
(E) Reticulate bodies

Courtesy of Dr. William Kaplan, Centers for Disease Control


and Prevention.
High-Yield Principles
Chapter 4: Microbiology  •  Questions 73

5. A homeless 37-year-old woman with HIV in-


fection comes to the clinic with a four-week
history of worsening hemiparesis, visual field
deficits, and cognitive impairment. The pa-
tient’s CD4+ count is 22/mm3. MRI shows
several hyperintensities on T2-weighted im-
ages that do not enhance with contrast and are
not surrounded by edema. A lumbar puncture
shows a normal opening pressure, and cerebro-
spinal fluid analysis shows a mildly elevated
protein level and the presence of myelin basic
protein, with a mild mononuclear pleocytosis.
Which of the following entities is most likely Courtesy of the Centers for Disease Control and Prevention.
responsible for this patient’s clinical picture?

Microbiology
(A) Cortical tuberculoma
(B) Cytomegalovirus encephalitis (A) Inhibition of cell wall synthesis
(C) JC virus (B) Inhibition of DNA polymerase
(D) Primary central nervous system lymphoma (C) Inhibition of genome uncoating
(E) Toxoplasmosis (D) Inhibition of nucleoside reverse transcrip-
tase
6. A family who recently emigrated from Roma- (E) Inhibition of protein synthesis
nia brings their 7-year-old child to the pedia-
trician with complaints of conjunctivitis and 8. A 38-year-old man comes to the emergency
periorbital swelling. The child has had cough- department complaining of cyclic fevers and
ing with a runny nose and high fever for three headaches. The fevers began about one week
days. Small lesions with blue-white centers are ago; two weeks ago the patient returned from
seen in his oral cavity. Which of the following a trip to Africa. Physical examination reveals
is the most likely cause of this child’s symp- hepatosplenomegaly. Imaging of the brain
toms? shows signs of significant cerebral involve-
ment. Which of the following parasites most
(A) Diphtheria likely caused this patient’s symptoms?
(B) Pertussis
(C) Roseola (A)
Plasmodium falciparum
(D) Rubella (B)
Plasmodium malariae
(E) Rubeola (C)
Plasmodium ovale
(D)
Plasmodium vivax
7. A 5-year-old girl is brought to her pediatrician (E)
Plasmodium knowlesi
because of an eight-day history of a painful
rash confined to her flank. Physical examina-
tion reveals the crusted lesion shown in the
image. Which of the following describes the
mechanism of action of the treatment for this
lesion?
High-Yield Principles
74 Section I: General Principles  •  Questions

9. A 15-year-old boy presents to the pediatrician 11. Influenza virus type A usually produces a mild,
with a two-day history of fever and headache. self-limited febrile illness in the general popu-
The patient is unable to touch his chin to his lation. However, worldwide epidemics have
chest when asked to do so. He also asks that occurred at different times in history due to
the lights in the room be turned down. In ad- rapid changes in viral genetic makeup. Which
dition to performing a lumbar puncture to ob- of the following is the most important reason
tain cerebrospinal fluid (CSF), the physician why these sporadic worldwide epidemics oc-
begins empiric treatment. The CSF is sent for cur?
analysis and culture. The patient’s condition
(A) Antigenic drift
improves over the next week. In the meantime,
(B) Antigenic shift
bacterial and fungal culture results are nega-
(C) Hemagglutinin develops the ability to de-
tive. Which of the following is the most likely
stroy a component of mucin, becoming
result of the CSF analysis?
more infectious
(D) Neuraminidase develops the ability to at-
Microbiology

Choice Pressure Lymphocyte Protein Sugar


tach to sialic acid receptors, becoming
count more infectious
A normal normal normal normal
(E) RBCs agglutinate with certain strains
B normal normal normal
C normal or normal normal 12. A 66-year-old woman who recently emigrated
D from Mexico comes to the physician be-
E cause she has begun to have seizures. A test
for anticysticercal antibodies is positive. A T1
Reproduced, with permission, from USMLERx.com. weighted, non-contrast MRI of the head is
shown in the image below. Which of the fol-
lowing organs or tissues is most likely to have
(A) A
similar lesions?
(B) B
(C) C
(D) D
(E) E

10. Oncogenic viruses act through a variety of


mechanisms. Some introduce oncogenes di-
rectly into host cells, while others force cells to
repeatedly undergo cycles of proliferation that
eventually become unregulated. Still others in-
troduce oncogenic potential by manipulating
chromosomal structure through deletions or
translocations. Which of the following viruses
causes neoplasia by inactivating tumor sup-
pressor genes such as p53 and Rb?
(A) Epstein-Barr virus
(B) Hepatitis C virus Courtesy of Dr. Per-Lennart Westesson, University of Roches-
(C) Human immunodeficiency virus ter Medical Center.
(D) Human papillomavirus
(E) Human T-cell lymphotropic virus type 1
High-Yield Principles
Chapter 4: Microbiology  •  Questions 75

(A) Bladder 15. A 27-year-old woman presents to her physi-


(B) Bone cian complaining of fever, chills, and flu-like
(C) Kidney cortex symptoms. A sputum culture at 25°C (77°F) is
(D) Skeletal muscle shown in the image. Which of the following is
(E) Small bowel most likely to be elicited on further question-
ing?
13. A young girl living in rural New Mexico is
brought to her pediatrician with complaints
of fever, cough, and fatigue for the past two
weeks. The physician notices that the patient
is having intermittent bouts of many coughs in
a single breath, followed by a deep inspiration.
The parents report that this pattern of cough-
ing had started in the past two days. The phy-

Microbiology
sician informs them that their daughter will
most likely recover with only supportive care.
However, she wants to confirm his diagnosis. A
throat swab is sent for culture for a specific or-
ganism. Which of the following culture media
will be used?
(A) Bordet-Gengou medium
(B) Charcoal yeast extract with iron and cyste- Courtesy of Dr. Hardin, Centers for Disease Control and
Prevention.
ine
(C) Chocolate agar with factors V and X
(D) Loeffler medium (A) A recent trip to Namibia
(E) Thayer-Martin medium (B) A recent trip to New Mexico
(C) A recent trip to Ohio
14. A worried mother brings her infant to the
(D) Recent hiking in wooded areas
emergency department because he appears to
(E) Recent work in her large rose garden
be unable to swallow and continues to choke
on his formula. On physical examination, the 16. At birth, a newborn is noted to be unrespon-
physician notes generalized muscle weakness. sive to verbal stimulation from the doctors,
On further questioning, the patient’s mother nurses, and his parents. A routine physical ex-
says that she recently started sweetening the amination of the child reveals a split S2 heart
baby’s food with honey. Which of the follow- sound with an accentuated P2 component.
ing is a characteristic of the organism most The newborn has bounding pulses with a wide
likely responsible for this infant’s symptoms? pulse pressure. After a week the newborn’s par-
(A) Production of IgA protease ents notice that he has developed shortness of
(B) Production of exotoxin A breath and respiratory distress. What pathogen
(C) Production of lecithinase did the mother contract during her pregnancy
(D) Production of lipopolysaccharide that could explain the newborn’s current con-
(E) Production of spores that can only be dition?
killed by autoclaving (A) Cytomegalovirus
(B) Herpes simplex virus 2
(C) Rubivirus
(D) Toxoplasma gondii
(E) Treponema pallidum
High-Yield Principles
76 Section I: General Principles  •  Questions

17. A 45-year-old man presents to the clinic com- (C) Erythromycin


plaining of several weeks of vague abdominal (D) Prompt replacement of water and electro-
discomfort and early satiety. The physician or- lytes; tetracyclines shorten the disease’s
ders upper GI endoscopy as part of his workup. course
During the study, mucosal rigidity and hyper- (E) Supportive care only, without antibiotics
plasia are seen in the stomach, and a biopsy
is taken from the affected area. Microscopic 19. A 65-year-old man with a history of viral hepa-
analysis of the biopsy specimen shows sheets of titis presents to his primary care physician with
atypical lymphocytes. The organism believed complaints of early satiety, a 4.5-kg (10-lb)
to be associated with this condition is best de- weight loss over three months, upper abdomi-
scribed by which set of laboratory results? nal pain, and yellowing of his eyes. The pa-
tient says he has lived in Rochester, New York
(upstate) for his entire life, has not traveled
Choice Urease Catalase Oxidase outside of the country, and received two blood
Microbiology

A negative negative negative


transfusions in the early 1970s following an au-
tomobile accident. Work-up reveals extensive
B negative negative positive macronodular cirrhosis with a 2-×2-cm mass
C negative positive
in his liver. Which of the following viral in-
negative
fections is most likely to result in this patient’s
D negative positive positive current presentation?
E positive negative negative (A) Cytomegalovirus
F
(B) Hepatitis A
positive positive negative
(C) Hepatitis C
G positive positive positive (D) Hepatitis E
(E) HIV
Reproduced, with permission, from USMLERx.com.
20. A neonate with purulent umbilical discharge
for one day presents with fever, irritability, and
(A) A diffuse flushing. One day later she is covered
(B) B in large, fluid-filled blisters that rupture easily,
(C) C leaving raw red areas beneath. Blood cultures
(D) D are taken, which within 24 hours grow an or-
(E) E ganism that is subsequently Gram stained with
(F) F the results shown below. The skin symptoms
(G) G observed in this case are due to the involve-
ment of which of the following intercellular
18. A 36-year-old man comes to the physician structures?
because he is experiencing abdominal pain,
vomiting, and a non-bloody diarrhea. He last
ate chicken and rice about four hours ago at
a Chinese restaurant. He has no other symp-
toms. Which of the following treatments
should this man receive?
(A) Bismuth subsalicylate, metronidazole, and
amoxicillin
(B) Ciprofloxacin
High-Yield Principles
Chapter 4: Microbiology  •  Questions 77

Courtesy of Dr. Richard Facklam, Centers for Disease Control Courtesy of Drs. E. Arum and N. Jacobs, Centers for Disease

Microbiology
and Prevention. Control and Prevention.

(A) Desmosomes (A) Azithromycin


(B) Gap junctions (B) Ceftriaxone
(C) Hemidesmosomes (C) Fluconazole
(D) Intermediate junctions (D) Penicillin
(E) Tight junctions (E) Vancomycin

21. A 5-year-old boy develops diarrhea after eat- 23. A 47-year-old woman comes to the clinic com-
ing at a fast-food restaurant. The following plaining of fever and malaise. She reports hav-
day, his mother notices that he seems lethargic ing severe headaches associated with some
and brings him to the urgent care center. His nausea and vomiting, over the past few days.
blood pressure is 150/90 mm Hg. Laboratory Her urine has been exceptionally dark for the
tests show a hemoglobin level of 9 g/dL, plate- past few days. The patient is mildly jaundiced
let count of 40,000/mm3, and creatinine level with scleral icterus. Based on these symptoms
of 2.8 mg/dL. What is the most likely cause of the physician suspects hepatitis B and draws
this patient’s condition? blood for serologic testing for hepatitis B mark-
ers. If the patient had unprotected intercourse
(A) Campylobacter
during this infection, the presence of which of
(B) Escherichia coli
the following would be most concerning for
(C) Rotavirus
her partner?
(D) Shigella
(E) Vibrio cholerae (A) Hepatitis B e antibody
(B) Hepatitis B e antigen
22. A 32-year-old man presents to his doctor with (C) Hepatitis B surface antibody
painful urination and a purulent urethral dis- (D) IgG hepatitis B core antibody
charge. The discharge material is cultured, (E) IgM hepatitis B core antibody
and a sample from the culture is stained with
Giemsa and is shown in the image. Which of
the following is the treatment of choice for this
infection?
High-Yield Principles
78 Section I: General Principles  •  Questions

24. A 1-year-old girl presents to the emergency (C) Hageman factor


room because of a three-day history of cough- (D) C5a
ing attacks that are occasionally followed by (E) Interleukin-1
episodes of vomiting. Her parents are espe- (F) Nitric oxide
cially concerned because sometimes she be-
comes blue after an episode. She has markedly 27. A 30-year-old man from Mexico comes to the
injected sclera but no discharge. She is afe- physician because of a two-day history of a fe-
brile and has no other symptoms. Laboratory ver of 38.8º C (101.8º F), a sore throat, and
tests show a WBC count of 25,000/mm3 with shortness of breath. On physical examination,
marked lymphocytosis. A bacterial infection is the patient is found to have cervical adenopa-
diagnosed. Which of the following is a result of thy and a thick, gray, tissue-like material cover-
the exotoxin produced by the organism caus- ing his tonsils. Which of the following types of
ing this child’s symptoms? culture should be used to determine the organ-
ism infecting this patient?
(A) Blocked release of acetylcholine into the
Microbiology

synaptic cleft (A) Löwenstein-Jensen agar


(B) Formation of a pore in the plasma mem- (B) Bordet-Gengou agar
brane (C) Chocolate agar with factors V and X
(C) Inactivation of elongation factor 2, a pro- (D) Loeffler medium, blood agar, and tellurite
tein involved in translation plate
(D) Inhibition of G proteins to increase cAMP (E) Thayer-Martin media
(E) Release of lipopolysaccharide-lipid A
28. A 12-year-old boy presents to the emergency
25. A 50-year-old man develops profuse, non- room five days after returning from a camping
bloody, watery diarrhea while working as trip in Virginia. He complains of a fever, head-
an aid worker in Bangladesh. He arrived in ache, and muscle aches for several days, and
the area two days ago. A stool smear shows recently developed a rash on his wrists, palms,
no WBCs. His mucous membranes are dry, and legs that has since spread to his trunk, as
and his skin shows signs of tenting. He subse- seen in the image. Which of the following is
quently develops electrolyte abnormalities that the most appropriate treatment?
lead to cardiac and renal failure. What is the
mechanism of action of the exotoxin produced
by the most likely causative organism?
(A) Cleaves host cell rRNA
(B) Directly acts as an adenylate cyclase
(C) Inactivates elongation factor 2
(D) Permanently inactivates Gi
(E) Permanently activates Gs

26. A biotechnology company is developing a


small protein to block the cascade by which
allergens can cause shock. Patients often die
from vasodilation and massive edema; there-
fore, the new protein could lead to a life-saving
Courtesy of the Centers for Disease Control and Prevention.
drug. Which of the following molecules is the
most promising target to block the anaphylac-
tic pathway? (A) Ceftriaxone
(A) γ Interferon (B) Doxycycline
(B) IgE surface receptors
High-Yield Principles
Chapter 4: Microbiology  •  Questions 79

(C) Gentamicin
(D) Nystatin
(E) Penicillin

29. A 53-year-old obese man with poorly con-


trolled noninsulin-dependent diabetes mel-
litus presents with fever to 39.6°C (103.2°F),
jaundice, hypotension, and acute onset of right
upper quadrant pain. Right upper quadrant
imaging shows multiple gallstones and chole-
cystitis. Urgent cholecystectomy is performed,
and subsequent gall bladder fluid and blood
cultures grow aerobic, nonlactose-fermenting,
Courtesy of the Centers for Disease Control and Prevention.
oxidase-positive, gram-negative rods. Blood

Microbiology
tests show:
Hematocrit: 29% (A) Bloody and foul-smelling vaginal discharge
WBC count: 14,700/mm3 (B) Pelvic pain
Platelet count: 76,000/mm3 (C) Profuse, frothy vaginal discharge
International Normalized Ratio: 3.2 (D) Thick, white, cottage cheese-like vaginal
D-dimer: 8500 ng/mL discharge
Fibrinogen levels: low (E) Thin, gray-white, fishy-smelling vaginal
discharge
Microscopic inspection of peripheral blood
smear shows schistocytes and multiple helmet 31. A 40-year-old man goes on a camping vaca-
cells. Clinically, there is no evidence of active tion with his family. One day after swimming
bleeding. What is the most appropriate treat- in a freshwater lake near the campsite, he de-
ment for this patient’s coagulopathy? velops nausea and vomiting and starts to be-
(A) Amoxicillin have irrationally. His family takes him to the
(B) Aztreonam emergency department, where blood samples
(C) Fresh frozen plasma are taken and a spinal tap is performed. He is
(D) Vancomycin diagnosed with a rapidly progressing meningo-
(E) Vitamin K encephalitis and dies shortly thereafter. Which
of the following protozoa was most likely the
30. A 16-year-old girl complains of abnormal vagi- cause of the man’s illness?
nal discharge as well as itching, tenderness, (A)
Cryptosporidium species
and burning in the vulvovaginal area. On ex- (B)
Entamoeba histolytica
amination there is vulvar and vaginal erythema (C)
Leishmania donovani
and colpitis macularis. Results of a wet mount (D)
Naegleria fowleri
examination are shown in the image. Which (E)
Plasmodium falciparum
additional symptom would most likely be seen
in this patient?
High-Yield Principles
80 Section I: General Principles  •  Questions

32. A mother brings her 12-year-old daughter to an (C) Mosquito bites


outpatient clinic. The child complains of ach- (D) Saliva and respiratory secretions
ing pain localized to the joints of the extremi- (E) Sexual contact
ties. The mother recalls her daughter was sick
with a sore throat about a month ago, but re- 34. While hospitalized for treatment of an episode
covered completely without medical attention. of aspiration pneumonia, a 45-year-old man
The girl is admitted to the hospital for further begins to have episodes of severe non-bloody
examination and testing. A tissue biopsy is diarrhea and lower abdominal pain. Tempera-
taken, and the abnormal results are seen in the ture is 38°C (100.4°F). CT of the abdomen
image. Given the most likely diagnosis, what and pelvis with oral and intravenous contrast
finding would be expected in this patient on demonstrates marked diffuse colonic thicken-
cardiac physical examination as an adult? ing (see image). What is the pathophysiology
of the most likely cause of this patient’s condi-
tion?
Microbiology

Courtesy of Dr. Ed Uthman.

(A) A friction rub heard throughout the pre- Reproduced, with permission, from USMLERx.com.
cordium
(B) A harsh crescendo-decrescendo early sys-
tolic murmur heard at the right upper ster- (A) Directly damages the microvilli of the en-
nal border with radiation to the carotids terocytes but does not invade
(C) A late diastolic murmur heard best at the (B) Produces an exotoxin that can induce cyto-
apex kine release and cause hemolytic uremic
(D) A midsystolic click heard best at the apex syndrome
(E) An S4 gallop heard at the apex (C) Produces an exotoxin that increases the se-
cretory activity of enterocytes
33. A 23-year-old man from Kenya presents with (D) Produces an exotoxin that kills enterocytes
night sweats, fevers, oliguria, and a large sub- (E) Produces both heat-stable and labile toxins
mandibular mass. Biopsy of the mass shows an that promote secretions in the intestines
aggressive tumor with sheets of lymphocytes
staining positive for CD20, as well as very high 35. A 56-year-old man presents with sharp subster-
levels of nuclear c-myc with interspersed mac- nal pain radiating to his back and arms. The
rophages. What is the mode of transmission of patient is seated and leaning forward. He states
the virus associated with this malignancy? that the pain is less severe in this position, and
worsens when he lies down and takes a deep
(A) Blood products
breath. He recently recovered from a febrile
(B) Fecal-oral transmission
illness. On physical examination a scratchy,
High-Yield Principles
Chapter 4: Microbiology  •  Questions 81

leathery sound is heard at the left lower sternal cutaneous needle biopsy. A Gram stain of this
border. An ECG shows diffuse ST-segment el- sample is most likely to show which of the fol-
evation. Which of the following describes the lowing?
microorganism that is the most likely cause of
(A) Acid-fast bacilli
this condition?
(B) Catalase-negative, gram-positive cocci in
(A) Catalase- and coagulase-positive cocci chains
(B) Double-stranded, linear, enveloped, icosa- (C) Coagulase-negative, gram-positive cocci in
hedral DNA virus clusters
(C) Double-stranded, segmented RNA virus (D) Coagulase-positive, gram-positive cocci in
(D) Positive, single-stranded, helical RNA virus clusters
(E) Small, naked, single-stranded RNA virus (E) Oxidase-negative, gram-negative bacilli

36. A 28-year-old man complains of increasing 38. A 28-year-old woman presents to her primary
muscle weakness and numbness that began care physician complaining of a generalized

Microbiology
in his legs and feet three days ago and that body rash, especially on the inside of her wrists
now involves his arms and hands. The patient and ankles, and lesions on her genitals (see im-
reports recently experiencing a self-limited age). Physical examination reveals generalized
episode of gastroenteritis. Which organism is lymphadenopathy and a mild fever. Which of
commonly associated with this patient’s neuro- the following could be used to confirm the di-
logic symptoms? agnosis?
(A) a-Hemolytic, encapsulated, gram-positive
cocci that produce an IgA protease
(B) Comma-shaped, oxidase-positive, gram-
negative bacteria that can be grown at
42°C
(C) Non-lactose-fermenting, oxidase-positive,
gram-negative, aerobic bacilli
(D) Rod-shaped, gram-positive, spore-forming
anaerobe that produces a heat-labile toxin
(E) Spiral-shaped bacteria with axial filaments,
visualized using dark-field microscopy

37. An otherwise healthy 15-year-old boy sustains


a deep puncture wound in his left heel while Courtesy of Dr. J. Pledger, Centers for Disease Control and
playing in a junkyard. Four days later, his fa- Prevention.
ther brings him to the emergency department
because the boy has become lethargic and has
developed shaking chills. He refuses to bear (A) Culture on Thayer-Martin agar
weight on his left foot. The patient’s tempera- (B) Tzanck preparation
ture is 39.7°C (103.4°F). Physical examination (C) Venereal Disease Research Laboratory test
shows a warm, swollen, and extremely tender (D) Weil-Felix reaction
area around the puncture wound. A specimen (E) Ziehl-Neelsen stain
through uninfected tissue is obtained via per-
High-Yield Principles
82 Section I: General Principles  •  Questions

39. A 43-year-old HIV-positive man presents to his 40. An 8-day-old infant has developed fever, ir-
physician complaining of recent-onset abdomi- ritability, decreased level of consciousness,
nal pain and diarrhea, along with an increased apnea, and a full anterior fontanelle. Prenatal
level of general fatigue and occasional night laboratory tests results are not available, as the
sweats. Physical examination is significant for infant’s mother received no prenatal care and
a fever of 37.8°C (100.1°F), bilateral cervical delivered at home at 36 weeks’ gestation. What
adenopathy, and a weight loss of 4.5 kg (10 lb) routine prophylactic intrapartum antibiotic
compared to his last physician’s visit. A blood could the mother have been treated with in or-
sample is taken, which when cultured shows der to eradicate the likely cause of the infant’s
the presence of nonbranching bacilli that stain illness?
positively for Ziehl-Neelsen stain. His CD4+
(A) Fluconazole
cell count is 50/mm3. Which of the following
(B) Metronidazole
is the most worrisome for causing the patient’s
(C) Moxifloxacin
recent symptoms?
(D) Penicillin
Microbiology

(A)
Actinomyces israelli (E) Vancomycin
(B)
Mycobacterium avium-intercellulare
(C)
Mycobacterium marinum
(D)
Mycobacterium tuberculosis
(E)
Nocardia asteroides
High-Yield Principles
Chapter 4: Microbiology  •  Answers 83

An s w e r s

1. The correct answer is B. The differential diag- quired the illness and/or the morphology of
nosis of a genital ulcer in a sexually active pa- the organism. In this case, one cannot reach a
tient should include primary syphilis (though conclusion based on location. The man lives
these ulcers are usually painless), genital her- near the Mississippi River basin (where his-
pes, and chancroid. Because the Tzanck smear toplasmosis and blastomycosis are endemic),
(which looks for multinucleated giant cells and he has recently traveled to the Southwest
typical of herpes infection), is negative, as is (where coccidioidomycosis is endemic). Only
the VDRL test for syphilis, chancroid becomes paracoccidioidomycosis (endemic to Central
most likely. Chancroid is a bacterial infection and South America) can be eliminated as a
caused by Haemophilus ducreyi, which pre­ likely answer by location. Rather, the morphol-
sents typically as a painful genital ulcer with ogy of the yeast is the key to reaching the cor-

Microbiology
associated inguinal lymphadenopathy. It is typ- rect conclusion. In the lower-right portion of
ically treated with ceftriaxone given as a one- the image, an example of broad-based budding
time, 250-mg, intramuscular injection, or with can be seen, which is most consistent with
azithromycin as a single 1000-mg dose. blastomycosis. A common mnemonic regard-
ing the appearance of Blastomyces is that it is a
Answer A is incorrect. Acyclovir is an antiviral
Big, Broad-Based, Budding organism.
agent used to treat herpes infections. It is acti-
vated by viral thymidine kinase, whereupon it Answer B is incorrect. The distinguishing
inhibits the herpes viral polymerase. It can be morphologic feature of Coccidioides immitis,
used to treat herpes simplex virus types 1 and which is endemic to the southwestern United
2, varicella-zoster virus, and Epstein-Barr virus States, is its tendency to form large (up to 70
infections. µm in diameter) spherules filled with endo-
spores.
Answer C is incorrect. Foscarnet inhibits viral
DNA polymerase without the need of activa- Answer C is incorrect. Unlike the other an-
tion by thymidine kinase. It is used to treat cy- swer choices, Cryptococcus neoformans most
tomegalovirus (CMV) retinitis, but it can also commonly causes fungal meningitis (with
be used to treat acyclovir-resistant herpes sim- pneumonia as the second most common mani-
plex virus. festation) in the immunocompromised. Like
Blastomyces, it is a budding organism, but is
Answer D is incorrect. Ribavirin is used to
distinguishable morphologically by a thick
treat respiratory syncytial virus. It functions by
capsule that may be visualized with India ink
inhibiting inosine monophosphate dehydro-
stain.
genase, thus blocking the synthesis of guanine
nucleotides. Answer D is incorrect. Histoplasma capsula-
tum, which is endemic to the Mississippi and
Answer E is incorrect. Vancomycin is a bac-
Ohio River valleys, is unique among the an-
tericidal antibiotic used for multidrug-resistant
swer choices in that it can live as an intracel-
gram-positive organisms such as Staphylococ-
lular pathogen. Microscopic evaluation of a le-
cus aureus and Clostridium difficile. It func-
sion can show numerous small (1-5 µm) yeast
tions by binding to mucopeptide precursors,
forms within an individual macrophage.
preventing formation of the bacterial cell wall.
Answer E is incorrect. Paracoccidioides brasil-
2. The correct answer is A. This patient is suf- iensis, which is endemic to rural Latin Amer-
fering from a fungal pneumonia. In USMLE- ica, is often described as having a captain’s-
style questions, students are often asked to wheel or Mickey-Mouse-head appearance.
differentiate between candidate yeast species This is due to several smaller daughter cells
based on the location where the patient ac-
High-Yield Principles
84 Section I: General Principles  •  Answers

that are simultaneously budding from a single Answer C is incorrect. Hypertrophic cardio-
mother cell. myopathy is the most common cause of death
in young athletes in the United States. It is
3. The correct answer is D. Lyme disease is characterized by an asymmetric hypertrophic,
caused by infection with the spirochete Bor- nondilated left ventricle. Histopathologically,
relia burgdorferi, and is transmitted by the bite the myocardial architecture is disorganized
of the Ixodes tick. Initially, the disease presents and scarred. The typical ECG shows repolar-
with constitutional symptoms such as fever and ization changes or frank hypertrophy.
malaise, as well as a rash surrounding the bite
Answer E is incorrect. Classic third-degree
site. However, the bite site often goes unno-
heart block is “complete,” which means that
ticed, and erythema chronicum migrans is not
the atria and the ventricles beat independently
necessarily present in every case. Early dissem-
of each other, with the P waves and the QRS
inated disease presents four-six weeks after the
waves bearing no relation to one another. Se-
initial infection and is characterized by cardiac
vere bradycardia is usually present, and sudden
Microbiology

and neurologic abnormalities. Cardiac abnor-


death is a possibility. This condition is usually
malities include myocarditis, arrhythmias, and
treated with a pacemaker.
conduction disturbances. Lyme arthritis is a
late-stage finding, occurs in about 60% of pa-
4. The correct answer is B. The gradual onset
tients months to years later, and is associated
of her symptoms, together with the radiologic
with pain and swelling of large joints, most of-
findings of diffuse interstitial infiltrates, sug-
ten in one or both knees. Lyme disease is most
gests atypical pneumonia. Atypical pneumo-
prevalent in the northeast Atlantic Coast states,
nia is caused most commonly by Mycoplasma
but cases have been reported throughout the
pneumoniae, Legionella pneumophila, Chla-
United States.
mydia pneumoniae, and viruses; however, IgM
Answer A is incorrect. Brugada syndrome is cold agglutinin production is seen only with
a conductive heart disease that usually affects Mycoplasma infection.
young men and carries an increased risk of
Answer A is incorrect. Culture on buffered
sudden cardiac death. The disease has been as-
charcoal yeast extract medium is performed to
sociated with sodium ion channel abnormali-
diagnose L pneumophila pneumonia. L pneu-
ties. The typical ECG pattern is a right bun-
mophila causes atypical pneumonia that is
dle-branch block and ST-segment elevation on
seen most commonly in older individuals who
leads V1-V3.
smoke and abuse alcohol. Although Legionella
Answer B is incorrect. Chronic Chagas dis- is transmitted through environmental water
ease usually manifests during its earliest phase sources, infection does not imply aspiration.
with arrhythmias (eg, heart block and ven-
Answer C is incorrect. Phyocyanins, a product
tricular tachycardia). Dilated cardiomyopathy,
of Pseudomonas aeruginosa, lead to the blue-
megacolon, and megaesophagus occur later in
green color of the organisms. Pseudomonas can
the course of the disease. The disease presents
cause pneumonia but typically in patients who
acutely after the transfer of Trypanosoma cruzi
have cystic fibrosis or are severely immuno-
(found in the southern United States, Mexico,
compromised.
and Central and South America) by the redu-
viid bug (also called the kissing bug). Trans- Answer D is incorrect. Polysaccharide cap-
mission is associated with a hardened red area sules are a characteristic of Streptococcus pneu-
or chagoma. This is followed by fever, malaise, moniae and other organisms including certain
lymphadenopathy, tachycardia, and menin- strains of Haemophilus influenzae, Neisseria
goencephalitis that resolve within one month. meningitidis, and Escherichia coli. S pneu-
The patient’s ECG tracing is classic for Mobitz moniae is the cause of typical lobar pneumo-
type II heart block. Chagas disease would be a nia, which is characterized by sudden onset of
rare diagnosis in the northern United States. fever, chills, cough, and pleuritic pain. X-ray
High-Yield Principles
Chapter 4: Microbiology  •  Answers 85

of the chest usually shows focal lung consoli- Answer B is incorrect. CMV encephalitis can
dation rather than diffuse infiltrates, as seen in mimic the appearance of PML, but would
this case. be associated with enhancing periventricular
white matter lesions in cortical and subepen-
Answer E is incorrect. Reticulate bodies are
dymal regions. CMV encephalitis also is asso-
the intracellular form of Chlamydia species,
ciated typically with more systemic signs and
including C pneumoniae. C pneumoniae can
symptoms. Polymerase chain reaction analysis
cause atypical pneumonia that presents simi-
of CSF would be positive for CMV, and histo-
larly to Mycoplasma pneumonia. It is difficult
logic exam shows giant cells with eosinophilic
to distinguish between the two based on symp-
inclusions in both the cytoplasm and the nu-
toms and presentation, so treatment usually is
cleus.
designed to cover both organisms. Mycoplasma
infection, however, is much more common. Answer D is incorrect. Central nervous system
(CNS) lymphoma typically affects those with
5. The correct answer is C. The clinical picture CD4+ cell counts <50/mm3. MRI will dem-

Microbiology
and imaging are consistent with progressive onstrate one or more enhancing lesions (50%
multifocal leukoencephalopathy (PML) sec- are multiple and 50% are single) that typically
ondary to reactivation of latent JC virus infec- are surrounded by edema, and can produce a
tion, which can occur with CD4+ counts <50/ mass effect. CNS lymphoma can present with
mm3. It typically presents with rapidly progres- polymerase chain reaction findings positive for
sive focal neurologic deficits without signs of Epstein-Barr virus on CSF.
increased intracranial pressure. Ataxia, apha-
Answer E is incorrect. Space-occupying le-
sia, and cranial nerve deficits also may occur.
sions due to toxoplasmosis infection represent the
Lumbar puncture is nondiagnostic and fre-
most common cause of cerebral mass lesions in
quently demonstrates mild elevations in pro-
HIV-infected patients, and typically present with
tein and WBCs. Cerebrospinal fluid (CSF)
multiple enhancing lesions on MRI. The le-
analysis can reveal the presence of myelin
sions typically are located at the corticomedullary
basic protein, which is due to demyelination
junction, and are surrounded by edema that fre-
caused by the JC virus. PML typically presents
quently produces a mass effect and distinguishes
as multiple nonenhancing T2-hyperintense
its appearance from PML. Positive Toxoplasma
lesions. When it is suspected, stereotactic bi-
serologies can assist in diagnosis, and clinical
opsy is required for definitive diagnosis, but a
improvements will result from treatment with
positive CSF polymerase chain reaction for JC
sulfadiazine/pyrimethamine or trimethoprim/
virus is diagnostic in the appropriate clinical
sulfamethoxazole.
setting. Histology of the lesions shows nuclear
inclusions in oligodendrocytes. Although there
6. The correct answer is E. Rubeola, also called
is no definitive treatment, clearance of JC virus
measles, is a relatively rare illness in the
DNA can be observed with response to highly
United States because of the ubiquity of the
active antiretroviral therapy.
measles/mumps/rubella (MMR) vaccine. It
Answer A is incorrect. Uncommon in the de- presents with the prodrome described in this
veloped world, but presenting with increased patient. The rash that spreads from head to
risk in homeless and HIV patients, cortical toe over a three-day period develops one or
tuberculomas are caseating foci within the two days after the appearance of Koplik’s spots,
cortical parenchyma occurring from previous which are red oral lesions with blue-white cen-
hematogenous mycoplasma bacillemia. The ters.
clinical presentation may be similar to that
Answer A is incorrect. Diphtheria is an ill-
of the current patient; however, presentation
ness virtually unknown in the United States
would include enhancing nodular lesions on
because of the prevalence of the diphtheria/
imaging and elevated protein and low glucose
tetanus/pertussis (DTaP) vaccine. It is caused
on CSF examination.
High-Yield Principles
86 Section I: General Principles  •  Answers

by Corynebacterium diphtheriae and is charac- zolid, erythromycin, tetracycline & doxycy-


terized by a membranous pharyngitis. cline, and the aminoglycosides. Doxycycline is
the main treatment for both Rickettsia rickettsii
Answer B is incorrect. Pertussis, or whooping
(Rocky Mountain spotted fever) and Borrelia
cough, is also rare due to widespread vaccina-
burgdorferi (Lyme disease) infections. The rash
tions. It is a respiratory infection of children
of Rocky Mountain spotted fever is typically
that characteristically produces coughing
petechial and begins around the wrists and an-
spasms followed by a loud inspiratory whoop.
kles, although it may begin on the trunk or dif-
Answer C is incorrect. Roseola is a febrile dis- fusely. The rash of Lyme disease may be solid
ease of very young children that begins with a red or may form a ring or multiple rings with a
high fever and progresses to a rash similar to bulls-eye appearance.
measles. Infants and young children are most
at risk. It is believed to be caused by human 8. The correct answer is A. Four members of the
herpesvirus 6. Plasmodium genus of protozoa commonly in-
Microbiology

fect humans and cause malaria. All are spread


Answer D is incorrect. Rubella, also known
by the female Anopheles mosquito; diagnosis is
as German measles, is a less severe viral ex-
made through a blood smear. The species that
anthem. Many infections are subclinical, but
causes cerebral involvement is P falciparum,
rubella can cause severe birth defects when in-
which is almost entirely responsible for the se-
fection occurs during the prenatal period.
vere cases of disease that proceed to coma and
7. The correct answer is B. Based on the derma- death.
tomal and unilateral distribution of this rash, Answer B is incorrect. Plasmodium malariae
the patient most likely has shingles. This is a infection causes a 72-hour cyclic fever. P ma-
focal reactivation of a prior varicella-zoster vi- lariae, however, does not cause cerebral ma-
rus (VZV) infection. Most patients who de- laria.
velop shingles have a two- to three-day pro-
Answer C is incorrect. Plasmodium ovale in-
drome of pain, tingling, or burning in the
fection causes a 48-hour cyclic fever. A unique
involved dermatome, followed by the devel-
feature of P vivax and P ovale organisms is that
opment of a vesicular rash. The treatment of
they can form hypnozoites that can remain
choice for herpesvirus infections is acyclovir,
dormant in the liver for long periods, only to
ganciclovir, and (for VZV specifically) fam-
resurface later. However, P ovale does not
ciclovir, which work by inhibiting viral DNA
cause cerebral malaria.
polymerase.
Answer D is incorrect. Plasmodium vivax in-
Answer A is incorrect. Bacteria, not viruses,
fection causes a 48-hour cyclic fever. A unique
have cell walls. Inhibition of cell wall synthe-
feature of both P vivax and P ovale organisms
sis is accomplished by the penicillin family of
is that they can form hypnozoites that can re-
antibiotics.
main dormant in the liver for long periods,
Answer C is incorrect. The antiviral medica- only to resurface later. However, P vivax does
tion amantadine, used only in the treatment of not cause cerebral malaria.
influenza A virus infection, works by inhibiting
Answer E is incorrect. Plasmodium knowlesi is
viral genome uncoating in the host cell.
a simian malaria parasite that primarily infects
Answer D is incorrect. Nucleoside reverse macaques, although it has been reported to
transcriptase inhibitors are first-line medica- infect humans in southeast Asia. There are re-
tions for treating HIV infection. ports of cerebral involvement in monkeys, and
Answer E is incorrect. Inhibition of protein of isolated fatal human cases.
synthesis is achieved by five types of antibiot-
9. The correct answer is C. These lab results and
ics: chloramphenicol & clindamycin, line-
the clinical presentation (fever, headache, nu-
High-Yield Principles
Chapter 4: Microbiology  •  Answers 87

chal rigidity, and photophobia) are typical of are associated with an increased risk of devel-
viral meningitis. Because the cultures are neg- oping hepatocellular carcinoma. The liver has
ative, a viral cause should be considered. This a high regenerative potential, but if this pro-
patient would have recovered without compli- cess is overused, the chance of an oncogenic
cations with only symptomatic support. Viral mutation occurring during the regeneration of
aseptic meningitis usually is caused by entero- cells increases.
viruses and runs a milder course than bacterial
Answer C is incorrect. HIV as a direct onco-
meningitis.
genic agent is being intensely researched, but
Answer A is incorrect. This profile does not it is already known that immune suppression
suggest meningitis. However, because the clin- and dysregulation caused by HIV infection
ical presentation strongly suggests meningitis, give rise to lymphomas and Kaposi sarcoma.
it is not likely that the CSF analysis would be
Answer E is incorrect. Human T-cell lym-
completely normal.
photropic virus causes adult T-cell leukemia,

Microbiology
Answer B is incorrect. In this profile, only the and although the mechanism of oncogenesis
CSF sugar level is elevated. This does not sug- remains unclear, there is some evidence that
gest bacterial meningitis, in which sugar levels integration into the host genome at locations
would decrease. This profile may suggest sys- near cellular growth genes may play a role.
temic hyperglycemia, such as in uncontrolled
diabetes. 11. The correct answer is B. Influenza virus has
both hemagglutinin (HA) and neuramini-
Answer D is incorrect. This is a typical profile
dase (NA) molecules on its surface. These
of fungal or mycobacterial meningitis. Note
two molecules are responsible for the ability
that it is the same profile as that of bacterial
of the virus to be absorbed and penetrate the
meningitis, except the increase in WBCs is
host cells. After a human is infected with the
due to lymphocytes, not neutrophils. Fungal
influenza virus, that person will be immune
and mycobacterial meningitis also have a more
to infection by the same virus because of an-
subacute presentation than has bacterial men-
tibodies created against HA and NA. If either
ingitis.
HA or NA is changed, as can be the case if two
Answer E is incorrect. This is a typical profile different influenza viruses infect the same cell
of bacterial meningitis, in which neutrophils and exchange RNA, antigenic shift can occur.
predominate over lymhocytes. This creates a new virus that has never been
exposed to the human immune system before,
10. The correct answer is D. Human papillomavi- with potentially catastrophic consequences.
rus (HPV) causes carcinoma (usually cervical) This type of mixing is most commonly thought
by inactivating tumor suppressor genes such to be between a human and an avian strain
as p53 and Rb through the actions of viral pro- mixing in an intermediary porcine host, thus
teins E6 and E7, respectively. leading to the term “avian flu.”
Answer A is incorrect. Epstein-Barr virus Answer A is incorrect. Antigenic drift de-
(EBV) is associated with Burkitt lymphoma (a scribes mutations that can occur in hemag-
B-lymphocyte lymphoma) and nasopharyngeal glutinin and neuraminidase, making them
carcinoma. The t(8;14) translocation is consis- less antigenic to the preexisting antibod-
tently associated with Burkitt lymphoma, but ies in the human host. Since this results in
the translocation alone is not responsible for small changes in viral toxicity, it will lead to
the neoplasm and is not found in nasopharyn- a slightly different strain, but it is not likely to
geal carcinomas. The other factors that deter- lead to a global epidemic.
mine oncogenesis of EBV remain unclear.
Answer C is incorrect. Hemagglutinin has the
Answer B is incorrect. Both hepatitis C ability to attach to sialic acid receptors, which
(HCV) and hepatitis B virus (HBV) infections
High-Yield Principles
88 Section I: General Principles  •  Answers

activates fusion of the virus to the cell. All in- Answer B is incorrect. Bone is an extremely
fectious influenza viruses have this molecule. unlikely source for cysticerci due to its rela-
tively low blood flow.
Answer D is incorrect. Neuraminidase has the
ability to destroy neuraminic acid, a compo- Answer C is incorrect. The kidney can be a
nent of mucin. This helps break down the bar- location for cysticerci but is much less likely
rier to the upper airways and aids in infectivity. than cysts involving muscle tissue.
Answer E is incorrect. RBCs agglutinate in Answer E is incorrect. While the small bowel
the presence of hemagglutinin; hence the is the site of infection of primary hosts like the
name. This does not affect the infection rate of pig, secondary hosts (humans) do not develop
the influenza virus. an adult tapeworm infection.

12. The correct answer is D. The image shows 13. The correct answer is A. This patient is pre-
multiple lesions throughout the brain paren- senting with a classic case of whooping cough
Microbiology

chyma and subarachnoid space, which are caused by Bordetella pertussis. The initial
characterized by ring-shaped regions of low T1 phase is characterized by flu-like symptoms
intensity consistent with calcification. This ap- for the first one-two weeks. During this time,
pearance is most consistent with the nodular erythromycin is an effective treatment. The
calcified stage of neurocysticercosis and is seen second phase, the paroxysmal stage, is marked
only in individuals with long-standing, chronic by bouts of multiple coughs in a single breath
infection from endemic areas. Although this followed by a deep inspiration (the classic
patient’s presentation is highly suspicious for whooping cough). Treatment during this phase
malignancy, the image provided and labora- does not change the disease course, so only
tory data confirm a diagnosis of neurocysticer- supportive care is indicated and the infection
cosis, which is caused by infection with Taenia ought to pass in otherwise healthy individu-
solium, a pork tapeworm. It is the most com- als. In the United States, the diptheria/tetanus/
mon parasitic infection of the CNS worldwide, pertussis (DTaP) vaccine is supposed to be
and is particularly endemic to Central and given to all infants and protects them against
South America, Eastern Europe, and some diphtheria, tetanus, and pertussis. Infants who
parts of Asia. After humans ingest the tape- are not vaccinated are at risk for infection.
worm’s eggs, the eggs hatch and the larvae B pertussis can only be cultured on Bordet-
invade the wall of the small intestine and dis- Gengou medium.
seminate hematogenously. Cysticerci may be
Answer B is incorrect. Charcoal yeast extract
found in any organ, but are most commonly
when buffered with increased levels of iron
found in the brain, muscles, skin, and heart.
and cysteine is used to culture Legionella
Since we know that this patient is already suf-
pneumophila.
fering from cysts in her brain, the most likely
additional location would be her muscles. Answer C is incorrect. Chocolate agar with
Fortunately, the disease rarely results in death factor V and X is used to culture Haemophilus
and patients are often asymptomatic; however, influenzae.
when the disease does result in neurologic se- Answer D is incorrect. Loeffler medium is
quelae, specific symptoms depend on the loca- needed to culture Corynebacterium diphthe-
tion of the cysts. Cysticercosis is treated with riae.
administration of albendazole.
Answer E is incorrect. Thayer-Martin me-
Answer A is incorrect. Although the cysticerci dium is used to culture Neisseria gonorrhoeae.
may be found in virtually any organ, they al-
most never involve the urinary bladder. Schis- 14. The correct answer is E. This describes all

tosomiasis is a parasite that commonly invades spore-forming bacteria, which include Bacil-
the bladder. lus anthracis, Bacillus cereus, and Clostridium.
High-Yield Principles
Chapter 4: Microbiology  •  Answers 89

However, only Clostridium botulinum pro- common fungal infection in the US. About
duces the symptoms seen in this baby and 60% of these infections cause no symptoms,
also fits the mode of transmission. C botuli- and in the remaining 40% of cases, the symp-
num causes botulism via the production of a toms can range from mild to severe. Severe
heat-labile toxin that inhibits the release of forms of the infection can present with blood-
acetylcholine into the neuromuscular junc- tinged sputum, loss of appetite, weight loss,
tion. Infants may initially become constipated a painful red rash on the legs, and change in
and then develop generalized muscle weak- mental status. Cultures from sputum samples
ness (“floppy baby”). The organism is spread or biopsy show a dimorphic fungus seen as hy-
through the ingestion of contaminated canned phae at 25°C (77°F) and spherules filled with
or bottled food. Additionally, fresh honey has endospores at 37°C (98.6°F). Treatment with
been shown to harbor the organism. amphotericin B or fluconazole is usually re-
quired only in severe, disseminated disease.
Answer A is incorrect. IgA protease is pro-
duced by some bacteria so they can cleave Answer A is incorrect. In a patient with fever,

Microbiology
secretory IgA and colonize mucosal areas; chills, and flu-like symptoms who has recently
Neisseria gonorrhoeae, Neisseria meningitidis, returned from Namibia, there is concern for
Streptococcus pneumoniae, and Haemophilus infection by Plasmodium species, which cause
influenzae are the most well known. However, malaria. Malaria is transmitted by the female
none of these are typically transmitted via food, Anopheles mosquito. The time course and pat-
namely honey. tern of symptoms depend on the Plasmodium
species with which the patient is infected.
Answer B is incorrect. Exotoxin A is produced
Treatment is tailored to the geographic area
by Pseudomonas aeruginosa as well as some
of infection and the Plasmodium species in-
Streptococcus species. Exotoxin A has been as-
volved; agents include chloroquine, hydroxy-
sociated with toxic shock syndrome and scarlet
chloroquine, and atovaquone-proguanil. Ma-
fever. However, none of these organisms pro-
larial infection would be evident on a blood
duce the symptoms seen in this case or is trans-
smear.
mitted by honey ingestion.
Answer C is incorrect. Sickness after travel
Answer C is incorrect. Lecithinase is pro-
to the Mississippi and Ohio River valleys is
duced by Clostridium perfringens and is re-
suggestive of histoplasmosis. Although histo-
sponsible for the development of gas gangrene,
plasmosis typically does not present symptom-
cellulitis, and diarrhea. This organism is associ-
atically, some patients experience a flu-like
ated with contaminated wounds, which is not
illness with fever, cough, headaches, and myal-
a part of this baby’s history.
gias. Histoplasmosis can result in lung disease
Answer D is incorrect. Lipopolysaccharide, resembling tuberculosis (TB) and widespread
also called endotoxin, is produced by gram- disseminated infection affecting the liver,
negative bacteria and Listeria. It is highly anti- spleen, adrenal glands, mucosal surfaces, and
genic and can cause sepsis in severe infections. meninges. On microscopy, histoplasmosis ap-
However, generalized muscle weakness is not pears as spherules filled with endospores, as
characteristic of sepsis. opposed to the hyphae and spherules observed
in the sputum of those with coccidiomycosis.
15. The correct answer is B. This patient’s history
and sputum culture are suggestive of coccidi- Answer D is incorrect. Recent hiking in
oidomycosis, a fungal infection caused by the wooded areas carries the risk of contracting
inhalation of Coccidioides immitis or Coccidi- tick-borne illnesses, such as those carried by
oides posadasii. These organisms are found in the Ixodes tick: Babesia microti, a protozoon
soil in dry areas of the southwestern United that causes babesiosis; Borrelia burgdorferi, a
States, Mexico, and Central and South Amer- spirochete that causes Lyme disease; and Ehr­
ica. Coccidioidomycosis is the second most lichia chaffeensis, a rickettsial bacterium that
High-Yield Principles
90 Section I: General Principles  •  Answers

causes erlichiosis. None of these organisms ap- valve, such as “physiologic split” and, in this
pears as hyphae on microscopy. case, a large PDA, we hear the split S2 sound.
Answer E is incorrect. A wound while garden- Answer A is incorrect. Congenital CMV in-
ing, such as a thorn prick, can cause inocula- fection is marked most commonly by pete-
tion with Sporothrix schenckii. This fungus can chial rashes, jaundice, hepatosplenomegaly,
be found in various environments, including and sensorineural hearing loss. Cardiovascular
sphagnum moss, decaying vegetation, hay, abnormalities are not features of congenital
and soil. When S schenckii is introduced into CMV infection.
the skin, it causes a local pustule or ulcer with
Answer B is incorrect. Congenital herpes in-
nodules along the draining lymphatics (as-
fection most often either affects skin, eyes, and
cending lymphangitis). S schenckii is a dimor-
mouth or presents as localized CNS infection.
phic fungus, existing as hyphae at 25°C (77°F)
Symptoms usually develop within four weeks
and as a budding yeast form at 37°C (98.6°F).
of birth. CNS symptoms can include tempera-
Itraconazole or potassium iodide is used for
Microbiology

ture instability, respiratory distress, poor feed-


treatment. On microscopy, one would not ex-
ing, and lethargy. Herpes simplex virus type 2
pect to see spherules.
is also one of the most common causes of neo-
natal encephalitis.
16. The correct answer is C. All five answers

are part of ToRCHeS (Toxoplasmosis, Ru- Answer D is incorrect. Congenital toxoplas-
bella, Cytomegalovirus, Herpesvirus/HIV, mosis infection most often is asymptomatic ini-
and Syphilis) infections, the group of infec- tially. The class triad of symptoms that develop
tions for which every newborn is tested. If the include chorioretinitis, hydrocephalus, and
mother becomes infected during pregnancy, intracranial calcifications. Early symptoms can
the pathogens can cross the placenta and in- include a maculopapular rash, jaundice, and
fect the fetus. Rubivirus causes rubella in hepatomegaly. Most complications develop if
adults. The virus crosses the placenta in the the infection is not treated soon after birth.
first trimester and causes congenital abnor-
Answer E is incorrect. Newborns with con-
malities that range from deafness to cataracts
genital syphilis normally are asymptomatic
to cardiovascular abnormalities. The abnormal
at birth. When symptoms develop, the babies
heart exam findings in this newborn are clas-
often have hearing problems (based on cra-
sic for a patent ductus arteriosus (PDA) with
nial defect VIII) and cutaneous lesions that
delayed symptoms of heart failure. A PDA is
normally appear on the palms and soles first.
the most common cardiovascular abnormal-
More serious symptoms include anemia, jaun-
ity seen in congenital rubella syndrome. A
dice, and hepatomegaly. Patent ductus arterio-
PDA results from the failure of the ductus ar-
sus is not found with congenital syphilis.
teriosus to close in the first days of life. This
results in a left-to-right shunt from the aorta to 17. The correct answer is G. This patient is likely
the pulmonary artery. In a substantial shunt, presenting with a mucosa-associated lymphoid
deoxygenated blood returning from the body tissue (MALT) lymphoma. This type of indo-
to the heart bypasses the lung via the PDA to lent lymphoma is believed to be associated
the aorta. Because blood is shunted, there is a with infection by the organism Helicobacter
widening of pulse pressure (the difference be- pylori. H pylori is commonly identified by the
tween systolic and diastolic). A split S2 sound presence of urease, catalase, and oxidase. Erad-
occurs in PDA because of the increased flow ication of the infection with antibiotics and
through the pulmonary artery. The S2 sound is proton-pump inhibitors is often sufficient to
a composite of two distinct heart valves. Nor- cause regression of the lymphoma.
mally, the aortic valve closes just before the
pulmonary valve. When the pulmonary valve Answer A is incorrect. This pattern is seen in
is forced to stay open longer than the aortic benign flora such as the lactobacilli.
High-Yield Principles
Chapter 4: Microbiology  •  Answers 91

Answer B is incorrect. This pattern would be Moreover, an invasive process by the organism
commonly seen in streptococcal species such would likely take >4 hours to produce symp-
as Streptococcus pneumoniae. toms.
Answer C is incorrect. This pattern of results Answer D is incorrect. Vibrio cholerae causes
is commonly seen in facultative anaerobes large-volume, watery diarrhea. Treatment in-
such as Escherichia coli. volves prompt replacement of water and elec-
trolytes. Although antibiotics are not needed
Answer D is incorrect. This pattern represents
for treatment, tetracyclines have been shown
Pseudomonas aeruginosa, an opportunistic
to reduce the course of the disease. However,
lung pathogen.
there is nothing about this patient’s history to
Answer E is incorrect. This represents a pat- suggest that he has been exposed to cholera.
tern common of true anaerobes in the Bacter­ Rather, the history indicates a food-related
oides family. cause.

Microbiology
Answer F is incorrect. This pattern is seen in
19. The correct answer is C. This patient presents
Proteus mirabilis, a common cause of urinary
with classic symptoms of hepatocellular carci-
tract infection.
noma. Approximately 10%-30% of people in-
18. The correct answer is E. Food poisoning is fected with HCV will develop cirrhosis of the
the major cause of illness in this patient, and liver. Approximately 1%-5% of these patients
the most likely cause in this case is preformed develop hepatocellular carcinoma. HCV is
exotoxin from Bacillus cereus secreted into the transmitted via blood or blood transfusions
gastrointestinal (GI) tract. These exotoxins are and, rarely, by sexual contact. This patient is
fast acting, so the symptoms of food poisoning more likely to have HCV infection due to his
(nausea, vomiting, diarrhea) are usually rapid lack of travel, history of blood transfusion prior
in onset (within four-eight hours of ingestion). to the availability of sensitive screening meth-
Other major causes of food poisoning resulting ods, and extensive macronodular cirrhosis. Up
in nonbloody diarrhea include Staphylococcus to 90% of HCV-related hepatocellular carcino-
aureus, Clostridium perfringens, and entero- mas occur in patients with cirrhosis. Onset of
toxigenic Escherichia coli, which cause trav- hepatocellular carcinoma occurs on average
eler’s diarrhea. These organisms are typically 30 years after initial infection with HCV.
found in specific types of food, and B cereus is Answer A is incorrect. In the immunocom-
found in reheated rice. Because the food poi- petent host, CMV infection is often asymp-
soning in B cereus infection is caused by pre- tomatic, or it may produce a mononucleosis
formed enterotoxins, antibiotic treatment will syndrome. Disease manifestations are more
not help and supportive care is recommended. common in the immunocompromised host,
Answer A is incorrect. Bismuth subsalicylate, and include CMV colitis and CMV retinitis.
metronidazole, and amoxicillin are used to Patients with symptomatic CMV infection
treat Helicobacter pylori infection, which does may have subclinical transaminitis; however,
not result in the symptoms seen in this patient. there is no association with chronic hepatitis.

Answer B is incorrect. Fluoroquinolones Answer B is incorrect. Hepatitis A is transmit-


can be used to treat severe Shigella infection, ted via the fecal-oral route and causes an acute
which causes a bloody diarrhea. self-limited GI infection. It may rarely result
in fulminant hepatic failure requiring liver
Answer C is incorrect. Campylobacter jejuni transplantation, but it does not cause a chronic
enterocolitis can be treated with erythromy- hepatitis or cirrhosis.
cin or ciprofloxacin. Infection with this organ-
ism is not associated with eating reheated rice Answer D is incorrect. Hepatitis E is transmit-
and would typically result in bloody diarrhea. ted via the fecal-oral route and has been linked
to fatalities in pregnant women. It does not
High-Yield Principles
92 Section I: General Principles  •  Answers

cause chronic hepatitis except very rarely in 21. The correct answer is B. This boy is suffering
patients who have previously received solid or- from the classic hemolytic-uremic syndrome
gan transplants. (HUS) caused most often by the endotoxin
of Escherichia coli O157:H7 contracted from
Answer E is incorrect. HIV is transmitted via
undercooked beef. This disease is caused by
bodily fluids and causes the death of CD4+ T
endothelial injury and platelet aggregation
lymphocytes, resulting in an immunocompro-
that lead to the classic triad of microangio-
mised state and increased susceptibility to op-
pathic hemolytic anemia, thrombocytopenia,
portunistic infections. It does not cause hepa-
and acute renal injury. Up to 75% of cases of
titis.
classic HUS occur as a result of infection with
20. The correct answer is A. The image shows
E coli O157:H7.
gram-positive cocci in clusters. Staphylococcal Answer A is incorrect. Campylobacter in-
scalded skin syndrome (SSSS) is caused by the fections can cause diarrheal illnesses and
release of two exotoxins (epidermolytic toxins have been implicated in the development of
Microbiology

A and B) from Staphylococcus aureus. Des- Guillain-Barré syndrome, but would not be ex-
mosomes (also called “macula adherens”) are pected to cause HUS.
responsible for binding epithelial cells to one
Answer C is incorrect. Rotavirus is a common
another to form a coherent whole. The exotox-
cause of infantile gastroenteritis. It is caused
ins that are released bind to a molecule within
by a double-stranded virus (reovirus). It is typi-
the desmosome called desmoglein 1, thereby
cally spread throughout daycare centers but
disrupting cell adhesion. In SSSS, the epider-
does not cause HUS.
mis separates at the stratum granulosum due to
the binding of exotoxins to desmosomes in this Answer D is incorrect. Shigella is known to
layer. Clinically, this results in bullous lesions cause bloody diarrhea. It can cause HUS, but
and a positive Nikolsky sign. this is less common than E coli-induced HUS.
Answer B is incorrect. Gap junctions are cir- Answer E is incorrect. Vibrio cholerae causes
cular intercellular contact areas that permit massive watery diarrhea by secreting cholera
the passage of small molecules between adja- toxin. Vibrio infections are not implicated in
cent cells, allowing communication to facili- classic HUS. This bacterium is generally asso-
tate electrotonic and metabolic function. ciated with contaminated water sources rather
than ill-prepared food.
Answer C is incorrect. Hemidesmosomes are
present on the basal surface of epithelial cells
22. The correct answer is A. These symptoms are
adjacent to the basement membrane, and
typical of urethritis. The most common causes
serve to connect epithelial cells to the underly-
of urethritis in males are Chlamydia tracho-
ing extracellular matrix.
matis and Neisseria gonorrhoeae. The image
Answer D is incorrect. Intermediate junctions shows intracellular inclusions consistent with
lie deep to tight junctions, comprised of actin infection by C trachomatis. While they may be
filaments forming a continuous band around difficult to differentiate, C trachomatis infec-
the cell, providing structural support just be- tion induces a predominantly immunologic
low tight junctions. reaction, with only a few polymorphonuclear
leukocytes (PMNs), while N gonorrhoeae in-
Answer E is incorrect. Tight junctions are lo-
duces predominantly nonimmunologic in-
cated beneath the luminal surface of simple
flammation, with a PMN-rich infiltrate. The
columnar epithelium (eg, intestinal lining)
antibiotic of choice for chlamydia urethritis is
and seal the intercellular space to prevent dif-
azithromycin (macrolide) or doxycycline (tet-
fusion between cells.
racycline).
High-Yield Principles
Chapter 4: Microbiology  •  Answers 93

Answer B is incorrect. Ceftriaxone is an effec- tor of recent disease, given that IgM is the first
tive treatment for gonorrhea, but the cephalo- antibody produced in response to an antigen.
sporin class of antibiotics is relatively ineffec-
tive against Chlamydia trachomatis. 24. The correct answer is D. This patient likely
has pertussis, or whooping cough, which is
Answer C is incorrect. Fluconazole inhibits
caused by the gram-negative rod Bordetella
fungal steroid synthesis. It is used in the treat-
pertussis. This organism has four virulence fac-
ment of fungal infections, such as Candida al-
tors, including pertussis toxin. The A subunit
bicans.
of this exotoxin inhibits membrane-bound
Answer D is incorrect. Penicillin has been Gai proteins, which ultimately results in the
shown to suppress chlamydial multiplication. accumulation of cAMP. The effects of this ac-
However, it does not eradicate the organ- cumulation include histamine sensitization,
ism and thus is not the best treatment for this increased insulin synthesis, lymphocytosis, and
type of infection. Penicillin is the treatment of inhibition of phagocytosis.

Microbiology
choice for syphilis.
Answer A is incorrect. The botulinum toxin
Answer E is incorrect. Vancomycin has not released by Clostridium botulinum prevents
been shown to be effective in the treatment of the release of acetylcholine into the synaptic
chlamydial infection. It is used to treat drug- cleft, resulting in muscle weakness and paraly-
resistant Staphylococcus aureus and Clostrid- sis.
ium difficile.
Answer B is incorrect. The alpha toxin of
Staphylococcus aureus binds to the plasma
23. The correct answer is B. HBV is transmit-

membrane of host cells, forming a pore in
ted via parenteral, sexual, or maternal-fetal
the membrane that allows ions and small
routes. Of the markers listed, only hepatitis B
molecules to enter the cell. This leads to cell
e antigen (HBeAg) signifies active viral replica-
swelling and eventual lysis. Streptolysin O of
tion, and would therefore make transmission
Streptococcus pyogenes functions in a similar
of HBV to a partner more likely. HBeAg and
manner.
hepatitis B core antigen (HBcAg) are antigenic
markers of the virus core. They can be de- Answer C is incorrect. The exotoxin of Cory-
tected two-four months after exposure. nebacterium diphtheriae functions via the inac-
tivation of elongation factor 2, causing pharyn-
Answer A is incorrect. Hepatitis B e anti-
gitis and the formation of a pseudomembrane
body (HBeAb) is an antibody directed against
in the throat. Pseudomonas aeruginosa exo-
HBeAg. Its presence indicates low transmissi-
toxin A also works via this mechanism.
bility. HBeAb can be detected five months af-
ter exposure to HBV and one month after the Answer E is incorrect. Lipopolysaccharide-
detection of HBeAg. lipid A is an endotoxin released only by gram-
negative bacteria, with the exception of Liste-
Answer C is incorrect. Hepatitis B surface an-
ria monocytogenes, a gram-positive bacteria
tibody (HBsAb) provides immunity to HBV in-
responsible for meningitis in neonates and
fection. It can be detected in former carriers of
immunosuppressed patients. Endotoxins are
HBV or in patients immunized with the HBV
a normal part of the bacterial membrane re-
vaccine.
leased upon lysis of the cell.
Answer D is incorrect. IgG HBcAb is an indi-
cator of chronic disease. 25. The correct answer is E. Vibrio cholerae

causes watery stools, often called “rice-water”
Answer E is incorrect. Hepatitis B core anti-
stool. This illness is not accompanied by ab-
body (HBcAb) is produced in response to hep-
dominal pain, but the symptoms are due to
atitis core antigen (HBcAg). IgM is an indica-
dehydration, which leads to electrolyte imbal-
ances. Cholera toxin binds to the GM1 entero-
High-Yield Principles
94 Section I: General Principles  •  Answers

cyte receptor via the pentameric B subunit. mechanism of shock through anaphylaxis is
Once inside the cell, the toxin must undergo not related to DIC.
cleavage of the active, A1 component, which
Answer D is incorrect. The C5a component
goes on to constitutively activate the Gs protein
of the complement cascade, activated by en-
through ADP ribosylation. This results in high
dotoxin, functions in neutrophil chemotaxis.
cyclic AMP levels, which activate the CFTR
This is not the mechanism involved in anaphy-
(cystic fibrosis transmembrane conductance
laxis.
regulator) channel, leading to a large efflux
of chlorine and other ions into the GI lumen. Answer E is incorrect. The cytokine
This results in extremely watery diarrhea ac- interleukin-1, released by macrophages acti-
companied by electrolyte imbalances. vated by endotoxin, causes fever. Fever is ab-
sent in anaphylaxis.
Answer A is incorrect. This is a characteristic
of Shiga toxin, which typically leads to bloody Answer F is incorrect. Nitric oxide, released
diarrhea. by macrophages activated by endotoxin, causes
Microbiology

hypotension (shock). However, this is not the


Answer B is incorrect. Bacillus anthracis pro-
mechanism of anaphylactic shock.
duces a toxin that acts as an adenylate cyclase
(edema factor) but is not associated with severe 27. The correct answer is D. Loeffler medium,

watery diarrhea. blood agar, and tellurite plate are used to cul-
Answer C is incorrect. Diphtheria toxin and ture a throat swab from patients with suspected
exotoxin A from Pseudomonas inactivate elon- Corynebacterium diphtheriae infection. This
gation factor 2, but neither is a likely cause of organism causes symptoms of pseudomembra-
watery diarrhea. nous pharyngitis (thick gray membrane) and
lymphadenopathy. A possible complication of
Answer D is incorrect. Bordetella pertussis
this disease is the extension of the membrane
produces an exotoxin that increases cAMP lev-
into the larynx and trachea, resulting in airway
els by inactivating the inhibitory Gi. However,
obstruction. Rapid treatment with diphtheria
pertussis causes whooping cough, not severe
antitoxin, penicillin, or erythromycin and the
watery diarrhea.
DTP vaccine is indicated in this patient
26. The correct answer is B. Anaphylactic shock Answer A is incorrect. Löwenstein-Jensen agar
is a life-threatening disorder that occurs when is used to culture Mycobacterium tuberculosis,
an allergen overactivates mast cells and ba- the bacterium that causes TB.
sophils, leading to widespread release of his-
Answer B is incorrect. Bordet-Gengou agar is
tamines, serotonins, and other compounds
used to culture Bordetella pertussis, the etio-
stored in the granules of these immune cells.
logic agent responsible for whooping cough.
These compounds lead to vasodilation and
leaky capillaries. The crosslinking of IgE re- Answer C is incorrect. Chocolate agar with
ceptors present on these cells causes the activa- factors V and X is used to culture Haemophilus
tion of these cell types. By blocking this cross influenzae. This organism typically causes up-
reaction, the anaphylactic pathway can be sty- per respiratory infections but is not responsible
mied. for pseudomembranous pharyngitis.
Answer A is incorrect. γ Interferon is pro- Answer E is incorrect. Neisseria gonorrhoeae
duced by T lymphocytes and, among other is cultured on Thayer-Martin media. This bac-
functions, activates tumoricidal macrophages. terium can cause gonorrhea, septic arthritis,
neonatal conjunctivitis, and pelvic inflamma-
Answer C is incorrect. Endotoxin can directly
tory disease.
activate Hageman factor, activating the coagu-
lation cascade and leading to disseminated in-
travascular coagulation (DIC). However, the
High-Yield Principles
Chapter 4: Microbiology  •  Answers 95

28. The correct answer is B. This patient presents treonam is a b-lactamase-resistant monobac-
with Rocky Mountain spotted fever (RMSF) tam that interferes with cell wall biosynthesis
caused by Rickettsia rickettsii, a small, gram- by binding to penicillin-binding protein 3. Az-
negative bacterium carried by the American treonam is a potent antipseudomonal agent in-
dog tick (Dermacentor variabilis). Despite dicated for pseudomonal sepsis.
its name, RMSF is more common in the
Answer A is incorrect. Amoxicillin is an ami-
southeastern United States than in the Rocky
nopenicillin antibiotic that interferes with cell
Mountains. Patients often present first with
wall synthesis. Although amoxicillin has an
severe headache, fever (>38.9°C or >102°F),
extended spectrum compared with penicillin
and myalgias followed by a petechial rash on
(covering Haemophilus influenzae, Escherichia
the palms and soles (or wrists and ankles) that
coli, Listeria, Proteus, Salmonella, and Entero-
spreads to the trunk. The Weil-Felix assay re-
cocci), it does not provide antipseudomonal
action is the classic test for rickettsial disease.
coverage.
The treatment of choice for adults with RMSF

Microbiology
is doxycycline; chloramphenicol is also used Answer C is incorrect. Use of fresh frozen
but has more significant adverse effects. plasma (FFP) is reserved for patients with co-
agulopathy and signs of active, life-threatening
Answer A is incorrect. Cephalosporins (cef-
bleeding. Although provision of FFP will tem-
triaxone) are not effective against Rickettsia
porarily reverse some of this patient’s labo-
rickettsii. They are generally used to treat neo-
ratory signs of DIC (elevated International
natal/infant sepsis or gonorrhea, as well as bac-
Normalized Ratio, decreased fibrinogen),
terial meningitis.
treatment of the underlying cause (ie, Pseudo-
Answer C is incorrect. Aminoglycosides (gen- monas sepsis) is most important.
tamicin) are not effective against Rickettsia
Answer D is incorrect. Vancomycin is an an-
rickettsii. Aminoglycosides are effective against
tibiotic used for serious multidrug-resistant,
many gram-negative bacteria and some strains
gram-positive infections. Major uses are for
of Staphylococcus aureus. It is also used as
methicillin-resistant Staphylococcus aureus
broad-spectrum therapy when combined with
and moderate to severe Clostridium difficile in-
a penicillin or metronidazole.
fections. Its mechanism of action is to inhibit
Answer D is incorrect. Nystatin is used in the cell wall mucopeptide formation by binding
treatment of fungal infections such as oral can- the D-ala-D-ala portion of cell wall precursors.
didiasis.
Answer E is incorrect. Coagulopathy caused
Answer E is incorrect. Penicillin is not effec- by warfarin overdose is reversed by pharma-
tive against Rickettsia rickettsii. Penicillin is cologic administration of vitamin K. This pa-
still the treatment of choice for syphilis. Other tient’s coagulopathy is caused by Pseudomonas
derivatives that are used more frequently in- sepsis, so vitamin K therapy plays no role here.
clude oxacillin, cloxacillin, dicloxacillin, and
amoxicillin when Staphylococcus and Strepto- 30. The correct answer is C. Trichomoniasis is

coccus species prove sensitive. caused by Trichomonas vaginalis. Symptoms
include a profuse, frothy discharge that is as-
29. The correct answer is B. This patient has leu- sociated with vulvovaginal pruritus, tender-
kocytosis and Charcot’s triad (fever, jaundice, ness, and burning. The vulva and vagina are
right upper quadrant pain), along with the frequently inflamed. The cervix and vagina
ominous sign of hypotension, a clear clini- may develop small, red, punctate lesions, caus-
cal picture of cholecystitis. In addition, he ing the classic “strawberry” appearance. Diag-
has Pseudomonas aeruginosa sepsis and DIC. nosis is made by wet mount, on which small,
Gram-negative rod sepsis is the clear cause of pear-shaped, flagellated organisms can be seen
this patient’s DIC, and antipseudomonal cov- moving around.
erage with aztreonam is most appropriate. Az-
High-Yield Principles
96 Section I: General Principles  •  Answers

Answer A is incorrect. Bloody and foul- Answer B is incorrect. Entamoeba histolytica


smelling vaginal discharge suggest a vaginal infection presents with bloody diarrhea (dysen-
foreign body. tery), abdominal cramps with tenesmus, and
pus in the stool. It can also cause right upper
Answer B is incorrect. Pelvic or lower ab-
quadrant pain and liver abscesses. E histolyt-
dominal pain with an abnormal, foul-smelling
ica is transmitted via cysts in water (fecal-oral
vaginal discharge is suggestive of pelvic inflam-
transmission). On microscopy, one observes
matory disease, which can involve the endo-
amebas with ingested RBCs. Treatment for
metrium, fallopian tubes, ovaries, and peri-
E histolytica infection includes metronidazole
toneum. Fever, nausea, and vomiting may be
and iodoquinol.
present. The typical organisms involved are
Neisseria gonorrhoeae and Chlamydia tracho- Answer C is incorrect. Leishmania donovani
matis. infection presents with hepatomegaly and sple-
nomegaly, malaise, anemia, and weight loss.
Answer D is incorrect. Candida species typi-
L donovani is transmitted via the sandfly. Mi-
Microbiology

cally cause intense pruritus and a thick, odor-


croscopically, macrophages containing amas-
less, white, cottage cheese-like vaginal dis-
tigotes are observed. Sodium stibogluconate is
charge. Erythema, edema, dysuria, and urinary
used to treat L donovani infection.
frequency may be present. Budding yeasts and
hyphae are seen on a wet mount. Answer E is incorrect. The Plasmodium falci-
parum parasite is responsible for causing ma-
Answer E is incorrect. Bacterial vaginosis
laria. It is spread by the Anopheles mosquito.
typically presents with an unpleasant fishy-
Diagnosis of Plasmodium falciparum infection
smelling discharge that is thin, gray-white, and
is made through a blood smear.
homogeneous. Pruritus and inflammation are
unusual in bacterial vaginosis, thus the term
32. The correct answer is C. This image of the
“vaginosis” rather than “vaginitis.” On a wet
mitral valve reveals Aschoff’s nodule, which
mount, one will see clue cells that are formed
is pathognomonic for rheumatic fever. Rheu-
from bacteria-coated epithelial cells.
matic fever is caused by group A streptococci.
Years after a bout of rheumatic fever, rheu-
31. The correct answer is D. Naegleria fowleri

matic heart disease can develop due to calcifi-
presents with a rapidly progressing meningoen-
cation of warty vegetations on fibrotic healing,
cephalitis that can progress to coma or death
most commonly on the mitral valve. As a re-
within six days. Other symptoms include nau-
sult, it most commonly causes mitral stenosis,
sea, vomiting, and irrational behavior. Trans-
which in mild disease causes a late diastolic
mission occurs through swimming in fresh-
murmur heard best at the apex. In the US, the
water lakes. Microscopic analysis will reveal
incidence of this mitral stenosis secondary to
amebas in the spinal fluid. Unfortunately,
rheumatic fever is low given the widespread
there is no treatment for N fowleri.
use of antibiotics. However, rheumatic mitral
Answer A is incorrect. Cryptosporidium spe- stenosis is prevalent in immigrant populations.
cies infection presents with severe diarrhea in
Answer A is incorrect. A friction rub heard
HIV-positive patients and mild watery diarrhea
throughout the precordium would correspond
in HIV-negative patients. Cryptosporidium spe-
to pericarditis, an inflammation of the pericar-
cies are transmitted via cysts in water (fecal-
dial sac that leads to friction with an expanding
oral transmission). Microscopically, acid-fast
and contracting myocardium. It is not a long-
staining cysts are found. Unfortunately, there
term complication of rheumatic fever. Rather,
is no treatment available for Cryptosporidium
a friction rub may be associated with malig-
species infection; however, in healthy patients,
nancy, uremia, active infections such as viral
cryptosporidiosis is self-resolving.
or tuberculous, and other rare causes.
High-Yield Principles
Chapter 4: Microbiology  •  Answers 97

Answer B is incorrect. Aortic stenosis is repre- munoglobulin heavy chain enhancer. There
sented by a harsh crescendo-decrescendo early is a strong association between Burkitt lym-
systolic murmur heard best at the right upper phoma and Epstein-Barr virus (EBV). In en-
sternal border with radiation to the carotids. demic regions, children are typically infected
Rheumatic heart disease can lead to calcifica- with EBV by the age of three years, compared
tion of the aortic valve, which leads to aortic with infection during adolescence in other re-
stenosis, but for unclear reasons, this occurs gions. EBV, the cause of infectious mononu-
much less frequently than on the mitral valve. cleosis, is spread by saliva and respiratory secre-
An aortic stenosis murmur in a middle-aged tions.
woman should raise the suspicion for a calci-
Answer A is incorrect. HBV and HCV are
fied bicuspid aortic valve (remember the “fish
spread by blood-borne contacts. Although
mouth”).
these viruses are associated with hepatocellu-
Answer D is incorrect. A systolic click heard lar carcinoma, there is no association between
best at the apex corresponds to mitral valve HBV or HCV and Burkitt lymphoma.

Microbiology
prolapse, which is a relatively common and
Answer B is incorrect. Hepatitis A virus (HAV)
benign finding in middle-aged women. It oc-
is transmitted by the fecal-oral route. There is
curs when an abnormally thickened mitral
no association between HAV and Burkitt lym-
valve leaflet displaces into the left atrium dur-
phoma.
ing ventricular systole. It is not related to rheu-
matic heart disease. It may lead to mitral regur- Answer C is incorrect. Arboviruses and flavi-
gitation and ultimately valve surgery may be ruses can be transmitted through mosquito
required, but typically it is followed by a car- bites. These viruses cause tropical diseases
diologist with serial exams and echocardiogra- such as dengue fever and yellow fever, but not
phy. Burkitt lymphoma.
Answer E is incorrect. An S4, which is heard Answer E is incorrect. Viruses such as HIV,
best at the apex and is associated with concen- herpes simplex virus type 2, and human her-
tric left ventricular hypertrophy, is indicative of pesvirus type 8 are spread by sexual contact.
either chronic extensive afterload (ie, from un- None of these pathogens is associated with
controlled hypertension) or long-standing aor- Burkitt lymphoma.
tic stenosis. It also can be caused by diastolic
heart failure because the left ventricle is not as 34. The correct answer is D. This patient’s con-
compliant as it should be. The exam finding is stellation of symptoms and prior hospitaliza-
due to turbulent blood flow caused by blood tion points toward pseudomembranous coli-
filling a stiff ventricle. An S4 is not associated tis. Clostridium difficile proliferation causes
with rheumatic heart disease. the severe non-bloody diarrhea associated
with pseudomembranous colitis by produc-
33. The correct answer is D. This patient has
ing an exotoxin that kills enterocytes. Antibi-
Burkitt lymphoma, a lymphoma endemic otic treatments suppress the normal flora of
to Africa. Clinically, Burkitt lymphoma of- the GI tract, allowing C difficile to multiply.
ten presents with “B symptoms” (fever, night Clindamycin was the first antibiotic associated
sweats, weight loss), signs of tumor lysis syn- with C difficile gastroenteritis and is used of-
drome such as oliguria, and solitary jaw ten to treat anaerobic infections above the dia-
masses. Histopathologically, Burkitt lymphoma phragm, such as aspiration pneumonia. How-
typically assumes a “starry sky” appearance ever, many antibiotics have been implicated
with sheets of lymphocytes interspersed with since then, especially cephalosporins and am-
occasional macrophages. The cytogenetic ab- picillin. Always consider C difficile in patients
normality associated with Burkitt lymphoma is with gastroenteritis and recent antibiotic use.
a t(8;14) translocation in which the oncogene
c-myc is placed under the expression of the im-
High-Yield Principles
98 Section I: General Principles  •  Answers

Answer A is incorrect. This describes how the drome in young adults similarly characterized
Norwalk virus can cause gastroenteritis charac- by fever and pharyngitis. Both cause pericardi-
terized by nausea, vomiting, and diarrhea that tis only rarely.
resolves spontaneously within 12-24 hours.
Answer C is incorrect. Reoviridae is the only
Answer B is incorrect. This describes the exo- double-stranded RNA virus. Members of this
toxin produced by Shigella species, which can family include rotavirus, which is responsible
cause a bloody and mucus-rich diarrhea. for diarrhea in children, and reovirus, which
causes Colorado tick fever.
Answer C is incorrect. This describes the
exotoxin produced by Vibrio cholerae, which Answer D is incorrect. This answer choice de-
causes a large volume of watery diarrhea de- scribes the structure of Coronaviridae. Coro-
void of RBCs or WBCs (sometimes called rice- navirus (CoV) is a common virus that causes
water stool). It is not associated with prior anti- a self-limited cold-like syndrome. SARS-CoV,
biotic use. however, has been identified as the cause of se-
Microbiology

vere acute respiratory syndrome.


Answer E is incorrect. This describes Esch-
erichia coli, which causes the abrupt onset of
36. The correct answer is B. Guillain-Barré syn-
profuse watery diarrhea.
drome (GBS) is a common cause of acute
peripheral neuropathy that results in progres-
35. The correct answer is E. The patient presents
sive weakness over a period of days. Labora-
with classic signs and symptoms of pericardi-
tory abnormalities associated with GBS in-
tis, including precordial chest pain. This pain
clude elevated gamma-globulin, decreased
is relieved when leaning forward and worsens
nerve conduction velocity indicative of demy-
with inspiration. On physical exam the patient
elination, and albuminocytologic dissociation
has a pericardial friction rub, which accounts
(CSF shows increased protein concentration
for the scratchy, leathery sound heard during
with normal cell count in the setting of nor-
both systole and diastole. Classic ECG find-
mal glucose). Although one third of patients
ings include diffuse ST-segment elevation and
report no history of an infection, the other
depression of the PR segment. Pericarditis is
two thirds will have recently experienced an
frequently preceded by a viral upper respira-
acute GI or influenza-like illness prior to de-
tory infection. Although many viruses may
veloping the neuropathy. The most common
cause pericarditis, coxsackie B is the most
epidemiologic associations involve infections
common cause of inflammation of the pericar-
with Campylobacter jejuni, a comma-shaped,
dial membrane. Coxsackievirus is a picornavi-
oxidase-positive, gram-negative bacterium that
rus, the smallest of the RNA viruses. They are
can be grown at 42°C. C jejuni causes a gas-
positive, single-stranded, naked, icosahedral
troenteritis that often presents with bloody di-
RNA viruses.
arrhea. Although a recent history of C jejuni
Answer A is incorrect. Staphylococcus au- enteritis is epidemiologically linked with GBS,
reus is a gram-positive, catalase-positive, and the infection does not directly cause GBS. The
coagulase-positive bacterium. Infection with proposed mechanism of GBS is that a preced-
S aureus may lead to acute bacterial endocar- ing infection incites an immune response that
ditis from seeding secondary to bacteremia. cross-reacts with peripheral nerve components,
The bacterium rarely causes pericarditis. leaving them susceptible to damage also.
Answer B is incorrect. This describes the Answer A is incorrect. Streptococcus pneu-
structure of Herpesviridae. Herpesvirus is char- moniae is an a-hemolytic, encapsulated, gram-
acterized by multinucleated giant syncytial positive coccus that produces an IgA protease.
cells with intranuclear inclusion bodies. Mem- Pneumococcal pneumonia can result in bac-
bers of this family include CMV and Epstein- teremia, meningitis, osteomyelitis, or septic
Barr virus. Both cause a mononucleosis syn-
High-Yield Principles
Chapter 4: Microbiology  •  Answers 99

arthritis, but is not associated with the develop- Group A streptococci can cause cellulitis, nec-
ment of GBS syndrome. rotizing fasciitis, and streptococcal toxic shock
syndrome. Group B streptococci are a com-
Answer C is incorrect. Pseudomonas aeru-
mon cause of neonatal bacterial meningitis.
ginosa is a non-lactose-fermenting, oxidase-
Group D streptococci (Enterococcus) are a fre-
positive, gram-negative, aerobic bacillus that
quent cause of urinary tract infections and sub-
can cause otitis externa, urinary tract infection,
acute bacterial endocarditis.
pneumonia, and sepsis in immunocompro-
mised hosts. It is not associated with GBS syn- Answer C is incorrect. Staphylococcus epider-
drome or other neurologic conditions. midis is a coagulase-negative, gram-positive
coccus that grows in clusters. It is common
Answer D is incorrect. Clostridium botulinum
skin flora. It can be a cause of osteomyelitis,
is a rod-shaped, gram-positive, spore-forming,
but this is more common after implantation
anaerobe that produces a heat-labile toxin that
with orthopedic appliances.
inhibits acetylcholine release at the neuromus-

Microbiology
cular junction, causing flaccid paralysis. Botu- Answer E is incorrect. Salmonella species are
lism, therefore, does not lead to sensory find- oxidase-negative, gram-negative bacilli. Salmo-
ings, and is not associated with increased CSF nella osteomyelitis is often associated with pa-
protein on lumbar puncture. tients who have sickle cell disease.
Answer E is incorrect. Treponema pallidum,
38. The correct answer is C. The patient is now
the causative agent of syphilis, is a spiral-
in the secondary stage of a syphilis infection,
shaped bacterium with axial filaments, visu-
characterized by wart-like lesions known as
alized using dark-field microscopy. Tertiary
condylomata lata, generalized rash, and sys-
syphilis causes sensory deficits rather than
temic symptoms such as lymphadenopathy,
muscle weakness and is preceded by a painless
weight loss, and fever. The fluorescent trepo-
chancre and maculopapular rash. Neurologic
nemal antibody absorption (FTA-ABS) test,
findings develop after years, not weeks, of un-
the Venereal Disease Research Laboratory
treated infection.
(VDRL) test, and the rapid plasma reagin test
approach 100% sensitivity for detecting syphi-
37. The correct answer is D. This teenager most
lis.
likely has osteomyelitis secondary to a contigu-
ous focus of infection, such as bites, puncture Answer A is incorrect. Thayer-Martin agar is
wounds, and open fractures. Most of these used to culture Neisseria gonorrhoeae.
cases are caused by Staphylococcus aureus, a
Answer B is incorrect. Tzanck preparation
gram-positive, coagulase-positive coccus that
using Giemsa stain reveals multinucleated gi-
occurs in clusters. This organism expresses re-
ant cells indicative of herpes simplex virus
ceptors for bone matrix components, such as
(HSV) or vesicular stomatitis virus infection.
collagen, which help it to attach to and infect
Although HSV can cause genital lesions, they
bone. When compared to hematogenous os-
are usually painful and come in clusters.
teomyelitis, continguous-focus infections are
more likely to also include gram-negative and Answer D is incorrect. The Weil-Felix re-
anaerobic bacteria. action assay tests for antirickettsial antibod-
ies. This test would be positive if the patient
Answer A is incorrect. Acid-fast bacilli such
were infected with Rickettsia rickettsii (Rocky
as Mycobacterium tuberculosis are found in
Mountain spotted fever). Rocky Mountain
patients with TB. Infection of the bone by TB
spotted fever can also present with a rash on
is known as Pott disease, but its usual presenta-
the palms and soles but would not present with
tion is vertebral osteomyelitis.
genital lesions.
Answer B is incorrect. Streptococcus species
are catalase-negative, gram-positive cocci.
High-Yield Principles
100 Section I: General Principles  •  Answers

Answer E is incorrect. Ziehl-Neelsen stain is 40. The correct answer is D. This infant is show-
used to stain acid-fast mycobacteria such as ing signs of neonatal sepsis and meningitis.
Mycobacterium tuberculosis. The most common cause of neonatal sep-
sis and meningitis is group B Streptococcus
39. The correct answer is B. The first step in
(GBS), which frequently colonizes the genito-
answering this question is to recognize that urinary tract of women and can be passed to
Ziehl-Neelsen stain is what is used for the the infant during delivery. Penicillin G would
acid-fast test. The presence of acid-fast rods have provided adequate coverage for the moth-
in the blood indicates that this man has a dis- er’s likely colonization with GBS.
seminated mycobacterial infection. In order of
Answer A is incorrect. Fluconazole, an an-
prevalence, the three main candidate organ-
tifungal agent that interferes with fungal er-
isms in patients with HIV are: (1) M avium-
gosterol synthesis, would not have been an
intracellulare (MAC), (2) Mycobacterium tu-
appropriate antibiotic for GBS prophylaxis.
berculosis, and (3) Mycobacterium kansasii.
Fluconazole has good coverage against fungal
Microbiology

Disseminated MAC infection can present


infections.
with generalized symptoms (such as fatigue,
night sweats, fever, and weight loss) as well as Answer B is incorrect. Metronidazole, a dis-
organ-specific symptoms, and occurs when the ruptor of nucleic acid structure, would not
patient’s CD4+ cell count has dropped to <50/ have been an appropriate antibiotic for GBS
mm3. It can include gut pain (from mycobac- prophylaxis. Metronidazole has good activity
terial enteritis), pulmonary symptoms, or ad- against protozoans and anaerobic organisms.
enopathy. Although M tuberculosis infection
Answer C is incorrect. Moxifloxacin, a DNA
is frequently seen in HIV-infected patients, it is
gyrase inhibitor, would not have been an ap-
rare to recover it from blood cultures.
propriate antibiotic for GBS prophylaxis.
Answer A is incorrect. Actinomyces israelii is a Fluoroquinolones have good coverage against
bacterial pathogen with a fungal-like morphol- gram-negative and select gram-positive organ-
ogy. Unlike Nocardia, it is not acid-fast. isms, and are often used to treat urinary tract
infections and community-acquired pneumo-
Answer C is incorrect. Mycobacterium mari-
nia. Additionally, fluoroquinolones are a class
num is a species native to fresh-water and salt-
C substance, as they have the potential to
water environments. It is a rare cause of cuta-
cause teratogenic or embryocidal effects. Giv-
neous wound infection in anglers, swimmers,
ing fluoroquinolones during pregnancy is not
and aquarium owners. It is not commonly as-
recommended unless the benefits justify the
sociated with disseminated infection in immu-
potential risks to the fetus.
nocompromised patients.
Answer E is incorrect. Vancomycin, a cell
Answer D is incorrect. M tuberculosis could
wall synthesis inhibitor, would not have been
be a causative agent for the symptoms found
an appropriate antibiotic for GBS prophylaxis
in this patient, except that his CD4+ count
in this case. While vancomycin has good cov-
is <50/mm3, which makes infection with
erage against GBS, its use is reserved for cases
M avium-intracellulare most likely.
of known resistance. Vancomycin could have
Answer E is incorrect. Nocardia is eliminated been used if the mother were known to harbor
as the correct answer by the given morphology GBS that was resistant to other first-line antibi-
of the observed bacteria. Although Nocardia otics, or had an allergy that prevented the use
can stain weakly acid-fast, they demonstrate of other agents.
a branching filamentous morphology that re-
sembles fungal hyphae.
Chapter 5

Immunology

101
High-Yield Principles
102 Section I: General Principles  •  Questions

Q u e st i o n s

1. A 48-year-man with chronic renal failure un- flammatory cytokines. Which of the following
derwent a cadaveric renal transplant. The op- opposes the action of cytokines?
eration was a success, and the transplanted kid-
(A) Interferon-γ
ney started producing urine “on the operating
(B) Interleukin-7
table.” However, one week later, the patient’s
(C) Transforming growth factor-b
creatinine level begins to rise and his urine
(D) Tumor necrosis factor-a
output drops. He does not experience dysuria,
(E) Tumor necrosis factor-b
and urinalysis shows no bacteria or crystals.
A biopsy taken from the transplanted kidney 4. The image depicts a cell that is activated by
shows a cellular infiltrate. Which of the follow- bacterial products, upregulates costimulatory
ing would most likely be found on the surface molecules, and migrates to the draining lymph
Immunology

of the cells responsible for this patient’s current node. Which of the following types of cells is
condition? shown in this image, and of which type of im-
(A) CD20 mune cell is it a specialized form?
(B) CD27
(C) CD34
Ag
(D) CD4 Ag
Ag
(E) CD8 TNF Ag
CD40L Ag
MCM Ag
2. An infant boy experiences multiple bacterial, LPS Ag
viral, and fungal infections during his first Ag

year of life. He has also been suffering diar- Ag Ag

rhea since birth. Tests are performed to deter-


Ag
mine the likely cause of his symptoms. Serum Ag Ag
calcium levels are normal, and the patient’s
white blood cells change nitroblue tetrazolium Reproduced, with permission, from USMLERx.com.
from clear to bright blue; however, his B- and
T-lymphocyte counts are very low. What is a
possible mechanism accounting for his symp- (A) Kupffer cell, dendritic cell
toms? (B) Kupffer cell, macrophage
(C) Langerhans cell, dendritic cell
(A) Defective interleukin-2 receptors (D) Langerhans cell, macrophage
(B) Failure of the third and fourth pharyngeal (E) Microglia, dendritic cell
pouches to descend
(C) Inability of helper T lymphocytes to switch 5. To assess the risk of erythroblastosis fetalis dur-
classes ing the future pregnancy of an Rh-negative
(D) Lack of NADPH oxidase activity woman, a clinician sends a sample of serum
(E) X-linked tyrosine kinase defect for detection of anti-Rh blood group anti-
bodies. The laboratory performs an indirect
3. A 45-year-old woman presents to her fam- Coombs test by mixing the patient’s serum
ily physician with complaints of two months with Rh-positive RBCs and then adding an
of joint stiffness and pain that is worst in the anti-IgG antibody. In doing so, the laboratory
morning. Physical examination reveals swell- technician observes agglutination of the RBCs.
ing of the left metacarpophalangeal joints and After receiving this test result, the clinician
of the wrists bilaterally. She is diagnosed with a would be correct to conclude which of the fol-
condition associated with the release of proin- lowing?
High-Yield Principles
Chapter 5: Immunology  •  Questions 103

(A) The laboratory performed the test incor- 7. A 1-year-old child whose parents just emi-
rectly; it should have mixed the patient’s grated from Mexico presents to the emergency
serum with Rh-negative rather than Rh- room with stridor. The child is in obvious dis-
positive RBCs tress and is drooling. Examination of the phar-
(B) The patient has had previous pregnancies ynx reveals a cherry-red mass at the base of
and all of her children are Rh-negative the tongue. The vaccine that could have pre-
(C) The patient is currently pregnant with an vented this child’s illness contains two parts:
Rh-positive fetus purified bacterial capsule and mutant diphthe-
(D) The presence of anti-Rh antibodies in the ria toxoid. Which of the following characteris-
patient’s serum suggests that she has been tics of the causative organism makes necessary
pregnant with an Rh-positive fetus the addition of the diphtheria toxoid to the vac-
(E) The results of the test cannot be used to cine?
determine the Rh status of a current fetus;
(A) Attachment pili
the results only assess the presence of anti-
(B) Gram-negative rod

Immunology
bodies in the mother. To determine fetal
(C) IgA protease
Rh status, direct typing of fetal blood must
(D) Polyribitol-ribose phosphate capsule
be done.
(E) Protein A
6. A 68-year-old woman has been hospitalized
8. A type B blood group, Rh-positive recipient
for three days after an exacerbation of emphy-
mistakenly receives a kidney from a type A
sema. Her clinical course progresses well un-
blood group, Rh-negative donor. Which of
til the fourth hospital day, when she develops
the following best describes the mechanism of
shortness of breath, fatigue, and cough produc-
transplant rejection that is most likely to ensue
tive of yellow sputum. Her oxygen saturation
in this recipient?
drops by 10%, and she is started on vancomy-
cin and gentamicin via rapid infusion. Thirty (A) Pre-formed recipient antibodies
minutes after the initiation of antibiotics, the (B) Macrophages
patient develops erythema of the face and (C) Pre-formed donor antibodies
neck, itchiness, and hypotension. The patient (D) Donor T lymphocytes
has no known drug allergies and has not been (E) Recipient T lymphocytes
treated with vancomycin prior to this hospital-
ization. Which of the following is the mecha- 9. A 3-year-old boy is admitted to the hospital
nism of this reaction? with aspergillosis. He has had multiple admis-
sions for viral and fungal infections previously.
(A) Binding of antibody-complement to target His serum calcium level is 7.8 mg/dL. Biopsy
cells of one of this patient’s lymph nodes would
(B) Cross-linking of IgE by Fc receptors on most likely reveal which of the following ab-
mast cells normalities?
(C) Deposition of antibody-drug immune com-
plexes (A) Decreased number of follicles
(D) Release of cytokines by activated T lym- (B) Hypocellular paracortex
phocytes (C) Hypocellular medullary sinuses
(E) Release of histamine by mast cells (D) Enlarged follicles throughout the entire
lymph node
(E) Increased number of the medullary cords
High-Yield Principles
104 Section I: General Principles  •  Questions

10. A 60-year-old postmenopausal woman presents the following symptoms is this patient most
with fatigue, mild jaundice, and tingling in the likely experiencing?
lower extremities. Laboratory studies show ele-
(A) Episodic palpitations, sweating, and head-
vated serum levels of homocysteine and meth-
aches with no apparent trigger
ylmalonic acid, and mild thrombocytopenia. A
(B) Intermittent ischemia of fingers and toes
peripheral blood smear supports the diagnosis.
associated with pallor and pain
In which of the following disorders would a pe-
(C) Joint pain, dry eyes and mouth
ripheral blood smear be similar to the one seen
(D) Urethritis
in this case?
(E) Widespread tightening of the skin
(A) Acute blood loss
(B) Anemia of chronic disease 13. A 10-month-old boy is brought to the pedia-
(C) Iron deficiency anemia trician by his parents because of fever, cough,
(D) Pernicious anemia and difficulty breathing. A profile of the pa-
(E) Pyridoxine deficiency tient’s immunoglobulin isotypes shows low
Immunology

IgA, low IgG, and markedly elevated IgM lev-


11. The image below depicts a series of steps in an els. The number of T and B lymphocytes is
immunologic activation pathway. This path- normal. Which of the following is the most
way is most important for which activity of the likely etiology of the increased level of IgM in
immune system? this patient?
(A) A defect in DNA repair enzymes
(B) A defect in LFA-1 adhesion proteins on
phagocytes
(C) A defect in the CD40 ligand on CD4 T
helper cells
(D) Failure of interferon-γ production
(E) Failure of the thymus and parathyroid
glands to develop

14. A 40-year-old man presents to his physician


with numbness and tingling on the dorsal sur-
face of his right hand and forearm, and raised
“varicose veins” that are firm to the touch
along the same distribution. He also complains
of weight loss. His serum creatinine level is 2.0
Reproduced, with permission, from USMLERx.com. mg/dL. He has no previous medical history
of significance. An immune complex disease
is suspected, and assays for autoantibodies in-
(A) Antigen presentation and destruction of in-
side neutrophils are conducted. What diseases
tracellular pathogens
are associated with the identification of anti-
(B) Defense against parasites by eosinophils
myeloperoxidase and anti-proteinase-3 antibod-
(C) Immediate allergic hypersensitivity reac-
ies, respectively?
tion
(D) Neutrophil chemotaxis (A) Kawasaki disease, Buerger disease
(B) Microscopic polyangiitis, Wegener granu-
12. A 35-year-old woman presents to the clinician lomatosis
with symptoms suggestive of an autoimmune (C) Polyarteritis nodosa, Buerger disease
etiology. A biopsy is sent for immunofluores- (D) Takayasu arteritis, Wegener granulomato-
cent staining, and the results indicate the pres- sis
ence of anticentromere antibodies. Which of
High-Yield Principles
Chapter 5: Immunology  •  Questions 105

(E) Takayasu arteritis, Buerger disease (C) Patients with juvenile rheumatoid arthritis
(F) Temporal arteritis, Wegener granulomato- are more likely to have large-joint involve-
sis ment, but the likelihood of concurrent sys-
temic symptoms is the same
15. A 24-year-old woman presents with a four- (D) Patients with juvenile rheumatoid arthritis
month history of fever, night sweats, and are more likely to have systemic symptoms
weight loss. X-ray of the chest reveals enlarged and high levels of serum rheumatoid factor
mediastinal lymph nodes. The biopsy of one (E) Patients with juvenile rheumatoid arthritis
such lymph node is shown in the image. From are more likely to have systemic symptoms
which of the following cell types is the cell and large-joint involvement
with the bilobed nucleus derived?
17. A 6-month old boy is noted at his well-child
visit to have very poor growth and weight gain
since his last visit. His mother states that he

Immunology
was hospitalized for pneumococcal pneumo-
nia, and has had several bad colds with pu-
rulent nasal discharge since he was born. His
physician suspects an immunodeficiency, and
laboratory results indeed reveal deficiency of a
complement protein. Which of the following
antibodies functions in the same manner as
the missing complement protein in this child?
(A) IgA
(B) IgD
(C) IgE
(D) IgG
Reproduced, with permission, from USMLERx.com.
(E) IgM

18. A 29-year-old intravenous drug user has suf-


(A) B lymphocyte
fered from recurrent pneumonias, fungal
(B) Fibroblast
infections in the axillae, and a recent ear in-
(C) Macrophage
fection. He now presents with a cough and
(D) Neutrophil
painful swallowing. Physical examination re-
(E) T lymphocyte
veals a patchy, white oral lesion. Cells with
16. Patients who share similar clinical symptoms which of the following markers are most likely
and disease pathology may nevertheless pre­ to be deficient in this patient?
sent differently based on age at disease onset. (A) CD4
How does the presentation of the disease evi- (B) CD8
dent in the image differ between juvenile and (C) CD20
adult-onset disease? (D) CD40
(A) Juvenile rheumatoid arthritis is simply the (E) CD56
presence of rheumatoid arthritis that be-
gins before the age of 21 years
(B) Patients with juvenile rheumatoid arthritis
are less likely to have systemic symptoms,
but the likelihood of high levels of serum
rheumatoid factor is the same
High-Yield Principles
106 Section I: General Principles  •  Questions

19. A 2-year-old girl with a lifelong history of mal- tient does not have tonsils. His mother denies
absorptive and foul-smelling diarrhea, weak- a previous tonsillectomy. Analysis of the boy’s
ness, and general failure to thrive has just un- serum would most likely yield which of the fol-
dergone a small intestine biopsy (see image). lowing results?
Her parents believe her problems began at 6
(A) < 200/mm3 CD4 T lymphocytes
months of age, when she started eating solid
(B) Absence of T lymphocytes
foods, but have significantly worsened over
(C) IgA, IgG, and IgM levels normal
the past few months. The only recent change
(D) IgG and IgM levels markedly decreased,
in her diet is that she eats a bowl of cereal ev-
no IgA
ery morning with her parents before they go to
(E) IgG and IgM levels normal, IgA markedly
work. She tried a dairy-free diet a month ago,
decreased
but it did not improve her symptoms. Which
of the following is the most likely diagnosis? 21. A 26-year-old woman presents to the clinic
with joint pain in her hands and wrists, diffi-
Immunology

culty breathing, and redness over her cheeks


and nose. She also notes that her fingertips
change color from white to blue to red when
she is cold. Which of the following describes
the renal pathology commonly associated with
this patient’s condition?
(A) Histopathology reveals glomerulonephritis
with subendothelial immune deposits evi-
dent on immunofluorescence electron mi-
croscopy
(B) It has a “railroad track” or “tram track” ap-
pearance on light microscopy
(C) It is a homogeneous renal disease
Reproduced, with permission, from USMLERx.com. (D) It is associated with elevated serum com-
plement levels
(E) It is immune mediated by anti-mitochon-
(A) Abetalipoproteinemia drial antibodies
(B) Celiac sprue
(C) Lactase deficiency 22. A clinician is concerned that an Rh-negative
(D) Viral enteritis mother may be pregnant with an Rh-positive
(E) Whipple disease fetus. The potential pathology that the clini-
cian is concerned about is classified as which
20. A pediatrician becomes concerned after learn- of the following immune reactions?
ing about the family and the medical history
(A) Graft-versus-host disease
of an 18-month-old child who is currently suf-
(B) Type I hypersensitivity
fering from pneumonia, with a presumed diag-
(C) Type II hypersensitivity
nosis of Streptococcus pneumoniae infection.
(D) Type III hypersensitivity
Over the past year, the patient has suffered
(E) Type IV hypersensitivity
from erysipelas as well as a previous bout of
pneumococcal pneumonia; both were treated 23. A 14-year-old boy presents to the physician
successfully with antibiotics. The patient’s with recurrent pyogenic infections. Physical
mother says that her son’s maternal uncle also examination shows that the boy has pruritic
suffered from repeated bacterial infections and papulovesicular dermatitis. Laboratory tests
was successfully treated with antibiotics. On show low serum IgM level and a normal serum
physical examination, it appears that the pa-
High-Yield Principles
Chapter 5: Immunology  •  Questions 107

IgG level. Which of the following laboratory (A) Lyme disease


abnormalities is most likely to also be present? (B) Pneumonia
(C) Sarcoidosis
(A) Hypocalcemia
(D) Systemic lupus erythematosus
(B) Low IgA level
(E) Tuberculosis
(C) Low IgE level
(D) Lymphopenia
25. A 58-year-old man presents to his physician be-
(E) Thrombocytopenia
cause of fatigue, edema, and worsening kidney
function. After extensive laboratory work-up,
24. A 30-year-old woman presents to the emer-
his physician decides to perform a kidney bi-
gency department with a right-sided facial
opsy. The pathologist notes numerous green-
droop and bilateral swelling on the face near
colored, proteinaceous deposits when he uses
the angles of the mandible. She reports that
polarized light microscopy to view the sample.
these signs occurred suddenly while she was at
Which of the following is most likely abnormal
home. On further questioning she says that for

Immunology
in this patient?
the past few weeks she has been feeling short
of breath and tired, and has had a dry cough. (A) C2
She denies any recent exposure to wooded (B) CD8
areas. Her temperature is 38.4°C (101.2°F), (C) Fab fragment
heart rate is 93/min, blood pressure is 110/85 (D) Ig light chain
mm Hg, respiratory rate is 22/min, and oxygen (E) Major histocompatibility complex II
saturation is 93% on room air. Physical exami-
nation reveals red, tearing eyes; smooth, non- 26. A 7-year-old boy is brought to the physician by
tender bulging of both cheeks; a right-sided his parents because of recurrent sinus infec-
drooping of the mouth; cervical nodes; and tions. The parents state that the boy also has
bilateral dry râles. Results of a purified protein had multiple lung infections and intermittent
derivative test are negative. X-ray of the chest diarrheal infections since birth. Which of the
is shown in the image. What process is most following results would most likely be found
likely to be causing this patient’s symptoms? on further testing?
(A) A high IgE level
(B) A low IgA level
(C) A low IgM level, with increased IgA
(D) A negative nitroblue tetrazolium dye test
(E) A normal immunoglobulin level for all iso-
types

27. A patient is administered a tuberculin test.


Which of the following types of hypersensitiv-
ity reaction is being tested, and which cells
would be expected to mediate a positive test re-
sult?
(A) Type I hypersensitivity, mast cells
(B) Type II hypersensitivity, cytotoxic T lym-
Reproduced, with permission, from USMLERx.com. phocytes
(C) Type IV hypersensitivity, mast cells
(D) Type IV hypersensitivity, T-helper 1 cells
(E) Type IV hypersensitivity, T-helper 2 cells
High-Yield Principles
108 Section I: General Principles  •  Questions

28. A 24-year-old, previously healthy woman pre­ (A) A noninflamed skin abscess
sents to the outpatient clinic with a two-week (B) A repaired cleft palate
history of drooping eyelids and difficulty rising (C) Frequent falls
from a chair. Stimulation with a reversible ace- (D) Skin granulomas
tylcholinesterase inhibitor leads to the resolu- (E) Visual hallucinations
tion of her symptoms. Which of the following
is most responsible for the three-dimensional 30. A 50-year-old man comes to the physician with
structure of the key mediator in this woman’s hemoptysis and diffuse joint pain. He states
condition? that both his father and cousin had similar
symptoms and were diagnosed with micro-
(A) Disulfide bonds
scopic polyangiitis, a disease affecting me-
(B) Hydrogen bonds
dium- to small-sized arteries that is believed to
(C) Ionic bonds
have an autoimmune component to its patho-
(D) Triple covalent bonds
genesis. Antibody levels were measured to con-
(E) Van der Waals forces
Immunology

firm the diagnosis. What other disease is most


closely correlated with the same type of auto-
29. A 3-year-old boy is brought to his pediatrician
antibodies as present in this patient?
because of worsening cough and rhinorrhea.
His parents state that he has had multiple simi- (A) Churg-Strauss syndrome
lar episodes over the past year with two brief (B) CREST variant of scleroderma
hospitalizations for pneumonia. Physical ex- (C) Sjögren syndrome
amination of the skin reveals the lesion seen (D) Systemic lupus erythematosus
in the image. Laboratory tests show very low (E) Wegener granulomatosis
serum levels of IgA. Which of the following is
most likely to also be seen in this child?

Reproduced, with permission, from Wolff K, et al, Fitzpatrick’s


Color Atlas & Synopsis of Clinical Dermatology, 6th edition.
New York: McGraw-Hill, 2009.
High-Yield Principles
Chapter 5: Immunology  •  Answers 109

An s w e r s

1. The correct answer is E. This patient suffers viral, and fungal infections. Treatment in-
from rejection of his recently transplanted cludes bone marrow transplantation.
organ. There are three types of rejection: hy-
Answer B is incorrect. Failure of the third
peracute, acute, and chronic. A hyperacute
and fourth pharyngeal pouches to descend is
rejection is mediated by pre-formed antibod-
known as DiGeorge syndrome, and results in
ies to the transplanted organ. In most cases,
no thymus or parathyroid glands. Chromo-
it can be observed while still in the operating
some 22q11 deletions are associated with Di-
theater. Acute rejection can occur at any time
George syndrome. Tetany from hypocalcemia
but is most likely in the first three months after
and viral and fungal infections are common
transplantation. It is due to human leukocyte
due to the lack of T lymphocytes, while nor-
antigen discrepancies between host and graft,
mal B lymphocytes are present.
and is more common in highly vascular organs

Immunology
such as the liver or kidney. Finally, chronic re- Answer C is incorrect. A CD40 ligand defect
jection is a slow process that leads to fibrosis of causes hyper-IgM syndrome, because the CD4
the transplanted tissue due to increased fibro- T-helper cells are unable to switch classes from
blast activity. This patient experienced acute IgM. Although these patients present with
rejection, which is mediated by cytotoxic T multiple infections such as pneumonia, sinus-
cells (CD8). itis, infectious diarrhea, and central nervous
system (CNS) infections, the most common
Answer A is incorrect. CD20 is a B-cell
laboratory finding is neutropenia, not low lev-
marker. Because they are a precursor to plasma
els of lymphocytes.
cells, they would only be indirectly involved in
hyperacute rejection. Answer D is incorrect. The lack of reduced
NADPH oxidase activity results in chronic
Answer B is incorrect. CD27 is a marker of
granulomatous disease. Staphylococcus aureus,
plasma cells. Plasma cells are responsible for
Escherichia coli, and Aspergillus are common
antibody production and therefore would be
pathogens. The nitroblue tetrazolium dye re-
the culprit in hyperacute rejections.
duction test shows no activity in these patients.
Answer C is incorrect. CD34 is a fibrocyte Because the question stem says that the ni-
marker. Fibrocytes leave the bloodstream and troblue tetrazolium dye test turned blue, this
differentiate into fibroblasts in the transplanted patient must have normal NADPH oxidase
organ. They are the cell type that leads to activity, and thus cannot have chronic granulo-
chronic rejection of the graft. matous disease.
Answer D is incorrect. CD4 is the marker of Answer E is incorrect. An X-linked tyrosine
T-helper cells. They do not attack pathogens kinase defect causes Bruton agammaglobulin-
or tissues and are not implicated in the pathol- emia, resulting in decreased levels of all im-
ogy of acute rejection and negative selection. munoglobulin molecules. Infections begin to
occur after the maternal IgG antibodies de-
2. The correct answer is A. Severe combined cline, typically after six months of life.
immunodeficiency can be caused by multiple
factors: failure to synthesize major histocom- 3. The correct answer is C. This patient has
patibility complex II antigens, defective inter- signs and symptoms of rheumatoid arthritis, a
leukin-2 receptors, or adenosine deaminase disorder of immune dysregulation. Many par-
deficiency. This defect in early stem cell differ- allel and sequential events occur within the
entiation results in a lack of T and B lympho- synovium to cause joint destruction, includ-
cytes. Without T and B lymphocytes, patients ing activation of T lymphocytes and release
are at significantly increased risk of bacterial, of many proinflammatory cytokines such as
High-Yield Principles
110 Section I: General Principles  •  Answers

interleukin (IL) 1. Transforming growth fac- Answer B is incorrect. Kupffer cells are tissue-
tor-b plays a role in dampening the immune resident macrophages; the image depicts a
response, and in the case of rheumatoid ar- Langerhans cell.
thritis works as a reparative agent by inhibit-
Answer D is incorrect. Langerhans cells are
ing T lymphocyte activation and proliferation,
specialized, tissue-resident dendritic cells, not
inhibiting biosynthesis of metalloproteinases,
macrophages.
protecting cartilage from degradation by IL-1,
and inhibiting tumor necrosis factor secretion Answer E is incorrect. Microglia are tissue-
by macrophages. resident macrophages within the CNS.
Answer A is incorrect. Interferon-γ is secreted 5. The correct answer is D. When an Rh-
by helper T lymphocytes and helps stimulate negative mother gives birth to an Rh-positive
macrophages. fetus, fetal RBCs may enter the mother’s circu-
Answer B is incorrect. IL-7 is secreted by mac- lation, and the body may recognize the Rh an-
Immunology

rophages and is one of the main cytokines in- tigen as foreign and produce antibodies against
volved in mounting an acute phase response. it. As maternal IgG freely crosses the placenta,
IL-7 has been shown to stimulate bone erosion any subsequent Rh-positive fetus is at risk for
in rheumatoid arthritis. hemolytic disease. Thus, the indirect Coombs
test is an important laboratory tool to monitor
Answer D is incorrect. Tumor necrosis
for Rh incompatibilities that may complicate
factor-a is one of the main cytokines involved
fetal health. The test result given in this ques-
in mounting an acute-phase response; it is se-
tion indicates that the patient possesses anti-Rh
creted by macrophages.
antibodies in her serum. Therefore, it would
Answer E is incorrect. Tumor necrosis fac- be logical for the clinician to suspect previous
tor-b has functions similar to tumor necrosis pregnancy with an Rh-positive fetus.
factor-a in that it helps mount an acute-phase
Answer A is incorrect. The laboratory protocol
response. However, it is secreted by activated T
described in the question stem is correct. Rh-
lymphocytes instead of macrophages.
positive RBCs must be added because the test
is assaying for the presence anti-Rh antibodies,
4. The correct answer is C. Dendritic cells arise
which will bind the Rh antigen on the RBCs.
from both myeloid and lymphoid bone mar-
row precursors and then establish sites of resi- Answer B is incorrect. The presence of anti-
dence within the peripheral tissues. Immature Rh antibodies within the patient’s serum sug-
dendritic cells located within the epidermis are gests that she has been exposed to the blood of
named Langerhans cells, and they are highly at least one Rh-positive fetus.
phagocytic. The exact function of Birbeck
Answer C is incorrect. The results of the test
granules is unknown, but they are present only
cannot be used to determine the Rh status of a
with Langerhans cells. Upon antigen encoun-
current fetus; only direct typing of fetal blood
ter, Langerhans cells upregulate costimulatory
can determine its Rh status.
molecules (such as major histocompatibil-
ity complex class II) and then migrate to the Answer E is incorrect. The test yielded a
draining lymph node, where they play an in- positive result. Agglutination occurs when the
tegral part in the activation of T lymphocytes anti-IgG antibody binds to the anti-Rh antibod-
and the subsequent induction of an adaptive ies that are already bound to the Rh-positive
immune response. RBCs.
Answer A is incorrect. Kupffer cells are tissue- 6. The correct answer is E. Vancomycin can
resident macrophages within the liver. cause two types of hypersensitivity reactions:
anaphylaxis, which is a type I hypersensitiv-
ity reaction, and the “red man” syndrome
High-Yield Principles
Chapter 5: Immunology  •  Answers 111

(RMS), which is an anaphylactic reaction. 7. The correct answer is D. This child is suffer-
This patient most likely has RMS due to rapid ing from acute epiglottitis, caused by Hae-
infusion of vancomycin. Although her symp- mophilus influenzae type B (HiB). The inci-
toms also may be consistent with anaphylaxis, dence of this illness has decreased drastically
the patient has never been exposed to vanco- in recent years because of immunization ef-
mycin; therefore, she would not have pre- forts. The HiB vaccine consists of two parts:
formed IgE antibodies against the drug. By the bacterial capsule antigen and diphtheria
contrast, a vancomycin-induced anaphylactic toxoid protein. Because the bacterial capsule
reaction involves histamine release from mast is a polysaccharide, it alone will not generate a
cells and basophils without binding of pre- sufficient immune response. Therefore, a pro-
formed IgE antibodies to Fc receptors. RMS tein (the toxoid) is added to the vaccine in or-
can be prevented by slow infusion of vanco- der to generate an antibody response.
mycin, which is preferable to discontinuing
Answer A is incorrect. Whereas attachment
the drug. Another method of prevention is pre-
pili are an important virulence factor of H in-

Immunology
treatment with antihistamine.
fluenzae, they are not the reason why the vac-
Answer A is incorrect. Type II hypersensitiv- cine must contain a toxoid protein.
ity reactions are characterized by IgM or IgG
Answer B is incorrect. Whereas H influenzae
molecules binding to an antigen on a cell,
is a gram-negative rod, its Gram stain charac-
which leads to destruction of the cell via ac-
teristics do not play a role in the body’s ability
tivation of the complement cascade. Exam-
to generate antibodies against it.
ples include autoimmune hemolytic anemia,
Goodpasture syndrome, rheumatic fever, and Answer C is incorrect. IgA protease is an im-
Graves disease. portant virulence factor in H influenzae. It
functions to cleave IgA, which allows the bac-
Answer B is incorrect. Type I hypersensitivity
teria to colonize the mucosa. It does not, how-
reactions are mediated by pre-formed IgE anti-
ever, play a role in vaccination.
bodies that bind a particular antigen to which
the patient has already been exposed. Binding Answer E is incorrect. H influenzae does not
of the antigen-IgE complexes to Fc receptors generate protein A. This is a virulence factor
on mast cells results in degranulation and re- found in Staphylococcus aureus that binds the
lease of vasoactive substances such as hista- Fc portion of IgG and thus helps prevent opso-
mine. The reaction occurs rapidly after anti- nization.
gen exposure. Examples include anaphylaxis,
asthma, hives, wheals, and flare. 8. The correct answer is A. This clinical scenario
would result in hyperacute rejection (within
Answer C is incorrect. Type III hypersensitiv- minutes of transplantation; clinical presenta-
ity reactions involve antibodies binding to an- tion within minutes to hours) mediated by
tigen in plasma, leading to antibody-antigen pre-formed anti-donor antibodies in the recipi-
immune complexes. Examples include polyar- ent. The recipient would possess anti-type A
teritis nodosa, immune complex glomerulone- antibodies, which react to the A antigen pres-
phritis, lupus, rheumatoid arthritis, serum sick- ent not only on RBCs but on most other cell
ness, and the Arthus reaction (local complex types. Hyperacute rejection occurs almost im-
deposition). mediately, as the anti-donor antibodies bind
Answer D is incorrect. Type IV hypersensitiv- directly to vascular endothelial cells, initiating
ity reactions involve sensitized T lymphocytes complement and clotting cascades and result-
that encounter the antigen and release lym- ing in hemorrhage and necrosis of the trans-
phokines. Examples include a tuberculosis planted kidney. It should be noted that it is not
(TB) skin test, transplant rejection, and con- only ABO blood group mismatches but any
tact dermatitis (poison oak, poison ivy). anti-donor antibodies possessed by the recipi-
ent that can lead to hyperacute rejection; thus,
High-Yield Principles
112 Section I: General Principles  •  Answers

it is important to carefully cross-match donor Answer D is incorrect. This pattern is charac-


and recipient. teristic of follicular lymphoma, not DiGeorge
syndrome.
Answer B is incorrect. Macrophages provide
a secondary immune response in this situation Answer E is incorrect. The medullary cords
but are not initially involved in acute or hyper- contain lymphocytes and plasma cells, and are
acute reactions. not usually affected in DiGeorge syndrome.
Answer C is incorrect. Hyperacute rejection
10. The correct answer is D. The smear would
is typically mediated by pre-formed anti-donor
have a hypersegmented neutrophil, classically
antibodies that are possessed by the recipient.
associated with vitamin B12 deficiency or fo-
Answer D is incorrect. Graft-versus-host dis- late deficiency. In this patient’s case, symptoms
ease (GVHD) is a serious adverse effect of and laboratory findings suggest a deficiency
bone marrow transplantation mediated by in vitamin B12, most likely resulting in perni-
donor-derived T lymphocytes. Acute GVHD cious anemia, given the patient’s age. Autoim-
Immunology

usually occurs within the first three months af- mune gastritis/pernicious anemia is associated
ter an allogeneic bone marrow transplantation, with two forms of autoantibodies: (1) antibod-
whereas chronic GVHD usually develops after ies directed against the transmembrane proton
the third month post-transplant. pumps of parietal cells (the cells that secrete
gastric acid and intrinsic factor [IF]) and (2)
Answer E is incorrect. Acute rejection is cell-
antibodies directed against IF itself. Parietal
mediated and occurs within weeks after trans-
cell autoantibodies lead to their destruction,
plantation of an organ that is major histocom-
thereby decreasing IF and acid production
patibility (MHC)-mismatched; rejection is the
(which then hinders vitamin B12 release from
result of cytotoxic T lymphocytes reacting to
food). IF autoantibodies, on the other hand,
foreign MHC molecules. In this scenario, hy-
prevent either the binding of vitamin B12 to IF,
peracute rejection would be expected to occur
or the binding of the vitamin B12-IF complex
first (within the first few hours after transplant).
to its receptor in the distal ileum. Symptoms of
9. The correct answer is B. This patient’s recur- vitamin B12 deficiency include fatigue, pallor,
rent viral and fungal infections and hypocalce- mild to moderated jaundice, glossitis (a pain-
mia are consistent with DiGeorge syndrome. ful, beefy tongue), and neuropathies (particu-
This condition is caused by the failure of de- larly of the lower extremities due to atrophy of
velopment of the third and fourth pharyngeal the posterior and lateral columns in the spi-
pouches, and thus the thymus and parathyroid nal cord). Serum levels of homocysteine and
glands. The recurrent viral and fungal infec- methylmalonic acid can be elevated, and a
tions are caused by a T-lymphocyte deficiency. complete blood cell count can show thrombo-
In lymph nodes, T lymphocytes are found in cytopenia. Vitamin B12 deficiency also causes
the paracortical region, thus this region is of- a megaloblastic anemia with hypersegmented
ten underdeveloped in patients with DiGeorge neutrophils on peripheral blood smear. Folate
syndrome. deficiency also results in a similar megaloblas-
tic anemia, although is not accompanied by
Answer A is incorrect. Follicles are areas of neurologic symptoms.
mostly B-lymphocyte aggregation, and are not
usually affected in DiGeorge syndrome. Answer A is incorrect. Acute blood loss would
result in a normocytic normochromic anemia.
Answer C is incorrect. The medullary sinuses However, this patient shows signs of vitamin
contain macrophages and communicate with B12 deficiency, which would result in a macro-
lymphatics. cytic anemia with polysegmented neutrophils
on peripheral blood smear.
High-Yield Principles
Chapter 5: Immunology  •  Answers 113

Answer B is incorrect. Anemia of chronic dis- chemicals. IL-12 and IFNγ are not directly in-
ease results in a microcytic, hypochromic ane- volved.
mia (late manifestation) on peripheral blood
Answer D is incorrect. IFN-γ, IL-8, and C5a
smear with decreased serum iron levels and
are major chemotactic factors for neutrophils.
decreased total iron-binding capacity. This
Neutrophils are not antigen-presenting cells
patient shows signs of vitamin B12 deficiency,
and would not secrete IL-12.
which would result in a macrocytic anemia
with polysegmented neutrophils. 12. The correct answer is B. This patient is likely
Answer C is incorrect. Iron deficiency ane- suffering from one of the two major types of
mia can result in hypersegmented neutrophils scleroderma, known as CREST syndrome.
but also results in a microcytic, hypochromic The CREST acronym indicates the major
anemia (late manifestation) on peripheral symptoms: Calcinosis, Raynaud phenomenon,
blood smear with decreased serum iron and Esophageal dysmotility, Sclerodactyly, and
increased total iron-binding capacity. This Telangiectasia. Often, CREST syndrome pa-

Immunology
patient shows signs of vitamin B12 deficiency, tients test positive for anticentromere antibod-
which would result in a macrocytic anemia ies. The answer describes Raynaud phenom-
with polysegmented neutrophils. enon, which may be triggered by cold weather
or even emotional stimuli.
Answer E is incorrect. Pyridoxine deficiency
(vitamin B6), often caused by isoniazid ther- Answer A is incorrect. These symptoms are
apy, results in an acquired sideroblastic anemia not usually associated with CREST syndrome.
(often macrocytic, anisocytosis). However, this Answer C is incorrect. These symptoms are
patient shows signs of vitamin B12 deficiency, suggestive of Sjögren syndrome, not CREST
which would result in a macrocytic anemia syndrome.
with polysegmented neutrophils on peripheral
blood smear. Answer D is incorrect. This is not a symptom
usually associated with CREST syndrome.
11. The correct answer is A. The cell on the left Note that urethritis, conjunctivitis/anterior
is a macrophage and the cell on the right is uveitis, and arthritis make up the classic triad
a helper T cell. The image depicts a macro- in Reiter syndrome.
phage secreting interleukin-12 (IL-12), which Answer E is incorrect. This is not usually a
causes the naïve helper T cell to differentiate hallmark of CREST syndrome but rather of
into a Th1 cell. In turn, the T cell secretes diffuse scleroderma. Note that diffuse sclero-
interferon-γ (IFN-γ), stimulating the macro- derma is associated with anti-Scl-70 antibody.
phage to increase antigen presentation and ly­
sosome production. This series of events is an 13. The correct answer is C. This patient most
important mechanism for increasing killing of likely has hyper-IgM syndrome, which is
intracellular pathogens such as mycobacteria. caused by a defect in the CD40 ligand on
Answer B is incorrect. Eosinophils do not CD4 T helper cells. This defect leads to an in-
present antigens to Th1 cells and are not clasi- ability to class switch between the different im-
cally thought to secrete IL-12. Plasma levels munoglobulin isotypes. Since IgM is initially
of eosinophils increase in hypersensitivity dis- created and subsequently switched to the other
eases such as asthma and in parasitic infection. isotypes, an inability to do so leads to elevated
IgM levels and low levels of all other isotypes.
Answer C is incorrect. Basophils and mast Two prominent clinical problems are Pneumo-
cells are the primary mediators of the immedi- cystis jiroveci and neutropenia.
ate allergic, or type I, hypersensitivity reaction.
These cells are activated by attached IgE and Answer A is incorrect. Ataxia-telangiectasia
produce histamine and other inflammatory is caused by a defect in DNA repair enzymes.
The disease is associated with an IgA defi-
High-Yield Principles
114 Section I: General Principles  •  Answers

ciency. The typical presentation of the disease body titer is usually a good indicator of disease
is given away by the name, as symptoms in- severity, particularly in Wegener granulomato-
clude cerebellar problems (ataxia) and spider sis, for which specificity and sensitivity of anti-
angiomas (telangiectasia). body testing are both >90%.
Answer B is incorrect. Leukocyte adhesion Answer A is incorrect. Kawasaki disease is not
deficiency syndrome is caused by a defect in associated with serum P-ANCA, and Buerger
the LFA-1 adhesion protein on the surface of disease is not associated with serum C-ANCA.
neutrophils. The disease usually presents with
Answer C is incorrect. “Classic” polyarteri-
marked leukocytosis and localized bacterial
tis nodosa is not associated with P-ANCA,
infections that are difficult to detect until they
and Buerger disease is not associated with
have progressed to an extensive life-threatening
C-ANCA.
level. Since neutrophils are unable to adhere
to the endothelium and transmigrate into tis- Answer D is incorrect. Takayasu arteritis is not
sues, infections in patients with leukocyte ad- associated with serum P-ANCA.
Immunology

hesion deficiency syndrome act similarly to Answer E is incorrect. Takayasu arteritis


those observed in neutropenic patients. is not associated with serum P-ANCA, and
Answer D is incorrect. Job syndrome in- Buerger disease is not associated with serum
volves the failure of helper T lymphocytes to C-ANCA.
produce interferon-γ (IFN-γ). Because IFN-γ Answer F is incorrect. Temporal arteritis is not
is a potent activator of phagocytic cells, a de- associated with serum P-ANCA.
crease in its production leads to a failure of
neutrophils to respond to chemotactic stimuli. 15. The correct answer is A. The image reveals
Job syndrome presents with recurrent staphy- Reed-Sternberg cells, which are found in
lococcal abscesses, eczema, and high levels of Hodgkin lymphoma and can be identified by
IgE. their distinctive “owl’s eye” bilobular nuclei.
Answer E is incorrect. In thymic aplasia (Di- Most studies support a B-lymphocyte origin for
George syndrome), the third and fourth pha- Reed-Sternberg cells. Evidence supporting this
ryngeal pouches, and thus the thymus and includes rearrangements at immunoglobulin
parathyroid glands, fail to develop. The disease gene loci and the expression of MHC class II
often presents with many congenital defects, and B7 molecules. Additionally, they are CD
such as cardiac abnormalities, cleft palate, and 30+ and CD 15+, but CD45-.
abnormal facies. Thymic aplasia can also pre­ Answer B is incorrect. Fibroblasts are elevated
sent with tetany due to hypocalcemia. in chronic inflammation and in granulation
tissue. Tumors that are derived from fibroblasts
14. The correct answer is B. Patients suffering are called fibrosarcomas.
from microscopic polyangiitis often have au-
toantibodies against the enzyme myeloperoxi- Answer C is incorrect. Activated macrophages
dase, which stains in a perinuclear pattern and tend to form granulomas, which are groups
is therefore commonly called P-ANCA (peri- of epithelial-like macrophages that are sur-
nuclear anti-neutrophil cytoplasmic antibody). rounded by a collar of mononuclear lympho-
Wegener granulomatosis is a vasculitis charac- cytes and plasma cells. They lack the “owl’s-
terized by necrotizing granuloma formation eyes” nuclei that characterize Reed-Sternberg
in the lungs and kidneys. These patients often cells.
have autoantibodies specific for proteinase-3, Answer D is incorrect. Neutrophil levels are
which stains in a cytoplasmic distribution and elevated in acute infection and inflammation.
is therefore commonly called C-ANCA (cyto- Their appearance is characterized by a three-
plasmic anti-neutrophil cytoplasmic antibody). to four-lobed nucleus and basophilic granules.
In both of these diseases, the patient’s autoanti-
High-Yield Principles
Chapter 5: Immunology  •  Answers 115

Answer E is incorrect. T lymphocytes are el- immune system. Opsonins adhere to microor-
evated in T-lymphocyte leukemias such as ganisms and promote leukocyte chemoattrac-
T-cell acute lymphocytic leukemia and T-cell tion, antigen binding and phagocytosis, and
chronic lymphocytic leukemia. activation of macrophage and neutrophil kill-
ing mechanisms. The antibody IgG is also an
16. The correct answer is E. Juvenile rheumatoid opsonin, and C3b and IgG are the two primary
arthritis (JRA) may appear with a different pre- opsonins responsible for defense against bacte-
sentation than adult-onset rheumatoid arthritis ria.
(RA). By definition, JRA begins before age 16
Answer A is incorrect. IgA patrols mucosal
years and must include arthritis in at least one
barriers and prevents attachment of bacteria
joint for at least six weeks. Additionally, the
and viruses. It does not fix complement, nor is
morphologic joint pathology is similar to that
it an opsonin.
of adult-onset RA. However, there are several
signs and symptoms that occur more com- Answer B is incorrect. IgD is found on the

Immunology
monly in JRA than in adult-onset RA, includ- surface of many B lymphocytes and in the se-
ing an increased likelihood of systemic onset rum, but its function is unclear.
(with symptoms including high fevers, lymph-
Answer C is incorrect. IgE mediates immedi-
adenopathy, and hepatomegaly), large-joint in-
ate (type I) hypersensitivity and immunity to
volvement, and anti-nuclear antibody seroposi-
helminths by facilitating the activation of eo-
tivity. Furthermore, JRA patients are less likely
sinophils.
to have rheumatoid nodules and rheumatoid
factor. Answer E is incorrect. IgM fixes (activates)
complement but does not function by opso-
Answer A is incorrect. There are several signs
nization. It exists as a monomer on the surface
and symptoms that occur more commonly
of B lymphocytes, or as a pentamer in the se-
in JRA than in adult-onset RA, including in-
rum.
creased likelihood of systemic onset, increased
likelihood of large-joint involvement, and in- 18. The correct answer is A. This patient has be-
creased likelihood of anti-nuclear antibody se- come susceptible to opportunistic disease
ropositivity. Age at onset is <16 years. due to infection with HIV. Risk factors for
Answer B is incorrect. Systemic symptoms are contracting HIV include high-risk sexual be-
more likely, but high levels of serum rheuma- havior and intravenous drug use. HIV pref-
toid factor are less likely, in patients with JRA. erentially infects T-helper cells, which stain
positive for CD3 and CD4. However, CD3
Answer C is incorrect. Both systemic symp-
stains all T lymphocytes, while CD4 is spe-
toms and large-joint involvement are more
cific for T-helper cells. At this point in time,
likely in patients with JRA.
the patient can be diagnosed with AIDS based
Answer D is incorrect. Systemic symptoms are on the presence of AIDS-defining opportunis-
more likely, but high levels of serum rheuma- tic infections or by a CD4+ cell count <200/
toid factor are less likely, in patients with JRA. mm3. This patient should be started on highly
active antiretroviral therapy, which commonly
17. The correct answer is D. This patient has al- includes the nucleoside reverse transcriptase
ready had several bacterial infections, most inhibitor zidovudine.
notably an infection with Streptococcus pneu-
Answer B is incorrect. CD8 is specific for
moniae. Complement, part of the innate im-
cytotoxic T lymphocytes. HIV affects only
mune system, is a group of serum proteins that
T-helper cells and does not affect cytotoxic
work with antibody activity to eliminate patho-
T lymphocytes.
gens. Specifically, the protein C3b is respon-
sible for the opsonization of bacteria, and is Answer C is incorrect. CD20 is a marker that
likely the protein that is missing in this child’s is specific for B lymphocytes, which are activated
High-Yield Principles
116 Section I: General Principles  •  Answers

by T-helper cells. A person with HIV infec- 20. The correct answer is D. The infant’s fam-
tion will not be deficient in B lymphocytes. In ily history is suggestive of a trait with X-linked
fact, individuals with HIV may produce a large inheritance, and the preponderance of bacte-
amount of antibodies from B lymphocytes. rial infections suggests a defect in the humoral
(antibody-mediated) immune response. These
Answer D is incorrect. CD40 is found on B
two clues are most suggestive of a diagnosis of
lymphocytes. Costimulation with T lympho-
Bruton X-linked agammaglobulinemia. The
cytes occurs via the CD40 ligand and is re-
molecular defect occurs in a signaling mol-
quired for activation of B lymphocytes. CD40
ecule named BTK (Bruton tyrosine kinase),
is not found on T lymphocytes
leading to maturing arrest of developing B
Answer E is incorrect. CD56 is a marker that cells at the pre-B-cell stage. Arrest at the pre-
is specific for natural killer cells, which are un- B-cell stage would result in an inability to
affected in HIV infection. It should be noted produce immunoglobulins; thus, the patient
that natural killer cells are an important part of would have very low levels of all immunoglob-
Immunology

the body’s defense against tumors. ulins in his serum. In the first several months
of life, the infant can get by without problems
19. The correct answer is B. Celiac sprue is also because of the mother’s IgG (still present).
known as gluten-sensitive enteropathy, non- Indeed, it should be noted that these patients
tropical sprue, and celiac disease. It is due to a are particularly susceptible to extracellular
sensitivity to gluten, which is found in wheat, pyogenic bacterial infections with organisms
grains, and many cereals. Biopsy shows marked such as Haemophilus influenzae, Streptococ-
atrophy, total loss, or flattening of the villi of cus pyogenes, Staphylococcus aureus, and
the small bowel. Streptococcus pneumoniae. The only physical
Answer A is incorrect. Abetalipoproteinemia finding for Bruton patients is the absence, or
is an autosomal recessive disease that causes a near absence, of tonsils and adenoids, which
defect in the synthesis and export of lipids by are B-cell-rich tissues. Patients diagnosed with
mucosal cells because of the inability to syn- Bruton will need to be treated with replace-
thesize apolipoprotein B. These patients usu- ment immunoglobulin.
ally have acanthocytes (or spur cells, RBCs Answer A is incorrect. A CD4 T-lymphocyte
that have spiny projections) and do not have count <200/mm3 suggests a diagnosis of AIDS,
any characteristic features of the intestine not an inherited genetic defect. Without any
found in celiac disease. indication that his mother has HIV, there
Answer C is incorrect. Lactase deficiency would be no reason to suspect HIV for the boy.
causes osmotic diarrhea from the inability to AIDS patients have a defect in cell-mediated
break down lactose into glucose and galactose. immunity, but the repeated bacterial infections
in this case indicate the patient has a defect in
Answer D is incorrect. Viral enteritis, usually humoral immunity.
caused by a rotavirus, is common in children
and can cause diarrhea. However, the clinical Answer B is incorrect. Absence of T lympho-
time course, suggested gluten sensitivity, and cytes would result in a defect in cell-mediated
findings on biopsy make viral enteritis unlikely. immunity, and the patient would be more
highly susceptible to viral and intracellular
Answer E is incorrect. Whipple disease usu- bacterial pathogens. Decreased T cells are
ally presents in middle-aged men who have seen in DiGeorge syndrome because of ab-
malabsorptive diarrhea, and the hallmark is the sence of thymus.
presence of periodic acid-Schiff-positive mac-
rophages in the intestinal mucosa. Rod-shaped Answer C is incorrect. The physical exami-
bacilli of the causal agent, Tropheryma whip- nation and family and patient history are all
pelii, are found on electron microscopy. highly suggestive of an immunoglobulin defi-
ciency.
High-Yield Principles
Chapter 5: Immunology  •  Answers 117

Answer E is incorrect. Low serum IgA levels 22. The correct answer is C. This clinician is
are suggestive of selective IgA deficiency, the concerned that the fetus may have erythro-
most common inherited immunodeficiency blastosis fetalis (hemolytic disease of the new-
in the European population and, interestingly born). This disease is mediated by maternally
enough, one that appears to have no striking derived IgG anti-Rh antibodies developed in
disease associations. These patients may pro- Rh-negative mothers that are directed at the
duce autoantibodies to IgA that make them Rh antigen present on the fetal RBCs of a Rh+
susceptible to anaphylactic reactions when fetus in a previous pregnancy. If the mother
transfused with normal blood products con- possesses the antibodies developed from a pre-
taining IgA. vious exposure to an Rh+ fetus, they may cross
the placenta (antibodies of the IgG isotype
21. The correct answer is A. Systemic lupus ery- readily cross the placenta) and coat the fetal
thematosus (SLE) is a chronic autoimmune RBCs of a Rh+ fetus if the mother is now preg-
disease that affects multiple organ systems, nant with another Rh+ child. Antibody coating

Immunology
including the kidney. It presents with a vari- of the RBCs leads to phagocytosis of RBCs (via
ety of symptoms including arthropathy, malar Fc receptors) and/or destruction of the RBCs
rash, Raynaud phenomenon, and pleural and by the complement system and potentially fa-
pericardial effusions. SLE affects young adults tal anemia. This antibody-mediated cytotoxic
(women almost three times more than men) reaction is an example of a type II hypersensi-
and usually presents with a combined ne- tivity reaction.
phritic and nephrotic picture. Lupus nephrop-
Answer A is incorrect. Graft-versus-host dis-
athy is a heterogeneous renal disease that is a
ease is a potentially lethal side effect of bone
consequence of immune complexes deposited
marrow transplantation.
in the glomerulus. Anti-dsDNA antibodies and
low serum complement levels are markers for Answer B is incorrect. Type I hypersensitivity
renal involvement in lupus. Histologic sec- reactions are antibody-mediated but require
tions of biopsies from an SLE-affected kidney antigen binding to IgE, which is prebound to
have a peculiar wire-loop appearance with sub­ the surface of mast cells. Mast cell degranula-
endothelial basement membrane deposits. In tion then ensues. Examples include anaphy-
general, immunosuppressants are used to mini- laxis, asthma, hives, and local wheal and flare.
mize the inflammatory effects of lupus on the
Answer D is incorrect. Type III hypersensitiv-
kidney.
ity reactions are immune complex-mediated.
Answer B is incorrect. A “tram track” appear- Examples include polyarteritis nodosa, glomer-
ance is typically seen in membranoprolifera- ulonephritis, rheumatoid arthritis, and SLE.
tive glomerulonephritis.
Answer E is incorrect. Type IV hypersensitiv-
Answer C is incorrect. Lupus nephritis is con- ity reactions are a group of T-cell-mediated
sidered a heterogeneous renal disease that can pathologies. Examples include the tuberculin
have a variety of presentations including active skin test, transplant rejection, and contact der-
or inactive diffuse, segmental, or global glo- matitis.
merulonephritis.
23. The correct answer is E. This patient has
Answer D is incorrect. Immune complex de-
Wiskott-Aldrich syndrome, an X-linked disor-
position causes complement activation and
der resulting in the body’s inability to mount
leads to low serum complement levels.
an IgM response to bacteria. Recurrent pyo-
Answer E is incorrect. SLE is associated with genic infections, eczema, and thrombocytope-
anti-dsDNA and anti-Smith antibodies, not an- nia are the typical triad of symptoms. Wiscott-
timitochondrial antibodies. Aldrich syndrome does not present with any
specific enzyme abnormality.
High-Yield Principles
118 Section I: General Principles  •  Answers

Answer A is incorrect. Hypocalcemia is char- infected bite, patients may develop erythema
acteristic of thymic aplasia (DiGeorge syn- migrans at the site of the bite and additional
drome), in which the third and fourth pha- annular lesions. They may also display symp-
ryngeal pouches, and thus the thymus and toms of cardiac and neurologic involvement,
parathyroid glands, fail to develop. As a result including first-degree heart block, myopericar-
patients may experience tetany. Thymic apla- ditis, meningitis, cerebellar ataxia, and seventh
sia often presents with congenital defects such nerve facial palsy. In addition, patients often
as cardiac abnormalities, cleft palate, and ab- experience a migratory musculoskeletal pain.
normal facies. They do not, however, experience respiratory
or ocular symptoms.
Answer B is incorrect. IgA levels are elevated
in Wiskott-Aldrich syndrome. They are low Answer B is incorrect. Pneumonia is an infec-
in hyper-IgM syndrome, ataxia-telangiectasia, tion of the lung involving the alveoli, intersti-
and in selective IgA deficiency. tia, and distal airways whereupon the alveoli
become consolidated with WBCs, RBCs, and
Immunology

Answer C is incorrect. IgE levels are elevated


fibrin. The patient with pneumonia may have
in Wiskott-Aldrich syndrome. They are low
fever, cough, and an increased respiratory
in hyper-IgM syndrome, in which B cells are
rate, as this patient does. However, physical
unable to class switch because of a defect in
exam would display findings of dullness to per-
helper T cells; these patients have high levels
cussion in a lobar distribution in addition to
of IgM and low IgG, IgA and IgE levels.
other breath sounds such as wheezes or râles,
Answer D is incorrect. Wiskott-Aldrich syn- and findings would be unlikely to be bilateral.
drome is characterized by thrombocytopenia Coughing often produces sputum.
and small platelets, rather than lymphopenia.
Answer D is incorrect. SLE is an autoimmune
Lymphopenia is typical of patients with severe
disease in which circulating antibodies and
combined immunodeficiency, who also have
immune complexes cause systemic damage
reduced levels of all immunoglobulin isotypes
to the body. SLE is most prevalent in women
and are infected early in life by opportunistic
and African-American people. Symptoms can
pathogens.
affect all organ systems and cause polyarthri-
24. The correct answer is C. Sarcoidosis is a sys- tis, malar rash, nephritis, vascular occlusions,
temic inflammatory process that involves an and pericarditis. Neurologic symptoms include
exaggerated Th1 immune response of un- headache, cognitive dysfunction, seizures, and
known etiology. It generally causes respiratory myelopathy, but seventh nerve involvement is
symptoms, with the appearance of hilar lymph- not particularly common. Pulmonary symp-
adenopathy and noncaseating granulomas toms usually manifest as a pleuritis with or
in the lungs, but can affect any organ system. without pleural effusion, but one would not
Acute sarcoidosis can develop suddenly over a see granulomas on radiography.
period of weeks, with both constitutional and Answer E is incorrect. TB is caused by the
respiratory symptoms. The Heerfordt-Walden- acid-fast bacterium Mycobacterium tuberculo-
ström syndrome of acute sarcoidosis includes sis, which produces a primary infection of the
the development of fever, parotid enlargement, lungs. Symptoms include fever, night sweats,
anterior uveitis, and facial nerve palsy, as seen malaise, weakness, and dry cough. Radiogra-
in this patient. Of the neurologic symptoms phy may reveal paratracheal or hilar lymph-
seen in sarcoidosis, seventh nerve involvement adenopathy, as in this patient. However, while
is the most common. it is possible that her nonpulmonary symptoms
Answer A is incorrect. Lyme disease is a multi- may occur with extrapulmonary TB, her puri-
system infection caused by the spirochete Bor- fied protein derivative test is negative, making
relia burgdorferi, carried by the Ixodes scapu- it less likely that she has been exposed to the
laris tick. Within the first few weeks after an bacterium.
High-Yield Principles
Chapter 5: Immunology  •  Answers 119

25. The correct answer is D. This patient is likely normal IgG level is characteristic of Wiskott-
presenting with renal amyloidosis. (Remem- Aldrich syndrome. Wiskott-Aldrich syndrome
ber, amyloid deposits show apple-green bire- is an X-linked disorder that results in a reduced
fringence in polarized light). Clonal plasma IgM response to encapsulated bacteria. The
cells express abnormal amounts of light chain triad of symptoms consists of recurrent pyo-
protein. The organs most affected are the kid- genic infections, eczema, and thrombocytope-
neys and the heart. Symptoms include neurop- nia.
athy, proteinuria, edema, hepatosplenomegaly,
Answer D is incorrect. Normal immunoglobu-
and congestive heart failure.
lin levels and a negative nitroblue tetrazolium
Answer A is incorrect. C2 is a complement dye reduction test are characteristic of chronic
protein; complement is a system of proteins granulomatous disease. Chronic granuloma-
that aid in humoral immunity and inflamma- tous disease results from an NADPH oxidase
tion. It is not involved in amyloidosis patho- deficiency that reduces the ability of phago-
genesis. cytes to kill catalase-positive bacteria. It does

Immunology
not present with low levels of immunoglobu-
Answer B is incorrect. CD8 is a protein found
lins. The definitive test for this disorder is a
on cytotoxic T lymphocytes and is involved in
negative nitroblue tetrazolium dye reduction
immune response. It is not involved in amyloi-
test.
dosis.
Answer E is incorrect. Normal immunoglob-
Answer C is incorrect. The Fab fragment is
ulin levels can be seen in thymic aplasia (Di-
the part of the antibody that binds to the anti-
George syndrome). Thymic aplasia, in which
gen and includes both light and heavy chains.
the thymus and parathyroids fail to develop,
Amyloidosis is associated only with abnormal
results from the failure of the third and fourth
light chain protein.
pharyngeal pouches to develop. Consequently,
Answer E is incorrect. Major histocompatibil- T cells are completely absent. Patients with
ity complex II is a protein complex that is in- thymic aplasia typically present with recur-
volved in presenting antigens to immune cells. rent viral and fungal infections. They may also
It is not involved in amyloid protein deposi- have disorders of the great vessels and heart
tion. and may experience tetany due to hypocalce-
mia.
26. The correct answer is B. Selective immuno-
globulin deficiency is a deficit in a specific 27. The correct answer is D. Type IV hypersen-
class of immunoglobulins. IgA deficiency is sitivity reactions (also known as delayed-type
the most common of these. Because IgA is the hypersensitivity) are mediated by previously
most prominent immunoglobulin found in activated T-helper 1 (Th1) cells. These cells
mucous membranes, patients suffering from a recognize the TB-derived peptide and carbo-
deficiency of it can present with sinus and lung hydrate mixture that is administered during
infections. the tuberculin test. These Th1 cells become
Answer A is incorrect. A very high IgE level activated to secrete interferon-γ and tumor
and normal levels of all other immunoglobu- necrosis factor-b, which mediate a local in-
lins are characteristic of Job syndrome. Job flammatory response within 24-48 hours after
syndrome is a disorder of the immune system administration of the injection. These Th1 ef-
that involves the failure of helper T cells to fector cells are present only in individuals who
produce interferon-γ. It presents with multiple have previously been exposed to Mycobacte-
“cold” (or noninflamed) skin lesions and high rium tuberculosis or those who were vaccinated
IgE levels. with bacille Calmette-Guérin. It is important
to note that other type IV hypersensitivity reac-
Answer C is incorrect. A low IgM level in tions include celiac disease and contact hyper-
conjunction with an elevated IgA level and a sensitivities such as poison ivy.
High-Yield Principles
120 Section I: General Principles  •  Answers

Answer A is incorrect. While mast cells are telangiectasia. Ataxia-telangiectasia is an auto-


involved in the pathogenesis of type I hyper- somal recessive disorder caused by a mutation
sensitivity reactions, the tuberculin test is an on chromosome 11q22-23 that results in a de-
example of a type IV reaction. fect in DNA repair enzymes. It manifests as a
variable combination of progressive neurologic
Answer B is incorrect. While cytotoxic T lym-
impairment, cerebellar ataxia, variable im-
phocytes are involved in the pathogenesis of
munodeficiency (usually IgA deficiency) with
type II hypersensitivity reactions, the tubercu-
susceptibility to sinopulmonary infections, im-
lin test is an example of a type IV reaction.
paired organ maturation, x-ray hypersensitivity,
Answer C is incorrect. Mast cells are not di- ocular and cutaneous telangiectasia, and a pre-
rectly involved in the pathogenesis of type IV disposition to malignancy.
hypersensitivity reactions.
Answer A is incorrect. A noninflamed or
Answer E is incorrect. While a positive tuber- “cold” abscess is characteristic of Job syndrome
culin test is an example of a type IV hypersen- which results from a failure of interferon-γ
Immunology

sitivity reaction, it is mediated by T-helper 1 production by helper T cells. The lack of


(Th1) cells, not Th2 cells. Th2 cells are those interferon-γ leads to the failure of the neutro-
that help the humoral (antibody-mediated) phil response to chemotactic stimuli. Patients
arm of the immune response. with Job syndrome present with eczema,
coarse facies, retained primary teeth, and high
28. The correct answer is A. The patient in this levels of IgE.
scenario has myasthenia gravis. This condition
is characterized by ptosis, limb weakness, and Answer B is incorrect. Cleft palate is a com-
difficulty breathing. The key mediator is an mon feature of thymic aplasia (DiGeorge syn-
autoantibody to the acetylcholine receptor on drome). Thymic aplasia results from failure of
the postsynaptic membrane. All antibody mol- the third and fourth pharyngeal pouches (and
ecules consist of two identical heavy chains thus the thymus and parathyroid glands) to de-
and two identical light chains that are held to- velop. The disease often presents with congeni-
gether by disulfide bonds. tal defects such as cardiac abnormalities, cleft
palate, and abnormal facies. Thymic aplasia
Answer B is incorrect. Hydrogen bonds are can also present with tetany due to hypocalce-
weaker than disulfide bonds and do not con- mia.
nect the antibody chains.
Answer D is incorrect. Granulomas are col-
Answer C is incorrect. Ionic bonds are found lections of cells seen in (among other things)
in chemicals such as sodium chloride but are chronic granulomatous disease. This disease
not responsible for holding antibody chains to- is caused by an inability of neutrophils to kill
gether. bacteria once they have phagocytosed them.
Answer D is incorrect. Triple covalent bonds Answer E is incorrect. Visual hallucinations
are seen between some atoms, such as nitro- are not a symptom of any of the known im-
gen, but are not responsible for holding the mune deficiencies.
chains of antibody molecules together.
Answer E is incorrect. Van der Waals forces 30. The correct answer is A. Microscopic poly-
are weak attraction forces and do not play a angiitis is one of the trio of diseases (with
significant role in holding antibody chains to- Wegener granulomatosis and Churg-Strauss
gether. syndrome) that are referred to as the ANCA
(antineutrophil cytoplasmic antibody)-asso-
29. The correct answer is C. The image shows ciated vasculitides. Over 80% of patients with
spider angiomas, which are common on this disease have ANCA, namely the peri-
the ears and cheeks of patients with ataxia- nuclear pattern of staining (P-ANCA) type.
Inflammation of the pulmonary capillaries,
High-Yield Principles
Chapter 5: Immunology  •  Answers 121

which can lead to hemoptysis, is common in Answer D is incorrect. SLE is associated with
these patients, and 90% of patients have necro- anti-nuclear, anti-dsDNA, anti-Smith antibod-
tizing glomerulonephritis (leading to hematu- ies. None of these are particularly associated
ria). Other common symptoms include intesti- with microscopic polyangiitis.
nal pain/bleeding, muscle pain, and weakness.
Answer E is incorrect. While the clinical pre-
Churg-Strauss syndrome also involves the pul-
sentation is similar, Wegener granulomatosis is
monary vasculature and is associated with eo-
associated with cytoplasmic (not perinuclear)
sinophilia as well as elevated P-ANCA levels.
ANCA, which is not very common in patients
Answer B is incorrect. The CREST variant of with microscopic polyangiitis.
scleroderma is associated with anti-centromere
antibodies, which are not particularly associ-
ated with microscopic polyangiitis.
Answer C is incorrect. Sjögren syndrome is

Immunology
associated with antiribonucleoprotein (anti-
RNP) antibody SS-A (Ro), which is present
in 70%-95% of patients, and SS-B (La), which
is present in 60-90% of patients. Neither anti-
body is associated with microscopic polyangi-
itis.
This page intentionally left blank
Chapter 6

Pathology

123
High-Yield Principles
124 Section I: General Principles  •  Questions

Q u e st i o n s

1. A 19-year-old college student is admitted to the (C) Pseudopalisading tumor cells surrounding
hospital for bacterial meningitis. The patient necrotic regions
initially recovers and responds to treatment. (D) Sharply demarcated areas of tumor cells
However, later in her hospital stay she begins located at the grey-white matter junction
to experience headache and blurred vision. (E) Whorled pattern of concentrically ar-
Her physicians note cognitive decline and a ranged spindle cells with psammoma bod-
gait disturbance that was not evident during ies
her brief recovery phase. CT of the head shows
enlarged ventricles. Which of the following 3. A 43-year-old woman presents to her primary
mechanisms is most likely responsible for the care physician for a regular check-up. A neuro-
patient’s deterioration? logic examination shows that when the patient
looks to the left, the right eye stops at the mid-
Pathology

(A) Destruction of brain parenchyma


line and the left eye shows monocular horizon-
(B) Excessive cerebrospinal fluid production
tal nystagmus. When the patient looks to the
(C) Fusion of the cranial sutures
right, both eyes seem to move appropriately.
(D) Impaired cerebrospinal fluid resorption
Convergence is normal. What is the most
(E) Obstruction of the cerebral aqueduct
likely cause of this patient’s findings?
2. A 41-year-old man visits his physician because (A) Lesion of the left medial longitudinal fas-
of increasingly painful headaches. CT of the ciculus
head reveals an abnormality and an MRI of (B) Lesion of the left sixth cranial nerve
the brain with contrast (shown below) is per- (C) Lesion of the right medial longitudinal fas-
formed for further evaluation. If a biopsy of ciculus
this tumor were obtained, what would the pa- (D) Lesion of the right oculomotor nerve
thologist likely see under the microscope? (E) Pupillary light-near dissociation
(F) Relative afferent pupillary defect

4. A 22-year-old man presents to his primary care


physician with complaints of weakness and a
rash. Physical examination shows slight oral
mucosal bleeding and diffuse petechiae. He
has a history of generalized tonic-clonic sei-
zures that are well controlled with medication
but is otherwise healthy. Results of a bone mar-
row aspirate are shown in the image. Which
of the following is the most likely cause of his
symptoms?

Courtesy of Dr. Per-Lennart Westesson, University of Roches-


ter Medical Center.

(A) Densely packed cells with halos of cyto-


plasm surrounding large round nuclei
(B) Perivascular pseudorosettes with tumor
cells surrounding vessels
High-Yield Principles
Chapter 6: Pathology  •  Questions 125

(A) Compared to the general population,


smoking would present an increased risk of
bronchogenic cancer in this patient
(B) Lungs in this patient will have silicotic
nodules
(C) The findings are associated with exposure
to air pollution
(D) The patient does not have an increased
risk of mesothelioma
(E) The patient has a condition associated
with increased tuberculosis susceptibility
(F) The patient will have localized intra-
alveolar fibrosis
Reproduced, with permission, from USMLERx.com.
6. A 7-year-old African-American boy is brought

Pathology
to the pediatrician’s office by his mother after
(A) Adverse drug reaction he begins crying inconsolably and complain-
(B) Alcoholism ing that his fingers hurt. His mother reports
(C) Idiopathic thrombocytopenic purpura that he had been playing in the sun all day
(D) Pernicious anemia long. A peripheral blood smear is shown in the
(E) Vitamin C deficiency image. Which of the following complications
is associated with this patient’s disease?
5. A 57-year-old man presents with a cough and
progressively increasing shortness of breath.
The patient has been a plumber for 20 years,
and before that job he worked on ships. High-
resolution CT reveals areas of pleural thicken-
ing with calcified plaques. A specially stained
specimen from a patient with a similar condi-
tion is shown in the image. Which of the fol-
lowing best describes the patient’s condition?

Courtesy of the Sickle Cell Foundation of Georgia: Jackie


George, Beverly Sinclair.

(A) Myelofibrosis
(B) Cyclical fevers
(C) Eosinophilia
(D) Microangiopathic hemolytic anemia
(E) Acute chest syndrome
Courtesy of Dr. Edwin P. Ewing Jr, Centers for Disease
Control and Prevention.
High-Yield Principles
126 Section I: General Principles  •  Questions

7. A mother takes her previously healthy 7-year-


old son to the doctor because he appears
“puffy.” Physical examination reveals pitting
edema bilaterally in his lower limbs and swell-
ing of his eyelids. Urinalysis reveals a pH of 5
with 4+ proteinuria; glucose and ketones are
absent. What is most likely the cause of the pa-
tient’s edema?
(A) Congestive heart failure
(B) A thickening of the glomerular basement
membrane
(C) A loss of the negative charge in the glo-
merular filtration barrier
(D) Inadequate dietary protein Reproduced, with permission, from USMLERx.com.
Pathology

(E) Split glomerular basement membrane sec-


ondary to a mutation in type IV collagen
(A) To block attachment of bacteria and vi-
8. A 26-year-old woman visits her physician with ruses to mucous membranes
complaints of vaginal bleeding after sexual in- (B) To fix complement and serve as an antigen
tercourse. She started menses at age 14 years receptor on the surface of B lymphocytes
and has 32-day cycles. She acknowledges hav- in the primary immune response
ing unprotected sex with multiple partners. (C) To increase interleukin-2 receptor synthe-
Cytologic specimens are taken from the cervix sis by helper T-lymphocyte and B-lympho-
and vagina. On microscopy, cervical cells have cyte proliferation
large nuclei with open chromatin; several cells (D) To mediate a type I hypersensitivity reac-
have mitotic figures. What additional findings tion by causing the release of secretory
would most likely be present in the specimens products from basophils or mast cells
that account for these findings? (E) To opsonize bacteria, neutralize toxins and
viruses, and fix complement in the second-
(A) Double-stranded DNA virus
ary immune response
(B) Gram-negative diplococci
(F) To promote vasodilation and leukocyte ex-
(C) Gram-positive cocci
travasation
(D) Single-stranded RNA
(G) To stimulate growth of helper and cyto-
(E) Squamous cells covered with bacteria
toxic T lymphocytes
9. A 67-year-old man is admitted to the hospital
after fracturing the neck of his right femur. A
pathologic fracture is suspected. Serum pro-
tein electrophoresis shows an abnormal mono-
clonal paraprotein spike. Subsequent bone
marrow biopsy demonstrates an abnormal
proliferation of the cells shown in the image.
Which of the following describes the function
of the secretory product these cells normally
produce?
High-Yield Principles
Chapter 6: Pathology  •  Questions 127

10. Autopsy of a patient shows bilaterally enlarged, 12. A 51-year-old man complains lately of recur-
cystic kidneys and a vascular lesion at the base rent vomiting. He cannot remember when it
of the brain, as seen in the image. Which chro- began, but he says he is concerned because
mosome was most likely mutated in the pa- sometimes he throws up blood. On physi-
tient? cal examination, there is some lower extrem-
ity edema, and some tortuous veins on the
surface of his abdomen. He admits to having
several recent sexual partners, and has never
been tested for sexually transmitted diseases.
He also reports drinking a “few” drinks a night.
Results of a liver biopsy are shown in the im-
age. Which of the following best describes the
pathologic process occurring in this patient’s
liver?

Pathology
Reproduced, with permission, from USMLERx.com.

(A) 4
(B) 15
(C) 16
(D) 22
(E) X

11. A 3-year-old developmentally delayed girl pre­


sents to the pediatric neurologist for evalua-
Reproduced, with permission, from USMLERx.com.
tion of new onset seizures. The parents are also
concerned because the child frequently exhib-
its inappropriate outbursts of laughter. Physical (A) At this stage in disease, hepatocytes fail to
examination is significant for abnormal facies regenerate
marked by microcephaly, deep-set eyes, and (B) Lipid deposition is taking place
a large mouth with a protruding tongue. The (C) Mallory bodies are commonly seen in this
child’s gait is unstable. The most likely diagno- condition
sis is an example of which of the following ge- (D) The architectural changes that are occur-
netic phenomena? ring are irreversible
(A) Anticipation (E) The liver will be enlarged
(B) Heteroplasmy
(C) Imprinting
(D) Locus heterozygosity
(E) Mosaicism
High-Yield Principles
128 Section I: General Principles  •  Questions

13. A 55-year-old recent immigrant from Taiwan 15. A patient with AIDS and a CD4+ cell count
presents to the clinic with a three-month his- <50/mm3 is suffering from an infection that af-
tory of worsening nasal congestion, epistaxis, fects his lungs, eyes, gastrointestinal tract, and
and recurrent ear infections. Physical examina- central nervous system. Results of a biopsy are
tion reveals painless firm lymph node enlarge- shown in the image. With what is the patient
ment in the neck. CT of the head reveals a most likely infected?
large mass situated in the upper nasopharynx.
Biopsy of the lesion shows large epithelioid
cells intermixed with numerous infiltrating
lymphocytes. The infectious agent directly as-
sociated with this patient’s pathology is best de-
scribed by which category?
(A) DNA virus
(B) Eubacterium
Pathology

(C) Fungus
(D) Mycobacterium
(E) RNA virus

14. A 56-year-old man is admitted to the emer-


gency department with a chief complaint of se-
Courtesy of Dr. Edwin P. Ewing Jr, Centers for Disease Con-
vere abdominal and flank pain. The physician
trol and Prevention.
orders x-ray of the abdomen, as shown in the
image. The patient’s condition necessarily im-
plies which of the following? (A) Candida albicans
(B) Cryptococcus neoformans
(C) Cytomegalovirus
(D) Herpes simplex virus
(E) Mycobacterium avium
(F) Pneumocystis jiroveci

16. The attending pathologist reviews a hematoxy-


lin-eosin-stained slide from the liver biopsy of
a 50-year-old man suffering from dyspnea on
exertion, lower extremity edema, and orthop-
nea. In addition, the patient has recent onset
diabetes mellitus and testicular atrophy. One
of the patient’s four brothers also has the same
set of medical problems. The pathologist per-
forms one more study, which confirms the sus-
Reproduced, with permission, from USMLERx.com. pected diagnosis (see image). Which of the fol-
lowing is the most likely diagnosis?
(A) Acidic urine, pH <7.2
(B) Alkaline urine, pH >7.2
(C) Ammonium in the urinary tract
(D) Sodium chloride in the collecting ducts
(E) Uric acid in the renal pelvis
High-Yield Principles
Chapter 6: Pathology  •  Questions 129

(A) APC
(B) BRCA1
(C) MSH2
(D) NF1
(E) p53

18. A 5-year-old boy presents with an unsteady gait


and severe vertigo and nausea. A brain lesion
is seen on CT scan. A biopsy of the lesion is
shown in the image. Where in the brain is the
patient’s lesion?

Reproduced, with permission, from USMLERx.com.

Pathology
(A) a1-Antitrypsin deficiency
(B) Alcoholic hepatitis
(C) Cystic fibrosis
(D) Hereditary hemochromatosis
(E) Wilson disease

17. A 37-year-old HIV-positive man presents for


evaluation of anogenital lesions. He states that
the lesions have been present for years, but
have recently grown in size and become pru-
ritic and tender. On examination he is circum- Reproduced, with permission, from USMLERx.com.
cised and has multiple hyperkeratotic papules
on his penis shaft, perineum, and anal area.
(A) Cerebellar vermis
He also has a palpable rectal mass with guaiac-
(B) Intermediate section of the cerebellar
positive stool and conjunctival pallor. On fur-
hemisphere
ther questioning, he admits to recent uninten-
(C) Lateral section of the cerebellar hemi-
tional weight loss, constipation, and bloating.
sphere
His CD4+ cell count is 150/mm3 and his he-
(D) Occipital cortex
matocrit is 26%. CT of the abdomen shows a
(E) Postcentral gyrus of the parietal lobe
3 × 4-cm rectal mass with multiple metastatic
lesions in his liver. What tumor-suppressor
protein is targeted by the virus causing this pa-
tient’s rectal cancer?
High-Yield Principles
130 Section I: General Principles  •  Questions

19. A 64-year-old retired shipyard worker has been 20. A 12-year-old boy is brought to the doctor for
experiencing shortness of breath, a cough, and progressive fatigue and shortness of breath on
chest pain for five months. In that time he has exertion. He was born full term and has been
lost 14.5 kg (32 lb). He develops progressive as- generally healthy until a few months ago,
cites, and ultimately dies due to a pulmonary when his parents noticed that he became tired
embolus. Autopsy results are shown in the im- after only five minutes of playing with his sib-
age. Exposure to which substance is a risk fac- lings. Physical examination reveals a loud,
tor for this patient’s disorder? holosystolic murmur and mild cyanosis of his
lips. Examination of his hands reveals club-
bing of the fingers. Which of the following car-
diac defects is most likely to be found on echo-
cardiogram?
(A) Bicuspid aortic valve
(B) Left-to-right shunt
Pathology

(C) Mitral regurgitation


(D) Right-to-left shunt
(E) Valvular vegetations

Reproduced, with permission, from USMLERx.com.

(A) Aflatoxin B
(B) Asbestos
(C) Benzene
(D) Cadmium
(E) Silica
High-Yield Principles
Chapter 6: Pathology  •  Answers 131

An s w e r s

1. The correct answer is D. The arachnoid gran- 2. The correct answer is C. This patient has a le-
ulations are outgrowths of the arachnoid mater sion consistent with glioblastoma multiforme
through the dura mater and into the superior (GBM), which is the most common primary
sagittal sinus. They are responsible for return- brain tumor. The MRI shows an irregular mass
ing fluid from the cerebrospinal fluid (CSF) lesion with avid peripheral contrast enhance-
into general circulation. Following irritation of ment and central hypoenhancement, which is
the meninges secondary to meningitis or sub- due to central necrosis suggestive of high-grade
arachnoid hemorrhage, the arachnoid granu- malignancy. Note the “butterfly” shape of the
lations may become scarred and fail to resorb lesion due to fact that it narrows as it passes
adequate CSF. This causes CSF to build up through the corpus collosum while crossing
in both the subarachnoid space and the ven- the midline—this is a classic imaging finding
tricles, producing communicating hydrocepha- in GBM. On histopathology, the lesion con-

Pathology
lus. sists of highly malignant astrocyte tumor cells
that surround areas of necrosis; this is known
Answer A is incorrect. Enlarged ventricles are
as pseudopalisading. GBM has been associated
commonly seen in advanced neurodegenera-
with genetic alterations, including loss of p53
tive diseases such as Alzheimer’s. As brain mass
function, increased activity of the epidermal
is progressively lost, CSF accumulates to fill
growth factor receptor gene (EGFR), and loss
the void, a phenomenon known as hydroceph-
of heterozygosity on chromosome arm 10q.
alus ex vacuo. Substantial loss of brain paren-
GBM has a poor prognosis, with a mean sur-
chyma is not generally seen in meningitis and
vival of 8-10 months after diagnosis; most pa-
thus is an unlikely mechanism for the develop-
tients die within two years.
ment of hydrocephalus in this setting.
Answer A is incorrect. Densely packed cells
Answer B is incorrect. Overproduction of
with halos of cytoplasm surrounding large
CSF by the choroid plexus is an uncom-
round nuclei are characteristic of oligoden-
mon cause of hydrocephalus. It may occur in
drogliomas, which are slow-growing tumors
the setting of a choroid plexus papilloma but
originating in the cerebral hemispheres. These
would not be a consequence of meningitis.
lesions have a better prognosis than astrocyto-
Answer C is incorrect. Closure of the cranial mas, with a mean patient survival of 5-10 years.
sutures is a normal phenomenon that occurs as
Answer B is incorrect. Perivascular pseudo-
infants grow and is not responsible for the de-
rosettes with tumor cells surrounding vessels
velopment of hydrocephalus. In young infants,
are characteristic of ependymomas, which are
the flexibility of the cranium prior to suture fu-
tumors located in the periventricular space or
sion allows the head to enlarge noticeably in
in the spinal cord that may obstruct the flow of
the setting of hydrocephalus.
CSF.
Answer E is incorrect. Obstruction of the flow
Answer D is incorrect. Sharply demarcated
of CSF within the ventricular system leads to
areas of tumor cells located at the grey-white
noncommunicating hydrocephalus, in which
matter junction are characteristic of secondary
pressure builds up within the ventricles, but
metastatic lesions from a primary tumor else-
not in the subarachnoid space. Such obstruc-
where in the body, most commonly the breast,
tion is commonly the result of brain tumors
lung, thyroid, skin, kidney, and gastrointestinal
(particularly neoplasms of the ependymal cells
(GI) tract.
lining the ventricles), or congenital malforma-
tions. Answer E is incorrect. A whorled pattern of
concentrically arranged spindle cells with
calcified psammoma bodies is characteristic
High-Yield Principles
132 Section I: General Principles  •  Answers

of meningiomas, which are the second most Answer E is incorrect. Pupillary light-near
common primary intracranial neoplasm. Me- dissociation is also known as Argyll Robertson
ningiomas are benign, slow-growing tumors of pupil and is an absent miotic reaction to light
the meninges. with preserved accommodation. This condi-
tion can occur in neurosyphilis, diabetes, and
3. The correct answer is C. The description is systemic lupus erythematosus.
consistent with internuclear ophthalmoplegia,
Answer F is incorrect. A relative afferent
which consists of ipsilateral medial rectus palsy
pupillary defect is also known as a Marcus
on attempted lateral conjugate gaze away from
Gunn pupil. It results from a lesion in the
the lesion with associated monocular hori-
afferent limb of the pupillary light reflex and
zontal nystagmus in the contralateral abduct-
can be seen in the retrobulbar neuritis of mul-
ing eye with preserved convergence. It results
tiple sclerosis. It would not result in medial
from damage to the ipsilateral medial longitu-
rectus palsy on attempted lateral conjugate
dinal fasciculus (MLF), which is the connec-
gaze.
tion between the abducent and oculomotor
Pathology

nuclei, so that the two nerves’ actions become


4. The correct answer is A. The bone marrow
unlinked. The third nerve and medial rectus
aspirate is hypocellular without any abnormal
muscle function normally in convergence but
cells, suggestive of aplastic anemia. Given the
not during actions that require conjugate eye
clinical history, this patient’s seizures were
movements. In young patients, it is most com-
likely treated with carbamazepine, which can
monly the result of central nervous system in-
cause aplastic anemia. Other causes of aplas-
fection or multiple sclerosis, whereas in older
tic anemia include viruses (Epstein-Barr virus,
patients it is more commonly the result of vas-
hepatitis C, and parvovirus B19), toxins (ben-
cular disease.
zene and insecticides), and other drugs (can-
Answer A is incorrect. A lesion of the left me- cer chemotherapeutics). Aplastic anemia can
dial longitudinal fasciculus (MLF) would pro- also be idiopathic.
duce an internuclear opthalmoplegia, as in
Answer B is incorrect. Alcoholism has many
this case. However, the findings of a left MLF
metabolic and clinical sequelae, including
lesion would include impaired left (ipsilateral)
decreases in all cell lines due to marrow sup-
eye adduction with rightward gaze as well as
pression. Therefore, this patient’s clinical pic-
right (contralateral) eye horizontal nystagmus.
ture could be explained by alcoholic cirrhosis.
The clinical description in the vignette is the
However, the hypoplastic bone marrow and
mirror image, consistent with a lesion of the
known antiepileptic drug use make the diagno-
right MLF.
sis of aplastic anemia most likely.
Answer B is incorrect. A pure sixth-nerve
Answer C is incorrect. This patient does have
palsy would have almost the opposite effect of
symptoms of thrombocytopenia, but the bone
this scenario and would cause a lateral rectus
marrow aspirate should appear normal in idio-
palsy as opposed to a medial rectus palsy seen
pathic thrombocytopenic purpura.
here.
Answer D is incorrect. Pernicious anemia,
Answer D is incorrect. Recall that cranial
the leading cause of megaloblastic anemia,
nerve (CN) III, or the oculomotor nerve, in-
does uncommonly occur in this age group.
nervates all the extraocular muscles with the
However, the bone marrow aspirate is not con-
exception of the lateral rectus (innervated by
sistent with pernicious anemia, as this disease
CN VI) and the superior oblique (innervated
does not affect neutrophils, platelets, or any
by CN IV). In a lesion of CN III the affected
myeloid cell line other than erythrocytes.
eye looks “down and out” while at rest, or ab-
ducted and depressed.
High-Yield Principles
Chapter 6: Pathology  •  Answers 133

Answer E is incorrect. Although vitamin C is explained by a vasoocclusive crisis, a situa-


deficiency can cause gingival bleeding, it does tion in which sickled cells clog the microvas-
not cause anemia or a hypoplastic bone mar- culature and produce painful tissue ischemia.
row. Sickle cell anemia also is characterized by
splenic sequestration (with eventual functional
5. The correct answer is A. The image shows the asplenia and an increased risk for sepsis and
classic “barbell” ferruginous bodies seen in as- osteomyelitis), cerebrovascular accidents, and
bestosis. These bodies can only be seen when acute chest syndrome. Acute chest syndrome is
the slide is stained with Prussian blue. Asbes- a severe complication of vasoocclusive disease
tosis is most common in plumbers, construc- and presents with chest pain, hypoxemia, and
tion workers, and shipbuilders. Pleural plaques chest infiltrates. Because hypoxemia can insti-
are suggestive of asbestos exposure. Patients gate further sickling and pulmonary vasoocclu-
with asbestosis have an increased risk of bron- sion, acute chest syndrome is an indication for
chogenic carcinoma. This risk is compounded emergent exchange transfusion.
greatly by smoking, making smoking an even

Pathology
Answer A is incorrect. Myelofibrosis (agno-
riskier habit in someone with asbestosis.
genic myeloid metaplasia) is associated with
Answer B is incorrect. Silicotic nodules are teardrop cells and nucleated RBCs on periph-
seen in the lungs of patients with silicosis, eral blood smear. In this disorder, dysplastic
while ferruginous bodies and ivory-white pleu- megakaryocytes produce increased fibroblast
ral plaques are seen in lungs of patients with growth factors, leading to marrow fibrosis and
asbestosis. The ferruginous bodies can be de- marrow failure.
tected on biopsy by Prussian blue staining.
Answer B is incorrect. Cyclical fevers are
Answer C is incorrect. Anthracosis (carbon characteristic of malaria. Intracorpuscular
deposits) is seen in patients exposed to pollu- parasites would be anticipated on peripheral
tion (usually seen in city-dwellers). This would blood smear.
grossly look like black spots on the lung, and is
Answer C is incorrect. Eosinophilia is associ-
not considered to be pathologic.
ated with parasite infection and would not ex-
Answer D is incorrect. The patient has an in- plain the sickled cells seen in the image. In
creased risk of both mesothelioma and bron- addition parasite infections are associated more
chogenic carcinoma. often with iron deficiency anemia and fatigue,
Answer E is incorrect. Silicosis, a disease rather than with acute painful episodes.
caused by exposure to free silica dust, is associ- Answer D is incorrect. Microangiopathic he-
ated with increased tuberculosis susceptibility. molytic anemia is found in both disseminated
Asbestosis is not associated with this increased intravascular coagulation and thrombotic
susceptibility. thrombocytopenic purpura. Schistocytes (frag-
Answer F is incorrect. Lungs with asbesto- mented RBCs) are found on the blood smear.
sis have diffuse pulmonary interstitial fibrosis
7. The correct answer is C. The presentation
caused by inhaled asbestos fibers. The distribu-
described is typical of nephrotic syndrome.
tion is not localized or intra-alveolar.
Minimal change disease is the most common
6. The correct answer is E. This patient’s blood cause of nephrotic syndrome in children. It is
smear is strongly suggestive of sickle cell ane- characterized by a lack of abnormalities visible
mia, an autosomal recessive disorder caused by by light microscopy, and it usually does not in-
defective hemoglobin S (HbS). Hypoxic con- volve any abnormalities that can be seen using
ditions result in the polymerization and “sick- immunofluorescence. Only when examined
ling” of HbS, leading to vulnerable RBCs and with electron microscopy does the tissue show
hemolytic anemia. This patient’s history likely effacement of the foot processes of visceral epi-
thelial cells (podocytes). It is thought that the
High-Yield Principles
134 Section I: General Principles  •  Answers

selective proteinuria that is present in minimal Answer C is incorrect. There are no gram-
change disease is caused by a loss of the fixed positive cocci associated with cervical cancer.
negative charge in the glomerular filtration
Answer D is incorrect. There is no single-
barrier. The disease usually responds to treat-
stranded virus associated with cervical cancer.
ment with steroids.
Answer E is incorrect. Squamous cells cov-
Answer A is incorrect. Edema is most often
ered with bacteria can be found in bacterial
caused by cirrhosis, nephrotic syndrome, or
vaginosis, which causes foul-smelling dis-
congestive heart failure (CHF). CHF in a
charge. There is no known association be-
child is uncommon. If the patient were to have
tween bacterial vaginosis and cervical cancer.
CHF, a different set of symptoms would be
seen, including exertional dyspnea and parox- 9. The correct answer is E. This patient most
ysmal nocturnal dyspnea. likely has multiple myeloma, a hematologic
Answer B is incorrect. Thickening of the glo- cancer in which the primary pathologic pro-
merular basement membrane (GBM) is seen cess is a neoplastic, monoclonal proliferation
Pathology

in patients with membranous glomerulopa- of plasma cells within the bone marrow. Com-
thy, which is a leading cause of nephrotic syn- mon manifestations of the disease include
drome in adults. It has characteristic findings bone pain, lytic bone lesions (which can lead
on light microscopy (thickening of the GBM), to pathologic fractures), renal insufficiency,
and this is in contrast with the lack of light mi- susceptibility to infection, and hypercalce-
croscope changes for minimal change disease. mia. The primary secretory product of the
plasma cells in multiple myeloma is IgG (55%
Answer D is incorrect. Urinalysis shows that
of cases). IgG functions to opsonize bacteria,
the patient is losing protein in the urine, and
neutralize toxins and viruses, and fix comple-
his physical appearance is not that of protein-
ment in the immune response.
calorie malnutrition.
Answer A is incorrect. This choice describes
Answer E is incorrect. The answer choice de-
the function of IgA, which blocks attachment
scribes Alport syndrome, which often causes
of bacteria and viruses to mucous membranes.
a nephritic syndrome with gross hematuria,
Although IgA also is produced by plasma cells,
but less significant protein loss. This child has
IgA-producing multiple myelomas comprise
significant edema, suggesting that a nephrotic
only 25% of all cases of multiple myeloma.
syndrome may be the culprit.
Answer B is incorrect. This choice describes
8. The correct answer is A. The findings on the function of IgM. Although IgM also is
microscopy indicate a malignant transforma- produced by plasma cells, IgM-producing
tion of normal cells. Increased nuclear-to- multiple myelomas are rare. Waldenström
cytoplasm ratio, open chromatin, and mitotic macroglobulinemia is an uncommon form of
figures indicate that cells are actively dividing. lymphoma that secretes monoclonal IgM para-
Given the patient’s history of multiple sexual protein. This condition, however, typically is
partners and postcoital bleeding, these cells not associated with bone lesions.
likely indicate the presence of cervical cancer.
Answer C is incorrect. This answer describes
Cervical cancer is commonly linked to human
the function of tumor necrosis factor-a, a criti-
papillomavirus infection. The virus can be iso-
cal inflammatory cytokine produced by macro-
lated during cytology through various meth-
phages.
ods. It is a double-stranded DNA virus.
Answer D is incorrect. This choice describes
Answer B is incorrect. Gram-negative diplo-
the function of IgE, which mediates type I hy-
cocci are not associated with cervical cancer.
persensitivity reactions. Multiple myelomas
However, they can be found in gonorrhea cer-
producing IgE are rare.
vicitis.
High-Yield Principles
Chapter 6: Pathology  •  Answers 135

Answer F is incorrect. This answer describes mental retardation and speech impairment
the function of histamine, which is stored in are usually present, and 80%-90% of patients
granules in mast cells and basophils. It is re- have epilepsy. Angelman syndrome, along with
leased in response to the cross-linking of IgE Prader-Willi syndrome, is a classic example of
molecules on the surfaces of these cells. imprinting, which occurs when the phenotype
differs depending on whether the mutation is
Answer G is incorrect. This choice describes
of paternal or maternal origin. Deletions in
the function of interleukin-2, a cytokine that is
Prader-Willi syndrome, a phenotypically dis-
important in triggering the secondary immune
tinct disorder, occur exclusively on the pater-
response. It is produced by helper T lympho-
nal chromosome 15, whereas deletions at the
cytes rather than plasma cells.
same site of chromosome 15 on the maternal
10. The correct answer is C. The patient most chromosome result in Angelman syndrome.
likely had adult polycystic kidney disease Answer A is incorrect. Anticipation is the
(APKD). Eighty-five percent of patients with phenomenon in which the severity of a dis-

Pathology
this disease have a mutation in the PKD1 ease worsens in succeeding generations. This
gene, which is located on the short arm of occurs, for example, in triplet repeat diseases
chromosome 16. Patients with APKD suffer such as Huntington disease, wherein the trip-
from renal failure due to multiple, large cysts let repeat tends to lengthen, age of onset de-
in the kidney. These patients can also develop creases, and disease severity increases with
intracranial berry aneurysms (as seen in the successive generations. This does not occur in
image). Angelman syndrome.
Answer A is incorrect. Huntington disease is Answer B is incorrect. Heteroplasmy describes
caused by a nucleotide repeat expansion in the the presence of both normal and mutated mi-
Huntington gene on the short arm of chromo- tochondrial DNA. This phenomenon is re-
some 4. sponsible for the variable expression of mito-
Answer B is incorrect. Marfan syndrome is chondrial inherited diseases.
caused by a mutation in the fibrillin-1 gene on Answer D is incorrect. Locus heterozygosity
the long arm of chromosome 15. describes the phenomenon by which muta-
Answer D is incorrect. Neurofibromatosis type tions at different loci can result in the same
2 is caused by a mutation in the merlin tumor phenotype. An example of this is albinism,
suppressor gene on the long arm of chromo- which can be caused by a number of different
some 22. mutations.

Answer E is incorrect. Many diseases are due Answer E is incorrect. Mosaicism occurs
to mutations of the X chromosome. One ex- when cells in the body have different genetic
ample is fragile X syndrome, which is due to makeup. This sometimes occurs, for example,
a nucleotide repeat expansion in the FMR1 in Turner syndrome. This does not occur in
RNA-binding protein gene on the X chromo- Angelman syndrome.
some.
12. The correct answer is D. The image demon-
11. The correct answer is C. This child most strates micronodular cirrhosis, typically seen
likely has Angelman syndrome. Individuals with regenerative nodules with thick collag-
with this phenotype have a characteristic fa- enous septa. Some proliferation of bile ducts
cies with microcephaly, maxillary hypoplasia, is also seen. This microscopic view of the
deep-set eyes, and a large mouth with tongue liver is most consistent with cirrhosis. In end-
protrusion. Their gait is jerky and “puppet- stage disease, it is often difficult to determine
like,” and their behavior is marked by frequent the primary etiology, but the most common
paroxysms of inappropriate laughter. Severe cause in the Western World is alcoholic liver
disease (60-70% of cases), followed by viral
High-Yield Principles
136 Section I: General Principles  •  Answers

hepatitis. An important feature of cirrhosis is 13. The correct answer is A. This patient has de-
that the normal architecture is disrupted. This veloped nasopharyngeal carcinoma, a condi-
can lead to impedance of blood and bile flow, tion common in certain parts of the world, in-
causing portal hypertension and/or cholestasis cluding Asia and Africa. Development of this
with jaundice. The early stages of alcoholic tumor is always associated with infection by
liver disease, such as steatosis (build-up of lipid Epstein-Barr virus (EBV), a DNA virus in the
droplets within cells) and alcoholic hepatitis, herpesvirus family. Development of this tumor
are reversible if alcohol use is discontinued. is believed to be related to a synergistic interac-
However, once the process of fibrosis begins, it tion between EBV and a diet high in carcino-
is irreversible. genic nitrosamines (common in foods that has
been smoked or preserved). Common symp-
Answer A is incorrect. In cirrhosis, hepatocyte
toms include nasal congestion, epistaxis, ear
regeneration continues between the fibrous
infections (due to tumor-induced blockage of
septae, forming uniform “micronodules.”
the eustachian tubes), and headache.
Thus, although the liver ultimately loses its
Pathology

function and normal architecture, regenera- Answer B is incorrect. Many bacteria are ca-
tion of hepatocytes still occurs. pable of infecting the nasopharynx; however,
none are directly associated with malignancy.
Answer B is incorrect. Steatosis is character-
ized by accumulation of lipid droplets within Answer C is incorrect. Nasopharyngeal zygo-
hepatocytes. This is a common characteris- mycosis is a condition that could present with
tic in many disease states such as alcoholic these symptoms in an immunocompromised
and nonalcoholic steatohepatitis, the latter of patient. However, biopsy would show filamen-
which is strongly associated with obesity and tous nonseptate hyphae and a granulomatous
hyperlipidemia. It is important to remember response.
that these disease states are often reversible
Answer D is incorrect. Although a tubercu-
when the offending agent (eg, alcohol use or
loma in the nasopharynx can be confused with
high serum lipid levels) is removed. However,
a nasopharyngeal tumor, biopsy would show
if untreated, these conditions can eventually
caseating granulomas with multinucleated gi-
progress to cirrhosis. Regenerative nodules and
ant cells.
fibrotic bands are not seen in steatosis.
Answer E is incorrect. Although a retrovirus
Answer C is incorrect. Mallory bodies, degen-
such as HIV can create an immunocompro-
erating hepatocytes full of eosinophilic cyto-
mised state favoring the development of a ma-
plasmic inclusions, are common in alcoholic
lignancy, it is not the direct cause of tumor
hepatitis. Alcoholic hepatitis is also character-
formation. Lymphomas can be associated with
ized by neutrophilic infiltrates, ballooning of
the retrovirus human T-cell lymphoma virus;
hepatocytes, and cytokeratin intermediate fila-
however, biopsy would show sheets of malig-
ments. However, the characteristic nodular re-
nant T lymphocytes typical of this lymphoma.
generation with scarring seen in cirrhosis is not
Other RNA viruses are not associated with ma-
a feature of alcoholic hepatitis. If alcohol use
lignancy.
continues over years, alcoholic hepatitis pro-
gresses to cirrhosis. However, Mallory bodies 14. The correct answer is B. The abdominal ra-
are rare in the cirrhotic liver. diograph shows staghorn renal calculus in
Answer E is incorrect. In alcoholic hepatitis, the left kidney. A stone that involves the renal
the fatty and inflamed liver will be larger than medulla and extends into at least two calyces
normal and usually weighs >2 kg. However, as is considered a staghorn calculus. Approxi-
cirrhosis develops over years of continued alco- mately three quarters of all staghorn calculi are
hol use, the liver becomes fibrotic and shrinks. caused by struvite stones. Struvite stones are
It can sometimes weigh as little as 1 kg. made up of a phosphate mineral that requires
an alkaline urine to precipitate.
High-Yield Principles
Chapter 6: Pathology  •  Answers 137

Answer A is incorrect. Acidic urine would hin- Answer D is incorrect. Herpes simplex virus
der formation of staghorn calculi. in HIV-infected individuals can cause recur-
rent orolabial, genital, and perianal lesions. A
Answer C is incorrect. Presence of ammonia
Tzanck smear is positive for multinucleated
(not ammonium) in urine is a requirement for
epithelial giant cells. It does not cause the
formation of staghorn calculi, which allows
large intranuclear inclusion body shown.
for crystallization of magnesium ammonium
phosphate and carbonate apatite. Answer E is incorrect. Mycobacterium avium
causes lung disease in immunocompromised
Answer D is incorrect. Sodium chloride is not
hosts and is subsequently spread via the blood
usually a component of renal calculi and is not
to the liver, spleen, bone marrow, and other
required for their formation.
sites. Histology of mycobacterium is not con-
Answer E is incorrect. Uric acid is a com- sistent with this image. Rather, acid-fast stain-
ponent of some renal calculi but is not radi- ing would show organisms in foamy macro-
opaque and is not part of staghorn calculi. phages, granulomas, giant cells, and cells with

Pathology
eosinophilic necrosis.
15. The correct answer is C. The image shows
the typical large, round, intranuclear inclu- Answer F is incorrect. Pneumocystis jiroveci
sion with perinuclear halo that is seen in cells (formerly carinii) causes pneumonia in immu-
affected by cytomegalovirus (CMV) infec- nocompromised individuals.
tion. These structures are called “owl’s eyes”
16. The correct answer is D. Hereditary hemo-
due to their microscopic appearance. In im-
chromatosis is an iron-overload disease caused
munocompromised patients, CMV infection
by mutations in the HFE gene. Patients clas-
can present as retinitis, pneumonitis, inflam-
sically present with the triad of diabetes, cir-
mation along the GI tract, polyradiculopathy,
rhosis, and bronze skin pigmentation. This
transverse myelitis, and focal encephalitis. In
patient has many of the signs of advanced
patients with HIV/AIDS, these sequelae occur
hereditary hemochromatosis: diabetes due
most prominently when the CD4+ cell count
to pancreatic iron deposition, heart failure
is <100/mm3 or when the HIV viral load is
as a result of cardiac iron deposition, cirrho-
>10,000 copies/mm3. CMV can cause further
sis from hepatic iron deposition, and testicu-
immunosuppression, leading to other opportu-
lar atrophy as a result of dysfunction of the
nistic infections such as Pneumocystis and As-
hypothalamic-pituitary system from iron de-
pergillus pneumonia.
position. In addition, one of his siblings is also
Answer A is incorrect. Candida is a fungus affected, indicating the autosomal recessive
that produces a wide spectrum of diseases, nature of the disease. The brown granular pig-
ranging from superficial mucocutaneous dis- ment in hepatocytes seen in the image sug-
ease in immunocompetent hosts to invasive ill- gests hemosiderosis, but an iron stain (Prussian
nesses in immunocompromised hosts. Histol- blue) is also necessary because the brown pig-
ogy reveals round or ovoid yeast cells, hyphae, ment resembles lipofuscin in hematoxylin and
or pseudohyphae. eosin stains.
Answer B is incorrect. Cryptococcus neofor- Answer A is incorrect. a1-Antitrypsin de-
mans causes meningitis and meningoencepha- ficiency is due to mutations in the pro-
litis in patients with AIDS. This fungus is dif- tease inhibitor gene on chromosome 14.
ficult to observe with routine hematoxylin and a1-Antitrypsin is a protein produced in the
eosin stains, so methenamine silver or periodic liver and has a number of functions, the most
acid-Schiff stains are used to identify the char- common being protection of the lungs from
acteristic narrow-based buds and round-to-oval elastase, an enzyme produced by neutrophils.
yeast, surrounded by a polysaccharide capsule. Patients with a deficiency in this protein may
present with respiratory complaints and can
High-Yield Principles
138 Section I: General Principles  •  Answers

develop emphysema. In some patients, cirrho- anorectal cancer, as seen here. Immunodefi-
sis may occur due to accumulation of a1-anti- ciency predisposes to the development of HPV-
trypsin in the liver. Histopathologic examina- induced transformation. The mechanism of
tion would not show brown granular pigment HPV-induced transformation involves the pro-
in hepatocytes. Rather, microscopic examina- duction of a viral protein, E6, which binds to
tion of a liver biopsy would show periodic acid a cellular ubiquitin ligase E6AP. On binding
Schiff-positive, diastase-resistant globules in to E6, E6AP polyubiquitinates the tumor sup-
hepatocytes. pressor p53, leading to dysregulated cell prolif-
eration and, eventually, oncogenesis.
Answer B is incorrect. Intracytoplasmic hya-
line inclusions derived from cytokeratin inter- Answer A is incorrect. APC is a tumor sup-
mediate filaments called Mallory bodies and pressor gene mutated in certain hereditary
microvesicular steatosis are common findings forms of colon cancer. The APC protein
in alcoholic hepatitis. Accumulation of iron is normally degrades the transcription factor
not associated with this condition. b-catenin, which is involved in colonic epithe-
Pathology

lial cell proliferation. In the absence of APC,


Answer C is incorrect. Cystic fibrosis is an
increased levels of b-catenin accumulate,
autosomal recessive disease caused by muta-
eventually leading to oncogenesis.
tions in the CFTR gene on chromosome 7.
This gene encodes for a chloride ion chan- Answer B is incorrect. BRCA1 is a tumor
nel. Patients typically present early in life with suppressor gene commonly mutated in he-
meconium ileus, multiple respiratory tract in- reditary forms of breast and ovarian cancers.
fections, or failure to thrive. Patients typically The BRCA1 protein functions in DNA repair
have a number of upper and lower pulmonary processes, and inherited mutations in BRCA1
manifestations, especially recurrent respiratory interfere with DNA repair, leading to the ac-
infections. Patients also have GI symptoms cumulation of mutations and, eventually, on-
due to insufficient release of pancreatic en- cogenesis.
zymes for digestion. This causes protein and fat
Answer C is incorrect. The MSH2 gene regu-
malabsorption, leading to steatorrhea and fat-
lates a mismatch repair enzyme and is mutated
soluble vitamin deficiencies.
in certain hereditary forms of colon cancer.
Answer E is incorrect. Wilson disease is an In the absence of MSH2, increased levels of
autosomal recessive disease of copper accumu- DNA mutations accumulate, leading to even-
lation in various tissues. It is due to mutations tual cellular transformation.
in the ATP7B gene, which encodes a copper-
Answer D is incorrect. NF1 is a tumor suppres-
transporting ATPase. Patients may have a vari-
sor gene mutated in neurofibromatosis type 1.
ety of hepatic manifestations, ranging from as-
The NF1 protein functions as a GTPase activat-
ymptomatic hepatomegaly to acute fulminant
ing protein for the small G protein Ras. Because
hepatitis. They also have neurologic manifes-
Ras is only active when it is GTP-bound, NF1-
tations such as tremor, dysarthria, or gait ab-
mediated GTP hydrolysis leads to inactivation
normalities. Psychiatric manifestations, such
of Ras. In the absence of NF1, Ras is hyperac-
as mood or personality changes, are also com-
tive, leading to enhanced growth factor signal
mon, and Kayser-Fleischer rings are evident
transduction and, eventually, oncogenesis.
on ophthalmic exam in almost all patients.
This patient’s liver biopsy does not show any 18. The correct answer is A. The boy’s brain bi-
changes specific to Wilson disease. opsy demonstrates medulloblastoma. This is a
poorly differentiated neuroectodermal tumor
17. The correct answer is E. This HIV-positive pa-
that occurs predominantly in children and
tient has multiple anogenital warts, or condylo-
exclusively in the cerebellum. The cerebellar
mata acuminata, which are caused by human
vermis is the medial section of the cerebellum
papillomavirus (HPV) types 6 and 11. A feared
and is responsible for proximal muscle coordi-
complication of condylomata acuminata is
High-Yield Principles
Chapter 6: Pathology  •  Answers 139

nation, balance, and vestibulo-ocular reflexes. Answer A is incorrect. Aflatoxin B is produced


As seen in this patient, lesions of the vermis by Aspergillus and is a common contaminant
lead to vertigo, nausea, and difficulties in co- of cereals, spices, and nuts. It has been associ-
ordinating movement of trunk and proximal ated with carcinoma of the liver.
limb muscles. The presence of medulloblas-
Answer C is incorrect. Long-term exposure to
toma commonly leads to obstruction of the
high levels of benzene can lead to leukemia
outflow of CSF and the potential for hydro-
and Hodgkin lymphoma. Benzene is the main
cephalus, a life-threatening condition.
component of light oil, and is found in gaso-
Answer B is incorrect. Medulloblastomas arise line and other fuels.
in the midline of the cerebellum in children.
Answer D is incorrect. Cadmium is a carcino-
The intermediate section of the cerebellar
gen associated with the development of pros-
hemisphere is more lateral than the vermis.
tate cancer. It can be found in batteries and in
Lesions in the intermediate section would
metal coatings.
cause deficits in coordinating movements of

Pathology
the ipsilateral distal extremities, not the vertigo Answer E is incorrect. Exposure to silica may
and proximal limb problems seen in this pa- occur in the manufacturing of several mate-
tient. rials, such as glass, ceramics, and electron-
ics. Silicosis is characterized by bilateral, fine
Answer C is incorrect. Medulloblastomas
nodularity in the upper lung lobes. It is slowly
arise in the midline of the cerebellum in chil-
progressive. Its role as a carcinogen is contro-
dren. Lesions of the lateral hemisphere would
versial.
likely cause deficits in planning movements of
the ipsilateral distal extremities, not the vertigo 20. The correct answer is D. This boy most likely
and proximal limb problems seen in this pa- suffers from Eisenmenger syndrome, which is
tient. demonstrated by the cyanosis and digital club-
Answer D is incorrect. Medulloblastomas bing. In Eisenmenger syndrome a ventricular
arise in the midline of the cerebellum in chil- septal defect (VSD), atrial septal defect (ASD),
dren, not in the occipital cortex. Occipital cor- or patent ductus arteriosus causes a left-to-right
tex lesions could lead to defects in vision, not shunt and thus an acyanotic lesion, which is
the vertigo and proximal limb problems seen why he has been asymptomatic for much of his
in this patient. life. Over time the uncorrected shunt reverses
as the right ventricle hypertrophies in response
Answer E is incorrect. Medulloblastomas arise
to increased pulmonary vascular resistance,
in the midline of the cerebellum in children,
and becomes a right-to-left shunt and thus acy-
not in the parietal cortex. The postcentral gy-
anotic heart disease. The resultant low blood
rus of the parietal lobe is the primary somato-
oxygen levels can lead to the development of
sensory cortex. Lesions of this area would
exercise intolerance, cyanosis, and clubbing of
cause sensory deficits, not the vertigo and prox-
the fingers as seen in this patient.
imal limb problems seen in this patient.
Answer A is incorrect. Patients with bicuspid
19. The correct answer is B. This image demon- aortic valves often have no symptoms, but later
strates an advanced case of mesothelioma, dif- in life the valve can become calcified, leading
fusely involving the pleura and encasing lung to premature aortic stenosis in the fifth and
parenchyma. The clinical history supports this sixth decades. However, bicuspid aortic valve
diagnosis. Asbestos is present in certain build- is an unlikely cause of symptoms such as cya-
ing materials and fire-resistant materials, and nosis and clubbing in a 12-year-old child.
exposure to asbestos is a risk factor for the de-
velopment of mesothelioma.
High-Yield Principles
140 Section I: General Principles  •  Answers

Answer B is incorrect. Before the develop- Answer E is incorrect. A holosystolic murmur


ment of Eisenmenger syndrome, congenital may be heard in the setting of endocarditis,
defects such as VSD, ASD, and patent ductus and valvular vegetations would be seen on
arteriosus produced a left-to-right shunt. How- echocardiogram. However, this would not ex-
ever, this boy’s symptoms indicate that the di- plain the boy’s other symptoms.
rection of the shunt has reversed.
Answer C is incorrect. Mitral regurgitation
classically produces a holosystolic murmur that
radiates to the axilla. Although chronic mitral
regurgitation can lead to CHF, it is an unlikely
cause of symptoms such as cyanosis and club-
bing in a 12-year-old child.
Pathology
Chapter 7

Pharmacology

141
High-Yield Principles
142 Section I: General Principles  •  Questions

Q u e st i o n s

1. AIDS is currently managed with highly active tient gastric emptying study shows esophageal
anti-retroviral therapy, combining a series of dysmotility. The best treatment option for this
medications to overwhelm the virus and min- patient is which of the following?
imize its ability to form resistance to any one
(A) Esophageal resection
of the medications. Currently, protease inhibi-
(B) Metoclopramide
tors, nucleoside reverse transcriptase inhibi-
(C) Omeprazole
tors, and non-nucleoside reverse transcriptase
(D) Ondansetron
inhibitors are three types of medications that
(E) Vagotomy
can be used in combination. Which of the fol-
lowing is a non-competitive reverse transcrip- 4. Patients with hyperthyroidism have several op-
tase inhibitor? tions to treat their disease, including subtotal
Pharmacology

(A) Didanosine thyroidectomy and radioactive ablation. When


(B) Lamivudine a patient is unwilling or unable to undergo
(C) Nevirapine these procedures, pharmacological therapy is
(D) Saquinavir often pursued. One particular medication in-
(E) Zidovudine hibits the conversion of iodide to iodine and
inhibits the organification of iodine with tyro-
2. A patient is being treated with b-blockers sine. Which of the following is an additional
for hypertension. Which of the following mechanism of action of this drug?
describes the effects of b-blockers on end-
(A) Permanently reducing thyroid activity
diastolic volume (EDV), blood pressure (BP),
(B) Binding to and blocking the iodide trans-
contractility, heart rate (HR), and ejection
port mechanism of thyroid follicular cells
time?
(C) Binding to intracellular nuclear receptors
of peripheral tissue and activating genes
Choice EDV BP Contractility HR Ejection time (D) Blocking the peripheral conversion of thy-
A no effect or little/no effect little/no effect roxine to triiodothyronine
B (E) Inhibiting the binding of thyroxine to
C thyroid-binding globulin
D
E 5. A 36-year-old man who works at an explo-
sives factory comes to the clinic for an annual
Reproduced, with permission, from USMLERx.com. check-up. He is concerned about long-term
exposure to industrial chemicals. He reports
that although he is in excellent health other-
(A) A wise, he experiences headaches, dizziness, and
(B) B palpitations every Monday. Laboratory studies
(C) C show:
(D) D
(E) E WBC count: 8000/mm3
Hematocrit: 46%
3. A 68-year-old woman with type 2 diabetes mel- Hemoglobin: 15 g/dL
litus and a 30-pound weight loss over the past Platelet count: 310,000/mm3
two months presents to the physician with a Na+: 137 mEq/L
history of nausea and bloating. Symptoms are K+: 3.5 mEq/L
most prominent following a meal. An outpa- Cl-: 102 mEq/L
High-Yield Principles
Chapter 7: Pharmacology  •  Questions 143

HCO3-: 24 mEq/L 7. A 70-year-old man presents to his cardiologist


Blood urea nitrogen: 12 mg/dL with shortness of breath, crackles along both
Creatinine: 1.0 mg/dL lung bases, and 1+ pitting edema in his lower
extremities. His cardiologist diagnoses him
An ECG shows normal sinus rhythm with no
with mild congestive heart failure and places
Q wave changes. Which of the following is the
him on a thiazide diuretic. Two days later, the
most likely serious complication that can oc-
patient comes to the emergency department
cur as a result of his exposure?
obtunded and oliguric, with a highly elevated
(A) Anemia creatinine level of 8.3 mg/dL. His wife reports
(B) Atherosclerosis that the only medication that he took besides
(C) Cardiac arrest his diuretic was “some ibuprofen for his head-
(D) Congestive heart failure ache.” Which of the following is the most
(E) Dementia likely reason for this patient’s sudden renal fail-
ure?

Pharmacology
6. A 30-year-old woman presents to her physician
with a two-month history of menorrhagia. She (A) Decrease of prostaglandin E2 production
has also noticed significant fatigue over the in both arterioles of the kidney
past five weeks and some blood on her tooth- (B) Decrease of prostaglandin E2 production
brush every day during this time. Laboratory in the afferent arterioles of the kidney
tests show a WBC count of 80,000/mm3, he- (C) Decrease of prostaglandin E2 production
moglobin of 8.6 g/dL, hematocrit of 25%, and in the efferent arterioles of the kidney
platelet count of 80,000/mm3. Bone marrow (D) Increase of prostaglandin E2 production in
smear reveals the following image. Which of the afferent arterioles of the kidney
the following is the best choice for therapy? (E) Increase of prostaglandin E2 production in
the efferent arterioles of the kidney

8. A 42-year-old woman comes to a follow-up


appointment complaining of weight gain two
weeks after beginning a new medication for
her refractory schizophrenia. Laboratory stud-
ies show a low WBC count and low absolute
neutrophil count. Peripheral blood smear
shows a total absence of neutrophils and
bands. The rest of the laboratory results are
within normal limits. Which of the following
drug therapies did this patient most likely be-
gin two weeks ago?
(A) Chloramphenicol
(B) Clozapine
Reproduced, with permission, from USMLERx.com.
(C) Metronidazole
(D) Penicillin
(A) All-trans retinoic acid (E) Polymyxin B
(B) Cytarabine
(C) Daunorubicin
(D) Fludarabine
(E) Irinotecan
High-Yield Principles
144 Section I: General Principles  •  Questions

9. A 64-year-old man develops chronic renal fail- (E) Inhibition of cytochrome P450 enzymes
ure. He has an extensive medical history, and (F) Inhibition of testosterone’s negative feed-
also complains of increasingly poor vision in back on gonadotropin secretion
his right eye. After a kidney biopsy is taken (see
image), his physician immediately starts him 11. Following the discovery of a suspicious aban-
on a new medication. What pharmacologic doned package on the subway, a number of
treatment has been shown to most effectively passengers present to the emergency depart-
delay the progression of the pathology shown ment with abdominal cramps, vomiting, short-
in this photomicrograph? ness of breath, and generalized weakness.
Physical examination of these patients reveals
excessive perspiration, bilateral wheezes,
bradycardia, and miosis. Which of the follow-
ing is the most appropriate treatment for these
patients?
Pharmacology

(A) Echothiophate
(B) Hexamethonium
(C) Pancuronium
(D) Pralidoxime
(E) Pyridostigmine

12. The targets of multiple lipid-lowering agents


are labeled in the image. Which of the targets
corresponds to the therapy associated with the
most significant decrease in triglyceride levels?
Reproduced, with permission, from USMLERx.com.

Hepatocytes
(A) Angiotensin-converting enzyme inhibitors Ac-CoA
(B) b-Blockers C D
(C) Cyclophosphamide HMG-CoA
LDL IDL
+
(D) Prednisone –
Lipoprotein
(E) Thiazides Cholesterol LDL Blood
lipase

VLDL
Bile acids
10. A 74-year-old man comes to the physician VLDL

complaining of increased urinary frequency A


B
along with difficulty starting and stopping uri- Lipid
oxidation
nation. His wife states that he wakes her up – Endothelial cells
multiple times throughout the night when he
hurries to the bathroom, yet is unable to uri- Gut
nate. Diagnostic work-up reveals a benign con-
dition. Which of the following is the mecha- Reproduced, with permission, from USMLERx.com.
nism of action of a common medication used
to treat this condition?
(A) A
(A) Formation of superoxide radicals that at- (B) B
tack DNA bonds (C) C
(B) Gonadotropin-releasing hormone analog (D) D
(C) Inhibition of 5-a-reductase
(D) Inhibition of cGMP-specific phosphodies-
terase type 5
High-Yield Principles
Chapter 7: Pharmacology  •  Questions 145

13. A 67-year-old man previously diagnosed with 16. A 33-year-old immigrant from Peru comes to a
Hodgkin lymphoma complains of severe short- women’s health clinic because she has missed
ness of breath when he walks or climbs stairs. her period for the past two months. When
Examination reveals regular venous pressure of her pregnancy test comes back positive, she
18 cm H2O, bi-basilar rales, an S3 gallop, and becomes distraught, saying that she has been
2+ lower-extremity edema. His physician sus- taking oral contraceptive pills for the past
pects an adverse effect of treatment is causing year and has not missed a single dose. As she
this patient’s symptoms. The pharmacologic starts to cry, her tears are noted to have an or-
agent most likely responsible for the symptoms ange tint. The physician tells her that the most
in this patient has the same mechanism of ac- likely reason her oral contraceptives were inef-
tion as which of the following cancer drugs? fective is an interaction with one of her other
medications. What is the mechanism of action
(A) Cisplatin
of the drug that the patient is taking?
(B) Cyclophosphamide

Pharmacology
(C) Dactinomycin (A) Formation of toxic metabolites that has a
(D) Methotrexate bactericidal effect
(E) Paclitaxel (B) Blocks sodium channels, which prevents
the release of the excitatory neurotransmit-
14. A 68-year-old man with a history of stroke and ter glutamate from the presynaptic neu-
hypertension comes to the emergency depart- rons
ment because of a five-hour history of palpi- (C) Cationic and basic proteins bind to the
tations and light-headedness. He states that cell membrane and disrupt the osmotic
he has experienced shorter episodes of palpi- properties
tations before, but nothing as severe as this. (D) Blocks RNA synthesis by inhibiting DNA-
Physical examination reveals an irregular heart dependent RNA polymerase
rhythm that eventually improves with various (E) Prolongs action potential phases I and III
atrioventricular node blocking agents. The pa- via blockage of sodium and potassium
tient is prescribed a regimen of daily medica- channels
tions including a drug to prevent a potential
complication of his condition. Which of the 17. A patient presents to his primary care physi-
following drugs is most likely prescribed long- cian. On his last visit, he was diagnosed with
term to prevent such complications? hypertension and started on hydrochlorothia-
zide. What electrolyte changes would you now
(A) Aspirin
expect to see in this patient with diuretic use?
(B) Protamine sulfate
(C) Streptokinase
(D) Unfractionated heparin Urine Urine Blood
Choice
(E) Warfarin sodium potassium pH

A
15. Drug X has a half-life of 50 hours and a vol-
ume of distribution of 5 L. Which of the fol- B
lowing is the clearance of drug X? C

(A) 0.07 L/h D

(B) 0.1 L/h


Reproduced, with permission, from USMLERx.com.
(C) 70 L/h
(D) 175 L/h
(E) 250 L/h (A) A
(B) B
(C) C
(D) D
High-Yield Principles
146 Section I: General Principles  •  Questions

18. A 35-year-old African-American woman pre­


sents to her gynecologist because of lower ab-
dominal pain. She is diagnosed with an eight-
week gestational age, ectopic pregnancy and
treated medically. A few months later she re-
turns complaining of vaginal bleeding and is
diagnosed with a gestational choriocarcinoma.
Her physician chooses the same drug to treat
this cancer that he used to treat her ectopic
pregnancy. Which of the following drugs is in-
dicated in the treatment of both ectopic preg-
nancy and gestational choriocarcinoma?
Courtesy of Wikipedia.
(A) Methotrexate
Pharmacology

(B) Mifepristone
(C) Misoprostol (A) The current symptoms are only manifest
(D) Rifampin once the virus has developed a novel RNA
(E) Tamoxifen polymerase
(B) The infective virions contain viral thymi-
19. A 24-year-old law student has been experienc- dine kinase
ing frequent headaches over the course of the (C) The viral reverse transcriptase may be in-
last several months, for which he has been hibited by acyclovir
taking increasing doses of aspirin. After a long (D) The virus incorporates host cell proteases
night of studies, he takes a particularly large into its genome in order to cleave and acti-
dose of aspirin; he later becomes disoriented, vate the viral polyprotein
confused, and then experiences a seizure. He (E) Treatment with acyclovir is preventing the
is brought to the emergency department by his virus from synthesizing its own guanine
roommate, where his serum salicylate level is nucleotides
130 mg/dL. Which of the following is the most
appropriate treatment? 21. A 4-year-old girl is brought by her mother to
(A) Protamine the emergency department complaining of se-
(B) Glucagon vere abdominal pain. Her mother reports that
(C) N-acetylcysteine the girl has ingested 10-15 chocolate-coated
(D) Bicarbonate iron tablets she had found in her grandmoth-
(E) Vitamin K er’s purse, presumably thinking they were
candy. Soon after arriving the patient begins to
20. A 59-year-old man who is receiving immuno- vomit bright red blood. Physical examination
suppressive therapy develops the tender red shows hypotension and tachycardia. Which of
vesicles seen in this image. His lesions respond the following is the best initial treatment for
to acyclovir. Which of the following best de- this patient?
scribes the infection shown in this image? (A) Aminocaproic acid
(B) Deferoxamine
(C) Dimercaprol
(D) Oral bicarbonate
(E) Penicillamine
High-Yield Principles
Chapter 7: Pharmacology  •  Questions 147

22. A 43-year-old woman presents to the emer- 24. Class I antiarrhythmics are sodium channel
gency department following a motor vehicle blockers that slow or block cardiac conduction,
collision. She did not sustain any serious physi- especially in depolarized cells. Which of the
cal injuries, but she appears drowsy and states following antiarrhythmics will increase both
that she is feeling very sleepy. Emergency the action potential and the effective refractory
medical staff collected several open medica- period?
tion bottles from the floor of her automobile.
(A) Encainide
Although the contents of two of the bottles
(B) Mexiletine
could not be determined, the patient’s recently
(C) Procainamide
filled bottle of diazepam was almost empty.
(D) Propafenone
The patient quickly becomes unresponsive
(E) Tocainide
and stops breathing. Administration of which
of the following agents will reverse these symp- 25. A 68-year-old man complains of gradually pro-
toms? gressive fatigue, shortness of breath, and a 5.9-

Pharmacology
(A) Flumazenil kg (13-lb) unintentional weight gain. He de-
(B) Glucagon nies chest pain, fevers, chills, and night sweats.
(C) Midazolam Physical examination reveals 3+ bilateral pit-
(D) Naloxone ting edema in the lower extremities. X-ray of
(E) Physostigmine the chest shows cardiomegaly and pulmonary
venous congestion. Echocardiography shows
23. The image shows the dose-response curves for severe left ventricular dilatation and global hy-
the agonist drug X administered alone, and pokinesis, with an ejection fraction of 25%. He
several possible curves representing drug X is placed on a pharmacologic regimen includ-
administered with other agents. Which of the ing digoxin. At which point in the image is di-
following curves, A through E, represents the goxin a direct inhibitor?
activity of drug X when it is administered with
a noncompetitive antagonist? Na+ Ca2+
Extracellular
space

A B C
Percent of maximum effect

Cytoplasm
Drug X
Ca2+ K+ Na+

A D

B C E Ca2+ Sarcoplasmic
reticulum

D E

Ca2+
Dose

Reproduced, with permission, from USMLERx.com. Reproduced, with permission, from USMLERx.com.

(A) Curve A (A) A


(B) Curve B (B) B
(C) Curve C (C) C
(D) Curve D (D) D
(E) Curve E (E) E
High-Yield Principles
148 Section I: General Principles  •  Answers

An s w e r s

1. The correct answer is C. Nevirapine is a Answer E is incorrect. Answer E is compatible


non-nucleoside reverse transcriptase inhibi- with the effects of nitrates. These drugs serve to
tor (NNRTI). NNRTIs function by binding to decrease the afterload on the heart by vasodi-
the reverse transcriptase enzyme and acting as lation. However, vasodilation produces a reflex
a noncompetitive inhibitor. Noncompetitive increase in both contractility and heart rate.
inhibitors bind to a site on the enzyme other An increase in heart rate causes a decrease in
than the active site. EDV and ejection time.
Answer A is incorrect. Didanosine is a nu-
3. The correct answer is B. This patient pre­
cleoside reverse transcriptase inhibitor. It can
sents with gastroparesis, specifically esophageal
cause pancreatitis in some individuals.
dysmotility, secondary to her diabetes. Other
Pharmacology

Answer B is incorrect. Lamivudine is a causes of esophageal dysmotility include dia-


nucleo­side reverse transcriptase inhibitor. It betic gastroparesis, Chagas disease, lupus, and
functions by binding to the active site of the other collagen vascular diseases. Initial treat-
reverse transcriptase enzyme to inactivate it. ment consists of promotility agents, with meto-
It has very few adverse effects and can also be clopramide being first-line therapy.
used to treat hepatitis B.
Answer A is incorrect. Esophageal resection is
Answer D is incorrect. Saquinavir is a protease the treatment for squamous cell carcinoma or
inhibitor that blocks the enzyme necessary for adenocarcinoma of the esophagus or for high-
the creation of new viral proteins. grade Barrett esophagus.
Answer E is incorrect. Zidovudine, or AZT, is Answer C is incorrect. Omeprazole is a pro-
a nucleoside reverse transcriptase inhibitor. It ton pump inhibitor used to treat gastroesopha-
functions by binding to the active site of the re- geal reflux disease, peptic ulcer disease, and
verse transcriptase enzyme to inactivate it. HIV acid hypersecretion.
will quickly become resistant to AZT if not
Answer D is incorrect. Ondansetron is a
taken in conjunction with other antiretroviral
5-HT3 receptor antagonist used to treat refrac-
drugs.
tory or severe nausea and vomiting.
2. The correct answer is D. b-Blockers decrease Answer E is incorrect. Vagotomy is a treat-
contractility and heart rate (resulting in de- ment option for peptic ulcer disease or acid hy-
creased oxygen consumption) by inhibiting persecretion states such as in Zollinger-Ellison
b-1 receptors in the heart. A decrease in heart syndrome.
rate will then allow more time for diastolic fill-
ing (increasing EDV) and systolic ejection (in- 4. The correct answer is D. The drug described
creasing ejection time). In addition, b-blockers is propylthiouracil (PTU), a standard agent in
will decrease the secretion of renin, which will the treatment of hyperthyroidism. This agent
decrease aldosterone by extension and further inhibits both the conversion of iodide to el-
reduce blood pressure. emental iodine and the organification of io-
dine with tyrosine, therefore blocking the pro-
Answer A is incorrect. Answer A does not re-
duction of mono- and diiodotyrosine within
flect the effects of β-blockers.
thyroid follicular cells. In addition, PTU in-
Answer B is incorrect. Answer B does not re- hibits the peripheral conversion of thyroxine
flect the effects of β-blockers. (T4) to triiodothyronine (T3), the most active
biological form of thyroid hormone; T3 is ap-
Answer C is incorrect. Answer C does not re-
proximately 10 times more potent than T4.
flect the effects of β-blockers.
The majority of T3 is converted in the periph-
High-Yield Principles
Chapter 7: Pharmacology  •  Answers 149

eral tissue, liver, and kidneys. Two important Answer D is incorrect. Congestive heart fail-
adverse effects of PTU are exfoliative skin rash ure (CHF) can be a complication of long-term
(most common) and agranulocytosis (most se- coronary artery disease or a myocardial infarc-
vere, but fortunately quite rare). tion (MI), but not of nitroglycerin exposure.
Answer A is incorrect. Propylthiouracil has no Answer E is incorrect. Dementia can be a re-
effect on the binding of T3 to the thyroid hor- sult of neurotoxin exposures (eg, mercury). Ni-
mone receptor. troglycerin exposure does not cause dementia.
Answer B is incorrect. Perchlorate and thio-
6. The correct answer is A. This patient is pre-
cyanate are two ionic inhibitors that block
senting with acute promyelocytic leukemia
the thyroid’s ability to sequester iodide. These
(APL), the cancer most frequently associated
drugs are rarely used today but have been
with disseminated intravascular coagulation
used to treat Graves disease and amiodarone-­
and bleeding diatheses. Typically these pa-
induced thyrotoxicosis.
tients present with anemia, fatigue, and bleed-

Pharmacology
Answer C is incorrect. This describes the ing. The age of this patient points to acute
mechanism of levothyroxine, the drug most myelogenous leukemia (AML), and the image
commonly used in to treat hypothyroidism. It confirms that this patient has the promyelo-
is an analog of endogenous T4. Also used in cytic type by showing abundant promyelocytes
neonates with thyroid deficiency, this medica- with azurophilic granules. APL can often be
tion can prevent mental retardation if adminis- treated by differentiating the malignant cells
tered within two weeks of delivery. into mature neutrophils, which have a life
span of approximately seven days. The cells
Answer E is incorrect. This describes radioac-
harbor a t(15;17) translocation, making the ret-
tive iodine, which is used patients >21 years
inoic acid receptor responsible for their trans-
old who have hyperthyroidism. It is the pre-
formation, and the cells can be differentiated
ferred drug to treat Graves disease that it is re-
using high-dose all-trans retinoic acid.
fractory to antithyroid drugs or when surgery
fails. Answer B is incorrect. Cytarabine is a nucle-
otide analog that is incorporated into DNA,
5. The correct answer is C. This patient’s symp- subsequently inhibiting DNA polymerase and
toms suggest that he is experiencing nitro- RNA polymerase. It is used to treat AML, but
glycerin withdrawal. Nitroglycerine is a vaso- is not as specific or effective in APL therapy.
dilator. Chronic industrial exposure leads to
Answer C is incorrect. Daunorubicin is an
tolerance for vasodilatation on work days and
anthracycline that inhibits topoisomerase II
a compensatory vasoconstriction on weekends
and intercalates into DNA, preventing relega-
when the exposure is removed, thus resulting
tion during mitosis and, in turn, introducing
in the “Monday disease.” The most severe con-
double­-strand breaks. It is used in acute lym-
sequence is when the compensatory vasocon-
phocytic leukemia and AML, but is not as spe-
striction is unopposed in critical areas such as
cific or helpful in treating APL.
the coronary vessels, leading ischemia due to
decreased blood flow. Answer D is incorrect. Fludarabine is an ATP
analog that inhibits DNA polymerase, DNA
Answer A is incorrect. Anemia can be a result
primase, and RNA reductase, blocking DNA
of lead or other heavy metal poisoning, but not
synthesis. It is often used to treat chronic lym-
nitroglycerin.
phocytic leukemia. It can also be used to treat
Answer B is incorrect. Atherosclerosis is AML, but is not specific to APL.
formed by fatty streak deposition, which along
Answer E is incorrect. Irinotecan is a topo­
with endothelial damage results in plaque for-
isomerase I inhibitor, preventing ligation of
mation over time. It is not linked to nitroglyc-
erin exposure.
High-Yield Principles
150 Section I: General Principles  •  Answers

single-strand breaks when it forms a complex shown to reduce the progression of diabetic ne-
with DNA and this enzyme. As the cell at- phropathy.
tempts to replicate its DNA, much damage is
done when DNA polymerase encounters the 8. The correct answer is B. Clozapine is an
complex, leading to cell death. Again, this atypical antipsychotic used primarily to treat
drug is not specific to APL. schizophrenia. The patient here is exhibiting
agranulocytosis, which is the most serious risk
7. The correct answer is A. The photomicro- associated with clozapine use. Consequently,
graph shows Kimmelstiel-Wilson nodules, clozapine is reserved for schizophrenia that is
which are pathognomonic for diabetic ne- otherwise refractory to treatment. Patients must
phropathy. Even without recognizing this spe- be regularly monitored for neutropenia for the
cific histopathology, however, one should be first six months of treatment. Other adverse ef-
reminded of diabetes due to the combination fects include weight gain, hypotension, mild
of renal and visual findings (diabetic nephrop- sedation, and, in some cases, extrapyramidal
Pharmacology

athy and retinopathy). Angiotensin-converting effects.


enzyme (ACE) inhibitors are the drugs of
Answer A is incorrect. Chloramphenicol is as-
choice in the control of diabetes-induced re-
sociated with aplastic anemia, which is charac-
nal disease because they reduce systemic blood
terized by neutropenia, thrombocytopenia, and
pressure, reduce the effects of angiotensin II
decreased RBCs. It would not be used to treat
(AT II) on efferent arterioles, and attenuate the
schizophrenia.
stimulatory effect of AT II on glomerular cell
growth and matrix production. ACE inhibi- Answer C is incorrect. Metronidazole is as-
tors have been conclusively shown to delay the sociated with disulfiramlike reactions and not
time to end-stage renal disease by 50% in type agranulocytosis.
1 diabetics and to significantly delay progres- Answer D is incorrect. Penicillin is associated
sion of renal disease in type 2 diabetics. All with gastrointestinal (GI) distress, urticaria,
diabetics should begin ACE inhibitor therapy and anaphylaxis, not agranulocytosis.
at the onset of microalbuminuria, even in
the absence of hypertension. In addition, one Answer E is incorrect. Polymyxins are associ-
should consider starting patients with diabetes ated with neurotoxicity and renal tubular aci-
on statins and low-dose aspirin as they are at a dosis, not agranulocytosis.
higher risk of coronary artery disease.
9. The correct answer is B. Renal failure is a very
Answer B is incorrect. b-Blockers are used to dangerous adverse event associated with non-
control essential hypertension, not diabetic ne- steroidal anti-inflammatory drugs (NSAIDs).
phropathy. These drugs have been shown to The patient was in CHF when he first pre-
reduce mortality in patients with CHF. sented. His cardiologist consequently treated
Answer C is incorrect. Cyclophosphamide is him with a diuretic, intending to reduce his to-
often used in conjunction with prednisone to tal body fluids. When the amount of fluids in
treat immunologically mediated kidney dis- the body contracts, the body attempts to com-
ease. pensate by releasing angiotensin II, a potent
vasoconstrictor. In order to protect the kidney
Answer D is incorrect. Prednisone is used to from losing its perfusion due to this vasocon-
treat immune-mediated nephropathy, not dia- striction, the kidney simultaneously releases
betic nephropathy. prostaglandins at both the afferent and efferent
Answer E is incorrect. Thiazides are often arterioles, where they act as vasodilators. By
used in the initial treatment of hypertension, taking an NSAID like ibuprofen and inhibit-
which could ultimately lead to renal fail- ing the cyclooxygenase (COX)-1 and COX-2
ure. However, they have not specifically been enzymes, this patient blocked the pathway pro-
ducing the prostaglandins that were keeping
High-Yield Principles
Chapter 7: Pharmacology  •  Answers 151

the afferent arterioles dilated and thus keeping tion leads to a reduction in the size of the pros-
his kidneys perfused. His renal failure is pre­ tate, providing symptomatic relief.
renal in origin, resulting from the constriction
Answer A is incorrect. Bleomycin acts by che-
of these arterioles.
lating mechanisms to attack the phosphodies-
Answer A is incorrect. NSAIDs will block the ter bonds of DNA. It is used to treat testicular
production of prostaglandin E2 at both arteri- tumors and lymphomas (especially Hodgkin),
oles, but the constriction of the afferent arte- not benign prostatic hypertrophy.
riole is the primary cause of this man’s renal
Answer B is incorrect. Leuprolide is a
failure.
gonadotropin-releasing hormone analog that
Answer C is incorrect. While NSAIDs would binds the luteinizing hormone-releasing hor-
also have blocked the production of prosta- mone receptor in the pituitary. This leads to
glandin E2 at the efferent arteriole, this would desensitization of the receptor and, subse-
not cause renal failure, it would actually in- quently, to reduced release of luteinizing hor-

Pharmacology
crease glomerular filtration rate (blocking the mone (LH). Leuprolide is used to treat meta-
outflow without blocking the inflow will in- static carcinoma of the prostate, not BPH, and
crease filtration). This would not cause the pa- also plays a role in in vitro fertilization.
tient to present with oliguria or a rising creati-
Answer D is incorrect. Sildenafil inhibits
nine level.
cGMP-specific phosphodiesterase type 5, re-
Answer D is incorrect. Ibuprofen blocks the sulting in increased concentrations of cGMP,
synthesis of prostaglandins, and thus a decrease which increases vasodilation leading to in-
in the prostaglandin level would be seen, not creased blood flow to the corpus cavernosum.
an increase. Sildenafil is used primarily to treat erectile dys-
function.
Answer E is incorrect. Ibuprofen blocks the
synthesis of prostaglandins, and thus a decrease Answer E is incorrect. Ketoconazole is an an-
in the prostaglandin level would be seen, not tifungal with antiandrogenic properties that
an increase. acts by inhibiting cytochrome P450 enzymes
(CYP450). It is not used to treat BPH, but is
10. The correct answer is C. This man presents used commonly to treat fungal infection such
with classic symptoms of benign prostatic hy- as athlete’s foot and ringworm.
pertrophy (BPH), which include difficulty
Answer F is incorrect. Flutamide is a potent
starting and maintaining a urine stream, feel-
androgen receptor antagonist that has lim-
ing as though the bladder is never emptied,
ited efficacy when used alone because the in-
having the urge to urinate again soon after
creased LH secretion stimulates higher serum
voiding, and pain on urination or dysuria. Fi-
testosterone levels. Thus the increase in serum
nasteride is most commonly used to treat this
testosterone can overcome the androgen re-
condition. Finasteride acts by inhibiting the
ceptor antagonism. This drug is used primarily
conversion of testosterone to dihydrotestoster-
in conjunction with a gonadotropin-releasing
one (DHT) by inhibiting 5a-reductase. Nor-
hormone analog in the treatment of metastatic
mally DHT binds to the nuclear androgen
prostate cancer.
receptors and stimulating mitogenic growth
factors that cause stromal and epithelial hyper-
11. The correct answer is D. This scenario is com-
plasia along with promoting secondary sexual
monly observed in cases of organophosphate
characteristics (in men and women). DHT,
poisoning. Organophosphates lead to phos-
not testosterone, is the culprit behind prostatic
phorylation and deactivation of acetylcholin-
hyperplasia because of DHT’s slow dissociation
esterase, causing an increase in acetylcholine
from the prostatic nuclear androgen receptor.
levels and their associated cholinergic effects
Inhibiting 5a-reductase and thus DHT forma-
and subsequent symptoms described in the
High-Yield Principles
152 Section I: General Principles  •  Answers

vignette. Pralidoxime dephosphorylates the Answer C is incorrect. 3-Hydroxy-3-methyl­


acetylcholinesterase and reactivates it, primar- glutaryl coenzyme A (HMG CoA) reductase
ily in the neuromuscular junction. It thus re- inhibitors act at point C by competitively in-
verses the toxic process triggered by organo- hibiting the synthesis of mevalonate by HMG
phosphates. Atropine, a muscarinic receptor CoA reductase, an essential step in the produc-
antagonist, is commonly added for sympto­ tion of cholesterol in the liver. Serum LDL
matic relief of salivation, cramping, sweating, cholesterol levels are, in turn, decreased as the
and wheezing. liver upregulates LDL receptor expression to
compensate for diminished capacity to endog-
Answer A is incorrect. Echothiophate is an
enously synthesize cholesterol.
acetylcholinesterase inhibitor; it would further
increase acetylcholine levels in the neuromus-
13. The correct answer is C. This patient has
cular junction and worsen the symptoms.
dysp­nea on exertion but normal pulmonary
Answer B is incorrect. Hexamethonium is a function tests, and most likely has dilated
Pharmacology

nicotinic receptor antagonist and would be in- cardiomyopathy due to the toxic effects of
appropriate, as it does not interact with musca- doxorubicin, a DNA intercalating agent that
rinic receptors. is used as a part of the ABVD (Adriamycin
[doxorubicin], Bleomycin, Vinblastine, and
Answer C is incorrect. Pancuronium is a long-
Dacarbazine) treatment regimen for Hodgkin
lasting nicotinic receptor antagonist. It does
lymphoma. Doxorubicin also is used to treat
not act at the muscarinic receptors responsible
myelomas, sarcomas, and some solid-tissue
for the patient’s sweating, salivation, cramping,
tumors (breast, lung, and ovary). Dactinomy-
and wheezing.
cin has a mechanism of action similar to that
Answer E is incorrect. Pyridostigmine is an of doxorubicin, as it also acts via DNA inter-
acetylcholinesterase inhibitor; it would further calation. Dactinomycin is used to treat Wilm
increase acetylcholine levels in the neuromus- tumor, germ cell tumors, rhabdomyosarcoma,
cular junction and worsen the symptoms. and various other sarcomas.

12. The correct answer is D. Fibrates like gem­ Answer A is incorrect. Cisplatin is an
fibrozil act at point D. They are ligands for the alkylating-like agent that cross-links DNA,
peroxisome proliferator-activated receptor-a thus causing apoptosis. It is used to treat tes-
(PPAR-a) protein, a receptor that regulates the ticular, bladder, ovarian, and lung carcinomas.
transcription of genes involved in lipid me- Nephrotoxicity and acoustic nerve damage are
tabolism. Increased expression of the PPAR-a prominent adverse effects of cisplatin treat-
protein results in increased expression of lipo- ment.
protein lipase on endothelial cells and thus Answer B is incorrect. Cyclophosphamide is
increased clearance of triglyceride-rich lipo- an alkylating agent that acts by attaching alkyl
proteins. Fibrates have been shown to decrease groups to DNA bases, ultimately causing DNA
triglyceride levels by as much as 35%-50%. damage. It is used to treat non-Hodgkin lym-
Answer A is incorrect. There are no lipid- phomas, and breast and ovarian carcinomas.
lowering drugs that act at point A. Hemorrhagic cystitis is an adverse effect of cy-
clophosphamide treatment.
Answer B is incorrect. Bile acid sequestrants
such as cholestyramine act at point B by block- Answer D is incorrect. Methotrexate inhib-
ing the reabsorption of bile acids. The liver its the metabolism of folic acid by inhibiting
must then metabolize more cholesterol to re- dihydrofolate reductase, thereby preventing
place the bile acids, thereby primarily lowering the conversion of dihydrofolate to tetrahydro-
LDL-cholesterol levels. folate. Because tetrahydrofolate is essential
for thymidine synthesis, methotrexate has a
toxic effect on replicating cells. Methotrexate
High-Yield Principles
Chapter 7: Pharmacology  •  Answers 153

is used to treat many cancers including cho- 16. The correct answer is D. Rifampin suppresses
riocarcinoma, leukemia in the spinal fluid, RNA synthesis by inhibiting DNA-dependent
osteosarcoma, breast cancer, lung cancer, non- RNA polymerase and is an antibiotic used to
Hodgkin lymphoma, and head and neck can- treat tuberculosis. One major adverse effect of
cers. rifampin is that it is metabolized by and also
induces the CYP450 isoenzyme system; thus
Answer E is incorrect. Paclitaxel inhibits mi-
drugs such as oral contraceptives, warfarin,
crotubule disassembly by binding to the b sub-
and ketoconazole may need to be given in
unit of tubulin, which ultimately disrupts cel-
higher doses in order to be therapeutic. This
lular function and leads to apoptosis. Paclitaxel
is probably the reason why this woman’s oral
is used to treat ovarian, breast, and lung cancer
contraceptive pills failed. Another well-known
in addition to Kaposi sarcoma.
adverse effect that can be frightening to pa-
14. The correct answer is E. This patient has tients is that rifampin turns all bodily fluids
chronic atrial fibrillation, which is a risk factor (tears, sweat, and urine) orange. Other uses of

Pharmacology
for clot formation and systemic embolization. rifampin include treatment of leprosy, for me-
Given his age, history of hypertension, and ningococcal prophylaxis, and for Haemophilus
previous stroke, he needs ongoing anticoagula- influenzae type b chemoprophylaxis.
tion to prevent possible complications, such as Answer A is incorrect. Metronidazole forms
cerebrovascular accidents or mesenteric infarc- toxic metabolites that have a bactericidal ef-
tion. Warfarin inhibits gamma-carboxylation fect. It is an effective antibiotic against amoe-
of vitamin K-dependent clotting factors and bae and anaerobes. It is also used in Giardia
is used for chronic anticoagulation. It is taken infection. One of metronidazole’s best known
orally and has a long half-life. The degree of adverse effects is a disulfiram-like reaction
anticoagulation must be followed by measur- when taken with ethanol. Metronidazole is
ing the International Normalized Ratio (INR). also highly teratogenic and should not be
Answer A is incorrect. Aspirin is an antiplate- taken by pregnant women.
let agent used to prevent MI. It also has anti- Answer B is incorrect. Phenytoin acts by
pyretic, analgesic, and anti-inflammatory ef- blocking sodium channels, which inhibits
fects. glutamate release from excitatory presynaptic
Answer B is incorrect. Protamine sulfate is neurons. It is used to treat epilepsy, particularly
used for rapid reversal of heparinization in the tonic-clonic and partial seizures. It has many
setting of overzealous anticoagulation. adverse effects, including induction of the
CYP450 isoenzyme system, and thus would
Answer C is incorrect. Streptokinase is used to interact with oral contraceptive pills. However,
break down existing clots. phenytoin does not cause red-orange bodily
Answer D is incorrect. Heparin is used for fluids. Some other adverse effects of pheny­
acute, not long-term, anticoagulation. toin include gingival hyperplasia, megaloblas-
tic anemia secondary to folate deficiency, and
15. The correct answer is A. The formula for half- central nervous system depression. Phenytoin
life (t1/2) is: (0.7 × volume of distribution) / is also teratogenic and causes fetal hydantoin
(clearance). Rearranging the formula yields: syndrome (prenatal growth deficiency, mental
Clearance = (0.7 × volume of distribution) / retardation, and congenital malformations).
(t1/2) = (0.7 × 5 L) / (50 h) = 0.07 L/h. Answer C is incorrect. Polymyxins B and E
Answer B is incorrect. Calculation error. are cationic basic proteins that act as deter-
gents that bind to cell membranes and disrupt
Answer C is incorrect. Calculation error. the osmotic and cell membrane integrity of
Answer D is incorrect. Calculation error. the bacteria. They are used in resistant gram-
Answer E is incorrect. Calculation error.
High-Yield Principles
154 Section I: General Principles  •  Answers

negative infections. Adverse effects include agent. It is used to terminate an ectopic preg-
neurotoxicity and acute renal tubular necrosis. nancy, or in combination with misoprostol to
terminate an intrauterine first-trimester preg-
Answer E is incorrect. Amiodarone prolongs
nancy. It also can be used alone or in combi-
the action potential in cardiac phases I and III
nation with other agents to treat malignant
via blockage of sodium and potassium chan-
gestational trophoblastic disease (GTD) such
nels. It is an antiarrhythmic drug that has prop-
as choriocarcinoma. Most patients with chorio-
erties of both class I and class III antiarrhyth-
carcinoma have metastatic disease at the time
mics, and is most often used to treat refractory
of diagnosis, with spread to the lungs, liver,
atrial fibrillation and ventricular tachyarrhyth-
vagina, or central nervous system. Of note,
mias. Amiodarone is infamous for its many ad-
malignant GTD after an ectopic pregnancy is
verse effects, including interstitial pulmonary
rare; it is much more likely to develop after a
fibrosis, thyroid dysfunction (both hyper- and
molar pregnancy.
hypothyroidism), and hepatocellular necrosis.
Pharmacology

Amiodarone and other antiarrhythmic drugs Answer B is incorrect. Mifepristone is a partial


do not cause orange bodily fluids, though it progesterone agonist that can be used alone
does turn the skin bluish with chronic use. or in combination with misoprostol to induce
a first-trimester medical abortion. It has been
17. The correct answer is A. Hydrochlorothiazide studied as an agent to treat ectopic pregnancy,
decreases blood, not urine, sodium by block- but it is not used to treat GTD.
ing reuptake of NaCl in the distal convoluted
Answer C is incorrect. Misoprostol is a syn-
tubule.
thetic analog of prostaglandin1. It promotes
Answer B is incorrect. Hydrochlorothiazide uterine contractions and so is used in com-
causes decreased blood potassium and in- bination with mifepristone to induce a first-
creased urine potassium through loss of potas- trimester medical abortion. It is not used to
sium in the distal nephron. treat either ectopic pregnancy or GTD.
Answer C is incorrect. Hydrochlorothiazide Answer D is incorrect. Rifampin inhibits
will cause a metabolic alkalosis, not acidosis, DNA-dependent RNA polymerase. It is used
through volume contraction, loss of hydrogen to treat Mycobacterium tuberculosis infection,
ion into cells in exchange for potassium, and to delay resistance to dapsone in patients with
loss of hydrogen ion in the urine. leprosy, and for chemoprophylaxis in patients
Answer D is incorrect. Patients taking exposed to Neisseria meningitides and unim-
any type of diuretic, with the exception of munized children exposed to Haemophilus in-
potassium-sparing diuretics such as spirono- fluenzae type B. It is not used to treat ectopic
lactone, will have increased loss of potassium pregnancy or GTD.
into the urine. All diuretics also increase urine Answer E is incorrect. Tamoxifen is a partial
sodium, as they inhibit reuptake of sodium at estrogen agonist that is used to treat estrogen-
various points along the nephron. Thiazide dependent cancers of the breast. It is not indi-
diuretics inhibit sodium reuptake in the distal cated in the treatment of ectopic pregnancy or
tubule. Finally, thiazide diuretics cause a met- GTD.
abolic alkalosis through several mechanisms:
volume contraction, low potassium leading 19. The correct answer is D. Administration of
to loss of hydrogen ion in principle cells, and bicarbonate alkalinizes the urine, thereby pro-
potassium exiting cells into the blood in ex- moting the excretion of acidic drugs such as as-
change for hydrogen ion going into cells. pirin and not allowing them to be reabsorbed.
Highly basic urine deprotonates acids such as
18. The correct answer is A. Methotrexate, a folic salicylates within the renal tubules, resulting
acid analog, can be used as an immunosup- in an ionic charge. In general, charged mol-
pressant, an abortifacient, or an antineoplastic ecules are cleared, while uncharged molecules
High-Yield Principles
Chapter 7: Pharmacology  •  Answers 155

are easily reabsorbed from the tubules. Thus 21. The correct answer is B. Symptoms of iron
bicarbonate administration is indicated be- overdose include onset of nausea, vomiting
cause it promotes “trapping” of salicylates in (including hemorrhagic gastroenteritis), and
the urine and their subsequent excretion. abdominal pain within four hours of inges-
tion. Other symptoms may include hypergly-
Answer A is incorrect. Protamine is used to
cemia, leukocytosis, shock, and coma. De­
treat heparin toxicity.
feroxamine is an iron-specific chelating agent
Answer B is incorrect. Glucagon is used to that binds to both ferric and ferrous ions to
treat b-blocker toxicity. form ferrioxamine. Deferoxamine can capture
Answer C is incorrect. N-acetylcysteine is iron from ferritin and hemosiderin outside the
used to treat acetaminophen toxicity. bone marrow, but leaves iron in hemoglobin,
cytochromes, and myoglobin untouched. Al-
Answer E is incorrect. Vitamin K is used to though deferoxamine can effectively and rap-
treat warfarin toxicity. idly reduce effective blood iron levels, adverse

Pharmacology
effects include hypotensive shock secondary to
20. The correct answer is B. Herpes zoster is char- histamine release, allergic reaction, and rare
acterized by reactivation of a latent varicella- cases of neural and renal toxicity.
zoster infection. Original infection is charac-
terized by chickenpox, after which the virus Answer A is incorrect. Aminocaproic acid is
lives in the ganglia of spinal nerve roots. When used in the treatment of tissue plasminogen ac-
the infection is reactivated, as often happens tivator or streptokinase overdose.
in immunosuppressed patients, eruptive vesi- Answer C is incorrect. Dimercaprol is used to
cles develop in a dermatomal pattern and do treat poisoning by arsenic, mercury, and gold.
not cross the midline. Acyclovir is the primary
treatment of zoster infection. Acyclovir is an Answer D is incorrect. Bicarbonate can be
inactive precursor that is activated by viral thy- used to inactivate iron in the GI tract. Al-
midine kinase when absorbed by infected cells. though this treatment may prevent iron from
When activated, it forms a guanine analog that being absorbed in the GI tract, it would do
results in chain termination when read by viral nothing to remove excess iron already in the
DNA polymerase. bloodstream.

Answer A is incorrect. Reactivation of the pa- Answer E is incorrect. Penicillamine is used to


tient’s latent varicella zoster virus infection is treat lead and arsenic toxicity, not iron poison-
most likely secondary to his immunosuppres- ing.
sion. While herpesviruses can develop resis-
22. The correct answer is A. The patient has over-
tance to acyclovir by modifications to the viral
dosed on diazepam, a benzodiazepine. Fluma-
DNA polymerase, mutations in the viral RNA
zenil is a competitive antagonist at the ben-
polymerase do not lead to an increased inci-
zodiazepam GABAergic receptor. Its use can
dence of reactivation.
prevent or reverse the central nervous system
Answer C is incorrect. Reverse transcriptase is effects of benzodiazepine overdose.
not present in varicella-zoster virus.
Answer B is incorrect. Glucagon can be used
Answer D is incorrect. Herpes zoster virus en- to counter b-blocker overdose. It would not be
codes its own proteases. Host cell proteases are used to treat diazepam overdose.
not incorporated into the zoster DNA, nor do
Answer C is incorrect. Midazolam is a shorter-
they cleave the viral polyprotein.
acting benzodiazepine and would exacerbate
Answer E is incorrect. The action of acyclovir this patient’s symptoms.
does not affect the production of the virion’s
native nucleotides. Chain termination occurs
as long as the virus contains thymidine kinase.
High-Yield Principles
156 Section I: General Principles  •  Answers

Answer D is incorrect. Naloxone is used to Answer B is incorrect. Curve B represents


reverse the effects of opiates such as heroin or a drug with a lower efficacy but a greater po-
morphine. tency than drug X. This could occur with a
type of drug known as a partial agonist, which
Answer E is incorrect. Physostigmine is used
binds to the receptor as well as (or better than)
to reverse the effects of anticholinergic agents.
the original agonist, but when bound, exerts
23. The correct answer is C. Graphs of this na- only a fraction of the effect that could be pro-
ture display two important characteristics about duced by the original agonist.
a drug in a given reaction—its potency and Answer D is incorrect. Curve D represents a
its efficacy. Potency is used to describe the drug with a decreased potency but the same
amount of drug needed to produce a given ef- efficacy. This could occur with the addition of
fect. A drug with high potency will be given a competitive antagonist. Remember that this
at a lower dose to produce the same effect as antagonist would bind to the same receptor
Pharmacology

a drug with low potency. On the graph, more site as the agonist and take up its binding spots,
potent drugs are located toward the left (indi- effectively making it appear as though there is
cating lower doses). Efficacy is a property in- less of the agonist around to bind the receptor.
trinsic to a drug that reflects how well it gen-
Answer E is incorrect. Curve E represents a
erates a response on binding to its receptor. A
drug with lower efficacy and potency. This
drug that is highly efficacious will better gen-
could occur with the addition of a partial an-
erate a response than the comparable quan-
tagonist, which operates with logic similar to
tity of a less efficacious drug. On the graph,
that of a partial agonist.
more efficacious drugs are located toward the
top (indicating higher maximal effects). Using
24. The correct answer is C. Class IA antiar-
this framework, we can now understand the
rhythmics, such as procainamide, affect both
definition of a noncompetitive antagonist. An-
atrial and ventricular arrhythmias. They block
tagonist indicates that it will decrease the effi-
sodium channels and thus slow conduction
cacy of the drug in question. The fact that it is
velocity in the atria, ventricles, and Purkinje
noncompetitive means that it cannot be over-
fibers. This decreased conduction veloc-
come (or “out-competed”) simply by adding
ity slows phase 0 of the action potential (AP)
more of the agonist drug. Molecularly, a com-
and is manifested as an increased QRS dura-
petitive antagonist is one that resembles the
tion on ECG. In addition to blocking sodium
agonist and thus can fill up receptor sites that
channels, class IA antiarrhythmics also block
the agonist would normally bind. A noncom-
potassium channels and thus increase the AP
petitive antagonist does not bind to the same
duration by prolonging the effective refractory
receptor as the agonist. Instead, it binds to the
period (ERP).
receptor to change its conformation, so that
when the agonist binds, it is less able to do its Answer A is incorrect. Encainide is a class IC
job (decreased efficacy). This phenomenon is antiarrhythmic.
displayed by curve C in the graph, which will Answer B is incorrect. Mexiletine is a class IB
never reach the same height (efficacy) as the antiarrhythmic.
agonist administered alone.
Answer D is incorrect. Class IC antiarrhyth-
Answer A is incorrect. Curve A represents a mics, such as propafenone, slow phase 0 of
dose-response curve with the same efficacy but the AP, but have no effect on the AP duration.
increased potency. In order to be a noncom- These antiarrhythmics have no affect on the
petitive antagonist, by definition, it must lower ERP.
the drug’s efficacy.
High-Yield Principles
Chapter 7: Pharmacology  •  Answers 157

Answer E is incorrect. Class IB antiarrhyth- Answer A is incorrect. A is the voltage-gated


mics, such as tocainide, slow phase 0 of the calcium channel, which is not inhibited by di-
AP, but decrease the AP duration. These an- goxin.
tiarrhythmics affect ischemic or depolarized
Answer C is incorrect. C is the Na+/Ca2+ ex-
Purkinje and ventricular tissues. These antiar-
changer, which is not the direct site of action
rhythmics have no affect on the ERP.
for digoxin.
25. The correct answer is B. Digoxin is a direct Answer D is incorrect. D is the ryanodine cal-
inhibitor of Na+/K+-ATPase (point B on the cium channel, which releases calcium from
image). The increased levels of intracellular the sarcoplasmic reticulum. It is not the site of
Na+ indirectly inhibit the function of the Na+/ action for digoxin.
Ca2+ exchanger (point C on the image), result-
Answer E is incorrect. E is the ATP-
ing in increased intracellular Ca2+ concentra-
dependent calcium pump, which pumps cal-
tion. This in turn results in the inotropic prop-
cium back into the sarcoplasmic reticulum.

Pharmacology
erties of digoxin.
This page intentionally left blank
S e c t i o n II

Organ Systems

 Cardiovascular

 Endocrine

 Gastrointestinal

 Hematology-Oncology

 Musculoskeletal

 Neurology

 Psychiatry

 Renal

 Reproductive

 Respiratory

159
This page intentionally left blank
Chapter 8

Cardiovascular

161
162 Section II: Organ Systems  •  Questions

Q u e st i o n s

1. A 36-year-old man presents with sudden-onset 3.


3-Hydroxy-3-methylglutaryl coenzyme A
dizziness and chest palpitations. He had been (HMG CoA) reductase inhibitors, commonly
healthy previously. An ECG is shown in the known as the statins, are the most effective
image. Laboratory work-up reveals normal lev- drugs available for reducing LDL cholesterol
els of RBCs and WBCs and a normal cardiac levels. Statins cause several downstream effects
panel. The drug commonly used to treat this by inhibiting the rate-limiting step in hepatic
condition inhibits which of the following? cholesterol synthesis. What is the primary ef-
fect of statins on hepatocytes?
(A) Decreased hepatic production of LDL
High-Yield Systems

cholesterol
(B) Decreased production of triglycerides
(C) Downregulation of cell surface LDL cho-
lesterol receptors
(D) Increased production of HDL cholesterol
(E) Upregulation of cell surface LDL choles-
terol receptors

4. A 45-year-old man who takes spironolactone


Courtesy of Dr. James Heilman. and digoxin for his congestive heart failure is
admitted to the hospital because he is experi-
encing an altered mental status. The ECG
(A) b-Adrenergic receptor changes shown in the image are noted on test-
(B) Acetylcholine receptor ing. Urinalysis would most likely reveal which
(C) Calcium channel of the following?
(D) Potassium channel
(E) Sodium-potassium ATPase enzyme
Cardiovascular

2. A 64-year-old woman with a history of diabe-


tes, hypertension, and congestive heart failure
was brought to the emergency department af-
ter she complained of a headache and blurred
vision and was found to have a blood pressure
of 220/95 mm Hg. The intern who saw her
wanted to treat her with drug X, but the attend- Reproduced, with permission, from USMLERx.com.
ing physician rejected this choice because of
its tendency to cause compensatory tachycar-
dia and exacerbate fluid retention, as well as its (A) High K+, high Na+, high-normal urine vol-
potential to cause a lupus-like syndrome with ume
long-term use. What is the mechanism of ac- (B) High K+, low Na+, low urine volume
tion of drug X? (C) High K+, low Na+, normal urine volume
(D) Low K+, high Na+, high-normal urine vol-
(A) Block calcium channels
ume
(B) Decreased production of cGMP
(E) Low K+, low Na+, normal urine volume
(C) Increased production of cGMP
(D) Inhibit angiotensin-converting enzyme
(E) Inhibit carbonic anhydrase
(F) Inhibit sodium chloride reabsorption
Chapter 8: Cardiovascular  •  Questions 163

5. A 2-year-old boy is brought to the clinic by vere. After a thorough work-up he is diagnosed
his parents because he suffers from sudden with an acute myocardial infarction (MI).
cyanotic attacks that can be improved only by Which of the following laboratory test eleva-
squatting. He is referred to a cardiologist, who tions is most specific for MI?
informs the parents that their son’s right ventri-
(A) Alanine aminotransferase
cle is abnormally large. Which of the following
(B) Aspartate aminotransferase
is most likely the root cause of this boy’s heart
(C) Creatine kinase-MB fraction
defect?
(D) Lactate dehydrogenase
(A) Anterosuperior displacement of the infun- (E) Troponin I
dibular septum
(B) Overriding aorta 8. Following the administration of drug X, there
(C) Patent ductus arteriosus is an increase in systolic, diastolic, and mean

High-Yield Systems
(D) Pulmonary stenosis arterial pressures. After the effect of drug
(E) Ventricular septal defect X has worn off completely, drug Y is then
added, resulting in little or no change to the
6. A 55-year-old man with hypertension is pre- baseline blood pressure. When drug X is re-
scribed an antiarrhythmic agent that alters the administered, there is a net decrease in blood
flow of cations in myocardial tissue. The im- pressure (see image). Which of the following
age is a trace of a myocardial action potential. drug combinations represents drug X and drug
Each phase is associated with the opening and/ Y, respectively?
or closing of various ion channels. Which of
the following would be affected by an agent
that affects phase 0 of the myocardial action X Y X
potential?

Ventricular muscle fiber

1
2 Reproduced, with permission, from USMLERx.com.

Cardiovascular
30 ms
(A) Epinephrine, phentolamine
0
3
(B) Isoproterenol, clonidine
(C) Norepinephrine, propranolol
(D) Phenylephrine, metoprolol
(E) Phenylephrine, phentolamine

Reproduced, with permission, from USMLERx.com. 9. A 56-year-old woman arrives in the emer-
gency department complaining of dizziness
and headache. Her blood pressure is 210/140
(A) Ligand-gated calcium channels opening mm Hg. She is currently not taking any med-
(B) Ligand-gated potassium channels closing ications and has not seen a doctor for several
(C) Ligand-gated sodium channels opening years. The physician decides to address her
(D) Voltage-gated calcium channels opening hypertension urgently. Which of the following
(E) Voltage-gated potassium channels closing drugs is contraindicated in this patient?
(F) Voltage-gated sodium channels closing
(G) Voltage-gated sodium channels opening (A) Intravenous diltiazem
(B) Intravenous labetalol
7. A 50-year-old man presents to the emergency (C) Intravenous metoprolol
department because of substernal chest pain (D) Oral captopril
that started four hours ago and is becoming se- (E) Sublingual nifedipine
164 Section II: Organ Systems  •  Questions

10. A 64-year-old man is brought to the emergency (D) Protein deposits such as transthyretin
department for chest pain. An electrocardio- (E) Trypanosome transmitted by insect bite
gram demonstrates an ST-elevation myocardial
infarction in the posterior leads. After the ini- 12. A 72-year-old African-American man under-
tial management, he is admitted to the critical goes hip surgery. On his third hospital day, he
care unit for monitoring. On the fifth hospital develops chest pain, tachycardia, dyspnea, and
day, he experiences a sudden onset of dyspnea a low-grade fever. The man goes into cardiac
and hypotension. An echocardiogram shows arrest and efforts to resuscitate him are unsuc-
severe mitral regurgitation. An occlusion in cessful. On autopsy a massive pulmonary em-
which vessel is responsible for these findings? bolus is discovered. Which of the following
would most likely predispose the patient to this
(A) Left anterior descending artery
event?
(B) Left circumflex artery
High-Yield Systems

(C) Left marginal artery (A) Factor VIII deficiency


(D) Posterior descending artery (B) Low serum homocysteine levels
(E) Right marginal artery (C) Mutation in the factor V gene
(D) Overproduction of protein C
11. A 17-year-old boy dies suddenly while playing (E) Von Willebrand factor deficiency
basketball for his high school team. His heart
is examined at autopsy. According to the im- 13. A 16-year-old Asian girl sees her physician be-
age, which of the following is the most likely cause she has been experiencing fever, night
cause of this person’s death? sweats, and arthralgias. The physician diagno-
ses her with a disorder that is characterized by
thickening of the great vessels. Which of the
following signs and symptoms most likely will
be detected on history and physical examina-
tion?
(A) Abdominal pain and melena
(B) Intermittent jaw claudication
Cardiovascular

(C) Pneumonitis
(D) Strawberry tongue
(E) Weak pulses in the upper extremities

14. A 67-year-old woman presents to the emer-


gency department complaining of dizziness.
During the interview, she experiences two epi-
Reproduced, with permission, from USMLERx.com.
sodes of near-syncope. Physical examination
reveals palpitations and slight bradycardia. Her
daughter, who accompanies her, states that the
(A) Enterovirus infection patient is taking a medication for “heart trou-
(B) Immune-mediated destruction of cardio- bles,” but she cannot remember its name. Re-
myocytes sults of ECG are shown in the image. Which
(C) Mutation in a sarcomere gene such as of the following medications is this patient
myosin-binding protein C likely taking that could both explain her symp-
toms and produce the abnormalities shown in
this ECG?
Chapter 8: Cardiovascular  •  Questions 165

17. Atherosclerosis is associated with numerous


well-known risk factors, such as age, smoking,
diabetes, hyperlipidemia, and a family history
of atherosclerosis. LDL cholesterol is believed
to be a key factor in the pathway through
which hyperlipidemia causes atherosclerosis.
Reproduced, with permission, from USMLERx.com.
What is most likely the first step in the patho-
genesis of atherosclerosis caused by hyperlipid-
(A) Adenosine emia?
(B) Bretylium (A) Endothelial dysfunction
(C) Propranolol (B) Foam cell formation
(D) Quinidine (C) LDL cholesterol oxidation

High-Yield Systems
(E) Verapamil (D) Monocyte activation
(E) Plaque formation
15. A 65-year-old African-American man is ad-
mitted to the hospital for severe shortness of 18. Jugular venous pressure (JVP) curves are de-
breath. He states that he has been having in- signed to show the pressure changes that nor-
creased difficulty breathing when performing mally take place in the right atrium through-
physical activity. Lung auscultation reveals bi- out the cardiac cycle. A JVP curve consists of
lateral crackles. His blood pressure is 125/85 two, or sometimes three, positive waves and
mm Hg, and his heart rate is 85/min. Cardiac two negative troughs. A normal JVP curve is
auscultation detects no rubs or murmurs. shown in the image. Which of the following
Blood tests are not suggestive of infection. points on the normal jugular venous tracing
Coronary artery angiography is performed and below would be most prominently affected in
shows no significant vascular disease. What is tricuspid regurgitation?
the most likely cause of this patient’s disease
manifestations?
R
(A) Aortic stricture

Cardiovascular
(B) Fibrosis of the endocardium
(C) Fibrotic debris within the pericardial
P T
membrane ECG
(D) Ischemia of the myocardium tracing
Q
(E) Ventricular dysfunction
mm Hg

S
a c
16. A 25-year-old pregnant woman goes to her v
y
Jugular x
gynecologist for her 36-week checkup. She tracing
complains of light-headedness when she lies
down in bed at night. In the office her blood
pressure is 120/70 mm Hg while sitting upright Reproduced, with permission, from USMLERx.com.
and 90/50 mm Hg while lying supine. Which
of the following is the most likely cause of this
hypotension? (A) C and X
(B) A and Y
(A) Anemia (C) A and C
(B) Cardiogenic shock (D) V and Y
(C) Inferior vena cava compression
(D) Third spacing of fluid
(E) Vasodilation
166 Section II: Organ Systems  •  Questions

19. A 56-year-old Asian man with hypertension,


hypercholesterolemia, and type 2 diabetes mel-
litus comes to a physician for a check-up. It
has been several years since he has been to the
doctor. His past medical history is significant
for an acute illness at the age of nine, which
involved a high fever, pleuritic chest pain, mi-
grating joint pain, and a pink, nonpruritic rash
on his torso. His blood pressure is 155/100 mm
Hg and heart rate is 70/min. Auscultation of
the heart reveals a low-pitched diastolic rum-
ble heard best at the apex. What is the most
High-Yield Systems

likely pressure change that would be seen in


Courtesy of Dr. Susan Lindsley, Centers for Disease Control
this patient’s heart?
and Prevention.
(A) Decreased left atrial pressure
(B) Decreased left ventricular pressure
(C) Increased left atrial pressure (A) Atherosclerosis
(D) Increased left ventricular pressure (B) Congenital medial weakness
(E) Increased right atrial pressure (C) Cystic medial necrosis
(D) Disruption of the vasa vasorum
20. A common location for an abdominal aortic (E) Hypertension
aneurysm is inferior to the renal arteries and
extending to the bifurcation of the common 22. A baby is born to a mother who was not im-
iliac arteries. Repair involves resecting the munized as a child. On gross physical ex-
diseased portion of the aorta and replacing it amination, the infant appears normal, but on
with a synthetic graft. Based on anatomic con- cardiac examination is found to have a con-
siderations, which structure is most at risk of tinuous murmur in both systole and diastole.
is­chemia during repair of an aneurysm at this The physician prescribes a particular drug for
the infant, and on follow-up the murmur has
Cardiovascular

specific location?
disappeared. What is the mechanism of action
(A) Ascending colon of the drug most likely prescribed?
(B) Sigmoid colon
(C) Small intestine (A) Increases thromboxane A2
(D) Spleen (B) Inhibits prostaglandin E2 formation
(E) Stomach (C) Stimulates M2-receptors
(D) Stimulates prostaglandin E2-receptors
21.
A 52-year-old African-American man is (E) Stimulates prostaglandin F2a-receptors
brought to the emergency department unre-
sponsive. Efforts to resuscitate him are unsuc- 23. A 25-year-old white woman with no past medi-
cessful. On autopsy, they discover that he suf- cal history presents to the emergency depart-
fered from a ruptured aneurysm of the aortic ment for “a racing heartbeat.” It is determined
root. A picture of his dilated aorta is shown be- that she has paroxysmal supraventricular tachy-
low. In addition, inspection of the man’s skin cardia. Which of the following is the drug of
reveals several nodular lesions that are present choice used for diagnosing and abolishing
throughout his trunk and extremities. Which atrioventricular nodal arrhythmias by virtue of
of the following is most likely associated with its effectiveness and its low toxicity?
the underlying etiology of this patient’s aneu- (A) Adenosine
rysm? (B) Bretylium
Chapter 8: Cardiovascular  •  Questions 167

(C) Encainide lar volume for a single cardiac cycle. This pa-
(D) Lidocaine tient’s S4 heart sound heard on auscultation
(E) Sotalol would best correspond to which of the follow-
ing points?
24. Gemfibrozil has been proven to modestly
decrease LDL cholesterol levels, modestly
140
increase HDL cholesterol levels, and signifi- F
cantly decrease triglyceride levels. The mecha- 120 E
nism of gemfibrozil is best described by which D
100
of the following?

Pressure (mmHg)
80
(A) Slows the conversion of VLDL to LDL
cholesterol 60

High-Yield Systems
(B) Increases expression of triglyceride recep- 40
tors C
(C) Inhibits peroxisome proliferator-activated 20 B
receptor a nuclear transcription (PPAR-a) 0
regulator 40 85 130
Ventricular volume (mL)
(D) Promotes hepatocyte lipolysis A
(E) Activates lipoprotein lipase
Reproduced, with permission, from USMLERx.com.
25. A 48-year-old executive presents to the emer-
gency department because of chest tight-
(A) A
ness and shortness of breath. ECG shows ST-
(B) B
segment elevations in leads V4, V5, and V6.
(C) C
He has a history of high blood pressure, and
(D) D
his father died of heart problems at a young
(E) E
age. Assuming no other cardiac history, which
(F) F
of the following myocardial abnormalities

Cardiovascular
would most likely be seen via light microscopy
27. A 67-year-old woman with a long history
eight hours after his symptoms began?
of poorly controlled diabetes mellitus and
(A) Contraction bands chronic renal failure is admitted to the hospi-
(B) Granulation tissue tal for treatment of cellulitis. Two days into her
(C) Monocytic infiltrate hospital stay she complains of chest pain that
(D) Neutrophilic infiltrate is relieved when she leans forward. An ECG
(E) No change can be detected with light mi- shows diffuse ST segment elevations with PR
croscopy at this time depressions; her echocardiogram is normal.
Which of the following is the most appropriate
26. A 58-year-old man with a past medical his- treatment at this time?
tory of hypertension goes to his physician for
(A) Cardiac catheterization
a routine visit. On physical examination the
(B) Dialysis
physician is able to detect an S4 heart sound,
(C) Nonsteroidal anti-inflammatory drugs
and refers the patient to a cardiologist. After a
(D) Pericardiocentesis
thorough work-up, he is found to have left ven-
(E) Switch her to another antibiotic regimen
tricular hypertrophy. The image below plots
left ventricular pressure versus left ventricu-
168 Section II: Organ Systems  •  Questions

28. An 87-year-old man suffered an acute ST- 30. A 65-year-old woman with type 2 diabetes mel-
elevation myocardial infarction a few minutes litus is prescribed a drug by her physician to
ago. He was subsequently treated with aspirin, treat her hypertension. She returns complain-
metoprolol, and heparin. Immediately be- ing of facial swelling and a cough. Her physi-
fore being taken to the catheterization labora- cian promptly switches her to a new drug,
tory, the patient becomes unresponsive. He which has the same desired effect but fewer
is placed on telemetry and his rhythm strip is adverse effects. What is the mechanism of ac-
shown in the image. Although the patient re- tion of the replacement drug?
ceives a series of emergent defibrillations, he
(A) Blockade of b1 receptors
does not convert to sinus rhythm. Due to fail-
(B) Inhibition of angiotensin II receptors
ure to respond, the patient is given an agent as-
(C) Inhibition of angiotensin-converting en-
sociated with which of the following potential
zyme
High-Yield Systems

adverse effects?
(D) Inhibition of calcium channels
(E) Inhibition of NaCl reabsorption in the
early distal tubule

31. A 75-year-old man with a history of small cell


lung cancer diagnosed six weeks ago presents
to the emergency department in shock. He
Reproduced, with permission, from USMLERx.com. is found to have a heart rate of 137/min, res­
piratory rate of 25/min, and blood pressure
of 85/45 mm Hg that decreases to 70/40 mm
(A) Bleeding Hg with inspiration. On examination the pa-
(B) Increased post-myocardial infarction mor- tient has jugular venous distension and distant
tality heart sounds but clear lungs. ECG shows sinus
(C) Malar rash tachycardia with electrical alternans. Which of
(D) Pulmonary fibrosis the following is the best next step in manage-
(E) Yellow-green vision ment?
Cardiovascular

29. A 24-year-old man presents to the emergency (A) Administer diltiazem


department with fever, chills, night sweats, (B) Administer metoprolol
malaise, and fatigue that started three days ago. (C) Diuresis
In the past 24 hours, he has become short of (D) Intravenous fluids
breath. He admits to using intravenous drugs (E) Surgery
regularly. At presentation the patient is shaking
32. A 25-year-old college student presents to his
and appears pale. Physical examination is re-
primary care physician. He said he first started
markable for a temperature of 39.4°C (103°F),
to notice problems a few months ago after
hypoxia to 88% on room air, jugular venous
returning from a hike in the woods at a park
distention, bilaterally decreased breath sounds
in upstate New York. He originally had an
at the bases of the lungs, and a grade III/VI sys-
expanding rash starting on his calf and flu-
tolic murmur heard best at the lower left ster-
like symptoms that resolved spontaneously.
nal border. Which pathogen is most likely re-
Recently, he started having symptoms of diz-
sponsible for this patient’s condition?
ziness, syncope, dyspnea, chest pain, and
(A)
Enterococcus faecalis palpitations for several weeks’ duration. His
(B)
Haemophilus aphrophilus physician obtains an ECG, as shown in the
(C)
Staphylococcus aureus image. The vector that carries the organism
(D)
Streptococcus bovis responsible for the student’s symptoms is also
(E)
Streptococcus viridans responsible for transmitting which of the fol-
lowing diseases?
Chapter 8: Cardiovascular  •  Questions 169

B
A C D
P P P P P P P P P P P P

1 2
Reproduced, with permission, from USMLERx.com. Pain Days

Reproduced, with permission, from USMLERx.com.

(A) Babesiosis
(B) Epidemic typhus (A) Aspartate aminotransferase
(C) Malaria (B) Brain natriuretic peptide

High-Yield Systems
(D) Plague (C) C-reactive protein
(E) Rocky Mountain spotted fever (D) Creatine kinase-MB fraction
(E) Lactate dehydrogenase
33. A 55-year-old woman with a history of myocar- (F) Myoglobin
dial infarction (MI) in the area of distribution (G) Troponin
of the left anterior descending artery (LAD)
presents to her cardiologist because of fatigue, 35. A girl is born prematurely at 26 weeks’ gesta-
orthopnea, nocturnal dyspnea, weight gain, tion. She is placed in an incubator and appears
and “swollen ankles.” Echocardiography re- to be in stable condition. During a cardiac ex-
veals normal timing of the cardiac cycle and amination on her second day of life, the physi-
normal cardiac valves. Which of the following cian hears a machine-like murmur in the sec-
would most likely be heard in this patient on ond intercostal space at the left sternal border.
auscultation? He prescribes indomethacin to eliminate the
(A) Holosystolic, harsh-sounding murmur, condition before complications arise. What is
loudest at tricuspid area the embryonic origin of the structure targeted
(B) Paradoxically split S2 by this drug?

Cardiovascular
(C) Pulmonary flow murmur (A) First aortic arch
(D) S3 (B) Fourth aortic arch
(E) S4 (C) Second aortic arch
(F) Widened splitting of S2 (D) Sixth aortic arch
(E) Third aortic arch
34. Elevations in cardiac enzymes are used to di-
agnose a myocardial infarction (MI). The
diagram below shows the duration of eleva-
tion of several of these enzymes following an
MI. Which cardiac enzyme becomes elevated
approximately four hours after an MI and re-
mains elevated 7-10 days afterward?
170 Section II: Organ Systems  •  Questions

36. A 61-year-old man is rushed to the emergency 38. A 2-year-old Japanese girl is brought to the
department after losing consciousness for a few clinic by her parents, who are concerned about
minutes at a shopping mall. His wife explained a persistent fever of 40°C (104°F) and an ery-
to the paramedics that he was walking up a thematous rash on her trunk that has been
flight of stairs when he became short of breath present for the past week. Laboratory tests
and fell to the ground. Once at the hospital, show an erythrocyte sedimentation rate of 80
the patient reports to the physician that over mm/hr, a C-reactive protein level of 15 mg/L,
the past three months, he has been feeling and a platelet count of 700,000/mm³. The
dizzy frequently, in addition to becoming short doctors decide to treat her with intravenous
of breath after walking five blocks. He reports immunoglobulin and aspirin to avoid a life-
that two years ago he was able to walk a mile threatening complication. What other signs
with no difficulty. On physical examination, might this patient’s doctors have seen that led
High-Yield Systems

the physician notes some crackles in the lung them to initiate this therapy?
bases bilaterally and carotid pulses that seem
(A) Hypothermia
somewhat weak and delayed in comparison to
(B) Optic neuritis
the patient’s heart sounds. Which other physi-
(C) Proteinuria
cal finding is likely to be found in this patient?
(D) Pulmonary edema
(A) Blowing holosystolic murmur that radiates (E) Strawberry tongue
to the axilla
(B) Crescendo-decrescendo systolic murmur 39. A 54-year-old woman comes to the physician
that radiates to the axilla three months after a undergoing a root canal,
(C) Crescendo-decrescendo systolic murmur with primary complaints of persistent general
that radiates to the carotids malaise and fever. The symptoms developed
(D) Diastolic decrescendo murmur that radi- slowly over the weeks following her root canal,
ates to the apex but have not abated. The patient has a temper-
(E) Diastolic decrescendo murmur that radi- ature of 38.3°C (100.1°F). Ophthalmic exami-
ates to the axilla nation reveals retinal hemorrhages with clear
(F) Opening snap followed by a mid-diastolic central regions. Examination of the extremi-
Cardiovascular

rumble that radiates to the axilla ties reveals painful red nodules on her dig-
its and dark macules on her palms and soles.
37. A 65-year-old man presents to the emergency On cardiac examination, a click and a systolic
department because of chest pain. He is found murmur are auscultated over the mitral valve.
to have a large inferior wall myocardial infarc- She tells the physician that the click is due to
tion with 2-mm ST-segment elevations in leads a mechanical valve replacement done four
II, III, and aVF. ECG also demonstrates grad- years ago because she had rheumatic fever as
ual lengthening of the PR interval until one a child. Which of the following is the most ap-
QRS complex is dropped. He is sent to the car- propriate treatment?
diac catheterization laboratory, where a stent is
(A) Acyclovir
placed in one of his major coronary arteries. In
(B) Caspofungin
which coronary artery was the stent most likely
(C) Clindamycin
placed?
(D) Mebendazole
(A) Diagonal branch of the LAD (E) Metronidazole
(B) Left anterior descending artery (F) Penicillin
(C) Left circumflex artery (G) Pentamidine
(D) Left marginal artery
(E) Right coronary artery
Chapter 8: Cardiovascular  •  Questions 171

40. A 60-year-old diabetic man is brought to the


emergency department by ambulance. His
wife told paramedics that he had been pro- 110
gressively drowsy over the previous two-three III

Pressure, mm Hg
hours. Forty-five minutes earlier he had an-
nounced that he was going to the bedroom to
administer his nightly dose of insulin. When II
IV
he didn’t return to the living room, she went to
20
check on him, and found him lying face down
on the bed with an empty insulin needle be-
side him and an open bottle of propranolol on I
the nightstand. Upon arrival in the emergency

High-Yield Systems
department the patient is unresponsive. Heart
rate is 45/min and his serum glucose level is 80 40 80 120 160
mg/dL. Which of the following is the most ap- Volume, cc
propriate treatment to reverse the most likely
Reproduced, with permission, from USMLERx.com.
cause of this patient’s condition?
(A) Albuterol
(B) Cocaine (A) Phase I
(C) Digitalis (B) Phase II
(D) Glucagon (C) Phase III
(E) Phenylephrine (D) Phase IV

41. Nitrates often are given as part of the manage- 43. A 32-year-old man with diabetes presents to his
ment of angina. What is the primary mecha- physician with orthostatic hypotension. A defi-
nism by which nitrates work in the treatment ciency in the normal physiologic response car-
of this condition? ried out by arterial baroreceptors located in the
aortic arch and the carotid sinus is suspected.
(A) Decrease atrioventricular node conduction

Cardiovascular
What is the mechanism of the normal physi-
velocity ologic response to hypotension?
(B) Decrease heart rate and contractility
(C) Increase oxygen delivery to myocytes (A) Decreased baroreceptor afferent firing in
(D) Stimulate dilation of coronary arterioles the aortic arch leads to increased sympa-
(E) Stimulate venodilation thetic efferent firing
(B) Decreased baroreceptor afferent firing in
42. The image depicts the relationship of left ven- the carotid sinus leads to increased para-
tricular pressure and volume in the cardiac cy- sympathetic efferent firing
cle. The various phases of the cardiac cycle are (C) Decreased baroreceptor afferent firing in
labeled I through IV. During which phase are the carotid sinus leads to increased sympa-
the pressures in the left atrium and left ventri- thetic efferent firing
cle most equal? (D) Increased baroreceptor afferent firing in
the aortic arch leads to increased parasym-
pathetic efferent firing
(E) Increased baroreceptor afferent firing in
the carotid sinus leads to increased sympa-
thetic efferent firing
172 Section II: Organ Systems  •  Questions

44. A 56-year-old woman presents to her physician (A) Cocaine abuse


due to recent onset of chest pain and dyspnea. (B) Diabetes
Six weeks prior, the patient suffered a myocar- (C) Hypercholesterolemia
dial infarction. Her physical examination is (D) Hypertension
remarkable for a friction rub over the fifth in- (E) Sickle cell disease
tercostal space in the midclavicular line, along
with an elevated jugular venous pressure. 46. A 76-year-old man receives a pacemaker to
What is the most likely cause of this patient’s treat a dangerous form of heart block. His con-
presentation? dition is characterized by an ECG with a con-
stant PR interval, with the random absence of
(A) Cardiac arrhythmia
QRS complexes. Which of the following ab-
(B) Dressler syndrome
normalities is the most likely cause of this type
(C) Left ventricular failure
High-Yield Systems

of heart block?
(D) Thromboembolism
(E) Ventricular rupture (A) Atrioventricular node abnormality
(B) Defect in the His-Purkinje system
45.
A 45-year-old African-American man is (C) Independently contracting atria and ventri-
brought to the emergency department because cles
of sudden chest pain radiating to the back. The (D) Retrograde conduction
pain started while the patient was taking his (E) Sinoatrial node abnormality
morning jog. X-ray of the chest is immediately
done and shows a widened mediastinum. The 47. A 56-year-old white man is rushed to the emer-
patient has a history of anaphylaxis related to gency department after complaining of crush-
the type of iodinated contrast agent used in ing substernal chest pain. He is morbidly
CT. Therefore, an MRI is performed to con- obese, diaphoretic, tachypneic, and clutching
firm the expected diagnosis. Findings on a sag- his chest. Initial ECG reveals ST-segment el-
ittal bright blood MRI are shown in the image. evations in the anterior leads. The patient is
Which of the following co-morbidities is most stabilized and seems to be doing well, but then
directly responsible for the patient’s potentially he suddenly experiences cardiac arrest and
Cardiovascular

fatal condition? dies. Which of the following is the most likely


cause of death in this patient?
(A) Arrhythmia
(B) Formation of ventricular septal defect
(C) Mural thrombosis
(D) Myocardial pump failure
(E) Rupture of ventricular free wall
(F) Ruptured papillary muscle

Reproduced, with permission, from USMLERx.com.


Chapter 8: Cardiovascular  •  Questions 173

48. The image is a representation of the pressure- 49. Cardiac output is a function of stroke volume
volume (P-V) relationship in the left ventricle and heart rate. Stroke volume increases when
during a typical cardiac cycle. The phases contractility increases, preload increases, or af-
of the cardiac cycle are labeled I through IV. terload decreases. There are a number of fac-
Which of the following occurrences alone tors that affect each of these components and
would increase the width of the P-V loop? ultimately cardiac output. Which of the follow-
ing variations would increase cardiac output in
an otherwise normal patient?
(A) b-Blocker treatment
110 (B) Cardiac glycoside administration
III (C) Increasing extracellular sodium concentra-
Pressure, mm Hg

tion

High-Yield Systems
(D) Lowering intracellular calcium concentra-
tion
IV II
(E) Metabolic acidosis
20

50. A 54-year-old woman presents to her physician


I with swelling in her extremities. Palpation pro-
duces significant pitting. Which of the follow-
ing conditions represents the most likely physi-
40 80 120 160 ologic basis for this physical finding?
Volume, cc
(A) Decreased capillary permeability
Reproduced, with permission, from USMLERx.com. (B) Decreased capillary pressure
(C) Increased interstitial fluid pressure
(D) Increased interstitial oncotic pressure
(A) Decreased afterload (E) Increased plasma protein levels
(B) Decreased contractility
(C) Decreased preload

Cardiovascular
(D) Increased arterial pressure
(E) Increased end-systolic volume
174 Section II: Organ Systems  •  Answers

An s w e r s

1. The correct answer is D. This patient has Answer A is incorrect. This is the mechanism
Wolff-Parkinson-White (WPW) syndrome, a of calcium channel blockers such as nifedi­
condition whereby the heart is abnormally pine, not of hydralazine.
stimulated by an accessory electrical pathway
Answer B is incorrect. Hydralazine works by
(often known as the bundle of Kent). The
increasing cGMP, not decreasing it.
presence of Δ-wave (a sloping upstroke of the
QRS complex) is diagnostic. Amiodarone is Answer D is incorrect. This is the mechanism
an effective pharmacologic agent for this con- of angiotensin-converting enzyme (ACE) in-
dition. This is a potassium channel blocker hibitors, not of hydralazine.
(class III antiarrhythmic). Procainamide also
High-Yield Systems

Answer E is incorrect. This is the mechanism


can be used; however, it is a sodium channel of acetazolamide, not hydralazine.
blocker (class Ia), which is not one of the an-
swer choices. Answer F is incorrect. Hydrochlorothiazide
acts on the early distal tubule in the kidney.
Answer A is incorrect. An example of a b-
adrenergic receptor blocker is metoprolol. It is 3. The correct answer is E. The inhibition of de
used to treat hypertension and atrial fibrillation novo cholesterol synthesis results in reduced
(to slow down the ventricular rate) but is con- intrahepatic cholesterol concentrations, which
traindicated WPW sndrome. This is a class II then stimulates hepatocytes to upregulate sur-
antiarrhythmic agent. face LDL cholesterol receptors. This facilitates
Answer B is incorrect. Acetylcholine recep- the clearance of LDL cholesterol from the cir-
tor is blocked by atropine, among other anti- culation. In addition, circulated LDL choles-
cholinergic agents. Atropine has a wide variety terol precursors, including VLDL cholesterol
of clinical uses such as increasing sinoatrial remnants, are cleared more rapidly from circu-
node firing (to treat bradycardia) and decreas- lation because of their cross-recognition with
ing bronchiole secretion (such as during anes- hepatic LDL cholesterol receptors.
Cardiovascular

thesia). Answer A is incorrect. Although statins de-


Answer C is incorrect. An example of a cal- crease hepatic synthesis of cholesterol, they
cium channel blocker is verapamil (class IV have no direct effect on LDL cholesterol pro-
antiarrhythmic). It is used to treat hypertension duction.
and atrial fibrillation (to slow down the ven- Answer B is incorrect. Statins only mildly re-
tricular rate) but is contraindicated in WPW duce the plasma concentrations of triglycer-
syndrome. ides.
Answer E is incorrect. The sodium-potassium Answer C is incorrect. Statins cause upregula-
ATPase enzyme is inhibited by digoxin, which tion, not downregulation, of surface LDL cho-
is used to increase myocardial contraction in lesterol receptors.
the symptomatic treatment of congestive heart
failure (CHF). Answer D is incorrect. Statins do not directly
increase HDL cholesterol production. They
2. The correct answer is C. Drug X is hydrala- can have a mild beneficial effect on HDL cho-
zine, a vasodilator used to treat essential hyper- lesterol in some patients, but it is not a consis-
tension and CHF (although rarely used today). tent result.
It works by increasing cGMP levels, which
causes smooth muscle relaxation. The effect 4. The correct answer is D. The key is to real-
is greater on arterioles than veins, causing a re- ize that the question is asking for results of
duction in afterload. urinalysis (not serum electrolyte values). The
Chapter 8: Cardiovascular  •  Answers 175

ECG shows peak T waves and a widened Answer C is incorrect. Patent ductus arterio-
QRS interval, which are classic changes seen sus is not the cause of tetralogy of Fallot. In
in hyperkalemia. Spironolactone is the most fact, a patent ductus arteriosus is protective
likely medication to affect urinary electrolytes. in patients with tetralogy of Fallot because it
As an inhibitor of aldosterone receptors in causes some of the unoxygenated blood from
the collecting tubule and an inhibitor of Na+ the overriding aorta to return to the pulmonary
channels, spironolactone greatly decreases the artery to be oxygenated.
excretion of K+ and mildly increases the excre-
Answer D is incorrect. Pulmonary stenosis is
tion of Na+. Urine volume will be high-normal
one of the four manifestations of tetralogy of
because the diuretic will increase saltwater
Fallot, not the cause.
wasting.
Answer E is incorrect. VSD is one of the four
Answer A is incorrect. Spironolactone de-

High-Yield Systems
manifestations of tetralogy of Fallot, not the
creases K+ excretion, so there will be decreased
cause.
levels of K+ in the urine sample.
Answer B is incorrect. Na+ excretion will be 6. The correct answer is G. Voltage-gated Ca+
increased with the use of spironolactone; also, channels (L type) open slowly in response to
diuretics will increase the amount of urine vol- the sodium upstroke (approximately around
ume excreted. -40 mV), allowing calcium to flow down its
concentration gradient and into the cell. Con-
Answer C is incorrect. Spironolactone will
currently, there is an outward potassium cur-
increase Na+ excretion and decrease K+ ex-
rent via voltage-gated channels that leads to
cretion so that K+ concentrations will be de-
the plateau. The result is a slow conduction ve-
creased in the urine and Na+ concentrations
locity that prolongs the transmission from the
will be increased. Treatment involves cystidine
atria to the ventricles.
or uridine to bypass this step in pyrimidine
synthesis and also to negatively downregulate Answer A is incorrect. Ion channels in the
orotic acid production. myocardium are voltage gated.
Answer E is incorrect. Spironolactone de- Answer B is incorrect. During phase 2,

Cardiovascular
creases K+ excretion but increases Na+ excre- voltage-gated potassium cells open to allow po-
tion; therefore, Na+ concentrations will be el- tassium efflux.
evated in the urine. Answer C is incorrect. Ion channels in the
myocardium are voltage gated.
5. The correct answer is A. This patient has te-
tralogy of Fallot, which is defined by the com- Answer D is incorrect. Voltage-gated sodium
bined symptoms of ventricular septal defect channels are responsible for the upstroke in
(VSD), overriding aorta, pulmonary stenosis, ventricular cells (phase 0).
and right ventricular hypertrophy. The cause Answer E is incorrect. During phase 2,
of these abnormalities, however, is an antero- voltage-gated potassium cells open to allow po-
superior displacement of the infundibular tassium efflux.
septum during heart development in utero.
Patients with tetralogy of Fallot learn to squat Answer F is incorrect. Closing voltage-gated
during cyanotic spells, which causes compres- sodium channels would hyperpolarize the cell.
sion of the femoral arteries, thereby decreases Voltage-gated sodium channels are responsible
their right-to-left shunt. for the upstroke in ventricular cells (phase 0).
These open in response to depolarization to
Answer B is incorrect. An overriding aorta is the -55mV threshold value, allowing sodium
one of the four manifestations of tetralogy of to rapidly flow down its concentration gradient
Fallot, not the cause. into the cell. These channels are then inacti-
176 Section II: Organ Systems  •  Answers

vated and cannot be opened again until the 8. The correct answer is A. Epinephrine is a
cell is repolarized. nonselective agonist of a- and b-adrenergic
receptors. Administering a large dose of epi-
7. The correct answer is E. Troponin is a protein nephrine causes an increase in blood pressure
found along the sarcomeres that assist in mus- via an increased heart rate and in contractil-
cle contraction. With muscle injury, troponin ity through stimulation of b1 receptors; and
is leaked into the serum. Different fractions net a increase in systemic vascular resistance
show different specificities for different tissues. through a1-mediated vasoconstriction (the
In cardiac tissue, troponin I has been shown to b2-mediated vasodilation is negligible com-
be more specific and equally if not more sensi- pared to the a1 effects). Adding phentolamine,
tive than cardiac enzymes, CK-MB in particu- a selective a1 antagonist, blocks the a effects
lar. of epinephrine. Therefore re-administration
High-Yield Systems

Answer A is incorrect. Alanine aminotransfer- leaves only the b1-receptor actions (increased
ase (ALT) is found mostly in the liver and is contractility and heart rate) and the b2-
therefore a specific marker of damage to hep­ mediated increase in vasodilation, causing a
atocytes. Elevations of ALT are not associated net decrease in blood pressure.
with myocardial infarction (MI). Answer B is incorrect. Isoproterenol is an ago-
Answer B is incorrect. Aspartate aminotrans- nist of b- and a-adrenergic receptors, although
ferase (AST) is an enzyme found in the heart, its primary action is at the b receptor. Hence,
liver, and skeletal muscle, and an elevation in adding isoproterenol actually would cause a
it is nonspecific for damage to those tissues. decrease in pressure through b2-mediated vaso-
Although commonly elevated in the setting of dilation. Clonidine is an a agonist, and would
liver disease, AST levels peak around day two lead to decreased sympathetic outflow and pos-
post-MI. sibly cause an additional decrease in pressure.
Adding isoproterenol after clonidine admin-
Answer C is incorrect. Creatinine kinase (CK) istration would lead to a further decrease in
is located on the inner mitochondrial mem- blood pressure.
brane, on myofibrils, and in the muscle cyto-
Answer C is incorrect. Norepinephrine is an
Cardiovascular

plasm and catalyzes the production of ATP.


It is released during skeletal muscle injury agonist at mainly a-adrenergic receptors but
(MM), cardiac injury (MB), and brain injury also some b1 activity. This leads to an increase
(BB). In this case, there is an infiltration of in vasoconstriction. Propranolol is a nonspe-
neutrophils and the cardiac muscle strand is cific b blocker, which would not block the ef-
lysing and losing its striations. These findings fects of norepinephrine, so readministration
are consistent with an acute MI one-three days would cause an increase in blood pressure.
earlier. During an acute MI, the most specific Answer D is incorrect. Phenylephrine is an a1
serum elevations are seen in CK-MB and tro- agonist that would cause an increase in pres-
ponin I. Between the two, however, troponin I sure through a1-stimulated vasoconstriction.
has been proven to be more specific, and Metoprolol is a b1 blocker, which would not
equally if not more sensitive than CK-MB. inhibit the effects of phenylephrine. There-
Answer D is incorrect. Lactate dehydrogenase fore re-administration of phenylephrine would
(LDH) is an enzyme found in many tissues cause another increase in blood pressure.
throughout the body. It is a useful marker for Answer E is incorrect. Phenylephrine is an a1
liver disease, MI, and hemolysis but is not spe- agonist that would cause an increase in pres-
cific for any one tissue. Before CK-MB (car- sure through a1-stimulated vasoconstriction.
diac injury) and troponin I detection, it was Phentolamine is a nonselective a blocker,
the test of choice in diagnosing MI. which would block the increase in pressure
caused by phenylephrine and cause no change
Chapter 8: Cardiovascular  •  Answers 177

in pressure after repeat phenylephrine admin- Answer A is incorrect. The anterior papillary
istration. muscle is supplied by both the left anterior de-
scending artery and the left circumflex artery.
9. The correct answer is E. Nifedipine is a dihy- Because of its dual blood supply, the anterior
dropyridine class calcium channel blocker that papillary muscle is less likely to rupture after
could be used in the long-term control of hy- a MI. Furthermore, the patient’s initial MI in-
pertension. However, in the case of a hyperten- volved the posterior aspect of the myocardium.
sive emergency, nifedipine used sublingually
Answer B is incorrect. The anterior papillary
can cause dangerous fluctuations in blood
muscle is supplied by both the left anterior de-
pressure that are difficult to control and can
scending artery and the left circumflex artery.
lead to more harm than good.
Because of its dual blood supply, the anterior
Answer A is incorrect. Diltiazem is a benzo- papillary muscle is less likely to rupture after

High-Yield Systems
thiazepine class calcium channel blocker that a MI. Furthermore, the patient’s initial MI in-
reduces myocardial demand and also causes volved the posterior aspect of the myocardium.
vasodilation. It is not contraindicated in this
Answer C is incorrect. The left (or obtuse)
patient.
marginal artery, which is a branch of the left
Answer B is incorrect. Labetalol is a com- circumflex artery, follows the left border of the
bined a/b-blocker that has effects on both re- heart to supply the left ventricle.
ceptors. It can be used in a hypertensive situa-
Answer E is incorrect. The right marginal ar-
tion as an emergent option for treatment. It is
tery follows the inferior border of the heart to
not contraindicated in this patient.
supply the right ventricle.
Answer C is incorrect. Metoprolol is a
b-blocker used to treat angina by reducing 11. The correct answer is C. The key finding is
heart rate and contractility. It also reduces the hypertrophy with asymmetric septal enlarge-
metabolic demand of the myocardium. It is ment without free ventricular wall enlarge-
often used to control hypertension, but is not ment. This is seen in patients with hyper­
contraindicated in this patient. trophic obstructive cardiomyopathy (HOCM).

Cardiovascular
Mutation in a sarcomere gene such as myosin-
Answer D is incorrect. Captopril is an ACE
binding protein C is one of the most common
inhibitor used in the control of chronic hyper-
genetic causes of HOCM. This type of cardio-
tension. It is especially useful for patients who
myopathy is inherited in an autosomal domi-
have signs of renal disease and can slow the
nant fashion. The anatomic distortion can lead
progression of damage to the kidneys. It is not
to a dynamic ventricular outflow obstruction
contraindicated in this patient.
during systolic ejection, which leads to a sys-
10. The correct answer is D. This patient most tolic murmur, dyspnea, lightheadedness, syn-
likely suffered from acute mitral regurgitation cope, and in many cases sudden death. On
secondary to rupture of the posterior papillary histologic examination, HOCM would show
muscle. The anterior and posterior papillary disoriented, tangled, hypertrophied myocardial
muscles anchor the chordae tendineae, which fibers.
prevent the cusps of the mitral valve from be- Answer A is incorrect. Infection with an En-
ing forced into the left atrium. An occlusion terovirus, namely coxsackie B virus, initially
of the posterior descending artery can lead to causes a subacute myocarditis but can eventu-
an infarction of the posterior papillary muscle ally lead to dilated cardiomyopathy (DCM).
and subsequent rupture of the muscle several Pathologically, DCM produces a grossly en-
days later. Patients will present with a sudden larged heart with dilation of all four chambers
onset of pulmonary edema and frequently car- and normal ventricular wall thickness. Other
diogenic shock. etiologies for DCM include alcohol abuse, wet
beriberi, cocaine use, Chagas disease, doxoru-
178 Section II: Organ Systems  •  Answers

bicin toxicity, hemochromatosis, and peripar- Answer D is incorrect. Proteins C and S act as
tum cardiomyopathy. negative regulators of the coagulation cascade.
Therefore a deficiency, rather than overpro-
Answer B is incorrect. Immune-mediated de-
duction, will lead to a hypercoagulable state.
struction of cardiomyocytes is the cause of car-
diac damage in rheumatic fever. This disease Answer E is incorrect. Von Willebrand factor
is a consequence of pharyngeal infection with allows platelets to adhere to a defect where col-
group A b-hemolytic streptococci. Early deaths lagen is exposed and binds inactive factor VIII
from rheumatic heart disease are due to myo- in circulation. A deficiency (von Willebrand
carditis, whereas late sequelae include damage disease) leads to bleeding complications such
to the heart valves. Histologically, this disease as epistaxis, menorrhagia, and gastrointestinal
is associated with Aschoff bodies, which are (GI) bleeds.
granulomas with giant cells. The heart muscle
High-Yield Systems

itself, however, would appear grossly normal. 13. The correct answer is E. This girl has
Takayasu arteritis, a large-vessel vasculitis usu-
Answer D is incorrect. Deposits of protein
ally found in women <40 years of age of Asian
such as light chains, heavy chains, or trans-
descent. It is characterized by a thickening of
thyretin are associated with amyloidosis. Amy-
the aortic arch and/or the proximal great ves-
loidosis produces a restrictive cardiomyopathy,
sels. The most prominent feature is weak
in which the ventricular wall and chamber
pulses in the upper extremities. Complications
size grossly appear normal, which is not con-
of untreated late-stage Takayasu include aor-
sistent with the reduced chamber size seen in
tic aneurysms and typically involve the aortic
this specimen. In addition, amyloidosis is gen-
arch. This leads to narrowing, or possible oblit-
erally a disease of the elderly, and is thus not as
eration, of the major arteries associated with
likely in this 17-year-old patient.
the aortic arch. Treatment involves high doses
Answer E is incorrect. Chagas disease is of oral prednisone that are tapered over many
caused by a trypanosome that is primarily months as the clinician and patient agree to
endemic to South America. Eighty percent minimize the adverse effects of corticosteroids.
of those infected will develop myocarditis,
Answer A is incorrect. Abdominal pain and
Cardiovascular

and 10% will suffer acute cardiac death later.


melena can be present in patients with polyar-
Grossly, the myocardium can appear normal
teritis nodosa, a vasculitis of small- or medium-
or slightly dilated, with minute hemorrhagic
sized arteries. These symptoms are due to
lesions.
ischemia and infarction of the intestines.
Classically, Takayasu disease affects the aortic
12. The correct answer is C. A mutation in the
arch, although the aortic root (dilation with
factor V gene, also known as factor V Leiden,
subsequent valvular insufficiency) and coro-
causes resistance to deactivation of factor V by
nary arteries (narrowing and potential infarc-
protein C. Uninhibited factor V activity leads
tion) may be involved in the later stages of the
to a hypercoagulable state, which predisposes
disease.
to deep vein thrombosis (DVT) and subse-
quent pulmonary embolism (PE). Answer B is incorrect. Intermittent jaw clau-
dication is indicative of temporal (giant-cell)
Answer A is incorrect. Factor VIII deficiency
arteritis. Giant-cell arteritis is a medium/large
(hemophilia A) would predispose an individual
vessel vasculitis that typically affects arteries
to bleeding. Factor VIII is an integral part of
on the head. It usually is seen in women >70
the intrinsic coagulation cascade.
years, and common symptoms include fever,
Answer B is incorrect. High, rather than low, headache (temporal area), jaw claudication,
homocysteine levels lead to a hypercoagulable reduced visual acuity, and sudden vision loss.
state. The diagnosis is achieved via a tissue biopsy of
the temporal artery and treatment is with cor-
Chapter 8: Cardiovascular  •  Answers 179

ticosteroids. Giant-cell arteritis may present as diac cycle; in other words, it slows the diastolic
sudden monocular blindness, and emergent depolarization of pacemaker cells.
corticosteroids are needed to save that eye.
Answer E is incorrect. Bepridil, not verapamil,
Answer C is incorrect. Pneumonitis is a clas- is a calcium channel blocker and class IV an-
sic symptom of Wegener granulomatosis, a tiarrhythmic known to be associated with tor-
necrotizing small-vessel vasculitis primarily af- sades de pointes. Verapamil and diltiazem are
fecting the kidneys and the lungs. The classic two calcium channel blockers used in the pre-
clinical vignette for Wegener will describe a vention of nodal arrhythmias (eg, supraventric-
patient who presents with hemoptysis and he- ular tachycardia).
maturia and who has classical antineutrophil
cytoplasmic antibodies on serologic evaluation. 15. The correct answer is E. This patient presents
with shortness of breath and dyspnea on exer-
Answer D is incorrect. Strawberry tongue is

High-Yield Systems
tion. The bilateral crackles detected on physi-
seen in children with Kawasaki disease, an ar-
cal examination indicate pulmonary edema.
teritis that often involves the coronary arteries,
This spectrum of signs and symptoms are
but can affect vessels of any size. In children,
suggestive of left ventricular dysfunction lead-
the symptoms of Kawasaki’s can relent with-
ing to CHF. CHF itself has many causes, but
out treatment and the major complications
the lack of other clinical findings (no rubs or
are coronary artery aneurysms and MIs. The
murmurs) indicates either hypertensive heart
diagnostic criteria for this disease are five days
disease or cardiomyopathy as likely etiologies.
of fever plus four of five of the following: (1)
Cardiomyopathy can be divided into dilated,
erythema of the lips or oral cavity or cracking
hypertrophic, or restrictive. Alcoholism and
of the lips; (2) rash on the trunk; (3) swelling
coxsackie virus B have been strongly associated
or erythema of the hands or feet; (4) red eyes
with dilated cardiomyopathy.
(conjunctival injection); and (5) a swollen
lymph node in the neck >15 mm. Answer A is incorrect. Aortic strictures are
typically associated with the congenital de-
14. The correct answer is D. The ECG demon- fect known as coarctation of the aorta. Twice
strates a pattern characteristic of torsades de as common in men as in women, coarctation

Cardiovascular
pointes. Quinidine, a class IA antiarrhythmic is fairly common in patients with Turner syn-
agent, is used to treat supraventricular arrhyth- drome. While pumping against a permanently
mias by slowing conduction. A simultaneous elevated afterload may cause left ventricular
increase of the QT interval caused by quini- hypertrophy and eventually failure, this condi-
dine risks torsades de pointes. tion would result in a slightly different clinical
picture. The patient would have higher blood
Answer A is incorrect. Adenosine is used
pressure in the upper extremities than in the
both to diagnose and to treat supraventricular
lower extremities, causing additional symptoms
tachyarrhythmias. However, it is not associated
of claudication and coldness in the feet. Also,
with torsades des pointes.
coarctations often produce a detectable sys-
Answer B is incorrect. Although the class III tolic murmur.
antiarrhythmics tend to be associated with tor-
Answer B is incorrect. Fibrosis of the endocar-
sades des pointes, bretylium is an exception to
dium is associated with the effects of a carci-
this rule and has no association with this con-
noid tumor on the heart. Signs that are usually
dition.
detected on auscultation are pulmonic stenosis
Answer C is incorrect. Propranolol is a class II or tricuspid regurgitation. Classic symptoms of
antiarrhythmic, but it is not associated with tor- carcinoid syndrome include flushing, diarrhea,
sades de pointes. b Blockers such as proprano- and bronchospasm.
lol are used to suppress abnormal pacemakers
Answer C is incorrect. Fibrotic debris within
by decreasing the slope of phase 4 of the car-
the pericardial membrane occurs in constric-
180 Section II: Organ Systems  •  Answers

tive pericarditis. A pericardial friction rub Answer E is incorrect. Vasodilation will re-
would be detectable on auscultation. duce blood pressure, and pregnant women do
have a constant amount of vasodilation that
Answer D is incorrect. Ischemia of the myo-
is greater than that in nonpregnant women.
cardium is usually the result of coronary artery
Blood pressure decreases by about 10 mm Hg
disease and typically presents with complaints
during pregnancy, particularly during the first
of chest pain. An active coronary artery disease
and second trimesters. However, this vasodila-
process has been ruled out by angiography.
tion should remain relatively constant when
16. The correct answer is C. Inferior vena cava transferring from an upright to a supine posi-
(IVC) compression is common in women dur- tion, and would not account for the sudden hy-
ing the third trimester of pregnancy. The large potension seen in this case.
uterus compresses the IVC, decreasing venous
High-Yield Systems

17. The correct answer is C. Hyperlipidemia is a


return to the heart. This reduction in preload
recognized risk factor for atherosclerosis. Cur-
reduces stroke volume, thus reducing cardiac
rent concepts describing the pathogenesis of
output. Recall that mean arterial pressure =
this process suggest that the first step is the
cardiac output × total peripheral resistance;
oxidation of LDL cholesterol. This then causes
an acute decrease in either of these parameters
endothelial damage that leads to monocyte
will reduce blood pressure. Pregnant women
activation, formation of foam cells, smooth
can avoid this problem by placing a pillow un-
muscle proliferation, and subsequent arterial
der their right side or by lying on their left side
plaques. The oxidation of LDL cholesterol in-
to remove the weight of the gravid uterus from
creases monocyte receptor affinity for the mol-
the IVC.
ecule and stimulates the monocytes to migrate
Answer A is incorrect. During pregnancy the into the subendothelial space of the artery.
total plasma volume increases to 30%-50% Macrophages that phagocytose these mol-
greater than normal at term. RBC mass in- ecules become foam cells, which then lead
creases during pregnancy as well due to in- to other downstream effects of atherosclerosis
creased circulating erythropoietin levels. The such as calcification and occlusion of the ves-
expansion of intravascular volume is relatively sel lumen. This in turn can lead to ischemia of
Cardiovascular

greater than the increase in RBC mass, result- various tissues, most notably the heart.
ing in a physiologic anemia. However, this
Answer A is incorrect. Endothelial dysfunc-
generally does not cause adverse symptoms for
tion is one of the primary steps in the patho-
the mother, and would not cause hypotension
genesis of atherosclerosis. Endothelial dysfunc-
in a supine position.
tion can be caused by numerous factors, such
Answer B is incorrect. Cardiogenic shock can as the toxins found in cigarette smoke, and is
cause hypotension by decreasing the stroke not necessarily caused by oxidized LDL cho-
volume and cardiac output, but it would not lesterol. However, when looking specifically at
occur only in the supine position. the relationship between hyperlipidemia and
Answer D is incorrect. When fluid leaves the atherosclerosis, LDL oxidation generally oc-
intravascular space and enters the interstitial curs before endothelial dysfunction.
space, it is referred to as third spacing. In preg- Answer B is incorrect. Foam cells are formed
nancy, there is a physiologic amount of third from macrophages that phagocytosed oxi-
spacing, which causes dependent edema in the dized LDL cholesterol. In fact, they only up-
hands and feet. Some women may even expe- take LDL cholesterol once it has undergone
rience pulmonary edema, which can be dan- oxidation, which then causes changes in the
gerous. Third spacing does cause hypotension LDL cholesterol that signal activation of mac-
if the intravascular volume is not replaced, but rophage “scavenger” receptors (CD36). Foam
it would not cause isolated hypotension in the cells play an important role in the pathogen-
supine position. esis of atherosclerosis, but they are not the first
Chapter 8: Cardiovascular  •  Answers 181

step in the pathway between hyperlipidemia erythema marginatum) is indicative of rheu-


and atherosclerosis. matic fever. If left untreated, rheumatic fever
can evolve into rheumatic heart disease, which
Answer D is incorrect. Monocyte activation
typically presents with mitral valve stenosis.
plays a role in the pathogenesis of atheroscle-
This can be heard on auscultation as a high-
rosis, but it is not the first step in the pathway
pitched opening snap that follows S2, and a
between hyperlipidemia and atherosclerosis.
low-frequency decrescendo diastolic murmur
Answer E is incorrect. Plaque formation is the heard best over the apex of the heart. The in-
result of many of these steps, and it contributes creased resistance to flow from the left atrium
to the pathogenesis of atherosclerosis. How- to the left ventricle leads to an increase in left
ever, it is not the first step in the pathway be- atrial pressure.
tween hyperlipidemia and atherosclerosis.
Answer A is incorrect. Left atrial pressure

High-Yield Systems
18. The correct answer is A. In tricuspid regurgi- would increase, not decrease, with mitral ste-
tation, blood flows backward into the atria dur- nosis.
ing ventricular systole. This would affect the Answer B is incorrect. Mitral stenosis would
C and X waves, replacing them with a large not result in an overall decrease in left ventric-
positive deflection. This positive deflection ef- ular pressure; pressure will only be decreased
fectively joins the C wave and the V wave, cre- relative to the left atrium.
ating the “CV wave.” The C wave is thought to
Answer D is incorrect. Left ventricular pres-
be due to pressure on the tricuspid valve dur-
sure would increase with aortic stenosis. Rheu-
ing ventricular systole. If the valve allows back-
matic heart disease most typically presents as
flow during ventricular systole, the increased
mitral stenosis, although aortic stenosis is the
ventricular pressures would be transmitted
next most common presentation.
back into the right atrium and the jugular
vein. The downward movement of the ventri- Answer E is incorrect. Right atrial pressure
cle causes the x descent during ventricular sys- would increase in response to pulmonic valve
tole. This would also be replaced by a positive stenosis or tricuspid valve stenosis. The tricus-
deflection from blood regurgitating into the pid valve is sometimes affected in rheumatic

Cardiovascular
atria during ventricular systole. heart disease and would also cause a diastolic
murmur. However, this is far less common
Answer B is incorrect. These points are not
than mitral stenosis in the setting of rheumatic
the most likely to be affected in tricuspid re-
heart disease. In fact, the pulmonic valve is the
gurgitation.
least likely to be affected in rheumatic heart
Answer C is incorrect. These points are not disease.
the most likely to be affected in tricuspid re-
gurgitation. 20. The correct answer is B. The inferior mesen-
teric artery (IMA) originates from the aorta in-
Answer D is incorrect. These points are not
ferior to the renal arteries and superior to the
the most likely to be affected in tricuspid re-
bifurcation of the aorta into the common iliac
gurgitation. The V wave is increased pressure
arteries. The IMA supplies blood to the distal
because of right atrial filling against a closed
one-third of the transverse colon, descending
tricuspid valve. With tricuspid regurgitation,
and sigmoid colons, and the upper portion of
there is little effect on the V wave itself, but
the rectum. The IMA may sometimes be sacri-
rather it becomes the end point for the new
ficed during an infrarenal aortic aneurysm re-
CV-wave change.
pair rather than being reattached to a healthy
19. The correct answer is C. The patient’s history segment of aorta. Usually there is enough col-
of a prior illness with features of fever, pleu- lateral flow to the hindgut from the superior
ritic chest pain, joint pain, and rash (probably mesenteric artery and the hypogastric arteries
that the loss of the IMA does not result in co-
182 Section II: Organ Systems  •  Answers

lonic ischemia. However, ischemia of the sig- Answer B is incorrect. Congenital medial
moid colon occurs in 1%-7% of repairs, and weakness is actually associated with the devel-
should be considered if bloody diarrhea or an opment of berry aneurysms, which typically
increased WBC count occurs postoperatively. occur along the circle of Willis. They are the
most frequent cause of subarachnoid hemor-
Answer A is incorrect. The ascending colon
rhage and are also associated with adult poly-
receives its blood supply from the superior
cystic kidney disease.
mesenteric artery. This vessel is located supe-
rior to the renal arteries and thus would not Answer C is incorrect. Cystic medial necrosis
be disrupted during resection of the infrarenal (cystic degeneration of the tunica media of the
aorta. aorta) is the most frequent pre-existing histo-
logic lesion in aortic dissection. It is associated
Answer C is incorrect. The small intestine, as
with dilation of the ascending aorta, particu-
High-Yield Systems

well as the distal duodenum and proximal two-


larly in relation to Marfan syndrome. Although
thirds of the transverse colon, receive blood
this answer choice is possible, the skin lesions
supply from the superior mesenteric artery,
point to tertiary syphilis as the most likely
which branches off the aorta above the level
cause of the cardiac pathology.
of the renal arteries. Thus blood flow to these
structures would not be affected. Answer E is incorrect. Hypertension is often
implicated in the etiology of dissecting aneu-
Answer D is incorrect. The spleen receives its
rysms due to a longitudinal intraluminal tear.
blood supply from the splenic artery. This ves-
Dissection is usually not associated with aortic
sel is a branch of the celiac trunk that emerges
dilation.
from the aorta superior to the renal arteries.
Thus blood flow to the spleen would not be af-
22. The correct answer is B. This infant is pre-
fected.
senting with a patent ductus arteriosus (PDA),
Answer E is incorrect. The stomach receives a left-to-right shunt that rarely causes cyano-
its blood supply from various branches of the sis. PDA is associated with maternal rubella
celiac trunk, which emerges from the aorta infection during pregnancy. During fetal
above the level of the renal arteries, and would development, the ductus arteriosus remains
Cardiovascular

thus not be affected. patent through the action of prostaglandin


E2 (PGE2). PDA at birth is closed with indo-
21. The correct answer is D. Syphilitic aortitis is methacin, a nonsteroidal anti-inflammatory
characterized by obliterative endarteritis of the drug (NSAID) that inhibits PGE2 formation.
vasa vasorum of the tunica media. This disrup- Remember, there are, in general, three con-
tion of the vasa vasorum can lead to an aneu- genital heart lesions that cause late cyanosis as
rysm, typically involving the ascending aorta, a result of left-to-right shunt: VSD, atrial septal
and is a manifestation of the tertiary stage of defect (ASD), and PDA. The classic murmur
the disease. Syphilitic aneurysms are often heard with PDA is a continuous, machine-like
complicated by atherosclerosis. The patient’s murmur.
skin lesions are likely “gummas” of tertiary
Answer A is incorrect. Thromboxane A2
syphilis, which appear as fibrous nodules on
causes platelet aggregation and vasoconstric-
the skin surface. Less commonly, these lesions
tion. Indomethacin would decrease thrombox-
can be ulcerative.
ane formation by inhibiting cyclooxygenase-1
Answer A is incorrect. Atherosclerosis is most and -2 enzymes.
frequently associated with a descending aortic
Answer C is incorrect. M2-receptors are
aneurysm, especially one involving the abdom-
G-protein-linked and are responsible for lower-
inal aorta, and is rarely associated with ascend-
ing both heart rate and heart contractility.
ing aortic aneurysms in the absence of under-
lying pathology, such as that of tertiary syphilis.
Chapter 8: Cardiovascular  •  Answers 183

Answer D is incorrect. Prostaglandin E2-­ Answer D is incorrect. Gemfibrozil inhibits li-


receptor activation increases body temperature. polysis.
Answer E is incorrect. Prostaglandin F2a-­
25. The correct answer is A. This man has suf-
receptor activation causes uterine contractions
fered a MI. He demonstrates two of the five
and bronchoconstriction.
important risk factors for developing heart dis-
ease, which include hypertension, hyperlipid-
23. The correct answer is A. Adenosine is ex-
emia, tobacco use, diabetes, and a family his-
tremely useful in abolishing atrioventricular
tory of heart disease. The changes that occur
(AV) nodal arrhythmias when given in high-
in the affected cardiac tissue can be helpful
dose intravenous (IV) boluses. Adenosine
in assessing when the infarct occurred. Dur-
works by hyperpolarizing AV node tissue by
ing the first day after an MI, the affected tissue
increasing the conductance of potassium and

High-Yield Systems
begins to undergo coagulative necrosis and
by reducing calcium current. As a result, the
releases enzymes such as troponin I and CK-
conduction through the AV node is markedly
MB from the dying cells. Coagulative necrosis
reduced. In addition to this, adenosine’s ex-
is marked in the early stages by preservation of
tremely short duration of action (15 seconds)
general tissue architecture, with myocytes be-
limits the occurrence of its toxicities (ie, hypo-
coming increasingly eosinophilic. Contraction
tension, flushing, chest pain, and dyspnea).
bands will also be seen, causing myocytes to
Answer B is incorrect. Bretylium, a potassium take on a wavy appearance.
channel blocker (class III), is used when other
Answer B is incorrect. The presence of granu-
antiarrhythmics fail.
lation tissue indicates that remodeling of dam-
Answer C is incorrect. Encainide is used aged tissue is occurring. It is characterized by
when ventricular tachycardia progresses to the presence of fibroblasts and vascular prolif-
ventricular fibrillation; it is also used in intrac- eration. This would first be seen, along with a
table supraventricular tachycardia. mixed inflammatory picture, beginning 5-10
Answer D is incorrect. Lidocaine, a class Ib days after a MI.
antiarrhythmic, is used in the treatment of Answer C is incorrect. Macrophages are seen

Cardiovascular
acute ventricular arrhythmias such as post-MI in the development of chronic inflammation.
arrhythmias. Five to ten days after an MI, macrophages
Answer E is incorrect. Sotalol, which is both come to the scene to aid in cleaning up areas
a b-adrenergic-receptor blocker (class II) and a of dead tissue. It is important to remember
potassium channel blocker (class III), is used that it usually takes time for macrophages and
when other antiarrhythmics fail. other signs of chronic inflammation to appear
in any setting.
24. The correct answer is E. Gemfibrozil is a fi- Answer D is incorrect. Neutrophils are the
brate that stimulates lipoprotein lipase by ac- hallmark of an acute inflammatory process. A
tivation of the PPARa protein. In so doing, it massive influx of neutrophils, along with ex-
has beneficial effects on the serum lipid pro- tensive coagulative necrosis, begins at 12-24
file. Adverse effects of fibrates include eleva- hours and continues until about four days af-
tion of liver enzymes and myositis. ter an MI. Although neutrophils are generally
Answer A is incorrect. Gemfibrozil speeds the considered the first responders in any inflam-
conversion of VLDL to LDL cholesterol. matory process, they appear after the start of
coagulative necrosis in the evolution of an MI.
Answer B is incorrect. Gemfibrozil has no ef-
fect on triglyceride receptors. Answer E is incorrect. In the first 2-4 hours
following an MI, no change can be de-
Answer C is incorrect. Gemfibrozil promotes tected using light microscopy. However, by
PPAR-a activation.
184 Section II: Organ Systems  •  Answers

eight hours changes associated with coagula- are stiff. In addition, in some cases a third
tive necrosis would certainly be visible. heart sound (S3) may be heard shortly after
point A at the beginning of diastole. S3 is due
26. The correct answer is B. Point A corresponds to the vibration of the distended ventricular
to the opening of the mitral valve at the be- wall during rapid filling and is usually soft and
ginning of diastole, and the line from A to C low in frequency. While the presence of an S3
shows the increase in ventricular volume dur- is normal in children, in adults it usually sug-
ing diastole. Point C marks the beginning of gests volume overload, such as occurs in CHF.
systole as left ventricular pressure becomes
Answer C is incorrect. Point C corresponds to
greater than left atrial pressure, causing the
S1, which is heard normally when the mitral
mitral valve to close. This closure (in conjunc-
and tricuspid valves close at the end of dias-
tion with the closure of the tricuspid valve)
tole. The S4 sound would be heard just before
High-Yield Systems

represents S1. The line from point C to D cor-


this at point B.
responds to isovolumetric contraction, dur-
ing which both the mitral and aortic valves Answer D is incorrect. Point D represents the
remain closed as the left ventricular pressure opening of the aortic valve. While this nor-
increases. At point D the left ventricular pres- mally creates no audible sound on ausculta-
sure becomes greater than the aortic pressure tion, there may be an ejection click at this
and the aortic valve opens. Between points D point in some cases of aortic stenosis.
and F the left ventricular pressure continues
Answer E is incorrect. Point E may corre-
to increase as the ventricle continues to con-
spond to an audible ejection murmur in cases
tract and blood is ejected from the left ventri-
of aortic stenosis. However, this is not the point
cle into the aorta. At point F the aortic valve
in the cardiac cycle when one expects to hear
closes when the left ventricle begins to relax
an S4 heart sound.
and the left ventricular pressure becomes less
than aortic pressure. This closure (in conjunc- Answer F is incorrect. Point F represents the
tion with the closure of the pulmonic valve) sound of the aortic valve closing when the left
represents S2. The line from point F to point ventricle begins to relax and the left ventricu-
A represents the isovolumetric relaxation at the lar pressure becomes less than aortic pressure.
Cardiovascular

end of ventricular systole. When the left ven- The closure of the aortic valve (in conjunction
tricular pressure becomes less than the pres- with the closure of the pulmonic valve) can
sure in the left atrium, the mitral valve opens, be heard on auscultation as the second heart
thus beginning a new loop of the cardiac cycle sound (S2).
(diastole plus systole). Point B corresponds
to the point near the end of diastole when S4 27. The correct answer is B. The patient is expe-
may be heard. An S4, commonly called the riencing pericarditis due to uremia second-
“atrial kick,” is not normally present in adults. ary to chronic kidney disease in the setting of
Its presence suggests a decrease in ventricular long-standing diabetes mellitus (DM). Pericar-
compliance, such as occurs in ventricular hy- ditis presents with pleuritic, positional chest
pertrophy resulting from chronic hypertension. pain that is often relieved by sitting forward
S4 is thought to result from vibration of a stiff, and with a pericardial friction rub on physical
noncompliant ventricular wall as blood is rap- examination. Diffuse ST segment elevations
idly ejected into the ventricle from the atrium. may be found on ECG, while an echocardio-
gram may be normal unless an effusion is also
Answer A is incorrect. Point A represents the present. Pericarditis has multiple etiologies,
opening of the mitral valve at the beginning of including viral (coxsackie virus, echovirus, ad-
diastole, not an S4 heart sound. Normally no enovirus, and HIV), bacterial (tuberculosis or
sound is heard when the mitral valve opens. Streptococcus pneumoniae or Staphylococcus
However, in cases of mitral stenosis, an open- aureus in the setting of endocarditis, pneumo-
ing click may be audible if the valve leaflets nia, or post-cardiac surgery), neoplastic, auto-
Chapter 8: Cardiovascular  •  Answers 185

immune, uremic, cardiovascular, or idiopathic. There are four general classes of antiarrhyth-
Treatment of pericarditis secondary to uremia mics: class I blocks sodium channels, class II
is dialysis. blocks b-adrenergic receptors, class III blocks
potassium channels, and class IV blocks cal-
Answer A is incorrect. Cardiac catheteriza-
cium channels. Epinephrine, amiodarone
tion is indicated in patients who are experi-
(class IA and class III properties), or lidocaine
encing acute coronary syndrome. While this
(class IB) are the agents indicated after a round
patient is certainly at risk for ischemic heart
of unsuccessful defibrillation. Of these agents,
disease given her age and history of diabetes,
only amiodarone may produce the adverse ef-
her symptoms and ECG findings of diffuse ST
fect of pulmonary fibrosis. Amiodarone is also
segment elevations are more indicative of peri-
associated with hypotension, thyroid dysfunc-
carditis. In contrast, acute myocardial ischemia
tion (both hypo- and hyperthyroidism), hepa-
is more likely to present with chest pain that is

High-Yield Systems
totoxicity, ocular changes, and other arrhyth-
not relieved by changes in position and ECG
mias (namely, bradyarrhythmias and torsades
findings that show ST segment elevations in
de pointes). In patients taking amiodarone,
contiguous leads only.
remember to check pulmonary function tests,
Answer C is incorrect. NSAIDs are the ap- liver function tests, and thyroid function tests.
propriate treatment for viral or idiopathic peri-
Answer A is incorrect. There is no increased
carditis. However, this patient is experienc-
risk of bleeding associated with any of the an-
ing pericarditis due to uremia secondary to
tiarrhythmic drugs. Warfarin and heparin are
chronic kidney disease in the setting of long-
commonly used drugs that can cause bleeding.
standing DM, and therefore the most effective
However, they would not be used during ven-
treatment for her is dialysis.
tricular fibrillation.
Answer D is incorrect. Pericardiocentesis
Answer B is incorrect. Class IC agents (ie, fle-
would be indicated if this patient had evidence
cainide and encainide) are contraindicated in
of a pericardial effusion, such as distant heart
the post-MI population, due to their associa-
sounds on physical examination, electrical al-
tion with increased post-MI mortality. These
ternans on ECG, cardiomegaly on x-ray of the
drugs work at phase 3 of the action potential

Cardiovascular
chest, or fluid in the pericardial space on echo-
and have no effect on action potential dura-
cardiogram. However, this patient does not
tion. They are most often used in cases of ven-
have any of these findings, and therefore peri-
tricular tachycardia that progress to ventricu-
cardiocentesis is not indicated.
lar fibrillation, or intractable supraventricular
Answer E is incorrect. Changing the patient’s tachycardia.
antibiotic regimen is indicated if she is sus-
Answer C is incorrect. Procainamide, a type
pected of having an allergic reaction (such as
1A antiarrhythmic drug, is a common cause of
the development of a rash or anaphylaxis) to
drug-induced lupus, but it is not used to treat
a particular medication. However, in this case
a patient in ventricular fibrillation. It acts by
her symptoms are not consistent with an aller-
increasing action potential duration and in-
gic reaction, and therefore her antibiotic regi-
creasing QT interval. It is most often used for
men does not need to be changed.
treatment of re-entrant and ectopic supraven-
28. The correct answer is D. Ventricular fibrilla- tricular and ventricular tachycardia. Other
tion is an irregular ventricular rhythm without drugs in this class include quinidine (which
any distinct QRS complexes, ST segments, or has the adverse effect of cinchonism), amioda-
T waves. This is an important cause of sud- rone (which is also a class III antiarrhythmic),
den cardiac death, as well as mortality within and disopyramide.
the first 24 hours of an acute MI. If defibrilla- Answer E is incorrect. Digoxin toxicity can
tion fails to convert to sinus rhythm, the next lead to disturbances in color vision, including
treatment choice is the use of antiarrhythmics. yellow-green vision, as well as nausea, vomit-
186 Section II: Organ Systems  •  Answers

ing, diarrhea, and arrhythmias. Digoxin di- gram-negative bacilli (Haemophilus aphrophi-
rectly inhibits Na+/K+-ATPase, leading to in- lus, Actinobacillus actinomycetemcomitans,
direct inhibition of the Na+/Ca2+ exchanger. Cardiobacterium hominis, Eikenella corrodens,
Therefore there is an increase in intracellular and Kingella kingae) that cause 5%-10% of
calcium, leading to positive inotropy. This cases of bacterial endocarditis that are not re-
drug is most often used for chronic heart fail- lated to IV drug use. These organisms are slow
ure and control of atrial fibrillation, not for growing and difficult to culture from blood
ventricular fibrillation. samples, making diagnosis more complex.
Answer D is incorrect. Streptococcus bovis also
29. The correct answer is C. This is a classic case
causes subacute bacterial endocarditis, which
of acute bacterial endocarditis (ABE). Endo-
presents with low-grade fever and insidious on-
carditis often is characterized by constitutional
set. It normally inhabits the lower GI tract, and
High-Yield Systems

symptoms (fever, malaise, chills), new-onset


lesions in the colon, such as those that occur
cardiac murmur, and a combination of other
in colon cancer, allow the bacteria access to
signs and symptoms (eg, Janeway lesions, Os-
the bloodstream. It most commonly affects the
ler’s nodes, Roth’s spots). Acute and subacute
aortic valve.
endocarditis can be differentiated based on
history, because the acute case will have the Answer E is incorrect. Streptococcus viridans
most severe and sudden onset, as in this pa- is the most common cause of bacterial en-
tient. ABE also is seen most often in cases of docarditis overall. This group of bacteria is
IV drug use and indwelling catheters, and seen most often in subacute cases in which
Staphylococcus aureus is the most common the onset of symptoms usually is chronic and
bacterial pathogen isolated in these cases, be- low-grade fevers are common. Streptococ-
cause it is part of the skin flora and enters the cus viridans commonly colonizes heart valves
blood at needle sites. This patient’s history previously damaged by rheumatic fever, thus
of IV drug abuse, as well as auscultation of a causing left-sided infective endocarditis as op-
murmur consistent with tricuspid regurgita- posed to the right-sided version seen more
tion, point to a right-sided ABE infection. In commonly with Staphylococcus aureus. One
right-sided endocarditis, one more often sees common source of infection is dental proce-
Cardiovascular

septic emboli to the lungs, leading to bilateral dures during which normal flora of the oro-
infiltrates. This patient is manifesting signs of pharynx can enter the bloodstream.
bilateral infiltrates with hypoxia, decreased
breath sounds, and dullness to percussion. It 30. The correct answer is B. Angiotensin II recep-
is important to note that many of the classic tor blockers such as losartan have similar blood
signs of endocarditis, such as Janeway lesions, pressure-lowering properties as ACE inhibi-
Osler’s nodes, and Roth’s spots, are seen mostly tors, but have fewer adverse effects. Adverse ef-
as a complication of left-sided endocarditis, in fects of angiotensin II receptors include hyper-
which septic emboli leave the heart and enter kalemia and fetal renal toxicity.
the systemic circulation. Answer A is incorrect. Blockade of b1 recep-
Answer A is incorrect. Enterococcus faecalis tors is the mechanism of action of acebutolol,
also causes subacute endocarditis. The classic betaxolol, esmolol, atenolol, and metoprolol.
picture is a slow onset of constitutional symp- These drugs are used to treat hypertension,
toms with low-grade fever. Enterococcus infec- in addition to angina, supraventricular tachy-
tion is not seen as frequently as Streptococcus cardia, CHF, and glaucoma. These drugs also
viridans, but it is known to colonize damaged decrease post-MI mortality. Common toxicities
heart valves, especially in patients with a his- include impotence, exacerbation of asthma,
tory of rheumatic fever. sedation, bradycardia, and atrioventricular
block.
Answer B is incorrect. Haemophilus aphrophi-
lus is part of the HACEK group of fastidious
Chapter 8: Cardiovascular  •  Answers 187

Answer C is incorrect. ACE inhibitors, such tolic blood pressure drops by >10 mm Hg on
as captopril and enalapril, are antihypertensive inspiration. ECG is often low voltage and may
drugs commonly prescribed to diabetics due to reveal electrical alternans. Because patients in
their possible renal-sparing properties. How- cardiac tamponade are in a low-output state,
ever, these drugs have common adverse effects they are preload dependent and require imme-
such as hyperkalemia, cough, angioedema, diate volume resuscitation to maintain cardiac
taste changes, hypotension, and rash. In addi- output. Positive inotropes such as dobutamine
tion, ACE inhibitors are teratogenic, causing and pericardiocentesis would also be indicated
fetal renal problems. This patient was taking in this patient after he has begun receiving IV
an ACE inhibitor before. The adverse effects hydration.
of cough and angioedema prompted her physi-
Answer A is incorrect. Diltiazem is a calcium
cian to switch her to an angiotensin II receptor
channel blocker that has a negative inotropic

High-Yield Systems
blocker.
effect on the heart. In the setting of cardiac
Answer D is incorrect. Nifedipine, verapamil, tamponade, a negative inotrope like diltiazem
and diltiazem are drugs that act through in- is contraindicated because it would decrease
hibition of calcium channels in cardiac and his already low cardiac output and therefore
smooth muscle. They are commonly used worsen his hypotension and shock.
in the treatment of hypertension, angina,
Answer B is incorrect. Metoprolol is a selec-
Prinzmetal’s angina, and Raynaud syndrome.
tive b1-blocker that has negative inotropic ef-
Verapamil and diltiazem are also used as anti-
fects on the heart. In the setting of cardiac
arrhythmics.
tamponade, a negative inotrope like meto-
Answer E is incorrect. This is the mechanism prolol is contraindicated because it would
of action of thiazide diuretics such as hydro- decrease his already low cardiac output and
chlorothiazide, which are commonly used an- therefore worsen his hypotension and shock.
tihypertensive agents. Adverse effects include
Answer C is incorrect. Because patients in
hypokalemic metabolic alkalosis, hyponatre-
cardiac tamponade are in a low-output state
mia, hyperglycemia, hyperlipidemia, hyperuri-
due to the compression of the heart by the sur-
cemia, hypercalcemia, and allergic reactions.

Cardiovascular
rounding fluid within the pericardial sac, their
cardiac output is preload dependent. Any in-
31. The correct answer is D. This patient is suffer-
tervention that decreases his preload would be
ing from cardiogenic shock due to pericardial
contraindicated in this setting because it would
tamponade secondary to his small cell lung
lead to decreased cardiac output and worsen-
cancer. Cardiac tamponade can occur sec-
ing hypotension and shock; therefore, diuresis
ondary to trauma, hypothyroidism, myocardial
is not indicated in this patient.
rupture, or as a complication of pericarditis
(especially in the setting of malignancy or ure- Answer E is incorrect. In the setting of cardiac
mia). Specifically, cardiac tamponade results tamponade, surgery is indicated only if fluid
when the pericardial space fills with enough has reaccumulated after catheter drainage, the
fluid to cause increased intrapericardial pres- effusion is loculated, there is a special need for
sure, compression of the heart throughout its biopsy material, or the patient has a coagulopa-
cycle, and subsequent decreased diastolic fill- thy. Moreover, general anesthesia is usually re-
ing of the heart. As a result of the decreased quired, and may be unsafe if needle drainage
preload, stroke volume falls and cardiogenic is not performed first to reduce the severity of
shock (in the absence of pulmonary edema) the tamponade. Therefore, surgery is not the
results. Classic physical examination findings most appropriate next step in the management
in cardiac tamponade include Beck’s triad of of this patient.
distant heart sounds, increased jugular venous
pressure, and hypotension. Pulsus paradoxus 32. The correct answer is A. This patient presents
may also be seen, which occurs when the sys- with symptoms consistent with Lyme disease.
188 Section II: Organ Systems  •  Answers

He had a characteristic expanding rash (ery- Answer A is incorrect. VSD causes a harsh
thema migrans) and resolving flu-like symp- holosystolic murmur heard best over the tricus-
toms. Lyme disease can often lead to cardiac pid area. VSD is a relatively common congeni-
symptoms such as those described, as well tal cardiac anomaly; however, this murmur
as heart block that can require cardiac pac- would not be heard in heart failure, which is
ing. Lyme disease is carried by the Ixodes tick. the most likely disease process occurring in
I scapularis is also the vector of disease for ba- this case.
besiosis, a malaria-like parasitic disease com-
Answer B is incorrect. In the absence of dis-
mon in the northeastern corner of the United
ease, the sounds made by the closing of the
States.
aortic and pulmonic valves (S2) occur simul-
Answer B is incorrect. Epidemic typhus is taneously during expiration, but are split dur-
known as “louse-borne typhus.” The vector for ing inspiration as the decrease in intrathoracic
High-Yield Systems

transmission is Pediculus corporis. Epidemic ty- pressure causes a delay in the closing of the
phus is unusual because the vector for disease pulmonic valve. Paradoxical splitting occurs
feeds only on humans and not other animals. in cases of aortic stenosis or left bundle branch
The bacterium responsible is Rickettsia prowa- block, when the closing of the aortic valve is
zekii. delayed and thus the pulmonic valve closes
before the aortic valve on expiration, but the
Answer C is incorrect. Malaria is a protozoan
delayed closure of the pulmonic valve on in-
parasitic disease responsible for one-three mil-
spiration causes the sounds to be simultaneous
lion deaths per year worldwide. Its vector of
on inspiration. This patient does not have any
transmission (and target for disease control) is
signs or symptoms of aortic stenosis or a left
the female Anopheles mosquito.
bundle branch block, but she does have signs
Answer D is incorrect. Plague is an infectious of volume overload due to CHF. Thus she
disease caused by the bacterium Yersinia pes- would likely have an abnormal S3, not a para-
tis. It is mainly transmitted by fleas that live on doxically split S2.
infected rodents such as the oriental rat flea,
Answer C is incorrect. A pulmonary flow mur-
Xenopsylla cheopis.
mur is a systolic murmur heard best over the
Cardiovascular

Answer E is incorrect. Rocky Mountain spot- pulmonic area, associated with increased flow
ted fever is caused by Rickettsia rickettsii, a spe- across the pulmonary valve. This occurs in
cies of bacteria spread to humans by the ticks conditions such as an ASD, in which blood
of the Dermacentor family such as D variabilis. from the left heart flows to the right heart,
thus increasing the volume of blood that flows
33. The correct answer is D. This woman has through the pulmonic valve. ASD can also
CHF, which is causing pulmonary and sys- cause a diastolic rumble due to increased flow
temic edema due to elevated venous pressures over the tricuspid valve. However, an ASD
(“backward failure”) and fatigue due to an in- murmur would not be heard in this clear case
sufficient cardiac output to meet the metabolic of heart failure.
demands of the body (“forward failure”). In an
attempt to compensate for the decreased car- Answer E is incorrect. The fourth heart sound
diac output, the heart operates at higher end- (S4) occurs in late diastole and coincides with
diastolic and end-systolic volumes, which often atrial contraction in cases in which the atrium
produces a third heart sound (S3), most likely contracts against a stiffened ventricle. An S4 is
due to the increased tension of the chordae not present in normal children or adults, and
tendinae during the rapid filling phase of early suggests a decrease in ventricular compliance,
ventricular diastole. While the presence of an as is seen in the ventricular hypertrophy that
S3 is normal in children, in adults it is often a develops in chronic hypertension. This pa-
sign of volume overload, as in CHF. tient has signs of volume overload and would
be more likely to present with an S3 due to
Chapter 8: Cardiovascular  •  Answers 189

systolic dysfunction than an S4 due to diastolic Answer E is incorrect. Lactate dehydrogenase


dysfunction. (line D) is the last cardiac enzyme to become
significantly elevated, on approximately day
Answer F is incorrect. In the absence of dis-
two post-MI, and it remains elevated for up to
ease, the sounds made by the closing of the
seven days.
aortic and pulmonic valves (S2) occur simul-
taneously during expiration, but are split dur- Answer F is incorrect. Myoglobin typically
ing inspiration, as the decrease in intrathoracic rises and falls within 6 hours post-MI. It, like
pressure causes a delay in the closing of the AST, is nonspecific, as it is also found in skel-
pulmonic valve. In cases of pulmonic valve ste- etal muscle throughout the body. It is not typi-
nosis or right bundle branch block, there may cally used clinically in the diagnosis of MI and
be an increased delay in the closure of the pul- is also not represented in the image.
monic valve, causing an accentuation of the

High-Yield Systems
normal splitting of S2 during inspiration. S2 35. The correct answer is D. This infant has a pat-
may be audibly split during expiration as well, ent ductus arteriosus (PDA). The sixth aortic
as the pulmonic valve closes after the aortic arch gives rise to the proximal pulmonary ar-
valve, regardless of respiratory cycle. teries and, on the left side, to the ductus arte-
riosus. In the fetus the ductus arteriosus con-
34. The correct answer is G. Cardiac troponin I nects the pulmonary trunk to the aorta and
levels (line A) become elevated in the first allows blood from the right ventricle to bypass
four hours after an MI and remain elevated for the lungs (which do not function at this time),
7-10 days. It is the most specific protein marker enter the aorta, and return to the umbilical ar-
for MI. However, creatine kinase (CK)-MB is teries. At birth, the increase in oxygen with the
the enzyme of choice for the detection of re- infant’s first breath results in decreased prosta-
infarction within the first week. If re-infarction glandin levels, which allows the ductus arterio-
occurs, CK-MB levels would again increase, sus to close. Closure is assisted by increased ox-
whereas troponin levels remain elevated from ygen stimulating the opening of the pulmonary
the previous event. vessels, which decreases vascular resistance,
thus leading to increased blood flow to the
Answer A is incorrect. AST (line C) is the

Cardiovascular
lungs. Failure of the ductus arteriosus to close
third enzyme to become elevated, as it gradu-
completely results in a PDA. This condition
ally increases over the first two days, then
is almost always present in premature infants
slowly declines. The serum level of this en-
with low surfactant production and low oxy-
zyme is not specific for cardiac cell damage.
gen levels. Indomethacin, a NSAID, inhibits
Answer B is incorrect. Brain natriuretic pep- prostaglandins and is frequently used to close a
tide becomes elevated as the atria are stretched PDA in neonates.
chronically as a result of volume overload, as
Answer A is incorrect. The first aortic arch
occurs in CHF. It is not represented in the im-
contributes part of the maxillary artery.
age, as it is not acutely elevated in MI.
Answer B is incorrect. The fourth aortic arch
Answer C is incorrect. C-reactive protein is a
gives rise to the aortic arch on the left and the
marker of inflammation. It has recently been
proximal right subclavian artery on the right.
shown in the JUPITER trial as possibly valu-
able as beginning statin use to lower cardiac Answer C is incorrect. The second aortic arch
risk in a person with a normal LDL cholesterol produces the stapedial artery and the hyoid ar-
level. It is not represented in the image, as it is tery.
not used to diagnose MI.
Answer E is incorrect. The third aortic arch
Answer D is incorrect. CK-MB (line B) levels gives rise to the common carotid artery and the
peak in the first 24 hours and then decrease. proximal part of the internal carotid artery.
190 Section II: Organ Systems  •  Answers

36. The correct answer is C. This patient is most the axilla usually involve pathology of the mi-
likely presenting with symptomatic aortic tral valve.
stenosis. One of the most common presenta-
Answer D is incorrect. An early diastolic de-
tions of this condition is new-onset syncope in
crescendo murmur radiating to the apex would
an older adult during an episode of exertion.
be associated with aortic insufficiency (also
This results from the inability to increase car-
known as aortic regurgitation). This condition
diac output during exertion due to a stenotic
can result in dyspnea, but generally is not as-
(usually calcified) valve. In addition, the ste-
sociated with syncope unless there is associ-
notic valve also causes a pressure build-up on
ated severe left ventricular dysfunction. Aortic
the left side of the heart, resulting in pulmo-
regurgitation will also present with a widened
nary congestion, as suggested by the bilateral
pulse pressure.
crackles in this patient. The classic heart mur-
High-Yield Systems

mur of aortic stenosis is a harsh crescendo-­ Answer E is incorrect. An early diastolic de-
decrescendo systolic murmur usually heard crescendo murmur could be associated with
best along the right upper sternal border, aortic insufficiency; however, the murmur of
which radiates to the carotids or the apex. aortic insufficiency would not radiate to the ax-
Other findings on cardiovascular examination illa. A murmur of this nature in the axilla may
that suggest aortic stenosis include a weak and be suggestive of mitral regurgitation; however,
delayed carotid pulse (pulsus parvus et tardus), this patient’s symptoms do not match with this
soft or absent A2 component of S2, displaced condition.
point of maximal impulse (PMI) with left ven- Answer F is incorrect. Mitral stenosis is associ-
tricular hypertrophy, and later left ventricular ated with an opening snap followed by a mid-
dysfunction (which results in a wide and dis- diastolic rumble. Mitral stenosis most com-
placed PMI). monly presents with dyspnea, but is generally
Answer A is incorrect. Mitral regurgitation is not associated with syncope, and does not pro-
associated with a blowing holosystolic murmur duce a weak and delayed carotid pulse. Mitral
best heard along the apex. When moderate to valve stenosis is often associated with a history
severe, this condition can result in dyspnea, of rheumatic fever.
Cardiovascular

but it generally is not associated with syncope,


unless there is associated severe left ventricu- 37. The correct answer is E. The ECG find-
lar dysfunction. This condition generally is ings are consistent with MI and a second-
not associated with a weak and delayed carotid degree Mobitz type 1 atrioventricular (AV)
pulse. Mitral regurgitation is often caused by block or Wenckebach block. AV block oc-
myxomatous degeneration of the valve, ische­ curs in 12%-25% of patients with acute MI;
mic heart disease, infective endocarditis, or first-degree AV block occurs in 2%-12%, sec-
collagen vascular disease. ond-degree AV block occurs in 3%-10%, and
third-degree AV block occurs in 3%-7%. Sec-
Answer B is incorrect. While a crescendo- ond-degree type I AV block occurs more com-
decrescendo systolic murmur certainly fits with monly in inferior than anterior MIs. The right
this patient’s likely aortic stenosis, the murmur coronary artery (RCA) supplies the inferior wall
would not radiate to the axilla. Cardiac mur- of the heart. It also supplies the sinoatrial and
murs radiate in the direction of flow that is AV nodes. The sinus node receives blood sup-
being affected. Flow through the aortic valve ply from the right coronary artery in 59% of pa-
goes from the apex of the heart (the left ventri- tients, from the left circumflex artery in 38%,
cle) up through the aortic valve into the aortic and from both arteries with a dual blood supply
arch and carotid arteries. Thus on auscultation in 3%. The AV node is supplied by the RCA in
you would expect to hear the murmur radiat- 90% of patients. Thus the AV block is likely due
ing through the carotids and to the apex of the to injury to the AV node as a result of an occlu-
heart. Murmurs associated with radiation to sion in the RCA causing ischemia to the right
Chapter 8: Cardiovascular  •  Answers 191

posterior atrium, where the AV nodal cells are ease is the development of coronary artery an-
located. eurysms, which can result in MI. IV immuno-
globulin and aspirin is the appropriate therapy.
Answer A is incorrect. The diagonal branches
of the LAD are one or two large branches of Answer A is incorrect. Kawasaki disease is as-
the LAD that descend anteriorly across the sur- sociated with a high fever rather than with hy-
face of the left ventricle. A lesion here will not pothermia.
cause an inferior wall infarct of the heart.
Answer B is incorrect. Kawasaki disease is not
Answer B is incorrect. The left anterior de- associated with optic neuritis. This finding is
scending (LAD) branch of the left coronary more commonly associated with multiple scle-
artery supplies the anterior two-thirds of the rosis.
interventricular septum and the entire ante-
Answer C is incorrect. Kawasaki disease typi-

High-Yield Systems
rior wall of the left ventricle, as it travels along
cally does not involve the kidney, which is in
the anterior interventricular groove toward the
contrast to Henoch-Schönlein purpura, a vas-
apex of the heart. A lesion in the LAD would
culitic disease affecting mostly children.
result in an anterior wall MI. It does not sup-
ply the inferior wall of the heart, as described Answer D is incorrect. Kawasaki disease of-
in the vignette. A large proximal LAD infarc- ten occurs after an upper respiratory infection,
tion may also affect the conduction system, but it is not usually associated with pulmonary
but it typically affects the area below the AV symptoms.
node that courses through the interventricular
septum. This can sometimes result in bundle- 39. The correct answer is F. This woman is likely
branch blocks, higher-degree AV blocks, and suffering from prosthetic valve endocarditis.
even complete heart block. She may not have taken appropriate prophy-
lactic antibiotics before her root canal proce-
Answer C is incorrect. The left circumflex ar- dure, and her susceptible mitral valve after
tery is a branch of the LAD that travels toward rheumatic fever has been exposed to transient
the left side of the heart. A lesion here will bacteremia. Her symptoms, including low-
not cause an infarct on the inferior wall of the grade persistent fever, new-onset murmur,

Cardiovascular
heart. and insidious onset, suggest subacute bacterial
Answer D is incorrect. The left marginal ar- endocarditis. This is further supported by her
tery is a large branch of the left circumflex ar- physical examination, which reveals the pres-
tery. It continues on the left side of the heart ence of Roth spots (retinal hemorrhages), Os-
across its rounded obtuse margin. A lesion ler’s nodes (painful red nodules on digits), and
here does not cause an AV block or an inferior Janeway lesions (dark macules on palms and
wall infarct. soles). Given her clinical history and symp-
toms, the bacterium most likely to have caused
38. The correct answer is E. The child in this this episode is Streptococcus sanguis, part of
question likely has Kawasaki disease, a vascu- the viridans group. The most appropriate treat-
litis of unknown etiology that is hypothesized ment for such an infection is penicillin G.
to be an infectious or autoimmune response
Answer A is incorrect. Acyclovir is a guano-
(ie, molecular mimicry). The major symptoms
sine analogue antiviral drug used to treat her-
include a high fever of more than five days’
pes simplex and herpes zoster. This is a bacte-
duration, bilateral conjunctivitis, lip fissures,
rial endocarditis and not a viral infection.
strawberry tongue, palmar and plantar desqua-
mation, and cervical lymphadenopathy. Minor Answer B is incorrect. Caspofungin is an anti-
criteria may include elevated acute phase reac- fungal used to treat aspergillosis. It would not
tants, leukocytosis, thrombocytosis, and mild treat a gram-positive cocci infection.
elevation of liver function test results. The
most important complication of Kawasaki dis-
192 Section II: Organ Systems  •  Answers

Answer C is incorrect. Clindamycin, the treat- metic actions. It would thus not be indicated
ment for several important anaerobic infec- in this clinical scenario.
tions, works by blocking peptide bond forma-
Answer E is incorrect. Phenylephrine is an
tion at the 50S ribosomal subunit. This is not
a-adrenergic receptor agonist that causes vaso­
an anaerobic infection.
constriction and rapid increases in total pe-
Answer D is incorrect. Mebendazole is an ripheral resistance. This dramatic increase in
antiparasitic drug used to treat roundworm afterload would be very dangerous for this pa-
infections such as pinworm and whipworm. tient.
Mebendazole is not used to treat bacterial en-
docarditis. 41. The correct answer is E. Nitrates are first-line
treatment in the management of angina. They
Answer E is incorrect. Metronidazole is a
act by stimulating the release of nitric oxide in
High-Yield Systems

bactericidal agent used to treat protozoal in-


smooth muscle, causing an increase in cGMP
fections, specifically Giardia, Entamoeba, and
levels and subsequent smooth muscle relax-
Trichomonas species, as well as anaerobes, spe-
ation, primarily in the venous system. Such
cifically Bacteroides and Clostridium species.
venodilation causes a decrease in preload,
Gram-positive cocci are not within metronida-
which reduces left ventricular wall stress and
zole’s spectrum.
in turn minimizes myocardial oxygen con-
Answer G is incorrect. Pentamidine is an an- sumption.
tiparasitic drug used for prophylaxis against
Answer A is incorrect. Many antiarrhyth-
Pneumocystis jiroveci pneumonia. Pentamidien
mic medications, including calcium channel
is not used to treat bacterial endocarditis.
blockers and adenosine, act by decreasing the
conduction velocity across the atrioventricular
40. The correct answer is D. This patient is expe-
node. However, this is not the mechanism by
riencing a propranolol overdose. Propranolol,
which nitrates work in the treatment of angina.
a b-adrenergic receptor blocker, reduces heart
rate and contractility due to antagonism of Answer B is incorrect. b-Blockers are also
b1-receptors in the sinoatrial node. This low- used to treat angina. They act by decreasing
Cardiovascular

ers cardiac output. The goal of therapy should heart rate and contractility to effectively re-
be restoring myocardial contractility and in- duce myocardial oxygen consumption. How-
creasing cardiac output. Glucagon acts as a ever, this is not the mechanism by which ni-
positive inotropic agent. It increases intracel- trates work in the treatment of angina.
lular cAMP levels independently of adrenergic
Answer C is incorrect. Nitrates act primarily
receptor signaling. Because its mechanism is
by stimulating venodilation to decrease preload
unaffected by adrenergic blockade, glucagon is
and thereby reduce myocardial oxygen con-
the drug of choice in b-blocker toxicity.
sumption. They do not act by increasing the
Answer A is incorrect. Albuterol is primarily oxygen supply to the myocardium.
a b2 agonist and has little positive inotropic ef-
Answer D is incorrect. Angina is caused by
fect.
atherosclerotic stenosis within coronary arter-
Answer B is incorrect. Cocaine causes vaso- ies that limit blood flow through those ves-
constriction and may lead to arrhythmia. It is sels. Coronary arterioles in patients with flow-
not indicated as treatment for b-blocker over- limiting coronary stenosis are already dilated
dose. to maintain resting blood flow. Therefore, any
vasodilating effects nitrates have on coronary
Answer C is incorrect. Although digitalis is
arteries are negligible in the setting of already
also a positive inotropic agent, it commonly
maximally dilated coronary arteries. Thus,
leads to atrioventricular block in the setting of
stimulating the vasodilation of coronary arteri-
bradycardia due to its strong parasympathomi-
Chapter 8: Cardiovascular  •  Answers 193

oles is not the primary mechanism of nitrates Answer E is incorrect. The afferent firing rate
in the treatment of angina. would decrease, not increase, with hypoten-
sion.
42. The correct answer is B. Isovolumetric con-
traction (phase II in the image) is the period 44. The correct answer is B. Dressler syndrome is
between mitral valve closing and aortic valve an autoimmune phenomenon that results in
opening. The cardiac musculature contracts fibrinous pericarditis. This delayed pericardi-
against the closed aortic valve to drastically el- tis typically develops 2-10 weeks post-MI and
evate ventricular pressure. presents clinically as chest pain and a pericar-
dial friction rub. It is generally treated with
Answer A is incorrect. Ventricular filling
nonsteroidal anti-inflammatory agents or corti-
(phase I in the image) is the period between
costeroids.
mitral valve opening and closing. Ventricu-

High-Yield Systems
lar pressure remains roughly equal to atrial Answer A is incorrect. Cardiac arrhythmia is a
pressure, as they are in direct communication common cause of post-MI death, typically oc-
while the mitral valve is open. curring the first few days following the event. It
is not associated with a friction rub.
Answer C is incorrect. Ventricular ejection
(phase III in the image) is the period between Answer C is incorrect. Left ventricular failure
aortic valve opening and closing. Volume falls occurs in 60% of people who suffer an MI and
precipitously as blood rushes into the aorta. can present as CHF, which can cause chest
pain, dyspnea, and an elevated jugular venous
Answer D is incorrect. Isovolumetric relax-
pressure. No friction rub is typically present.
ation (phase IV in the image) is the period in
which both the aortic and mitral valves are Answer D is incorrect. Because ischemic/
closed, thus keeping ventricular volume con- scarred myocardial tissue lacks normal contrac-
stant. This phase ends when the ventricular tility, there is increased blood stasis and forma-
pressure falls below the level of the atrial pres- tion of large mural thrombi. Smaller throm-
sure, and the mitral valve opens to allow fill- boemboli can break off these large mural
ing. thrombi and lead to cerebrovascular accidents,

Cardiovascular
transient ischemic attacks, and renal artery
43. The correct answer is C. The carotid sinus thrombosis. Post-MI arrhythmias are also a pro-
baroreceptor sends an afferent signal via the moter of blood stasis and subsequent thrombo-
glossopharyngeal nerve to the medulla, which embolic events. However, a friction rub does
in turn responds by increasing sympathetic not indicate thromboembolism.
outflow. This results in systemic vasoconstric-
Answer E is incorrect. Ventricular rupture is
tion, increased heart rate, increased contractil-
a serious cause of post-MI death that typically
ity, and increased blood pressure.
occurs 4-10 days after the initial event. It can
Answer A is incorrect. The baroreceptor lo- present with persistent chest pain, syncope,
cated in the aortic arch responds only to an in- and distended jugular veins, but most often
crease in blood pressure. it presents with sudden death. A friction rub
Answer B is incorrect. The correct efferent re- would not be observed.
sponse to a decreased baroreceptor afferent fir-
45. The correct answer is D. This patient experi-
ing rate would be increased sympathetic activ-
enced an aortic dissection, characterized by a
ity and decreased parasympathetic activity.
transverse tear through the intima and internal
Answer D is incorrect. The baroreceptor lo- media of the aortic wall. A blood-filled chan-
cated in the aortic arch responds only to an in- nel subsequently forms within the wall and is
crease in blood pressure. The parasympathetic at great risk of rupture, resulting in massive
nervous system is not activated to correct hypo- hemorrhage (the “pseudolumen” is the darker
tension. of the two lumens in the image). Hyperten-
194 Section II: Organ Systems  •  Answers

sion is the major risk factor contributing to the all automaticity of the conduction system and
damage of the blood vessel. Other risk factors would not result in randomly dropped QRS
include connective tissue diseases (eg, Ehlers- complexes.
Danlos and Marfan syndromes), pregnancy,
trauma, and aortic coarctation. 47. The correct answer is A. The patient died
suddenly a few hours after an acute MI. Fatal
Answer A is incorrect. Cocaine abuse can lead
arrhythmias are the most common cause of
to hypertension, which can later contribute to
death (also known as sudden cardiac death)
aortic dissection. Cocaine abuse would not be
in the first few hours of an infarction. Arrhyth-
the direct cause of the aortic dissection, how-
mias are due to disruption of the vascular sup-
ever.
ply to the conduction system, combined with
Answer B is incorrect. Diabetes is a risk factor myocardial irritability after injury. The patient
High-Yield Systems

for MI but not for aortic dissection. likely suffered from polymorphic ventricular
tachycardia or ventricular fibrillation. Also,
Answer C is incorrect. Hypercholesterolemia
ventricular tachyarrhythmias and intraventric-
contributes more significantly to atherosclero-
ular conduction abnormalities (such as a left
sis and subsequent MI than aortic dissection.
bundle branch block) are common long-term
Answer E is incorrect. Sickle cell disease is complications of an MI after the myocardium
not a direct cause of aortic dissection. becomes scar tissue and loses its intrinsic con-
ducting abilities.
46. The correct answer is B. The ECG findings
describe Mobitz type II second-degree heart Answer B is incorrect. Although less likely
block. A defect in the His-Purkinje system than a ventricular free-wall rupture, the in-
transmitting impulses from the atrioventricular terventricular septum also may rupture three-
(AV) node to the myocardium is usually, but seven days after infarction, leading to a VSD,
not always, responsible for this type of heart a left-to-right intracardiac shunt, and a low car-
block. diac output.

Answer A is incorrect. In contrast to this Answer C is incorrect. After MI, there is a


risk of thrombus formation over the infarcted
Cardiovascular

patient’s findings, AV node abnormalities


lengthen the PR interval and are responsible area of endocardium due to turbulence in the
for first-degree heart block and Mobitz type I blood flow. Clot formation can lead to a left-
second-degree heart block. While AV nodal sided embolism; however, this is not the most
abnormalities can occasionally cause Mobitz common cause death in the acute post-MI set-
type II second-degree heart block, defects in ting. Additionally, left ventricular emboli more
the His-Purkinje system are more commonly likely would lead to ischemic strokes, not sud-
the culprit. den cardiac death.

Answer C is incorrect. Independently con- Answer D is incorrect. MI can lead to fur-


tracting atria and ventricles occur in the com- ther complications such as CHF and cardio-
plete absence or ablation of the His-Purkinje genic shock. However, these complications
system. There is no consistency in the length rarely cause sudden cardiac death in a sta-
of the PR interval. bilized patient in the acute setting. Myocar-
dial failure leading to CHF and poor cardiac
Answer D is incorrect. Retrograde conduc- output may take weeks to months to occur,
tion would result in an increase in the number and is the result of ventricular remodeling.
of P waves and a decrease in the PR interval. Hence b-blockers and ACE inhibitors are
P waves would also show flipping or other ab- used widely in the post-MI period to alter the
normalities. neurohormonal milieu imposed by the renin-
Answer E is incorrect. Sinoatrial node abnor- angiotensin-aldosterone system, and to coun-
malities are responsible for problems in over- teract deleterious ventricular remodeling.
Chapter 8: Cardiovascular  •  Answers 195

Answer E is incorrect. Ventricular free-wall Answer E is incorrect. Increased end-systolic


rupture is a complication that usually occurs volume in the ventricle without a correspond-
three-seven days after infarction because of the ing increase in preload would represent a de-
weakened wall of the damaged area during the creased ejection fraction and therefore a de-
inflammatory cellular reorganization process. creased stroke volume.
The left ventricular free wall is the most likely
site of rupture. Rupture, should it occur, leads 49. The correct answer is B. Cardiac glycosides
to bleeding into the pericardial space and fatal such as digoxin inhibit the Na+-K+-ATPase
cardiac tamponade. transport system to increase intracellular so-
dium concentration, which in turn increases
Answer F is incorrect. A ruptured papillary
intracellular calcium concentration via the
muscle is a possible complication of MI, but
sodium-calcium exchange carrier mechanism.
it most commonly occurs three-seven days af-

High-Yield Systems
This increased intracellular calcium level al-
ter the ischemic event. Papillary muscle rup-
lows greater amounts of calcium to be released
ture is not typically the underlying etiology
to the myofilaments during excitation, result-
of death in the acute setting. Rather, it could
ing in a positive inotropic effect. Increased
cause a low cardiac output and acute pulmo-
contractility of the heart increases stroke vol-
nary edema, likely requiring intubation until
ume, which in turn increases cardiac output.
surgery can be performed to repair the valve.
Glycosides are largely not used today because
of the advent of newer drugs that have fewer
48. The correct answer is A. The width of the P-V
adverse effects. The outstanding exception is
loop represents the stroke volume. Afterload
digoxin, which is still widely used to treat heart
refers to the aortic pressure against which the
failure and atrial fibrillation.
left ventricle pumps. Decreases in afterload
decrease the resistance against which the left Answer A is incorrect. b-Blockers inhibit sym-
ventricle must pump and, therefore, increase pathetic cardiac activation by blocking the ac-
the stroke volume of the cardiac cycle. Other tivity of catecholamines on the heart, decreas-
physiologic changes that increase the stroke ing heart rate and thus cardiac output. These
volume include increased preload and in- agents are specifically used in CHF in order

Cardiovascular
creased contractility. to decrease myocardial energy use and cardiac
oxygen demand.
Answer B is incorrect. Contractility describes
the intrinsic ability of the myocardium to Answer C is incorrect. Increased extracel-
pump against a given resistance. Decreased lular sodium levels in isolation would not in-
contractility in effect weakens the heart, reduc- crease cardiac output. Increasing cardiac out-
ing stroke volume. put would require an increase in intracellular
calcium levels, which is released from the sar-
Answer C is incorrect. Decreased preload is a
coplasmic reticulum. This may be achieved
reduction in the volume of blood that fills the
by the calcium influx triggered by sodium-
ventricle during diastole. Based on the Star-
calcium exchange channels. Thus increasing
ling relationship, in which force of contraction
intracellular sodium concentrations would in-
is proportional to the initial length of cardiac
crease contractility, but increased extracellular
muscle fibers, this decreased ventricular filling
sodium would not.
results in a reduction of stroke volume.
Answer D is incorrect. A decreased intracellu-
Answer D is incorrect. Increased arterial pres-
lar calcium level would decrease the contrac-
sure is synonymous with increased afterload.
tility of the heart, resulting in decreased stroke
Increased afterload results in decreased stroke
volume and thus decreased cardiac output.
volume based on an increase in pressure
against which the left ventricle must pump. Answer E is incorrect. Metabolic acidosis de-
creases contractility and stroke volume; thus
this would decrease cardiac output.
196 Section II: Organ Systems  •  Answers

50. The correct answer is D. Net fluid movement Answer B is incorrect. Decreased capillary
is governed by the equation JV = Kf[(Pc − Pi) pressure would decrease the pressure differen-
−(pc − pi)], where Kf is the filtration coefficient tial driving fluid into the interstitial space.
(factoring in membrane permeability), Pc is
Answer C is incorrect. Increased interstitial
capillary pressure, Pi is interstitial fluid pres-
fluid pressure would lead to greater resistance
sure, pc is capillary oncotic pressure, and pi is
to net fluid flow from the capillaries.
interstitial oncotic pressure. Increasing Pc, pi,
or the permeability of the capillaries will lead Answer E is incorrect. Increased plasma pro-
to a net flow of fluid from the capillaries into tein levels would cause an increase in capillary
the interstitium, leading to the edema that is oncotic pressure, leading to fluid retention in
seen in this patient. Likewise, decreasing pc the vascular space.
and Pi will also lead to net outward flow and
High-Yield Systems

edema.
Answer A is incorrect. Decreasing capil-
lary permeability would result in fluid being
trapped in the vascular space.
Cardiovascular
Chapter 9

Endocrine

197
198 Section II: Organ Systems  •  Questions

Q u e st i o n s

1. A 42-year-old woman with a history of perni- (A) Anaplastic


cious anemia presents to her physician com- (B) Follicular
plaining of anxiety and occasional palpita- (C) Hürthle cell
tions. She has unexplained weight loss of 10 lb (D) Medullary
(4.5 kg) and multiple daily bowel movements. (E) Papillary
She has not had a menstrual period in four
months. She has a thyroid bruit and a 10 × 4 3. A 21-year-old woman presents to the emer-
cm oval, nontender soft-tissue mass anterior to gency department complaining of diarrhea,
the thyroid cartilage. Which of the following is heart palpitations, anxiety, and diffuse abdomi-
the most likely etiology of this patient’s disease? nal pain. Vital signs show tachycardia. The pa-
High-Yield Systems

tient wanted to lose weight and started taking


(A) Autoantibody stimulation of thyroid-
her mother’s medication to do so; she does not
stimulating hormone receptors
know for which condition the medication was
(B) Idiopathic replacement of thyroid tissue
prescribed. Her friend had similar symptoms
with fibrous tissue
not long ago and was treated for her condition.
(C) Papillary thyroid cancer
Which of the following most likely accounts
(D) Thyroid adenoma
for this patient’s presentation?
(E) Viral infection leading to destruction of
thyroid tissue (A) Dobutamine
(B) Iodide
2. A 57-year-old man with hypertension and coro- (C) Levothyroxine
nary artery disease presents to the clinic for his (D) Methimazole
annual check-up. His physical examination is (E) Propylthiouracil
notable for a solitary thyroid nodule. The pa-
tient denies any symptoms such as cough, diffi- 4. The parents of an 18-month-old girl bring her
culty swallowing, or hoarseness. The physician to the pediatrician because she has been hav-
refers him to an ear, nose, and throat specialist ing frequent fevers, has developed dark circles
for further evaluation. Results of a fine-­needle around her eyes, and has seemed paler than
Endocrine

aspiration biopsy are shown in the image. usual. On physical examination the physician
Which of the following thyroid tumors does notices an abdominal mass that is palpable in
this patient have? the right upper and left upper quadrants. Im-
aging studies show involvement of the left ad-
renal gland and the liver. Which the following
would be consistent with this patient’s likely di-
agnosis?
(A) Bilateral underdevelopment of the iris
(B) Elevated urine free cortisol
(C) Elevated urine vanillylmandelic acid
(D) Symptoms of hypotensive shock
(E) WBC count >60,000/mm³

Courtesy of Armed Forces Institute of Pathology.


Chapter 9: Endocrine  •  Questions 199

5. A 60-year-old patient from an underserved, (A) Carbamazepine


impoverished family complains of being “re- (B) Lithium
ally thirsty, really hungry, and always having to (C) Fluoxetine
urinate.” The patient also reports worsening fa- (D) Clomipramine
tigue and weight loss. Urinalysis reveals severe (E) Phenobarbital
proteinuria, which prompted a renal biopsy;
the histologic section is shown in the image. 7. A 45-year-old man with type 2 diabetes melli-
Which of the following histologic findings is tus presents to his physician for regular follow-
apparent in the renal tissue? up. On neurologic examination, the patient
demonstrates the most common initial sensory
impairment in patients with diabetes mellitus.
Which of the following receptors is most likely

High-Yield Systems
affected in this patient?
(A) Krause end bulbs
(B) Meissner corpuscle
(C) Merkel nerve endings
(D) Pacinian corpuscle
(E) Ruffini corpuscle

8. A certain tumor-associated syndrome may


manifest as diarrhea, cutaneous flushing, asth-
matic wheezing, and right-sided valvular heart
disease. These symptoms are attributable to
high circulating levels of particular tumor-
Reproduced, with permission, from USMLERx.com. secreted substance. Which of the following
acts primarily by blocking the reuptake of this
same substance?
(A) Crescent formation
(B) Hyaline arteriolosclerosis (A) Buspirone
(C) Leukocytic infiltration (B) Fluphenazine

Endocrine
(D) Nodular glomerulosclerosis (C) Isocarboxazide
(E) Wire-loop abnormality (D) Maprotiline
(E) Quetiapine
6. A 35-year-old man presents to his physician (F) Trazodone
complaining of increased urinary frequency,
polyuria, and an insatiable thirst. He was re-
cently started on a new medication after visit-
ing his psychiatrist. Laboratory studies show
that the patient is hypernatremic and has de-
creased urine osmolarity. Which of the follow-
ing drugs is most likely contributing to this pa-
tient’s current condition?
200 Section II: Organ Systems  •  Questions

9. A 45-year-old woman presents to her doctor 11. A normal-appearing, 23-year-old man sees a
with feelings of fatigue, increased appetite, in- physician, because he was awakened on sev-
creased sweating, and palpitations. Her doctor eral occasions by severe headaches, anxiety,
also notes that her eyes appear unusual (see and heart palpitations. Vital signs are within
image). She receives pharmacologic treatment normal limits. On physical examination, he
for her condition, but soon develops a fever has pectus excavatum, a high arched palate,
and multiple infections in her throat and gas- bilateral pes cavus, and scoliosis. Which of the
trointestinal tract. Her doctor quickly discon- following laboratory measures would likely be
tinues the medication. Which medication was elevated in this patient?
she most likely prescribed?
(A) Calcitonin
(B) Calcium
(C) Insulin
High-Yield Systems

(D) Phosphate
(E) Thyroglobulin

12. A 23-year-old woman visits her primary care


physician because of fatigue. Laboratory stud-
ies show:
WBC count: 5000/mm³
Hematocrit: 28.0%
Hemoglobin: 9.8 g/dL
Mean corpuscular volume: 105 fL
Glucose: 98 mg/dL
Reproduced, with permission, from USMLERx.com. Platelet count: 322,000/mm³
Na+: 139 mEq/L
K+: 4.3 mEq/L
(A) Folic acid
Cl-: 101 mEq/L
(B) Levothyroxine
Blood urea nitrogen: 15 mg/dL
(C) Propranolol
Creatinine: 0.9 mg/dL
(D) Propylthiouracil
Endocrine

(E) Radioactive iodine A Schilling test is performed: A dose of radio-


labeled vitamin B12 is administered orally, and
10. A 40-year-old man with AIDS and chronic ad- shortly thereafter a nonradiolabeled dose is
renocortical insufficiency presents to the emer- given via injection. A 24-hour urine collection
gency department with acute-onset nausea, reveals no vitamin B12 excretion. One week
vomiting, and abdominal pain. He is febrile later, another dose of radiolabeled vitamin B12
and hypotensive. He was started recently on is administered, along with intrinsic factor.
a new drug for a fungal infection. Laboratory This time, vitamin B12 excretion in the urine
tests reveal a potassium level of 5.2 mEq/L. is normal. The secretion of what substance is
Which of the following agents is most likely to likely altered?
have caused this patient’s symptoms?
(A) Bicarbonate
(A) Amphotericin B (B) Gastrin
(B) Caspofungin (C) Hydrochloric acid
(C) Flucytosine (D) Insulin
(D) Ketoconazole (E) Trypsin
(E) Trimethoprim-sulfamethoxazole
Chapter 9: Endocrine  •  Questions 201

13. Overactivity of the smaller, pale cells with 15. A 65-year-old woman comes to her primary
round nuclei shown in the image can lead to care physician complaining of progressive
fibrous tissue with hemorrhagic foci and cyst weakness and fatigue. On further questioning
formation in bone marrow. What are these she notes recent weight gain and constipation,
cells called? and states that she often feels cold. Physical ex-
amination shows a moderate-sized, non-tender
goiter. Histologic examination of the goiter
shows a lymphocytic infiltrate. Which of the
following best describes this patient’s thyroid-
stimulating hormone and thyroid hormone lev-
els relative to normal baseline values?

High-Yield Systems
Thyroid-
Total Free
Choice stimulating
thyroxine thyroxine
hormone
A

Reproduced, with permission, from USMLERx.com. E

Reproduced, with permission, from USMLERx.com.


(A) Adipocytes
(B) Parathyroid oxyphil cells
(C) Parathyroid chief cells (A) A
(D) Thyroid follicle cells (B) B
(E) Thyroid C cells (C) C
(D) D
14. A 66-year-old man with a 50-pack-year his- (E) E

Endocrine
tory of cigarette smoking comes to the clinic
complaining of chronic cough, dyspnea, and 16. A 25-year-old woman is brought to the hospital
blood in his sputum. He says he has been feel- by her parents because she told them she con-
ing lethargic and has lost 18 kg (40 lb) over the stantly hears voices that insult her and tell her
past three months with no changes in diet or to perform various actions. After a brief stay on
exercise. While awaiting x-ray of the chest, the the psychiatric floor she was discharged with a
patient suffers a seizure and is rushed to the prescription for haloperidol. Which of the fol-
emergency department of the nearest hospi- lowing is an adverse effect of this medication?
tal. Laboratory studies show a serum sodium (A) Ataxia
level of 120 mEq/dL. Which of the following (B) Diarrhea
is most likely to be elevated in this patient? (C) Galactorrhea
(A) ACTH (D) Hypertension
(B) ADH (E) Insomnia
(C) Parathyroid hormone
(D) Renin
(E) Tumor necrosis factor-a
202 Section II: Organ Systems  •  Questions

17. A 43 year-old woman presents with fatigue, a (B) Inhibition of intestinal brush border
4.5-kg (9.9-lb) weight gain over the past three a-glucosidase and delayed sugar hydrolysis
months, cold intolerance, hair loss, and con- (C) Stimulation of glycolysis in peripheral tis-
centration problems. Physical examination is sues and decrease in hepatic gluconeogen-
significant for dry, coarse skin and bradycardia. esis
She states that she had some slight swelling of (D) Stimulation of peroxisome proliferator-
her lower neck several months ago, which re- activated receptor-gamma, leading to in-
solved without treatment. Results of antithy- creased glucose uptake in muscle and adi-
roglobulin antibody and antinuclear antibody pose tissue
tests are negative, but a thyroid peroxidase (E) Stimulation of the release of endogenous
antibody test is positive. For which other auto- insulin stores
immune diseases does this patient have an in-
High-Yield Systems

creased risk? 20. A 42-year-old white man presents with a com-


plaint of a “sugar problem.” He brings a copy
(A) Graves disease and pernicious anemia
of the report that was obtained on his routine
(B) Osteoarthritis and Addison disease
insurance screening test. The report shows his
(C) Rheumatoid arthritis and vitiligo
fasting glucose level was 136 mg/dL, and a
(D) Type 1 diabetes mellitus and celiac disease
follow-up fasting glucose level was 142 mg/dL.
(E) Whipple disease and type 1 diabetes melli-
During a review of systems the patient com-
tus
plains of weight gain, trouble speaking, and
vision problems over the past several months.
18. A 26-year-old man presents to a clinic com-
He specifically comments that his favorite hat
plaining of intermittent muscle cramping for
is now too small on him. During examination
the past three months. On physical examina-
of the patient’s cranial nerves, which of the fol-
tion, his blood pressure is found to be 190/105
lowing visual field cuts would be expected?
mm Hg. Blood is drawn, and this reveals his
sodium level to be 155 mEq/L, potassium level
3.2 mEq/L, and bicarbonate level 33 mEq/L. Defect in visual field of
Otherwise, his extended metabolic panel and Left eye Right eye
complete blood count are normal. What is the
Endocrine

most likely cause of this patient’s findings? A


(A) Addison disease
(B) Conn syndrome B
(C) Cushing syndrome
(D) Hyperparathyroidism C
(E) Pheochromocytoma
D
19. A 44-year-old woman presents to her physi-
cian with a one month history of fatigue, poly- E
uria, and polydipsia. Laboratory studies show
a glucose level of 350 mg/dL. The physician Reproduced, with permission, from USMLERx.com.
decides to prescribe a medication to treat dia-
betes mellitus. He warns the patient that an
adverse effect of the medication is lactic acido- (A) A
sis. Which of the following is the most likely (B) B
mechanism of action of the medication? (C) C
(D) D
(A) Binding to the insulin receptor, increasing
(E) E
glycogen storage in the liver and glycogen
and protein synthesis in muscle
Chapter 9: Endocrine  •  Questions 203

21. A 34-year-old woman gives birth to a child 24. A 42-year-old woman with a history of breast
with ambiguous genitalia. The child is hypo- cancer presents to the emergency department
tensive. A geneticist tells the mother that her with hematuria and costovertebral tenderness.
child is genotypically a female, although the She rates her pain as 8 out of 10 in severity,
child seems to have partially virilized external but says that it seems to “come and go,” fluctu-
genitalia. Which of the following enzymes is ating in intensity. She also complains of mild
most likely deficient in this infant? fatigue, along with nausea and constipation for
the past few months. Physical examination re-
(A) 5a-Reductase
veals tenderness of the joints and muscle weak-
(B) 17a-Hydroxylase
ness. Radionuclide imaging demonstrates one
(C) 11b-Hydroxylase
area of increased uptake corresponding to her
(D) 21-Hydroxylase
superior left parathyroid gland. Which labora-

High-Yield Systems
22. Glucocorticoids are important in the treat- tory values are most likely to be seen in this pa-
ment of inflammatory diseases; however, their tient?
use is associated with many adverse effects on
multiple systems. The utility of glucocorticoids Choice Parathyroid 1,25-Dihydroxy- Serum Serum
has to be weighed against the patient’s ability hormone cholecalciferol calcium phosphate
A
to withstand the problems that are likely to
arise. High-dosage glucocorticoid treatment B

can result in which ECG changes? C

D normal to
(A) Appearance of delta wave E
(B) Appearance of U wave
(C) Peaked T wave Reproduced, with permission, from USMLERx.com.
(D) PR segment elongation
(E) ST segment elevation
(A) A
23. A 30-year-old woman with no prior medi- (B) B
cal history is diagnosed with diastolic hyper- (C) C
tension. The patient reports symptoms of (D) D

Endocrine
weakness, fatigue, polyuria, polydipsia, and (E) E
headache. Laboratory results indicate she is hy-
pokalemic and hypernatremic, has decreased 25. A 27-year-old white woman comes to the phy-
renin levels, and has a metabolic alkalosis. CT sician because of a six-month history of pro-
of the abdomen shows a 4-cm mass located gressive weakness and fatigue, and occasional
on the right side of the body. In order to im- mild abdominal pain. She also has noticed
mediately relieve the patient’s condition surgi- that her skin has become more tan, especially
cally, the surgeon must first ligate the primary at the elbows, knees, and knuckles, despite the
venous drainage of the mass. Into which of the fact that she is not usually in the sun. Labora-
following structures does the primary venous tory tests show decreased serum levels of so-
drainage flow? dium, bicarbonate, chloride, and glucose and
(A) Abdominal aorta increased levels of potassium. Which of the
(B) Inferior vena cava following is the most likely diagnosis?
(C) Portal vein (A) Addison disease
(D) Right gonadal vein (B) Conn syndrome
(E) Right renal vein (C) Cushing syndrome
(D) Pheochromocytoma
(E) Waterhouse-Friderichsen syndrome
204 Section II: Organ Systems  •  Questions

26. A 34-year-old woman goes to her primary care pH: 7.25


physician complaining of visual changes and Partial oxygen pressure: 90 mm Hg
a recent feeling that “her heart was racing.” Partial carbon dioxide pressure: 30 mm Hg
During the interview, the physician notices Bicarbonate: 18 mEq/L
that the patient is clearly anxious. During the
Based on the probable diagnosis, which of the
review of systems, the patient reveals a recent
following is the child’s most likely human leu-
unintentional 4-kg (8.8-lb) weight loss. On
kocyte antigen type?
physical examination, the physician notes that
the patient is tachycardic and has 2+ nonpit- (A) HLA-A3
ting edema in her lower extremities. Which of (B) HLA-B27
the following is the most likely etiology of this (C) HLA-DQ2 and -DQ8
disease? (D) HLA-DR2 and -DR3
High-Yield Systems

(E) HLA-DR3 and -DR4


(A) Autoantibodies to the thyroid-stimulating
hormone receptor 29. The image below depicts the arterial network
(B) Circulating antibodies to thyroid peroxi- of the brain. Infarction of which artery would
dase and thyroglobulin lead to contralateral deficits of the leg and
(C) Hyperfunctioning thyroid nodule foot?
(D) Iodine deficiency
(E) Reaction to radiation
A
27. An 18-year-old woman is referred to a special- B
ist, because she never began menstruating.
She reports generalized weakness and oc-
casional bouts of nausea and vomiting. Her
blood pressure is 160/99 mm Hg. Laboratory C
studies show a serum potassium level of 2.2
mEq/L. She is diagnosed with a condition that
affects the production of two of the three ma- D
jor adrenal hormones, leaving only one func-
Endocrine

tioning hormone. In which area of the adrenal


gland is this one functioning hormone pro-
E
duced?
(A) Capsule Reproduced, with permission, from USMLERx.com.
(B) Medulla
(C) Zona fasciculata
(D) Zona glomerulosa (A) A
(E) Zona reticularis (B) B
(C) C
28. A 7-year-old girl with a viral upper respiratory (D) D
infection is brought to the emergency depart- (E) E
ment because of severe confusion, abdominal
pain, and vomiting. Her respirations are deep
and rapid, and her breath has a fruity odor.
The child’s mother notes that she has been
drinking large quantities of water and has had
an increased appetite. Despite her increased
appetite, she has been losing weight. An arte-
rial blood gas analysis shows:
Chapter 9: Endocrine  •  Questions 205

30. A researcher is investigating hormones that act 33. A 59-year-old man with no prior medical his-
via nuclear hormone receptors. She has devel- tory presents to his physician with marked hy-
oped an assay to analyze the activity of these perglycemia, diarrhea, and weight loss. CT
hormones by assessing mRNA levels of known scan of the abdomen reveals a pancreatic mass.
downstream products in various tissues. What A trial period on an oral hypoglycemic agent
hormone can be studied by this technique? has not succeeded in reducing his glucose lev-
els. Physical examination is significant for the
(A) Glucagon
painful rash shown in the image. Which of the
(B) Histamine
following is the most likely diagnosis?
(C) Insulin
(D) Norepinephrine
(E) Thyroid hormone

High-Yield Systems
31. A 6-year-old girl is brought to the clinic by her
parents, because they thought she looked like
she was “growing breasts.” On physical exami-
nation, she indeed is developing breasts, but
she also has pubic hair. The diagnosis is pre-
cocious puberty, and leuprolide is prescribed.
The parents are told that leuprolide will sup-
press their daughter’s increased expression of
sex hormones. The vasculature of the hypo-
thalamus and pituitary gland is uniquely de-
signed and functionally resembles the design
of the gastrointestinal vasculature in that it in-
cludes a portal system. Which of the following Reproduced, with permission, from Wolff K, Johnson RA, Su-
urmond D. Fitzpatrick’s Color Atlas & Synopsis of Clinical
is the functional significance of this special-
Dermatolgy, 5th edition. New York: McGraw-Hill, 2005; Fig.
ized vascular system?
17-12.
(A) Delivery of hormones for processing
(B) Delivery of hormones for storage

Endocrine
(C) Delivery of hormones in high concentra- (A) Glucagonoma
tions (B) Insulinoma
(D) Delivery of hormones to the hypothalamus (C) Pancreatic adenocarcinoma
(E) Delivery of pre-formed trophic hormones (D) Type 1 diabetes mellitus
systemically (E) VIPoma

32. A 24-year-old nulliparous woman presents to 34. A particular hormone stimulates the enzyme
her physician with galactorrhea and bilateral that converts testosterone to 17b-estradiol in
hemianopsia. Medical history is significant for granulosa cells. What is a second function of
hypercalcemia and recurrent duodenal ulcers. this hormone?
Which of the following shares the genetic in- (A) Causes endometrial proliferation
heritance pattern of this patient’s disorder? (B) Induces inhibin secretion
(A) Fabry disease (C) Stimulates desmolase
(B) Familial dysautonomia (D) Stimulates secretion of luteinizing hor-
(C) Hypokalemic periodic paralysis mone and follicle-stimulating hormone
(D) Mitochondrial myopathy, encephalopathy, (E) Triggers ovulation
lactic acidosis, and stroke-like (MELAS)
episodes
(E) X-linked hypophosphatemic rickets
206 Section II: Organ Systems  •  Questions

35. A 44-year-old man is brought to the emergency


department after collapsing at his office. The
patient has diabetes mellitus and was recently
diagnosed with hypertension. He is complain-
ing of blurry vision, fatigue, and dizziness. His
coworkers report that he was confused and
agitated before he collapsed. He denies chest
pain, palpitations, tremor, fever, or chills. His
diabetes has been poorly controlled with gly-
buride, and his most recent HbA1c level was
8.5%. He is unable to recall the name of his
antihypertensive drug. Which of the following
High-Yield Systems

agents is most likely responsible for this pa-


tient’s condition?
Reproduced, with permission, from USMLERx.com.
(A) Enalapril
(B) Hydrochlorothiazide
(C) Metoprolol (A) C cells
(D) Propranolol (B) Chromaffin cells
(E) Triamterene (C) Islet beta cells
(D) Plasma cells
36. A 10-year-old girl with no prior medical history (E) Somatotrophs
presents to the clinic because she’s not feeling
well. Her mother says the girl has been very 38. A 25-year-old woman who was recently diag-
thirsty lately and urinating more frequently; nosed with hypothyroidism comes to the clinic
she has also lost 4.5 kg (10 lb) over the past for a follow-up examination. In addition to le-
year. Laboratory studies show a markedly el- vothyroxine, she has been taking a number of
evated blood glucose level and a decreased daily supplements to help accelerate her re-
insulin level; levels of a specific antibody are covery. Although she initially reported an im-
also elevated. Antibodies against which of the provement in her symptoms, the patient now
complains of constipation, brittle hair, and
Endocrine

following organs are most likely present in this


girl? fatigue. Which of the following supplements
best accounts for the decline in the patient’s
(A) Adrenal gland clinical course?
(B) Kidney
(C) Liver (A) Iodine
(D) Pancreas (B) Magnesium
(E) Spleen (C) Vitamin C
(D) Vitamin E
37. A tumor biopsy reveals cells with numerous (E) Zinc
large vacuolar spaces within the cytoplasm
(see image). The patient is later prescribed 39. A 65-year-old man presents to his physician
phenoxybenzamine. From what cells is this tu- complaining of nocturia. He reports awaken-
mor derived? ing five to six times per night to urinate and
difficulty starting a stream. Biopsy of the pros-
tate shows enlargement and dilation of the
prostatic glands but no dysplasia. Which of the
following is the most appropriate pharmaco-
logical treatment for this patient?
Chapter 9: Endocrine  •  Questions 207

(A) Finasteride (A) Esophagus


(B) Flutamide (B) First branchial arch
(C) Ketoconazole (C) First branchial pouch
(D) Spironolactone (D) Pharynx
(E) Yohimbine (E) Second branchial arch
(F) Second branchial pouch
40. A mother brings her 2-year-old daughter to the (G) Trachea
pediatrician because she has noticed that her
daughter’s ribs appear knobby on either side of 42. A 25-year-old woman with a history of obesity
her midline. The mother says that she and her presents for her annual physical examination.
daughter live in an inner-city apartment and She is seeking advice on how to lose weight.
rarely go outside. She also notes that she is still She has been on a diet and exercise program

High-Yield Systems
breast-feeding her daughter. Which laboratory for the past year without much success and
result would the pediatrician expect to see in wants to try pharmaceutical intervention. The
this patient? mechanism of action of which of the following
drugs is most similar to that of sibutramine?

Parathyroid
(A) Methylphenidate
Choice Calcium Phosphate
hormone (B) Orlistat
(C) Selegiline
A
(D) Tranylcypromine
B

C 43. A 19-year-old female college student presents


D with rapid onset of malaise, myalgias, vomit-
E
ing, photophobia, and a temperature of 39.8°C
(103.6°F). Because she quickly develops leg
Reproduced, with permission, from USMLERx.com. pains along with a purpuric rash, she is trans-
ferred from university health services to a local
hospital, where she is found to have a blood
(A) A pressure of 82/49 mm Hg after receiving 2 L
(B) B intravenous normal saline. Physical examina-

Endocrine
(C) C tion reveals positive Kernig and Brudzinski
(D) D signs, a petechial rash on her lower extremity,
(E) E and diffuse abdominal tenderness. Her arterial
oxygen pressure is 58 mm Hg, platelet count
41. A 12-year-old girl presents with her mother to is 81,000/mm³, International Normalized
her pediatrician complaining of a lump in her Ratio is 2.1, and D-dimer levels are elevated
neck. She states that the lump has been there at 41,000 ng/mL. Cerebrospinal fluid shows
as long as she can remember, but within the 1300 WBCs/mm³ and 10. RBCs/mm³. Gram
past three days has become red, swollen, and stain shows multiple gram-negative diplococci.
very painful. On physical examination, the pe- Which aspect of this disease process most
diatrician notes a warm, fluctuant mass in the likely contributes to her hypotension, vomit-
anterior midline of the neck that is exquisitely ing, and diffuse abdominal tenderness?
sensitive to touch and moves with swallowing.
(A) Bilateral adrenal hemorrhage
During embryologic development, this pa-
(B) Disseminated intravascular coagulation
tient’s lesion most likely originated from which
(C) Hypoxemia
of the following structures?
(D) Meningitis
(E) Viral gastroenteritis
208 Section II: Organ Systems  •  Questions

44. A 65-year-old man is diagnosed with the 46. A 25-year-old man with no significant medical
syndrome of inappropriate ADH secretion history comes to the emergency department
(SIADH) in the setting of small cell lung can- after experiencing tremors. On questioning he
cer. To treat his SIADH, the patient is started admits to a recent history of sweating, nausea,
on drug X. He soon begins producing large vomiting, and lightheadedness. On physical
quantities of dilute urine and drinking copious examination he is visibly anxious. Laboratory
amounts of water. Despite fluid restriction, the studies show a blood glucose level of 50 mg/
patient continues to produce dilute urine. Lab- dL. CT of the abdomen shows a 1.5-cm mass
oratory tests reveal an increased serum ADH in the head of an organ. Surgical resection of
level, serum hyperosmolarity, and hypernatre- this mass will necessitate ligation of branches
mia. Drug X might also be used to treat which from which of the following vascular struc-
of the following? tures?
High-Yield Systems

(A) Acute gouty arthritis (A) The gastroduodenal and inferior mesen-
(B) Conn syndrome teric arteries
(C) Enuresis (B) The gastroduodenal and superior mesen-
(D) Lyme disease teric arteries
(E) Pheochromocytoma (C) The proper hepatic and inferior mesen-
teric arteries
45. A neonate with a cleft palate undergoes a full (D) The proper hepatic and superior mesen-
evaluation. Abnormalities on physical exami- teric arteries
nation include low-set ears and ocular hyper- (E) The left gastric and inferior mesenteric ar-
telorism. Echocardiogram reveals a ventricu- teries
lar septal defect, and x-ray of the chest reveals (F) The left gastric and superior mesenteric ar-
absence of a thymic shadow. Complete blood teries
cell count with differential reveals lymphope-
nia. What would be the expected serum cal- 47. A 54-year-old woman presents to the physi-
cium, parathyroid hormone, and phosphorus cian with diabetes mellitus, osteoporosis, and
levels for this infant relative to normal? hypertension. She has noted a recent weight
gain and abdominal striae. Laboratory studies
Endocrine

show a decreased ACTH level. A single mass


Choice Calcium Parathyroid Phosphorus is noted adjacent to the right kidney on ab-
hormone
dominal CT scan. Neither low- nor high-dose
A dexamethasone suppresses the patient’s cortisol
B production. Which of the following is the most
likely explanation for these findings?
C

D (A) Adrenal adenoma


(B) Bilateral adrenal hyperplasia
Reproduced, with permission, from USMLERx.com.
(C) Ectopic ACTH secretion
(D) Exogenous corticosteroid administration
(E) Pituitary adenoma
(A) A
(B) B
(C) C
(D) D
Chapter 9: Endocrine  •  Questions 209

48. A 60-year-old woman with a 55-pack-year 50. A 48-year-old man with a history of thyroid
smoking history presents to the emergency carcinoma for which he underwent thyroidec-
department complaining of nausea and vomit- tomy five years ago presents to his primary care
ing, headache, malaise, and diffuse bone pain. physician complaining of a six-week history of
CT shows a solitary nodule in the upper lobe intermittent palpitations, diaphoresis, head-
of the right lung. Laboratory studies are sig- aches, and anxiousness. His blood pressure is
nificant for a serum calcium level of 14.2 mg/ 180/90 mm Hg and heart rate is 135/min. After
dL, serum phosphate of 1.5 mg/dL, and serum a complete work-up, the patient is diagnosed
alkaline phosphatase activity of 81 U/L. The with a pheochromocytoma, and his physician
factor that accounts for this patient’s laboratory recommends surgery. Which of the following
findings acts primarily at which of the follow- is the most appropriate preoperative manage-
ing locations? ment for this patient?

High-Yield Systems
(A) Adrenal cortex and intestines (A) a-Blockade followed by b-blockade
(B) Adrenal cortex and renal tubules (B) b-Blockade followed by a-blockade
(C) Intestine and bones (C) Levothyroxine
(D) Renal tubules and bones (D) Prednisone
(E) Renal tubules and pancreas (E) Propylthiouracil

49. A researcher studying type 2 diabetes mellitus


is attempting to induce insulin resistance in
normal mice. Inhibition of which of the fol-
lowing would produce this effect?
(A) Adenylate cyclase
(B) Guanylate cyclase
(C) Serine kinases
(D) Threonine kinases
(E) Tyrosine kinases

Endocrine
210 Section II: Organ Systems  •  Answers

An s w e r s

1. The correct answer is A. This patient pre­ Answer D is incorrect. Most thyroid adenomas
sents with Graves disease. In Graves disease, present as solitary nodules and usually are non-
thyroid-stimulating IgG antibodies bind to functional.
thyroid-stimulating hormone (TSH) recep-
Answer E is incorrect. Viral infections such
tors on the thyroid, leading to excess thyroid
as mumps or coxsackievirus can lead to de-
hormone production. This causes glandular
struction of thyroid tissue and granulomatous
hyperplasia and enlargement, resulting in a
inflammation, as seen in subacute granuloma-
characteristic goiter (the 10 × 4 cm nontender
tous thyroiditis. Patients typically present with
soft-tissue mass is the goiter, which has oblit-
flu-like symptoms and thyroid tenderness. The
erated the normal thyroid anatomy.) Graves
High-Yield Systems

disease is typically self-limited and can include


disease is the most common cause of thyro-
a transient hyperthyroid state.
toxicosis. Patients with Graves disease are at
higher risk of other autoimmune diseases, such 2. The correct answer is E. As shown in the
as pernicious anemia or type 1 diabetes mel- image, psammoma bodies, which are calci-
litus (DM), than are patients without Graves fied remnants belonging to the tumor that
disease. Presenting symptoms of Graves dis- likely infarcted, are present in about one-half
ease include anxiety, irritability, tremor, heat of patients with papillary carcinoma. Papil-
intolerance with sweaty skin, tachycardia and lary thyroid cancer is the most common type
cardiac palpitations, weight loss, increased ap- of thyroid cancer and has the best prognosis.
petite, fine hair, diarrhea, and amenorrhea or Papillary carcinoma is the most common thy-
oligomenorrhea. Signs include diffuse goiter, roid tumor, and results from neck irradiation, a
exophthalmos, periorbital edema, and hyperre- common therapy between 1920 and 1960. Pa-
flexia. Laboratory values reveal increased thy- tients often present as the man in our scenario:
roid hormone levels and decreased TSH levels. asymptomatic with an incidental finding on
Answer B is incorrect. Idiopathic replacement palpation of the thyroid gland.
of thyroid and surrounding tissue with fibrous Answer A is incorrect. Anaplastic thyroid can-
tissue is seen in Riedel thyroiditis; patients may
Endocrine

cer is rare, undifferentiated, and tends to have


present with dysphagia, stridor, dyspnea, and a worse prognosis. It is the most aggressive type
hypothyroidism, although more than 50% of of thyroid gland carcinoma. Patients often pre­
patients are euthyroid. The disease can mimic sent with a rapidly-growing nodule or mass and
thyroid carcinoma, which is high on the list of a complaint of dysphagia, dyspnea, neck pain,
differential diagnoses for a patient with Riedel and/or cough. Histologically, this tumor shows
thyroiditis. infiltration of adjacent structures with regions
Answer C is incorrect. A large, nontender of necrosis and hemorrhage. Treatment for
mass in the region where one usually would this disease is mostly palliative, as this cancer
expect to palpate the thyroid could be con- is very aggressive and has often metastasized by
cerning for a locally advanced thyroid malig- the time of diagnosis.
nancy; however, the symptoms of hyperthy- Answer B is incorrect. Follicular thyroid can-
roidism that this patient is experiencing make cer tends to have a good prognosis, but there
this much less likely, because the vast majority is the risk of early metastasis. It is a well-
of thyroid malignancies do not produce thyroid differentiated tumor, but has the potential to
hormones. Furthermore, the exam is more become invasive. The most common presenta-
consistent with a diffuse goiter than a focal tion is subclinical, with a thyroid mass or nod-
nodule, which tends to be more discrete and ule felt upon palpation of the neck. Histologi-
firm. cally, this tumor appears encapsulated and is
Chapter 9: Endocrine  •  Answers 211

often found in necrotic or hemorrhagic areas. Answer B is incorrect. Pharmacologic doses of


It can also have well-defined follicles. iodide are used to treat hyperthyroidism. They
inhibit the synthesis of thyroid hormone and
Answer C is incorrect. Hürthle cell is a rare
the release of pre-formed thyroid hormone. Io-
variant of follicular cell carcinoma with the
dide is administered orally, and adverse effects
following differences: (1) it tends to be bilat-
include sore mouth and throat, rashes, ulcer-
eral and multifocal, and (2) it is more likely to
ations of mucous membranes, and a metallic
metastasize to lymph nodes than follicular car-
taste in the mouth.
cinoma. Patients often present with a palpable
mass in the neck, and complaints of pressure Answer D is incorrect. Methimazole is used
on the throat resulting in pain, coughing, dysp­ to treat hyperthyroidism by inhibiting the addi-
nea, dysphagia, or hoarseness. tion of iodine to thyroglobulin by the enzyme
thyroperoxidase. Adverse effects include agran-

High-Yield Systems
Answer D is incorrect. Medullary thyroid
ulocytosis, which should be suspected if the
cancer arises from the parafollicular cells (un-
patient develops a fever or sore throat while on
like papillary, follicular, and anaplastic, which
this medication. In the event of an overdose,
arise from the epithelial cells). Since the para-
iatrogenic hypothyroidism would result.
follicular cells (C cells) produce calcitonin,
this can serve as a marker by which the tumor Answer E is incorrect. Propylthiouracil, which
can be monitored. Carcinoembyronic antigen inhibits the synthesis of T4 and the peripheral
can also serve as a marker for medullary thy- conversion of T4 to tri-iodothyronine, is used
roid cancer, but it is not as sensitive. This tu- to treat hyperthyroidism. Rare toxicities caused
mor can be seen alone or as a part of multiple by this agent include agranulocytosis, rash, and
endocrine neoplasia (MEN) types II and III. edema.

3. The correct answer is C. This patient’s symp- 4. The correct answer is C. Neuroblastoma is a
toms are caused most likely by surreptitious tumor that often affects the adrenal medulla,
use of levothyroxine. This medication can although it can involve any site along the sym-
cause weight loss but also thyrotoxicosis. Levo- pathetic chain. It is most commonly seen in
thyroxine is a synthetic form of thyroxine (T4) children, with a median age of presentation of
that is used to treat hypothyroidism. Exces- 17 months. The condition may present with

Endocrine
sively high serum levels of levothyroxine result constitutional symptoms as well as periorbital
in symptoms of thyrotoxicosis, including those ecchymosis, proptosis, limp, bone pain, hyper-
described in the vignette in addition to heat tension, and/or involvement of the skin (blu-
intolerance, unexplained weight loss, agita- ish, nontender subcutaneous nodules). Urine
tion, and confusion. Her friend also had these vanillylmandelic acid and homomandelic acid
symptoms because of her thyrotoxicosis caused levels are typically elevated in these patients.
by Graves disease, in which TSH receptor Therapy involves resection, radiation, and/or
antibodies are produced. Graves disease also chemotherapy.
causes exophthalmos; this autoimmune disease
Answer A is incorrect. Bilateral underdevelop-
affects the periorbital region, resulting in prop­
ment of the iris, also known as aniridia, would
tosis and extraocular muscle swelling.
be consistent with symptoms of WAGR, in
Answer A is incorrect. Dobutamine is an which the W stands for Wilms tumor (WAGR
a-adrenergic agonist that is useful in the acute means Wilms tumor, Aniridia, Genitouri-
treatment of congestive heart failure (CHF). nary malformations, and mental Retardation).
Whereas the effects of this drug can mimic Wilms tumor is the most common primary
the symptoms and signs of thyrotoxicosis, this renal tumor, commonly presenting with an
agent cannot account for the gastrointestinal asymptomatic flank mass that classically does
(GI) symptoms, weight loss, and heat intoler- not cross the midline, while this patient’s mass
ance that are characteristic of thyrotoxicosis.
212 Section II: Organ Systems  •  Answers

does cross the midline on physical examina- sclerosis often seen in long-standing hyperten-
tion. Mean age at presentation is 42 months. sion, a common cause of chronic renal failure.
The image shown is of a glomerulus, not a re-
Answer B is incorrect. Adrenal adenocarci-
nal arteriole.
noma may cause Cushing syndrome, result-
ing in findings such as truncal obesity, moon Answer C is incorrect. Leukocytic infiltration
facies, buffalo hump, purple striae, muscle is characteristic of inflammatory diseases of the
wasting, osteoporosis, and psychiatric distur- glomerulus; it is observed most often in post-
bances. However, this patient’s age and symp- streptococcal glomerulonephritis, but also can
toms are far more consistent with neuroblas- be seen in membranoproliferative glomerulo-
toma. nephritis and rapidly progressive glomerulone-
phritis.
Answer D is incorrect. Symptoms of hypoten-
High-Yield Systems

sive shock without hypovolemia in this young Answer E is incorrect. The wire-loop abnor-
child would be consistent with adrenal insuf- mality is characteristic of type IV lupus ne-
ficiency due to adrenocortical atrophy. Other phropathy. This condition is caused by thick-
symptoms may include nausea and vomiting, ening of the glomerular basement membrane
which this patient does not have. Adrenocorti- associated with immune complex deposition.
cal atrophy would not present with a mass on
physical examination or imaging. 6. The correct answer is B. This patient is pre-
senting with nephrogenic diabetes insipidus
Answer E is incorrect. A WBC count
(DI), which is a rare toxicity of lithium. Pa-
>60,000/mm³ in this patient would be consis-
tients with DI either lack the ability to pro-
tent with an acute leukemia. However, while
duce ADH (central DI), or cannot produce an
acute leukemias can present with systemic
appropriate renal response to ADH (nephro-
symptoms, they do not involve the presence of
genic DI). Patients present with symptoms of
abdominal masses.
insatiable thirst, polyuria, and an inability to
concentrate urine. Lithium, which is used as
5. The correct answer is D. This is a classic case
a mood stabilizer for bipolar affective disorder,
of a patient with type 2 DM: polydipsia, poly-
is an iatrogenic cause of nephrogenic diabetes
uria, and polyphagia in an individual >40 years
insipidus.
of age. This condition is due to increased re-
Endocrine

sistance to insulin. The image shows nodular Answer A is incorrect. Carbamazepine is an


glomerulosclerosis, also known as Kimmelstiel- antiepileptic drug that can cause diplopia,
Wilson glomerulosclerosis. This represents the ataxia, and liver toxicity.
accumulation of nodules in the mesangial ma-
Answer C is incorrect. Fluoxetine is a selec-
trix, which is pathognomonic for DM. Kidneys
tive serotonin reuptake inhibitor that can
of diabetic patients also show increased base-
cause symptoms of central nervous system
ment membrane thickness and diffuse mesan-
(CNS) stimulation such as anxiety, tremor,
gial matrix proliferation.
and nausea.
Answer A is incorrect. Crescent formation re-
Answer D is incorrect. Clomipramine is a tri-
sults from a proliferation of Bowman capsule
cyclic antidepressant that can cause convul-
epithelial cells, which appear to “crowd out”
sions, coma, and arrhythmias.
the glomerular tufts. It is classically seen in ne-
phritic syndromes and suggests a poor progno- Answer E is incorrect. Phenobarbital is a
sis. barbiturate that can cause symptoms of CNS
depression and induce the cytochrome P450
Answer B is incorrect. Hyaline arterioloscle-
(CYP450) enzyme system.
rosis is a homogeneous, eosinophilic thicken-
ing of arteriolar walls that results in a narrowed 7. The correct answer is D. The most common
vessel lumen. It is a feature of benign nephro- initial sensory impairment in patients with
Chapter 9: Endocrine  •  Answers 213

diabetes is a loss of vibrational sensation. This increased in these patients. Trazodone is an


finding is most clearly demonstrated on physi- antidepressant whose primary mechanism of
cal examination by placing a vibrating tun- action is inhibiting serotonin reuptake in the
ing fork on the big toe. The sensory receptors CNS.
responsible for transducing the sensation of
Answer A is incorrect. Buspirone is a 5-HT1A
vibration, pressure, and tension are the large,
agonist used to treat generalized anxiety disor-
encapsulated pacinian corpuscles, which are
der.
located in the deeper layers of the skin, liga-
ments, and joint capsules. They can be dis- Answer B is incorrect. Fluphenazine is a typi-
tinguished histologically by their onion-like cal antipsychotic that acts by blocking dopa-
appearance on cross section. This patient is mine D2 receptors. It does not significantly in-
presenting with one of the complications of hibit serotonin reuptake.

High-Yield Systems
diabetes, neuropathy, and because pacinian Answer C is incorrect. Isocarboxazide is an
corpuscles are responsible for transducing vi- antidepressant whose mechanism of action
bratory stimuli, it is these receptors that are in- consists of inhibiting an enzyme called mono-
volved in this patient’s presentation. amine oxidase. While this results in elevated
Answer A is incorrect. Krause end bulbs are serotonin levels, isocarboxazide itself does not
sensory receptors found in the oropharynx and inhibit serotonin reuptake.
conjunctiva of the eye. Answer D is incorrect. Maprotiline is an an-
Answer B is incorrect. Meissner corpuscles, tidepressant that acts by blocking the reuptake
which are responsible for conveying the sensa- of norepinephrine.
tion of light touch, are small and encapsulated Answer E is incorrect. Quetiapine is an atypi-
sensory receptors found just beneath the der- cal antipsychotic. These agents act by blocking
mis of hairless skin, most prominently in the 5-HT2 and dopamine receptors.
fingertips, soles of the feet, and lips. Meissner
corpuscles are involved in the reception of 9. The correct answer is D. The patient most
light discriminatory touch, not vibratory sensa- likely has Graves disease-induced hyperthy-
tion, as is being tested in this case. roidism, the most prominent feature being
the exophthalmos seen in the image. In addi-

Endocrine
Answer C is incorrect. Merkel nerve endings
are non-encapsulated and found in all skin tion, her fatigue, sweating, palpitations, and
types (both hairy and hairless) and, along with increased appetite are also symptoms of hyper-
Meissner corpuscles, are believed to be respon- thyroidism. She could be treated with an agent
sible for discriminatory touch. such as propylthiouracil at this point, which
would help with the symptoms, although it
Answer E is incorrect. Ruffini corpuscles are will not reverse the eye changes. Unfortu-
spindle-shaped, encapsulated mechanorecep- nately, it can have the adverse effect of agran-
tors that are found in the soles of the feet and ulocytosis, and thus blood work needs to be
are responsible for transducing pressure. done when first prescribing this agent to a pa-
tient. Agranulocytosis can lead to infection and
8. The correct answer is F. Carcinoid syndrome
may be life threatening.
is an uncommon syndrome caused by tumors
of neuroendocrine origin. When these tumors Answer A is incorrect. Folic acid is not used in
metastasize to the liver, the high levels of sero­ the treatment of hyperthyroidism. It also does
tonin (the substance referenced in the ques- not cause agranulocytosis. It is often given to
tion stem) secreted by the tumor are no longer patients who are anemic. It is important that
metabolized by first-pass hepatic metabolism, pregnant women receive adequate amounts
leading to characteristic symptoms of diarrhea, of folic acid to reduce the incidence of neural
cutaneous flushing, asthmatic wheezing, and tube defects.
right-sided heart disease. Serotonin levels are
214 Section II: Organ Systems  •  Answers

Answer B is incorrect. Levothyroxine is used 11. The correct answer is A. This patient has
to treat hypothyroidism and would be contra- a presentation suspicious of MEN type 2B.
indicated in this patient as it would worsen her These patients have a classic triad of pheo-
current symptoms. chromocytoma, marfanoid body habitus, and
medullary thyroid carcinoma. This patient’s
Answer C is incorrect. Propranolol is also
paroxysms of severe headaches, anxiety, and
used to treat hyperthyroidism. However, it is
heart palpitations suggest a possible excess of
not associated with agranulocytosis. Its most
catecholamines, the hallmark of pheochro-
common adverse effect is fatigue.
mocytoma. The physical examination findings
Answer E is incorrect. Radioactive iodine is suggest a marfanoid body habitus. Because the
used to treat hyperthyroidism. The treatment patient has two of the three signs of MEN-2B,
is permanent and often results in hypothyroid- one must have a high level of suspicion for
High-Yield Systems

ism. It does not cause agranulocytosis. the third, medullary thyroid carcinoma. This
is a neoplasm of the medullary C cells, which
10. The correct answer is D. HIV-infected pa- produce calcitonin. Hence, the patient’s labo-
tients commonly have endocrine dysfunc- ratory findings would be significant for an el-
tion caused by HIV itself, other infections, or evated serum calcitonin level.
medications. In this patient, who has baseline
chronic adrenal insufficiency, ketoconazole Answer B is incorrect. An elevated serum cal-
can precipitate acute adrenal crisis, which of- cium is seen in patients with MEN type 1 or
ten presents with nonspecific symptoms such type 2A. These patients present with hypercal-
as nausea, vomiting, abdominal pain, fever, cemia in the setting of a parathyroid adenoma.
weakness, and fatigue. Ketoconazole is an im- However, neither of these two syndromes pre­
idazole whose mechanism of action is primar- sents with a marfanoid body habitus. Thus, the
ily inhibition of ergosterol synthesis. As ergos- patient’s physical exam findings should lead to
terol is a major component of the fungal cell a diagnosis of MEN-2B, in which serum cal-
membrane, ketoconazole is an effective anti- cium levels would be normal.
fungal agent. Unfortunately, CYP450 activity Answer C is incorrect. An insulinoma is a
and steroid synthesis are also inhibited by this pancreatic islet cell tumor that could be seen
agent, as seen in this patient. in a patient with MEN type 1 and would se-
Endocrine

Answer A is incorrect. Amphotericin B is an crete large amounts of insulin, causing hypo-


antifungal medication that can cause fevers glycemia with symptoms of nervousness, sweat-
and chills, hypotension, nephrotoxicity, and ar- ing, trembling, and weakness. MEN-1 consists
rhythmias. of pancreatic islet cell tumors (eg, gastrinomas,
vasoactive intestinal peptide tumors [VIPo-
Answer B is incorrect. Caspofungin is an anti- mas], and insulinomas), parathyroid adeno-
fungal medication that can cause GI upset and mas, and pituitary adenomas. This patient has
flushing. a history suggestive of MEN-2B, so an insuli-
Answer C is incorrect. Flucytosine is an anti- noma is unlikely and thus serum insulin levels
fungal medication that can cause nausea, vom- would be normal.
iting, diarrhea, and bone marrow suppression. Answer D is incorrect. An elevated serum
Answer E is incorrect. Trimethoprim- phosphate is not present in any of the MEN
sulfamethoxazole is used as treatment and syndromes. In patients with MEN-1 or MEN-
prophylaxis against Pneumocystis jiroveci pneu- 2A who have parathyroid adenomas, the serum
monia in patients with AIDS. It may cause ad- phosphate will actually be low. This is because
verse effects such as hypersensitivity reactions, these patients produce excess amounts of para-
nephrotoxicity, and leukopenia. thyroid hormone (PTH), which acts on the
kidneys to cause secretion of phosphate. The
patient in the vignette has MEN-2B, a syn-
Chapter 9: Endocrine  •  Answers 215

drome that does not produce any pathophysiol- nuclei. These cells secrete PTH, which raises
ogy in phosphate balance. Thus, this patient’s serum calcium levels in three ways: (1) it acts
serum phosphate levels would be normal. directly on bone to increase osteoclastic re-
sorption; (2) it acts directly on the kidney to
Answer E is incorrect. Although patients with
increase resorption of calcium and inhibit re-
MEN-2B have medullary thyroid carcinoma,
sorption of phosphate; and (3) it promotes GI
laboratory values of thyroglobulin will not be
absorption of calcium via increased levels of
elevated. This is because the neoplasm consists
activated vitamin D. Hyperparathyroidism can
of the medullary C cells, which produce cal-
lead to osteitis fibrosa cystica, which consists of
citonin. Serum thyroglobulin is a useful post-
fibrous tissue with cysts and hemorrhagic foci
operative tumor marker in papillary thyroid
in the bone marrow with a very thin cortex.
carcinoma, but it would not be elevated in this
patient. Answer A is incorrect. Adipose tissue in the

High-Yield Systems
parathyroid gland increases with age but does
12. The correct answer is C. The patient suffers not secrete hormones related to calcium regu-
from a megaloblastic anemia secondary to per- lation. The cells contain large vacuoles that
nicious anemia, as evidenced from the mean appear white on hematoxylin and eosin stain.
corpuscular volume >100 fL and the hemato-
Answer B is incorrect. Parathyroid oxyphil
crit <35%. The Schilling test is performed to
cells tend to occur in nodules and have abun-
determine whether the body is absorbing vita-
dant eosinophilic cytoplasm. They are larger
min B12, and to identify why if it is not. During
than chief cells and do not secrete PTH.
part 1 of the test, nonradiolabeled vitamin B12
is given to saturate tissue stores and to prevent Answer D is incorrect. Thyroid follicular cells
the binding of radioactive vitamin B12 in body are simple cuboidal cells that line colloid folli-
tissues. The absence of vitamin B12 excretion cles. They are responsible for the synthesis and
indicates that the intestines are not absorbing secretion of triiodothyronine (T3) and T4.
the vitamin normally. Results of part 2 of the
Answer E is incorrect. Thyroid C cells secrete
Schilling test indicate a problem with the se-
calcitonin, which decreases bone resorption
cretion of intrinsic factor (IF). Because IF is
of calcium, leading to a decrease in serum
secreted by the parietal cells, which are de-
calcium levels. C cells are distinguished by

Endocrine
stroyed by an autoimmune process, other sub-
their extensive clear cytoplasm. Think “C” for
stances secreted by these cells, such as hydro-
“Clear Cytoplasm.”
chloric acid, are also likely altered.
Answer A is incorrect. Bicarbonate is secreted 14. The correct answer is B. This vignette is most
to buffer the hydrochloric acid from the pari- consistent with the syndrome of inappropriate
etal cells; however, bicarbonate is not secreted ADH secretion (SIADH) due to a lung neo-
by the parietal cells themselves. plasm. ADH is normally secreted by the poste-
rior pituitary and stimulates the expression of
Answer B is incorrect. Gastrin is secreted by
aquaporins in the renal collecting ducts. This
G cells, not by parietal cells.
results in transport of water into the renal me-
Answer D is incorrect. Insulin is secreted by dulla from the ductal lumen, thus increasing
the pancreatic b cells in the islets of Langer- water reabsorption by the kidneys. When levels
hans, not by parietal cells of the stomach. of this hormone are inappropriately elevated,
excessive water retention results in hyponatre-
Answer E is incorrect. Trypsin is secreted in
mia, which can lead to seizures. ADH can be
the pancreas by acinar cells, not by the parietal
produced ectopically in the setting of malig-
cells.
nancy such as small cell lung cancer.
13. The correct answer is C. Parathyroid chief Answer A is incorrect. ACTH can be pro-
cells are small, pale cells with round central duced ectopically in the setting of small cell
216 Section II: Organ Systems  •  Answers

lung cancer. However, excessive levels of antibodies against one or more thyroid anti-
ACTH would result in Cushing syndrome, gens, leading to autoimmune-mediated de-
with symptoms of hypertension, weight gain, struction of the thyroid gland (as reflected by
moon facies, truncal obesity, buffalo hump, in- the lymphocytic infiltrate). It is characterized
sulin resistance, skin thinning/striae, osteopo- by reduced secretion of both free and total T4.
rosis, and amenorrhea. The vignette provides This leads to an increase in TSH levels due to
no signs or symptoms that would be consistent a decrease in negative feedback by T4 and T3
with Cushing syndrome. on the anterior pituitary.
Answer C is incorrect. PTH-related peptides Answer A is incorrect. In the setting of pri-
can be produced ectopically in the setting of mary hypothyroidism, both total and free T4
malignancy and are associated with a variety of levels should be decreased rather than in-
neoplasms, including squamous cell lung can- creased. An elevated TSH level in the setting
High-Yield Systems

cer, breast cancer, renal cell carcinoma, and of high total and free T4 levels could be seen
multiple myeloma. However, excessive levels in a TSH-secreting pituitary adenoma or, less
of PTH would result in hypercalcemia, and commonly, in patients with thyroid hormone
the vignette does not provide any indication resistance syndrome. A TSH-secreting ad-
that would be consistent with this condition. enoma, however, would result in symptoms of
hyperthyroidism, such as weight loss, hyperac-
Answer D is incorrect. Hyperreninemia does
tivity, heat intolerance, and palpitations.
not typically occur as a paraneoplastic syn-
drome and is most commonly caused by re- Answer B is incorrect. Both total and free T4
nal artery stenosis. Renin is normally released levels should be decreased in the setting of
by the kidneys upon sensing decreased blood primary hypothyroidism. An elevated T4 level
pressure. Renin plays a role in the conversion is usually associated with hyperthyroidism,
of angiotensinogen to angiotensin I, which is not hypothyroidism. Free and total T4 levels
then converted by angiotensin-converting en- should vary in opposite directions only when
zyme (ACE) into angiotensin II. Angiotensin there is a change in the amount of thyroxine-
II stimulates the release of aldosterone from binding globulin (TBG), which binds thy-
the adrenal glands. Therefore, hyperreninemia roid hormone in the blood. A good example
leads to increased levels of angiotensin II and is in pregnancy, where TBG levels increase
Endocrine

aldosterone, resulting in hypertension and hy- in response to estrogen, thus decreasing the
pokalemia. The vignette does not mention any amount of free T4. However, TSH levels are
signs or symptoms of hypokalemia, such as high due to decreased negative feedback to the
nausea, vomiting, muscle weakness, and car- pituitary, and therefore the total T4 level will
diac arrhythmias. be high.
Answer E is incorrect. Tumor necrosis factor- Answer D is incorrect. In Hashimoto thyroid-
a (TNF-a) can be produced ectopically in the itis, both total and free T4 levels should be de-
setting of malignancy. Increased TNF-a secre- creased and TSH levels should be increased
tion can be caused by the same cancers that due to a decrease in negative feedback on the
produce PTH-related peptide, including squa- pituitary gland.
mous cell lung cancer, breast cancer, renal
Answer E is incorrect. There are three types
cell carcinoma, and multiple myeloma. TNF-
of hypothyroidism: primary, secondary, and
a is associated with fever and cachexia, among
tertiary. In primary hypothyroidism, such as
many other findings.
Hashimoto thyroiditis, the cause is malfunc-
tioning of the thyroid gland which leads to re-
15. The correct answer is C. The vignette de-
duced free and total T4 levels and increased
scribes a classic history for primary hypo-
TSH levels. In secondary hypothyroidism
thyroidism caused by Hashimoto thyroiditis.
(much less prevalent than the primary form),
Hashimoto thyroiditis is caused by circulating
the pituitary gland does not produce sufficient
Chapter 9: Endocrine  •  Answers 217

TSH, leading to decreased TSH levels in ad- ing, and chewing. Ataxia, or poor coordina-
dition to low total and free T4 levels. Tertiary tion, is a known adverse effect of lithium but
hypothyroidism (also rare) presents with low not neuroleptics.
levels of TSH and thyroid hormones, but is
Answer B is incorrect. Typical antipsychotics
caused by a hypothalamic abnormality leading
have an antimuscarinic effect that can cause
to decreased thyrotrophin-releasing hormone
constipation, not diarrhea.
production in the hypothalamus.
Answer D is incorrect. Typical antipsychotics
16. The correct answer is C. The patient presents have anti-a-receptor effects that can cause hy-
with auditory hallucinations, which are sug- potension, not hypertension.
gestive of schizophrenia. Schizophrenia is a
Answer E is incorrect. Typical antipsychotics
psychiatric disorder characterized by positive
have an antihistamine effect that causes seda-

High-Yield Systems
symptoms such as delusions, hallucinations
tion, not insomnia.
(mainly auditory), disorganized thought, and
disorganized behavior, and negative symptoms 17. The correct answer is D. This patient has
that include flat or blunted affect, apathy, and Hashimoto thyroiditis, an autoimmune disor-
anhedonia. Although not fully understood, it der in which patients have antibodies attacking
is believed that schizophrenia is related to in- thyroglobulin, thyroid peroxidase, or another
creased dopamine activity in certain neural part of the thyroid gland or thyroid hormone
pathways. Consequently, typical antipsychot- synthesis pathway. Patients with Hashimoto
ics, such as haloperidol, that block dopamine thyroiditis have a 20 times greater prevalence
D2 receptors, have been used. Unfortunately, of celiac disease and type 1 DM than the gen-
dopamine antagonism has adverse effects, eral population.
such as interfering with the normal feedback
inhibition of dopamine in the hypothalamic- Answer A is incorrect. This patient has Hashi-
pituitary axis. Since dopamine inhibits prolac- moto thyroiditis. She would not have Graves
tin secretion from the anterior pituitary, the disease as well.
blockade of dopamine receptors may cause hy- Answer B is incorrect. Addison disease does
perprolactinemia and galactorrhea. Although have a high prevalence in patients with Hashi-
the cause is unknown, neuroleptic malignant moto thyroiditis. However, osteoarthritis is not

Endocrine
syndrome (NMS) is another important adverse an autoimmune disease.
event, and is believed to be due to dopamine
antagonism. NMS is a neurologic emergency, Answer C is incorrect. Rheumatoid arthritis
characterized by mental status change, rigidity, and vitiligo are both autoimmune diseases, but
fever, and dysautonomia. they do not have as high of an association with
Hashimoto thyroiditis as do type 1 DM and
Answer A is incorrect. Antipsychotics may celiac disease.
cause an imbalance in dopamine and musca-
rinic receptor antagonism, which results in Answer E is incorrect. Whipple disease is
extrapyramidal adverse effects, such as dysto- caused by Tropheryma whippelii.
nia, akinesia, akathisia, and tardive dyskinesia.
Dystonia is characterized by sustained and pro- 18. The correct answer is B. Conn syndrome,
longed contraction of agonist and antagonist which is also known as primary hyperaldoster­
muscles producing abnormal postures. Aki- onism, usually results from a solitary
nesia is the absence of movement, while aka­ aldosterone-secreting adenoma of the adrenal
thisia refers to the feeling of restlessness that cortex. In patients with Conn syndrome, the
is relieved by movement. Tardive dyskinesia high levels of aldosterone cause sodium reten-
commonly presents as involuntary choreiform tion; this leads to hypertension and excess po-
movements of the lower face, characterized by tassium excretion, which likely caused the pa-
rhythmic protrusion of the tongue, lip smack- tient’s muscle cramping. Metabolic alkalosis
218 Section II: Organ Systems  •  Answers

is another finding of Conn syndrome. The glycogen storage in the liver and glycogen and
plasma renin level is decreased in patients with protein synthesis in muscle.
this condition.
Answer B is incorrect. a-Glucoside inhibitors
Answer A is incorrect. Addison disease, un- such as acarbose and miglitol are carbohydrate
like Conn syndrome, results from a defi- analogs. They inhibit intestinal brush border
ciency of aldosterone. Signs of this condition a-glucosidase and decrease sugar hydrolysis.
include hypotension, skin hyperpigmentation,
Answer D is incorrect. Glitazones such as rosi-
hyponatremia, and hyperkalemia. Addison dis-
glitazone and pioglitazone are thiazolidinedio-
ease does not explain the clinical scenario de-
nes used to increase sensitivity to insulin. They
scribed.
stimulate peroxisome proliferator-activated
Answer C is incorrect. Cushing syndrome oc- receptor-gamma, leading to increased glucose
High-Yield Systems

curs from excess secretion of cortisol. The clin- uptake in muscle and adipose tissue.
ical picture is marked by hypertension, weight
Answer E is incorrect. Sulfonylureas such as
gain, moon facies, truncal obesity, hypergly-
glyburide and glipizide stimulate the release of
cemia, striae, and osteoporosis. Cushing syn-
endogenous insulin stores. They close potas-
drome does not explain the clinical scenario
sium channels in b cells, leading to an influx
described.
of calcium, cell depolarization, and insulin re-
Answer D is incorrect. Hyperparathyroidism lease.
results in elevated calcium levels. Depression,
constipation, and fractures are symptoms that 20. The correct answer is B. This patient has
are associated with hyperparathyroidism. Other symptoms of acromegaly, which is caused by
commonly associated symptoms include kid- increased release of growth hormone (GH),
ney stones, chronic renal insufficiency, gall- most likely due to a pituitary adenoma. The
stones, pancreatitis, weakness, fatigue, and val- pituitary sits immediately behind the optic
vular calcifications. Hyperparathyroidism does chiasm, and increases in its size may lead to
not explain the clinical scenario described. impingement on the nasal tracts that cross in
the midline of the chiasm, causing bi-temporal
Answer E is incorrect. Pheochromocytoma
hemianopia.
usually results from neoplasms of the adrenal
Endocrine

medulla that secrete catecholamines. Classic Answer A is incorrect. Loss of vision only in
symptoms of Pheochromocytoma include the the right eye is caused by lesions of the right
“5 Ps”: elevated blood Pressure, Pain (head- eye or right optic nerve that are distal to the
ache), Perspiration, Palpitations, and Pallor optic chiasm.
with diaphoresis. Pheochromocytoma does not Answer C is incorrect. Left homonymous
explain the above patient presentation. hemianopia is caused by a lesion of the right
optic tract that is just proximal to the optic chi-
19. The correct answer is C. Though its mecha-
asm.
nism of action is poorly understood, metfor-
min, a biguanide, is thought to decrease serum Answer D is incorrect. Left-upper-quadrant
glucose levels by stimulating glycolysis in pe- anopia is caused by a lesion in Meyer loop in
ripheral tissues and decreasing hepatic gluco- the temporal lobe on the right.
neogenesis. Biguanides may cause lactic acido- Answer E is incorrect. Left-lower-quadrant an-
sis. opia is caused by a lesion in the dorsal optic
Answer A is incorrect. Insulin and insulin an- radiation that goes through the parietal lobe.
alogs bind the insulin receptor on cell mem-
branes, subsequently activating a tyrosine 21. The correct answer is D. This patient has
kinase that leads to the absorption of glucose symptoms consistent with congenital adrenal
into the cell. Insulin binding leads to increased hyperplasia, a group of syndromes involving
Chapter 9: Endocrine  •  Answers 219

deficiency of enzymes produced in the adre- 22. The correct answer is B. Acute high-dosage
nal glands involved in aldosterone production glucocorticoid treatment can cause a change
for salt retention (zona glomerulosa), cortisol in electrolyte levels by their cross-reactivity to
production (zona fasciculata), and androgen the mineralocorticoid receptors, thus caus-
production for sexual development (zona re- ing sodium retention and potassium deple-
ticularis). 21-Hydroxylase is part of the min- tion. Hypokalemia is manifested on ECG as a
eralocorticoid- and glucocorticoid-producing U wave, which is a small wave that follows the
pathways. Deficiency of this enzyme decreases T wave.
aldosterone and cortisol levels, causing de-
Answer A is incorrect. Delta waves are the
creased blood pressure. Additionally, the block
result of accessory conduction pathways, not
in the production of these hormones shunts
hypokalemia. They are often seen in Wolff-
additional intermediary products toward sex
Parkinson-White syndrome and look like

High-Yield Systems
hormone production, resulting in overproduc-
slurred upstrokes in the QRS complex.
tion of sex hormones, especially testosterone;
this results in masculinization of the fetus’ Answer C is incorrect. Peaked T waves on
phenotypic characteristics. ECG are suggestive of hyperkalemia. In this
case, the more likely result is hypokalemia in-
Answer A is incorrect. 5a-Reductase converts
stead of hyperkalemia; thus, T waves would
testosterone into dihydrotestosterone and is
not be expected to be peaked.
responsible for secondary sexual development
in males (ie, external genitalia). A deficiency Answer D is incorrect. PR segment elonga-
in this enzyme would not affect mineralocor- tion is indicative of increased time for conduc-
ticoid, glucocorticoid, or testosterone produc- tion between the atria and the ventricles. This
tion, but infants born XY would have female is typically seen with first-degree heart block,
genitalia at birth. At puberty, genitalia would which is not caused by glucocorticoid treat-
become masculinized as a result of the surge ment.
of testosterone that occurs during this time. Answer E is incorrect. ST segment eleva-
This deficiency cannot account for hypoten- tion suggests transmural myocardial ischemia,
sion and virilization. which is not an adverse effect of glucocorti-
Answer B is incorrect. 17a-Hydroxylase defi- coids.

Endocrine
ciency is a rare disorder that blocks glucocor-
ticoid and sex hormone production, resulting 23. The correct answer is B. This is a case of
in excess production of mineralocorticoid, primary hyperaldosteronism, or Conn syn-
namely aldosterone. This syndrome would drome. It is caused by increased aldosterone
present with sodium retention and hyperten- secretion, and an adrenal adenoma is the
sion, as well as a default genotypic female gen- most common cause, as in this case. In ad-
der (lack of müllerian inhibiting factor) with dition to the symptoms seen in this patient,
an immature female phenotype caused by a Conn syndrome is associated with failure to
lack of sex hormone production. suppress aldosterone with salt loading. Patho-
logic examination would reveal a single, well-
Answer C is incorrect. 11b-Hydroxylase defi- circumscribed adenoma with lipid-laden clear
ciency also affects the mineralocorticoid- and cells. In terms of primary venous drainage,
glucocorticoid-producing pathways (similar the right adrenal gland is drained via the right
to 21-hydroxylase deficiency). The effects of adrenal vein, which flows directly into the in-
11b-hydroxylase deficiency occur later in the ferior vena cava (IVC). Thus a right-sided hy-
adrenal pathways, and the result is the build- perfunctioning adrenal adenoma is drained
up of a precursor (11-deoxycorticosterone) that via the right adrenal vein into the IVC. In con-
can act as a weak mineralocorticoid, producing trast, the left adrenal gland is drained via the
masculinization, hypertension, and salt reten- left adrenal vein into the left renal vein, which
tion. then flows into the IVC.
220 Section II: Organ Systems  •  Answers

Answer A is incorrect. The abdominal aorta cholecalciferol in the kidney, leading to hy-
plays no role in the vascular drainage of any percalcemia. Hypophosphatemia is caused by
organ but rather provides arterial supply to the PTH inhibition of renal phosphate reabsorp-
abdominal organs, including the kidneys and tion in the proximal tubule.
adrenals glands.
Answer A is incorrect. These laboratory values
Answer C is incorrect. The portal vein is supe- are consistent with hypoparathyroidism, com-
rior and anterior to the adrenal and renal vas- monly caused by surgical removal of the thy-
culature and is not involved in the drainage of roid or congenital absence, such as in a patient
either of the adrenal glands. Instead, it drains with DiGeorge syndrome. The low PTH level
most of the GI tract down to the rectum into leads to decreased 1,25-dihydroxycholecal-
the liver. ciferol production, decreased serum calcium
(both through decreased uptake and decreased
High-Yield Systems

Answer D is incorrect. The right gonadal vein


reabsorption), and increased phosphate reten-
drains the testes or ovaries directly into the in-
tion. Patients with hypoparathyroidism gen-
ferior vena cava but does not drain the right
erally present with increased neuromuscular
adrenal gland in either sex.
excitability and tetany, symptoms of severe hy-
Answer E is incorrect. Drainage of the right pocalcemia.
adrenal gland and hence a right-sided adrenal
Answer B is incorrect. These laboratory values
adenoma does not flow through the right renal
are consistent with humoral hypercalcemia of
vein, but instead the adrenal vein flows directly
malignancy, caused by a PTH-related peptide
into the inferior vena cava.
(PTHrP) secreted by some types of malignant
24. The correct answer is E. This patient has tumors, such as breast or bronchogenic squa-
symptoms of nephrolithiasis or kidney stones, mous cell carcinoma. PTHrP has all the same
a common symptom of hyperparathyroid- physiologic functions of PTH, including in-
ism. The hypercalcemia and imaging findings creased bone resorption, increased renal cal-
suggest a parathyroid adenoma, which is a cium reabsorption, and increased phosphate
benign growth of the parathyroid gland. Para- excretion. Laboratory findings are similar to
thyroid adenomas are three times more often those of primary hyperparathyroidism, ex-
in women than men and generally presenting cept that in this case the increased calcium is
Endocrine

during middle age. They are the most com- caused by PTHrP, not PTH. Thus PTH levels
mon cause of primary hyperparathyroidism. are low due to the hypercalcemia, but all other
Parathyroid adenomas are often asympto­matic, laboratory values are the same.
but may present with the classic tetrad of Answer C is incorrect. These laboratory values
“stones, bones, abdominal groans, and psychic are associated with secondary hyperparathyroid-
moans,” including nephrolithiasis (stones); os- ism, commonly associated with chronic renal
teoporosis or osteitis fibrosa cystica (bones); failure. This is referred to as “secondary” be-
constipation, nausea, vomiting, ulcers, pan- cause other changes in body chemistry precede
creatitis, or gallstones (abdominal groans); and the increase in PTH. In the case of chronic
depression, lethargy, and eventually seizures renal failure, a decreased glomerular filtration
(psychic moans). Imaging studies are often rate leads to decreased phosphate excretion
used for diagnosis. Treatment is surgical. Pre- and, ultimately, hyperphosphatemia. Hyper-
operative scintigraphy is useful in distinguish- phosphatemia in turn decreases a-hydroxylase
ing adenomas from parathyroid hyperplasia, activity, lowering 1,25-dihydroxycholecalciferol
in which more than one gland would demon- production and decreasing serum calcium.
strate increased uptake. High levels of PTH In addition, 1-25-dihydroxycholecalciferol is
increase calcium levels by increasing renal already lowered due to poor renal function.
calcium absorption in the distal tubule and by Serum phosphate complexes with calcium in
stimulating the production of 1,25-dihydroxy- the blood, which further decreases the serum
Chapter 9: Endocrine  •  Answers 221

calcium level. This low calcium level stimu- pressure points such as the elbows, knees, and
lates an increase in PTH production. Individu- knuckles.
als with renal failure are generally older than
Answer B is incorrect. Conn syndrome is de-
this patient and have other metabolic imbal-
fined as a chronic excess of aldosterone secre-
ances, such as acidosis, hyperkalemia, and hy-
tion from an aldosterone-secreting adenoma
pertension.
in one adrenal gland. Unlike patients with
Answer D is incorrect. These laboratory val- Addison disease, people with Conn syndrome
ues are consistent with pseudohypoparathy- would have, among other findings, hyper-
roidism, also known as type Ia Albright’s he- tension, increased serum sodium, decreased
reditary osteodystrophy. This is an autosomal serum potassium, low serum renin, and in-
dominant disease caused by a defective Gs pro- creased serum aldosterone.
tein in kidney and bone, leading to resistance

High-Yield Systems
Answer C is incorrect. Cushing syndrome is a
to PTH. The inability to respond to PTH re-
name for any condition that causes an excess
sults in hypocalcemia and hyperphosphatemia,
of glucocorticoids, such as cortisol. Early signs
and the low calcium levels stimulate release of
include hypertension and weight gain, which
more PTH. Patients with this disease tend to
progress to truncal obesity, moon facies, and
have short stature, a round face, and shortened
a “buffalo hump” from accumulation of fat in
fourth and fifth digits (brachydactyly), along
the posterior neck and back.
with symptoms of hypocalcemia such as tetany
and positive Chvostek and Trousseau signs. Answer D is incorrect. A pheochromocytoma
Impaired mentation is also found in about is neoplasm of the chromaffin cells (neural
half of patients with pseudohypoparathyroid- crest derivatives that synthesize and release
ism. This patient’s presentation is not consis- mostly catecholamines). Most cases present
tent with this disease. In addition, this condi- with hypertension, headache, tremor, sweat-
tion generally presents before age 42 years, and ing, and a sense of apprehension. The hyper-
hypocalcemia, not hypercalcemia, would be tension may occur in isolated bursts or chroni-
found. cally and may be associated with palpitations,
Answer E is incorrect. Waterhouse-
25. The correct answer is A. This patient has Ad-
Friderichsen syndrome is characterized by

Endocrine
dison disease (also known as primary adreno-
rapidly developing adrenocortical insuffi-
cortical deficiency), which is adrenal insuffi-
ciency (days to weeks) accompanied by an
ciency (remember the “GFR” of the adrenal
overwhelming bacterial infection (classically
gland: Glomerulosa is the outer layer, which
Neisseria meningitidis), rapidly progressive hy-
makes mineralocorticoids, namely aldoster­
potension, shock, disseminated intravascular
one; Fasciculata is the middle layer, which
coagulation (DIC), and widespread purpura.
makes glucocorticoids such as cortisol; and
Reticularis is the inner layer, which makes 26. The correct answer is A. This patient has
sex steroids such as estrogens and androgens). Graves disease, an autoimmune disorder re­
Decreased levels of aldosterone lead to hypo- sulting from IgG-type autoantibodies to the
tension, increased levels of potassium, and TSH receptor. The three classic findings as-
decreased levels of sodium, chloride, and bi- sociated with Graves disease are hyperthyroid-
carbonate. Decreased levels of cortisol lead to ism, ophthalmopathy, and dermopathy/pretib-
decreased glucose levels. Decreased cortisol ial myxedema.
also leads to increased levels of ACTH (as the
pituitary attempts to increase stimulation of Answer B is incorrect. Hashimoto thyroiditis
the adrenal gland), causing hyperpigmenta- is an autoimmune disorder characterized by
tion (from a precursor hormone to ACTH that antibodies attacking thyroglobulin or thyroid
stimulates melanocytes), especially at physical peroxidase (the two most common autoanti-
bodies in these patients) or antibodies against
222 Section II: Organ Systems  •  Answers

another part of the thyroid or thyroid hormone “Salt, Sugar, and Sex” for the layers of the ad-
synthesis pathway. Although some cases of renal cortex and their respective products, with
Hashimoto thyroiditis may present as a tran- “salt” corresponding to the outer zona glo-
sient hyperthyroidism (with symptoms includ- merulosa, “sugar” corresponding to the middle
ing palpitations and increased metabolic rate) zona fasciculata, and “sex” corresponding to
from an initial disruption of thyroid follicles, the inner zona reticularis. 11b-Hydroxylase
the majority of cases present with signs and deficiency is another form of congenital adre-
symptoms of hypothyroidism, such as intoler- nal hyperplasia and causes decreased cortisol,
ance to cold weather, weight gain, and mental aldosterone, and corticosterone production
and physical slowness. and excess production of sex hormones and
11-deoxycorticosterone. The symptoms are
Answer C is incorrect. Plummer disease is
similar to those of 17a-hydroxylase deficiency,
characterized by a nodular goiter that has a hy-
High-Yield Systems

manifesting as hypertension and hypokalemia;


perfunctioning nodule, causing hyperthyroid-
however, virilization would also be present.
ism. As opposed to Graves disease, Plummer
disease is not accompanied by ophthalmopathy Answer A is incorrect. The capsule does not
or dermopathy/pretibial myxedema. produce any hormones.
Answer D is incorrect. Iodine deficiency Answer B is incorrect. The medulla produces
causes hypothyroidism, manifested with signs catecholamines (epinephrine and norepineph-
and symptoms that include intolerance to cold rine); neither 17a-hydroxylase nor 21b-hydrox-
weather, weight gain, and mental and physical ylase is required for the synthesis of catechol-
slowness. amines.
Answer E is incorrect. Papillary carcinoma of Answer C is incorrect. Cortisol is produced
the thyroid, the most common form of thyroid in the zona fasciculata of the adrenal cortex.
cancer, usually presents as an asymptomatic Cortisol synthesis requires 21b-hydroxylase and
thyroid nodule with signs of obstruction from 17a-hydroxylase and is therefore deficient in
the tumor such as hoarseness, cough, dyspha- this patient.
gia, or dyspnea or a cervical lymph node mass
Answer E is incorrect. Sex hormones are pro-
(as opposed to symptoms of hyper- or hypothy-
duced in the zona reticularis. Synthesis of the
Endocrine

roidism). Radiation is a common cause of thy-


sex hormones requires 17a-hydroxylase, but
roid cancer.
not 21b-hydroxylase, and is therefore deficient
in this patient, leading to her primary amenor-
27. The correct answer is D. 17a-Hydroxylase
rhea. 11b-Hydroxylase deficiency is another
deficiency, a form of congenital adrenal hy-
form of congenital adrenal hyperplasia, and
perplasia, is characterized by deficits in glu-
causes decreased cortisol, aldosterone, and cor-
cocorticoid and sex steroid synthesis. This is
ticosterone production and excess production
coupled with increased mineralocorticoid
of sex hormones and 11-deoxycorticosterone.
(aldosterone) production due to the shunt-
The symptoms are similar to those of 17a-­
ing of precursors, such as pregnenolone and
hydroxylase deficiency, manifesting as hyper-
progesterone, through mineralocorticoid path-
tension and hypokalemia; however, virilization
ways. The increased levels of aldosterone lead
would also be present.
to increased sodium retention and potassium
excretion (hence, the patient’s hypertension
28. The correct answer is E. This patient has ke-
and hypokalemia). The low sex steroid lev-
toacidosis due to uncontrolled type 1 DM.
els manifest clinically as a female phenotype
The stress caused by the viral upper respira-
with no sexual maturation. Aldosterone is pro-
tory infection likely increased her insulin re-
duced in the zona glomerulosa. Aldosterone
quirements, precipitating fat breakdown and
synthesis requires 21b-hydroxylase but not
ketogenesis. These are often the presenting
17a-­hydroxylase. Remember the mnemonic
events of previously undiagnosed type 1 DM:
Chapter 9: Endocrine  •  Answers 223

The patient has a subclinical disease that be- Answer E is incorrect. Label E points to the
comes a clinical disease in the setting of an posterior inferior cerebellar artery (PICA). A
acute comorbid illness. Signs and symptoms deficit in the PICA would lead to lateral med-
of ketoacidosis include rapid deep breathing ullary syndrome.
(Kussmaul’s respirations), nausea/vomiting,
abdominal pain, hyperthermia, psychosis/de- 30. The correct answer is E. Thyroid hormones
mentia, and dehydration. A fruity odor to the act via nuclear hormone receptors. On binding
breath from the ketone bodies is commonly with its ligand, the receptor translocates from
present. Labs demonstrate hyperglycemia, in- the cytoplasm into the cell nucleus, and the
creased ketone levels, leukocytosis, and a met- ligand-receptor complex acts as a transcription
abolic acidosis. Type 1 DM is associated with factor. This results in gene transcription and
HLA-DR3 and -DR4. new protein synthesis. Other hormones that

High-Yield Systems
act through nuclear steroid hormone receptors
Answer A is incorrect. HLA-A3 is associated
include cortisol, aldosterone, vitamin D, testos-
with hemochromatosis.
terone, estrogen, and progesterone.
Answer B is incorrect. HLA-B27 is associated
Answer A is incorrect. Glucagon acts via
with a number of inflammatory conditions,
G-protein receptors located in the plasma
including psoriasis, ankylosing spondylitis,
membrane. G proteins are activated, and the a
Rei­ter syndrome, and inflammatory bowel dis-
subunit activates adenylate cyclase. Glucagon
ease.
does not act via nuclear hormone receptors.
Answer C is incorrect. HLA-DQ2 and -DQ8
Answer B is incorrect. Histamine and vaso-
are associated with celiac disease.
pressin activate phospholipase C, resulting in
Answer D is incorrect. HLA-DR2 and -DR3 the cleavage of phosphatidylinositol diphos-
are associated with lupus fibrillation. phate into inositol triphosphate and diacyl-
glycerol. It does not act via nuclear hormone
29. The correct answer is A. The image depicts receptors.
the circle of Willis. The artery labeled A is
Answer C is incorrect. Intracellular insulin
the anterior cerebral artery (ACA), which sup-
acts via a tyrosine kinase cascade and not via
plies the medial surface of the brain, the area
nuclear hormone receptors.

Endocrine
responsible for the contralateral leg and foot
areas of the motor and sensory cortices. Thus, Answer D is incorrect. Norepinephrine acts
a lesion in the artery would lead to deficits in by binding to and activating adrenergic recep-
contralateral motor function of the leg and tors. It does not bind to nuclear hormone re-
foot. ceptors.
Answer B is incorrect. Label B points to the
31. The correct answer is C. The hypophyseal
middle cerebral artery (MCA). A deficit in the
portal system allows delivery of releasing hor-
MCA would cause contralateral face and arm
mones (GH-releasing hormone, corticotropin-
paralysis and sensory loss.
releasing hormone, thyrotropin-releasing hor-
Answer C is incorrect. Label C points to the mone, and gonadotropin-releasing hormone)
posterior cerebral artery (PCA). A deficit in the and inhibiting hormones (somatostatin and
PCA would cause contralateral hemianopia dopamine) from the neuroendocrine cells in
with macular sparing. the hypothalamus directly to the anterior pi-
tuitary gland, where they control the produc-
Answer D is incorrect. Label D points to the
tion and release of tropic hormones into the
anterior inferior cerebellar artery (AICA). A
systemic circulation. Portal systems consist of
deficit in the AICA would lead to lateral infe-
two capillary beds directly connected by veins;
rior pontine syndrome.
these keep hormones from being diluted be-
fore reaching the pituitary. Hormones are de-
224 Section II: Organ Systems  •  Answers

livered to the pituitary gland in high concen- lemia. Although not necessary as a part of the
trations and are not needed elsewhere in the question stem, as are all three characteristic
body. tumors, the USMLE often will mention that
multiple family members have been affected
Answer A is incorrect. The cells of the pitu-
by similar or a variety of tumors. This is a clear
itary gland do not carry out any processing of
indication that the disorder is most likely auto-
hypothalamic hormones. The posterior pitu-
somal dominant in inheritance pattern.
itary serves as a storage site for ADH and oxy-
tocin, which are synthesized in the hypotha- Answer A is incorrect. Fabry disease is an X-
lamic supraoptic and paraventricular nuclei linked recessive disorder characterized by pe-
and then transported to the posterior pituitary ripheral neuropathy, cardiovascular disease,
via the supraopticohypophyseal tract. and angiokeratomas. It results from a defi-
ciency in the enzyme a-galactosidase A.
High-Yield Systems

Answer B is incorrect. The supraopticohy-


pophyseal tract is the conduit through which Answer B is incorrect. Familial dysautonomia
ADH and oxytocin, both produced in the hy- is an autosomal recessive disorder in which in-
pothalamus, are delivered to the posterior pitu- dividuals have a poorly developed autonomic
itary for storage and later release. Note that this nervous system, resulting in such manifesta-
circuit does not involve any vascular structures. tions as labile blood pressure, reduced produc-
tion of tears, and an inappropriate response to
Answer D is incorrect. Hormones are deliv-
stress.
ered from the anterior pituitary to the hypo-
thalamus via the systemic circulation, just as Answer D is incorrect. MELAS (Mitochon-
they are delivered to other tissues. This process drial myopathy, Encephalopathy, Lactic Aci-
does not involve the hypophyseal portal sys- dosis, and Stroke-like episodes) is a disorder
tem. characterized by mitochondrial inheritance.
Mitochondrial inheritance is characterized by
Answer E is incorrect. Tropic hormones of
transmission of a trait to the offspring of an af-
the hypothalamic-pituitary axis (GH, ACTH,
fected mother but never to the offspring of an
TSH, luteinizing hormone (LH), and follicle-
affected father, because mitochondria are in-
stimulating hormone) are synthesized and
herited from only the egg and not the sperm.
secreted by the anterior pituitary gland in re-
Endocrine

sponse to hormonal stimulation by the hypo- Answer E is incorrect. Hypophosphatemic


thalamus, but they are not produced in the rickets is inherited in an X-linked dominant
hypothalamus. These hormones are secreted fashion.
from the pituitary into the systemic circulation.
33. The correct answer is A. This patient has
32. The correct answer is C. The patient has tu- symptoms of a glucagonoma, a rare glucagon-
mors involving the “3 Ps” of MEN type 1, secreting tumor that can cause hyperglycemia,
also known as Wermer syndrome. Her galac- diarrhea, and weight loss. The hyperglycemia
torrhea and bilateral hemianopsia likely are seen in these patients will not respond to oral
due to a Prolactin-secreting pituitary tumor hypoglycemic agents because of the uncon-
encroaching on her optic chiasm, and her hy- trolled excess glucagon production that con-
percalcemia likely is due to a Parathyroid ad- tinues despite increased insulin levels. Glu-
enoma. Her recurrent duodenal ulcers are a cagonomas also are associated with necrolytic
manifestation of a gastrin-secreting tumor that migratory erythema, a skin rash consisting of
frequently is located in the Pancreas, as seen painful, pruritic erythematous papules that
in Zollinger-Ellison syndrome. The genetic blister, erode, and crust over.
inheritance of MEN-1 is autosomal dominant,
Answer B is incorrect. An insulinoma would
as is the inheritance of hypokalemic periodic
be expected to cause hypoglycemia rather than
paralysis, a disorder characterized by episodes
hyperglycemia.
of paralysis/weakness accompanied by hypoka-
Chapter 9: Endocrine  •  Answers 225

Answer C is incorrect. Pancreatic adenocar- not directly affect the conversion of testoste­
cinoma could explain this patient’s weight rone to estradiol.
loss and abdominal mass. Adenocarcinoma of
Answer E is incorrect. LH acts on the theca
the pancreatic tail also could account for his
cells to produce testosterone. LH surges also
hyperglycemia, as these lesions sometimes in-
trigger ovulation.
filtrate and compromise the pancreatic islets,
resulting in insulin deficiency. However, this 35. The correct answer is D. This patient is most
patient’s painful skin rash would not be consis- likely suffering from hypoglycemia due to his
tent with pancreatic adenocarcinoma. Rather, sulfonylurea medication, glyburide. Symptoms
these patients may manifest migratory throm- of hypoglycemia can be categorized as neu-
bophlebitis (Trousseau syndrome), a condi- roglycopenic (resulting from the lack of glu-
tion in which the extremities become red and cose to the CNS) or neurogenic (adrenergic

High-Yield Systems
tender. symptoms resulting from the catecholamine
Answer D is incorrect. The patient’s age response to hypoglycemia). Neuroglycope-
makes less likely, though does not exclude, an nic symptoms include weakness, confusion,
initial presentation of type 1 DM. However, drowsiness, dizziness, syncope, difficulty speak-
the pancreatic mass and rash cannot be ex- ing and blurry vision. Neurogenic symptoms
plained by the diagnosis of type 1 DM. include diaphoresis, hunger, tingling, tremor,
palpitations, chest pain, and anxiety. Nonse-
Answer E is incorrect. VIPomas are rare endo-
lective b-blockers, such as propranolol, can
crine tumors that arise in the pancreatic islets.
block the sympathetic surge that triggers the
Although VIPomas can cause diarrhea, hyper-
body’s response to insulin-induced hypoglyce-
glycemia, and a pancreatic mass on CT, they
mia. This can cause “hypoglycemia unaware-
are not associated with the rash observed in
ness” and is especially seen in patients with
this patient.
diabetes. This is mainly a b2-mediated effect,
as noncardiac b2-receptors are the trigger for
34. The correct answer is B. Testosterone is pro-
catecholamine-induced glycogenolysis. Selec-
duced in the theca cells. From the theca cells,
tive b1 blockers, such as metoprolol, are there-
it diffuses into the granulosa cells. There,
fore less likely to cause this effect.
follicle-stimulating hormone (FSH) stimu-

Endocrine
lates the enzyme aromatase within the cells to Answer A is incorrect. Enalapril is an ACE
convert testosterone to 17b-estradiol. In males, inhibitor. Its most common adverse effects are
FSH is also responsible for inducing Sertoli’s cough, hypotension, and edema. Dizziness
cells to secrete inhibin, which works in a nega- and syncope have been associated with ACE
tive-feedback fashion to inhibit FSH formation. inhibitors, but agitation and confusion have
not been reported. Propranolol is therefore the
Answer A is incorrect. Estrogen causes the en-
better answer.
dometrium to proliferate. It does not stimulate
the granulosa cells to convert testosterone to Answer B is incorrect. Hydrochlorothiazide is
estradiol. a thiazide diuretic. Its adverse effects include
hyperglycemia, hyperlipidemia, hyperurice-
Answer C is incorrect. ACTH stimulates the
mia, and hypercalcemia. It is not associated
enzyme desmolase. Desmolase is responsible
with hypoglycemia unawareness.
for converting cholesterol to pregnenolone, a
key step in the synthesis of adrenocortical ste- Answer C is incorrect. Metoprolol is a b1-
roids. ACTH does not stimulate the granulosa selective blocker and is therefore less likely to
cells to convert testosterone to estradiol. cause hypoglycemia unawareness.
Answer D is incorrect. Gonadotropin- Answer E is incorrect. Triamterene is a potas-
releasing hormone stimulates the production of sium-sparing diuretic. Its most common ad-
LH and FSH in the anterior pituitary. It does
226 Section II: Organ Systems  •  Answers

verse effect is hyperkalemia. It is not associated 37. The correct answer is B. The biopsy reveals
with hypoglycemia unawareness. a pheochromocytoma, a tumor derived from
chromaffin cells of the adrenal medulla. The
36. The correct answer is D. This patient has punctate blue-black granules seen in the im-
type 1 DM. The pancreas contains the islets age are dense-core neurosecretory granules
of Langerhans, which secrete insulin (b cells), containing catecholamines, and episodic re-
glucagons (a cells), and somatostatin (δ-cells) lease of these granules produces classic hy-
and comprise the endocrine component of the peradrenergic symptoms (the “5 Ps”: elevated
pancreas. This patient has marked, absolute blood Pressure, Pain [headache], Perspiration,
insulin deficiency, resulting from diminished Palpitations, and Pallor/diaphoresis) as well as
b cell mass; the pathophysiology often in- increased urinary vanillylmandelic acid levels
volves islet antibodies. Insulin acts on the liver, (metabolites from catecholamine breakdown).
High-Yield Systems

muscle, and adipose, and ultimately decreases Initial treatment is with phenoxybenzamine,
blood glucose levels. An acute metabolic com- a nonspecific, irreversible a-blocker that can
plication seen primarily in type 1 diabetes is help manage symptoms related to pheochro-
diabetic ketoacidosis, which results from ac- mocytoma, such as hypertension and exces-
cumulation of ketones. Even though the blood sive sweating. Pheochromocytomas also should
glucose level is elevated, the body is unable to be remembered by the “rule of 10s”: 10% are
utilize it due to the lack of insulin. The high malignant, 10% are bilateral, 10% are extra-
blood glucose causes dehydration via an os- adrenal, 10% are calcified, 10% are in chil-
motic diuresis. Patients are treated with insulin dren, and 10% are familial.
to normalize the metabolism of carbohydrates,
Answer A is incorrect. Parafollicular C cells of
proteins, and fats and with fluids to correct the
the thyroid produce calcitonin and can lead to
dehydration.
medullary carcinoma, which appears histologi-
Answer A is incorrect. The adrenal gland had cally as sheets of cells in an amyloid stroma.
several important synthetic functions, includ-
Answer C is incorrect. Malignancy of the
ing production of aldosterone, cortisol, sex ste-
pancreatic islet beta cells can lead to insulin-
roids, and epinephrine, but it does not play a
secreting insulinomas. Patients present with se-
role in insulin production.
vere intractable hypoglycemia.
Endocrine

Answer B is incorrect. The kidney produces


Answer D is incorrect. Plasma cells produce
renin, the first component of the renin-
antibodies as part of the humoral immune re-
angiotensin-aldosterone system.
sponse and can become malignant in multiple
Answer C is incorrect. The liver has numer- myeloma. Monoclonal plasma cells can take
ous important synthetic functions, including on a “fried-egg” appearance in the marrow.
production of insulin-like growth factors/so-
Answer E is incorrect. Somatotrophs release
matomedins in response to GH stimulation,
GH from the anterior pituitary, and somato-
but is not involved in insulin synthesis or se-
troph adenomas can lead to acromegaly or gi-
cretion. It is, however, one of the targets of
gantism (in adolescents).
insulin. Insulin acts on the liver to increase
glucose uptake via an enzymatic effect, triglyc-
38. The correct answer is A. Iodine is essential for
eride synthesis, protein synthesis, and glycogen
the normal synthesis and secretion of T4; how-
synthesis and to decrease gluconeogenesis, gly-
ever, at excess levels iodine can actually in-
cogenolysis, lipolysis, protein catabolism, ure-
hibit this process by blocking its own transport
agenesis, ketogenesis, and blood glucose levels.
into the thyroid follicular cells. This occurs
Answer E is incorrect. The spleen is an impor- because of an escape mechanism by the thy-
tant component of the reticuloendothelial sys- roid: A shutdown in the presence of excessive
tem; however, it has no significant synthetic or iodine protects the body from excessive pro-
endocrine function. duction of T3, which can cause thyrotoxicosis.
Chapter 9: Endocrine  •  Answers 227

This inhibition can actually result in clinical is a 5-a-reductase inhibitor that inhibits the
hypothyroidism, manifesting in this patient as conversion of testosterone to DHT, therefore
constipation, brittle hair, and fatigue. In her preventing further growth of the prostate. Fi­
case, overzealous supplementation of iodine nasteride also promotes hair growth.
resulted in hypothyroidism secondary to iodine
Answer B is incorrect. Flutamide is a com-
excess.
petitive inhibitor of testosterone and its recep-
Answer B is incorrect. Magnesium is impor- tor and is used to treat prostatic carcinoma, not
tant in the maintenance of normal cardiac BPH. The biopsy did not show dysplasia, so
rhythms and in the generation and transduc- treatment of carcinoma is not appropriate.
tion of action potentials. Hypermagnesemia
Answer C is incorrect. Ketoconazole is a com-
is associated with dysrhythmias, neurologic
monly used antifungal that also has antiandro-
symptoms, neuromuscular deficits, and pulmo-

High-Yield Systems
gen effects. In the latter capacity, it is used in
nary symptoms, but it is not associated with hy-
the treatment of polycystic ovarian syndrome
pothyroidism.
to prevent hirsutism.
Answer C is incorrect. Vitamin C (ascorbic
Answer D is incorrect. Spironolactone is a K+-
acid) is a necessary cofactor for collagen syn-
sparing diuretic that also has antiandrogenic
thesis and the conversion of dopamine to nor-
effects. In addition to use in the treatment of
epinephrine, and it facilitates iron absorption
hyperaldosteronism, hypokalemia, and CHF, it
by keeping iron in a reduced oxidation state.
can be used in preventing hirsutism in polycys-
Deficiency of vitamin C causes scurvy, which
tic ovarian syndrome.
involves skin and gum breakdown due to colla-
gen fragility. Excess vitamin C is not associated Answer E is incorrect. Yohimbine is an a2-­
with hypothyroidism. selective inhibitor with questionable usage in
the treatment of impotence. While this patient
Answer D is incorrect. Vitamin E is an anti-
has multiple genitourinary complaints, impo-
oxidant that protects RBCs from hemolysis.
tence is not one of them.
Deficiency of this vitamin is associated with
increased RBC fragility and hemolysis but not 40. The correct answer is B. To answer this ques-
hypothyroidism. Excessive vitamin E can actu- tion, one must first arrive at the diagnosis of

Endocrine
ally lead to an increased risk of bleeding, and rickets. The knobbiness of the patient’s ribs
can consequently cause hemorrhagic stroke. suggests the rachitic rosary often seen with
Answer E is incorrect. Zinc is important for rickets. The next step is to realize that in this
normal wound healing and immune function, case, rickets is secondary to vitamin D defi-
and deficiency results in symptoms of hypogo- ciency, as evidenced by the risk factors listed:
nadism, decreased skeletal muscle maturation, living in an inner-city apartment, rarely going
and cataracts. However, zinc intake is not as- outside, and continuing to be breast-fed (breast
sociated with hypothyroidism. milk has little or no vitamin D, unlike vitamin
D-supplemented cow’s milk). In vitamin D
39. The correct answer is A. Benign prostatic hy- deficiency one expects to see a decrease in cal-
perplasia (BPH) is a common entity in men cium and phosphate levels because vitamin D
>50 years old and increases in prevalence with has a key role in increasing their uptake from
increasing age. Pathophysiologically, estradiol the GI tract. The decrease in calcium levels
levels increase with age and they are thought would result in an increase in PTH levels,
to sensitize the prostate to the effects of dihy- making this the correct answer.
drotestosterone (DHT), causing the prostatic
Answer A is incorrect. This combination of
cells to grow. Common symptoms of BPH in-
laboratory values would rarely be seen.
clude increased frequency of urination, noc-
turia, problems with initiating and stopping Answer C is incorrect. This combination
urination, and pain on urination. Finasteride of laboratory values could be seen with renal
228 Section II: Organ Systems  •  Answers

insufficiency. In such a case, the poor kidney panic cavity. It is not involved in thyroid devel-
function would result in decreased excretion opment.
of phosphate. The increased serum phosphate
Answer E is incorrect. The second branchial
could then complex with serum calcium, caus-
arch gives rise to the stapes, styloid process, hy-
ing a decrease in calcium levels. This decrease
oid bone, the muscles of facial expression, and
in calcium would in turn stimulate an increase
other structures innervated by the facial nerve.
in PTH. This chronic stimulation of PTH lev-
It does not give rise to the thyroid.
els can result in secondary hyperparathyroid-
ism. Answer F is incorrect. The second branchial
pouch gives rise to the lining of the palatine
Answer D is incorrect. This combination of
tonsils. It is not involved in thyroid develop-
laboratory values could be seen with hyper-
ment.
parathyroidism because the increased PTH
High-Yield Systems

level would cause an increase in calcium lev- Answer G is incorrect. Thyroid development
els and a decrease in phosphate levels. This begins in the pharynx, and the thyroid eventu-
occurs because PTH stimulates calcium ab- ally moves to its final anatomic position ante-
sorption from the GI tract, calcium reabsorp- rior to the trachea. Thyroid development does
tion from the kidney, and calcium release from not involve the trachea.
bone. At the same time, PTH inhibits reab-
sorption of phosphate from the kidney. 42. The correct answer is A. Both methylpheni-
date and sibutramine are stimulants that func-
Answer E is incorrect. This combination tion by inhibiting neurotransmitter reuptake.
of laboratory values could be seen with vita- Sibutramine is a drug used to treat obesity that
min D intoxication. High levels of vitamin inhibits the reuptake of serotonin, norepineph-
D would result in high levels of calcium and rine, and dopamine; methylphenidate treats
phosphate. The high calcium levels would attention deficit/hyperactivity disorder by in-
suppress secretion of PTH. hibiting the reuptake of norepinephrine and
dopamine.
41. The correct answer is D. This is a case of a
thyroglossal duct cyst, resulting from a failure Answer B is incorrect. Orlistat is a diet drug
of the thyroglossal duct to involute during de- that inhibits pancreatic lipase. It is also used
Endocrine

velopment. It usually remains asymptomatic in the treatment of obesity with the idea that
unless it becomes infected, which often occurs reducing the activity of the lipase will reduce
during childhood. These cysts are differenti- absorption of dietary fats. Its function is not
ated from other conditions such as branchial similar to that of sibutramine.
cleft cysts by their midline location. Embryo- Answer C is incorrect. Selegiline is a selec-
logic development of the thyroid begins in the tive monoamine oxidase inhibitor used to treat
pharynx. Parkinson disease. Its main function is to in-
Answer A is incorrect. Thyroid development crease the amount of dopamine available in
does not originate in the esophagus. It origi- these patients.
nates in the pharynx, above the beginning of Answer D is incorrect. Tranylcypromine is a
the esophagus. monoamine oxidase inhibitor used as an an-
Answer B is incorrect. The first branchial arch tidepressant. Although it also increases the
gives rise to the malleus and incus, the mus- amount of neurotransmitters at the neuronal
cles of mastication, and other structures inner- synapse, it does so by preventing the break-
vated by cranial nerve V. It is not involved in down as opposed to the reuptake.
thyroid development.
43. The correct answer is A. This patient has dis-
Answer C is incorrect. The first branchial seminated meningococcal infection, character-
pouch gives rise to the auditory tube and tym- ized by meningitis (cerebrospinal fluid [CSF]
Chapter 9: Endocrine  •  Answers 229

pleocytosis and positive examination findings), and hypernatremia. In addition, serum ADH
hypoxemia, DIC, and acute onset adrenal in- levels tend to be elevated; this represents the
sufficiency. Acute onset adrenal insufficiency body’s effort to compensate for a diminished
in the context of disseminated meningococ- renal response to ADH. Demeclocycline is just
cemia is termed Waterhouse-Friderichsen syn- one cause of nephrogenic DI; others include
drome and is caused by bilateral adrenal hem- lithium toxicity and hypercalcemia. Demeclo-
orrhage. It is important to recognize that the cycline is a tetracycline analog (an antibiotic)
clinical presentation of adrenal insufficiency is that might also be used to treat Lyme disease.
often nonspecific, and may include fever, nau-
Answer A is incorrect. Acute gouty arthri-
sea, vomiting, diffuse abdominal tenderness,
tis is often treated with nonsteroidal anti-
and hypotension that is mainly refractory to
inflammatory drugs, including indomethacin.
large amounts of intravenous hydration.
The patient in this vignette has developed

High-Yield Systems
Answer B is incorrect. Although this patient nephrogenic DI. Indomethacin is actually
indeed has DIC (thrombocytopenia, coagulop- used to treat nephrogenic DI, and it does not
athy, elevated D-dimer, and a petechial rash), have a role in the treatment of SIADH.
these findings are not likely contributing to her
Answer B is incorrect. Conn syndrome (pri-
abdominal tenderness.
mary hyperaldosteronism) is a disorder char-
Answer C is incorrect. This patient’s hypox- acterized by excessive aldosterone secretion.
emia is likely due to sepsis secondary to me- Medical treatment includes spironolactone,
ningococcal bacteremia. Hypoxemia may a potassium-sparing diuretic that acts as an al-
cause cyanosis and dyspnea, but it does not dosterone antagonist. Spironolactone is not a
cause hypotension and diffuse abdominal ten- conventional treatment for SIADH, nor would
derness. it explain the development of nephrogenic DI
in this patient.
Answer D is incorrect. This patient also has
meningitis, characterized by photophobia, fe- Answer C is incorrect. Medical treatment op-
vers, positive Kernig and Brudzinski signs, and tions for enuresis (bedwetting) include desmo-
CSF pleocytosis with positive Gram stains. pressin (a synthetic form of ADH) and tricyclic
However, meningitis alone does not cause dif- antidepressants. Neither of these agents would

Endocrine
fuse abdominal tenderness. be used in the therapy of SIADH, nor would
they be expected to cause nephrogenic DI.
Answer E is incorrect. Viral gastroenteritis is a
The offending agent in this vignette (drug X) is
common cause of hypotension, vomiting, and
demeclocycline.
diffuse abdominal tenderness. However, in this
case of Waterhouse-Friderichsen syndrome Answer E is incorrect. Medical treatment
(which is due to disseminated meningococce- for pheochromocytoma includes a-blocking
mia), a concomitant viral gastroenteritis is un- agents such as phenoxybenzamine and phen-
likely. tolamine. This patient appears to have devel-
oped nephrogenic DI in response to demeclo-
44. The correct answer is D. Treatments for cycline therapy for SIADH. Demeclocycline
SIADH include demeclocycline (drug X in is an antibiotic and is not commonly used to
this vignette) and water restriction. This patient treat pheochromocytomas.
appears to have developed a complication of
demeclocycline therapy: nephrogenic diabetes 45. The correct answer is A. Parathyroid hypo-
insipidus (DI). In nephrogenic DI the kidney plasia results in a decreased PTH level, which
is unable to resorb sufficient water in response leads to elevated, not low, serum phosphorus
to ADH, resulting in the production of large levels.
quantities of relatively dilute urine. Symptoms
Answer B is incorrect. This patient most likely
of nephrogenic DI include polydipsia, poly-
has DiGeorge syndrome, a collection of signs
uria, hypotonic urine, serum hyperosmolarity,
230 Section II: Organ Systems  •  Answers

and symptoms associated with defective de- Answer A is incorrect. While the gastroduo-
velopment of the pharyngeal pouch system denal artery is an important source of vascular
(usually caused by a deletion on chromosome supply to the head of the pancreas, the inferior
22q11.2). The classic presentation for Di- mesenteric artery does not provide any vascu-
George syndrome is the triad of conotruncal lar supply to this structure and thus provides
cardiac anomalies, hypoplastic thymus, and no branches that would need to be ligated to
hypocalcemia. Cleft palate and abnormal fa- remove the mass described in the question
cies are also common. The thymus may be stem.
hypoplastic or completely absent, resulting in
Answer C is incorrect. Neither the proper he-
T-lymphocyte deficiency. Hypocalcemia re-
patic nor the inferior mesenteric arteries pro-
sults from parathyroid hypoplasia and is thus
vide any significant arterial supply to the head
accompanied by low PTH and elevated serum
of the pancreas; thus no branches from either
High-Yield Systems

phosphorus levels.
of these vessels would need to be ligated to
Answer C is incorrect. Parathyroid hypopla- complete the resection.
sia results in a decreased PTH level, which in
Answer D is incorrect. While the superior
turn leads to decreased, not elevated, serum
mesenteric artery is an important source of
calcium levels.
vascular supply to the head of the pancreas,
Answer D is incorrect. Parathyroid hypopla- the proper hepatic artery does not provide any
sia results in a decreased rather than elevated vascular supply to this structure and therefore
PTH level, which in turn leads to decreased, provides no branches that would need to be li-
not elevated, serum calcium levels. gated to remove the mass.
Answer E is incorrect. Neither the left gastric
46. The correct answer is B. This patient has
nor the inferior mesenteric arteries provide
a mass in the head of the pancreas, an organ
any significant arterial supply to the head of
that is both an exocrine and endocrine gland,
the pancreas; thus no branches from either of
and is both secondarily retroperitoneal and
these vessels would need to be ligated to com-
peritoneal. As an exocrine gland it produces
plete the resection.
bicarbonates and digestive enzymes, and as
an endocrine gland it produces glucagons, in- Answer F is incorrect. While the superior
Endocrine

sulin, somatostatin, and pancreatic polypep- mesenteric artery is an important source of vas-
tide. This patient’s symptoms and low blood cular supply to the head of the pancreas, the
glucose level suggest that there is an abun- left gastric artery does not provide any vascular
dance of insulin production, such as in a tu- supply to this structure and thus provides no
mor. The CT findings support a mass, which branches that would need to be ligated to re-
likely is the source. Treatment involves resec- move the mass.
tion of the mass, and ligation of its blood sup-
ply in the case of a possible tumor. The head 47. The correct answer is A. The patient has signs
of the pancreas and the duodenum share a and symptoms suggestive of hypercortisolism,
dual blood supply from the gastroduodenal ar- also known as Cushing syndrome. Etiologies of
tery, a branch of the celiac trunk. This artery hypercortisolism include a cortisol-producing
supplies the anterior and posterior superior adrenal adenoma, an ACTH-producing pi-
pancreaticoduodenal arteries as well as the tuitary adenoma, paraneoplastic ectopic pro-
superior mesenteric artery, which supplies the duction of ACTH, and exogenous cortisol or
anterior and posterior inferior pancreaticoduo- ACTH administration. The dexamethasone
denal arteries. Therefore, to resect any portion suppression test can help distinguish between
of the duodenum or the head of the pancreas, possible etiologies of hypercorticism. In nor-
branches from both the gastroduodenal and mal individuals, low doses of dexamethasone
superior mesenteric arteries must be ligated. suppress cortisol production. In patients with
ACTH-producing pituitary adenomas, high
Chapter 9: Endocrine  •  Answers 231

doses of dexamethasone are needed to sup- nal reabsorption of phosphate (resulting in


press cortisol production. In patients with ad- decreased serum phosphate levels), and in-
renal adenomas or ectopic sources of ACTH, creased vitamin D activation. Squamous cell
both low and high doses of dexamethasone fail lung cancer is known to be associated with ec-
to suppress cortisol production. Unlike patients topic PTHrP production.
with ectopic ACTH production, patients with
Answer A is incorrect. PTH and PTHrP
an adrenal adenoma are expected to have low
do not act at the adrenal cortex or the intes-
levels of ACTH due to negative feedback in-
tine. The adrenal cortex is the primary site
hibition from the increased cortisol levels, as
of action for ACTH and ACTH-like peptide;
noted in this patient.
the intestine is the primary site of action for
Answer B is incorrect. Bilateral adrenal hy- 1,25-­dihydroxycholecalciferol. Additionally, the
perplasia suggests increased stimulation of the adrenal glands are not normally involved in reg-

High-Yield Systems
adrenal glands due to increased ACTH pro- ulating serum calcium and phosphate levels.
duction from either a pituitary adenoma or an
Answer B is incorrect. PTH and PTHrP have
ectopic ACTH source.
no action at the adrenal cortex. ACTH and
Answer C is incorrect. Ectopic ACTH pro- ACTH-like peptide (both of which can be se-
duction is seen in paraneoplastic syndromes creted by neoplastic cells, resulting in Cushing
associated with bronchogenic cancer, pancre- syndrome) act primarily at the adrenal cortex.
atic cancer, and thymomas. Bilateral adrenal Additionally, the adrenal glands are not nor-
hyperplasia and failed dexamethasone sup- mally involved in regulating serum calcium
pression are characteristics of ectopic ACTH and phosphate levels.
production. A single mass noted on abdominal
Answer C is incorrect. The intestine
CT scan adjacent to a kidney is more sugges-
and bones are primary sites of action for
tive of an adrenal adenoma than bilateral adre-
1,25-­dihydroxycholecalciferol (activated vita-
nal hyperplasia.
min D), which causes increased calcium and
Answer D is incorrect. Although exogenous phosphate absorption in the intestines and in-
corticosteroid administration, like adrenal ad- creased bone resorption of calcium and phos-
enomas, results in decreased levels of ACTH, phate. While PTH and PTHrP stimulate the

Endocrine
a mass on abdominal CT scan would be un- production of 1,25-dihydroxycholecalciferol,
likely in a patient with exogenous corticoste- producing secondary effects at the intestine,
roid administration. these hormones do not act primarily on the in-
testine.
Answer E is incorrect. An ACTH-secreting pi-
tuitary adenoma would cause bilateral adrenal Answer E is incorrect. PTH and PTHrP act
hyperplasia and elevated ACTH levels, which primarily at the renal tubules and bones to
are usually suppressed with high-dose dexa- increase serum calcium levels and decrease
methasone. phosphate levels. These hormones do not in-
fluence the pancreas, although pancreatic tu-
48. The correct answer is D. The patient’s hyper- mors have been shown to occasionally secrete
calcemia and hypophosphatemia in the set- PTHrP. Additionally, the pancreas is not nor-
ting of increased serum alkaline phosphatase mally involved in regulating serum calcium
activity are consistent with elevated PTH lev- and phosphate levels.
els. This is likely caused by ectopic produc-
tion of PTH-related peptide (PTHrP), which 49. The correct answer is E. The actions of insu-
produces physiologic effects that mimic those lin are mediated at the cellular level by bind-
of PTH: increased bone resorption (causing el- ing of the insulin to its receptor followed by
evated alkaline phosphatase activity), increased autophosphorylation of tyrosine residues on
renal reabsorption of calcium (resulting in the insulin receptor; this generates a tyrosine
elevated serum calcium levels), decreased re- kinase that participates in an intracellular sig-
232 Section II: Organ Systems  •  Answers

naling cascade. Inhibition of tyrosine kinase of autosomal dominant syndromes in which


function would preclude downstream signal- more than one endocrine organ is dysfunc-
ing and block the physiologic changes asso- tional. Given his history of thyroid carcinoma
ciated with insulin action, regardless of the requiring a thryoidectomy, this patient most
amount of insulin present in the blood. likely has MEN type II, which is character-
ized by the combination of medullary carci-
Answer A is incorrect. Adenylate cyclase and
noma of the thyroid, pheochromocytoma, and
its product, cAMP, are involved in numerous
parathyroid hyperplasia. Pheochromocytomas
important intracellular signaling systems, in-
are treated surgically, but must first be man-
cluding the systems that mediate autonomic
aged preoperatively with both a nonselective
sympathetic nervous stimulation, ADH action,
a-antagonist (usually phenoxybenzamine)
renal calcium and phosphate transport, and
to normalize blood pressure, followed by a
glucagon action. However, adenylate cyclase
High-Yield Systems

b-blocker (eg, propranolol) to control tachy-


and cAMP are not involved in the system that
cardia. The b-blocker should never be started
mediates insulin action.
prior to a-blockade because it would result in
Answer B is incorrect. Guanylate cyclase and unopposed a-receptor stimulation, leading to a
its product, cGMP, are involved in many in- further elevation in blood pressure.
tracellular signaling systems, including those
Answer B is incorrect. Pheochromocytomas
that mediate the transduction of visual stimuli
are treated surgically, but must first be man-
into electrical signals in the nervous system,
aged preoperatively with both a nonselective
and the relaxation of vascular smooth muscle
a-antagonist (usually phenoxybenzamine)
throughout the body. However, guanylate cy-
to normalize blood pressure, followed by a
clase and cGMP are not known to be involved
b-blocker (eg, propranolol) to control tachy-
in the system that mediates insulin action.
cardia. The b-blocker should never be started
Answer C is incorrect. Serine kinases are in- prior to a-blockade because it would result in
volved in a number of intracellular signaling unopposed a-receptor stimulation, leading to a
cascades, but they are not known to be in- further elevation in blood pressure.
volved in the signaling cascade that mediates
Answer C is incorrect. Levothyroxine is thy-
insulin action.
roid hormone replacement that is used to treat
Endocrine

Answer D is incorrect. Threonine kinases are hypothyroidism and myxedema. It is not indi-
involved in a number of intracellular signal- cated in the treatment of pheochromocytoma.
ing cascades, but they are not known to be in-
Answer D is incorrect. Prednisone is a gluco-
volved in the signaling cascade that mediates
corticoid that is used to treat many inflamma-
insulin action.
tory, allergic, and immunologic disorders, but
50. The correct answer is A. This patient has a is not indicated in the treatment of pheochro-
pheochromocytoma, a chromaffin cell tumor mocytoma.
of the adrenal medulla that secretes excess Answer E is incorrect. Propylthiouracil is used
epinephrine and norepinephrine. Signs and to treat hyperthyroidism. It acts by inhibiting
symptoms of pheochromocytoma include epi- the organification and coupling of thyroid hor-
sodic hypertension, headache, sweating, tachy- mone synthesis, and by decreasing the periph-
cardia, palpitations, and pallor. Pheochro- eral conversion of T4 to T3. It is not indicated
mocytomas may occur sporadically or as part in the treatment of pheochromocytoma.
of the MEN syndromes, which are a group
Chapter 10

Gastrointestinal

233
234 Section II: Organ Systems  •  Questions

Q u e st i o n s

1. A 35-year-old woman who is HIV positive pre­ (D) Perforated esophagus


sents to the physician because of jaundice and (E) Volvulus
right upper quadrant abdominal pain. She re-
ports having had multiple episodes of jaundice 3. A 22-year-old woman with no significant medi-
over the past 10 years. Physical examination is cal history complains of diffuse abdominal
remarkable for scleral icterus, marked ascites, pain. Physical examination reveals rebound
and splenomegaly. A hepatitis panel is positive tenderness in the right lower quadrant. The pa-
for HBsAg and anti-HBc IgM, but negative for tient denies being sexually active and has not
HBsAb and anti-HAV IgM. Which of the fol- traveled recently. However, she does mention
lowing would most likely be lower than the eating “funny tasting” potato salad at an out-
High-Yield Systems

normal reference range in this patient? door party three days ago. Which of the follow-
ing is the most appropriate next step in man-
(A) Alkaline phosphatase
agement?
(B) Bilirubin
(C) Platelet count (A) Emergency appendectomy
(D) Prothrombin time (B) Initiation of metronidazole, bismuth, and
(E) Transaminases amoxicillin therapy
(C) Measurement of serum b-human chori-
2. An 85-year-old woman presents to the emer- onic gonadotropin level
gency department because of sudden onset of (D) Measurement of serum lipase level
abdominal pain, maroon-colored stools, and (E) Stool culture
abdominal distention. She denies any past ab-
dominal surgery. An upper gastrointestinal 4. A 43-year-old overweight woman presents to
fluoroscopy study is performed with the results her doctor’s office because of right upper quad-
shown below. Which of the following is the rant abdominal pain. She has experienced sim-
most likely cause of this patient’s symptoms? ilar episodes of this type of pain in the past and
Gastrointestinal

admits that it is worse after meals. Increased se-


cretion of which of the following is responsible
for this patient’s postprandial pain?
(A) Cholecystokinin
(B) Decreasing the secretion of gastrin
(C) Pepsin
(D) Somatostatin
(E) Vasoactive intestinal peptide

5. A 25-year-old man presents to his physician


with a complaint of “yellow eyes” for the past
day. For the past five days, he has been ill with
a low-grade fever, rhinorrhea, myalgias, and
generalized malaise. The physical examination
confirms scleral icterus, but is otherwise un-
Reproduced, with permission, from USMLERx.com. remarkable. Electrolytes and complete blood
cell count are all within normal limits. Labora-
tory tests show:
(A) Duodenal hematoma
(B) Esophageal stricture Aspartate aminotransferase: 31 IU/L
(C) Intussusception Alanine aminotransferase: 25 IU/L
Alkaline phosphatase: 45 IU/L
Chapter 10: Gastrointestinal  •  Questions 235

Total bilirubin: 3 mg/dL 8. A 57-year-old white man is brought to the


Lactate dehydrogenase: 40 IU/L emergency department by ambulance be-
Haptoglobin: 76 mg/dL (normal: 46-316 mg/ cause of sudden-onset, bright red emesis. His
dL) blood pressure is 80/40 mm Hg and heart rate
is 124/min. Physical examination is notable
His urinalysis demonstrates a normal bilirubin
for jaundice and an enlarged abdomen that is
level. What is the most appropriate treatment
dull to percussion and positive for a fluid wave.
for this patient’s condition?
Which of the following vessel anastomoses is
(A) Corticosteroids responsible for the patient’s bleeding?
(B) No treatment is required
(A) Left gastric artery and left gastric vein
(C) Pegylated interferon
(B) Left gastric vein and azygos vein
(D) Phenobarbital
(C) Paraumbilical vein and inferior epigastric

High-Yield Systems
(E) Ursodeoxycholic acid
vein
6. A 39-year-old white woman who suffers from (D) Portal vein and inferior vena cava
polycythemia vera presents to the clinic com- (E) Splenic vein and left renal vein
plaining of severe and constant right upper
9. An 18-year-old man with no significant medi-
quadrant pain over the past two days. Physical
cal history presents to the clinic with pain in
examination reveals an enlarged liver. What
the right lower quadrant, mild diarrhea, and
other finding would most likely be seen at pre-
fever. This has happened twice within the past
sentation?
12 months, but he has been asymptomatic
(A) Ascites between episodes. The patient denies recent
(B) Asterixis travel or camping. Physical examination re-
(C) Esophageal varices veals a perianal fistula. The gross appearance
(D) Hyperpigmented skin of the terminal ileum from a similar patient is
(E) Spider angiomata shown in the image. Which of the following
screening measures would be least useful in
7. A 29-year-old woman presents to her primary this patient?

Gastrointestinal
care physician complaining of “trouble eat-
ing.” She says she has had pain when swallow-
ing both solids and liquids for the past nine
months. She states that it has been difficult to
maintain an appetite over this time and reports
a weight loss of 2.3 kg (5 lb). Symptoms have
remained constant since they appeared nine
months ago. The patient does not exhibit tight-
ening of the facial skin, claw-like hands, or any
other systemic symptoms. Which of the follow-
ing drug mechanisms of action is most likely to
improve the patient’s symptoms? Reproduced, with permission, from USMLERx.com.

(A) Decreasing calcium availability in smooth


muscle cells (A) Fundoscopic examination
(B) Decreasing proton secretion into the stom- (B) Yearly alkaline phosphatase level
ach lumen (C) Intermittent hemoglobin levels
(C) Enhancing the phosphorylation of myosin (D) Intermittent blood urea nitrogen and creat-
light chains inine levels
(D) Inhibiting the degradation of acetylcholine (E) Yearly colonoscopy
236 Section II: Organ Systems  •  Questions

10. A 16-year-old boy presents to the clinic with


“skin boils.” The lesions are erythematous and
tender, concerning for skin abscesses due to
methicillin-resistant Staphylococcus aureus.
The patient is promptly given an oral medica-
tion to treat the infection. The abscesses even-
tually resolve. Three days later, the boy devel-
ops fever and watery, foul-smelling diarrhea.
What is the mechanism of action of the antibi-
otic the patient is taking?
(A) Blocking the 30S ribosomal subunit
(B) Blocking the 50S ribosomal subunit
High-Yield Systems

(C) Forming toxic metabolite


(D) Inhibiting cell-wall synthesis Reproduced, with permission, from USMLERx.com.
(E) Inhibiting the translocation step of protein
synthesis
(A) Infliximab
11. A 65-year-old woman presents to the emer- (B) Nizatidine
gency department with persistent right upper (C) Omeprazole
quadrant pain with nausea and vomiting. CT (D) Ondansetron
of the abdomen reveals a polypoid mass of the (E) Sucralfate
gallbladder protruding into the lumen, diffuse (F) Sulfasalazine
thickening of the gallbladder wall, and en-
larged lymph nodes. This patient most likely 13. A fourth-year medical student is working in
has a history of which of the following? a medical relief group in Haiti for several
months. Several parents bring their children
(A) Ascaris lumbricoides to the clinic and explain that the children
(B) Cigarette smoking have had profuse, watery stool along with wa-
(C) Gallstones
Gastrointestinal

tery vomiting. All of the children are afebrile,


(D) Schistosoma haematobium slightly hypotensive, and tachycardic but have
(E) Tuberculosis a normal respiratory rate. Urine output is re-
duced. What would be the best immediate
12. A 35-year-old woman presents to the emer-
management of this diarrhea?
gency department because of abdominal pain
and diarrhea mixed with mucus and blood. (A) Antibiotics
She also has ulcerated lesions with violaceous (B) Diphenoxylate
borders on her legs. Gross blood is present on (C) Intravenous normal saline with 5% dex-
rectal examination. A biopsy of her colon re- trose
veals inflammation confined to the mucosa (D) Loperamide
and submucosa, as shown in the image. Which (E) Oral rehydration solutions
of the following would most likely be used to
treat this patient?
Chapter 10: Gastrointestinal  •  Questions 237

14. A 79-year-old woman presents with 5.4-kg (12- 16. A 34-year-old man is brought to the emergency
lb) weight loss over the past two months, asso- department after being involved in a high-
ciated with progressively worsening dull, con- speed collision with an oncoming car. He has
stant abdominal pain, early satiety, and nausea. multiple fractures and contusions. Results of
Her examination is notable for a palpable fundoscopy are shown in the image. The pa-
periumblicial node as well as left supraclavicu- tient is stabilized and transferred to the inten-
lar adenopathy. Which of the following is the sive care unit. Two days later, there is evidence
most likely diagnosis? of gastrointestinal (GI) hemorrhage. What is
the most likely mechanism of the GI bleeding?
(A) Adenocarcinoma of the pancreas
(B) Adenocarcinoma of the stomach
(C) Ductal carcinoma in situ of the breast
(D) Esophageal carcinoma

High-Yield Systems
(E) Krukenberg tumor

15. A medical student presents to an infectious dis-


ease specialist complaining of abdominal dis-
tention and tenderness. The patient reports no
recent changes in normal bowel habits. Physi-
cal examination shows hepatosplenomegaly.
Bowel sounds are normal. On questioning, the
patient reports that he traveled to Brazil several
months ago to study tribal medical practices.
He frequently went swimming in the Amazon
River to wash himself. Several weeks after re-
turning from his trip, he recalls having fever, Reproduced, with permission, from USMLERx.com.
diarrhea, weight loss, and “funny looking”
stools. Which of the following conditions is
(A) Chronic irritation of the gastroesophageal
most likely responsible for this patient’s present
junction by caustic acid secretions

Gastrointestinal
symptoms?
(B) Dilated submucosal veins in the esophagus
(A) Appendicitis due to portal hypertension
(B) Bowel obstruction (C) Increased vagal stimulation secondary to
(C) Enterocolitis elevated intracranial pressure
(D) Portal hypertension (D) Inhibited gastric secretion of mucus sec-
(E) Ruptured viscus ondary to bacterial infection in the stom-
ach
(E) Overuse of oral analgesics leading to super-
ficial gastric erosions
238 Section II: Organ Systems  •  Questions

17. A 20-year-old man has Crohn disease that is 19. An 8-year-old boy presents to the emergency
refractory to treatment with high-dose meth- department because of 18 hours of severe vom-
ylprednisolone. He is started on therapy with iting. Arterial blood gas analysis reveals a pH of
infliximab, a chimeric monoclonal antibody 7.48, a bicarbonate level of 35 mEq/L, and a
with anti-inflammatory effects. This drug is partial carbon dioxide pressure of 48 mm Hg.
administered intravenously every two months Which of the following best describes the acid-
and produces substantial improvement in the base disturbance occurring in this patient?
patient’s symptoms between doses. Which of
(A) Metabolic acidosis
the following best describes infliximab’s mech-
(B) Metabolic acidosis with respiratory com-
anism of action?
pensation
(A) Binds to a growth factor receptor to target (C) Metabolic alkalosis and metabolic acidosis
a cell for killing (D) Metabolic alkalosis and respiratory acidosis
High-Yield Systems

(B) Binds to and neutralizes soluble tumor ne- (E) Metabolic alkalosis and respiratory alkalo-
crosis factor-a sis
(C) Binds to and neutralizes tumor necrosis (F) Metabolic alkalosis with respiratory com-
factor-a receptors on T cells pensation
(D) Inhibits a fusion protein with tyrosine ki-
nase activity 20. A 43-year-old multiparous woman with no
(E) Inhibits macrophage production of tumor other medical history presents to her physician
necrosis factor-a because of crampy abdominal pain, fever, and
jaundice. Laboratory studies show:
18. A 2-month-old boy is brought to his pediatri-
Total bilirubin: 4.8 mg/dL
cian for a regular check-up. His parents report
Direct bilirubin: 4.2 mg/dL
that he has a poor appetite and is very consti-
Amylase: 50 U/L
pated. He has small bowel movements once a
Lipase: 70 U/L
week, which his parents say appear to be very
Aspartate aminotransferase: 75 U/L
painful. Although he was at the 75th percen-
Alanine aminotransferase: 70 U/L
tile for both height and weight at birth, he is
Gastrointestinal

currently at the 25th percentile for height and The patient subsequently is sent for endo-
is below the fifth percentile for weight. His ab- scopic retrograde cholangiopancreatography;
domen is distended, but his bowel sounds are results are shown in the image. What is the
normal and his abdomen does not appear to most likely cause of the obstruction seen in the
be tender. Barium enema shows a narrow rec- image?
tosigmoid with a dilation of the segment above
the narrowing, and a rectosigmoid biopsy
shows a conspicuous absence of ganglion cells.
Which of the following genetic conditions is
most commonly associated with this patient’s
disease?
(A) Cystic fibrosis
(B) Down syndrome
(C) Sickle cell disease
(D) Tay-Sachs disease
(E) Turner syndrome
Chapter 10: Gastrointestinal  •  Questions 239

(A) Autosomal dominant


(B) Autosomal recessive
(C) Autosomal trisomy
(D) Sex chromosome abnormality
(E) X-linked recessive

23. A 10-year-old girl living in Grand Haven,


Michigan, is brought to the physician because
she has had a fever and headache accompa-
nied by vomiting and bloody diarrhea over
the last few days. She has no history of recent
travel or sick contacts but has a pet puppy,

High-Yield Systems
which the mother says has also had diarrhea
for the past week. A stool culture incubated at
Fauci AS, et al, eds. Harrison’s Principles of Internal Medicine,
42°C in a microaerophilic environment shows
17th ed. New York: McGraw-Hill, 2008; Fig. 305-2C.
many gram-negative, comma-shaped organ-
isms, each with a single polar flagellum. The
(A) Choledocholithiasis organism responsible for this patient’s sickness
(B) High-fiber diet is associated with the possible development of
(C) Malignancy which of the following symptoms?
(D) Primary biliary cirrhosis (A) Acute renal failure and thrombocytopenia
(E) Primary sclerosing cholangitis with hemolytic anemia
(B) Fever, migratory polyarthritis, and carditis
21. A 75-year-old woman is taken to the hospital (C) Fever, new murmur, small erythematous
by her son after two bouts of bilious vomiting. lesions on the palms, and splinter hemor-
Although she normally has a healthy appetite, rhages on the nail bed
over the past three days she has had little in- (D) Petechial rash and bilateral hemorrhage
terest in eating. Furthermore, her belly has into the adrenal glands

Gastrointestinal
become rigid and diffusely tender. X-ray of the (E) Symmetric ascending muscle weakness be-
abdomen reveals dilated loops of small intes- ginning in the distal lower extremities
tines. Which of the following predisposes this
patient to this condition? 24. A 2-year-old girl who has recently been
(A) Celiac sprue adopted from Southeast Asia is brought to the
(B) Chronic Helicobacter pylori infection clinic by her adopted parents. They are con-
(C) History of abdominal surgery cerned because the child seems to be having
(D) Smoking trouble with her vision in low-light conditions.
(E) Ulcerative colitis The nutrient most likely deficient in this child
is absorbed by the gastrointestinal system using
22. A 27-year-old man presents to his family phy- what mechanism?
sician for an annual physical examination. On (A) Apoferritin-mediated transport
rectal examination, masses are palpated. The (B) Intrinsic factor-mediated transport
patient is referred for a colonoscopy, which (C) Micelle-mediated transport
reveals adenomatous polyps located diffusely (D) Sodium-dependent cotransport
throughout the colon. When asked about his (E) Vitamin D-dependent binding protein-
family history, the patient states that his father mediated transport
passed away from colon cancer. Which of the
following inheritance patterns is characteristic
of this condition?
240 Section II: Organ Systems  •  Questions

25. A 34-year-old man visits his physician because and biopsy of the upper part of the small intes-
he has experienced increasing “itchiness” and tine demonstrates diffuse blunting of villi and
fatigue over the past three months. Medical a chronic inflammatory infiltrate in the lamina
history is significant for a total colon resec- propria. Which therapeutic option will most
tion; pathologic findings at resection are shown likely benefit this patient?
in the image. Physical examination reveals
(A) Gluten-free diet
scleral icterus. Given his past medical history,
(B) Corticosteroid therapy
ultrasound studies are performed, which re-
(C) Antibiotic therapy
veal obliteration of the intrahepatic bile ducts.
(D) Thiamine and vitamin B12
What is this man’s most likely underlying con-
(E) Vitamins A, D, E, and K
dition?
27. A 26-year-old man with hepatitis C is be-
High-Yield Systems

ing treated medically while he awaits liver


transplantation. One of the drugs he is tak-
ing causes him to have periodic fevers and
chills and a sense of depression that he did not
have prior to treatment. Which of the follow-
ing drugs most likely is responsible for this pa-
tient’s adverse effects?
(A) Intravenous immunoglobulin
(B) Lamivudine
(C) Pegylated interferon
(D) Ribavirin
(E) Tumor necrosis factor-a

Reproduced, with permission, from USMLERx.com. 28. A 10-year-old boy is brought to the emergency
department by his parents with a low-grade fe-
ver, anorexia, nausea, vomiting, and abdomi-
Gastrointestinal

(A) Hepatitis C infection nal pain. The parents report that the pain ini-
(B) Primary biliary cirrhosis tially began periumbilically and developed
(C) Systemic lupus erythematosus into severe right lower quadrant pain after sev-
(D) Ulcerative colitis eral hours. On physical examination the child
(E) Wilson disease is diaphoretic and lies still; involuntary guard-
ing and rebound are present. Pain is elicited
26. A 29-year-old man complains to his physician when the child is placed on his left side and
of chronic diarrhea. On further questioning the right leg is hyperextended against resis-
he reveals that the diarrhea is watery and in- tance. Which of the following provides inner-
termittent, and that he also suffers from flatu- vation to the muscle involved in this maneu-
lence and weight loss of 3.6 kg (8 lb) over the ver?
past year. He denies fever, nausea, vomiting,
(A) Inferior gluteal nerve
abdominal pain, and recent travel. Stool ex-
(B) Lumbar plexus and femoral nerve
aminations for ova and parasites and for occult
(C) Obturator nerve
blood are negative, and stool culture does not
(D) Sciatic nerve
grow any pathogens. Endoscopy is performed
(E) Superior gluteal nerve
Chapter 10: Gastrointestinal  •  Questions 241

29. A 4-year-old child is brought to the pediatri- (D) Somatostatin


cian because of abdominal pain, vomiting, (E) Vasoactive intestinal peptide
and diarrhea containing mucus and blood.
The child has a fever of 39.4°C (103°F). On 31. A 27-year-old woman with no significant medi-
stool culture, the causative organism is shown cal history complains of a month of sharp,
to be a non-lactose-fermenting, non-hydrogen nonradiating, epigastric pain. Her pain is re-
sulfide-producing bacterium that is extremely lieved after eating food, and she has experi-
virulent. Which of the following is/are most enced weight gain. What is the most likely pri-
likely to result from continued infection by this mary treatment for this patient?
organism? (A) Amoxicillin, clarithromycin, and omepra-
(A) Ascending muscle weakness zole
(B) Headache and rose spots on abdomen (B) Avoidance of nonsteroidal anti-

High-Yield Systems
(C) Pulmonary hemorrhage, mediastinitis, and inflammatory drugs
shock (C) Gastrinoma resection
(D) Renal failure, microangiopathic hemolytic (D) Metoclopramide
anemia, and thrombocytopenia (E) Ranitidine
(E) Subcutaneous nodules, polyarthritis, cho-
rea, and a heart murmur 32. A 34-year-old woman presents with three
weeks of abdominal pain and diarrhea. She
30. A 62-year-old man with a long history of alco- says the diarrhea appears to be greasy. She also
holism presents to the emergency department admits to a lot of flatulence since the gastroin-
with steatorrhea and abdominal pain. CT of testinal symptoms began. A stain of the stool is
the abdomen is shown in the image. The in- shown in the image. What drug(s) should be
tern on duty recalls learning about a drug in- used to treat the organism causing her symp-
dicated for acromegaly that may also reduce toms?
the secretion of pancreatic fluids and possibly
decrease the patient’s pain. The drug works by
mimicking the levels of which hormone?

Gastrointestinal
Courtesy of Dr. Mae Melvin, Centers for Disease Control and
Prevention.

(A) Chloroquine
Reproduced, with permission, from USMLERx.com.
(B) Dapsone
(C) Melarsoprol
(A) Cholecystokinin (D) Metronidazole
(B) Gastrin (E) Nifurtimox
(C) Secretin (F) Sulfadiazine and pyrimethamine
242 Section II: Organ Systems  •  Questions

33. A 40-year-old man with no significant past (A) Cholecystitis


medical history presents to the emergency de- (B) Chronic gastritis
partment because of a two day history of fever, (C) Pancreatic carcinoma
vomiting, and diarrhea. His blood pressure is (D) Pancreatic pseudocyst
90/65 mm Hg and pulse is110/min. An intra- (E) Small bowel obstruction
venous line is started and he is given 3 L of
fluid and then admitted for monitoring. On 36. A 51-year-old man with a lengthy history of
admission, laboratory studies are unremarkable medication-dependent reflux esophagitis sees
except for a serum albumin level of 3.0 g/dL. his physician for an annual physical examina-
Which of the following is the most likely cause tion. Laboratory tests reveal a blood gastrin
of this patient’s laboratory abnormality? level three times the upper limit of normal.
His physician expresses concern that the pa-
(A) Hemodilution
High-Yield Systems

tient is at risk of developing atrophic gastritis.


(B) Liver disease
Which of the following medications is this pa-
(C) Malabsorption
tient most likely taking?
(D) Nephrotic syndrome
(E) Poor nutritional status (A) Aluminum hydroxide
(B) Bismuth
34. A 33-year-old man with gastroesophageal re- (C) Cimetidine
flux disease returns to his physician for the sec- (D) Misoprostol
ond time in two weeks complaining of worsen- (E) Omeprazole
ing soreness in his throat. Two weeks earlier he
was diagnosed with penicillin-sensitive Strep- 37. A 25-year-old man presents to his primary
tococcus pyogenes on throat culture and was care physician after several episodes of severe
prescribed ciprofloxacin (since he is allergic crampy abdominal pain relieved by the pas-
to penicillin). Review of the patient’s medica- sage of loose stool mixed with blood and mu-
tion history reveals a possible drug interaction. cus. He says he has been feeling fatigued for
Which of the following medications is this pa- the past three months and has lost 6.8 kg (15
tient most likely taking that would reduce the lb). He has a fever of 37.3°C (99.1°F); ab-
Gastrointestinal

effectiveness of his antibiotic? dominal examination is notable for hypoac-


tive bowel sounds and diffuse tenderness with
(A) Aspirin
guarding. Colonoscopy reveals diffuse, con-
(B) Calcium carbonate
tinuous ulcerations of the intestinal mucosa
(C) Cimetidine
extending proximally from the rectum to the
(D) Misoprostol
splenic flexure. A diagnosis is made and ge-
(E) Omeprazole
netic testing reveals the patient carries an HLA
subtype commonly associated with the disease.
35. A 46-year-old man presents to the emergency
Which other disease is associated with the
department complaining of severe abdominal
same human leukocyte antigen subtype?
pain following a weekend of tailgating during
which he consumed “a ton” of beer. On physi- (A) Ankylosing spondylitis
cal examination the patient has a tempera- (B) Diabetes mellitus type 1
ture of 38.2°C (100.8°F), with pain located (C) Graves disease
in the epigastric region that periodically radi- (D) Multiple sclerosis
ates to his back. Laboratory tests show a serum (E) Rheumatoid arthritis
amylase level of 400 U/L and WBC count of
16,000/mm³. What is the most likely compli-
cation of this disease?
Chapter 10: Gastrointestinal  •  Questions 243

38. A 67-year-old Chinese immigrant with a his- 39. A 35-year-old man with a history of drinking
tory of alcohol abuse and chronic hepatitis B one-two bottles of vodka per day for the past
virus infection has been experiencing fatigue, 15 years presents to the emergency department
weight loss, and vague abdominal pain for sev- because of massive hematemesis and severe
eral months. Physical examination reveals a epigastric pain. He takes antacids to manage
palpable mass in the liver. Before the mass can mild acid reflux but has no other known medi-
be surgically resected, the patient dies of respi- cal problems or medications. His temperature
ratory failure. The appearance of his liver at is 36.7°C (98.1°F), pulse is 110/min, respira-
autopsy is shown in the image. How does this tory rate is 23/min, and blood pressure is 80/40
lesion migrate to other organs in the body? mm Hg. Physical examination reveals a regu-
lar rate and rhythm with no murmurs and his
lungs are clear to auscultation. No jaundice

High-Yield Systems
is present. There is no abdominal tenderness
or distension, no hepatosplenomegaly, and
bowel sounds are present. His stool is negative
for blood. Which of the following is the most
likely diagnosis?
(A) Esophageal laceration
(B) Esophageal metaplasia
(C) Esophageal squamous cell carcinoma
(D) Esophageal stricture
(E) Esophageal varices
Reproduced, with permission, from USMLERx.com.
40. A 32-year-old woman complains of alternating
bouts of diarrhea and constipation and reports
(A) Contiguous spread
chronic abdominal pain relieved by frequent
(B) Direct dissemination into the peritoneal
bowel movements. Her symptoms are exac-
cavity
erbated by stress. The patient denies fever or
(C) Hematogenous dissemination

Gastrointestinal
weight loss. She has a negative fecal occult
(D) Lymphatic drainage
blood test. Colonoscopy and endoscopy reveal
(E) Secretion into bile
no abnormalities. The most likely diagnosis in
this patient is commonly associated with which
of the following findings?
(A) Leukocytosis
(B) Normal biopsy
(C) Primary sclerosing cholangitis
(D) Strictures in the small bowel
(E) Villous flattening in the small intestine
244 Section II: Organ Systems  •  Questions

41. An obese 40-year-old multiparous woman (A) Adenoid cystic carcinoma


comes to the physician because she has been (B) Mucoepidermoid carcinoma
experiencing right upper quadrant pain with (C) Pleomorphic adenoma
nausea and vomiting precipitated by fatty (D) Sialic duct stone
foods. Results of a right upper quadrant ul- (E) Warthin tumor
trasound are shown in the image. Laboratory
studies show: 43. A 45-year-old woman who was diagnosed with
scleroderma five years ago presents to her
Total cholesterol: 280 mg/dL
physician with increasing difficulty swallow-
LDL cholesterol: 170 mg/dL
ing. Which of the following abnormalities of
HDL cholesterol: 33 mg/dL
esophageal muscle function is the most likely
Triglycerides: 420 mg/dL
cause of these symptoms?
High-Yield Systems

Which of the following drugs is relatively con-


(A) Atrophy of smooth muscle in the lower
traindicated in this patient’s treatment?
half of the esophagus
(B) Atrophy of smooth muscle in the lower two
thirds of the esophagus
(C) Atrophy of smooth muscle in the upper
two thirds of the esophagus
(D) Atrophy of striated muscle in the lower
half of the esophagus
(E) Atrophy of striated muscle in the lower
two-thirds of the esophagus
(F) Atrophy of striated muscle in the upper
two-thirds of the esophagus

44. A 78-year-old man is brought to the hospital


because of fever and acute onset of left lower
quadrant abdominal pain. About a week ago,
Gastrointestinal

he was seen by his family physician for painless


Reproduced, with permission, from USMLERx.com. rectal bleeding. Laboratory tests show:
RBC count: 5 million/mm³
(A) Ezetimibe Hematocrit: 36%
(B) Gemfibrozil Hemoglobin: 12 g/dL
(C) Niacin WBC count: 93,000/mm³
(D) Pravastatin Mean corpuscular volume (MCV): 75 fL
(E) Simvastatin
Which of the following is the most appropriate
follow-up test after the patient is discharged?
42. A healthy 55-year-old woman presents to the
physician with a one-year history of an un- (A) Abdominal ultrasound
changing, non-painful palpable mass in her (B) Colonoscopy
left cheek. A parotid gland biopsy reveals (C) CT of the abdomen
groups of well-differentiated epithelial cells (D) MRI of the abdomen
in a chondromyxoid stroma surrounded by a (E) Upright abdominal X-ray
fibrous capsule; multiple cell types are visible
on light microscopy. The pathologic descrip-
tion of the mass is most consistent with which
of the following conditions?
Chapter 10: Gastrointestinal  •  Questions 245

45. A 26-year-old man presents to the clinic with 47. A newborn develops marked jaundice and
bradykinesia, rigidity, and resting tremor. Se- kernicterus within weeks of birth. Blood tests
rum aminotransferase levels are mildly el- reveal alanine aminotransferase = 16 U/L, as-
evated. A liver biopsy is shown in the image. partate aminotransferase = 14 U/L, and total
What is the chance that this patient’s sister will bilirubin = 3.8 mg/dL. Urine tests are negative
have the same condition? for bilirubin. Treatment attempts with pheno-
barbital, plasmapheresis, and phototherapy
are unsuccessful. The infant’s condition dete-
riorates over the next two months and he dies.
What is the underlying cause of the infant’s
death?
(A) Elevated antimitochondrial antibodies

High-Yield Systems
(B) Problem with bilirubin conjugation
(C) Problem with bilirubin uptake
(D) Problem with excretion of conjugated bili-
rubin
(E) Problem with hepatic copper excretion

48. A 45-year-old man presents to the emergency


department complaining of a high fever, mal-
Courtesy of Wikipedia.
aise, and confusion since waking up earlier in
the morning. He underwent abdominal sur-
(A) 0% gery two weeks ago and was discharged two
(B) 25% days postoperatively without complication.
(C) 50% On examination, his temperature is 39.0°C
(D) 66.6% (102.2°F), heart rate is 110/min, blood pres-
(E) 100% sure is 80/50 mm Hg, and respiratory rate is
18/min. His abdomen is warm and erythema-

Gastrointestinal
46. A woman comes to the physician because of tous, there is purulent discharge draining from
profuse vomiting and watery, non-bloody diar- the surgical incision site, and a rash is evident
rhea that developed five hours after she had on his chest and abdomen. The patient re-
eaten tuna salad. She is diagnosed with food ceives appropriate therapy. Days later, a blood
poisoning. Which of the following is the most culture reveals high levels of gram-positive
likely cause of her symptoms? bacteria, and molecular studies reveal high lev-
els of interleukin (IL)-1, IL-6, and tumor ne-
(A) A b-hemolytic, gram-positive rod crosis factor-a. What cellular process initiated
(B) A gram-positive, catalase-positive, this patient’s presentation?
coagulase-positive coccus
(C) A pre-formed enterotoxin (A) Antigen binding to normal cellular recep-
(D) A pre-formed superantigen tors and interfering with proper function
(E) An invasive gram-negative rod (B) Binding to the macrophage CD14 recep-
tor
(C) Free antigen cross-linking of mast cell IgE
(D) Protein A binding of the IgG Fc receptor
(E) Simultaneous antigen binding of the T-cell
receptor and major histocompatibility
complex class II
246 Section II: Organ Systems  •  Questions

49. A 40-year-old white man presents to the emer- 50. A 50-year-old man presents to his physician
gency department complaining of burning because of a 6.8-kg (15-lb) weight loss over the
retrosternal chest pain after meals. The pain past month, epigastric pain radiating to the
is relieved by antacids. The patient’s ECG is back, and jaundice. He also complains of red-
normal, and x-ray of the chest is remarkable ness, swelling, and tenderness of his left lower
for an 8-cm hiatal hernia. This patient is at risk extremity. Laboratory studies show an amylase
for developing which of the following types of level of 500 U/L, a lipase level of 300 U/L,
cancer? and an alkaline phosphatase level of 500 U/L.
Which tumor markers are most likely to be el-
(A) Adenocarcinoma of the esophagus
evated in this patient?
(B) Gastric adenocarcinoma
(C) Krukenberg tumor (A) a-Fetoprotein and b-human chorionic
(D) Non-small cell adenocarcinoma of the gon­adotropin
High-Yield Systems

lung (B) a-Fetoprotein and CA 19-9


(E) Squamous cell carcinoma of the esopha- (C) a-Fetoprotein and carcinoembryonic anti-
gus gen
(D) b-Human chorionic gonadotropin and car-
cinoembryonic antigen
(E) CA 19-9 and carcinoembryonic antigen
Gastrointestinal
Chapter 10: Gastrointestinal  •  Answers 247

An s w e r s

1. The correct answer is C. This patient has a tion of all major coagulation factors except
flare-up of her chronic hepatitis B virus (HBV) factor VIII.
infection, as evidenced by the presence of
Answer E is incorrect. A patient with chronic
HBsAg and anti-HBc IgM and lack of HBsAb.
liver disease would be more likely to have an
HBV typically presents with jaundice and right
elevation of transaminase levels, as these in-
upper quadrant pain and can be transmit-
dicate hepatocellular damage. If the patient’s
ted via parenteral, sexual, and maternal-fetal
liver disease has progressed to cirrhosis, one
routes. About 5% of adults with acute HBV
might expect the transaminase levels to be
infection will develop chronic hepatitis, and
normal since the liver would be small and fi-
12%-20% of these will go on to develop cirrho-

High-Yield Systems
brotic, thus no longer capable of releasing
sis. Chronic HBV infection is marked by the
large amounts of these enzymes. However, an
presence of HBsAg for >6 months. While most
abnormally low value would not be the most
patients will develop HBsAb and eliminate
likely finding.
HBsAg from the blood, chronically infected
patients do not. A patient with HIV may have 2. The correct answer is E. This is an example of
a history of risky sexual behavior and would volvulus, a twisting of the large intestine in a
be at increased risk for HBV infection; in fact, closed-loop obstruction. The elderly and debil-
chronic HBV infection affects about 10% of itated are at a particular risk for volvulus. This
HIV-infected patients worldwide. This patient’s patient has the common symptoms of colonic
signs of scleral icterus, ascites, and spleno- obstruction, with abdominal pain, abdomi-
megaly indicate that her chronic liver disease nal distention, and bloody stools. The image
may have progressed to cirrhosis. The fibrotic shows the nonspecific “double-bubble,” which
liver can induce portal hypertension, causing is a sign of proximal small-bowel obstruction.
engorgement of the spleen due to increased Colonoscopy frequently is both diagnostic and
pressures within the portal circulation. This therapeutic, as insufflations of air and passing
enlarged spleen sequesters increased numbers

Gastrointestinal
the colonoscope through the point of volvu-
of platelets within it. Thus in this patient we lus frequently results in reduction of the vol-
might expect a low platelet count. vulus. The condition frequently recurs, and
Answer A is incorrect. An elevation in alkaline definitive surgical treatment is a sigmoid co­
phosphatase activity occurs most commonly lectomy, though a sigmoidopexy (fixing of the
in obstructive liver disease due to a blockage sigmoid colon to anterior abdominal) can be
in the biliary tree caused by acute cholecysti- performed in those patients who are too sick to
tis, primary biliary cirrhosis, and other causes. tolerate an intestinal resection.
Because viral hepatitis is not an obstructive Answer A is incorrect. Duodenal hematoma is
disease, alkaline phosphatase level would not a potential consequence of abdominal trauma.
likely be abnormal. Signs and symptoms include pain, gastric dis-
Answer B is incorrect. A patient with chronic tention, and anorexia. Radiologic findings
liver disease would have an elevated serum can resemble an obstruction in the upper or
bilirubin level, because damage to hepatocytes middle gastrointestinal (GI) tract, and hence
would lead to difficulty releasing bilirubin into are not congruent with the image shown. The
the bile, causing it to leak out into the blood. treatment for duodenal hematomas usually
is nonoperative, as these hematomas often re-
Answer D is incorrect. A patient with chronic
solve on their own, except in the case of pene-
liver disease would have an increase in pro-
trating abdominal trauma, when the abdomen
thrombin time due to a decrease in coagula-
must be explored to the examine the duodenal
tion factors, as the liver is the site of produc-
248 Section II: Organ Systems  •  Answers

hematoma as well as search for other sites of agnosed ectopic pregnancy can result in mor-
injury). bidity and even death.
Answer B is incorrect. Esophageal strictures Answer A is incorrect. Surgery to remove the
can cause an obstruction in the upper GI tract, appendix would be indicated once appendicitis
leading to impaired passage of both food and had been confirmed by CT or upon ruling out
liquid. The consequences include vomiting, other significant diagnostic possibilities. Pro-
anorexia, and aspiration. On x-ray of the chest ceeding with appendectomy prior to ruling out
the esophagus is greatly distended, and stom- ectopic pregnancy would be inappropriate in a
ach often is collapsed. Because the obstruction young woman.
is proximal to the stomach, the vomitus usually
Answer B is incorrect. The triple therapy
does not include bile, and the ingested food
listed is indicated for the eradication of Helico-
usually is not digested.
High-Yield Systems

bacter pylori, the causative agent of most gas-


Answer C is incorrect. Intussusception occurs tric and duodenal ulcers. This patient’s severe,
when one segment of the intestine (frequently acute-onset pain in the right lower quadrant is
small intestine, or distal ileum into cecum) not suggestive of an ulcer.
telescopes into the immediately distal segment
Answer D is incorrect. Acute pancreatitis clas-
of the bowel. Intussusception is a common
sically presents with epigastric pain radiating to
cause of small-bowel obstruction in children,
the back, often preceded by nausea and vom-
but not in the adult or elderly populations.
iting. Elevated serum lipase levels are often
Furthermore, the radiologic findings make
present. This patient’s pain is not demonstra-
volvulus, and not intussusceptions (the classic
tive of that typically seen in acute pancreatitis.
radiographic finding of intussusceptions is the
“target sign”), the better answer. Answer E is incorrect. A stool culture would
be performed if the physician were suspicious
Answer D is incorrect. A perforated esopha-
of an infectious diarrhea. Although the “funky
gus, also known as Boerhaave syndrome, is a
tasting” potato salad might suggest infectious
medical emergency, because gastric contents
causes, this patient denies diarrhea, and thus
and air are released into the mediastinum.
the clinical suspicion remains low. Further-
Gastrointestinal

The patient usually experiences excruciating


more, a stool culture is certainly not the first
chest pain, dyspnea, dysphagia, and hemody-
test to be performed, as more pressing tests
namic instability. Radiography would reveal
such as measuring the b-hCG level would
air within the mediastinum. There would be
need to be done first.
no bilious vomiting or hematochezia. Fur-
thermore, the patient usually has a history of 4. The correct answer is A. This is a classic pre-
trauma or violent retching (most commonly sentation of cholelithiasis, or gallstones. Pa-
secondary to alcohol consumption). tients with cholelithiasis experience pain after
meals as a result of the duodenal release of
3. The correct answer is C. The possibility of ec-
cholecystokinin (CCK), which causes the gall-
topic pregnancy should always be considered
bladder to contract while the stone obstruct
in a woman of reproductive age who presents
the cystic duct. CCK is stimulated by fatty ac-
with abdominal pain, regardless of the patient’s
ids and amino acids.
history. The history of eating potato salad is
a distracter, and should not change the clini- Answer B is incorrect. Gastrin is released by
cian’s decision to measure the b-human cho- the G cells of the stomach in response to pro-
rionic gonadotropin (b-hCG). The symptoms teins or peptides in the stomach. Gastrin leads
of ectopic pregnancy may closely mimic those to increased secretion of gastric acid and low
of acute appendicitis, making measurement of pH inhibits its secretion, leading to a negative
the b-hCG level especially critical in cases of feedback loop. Gastrin does not have an effect
suspected appendicitis. Furthermore, an undi- on gallbladder contraction.
Chapter 10: Gastrointestinal  •  Answers 249

Answer C is incorrect. Pepsin is a digestive these levels are normal in this patient, so auto-
protease released by chief cells in the stom- immune hemolytic anemia is unlikely.
ach. Pepsinogen, which is the precursor pro-
Answer C is incorrect. Pegylated interferon is
tein, autocleaves in the acidic environment of
useful in treating chronic hepatitis C infection
the stomach. It is released under the influence
(HCV). HCV would present with increased
gastrin and the influence of the vagus nerve. It
aspartate aminotransferase and alanine amino-
does not cause gallbladder contraction.
transferase, and if advanced, signs of cirrhosis.
Answer D is incorrect. Somatostatin is release This patient has an isolated hyperbilirubin­
by the D cells of the duodenum. It reduces emia, so HCV is unlikely. An acute HCV infec-
smooth muscle contractions and inhibits the tion can present in a variety of ways, including
release of both insulin and glucagon from the hyperbilirubinemia, but interferon is not used
pancreas. It does not cause gallbladder con- for acute HCV, so this is not the best answer.

High-Yield Systems
traction.
Answer D is incorrect. Crigler-Najjar syn-
Answer E is incorrect. Vasoactive intestinal drome is similar to Gilbert syndrome in that
peptide induces smooth muscle relaxation in it is caused by a mutation in the gene coding
the lower esophageal sphincter, stomach, gall- for UDPGT. However, in Crigler-Najjar syn-
bladder and stimulate secretion of water into drome the enzyme is severely lacking or com-
pancreatic juice and bile. It also inhibits gas- pletely absent. Type I is most severe, causing
tric acid secretion and absorption from the in- death early in life from kernicterus. Type II is
testinal lumen. It causes relaxation rather than milder and more responsive to treatment. Phe-
contraction of the gallbladder. nobarbital is useful in lowering the bilirubin
concentration in type II but not type I disease.
5. The correct answer is B. This patient has Gil- This patient is asymptomatic and is presenting
bert syndrome, the most common inherited later in life; thus this cannot be Crigler-Najjar
disorder of bilirubin conjugation. It is due to a syndrome.
gene mutation that results in a decreased level
Answer E is incorrect. Ursodeoxycholic acid
of uridine diphosphate glucuronyl transferase
(ursodiol) is used for gallstone dissolution in

Gastrointestinal
(UDPGT). The disease is autosomal reces-
patients who cannot tolerate surgery. This pa-
sive; in the Western world, approximately 9%
tient has normal alkaline phosphatase levels,
of people are homozygous for the mutation,
which would be elevated in cholelithiasis. Fur-
and 30% are heterozygous (heterozygotes are
thermore, if this patient did have gallstones, he
asymptomatic). The disease commonly first
would likely be treated with surgery, which is
manifests in young adults after an inciting
much more effective than ursodiol.
event, such as a febrile illness, physical exer-
tion, stress, or fasting. Laboratory tests will
6. The correct answer is A. This is an acute pre-
demonstrate unconjugated hyperbilirubin­
sentation of Budd-Chiari syndrome, or throm-
emia, but will be otherwise normal. The dis-
bosis of two or more hepatic veins. This condi-
ease is benign and requires no treatment. This
tion is associated with hypercoagulable states
patient has a mild unconjugated hyperbilirubi-
such as myeloproliferative disorders, inherited
nemia and is otherwise healthy besides a mild
coagulation disorders, intra-abdominal can-
flu-like illness, so Gilbert is the most likely di-
cers, oral contraceptive use, and pregnancy.
agnosis.
The increased intrahepatic pressure leading to
Answer A is incorrect. Corticosteroids would ascites is present in 90% of patients with Budd-
be useful for treatment of autoimmune hemo- Chiari syndrome. The disease can also present
lytic anemia. In hemolytic anemia, lactate de- in a subacute manner or in a chronic man-
hydrogenase is commonly elevated, haptoglo- ner, and diagnosing this condition may then
bin is decreased, and hemoglobin is low. All of be more challenging because the classic triad
250 Section II: Organ Systems  •  Answers

of abdominal pain, hepatomegaly, and ascites toxin causes muscle paralysis by inhibiting the
may not be present. exocytosis of acetylcholine from presynpatic
neurons.
Answer B is incorrect. Asterixis is associated
with hepatic encephalopathy in the setting of Answer B is incorrect. Decreasing proton se-
liver failure. The underlying mechanisms link- cretion into the stomach lumen is the mecha-
ing liver failure to asterixis are not completely nism of action of proton pump inhibitors (eg,
understood. omeprazole), which are used to treat gastro-
esophageal reflux disease and dyspepsia as well
Answer C is incorrect. Esophageal varices are
as peptic ulcer disease. This patient has achala-
associated with portal hypertension, in which
sia, which is unrelated to acid secretion. This
portosystemic shunts occur to bypass the he-
mechanism would not relieve the patient’s
patic circulation and return blood to the heart.
symptoms.
High-Yield Systems

Answer D is incorrect. Skin hyperpigmenta-


Answer C is incorrect. Enhancing phosphory-
tion can occur in several settings, such as he-
lation of myosin light chains would enhance
mochromatosis or Addison disease (adrenal
muscle contraction, which is the opposite of
atrophy), but none of these diseases fit the
the mechanism of action of nitroglycerin, and
present clinical picture.
would worsen this patient’s symptoms.
Answer E is incorrect. Spider angiomata are
Answer D is incorrect. Inhibiting the degra-
usually associated with cirrhosis; their patho-
dation of acetylcholine is the mechanism of
genesis is not completely understood but they
action of acetylcholinesterase inhibitors (eg,
are believed to occur in the setting of defects
physostigmine), which are used to treat or-
in the metabolism of sex hormones. They can
ganophosphate poisoning and to reverse the
also be present in pregnant women and mal-
effects of neuromuscular junction blockers (eg,
nourished patients.
pancuronium). This mechanism would lead
7. The correct answer is A. The patient’s dys- to increased availability of acetylcholine at the
phagia for both solids and liquids suggest a neuromuscular junction, leading to more ac-
motility problem. Due to loss of the myen- tive muscle contraction, and would actually
Gastrointestinal

teric (Auerbach’s) plexus, the lower esophageal worsen this patient’s symptoms.
sphincter (LES) fails to relax. The majority of
8. The correct answer is B. The patient’s presen-
patients with achalasia have difficulty swallow-
tation is consistent with ruptured esophageal
ing both solids and liquids for >6 months, as
varices, a dangerous complication of portal hy-
well as an increased risk of esophageal cancer.
pertension. When the liver becomes extremely
Esophageal manometry is the gold standard
fibrotic, as it does with years of exposure to
of diagnosing achalasia by documenting loss
alcohol (note the jaundice and ascites in this
of coordinated peristalsis along the esophagus
patient), there is an increase in resistance in
and abnormally high lower esophageal sphinc-
blood flow through the liver, causing portal
ter tone. Upper endoscopy is also performed to
hypertension. When the pressure in the portal
rule out cancer. Surgical corrections include
system is greater than the systemic venous pres-
pneumatic dilation and esophageal myotomy.
sure, blood will find alternate routes to return
Medical approaches to treating achalasia in-
to the heart. One of those alternate routes is
clude using a calcium channel blocker, nitro-
from the left gastric vein into the azygos vein,
glycerin, or botulinum toxin. Calcium chan-
which leads to esophageal varices.
nel blockers such as nifedipine decrease the
availability of calcium to the myosin-actin Answer A is incorrect. The portal system is
complex, leading to smooth muscle relax- a system of veins that drain the GI tract and
ation. Nitroglycerin works through a cGMP- deliver the blood to the liver. The left gastric
mediated mechanism to dephosphorylate and artery is upstream of the portal system, and an
inactivate myosin light chains. Botulinum anastomosis between the left gastric artery and
Chapter 10: Gastrointestinal  •  Answers 251

vein would bypass the portal system altogether Answer E is incorrect. There is a fivefold in-
and therefore would not contribute to the for- creased incidence of GI tract cancer in pa-
mation of esophageal varices. tients with long-standing progressive Crohn
disease compared to age-matched patients.
Answer C is incorrect. Anastomoses between
Screening with yearly colonoscopy is usually
the paraumbilical vein and the inferior epi-
started approximately eight years after diagno-
gastric vein lead to the formation of caput me-
sis of Crohn disease.
dusae, the spokes-of-a-wheel veins that radiate
from the umbilicus in patients with portal hy-
10. The correct answer is B. Methicillin-resistant
pertension.
Staphylococcus aureus (MRSA) is an organ-
Answer D is incorrect. There is no natural ism that is resistant to traditional penicillin
route for blood to flow from the portal vein to family of antibiotics. Therefore, MRSA must

High-Yield Systems
the inferior vena cava. Creating a portacaval be treated with clindamycin, trimethoprim-
shunt, which is a current treatment option for sulfamethoxazole, or vancomycin. Watery,
portal hypertension, can relieve the pressure in foul-smelling stool in the presence of fever fol-
the portal system and reduce the risk of bleed- lowing antibiotic treatment usually is caused
ing from varices. by Clostridium difficile superinfection. Of
these various therapies for MRSA, clindamy-
Answer E is incorrect. Anastomoses between
cin is the only one that is known to cause a
the splenic vein and the left renal vein are
C difficile colitis from antibiotic-induced bac-
retroperitoneal vessels that are not near the
terial overgrowth. Clindamycin disrupts bac-
esophagus.
terial protein synthesis by blocking the 50S
9. The correct answer is D. The picture of in- subunit of the ribosome. It is used frequently
termittent abdominal pain, fever, and diar- to treat anaerobic infections and has proven ef-
rhea should lead you to a diagnosis of irritable ficacy against MRSA infections. In this patient,
bowel disease. The presence of an anal fistula clindamycin would have been a good choice
strongly suggests Crohn disease rather than for treatment of a community acquired MRSA
ulcerative colitis, given that it causes transmu- infection.

Gastrointestinal
ral inflammation. There are no renal disorders Answer A is incorrect. Blocking the 30S ribo-
associated with Crohn disease, so blood urea somal subunit is the main mechanism of the
nitrogen and creatinine would not be reason- aminoglycosides (for example, gentamicin)
able screening tests. and tetracycline. Gentamicin is an aminogly-
Answer A is incorrect. A much-feared extra- coside that works by binding to bacterial ri-
intestinal manifestation of Crohn disease is an- bosomes and preventing protein synthesis. It
terior uveitis. If untreated, it can lead to blind- is used to treat infections with gram-negative
ness. Migratory polyarthritis may also travel with rods. Adverse effects include nephrotoxicity
this condition and can develop either before or and ototoxicity. It is not a common treatment
soon after intestinal symptoms develop. for MRSA, nor a well-documented cause of
C difficile. Tetracyclines such as doxycycline
Answer B is incorrect. Primary scleros- may be used to treat MRSA but do not cause
ing cholangitis can occasionally occur with C difficile colitis.
Crohn disease, but the association is much
stronger in ulcerative colitis. Testing for el- Answer C is incorrect. Forming toxic metabo-
evated alkaline phosphatase would prompt in- lite is the main mechanism of metronidazole.
vestigation and further testing for primary scle- Metronidazole is the treatment of choice for
rosing cholangitis. C difficile superinfection, yeast infections, and
bacterial vaginosis. It destroys bacteria through
Answer C is incorrect. Iron deficiency anemia the production of toxic free radicals and is
may develop as a result of blood loss secondary used commonly to treat anaerobic and proto-
to GI tract inflammation. zoan infections. Metronidazole is not used to
252 Section II: Organ Systems  •  Answers

treat MRSA infections. Adverse effects of met- Answer A is incorrect. Ascaris lumbricoides is
ronidazole include a disulfiram-like reaction to associated with GI irritation, cough, and eo-
alcohol, GI upset, and headache. sinophilia.
Answer D is incorrect. Many antibiotics work Answer B is incorrect. Cigarette smoking is as-
by inhibiting cell-wall synthesis. A classic ex- sociated with many malignancies, particularly
ample is penicillin, which inhibits peptido- of the lung, pancreas, and esophagus; it has
glycan crosslinking. Another class of cell-wall not been linked to adenocarcinoma of the gall-
synthesis blocker is vancomycin, which binds bladder.
to the D-ala-D-ala portion of cell-wall precur-
Answer D is incorrect. Schistosoma haemato-
sors. Vancomycin, although a good treatment
bium infection is associated with the develop-
option for MRSA, rarely causes C difficile su-
ment of squamous cell carcinoma of the blad-
perinfection. It can be administered orally to
High-Yield Systems

der.
treat C difficile superinfection, but is generally
a second-line agent because of concern about Answer E is incorrect. Tuberculosis is associ-
the spread of vancomycin-resistant entero- ated with hemoptysis, cough, and weight loss.
cocci. Adverse effects of vancomycin include
nephrotoxicity, ototoxicity, and red man syn- 12. The correct answer is F. The patient has ul-
drome. cerative colitis. The leg lesions represent pyo-
derma gangrenosum and are the first clue of
Answer E is incorrect. Inhibiting the translo- an extra-intestinal manifestation of ulcerative
cation step of protein synthesis is the mecha- colitis. The diagnosis is confirmed with the
nism of macrolides (azithromycin, erythromy- biopsy showing that the inflammation is con-
cin). Macrolides are good choices for treating tained to the mucosal and the submucosal
community acquired pneumonia (particularly layers (remember that in Crohn disease the
azithromycin), but not MRSA-related infec- inflammation is transmural, leading to fistula
tions. Moreover, macrolides are not commonly formation). Sulfasalazine is a combination of
associated with pseudomembranous colitis. sulfapyridine, which is an antibacterial drug,
and mesalamine, which is an anti-inflamma-
11. The correct answer is C. The patient’s clini-
Gastrointestinal

tory drug. Its adverse effects include malaise,


cal presentation is consistent with adenocar-
nausea, sulfonamide toxicity, and reversible
cinoma of the gallbladder. Gallbladder ad-
oligospermia. Immunosuppressive drugs such
enocarcinoma is associated with chronic
as 6-mercatopurine and methotrexate can be
gallbladder inflammation, typically from a
used to treat ulcerative colitis and Crohn dis-
history of gallstones, which can be seen with
ease.
the thickening of the gallbladder wall on CT.
Gallbladder polyps, the polypoid lesion, are Answer A is incorrect. Infliximab is a mono-
also associated with an increased risk of gall- clonal antibody to tumor necrosis factor-a. It
bladder adenocarcinoma. The enlarged lymph is used to treat patients with Crohn disease, es-
nodes point to local invasion and spread, pecially when anal fistulas are present, but it is
which is unfortunately common on initial pre- second-line therapy for ulcerative colitis, after
sentation. Gallbladder cancer is a disease of the aminosalicylates (eg, sulfasalazine).
the elderly and is more common in women Answer B is incorrect. Nizatidine is a revers-
than men. Most (90%) patients with gallblad- ible histamine-2 blocker. It is used to treat
der cancer have concomitant stones. In gen- patients with peptic ulcer, gastritis, and mild
eral, the treatment for adenocarcinoma of the esophageal reflux. It has no role in the treat-
gallbladder is surgical excision but prognosis is ment of ulcerative colitis.
generally poor if not found incidentally.
Answer C is incorrect. Omeprazole is a pro-
ton pump inhibitor that irreversibly inhibits
sodium-potassium-ATPase in stomach parietal
Chapter 10: Gastrointestinal  •  Answers 253

cells. It is used in cases of peptic ulcer, gastri- Answer B is incorrect. Diphenoxylate is an


tis, esophageal reflux, and Zollinger-Ellison antidiarrheal opiate that is used in the manage-
syndrome. It has no role in the treatment of ul- ment of diarrhea. However, it would not be the
cerative colitis. treatment of choice, as this is likely a secretory
diarrhea and not due to increased peristalsis.
Answer D is incorrect. Ondansetron is a
Anti-peristaltic medications also exacerbate in-
5-­
hydroxytryptamine-3 antagonist that serves
fections and would be undesirable in this case
as a powerful central-acting anti-emetic. It can
of possible viral/bacterial infection.
be used to treat patients symptomatically for
nausea and vomiting, but it has no role in the Answer C is incorrect. A bolus of normal sa-
treatment of ulcerative colitis. line could be given immediately to a child
presenting with dehydration. However, in-
Answer E is incorrect. Sucralfate binds to an
travenous (IV) normal saline with dextrose is

High-Yield Systems
ulcer base to provide physical protection. It
considered maintenance IV fluid and usually
improves ulcer healing, and it also is used to
would not be given immediately. In addition,
treat traveler’s diarrhea. It has no role in the
IV fluids would be less desirable than ORT
treatment of ulcerative colitis.
because of the potential risk of developing
13. The correct answer is E. The presentation iatrogenic electrolyte imbalances such as hy-
and history suggest cholera. Cholera is caused pernatremia. When patients present with se-
by Vibrio cholerae, a gram-negative comma- vere dehydration (hypotensive, anuric, sunken
shaped bacterium. It can cause profuse secre- eyes, cool skin with acrocyanosis, mental status
tory diarrhea with watery vomiting in some changes, tachypneic, >10% volume loss), then
patients. It has been a problem in many parts rapid IV infusion of isotonic saline is indicated.
of the world, especially after natural disasters, Answer D is incorrect. Loperamide is an an-
as mortality in untreated patients exceeds 50%. tidiarrheal opiate that inhibits peristalsis by in-
Initial management in the setting of moder- terfering with calcium channels. Although this
ate or mild dehydration secondary to diarrhea medication is used in diarrhea management, it
is oral rehydration therapy (ORT). The pre- would not be the treatment of choice, as this
sentation of these patients suggests moderate

Gastrointestinal
is likely a secretory diarrhea and not due to in-
dehydration. ORT consists of administering creased peristalsis. Anti-peristaltic medications
glucose-containing sodium solution in a ratio also exacerbate infections and would be unde-
not exceeding two glucose molecules per one sirable in this case of possible viral/bacterial in-
sodium molecule. Dehydration is prevented fection.
by shifting fluid from the intestinal lumen into
the circulation, secondary to glucose-coupled 14. The correct answer is B. Patients with gastric
sodium transport in the mucosal cells. cancer tend to present with abdominal pain,
anorexia, early satiety, nausea, and/or dyspha-
Answer A is incorrect. Although antibiotics
gia. Weight loss tends to be secondary to in-
might be helpful in treating a bacterial diar-
sufficient food intake, but may also be due to
rhea, the first-line treatment for dehydration of
gastric stasis or outlet obstruction. The abdom-
any etiology is to correct the volume loss. This
inal pain progressively worsens as the disease
is achieved by ORT. Although one could make
spreads. Lymphatic spread is common and re-
a case for using antibiotics concomitantly with
sulting examination findings may show a peri-
ORT, almost all cases of cholera subside if the
umbilical nodule (Sister Mary Joseph’s node)
patient is well hydrated. Furthermore, antibiot-
as well as left supraclavicular adenopathy (Vir-
ics are not recommended in certain bacterial
chow node).
diarrheas such as with Salmonella infection
in which antibiotics can, in fact, lengthen the Answer A is incorrect. Cancer of the body
course and severity of the disease. or tail of the pancreas typically presents with
pain and weight loss, while cancer in the head
254 Section II: Organ Systems  •  Answers

of the pancreas tends to present earlier, with also be noted that this patient reports no
steatorrhea, weight loss, and jaundice. Other changes in his normal bowel habits. If a bowel
presenting symptoms include recent onset of obstruction were producing his symptoms,
atypical diabetes mellitus (DM), unexplained he would not be passing gas or having bowel
thrombophlebitis, and unexplained pancreati- movements.
tis.
Answer C is incorrect. Enterocolitis would not
Answer C is incorrect. Nonmetastatic breast present with signs and symptoms of ascites. It
cancer does not present with abdominal pain. usually manifests with diarrhea; however, this
patient’s current symptoms do not include
Answer D is incorrect. Esophageal cancer
changes in bowel habits. It is not a chronic dis-
tends to present with dysphagia associated with
ease and cannot cause portal hypertension.
weight loss as well as regurgitation of food.
High-Yield Systems

Hoarseness may be a symptom if the tumor has Answer E is incorrect. Ruptured viscera may
invaded the recurrent laryngeal nerve. present with signs of peritonitis such as re-
bound tenderness. Rupture may result from
Answer E is incorrect. Krukenberg tumor
ischemic bowel disease or obstruction. This pa-
is secondary to peritoneal spread of gastric
tient’s history, physical examination, and imag-
cancer and tends to present with an enlarged
ing studies are inconsistent with this etiology.
ovary.
A ruptured viscus is often fatal within several
15. The correct answer is D. Schistosomiasis is days because of infection and subsequent sep-
a parasitic disease with hepatic involvement. sis.
Schistosoma mansoni larva, which are com-
16. The correct answer is C. Cushing ulcer is
monly found in fresh waters of South America,
a type of acute stress ulcer that is associated
penetrate the host’s skin, invade the peripheral
with elevated intracranial pressure in trauma
vasculature, and eventually settle in the portal
or severe illness. Increased intracranial pres-
or pelvic venous vasculature. Several weeks
sure stimulates the vagus nucleus, causing in-
following infection, patients may develop
creased acid secretion in the stomach. A Curl-
symptoms similar to the ones described, such
ing’s ulcer tends to occur in the fundus of the
Gastrointestinal

as fever, diarrhea, and weight loss; the “funny


stomach and results in superficial capillary
looking” stools likely represent S mansoni eggs.
bleeding, but may also involve heavier bleed-
Chronic infection may eventually lead to por-
ing if the submucosa is eroded. In this case,
tal hypertension and hepatosplenomegaly,
the elevated intracranial pressure from the
leading in turn to ascites and eventually cirrho-
high-speed collision is markedly demonstrated
sis. In addition, the hepatosplenomegaly leads
as papilledema in the image.
to esophageal varices, producing bleeding that
can often be the first clinical sign. Answer A is incorrect. This answer choice de-
scribes the mechanism behind gastroesopha-
Answer A is incorrect. Appendicitis commonly
geal reflux disease (GERD). Esophageal irrita-
presents with right lower quadrant abdominal
tion by acid reflux is generally associated with
pain, fever, nausea, vomiting, and leukocytosis.
intestinal metaplasia of the esophagus and Bar-
The ascites seen in this patient is not a typical
rett esophagus with predisposition to esopha-
finding with appendicitis.
geal cancer. GERD is unlikely to be the cause
Answer B is incorrect. Bowel obstruction gen- for bleeding ulcers.
erally presents with nausea, vomiting, and de-
Answer B is incorrect. Esophageal varices
creased or absent bowel sounds. This patient
are commonly caused by portal hypertension,
has none of these signs or symptoms. The pa-
which is frequently seen in chronic alcoholics
tient does not have risk factors for bowel ob-
or in patients with cirrhosis. Elevated hepatic
structions, which include hernias, previous
portal pressure can cause the development
abdominal surgeries, or colon cancer. It should
of lower esophageal varices, which can be a
Chapter 10: Gastrointestinal  •  Answers 255

source of massive bleeding. This patient does ecules and inhibits binding to the TNF-a re-
not have a history of alcohol use or cirrhosis. ceptor, thereby preventing receptor activation
and its subsequent effects. Infliximab does not
Answer D is incorrect. Duodenal ulcers are
bind to cell surface receptors.
a type of peptic ulcer that is highly associated
with Helicobacter pylori. The patient such an Answer D is incorrect. Imatinib (Gleevec) is
ulcer often complains of abdominal pain that a tyrosine kinase receptor inhibitor. The tyro-
is ameliorated by eating. The peptic ulcers sine kinase receptor is mutated in the bcr-abl
are characteristically more insidious and often protein that results from the 9;22 translocation
linked to H pylori, not to the acute stress asso- (Philadelphia chromosome). This mutation
ciated with trauma or severe illness. has been linked with chronic myelogenous
leukemia and some GI stromal tumors. It re-
Answer E is incorrect. Oral analgesics such
sults in an overly active abl tyrosine kinase that

High-Yield Systems
as nonsteroidal anti-inflammatory drugs
has oncogenic effects.
(NSAIDs), when used long-term, are associ-
ated with gastric ulcerations and superficial Answer E is incorrect. Infliximab binds
erosions. Given that the patient was just in- soluble TNF-a but does not affect the total
jured two days ago, not only is there insuffi- amount of TNF-a produced by macrophages.
cient exposure to oral NSAIDs, but he is likely
still on IV analgesia. Also, NSAID use is not 18. The correct answer is B. This patient suffers
associated with the papilledema shown in the from Hirschsprung disease, which manifests
image. when neural crest cells fail to migrate to the
distal colon. Consequently, enteric neurons
17. The correct answer is B. Recent studies have do not form in a segment of the rectosigmoid;
demonstrated the benefit of infliximab (Remi- these neurons are normally responsible for re-
cade) in the treatment of Crohn disease that laxation of the rectum to allow defecation. If
is refractory to steroid treatment. It also is ap- this condition is left untreated, these infants
proved for use in a variety of other autoim- run the risk of developing enterocolitis or a
mune diseases such as ulcerative colitis, an- bowel perforation. Ten percent of cases of
kylosing spondilitis, psoriasis, and psoriatic Hirschsprung disease occur in children with

Gastrointestinal
arthritis. Infliximab is a monoclonal chimeric Down syndrome, caused by trisomy 21. Chil-
antibody that binds soluble tumor necrosis dren with Down syndrome also have an in-
factor-a (TNF-a), and as a result blocks its ef- creased risk of duodenal atresia, congenital
fects. TNF-a is a pro-inflammatory cytokine heart disease, and acute lymphoblastic leuke-
secreted by macrophages that is found in high mia.
concentrations in the stool of Crohn patients.
Answer A is incorrect. Cystic fibrosis (CF) is
The chimeric antibody is 75% human and
caused by recessive mutations in the CFTR
25% murine. A single infusion produces a
gene on chromosome 7. Infants with CF have
clinical response in 65% of patients. Common
an increased risk of meconium ileus, and also
adverse effects are increased susceptibility to
show a variable spectrum of pancreatic insuffi-
upper respiratory infections, headache, and GI
ciency. Later in life, patients with CF develop
distress.
characteristic bronchiectasis with chronic lung
Answer A is incorrect. The drug trastuzumab infections with Pseudomonas species. There
(Herceptin) is a monoclonal antibody spe- is, however, no association between CF and
cific for c-erbB2 (HER-2/neu), the epidermal Hirschsprung disease.
growth factor receptor. Over-expression of this
Answer C is incorrect. Sickle cell disease
cell marker is a poor prognostic factor for car-
is caused by homozygous mutations in the
cinoma of the breast.
b-globin gene. Neonates with sickle cell disease
Answer C is incorrect. Infliximab remains are often asymptomatic as long as fetal hemo-
in the serum, and binds soluble TNF-a mol- globin persists. There is no known association
256 Section II: Organ Systems  •  Answers

between sickle cell disease and Hirschsprung Answer C is incorrect. Vomiting typically in-
disease. duces a metabolic alkalosis due to a loss of hy-
drogen ions from the stomach, leading to an
Answer D is incorrect. Tay-Sachs disease is
increase in pH. This leaves an increased bicar-
caused by a deficiency in the hexosaminidase
bonate concentration (generally >24 mEq/L)
A enzyme, leading to accumulation of GM2
in the bloodstream; therefore this patient does
ganglioside. Infants with Tay-Sachs disease
indeed have a metabolic alkalosis. However,
show progressive neurodegeneration, cherry
some patients present with more than one con-
red spots on the retina, and developmental de-
dition causing an acid-base imbalance, which
lay. There is, however, no known association
is known as a complicated or mixed condition.
between Tay-Sachs disease and Hirschsprung
If a metabolic acidosis were occurring simulta-
disease.
neously (such as in ketoacidosis or diarrhea),
High-Yield Systems

Answer E is incorrect. Infants with Turner the bicarbonate level would be closer to nor-
syndrome (XO) commonly show coarctation of mal because the two processes would have op-
the aorta, but there is no association between posing effects on bicarbonate levels and effec-
Turner syndrome and Hirschsprung disease. tively cancel each other out.

19. The correct answer is F. This patient is pre- Answer D is incorrect. This patient has a met-
senting with only slight alkalemia (normal abolic alkalosis, as evidenced by the increased
arterial pH is 7.35-7.45). However, the bicar- pH with increased bicarbonate level. PCO2
bonate level is substantially elevated, about 11 is elevated as well, indicating that appropriate
mEq/L above normal. This implies that a met- res­piratory compensation is occurring. If a si-
abolic alkalosis is occurring, which can be ex- multaneous respiratory acidosis were occur-
plained by the patient’s recent history of severe ring, the PCO2 would be even higher than
protracted vomiting. Vomiting causes a loss of anticipated based on normal compensatory
hydrochloric acid from the GI tract; this acid mechanisms. Generally, PCO2 should rise by
must be replaced, which is done by drawing 0.7 mm Hg for every 1-mEq/L increase in bi-
hydrogen from body stores and leaving bicar- carbonate level. So if carbon dioxide levels in-
bonate behind. Normally with this level of bi- crease by more than that, a coexistent respira-
Gastrointestinal

carbonate elevation alone, a higher pH would tory acidosis should be suspected.


be expected. However, this patient’s partial car- Answer E is incorrect. This patient has an al-
bon dioxide pressure (PCO2) is elevated to 48 kalosis, as evidenced by the elevated pH with
mm Hg. This implies that a normal respiratory increased bicarbonate. PCO2 is elevated as
compensation has occurred in order to nor- well, indicating that there is appropriate res­
malize the pH by retaining acid by the forma- piratory compensation occurring. If a simul-
tion of carbonic acid from carbon dioxide. The taneous respiratory alkalosis were occurring,
expected compensation is an increase of 0.7 the PCO2 would be normal (meaning there is
mm Hg of carbon dioxide for every 1-mEq/L a failure of respiratory compensatory mecha-
increase in bicarbonate. This patient’s bicar- nisms) or it would be decreased. In an isolated
bonate level has increased by about 11 mEq/L, metabolic alkalosis, respiratory compensation
and the PCO2 has increased by about 8 mm should occur. If it does not or if PCO2 is low, a
Hg, an appropriate compensation. concurrent respiratory alkalosis should be sus-
Answer A is incorrect. Because this patient’s pected.
pH is greater than normal, an alkalosis must be
20. The correct answer is A. The elevated direct
present.
bilirubin level and the imaging findings are
Answer B is incorrect. Metabolic acidosis with consistent with a common duct obstruction,
respiratory compensation would result in a low most likely secondary to a gallstone (consider
pH, a low bicarbonate level, and a decreased this woman’s risk factors: female, fertile, and
PCO2. >40 years). An obstruction in the common
Chapter 10: Gastrointestinal  •  Answers 257

duct (choledocholithiasis) does not allow gery. Other conditions that predispose patients
drainage of bile from the liver or gallbladder, to small bowel obstructions are hernias and in-
and can lead to cholangitis, which is charac- traluminal cancers of the small intestine.
terized by Charcot’s triad of jaundice, fever,
Answer A is incorrect. Celiac sprue can pre­
and right upper quadrant pain. About 80% of
sent with GI upset, diarrhea, and flatulence,
gallstones are made of cholesterol; these oc-
but does not have a correlation with small
cur when solubilizing bile acids and lecithin
bowel obstruction. Dermatitis herpetiformis
are overwhelmed by excess cholesterol. The
is a dermatologic manifestation of gluten in-
remaining 20% are pigment stones containing
sensitivity that presents with erythematous,
mainly calcium bilirubinate; these can occur
grouped, pruritic papules.
during periods of increased hemolysis.
Answer B is incorrect. Infection with Helico-
Answer B is incorrect. No clear relationship

High-Yield Systems
bacter pylori is limited to gastric and duodenal
has been established between gallstone forma-
mucosa and is not known to cause strictures
tion and diet, but it has been suggested that
or adhesions. Therefore even chronic infec-
low-fiber and high-cholesterol diets are con-
tions have no potential to cause small bowel
tributing factors.
obstructions. Long-standing H pylori infection
Answer C is incorrect. A tumor at the head of can cause MALT lymphoma and peptic ulcers.
the pancreas could cause this patient’s signs
Answer D is incorrect. Although smoking
and symptoms, but the patient would likely be
has multiple detrimental effects on health, it
older and present with other associated symp-
has no correlation to the progression of small
toms such as weight loss and back pain.
bowel obstructions.
Answer D is incorrect. Although the patient’s
Answer E is incorrect. Ulcerative colitis does
age and sex are consistent with a diagnosis of
not predispose patients to small bowel obstruc-
primary biliary cirrhosis, endoscopic retrograde
tion because this inflammatory disease process
cholangiopancreatography would demonstrate
is limited to mucosa and submucosa. Crohn
the small intrahepatic bile duct destruction as-
disease, on the other hand, can lead to small
sociated with this autoimmune disease. Addi-
bowel obstruction, as it can cause transmural

Gastrointestinal
tionally, the patient’s lab work-up would reveal
inflammatory and bowel strictures.
increased serum mitochondrial antibodies.
Answer E is incorrect. Primary sclerosing 22. The correct answer is A. Familial adenoma-
cholangitis also may cause Charcot’s triad. tous polyposis (FAP) is an autosomal domi-
However, endoscopic retrograde cholangio- nant condition characterized by a germline
pancreatography would reveal “beading,” or mutation on chromosome 5, specifically at
alternating strictures and dilation of the biliary the adenomatous polyposis coli (APC) lo-
tree due to intra- and extrahepatic inflamma- cus. The APC gene is thought to have tumor-
tion and fibrosis of the bile ducts. This disease suppressive effects, and its loss is associated
frequently is associated with ulcerative colitis. with more than colonic cancers. In addition to
duodenal neoplasms for which these patients
21. The correct answer is C. This patient has a with FAP must undergo lifelong upper en-
small bowel obstruction. Dilated loops of small doscopic surveillance, increased risk exists in
intestines on x-ray of the abdomen and a clini- these patients for developing gastric, liver, thy-
cal history of anorexia, vomiting, and abdomi- roid, and central nervous system neoplasms.
nal pain are usually sufficient to make the di-
Answer B is incorrect. Examples of autosomal
agnosis. In the United States, the leading cause
recessive conditions include cystic fibrosis,
of small bowel obstructions is adhesion forma-
sickle cell anemia, and hemochromatosis.
tion, which obstructs the lumen of the small
bowel. These adhesions are formed during the Answer C is incorrect. Examples of autosomal
healing process secondary to abdominal sur- trisomy conditions include Down syndrome
258 Section II: Organ Systems  •  Answers

(trisomy 21), Edwards syndrome (trisomy 18), pharyngitis or culture findings of gram-positive
and Patau syndrome (trisomy 13). cocci in chains.
Answer D is incorrect. Examples of conditions Answer C is incorrect. Fever, a new murmur,
related to sex chromosome abnormalities in- Janeway lesions, and nail bed hemorrhages
clude Klinefelter syndrome (XXY), Turner syn- are signs of bacterial endocarditis. Acute en-
drome (XO), and double-Y males (XYY). docarditis is most often caused by Staphylococ-
cus aureus, and subacute infection are often
Answer E is incorrect. Examples of X-linked
caused by viridans group streptococci. Endo-
recessive conditions include hemophilia A and
carditis is not a known sequelae of C jejuni in-
B, glucose-6-phosphate dehydrogenase defi-
fection.
ciency, and Lesch-Nyhan syndrome.
Answer D is incorrect. Waterhouse-
23. The correct answer is E. Guillain-Barré syn-
High-Yield Systems

Friderichsen syndrome is characterized by


drome is characterized by rapidly progress- high fever, shock, purpura, and adrenal insuffi-
ing ascending paralysis. It is an autoimmune- ciency. It is classically associated with Neisseria
mediated illness that can occur following a meningitides septicemia. This patient’s symp-
variety of infectious diseases, but is particularly toms do not resemble meningitis nor does the
associated with prior infection by Campylo- culture match the gram-negative diplococci of
bacter jejuni. C jejuni gastroenteritis is char- Neisseria.
acterized in this patient by her vomiting and
bloody diarrhea together with the finding of 24. The correct answer is C. This patient most
comma-shaped organisms with a single polar likely has vitamin A deficiency, which is char-
flagellum when cultured at 42°C in a micro- acterized by early symptoms of night blind-
aerophilic environment. Other enteric patho- ness, dry conjunctivae, and gray plaques, or
gens with this morphology include bacteria of late symptoms of corneal ulceration and ne-
the Vibrio genus (V cholera and V parahaemo- crosis leading to perforation and blindness.
lyticus). These species, however, are not en- This deficiency is typically seen in children
demic to the United States and would not be and pregnant women whose diets are deficient
expected in a patient without a recent travel in vitamin A. It can also be seen in alcoholics,
Gastrointestinal

history, nor do the symptoms match. C jejuni after intestinal surgery (especially when the il-
is transmitted to humans via the fecal-oral eum is involved), and in patients with fat mal-
route from either domestic animals or eating absorption, cholestasis, or inflammatory bowel
undercooked poultry. disease. Vitamins A, D, E, and K (fat-soluble
vitamins) are absorbed in the small intestine
Answer A is incorrect. Hemolytic-uremic syn-
and absorption requires micelles formed with
drome (HUS) is characterized by acute renal
bile salts.
failure and thrombocytopenia with hemolytic
anemia. HUS can be a complication of infec- Answer A is incorrect. Ferrous iron is ab-
tion caused by Escherichia coli O157:H7, not sorbed in the small intestine and absorption re-
Campylobacter jejuni. Like C jejuni, E coli quires apoferritin binding. Iron deficiency may
O157:H7 is a gram-negative rod that can cause be caused by chronic bleeding, inadequate
enteritis. E coli, however, is characterized by intake of iron, or malabsorption syndromes.
numerous flagella and aerobic metabolism. Symptoms include fatigue, pallor, or weakness.
Iron deficiency may affect many other pro-
Answer B is incorrect. Rheumatic fever is
cesses because iron is needed for the normal
characterized by fever, migratory polyarthritis,
functioning of many enzymes.
and carditis. It is known to occur after Group A
streptococcal pharyngitis. There is no evidence Answer B is incorrect. Vitamin B12 is a water-
of Streptococcus pyogenes infection in this soluble vitamin that is absorbed in the termi-
patient, as she does not have symptoms of nal ileum. Its absorption requires binding to
intrinsic factor, which is secreted by gastric pa-
Chapter 10: Gastrointestinal  •  Answers 259

rietal cells. Thus, in pernicious anemia, where stellation of immunologically related condi-
there is damage to gastric parietal cells, vita- tions, PSC is not one of them.
min B12 deficiency may develop and may lead
Answer E is incorrect. Wilson disease does
to megaloblastic anemia.
cause hepatic manifestations such as chronic
Answer D is incorrect. Water-soluble vitamins hepatitis, which may lead to jaundice and
are absorbed in the small intestine and absorp- cirrhosis. However, the obliteration of intra-
tion requires sodium cotransport. Vitamin A is hepatic ducts seen on imaging studies is con-
a fat-soluble vitamin. sistent with PSC, rather than hepatitis. Fur-
thermore, hepatitis would not explain the
Answer E is incorrect. Calcium is absorbed in
purpose of his total colectomy nor his colon
the small intestine and absorption is facilitated
pathology shown in image.
by a vitamin D-dependent calcium binding

High-Yield Systems
protein. Hypocalcemia may result in numer-
26. The correct answer is A. The patient’s clinical,
ous situations, including parathyroid hormone
laboratory, and histologic features are indica-
deficiency, eating disorders, and following
tive of celiac sprue. The classical presentation
parathyroidectomy. Symptoms include perioral
of this disease is during infancy, but it may
tingling, ECG changes, tetany, and Chvostek
also present any time between the ages of 10
sign (tapping the facial nerve will produce fa-
and 40. Classic signs include diarrhea, foul-
cial spasms on the same side).
smelling, bulky, floating stools, weight loss,
growth failure, and vitamin deficiencies. These
25. The correct answer is D. The image shows
symptoms follow exposure to the protein glia-
inflammatory pseudopolyps in a patient with
din, which results in intestinal inflammation.
ulcerative colitis. Primary sclerosing cholangi-
A gluten-free diet usually relieves symptoms
tis (PSC) is an extraintestinal complication of
and restores mucosal histology, and therefore it
ulcerative colitis. About 70% of patients with
is the most appropriate therapeutic measure.
PSC also have ulcerative colitis; however, only
about 4% of patients with ulcerative colitis will Answer B is incorrect. Corticosteroids may be
develop PSC. PSC leads to obliterative fibrosis employed in the treatment of refractory sprue,
of intrahepatic and extrahepatic bile ducts, and but a gluten-free diet is the most appropriate

Gastrointestinal
can over time lead to cirrhosis. Patients with step at this time.
PSC also have an increased risk of developing
Answer C is incorrect. Antibiotic therapy does
cholangiocarcinoma.
not have a place in the treatment of celiac
Answer A is incorrect. HCV infection can sprue.
cause cirrhosis, and subsequent hepatic adeno-
Answer D is incorrect. While patients with
carcinoma, but it is not associated with PSC.
celiac sprue are often deficient in B vitamins,
Answer B is incorrect. Primary biliary cirrho- treating the underlying pathology is the best
sis is a disease commonly found in middle- course.
aged women that affects smaller bile ducts
Answer E is incorrect. While patients with ce-
and, as the name suggests, can lead to cirrhosis
liac sprue are often deficient in fat-soluble vita-
over time. The etiology is likely autoimmune,
mins, treating the underlying pathology is the
and 90% of patients have antimitochondrial
best course.
antibodies. This disease can manifest with de-
struction of intrahepatic ducts. However, this 27. The correct answer is C. Pegylated inter-
would not explain this patient’s prior colon re- feron is a cytokine derivative that improves
section nor his colon pathology. the body’s antiviral response. It is used in the
Answer C is incorrect. While systemic lupus treatment of HBV and HCV. Adverse effects
erythematosus (SLE) is associated with a con- of interferon therapy include a flu-like reac-
tion that manifests as episodic fevers and chills,
260 Section II: Organ Systems  •  Answers

as well as occasional profound depression. As Answer D is incorrect. The sciatic nerve in-
a result, interferon is contraindicated in se- nervates the hip joint, the muscular knee
verely depressed or suicidal patients. Although flexors in the thigh, and all the leg and foot
interferon is not a cure for hepatitis, it is rec- muscles. These muscles are not utilized in the
ommended to slow the progression of cirrhotic psoas sign maneuver.
liver disease in some patients. Pegylated inter-
Answer E is incorrect. The superior gluteal
feron is a longer-acting form of interferon.
nerve innervates the gluteus medius and glu-
Answer A is incorrect. IV immunoglobulin is teus minimus muscles. These muscles are not
an engineered antibody that is used to clear utilized in the psoas sign maneuver.
the serum of protein products. It often is used
in treatment of autoimmune diseases such as 29. The correct answer is D. Shigella species pro-
Guillain-Barré syndrome. Adverse effects in- duce gastroenteritis characterized by abdomi-
High-Yield Systems

clude flu-like reaction and anaphylactic reac- nal pain, bloody diarrhea, and nausea and/
tion. or vomiting. Additionally, because Shigella
species invade intestinal epithelial cells, the
Answer B is incorrect. Lamivudine is a
illness is accompanied by fever. Shigella is a
nucleo­tide reverse transcriptase inhibitor used
nonlactose fermenter, and it does not produce
in the treatment of HIV and HBV. Its principal
gas or hydrogen sulfide. Infection usually af-
adverse effect is hepatotoxicity.
fects preschool-age children and populations
Answer D is incorrect. Ribavirin is an antiviral in nursing homes. Transmission occurs by the
drug used in the treatment of HCV, respiratory fecal-to-oral route via fecally contaminated wa-
syncytial virus, and, occasionally, other viral ill- ter and hand-to-hand contact. It’s an extremely
nesses. It is not associated with depression or a virulent organism requiring only 10 organisms
flu-like reaction. for infection. Shigella also produces Shiga
toxin, which can cleave host rRNA and en-
Answer E is incorrect. Tumor necrosis
hance cytokine release, resulting in hemolytic
factor-a is a cytokine involved in the antiviral
uremic syndrome. This syndrome develops af-
and antitumor response. It is not used cur-
ter the endothelium is damaged in the kidney
rently as a treatment for HCV.
Gastrointestinal

and results in renal failure, thrombocytopenia,


28. The correct answer is B. This patient presents and microangiopathic hemolytic anemia.
with the classic signs and symptoms of appen- Answer A is incorrect. This describes
dicitis. When the right leg is hyperextended, Guillain-Barré syndrome, which is often as-
the iliopsoas muscle group pushes against the sociated with Campylobacter jejuni infection.
appendix and causes significant pain and ir- While C jejuni is a common cause of dysen-
ritation. Pain with hyperextension will also tery, it does not fit this lab description (it’s a
be present in pancreatic cancers and inflam- gram-negative, curved rod).
mation of the cecum and the sigmoid colon.
Answer B is incorrect. These two symptoms,
The psoas muscle is innervated by the lumbar
along with diarrhea and fever, are character-
plexus, and the iliacus muscle is innervated by
istic of typhoid fever caused by Salmonella ty-
the femoral nerve.
phi. However, unlike S enteritidis, S typhi does
Answer A is incorrect. The inferior gluteal not cause dysentery. Furthermore, Salmonella
nerve innervates the gluteus maximus. This produce hydrogen sulfide, which Shigella does
muscle is not utilized in the psoas sign maneu- not. Salmonella infection results from fecal-
ver. oral transmission and the pathogen lives in the
Answer C is incorrect. The obturator nerve GI tract of animals. It infects humans when
originates from the lumbar plexus and inner- there is contamination of food or water by ani-
vates the medial thigh muscles. These muscles mal feces.
are not utilized in the psoas sign maneuver.
Chapter 10: Gastrointestinal  •  Answers 261

Answer C is incorrect. This describes an in- acidity. It maintains duodenal pH by stimulat-


haled infection by Bacillus anthracis, which ing bicarbonate secretion by the pancreas, thus
is not connected to any of the other symptoms neutralizing gastric acid. It is not indicated for
this child has. the treatment of acromegaly and does not con-
trol pancreatic enzyme secretion.
Answer E is incorrect. These symptoms de-
scribe rheumatic fever, which is caused by an Answer E is incorrect. VIP is a hormone pro-
infection by Streptococcus pyogenes. S pyo- duced in the intestines and pancreas. In the
genes does not cause dysentery, nor does it fit intestines VIP induces smooth muscle relax-
the laboratory description of the causative or- ation, whereas in the pancreas it stimulates
ganism in this case. pancreatic bicarbonate secretion. VIP is not in-
dicated for the treatment of acromegaly.
30. The correct answer is D. This patient’s symp-

High-Yield Systems
toms and pancreatic calcifications on the CT 31. The correct answer is A. This patient is likely
scan are consistent with chronic pancreatitis. suffering from a duodenal ulcer. Helicobacter
The intern is thinking about octreotide, a so- pylori is the most common cause of duodenal
matostatin analog used to treat acromegaly. and gastric ulcers (involved in 100% and 70%
Among its various actions, somatostatin is a of lesions, respectively). It can be diagnosed
potent inhibitor of growth hormone secre- with esophagogastroduodenoscopy or a urease
tion; it also suppresses the release of a number breath test. A key distinction between these
of digestive hormones, such as gastrin, chole- two ulcers is that eating food often relieves du-
cystokinin, secretin, and vasoactive intestinal odenal ulcer pain and patients tend to report
peptide (VIP). It also decreases the secretion of resulting weight gain. Duodenal ulcer symp-
pancreatic fluids. By inhibiting the secretion of toms are exacerbated when acid is secreted
pancreatic fluids, octreotide may be able to al- without any food to act as a buffer, causing
leviate this patient’s chronic abdominal pain. pain on an empty stomach. The standard first-
line therapy is one-week triple therapy consist-
Answer A is incorrect. Cholecystokinin
ing of the antibiotics amoxicillin and clarithro-
(CCK) is a hormone synthesized by duodenal
mycin, and a proton pump inhibitor such as

Gastrointestinal
I cells. Once chyme enters the duodenum,
omeprazole.
CCK is secreted to decrease gastric acid se-
cretion and to slow the release of chyme into Answer B is incorrect. NSAIDs are the second
the duodenum. CCK stimulates the release of leading cause of both gastric and duodenal ul-
pancreatic digestive enzymes and induces gall- cers, but are more commonly associated with
bladder contraction. CCK is not indicated for gastric ulcers. Unlike duodenal ulcers, gastric
the treatment of acromegaly, and no analogs ulcers are worsened by food; patients usually
have been created. complain of resulting early satiety and weight
loss. Although NSAIDs should be avoided, this
Answer B is incorrect. Gastrin is a hormone
would not be a primary part of the treatment
released by stomach G cells in response to
plan for this patient with confirmed Helico-
stomach distention, vagal stimulation, and
bacter pylori infection.
proteins. Through the actions of gastrin on pa-
rietal cells, the end result of gastrin release is Answer C is incorrect. Patients with Zollinger-
increased acid secretion. Gastrin also causes Ellison syndrome have gastrin-secreting tu-
chief cells to release pepsinogen for break- mors of the pancreas and duodenum. They
down of proteins. Gastrin is not indicated for will have findings indicative of acid hyperse-
the treatment of acromegaly and does not have cretion, such as several refractory ulcers. It is
a role in exocrine pancreas function. associated with multiple endocrine neoplasia
type 1 (MEN-1). Although this syndrome can
Answer C is incorrect. Secretin is a hormone
result in a duodenal ulcer, the question stem
produced by duodenal S cells and secreted in
makes no mention of other tumors associated
response to increased duodenal fatty acids and
262 Section II: Organ Systems  •  Answers

with MEN-1 (pituitary adenoma, parathyroid Answer F is incorrect. Sulfadiazine plus pyri-
adenoma), and does not indicate that the pa- methamine is the primary treatment for Toxo-
tient has an elevated serum gastrin. Thus, the plasma, which causes brain abscesses in HIV
patient’s pathology is more likely due to Heli- and birth defects.
cobacter pylori infection.
33. The correct answer is A. This patient is most
Answer D is incorrect. Metoclopramide is
likely suffering from acute gastroenteritis,
used primarily as an anti-emetic and a proki-
probably of viral origin. While there are sev-
netic agent. It would not be first-line therapy
eral causes of hypoalbuminemia, the most
for a patient with a duodenal ulcer.
likely cause in this otherwise healthy man
Answer E is incorrect. Ranitidine is a is simply a dilutional effect due to the large
histamine2-receptor antagonist. It is frequently amounts of fluid he was given. This type of hy-
High-Yield Systems

used as a treatment for gastroesophageal reflux poalbuminemia is also seen in congestive heart
disease to reduce the amount of acid in the failure.
stomach. It would not target Helicobacter pylori.
Answer B is incorrect. Chronic liver disease
causes hypoalbuminemia as albumin is pro-
32. The correct answer is D. The organism caus-
duced in the liver. However, this loss of syn-
ing the patient’s symptoms is the protozoan
thetic liver function typically does not occur
Giardia lamblia. G lamblia trophozoites com-
until the final stage of liver disease, cirrhosis.
monly cause chronic diarrhea. This parasite
It is unlikely that this young and healthy indi-
is treated with metronidazole. It spreads via
vidual has severe enough liver disease to cause
oral-fecal transmission in its cyst form and then
a dysfunction in albumin synthesis.
colonizes the GI tract in its trophozoite form.
G lamblia is found primarily in the duode- Answer C is incorrect. Malabsorptive states,
num and jejunum, and causes a combination such as short bowel syndrome and celiac
of malabsorption with diarrhea through a still sprue, can lead to decreased ability to absorb
incompletely understood mechanism. The pa- protein. Over time this leads to hypoalbumin-
tient’s complaints of greasy stool and flatulence emia. This patient’s two day illness would not
are classic signs of this type of infection. Diag- cause a protein deficiency severe enough to
Gastrointestinal

nosis is made via direct examination of stool cause hypoalbuminemia.


for cysts as well as duodenal fluid sampling
Answer D is incorrect. Nephrotic syndrome
and small-bowel biopsy.
causes hypoalbuminemia due to excessive pro-
Answer A is incorrect. Chloroquine is used tein loss. This syndrome would also be clas-
to treat Plasmodium, the organism that causes sically accompanied by massive proteinuria,
malaria. peripheral and periorbital edema, and hy-
perlipidemia. However, the vignette does not
Answer B is incorrect. Dapsone is used to
describe any of these findings, and the recent
treat Pneumocystis jiroveci, a common cause of
large fluid load is the most likely culprit for the
pneumonia in HIV.
hypoalbuminemia.
Answer C is incorrect. Melarsoprol is used to
Answer E is incorrect. Poor nutritional status
treat Trypanosoma brucei gambiense and Try-
causes hypoalbuminemia due to insufficient
panosoma brucei rhodesiense, the causes of Af-
protein intake. However, this patient’s two-day
rican sleeping sickness. Specifically, it is used
history of vomiting would not cause a severe
in cases of central nervous system penetration.
enough nutritional deficit to induce hypoal-
Answer E is incorrect. Nifurtimox is the pri- buminemia, which is generally seen only after
mary treatment for Trypanosoma cruzi, the long-term protein deficiency.
cause of Chagas disease.
34. The correct answer is B. The oral absorption
of ciprofloxacin is impaired by divalent cations,
Chapter 10: Gastrointestinal  •  Answers 263

including those in common antacids such as rhagic debris with pancreatic enzymes. These
calcium carbonate, which this patient is likely cysts lack a true epithelial lining.
taking to treat GERD. Ciprofloxacin belongs
Answer A is incorrect. Cholecystitis is not a
to the family of fluroquinolones, which block
common complication of acute pancreatitis.
bacterial DNA synthesis by inhibiting bacterial
DNA gyrase. Inhibition of DNA gyrase pre- Answer B is incorrect. Chronic gastritis is not
vents the relaxation of positively supercoiled a common complication of acute pancreatitis.
DNA that is required for normal transcription Answer C is incorrect. Pancreatic carcinoma
and replication. does not arise from a bout of acute pancre-
Answer A is incorrect. Unlike the other drugs atitis, although evidence exists to suggest that
listed, aspirin is not used as a pharmacothera- chronic pancreatitis may be a risk factor for
peutic agent for treating GERD. In fact, it is pancreatic carcinoma.

High-Yield Systems
known to cause gastric ulcers. Although many Answer E is incorrect. Small bowel obstruc-
patients take aspirin for its antiplatelet effect, tion is not a common complication of acute
this patient has no reason to be using aspirin. pancreatitis.
Furthermore, no drug interactions would be
expected between aspirin and ciprofloxacin. 36. The correct answer is E. Omeprazole is a
Answer C is incorrect. Cimetidine, an H2- proton pump inhibitor (PPI) that works by
receptor antagonist, is associated with gyneco- covalently binding, and irreversibly inactivat-
mastia, thrombocytopenia, and inhibition of ing the H+/K+/ATPase on the luminal surface
the cytochrome P450 (CYP450) system. How- of the gastric parietal cell. Gastrin levels are
ever, it does not significantly affect the metab- regulated by a feedback loop. Intragastric acid-
olism or pharmacokinetics of ciprofloxacin. ity stimulates D-cells in the gastric antrum to
release somatostatin, which works in a para-
Answer D is incorrect. Misoprostol is a prosta- crine fashion, binding to G-cells in the gastric
glandin E1 analog that can be used to prevent antrum and inhibiting gastrin release. PPIs will
NSAID-induced peptic ulcers. It is also used as effectively raise the intragastric pH so that gas-
a medical abortifacient in many countries and trin levels rise two- to four-fold. Omeprazole

Gastrointestinal
is therefore strictly contraindicated in pregnant is associated with atrophic gastritis due to hy-
women. Misoprostol would not decrease the pergastrinemia. It may also be associated with
absorption nor increase the metabolism of cip- carcinoid tumors, headaches, and GI distur-
rofloxacin. bances.
Answer E is incorrect. Omeprazole is a proton Answer A is incorrect. Aluminum hydroxide is
pump inhibitor considered first-line therapy an antacid that is not absorbed and thus does
for GERD. Omeprazole decreases intragas- not cause systemic adverse effects. It is associ-
tric acidity, but does not affect the intestinal ated with constipation and hypophosphatemia.
absorption, metabolism, or excretion of cipro-
floxacin. Answer B is incorrect. Bismuth binds to ul-
cers, providing a physical protective barrier. It
35. The correct answer is D. This patient’s presen- has no known adverse effects.
tation is classic for acute pancreatitis. This pro- Answer C is incorrect. Cimetidine is an H2-
cess often occurs in young patients after con- antagonist and is associated with headache,
suming large amounts of alcohol. Other causes confusion, thrombocytopenia, and inhibition
include gallstone obstruction, medications, of the CYP450 system. Inhibition of the pro-
infection, hypertriglyceridemia, and trauma. ton pumps by PPIs such as omeprazole has a
Pseudocysts often arise after a bout of acute greater effect on the intragastric acidity that
pancreatitis and consist of necrotic, hemor- regulates the gastrin feedback loop than an-
264 Section II: Organ Systems  •  Answers

tagonizing the H2-receptors by drugs such as spread in a contiguous manner without metas-
cimetidine. tasizing to distant sites.
Answer D is incorrect. Misoprostol is a prosta- Answer B is incorrect. Ovarian cancer and
glandin E1 analog that can be used to prevent appendiceal cancers can disseminate directly
NSAID-induced peptic ulcers. It is also used as throughout the peritoneal cavity. This often
a medical abortifacient in many countries and happens well before the diagnosis of ovarian
is therefore strictly contraindicated in pregnant cancer is made. This process is known as pseu-
women. domyxoma peritonei, and is characterized by
a diffuse collection of gelatinous materials in
37. The correct answer is A. This patient suffers the abdomen, peritoneal surfaces, and pelvis.
from ulcerative colitis (UC), an inflamma- HCC does not spread in this manner.
tory bowel disease of unknown etiology that
High-Yield Systems

Answer D is incorrect. Many cancers, includ-


affects rectum and may extend proximally to
ing breast cancer, colon cancer, papillary thy-
the colon; disease is rarely found in the small
roid carcinoma, and melanoma, metastasize
intestine. UC is characterized by mucosal and
via lymphatic drainage. HCC, however, is no-
submucosal inflammation. Friable mucosal
table for hematogenous dissemination.
pseudopolyps may be evident on colonoscopy.
Patients typically present with crampy abdomi- Answer E is incorrect. There is no evidence
nal pain and bloody diarrhea. UC is associated that HCC or any other cancer metastasizes in
with the HLA B27 subtype. Diseases associated this manner.
with the HLA B27 subtype can be remem-
bered with the mnemonic PAIR, and include 39. The correct answer is A. This patient has
Psoriasis, Ankylosing spondylitis, Inflammatory Mallory-Weiss syndrome. Repeated bouts of
bowel disease, and Reiter syndrome. prolonged vomiting (such as after an alcohol
binge or in eating disorders) can cause longi-
Answer B is incorrect. DM is associated with
tudinal lacerations in the distal esophagus,
the HLA-DR3 and -DR4 subtypes.
normally at the gastroesophageal junction or
Answer C is incorrect. Graves disease and ce- in the proximal gastric mucosa, with exten-
Gastrointestinal

liac sprue are associated with the HLA-B8 sub- sion to submucosal arteries that can bleed mas-
type. sively. Left untreated, this bleeding can be fa-
tal. Mallory-Weiss syndrome generally presents
Answer D is incorrect. Multiple sclerosis, hay
with hematemesis after a bout of retching or
fever, SLE, and Goodpasture syndrome are as-
vomiting; however, new research suggests that
sociated with the HLA-DR2 subtype.
this classic history may be obtained in only
Answer E is incorrect. Rheumatoid arthritis is about 50% of patients. Bleeding from esopha-
associated with the HLA-DR4 subtype. geal varices might also be expected if the pa-
tient has chronic liver disease. However, this
38. The correct answer is C. This is a case of he- patient is relatively young and shows no other
patocellular carcinoma (HCC). As are renal signs of liver disease. Furthermore, variceal
cell carcinoma and follicular thyroid carci- bleeding is usually painless, while Mallory-
noma, HCC is spread commonly via hema- Weiss tears are more commonly associated
togenous dissemination. Accordingly, metas- with pain. Thus of the two choices, Mallory-
tases often develop in the lung, portal vein, Weiss syndrome is the better answer.
periportal nodes, brain, or bones. The patient’s
chronic HBV infection and alcohol abuse Answer B is incorrect. Metaplasia of esopha-
likely led to the development of cirrhosis and geal mucosa, called Barrett esophagus, is as-
then HCC. sociated with chronic reflux causing inflam-
mation and possibly ulceration. This can lead
Answer A is incorrect. Hodgkin lymphoma, to bleeding, but it is usually not massive as in
many benign neoplasms, and some sarcomas this patient. Barrett esophagus is of concern
Chapter 10: Gastrointestinal  •  Answers 265

primarily because of its strong association with Answer D is incorrect. Crohn disease has a
adenocarcinoma. highly variable presentation; however, skip le-
sions, fissures, and strictures are generally evi-
Answer C is incorrect. Esophageal squamous
dent on colonoscopy, endoscopy, and radiogra-
cell carcinoma might cause ulceration of the
phy.
esophageal mucosa, but massive bleeding is
uncommon. Answer E is incorrect. Celiac sprue is a dis-
ease of malabsorption characterized by bulky,
Answer D is incorrect. Esophageal stricture is
fatty stools following meals. Endoscopy reveals
typically caused by scarring from reflux, inges-
villous flattening in the small intestine.
tion of toxic or caustic substances, or sclero-
derma. It most commonly manifests as chronic
41. The correct answer is B. The patient has gall-
dysphagia, not hematemesis.
stones, as evidenced by her symptoms and ul-

High-Yield Systems
Answer E is incorrect. Esophageal varices are trasound findings of multiple gallstones. She
seen in chronic liver disease that has resulted fits the demographics of the classic patient
in portal hypertension. The increased pres- with cholesterol gallstones: Fat, Female, Fer-
sure in the portal circulation causes dilation tile, and Forty (the “4 F’s”). Gemfibrozil, a
of submucosal veins in the lower esophagus. fibrate, is contraindicated in the treatment of
Bleeding from varices can be massive and even hypertriglyceridemia in the presence of gall-
fatal, but it is generally not painful. Further- stones. Fibrates can increase the development
more, this patient is rather young and shows of gallstones, thus increasing the risk of chole-
no symptoms of liver disease. cystitis.
Answer A is incorrect. Ezetimibe, in combina-
40. The correct answer is B. Any biopsy would
tion with a statin, is an appropriate choice for
likely show normal structures. Irritable bowel
treatment of this patient.
syndrome is a functional GI disorder charac-
terized by abdominal pain and altered bowel Answer C is incorrect. Niacin would not be
habits in the absence of demonstrable organic the first line of treatment for this patient, but it
pathology. It is a diagnosis of exclusion based is not contraindicated.

Gastrointestinal
on clinical features such as the ones presented.
Answer D is incorrect. Pravastatin is an appro-
Most commonly, patients have alternating di-
priate choice for treatment of her metabolic
arrhea and constipation, chronic abdominal
profile.
pain that improves with stools, a change in
stool frequency and consistency, and onset af- Answer E is incorrect. Simvastatin is an ap-
ter emotional and/or stressful life events. These propriate choice for treatment of this patient.
symptoms occur in the absence of fevers, lower
GI bleeding, leukocytosis, and weight loss. 42. The correct answer is C. The most common
tumor of the parotid gland is the pleomorphic
Answer A is incorrect. Pseudomembranous adenoma or the mixed tumor, accounting for
colitis usually follows antibiotic therapy and 50% of salivary tumors. The pleomorphic ad-
is characterized by bloody diarrhea, fever, and enoma is a benign, well-differentiated, well-
leukocytosis. Absence of these manifestations circumscribed mass that grows slowly over the
in this patient argues against this diagnosis. course of months to years. On histopathology,
Answer C is incorrect. Primary sclerosing it is characterized by the presence of multiple
cholangitis is an extra-intestinal manifestation cell types, classically epithelial cells in a chon-
of ulcerative colitis. On colonoscopy, the mu- dromyxoid stroma.
cosa demonstrates continuous superficial ul- Answer A is incorrect. Adenoid cystic carci-
cerations with resultant inflammatory pseudo- noma (ACC) is an invasive, poorly differenti-
polyps. ated cancer characterized by gland-forming
tissue and a cystic, fluid-filled cavity. It tends
266 Section II: Organ Systems  •  Answers

to infiltrate perineurial spaces and cause pain. Answer C is incorrect. Patients with sclero-
ACC constitutes 5% of salivary tumors and is derma develop dysphagia secondary to atro-
malignant. phy of smooth muscle in the lower (not upper)
two-thirds of the esophagus and incompetence
Answer B is incorrect. Mucoepidermoid car-
of the LES.
cinoma is an invasive, poorly differentiated
cancer composed of mucosal and epidermal Answer D is incorrect. Patients with sclero-
cell types. This type constitutes 15% of salivary derma develop dysphagia secondary to atrophy
gland tumors and is malignant. The fact that of smooth (not striated) muscle in the lower
this patient’s tumor has not changed over the two-thirds (not the lower half) of the esophagus
course of one year suggests that the mass is not and incompetence of the LES.
an invasive, malignant carcinoma.
Answer E is incorrect. Patients with sclero-
High-Yield Systems

Answer D is incorrect. A sialic duct stone is an derma develop dysphagia secondary to atrophy
inorganic precipitate mechanically obstructing of smooth (not striated) muscle in the lower
the opening of the sialic duct, resulting in an two-thirds of the esophagus and incompetence
erythematous and inflamed oral mass. Sialic of the LES.
duct stones typically present with pain while
Answer F is incorrect. Patients with sclero-
eating due to the stone obstructing saliva from
derma develop dysphagia secondary to atrophy
exiting the gland through the salivary duct.
of smooth (not striated) muscle in the lower
Answer E is incorrect. Warthin tumor is (not upper) two-thirds of the esophagus and in-
a benign mass of lymphoid cells. A well- competence of the LES.
circumscribed mass of lymphoid cells in a
salivary gland is virtually pathognomonic for a 44. The correct answer is B. Painless rectal bleed-
Warthin tumor, which make up 5-10% of sali- ing in an elderly individual (especially with
vary tumors. Although this patient’s tumor is a history of constipation or poor fiber intake)
well-circumscribed (surrounded by a fibrous suggests diverticulosis, a condition in which
capsule), it is composed of tissues of mixed ori- the mucosa and submucosa herniate through
gin—not lymphocytes. the muscular layer of the GI tract (frequently
Gastrointestinal

along the sigmoid colon), forming pockets


43. The correct answer is B. The upper third of called diverticula. This patient’s lower left
the esophagus is made up of striated muscle quadrant abdominal pain indicates that he is
(allows some voluntary control). The middle now suffering from acute diverticulitis, which
third of the esophagus is made up of both stri- is inflammation of one or more diverticula.
ated and smooth muscle. The lower third of The laboratory values show marked leukocy-
the esophagus is made up of smooth muscle tosis, which is actually a common finding in
(entirely involuntary). Patients with sclero- acute diverticulitis. More notably, the patient
derma develop dysphagia (usually to solids) appears to have iron-deficient, microcytic ane-
secondary to atrophy of smooth muscle of the mia (low hemoglobin and low MCV) This
lower two-thirds of the esophagus and incom- could result from his past bleeding episodes;
petence of the lower esophageal sphincter however, it could also be a sign of chronic oc-
(LES). The wall of the esophagus becomes cult bleeding from an undiagnosed carcinoma.
thin and atrophic and can have regions of fi- After the patient is stabilized and the acute
brosis. diverticulitis has resolved, the patient should
undergo colonoscopy to rule out malignancy.
Answer A is incorrect. Patients with sclero-
Colonoscopy is contraindicated during an
derma develop dysphagia secondary to atrophy
acute episode of diverticulitis due to increased
of smooth muscle in the lower two-thirds (not
risk of bowel perforation.
the lower half) of the esophagus and incompe-
tence of the LES. Answer A is incorrect. An abdominal ultra-
sound can be quickly carried out in the emer-
Chapter 10: Gastrointestinal  •  Answers 267

gency setting. It is helpful in the diagnosis of Answer C is incorrect. Fifty percent would be
some genitourinary, gynecologic, and biliary the answer in the case of an autosomal domi-
pathologies as well as appendicitis. Its useful- nant disorder such as hereditary spherocyto-
ness in diagnosing diverticulitis is limited. It sis, familial adenomatous polyposis, and adult
has no role in screening for colon cancer. polycystic kidney disease.
Answer C is incorrect. CT of the abdomen Answer D is incorrect. If the sister were
with infusion is useful in the diagnosis of healthy, she would have a 66.6% chance of be-
acute diverticulitis. It may show the presence ing a carrier of the mutated gene.
of diverticula and signs of adjacent inflamma-
Answer E is incorrect. A 100% chance of get-
tory changes such as edema and fat stranding.
ting a disease is not common, unless one is re-
Once the patient’s symptoms resolve, there is
ferring to the likelihood a child has of inher-
no need for a follow-up CT scan. This imag-

High-Yield Systems
iting a disease with a mitochondrial pattern of
ing modality has no role in screening for colon
inheritance from a mother with the condition.
cancer.
Examples of diseases with this inheritance pat-
Answer D is incorrect. MRI of the abdomen tern include mitochondrial myopathies and
is not useful in the diagnosis or management Leber hereditary optic neuropathy.
of diverticulitis. It has no role in screening for
colon cancer. 46. The correct answer is C. The rapid onset of
vomiting and diarrhea associated with Staphy-
Answer E is incorrect. An upright abdominal
lococcus aureus food poisoning is not due to
x-ray is cheap and easily obtained. However,
the bacterium itself but rather to ingestion of
it is not used to diagnose diverticulitis nor is it
pre-formed enterotoxin. Mayonnaise and egg
used to rule out colon cancer.
products are common sources.
45. The correct answer is B. This is a presentation Answer A is incorrect. Bacillus cereus is a
of Wilson disease. The patient has parkinso- gram-positive, b-hemolytic rod that is often
nian symptoms due to the death of neurons in associated with food poisoning from reheated
the basal ganglia (particularly in the putamen rice. Much like S aureus, it can produce a

Gastrointestinal
and globus pallidus). In addition, the liver bi- pre-formed toxin that can result in food poison-
opsy shows evidence of cirrhosis (although this ing.
is a trichrome stain and not a copper stain, so
Answer B is incorrect. Gram-positive,
the histopathology findings alone in this case
catalase-positive, coagulase-positive bacteria
are not specific). Wilson disease is an autoso-
include S aureus, which is likely the source of
mal recessive disease in which there is a mu-
the pre-formed enterotoxin. However, the pre-
tation in ATP7B, a gene in chromosome 13
formed enterotoxin is responsible for this pa-
that encodes for a copper-transporting ATPase.
tient’s symptoms of food poisoning.
Copper accumulates in the liver, basal ganglia,
bones, joints, kidneys, and Descemet mem- Answer D is incorrect. S aureus can also pro-
brane in the cornea (Kayser-Fleischer rings). duce the superantigen TSST-1, which medi-
Because Wilson disease follows an autosomal ates toxic shock syndrome. This patient is not
recessive pattern of inheritance, the patient’s exhibiting symptoms of toxic shock syndrome,
sister has a 25% chance of also having the dis- which include fever, rash, and shock.
ease. Answer E is incorrect. Raw or undercooked
Answer A is incorrect. The patient’s sister seafood is a common cause of Vibrio para-
would have nearly 0% chance of getting the haemolyticus or V vulnificus infection. These
disease if this condition followed an X-linked pathogens cause disease by direct invasion of
pattern of inheritance. Examples of X-linked the intestinal mucosa. The incubation period
disorders include Duchenne muscular dystro- from ingestion to illness is about three days;
phy and Lesch-Nyhan syndrome.
268 Section II: Organ Systems  •  Answers

S aureus toxin causes illness within hours of Answer E is incorrect. Inadequate hepatic
ingestion. copper excretion is the cause of Wilson dis-
ease, an autosomal recessive disease associated
47. The correct answer is B. The patient has with asterixis, parkinsonian symptoms, chorei-
Crigler-Najjar syndrome type 1. Patients with form movements, dementia, and characteris-
this condition lack uridine diphosphate glu­ tic corneal deposits known as Kayser-Fleischer
curonyl transferase, leading to an inability to rings.
conjugate bilirubin. This leads to increased
unconjugated bilirubin, which causes jaun- 48. The correct answer is E. The clinical presen-
dice, kernicterus, and bilirubin deposition in tation (fever, rash, hypotension) is character-
the brain. Crigler-Najjar syndrome type 1 pre­ istic of toxic shock syndrome (TSS) produced
sents early in life and it is fatal. Type 2 of the by Staphylococcus aureus. The S aureus
High-Yield Systems

syndrome is less severe and responds to pheno- TSST-1 superantigen simultaneously binds
barbital. the b region of the T-cell receptor and the
major histocompatibility complex class II of
Answer A is incorrect. Elevated antimito-
antigen-presenting cells, leading to the release
chondrial antibodies are found in the serum
of interferon-γ from T helper cells type 1 and
of patients with primary biliary cirrhosis, an
IL-1, IL-6, and tumor necrosis factor-a from
intrahepatic autoimmune disorder often associ-
macrophages. Prolonged use of a single tam-
ated with scleroderma and CREST syndrome
pon is a common cause of TSS in women, but
(CREST = Calcinosis and anti-centromere
other causes in either sex include staphylococ-
antibodies, Raynaud phenomenon, Esopha-
cal infection of surgical wounds, burns, and
geal dysmotility, Sclerodactyly, and Telangiec-
catheters.
tasias). Patients present with severe obstructive
jaundice, steatorrhea, pruritus, and hypercho- Answer A is incorrect. This answer choice de-
lesterolemia. scribes one type of antibody-mediated (type II)
hypersensitivity, such as that of myasthenia gra-
Answer C is incorrect. A problem with bili-
vis.
rubin uptake describes Gilbert syndrome.
Answer B is incorrect. Binding of the macro-
Gastrointestinal

Gilbert syndrome is a benign unconjugated


hyperbilirubinemia that is caused directly by phage CD14 receptor is characteristic of en-
decreased bilirubin uptake or by mildly de- dotoxin/lipopolysaccharide of gram-negative
creased uridine diphosphate glucuronyl trans- bacteria. This process is capable of initiating a
ferase. It is asymptomatic except for jaundice clinical presentation similar to that described
during periods of psychological or physiologic here, but it is specific to gram-negative bacte-
stress and has no clinical consequences. ria, thus contradicting other information con-
tained in the stem.
Answer D is incorrect. A problem with ex-
cretion of conjugated bilirubin is the etiology Answer C is incorrect. Antigen cross-linking
of both Dubin-Johnson syndrome and Rotor of IgE on mast cells and basophils describes
syndrome. Dubin-Johnson syndrome is a con- the initiating event of anaphylaxis, a type I hy-
jugated hyperbilirubinemia that is the result persensitivity reaction that results in histamine-
of defective liver excretion. This patient has induced vasodilation. Although anaphylaxis
unconjugated hyperbilirubinemia, evident in can result in some of the signs and symptoms
the absence of bilirubin in the urine (uncon- seen in this patient, the specific molecular
jugated bilirubin is not water soluble). Gross cascade described is indicative of S aureus
examination would reveal a black liver. Rotor superantigen infection.
syndrome is a rare idiopathic form of hyperbili-
Answer D is incorrect. Protein A binding of
rubinemia that affects both sexes, with onset
IgG Fc receptors is a virulence mechanism
occurring shortly after birth or during child-
utilized by S aureus that leads to the inhibition
hood. Rotor syndrome, like Dubin-Johnson
of complement fixation and phagocytosis. Al-
syndrome, has a relatively benign course.
Chapter 10: Gastrointestinal  •  Answers 269

though protein A is seen with the organism im- metastatic lesions. There is no agreed-upon
plied in the stem, it does not initiate the signs medical treatment, but survival can be im-
and symptoms of TSS. Instead, it inhibits im- proved with bilateral oophrectomy if the me-
mune function. tastasis is limited to the ovaries. This would ac-
company treatment of the underlying primary
49. The correct answer is A. The quality and loca- tumor.
tion of this patient’s pain, combined with its al-
Answer D is incorrect. Non-small cell adeno-
leviation with medication and the radiographic
carcinoma of the lung (NSCLC) is commonly
findings, are suggestive of GERD. GERD
caused by cigarette smoking but is not associ-
increases the risk of developing esophageal
ated with GERD. A patient with NSCLC
mucosal metaplasia (squamous is replaced
might present with hemoptysis, cough, chronic
by columnar), called Barrett esophagus. In
lung infections, history of smoking, chest

High-Yield Systems
turn, patients with Barrett esophagus are at in-
wall pain, shortness of breath, and/or hoarse-
creased risk for developing adenocarcinoma of
ness. The only current treatment regimens for
the distal esophagus. Barrett esophagus is di-
NSCLC that have proven to improve survival
agnosed by endoscopy, and must be followed
are the cisplatin-based regimens, which com-
with annual endoscopy and biopsy to monitor
bine cisplatin with various other antineoplastic
for adenocarcinoma.
agents (eg, paclitaxel, docetaxel, gemcitabine,
Answer B is incorrect. Stomach cell adeno- and vinorelbine).
carcinoma is not associated with GERD. Risk
Answer E is incorrect. Squamous cell carci-
factors for gastric adenocarcinoma include
noma is not known to correlate with GERD.
Helicobacter pylori infection, nitrosamine ex-
Squamous cell carcinoma is thought to be as-
posure, excessive salt intake, and low intake
sociated with tobacco and alcohol use. It tends
of fresh fruits and vegetables. This type of car-
to involve the proximal and mid-sections of the
cinoma is predisposed by achlorhydria and
esophagus rather than the distal esophagus.
chronic gastritis. Alarming signs and symp-
Combination therapy with 5-fluorouracil and
toms of gastric adenocarcinoma include un-
cisplatin is currently the mainstay of treatment
explained weight loss, persistent vomiting,

Gastrointestinal
in esophageal squamous cell carcinoma.
dysphagia, family history of upper GI tract
cancer, left supraclavicular adenopathy, and
50. The correct answer is E. This patient has signs
hematemesis. Current treatment of gastric ad-
and symptoms characteristic of pancreatic ad-
enocarcinoma consists of different multidrug
enocarcinoma. This malignancy often presents
therapies with agents such as 5-fluoroura-
with jaundice, epigastric pain radiating to the
cil, doxorubicin, cisplatin, etoposide, and/or
back, and weight loss. The red, swollen, ten-
irinotecan.
der lower extremity indicates a possible deep
Answer C is incorrect. Krukenberg tumor is vein thrombosis. Patients with pancreatic ad-
involvement of the ovaries (typically bilateral) enocarcinoma may present with migratory
by metastatic carcinoma of the stomach, intes- thrombophlebitis, which is called Trousseau
tines, or breasts. This neoplasm bears no rela- sign. Laboratory studies show increased amy-
tion to GERD nor is it possible for this male lase, lipase, and alkaline phosphatase levels.
patient to have ovarian metastases assuming Tumor markers such as CA 19-9 and carcino-
normal reproductive development. Patients embryonic antigen (CEA) are often elevated in
with Krukenberg tumor might present with pancreatic cancer. These markers are generally
disproportionate abdominal pain, left supra- not sensitive or specific enough to be used for
clavicular adenopathy, bloody stools, iron de- diagnosis, but they do have use in monitor-
ficiency anemia, palpable pelvic masses, and/ ing the course of the disease and response to
or cystic masses on ultrasound. Current treat- therapy. CEA is also a marker for colorectal
ment includes obtaining a tissue diagnosis of cancer. Pancreatic cancers are more common
the ovarian mass to differentiate primary from in patients with a history of smoking, DM, and
270 Section II: Organ Systems  •  Answers

chronic pancreatitis. Treatment for pancreatic levels. Furthermore, the characteristic signs of
adenocarcinoma is surgical removal, yet for migratory thrombophlebitis (Trousseau sign)
most patients this is impossible, as the cancer should lead you to think of pancreatic adeno-
has already metastasized prior to its discovery. carcinoma as the most likely diagnosis.
If possible, pancreaticoduodenectomy or distal
Answer C is incorrect. The CEA level is ele-
pancreatectomy is preferred to a total pancre-
vated in both pancreatic adenocarcinoma and
atectomy in order to preserve some of the pan-
colorectal cancer. However, the a-fetoprotein
creatic function.
level is elevated in hepatocellular carcinomas
Answer A is incorrect. This patient clearly and nonseminomatous germ cell tumors of the
shows signs of pancreatic adenocarcinoma, testis. Neither of these is likely in this patient,
and neither a-fetoprotein nor b-hCG eleva- so this is not the best answer.
tions are associated with this neoplasm.
High-Yield Systems

Answer D is incorrect. The CEA level is el-


Answer B is incorrect. The CA 19-9 level is evated in both pancreatic adenocarcinoma
indeed elevated in many cases of pancreatic and colorectal cancer. However, b-hCG is a
adenocarcinoma. However, an a-fetoprotein tumor marker for gynecologic neoplasms such
level is not found in pancreatic malignancies; as gestational trophoblastic tumors, choriocar-
rather, it is elevated in hepatocellular carcino- cinomas, and hydatidiform moles. It is also el-
mas and nonseminomatous germ cell tumors evated in the setting of several types of testicu-
of the testis. While hepatocellular carcinomas lar germ cell tumors, such as seminomas and
can occasionally present with jaundice and de- choriocarcinomas. Given the clinical presenta-
rangements in liver function tests, they are un- tion, none of these is the best diagnosis for this
likely to cause elevations in amylase and lipase patient, so this is not the best answer.
Gastrointestinal
Chapter 11

Hematology-Oncology

271
272 Section II: Organ Systems  •  Questions

Q u e st i o n s

1. The image shows a section of biopsied tissue (D) Nasopharyngeal carcinoma


from a skin lesion on the forehead of a 60-year- (E) Transitional cell carcinoma of the bladder
old farmer. Which of the following is the most
likely gross description of the skin lesion? 3. A 43-year-old man comes to the physician
complaining of heartburn and black, tarry
stools. He states that he has been eating much
less than usual, although he still manages to
drink at least one beer per day, which he re-
ports having done for the past 20 years. On
physical examination the patient exhibits dif-
High-Yield Systems

fuse tenderness and guarding over the entire


epigastric area. Serum levels of which of the
following substances are likely to be elevated
in this patient?
(A) Cholecystokinin
(B) Gastrin
(C) Intrinsic factor
(D) Secretin
(E) Somatostatin
Reproduced, with permission, from USMLERx.com.
4. A 65-year-old man presents with a two-month
history of cough, severe left-sided shoulder
(A) 5- to 10-mm oval, tan-brown patches that
pain, and hoarseness. Most concerning to him,
do not darken with sunlight
however, is the droop of his left eyelid, which
(B) Raised, pearly borders surrounded by fine
Hematology-Oncology

developed over the previous few weeks. The


telangiectasias
patient worked as a technician in a nuclear
(C) Sharply defined red, scaling plaques
power plant before retiring last year and has a
(D) Tan-brown, rough lesion <1 cm in diame-
40-year history of smoking. Physical examina-
ter
tion reveals that his left pupil is smaller than
(E) Thickened, hyperpigmented skin with
his right, and the skin on the left side of his
velvet-like texture
face is extremely dry. CT of the chest reveals
a 3-cm nodule in one lung. Which type and
2. A 55-year-old man comes to his physician for
in which part of the lung is this man’s tumor
a routine health maintenance examination. He
most likely located?
has never smoked and does not drink alcohol.
Toward the end of the visit, he tells his physi- (A) Adenocarcinoma located in the apex of left
cian that he has worked in the textiles indus- lung
try for 30 years. He knows that he has been ex- (B) Adenocarcinoma located in the apex of the
posed to aniline dyes and is concerned about right lung
how this may affect his health. This patient’s (C) Squamous cell carcinoma located in the
occupational exposure increases his risk for apex of left lung
which of the following neoplasms? (D) Squamous cell carcinoma located in the
apex of the right lung
(A) Esophageal adenocarcinoma
(E) Squamous cell carcinoma located in the
(B) Hepatocellular carcinoma
hilum of the left lung
(C) Mesothelioma
Chapter 11: Hematology-Oncology  •  Questions 273

5. A 58-year-old man presents to his primary care 7. A 4-year-old girl is brought to the emergency
physician complaining of years of heartburn department with an eight-hour history of pro-
that has not resolved with over-the-counter jectile vomiting and headache. Her parents say
drugs. His physician refers him to have an that the patient was well until two months ago,
esophagoduodenostomy (EGD) performed. when they noted that she was becoming in-
The image was taken during the patient’s creasingly clumsy. Physical examination shows
EGD. Following the EGD, what medication nystagmus in all directions of gaze, as well as
should this patient start taking? truncal ataxia. Laboratory blood studies show a
WBC count of 7200/mm³, a hemoglobin level
of 12.3 g/dL, and a platelet count of 225,000/
mm³. Which of the following is most likely to
be evident on histopathologic examination of

High-Yield Systems
the lesion?
(A) Deeply staining nuclei with scant cyto-
plasm arranged in pseudorosettes
(B) Pleomorphic, anaplastic cells with foci of
necrosis in a palisading pattern
(C) Regular round cells featuring spherical nu-
clei with finely granular chromatin sur-
rounded by clear cytoplasm
(D) Stratified squamous epithelial cells embed-
ded in spongy reticular stroma with promi-
nent peripheral gliosis
Reproduced, with permission, from Greenberger NJ, et al.
(E) Whorls of meningothelial cells with oval-
Current Diagnosis & Treatment: Gastroenterology, Hepatology,
shaped nuclei with indistinct cytoplasm
& Endoscopy, New York: McGraw-Hill, 2009; Plate15.
and psammoma bodies

Hematology-Oncology
(A) Bismuth subsalicylate 8. A 17-year-old boy presents to the emergency
(B) Calcium carbonate department with severe abdominal pain. Labo-
(C) Omeprazole ratory tests show a deficit in uroporphyrinogen
(D) Ranitidine I synthetase and excess δ-aminolevulinate and
(E) Sucralfate porphobilinogen in the urine. Which of the
following symptoms would most likely also be
6. A 38-year-old woman returning to the US from present in this patient?
a trip to Japan complains of sudden chest pain (A) Chest pain
and shortness of breath. She is taking no medi- (B) Hypotension
cation other than an oral contraceptive pill. (C) Neuropsychiatric disturbances
She is tachycardic and tachypneic, and her (D) Polyphagia
right calf is swollen and tender. She is admit- (E) Stiff neck
ted for appropriate therapy. Which of the fol-
lowing laboratory tests is important in monitor-
ing her initial therapy?
(A) Bleeding time
(B) International Normalized Ratio
(C) No monitoring is needed
(D) Partial thromboplastin time
(E) Prothrombin time
274 Section II: Organ Systems  •  Questions

9. A 7-year-old child from Africa presents with 11. A 9-year-old African-American boy is brought
a neck mass and painless cervical lymphade- to the emergency department with sudden
nopathy. On nasopharyngoscopy, a mass aris- onset of chest pain and dyspnea. Initial exami-
ing in the nasopharynx is visible. The mass is nation reveals tachycardia and hypoxemia. A
biopsied. Under light microscopy, the cells are chest x-ray demonstrates bilateral patchy in-
arranged in a syncytial pattern with vesiculated filtrates. Last year, the patient was diagnosed
nuclei and prominent nucleoli. Immunohis- with cholecystitis with multiple radiopaque
tochemistry is consistent with nonkeratinizing stones. There is a history of a “blood problem”
nasopharyngeal carcinoma, undifferentiated. that runs in the family. Which of the following
The virus associated with this carcinoma is peripheral blood smear findings is most consis-
also associated with what other malignancy, as tent with his clinical condition?
shown in the image?
(A) Heinz bodies
High-Yield Systems

(B) Howell-Jolly bodies


(C) Microcytes
(D) Ringed sideroblasts
(E) Spherocytes

12. A 55-year-old man presents to his physician


because of easy bruising, splenomegaly, and
fatigue. Results of peripheral blood smear are
shown in the image. Rapid treatment of this
patient’s condition could lead to release of the
contents of the pictured cells and the develop-
ment of a serious hematologic complication.
If this complication were to develop, which of
the following would most likely be seen on a
peripheral blood smear?
Hematology-Oncology

Reproduced, with permission, from USMLERx.com.

(A) Adult T-lymphocyte leukemia


(B) Burkitt lymphoma
(C) Cervical carcinoma
(D) Hepatocellular carcinoma
(E) Kaposi sarcoma

10. A 22-year-old man presents to the emergency


department with mucosal bleeding and epi-
staxis. Laboratory tests show an increased
bleeding time and an increased partial throm-
boplastin time. His mother was anemic
throughout her life and required several blood
transfusions after a minor operation. Which of
the following is the most likely diagnosis? Courtesy of Wikipedia.

(A) Bernard-Soulier syndrome


(B) Glanzmann thrombocytopenia (A) Acanthocytes
(C) Hemophilia A (B) Burr cells
(D) Vitamin K deficiency (C) Schistocytes
(E) Von Willebrand disease
Chapter 11: Hematology-Oncology  •  Questions 275

(D) Target cells (A) Essential thrombocytosis


(E) Teardrop cells (B) Immune thrombocytopenic purpura
(C) Polycythemia vera
13. An autopsy is performed on a 65-year-old (D) Sickle cell disease
woman. On examination of the liver, the pa- (E) Thrombotic thrombocytopenic purpura
thologist finds multiple tumors of various sizes
throughout both lobes. Without direct patho- 16. Physical examination of a 60-year-old woman
logic correlation, which of the following is reveals gait instability and decreased proprio-
most likely the location of the primary tumor? ception in her lower extremities. Blood tests
show a normal hematocrit level and near-
(A) Breast
normal mean cell volume. Her physician or-
(B) Colon
ders additional tests. Which of the following
(C) Kidney

High-Yield Systems
laboratory results supports a diagnosis of cobal-
(D) Liver
amin deficiency?
(E) Lung
(A) Decreased homocysteine levels
14. A child is brought to the pediatrician because (B) Decreased level of lactate dehydrogenase
her parents are concerned about lead poison- (C) Elevated methylmalonic acid
ing since their house is known to contain lead- (D) Elevated WBC count
based paint. A complete blood count reveals (E) Negative anti-intrinsic factor antibody
anemia. Lead poisoning causes anemia be- (F) Negative result of Schilling’s test
cause it does which of the following?
17. A 28-year-old previously healthy woman who
(A) Disrupts heme synthesis by causing de-
is six months pregnant comes to the physician
creased iron absorption from the gut
complaining of excessive fatigue for the past
(B) Disrupts heme synthesis by increasing the
several months. Physical examination reveals
activity of aminolevulinate dehydratase
pallor of her mucous membranes. A periph-
(C) Disrupts heme synthesis by inhibiting fer-
eral blood smear shows small RBCs containing

Hematology-Oncology
rochelatase
a narrow rim of hemoglobin at the periphery.
(D) Disrupts hemoglobin function by binding
Which of the following laboratory values are
to hemoglobin with high affinity, prevent-
most likely to be found in this patient?
ing oxygen binding
(E) Disrupts RBC DNA synthesis, causing
megaloblastic changes in RBCs Iron-binding Mean
Choice Ferritin corpuscular
capacity
volume
15. A 59-year-old woman is admitted to the hos-
A normal
pital because of a brief episode of right-sided
hemiparesis. Medical history is significant for B

an eight-week history of bleeding from the C


gums, nosebleeds, throbbing and burning sen- D
sations in the hands and feet, and mild left up- E
per quadrant pain. A blood panel shows a he-
moglobin level of 15 g/dL, hematocrit of 45%, Reproduced, with permission, from USMLERx.com.
and platelet count of 900,000/mm³. Erythro-
cyte sedimentation rate, C-reactive protein,
and WBC count are within normal range. (A) A
Based on the patient’s symptoms and abnormal (B) B
laboratory results, the patient begins treatment (C) C
with hydroxyurea. Which of the following is (D) D
the most likely diagnosis? (E) E
276 Section II: Organ Systems  •  Questions

18. A 40-year-old woman is diagnosed with meta- 21. A 52-year-old heart transplant patient receiving
static breast cancer. Analysis of her biopsy chronic immunosuppressive therapy develops
specimen shows tumor cells that overexpress bacterial sinusitis. His list of medications in-
HER2, a member of the epidermal growth fac- cludes cyclosporine. The patient’s physician
tor receptor family. She begins chemotherapy decides to start him on antibiotic therapy but is
that will specifically target the extracellular having difficulty choosing between amoxicillin
domain of the HER2 receptor on the cancer and erythromycin to treat the infection. Com-
cells. Which of the following drugs has most pared to using each agent alone, concurrent
likely been prescribed for this patient? use of erythromycin and cyclosporine would
most likely lead to which of the following?
(A) All-trans retinoic acid
(B) Imatinib mesylate (A) Decreased risk of renal failure
(C) Methotrexate (B) Increased cyclophilin concentration
High-Yield Systems

(D) Rituximab (C) Increased cyclosporine serum concentra-


(E) Trastuzumab tions
(D) Increased risk of infection with Clostrid-
19. A 47-year-old woman from the Middle East ium difficile
presents to the clinic with fever, general mal- (E) Increased risk of QTc prolongation
aise, and weight loss. Physical examination re-
veals hepatomegaly and massive splenomegaly, 22. A 34-year-old man presents to his primary
along with edema. Laboratory tests show mod- care physician complaining of a low-grade fe-
erate anemia and a peripheral WBC count ver, drenching sweats at night, and an unin-
<4000/mm3. Macrophages containing amas- tentional 5-kg (12-lb) weight loss over the past
tigotes are seen on histologic analysis. From three months. CT of the chest reveals medi-
which of the following hosts did this woman astinal lymphadenopathy. Subsequent biopsy
most likely acquire the parasite that she now of an involved node is remarkable for the cell
harbors? shown in the image. Which of the following is
part of the multidrug regimen that would be
Hematology-Oncology

(A) Aedes mosquito


used to manage this patient’s disease?
(B) Anopheles mosquito
(C) Ixodes tick
(D) Reduviid bug
(E) Sandfly

20. A type of lymphoma is characterized by on-


set in middle age and by neoplastic cells that
resemble normal germinal center B lympho-
cytes. What characteristic chromosomal trans-
location and protein are produced by this
translocation?
(A) t(8;14), c-Myc
(B) t(9;22), Bcr-Abl
(C) t(11;22), EWS-Fli-1
(D) t(14;18), Bcl-2
(E) t(15;17), PML/RARa Reproduced, with permission, from USMLERx.com.
Chapter 11: Hematology-Oncology  •  Questions 277

(A) Cyclosporine 25. A 44-year-old woman comes to the physician


(B) Hydroxyurea because of a four-month history of fatigue,
(C) Imatinib joint pain, malaise, and morning stiffness that
(D) Isoniazid sometimes persist for more than an hour after
(E) Vinblastine waking. Laboratory tests show:

23. A 30-year-old woman comes to the physician WBC count: 15,100/mm³


because of bruising easily. She is currently tak- Hemoglobin: 8.8 g/dL
ing no medications and has no significant past Hematocrit: 26.7%
medical history. Laboratory studies are signifi- Platelet count: 267,000/mm³
cant for a platelet count of 25,000/mm3 and Mean corpuscular volume: 87 fL
the presence of high levels of antiplatelet anti- Total iron binding capacity: 200 μg/dL
Haptoglobin: 100 mg/dL

High-Yield Systems
bodies. Which of the following features is most
likely to be seen on peripheral blood smear? LDH cholesterol: 85 mg/dL

(A) Bite cells Peripheral blood smear shows a normochro-


(B) Drepanocytes mic, normocytic anemia. Which of the follow-
(C) Enlarged platelets ing is the most likely cause of her fatigue?
(D) Howell-Jolly bodies (A) Anemia of chronic disease
(E) Pappenheimer bodies (B) Autoimmune hemolytic anemia
(C) Iron deficiency anemia
24. A 2-year-old boy is brought to a clinic because (D) Megaloblastic anemia
of a large, unilateral, painless abdominal mass (E) Sideroblastic anemia
his mother noticed while bathing him. While
performing an ultrasound-guided biopsy, the 26. A 32-year-old man is diagnosed with Hodg-
technician notes that the kidney calyces are kin lymphoma and begins chemotherapy.
highly distorted by the mass. Which of the fol- Shortly after completing treatment, the patient
lowing is most likely the origin of this lesion? complains of a chronic cough and difficulty

Hematology-Oncology
(A) A mutation of the APKD1 gene on chro- catching his breath after climbing stairs. He
mosome 16 also says he thinks his skin is “getting darker.”
(B) Embryonic renal cells from the embryonic Throughout chemotherapy his complete blood
kidney cell counts revealed minimal myelosuppres-
(C) Malignant transformation of renal tubular sion. Which of the following drugs is most
cells likely responsible for these adverse effects?
(D) Malignant transformation of uroepithelial (A) Bleomycin
cells (B) Busulfan
(E) Primitive neural crest cells (C) Dactinomycin
(D) Doxorubicin
(E) Vinblastine
278 Section II: Organ Systems  •  Questions

27. A 61-year-old woman complains of a tingling 29. A 45-year-old woman arrives at the emergency
sensation in her feet that has become progres- department complaining of intense pain in
sively worse over the past several months. On her upper abdomen for the past four hours.
physical examination she appears mildly jaun- She had a similar episode many months ago,
diced and her tongue has a glazed appear- but the pain resolved within an hour without
ance. Neurologic findings include decreased treatment. Physical examination reveals scleral
vibrational sense in her lower extremities. A icterus, jaundice, and splenomegaly. Ultra-
peripheral blood smear is shown in the image. sound demonstrates radiopaque gallstones. Re-
Which of the following is the most common sults of a Coombs test are negative. A periph-
cause of the disorder seen in this patient? eral blood smear shows small RBCs, several of
which have no central pallor. Which of the fol-
lowing is the most likely cause of this patient’s
High-Yield Systems

condition?
(A) A mechanical heart valve
(B) A mutation in the gene encoding ankyrin
(C) A mutation in the gene encoding glucose-
6-phosphate dehydrogenase
(D) Circulating antibodies targeted against
RBCs
(E) Iron deficiency

30. A 19-year-old man is referred to an oncologist


after his primary care physician detects a soft
tissue mass along the distal femur. The oncolo-
gist diagnoses the patient with osteosarcoma
Reproduced, with permission, from USMLERx.com. and places him on a chemotherapeutic regi-
men that includes a dihydrofolate reductase
Hematology-Oncology

inhibitor. A week later, he returns to the clinic


(A) Bacterial overgrowth
with nausea, vomiting, and diarrhea. Which of
(B) Gastrectomy
the following is the best course of intervention?
(C) Infection with Diphyllobothrium latum
(D) Nutritional deficiency (A) Administer erythropoietin
(E) Pancreatic insufficiency (B) Administer filgrastim
(F) Pernicious anemia (C) Folic acid supplementation
(D) Iron supplementation
28. A 57-year-old woman is diagnosed with small- (E) Vitamin B12 supplementation
cell lung carcinoma. She has gained weight,
and she says that her face has recently “bal- 31. A 72-year-old man with a chronic cough
looned.” Which of the following symptoms is comes to a local clinic complaining of a sharp
this patient likely to be experiencing in addi- pain in his ribs that started this morning af-
tion to the ones mentioned? ter he had an episode of severe coughing. He
notes that he has also been feeling very fa-
(A) Cold intolerance and constipation
tigued lately and has lost 10 lb (4.5 kg) in the
(B) Increased bone density
past three months. The physician orders a
(C) Insatiable thirst and polyuria
chest x-ray, which reveals a rib fracture on the
(D) Poor wound healing and facial plethora
lateral view. Suspicious, the physician then
(E) Proximal muscle weakness and palpita-
orders a bone marrow biopsy. The results are
tions
shown in the image. Which of the following
additional pathologic findings is this patient
most likely to have?
Chapter 11: Hematology-Oncology  •  Questions 279

33. A 60-year-old woman presents with weight loss,


fatigue, and new-onset ascites. Ascitic fluid
cytology shows atypical glandular cells with
dysplastic nuclei, blood, and increased pro-
tein content. The patient has a palpable left
clavicular mass, a hard mass beneath her um-
bilicus, and diffuse abdominal and pelvic pain.
Results of biopsy of the stomach are shown in
the image. Which of the following factors most
likely contributed to the development of this
patient’s pathology?

High-Yield Systems
Reproduced, with permission, from USMLERx.com.

(A) Aplastic anemia


(B) Hypercalcemia
(C) Hypocalcemia
(D) Reed-Sternberg cells
(E) Severely elevated prostate-specific antigen
level

32. A 62-year-old man with a history of hyperlipid-


emia and multiple transient ischemic attacks
presents to the emergency department after
two hours of left-sided weakness in his upper Reproduced, with permission, from USMLERx.com.

Hematology-Oncology
extremity and face. After a CT of the head
shows no evidence of intracranial hemorrhage,
(A) Dietary nitrosamines
a fibrinolytic enzyme is administered. Two
(B) History of oral contraceptive use
hours later the man develops bleeding from his
(C) Low dietary folate
gums as well as several large subcutaneous ec-
(D) Polyvinyl chloride exposure
chymoses. Which of the following drugs could
(E) Prior tuberculosis
stop this man’s bleeding?
(A) Aminocaproic acid 34. A patient with relapsing Hodgkin disease
(B) Protamine sulfate presents with weight gain, foot ulcers, vision
(C) Recombinant factor VIII problems, elevated blood sugar, oral candi-
(D) Tissue plasminogen activator diasis, and new onset of wildly swinging mood
(E) Vitamin K changes. What is the most likely etiology of
this patient’s psychiatric symptoms?
(A) Adverse effects of bleomycin
(B) Adverse effects of prednisone
(C) Adverse effects of vincristine
(D) Normal psychiatric response to having
cancer
(E) Progression of disease
280 Section II: Organ Systems  •  Questions

35. A 41-year-old pregnant woman sees her obste- increased in this patient is also likely to be el-
trician because of new-onset vaginal bleeding. evated in which of the following malignancies?
Although she is only four months pregnant,
(A) Choriocarcinoma
her doctor notes that her uterus is the size usu-
(B) Colorectal carcinoma
ally seen at six months of gestation. Maternal
(C) Melanoma
blood works shows a b-human chorionic gon­
(D) Neuroblastoma
adotropin level >5 times the upper limit of
(E) Prostatic carcinoma
normal. If left untreated, what is a possible
(F) Yolk sac carcinoma
consequence of the patient’s condition?
(A) Choriocarcinoma 38. A 57-year-old man presents to his internist be-
(B) Coma cause of abnormal bleeding following a rou-
(C) Fetal neural tube defects tine dental cleaning. Over the past several
High-Yield Systems

(D) Ovarian cancer months he has experienced symptoms of head-


(E) Uterine bleeding and exsanguination ache, dizziness, fatigue, decreased vision, and
occasional epistaxis. He has also begun notic-
36. A 9-month-old girl is found to be anemic. Her ing “lumps” on his armpits and groin region.
mother and several great aunts have a similar The spleen tip is palpable on physical exami-
history. Results of a peripheral blood smear nation. Laboratory tests show a hemoglobin
suggest thalassemia major. What laboratory level of 11 g/dL, blood urea nitrogen of 25 mg/
values are most likely to be seen in this pa- dL, calcium of 9.1, and total protein of 15 g/
tient? dL. Immunofixation shows high levels of IgM.
Results of bone marrow biopsy are shown in
Serum
the image. Which of the following is the most
Mean corpuscular Mean
Choice Hemoglobin
(g/dL)
Reticulocyte
count (%)
hemoglobin
(pg/cell)
corpuscular iron
(µg/dL)
likely explanation for the patient’s symptoms?
volume (fL)

A 5.5 6% 18 60 100

B 7.5 3% 26 65 65
Hematology-Oncology

C 8.0 1% 27 100 75

D 9.0 2% 31 92 225

E 10.5 2% 30 91 85

Reproduced, with permission, from USMLERx.com.

(A) A
(B) B
(C) C
(D) D
(E) E
Reproduced, with permission, from USMLERx.com.
37. A 56-year-old man who is a health care worker
presents to his physician with vague abdominal
discomfort. A physical examination reveals a (A) Heavy-chain disease
tender liver, palpable to 6 cm below the costal (B) Monoclonal gammopathy of undeter-
margin and scleral icterus. His laboratory stud- mined significance
ies are significant for an aspartate aminotrans- (C) Multiple myeloma
ferase activity of 200 U/L and an alanine ami- (D) Primary amyloidosis
notransferase activity of 450 U/L. CT of the (E) Waldenström macroglobulinemia
abdomen shows a dominant solid nodule in
the liver. The tumor marker most likely to be
Chapter 11: Hematology-Oncology  •  Questions 281

39. A 29-year-old man presents to his primary 42. A 62-year-old woman presents to the clinic
care physician with a painless testicular mass. complaining of frequent bleeding while brush-
Which of the following lymph nodes are most ing her teeth and easy bruising. She reports
likely involved? she recently had pneumonia and was treated
with a broad-spectrum antibiotic. Laboratory
(A) Deep inguinal
tests show:
(B) External iliac
(C) Gluteal Prothrombin time: 18 seconds
(D) Para-aortic Partial thromboplastin time: 37 seconds
(E) Superficial inguinal Platelet count: 231,000/mm³
Hematocrit: 37%
40. A 20-year-old woman comes to her family WBC count: 4800/mm³
physician for a routine check-up. She exhibits

High-Yield Systems
The cofactor that is deficient in this patient is
mild mental retardation. Physical examina-
needed for the carboxylation of glutamate resi-
tion shows an oval-shaped, slightly pigmented
dues of which of the following?
nevus on the lower back and an angiofibroma
on the forehead that the patient reports having (A) Factors II, VII, VIII, and X
had since birth. The patient has a history of a (B) Factors VII, VIII, IX, and XII
cardiac rhabdomyoma that spontaneously re- (C) Proteins C and S and factors IX, X, XI, and
gressed in childhood. A previous MRI showed XII
multiple nodules of glial proliferation in the (D) Proteins C and S and factors XII, IX, and X
basal ganglia, ventricles, and cortex. This pa- (E) Proteins C and S, prothrombin, and fac-
tient most likely also has which of the follow- tors VII, IX, and X
ing conditions?
43. A 50-year-old man comes to the physician’s of-
(A) Bilateral hearing loss
fice with weight loss, fatigue, night sweats, easy
(B) Central scotoma
bruising, and nosebleeds. He also complains
(C) Epilepsy
of pain near his first metatarsophalangeal joint

Hematology-Oncology
(D) Retinal detachment
in his left foot. Physical examination is remark-
(E) Severe sun sensitivity
able for hepatosplenomegaly. Laboratory stud-
ies show a preponderance of WBCs (>50,000/
41. A 29-year-old woman, who is 32 weeks preg-
mm³) and a full spectrum of myeloid cells in
nant and has been in the hospital for 3 days
the peripheral blood smear. On genetic analy-
because of pyelonephritis, starts oozing blood
sis, a translocation of chromosomes 9 and 22 is
from her intravenous lines and bleeding from
found. Which of the following is the first-line
her gums. Petechiae are also noted in her skin.
treatment for this condition?
Laboratory tests show:
(A) Allopurinol
Platelet count: 98,000/mm³
(B) Busulfan
Hematocrit: 38%
(C) Hydroxyurea
WBC count: 8000/mm³
(D) Imatinib
Prothrombin time: prolonged
(E) Interferon-a
What other laboratory anomaly would also be
expected?
(A) Elevated D-dimer levels
(B) Elevated factor VII levels
(C) Elevated fibrinogen levels
(D) Elevated protein C levels
(E) Elevated protein S levels
282 Section II: Organ Systems  •  Questions

44. Several drugs are used to prevent myocardial D-dimers. A peripheral blood smear is shown
infarction in patients with acute coronary syn- in the image. Which of the following is the
drome. One class of drugs binds to the glyco- most likely underlying cause of this woman’s
protein receptor IIb/IIIa on activated platelets, acute symptoms?
thereby interfering with platelet aggregation.
This prevents renewed formation of clots that
could block the lumen of the cardiac vessels.
Which of the following is an example of this
class of drug?
(A) Abciximab
(B) Clopidogrel
(C) Leuprolide
High-Yield Systems

(D) Selegiline
(E) Ticlopidine

45. A 36-year-old white woman with a history of


uncomplicated systemic lupus erythemato-
sus presents to her physician with edema and
Reproduced, with permission, from USMLERx.com.
pain in her right foot that began two days ago.
Physical examination reveals a positive Homan
sign (pain on dorsiflexion of the foot), and an (A) Absence of glycoprotein Ib
ultrasound reveals a non-occlusive thrombus (B) Absence of protease responsible for cleav-
in the right popliteal vein. Laboratory stud- ing von Willebrand factor
ies show a platelet count of 175,000/mm³, a (C) Activation of the coagulation cascade
prothrombin time of 26 seconds, and partial (D) Deficiency of factor VIII
thromboplastin time of 89 seconds. What is (E) An invasive gram-negative rod
the most likely cause of this patient’s condi-
Hematology-Oncology

tion? 47. A 70-year-old African-American man presents


(A) Antibodies directed against factor VIII to his physician for a routine physical exami-
(B) Antibodies directed against heparin nation. The patient denies any current or past
(C) Antibodies directed against platelet glyco- medical problems, and results of physical ex-
protein IIb/IIIa amination are normal. Routine laboratory tests
(D) Antibodies directed against platelet phos- show an abnormal monoclonal serum immu-
pholipids noglobulin level. Additional studies including
(E) Antibodies directed against RBCs a bone scan, CT of the abdomen, and a 24-
hour urine collection reveal no abnormalities.
46. A 58-year-old hospitalized woman who has Which of the following is the best next step in
been complaining of dysuria develops a management?
high fever and a sudden drop in blood pres- (A) Prescribe a bisphosphonate
sure. The patient has petechiae and purpura, (B) Prescribe high-dose steroids
bleeding from her intravenous sites, and epi- (C) Prescribe vinca alkaloids
staxis. Laboratory tests show an elevated white (D) Recommend daily anticoagulation with as-
blood cell count, low platelet count, elevated pirin
creatinine, increased prothrombin time and (E) Repeat laboratory tests in six months
partial thromboplastin time, and elevated of
Chapter 11: Hematology-Oncology  •  Questions 283

48. An 8-year-old boy has a history of chronic and thology report indicates that the specimen
severe hemolytic anemia, hepatosplenomegaly, consists of compact areas of spindle cells with
and maxillary overgrowth. He has received pink cytoplasm that form whorls and palisades.
blood transfusions since early infancy but has Which of the following types of tumors would
not received a transfusion in >4 months. He- most likely result in these findings?
moglobin (Hb) electrophoresis shows marked
(A) Medulloblastoma
elevation of HbF, increased HbA2, and ab-
(B) Meningioma
sence of HbA1. Which of the following diagno-
(C) Neurofibroma
ses is most consistent with this patient’s electro-
(D) Oligodendroglioma
phoresis?
(E) Schwannoma
(A) a-Thalassemia minor
(B) b-Thalassemia major 50. A 22-year-old man is diagnosed with medul-

High-Yield Systems
(C) b-Thalassemia minor lary thyroid carcinoma, and a comprehensive
(D) Glucose 6-phosphate dehydrogenase defi- metabolic panel is significant for hypercal-
ciency cemia. He notes that he has episodes of diz-
(E) HbH disease histocompatibility complex ziness accompanied by sweating and feeling
class II lightheaded. He says he remembers his mother
(F) RBCs containing Hb Bart having her thyroid gland removed when he
was a young child but cannot remember the
49. A 69-year-old man has a tumor removed from exact reason. These findings suggest a possible
the cerebellopontine angle because a CT scan mutation in which of the following genes?
shows a 2-cm, sharply circumscribed mass ad-
(A)
braf
jacent to the right pons and extending into the
(B)
erb-B2
right cerebellar hemisphere. The patient re-
(C)
MEN1
ports a three-month history of dizziness. The
(D)
ras
tumor specimen appears as a single irregular
(E)
ret
fragment of tan-pink soft tissue that measures

Hematology-Oncology
slightly less than two cm. A microscopic pa-
284 Section II: Organ Systems  •  Answers

An s w e r s

1. The correct answer is B. The classic gross 2. The correct answer is E. Aniline dyes such as
signs of pearly borders and fine telangiectasias naphthalene increase the risk of transitional
lead one to suspect basal cell carcinoma. The cell carcinoma of the bladder. Other general
photomicrograph confirms this, as the nuclei risk factors include advanced age (typically a
are arranged in palisades, and there are islands patient’s sixth or seventh decade), tobacco use,
of tumor cells. These histologic characteristics and exposure to nitrosamines. Clinical symp-
are hallmarks of basal cell carcinoma. Also toms of bladder cancer include hematuria,
note that the lesion is on the face of a farmer, dysuria, and incontinence.
someone who presumably has extensive sun
Answer A is incorrect. Exposure to nitros­
exposure.
High-Yield Systems

amines (compounds often found in smoked


Answer A is incorrect. This description is that foods) increases one’s risk of esophageal and
of lentigo, which is characterized histologically gastric cancer.
by linear basal hyperpigmentation from mela-
Answer B is incorrect. Infection with hepati-
nocyte hyperplasia. This skin lesion most often
tis B (HBV) and hepatitis C (HCV) viruses, as
occurs in childhood and is benign.
well as exposure to carbon tetrachloride, afla-
Answer C is incorrect. Squamous cell carci- toxins, and vinyl chloride, increase one’s risk of
noma of the skin may also be seen in a patient hepatocellular carcinoma.
such as this farmer, who has had frequent sun
Answer C is incorrect. Asbestos exposure in-
exposure over many years. However, it is fre-
creases the risk of mesothelioma and lung can-
quently preceded by actinic keratosis, can oc-
cer.
cur on mucous areas (such as lips, unlike basal
cell carcinoma), and is characterized histologi- Answer D is incorrect. Infection with Epstein-
cally by keratin pearls. Keratin pearls appear as Barr virus increases the risk of nasopharyngeal
carcinoma.
Hematology-Oncology

pink, extracellular concretions on hematoxylin


and eosin staining, and these are not evident
on the image shown here. 3. The correct answer is B. The question de-
scribes a case of Zollinger-Ellison syndrome.
Answer D is incorrect. Actinic keratosis, Although gastrin-secreting tumors (called gas-
which can progress to squamous cell carci- trinomas) are just as likely to arise in the duo-
noma, is the most common precancerous der- denum and peripancreatic soft tissue as in the
matosis. The histologic description is charac- pancreas itself, the patient’s long history of
terized by cytologic atypia in the lower-most alcohol consumption makes a gastrinoma of
layers of the epidermis, and a thickened stra- pancreatic origin most likely secondary to ma-
tum corneum in which nuclei are often re- lignant pancreatic islet cell tumors. Zollinger-
tained. In contrast to basal cell carcinoma, in- Ellison syndrome arises from hypergastrin-
tercellular bridges are also present. emia. Gastrin, which is normally produced by
Answer E is incorrect. Acanthosis nigricans the G cells of the stomach antrum and duo-
involves thickened hyperpigmented zones. denum, stimulates acid secretion by parietal
These lesions can be associated with benign or cells in the stomach. Excess gastrin typically
malignant conditions elsewhere in the body, leads to excess acid production. Laboratory val-
such as endocrine disorders or adenocarci- ues commonly seen in this condition include
nomas. Histologically, there is hyperkeratosis increased basal and maximal acid output and
with prominent rete ridges and basal hyperpig- serum gastrin levels >1000 pg/mL. Multiple
mentation without melanocyte hyperplasia. peptic ulcerations are often the result. Excess
gastrin secretion also promotes hypertrophy of
the stomach mucosa. Increased stomach acid
Chapter 11: Hematology-Oncology  •  Answers 285

leads to symptoms of gastroesophageal reflux Answer D is incorrect. A tumor in the right


disease, and to ulceration and gastric bleeding, apex would not result in left-sided symptoms.
as evidenced by this patient’s melena. An ele-
Answer E is incorrect. Although squamous
vated serum gastrin level would be diagnostic
cell carcinoma of the lung usually appears
of gastrinoma.
centrally about the hilum, a tumor in this loca-
Answer A is incorrect. Cholecystokinin tion would not cause Horner syndrome.
(CCK) is produced by the I cells of the duode-
num and jejunum. CCK stimulates gallblad- 5. The correct answer is C. The lighter epithe-
der contraction and pancreatic enzyme secre- lium well above the gastroesophageal junction
tion. CCK inhibits gastric acid secretion. in the image is the characteristic appearance
of Barrett esophagus. It may also be described
Answer C is incorrect. Intrinsic factor is re-
as velvety Barrett esophagus, which is a precur-

High-Yield Systems
leased by the parietal cells of the stomach. Its
sor lesion to esophageal adenocarcinoma in
principle function is to bind vitamin B12 to
which intestinal epithelium replaces normal
promote its absorption.
squamous epithelium of the distal esophagus.
Answer D is incorrect. Secretin is produced If not already taking a proton-pump inhibitor,
normally by the S cells of the duodenum and the patient should be started on one to prevent
promotes pancreatic bicarbonate secretion. It and reverse dysplasia.
inhibits gastric acid secretion.
Answer A is incorrect. Bismuth can be used to
Answer E is incorrect. Somatostatin is pro- acutely treat reflux and is part of “triple ther-
duced by the pancreatic islets and gastrointesti- apy” to eradicate Helicobacter pylori; however,
nal (GI) mucosa. Somatostatin inhibits gastric it is not used to treat Barrett esophagus.
acid and pepsinogen secretion and also coun-
Answer B is incorrect. Calcium carbonate is
teracts cholecystokinin and secretin activity.
an over-the-counter treatment for the acute
Somatostatin also inhibits the release of insu-
relief of gastroesophageal reflux disease. It has
lin and glucagons.
no role in preventing reflux or treating Barrett

Hematology-Oncology
esophagus.
4. The correct answer is C. The vignette de-
scribes a superior sulcus tumor, otherwise Answer D is incorrect. Ranitidine is an
known as Pancoast tumor. Although Pancoast H2-blocker that can be used to treat mild re-
tumors may be lung neoplasms of any type, flux. This patient has a precancerous lesion
primary squamous cell carcinomas are the (Barrett esophagus) as a result of his long-
most commonly seen. Located in the apex of standing gastroesophageal reflux disease; thus,
the affected lung, the tumor compresses the treatment with an H2-blocker alone is inappro-
cervical sympathetic plexus as it grows, result- priate.
ing in shoulder pain, Horner syndrome (ip-
Answer E is incorrect. Sucralfate works by
silateral ptosis, miosis, and anhidrosis), and
coating the gastric mucosa. It is not indicated
sometimes neurologic deficits such as ipsilat-
to treat Barett’s esophagus.
eral hand weakness and hoarseness. Because
the patient’s symptoms are ipsilateral to the 6. The correct answer is D. This patient likely
damaged plexus, the tumor must be located in has a deep vein thrombosis (DVT) compli-
the apex of the left lung. cated by a pulmonary embolus. She is pre-
Answer A is incorrect. Pancoast tumor is a disposed to DVT by long hours of inactivity
squamous cell carcinoma. and use of oral contraception. Unfractionated
heparin is the most appropriate initial therapy.
Answer B is incorrect. Pancoast tumor is a
It works by catalyzing the activation of anti-
squamous cell carcinoma. A tumor in the right
thrombin III, decreasing the level of avail-
apex would not result in left-sided symptoms,
able thrombin, and inhibiting factor Xa. The
as described in this patient.
286 Section II: Organ Systems  •  Answers

degree of heparinization can be monitored symptoms consistent with increased intracra-


by measuring the partial thromboplastin time nial pressure. The prognosis is poor, with most
(PTT), which is a measure of the intrinsic co- mortality occurring within the first year, and
agulation pathway. a five-year survival rate of near zero. Glioblas-
toma multiforme is rarely found in children.
Answer A is incorrect. Bleeding time is a mea-
sure of platelet function. Any disorder of plate- Answer C is incorrect. This description is con-
let function or one that affects platelet func- sistent with a diagnosis of oligodendroglioma,
tion, such as von Willebrand disease, would a relatively rare, slow-growing, benign tumor
result in an increase in bleeding time. that is most often found in the frontal lobes.
General symptoms include headaches, sei-
Answer B is incorrect. The International Nor-
zures, and changes in cognition, while focal
malized Ratio (INR) is a means of standard-
lesions can present with localized weakness,
High-Yield Systems

izing prothrombin time (PT) measurements


sensory loss, or aphasia. The tumor consists of
from laboratory to laboratory. INR must be
cells that have round nuclei with clear cyto-
monitored in patients who are treated with
plasm (an appearance resembling a fried egg),
warfarin. Therapeutic INR is between 2 and 3.
and are often associated with areas of calcifica-
Answer C is incorrect. A patient who is treated tion.
with low-molecular-weight heparin (LMWH)
Answer D is incorrect. This describes a cra-
needs no laboratory monitoring. However, this
niopharyngioma, the most common supraten-
patient is unlikely to be treated with LMWH,
torial tumor in children, which is derived from
which is more expensive and more difficult to
embryological remnants of Rathke pouch. The
reverse in the case of supratherapeutic antico-
clinical presentation typically involves severe
agulation. The initial therapy of choice for in-
headaches, visual changes, and growth failure
patient anticoagulation (absent specific contra-
secondary to pituitary dysfunction. Incidence
indications) is still heparin.
peaks in children and in the fifth decade of
Answer E is incorrect. PT is a measure of the life.
Hematology-Oncology

extrinsic coagulation pathway. This value must


Answer E is incorrect. This is a description of
be monitored when warfarin is used as an anti-
a meningioma, a benign primary intracranial
coagulant.
neoplasm localized to the meninges. Clini-
7. The correct answer is A. Symptoms of in- cally, these tumors often present with seizures
creased intracranial pressure (nausea and vom- or a neurologic deficit that gradually wors-
iting), in conjunction with cerebellar signs of ens over time secondary to mass effect. Focal
nystagmus and truncal ataxia in a young child symptoms can vary by location; however, a
are highly suspicious for a diagnosis of medul- meningeally-derived neoplasm is unlikely to
loblastoma, which arises in the cerebellum. result in radiologic findings localized to the
This lesion is the most common malignant cerebellar vermis, as in this case.
brain tumor in children. Histologically, tu-
8. The correct answer is C. This patient suffers
mor cells are small, with reduced cytoplasm
from acute intermittent porphyria (AIP), an
and crescent-shaped, deeply-staining nuclei
autosomal-dominant disorder caused by a lack
(due to high mitotic activity) arranged in pseu-
of uroporphyrinogen I synthetase. The buildup
dorosettes that form sheets of anaplastic cells.
of toxic levels of δ-aminolevulinate (ALA) and
These tumors often respond to radiotherapy.
porphobilinogen lead to the associated symp-
Answer B is incorrect. This description is toms of abdominal pain (more than 90% of
characteristic of glioblastoma multiforme, the cases), neuropathy, high sympathetic tone,
most common primary adult brain tumor. It is and neuropsychiatric disturbances, including
generally localized to the cerebral cortex, and anxiety, depression, seizures, and paranoia.
typically presents with severe headache and AIP almost never presents before puberty, and
Chapter 11: Hematology-Oncology  •  Answers 287

it can be hard to diagnose because of its acute Answer C is incorrect. Epidemiologic studies
nature. Untreated, it can lead to paralysis and indicate human papilloma virus (HPV) infec-
death. tions as the most important risk factor for the
development of cervical cancer. The majority
Answer A is incorrect. The differential diag-
of anal, perianal, vulvar, and penile cancers,
nosis for chest pain is long and includes car-
as well as some oropharyngeal squamous cell
diac, pulmonary, GI, and musculoskeletal eti-
cancers, also appear to be linked to HPV in-
ologies. However, attacks of acute intermittent
fection. There are high- and low-risk strains
porphyria are not associated with chest pain.
of HPV, the major difference being that the
Answer B is incorrect. Due to the high sym- low-risk strains are maintained as extrachro-
pathetic tone caused by the pain of the crisis, mosomal DNA episomes in infected cells,
hypertension may be associated with acute in- whereas the high-risk HPV genome becomes

High-Yield Systems
termittent porphyria, but not hypotension. integrated into the host cellular DNA. This
Answer D is incorrect. Polyphagia, or greatly recombination event often leaves the viral on-
increased hunger, is one of the cardinal symp- cogenes E6 and E7 coupled to the viral pro-
toms associated with diabetes mellitus, not moter and enhancer sequences, allowing their
acute intermittent porphyria. continued expression after integration. E7
inactivates the Rb protein, whereas E6 causes
Answer E is incorrect. A stiff neck may be as- destruction of p53. This leads to resistance to
sociated with meningeal irritation and can be apoptosis and allows cells with DNA damage
found in meningitis or with musculoskeletal to survive and proliferate. Ultimately, there
problems, but it is not found in acute intermit- may be progression to malignancy. Histologi-
tent porphyria. cally, 90%-95% of invasive cervical cancers are
squamous cell carcinomas (<5% are adenocar-
9. The correct answer is B. Epstein-Barr vi- cinoma). On light microscopy, early invasive
rus is closely associated with the African and cancers appear as a tiny bud of invasive cells
AIDS cases of Burkitt lymphoma. By light penetrating through the basement membrane

Hematology-Oncology
microscopy, Burkitt lymphoma has a classic and pushing into the underlying stroma. Lym-
“starry-sky” appearance, made up of mono- phocytic collections in the stroma represent a
morphic medium-size cells with deep baso- reaction to invasion.
philic cytoplasm and numerous tingible body
macrophages. Immunohistochemistry stains Answer D is incorrect. Both HBV and HCV
demonstrate that the tumor cells are made up infections are associated with the develop-
of CD10-positive monoclonal B lymphocytes. ment of hepatocellular carcinoma (HCC).
Ninety percent of Burkitt lymphoma cases HBV causes HCC through a combination of
contain a c-myc translocation. chronic hepatic inflammation and integration
of the viral genome into the host DNA. Histo-
Answer A is incorrect. Adult T-lymphocyte pathology of HCC from HBV or HCV varies,
leukemia is most commonly associated with depending on the degree of differentiation of
human T-cell lymphotropic virus-1 (HTLV-1). the tumor, but most commonly has a trabecu-
HTLV-1 is endemic to southwest Japan, lar pattern and is closely associated to cirrhosis.
South America, central Africa, and the Ca-
ribbean, and is transmitted through blood Answer E is incorrect. Human herpesvirus
products, breast milk, and sexual secretions. 8 genome segments have been identified in
Adult T-lymphocyte leukemia arises in 1%- >90% of patients with Kaposi sarcomas, sug-
5% of HTLV-1-positive patients. In adult gesting a causative role. Kaposi sarcoma is
T-lymphocyte leukemia, the neoplastic cells caused by a proliferation of spindle cells from
have multilobated nuclei (in a clover leaf pat- a single clone of endothelial cell origin. Histo-
tern), and will often invade the dermis and logic findings include proliferation of spindle
subcutaneous tissue layers. cells, prominent slit-like vascular spaces, and
extravasated RBCs.
288 Section II: Organ Systems  •  Answers

10. The correct answer is E. This patient suffers are at risk for RBC sickling, leading to micro-
from von Willebrand disease (vWD), the most vascular occlusions and tissue ischemia. Vaso-
common inherited bleeding disorder. This dis- occlusion of the pulmonary capillaries causes
order is a result of functional problems with acute chest syndrome. In addition, other com-
vWD factor. This factor serves as the ligand for plications include aseptic necrosis of the femo-
platelet adhesion to a damaged vessel wall. It ral head, aplastic crisis, sequestration crisis in
also is the plasma carrier for factor VIII. The the spleen, autosplenectomy, and increased
disease may be acquired (via malignancy, auto- susceptibility to infection by encapsulated
immunity, or drug therapy) through antibodies bacteria. Chronic intravascular hemolysis pre-
directed against the factor, or it may be inher- disposes these patients to calcium bilirubinate
ited (typically in an autosomal dominant pat- gallstones, which are radiopaque more often
tern). The unique lab findings in vWD (plate- than cholesterol stones (10%-15% of choles-
High-Yield Systems

let dysfunction and lack of a carrier for factor terol stones and ~50% of pigment stones con-
VIII) result in increased bleeding time and an tain enough calcium to be radiopaque). On
increased PTT. the peripheral blood smear, one would detect
normocytic RBCs admixed with sickle-shaped
Answer A is incorrect. Bernard-Soulier disease
RBCs, increased reticulocytes, and Howell-
is an inherited defect in platelet adhesion due
Jolly bodies. Howell-Jolly bodies are nuclear
to decreased surface expression of glycoprotein
remnants in RBCs that are a result of splenic
Ib. Laboratory tests would reveal an increased
dysfunction.
bleeding time, normal platelet count, normal
PT, and normal PTT. Answer A is incorrect. Heinz bodies are pre-
cipitated hemoglobin, often associated with
Answer B is incorrect. Glanzmann thrombo-
glucose-6-phosphate dehydrogenase (G6PD)
cytopenia is an autosomal recessive disorder in
deficiency. Heinz bodies can cause membrane
which platelets do not have the glycoprotein
damage, which the spleen recognizes and re-
IIb/IIIa receptor, which normally binds fibrin-
moves, producing the so-called “bite cells.”
ogen. This defect results in abnormal platelet
Patients with G6PD deficiency do not present
Hematology-Oncology

aggregation. However, PTT is normal.


with acute chest syndrome.
Answer C is incorrect. Hemophilia A is an
Answer C is incorrect. Microcytic RBCs are
X-linked disorder that causes factor VIII de-
seen in iron deficiency anemia, not in sickle
ficiency. Bleeding frequently occurs in joints
cell patients. In fact, peripheral smears from
and the retroperitoneal space. Laboratory tests
sickle cell patients show normocytic RBCs
would reveal an increased PTT and normal
with increased reticulocytes.
bleeding time (because there is no inherent
platelet disorder). Answer D is incorrect. Ringed sideroblasts re-
sult from iron accumulation in the mitochon-
Answer D is incorrect. Vitamin K deficiency
dria secondary to defective heme synthesis.
impairs coagulation factors II, VII, IX, and
This is seen in patients with chronic alcohol-
X, the function of which depends on vitamin
ism, vitamin B6 deficiency, and lead poisoning.
K-mediated γ-carboxylation. Laboratory tests
Clinical symptoms depend largely on the spe-
would reveal an elevated PT, elevated PTT,
cific cause. However, ringed sideroblastic ane-
and a deficiency in factors II, VII, IX, and X.
mia does not produce vasoocclusive symptoms.
11. The correct answer is B. This patient has Answer E is incorrect. Spherocytes are RBCs
sickle cell disease, an autosomal recessive dis- that appear round and lack the central in-
order common in African-Americans. It arises dentation found normally in mature RBCs.
from a missense mutation on the b-globin Spherocytes may be acquired or congenital.
chain (valine to glutamic acid substitution). Congenital or hereditary spherocytosis is an
Under certain conditions such as dehydration, autosomal dominant disorder involving muta-
acidosis, and hypoxemia, sickle cell patients tions in the ankyrin or spectrin proteins; these
Chapter 11: Hematology-Oncology  •  Answers 289

proteins provide RBC membrane with its the RBCs trying to squeeze out of fibrotic bone
structural integrity. A portion of this unstable marrow.
membrane is removed by the spleen during
the RBCs’ life cycle, resulting in the charac- 13. The correct answer is B. The liver and lung
teristic spherical shape of this disease. These are the most common sites of metastasis (after
patients may present with splenomegaly and lymph nodes) because of the high blood flow
aplastic crisis but not vasoocclusive symptoms. through these organs. Therefore, primary tu-
mors in any of the locations listed as possible
12. The correct answer is C. This patient has answers may metastasize to the liver. However,
symptoms, physical findings, and a peripheral given that the blood vessels from the GI tract
blood smear suggestive of acute myelogenous drain into the hepatic circulation, the most
leukemia (AML). The image has a classic likely primary tumor that metastasizes to the

High-Yield Systems
“Auer rod” in the cytoplasm. Auer rods are are liver would be from a GI source such as the
fused lysosomal granules. A sudden release of colon.
Auer’s rods may cause acute disseminated in-
Answer A is incorrect. Breast tumors may also
travascular coagulation (DIC) and fatal hem-
metastasize to the liver, but with less frequency
orrhage. DIC is a disorder in which activation
than GI cancers. Breast metastases are found
of the coagulation cascade leads to the devel-
more often in brain and bone.
opment of microthrombi and the consumption
of platelets, fibrin, and coagulation factors. Answer C is incorrect. Tumors of the kidney,
Lab findings in DIC include increased PT and such as renal cell carcinoma, metastasize to
PTT, an elevation in fibrin split products, and the brain and bone, and less so to the liver.
thrombocytopenia. A peripheral blood smear Answer D is incorrect. Metastatic involve-
of a patient with DIC would reveal helmet- ment of the liver is far more common than
shaped cells and schistocytes. Schistocytes are primary neoplasia. Multiple tumors in the liver
irregularly shaped or fragmented RBC forms suggests that the primary tumor exists in an-
that are generated when RBCs attempt to other organ.

Hematology-Oncology
squeeze through the fibrin meshwork associ-
ated with small vessel thrombi. Answer E is incorrect. Lung tumors often me-
tastasize to brain and bone, and less often to
Answer A is incorrect. Acanthocytes (some- the liver.
times called spur cells) are spiny RBCs that are
associated with abetalipoproteinemia as well as 14. The correct answer is C. Lead inhibits ALA
severe liver disease. dehydratase and ferrochelatase, preventing
Answer B is incorrect. Burr cells (also known both porphobilinogen formation and the in-
as echinocytes) are abnormal RBCs with short, corporation of iron into protoporphyrin IX,
blunt projections around the periphery. These the final step in heme synthesis. Inhibition
cells can be seen in hemolytic-uremic syn- of both of these steps results in ineffective
drome, pyruvate kinase deficiency, uremia, heme synthesis and subsequent microcytic
and other disorders. (hemoglobin-poor) anemia.
Answer D is incorrect. Target cells are ab- Answer A is incorrect. Lead poisoning does
normal RBCs with a “bull’s-eye” appearance. not affect iron absorption from the gut.
They are seen in a variety of conditions includ- Answer B is incorrect. Lead inhibits ALA
ing asplenia, liver disease, and thalassemia. dehydratase, preventing porphyrin synthesis
Answer E is incorrect. Teardrop cells are ab- beyond the formation of ALA; it does not in-
normal RBCs seen in the setting of myeloid crease its actions. This causes ALA to accumu-
metaplasia with myelofibrosis. Some have pos- late in the urine.
ited that the teardrop morphology results from
290 Section II: Organ Systems  •  Answers

Answer D is incorrect. Lead does not have a (erythrocytosis), WBC production (leukocyto-
high affinity for hemoglobin. This type of pa- sis), and platelet production (thrombocytosis).
thology is seen in carbon monoxide (CO) poi- Patients can have constitutional findings such
soning. CO binds to hemoglobin with much as headache or weakness; neurologic symp-
higher affinity than oxygen, resulting in de- toms such as dizziness, tinnitus, or changes in
creased oxygen-carrying capacity. vision; or abdominal pain from massive hepa-
tosplenomegaly. The thrombocytosis associ-
Answer E is incorrect. Lead does not interrupt
ated with polycythemia vera is milder than that
RBC DNA synthesis. Folate and/or vitamin
seen in essential thrombocytosis. In addition,
B12 deficiencies disrupt DNA synthesis, specifi-
this patient has no leukocytosis or changes in
cally thiamine synthesis, resulting in megalo-
hemoglobin or hematocrit (Hct), making poly-
blastic changes in RBCs.
cythemia vera an unlikely diagnosis.
High-Yield Systems

15. The correct answer is A. Based on the pa- Answer D is incorrect. Sickle cell disease is
tient’s symptoms and blood count, the physi- caused by hemoglobin S, a mutated variant
cian suspects essential thrombocytosis and of hemoglobin that leads to RBC “sickling”
begins the patient on hydroxyurea. Essential in deoxygenated blood. The sickled RBCs oc-
thrombocytosis is a myeloproliferative disor- clude blood vessels, depriving tissue of oxygen
der characterized by platelet overproduction and leading to ischemia and vasoocclusive cri-
by megakaryocytes; its cause is unknown. Pa- ses. Patients with sickle cell disease are anemic
tients exhibit epistaxis, thrombosis, bruising, and functionally asplenic; this patient has nei-
bleeding, and mild splenomegaly. Patients also ther condition. In addition, platelet levels are
complain of erythromelalgia, which is burning normal in sickle cell disease. Hydroxyurea is
and redness of the hands and feet caused by approved for use in sickle cell disease, because
platelets obstructing blood flow in capillaries it increases expression of fetal globin chains,
and arterioles. Older patients also may experi- thereby providing a pool of functional hemo-
ence transient ischemic attacks. Hydroxyurea globin.
interferes with DNA synthesis by inhibiting ri-
Hematology-Oncology

Answer E is incorrect. Thrombotic thrombo-


bonucleotide reductase, preventing conversion
cytopenic purpura (TTP) is characterized by
of ribonucleotides to deoxyribonucleotides.
the formation of many blood clots, leading to
Hydroxyurea would thus decrease platelet pro-
a variety of symptoms. The classical findings
duction in this patient.
are hemolytic anemia, thrombocytopenia,
Answer B is incorrect. Immune thrombocyto- neurologic findings, impaired kidney func-
penic purpura (ITP) is characterized by a low tion, and fever. TTP is a result of aggregation
platelet count due to antibodies against plate- of platelets, setting off the coagulation cascade
lets. The antibodies opsonize the platelets for in blood vessels. When these microthrombi
phagocytosis by macrophages, causing plate- form, they circulate. Treatment is with plasma-
let levels to fall. Patients usually are asympto­ pheresis. This patient has none of these signs
matic, but once the platelet level falls below or symptoms; in fact, the patient has increased
20,000/mm3, they may have bleeding, mani- platelet levels.
fested as bruising, petechiae, nosebleeds, or in-
tracerebral hemorrhage. ITP usually is treated 16. The correct answer is C. In a minority of pa-
with steroids. This patient has thrombocytosis, tients with cobalamin (vitamin B12) deficiency,
not thrombocytopenia, so ITP is not a correct the Hct and mean corpuscular volume are
choice. normal. In these cases, laboratory testing for
elevated methylmalonic acid can be used to
Answer C is incorrect. Polycythemia vera is a
make the diagnosis.
chronic myeloproliferative disorder character-
ized by increases in all three hematopoietic Answer A is incorrect. In patients with cobal­
lineages, leading to excess RBC production amin deficiency, homocysteine levels typically
Chapter 11: Hematology-Oncology  •  Answers 291

are elevated. Homocysteine levels can be used binding capacity, and low mean corpuscular
to diagnose cobalamin deficiency in symptom- volume (MCV).
atic patients in whom the Hct and mean cor-
Answer A is incorrect. A blood smear show-
puscular volume are normal.
ing small RBCs is suggestive of a microcytic
Answer B is incorrect. Lactate dehydrogenase anemia with a low mean corpuscular volume
is increased in cobalamin deficiency because (MCV), not a normocytic anemia with a nor-
of failed hematopoiesis. mal MCV. Normocytic anemia can be caused
by acute hemorrhage, enzyme defects (eg,
Answer D is incorrect. Because cobalamin
glucose-6-phosphate dehydrogenase deficiency),
deficiency impairs DNA synthesis in all cell
RBC membrane defects, bone marrow disor-
lines, neutropenia with hypersegmented neu-
ders, hemoglobinopathies, autoimmune hemo-
trophils typically is seen in cobalamin-deficient
lytic anemia, and anemia of chronic disease.

High-Yield Systems
patients.
Answer B is incorrect. Iron deficiency anemia
Answer E is incorrect. A positive anti-intrinsic
is a microcytic anemia and thus is accompa-
factor antibody test suggests inhibition of the
nied by low, not high, MCV. High MCV indi-
binding between intrinsic factor and cobal­
cates a macrocytic anemia, such as folate defi-
amin. Because this decreases absorption of the
ciency or vitamin B12 deficiency. Furthermore,
vitamin, antibodies against intrinsic factor can
iron deficiency anemia is associated with a
result in cobalamin deficiency and pernicious
high, not low, iron-binding capacity.
anemia.
Answer D is incorrect. Iron deficiency ane-
Answer F is incorrect. The Schilling test lo-
mia is a microcytic anemia and thus is accom-
calizes the cause of cobalamin deficiency,
panied by low, not high, MCV. High MCV
once a diagnosis has been made. Patients sus-
indicates a macrocytic anemia, which can be
pected of cobalamin deficiency are adminis-
caused by a folate or vitamin B12 deficiency.
tered combinations of radioactive cobalamin,
antibiotics, intrinsic factor, and pancreatic ex- Answer E is incorrect. Iron deficiency ane-

Hematology-Oncology
tracts, after which radioactivity in the urine is mia is accompanied by a low, not high, ferritin
measured. A positive Schilling test implies a level.
defect in cobalamin absorption.
18. The correct answer is E. Trastuzumab is a
17. The correct answer is C. This woman most recombinant human monoclonal antibody
likely suffers from iron deficiency anemia, to human epidermal growth factor receptor 2
as supported by her fatigue and signs of mu- (HER-2). Trastuzumab is used to treat women
cous membrane pallor. Furthermore, the pe- with metastatic breast cancer that overex-
ripheral blood smear showing small RBCs is presses the HER-2 oncogene. It acts by bind-
consistent with the microcytic, hypochromic ing to the extracellular domain of the HER-2
anemia of iron deficiency. In premenopausal receptor on cancer cells, preventing receptor
women, pregnancy is a common cause of iron stimulation and inhibiting cell growth.
deficiency anemia due to increased iron de-
Answer A is incorrect. All-trans retinoic acid
mands. Other causes of iron deficiency anemia
(ATRA) is a vitamin A derivative used to in-
include blood loss, menorrhagia or GI bleed-
duce remission in patients with acute promy-
ing, hookworm, poor diet, and malabsorption.
elocytic leukemia. ATRA is thought to bind to
Other causes of microcytic anemia include
nuclear receptors and induce terminal cell di-
thalassemia, lead poisoning, and sideroblastic
vision and differentiation of the neoplastic pro-
anemia. These are less likely causes of anemia
myelocytes into mature WBCs.
than iron deficiency in a previously healthy
pregnant woman. Iron deficiency anemia is Answer B is incorrect. Imatinib mesylate is
characterized by a low ferritin level, high iron- used to treat chronic myeloid leukemia. It
works by blocking the ATP-binding site in the
292 Section II: Organ Systems  •  Answers

bcr-abl tyrosine kinase domain, thereby pre- relia burgdorferi, a spirochete bacterium) and
venting autophosphorylation, and repressing erlichiosis (Ehrlichia chaffeensis, a rickettsial
signals for cell proliferation. bacterium). Infection with Babesia species
presents with a malaria-like syndrome. On
Answer C is incorrect. Methotrexate is an
microscopic examination one observes the
antimetabolite that acts as a folic acid analog.
Maltese cross-appearing parasite but no RBC
Methotrexate is used to treat trophoblastic neo-
pigment. Quinine is used to treat babesiosis.
plasms, leukemias, and cancers of the breast,
Lyme disease classically presents with a circu-
head and neck, and lung. Given that this pa-
lar, expanding rash that takes on the appear-
tient’s cancer is known to overexpress HER-2,
ance of a bull’s eye; it can proceed to involve
trastuzumab is the best answer.
many organs, notably the central and periph-
Answer D is incorrect. Rituximab is a mono- eral nervous systems, the joints, the eyes, and
High-Yield Systems

clonal antibody that interacts with the surface the heart. Lyme disease is diagnosed by exam
protein found on non-Hodgkin lymphoma findings and exposure history, with corrobo-
cells. It is not used to treat breast cancer. ration from serological testing. Treatment in
adults is usually with doxycycline. Erlichiosis
19. The correct answer is E. This patient is in- presents with a high fever, fatigue, and myal-
fected with Leishmania donovani, which is gias, and can cause leukopenia, thryombocy-
transmitted by the sandfly. Infection presents topenia and renal insufficiency. Diagnosis is
with hepatosplenomegaly, malaise, anemia, again by exam and exposure history, with cor-
leukopenia, and weight loss. Microscopically, roboration by serological testing. Treatment is
macrophages containing amastigotes are ob- with doxycycline.
served. Sodium stibogluconate is used to treat
L donovani infection. Answer D is incorrect. The reduviid bug
spreads the protozoan Trypanosoma cruzi.
Answer A is incorrect. The Aedes mosquito Chronic infection with T cruzi causes Chagas
spreads the flaviviruses responsible for dengue disease, a condition characterized by cardio-
fever and yellow fever. Dengue fever is charac- megaly and, often, dilation of the intestinal
Hematology-Oncology

terized by headache, myalgais, arthralgias, and tract. Microscopic examination reveals flagel-
a petechial rash; laboratory tests show throm- lated trypomastigotes in the blood and nonmo-
bocytompenia and a relative leukopenia. Treat- tile amastigotes in tissue culture. Nifurtimox is
ment is supportive. Yellow fever presents with used to treat T cruzi.
fever, headache, back pain, and jaundice; the
best treatment is prevention via vaccination. 20. The correct answer is D. This answer de-
Answer B is incorrect. The Anopheles mos- scribes follicular lymphoma, the most com-
quito transmits the Plasmodium species of mon type of non-Hodgkin lymphoma in the
protozoa that are responsible for malaria. Ma- US. The characteristic chromosomal trans-
laria presents with fevers accompanied by location is t(14;18), which juxtaposes the im-
headaches, sweats, malaise, and anemia (due munoglobulin heavy-chain (IgH) locus on
to lysed RBCs), but not leukopenia. Diagno- chromosome 14 next to the BCL2 locus on
sis is made by examining the patient’s RBCs chromosome 18. This causes overproduction
on blood films. Treatment is tailored to the of the Bcl-2 protein, an anti-apoptotic fac-
geographic area of infection and the Plasmo- tor, facilitating the survival of the cancer. An
dium species involved; agents include chloro- important simplifying fact to help remember
quine, hydroxychloroquine, and atovaquone- the different chromosomal translocations is
proguanil. that those involving the immunoglobulin loci
on chromosome 14 tend to be cells that nor-
Answer C is incorrect. The Ixodes tick trans- mally produce antibodies (eg, B lymphocytes).
mits the pathogens that cause babesiosis (Ba- Thus these translocations are common in B-
besia microti, a protozoan), Lyme disease (Bor- lymphocyte lymphomas.
Chapter 11: Hematology-Oncology  •  Answers 293

Answer A is incorrect. The t(8;14) rearrange- term survival rates of 70%-80%, unique among
ment is found most commonly in Burkitt the acute leukemias. APL patients typically
lymphoma as well as in some cases of acute also present with dysfunctional coagulopathies,
lymphocytic leukemia (ALL). Translocation predisposing them to excess bleeding, a major
of the c-myc gene next to the immunoglobulin source of mortality.
heavy-chain (IgH) locus results in constitutive
overproduction of the c-myc oncogene, pro- 21. The correct answer is C. Cyclosporine is an
moting neoplastic proliferation. immunosuppressant that binds to cyclophilins.
This complex inhibits calcineurin, triggering
Answer B is incorrect. The t(9;22) transloca-
the inhibition of the production of interleukin-2
tion results in the Philadelphia chromosome,
(IL-2) and blocking the differentiation and
which is found most commonly in chronic
activation of T lymphocytes. Cyclosporine is
myelogenous leukemia (CML) as well as in

High-Yield Systems
metabolized by the cytochrome P450 system
other chronic myeloproliferative disorders and
in the liver. Erythromycin is an inhibitor of
ALL, where it confers a poor prognosis. The
P450 and causes increased concentrations of
translocation results in a Bcr-Abl fusion protein
drugs processed via the system. Other inhibi-
that functions as a constitutively active tyrosine
tors include isoniazid, sulfonamides, cimeti­
kinase to promote leukemia growth. This fu-
dine, ketoconazole, and grapefruit juice.
sion has allowed not only easier diagnosis and
monitoring of disease, but the recent develop- Answer A is incorrect. Cyclosporine is notori-
ment of imatinib mesylate (Gleevec) for the ous for causing nephrotoxicity. Erythromycin
treatment of CML. Imatinib is a competitive alone does not cause renal failure. However,
inhibitor of Bcr-Abl, platelet-derived growth by increasing the serum levels of cyclosporine,
factor, and c-kit tyrosine receptor kinases. erythromycin treatment will increase the likeli-
hood of renal failure.
Answer C is incorrect. The t(11;22) chromo-
somal translocation is associated with Ewing Answer B is incorrect. Cyclophilin concentra-
sarcoma. (Note that it does not involve the im- tion is decreased as cyclosporine is increased

Hematology-Oncology
munoglobulin locus.) It overproduces a chime- and binds cyclophilin. Because erythromycin
ric transcription factor that activates the c-myc increases cyclosporine, cyclophilin levels fall.
promoter and produces excessive amounts Answer D is incorrect. Erythromycin and
of the EWS-Fli-1 protein. Ewing sarcoma is other macrolide antibiotics are known to cause
a small, round cell tumor of the bone usually C difficile colitis. C difficile infection is not
found in the long bones of teenagers. X-ray will opportunistic but is instead caused by over-
show a lytic tumor with reactive bone deposited growth of C difficile in the colon when normal
around it in an onion-skin fashion. gut flora are killed by antibiotic treatment. Al-
Answer E is incorrect. The t(15;17) translo- though the combination of erythromycin and
cation denotes the acute pro-myelocytic leu- cyclosporine therapy increases the serum con-
kemia (APL) subtype of AML. The charac- centration of cyclosporine, it has no effect on
teristic fusion of the pro-myelocytic leukemia the concentration of erythromycin. There is
gene with the retinoic acid receptor-a(RARa) therefore no increased risk of C difficile infec-
gene blocks differentiation of immature my- tion.
eloid blasts, most likely by blocking activity of Answer E is incorrect. One rare complication
other retinoic acid receptors. Treatment with of erythromycin treatment is prolongation of
all-trans retinoic acid (termed differentiation the QTc interval and the risk for cardiac ar-
therapy) overwhelms the blockade of the other rhythmia. Although the combination of eryth-
retinoic acid receptors, restores differentiation, romycin and cyclosporine therapy increases
and can induce temporary remission. Combi- the serum concentration of cyclosporine, it
nation differentiation treatment together with has no effect on the concentration of erythro-
conventional chemotherapy can result in long- mycin. There is therefore no increased risk of
294 Section II: Organ Systems  •  Answers

QTc prolongation with this drug combination gain, GI distress, musculoskeletal pain, and
compared with the use of erythromycin alone. myelosuppression.
Answer D is incorrect. Isoniazid is one of the
22. The correct answer is E. This patient presents
drugs used to treat tuberculosis (TB), along
with the classic signs and “symptoms” (night
with rifampin, pyrazinamide, streptomycin,
sweats, fever, and weight loss) of Hodgkin
and ethambutol. Night sweats, fever, and
lymphoma. The diagnosis is confirmed by the
weight loss are common in patients with TB;
presence of a Reed-Sternberg cell, which is
however, the presence of Reed-Sternberg cells
shown in the image and is diagnostic for Hodg-
in this patient strongly suggests Hodgkin lym-
kin lymphoma. These cells are large, with
phoma. Isoniazid inhibits mycolic acid syn-
lobed nuclei that look like “owl eyes.” Vin-
thesis, thereby disrupting the Mycoplasma cell
blastine is part of the ABVD regimen (Adria-
wall. Adverse effects include hepatotoxicity,
High-Yield Systems

mycin [doxorubicin], Bleomycin, Vinblastine,


neuropathy, and potentially psychiatric symp-
and Dacarbazine) used to treat Hodgkin lym-
toms.
phoma. It inhibits microtubular formation of
the mitotic spindle, so affected cells cannot
23. The correct answer is C. This patient most
pass through metaphase. Vinblastine is used to
likely has immune thrombocytopenic purpura,
treat both Hodgkin and non-Hodgkin lympho-
an autoimmune disease characterized by a
mas as well as many solid tumors. Adverse ef-
low platelet count and easy bruising or bleed-
fects include alopecia, constipation, myelosup-
ing through skin or mucous membranes. On
pression, and, rarely, neurotoxicity.
blood smear platelets may be abnormally large,
Answer A is incorrect. Cyclosporine is an im- because of increased platelet production; there
munosuppressant used in transplant patients also tend to be more megakaryocytes in the
and patients with autoimmune disorders. It marrow.
works by inhibiting the production and release
Answer A is incorrect. Bite cells are RBCs
of IL-2, a cytokine involved in the activation
with portions removed by splenic macro-
of cytotoxic T lymphocytes. Adverse effects of
Hematology-Oncology

phages. They are often found in patients with


cyclosporine include GI upset, headache, and
hemolysis related to an enzyme deficiency.
tremor. Notably, myelosuppression is not a
common adverse effect. Answer B is incorrect. Drepanocytes, or sick-
led RBCs, are found in patients with sickle cell
Answer B is incorrect. Hydroxyurea is used
disease.
to treat sickle cell anemia, and to decrease
the burden of high WBC counts in acute Answer D is incorrect. Howell-Jolly bodies are
leukemia and CML. Hydroxyurea is an anti- retained chromosomes found in the RBCs of
metabolite, and although its exact mechanism patients who have undergone splenectomy.
is unknown, it is believed to affect the synthe- Answer E is incorrect. Pappenheimer bod-
sis (S) phase of the cell cycle. Its major adverse ies are siderosomes, or iron bodies, seen on
effect is myelosuppression. Wright stain of RBCs in patients with excess
Answer C is incorrect. Imatinib is used to iron.
treat CML and GI stromal tumors, and would
not be helpful in a patient with Hodgkin lym- 24. The correct answer is B. Wilms tumor arises
phoma. Patients with CML present with in- from neoplastic embryonic renal cells of the
creased neutrophils and metamyelocytes. metanephros. Wilms tumor is the most com-
Imatinib is a tyrosine kinase inhibitor that mon solid tumor of childhood (most com-
specifically targets the continuously active Bcr- monly occurring between the ages of two and
Abl receptor in CML, promoting apoptosis four years) and is rarely seen in adults. It com-
in these cells. Adverse effects include weight monly presents with a large palpable flank
mass and hemihypertrophy (abnormal enlarge-
Chapter 11: Hematology-Oncology  •  Answers 295

ment of one side of the body). It is associated tion into developing RBCs. ACD is associated
with the deletion of tumor suppressor gene with low total iron binding capacity as well as
WT1 on chromosome 11. Because it arises elevated ferritin levels. Other causes of nor-
from the kidney parenchyma, it distorts the mocytic anemia include acute hemorrhage,
kidney calyces as it grows enzyme defects, RBC membrane defects, bone
marrow disorders, hemoglobinopathies, and
Answer A is incorrect. Adult polycystic kid-
autoimmune hemolytic anemia.
ney disease is an autosomal dominant disorder
that presents with bilateral cystic enlargement Answer B is incorrect. Autoimmune hemo-
of the kidneys. Individuals with this disorder lytic anemia (AIHA) is a normocytic anemia
also suffer from cystic enlargement of the liver, caused by antibody-mediated RBC destruc-
berry aneurysms, and mitral valve prolapse. tion. There is no association between AIHA
and RA. Significant laboratory findings include

High-Yield Systems
Answer C is incorrect. Clear cell carcinoma
a positive direct antiglobulin test (also known
of the kidney is a malignancy derived from the
as the Coombs test), low haptoglobin level,
renal tubular cells. It is common for patients to
and elevated LDH cholesterol level. AIHA can
present with an abdominal mass, but patients
be further categorized as warm or cold depend-
with clear cell carcinoma are commonly men
ing on whether the antibody causes a stronger
around 50-70 years of age, with an increased
reaction at 37°C or 4°C. Blood smears show
incidence found in smokers. Patients with re-
numerous microspherocytes (warm) or marked
nal cell carcinoma present with a range of
RBC agglutination (cold).
symptoms, such as hematuria, a palpable mass,
polycythemia, flank pain, and fever. Answer C is incorrect. Iron deficiency anemia
is a common cause of anemia that is often due
Answer D is incorrect. Transitional cell carci-
to occult blood loss in adults and dietary defi-
noma (TCC) is a malignant tumor that arises
ciency in young children. This type of anemia
from the uroepithelial cells of the urinary tract.
has no direct association with RA or most other
TCC is the most common tumor of the uri-
chronic diseases. Blood smear should show a
nary tract, and can occur in the renal calyx, re-

Hematology-Oncology
microcytic, hypochromic anemia, instead of
nal pelvis, ureters, and bladder. Painless hema-
a normocytic anemia. Other significant labo-
turia and urinary outflow obstruction are the
ratory values include a decreased serum iron
most common presenting signs of TCC.
level, decreased serum ferritin level, decreased
Answer E is incorrect. Neuroblastoma results MCV, and increased total iron binding capac-
from primitive neural crest cells and presents ity.
as an abdominal mass in young children. The
Answer D is incorrect. Megaloblastic anemia
tumor does not arise in the kidney; instead, it
is usually due to lack of either vitamin B12 or
forms from the adrenal medulla and paraspi-
folate. Common causes include pernicious
nal sympathetic ganglia. Therefore it does not
anemia, gastrectomy, disease of the terminal il-
distort the kidney architecture.
eum, and dietary deficiency. This patient has
25. The correct answer is A. This patient’s history a normocytic picture, not a macrocytic one,
and physical exam, including prolonged morn- which makes megaloblastic anemia unlikely.
ing stiffness and symmetrical joint involve- Answer E is incorrect. Sideroblastic anemia
ment, are consistent with rheumatoid arthritis is a condition in which a defect exists in heme
(RA). RA is often associated with anemia of synthesis such that iron is deposited in a ring
chronic disease (ACD). ACD typically presents around the nucleus of the erythroblast. Blood
as a normocytic anemia as evidenced by a nor- smear should show the presence of these dis-
mal mean corpuscular volume (MCV). ACD tinctive ring sideroblasts. Furthermore, sidero-
occurs in patients with chronic inflammatory, blastic anemia is a microcytic, microchromic
infectious, malignant, or autoimmune diseases anemia, not a normocytic anemia. This type
and is caused by ineffective iron incorpora- of anemia can be found congenitally through
296 Section II: Organ Systems  •  Answers

gene defects such as a mutation in the alanine erythropoiesis. Other physical findings include
synthase gene or can be seen as a part of ac- angular stomatitis (fissuring at the corners
quired diseases such as the myelodysplastic of the mouth) and painful glossitis (a shiny,
syndromes. “beefy” tongue). Patients with cobalamin de-
ficiency also experience symmetric neuropa-
26. The correct answer is A. This patient is expe- thy, especially of the lower extremities. The
riencing bleomycin toxicity, which causes pul- peripheral blood smear demonstrates macro-
monary fibrosis, interstitial pneumonitis, and cytosis and hypersegmented neutrophils. The
skin changes, especially hyperpigmentation. most common cause of cobalamin deficiency
The pulmonary complications of bleomycin is pernicious anemia, a disorder in which di-
are dose dependent and therefore occur late etary cobalamin is not absorbed as a result of
in the treatment course when the cumulative gastric parietal cell atrophy and the subsequent
High-Yield Systems

dose becomes high. The incidence of pulmo- absence of intrinsic factor.


nary toxicity is approximately 10%. Bleomycin
Answer A is incorrect. Bacterial overgrowth in
is notable for causing minimal myelosuppres-
an intestinal blind loop can lead to competi-
sion compared to other chemotherapeutic
tion for dietary cobalamin. Provided cobala-
agents.
min intake is adequate, floral overgrowth is a
Answer B is incorrect. Busulfan, an alkylating less common cause of cobalamin deficiency.
agent, is also known for causing pulmonary fi-
Answer B is incorrect. Gastrectomy can result
brosis and hyperpigmentation. Unlike bleomy-
in insufficient intrinsic factor secretion and
cin, however, busulfan causes myelosuppres-
subsequent cobalamin deficiency. However, it
sion in almost 100% of patients.
is not the most common cause of this megalo-
Answer C is incorrect. Dactinomycin is an blastic anemia.
antineoplastic agent that intercalates in DNA,
Answer C is incorrect. Infection with the fish
inhibiting DNA and RNA synthesis. Its adverse
tapeworm Diphyllobothrium latum can also re-
effects include cardiotoxicity, nausea, vomit-
sult in competition for dietary cobalamin.
Hematology-Oncology

ing, and myelosuppression.


Answer D is incorrect. Nutritional deficiency
Answer D is incorrect. The toxic effects of
is a rare cause of cobalamin deficiency and
doxorubicin include cardiotoxicity, myelo-
is seen in patients who have adhered to strict
suppression, and alopecia. Toxic injury to the
vegan diets over several years. Bodily stores of
myocardium is dose dependent, and cardiotox-
cobalamin are large (on the order of years) and
icity results from doses >500 mg/m². Conges-
must be depleted before a deficiency can de-
tive heart failure could present with a nonpro-
velop.
ductive cough and exertional dyspnea, but this
patient is also suffering from hyperpigmenta- Answer E is incorrect. Pancreatic insuffi-
tion and minimal myelosuppression, making ciency results in decreased or absent levels of
bleomycin the best answer. pancreatic proteases, which free cobalamin
from carrier proteins and allow it to bind in-
Answer E is incorrect. The toxic effects of vin-
trinsic factor. Pancreatic insufficiency is a less
blastine include neurotoxicity (areflexia, pe-
common cause of cobalamin deficiency.
ripheral neuropathy, paralytic ileus) and bone
marrow suppression. Remember the mne-
28. The correct answer is D. Although her symp-
monic, “Vinblastine blasts bone marrow.”
toms may be somewhat nonspecific, the fact
that this patient has small-cell lung carcinoma
27. The correct answer is F. This patient has signs
(SCLC) hints that her symptoms may be part
and symptoms of cobalamin (vitamin B12) defi-
of a paraneoplastic syndrome. SCLC is noto-
ciency. Cobalamin deficiency can present with
rious for production of ACTH and ADH. In
pallor and mild jaundice because of increased
this case, excessive ACTH production has led
RBC breakdown stemming from ineffective
Chapter 11: Hematology-Oncology  •  Answers 297

to increased glucocorticoids. Weight gain and (G6PD) can cause G6PD deficiency. This is
redistribution of body fat (in contrast to the an X-linked mutation and therefore typically
cachexia typical of cancers alone) and moon occurs only in male patients. Heinz bodies
facies are classic signs of Cushing syndrome. may be found in the peripheral blood smear.
Poor wound healing (due to inhibition of col-
Answer D is incorrect. Circulating antibodies
lagen synthesis by glucocorticoids) and facial
targeted against RBCs can cause autoimmune
plethora are also part of Cushing syndrome.
hemolytic anemia. This will result in a positive
Answer A is incorrect. Cold intolerance and Coombs test.
constipation are symptoms of hypothyroidism.
Answer E is incorrect. Iron deficiency can
Answer B is incorrect. This patient is likely cause a nonhemolytic, microcytic anemia that
suffering from decreased bone density second- can result in microcytic RBCs with increased

High-Yield Systems
ary to increased glucocorticoid activity. Pa- central pallor in the peripheral blood smear,
tients with Cushing syndrome often develop not spherocytes.
osteoporosis.
30. The correct answer is C. This patient’s can-
Answer C is incorrect. Insatiable thirst and
cer was treated with methotrexate, a folic acid
polyuria are symptoms of ADH deficiency or
analog that inhibits dihydrofolate reductase,
diabetes insipidus.
halting DNA synthesis. It is specific for the
Answer E is incorrect. Proximal muscle weak- S-phase of the cell cycle, so rapidly dividing
ness and palpitations are symptoms of hyper- cancer cells are targeted. Myelosuppression
thyroidism. Although Cushing syndrome can may occur, because the production of blood
also cause proximal limb weakness from selec- cell progenitors in the bone marrow is also
tive atrophy of fast-twitch (type 2) myofibers, it inhibited. Myelosuppression can be reversed
does not have the cardiac effects of hyperthy- with leucovorin, an analog of folate, which
roidism. counters the effects of methotrexate.
Answer A is incorrect. Epoetin can be used to

Hematology-Oncology
29. The correct answer is B. This woman suffers
ameliorate bone marrow toxicity through stim-
from hereditary spherocytosis (HS), typically
ulation of erythroid proliferation. However,
caused by mutations in the genes that code
this will not correct the pancytopenia caused
for either ankyrin or spectrin; these are pro-
by methotrexate.
teins that contribute to the RBC cytoskeleton.
Instability in the RBC cytoskeleton results in Answer B is incorrect. Administration of a
the formation of spherocytes, which are small granulocyte colony stimulating factor drug
RBCs with a characteristic lack of central pal- such as filgrastim is useful in correcting neu-
lor. HS is a type of hemolytic anemia often as- tropenia post-chemotherapy, but it does not
sociated with splenomegaly, jaundice, and pig- correct pancytopenia as seen in this patient.
mented gallstones. An osmotic fragility test is
Answer D is incorrect. Iron supplementation
used to confirm the diagnosis. Coombs test is
is useful to correct for iron deficiency ane-
negative in patients with HS.
mia, which is a microcytic, hypochromic ane-
Answer A is incorrect. A mechanical heart mia, and usually results from blood loss due
valve can cause a hemolytic anemia due to to menstruation or GI bleeding. This patient
mechanical trauma to the RBCs as they flow has pancytopenia due to methotrexate therapy,
across the foreign surface. This results in schis- which is reversed with leucovorin.
tocytes in the peripheral blood smear, not
Answer E is incorrect. A vitamin B12 replace-
spherocytes. Furthermore, it would not ac-
ment is often administered via intramuscular
count for this patient’s symptoms.
injection. This vitamin reverses megaloblastic
Answer C is incorrect. A mutation in the gene anemia due to autoimmune-mediated defi-
encoding glucose 6-phosphate dehydrogenase ciency of intrinsic factor produced by the pari-
298 Section II: Organ Systems  •  Answers

etal cells of the stomach. This patient has pan- location. However, bone marrow biopsies
cytopenia due to methotrexate therapy, which should be normal.
is reversed with leucovorin.
32. The correct answer is A. This man appears to
31. The correct answer is B. Multiple myeloma be suffering from an acute stroke (cerebrovas-
(MM) must be on the differential diagnosis in cular accident or CVA). After confirming that
any elderly patient with pathologic fractures. it is not a hemorrhagic stroke with a CT scan,
MM is a bone marrow neoplasm, specifically the next step in treatment is administration of
the plasma cell. The bone marrow biopsy find- tissue plasminogen activator (tPA) if the dura-
ings are consistent with MM. Plasma cells can tion of stroke has been <3 hours. The major
be seen throughout this image, recognized by adverse effect of tPA is bleeding. Excess bleed-
their off-center nuclei and clock-face chro- ing must be reversed with aminocaproic acid,
High-Yield Systems

matin distribution. Also commonly seen on which blocks the conversion of plasminogen to
a blood smear are stacked RBCs, in what is plasmin.
known as a rouleaux formation. Because MM
Answer B is incorrect. Protamine sulfate is
is a tumor arising from bone, it causes destruc-
used to reverse the effects of heparin. It is a
tive bone lesions, which causes hypercalcemia.
positively-charged molecule that binds to neg-
Answer A is incorrect. Aplastic anemia is a atively charged heparin molecules, thereby
pancytopenia that manifests as anemia, throm- neutralizing the molecule and rendering it in-
bocytopenia, and neutropenia. Biopsies show effective. Protamine sulfate would not reverse
a hypocellular marrow with fatty infiltration. bleeding associated with tPA.
This slide (image) has too many cells to repre-
Answer C is incorrect. Recombinant fac-
sent aplastic anemia.
tor VIII is used to treat hemophilia A. These
Answer C is incorrect. Because MM is a tu- patients do not make factor VIII so it must be
mor arising from within bone, it causes de- replaced. It would not reverse bleeding associ-
structive bone lesions that result in elevated ated with tPA.
Hematology-Oncology

(not reduced) serum calcium levels.


Answer D is incorrect. tPA is an enzyme that
Answer D is incorrect. Lymphoma is a neo- normally cleaves plasminogen to plasmin.
plastic disorder of the lymphoid tissue. There Plasmin then facilitates the breakdown of fi-
are many different types of lymphoma, two brin clots. It is the cause of the bleeding in this
of the most distinctive histologic types being individual, not the antidote.
Burkitt lymphoma and Hodgkin lymphoma.
Answer E is incorrect. Vitamin K is cofactor
Burkitt lymphoma shows a “starry sky” pattern
for epoxide reductase, which is responsible for
on histology. This pattern is created by macro-
making clotting factors II, VII, IX, and X. It is
phage ingestion of tumor cells. Hodgkin lym-
used to treat warfarin overdose along with fresh
phoma is distinguished by its Reed-Sternberg
frozen plasma. Vitamin K would not reverse
cells. The Reed-Sternberg cells are binucleate
bleeding associated with tPA.
and display prominent nucleoli. There are no
Reed-Sternberg cells in this image, and the 33. The correct answer is A. This patient presents
clinical picture is more consistent with MM with metastatic gastric adenocarcinoma and
than lymphoma. peritoneal carcinomatosis. The image shows a
Answer E is incorrect. While a slight increase diffusely infiltrating signet ring cell carcinoma
in prostate-specific antigen is normal with ag- of the stomach, with large vacuoles of mucin
ing, severely elevated levels indicate prostate displacing the nuclei of cells. Physical find-
cancer. It commonly metastasizes to the axial ings may include Virchow node (left clavicu-
skeleton and can cause back pain and spinal lar node), Sister Mary Joseph node (umbilical
cord compression. It causes osteoblastic lesions nodes), Blumer’s shelf node (superior rectum),
in bone, although the rib would be an unusual and new-onset ascites. Gastric cancer is associ-
Chapter 11: Hematology-Oncology  •  Answers 299

ated with dietary nitrosamines, Japanese eth- nosed until the patient has experienced two
nicity, Helicobacter pylori infection, and perni- years of mood symptoms.
cious anemia.
Answer E is incorrect. The progression of
Answer B is incorrect. Oral contraceptives are Hodgkin disease typically does not involve pro-
associated with hepatic adenomas. found psychiatric symptoms.
Answer C is incorrect. Low dietary folate is as-
35. The correct answer is A. The patient has a
sociated with neural tube defects in pregnancy.
hydatidiform mole. Hydatidiform moles are
Answer D is incorrect. Polyvinyl chloride is as- cystic swellings of the chorionic villi. They
sociated with hepatic angiosarcomas. usually present in the fourth and fifth months
of pregnancy with vaginal bleeding. On exami-
Answer E is incorrect. Prior TB is associated
nation, the uterus is larger than expected for
with disseminated, miliary, or reactivation TB,

High-Yield Systems
gestational age, and the serum b-human cho-
with classic lesions in vertebral bodies (Pott dis-
rionic gonadotropin (b-hCG) level is much
ease) and the psoas muscle.
higher than normal. Moles can be either par-
34. The correct answer is B. This patient presents tial or complete and are caused by either fertil-
with some of the classic adverse effects of ste- ization of an egg that has lost its chromosomes
roid therapy, which is often part of treatment or fertilization of a normal egg with two sperm.
for Hodgkin disease via the MOPP cancer Partial moles may contain some fetal tissue
chemotherapy regimen: Mechlorethamine, but no viable fetus, and a complete mole con-
vincristine (Oncovin), Procarbazine, and tains no fetal tissue. Hydatidiform moles must
Prednisone. These include the physical signs be surgically removed, because the chorionic
of Cushing syndrome (weight gain, moon villi may embolize to distant sites and because
facies, thin skin, muscle weakness, and brittle hydatidiform moles may lead to choriocarci-
bones), along with cataracts, hypertension, in- noma, an aggressive neoplasm that metasta-
creased appetite, elevated blood sugar level, in- sizes early but is very responsive to chemother-
apy.

Hematology-Oncology
digestion, insomnia, nervousness, restlessness,
and immunosuppression. Prednisone is known Answer B is incorrect. Coma is a possible out-
to produce profound mood changes known as come of eclampsia, not an outcome of a hy-
glucocorticoid psychosis. datidiform mole. Preeclampsia is the triad of
Answer A is incorrect. The typical adverse ef- hypertension, proteinuria, and edema seen in
fects of bleomycin are pulmonary fibrosis, skin pregnancy. Eclampsia occurs when seizures
changes, and myelosuppression. Bleomycin is accompany the symptoms of preeclampsia.
part of the ABVD cancer chemotherapy regi- This patient has none of these symptoms.
men against Hodgkin: Adriamycin (doxorubi- Answer C is incorrect. Neural tube defects
cin), Bleomycin, Vinblastine, and Dacarba- are usually detected in utero by increased
zine. a-fetoprotein levels in amniotic fluid and ma-
Answer C is incorrect. Common adverse ef- ternal serum; b-hCG levels are normal in
fects of vincristine are areflexia and peripheral these patients.
neuritis. Vincristine is part of the MOPP can- Answer D is incorrect. Ovarian cancers are of-
cer chemotherapy regimen used against Hodg- ten accompanied by an increase in blood can-
kin disease: Mechlorethamine, vincristine cer antigen 125 levels, not b-hCG levels. This
(Oncovin), Procarbazine, and Prednisone. patient has a hydatidiform mole, not ovarian
Answer D is incorrect. Wildly swinging mood cancer. Hydatidiform moles do not predispose
is suggestive of cyclothymic disorders, which patients to ovarian cancer.
are common in patients with chronic medical Answer E is incorrect. If undetected, tubal
illness. Cyclothymic disorders cannot be diag- pregnancies may rupture, causing unilateral
300 Section II: Organ Systems  •  Answers

lower quadrant pain and uterine bleeding. Be- Yolk sac tumors are the most common malig-
cause the patient may exsanguinate, ruptured nant testicular and ovarian tumors in children.
ectopic pregnancy is a surgical emergency. Remember, tumor markers should not be used
This patient does not have the symptoms of to make the primary diagnosis, but for confir-
a tubal pregnancy. Also, because of the small mation and to monitor the response to therapy.
size of the fallopian tubes, tubal pregnancies
Answer A is incorrect. The marker for cho-
present long before four months of gestation.
riocarcinoma is b-hCG. This marker is also
elevated with hydatidiform moles and gesta-
36. The correct answer is A. In thalassemia major,
tional trophoblastic tumors. Hepatomas are
hypochromic, microcytic anemia is seen, and
not known to elevate b-hCG levels.
the reticulocyte count is elevated due to in-
creased production of RBCs by the bone mar- Answer B is incorrect. The marker for
High-Yield Systems

row. colorectal carcinoma is carcinoembryonic an-


tigen (CEA). This marker is nonspecific and
Answer B is incorrect. These indices are con-
is also produced by pancreatic, gastric, and
sistent with iron-deficiency anemia.
breast carcinomas. Most tumors of endoderm
Answer C is incorrect. A mean corpuscular origin can release CEA excluding hepatomas.
volume >100 suggests megalobastic anemia, However, secondary tumors in the liver from
which is not consistent with the patient’s thal- malignant colon cancer may also present with
assemia major observed on the peripheral increased CEA levels.
blood smear. Classically, megaloblastic anemia
Answer C is incorrect. The marker for mela-
is caused by either vitamin B12 or folate defi-
noma is S-100. This marker also is elevated
ciency.
with neuroendocrine tumors such as astrocy-
Answer D is incorrect. The mean corpuscular tomas and even carcinoid tumors. Hepatomas
volume suggests a normocytic, normochromic are not known to elevate S-100 levels.
anemia, which includes the hemolytic ane-
Answer D is incorrect. The marker for neuro-
mias. However, because the serum iron level is
Hematology-Oncology

blastoma is bombesin. This marker also is el-


increased, sideroblastic anemia should be sus-
evated with lung and gastric cancers. Hepato-
pected if these lab values are found.
mas are not known to elevate bombesin levels.
Answer E is incorrect. These RBC parameters
Answer E is incorrect. The marker for pros-
(Hct, hemoglobin) are consistent with anemia,
tatic carcinoma is prostate-specific antigen.
and the mean corpuscular volume suggests a
Hepatomas are not known to elevate prostate-
normocytic, normochromic anemia. The ane-
specific antigen levels.
mia is most likely due to chronic disease.
38. The correct answer is E. High levels of IgM
37. The correct answer is F. This vignette suggests
on immunofixation, coupled with the pre-
a hepatoma (hepatocellular carcinoma), which
dominance of plasma cells in the bone marrow
is associated with an elevated a-fetoprotein
biopsy, are consistent of Waldenström macro-
level. Hepatomas are highly associated with
globulinemia. Waldenström macroglobulin-
chronic HBV and HCV infections. Other risk
emia is a B-lymphocyte lymphoproliferative
factors include Wilson disease, hemochroma-
disorder that often results in a syndrome of
tosis, alcoholic cirrhosis, a1-antitrypsin defi-
blood hyperviscosity secondary to increased
ciency, and exposure to toxins and carcinogens
levels of IgM (both heavy and light chains),
such as aflatoxin. a-Fetoprotein is a marker for
which are secreted by neoplastic plasma cells.
hepatomas, but levels can also be elevated in
It is most commonly seen in adults who have
patients with germ cell tumors, such as yolk
lymphoplasmacytic lymphoma. Bone mar-
sac tumors. Yolk sac tumors (also known as en-
row biopsy reveals diffuse infiltrates of lym-
dodermal sinus tumors) arise from the germ
phocytes, plasma cells, and intermediate
cells that eventually become the adult gonads.
Chapter 11: Hematology-Oncology  •  Answers 301

plasmacytoid lymphocytes; there is often re- with hypercalcemia due to osteoclast activa-
active hyperplasia of mast cells. Patients with tion. The patient in this vignette, on the other
Waldenström macroglobulinemia often pre­ hand, has normal calcium levels. High levels
sent with similar abnormalities to those listed of monoclonal Ig chain, usually IgG or IgA,
(mild anemia, blood urea nitrogen on the up- are secreted into the serum. Ig light chains,
per limit of normal, and increased total pro- known as Bence-Jones proteins, are often se-
tein). They experience a variety of signs and creted in the urine, leading to renal failure. In
symptoms such as constitutional symptoms, this patient, immunofixation shows high levels
episodic bleeding, lymphadenopathy, and hep- of IgM, which is consistent with Waldenström
atosplenomegaly. Visual changes, dementia, macroglobulinemia.
ataxia, and vertigo are symptoms of hypervis-
Answer D is incorrect. Primary amyloido-
cosity.
sis results from a monoclonal proliferation of

High-Yield Systems
Answer A is incorrect. Heavy-chain disease plasma cells that secrete abnormal immuno-
encompasses a number of disorders including globulins containing light chains (usually λ)
chronic lymphocytic leukemia/small lympho- that deposit in various tissues. Bone marrow bi-
cytic lymphoma, lymphoplasmacytic lym- opsy may show normal bone marrow or a mod-
phoma, and Mediterranean lymphoma. The est increase in the number of plasma cells with
common feature of these disorders is that they pale pink amorphous deposits and increased
secrete free heavy-chain fragments. Bone mar- histiocytes. Diagnosis is made by positive
row biopsy reflects the underlying disorder. Al- Congo red staining and apple green birefrin-
though the image depicts lymphoplasmacytic gence by polarizing microscopy.
lymphoma, the findings of increased IgM,
rather than only increased free heavy chains, 39. The correct answer is D. A painless testicular
makes this diagnosis less likely. mass is the classic presentation of testicular
cancer. Young and middle-aged men are most
Answer B is incorrect. Monoclonal gammopa-
commonly affected. In development, the tes-
thy of undetermined significance (MGUS) is
tes begin high in the abdomen and descend

Hematology-Oncology
common in elderly asymptomatic patients with
to their final resting place in the scrotum. The
high M components in the blood. One per-
lymphatic drainage from the testes, therefore,
cent of cases progress to a symptomatic mono-
is to the para-aortic lymph nodes in the lumbar
clonal gammopathy (eg, multiple myeloma).
region just inferior to the renal arteries.
Bone marrow biopsy is normal with <10% ab-
normal plasma cells. MGUS is distinguished Answer A is incorrect. The deep inguinal
from myeloma by the lack of lytic lesions and nodes drain the vessels in the spongy urethra
little to no plasmacytosis. and may become enlarged in some sexually
transmitted diseases or other causes of urethri-
Answer C is incorrect. Multiple myeloma is
tis, or in anal cancer.
also a monoclonal plasma cell neoplasm char-
acterized by lytic bone lesions at multiple sites. Answer B is incorrect. External iliac nodes
It can also spread to lymph nodes and other ex- drain the bladder.
tranodal sites. Bone marrow biopsy reveals the Answer C is incorrect. Gluteal lymph nodes
replacement of normal marrow cells with ab- drain the deep tissue of the buttocks.
normal plasma cells featuring multiple nuclei,
prominent nucleoli, and cytoplasmic droplets Answer E is incorrect. Tumors of the scrotum
containing Ig molecules. Although multiple itself, but not of the testes, may spread to the
myeloma can also demonstrate similar bone superficial inguinal lymph nodes. The scro-
marrow biopsy findings as Waldenström mac- tum is an outpouching of abdominal skin, and
roglobulinemia, multiple myeloma is not as- drainage of this skin is to the superficial ingui-
sociated with hyperviscosity syndrome. Addi- nal nodes. The testes, however, which lie in-
tionally, multiple myeloma typically presents side the scrotum, begin life in the abdomen,
302 Section II: Organ Systems  •  Answers

and lymph drainage follows embryologic ori- clear cell renal carcinoma. The disease is due
gins. to deletion of the VHL tumor suppressor gene
on the short arm of chromosome 3. Untreated
40. The correct answer is C. This patient has tu- retinal hemangiomas can rupture, leading to
berous sclerosis, an autosomal dominant dis- retinal detachment.
order affecting tuberin and hamartin proteins,
Answer E is incorrect. Sun sensitivity would
which regulate cellular growth and differen-
be associated with xeroderma pigmentosum,
tiation. Two gene loci for tuberous sclerosis
an autosomal recessive disease caused by a
have been identified on chromosomes 9 and
deficiency in DNA repair of thymine dimers.
16. Tuberous sclerosis has a strong association
Patients are very sensitive to UV radiation-
with tumors such as cortical tubers, renal angi-
induced thymine dimers, which can cause
omyolipomas, cardiac rhabdomyomas, astrocy-
keratoses, skin cancers, and premature aging.
High-Yield Systems

tomas, and pulmonary lymphangioleiomyoma-


These patients have an increased tendency to
tosis. Typical skin findings include the ash-leaf
develop basal cell and squamous cell carcino-
spot, Shagreen patch, and facial angiofibro-
mas of the skin. Lesions are often seen around
mas. Patients with tuberous sclerosis typically
the eyes and eyelids.
present with mental retardation and epilepsy.
Answer A is incorrect. Neurofibromatosis is an 41. The correct answer is A. DIC can occur in
autosomal dominant disorder caused by muta- the setting of obstetric complications, sepsis,
tions in either the NF1 gene on chromosome malignancy, and other conditions. It is de-
17 (neurofibromatosis type 1) or the NF2 gene scribed as a thrombohemorrhagic process be-
on chromosome 22 (neurofibromatosis type cause there are microthrombi throughout the
2). Patients with neurofibromatosis type 1 have body, and coagulation factors and platelets are
characteristic café-au-lait spots, cutaneous consumed actively. The active conversion of fi-
neurofibromas, and Lisch nodules in the iris, brinogen to fibrin as part of the convergence of
along with central nervous system tumors such both clotting cascades leads to decreased levels
as optic gliomas and astrocytomas. Patients of fibrinogen. At the same time, anticoagula-
Hematology-Oncology

with neurofibromatosis type 2 have bilateral tion factors such as plasmin and protein C are
acoustic neuromas as a characteristic finding, being activated, leading to fibrinolysis and in-
and may present with bilateral tinnitus and creased levels of D-dimers in the circulation.
hearing loss.
Answer B is incorrect. DIC leads to consump-
Answer B is incorrect. Leber hereditary op- tion of coagulation factors; therefore, a drop in
tic neuropathy is due to a mitochondrial mu- factor VII levels would be expected.
tation that causes degeneration of the optic
Answer C is incorrect. Fibrinogen is actively
nerve with rapid loss of central vision, leading
converted to fibrin in the setting of DIC; there-
to a central scotoma that is permanent. More
fore, a decrease in the levels of fibrinogen
males than females are affected, with symp-
would be expected.
toms usually starting in the third decade.
Answer D is incorrect. Because the antico-
Answer D is incorrect. Von Hippel-Lindau
agulation factors are also being activated and
syndrome is an autosomal dominant disor-
consumed during DIC, protein C levels would
der characterized by abnormal blood ves-
decrease.
sel growth. The overgrowth of blood vessels
leads to angiomas and hemangioblastomas in Answer E is incorrect. The patient has a pro-
the retina, brain, and spinal cord as well as in longed PT, likely indicating a deficiency in
other regions of the body. Patients also show one of the factors involved with the extrinsic
cystic growths in the kidneys and pancreas, pathway. Vitamin K is a fat-soluble vitamin
pheochromocytomas (resulting in apparently that is a cofactor for the γ-carboxylation of glu-
essential hypertension), islet cell tumors, and tamate residues of prothrombin; factors VII,
Chapter 11: Hematology-Oncology  •  Answers 303

IX, and X; and proteins C and S. Vitamin K of gout due to release of nucleic acids into the
deficiency is uncommon; however, it can oc- plasma from dying WBCs. It does not treat the
cur in the setting of oral broad-spectrum anti- underlying CML.
biotics, which suppress the flora of the bowel
Answer B is incorrect. Busulfan used to be a
and interfere with the absorption and synthesis
treatment in patients intolerant of hydroxy-
of this vitamin. It can also be associated with
urea. Like the others, it has now been sup-
other conditions related to fat malabsorption
planted by imatinib.
and diffuse liver disease, or in the neonatal
period when the intestinal flora have not de- Answer C is incorrect. Hydroxyurea is mod-
veloped and the liver reserves of vitamin K are erately effective in bringing the disease un-
small. Vitamin K deficiency usually presents der control and maintaining a normal white
with bleeding diathesis, hematuria, melena, count, but its use has been superseded by ima-

High-Yield Systems
bleeding gums, and ecchymoses. tinib.
Answer E is incorrect. Interferon-a was once
42. The correct answer is E. The most common
the treatment of choice for this condition but
tumor of the parotid gland is the pleomorphic
has been superseded by imatinib.
adenoma or the mixed tumor, accounting for
50% of salivary tumors. The pleomorphic ad- 44. The correct answer is A. Abciximab functions
enoma is a benign, well-differentiated, well- by binding to the glycoprotein receptor IIb/
circumscribed mass that grows slowly over the IIIa on activated platelets, preventing fibrino-
course of months to years. On histopathology, gen from binding and interfering with platelet
it is characterized by the presence of multiple aggregation. It is used in acute coronary syn-
cell types, classically epithelial cells in a chon- drome and angioplasty.
dromyxoid stroma.
Answer B is incorrect. Both clopidogrel and
Answer A is incorrect. The activity of factor ticlopidine function by inhibiting the ADP
VIII does not depend on vitamin K. pathway involved in the binding of fibrinogen

Hematology-Oncology
Answer B is incorrect. The activities of factors to platelets during platelet aggregation.
VIII and XII do not depend on vitamin K. Answer C is incorrect. Leuprolide is a gona­
Answer C is incorrect. The activities of factors dotropin-releasing hormone analog that acts
IX and XII do not depend on vitamin K. as an agonist when administered in a pulsatile
fashion and as an antagonist when adminis-
Answer D is incorrect. The activity of factor
tered in a continuous fashion. It is used to treat
XII does not depend on vitamin K.
infertility (when administered as an agonist),
prostate cancer (when administered as an an-
43. The correct answer is D. This patient is suffer-
tagonist), and uterine fibroids.
ing from CML caused by the t(9;22) chromo-
somal translocation, creating the Philadelphia Answer D is incorrect. Selegiline is a selective
chromosome. This translocation generates a monoamine oxidase B inhibitor that causes an
fusion protein, Bcr-Abl, that functions as a con- increase in the availability of dopamine. It is
stitutively active tyrosine kinase, promoting used with levodopa in the treatment of Parkin-
dysregulated cell growth and division. Imatinib son disease.
(Gleevec or STI571) is a specific inhibitor of
Answer E is incorrect. Both clopidogrel and
tyrosine kinase. It acts by binding to the ATP-
ticlopidine function by inhibiting the ADP
binding pocket of tyrosine kinase, thus inhibit-
pathway involved in the binding of fibrinogen
ing its ability to phosphorylate.
to platelets during platelet aggregation.
Answer A is incorrect. Allopurinol is often
used with other chemotherapeutic agents in 45. The correct answer is D. This patient has de-
the initial treatment of CML to prevent attacks veloped a lupus anticoagulant, which is an-
304 Section II: Organ Systems  •  Answers

other name for acquired antiphospholipid an- Answer A is incorrect. This abnormality un-
tibody syndrome. Platelet phospholipids are derlies Bernard-Soulier disease, an inherited
required for both the intrinsic and extrinsic disorder in platelet adhesion due to the ab-
clotting pathways. Antiphospholipid antibodies sence of the glycoprotein Ib receptor. Periph-
bind to platelet phospholipids, thereby making eral blood smear may show increased platelet
them accessible to clotting factors and leading size (macrothrombocythemia) but no schis-
to recurrent venous and arterial thrombosis. tocytes. Laboratory findings include a nor-
Because both the PT and PTT assays use ex- mal PT, normal PTT, and decreased platelet
ogenous phospholipids, the antibodies inhibit count.
their function and paradoxically show an in-
Answer B is incorrect. This abnormality un-
crease in coagulation time.
derlies thrombotic thrombocytopenic purpura,
Answer A is incorrect. Factor VIII antibodies which is characterized by the classic pentad
High-Yield Systems

are a rare cause of acquired hemophilia in the of fever, thrombocytopenia, microangiopathic


elderly. Their presence would cause bleeding hemolysis, neurologic symptoms, and renal in-
rather than thrombus formation. They would sufficiency. Schistocytes may be present on the
also elevate the PTT but not the PT. peripheral blood smear. However, laboratory
findings include a decreased platelet count
Answer B is incorrect. Heparin-induced
with normal PT and PTT.
thrombocytopenia is a hypercoagulable state
caused by an immune reaction to exogenous Answer D is incorrect. Hemophilia A is an
heparin. This patient has no history of heparin X-linked disorder leading to deficiency of fac-
exposure. tor VIII that may manifest as excessive or spon-
taneous bleeding. Laboratory findings include
Answer C is incorrect. Antibodies directed
a normal PT, elevated PTT, and normal plate-
against platelet glycoproteins, especially IIb/
let count. Schistocytes would not be present
IIIa, lead to idiopathic thrombocytopenic pur-
on the peripheral blood smear.
pura. This is characterized by extremely low
platelet levels but normal PT and PTTs. Answer E is incorrect. This answer describes
Hematology-Oncology

the abnormality underlying von Willebrand


Answer E is incorrect. Antibodies directed
disease, an inherited bleeding disorder that af-
against RBC antigens lead to hemolytic ane-
fects both platelet adhesion and factor VIII
mia. They are unrelated to clotting disorders.
half-life. Schistocytes would not be present on
46. The correct answer is C. This woman is suf- the peripheral blood smear. Laboratory find-
fering from DIC secondary to gram-negative ings include a normal PT, elevated PTT, and
sepsis that resulted from a hospital-acquired normal platelet count.
urinary tract infection. Common causes of
47. The correct answer is E. This patient likely
DIC are gram-negative sepsis, malignancy,
has MGUS, a premalignant dysproteinemia
acute pancreatitis, trauma, transfusion reac-
affecting nearly 1%-3% of asymptomatic indi-
tions, and obstetric complications. The un-
viduals >50 years old. MGUS is defined by the
derlying mechanism of DIC is activation of
presence of <3 g/dL of monoclonal immuno-
the coagulation cascade, which leads to mi-
globulin (M protein) in the serum in the ab-
crothrombi and global consumption of plate-
sence of evidence for malignancy. This rule-
lets, fibrin, and coagulation factors. Thrombus
out includes performing a metastatic bone
formation in the microvasculature results in
survey, CT of the abdomen, 24-hour urine col-
microangiopathy with schistocytes and helmet-
lection, serum protein electrophoresis, and po-
shaped cells, which are shown in the image.
tentially bone marrow aspiration. There is no
Complications include bleeding, thrombosis,
treatment for MGUS, and approximately 1%
and organ failure. Laboratory findings include
of patients per year with MGUS will progress
elevated PT, elevated PTT, elevated D-dimer
to an overt plasma cell dyscrasia. It is essential
level, and decreased platelet count.
Chapter 11: Hematology-Oncology  •  Answers 305

to monitor M protein levels, as well as Bence a-globin (two gene regions out of four are in-
Jones proteinuria, in these patients at periodic tact). A mild anemia will be present, but the
intervals for the remainder of their lives. electrophoresis results will be normal, because
the two remaining a-globin genes produce suf-
Answer A is incorrect. Bisphosphonates such
ficient a-globin chains for normal HbA1 lev-
as alendronate and risedronate are used to treat
els. Deletion of only a single a-gene results in
multiple myeloma, which is known to cause
an asymptomatic carrier with no hematologic
bone destruction as a result of increased os-
manifestations.
teoclast activity. Bisphosphonates have been
shown to decrease pain and fractures in mul- Answer C is incorrect. b-Thalassemia minor
tiple myeloma by reducing the number and (the heterozygous defect) would show de-
activity of osteoclasts. creased but not absent HbA1. This is a less se-
vere form of the disease. b-Thalassemia minor

High-Yield Systems
Answer B is incorrect. High-dose steroids are
confers a mild protective effect against Plasmo-
not a treatment modality for MGUS, as there
dium falciparum malaria because of the short-
is no treatment for MGUS.
ened lifespan of the RBCs.
Answer C is incorrect. Vinca alkaloids such as
Answer D is incorrect. Glucose-6-phosphate
vincristine and vinblastine are microtubule in-
dehydrogenase deficiency does not present
hibitors used to treat some cancers, including
with an abnormal Hb electrophoresis.
leukemias and lymphomas.
Answer E is incorrect. HbH is associated with
Answer D is incorrect. A patient with an ab-
three-gene deletion of a-globin. The abnormal
normal monoclonal serum immunoglobu-
Hb molecule (HbH) contains four b-chains,
lin level but no other signs of malignancy
and is detected by electrophoresis. The b-thal-
(MGUS) should be monitored at regular in-
assemias are more prevalent in Mediterranean
tervals for progression of disease, but no treat-
people, whereas the a-thalassemias are more
ment is indicated.
prevalent in Asian and African populations.

Hematology-Oncology
48. The correct answer is B. Thalassemias are Answer F is incorrect. Hb Bart is the most
inherited diseases involving decreased syn- severe of the hemoglobinopathies, involving
thesis or complete absence of either the deletion of all four a-globin genes. This re-
a-globin chain or the b-globin chain of Hb. sults in the absence of all hemoglobins that
This patient has classic symptoms of severe require this chain and the sole production of
b-thalassemia (Cooley anemia): hemolytic Hb Bart, a tetramer of the γ-chain (normally a
anemia, hepatosplenomegaly, and “chipmunk component of HbF). This condition leads to
facies” (reflecting the extramedullary hemato- hydrops fetalis and intrauterine fetal death. Hb
poiesis in the bones of the face). The require- Bart is detected by electrophoresis.
ment for blood transfusions since birth should
raise the suspicion for b-thalassemia major, 49. The correct answer is E. This patient had an
but the Hb electrophoresis results alone can acoustic schwannoma removed from the cer-
be used to arrive at this conclusion. This pa- ebellopontine angle. Schwann cell tumors
tient shows increased HbF (a2γ2) and HbA2 are the third most common primary intracra-
(a2δ2); thus synthesis of the a-chain is intact. nial tumor, are often localized to cranial nerve
Absence of HbA1 (a2b2) supports an absence VIII (acoustic neuroma), and are commonly
of b-chain synthesis and, therefore, a diagnosis seen at the cerebellopontine angle. The most
of b-thalassemia major. Death in these individ- common signs and symptoms of schwannomas
uals often is caused by cardiac failure second- include hearing loss, tinnitus, vertigo, hydro-
ary to hemochromatosis. cephalus, and increased intracranial pressure.
Most are benign, slow-growing tumors that can
Answer A is incorrect. a-Thalassemia mi-
be resected. Histologically, two patterns are
nor is associated with a two-gene deletion of
found: (1) Antoni A, or spindle cells palisad-
306 Section II: Organ Systems  •  Answers

ing and forming a whorl appearance; and (2) hyperplasia or tumor leading to hypercal-
Antoni B, or loosely arranged tissue after de- cemia, medullary carcinoma of the thyroid,
generation in the tumor. The histopathologic and pheochromocytoma (which commonly
description of this patient’s tumor is consistent causes elevated plasma catecholamine levels).
with Antoni A. The patient’s episodes of lightheadedness and
sweating may be attributed to the elevated
Answer A is incorrect. Medulloblastomas are
catecholamine level found in pheochromo-
highly malignant radiosensitive tumors that are
cytoma. The MEN syndromes follow an au-
typically found in the posterior fossa. These tu-
tosomal dominant pattern of inheritance, and
mors are of neuroectodermal origin, and histo-
MEN-2A and MEN-2B are both linked to dis-
pathologic examination shows a rosette or peri-
tinct mutations in the ret proto-oncogene.
vascular pseudorosette pattern. Peak incidence
occurs in childhood. Answer A is incorrect. Mutations in braf have
High-Yield Systems

been associated with papillary carcinoma of


Answer B is incorrect. Meningiomas are
the thyroid. This is the most common thyroid
slow-growing tumors that occur most often in
malignancy, and it has been associated with a
the hemispheric convexities and parasagittal
remote history of radiation exposure. It is not
regions. They rarely appear in the cerebello-
associated with any of the MENs.
pontine angle. However, the histology would
classically show psammoma bodies, or areas of Answer B is incorrect. The erb-B2 oncogene is
calcification. associated with breast, ovarian, and gastric car-
cinomas.
Answer C is incorrect. Neurofibromas are tu-
mors of peripheral origin. This patient’s tumor Answer C is incorrect. Mutations in the
is intra-axial, as shown on the CT scan. Histo- MEN1 tumor suppressor gene are found in
logically, these cells appear as loosely arranged patients with MEN type 1, also known as
spindle cells with intervening collagen. Wermer syndrome. This disease is character-
ized by hyperplasia or tumor of the “3 Ps”:
Answer D is incorrect. Oligodendrogliomas
Pituitary, Parathyroid, and Pancreas. It may
Hematology-Oncology

are relatively uncommon, slow-growing tumors


manifest its pancreatic component by the
that occur most often in the frontal lobes. The
Zollinger-Ellison syndrome, hyperinsulinemia,
tumor is composed of homogeneous sheets of
or pancreatic cholera. Although this patient
cells with uniformly rounded nuclei and an as-
does have hypercalcemia that may be attrib-
sociated network of finely branching blood ves-
uted to hyperplasia or tumor of the parathyroid
sels.
gland, he does not have any pancreatic symp-
50. The correct answer is E. The characteris- toms. MEN-1 syndrome is not associated with
tics described in the question are found in medullary thyroid carcinoma.
patients with multiple endocrine neoplasia Answer D is incorrect. The ras oncogene mu-
(MEN) type 2, also known as Sipple’s syn- tation is associated with follicular thyroid carci-
drome. MEN-2 is associated with parathyroid noma, not medullary thyroid carcinoma.
Chapter 12

Musculoskeletal

307
308 Section II: Organ Systems  •  Questions

Q u e st i o n s

1. A 68-year-old man presents to his physician’s coursing through the inguinal canal. Which of
office with diffuse pelvic pain. X-ray of the ab- the following types of hernia does this patient
domen is shown in the image. Which of the most likely have?
following would help diagnose the primary tu-
(A) Diaphragmatic hernia
mor most commonly responsible for these ra-
(B) Direct inguinal hernia
diologic findings?
(C) Femoral hernia
(D) Hiatal hernia
(E) Indirect inguinal hernia
High-Yield Systems

4. A 22-year-old college student presents to the


school health service complaining of wors-
ening weakness in his arms and legs. He says
that over the past day he has also begun to feel
weakness in his chest and back. He mentions
that he thinks he had food poisoning earlier
in the week, which caused stomach pain and
bloody diarrhea. Physical examination reveals
that the deep tendon reflexes in his lower ex-
tremities are absent. His physician sends his
stool to be cultured. The results show infection
Reproduced, with permission, from Skinner HB. Current with Campylobacter jejuni. Which one of the
Diagnosis & Treatment in Orthopedics, 4th ed. New York: following statements is most consistent with
McGraw-Hill, 2006; Fig. 6-78A.
the disease process connecting the patient’s
gastrointestinal infection and his neurological
(A) Digital rectal examination symptoms?
Musculoskeletal

(B) Palpation of the abdomen (A) An excessive immune response to a gastro-


(C) Palpation of the costovertebral angle intestinal Campylobacter jejuni infection
(D) Palpation of the neck led to an autoimmune inflammation of pe-
(E) Thorough skin examination ripheral nerves
(B) Dissemination of Campylobacter jejuni
2. A 17-year-old boy presents to his pediatrician gastrointestinal infection led to infiltration
with pain in his right leg. Biopsy reveals a ma- of peripheral muscle cells with C jejuni
lignancy in the distal femur. He has a family his- (C) Dissemination of Campylobacter jejuni
tory of “eye cancer” in a younger sister. Which gastrointestinal infection led to infiltration
of the following is the most likely diagnosis? of peripheral nerves with C jejuni
(A) Chondrosarcoma (D) Toxins secreted by Campylobacter jejuni
(B) Enchondroma infiltrated into peripheral muscle cells
(C) Ewing sarcoma (E) Toxins secreted by Campylobacter jejuni
(D) Osteochondroma infiltrated into peripheral nerves, causing
(E) Osteosarcoma their destruction

3. A 76-year-old man is scheduled to undergo 5. A 37-year-old man presents to his physician


elective repair of an abdominal hernia that with a rash. Physical examination reveals the
is easily reducible. During the repair, the lesions shown in the image; the man says the
surgeon sees that the sac protrudes directly lesions do not itch. Which family of viruses is
through the external inguinal ring without responsible for causing this rash?
Chapter 12: Musculoskeletal  •  Questions 309

takes a fluid sample from the affected area. Ex-


amination of the contents reveals the presence
of intracellular crystals. Which of the follow-
ing medications would prevent this problem
from recurring?
(A) Allopurinol
(B) Calcitonin
(C) Collagenase
(D) Furosemide
(E) Warfarin

8. A 16-year-old gymnast presents to the emer-

High-Yield Systems
Reproduced, with permission, from Fauci AS, et al, eds.
gency department after landing awkwardly on
Harrison’s Principles of Internal Medicine, 17th ed. New York:
her ankle subsequent to dismounting from the
McGraw-Hill, 2008; Fig. 176-1.
parallel bars. She reports that she felt her ankle
“roll inward,” essentially inverting her ankle as
(A) Adenovirus a result of a misaligned axial load. On physi-
(B) Hepadnavirus cal examination of the injured ankle, there ap-
(C) Herpes simplex virus pears to be a forward displacement of the talus.
(D) Papillomavirus Which of the following ligaments has she most
(E) Polyomavirus likely injured?
(F) Poxvirus (A) Anterior talofibular ligament
(B) Calcaneofibular ligament
6. A 17-year-old girl complains of a painful, (C) Talonavicular ligament
swollen left elbow and fever. In the previous (D) Tibiocalcaneal ligament
few days her right knee was also swollen and (E) Tibiotalar ligament
slightly painful. The physician notices several
oral ulcers and an edematous and tender left 9. A 3-year-old boy is brought to the pediatrician

Musculoskeletal
elbow. Laboratory tests are notable for weakly by his mother, who notes that, despite no his-
positive antinuclear antibodies and anemia, tory of trauma, her son has been crying, rub-
and the results of Venereal Disease Research bing his forearm, and guarding this area. This
Laboratory testing are positive. The patient is child’s medical history is significant for nor-
shocked when informed of her positive result mal, full-term birth and satisfactory achieve-
for syphilis, stating that she has no sexual his- ment of developmental milestones, but poor
tory. Which of the following is the most spe- dentition and two previous instances of skel-
cific antibody for the patient’s condition? etal fracture. The posterior area of the forearm
(A) Anti-IgG is tender and erythematous. There is a blu-
(B) Anti-Jo-1 ish discoloration to the sclera and diminished
(C) Anti-Smith auditory acuity. Which of the following is the
(D) Anticentromere most likely diagnosis?
(E) Antimicrosomal (A) A defect in dystrophin
(B) A defect in fibrillin
7. A 32-year-old man recently underwent a liver (C) A defect in type I collagen
transplant and was prescribed cyclosporine to (D) A defect in type III collagen
reduce the chance of immune rejection. After (E) Child abuse
a few weeks of taking the drug, he started expe- (F) Hemophilia A
riencing acute pain, tenderness, warmth, and
swelling of his right great toe. His physician
310 Section II: Organ Systems  •  Questions

10. A 32-year-old man is brought into the emer- (C) Macula adherens
gency department by ambulance after fall- (D) Zona adherens
ing from a ladder while cleaning his roof gut- (E) Zona occludens
ters. His vital signs are stable, he is fully alert
and oriented, and he reports having no past 12. A 27-year-old man presents to his dermatolo-
medical problems. He is in excruciating pain, gist with several red, tender nodules on his
which he states is located in his left arm. An lower legs. On histopathology, there is inflam-
x-ray of the left upper extremity is shown in the mation of the subcutaneous fat, tissue septal
image. If left untreated, which of the following widening, neutrophilia, and fibrin exudation.
muscles is at risk of losing function due to this Which of the following diseases often goes
injury? along with these skin findings?
(A) Acne vulgaris
High-Yield Systems

(B) Crohn disease


(C) Eczema
(D) Pancreatitis
(E) Psoriasis

13. A previously healthy 30-year-old woman who


performs as a contortionist in the circus is
found unconscious in her dressing room. By
the time she receives medical attention, she
cannot be revived. An autopsy is performed
and an abnormality is discovered in the circle
of Willis, as shown in the image. The patient
was also reported to have a history of easy
bruising and bleeding. Which of the following
Reproduced, with permission, from Skinner HB. Current
proteins was most likely defective in this pa-
Diagnosis & Treatment in Orthopedics, 4th ed. New York:
tient?
McGraw-Hill, 2006; Fig. 11-44.
Musculoskeletal

(A) First and second lumbricals


(B) Brachioradialis
(C) Flexor carpi ulnaris
(D) Opponens pollicis
(E) Palmaris longus
(F) Pronator teres

11. A 52-year-old woman presents to her physician


with three weeks of skin blisters. She says when
these blisters rupture they leave painful ulcer-
ations. On examination, tender ulcerations
are found on her back and trunk. Biopsy is
performed and direct immunofluorescence of Reproduced, with permission, from USMLERx.com.
the lesion reveals a net-like deposit of immu-
noglobulin surrounding keratinocytes. What
aspect of epithelial cell junctions is targeted by (A) Fibrillin-1
autoantibodies in this condition? (B) Keratin 14
(C) Type I procollagen
(A) Gap junctions
(B) Hemidesmosomes
Chapter 12: Musculoskeletal  •  Questions 311

(D) Type III procollagen Treatment is initiated, and the patient’s uric
(E) Type IV collagen acid levels gradually fall. What is the mecha-
nism of action of the best treatment?
14. A couple brings their 3-year-old son to the
emergency department, reporting that he fell
down the stairs and broke his arm. The boy
has a tearful face and gingerly holds his right
arm by the elbow, but refuses to look the physi-
cian in the eye or to answer any questions. An
x-ray of the boy’s arm is performed. Which of
the following types of fracture is most likely to
suggest an etiology of child abuse?

High-Yield Systems
(A) Bowing fracture
(B) Buckle fracture
(C) Greenstick fracture
(D) Spiral fracture
Reproduced, with permission, from Fauci AS, et al, eds. Harri-
15. A 31-year-old man comes to the clinic com- son’s Principles of Internal Medicine, 17th edition. New York:
plaining of red and itchy eyes for the past eight McGraw-Hill, 2008; Figure 327-1.
hours. The patient has had pain on urination
and diffuse joint pain for 1 month, but tested
negative for gonorrhea and chlamydial infec- (A) Binds tubulin
tion on a previous visit three weeks ago. He (B) Blocks formation of prostaglandins and
has also tested negative for rheumatoid factor, thromboxane from arachidonic acid
and his human leukocyte antigen (HLA) sta- (C) Inhibits release of phospholipase A2
tus is HLA-B27. When asked about any recent (D) Inhibits xanthine oxidase
illnesses, the patient recalls going to the emer- (E) Selective, competitive angiotensin II re-
gency department two months ago for a bad ceptor inhibition

Musculoskeletal
case of diarrhea. Which of the following is the
most likely diagnosis? 17. A 57-year-old automotive factory worker with
no significant past medical history reports to
(A) Anklyosing spondylitis
the clinic complaining of weakness and pain
(B) Lyme arthritis
in his right shoulder with any movement. He
(C) Psoriatic arthritis
says that for the past two weeks he has had
(D) Reactive arthritis
trouble lifting his right arm above his head. He
(E) Systemic lupus erythematosus
reports using his left arm to lift his right arm
16. A 70-year-old man complains of a long his- slightly, and then he can lift his right arm the
tory of pain in his ankles, toes, and fingers. He rest of the way without help. Which of the fol-
has experienced intermittent acute attacks of lowing muscles is most likely injured in this
exquisite pain every few months, followed by patient?
completely asymptomatic periods. In recent (A) Deltoid
years, he has had near-constant discomfort at (B) Infraspinatus
baseline, and now has permanent “swelling” in (C) Subscapularis
many of the joints of his fingers and toes. Joint (D) Supraspinatus
fluid is aspirated, and the fluid is examined (E) Teres minor
under polarized light (shown in the image).
312 Section II: Organ Systems  •  Questions

18. A 49-year-old man is lifting a heavy box above tient has asymmetric left-sided ptosis, a subjec-
his head when he experiences a sudden tear- tive feeling of “blurred vision” when reading
ing sensation and pain that travels along his the eye chart on the wall, and a facial droop
left arm. When he attempts to move his arm bilaterally. What test would confirm the most
he is not able to medially rotate it. Which of likely diagnosis in this patient?
the following muscles is most likely injured?
(A) Deltoid muscle
(B) Infraspinatus muscle
(C) Subscapularis muscle
(D) Supraspinatus muscle
(E) Teres minor
High-Yield Systems

19. A 22-year-old emergency room nurse has a


positive purified protein derivative test. Results
of radiography of the chest are unremarkable.
She begins a nine-month prophylaxis regimen,
but two months into treatment she complains
of the sudden onset of fever, rash, and swell-
ing of her joints. What is the mechanism of the Reproduced, with permission, from USMLERx.com.
medication that caused these adverse effects?
(A) Decreases synthesis of folate
(A) Acetylcholine receptor antibody assay
(B) Decreases synthesis of messenger RNA
(B) Complete blood cell count
(C) Decreases synthesis of mycolic acids
(C) Parathyroid hormone level
(D) Decreases synthesis of peptidoglycan
(D) Thyroid stimulating hormone level
(E) Increases synthesis of cGMP
(E) Urine catecholamine levels
20. A young man complains of difficulty eating.
22. A 12-year-old African-American girl is brought
His neurologic examination reveals a symmet-
Musculoskeletal

to the physician with complaints of fever, mal-


ric smile, symmetric palate elevation, midline
aise, and pain in her left forearm for the past
tongue, as well as good shoulder shrug and
four days. Her mother reports no history of
head turning strength. However, he has dif-
trauma or fracture, but recounts an upper res­
ficulty opening his mouth and biting down
piratory infection a few days ago. The patient
firmly. Damage to what cranial nerve could ex-
has been hospitalized three times for abdomi-
plain his condition?
nal pain, for which she takes hydroxyurea. Her
(A) V2 temperature is 39.8°C (103.6°F) and the rest
(B) V3 of her vital signs are stable. On examination
(C) VII the left forearm is erythematous, warm, and
(D) X tender to palpation. Blood samples for culture
(E) XII are drawn. Which of the following organisms
is most likely to be isolated from this patient’s
21. A 32-year-old woman is incidentally found to blood?
have a mediastinal mass (see image) while hav-
(A)
Escherichia coli
ing a plain film x-ray of the chest performed.
(B)
Pseudomonas aeruginosa
She subsequently reports that lately she has
(C)
Salmonella species
been “feeling more fatigued, especially at the
(D)
Staphylococcus aureus
end of the day.” In the office, she has difficulty
(E)
Streptococcus pyogenes
rising from her chair to move to the examina-
tion table. On physical examination the pa-
Chapter 12: Musculoskeletal  •  Questions 313

23. A 9-month-old girl presents to her pediatrician 25. A 42-year-old woman comes to the clinic com-
because of abnormal stature and growth. The plaining of blurry vision. She states that for the
child displays short stature, shortening of the past three weeks her eyes have been very dry
proximal limbs, short fingers, and frontal boss- and itchy, and she is unable to make tears. She
ing. Which of the following is the most likely also states that she has had a very dry mouth
etiology of this infant’s condition? despite drinking adequate fluids. Physical ex-
amination reveals bilateral dry, ulcerated cor-
(A) Bone resorption
neas and fissures on the sides of her lips. In
(B) Collagen formation
addition, both of her knees are erythematous
(C) Endochondral ossification
and swollen. When asked about her knees, she
(D) Mucopolysaccharide degradation
says, “Yes, my knees and wrists tend to be swol-
24. A 64-year-old woman with no prior medical len and stiff in the morning, but my mom had

High-Yield Systems
history has had increasing back pain and right arthritis.” Testing for several autoantibodies
hip pain for the past decade. The pain is worse reveals she is rheumatoid factor-positive and
at the end of the day. On physical examina- antibody-SS-B (La)-positive. Which of the fol-
tion, she has bony enlargement of the distal lowing is the most likely diagnosis?
interphalangeal joints (see image). Which of (A) CREST syndrome
the following diseases is most likely the cause (B) Cystic fibrosis
of this patient’s symptoms? (C) Sicca syndrome
(D) Sjögren syndrome
(E) Sjögren-Larsson syndrome

26. An 18-year-old man is injured during a soc-


cer game when the goalie dives for the ball
but tackles the player on the lateral aspect of
his leg. He is helped off the field and brought
to the emergency department, where he tells
the physician that he heard a pop when he was

Musculoskeletal
tackled. Physical examination reveals an ante-
rior drawer sign and MRI reveals a torn ante-
rior cruciate ligament and medial collateral
ligament. Which one of the following struc-
tures is also likely to be injured?
Reproduced, with permission, from Fauci AS, et al, eds. Harri- (A) Lateral collateral ligament
son’s Principles of Internal Medicine, 17th edition. New York:
(B) Lateral meniscus
McGraw-Hill, 2008; Figure 326-2.
(C) Medial meniscus
(D) Patellar ligament
(A) Gout (E) Posterior cruciate ligament
(B) Osteoarthritis
(C) Osteomyelitis
(D) Pseudogout
(E) Rheumatoid arthritis
314 Section II: Organ Systems  •  Questions

27. A mother brings her 12-year-old son to the pe- (A) Increased intact parathyroid hormone
diatrician’s office because he is complaining of level
leg pain. The pain is located at the distal right (B) Increased serum 1,25-dihydroxycholecal-
tibia and has persisted over the past two weeks. ciferol level
The mother also notes that the child has had (C) Increased serum 25-hydroxycholecalciferol
intermittent fevers during this time. On ex- level
amination, the site is erythematous and swol- (D) Increased serum calcium level
len. A plain film x-ray is taken, the results of (E) Increased serum phosphorus level
which are shown in the image. Biopsy of the
site shows sheets of many uniform cells with 29. A 45-year-old woman who recently underwent
scant, clear cytoplasm and no evidence of nor- a left mastectomy and axillary dissection for
mal bony matrix. Which of the following is the breast cancer now presents to her physician
High-Yield Systems

most likely diagnosis? with a chief complaint that she feels like her
shoulder blade sticks out sometimes. She de-
nies any pain. This patient’s injured nerve orig-
inates at which spinal levels?
(A) C3, C4, and C5
(B) C5 and C6
(C) C5, C6, and C7
(D) C7 and C8
(E) C7, C8, and T1

30. A 42-year-old woman has had increasing pain


and swelling of the joints of her hands and feet
for several months. It is becoming very difficult
for her to perform common household tasks. A
microscopic image of the synovium of a proxi-
mal interphalangeal joint in her hand is shown
Musculoskeletal

in the image. Which of the following labora-


Courtesy of Dr. Jeevak Almast, University of Rochester Medi- tory serologic findings would most likely be
cal Center. positive in this patient?

(A) Ewing sarcoma


(B) Giant cell tumor
(C) Osteochondroma
(D) Osteomyelitis
(E) Osteosarcoma

28. A 1-year-old adopted, darkly pigmented boy


is brought to the pediatrician for his first well-
child check-up. The adoptive parents do not
know any of the infant’s past medical history
or family history. Physical examination reveals
an unusual widening of the child’s wrists and
ankles and marked enlargement of the child’s
costochondral junctions. What is a characteris- Reproduced, with permission, from USMLERx.com.
tic laboratory finding used to support the most
likely diagnosis?
Chapter 12: Musculoskeletal  •  Questions 315

(A) Anti-centromere antibody 33. A 27-year-old homeless man presents to the


(B) Anti-nuclear antibody clinic because of a five-day history of pain and
(C) Borrelia burgdorferi antibody swelling in his right upper arm. MRI of the
(D) HLA-B27 area reveals diffuse soft tissue and bone inflam-
(E) IgM anti-IgG mation. Results of bone biopsy are shown in
the image. Blood cultures grow Pseudomonas.
31. A 6-year-old child is brought to the emergency Which of the following would a complete his-
department by his parents after they all re- tory of this patient most likely reveal?
turned from a trip to East Africa. His parents
report that approximately two weeks ago he
had a fever and diarrhea that resolved. How-
ever, he now has a fever and weakness of his

High-Yield Systems
left leg. On further questioning, his parents
state that he is home-schooled and that he has
never received vaccinations. Which of the fol-
lowing sequelae is most likely to result in this
patient?
(A) Neuron loss in the posterior horns
(B) Reflex preservation in affected limbs
(C) Respiratory muscle paralysis
(D) Sensory loss in affected limbs
(E) Short-term memory loss

32. A 15-year-old boy presents to the emergency Reproduced, with permission, from USMLERx.com.
department after falling off his skateboard. On
physical examination he is unable to dorsiflex
(A) Corticosteroid use
or evert at the ankle. In addition, the patient
(B) Intravenous drug use
reports pain and numbness in the lateral leg
(C) Miliary tuberculosis

Musculoskeletal
and dorsum of the foot. When asked to walk,
(D) Multiple sexual partners
he raises his affected leg high off the ground
(E) Sickle cell anemia
and his foot slaps the ground when walking.
He is diagnosed with a fracture. Which of the
34. A 22-year-old woman who is a professional ten-
following structures is most likely to be com-
nis player presents to her physician because of
promised by this fracture?
pain on the lateral aspect of her elbow radiat-
(A) Common peroneal nerve ing down her forearm. She describes the pain
(B) Femoral nerve as shooting and constant. Repetitive use of
(C) L4 nerve root which of the following muscles most likely led
(D) Obturator nerve to this patient’s condition?
(E) Tibial nerve
(A) Biceps
(B) Extensor carpi radialis
(C) Extensor carpi ulnaris
(D) Flexor carpi ulnaris
(E) Pronator teres
316 Section II: Organ Systems  •  Questions

35. A 50-year-old man who recently returned from (A) Lengthening of A band; no change in H
visiting family in northern New Mexico comes band; shortening of I band
to the physician with exquisitely tender and (B) No change in A band; lengthening of H
enlarged lymph nodes. He also complains of band; lengthening of I band
fever, chills, and general weakness. On physi- (C) No change in A band; shortening of H
cal examination, the physician notes a painful band; shortening of I band
ulcer surrounded by dark, hemorrhagic pur- (D) Shortening of A band; no change in H
pura on the right arm in the area where, ac- band; shortening of I band
cording to the patient, a flea had bitten him (E) Shortening of A band; shortening of H
five days ago. After being admitted to the hos- band; no change in I band
pital, the patient soon develops abnormal co-
agulation times and is quickly started on a regi- 37. A 38-year-old man comes to the clinic with a
High-Yield Systems

men of streptomycin and tetracycline. Which swollen, sausage-like left middle finger along
of the following organisms is most likely re- with diffuse joint swelling of his left hand and
sponsible for this patient’s symptoms? right foot over the past three days. The patient
also has scaly plaques with well-defined bor-
(A)
Babesia microti
ders along the skin just distal to both elbows.
(B)
Bacillus anthracis
His uric acid level is within normal limits.
(C)
Leishmania donovani
Which of the following is most likely to be
(D)
Trichinella spiralis
seen in this patient?
(E)
Yersinia pestis
(A) Antigliadin antibodies
36. A 63-year-old man and known alcoholic has (B) Elevated erythrocyte sedimentation rate
been unable to afford alcohol. He now pre­ (C) Negatively birefringent crystals in joint
sents with hypertension and profuse sweating, fluid
and is agitated because he believes that insects (D) Positive rheumatoid factor
are crawling all over his skin. During the ex- (E) Weakly positively birefringent crystals in
amination, he loses consciousness and begins joint fluid
to seize on the stretcher. The seizure-induced
Musculoskeletal

muscle contractions result in which of the fol- 38. A 20-year-old man presents to the physician
lowing changes in length within each sarco- with a nontender indurated mass over his man-
mere? dible. He has had this mass for four months
after undergoing oral surgery and decided
to come to the physician because the mass
Sarcomere
started to ooze a thick yellow exudate. Yellow
granules are seen on microscopic examina-
I A tion of the discharge and an antibiotic is pre-
M line band band Z line
scribed. Which of the following best describes
the mechanism of action of the antibiotic most
H
band likely prescribed?
Myofibril
(A) Binds ergosterol, forming pores in the
membrane
(B) Block bacterial cell wall synthesis by inhib-
iting transpeptidase crosslinking
(C) Block bacterial nucleotide synthesis
(D) Block bacterial protein synthesis
(E) Inhibits ergosterol synthesis
Reproduced, with permission, from USMLERx.com.
Chapter 12: Musculoskeletal  •  Questions 317

39. A 74-year-old man presents to his physician 41. A 25-year-old man develops acute onset of fe-
with a bulge in his scrotum. He is diagnosed ver, malaise, muscle pain, hypertension, ab-
with an inguinal hernia on his left side, and dominal pain, bloody stool, and prerenal fail-
undergoes surgery two days later to repair the ure six months after recovering from an acute
hernia. On postoperative day two he com- hepatitis B infection. Which of the following
plains of numbness and tingling of his scro- disease processes is most likely responsible for
tum. Which nerve root contributes to the af- the patient’s findings?
fected nerve?
(A) Buerger disease
(A) L1-L2 (B) Giant cell (temporal) arteritis
(B) L2-L3 (C) Kawasaki syndrome
(C) S1-S3 (D) Polyarteritis nodosa
(D) S2-S4 (E) Takayasu arteritis

High-Yield Systems
40. The diagram shows a cross-section of normal 42. A 36-year-old woman presents to the clinic
human skin. Pemphigus vulgaris patients suf- with a new complaint of fatigue of several
fer from production of autoantibodies against months’ duration. She also reports stiffness
which of the following labeled layers in this in both hands that is worse in the morning
image? and decreases after soaking them in her warm
morning bath each day. Physical examination
reveals a low-grade fever. Subcutaneous nod-
ules are palpated along her forearm bilaterally.
What type of hypersensitivity reaction is caus-
ing this patient’s arthritis?
(A) Arthus reaction
A B
(B) Delayed cell-mediated hypersensitivity re-
action
C (C) Immune complex hypersensitivity reaction
(D) Type I hypersensitivity reaction

Musculoskeletal
(E) Type II hypersensitivity reaction

D 43. A 27-year-old man comes to the physician’s of-


fice with a six-month history of low back pain
and stiffness that wakes him up during the
night and is worst in the morning. The pa-
tient was diagnosed with bilateral sacroiliitis
E four months ago because of his tenderness to
percussion of the sacroiliac joints and pain on
springing the pelvis up. He has severe limita-
Reproduced, with permission, from USMLERx.com. tion of motion of his lumbar spine. Laboratory
tests are most likely to be positive for which of
the following?
(A) A
(A) Human leukocyte antigen B27
(B) B
(B) Antinuclear antibodies
(C) C
(C) Anti-neutrophil cytoplasmic antibodies
(D) D
(D) IgM antibodies to B burgdorferi
(E) E
(E) Rheumatoid factor
(F) Vertebral compression fracture
318 Section II: Organ Systems  •  Questions

44. A visibly upset 15-year-old boy is brought 45. A 38-year-old woman presents to the emer-
to the emergency department because he gency department complaining of increas-
punched a wall and now has pain in his hand. ing muscle weakness and pain. She first no-
The physician tells the patient that he has bro- ticed the muscle weakness approximately one
ken his hand. Which of the following is the month ago, and it has gradually worsened
most likely site of this patient’s fracture? since then. During the same time she has had
increasing difficulty swallowing her meals.
(A) Distal radius
Two weeks prior to this visit, she recalls swell-
(B) Hamate
ing and a rash over her eyelids. On physical
(C) Metacarpals
examination, deltoid and quadriceps strength
(D) Phalanges
are 2/5 bilaterally. Creatine kinase, lactate de-
(E) Scaphoid
hydrogenase, and aldolase levels are elevated.
High-Yield Systems

Which auto-antibody would diagnostic testing


find to be elevated?
(A) Anti-dsDNA
(B) Anti-Jo-1
(C) Anti-IgG
(D) Anti-microsomal
(E) Anti-mitochondrial
Musculoskeletal
Chapter 12: Musculoskeletal  •  Answers 319

An s w e r s

1. The correct answer is A. The image demon- Answer A is incorrect. Chondrosarcomas are
strates osteoblastic lesions of the pelvis sec- malignant cartilaginous tumors that occur
ondary to metastatic cancer. Prostate cancer, most commonly in men 30-60 years old, usu-
the most common cause of cancer in men, is ally in the pelvis, spine, scapula, humerus,
notorious for producing osteoblastic lesions tibia, or femur.
upon metastasis. Prostate cancer may often be
Answer B is incorrect. Enchondromas are be-
detected on physical examination through the
nign cartilaginous tumors found in intramed-
use of a digital rectal examination. Digital rec-
ullary bone, and are most often found in the
tal examination can also be useful in detecting
distal extremities.
a small number of colorectal cancers, though

High-Yield Systems
most bony lesions of metastatic colorectal can- Answer C is incorrect. Ewing sarcoma is the
cer are osteolytic. second most common primary bone cancer.
Most patients present at the onset of puberty
Answer B is incorrect. Palpation of the abdo-
(average is between 10 and 15 years old). Ew-
men may assist in the detection of leukemia or
ing sarcoma most commonly occurs in the di-
lymphoma, as splenomegaly is a common pre-
aphyses of long bones, pelvis, scapula, and ribs.
senting symptom. These cancers occur much
less frequently than prostate cancer, and more Answer D is incorrect. Osteochondromas are
often produce osteolytic lesions. benign growths that are often first diagnosed in
late adolescence or early adulthood. They are
Answer C is incorrect. Signs of renal cell can-
the most common benign bone tumor.
cer include flank pain (noted upon palpation
of the costovertebral angle), the presence of a 3. The correct answer is B. Direct inguinal her-
flank mass, hematuria, and/or weight loss. Re- nias protrude directly through the abdominal
nal metastases are more commonly osteolytic. wall in Hesselbach’s triangle, which is bor-
Answer D is incorrect. Palpation of the neck dered by the inguinal ligament (inferiorly),

Musculoskeletal
may be helpful in detecting thyroid cancer. rectus abdominis muscle (medially), and infe-
Thyroid metastases are more often lytic. rior epigastric vessels (laterally). Direct versus
indirect distinguishes where the hernia enters
Answer E is incorrect. A thorough skin ex-
the inguinal canal, either through the inter-
amination would be vital in detecting a mela-
nal ring (indirect) or straight through the ab-
noma. However, melanoma is much less com-
dominal wall (for direct hernias, think directly
mon than prostate cancer, and most often
through the wall). In this case the abdominal
produces osteolytic lesions when it metasta-
contents pierced through the abdominal wall
sizes to bone.
and through the external inguinal ring. Direct
hernias often are discovered in older patients
2. The correct answer is E. Osteosarcoma is the
as a result of pressure and tension exerted over
most common primary malignant bony tumor.
time that eventually leads the abdominal wall
These tumors are seen predominantly in males
to give way. Direct hernias are less common
<20 years old and occur at the metaphyseal re-
and have less risk of strangulation than indirect
gion of long bones. Plain films of the affected
hernias.
bone often reveal a characteristic “sunburst”
pattern. Genetic mutations of the Rb gene are Answer A is incorrect. Diaphragmatic hernias
associated with both osteosarcoma and retino- are serious birth defects that are lethal unless
blastoma, which was likely present in this pa- repaired soon after birth. A defective develop-
tient’s sister. ment of the diaphragm leads to herniation of
abdominal contents into the thorax, displac-
ing the lung. Even after repair of such lesions,
320 Section II: Organ Systems  •  Answers

children may have poor pulmonary function Answer D is incorrect. Guillain-Barré syn-
due to poor lung development in utero. drome is a disease that primarily attacks pe-
ripheral nerves and Schwann cells. The as-
Answer C is incorrect. Femoral hernias pro-
cending paralysis and muscle weakness that
trude inferior to the inguinal ligament and
occur as a consequence are secondary to the
do not go through the external inguinal ring.
neuropathy.
They protrude below and lateral to the pubic
tubercle and are more common in women. Answer E is incorrect. While Campylo-
bacter jejuni does produce an enterotoxin, the
Answer D is incorrect. Hiatal hernias are
mechanism described in this answer choice
hernias of the stomach protruding superiorly
is not the major hypothesized pathogenesis of
through the diaphragm. This is common in
Guillain-Barré syndrome.
premature neonates and the elderly.
High-Yield Systems

Answer E is incorrect. Indirect inguinal her- 5. The correct answer is F. Molluscum contagio-
nias occur when abdominal contents enter the sum is a member of the poxvirus family that
internal inguinal ring through a patent proces- causes a localized infection consisting of non-
sus vaginalis, exit the inguinal canal through erythematous, pearly, dome-shaped papules
the external ring, and usually descend into the on the skin of an infected individual. Children
scrotum. These hernias are the most common and immunosuppressed patients are often in-
type found both in men and in women, but fected with this virus. The infection is usually
overall occur more commonly in men. They self-limited and spontaneously resolves after a
are found often in young individuals. They few months.
should be repaired when they are discovered
Answer A is incorrect. Adenovirus is a com-
to avoid the complications of strangulation and
mon cause of upper respiratory infections.
bowel infarction.
Answer B is incorrect. Hepadnavirus causes
4. The correct answer is A. This patient has hepatitis B with jaundice being a possible der-
Guillain-Barré as a result of his gastroenteritis. matologic sequela.
The important message here is that Guillain-
Answer C is incorrect. The various herpes
Musculoskeletal

Barré syndrome is thought to be primarily an


simplex viruses (HSVs) cause several different
autoimmune disorder against peripheral nerves
diseases resulting in rash. HSV-1 and HSV-2
and the cells that myelinate them (Schwann
cause genital herpes and cold sores. Varicella-
cells). Thus, it makes sense that an excessive
zoster virus causes chickenpox, which can re-
immune response to an infection (such as
activate and result in shingles. The lesions are
from a pathogen like Campylobacter jejuni)
painful vesicles with an erythematous base.
can lead to an autoimmune process. Histologi-
cally, this disease is characterized by perivenu- Answer D is incorrect. Papillomavirus causes
lar and endoneurial infiltration with lympho- warts, which can be flat, raised, or resemble a
cytes, macrophages, and plasma cells. cauliflower.
Answer B is incorrect. Guillain-Barré syn- Answer E is incorrect. Reactivation of poly-
drome is a disease that primarily attacks pe- omavirus results in progressive multifocal leu-
ripheral nerves and Schwann cells. The as- koencephalopathy in immunosuppressed pa-
cending paralysis and muscle weakness that tients.
occur as a consequence are secondary to the
neuropathy. 6. The correct answer is C. This patient has sys-
temic lupus erythematosus (SLE), which is
Answer C is incorrect. The mechanism sug- diagnosed by the presence of four of the fol-
gested in this answer choice is not thought to lowing 11 findings designated by the American
be the primary disease process in Guillain- Rheumatism Association, and summarized by
Barré syndrome. the mnemonic “BRAIN SOAP, MD:” Blood
Chapter 12: Musculoskeletal  •  Answers 321

dyscrasias (such as hemolytic anemia or throm- symptoms of hypothyroidism and a moderately


bocytopenia), Renal disorder, Arthritis (in two enlarged, nontender thyroid.
or more peripheral joints), Immunologic dis-
order (such as anti-DNA antibody and anti- 7. The correct answer is A. The main adverse
Smith antibody), Neurologic disorder, Serositis reactions to cyclosporine therapy are renal
(such as pleuritis or pericarditis), Oral ulcers, dysfunction, tremor, hirsutism, hypertension,
Antinuclear antibody (elevated titers in the ab- and gum hyperplasia. Cyclosporine can dam-
sence of drugs associated with drug-induced age renal tubules irreversibly, thus decreasing
lupus syndrome), Photosensitivity, Malar rash, the kidney’s ability to excrete potentially toxic
and Discoid rash. Many patients with SLE metabolites. This patient is suffering from
have antiphospholipid antibodies, which actu- acute gouty arthritis secondary to impaired re-
ally are believed to be antibodies against pro- nal excretion of uric acid and thus increased

High-Yield Systems
teins that complex to phospholipids. Because serum levels of urate, which can precipitate as
these antibodies also bind to the cardiolipin monosodium urate crystals in joints. Gout clas-
antigen used in syphilis serology, patients sically presents with a warm, red, painful joint.
with SLE may have a false-positive result for The most common joint affected in gout is the
syphilis. Whereas antinuclear antibodies are great toe. Other potential adverse effects of cy-
sensitive for SLE, anti-Smith and anti-double- closporine that occur secondary to renal fail-
stranded DNA are the most specific. Many ure include hyperkalemia, hypophosphatemia,
other autoantibodies may be present in SLE hypomagnesemia, hypercalciuria, and meta-
that are not necessarily sensitive or specific for bolic acidosis. Allopurinol is used to prevent
the condition. gouty arthritis by inhibiting xanthine oxidase,
an enzyme involved in uric acid synthesis. The
Answer A is incorrect. Anti-IgG, or rheuma-
mainstay of therapy for acute gout is nonste-
toid factor, is associated with rheumatoid ar-
roidal anti-inflammatory drugs (NSAIDs) and
thritis (RA). Although arthritis can be a symp-
corticosteroids.
tom of SLE, a patient with RA classically will
present with morning stiffness, systemic in- Answer B is incorrect. Calcitonin counter-
flammatory symptoms, and symmetric swell- acts the action of parathyroid hormone (PTH),

Musculoskeletal
ing, particularly in the metacarpophalangeal thus reducing serum calcium level. Whereas
and proximal interphalangeal joints. cyclosporine is known to cause hypercalciuria,
it is not associated with the formation and pre-
Answer B is incorrect. Anti-Jo-1 antibodies are
cipitation of calcium pyrophosphate crystals
associated with inflammatory myopathies such
in joints and connective tissues, also known as
as polymyositis and dermatomyositis. These
pseudogout. The development of pseudogout
conditions are characterized by proximal mus-
is associated with joint trauma, familial chon-
cle weakness and, in dermatomyositis, skin in-
drocalcinosis, hemochromatosis, and certain
volvement and increased incidence of malig-
other metabolic or endocrine disorders. Thus
nancy.
calcitonin would not prevent acute gouty ar-
Answer D is incorrect. Anticentromere anti- thritis.
bodies are associated with CREST syndrome,
Answer C is incorrect. Collagenase is an en-
a more limited type of scleroderma character-
zyme involved in collagen degradation. It is
ized by Calcinosis (tissue deposits of calcium),
key to pathogenesis of various bacteria, but it
Raynaud phenomenon, Esophageal dysmotil-
also can be induced during an immune re-
ity, Sclerodactyly (tightness and thickening of
sponse and lead to indirect tissue damage. Cy-
digits), and Telangiectasia.
closporine does not affect collagen turnover in
Answer E is incorrect. Antimicrosomal an- joints. Collagenase would not counteract the
tibodies are associated with Hashimoto thy- effects of cyclosporine and should not lead to
roiditis. Clinically this would present with deposition of crystals in joint fluid.
322 Section II: Organ Systems  •  Answers

Answer D is incorrect. Furosemide is a loop ligaments, since they are weaker. The talona-
diuretic used in treatment of congestive heart vicular ligament is a medial ligament so it is
failure and acute renal failure. Loop diuretics much less likely to be injured.
may be used to increase calcium diuresis in
Answer D is incorrect. The lateral ligaments
hypercalcemic states. However, loop diuretics
are more commonly injured than the medial
are known to cause hyperuricemia, which may
ligaments, since they are weaker. The tibio-
precipitate gout in some patients and should
calcaneal ligament is a medial ligament so it is
be avoided in this patient specifically.
much less likely to be injured.
Answer E is incorrect. Warfarin is an antico-
Answer E is incorrect. The lateral ligaments
agulant that inhibits vitamin K-dependent clot-
are more commonly injured than the medial
ting factors II, VII, IX, and X. Neither elevated
ligaments, since they are weaker. The tibiota-
uric acid levels nor cyclosporine itself predis-
High-Yield Systems

lar ligament is a medial ligament so it is much


poses patients to thrombus formation; there-
less likely to be injured.
fore antithrombotic therapy is unnecessary.
9. The correct answer is C. Osteogenesis imper-
8. The correct answer is A. The lateral ligaments
fecta, or “brittle bone disease,” is a group of
of the foot are more commonly injured than
hereditary disorders characterized by abnormal
the medial ligaments, since they are weaker.
type I collagen synthesis. In this disease a num-
The anterior talofibular ligament is the most
ber of mutations can result in either defective
common of the lateral ligaments to be injured.
synthesis or secretion of type I collagen. The
Injuries to the ligaments about the ankle usu-
result is increased bone fragility, abnormal
ally result from inversion and internal rotation
dentition, hearing loss, and a blue appearance
of the foot combined with ankle plantar flex-
to the sclera, as seen in this child.
ion. With complete disruption of the anterior
talofibular ligament, forward displacement of Answer A is incorrect. Deletion of the dys-
the talus in the ankle mortise is present. Lat- trophin gene results in Duchenne muscular
eral ankle sprains represent 16%-21% of all dystrophy, which causes accelerated muscle
sports-related traumatic lesions. The anterior breakdown, proximal muscle weakness, and
Musculoskeletal

talofibular ligament is the weakest ligament pseudohypertrophy of muscles due to fatty in-
and therefore the most frequently torn. There filtration.
is usually a predictable pattern of injury involv- Answer B is incorrect. Marfan syndrome re-
ing the anterior talofibular ligament followed sults from a defect in fibrillin, the major com-
by the calcaneofibular ligament and the poste- ponent of microfibrils found in the extracel-
rior talofibular ligament. First-degree sprain is lular matrix. Although more than 100 distinct
characterized by a partial or complete tear of mutations in this gene can result in Marfan
the anterior talofibular ligament. In second- syndrome, these patients usually have prob-
degree sprain both the anterior talofibular lems with their eyes, skeleton, and cardiovascu-
and calcaneofibular ligaments are either par- lar system. These patients display bilateral lens
tially or completely torn. Third-degree sprain subluxation, or dislocation, due to weakness
consists of injuries to the anterior talofibular, in the suspensory ligaments. These patients
calcaneofibular, and posterior talofibular liga- also manifest distinctive skeletal abnormalities.
ments. They have a slender elongated habitus, arach-
Answer B is incorrect. The calcaneofibular nodactyly, high arched palate, and hyperexten-
ligament is a lateral ligament that is injured sibility of joints. Finally, as the result of a weak-
frequently, but less frequently than the anterior ened extracellular matrix, these patients tend
talofibular ligament. to display aortic aneurysms, dilation of the aor-
tic ring resulting in aortic incompetence, and
Answer C is incorrect. The lateral ligaments
incompetent mitral and tricuspid valves.
are more commonly injured than the medial
Chapter 12: Musculoskeletal  •  Answers 323

Answer D is incorrect. Ehlers-Danlos syn- though, so their function would likely be com-
drome is characterized by defects in collagen promised for this patient.
synthesis or structure. Accordingly, patients
Answer B is incorrect. The brachioradialis is
with this disorder have collagen that lacks ten-
innervated by the radial nerve, which is more
sile strength, and patients can have hyperex-
classically damaged by a mid-humeral fracture.
tensible skin and hypermobile joints. Because
of a defect in connective tissue, patients with Answer D is incorrect. The opponens pollicis
this disorder are more susceptible to berry an- muscle is innervated by the recurrent branch
eurysms. of the median nerve, so it would not be af-
fected by an ulnar nerve injury.
Answer E is incorrect. Fractures do occur in
many instances of child abuse. However, iso- Answer E is incorrect. The palmaris longus
lated long-bone fracture is considered a low- muscle is innervated by the median nerve, so it

High-Yield Systems
specificity fracture. Fractures more suggestive would not be affected by an ulnar nerve injury.
of child abuse include metaphyseal corner Answer F is incorrect. The pronator teres
fractures and posterior rib fractures. In addi- muscle is innervated by the median nerve, so it
tion, this patient does not display other features would not be affected by an ulnar nerve injury.
such as abnormal bruising, retinal dislocations,
or retinal hemorrhages that would be more 11. The correct answer is C. This patient is suf-
consistent with this diagnosis. fering from pemphigus vulgaris, an autoim-
Answer F is incorrect. Hemophilia A is an mmune blistering disorder. This disease most
X-linked deficiency in factor VIII. These pa- commonly presents in the fourth to sixth de-
tients demonstrate prolonged partial throm- cades of life. It is characterized by fragile blis-
boplastin times, easy bruising, and massive ters over the face, axilla, trunk, and mucosa.
hemorrhage after trauma. Bleeding into joint Large lesions can jeopardize fluid balance and
spaces (hemarthroses) are the primary skeletal temperature regulation, and can be sources
manifestation. of infection; thus severe cases may be life-
threatening. The disease is caused by an auto-
10. The correct answer is C. This patient has frac- immune reaction against desmoglein 3, a com-

Musculoskeletal
tured his distal humerus, which is a common ponent of desmosomes (also called macula
way to injure the ulnar nerve. Remember, the adherens). Desmosomes are “spot-junctions”
ulnar nerve courses through the medial epi- that attach epithelial cells to one another. Im-
condyle of the humerus just below the skin, so munoglobulin is deposited in a net-like pattern
it is not well-protected at all. This vulnerability surrounding keratinocytes. This condition is
is famous with the so-called “funny bone” in- treated with corticosteroids and other immuno-
juries. Our patient’s radiograph shows a much suppressive medications.
more serious injury—a completely fractured Answer A is incorrect. Gap junctions, com-
humerus, so it should come as no surprise that prised of connexons, allow for communication
muscles innervated by the ulnar nerve would between adjacent cells, classically in cardiac
have decreased function. Of the muscles listed muscle cells. These are not involved in pem-
here, only the flexor carpi ulnaris is innervated phigus vulgaris.
by the ulnar nerve, and on exam the patient
would not be able to flex his fingers. Answer B is incorrect. Hemidesmosomes are
similar to desmosomes, but are found exclu-
Answer A is incorrect. The first and second sively between the basal layer of keratinocytes
lumbrical muscles are innervated by the ra- and the basement membrane. Autoimmune
dial nerve, which is more classically injured by disease targeting hemidesomsomes causes bul-
a mid-humeral frature. The third and fourth lous pemphigoid, a separate, milder disease
lumbricals are innervated by the ulnar nerve, characterized by tense bullae that do not easily
rupture. Bullous pemphigoid tends to occur in
324 Section II: Organ Systems  •  Answers

the elderly, and oral involvement is less com- certain human leukocyte antigen (HLA) types,
mon. Direct immunofluorescence imaging in including HLA-B27, HLA-13, and HLA-17.
this condition demonstrates a linear pattern
along the basement membrane, rather than 13. The correct answer is D. This image depicts a
the net-like pattern seen in the case above. berry aneurysm. Berry aneurysms are congeni-
tal and associated with several syndromes. In
Answer D is incorrect. Zona adherens, or in-
this patient, the combination of a berry aneu-
termediate junctions, are found just deep to
rysm, work as a contortionist (implying hyper-
the zona occludens. These junctions are not
extensible joints), and a history of easy bruising
involved in pemphigus vulgaris.
and bleeding suggests a collagen disorder, spe-
Answer E is incorrect. Zona occludens, or cifically Ehlers-Danlos syndrome (EDS). EDS
tight junctions, are located close to the surface is a group of disorders resulting from defects in
High-Yield Systems

of epithelial cells. They prevent diffusion be- collagen synthesis and processing. This patient
tween cells. These junctions are not involved likely had vascular EDS (type IV), which is as-
in pemphigus vulgaris. sociated with a defect in type III procollagen,
a precursor of the collagen found in many tis-
12. The correct answer is B. This patient has sues.
erythema nodosum, an inflammation of sub-
Answer A is incorrect. Marfan syndrome,
cutaneous fat that is often accompanied by
caused by a mutation of the fibrillin-1 gene on
fever and malaise that is described clinically
chromosome 15, is associated with long, thin
and pathologically in this vignette. The exact
extremities, loose and occasionally hyperexten-
mechanism is unknown, but it often occurs
sible joints, and aortic aneurysms. Mucocuta-
together with inflammatory-bowel diseases
neous bleeding and bruising are not frequently
(IBDs) such as Crohn disease or ulcerative
seen with Marfan syndrome; EDS is a better
colitis (UC); sarcoidosis; certain drugs (such as
answer.
oral contraceptives and sulfonamides); certain
malignant neoplasms; and certain infections Answer B is incorrect. Patients with epider-
(such as tuberculosis (TB), b-hemolytic strep- molysis bullosa have mutations in either kera-
tococci, coccidioidomycosis, histoplasmosis, tin 14 or keratin 5, two of the major keratins
Musculoskeletal

and leprosy). in basal epithelial cells, resulting in skin that


readily breaks and forms blisters with minor
Answer A is incorrect. Acne vulgaris is a dis-
trauma. Epidermolysis bullosa is not associated
order of the epidermis that has both inflam-
with hyperextensible joints or berry aneurysms.
matory and noninflammatory variants. It is as-
sociated with the bacterium Propionibacterium Answer C is incorrect. Osteogenesis im-
acnes. perfecta (OI), caused by mutations in the
pro-a1(I) chain or pro-a2(I) chain in type
Answer C is incorrect. Eczema is an inflam-
I procollagen, is characterized by multiple
matory skin disorder that is associated with
spontaneous bone fractures, retarded wound
contact allergies, asthma, ultraviolet light ex-
healing, and characteristically blue sclerae. Al-
posure, repeated physical skin rubbing, and
though caused by defective collagen synthesis,
certain drugs. It is not associated with ery-
OI is not associated with berry aneurysms.
thema nodosum.
Answer E is incorrect. A defect in type IV col-
Answer D is incorrect. Pancreatitis, which is
lagen is the underlying etiology for Alport syn-
associated with many cases of biliary tract dis-
drome. Alport syndrome is characterized by
ease and alcoholism, is not associated with ery-
nephritis with hematuria, hearing loss, and eye
thema nodosum.
disorders. Although it is commonly inherited
Answer E is incorrect. Psoriasis is a nonpru- in an X-linked pattern, autosomal-dominant
ritic inflammatory skin disorder associated with and autosomal-recessive inheritance has been
arthritis, enteropathy, spondylitic disease, and
Chapter 12: Musculoskeletal  •  Answers 325

exhibited. Alport syndrome is not associated their 20s or 30s, and 80% are positive for HLA-
with the above pathology. B27. Reactive arthritis is thought to be caused
by an autoimmune reaction to a gastrointesti-
14. The correct answer is D. A spiral fracture is nal (GI) or genitourinary infection.
one that is caused by a twisting, rotational
Answer A is incorrect. Ankylosing spondylitis
force to the bone. While child abuse may
is an inflammatory disease of the spine and
manifest with any kind of fracture, a spiral
sacroiliac joints causing stiffening of the back,
fracture should raise increased suspicion of in-
and it often is accompanied by uveitis and aor-
tentionally inflicted injury. The mechanism of
tic regurgitation. It is strongly associated with
injury is usually the twisting of the bone by an
the HLA-B27 allele.
angry adult.
Answer B is incorrect. Lyme arthritis, usually
Answer A is incorrect. A bowing fracture, also

High-Yield Systems
caused by a bite from a tick harboring the spi-
known as a bending fracture, is one in which
rochete Borrelia burgdorferi, presents with a lo-
the cortex of the diaphysis is deformed, but
cal skin rash followed by arthralgias and arthri-
without injury to the periosteum. The pediat-
tis (usually mono-articular). Its late sequelae
ric bone structure has greater compliance and
include myocardial, pericardial, and neuro-
porosity than adult bone, resulting in injuries
logic changes.
that deform or bend the bone rather than caus-
ing a line fracture. Bowing fractures are com- Answer C is incorrect. Patients with psoriatic
mon pediatric fractures. While a bowing frac- arthritis can have joint pain and conjunctivi-
ture could be caused by child abuse, it is not tis, but the diagnosis requires the presence of
the most likely of the listed choices to suggest psoriasis, which is characterized by nonpruritic
that etiology. scaly or silvery erythematous plaques with well-
defined borders.
Answer B is incorrect. A buckle fracture, also
known as a torus fracture, is caused by com- Answer E is incorrect. SLE is diagnosed by
pression, resulting in a bulging or buckling of the presence of four of the 11 symptoms sum-
the periosteum, rather than a complete frac- marized by the mnemonic “BRAIN SOAP,
ture line. Buckle fractures are one of the more MD:” Blood dyscrasias (such as hemolytic

Musculoskeletal
common pediatric fractures and are frequently anemia or thrombocytopenia), Renal disorder,
caused by accidents. While a buckle fracture Arthritis, Immunologic disorder (such as anti-
could be caused by child abuse, it is not the DNA antibody and anti-Smith antibody), Neu-
most likely of the listed choices to suggest that rologic disorder, Serositis (such as pleuritis or
etiology. pericarditis), Oral ulcers, Antinuclear antibody
(elevated titers in the absence of drugs associ-
Answer C is incorrect. A greenstick fracture is
ated with drug-induced lupus syndrome), Pho-
an incomplete cortical fracture in which corti-
tosensitivity, Malar rash, and Discoid rash.
cal disruption and periosteal tearing occurs on
the convex aspect of the bone, but the concave
16. The correct answer is D. The patient has a
aspect has an intact periosteum. Greenstick
classic case of chronic gout, with intermittent
fractures are one of the more common pediat-
attacks eventually giving rise to disfiguring to-
ric fractures and are frequently caused by ac-
phi. Gouty arthritis is a common manifesta-
cidents. While a greenstick fracture could by
tion of hyperuricemia, and tophi form as a re-
caused by child abuse, it is not the most likely
sult of the accumulation of monosodium urate
of the listed choices to suggest that etiology.
crystals surrounded by reactive fibroblasts and
chronic inflammatory cells in the joints and
15. The correct answer is D. Conjunctivitis in a
soft tissues. Common extra-articular sites of
patient who has had both urethritis (or cervici-
tophus formation include the Achilles tendon
tis) and arthritis for at least one month is sug-
and the helix of the external ear. Aspiration of
gestive of reactive arthritis. Most patients are in
the tophus usually reveals the presence of neg-
326 Section II: Organ Systems  •  Answers

atively birefringent needle-shaped crystals seen to treat hypertension and may delay progres-
in the question image, which are characteris- sion of diabetic nephropathy. It has a moder-
tic of gout. The best therapy of chronic gouty ately potent uricosuric effect and thus may be
arthritis aims to lower the levels or uric acid. used in treatment of chronic gout, but not as
Allopurinol blocks xanthine oxidase and thus first-line therapy.
reduces the generation of uric acid. Therefore
it should be used in patients who overproduce 17. The correct answer is D. The supraspinatus
uric acid and in patients at risk of tumor lysis muscle is the most frequently injured rotator
syndrome to prevent renal toxicity during ther- cuff muscle. Patients often report pain anteri-
apy for malignancies. It is the most effective orly and superiorly to the glenohumeral joint
urate-lowering agent. However, alcohol can during abduction. The primary motion of the
interfere with the effectiveness of allopurinol. supraspinatus is the first 15 degrees of abduc-
High-Yield Systems

Probenecid increases excretion of uric acid by tion of the arm, at which point the deltoid
the kidneys and can therefore also be used in muscle continues the motion of abduction.
treatment of chronic gout. Therefore, as long as a patient can assist the
injured rotator cuff at the beginning of abduc-
Answer A is incorrect. Colchicine binds tubu-
tion, he or she will be able to continue the mo-
lin, thereby inhibiting microtubule polymer-
tion unassisted thereafter. This muscle and its
ization, which blocks mitosis as well as neutro-
tendon can be torn traumatically, but it is most
phil migration. Colchicine is now considered
often injured in a more insidious manner be-
to be second-line treatment of acute gout be-
cause of repetitive movement, as is the likely
cause of its narrow therapeutic window and
cause in this case.
risk of toxicity. Colchicine causes GI upset and
diarrhea in 80% of people. Answer A is incorrect. The deltoid muscle
takes over for the supraspinatus muscle in ab-
Answer B is incorrect. NSAIDs block the en-
duction of the arm after about 15 degrees of
zymes cyclooxygenase-1 and/or -2, thereby
abduction. The deltoid muscle is not injured
blocking production of inflammatory media-
in this case, as demonstrated by the patient’s
tors. NSAIDs have been used successfully in
ability to initiate the lift motion by lifting the
Musculoskeletal

treatment of acute gout. Most NSAIDs can


arm on the injured side with his other arm and
be used in treatment of gout but care must be
then continue the motion unassisted.
taken with giving indomethacin to elderly pa-
tients (central nervous system adverse effects). Answer B is incorrect. The action of the infra-
An agent with a quick onset of action is de- spinatus muscle is to laterally rotate the arm.
sired, but aspirin should not be used because Answer C is incorrect. The subscapularis is re-
it can alter uric acid levels and potentially pro- sponsible for medial rotation and adduction of
long and intensify an acute attack. the arm.
Answer C is incorrect. Corticosteroids are Answer E is incorrect. The teres minor mus-
used to treat episodes of acute gout in patients cle assists in lateral rotation and adduction of
who cannot use NSAIDs. Steroids have spe- the arm.
cific and nonspecific anti-inflammatory func-
tions. Steroids inhibit release of phospholipase 18. The correct answer is C. The subscapularis
A2, an enzyme responsible for the formation muscle is one of the muscles that comprise the
of prostaglandins, leukotrienes, and other de- rotator cuff. It medially rotates and adducts the
rivatives of the arachidonic acid pathway. Intra- arm. It is the only rotator cuff muscle that acts
articular corticosteroid injections may be par- to medially rotate the arm.
ticularly useful in patients with mono-articular
gout flare-ups. Answer A is incorrect. The deltoid muscle is
not a muscle that comprises the rotator cuff
Answer E is incorrect. Losartan, an inhibitor
of angiotensin II receptor, is used traditionally
Chapter 12: Musculoskeletal  •  Answers 327

muscles. It is primarily involved in abduction enzyme system, and may cause tears and urine
of the arm at the shoulder. to turn orange in color, but it is not known to
cause a lupus-like syndrome.
Answer B is incorrect. The infraspinatus is an
adductor of the arm and a lateral rotator of the Answer D is incorrect. Inhibitors of peptido-
glenohumoral joint. glycan synthesis include penicillins, cepha-
losporins, monobactams, carbapenems, and
Answer D is incorrect. The supraspinatus is
vancomycin. These antibiotics are not used as
the most often injured rotator cuff muscle. It
prophylaxis against TB, and are not associated
is typically described as being the initiator of
with a lupus-like syndrome.
abduction for the first 15° of the arc. The del-
toid muscle becomes the main propagator for Answer E is incorrect. Hydralazine causes
abducting the arm beyond 15 degrees. vasodilation through increased production of

High-Yield Systems
cGMP and may cause a lupus-like syndrome.
Answer E is incorrect. The teres minor is a
However, it would not be used to treat TB.
narrow, elongated muscle of the rotator cuff
Other drugs that also may cause a lupus-like
that works to adduct and laterally rotate the
syndrome are procainamide and phenytoin.
arm.
20. The correct answer is B. Four muscles are in-
19. The correct answer is C. This woman has a
volved in jaw movement: the lateral pterygoid,
positive purified protein derivative test, signify-
masseter, temporalis, and medial pterygoid.
ing possible exposure to TB and latent disease.
The lateral pterygoid opens the jaw, and the
To decrease her risk of developing an active
other three close the jaw. One can remember
TB infection, a 6- to 12-month regimen of iso-
this with the mnemonic “M’s Munch” and
niazid is indicated. Isoniazid causes a decrease
“Lateral Lowers.” All four muscles are inner-
in the synthesis of mycolic acids that make up
vated by the mandibular branch of the trigemi-
the unique cell envelope of mycobacterium
nal nerve (V3).
TB. One of the possible adverse effects of iso-
niazid is a lupus-like syndrome. Drug-induced Answer A is incorrect. Cranial nerve (CN) V2,
lupus is characterized by an abrupt onset of the maxillary branch of the trigeminal nerve,

Musculoskeletal
symptoms, which may include fever, arthritis, relays sensory information from the middle
pleural pericarditis, and rash, along with the portion of the face in the palpebral fissure to
development of anti-nuclear antibodies. Other the mouth. V3 is the only branch of CN V
medications that may cause a similar lupus- with a motor component.
like syndrome include procainamide, hydrala-
Answer C is incorrect. CN VII innervates the
zine, minocycline, and penicillamine.
muscles of facial expression. A lesion to this
Answer A is incorrect. Dapsone is used in the nerve would result in Bell palsy, which would
treatment of Hansen disease caused by Myco- present with ipsilateral facial droop. The pa-
bacterium leprae. It disrupts the synthesis of tient has a symmetric smile, which suggests
folate in a method similar to that of sulfon- that his CN VII is intact.
amides. Other inhibitors of folate metabolism
Answer D is incorrect. CN X (vagus) inner-
such as trimethoprim-sulfamethoxazole are
vates most muscles of the soft palate (except
used as prophylactic treatment against pneu-
tensor veli palatini), pharynx (except stylopha-
mocystis pneumonia in immunocompromised
ryngeus), and larynx. Patients with a lesion to
patients. Neither of these medications causes a
CN X would have difficulty swallowing (dys-
lupus-like syndrome.
phagia). This patient has symmetric palate
Answer B is incorrect. Rifampin is used to elevation, suggesting that his CN X is intact.
treat TB through suppression of RNA synthesis Damage would lead to asymmetric palate el-
by inhibiting bacterial DNA-dependent RNA evation with the uvula pointing away from the
polymerase. It induces the cytochrome P450 lesion because the damaged side is lower.
328 Section II: Organ Systems  •  Answers

Answer E is incorrect. CN XII (hypoglossal) Answer D is incorrect. Hypothyroidism can


innervates the hypoglossus, genioglossus, stylo- present with generalized fatigue and musculo-
glossus, and all intrinsic muscles of the tongue. skeletal weakness, but is not routinely associ-
The only tongue muscle not innervated by CN ated with a mediastinal mass. Measurements of
XII is the palatoglossus, which is innervated by the thyroid-stimulating hormone (TSH) level
the vagus nerve (X). Lesions to CN XII would are useful in making the diagnosis in suspected
result in difficulty eating as well as slurred cases.
speech (dysarthria). This patient’s tongue is
Answer E is incorrect. Neuroblastoma can
midline, which suggests that his CN XII is in-
present with a mediastinal mass and constitu-
tact. Damage would lead to the tongue point-
tional symptoms, although it is typically a pe-
ing to the side of the lesion when protruded.
diatric malignancy. Testing of urine catechol-
amine levels is useful in making the diagnosis
High-Yield Systems

21. The correct answer is A. This patient has


in suspected cases.
symptoms and signs consistent with a diagnosis
of myasthenia gravis (MG). MG is an autoim-
22. The correct answer is C. This patient has
mune disorder characterized by the presence
acute osteomyelitis and a history of sickle cell
of acetylcholine receptor antibodies, which
disease as evidenced by her multiple hospital-
cause impaired signal transmission at the neu-
izations (likely for episodes of painful crises)
romuscular junction. Patients typically pre­sent
and medication (hydroxyurea reduces the inci-
with characteristic ocular (ptosis, diplopia)
dence of painful crises in sickle cell disease by
and musculoskeletal (facial muscle weakness,
increasing the amount of fetal hemoglobin).
proximal muscle weakness) symptoms, which
Osteomyelitis is an infection of the bone tis-
are classically worsened by repetitive use of the
sue. It is common in young children and usu-
involved muscles (which is why symptoms are
ally results from the hematogenous spread of
worse by the end of the day). Thymomas (evi-
organisms from another site of infection (up-
denced by the widened superior mediastinum
per respiratory infection in this case). Salmo-
in the image) occur in approximately 10% of
nella species are the most common organisms
patients with MG. The diagnosis of MG can
responsible for osteomyelitis in patients with
Musculoskeletal

be confirmed by measuring acetylcholine re-


sickle cell disease.
ceptor antibodies and performing electromy-
ography (EMG) and/or a Tensilon (edropho- Answer A is incorrect. Escherichia coli causes
nium) test. Treatment generally consists of osteomyelitis in infants, but Salmonella species
an acetylcholinesterase inhibitor such as pyr- are the most common organisms responsible
idostigmine. for osteomyelitis in patients with sickle cell dis-
ease.
Answer B is incorrect. Lymphoma can pre­
sent with a mediastinal mass and constitutional Answer B is incorrect. Pseudomonas aerugi-
symptoms, but is not routinely associated with nosa typically causes infections in immuno-
musculoskeletal weakness. A complete blood compromised hosts and patients with cystic
cell count is useful in making the diagnosis in fibrosis.
suspected cases. Answer D is incorrect. Staphylococcus aureus
Answer C is incorrect. A parathyroid gland is the most common cause of osteomyelitis
tumor can infrequently present with a medi- in the general population, but not in patients
astinal mass and commonly with generalized with sickle cell disease.
weakness secondary to high serum calcium Answer E is incorrect. Streptococcus pyogenes
levels. Measurements of PTH level are useful causes osteomyelitis in children, but Salmo-
in making the diagnosis in suspected cases. nella species are the most common organisms
responsible for osteomyelitis in patients with
sickle cell disease.
Chapter 12: Musculoskeletal  •  Answers 329

23. The correct answer is C. This child has Answer C is incorrect. Osteomyelitis is an
achondroplasia, which is the most common infection in the bone. It presents most com-
inherited form of dwarfism. This autosomal monly with tenderness, warmth, swelling, and
dominant disease results in a disturbance of more acute pain, rather than joint narrowing.
endochondral bone formation due to a muta- The pain typically is present with and without
tion in the fibroblast growth factor receptor-3 movement.
(FGF-3 gene). This change results in abnor-
Answer D is incorrect. Pseudogout causes
mal growth plates and impaired cartilage mat-
symptoms that mimic those of gout, but is
uration.
caused by the precipitation of calcium py-
Answer A is incorrect. Osteopetrosis is a dis- rophosphate crystals within the joint space.
ease characterized by reduced osteoclast re- These crystals are positively birefringent,
sorption. This condition usually manifests as which means they are blue under polarized

High-Yield Systems
repeated skeletal fractures early in life. light. This disease classically affects large
joints, most commonly the knee, in men over
Answer B is incorrect. Osteogenesis imper-
50 years old. It is not associated with osteo-
fecta involves a collagen I deficiency leading
phytes or sclerosis with narrowing.
to bone fragility, among other findings. It does
not explain the findings in this child. Answer E is incorrect. Rheumatoid arthritis is
an autoimmune arthritis caused by inflamma-
Answer D is incorrect. Mucopolysaccharido-
tory destruction of synovial joints. It is associ-
ses are lysosomal storage diseases caused by
ated with pain that is worst in the morning,
specific enzyme deficiencies. These can pre­
improving with use, and classically affects the
sent in a variety of ways with chest wall abnor-
proximal interphalangeal joints. It is not associ-
malities and malformed bones, but not as the
ated with osteophytes or sclerosis with narrow-
findings seen in this child.
ing.
24. The correct answer is B. Osteoarthritis is a
25. The correct answer is D. This patient has
disease of wear and tear leading to destruction
Sjögren syndrome. The vast majority of pa-
of articular cartilage, subchondral bone forma-
tients with this syndrome are women between

Musculoskeletal
tion, osteophytes, sclerosis, and other degen-
the ages of 35 and 45, and the disease is char-
erative changes. It is common and progressive,
acterized by dry eyes (keratoconjunctivitis
and becomes more so with age. It classically
sicca), dry mouth (xerostomia), and one other
presents in weight-bearing joints as pain after
connective tissue or autoimmune disease (such
use, improving with rest. It commonly affects
as rheumatoid arthritis). The eye and mouth
the distal interphalangeal joints as well. Com-
dryness is from autoimmune destruction of the
mon imaging findings include narrowing of
lacrimal and salivary glands.
the joint space, sclerosis, and the presence of
osteophytes. The image reveals Heberden’s Answer A is incorrect. CREST syndrome is
nodes, representing bony enlargement of the a variant of scleroderma (progressive systemic
distal interphalangeal (DIP) joints. sclerosis), which is a disease characterized by
extensive fibrosis throughout the body (most
Answer A is incorrect. Gout is a painful swell-
notably of the skin). CREST stands for Calci-
ing of a joint, most commonly the metatarso-
nosis, Raynaud phenomenon, Esophageal dys-
phalangeal joint, caused by precipitation of
function, Sclerodactyly, and Telangiectasia.
monosodium urate crystals. It is diagnosed by
viewing the crystals in the joint’s synovial fluid, Answer B is incorrect. Cystic fibrosis also
which are negatively birefringent. It is not as- causes dysfunction of the exocrine glands
sociated with osteophytes or sclerosis with nar- (such as the lacrimal and salivary glands), but
rowing. it is due to a mutation in the cystic fibrosis
transmembrane conductance regulator gene
on chromosome 7. Individuals with cystic fi-
330 Section II: Organ Systems  •  Answers

brosis also tend to have pulmonary and pancre- years old. Most of these are between the ages
atic dysfunction. of 10 and 15 years. Males are at slightly greater
risk than females, and a great proportion of
Answer C is incorrect. When only the first two
patients are white. Plain film x-ray classically
criteria are present (dry eyes and dry mouth),
reveals a lytic lesion with “onion skinning” of
the disorder is called sicca syndrome.
the periosteum. This reactive process occurs
Answer E is incorrect. Sjögren-Larsson is an as the tumor arises out of the medullary cavity
autosomal-recessive syndrome characterized and new layers of bone are deposited around
by congenital ichthyosis (dry and scaly, fishlike it by the periosteum (hyperlucent layers seen
skin) and associated with mental retardation in the radiograph). Microscopic analysis of
and spastic paraplegia; it is caused by a muta- biopsy specimens reveals sheets of uniform,
tion in the aldehyde dehydrogenase gene on small, round cells that are slightly larger than
High-Yield Systems

chromosome 17p. lymphocytes. Rosette formations may be seen


as cells arrange themselves around a central fi-
26. The correct answer is B. Anterior cruciate brous space. Otherwise, there is very little stro-
ligament (ACL) injuries are common injuries mal space and no normal bony matrix mate-
in sports and frequently occur without contact rial.
when people change direction on a planted
foot. ACL tears are commonly associated with Answer B is incorrect. Giant cell tumors dif-
other injuries and occur in isolation <10% of fer in their most common presentation, peak-
the time. The most common association is ing in incidence between the ages of 20 and
with the lateral meniscus and medial collat- 40 years, and arising from the epiphyseal end
eral ligament (MCL), which together with the of long bones. Plain film x-ray classically shows
ACL are referred to as the “unhappy triad” of a “soap bubble” appearance, and histology
knee injury. shows spindle-shaped cells with multinucle-
ated giant cells interspersed between them.
Answer A is incorrect. The valgus force of the
injury described in the question is unlikely to Answer C is incorrect. Osteochondroma, the
put tension on the lateral collateral ligament, most common benign tumor of bone, is a car-
Musculoskeletal

so it is less likely to be injured. tilaginous cap attached to the skeleton by a


mature bony stalk. Also known as exostosis, it
Answer C is incorrect. Although it is possible very rarely converts to a malignant neoplastic
to injure the medial meniscus, a medial menis- process.
cus injury is less likely to be associated with a
concurrent MCL and ACL injury. Answer D is incorrect. Ewing sarcoma may
mimic infection with systemic findings such
Answer D is incorrect. The patellar ligament as fever, elevated erythrocyte sedimentation
is a thick strong ligament that is unlikely to be rate (ESR), anemia, and leukocytosis, as well
injured in this type of mechanism. as local inflammation of the surrounding soft
Answer E is incorrect. The posterior cruciate tissue. However, the diagnostic work-up of this
ligament is a strong ligament, but it can be in- patient is not consistent with infection.
jured, especially in association with collateral Answer E is incorrect. Osteosarcoma is the
ligament tears; however, it is less likely to be most common primary malignant tumor of
injured via this mechanism. bone. Like Ewing, it peaks in occurrence be-
tween the ages of 10 and 20 years, and arises
27. The correct answer is A. The most likely diag- most often in the metaphyseal region of long
nosis is Ewing sarcoma. Made up of anaplas- bones. However, the key difference is in the re-
tic, small blue cells, Ewing sarcoma is classi- sults of the imaging and biopsy studies. Clas-
fied as one of the primitive neuroectodermal sically, osteosarcomas show Codman triangle
tumors. This patient’s presentation is typical in or a sunburst pattern on plain film x-ray. Histo-
that 80% of tumors are found in patients <20 logically, cellular patterning is highly variable,
Chapter 12: Musculoskeletal  •  Answers 331

with a number of subtypes. The most charac- long thoracic nerve is damaged, the scapula
teristic feature, however, is the formation of moves away from the thoracic cage, resulting
bony matrix material by the tumor cells, which in what is referred to as winging of the scapula.
does not occur in Ewing sarcoma. Long thoracic nerve injury is an occasional
complication of mastectomy. The long tho-
28. The correct answer is A. This patient has racic nerve originates from the brachial plexus,
symptoms and signs consistent with vitamin specifically from C5, C6, and C7.
D-deficient rickets, which results from the de-
Answer A is incorrect. C3, C4, and C5 are the
creased or absent mineralization of osteoid
origins of the phrenic nerve, which innervates
(bone matrix) secondary to decreased serum
the diaphragm.
calcium and/or phosphorous levels. Vitamin
D normally promotes absorption of calcium Answer B is incorrect. C5 and C6 join to-

High-Yield Systems
and phosphorous from the GI tract. In vita- gether to form the upper trunk, and do par-
min D-deficient patients, areas of bone growth ticipate in the formation of the long thoracic
(eg, wrists, ankles, costochondral junctions) nerve, but C7 is also involved.
contain patches of unmineralized, soft osteoid
Answer D is incorrect. The long thoracic
that give rise to the classically reported signs,
nerve does not originate from C7 and C8.
including widened wrists and/or ankles and
enlarged costochondral junctions (rachitic ro- Answer E is incorrect. C8 and T1 join to-
sary). In cases of severe vitamin D deficiency, gether to form the lower trunk of the brachial
laboratory studies typically demonstrate de- plexus, while C7 is a part of the middle trunk.
creased serum calcium, decreased serum Neither C8 nor T1 contributes to the long tho-
phosphorous, decreased serum 1,25-dihydroxy- racic nerve.
cholecalciferol, increased serum alkaline phos-
phatase, and increased serum intact PTH lev- 30. The correct answer is E. Rheumatoid factor
els. would most likely be positive in this woman,
who is suffering from RA. Eighty percent of
Answer B is incorrect. A decreased, not in- patients with RA have positive rheumatoid
creased, serum 1,25-dihydroxycholecalciferol factor (anti-IgG antibody). This autoimmune

Musculoskeletal
level is characteristic of vitamin D-deficient condition causes a marked influx of inflam-
rickets. matory cells into the joint synovium, as seen
Answer C is incorrect. A decreased, not in- here, resulting in destructive change, pannus
creased, serum 25-hydroxycholecalciferol level formation, and eventually joint deformity. The
is characteristic of vitamin D-deficient rickets. disease is more common in women, and classi-
cally symmetrically affects the proximal inter-
Answer D is incorrect. A decreased, not in-
phalangeal joints, as described here.
creased, serum calcium level (due to de-
creased GI absorption) is characteristic of vita- Answer A is incorrect. Anti-centromere anti-
min D-deficient rickets. body is associated with the CREST variant of
scleroderma (progressive systemic sclerosis).
Answer E is incorrect. A decreased, not in-
In this disease patients suffer from Calcinosis,
creased, serum phosphorous level (due to de-
Raynaud phenomenon, Esophageal dysmotil-
creased GI absorption) is characteristic of vita-
ity, Sclerodactyly, and Telangiectasia. Arthritis
min D-deficient rickets.
is not associated with this syndrome.
29. The correct answer is C. This woman has sus- Answer B is incorrect. Anti-nuclear antibody
tained an injury to her long thoracic nerve, is associated with SLE, an autoimmune disease
which innervates the serratus anterior muscle, with a wide variety of symptoms including fe-
as a complication of her surgery. The function ver, rash, joint pain, and photosensitivity. The
of the serratus anterior muscle is to anchor the joint pain in lupus is typically transient, asym-
scapula against the thoracic cage. When the metrical, and non-deforming.
332 Section II: Organ Systems  •  Answers

Answer C is incorrect. Borrelia burgdorferi is page gait, as described in the vignette. Com-
the gram negative bacteria that causes Lyme mon peroneal nerve injury can occur with
disease. While the third stage of Lyme disease fracture to the fibular neck and patients will
can manifest as migratory polyarthritis, this present with an inability to dorsiflex or evert, as
patient has no other associated signs or symp- well as a foot drop gait.
toms.
Answer B is incorrect. The femoral nerve
Answer D is incorrect. HLA-B27 is strongly courses in the anterior thigh and branches
associated with joint disease without rheuma- into the tibial nerve and the common peroneal
toid factor, such as ankylosing spondylitis. This nerve. It is not involved in fractures to the fibu-
condition most commonly affects men and lar neck.
causes severe stiffening of the spine and sa­
Answer C is incorrect. The L4 nerve root de-
croiliac joints, as well as uveitis. The hands are
High-Yield Systems

fines the sensory dermatome to the lateral as-


not typically involved.
pect of the foot and leg. Patients also can have
motor weakness in the L4 myotome. However,
31. The correct answer is C. Poliovirus infects the
the combination of impaired dorsiflexion and
Peyer patches of the intestine and the motor
eversion is most specific for damage to a pe-
neurons. It is passed by the fecal-oral route and
ripheral nerve (common peroneal).
can present as a spectrum of severity, depend-
ing on the age of the patient. Younger children Answer D is incorrect. Obturator nerve does
and infants often have a nonclinical infection not travel near the fibula. It is not involved in
or mild fever with diarrhea. Older children injuries to the fibula.
who have not previously been infected can de-
Answer E is incorrect. The tibial nerve is un-
velop meningitic signs. The most severe com-
likely to be injured because it is well protected
plications are respiratory muscle failure, para-
in the popliteal fossa, although it can be in-
plegia, and quadriplegia.
jured by deep lacerations.
Answer A is incorrect. The neuron loss that
occurs affects the motor neurons in the ante- 33. The correct answer is B. This patient has os-
rior horns, not the posterior horns. teomyelitis, most commonly caused by Pseu-
Musculoskeletal

domonas aeruginosa in intravenous drug users.


Answer B is incorrect. Because this is a lower
The image shows a bone abscess with polymor-
motor neuron disease, reflexes are lost in the
phonuclear leukocytes and foci of degraded
affected limbs, and the limbs atrophy.
bone collagen. Treatment would require sev-
Answer D is incorrect. The poliovirus infects eral weeks of antipseudomonal antibiotics
the motor neurons, leaving sensation spared. such as ticarcillin or piperacillin.
Answer E is incorrect. Poliovirus does not af- Answer A is incorrect. Chronic corticosteroid
fect short-term memory or cortical function- use can lead to immunosuppression and osteo-
ing. It can affect the cranial nerves, however. porosis, but does not predispose one to pseudo-
monal osteomyelitis..
32. The correct answer is A. The common per­
Answer C is incorrect. Patients with miliary
oneal nerve courses around the neck of the
Mycobacterium tuberculosis infection are at
fibula, making it vulnerable to damage by a
risk for vertebral osteomyelitis, known as Pott
fracture at the fibular neck. It is the most fre-
disease. It is uncommon for tubercular osteo-
quently lesioned nerve in the lower limb. Pa-
myelitis to affect the humerus.
tients experience foot drop, which results from
a loss of dorsiflexion at the ankle, and a loss of Answer D is incorrect. Multiple sexual part-
eversion. Patients will have pain and paresthe- ners would put the patient at increased risk
sia in the lateral leg and dorsum of the foot. of Neisseria gonorrhoeae infection, which can
Patients with foot drop may also have a step- cause osteomyelitis. More commonly, the pa-
Chapter 12: Musculoskeletal  •  Answers 333

tient would have urethritis, epidydimitis, or Unlike in the case of anthrax, the skin ulcers
perhaps Fitz-Hugh-Curtis syndrome, which seen in Y pestis infection are painful. Fur-
is the formation of postinfection adhesions thermore, prolonged infection and spread of
on the liver capsule, leading to chronic upper Y pestis can lead to disseminated intravascular
right quadrant pain. coagulation.
Answer E is incorrect. Patients with sickle cell Answer A is incorrect. Babesia microti is trans-
anemia are at risk for osteomyelitis due to Sal- mitted to humans through the bite of a tick. It
monella or Staphylococcus, but not to pseudo- causes a sickness similar to malaria with symp-
monal osteomyelitis. toms of fever and anemia.
Answer B is incorrect. Bacillus anthracis can
34. The correct answer is B. This patient suffers
cause cutaneous anthrax, which is character-
from lateral epicondylitis, better known as ten-

High-Yield Systems
ized by a painless ulcer with a black scab.
nis elbow. This condition stems from overuse
of the superficial extensor muscles of the fore- Answer C is incorrect. Leishmania donovani
arm and wrist, including the extensor carpi is transmitted through the bite of a sandfly and
radialis muscle. This muscle also inserts at the causes visceral leishmaniasis. This disease is
lateral epicondyle. The repeated forced exten- characterized by abdominal pain and disten-
sion and flexion of the forearm at the elbow tion, anorexia, weight loss, and fever.
causes an inflammation of the common exten-
Answer D is incorrect. A person infected with
sor tendon. Patients exhibit pain over the lat-
Trichinella spiralis presents with fever, perior-
eral epicondyle that may radiate down the pos-
bital and facial edema, myalgia, and eosino-
terior aspect of the forearm. Treatment options
philia.
include rehabilitation, which may include ex-
ercises, motion analysis, and straps or braces; 36. The correct answer is C. A sarcomere, the ba-
medication; open surgery; and arthroscopic sic functional unit of skeletal muscle, extends
(minimally invasive) surgery. from one Z line to an adjacent Z line. Upon
Answer A is incorrect. The biceps muscle muscle contraction, the power stroke results
functions to supinate and flex the forearm. from actin sliding on myosin, causing Z lines

Musculoskeletal
to move closer together. The absolute length
Answer C is incorrect. The extensor carpi ul-
of the thin (actin) and thick filaments (myosin)
naris muscle functions to extend and adduct
in a myofibril does not change during contrac-
the hand at the wrist but does not extend the
tion, but rather the overlap of the filaments in-
forearm.
creases. Thus, the A band, which is the length
Answer D is incorrect. The flexor carpi ul- of myosin, does not change upon contraction.
naris muscle functions to flex and abduct the The H band and I band are the two areas
hand at the wrist. where there is no overlap of actin and myosin;
both bands will then decrease when the over-
Answer E is incorrect. The pronator teres
lap increases with contraction.
muscle functions to pronate and flex the fore-
arm. Answer A is incorrect. The A band, which is
composed of myosin, does not change with
35. The correct answer is E. Yersinia pestis is contraction. H shortens and I shortens.
the organism responsible for the plague, also
Answer B is incorrect. This is what will occur
known as the Black Death. The bacterium can
upon relaxation. The A band will again remain
be spread to humans by fleas from rodents, es-
the same length, but as myosin and actin de-
pecially prairie dogs in the United States. The
crease their overlap, both the H and I bands
disease develops after two-eight days of incu-
will increase in length.
bation and is characterized by the presence of
exquisitely tender lymph nodes called buboes.
334 Section II: Organ Systems  •  Answers

Answer D is incorrect. The A band, which is ties of the interphalangeal joints. Generalized
composed of myosin, does not change with swelling and a sausage-like appearance of the
contraction. H shortens and I shortens. digits are not commonly seen. Patients with
rheumatoid arthritis can also have cutaneous
Answer E is incorrect. The A band, which is
manifestations, the most characteristic being
composed of myosin, does not change with
rheumatoid nodules that are usually located
contraction. H shortens and I shortens.
over bony prominences. About 80% of patients
37. The correct answer is B. This patient has with rheumatoid arthritis are positive for rheu-
psoriatic arthritis, which presents with psoria- matoid factor.
sis (nonpruritic scaly or silvery erythematous Answer E is incorrect. Pseudogout is a rheu-
plaques with well-defined borders) and joint matologic disorder characterized by calcium
symptoms that are of acute onset in one-third pyrophosphate dihydrate deposition in connec-
High-Yield Systems

of patients. Psoriatic arthritis is an inflamma- tive tissues. The presentation of pseudogout


tory arthritis and signs of inflammation, such is very similar to that of gout, except the joint
as an elevated ESR, are commonly seen in fluid contains calcium pyrophosphate dihy-
patients with this condition. More than 50% drate crystals, which are weakly bifringent and
of patients have an asymmetric distribution rhomboidal in shape.
of joint swelling in the distal interphalangeal
joints of the hands and feet. Some patients 38. The correct answer is B. The infection caused
may develop a sausage-like finger from inflam- by Actinomyces israelii typically presents as a
mation of the digital tendon sheaths. chronic, slowly progressing mass that eventu-
ally evolves into a draining sinus tract. Char-
Answer A is incorrect. Celiac disease is a
acteristic sulfur granules are seen in the thick
malabsorption syndrome in which patients
yellow exudate. Penicillin G is the first-line
produce autoantibodies to gluten (gliadin).
treatment.
Dermatitis herpetiformis is a skin disorder
commonly seen in patients with celiac dis- Answer A is incorrect. This describes the
ease; it causes pruritic papules and vesicles, mechanism for amphotericin B, which is used
not scaly plaques. Patients can present with to treat systemic mycoses. While Actinomyces
Musculoskeletal

arthralgias; however, the joints do not usually form long, branching filaments that resemble
become swollen. fungi, they are bacteria.
Answer C is incorrect. Gout is characterized Answer C is incorrect. Sulfonamides act by in-
by a raised serum uric acid level that leads to hibiting dihydropteroate synthetase, preventing
uric acid deposition in tissues, particularly the nucleotide synthesis. They are first-line medi-
joint spaces. Urate crystal deposition in a joint cations for Nocardia infection but not for Acti-
can cause an inflammatory reaction, leading to nomyces.
joint pain and inflammation. Diagnosis of gout
Answer D is incorrect. Many antibiotics act
is generally based on clinical symptoms and
by blocking protein synthesis including macro-
the presence of urate crystals (which are neg-
lides, aminoglycosides, and tetracyclines. How-
atively bifringent and needle-shaped) in the
ever, none of these are first-line treatments for
joint fluid. The presence of arthritic symptoms
Actinomyces infection, which best explains this
and the absence of elevated uric acid levels
patient’s presentation.
make the diagnosis of gouty arthritis unlikely.
Answer E is incorrect. This describes the
Answer D is incorrect. Rheumatoid arthritis
mechanism for azoles such as fluconazole
is another type of inflammatory arthritis; how-
and ketoconazole, which are used to treat fun-
ever, joint involvement is generally bilateral
gal infections. While Actinomyces form long,
and symmetric. Joint involvement of the hand
branching filaments that resemble fungi, they
often leads to ulnar deviation of the wrist, as
are bacteria.
well as swan-neck and boutonniere deformi-
Chapter 12: Musculoskeletal  •  Answers 335

39. The correct answer is A. The genitofemoral against desmoglein-3, a protein involved in
nerve arises from the L1 and L2 nerve roots. In cell-cell adhesion within the other layers of the
male subjects its genital branch travels through epidermis, not within the stratum corneum.
the superficial inguinal ring along with the
Answer B is incorrect. B is the dermoepider-
spermatic cord, and supplies the cremaster and
mal junction. The autoantibodies that mediate
the scrotal skin. Severing the genitofemoral
this disease are directed against a protein ex-
nerve during a hernia repair leads to numbness
pressed in the epidermis.
of the scrotum and/or inner thigh. The genito-
femoral nerve and the ilioinguinal nerve have Answer D is incorrect. D is the dermis. The
overlapping territory. Thus, severing one only autoantibodies that mediate this disease are
leads to transient anesthesia. directed against a protein expressed in the epi-
dermis.
Answer B is incorrect. The lateral femoral cu-

High-Yield Systems
taneous nerve originates from the L2-L3 roots Answer E is incorrect. E is the hypodermis.
of the lumbar plexus, passes deep to the ingui- The autoantibodies that mediate this disease
nal ligament, and innervates the skin on the are directed against a protein expressed in the
lateral aspect of the thigh. epidermis.
Answer C is incorrect. The posterior femo- 41. The correct answer is D. Polyarteritis nodosa
ral cutaneous nerve originates from the S1-S3 (PAN) is a vasculitis (ie, inflammation of a
roots of the sacral plexus, passes through the blood vessel) characterized by inflammation
greater sciatic foramen, and innervates the skin affecting small to medium-sized arteries, par-
overlying the buttock, the posterior aspect of ticularly the renal, cardiac, and GI-tract ves-
the thigh, and the popliteal fossa. sels (usually not the pulmonary vasculature).
Answer D is incorrect. Nerve roots S2-S4 are As many as 30% of patients have had prior
associated with the pudendal nerve, which in- hepatitis-B infections.
nervates the external genitals. Injury to this Answer A is incorrect. Buerger disease, also
nerve would be associated with bowel or blad- known as thromboangiitis obliterans, is a vas-
der incontinence and possible anesthesia in culitis that mostly affects arteries and veins of

Musculoskeletal
the perineum. This nerve is injured most com- the extremities. As such, patients often have in-
monly during childbirth or saddle injury. The termittent claudication and Raynaud phenom-
pudendal nerve is not near the inguinal liga- enon. The majority of patients are men who
ment are heavy smokers and show hypersensitivity to
tobacco injected into the skin.
40. The correct answer is C. C is the epidermis.
Pemphigus vulgaris is an autoimmune dis- Answer B is incorrect. Giant cell (temporal)
order in which pathogenic antibodies are di- arteritis is a type of vasculitis that affects the
rected against a cell-cell adhesion protein, arteries of the head, especially, of course, the
desmoglein-3, which is expressed by the ke- temporal arteries. The highlights of this disease
ratinocytes of the epidermis. The destruction can be remembered by the mnemonic, JOE:
of desmoglein leads to intraepidermal acan- patients get Jaw pain and Ocular disturbances
tholysis with sparing of the basal layer. Physical from ischemia to the arteries supplying them.
exam typically shows flaccid epidermal bullae Patients also often have markedly elevated
that easily slough off leaving large denuded ar- Erythrocyte sedimentation rates. The disease is
eas of skin (Nikolsky sign), subject to second- often associated with the presence of polymyal-
ary infection. Treatment is usually steroids. gia rheumatica.
Answer A is incorrect. A is the stratum cor- Answer C is incorrect. Kawasaki disease is
neum, which is composed of enucleated, kera- a self-limited vasculitis that normally occurs
tinized, flat keratinocytes. The autoantibodies in infants and children and is characterized
that mediate pemphigus vulgaris are directed by conjunctival and oral erythema, fever,
336 Section II: Organ Systems  •  Answers

erythema and edema of the palms and soles, In rheumatoid arthritis, CD4+ T lymphocytes
generalized rash, and cervical lymph node stimulate the immune cascade, leading to cy-
swelling. About 20% of patients may go on to tokine production such as tumor necrosis
develop coronary artery inflammation and/or factor-a and interleukin-1. However, the main
aneurysm. mechanism of injury is believed to be the for-
mation of immune complexes.
Answer E is incorrect. Takayasu arteritis is a
vasculitis characterized by fibrotic thickening Answer D is incorrect. Type I hypersensitiv-
of the aortic arch (it also affects the pulmo- ity reactions are characterized by antigens that
nary arteries, the branches of the aortic arch, cross-link IgE antibodies present on presen-
and the rest of the aorta in up to one-third of sitized mast cells and basophils. This cross-
patients). Clinically, patients often have lower linking results in the release of vasoactive
blood pressure and weaker pulses in the upper amines, like histamine.
High-Yield Systems

extremities than in the lower extremities; some


Answer E is incorrect. Type II hypersensitiv-
patients have ocular disturbances as well.
ity reactions are mediated by antibody binding
to a host antigen on a cell, leading to phago-
42. The correct answer is C. This patient has RA,
cytosis or lysis of the cell by complement. In
which is characterized by systemic symptoms
Graves disease, an example of a type II hyper-
of fever, fatigue, pleuritis, and pericarditis.
sensitivity reaction, autoantibodies bind to and
Women are affected by RA more frequently
activate the TSH receptor on thyroid gland
than men. Patients with RA classically experi-
cells, causing increased triiodothyronine and
ence symmetric morning stiffness of joints that
thyroxine production and release.
improves with use. Patients may also have sub-
cutaneous rheumatoid nodules, ulnar devia-
43. The correct answer is A. This patient has an-
tion of the fingers, and joint subluxation. RA is
kylosing spondylitis, a chronic inflammatory
mediated by a type III hypersensitivity reaction
disease of the spine and sacroiliac joints that
in which immune complexes form and acti-
often leads to the stiffening or consolidation of
vate complement. In RA, rheumatoid factor is
the bones that make up the joints. Common
an IgM autoantibody that is directed against
findings are low back pain, stiffness for over
Musculoskeletal

the Fc region of the patient’s IgG antibody,


three months, pain and stiffness in the thoracic
leading to immune complex formation and
region, limited movement in the lumbar area,
deposition. Rheumatoid factor antibodies are
and limited chest expansion. Around 90% of
present in a number of asymptomatic patients;
patients are positive for human leukocyte an-
more recently, anti-CCP (citrulline-containing
tigen (HLA) B27, and common complications
protein) antibodies have become more popular
include uveitis and aortic regurgitation.
as sensitive and specific serological diagnostic
indicators of RA. Answer B is incorrect. Antinuclear antibodies
are most commonly found in systemic auto-
Answer A is incorrect. The Arthus reaction is
immune diseases such as lupus, scleroderma,
a local, subacute, antibody-mediated hypersen-
Sjögren syndrome, and rheumatoid arthritis.
sitivity reaction. Hypersensitivity pneumonitis
from thermophilic actinomycetes, not RA, is Answer C is incorrect. Antineutrophil cyto-
explained by the Arthus reaction. plasmic antibodies (ANCA) are associated with
the vasculitides, including Wegener granulo-
Answer B is incorrect. Delayed, cell-mediated
matosis and Churg-Strauss syndrome.
hypersensitivity reactions are also type IV hy-
persensitivity reactions. In this type of reaction, Answer D is incorrect. IgM antibodies to
sensitized T lymphocytes encounter antigen B burgdorferi are suggestive of acute Lyme dis-
and release lymphokines, leading to macro- ease, which is transmitted by a bite from an
phage activation. A positive TB skin test is an Ixodes tick. Initially patients suffer from a lo-
example of a type IV hypersensitivity reaction. cal skin infection (often in a bull’s-eye pattern)
Chapter 12: Musculoskeletal  •  Answers 337

that can be followed by arthralgias and arthritis ulnar claw hand, which is caused by a weak-
(usually monoarticular). The late sequelae of ness of the medial two lumbricals that flex at
Lyme disease include myocardial, pericardial, the metacarpophalangeal joints and extend at
and neurologic changes. the interphalangeal joints of the ring and little
fingers. Patients will also experience weak-
Answer E is incorrect. Rheumatoid factor is
ness in the ability to abduct or adduct fingers
positive in about 80% of patients with rheu-
or adduct the thumb at the metacarpophalan-
matoid arthritis; it can also be positive in those
geal joints (interosseous muscles and adduc-
with other rheumatic disorders such as Sjögren
tor pollicis). They are unable to hold a piece
syndrome and lupus, as well as in healthy peo-
of paper between the thumb and index finger
ple. Rheumatoid arthritis is an autoimmune
or between adjacent fingers. Weakness of the
disorder of synovial joints and often presents
interosseous muscles may also result in a slight
with morning joint stiffness, subcutaneous

High-Yield Systems
clawing of the index and middle fingers. The
joint nodules (particularly in the proximal in-
muscles in the hypothenar eminence may also
terphalangeal joints), and symmetric joint in-
be affected; patients experience weakness in
volvement. The disease may also include sys-
flexion, abduction, and opposition of the fifth
temic symptoms such as fever, pleuritis, and
finger. Altered sensation in skin of the medial
pericarditis.
aspect of the hand and medial digits may be
Answer F is incorrect. Vertebral compres- evident. There are healing difficulties associ-
sion fractures are a complication of osteopo- ated with this type of fracture.
rosis and present with acute back pain, loss of
Answer D is incorrect. Fracture of the pha-
height, and kyphosis.
langes is a common injury and is often due to
44. The correct answer is C. This patient most crushing or hyperextension injuries.
likely has a “boxer’s fracture,” which occurs Answer E is incorrect. Fracture of the scaph-
when individuals strike a blow with a closed oid commonly occurs when individuals fall
fist against a hard, unyielding object. The most onto an outstretched hand. The scaphoid is
commonly injured sites for experienced boxers the most commonly fractured carpal bone, and
are the first and second metacarpals, whereas

Musculoskeletal
patients may exhibit pain and tenderness local-
for others, the neck of the fifth metacarpal is ized over the anatomic snuffbox.
the most common site of injury. The metacar-
pals have a good blood supply and thus heal 45. The correct answer is B. Anti-Jo-1 antibody is
rapidly. a myositis-specific antibody most commonly
associated with polymyositis and dermatomyo-
Answer A is incorrect. A complete transverse
sitis. Polymyositis and dermatomyositis are
fracture of the distal radius is called Colles’
diagnosed if five criteria are met: symmetric
fracture. This occurs most commonly in the
proximal muscle weakness; characteristic he-
elderly after forced dorsiflexion. It results in
liotrope rash; elevated serum muscle enzymes;
avulsion of the styloid process from the shaft
myopathic changes on EMG; and muscle bi-
of the radius. The radius may be shortened,
opsy abnormalities with the absence of histo-
and the styloid process of the ulna may project
pathologic signs of other myopathies. This pa-
farther distally than the styloid process of the
tient presented with the heliotrope rash, a skin
radius. The forearm and hand may exhibit a
manifestation that is highly specific for derma-
“dinner fork” deformity as a result of the pos-
tomyositis.
terior displacement of the distal part of the ra-
dius. Answer A is incorrect. Anti-dsDNA auto-
antibodies are associated with SLE.
Answer B is incorrect. Fracture of the hamate
is not common but can be complicated, as the Answer C is incorrect. Anti-IgG auto-
ulnar nerve can often be injured. Patients with antibodies are associated with rheumatoid ar-
an ulnar nerve lesion at the wrist may have an
338 Section II: Organ Systems  •  Answers

thritis. Anti-IgG is also known as rheumatoid Answer E is incorrect. Anti-mitochrondrial


factor. auto-antibodies are associated with primary
biliary cirrhosis, not polymyositis or dermato-
Answer D is incorrect. Anti-microsomal auto-
myositis.
antibodies are associated with Hashimoto thy-
roiditis, not polymyositis or dermatomyositis.
High-Yield Systems
Musculoskeletal
Chapter 13

Neurology

339
340 Section II: Organ Systems  •  Questions

Q u e st i o n s

1. Parents bring their 10-day-old infant to the (C) Contralateral motor deficits of the leg and
emergency department because of poor feed- foot
ing for the past two days. The infant is febrile (D) Ipsilateral motor deficits of the arm and
and appears lethargic and irritable. Blood and hand
cerebrospinal fluid (CSF) are taken for cul- (E) Ipsilateral motor deficits of the face
ture, and the infant is started on ampicillin (F) Ipsilateral motor deficits of the leg and foot
and gentamicin. Laboratory culture of CSF
reveals growth of gram-positive bacilli, with 3. During the autopsy of a patient noted to have
b-hemolysis on sheep blood agar. The micro- xanthochromia, the pathologist removes the
biologist notes that the same organism may calvarium and the attached dura. On the sur-
High-Yield Systems

cause meningitis in an immunocompromised face of the brain there is frank blood that can-
adult. What is the likely mode of pathogen not be removed by rubbing or scraping the
transmission to the immunocompromised surface. Which of the following most likely
adult? caused this finding?
(A) An ascending urinary tract infection (A) Ruptured aneurysm
(B) Direct inoculation into an open wound (B) Intraparenchymal hemorrhage
(C) Inhalation of aerosolized bacteria (C) Intradural hemorrhage
(D) Ingestion of unpasteurized milk (D) Tearing of bridging veins
(E) Temporoparietal bone fracture
2. The diagram below depicts the arterial net-
work of the brain. Infarction of the artery des- 4. A 34-year-old man comes to the physician be-
ignated by arrow “A” would lead to which of cause of the gradual onset of involuntary limb
the following deficits? and facial movements, mood swings, and trou-
ble with his memory. He says that his father
displayed similar symptoms when he was in
A his 40s. Which of the following changes would
most likely be seen in this patient?
Neurology

B
(A) Accumulation of neuritic plaques
(B) Copper accumulation in the basal ganglia
(C) Gliosis of the caudate nucleus
C (D) Loss of pigmentation in substantia nigra
(E) Scattered plaques of demyelination

5. Examination of a newborn shows a number of


D serious nervous system abnormalities. A diag-
nosis of meningohydroencephalocele is made.
Which of the following descriptions is most
consistent with this diagnosis?
E (A) Protrusion of the meninges, the brain, and
a portion of the ventricle through a defect
Reproduced, with permission, from USMLERx.com. in the skull
(B) Protrusion of the meninges and brain
through a defect in the skull
(A) Contralateral motor deficits of the arm and (C) Protrusion of the meninges and spinal
hand cord through a vertebral defect, forming a
(B) Contralateral motor deficits of the face sac
Chapter 13: Neurology  •  Questions 341

(D) Protrusion of the meninges through a de- 8. A researcher studying the function of the hy-
fect in the skull pothalamus ablates the supraoptic hypophyseal
(E) Protrusion of the meninges through a ver- tract in laboratory rats. Which of the following
tebral defect, forming a sac processes will be impaired in these animals?
(A) Concentration of urine
6. A 40-year-old man was admitted to the neurol-
(B) Milk synthesis
ogy service for evaluation of persistent numb-
(C) Ovulation
ness over his left jaw and lower face. MRI re-
(D) Salt retention
veals a mass that is compressing a cranial nerve
(E) Spermatogenesis
as the nerve exits the skull. The cranial nerve
involved in this case exits the skull through
9. A 71-year-old woman is diagnosed with non-
which of the following foramina?
Hodgkin lymphoma. Soon after starting her

High-Yield Systems
(A) Foramen ovale first cycle of chemotherapy, she reports severe
(B) Foramen rotundum nausea and vomiting. Prochlorperazine is pre-
(C) Foramen spinosum scribed to control this adverse effect. Upon
(D) Jugular foramen which of the following labeled areas of the
(E) Superior orbital fissure brain does prochlorperazine act?

7. A 47-year-old man presents with dysarthria and


progressive muscle weakness of the bilateral
upper and lower extremities in the absence of
any history of neurologic disease or recent ill-
ness, weight loss, or trauma. Physical examina-
tion is notable for muscle atrophy and weak-
ness in all extremities. Deep tendon reflexes 1
are absent in the upper extremities but are 3+ 2 5
in the lower extremities; some fasciculations 3
are present. Babinski’s sign is up-going bilater-
ally, and cranial nerves are intact. Laboratory 4

Neurology
and imaging studies are all within normal lim-
its. Which of the following would be expected
on microscopic examination of the central ner-
vous system? Reproduced, with permission, from USMLERx.com.
(A) Demyelination of axons in the dorsal col-
umns and spinocerebellar tracts in the spi-
(A) 1
nal cord
(B) 2
(B) Demyelination of axons in the posterior
(C) 3
limb of the internal capsule in the cere-
(D) 4
brum
(E) 5
(C) Neuronal loss in the region of the anterior
horn cells and corticospinal tracts in the
spinal cord
(D) Neuronal loss in the region of the anterior
horn cells and posterior columns in the
spinal cord
(E) Neuronal loss only in the region of the an-
terior horn cells in the spinal cord
342 Section II: Organ Systems  •  Questions

10. A patient has a sudden, almost-total occlusion


of his right internal carotid artery. Brain tissue
in which cerebral artery territory is likely to be
affected most by the resultant ischemia?
(A) The territory supplied by the anterior cere-
bral artery
(B) The territory supplied by the middle cere-
bral artery
(C) The territory supplied by the posterior ce-
rebral artery
(D) The watershed territory between the mid-
dle cerebral artery and the posterior cere-
High-Yield Systems

bral artery
(E) The watershed territory between the ante- Reproduced, with permission, from USMLERx.com.
rior cerebral artery and the middle cere-
bral artery
(A) Arachnoid cells
11. A 63-year-old homeless woman is brought to (B) Astrocytes
the emergency department (ED) by the po- (C) Melanocytes
lice because she is disoriented and confused. (D) Neurons
On questioning the patient frequently forgets (E) Oligodendrocytes
what she has been asked. She provides seem-
ingly plausible details of the events prior to her 13. A 24-year-old woman with no significant medi-
coming to the hospital, but her accounts are cal history complains of double vision that be-
entirely inconsistent with the police report. On gan two weeks ago. Additionally, she has felt
physical examination she is emaciated and has weakened after using the sauna at her local
nystagmus and an unsteady gait. ED records gym over the last few months. Her double vi-
indicate that she has presented multiple times sion is present only when she attempts to look
in the past for alcohol withdrawal and alcohol- to the side. On neurologic examination, when
Neurology

related injuries. The lesion accounting for the patient attempts to look to the left, her
the patient’s signs and symptoms is located in right eye does not adduct past the midline, and
which part of the brain? her left eye exhibits beating horizontal move-
ments. On looking to the other direction, the
(A) Amygdala left eye exhibits the same signs while the right
(B) Basal ganglia eye “beats.” Her physician suspects a particular
(C) Broca area condition, and decides to send the patient for a
(D) Mamillary bodies MRI of the brain. Where is the lesion respon-
(E) Wernicke area sible for this patient’s symptoms most likely lo-
cated?
12. An 89-year-old woman is brought to the clinic
after suffering from a number of recent minor (A) Arcuate fasciculus
falls and episodes of confusion. MRI of the (B) Bilateral lesion of nuclei of Edinger-
brain reveals a mass localized to a convexity of Westphal
the left hemisphere. The patient declines treat- (C) Left medial lemniscus
ment and dies a few months later. An autopsy (D) Medial longitudinal fasciculus
slide obtained from the patient’s brain lesion is (E) Right medial lemniscus
shown in the image. From which type of cell
does the tumor seen here typically arise?
Chapter 13: Neurology  •  Questions 343

14. A 55-year-old woman has received treatment right side of her face, decreased taste sensation
for years to manage a chronic, progressive dis- on the right side of her tongue, and increased
ease. Since her mid-40s the patient has had sensitivity to loud sounds in her right ear. The
difficulty initiating movements. She has a shuf- rest of her neurological examination is normal.
fling gait, an expressionless face, and tremor The physician suspects a short course of ste-
in her hands and fingers at rest. Over the years roids will improve her facial weakness over the
she has tried many medications but with little next few weeks. In addition to the symptoms
relief of her symptoms, and instead has experi- described above, this patient may also develop:
enced severe adverse effects. She is referred for
(A) Decreased sensation over her left upper
possible ablation surgery. The neurosurgeon
cheek
explains the different pathways involved in ini-
(B) Decreased sensation over her right upper
tiation and inhibition of movement, the foun-
cheek

High-Yield Systems
dation of her disease. The neurosurgeon ex-
(C) Deviation of the uvula and soft palate to
plains that by nullifying or accentuating some
the left when the patient is asked to say
of the pathways, some of her symptoms may be
“Ahh”
alleviated. The introduction of an ablative le-
(D) Deviation of the uvula and soft palate to
sion into which structure labeled in the image
the right when the patient is asked to say
would be expected to improve this patient’s
“Ahh”
bradykinesia?
(E) Dryness in her left eye
(F) Dryness in her right eye
E
D
16. Both neostigmine and physostigmine act as
C acetylcholinesterase inhibitors. Although they
share many of the same effects, only physostig-
mine is used to treat anticholinergic toxicity,
whereas neostigmine is preferentially used to
treat myasthenia gravis. What is the justifica-
tion for using physostigmine over neostigmine
B
as an antidote for anticholinergic toxicity?

Neurology
(A) Physostigmine does not enter the central
nervous system, whereas neostigmine does
cross the blood-brain barrier
A
(B) Physostigmine enters the central nervous
Reproduced, with permission, from USMLERx.com. system, whereas neostigmine does not
cross the blood-brain barrier
(C) Physostigmine has a greater effect on skele-
(A) A tal muscle, whereas neostigmine has a
(B) B greater effect on the bladder
(C) C (D) Physostigmine has a greater effect on the
(D) D bladder, whereas neostigmine has a greater
(E) E effect on skeletal muscle
(E) Physostigmine irreversibly binds acetylcho-
15. A 33-year-old woman presents to the emer- linesterase, whereas neostigmine acts as a
gency department complaining of pain behind reversible inhibitor
her right ear since that morning. Neurologi- (F) Physostigmine reversibly binds acetylcho-
cal examination is notable for paralysis of the linesterase, whereas neostigmine acts as an
irreversible inhibitor
344 Section II: Organ Systems  •  Questions

17. The high-power micrograph shown in the im- 18. A 67-year-old woman presents to the physi-
age demonstrates a key histologic finding ob- cian with right-sided Horner syndrome and
tained from the brain of a 75-year-old man at face pain, a hoarse voice, dysarthria, diplopia,
autopsy. In the years leading up to his death, numbness, and ataxia with contralateral im-
the patient had exhibited the gradual onset of pairment of pain and temperature sensation
motor symptoms including bradykinesia, rest- in her arm and leg. No other motor or hearing
ing tremor, shuffling gait, and stooped posture. deficits are evident. Which parts of the central
His medical history was otherwise unremark- nervous system are involved in this patient?
able. Which of the following best describes the
(A) Caudal lateral pontine tegmentum, in-
pathology underlying this patient’s disease pro-
cluding spinal tegmental tract of cranial
cess?
nerve V, and the inferior surface of the cer-
ebellum
High-Yield Systems

(B) Cochlea and vestibular apparatus


(C) Dorsolateral quadrant of the medulla and
the inferior surface of the cerebellum
(D) Internal capsule, caudate nucleus, puta-
men, and globus pallidus
(E) Lateral geniculate body, globus pallidus,
and posterior limb of the internal capsule

19. A 26-year-old man presents with four days of


progressive, bilateral, lower extremity weakness
and dysesthesia. The patient denies any history
of trauma, but states that he stayed home from
work last week because of a fever accompanied
by diarrhea. Neurologic examination demon-
Reproduced, with permission, from USMLERx.com. strates the absence of reflexes in the lower ex-
tremities with no cranial nerve deficits. What is
the best therapy for this patient’s condition?
(A) Cortical atrophy associated with b-amyloid
Neurology

plaques, neurofibrillary tangles, and de- (A) Plasmapheresis and immunoglobulin


creased cholinergic activity (B) Glucocorticoids
(B) Defective copper transport leading to the (C) Acetaminophen
accumulation of copper in tissues (D) Radiation therapy
(C) Loss of γ-aminobutyric acidergic neurons (E) Rest and elevation of the lower extremities
causing atrophy of the caudate nucleus
(D) Loss of pigmented dopaminergic neurons 20. A 70-year-old woman presents to her physician
in the substantia nigra with a history of memory loss and occasion-
(E) Malignant tumor cells derived from the ally becoming lost and disoriented in her own
neural crest leading to metastatic disease home. Brain biopsy of another patient suffer-
of the brain ing from the same disease process is shown in
the image. What is the most appropriate ther-
apy?
Chapter 13: Neurology  •  Questions 345

(C) Hydrochlorothiazide
(D) Insulin
(E) Metformin

22. A 43-year-old man who is HIV positive pre­


sents to emergency department complaining of
vision problems for the past two days. He has
not been seeing a physician regularly, and his
CD4 cell count is 24/mm³. Neurologic exami-
nation reveals problems with speech, memory,
and coordination. He is admitted to the hospi-
tal but his symptoms rapidly worsen, and three

High-Yield Systems
weeks after admission the patient dies. What is
the most likely cause of death in this patient?
Reproduced, with permission, from USMLERx.com.
(A) Cryptococcus neoformans
(B) Herpes simplex virus
(A) Bromocriptine (C) JC virus
(B) Diazepam (D) Pneumocystis jiroveci
(C) Donepezil (E) Toxoplasma gondii
(D) Levodopa/carbidopa
(E) Selegiline 23. A 71-year-old man is brought in to a demen-
tia clinic for a neurologic evaluation. His wife
21. A 27-year-old man presents to his primary reports his memory has been deteriorating for
care physician for a pre-employment physi- the past several years, but that she is particu-
cal examination. The patient states that he larly concerned about his behavioral changes.
has been healthy and has no complaints ex- The man has become impulsive and aggres-
cept that he has been drinking a lot of water sive, occasionally striking out when family
for what feels like an unquenchable thirst for members try to prevent him from doing some-
the past couple of weeks. He reports that he thing unsafe. Physical examination reveals

Neurology
has also been urinating excessively and is un- obvious cognitive deficits, and the patient is
able to sleep through the night due to his uncooperative. In addition, he displays a habit
thirst and frequent urination. Urine analysis is of putting everything he holds into his mouth.
significant only for a specific gravity of 1.002. There is no evidence of gait or movement
Serum analysis is significant for an osmolality abnormalities. Two years later, the man dies
of 320 mOsm/L and a serum glucose level of of pneumonia and an autopsy is performed.
120 mg/dL. The patient is admitted to the hos- There is a complete absence of senile plaques
pital, where subcutaneous vasopressin is ad- and neurofibrillary tangles; however, there are
ministered. Subsequent urine analysis revealed spherical, silver-staining protein tangles and
a specific gravity of 1.009, and serum analysis occasional ballooned neurons. Which of the
reveals an osmolality of 300 mOsm/L and a se- following diseases is most likely?
rum glucose level of 124 mg/dL. The patient
(A) Creutzfeldt-Jakob disease
is most likely to benefit from treatment with
(B) Dementia with Lewy bodies
which of the following?
(C) Normal-pressure hydrocephalus
(A) Desmopressin (D) Pick disease
(B) Fluid restriction (E) Wilson disease
346 Section II: Organ Systems  •  Questions

24. A 7-year-old boy is brought to the emergency (C) Migraine headache


department after falling off his grandparents’ (D) Temporomandibular joint dysfunction syn-
deck; an x-ray film shows that he has a mid- drome
shaft fracture of the humerus. Which of the (E) Tension headache
following defects is most likely to occur with
this type of fracture? 27. A 42-year-old man is brought to the emer-
gency department by police after they found
(A) A protruding scapula
him walking unsteadily in the middle of a busy
(B) Inability to fully abduct the arm
street harassing other pedestrians. On presenta-
(C) Inability to hold a piece of paper between
tion the patient appears minimally responsive
fingers
and his temperature is 36.8°C (98.2°F), respi-
(D) Pain over the palmar aspects of the first
ratory rate is 8/min, heart rate is 54/min, and
three and a half digits
High-Yield Systems

blood pressure is 101/54 mm Hg. The patient


(E) Weakness in wrist extension
is unresponsive to glucose and naloxone ad-
ministration. Laboratory tests show:
25. A 27-year-old man is thrown from a motorcycle
into a tree. He is taken to the emergency de- Na+: 137 mEq/L
partment, where he is alert and awake but in Cl-: 100 mEq/L
severe pain. On physical examination, he has Glucose: 69 mg/dL
very limited abduction of his left shoulder and WBC count: 8000/mm³
flexion of his left elbow. On observation, the Hemoglobin: 14.6 g/dL
left shoulder is externally rotated. His forearm Vitamin B1: 59 pg/dL (normal: 140-820 pg/mL)
is pronated, and his elbow is extended. Which
Which of the following findings is most likely
of the following muscles is most likely para-
to be present upon physical examination?
lyzed in this patient?
(A) Fruity odor to breath
(A) Flexor carpi ulnaris
(B) Miosis
(B) Flexor digitorum superficialis
(C) Nystagmus
(C) Latissimus dorsi
(D) Pill-rolling tremor
(D) Teres minor
(E) Shaking chills
(E) Trapezius
Neurology

28. A 40-year-old woman with Crohn disease pre­


26. A 32-year-old man comes to the physician be-
sents with a tingling sensation in her fingers
cause of headaches that occur at night and
and toes and a recent history of fatigue. A com-
without warning. They begin in his left eye
plete history reveals that three years ago she
and then generalize to the left side of his face.
underwent resection of her terminal ileum,
Alcohol can precipitate the attacks, which last
but since then, she has been feeling well and
for less than one hour. The patient rates the
eating a normal diet. Physical examination
pain as a 10/10, and multiple over-the-counter
demonstrates weakness in all four extremities,
analgesics have resulted in minimal benefit.
hyperreflexia, and a positive Romberg sign.
He is given a prescription for sumatriptan to
CBC count reveals a hematocrit of 22% and a
treat his symptoms and is prescribed verapamil
hemoglobin level of 6 g/dL. Which of the fol-
for prophylaxis. Which of the following is the
lowing sets of laboratory results is most likely to
most likely diagnosis?
be seen in this patient?
(A) Cluster headache
(B) Medication-overuse headache
Chapter 13: Neurology  •  Questions 347

Mean corpuscular Hypersegmented and that for the past week she has been self-
Folate
Choice volume
(fL)
level PMNs on blood medicating with normal daily doses of one of
smear?
A <100 normal no her friend’s antidepressant medications. What
antidepressant is the patient most likely taking?
B 100 normal no
(A) Amitriptyline
C >100 low yes
(B) Bupropion
D >100 normal no (C) Fluoxetine
E >100 normal yes
(D) Mirtazapine
(E) Selegiline
Reproduced, with permission, from USMLERx.com.
30. A 26-year-old woman is brought to the hospital
by ambulance after an automobile accident.

High-Yield Systems
(A) A She sustained no injuries in the collision but
(B) B is adamant that she did not see the car that hit
(C) C her from the side as it approached. On neu-
(D) D rologic examination she has bilateral visual
(E) E field defects affecting nearly half of her lateral
vision. On further questioning about recent
29. A 15-year-old girl is brought to the emergency changes in her health, she notes that she has
department by her mother after experiencing not menstruated in 10 months. From which
a first-time seizure. The thin-appearing girl embryologic layer does this woman’s neoplasm
has a heart rate of 55/min, signs suggestive of most likely originate?
dehydration, and fine, velvety hair covering (A) Endoderm
her arms and legs. The physician calculates (B) Mesoderm
her body mass index to be 16.4 kg/m². When (C) Neural crest
the patient’s mother leaves the room for a (D) Neuroectoderm
moment, the patient admits to the physician (E) Surface ectoderm
that she has been feeling depressed recently

Neurology
348 Section II: Organ Systems  •  Answers

An s w e r s

1. The correct answer is D. The organism de- leg and foot areas of the motor and sensory cor-
scribed Listeria monocytogenes, causes men- tices. Thus, a lesion in the artery would lead to
ingitis and sepsis in neonates and the im- deficits in contralateral motor function of the
munocompromised. Other bacteria causing leg and foot.
neonatal meningitis include Escherichia coli
Answer A is incorrect. The artery in question
and Group B streptococci (GBS). GBS are
does not supply the arm and hand; the middle
the most common cause of neonatal menin-
cerebral artery does.
gitis. Ingestion of poorly pasteurized milk, soft
cheeses, coleslaw, and ready-to-eat turkey and Answer B is incorrect. The artery in question
pork products are implicated in the pathogen- does not supply the face; the middle cerebral
High-Yield Systems

esis of listeriosis in the immunocompromised artery does.


population and pregnant women. Answer D is incorrect. The artery in question
Answer A is incorrect. While Escherichia coli, does not supply the arm and hand; the middle
a common gram-negative bacterial cause of cerebral artery does so in a contralateral fash-
neonatal meningitis, can produce urinary tract ion.
infections in both well and immunocompro- Answer E is incorrect. The artery in question
mised adults, the organism described in the does not supply the face; the middle cerebral
clinical case is the gram-positive bacilli Listeria artery does so in a contralateral fashion.
monocytogenes.
Answer F is incorrect. The artery in question
Answer B is incorrect. Direct inoculation is a supplies the medial side of the brain respon-
common route of transmission of infection, in- sible for contralateral motor function, not ipsi-
cluding gas gangrene produced by Clostridium lateral leg and foot motor functions.
perfringens, as well as tetanus caused by Clos-
tridium tetani. In contrast, listeriosis in the im- 3. The correct answer is A. The layers of the
munocompromised is most often from inges- head from superficial to deep are skin, perios-
tion of poorly pasteurized milk, soft cheeses, teum, bone, dura mater, arachnoid, pia, and
Neurology

coleslaw, and ready-to-eat turkey and pork brain parenchyma. The meningeal layers con-
products. sist of the dura, arachnoid, and pia. The sub-
Answer C is incorrect. Listeria monocytogenes, arachnoid space, which contains cerebrospinal
a gram-positive, b-hemolytic, catalase-positive fluid (CSF), is between the arachnoid and pia
bacillus, causes meningitis and sepsis in neo- as the name suggests. Subarachnoid hemor-
nates as well as the immunocompromised. rhages are usually caused by rupture of con-
While neonatal listeriosis may be contracted genital berry aneurysms, and less commonly
by passage through the birth canal, inhalation from arteriovenous malformations. Berry aneu-
of infected amniotic fluid, or nosocomial in- rysm rupture releases blood into the subarach-
fection, listeriosis in the immunocompromised noid space and covers the surface of the brain
is most often from ingestion of poorly pasteur- with blood. However, the blood cannot be
ized milk, soft cheeses, coleslaw, and ready-to- scraped off since it is trapped under the arach-
eat turkey. noid mater. CSF obtained in subarachnoid
hemorrhages via spinal taps will appear yel-
2. The correct answer is C. The specimen in the low (this indicates the presence of bilirubin in
photo is the circle of Willis. The artery desig- the CSF, a sign of hemorrhage and blood cell
nated by the arrow “A” is the anterior cerebral breakdown) because the subarachnoid space
artery, which supplies the medial surface of the is continuous with the spinal space. This find-
brain, the area responsible for the contralateral ing is called xanthochromia. Patients with sub-
Chapter 13: Neurology  •  Answers 349

arachnoid hemorrhages will often present with with each generation. The caudate and puta-
the “worst headache ever” and nuchal rigidity. men are mainly affected, altering the indirect
pathway of the basal ganglia, which results in
Answer B is incorrect. An intraparenchymal
loss of motor inhibition. Gliosis refers to the
hemorrhage such as those caused by chronic
proliferation of astrocytes in areas of central
hypertension would not appear as blood on the
nervous system (CNS) damage. On imaging
surface of the brain. It would likely be deeper
the lateral ventricles may appear dilated be-
in the brain, commonly affecting the basal
cause of the caudate atrophy. Reserpine has
ganglia and thalamus. An intraparenchymal
been shown to minimize the motor abnormali-
hemorrhage appears more like a bruise of the
ties observed in Huntington disease.
brain tissue and less like a frank pool of blood,
as described in the vignette. Answer A is incorrect. Alzheimer disease is
the most common cause of dementia in the el-

High-Yield Systems
Answer C is incorrect. The dura mater is a
derly. It is marked by progressive memory loss
thick, fibrous structure of dense connective tis-
and cognitive impairment. Pathophysiologi-
sue without space for a significant amount of
cally, this disease is associated with deposition
blood to pool. Blood collects either above or
of neuritic plaques (abnormally cleaved amy-
below the dura but not within it.
loid protein) and neurofibrillary tangles (phos-
Answer D is incorrect. A subdural hemor- phorylated tau protein) in the cerebral cortex.
rhage is defined as a hemorrhage under the Donepezil/vitamin E therapy has been shown
dura mater that is caused by damage to bridg- to slow down but not prevent the progression
ing veins. There is a potential space between of the disease.
the dura mater and the arachnoid mater.
Answer B is incorrect. Wilson disease, an au-
When the calvarium (and its adherent dura) is
tosomal recessive disease, is caused by failure
removed, this space is exposed, and any blood
of copper to enter circulation bound to ceru-
there should be readily scraped off. Blood
loplasmin due to a problem with excretion
that cannot be scraped off must be under the
of copper from the liver. This disorder results
arachnoid, which is under the dura. Subdural
in copper accumulation in the liver, corneas,
hemorrhages are commonly caused by blunt
and basal ganglia. Symptoms include asterixis,
trauma, especially in the elderly, alcoholics,
parkinsonian symptoms, cirrhosis, and Kayser-

Neurology
and children, who have atrophied or underde-
Fleischer rings (corneal deposits of copper).
veloped brains that causes extra strain on the
Penicillamine, a chelating agent, has been
bridging veins.
used for treatment with some success. Al-
Answer E is incorrect. An epidural hemor- though Wilson disease can cause chorea and
rhage is caused by temporoparietal bone frac- dementia, it is less likely in this scenario as it
tures that damage the middle meningeal ar- is inherited in an autosomal recessive fashion
tery. If the bony calvarium and the dura are and other expected manifestations are not pres-
removed, an epidural (above the dura) hem- ent.
orrhage would be removed, and one would
Answer D is incorrect. Parkinson disease re-
not see blood on the surface of the brain. The
sults from loss of dopaminergic neurons and
blood in an epidural hemorrhage is between
therefore loss of pigmentation in the substantia
the dura and the cranium.
nigra. These changes alter the direct pathway
4. The correct answer is C. Huntington disease of the basal ganglia, resulting in loss of excita-
is characterized by chorea, dystonia, altered tion. Patients with Parkinson disease present
behavior, and dementia. It is an autosomal with difficulty initiating movement, cogwheel
dominant disease caused by CAG triplet re- rigidity, shuffling gait, and pill-rolling tremor,
peats on chromosome 4p. It is the classic ex- not chorea. A levodopa/carbidopa combina-
ample of genetic anticipation, in disease sever- tion is used for treatment.
ity increases and age of onset becomes earlier
350 Section II: Organ Systems  •  Answers

Answer E is incorrect. Multiple sclerosis (MS) with the mnemonic Standing Room Only
is characterized by scattered plaques of demye- (SRO) for the Superior orbital fissure, fora-
lination that can occur anywhere in the CNS. men Rotundum, and foramen Ovale, which
Periventricular areas and the optic nerve are transmit cranial nerves (CNs) V1, V2, and V3,
commonly affected because of their high de- respectively. This patient has a schwannoma
grees of myelination. Oligodendrocytes, which of the mandibular division of the trigeminal
are responsible for CNS myelination, are the nerve (CN V3) as the nerve exits the skull
specific targets of this autoimmune disease. through the foramen ovale. Compression of
The classic patient with MS is a white woman CN V3 causes numbness over the ipsilateral
who presents in her 20s or 30s. Patients typi- jaw and lower face.
cally present with recurring multifocal lesions
Answer B is incorrect. The maxillary division
that are separated in time (intervening periods
of the trigeminal nerve (CN V2) exits the skull
High-Yield Systems

of recovery) and space and diagnosed by MRI.


through the foramen rotundum, and compres-
Common complications include optic neuri-
sion would cause decreased sensation over the
tis, internuclear ophthalmoplegia (difficulty
cheek and middle face.
with horizontal eye movements), sensory and
motor changes, and Lhermitte sign (an “elec- Answer C is incorrect. The meningeal (recur-
tric shock” felt down the spine with neck flex- rent) branch of the mandibular nerve (CN V3)
ion). Treatments are aimed at immunosuppres- exits the skull through the foramen spinosum,
sion. along with the middle meningeal artery. This
nerve innervates the dura mater and is respon-
5. The correct answer is A. The diagnosis of me- sible for pain sensation.
ningohydroencephalocele is extremely rare
Answer D is incorrect. The jugular foramen
and involves protrusion of the meninges, the
transmits the glossopharyngeal (CN IX), vagus
brain, and a portion of the ventricle through a
(CN X), and spinal accessory (CN XI) nerves.
defect in the skull. The prognosis is extremely
The glossopharyngeal nerve is responsible
poor.
for motor innervation of the stylopharyngeus
Answer B is incorrect. Protrusion of the me- muscle, parasympathetic innervation of the
ninges and brain through a defect in the skull parotid gland, and sensory innervation of the
Neurology

is consistent with a diagnosis of meningoen- pharynx, middle ear, and posterior third of
cephalocele, which also carries a grave progno- the tongue. It also innervates the chemorecep-
sis. Seventy-five percent of these infants die or tors and baroreceptors of the carotid body. The
are severely mentally retarded. vagus nerve is responsible for motor innerva-
tion of the pharyngeal and laryngeal muscles,
Answer C is incorrect. Protrusion of the me-
parasympathetic innervation to visceral organs,
ninges and spinal cord through a vertebral de-
and sensory innervation to the pharynx and
fect to form a sac is consistent with a diagnosis
meninges. It also innervates the chemorecep-
of spina bifida with meningomyelocele.
tors and baroreceptors of the aortic arch. The
Answer D is incorrect. Protrusion of the me- spinal accessory nerve innervates the sternomas-
ninges through a defect in the skull is consis- toid and upper part of the trapezius muscles.
tent with a diagnosis of meningocele.
Answer E is incorrect. CN III (oculomotor),
Answer E is incorrect. Protrusion of the me- CN IV (trochlear), CN V1 (ophthalmic), and
ninges through a vertebral defect to form a sac CN VI (abducens) exit the skull through the
is consistent with a diagnosis of spina bifida superior orbital fissure, together with the supe-
with meningocele. In this condition, the spinal rior ophthalmic vein. Lesions of these nerves
cord remains in its normal position. would lead to ipsilateral extraocular muscle
paralysis (CNs III, IV, and VI) and numbness
6. The correct answer is A. The foramina of the of the ipsilateral forehead and upper face (CN
trigeminal nerve divisions can be remembered V1).
Chapter 13: Neurology  •  Answers 351

7. The correct answer is C. This patient pre­ gland via the supraoptic hypophyseal tract,
sents with signs and symptoms consistent with where they are stored and eventually released
amyo­ trophic lateral sclerosis (ALS), which into the capillaries draining into the hypo­
affects both anterior horn cells in the spinal physeal vein. ADH mediates water absorption
cord and upper motor neurons in the spinal at the renal collecting ducts via V2 receptors,
cord. ALS results in a combination of upper allowing translocation of aquaporins, thus con-
and lower motor neuron signs, although the centrating the urine. Oxytocin facilitates milk
deficits may be asymmetric. More males are secretion but not synthesis, and also stimu-
affected than females, and the incidence rises lates uterine contractions during parturition.
after age 40. Riluzole (Rilutek) is the only Think “pituitary” when you see “hypophysis/
FDA-approved treatment for the disorder, and hypo­physeal.” Also remember that the impor-
it prolongs life by only three-six months; it is tant connection between the hypothalamus

High-Yield Systems
thought to function by reducing the presynap- and the anterior pituitary is the hypothalamic-
tic release of glutamate. hypo­physeal portal system. Unlike the neuro-
nal connection of the supraoptic hypophyseal
Answer A is incorrect. Demyelination of ax-
tract, the hypothalamic-hypophyseal portal
ons in the dorsal columns and spinocerebellar
system is a capillary system that transports hor-
tracts occurs in subacute combined degenera-
mones synthesized in the hypothalamus that
tion of the spinal cord, which is also known as
act on the anterior pituitary. Secretion of ADH
vitamin B12 neuropathy. It is associated with
in response to reduced plasma volume is acti-
pernicious anemia and results in loss of vibra-
vated by pressure receptors in the veins, atria,
tion and position sense (dorsal columns) and
and carotids. Secretion of ADH in response to
arm/leg ataxia (spinocerebellar tracts).
increases in plasma osmotic pressure is medi-
Answer B is incorrect. Demyelination of ax- ated by osmoreceptors in the hypothalamus.
ons in the posterior limb of the internal cap-
Answer B is incorrect. Milk synthesis is medi-
sule would cause contralateral spastic paralysis
ated by prolactin, which is secreted by the an-
secondary to disruption of the descending fi-
terior pituitary gland and hence would be un-
bers of the corticospinal tract, resulting in up-
affected by ablation of the transport tract from
per motor neuron signs.
the hypothalamus to the posterior pituitary

Neurology
Answer D is incorrect. Neuronal loss in the gland. Although oxytocin does not have a role
region of the anterior horn cells and posterior in milk synthesis, it allows milk letdown in lac-
columns in the spinal cord occurs in Charcot- tating women.
Marie-Tooth disease, also known as peroneal
Answer C is incorrect. Ovulation is stimulated
muscular atrophy. It results in loss of conscious
by the surge of luteinizing hormone (LH) just
proprioception (posterior columns) and lower
prior to the midpoint of the menstrual cycle.
motor neuron signs (anterior horn motor neu-
LH is secreted by the anterior pituitary gland
rons).
and hence would be unaffected by ablation of
Answer E is incorrect. Neuronal loss in the the transport tract from the hypothalamus to
region of the anterior horn cells in the spinal the posterior pituitary gland.
cord occurs in poliomyelitis, an acute inflam-
Answer D is incorrect. Salt retention is a pri-
matory viral infection that affects the lower
mary action of aldosterone, acting at the renal
motor neurons and results in a flaccid paralysis
distal tubules to increase sodium and chloride
(pure lower motor neuron disease).
reabsorption as well as increase potassium
8. The correct answer is A. ADH (vasopressin) and hydrogen secretion. Aldosterone is pro-
and oxytocin are synthesized by the neurons duced from cholesterol in a multistep path-
of the supraoptic and paraventricular nuclei way in response to ACTH stimulation. ACTH
in the hypothalamus, respectively. These hor- is secreted by the anterior pituitary gland and
mones are transported to the posterior pituitary hence would be unaffected by ablation of the
352 Section II: Organ Systems  •  Answers

transport tract from the hypothalamus to the during voluntary movement. It also aids in mo-
posterior pituitary gland. tor learning.
Answer E is incorrect. Spermatogenesis is
10. The correct answer is E. The right internal ca-
stimulated by follicle-stimulating hormone,
rotid artery supplies blood to both the right an-
which is secreted by the anterior pituitary
terior cerebral artery and the middle cerebral
gland and hence would be unaffected by abla-
artery. Decreased blood supply to both the an-
tion of the transport tract from the hypothala-
terior and middle cerebral arteries would most
mus to the posterior pituitary gland.
severely damage the tissue that lies between
the distributions of the two arteries. This area
9. The correct answer is D. The area postrema
is known as the watershed zone, the zone sup-
of the medulla contains the chemoreceptor
plied by the most distal sections of two differ-
trigger zone (CTZ), which controls vomit-
High-Yield Systems

ent arteries.
ing. The CTZ is located on the floor of the
fourth ventricle and communicates with the Answer A is incorrect. During occlusion of
vomiting center. It is important to note that the internal carotid artery, blood supply would
the CTZ is located outside of the blood-brain decrease to tissue supplied by the anterior ce-
barrier, which is critical to its role in detecting rebral artery. However, tissue in the watershed
toxic substances in the circulation. Prochlor- zone would be more susceptible to infarction
perazine is a typical anti-psychotic agent that is and ischemia because this area is downstream
more often used for its anti-emetic properties. of the tissue that is purely in the anterior cere-
It functions as a dopamine blocker at the CTZ. bral artery distribution.
Answer A is incorrect. The midbrain helps Answer B is incorrect. During occlusion of
regulate motor control, control of eye move- the internal carotid artery, blood supply would
ments, and acoustic relay. It contains several decrease to tissue supplied by the middle ce-
essential nuclei of auditory and visual systems. rebral artery. However, tissue in the watershed
zone would be more susceptible to infarction
Answer B is incorrect. The pituitary plays
and ischemia because this area is downstream
many roles in autonomic and endocrine con-
of the tissue that is purely in the middle cere-
trol. A helpful mnemonic is “TAN HATS”:
bral artery distribution.
Neurology

Thirst, Adenohypophysis control via releasing


factors (gonadotropin hormone-releasing hor- Answer C is incorrect. The internal carotid
mone, thyroid hormone-releasing hormone), artery does not supply the posterior cerebral
Neurohypophysis and median eminence, artery; therefore, tissue in the distribution of
Hunger, Autonomic regulation, Temperature the posterior cerebral artery would not be in-
regulation, and Sexual urges. farcted.
Answer C is incorrect. The pons plays a role Answer D is incorrect. The internal carotid
in many vital functions such as respiratory and artery supplies the middle cerebral artery, but
urinary bladder control. It also contains the re- does not supply the posterior cerebral artery.
ticular activating system, which is responsible Therefore during occlusion of the internal ca-
for regulating sleep-wake cycles and level of rotid artery, the watershed zone between the
arousal, as well as contributing vestibular con- middle and posterior cerebral arteries would
trol of eye movements. It also conveys motor still be supplied by the posterior cerebral ar-
information from the cerebral hemispheres to tery.
the cerebellum.
11. The correct answer is D. The patient presents
Answer E is incorrect. The cerebellum regu-
with signs of Wernicke-Korsakoff syndrome,
lates movement and posture by providing con-
which is caused by thiamine (vitamin B1) de-
stant feedback in order to allow for correction
ficiency. Malnourished chronic alcohol abus-
ers are thus particularly prone to this disease.
Chapter 13: Neurology  •  Answers 353

Thiamine pyrophosphate serves as a cofactor papillary cystadenocarcinoma, and mesothe-


for several enzymes that are needed in key lioma (mnemonic: PSaMMoma (Papillary
glucose metabolic pathways. Wernicke en- [thyroid], Serous [ovary], Meningioma, Meso-
cephalopathy presents first, and may progress thelioma). Given the CNS tumor and absence
to Korsakoff psychosis if left untreated. The of any clues to suggest metastatic disease, me-
classic triad of Wernicke encephalopathy is ningioma is the most likely culprit. Meningio-
confusion, ataxia, and ophthalmoplegia (weak- mas are the second most common type of pri-
ness of eye muscles that may result in diplopia mary brain tumor; they frequently occur in the
and/or nystagmus). Whereas Wernicke en- convexities of cerebral hemispheres and in the
cephalopathy may be reversible if treated early, parasagittal regions. Meningiomas arise from
Korsakoff psychosis is an irreversible condition arachnoid cells external to the brain. Know
characterized by anterograde amnesia, confab- what psammoma bodies look like and with

High-Yield Systems
ulation, and personality changes. Anterograde which tumors they are associated.
amnesia is the inability to create new memory,
Answer B is incorrect. Astrocytes give rise to a
and confabulation is the act of filling in gaps
variety of tumors, most notably gliomas. None
in one’s memory with fabrications that are
of the astrocyte-derived tumor types contain
believed to be true. The lesion in Wernicke-
dystrophic calcifications such as psammoma
Korsakoff syndrome is located in the mamillary
bodies.
bodies.
Answer C is incorrect. Melanocytes give rise
Answer A is incorrect. Bilateral lesions of
to melanomas, which do not present with
the amygdala causes Klüver-Bucy syndrome,
psammoma bodies.
which is characterized by hyperorality, hyper-
sexuality, and disinhibition. Hyperorality Answer D is incorrect. Neurons give rise to
means placing inappropriate objects into one’s neuromas, which do not undergo calcification
mouth. to form psammoma bodies.
Answer B is incorrect. Lesions in the basal Answer E is incorrect. Oligodendrocytes give
ganglia are associated with movement disor- rise to oligodendrogliomas, which tend to oc-
ders such as Parkinson disease. cur in the cerebral hemispheres in middle-
aged people, and the histology shows large,

Neurology
Answer C is incorrect. Broca area is located in
round nuclei with a clear halo of cytoplasm
the inferior frontal gyrus. Patients with lesions
(“fried egg”) cells. Oligodendrogliomas do not
here have motor/nonfluent/expressive aphasia,
contain psammoma bodies.
meaning that although they can understand
what others are saying, they have difficulty pro- 13. The correct answer is D. This patient presents
ducing coherent speech. with internuclear ophthalmoplegia (INO). In
Answer E is incorrect. Wernicke area is lo- young patients, bilateral INO is highly indica-
cated in the superior temporal gyrus. Patients tive of the development of MS. On gaze to the
with lesions here have sensory/fluent/receptive left, the primary movement is started in the left
aphasia, meaning that they can speak fluently CN VI nucleus, and cannot be transmitted to
but cannot understand what others or they the right CN III nucleus via the right medi-
themselves are saying. cal longitudinal fasciculus (MLF) because of
demyelination secondary to the underlying
12. The correct answer is A. The autopsy slide disease process. As a consequence, the right
demonstrates psammoma bodies, which are eye cannot adduct. The reverse is true for gaze
lamellated mineral deposits formed via calcifi- directed to the right. However, convergence is
cation of whorled clusters of cells found inside usually intact as this maneuver does not utilize
the tumor. Psammoma bodies are associated the MLF pathway.
with several neoplasms: meningioma, papillary
adenocarcinoma of the thyroid, ovarian serous
354 Section II: Organ Systems  •  Answers

Answer A is incorrect. The arcuate fasciculus this pathway is ineffective. Ablating part of the
is the tract of neurons connecting Broca area internal segment of the globus pallidus (palli-
and Wernicke area in the left cerebral hemi- dotomy) would reduce its tonic inhibition of
sphere. Lesions of the arcuate fasciculus most the thalamus and improve the patient’s parkin-
often manifest as impaired repetition. While sonian symptoms. Note that first-line treatment
MS can theoretically affect any white-matter for PD is pharmacologic, most classically with
tract in the brain, this patient does not present levodopa, which is converted into dopamine in
with any language deficits, thus her current le- the brain.
sion is not located here.
Answer B is incorrect. The putamen is one
Answer B is incorrect. The Edinger-Westphal of the components of the striatum. It contains
nucleus supplies preganglionic parasympa- neurons with dopamine receptors that are re-
thetic fibers to the eye that run on the outside sponsive to projections from dopaminergic
High-Yield Systems

of CN III. Its main function is to regulate pu- neurons of the substantia nigra. Some of the
pillary constriction and lens accommodation. striatal neurons provide inhibitory input to the
Despite its close association with the oculomo- internal segment of the globus pallidus, which
tor nerve, it does not itself participate in move- is necessary for the initiation of movement.
ment of the globe. Ablating the putamen may therefore make the
patient’s bradykinesia worse.
Answer C is incorrect. The medial lemniscus
is the tract in the brain stem that carries sen- Answer C is incorrect. The internal capsule is
sory information on light touch, propriocep- a white-matter tract that transmits the axons of
tion, and vibration in the extremities from the upper motor neurons to the brainstem and spi-
nucleus gracilis and cuneatus to the thalamus. nal cord. Ablating the internal capsule would
Although it could be affected by MS, it is not cause devastating hemiplegia with upper mo-
involved in the coordination of extraocular tor neuron symptoms (eg, spasticity, hyperre-
movements and thus cannot be responsible for flexia).
this patient’s symptoms.
Answer D is incorrect. The caudate nucleus is
Answer E is incorrect. The medial lemniscus one of the components of the striatum. It con-
is the tract in the brain stem that carries sen- tains neurons with dopamine receptors that are
Neurology

sory information on light touch, propriocep- responsive to projections from dopaminergic


tion, and vibration in the extremities from the neurons of the substantia nigra. Some of the
nucleus gracilis and cuneatus to the thalamus. striatal neurons provide inhibitory input to the
Although it could be affected by MS, it is not internal segment of the globus pallidus, which
involved in the coordination of extraocular is necessary for the initiation of movement.
movements, and thus cannot be responsible Ablating the caudate nucleus might therefore
for this patient’s symptoms. make the patient’s bradykinesia even worse.
Answer E is incorrect. The corpus callosum is
14. The correct answer is A. The patient in this
a white-matter tract that transmits axons from
question suffers from refractory Parkinson dis-
one side of the cerebral cortex to the other,
ease (PD). In this disease the internal segment
allowing the two halves of the brain to com-
of the globus pallidus produces excessive in-
municate. Ablating the corpus callosum could
hibition of the ventral lateral nucleus of the
cause perceptual abnormalities but would not
thalamus, making it difficult for patients to ini-
improve the patient’s parkinsonian symptoms.
tiate movements (bradykinesia). Normally, do-
paminergic neurons from the substantia nigra
15. The correct answer is F. This patient’s history
induce striatal neurons to inhibit the globus
and physical exam findings are consistent with
pallidus, thereby lifting the inhibition of the
Bell palsy, an acute peripheral facial nerve
thalamus. Because of decreased dopaminer-
palsy of unknown etiology. The symptoms
gic signaling from the substantia nigra in PD,
seen in this patient can be understood if one
Chapter 13: Neurology  •  Answers 355

remembers the different nerve fibers carried thus cause the uvula and soft palate to deviate
by the facial nerve (cranial nerve VII): Afferent away from the side with the lesion due to un-
taste fibers from the anterior two-thirds of the opposed action from the normal side. A lesion
ipsilateral tongue (decreased taste sensation), of the left vagus nerve would cause the uvula
general touch and pain sensory fibers from a and soft palate to deviate to the right; however,
small area around the ipsilateral ear (retroau- the patient in this case has symptoms indicat-
ricular pain), and motor fibers to the muscles ing palsy of the facial nerve (cranial nerve VII),
of facial expression (ipsilateral facial paralysis) not the vagus nerve (cranial nerve X).
and stapedius muscle (increased sensitivity
Answer E is incorrect. The facial nerve (cra-
to noise in the ipsilateral ear due to weakness
nial nerve VII) carries sensory and motor fibers
of the stapedius muscle, which normally pre-
that innervate the ipsilateral face; thus a lower
vents excessive movement of the stapes). This
motor nerve palsy of the facial nerve would

High-Yield Systems
patient may also experience dryness in her ip-
cause dryness of the ipsilateral (right), not con-
silateral (right) eye and mouth because the fa-
tralateral (left), eye.
cial nerve carries parasympathetic fibers to the
ipsilateral lacrimal gland and submandibular/ 16. The correct answer is B. Physostigmine and
sublingual glands, which provides lubrication neostigmine, both of which are amines, are re-
to the eye and mouth, respectively. In addition, versible anticholinesterases. Both compounds
weakness of the facial muscles prevents com- act as substrates for acetylcholinesterase,
plete eye closure, exacerbating the eye dry- thereby preventing the esterase from break-
ness. Patients with facial nerve paralysis should ing down acetylcholine in the neuromuscular
be given lubricating eye drops and instructed junction or synapse. Physostigmine has better
to tape their eye closed at night. While facial CNS penetration, whereas neostigmine has
nerve paralysis can be caused by head trauma, better peripheral action (especially on skel-
AIDS, Lyme disease, sarcoidosis, or brain stem etal muscle). Neostigmine does not cross the
lesions, in most cases no cause is discovered blood-brain barrier. Thus physostigmine is
and the diagnosis of Bell palsy is made. used as an antidote for anticholinergic toxicity
Answer A is incorrect. Decreased sensation of because of its ability to cross the blood-brain
the left upper cheek could be caused by lesion barrier and treat central anticholinergic toxic-

Neurology
of the left maxillary division of the trigeminal ity (delirium, agitation) as well as peripheral
nerve (cranial nerve V2); however, this patient anticholinergic toxicity.
shows signs of facial nerve palsy but no signs of Answer A is incorrect. Neostigmine does not
trigeminal nerve involvement. cross into the CNS, whereas physostigmine
Answer B is incorrect. Decreased sensation does.
on the right upper cheek could be caused by Answer C is incorrect. Neostigmine has a
lesion of the right maxillary division of the tri- greater effect on skeletal muscle than does
geminal nerve (cranial nerve V2); however, physostigmine. Neither compound has much
this patient shows signs of facial nerve palsy effect on the bladder.
but no signs of trigeminal nerve involvement.
Answer D is incorrect. Neostigmine does have
Answer C is incorrect. Lesion of the right va- a greater effect on skeletal muscle than does
gus nerve would lead to deviation of the uvula physostigmine. Neither compound has much
and soft palate to the left; however, all signs effect on the bladder.
and symptoms in this case suggest damage to
the facial nerve (CN VII) and not the vagus Answer E is incorrect. Both physostigmine
nerve (CN X). and neostigmine are reversible anticholines-
terases. Isoflurophate and other organophos-
Answer D is incorrect. Unilateral lesions of phates irreversibly bind acetylcholinesterases,
the vagus nerve or nucleus ambiguous pre- permanently preventing the breakdown of ace-
vent elevation of the uvula on that side and
356 Section II: Organ Systems  •  Answers

tylcholine until the protein can be replaced chromosome 4. Huntington disease is com-
with a new one. monly associated with chorea, athetosis, and
changes in personality. This disease generally
Answer F is incorrect. Both physostigmine
presents in the third to fifth decades and is not
and neostigmine are reversible anticholines-
associated with Lewy bodies.
terases. Isoflurophate and other organophos-
phates irreversibly bind acetylcholinesterases, Answer E is incorrect. This answer choice is
permanently preventing the breakdown of ace- likely referring to metastatic melanoma of the
tylcholine until the protein can be replaced brain. This choice may be appealing because
with a new one. of the pigmentation of the histology shown in
the image. Ultimately, though, Lewy bodies
17. The correct answer is D. This specimen is are seen only in Parkinson disease and Lewy
taken from the patient’s substantia nigra and body dementia.
High-Yield Systems

demonstrates a typical melanin-containing


neuron with pink-staining inclusions known as 18. The correct answer is C. The stroke syndrome
Lewy bodies. Lewy bodies are aggregations of described is the lateral medullary syndrome,
the protein a-synuclein that are seen primar- also known as the posterior inferior cerebellar
ily in two neurologic diseases: Parkinson dis- artery (PICA) syndrome or Wallenberg syn-
ease and Lewy body dementia. This patient’s drome. This syndrome results in numbness
symptoms are characteristic of Parkinson dis- of the ipsilateral face and contralateral limbs,
ease, and this answer choice describes the pro- diplopia, dysarthria, and an ipsilateral Horner
cess underlying this disease. Other common syndrome. It classically results from disruption
symptoms of Parkinson disease are cogwheel of the PICA, which supplies blood to the dor-
rigidity, postural instability, micrographia, and solateral quadrant of the medulla, including
masked facies. the nucleus ambiguus and the inferior surface
of the cerebellum. The infarcted dorsolateral
Answer A is incorrect. The pathology de-
quadrant of the medulla contains the tract of
scribed in this answer choice is characteris-
cranial nerve V (face pain), vestibular nuclei
tic of Alzheimer disease, the most common
(dysequilibrium), nucleus ambiguus (palate
cause of dementia in the elderly. This answer
problems and hoarse voice), the spinothalamic
choice can be definitively ruled out, because
Neurology

tract (contralateral pain and temperature loss),


Alzheimer disease is not associated with Lewy
and descending sympathetic fibers (Horner
bodies.
syndrome). Limb weakness and reflex changes
Answer B is incorrect. The pathology de- are not found because corticospinal fibers are
scribed in this answer choice is consistent with in the ventral medulla at this location.
Wilson disease, an autosomal recessive defect
Answer A is incorrect. This vascular territory
in copper transport that leads to the accumula-
is supplied by the anterior inferior cerebel-
tion of copper in tissues. Wilson disease can be
lar artery. Disruption in blood flow typically
associated with parkinsonian symptoms, as well
causes ipsilateral deafness from involvement of
as chorea, psychiatric symptoms, liver disease,
the labyrinthine artery, ipsilateral facial weak-
and the characteristic Kayser-Fleischer ring.
ness, and ataxia. It is the second most common
Although some aspects of this answer choice
brainstem stroke syndrome.
are possible, this patient’s symptoms are most
consistent with Parkinson disease. Further- Answer B is incorrect. The cochlea and ves-
more, Wilson disease is not associated with tibular apparatus are perfused via the labyrin-
Lewy bodies. thine artery, and an isolated disruption of this
end artery would result in isolated dysfunction
Answer C is incorrect. This answer choice de-
of these two structures.
scribes the pathologic process associated with
Huntington disease, an autosomal dominant Answer D is incorrect. The internal capsule,
disease caused by a trinucleotide repeat on caudate nucleus, putamen, and globus pal-
Chapter 13: Neurology  •  Answers 357

lidus are perfused by the penetrating branches trauma to the lower extremity (eg, sprain or
of the middle cerebral artery known as the strain).
lateral striate arteries. They are commonly in-
volved with lacunar strokes. 20. The correct answer is C. This patient has
symptoms of dementia, and the brain bi-
Answer E is incorrect. The lateral geniculate
opsy shows neurofibrillary tangles and senile
body, globus pallidus, and posterior limb of
plaques, which are characteristic of Alzheimer
the internal capsule are supplied by the ante-
disease. People suffering from Alzheimer dis-
rior choroidal artery. Syndromes affecting this
ease often have >90% loss of acetylcholine in
artery represent less than 1% of anterior circu-
their brains. Initial treatment of this disease is
lation strokes and typically occur in the setting
acetylcholinesterase inhibitors, which increase
of symptomatic internal carotid artery occlu-
the amount of acetylcholine in the presynaptic
sion.

High-Yield Systems
space. Donepezil is an acetylcholinesterase in-
hibitor that is often used for this purpose.
19. The correct answer is A. Guillain-Barré syn-
drome is an acute, autoimmune, demyelinat- Answer A is incorrect. Bromocriptine is a do-
ing polyradiculoneuropathy affecting the pe- pamine receptor agonist used to treat Parkin-
ripheral nervous system, usually triggered son disease.
by an acute infectious process (most notably
Answer B is incorrect. Diazepam is a benzo-
infection with Campylobacter jejuni). It is fre-
diazepine. It increases the frequency of GABA
quently severe and usually manifests as an as-
channel opening and is used to treat anxiety,
cending paralysis noted by weakness in the legs
status epilepticus, and alcohol withdrawal.
that spreads to the upper limbs and the face,
along with complete loss of deep tendon re- Answer D is incorrect. Levodopa/carbidopa
flexes. With prompt treatment by plasmaphe­ is a combination that increases levels of dopa-
resis followed by immunoglobulins and sup- mine in the brain. It is used to treat Parkinson
portive care, most will regain full functional disease.
capacity. However, death may occur if there is Answer E is incorrect. Selegiline is an inhibi-
paralysis of the musculature used for respira- tor of monoamine oxidase type B, which in-
tion. creases dopamine availability and is used to

Neurology
Answer B is incorrect. Glucocorticoids would treat Parkinson disease.
be indicated in the setting of acute injury to
the CNS (eg, cauda equina syndrome, meta- 21. The correct answer is A. The patient has cen-
static bone disease, or spinal cord injury). Ste- tral diabetes insipidus (DI), a disorder in which
roid therapy reduces edema secondary to in- the kidneys fail to concentrate urine due to a
flammation. lack of ADH secretion by the posterior pitu-
itary. Causes of central DI include head in-
Answer C is incorrect. Acetaminophen is a juries, hypothalamic or pituitary tumors, and
non-narcotic analgesic and anti-pyretic. Given idiopathic causes. Without ADH, the principal
the pathophysiologic basis of Guillain-Barré cells of the distal tubule and collecting ducts
syndrome, plasmapheresis and intravenous im- remain impermeable to water. Nephrogenic
mune globulin are the best therapy. DI is a related condition in which ADH is se-
Answer D is incorrect. Radiation therapy is a creted by the posterior pituitary, but the princi-
treatment used for CNS tumors or metastatic pal cells of the kidneys do not respond to ADH
brain cancer. Radiation is not useful in pa- because of defective ADH receptors. Causes
tients with Guillain-Barré syndrome. include lithium toxicity and hypercalcemia.
Both conditions are characterized by low
Answer E is incorrect. Rest and lower extrem-
urine specific gravity and high serum osmolal-
ity elevation is indicated in the setting of mild
ity. The two are differentiated by a test dose of
358 Section II: Organ Systems  •  Answers

vasopressin, which corrects serum and urine provement with the initiation of highly active
osmolalities in central DI, but not in neph- antiretroviral therapy.
rogenic DI. Desmopressin is an ADH analog
Answer A is incorrect. Cryptococcus neofor-
that is taken nasally to treat central DI.
mans is a common cause of meningitis is pa-
Answer B is incorrect. Fluid restriction is not tients with HIV/AIDS. The classic meningitis
indicated for, and is actually detrimental to, triad of fever, headache, and nuchal rigidity
this patient because his renal system is unable are usually present. Abnormalities on CSF ex-
to reabsorb water. In some clinical settings the amination would also be present.
fluid deprivation test may be used as part of the
Answer B is incorrect. Herpes simplex virus
work-up for DI. In central DI, testing reveals
can cause temporal lobe encephalitis in pa-
low ADG levels and highly concentrated se-
tients with HIV/AIDS. It is also seen in the
rum, yet lack of concentrated urine.
High-Yield Systems

general population but at a lower frequency.


Answer C is incorrect. Although it seems Rapid onset of fever and focal neurological
counterintuitive, hydrochlorothiazide is a di- deficits are the most common presenting fea-
uretic used to treat nephrogenic DI. Its exact tures. Deficits often stem from damage to the
mechanism of action is unclear. Desmopressin temporal lobe and can include memory prob-
would have no effect on nephrogenic DI, be- lems, personality changes, and potentially sei-
cause exogenous ADH is futile if its receptors zures.
are defective.
Answer D is incorrect. Pneumocystis jiroveci
Answer D is incorrect. Insulin is used to treat (formerly carinii) is a common cause of pneu-
type 1 diabetes mellitus (DM) and the later monia in patients with HIV whose CD4+ cell
stages of type 2 DM. It has no indication in counts are <200/mm³. On x-ray of the chest,
the treatment of DI. Although DM is part of the classic picture is one of “ground glass,”
the differential diagnosis of this patient, nor- although other radiological features are also
mal glucose levels and the positive vasopressin common.
challenge point toward DI.
Answer E is incorrect. Toxoplasmosis is the
Answer E is incorrect. Metformin is an oral most common cause of encephalitis in patients
hypoglycemic agent used to treat type 2 DM with HIV/AIDS and is seen mostly in patients
Neurology

by decreasing hepatic gluconeogenesis and in- with a CD4+ cell count <100/mm³. The most
creasing peripheral utilization of glucose. common manifestation of toxoplasmosis is
seizures and headache, although other focal
22. The correct answer is C. This patient is pre- neurological deficits may be seen. The clas-
senting with progressive multifocal leukoen- sic radiological picture is one or more ring-­
cephalopathy (PML), which is caused by the enhancing lesions with surrounding edema.
JC virus in patients with AIDS. PML is a re-
activation of a dormant virus to which the pa- 23. The correct answer is D. Although cognitive
tient has previously been exposed. Initial find- decline is common to all forms of dementia,
ings include neurological deficits of speech, the patient’s increased impulsivity and hyper-
memory, and coordination. Vision problems oral habits are suggestive of a frontotemporal
are also common. The disease causes a very dementia, such as Pick disease. This diagnosis
rapid decline in neurological function result- is confirmed by the observation of intracyto-
ing in coma and death. The three-week course plasmic, silver-staining Pick bodies on autopsy
for this patient is not uncommon. The dis- and occasional balloon neurons. The loss of
ease causes multiple areas of demyelination neurons in the frontal lobe causes extreme
throughout the white matter of the CNS. changes in personality and a decline in execu-
There is no specific treatment for PML, but tive functioning. Movement disorders are less
some patients have shown some clinical im- prominent compared to the other dementia di-
agnoses listed here.
Chapter 13: Neurology  •  Answers 359

Answer A is incorrect. Creutzfeldt-Jakob dis- testing, and Kayser-Fleischer rings are present
ease is characterized by spongiform changes in the iris of the eye.
throughout the cerebral cortex. It is a rapidly
progressive neurodegenerative disease charac- 24. The correct answer is E. A midshaft fracture
terized by severe dementia, myoclonus, and of the humerus can cause injury to the struc-
ataxia. Death generally occurs within 6-12 tures found in the radial groove, which are
months of the onset of symptoms. This disease the radial nerve and the deep brachial artery.
is believed to be caused by infectious prions The radial nerve is known as the great exten-
derived from aberrant proteins. sor nerve. Radial nerve injury results in “wrist
drop,” an inability to extend the wrist and
Answer B is incorrect. In dementia with Lewy
metacarpophalangeal joints of all digits.
bodies, the patient’s cognitive decline is fre-
quently combined with extrapyramidal symp- Answer A is incorrect. The long thoracic

High-Yield Systems
toms reminiscent of Parkinson disease, and nerve innervates the serratus anterior muscle,
visual hallucinations. As in Pick disease, intra- and can be damaged in breast surgery through
cytoplasmic inclusions (Lewy bodies) can be injury to the axilla or lateral wall of the thorax.
observed histologically. Dementia with Lewy The result is a “winged scapula,” or protrusion
bodies can also present with a fluctuating of the scapula from the back when the person
course, hallucinations, and prominent frontal pushes forward against resistance.
signs. This patient’s lack of parkinsonian move- Answer B is incorrect. The axillary nerve
ment abnormalities makes this diagnosis un- is damaged by injury to the surgical neck of
likely. the humerus or by anterior dislocation of the
Answer C is incorrect. Normal-pressure hy- shoulder. It innervates the deltoid muscles, of
drocephalus is characterized by the triad of which the middle fibers along with the supra-
urinary incontinence, progressive dementia, spinatus are responsible for arm abduction.
and ataxic gait (the “3 Ws:” Wet, Wacky, and Answer C is incorrect. The ulnar nerve can be
Wobbly). In this disorder, head imaging re- damaged by injury to the medial epicondyle of
veals enlarged ventricles, but lumbar puncture the humerus. Motor deficits include weakness
does not reveal markedly elevated intracranial in abduction and adduction of fingers, adduc-
pressure. Unlike most forms of dementia, it is

Neurology
tion of the thumb, and extension of fingers (re-
responsive to treatment, such as the placement sulting in “claw hand”).
of a shunt to remove excess CSF. Since this
patient has no ataxia or incontinence, this di- Answer D is incorrect. The median nerve
agnosis is very unlikely. is damaged by injury to the distal end of the
humerus in the supracondylar area. Median
Answer E is incorrect. Wilson disease is an nerve injury results in the inability to flex fin-
autosomal recessive disorder of copper metab- gers and abduct and oppose the thumb, as well
olism that can cause dementia and psychotic as pain or paresthesia over the palmar side of
symptoms as a result of accumulation of cop- the thumb, index, middle finger, and half of
per in different parts of the body. Because the the ring finger. Atrophy of the thenar muscles
putamen is particularly affected by the toxic may also occur. Similar changes are seen with
accumulation of copper, extrapyramidal symp- carpal tunnel syndrome, in which the median
toms are common. The finding would be nerve is compressed between the flexor ten-
an accumulation of copper in the putamen; dons and the flexor retinaculum.
however, it would be unlikely for this disease
to present so late in life. The gene for Wilson 25. The correct answer is D. This patient most
disease is located on chromosome 13, and this likely has an Erb palsy, which is characterized
disease typically presents in childhood. There by the “waiter’s tip” position of the affected
is a decrease in ceruloplasmin on laboratory upper extremity. It is due to downward com-
pression of the shoulder and damage to the C5
360 Section II: Organ Systems  •  Answers

and C6 nerve roots of the brachial plexus. Of odicity of the headaches precludes the regular
the choices, the teres minor is innervated by administration of analgesics, which would be
the axillary nerve (C5, C6) and therefore likely necessary for the consideration of this diagno-
to be paralyzed in this patient. sis.
Answer A is incorrect. The flexor carpi ulnaris Answer C is incorrect. Migraine headaches
is innervated by the ulnar nerve (C7, C8) and are typically preceded by prodromal symptoms
is not affected in an Erb palsy. and can also be bilateral in nature. Typically,
the headaches increase in severity and can last
Answer B is incorrect. The flexor digitorum
from 10-12 hours. They may be associated with
superficialis is innervated by the median nerve
nausea and vomiting. Frequently such patients
(C7, C8, T1) and is not affected in an Erb
have a family history of migraine. Effective
palsy.
treatment involves use of triptans as an abor-
High-Yield Systems

Answer C is incorrect. The latissimus dorsi tive agent and b-blockers for prophylaxis.
is innervated by the thoracodorsal nerve (C6,
Answer D is incorrect. A headache induced
C7, C8) and is not affected in an Erb palsy.
by temporomandibular joint dysfunction syn-
Answer E is incorrect. The trapezius is in- drome frequently presents with unilateral ear
nervated by the spinal accessory nerve (cranial or auricular pain radiating to the jaw. The pain
nerve XI) and is not affected in an Erb palsy. is deep and continuous, is most severe in the
morning, and can be associated with jaw dys-
26. The correct answer is A. This patient is suffer- function. Treatment is aimed at the underlying
ing from cluster headaches, which are repeti- joint malfunction.
tive headaches that occur for weeks to months
at a time, with intervening periods of remis- Answer E is incorrect. Tension headaches
sion. Men are affected more than women, with are the most common headache syndrome
a peak incidence in persons 25-50 years old. but typically present with pain that is bifron-
Attacks begin without any prodromal symp- tal, “squeezing,” and constant. They may be
toms (such as the vision changes characteristic accompanied by nausea, but not usually by
of migraines), typically around the eye or tem- either vomiting or photophobia, and are not
ple, and are excruciating. They are always uni- preceded by prodromal symptoms. Acetami­
Neurology

lateral and may last for minutes to hours, with nophen and nonsteroidal anti-inflammatory
a mean duration of 45 minutes. In contrast to drugs are typically effective for relief.
patients with migraines, who prefer remaining
27. The correct answer is C. The vitamin B1
in a dark, quiet room, cluster headache pa-
(otherwise known as thiamine) deficiency in
tients typically prefer to stay active. Treatment
this patient, accompanied by his gait ataxia is
can be difficult because of the short duration
highly suggestive of Wernicke encephalopathy,
of symptoms, but effective options include
a serious disorder. The classic triad of Wer-
oxygen, intranasal lidocaine, and triptans. Pro-
nicke encompasses encephalopathy, ataxic
phylaxis may consist of treatment with predni-
gait, and some variant of oculomotor dysfunc-
sone, verapamil, or methysergide for one-two
tion, most notably nystagmus. However, all
months.
three features of the triad are recognized in
Answer B is incorrect. Medication-overuse only about one-third of cases. It is important
headaches are secondary to excessive use of to consider Wernicke encephalopathy in the
analgesics and may occur in patients who have setting of alcohol abuse or malnutrition and
tension, migraine, or cluster headaches. The acute confusion, decreased level of conscious-
diagnosis should be considered in patients who ness, ataxia, ophthalmoplegia, memory distur-
have frequent or daily headaches despite the bance, hypothermia with hypotension, or de-
use of medications. Although this patient is lirium tremens.
taking over-the-counter medications, the peri-
Chapter 13: Neurology  •  Answers 361

Answer A is incorrect. A fruity odor to the min B12 is ingested, it combines with intrin-
breath is indicative of diabetic ketoacidosis sic factor secreted by the parietal cells in the
(DKA), a pathophysiologic state of inadequate stomach. This complex is then absorbed in
insulin levels resulting in high blood sugar lev- the terminal ileum. It is likely that she avoided
els and accumulation of organic acids and ke- this deficiency for some time due to the large
tones in the blood. It is also common in DKA pool of vitamin B12 stored in the liver. Folate
to have severe dehydration and significant lab- deficiency also presents with a macrocytic,
oratory abnormalities, including hyperglyce- megaloblastic anemia with hypersegmented
mia, hypernatremia, and anion gap metabolic neutrophils. However, it is not associated with
acidosis. None of these laboratory abnormali- neurologic problems. In this patient, folate lev-
ties is evident in this patient. els would be expected to be normal, inasmuch
as loss of the terminal ileum does not affect the
Answer B is incorrect. The presence of mio-

High-Yield Systems
intestinal absorption of folate, and this patient
sis (constricted pupils), which is indicative of
has been eating a normal diet, which should
increased parasympathetic tone, may suggest
provide adequate folate levels.
opioid intoxication. This patient presents with
bradycardia and decreased respiratory rate, two Answer A is incorrect. These laboratory values
other signs suggestive of opioid use. However, are consistent with a microcytic anemia (ie,
the patient was not responsive to naloxone ad- iron deficiency, thalassemia, or lead poison-
ministration. In patients with heroin (or other ing). Microcytic anemias are not associated
opioid) intoxication, naloxone (an opioid-­ with subacute combined degeneration. They
receptor antagonist) rapidly reverses the effects can present with fatigue and pallor, and are
of opioid intoxication. most often due to some form of blood loss.
Answer D is incorrect. Pill-rolling tremor is Answer B is incorrect. These laboratory values
suggestive of Parkinson disease, a disorder of could be from a healthy patient or from some-
the basal ganglia caused by degeneration of one with a normocytic anemia such as anemia
dopaminergic neurons in the substantia nigra. of chronic disease, autoimmune hemolytic
These patients are usually >60 years old and anemia, or anemia following an acute hemor-
present with a shuffling gait, masked facies, rhage.
resting pill-rolling tremor, and bradykinesia.

Neurology
Answer C is incorrect. These laboratory values
This patient is rather young to have Parkinson
are consistent with folate deficiency. Folate de-
disease and does not demonstrate the classic
ficiency is not associated with the neurologic
syndrome associated with this disease.
symptoms observed in this patient, and this pa-
Answer E is incorrect. Shaking chills is indica- tient appears to be receiving adequate amounts
tive of bacteremia and sepsis. Although this pa- of folate in her diet. Folate deficiency is most
tient is hypotensive, a normal temperature and often seen with chronic alcohol use and mal-
WBC count do not indicate any evidence of nutrition.
infection.
Answer D is incorrect. These laboratory val-
ues are consistent with a macrocytic, nonmeg-
28. The correct answer is E. This patient presents
aloblastic anemia, which can result second-
with subacute combined degeneration, a neu-
arily to liver disease, hypothyroidism, or drugs
rologic condition associated with vitamin B12
that impair DNA synthesis. Macrocytic, non-
deficiency that leads to abnormal myelin. Vi-
megaloblastic anemias are not associated with
tamin B12 deficiency causes macrocytic, mega-
subacute combined degeneration.
loblastic anemia (mean corpuscular volume
>100 fL) with hypersegmented neutrophils on
29. The correct answer is B. This patient has
blood smear. This patient likely has an isolated
physical signs consistent with anorexia nervosa,
vitamin B12 deficiency secondary to surgical
most notably a low body mass index, bradycar-
resection of the terminal ileum. When vita-
dia, evidence of hypotension, lanugo, and con-
362 Section II: Organ Systems  •  Answers

comitant depression. Anorexia nervosa is a se- been studied rigorously. Mirtazapine is not
rious condition that requires intensive mental known to increase the risk of seizure in ano­
health care, as well as close medical monitor- rexic patients.
ing of weight, electrolyte levels, and hydration
Answer E is incorrect. Selegiline is a mono-
status. The mainstay of therapy is a combina-
amine oxidase (MAO) inhibitor most com-
tion of cognitive behavioral therapy and se-
monly used as an antidepressant; it is not typi-
lective serotonin reuptake inhibitors (SSRIs).
cally used to manage anorexia nervosa. MAO
Use of the antidepressant bupropion is con-
inhibitors are not known to increase the risk of
traindicated in patients with anorexia nervosa
seizure in anorexic patients.
because it increases the risk of seizure in this
population. 30. The correct answer is E. This patient shows
Answer A is incorrect. Amitriptyline is a tri- the classic signs of a functional anterior pitu-
High-Yield Systems

cyclic antidepressant (TCA). TCAs, although itary adenoma, namely bitemporal hemianop-
effective, are not first-line therapy in the man- sia and amenorrhea due to prolactin hyperse-
agement of anorexia nervosa, given the poten- cretion. These tumors are typically benign and
tial for cardiac adverse effects in anorexic pa- slow-growing. The anterior pituitary develops
tients already suffering from bradycardia and from Rathke pouch, which is composed of sur-
electrolyte abnormalities. TCAs are not known face ectoderm that abuts the sella turcica.
to increase the risk of seizure in anorexic pa- Answer A is incorrect. The gut tube epithe-
tients. lium and its derivatives originate from endo-
Answer C is incorrect. Fluoxetine is an SSRI derm.
most commonly used as an antidepressant. It Answer B is incorrect. Most of the body’s con-
has also been used to treat anorexia nervosa, nective tissues are derived from mesoderm.
although with questionable efficacy. SSRIs
are not known to increase the risk of seizure in Answer C is incorrect. The parafollicular
ano­rexic patients. cells of the thyroid are derived from the neural
crest.
Answer D is incorrect. Mirtazapine in an
atypical antidepressant that induces weight Answer D is incorrect. The posterior pituitary
Neurology

gain, which may be beneficial in patients with is derived from neuroectoderm.


weight control issues, although this has not yet
Chapter 14

Psychiatry

363
364 Section II: Organ Systems  •  Questions

Q u e st i o n s

1. A 27-year-old man with a history of panic disor- Physical examination reveals coarse tremors,
der and generalized anxiety disorder is brought facial flushing, palmar erythema, and spider
to the emergency department after being angiomas. His blood pressure is 145/95 mm
found unconscious in his room by his parents. Hg, his pulse is 115/min, and his temperature
He is lethargic and can barely be aroused. He is 38.3°C (100.9°F). Thiamine is adminis-
nods “yes” when asked if he has had any alco- tered. Which of the following drugs is indi-
hol and “yes” when asked if he has taken any cated for the treatment of this patient’s condi-
pills. His parents are sure the only pills in the tion?
house are those prescribed by his psychiatrist.
(A) Chlordiazepoxide
His vital signs are normal, and his pupils are
High-Yield Systems

(B) Disulfiram
dilated to 2 mm and normally reactive. His
(C) Haloperidol
blood alcohol level is 100 mg/dL. Results of
(D) Methadone
the urine toxicology screen are pending. All of
(E) Naltrexone
a sudden, his breathing slows and his oxygen
saturation drops significantly. What should the 4. A 24-year-old woman is brought to the emer-
physician give to treat this patient’s condition? gency department by ambulance after she
(A) Benztropine is found collapsed and unresponsive on the
(B) Flucytosine street. It is not known how long she was lying
(C) Flumazenil on the street. Physical examination reveals
(D) Naloxone constricted pupils and a heart rate of 55. Ad-
(E) Naltrexone ministration of which of the following drugs
would be most appropriate?
2. A 22-year-old man presents to his family physi-
(A) Chlordiazepoxide
cian with complaints of insomnia. He was re-
(B) Flumazenil
cently honorably discharged from the army af-
(C) Fomepizole
ter finishing an 18-month tour of duty in Iraq.
(D) N-acetylcysteine
He states that the insomnia began about seven
Psychiatry

(E) Naloxone
months ago after a fierce night-time battle. He
(F) Naltrexone
reports having nightmares and flashbacks of
(G) Phenobarbital
the battle and is easily startled by loud noises.
Which of the following pharmacologic agents, 5. After several failed trials of various antipsy-
along with psychotherapy, would be most ap- chotic drugs, a 46-year-old woman is switched
propriate to treat this patient’s condition? to a new medication for her schizophrenia.
(A) Buspirone However, a few weeks later, she develops
(B) Carbamazepine pneumonia. A complete blood count is or-
(C) Fluoxetine dered and reveals a significantly reduced num-
(D) Propranolol ber of neutrophils, basophils, and eosinophils.
(E) Trazodone Which of the following agents is the most
likely cause of this clinical picture?
3. A 42-year-old man presents to the local crisis
(A) Chlorpromazine
center requesting alcohol detoxification. He
(B) Clozapine
has a 20-year history of heavy drinking, with
(C) Haloperidol
the longest period of abstinence being four
(D) Risperidone
months. His last drink was two nights ago, and
(E) Thioridazine
he now complains of discomfort and anxiety.
Chapter 14: Psychiatry  •  Questions 365

6. A 60-year-old African-American man has been 8. A 19-year-old man is brought to the emergency
reclusive, rarely leaving his home for the past department by his friends after suffering a sei-
40 years. His family describes him as an emo- zure. He is sweating, paranoid, tachycardic,
tionally cold person with few friends. Growing and his pupils are dilated. His friends say that
up, he preferred solitary activities like reading he has a history of using illicit drugs. What is
to engaging in activities with others. Members the mechanism of action of the drug that is
of his church have delivered groceries to his causing the patient’s symptoms?
front door once a week for the past 20 years,
(A) Blocks NMDA receptors
but he never opens the door to greet them.
(B) Increases GABA activity by increasing the
Which of the following is the most likely diag-
duration of chloride channel opening
nosis?
(C) Prevents reuptake of norepinephrine, do-
(A) Avoidant personality disorder pamine, and serotonin by presynaptic

High-Yield Systems
(B) Paranoid personality disorder pumps
(C) Schizoid personality disorder (D) Prevents the fusion of the presynaptic vesi-
(D) Schizophrenia cle with the presynaptic surface membrane
(E) Schizophreniform disorder (E) Prevents the uptake of acetylcholine at
(F) Schizotypal personality disorder cholinergic synapses

7. A 27-year-old man is brought to the emer- 9. A 45-year-old man who has received long-term
gency department by ambulance. Paramedics treatment for schizophrenia recently has been
report that he was found sitting on the side- displaying involuntary facial movements that
walk speaking as though engaged in a heated include lateral deviations of the jaw and “fly
argument, but nobody else was around. They catching” motions of the tongue. Which of the
say that the patient appeared to be in distress following agents is the most likely cause of his
and that he was quite disheveled. The man is involuntary movements?
evaluated by a psychiatrist, admitted to the hos-
(A) Clozapine
pital, and started on a medication to treat his
(B) Fluphenazine
symptoms. Two days later a medical student
(C) Lithium
notices that the patient has painful spasms in
(D) Selegiline

Psychiatry
his neck muscles. Which of the following is
(E) Ziprasidone
the most appropriate treatment for this man’s
condition? 10. A 28-year-old man who has been experienc-
(A) Benztropine ing delusions, hallucinations, and thought
(B) Dantrolene disorders for the past six months now begins
(C) Diazepam to display flattening affect, lack of motivation,
(D) Fluphenazine and social withdrawal. Which of the following
(E) Prochlorperazine agents would address his newest symptoms?
(A) Olanzapine
(B) Haloperidol
(C) Lithium
(D) Fluphenazine
(E) Phenelzine
366 Section II: Organ Systems  •  Questions

11. A 35-year-old man with depression has been thickened and hyperpigmented. A similar le-
treated with medication for the past seven sion is seen on the neck. The patient states
years. Recently he began seeing a new psy- that she has recently began to have diarrhea,
chiatrist who suggested changing this medica- and at times during the interview she forgets
tion to a newer class of antidepressants that has what she was saying. For the past six months
proven effective for many of her patients. Two she has been following a new low-calorie
weeks later he presents to the emergency de- fad diet. A defect in the absorption of which
partment because of flushing, diarrhea, sweat- amino acid would cause similar symptoms?
ing, and muscle rigidity. During the physical
(A) Arginine
examination, he admits that he was a bit sus-
(B) Histidine
picious of the new medication he was given
(C) Phenylalanine
to treat his depression since he was told he
(D) Tryptophan
High-Yield Systems

no longer needed to avoid certain foods. He


(E) Tyrosine
decided to use both medicines just to “make
sure” the new one was working. Which of the 14. A 28-year-old woman presents to her pri-
following medications did the new doctor most mary care provider complaining of difficulty
likely prescribe for this patient? sleeping. Although she reports trouble falling
(A) Lithium asleep despite waking up “before the sun” ev-
(B) Nortriptyline ery morning, her major complaint is awaken-
(C) Phenelzine ing from sleep multiple times each night. She
(D) Sertraline also complains of decreased energy and moti-
(E) Trazodone vation to complete tasks at work. Polysomnog-
raphy reveals >25% of total sleep time is spent
12. A 20-year-old man is seen by a physician for in REM sleep and <25% of total sleep time is
the third time in three months. At the first spent in stages 3 and 4 sleep. Which of the fol-
visit he was brought to the emergency depart- lowing is the most appropriate treatment?
ment by his mother after swallowing toilet
(A) Avoidance of caffeine
bowl cleaner. He told the doctor that he took
(B) Continuous positive airway pressure
the cleaning product to “cleanse his body from
(C) Fluoxetine
Psychiatry

the aliens” that had “forced their entry” and


(D) Methylphenidate
“possessed” him. Today the patient appears un-
(E) No intervention is necessary; these are nor-
clean and disheveled, and his mother reports
mal polysomnographic findings in a young
that he has become progressively withdrawn
adult
and expressionless. Four months ago the pa-
tient witnessed the gruesome death of his fa- 15. A 23-year-old man is brought to the emergency
ther in a drive-by shooting incident. Prior to department because his friends heard him say
this incident, he had a normal and healthy life. that he was talking to president Kennedy about
Which of the following is the most likely diag- a secret spy mission in the Soviet Union. He
nosis? appears quite anxious, agitated, and restless.
(A) Factitious disorder Physical examination reveals dilated pupils.
(B) Schizophreniform disorder His pulse is 80/min and his blood pressure is
(C) Schizophrenia 120/80 mm Hg. What is the most likely cause
(D) Schizoaffective disorder of his symptoms?
(E) Shared delusional disorder (A) Alcohol
(B) Cocaine
13. A 43-year-old woman comes to the clinic with
(C) Lysergic acid diethylamide
complaints of pruritus and burning of both
(D) Marijuana
forearms that initially looked like sunburn. On
(E) Phencyclidine
physical examination the affected skin appears
Chapter 14: Psychiatry  •  Questions 367

16. The parents of an 8-year-old boy bring him to


see a psychiatrist because they are frustrated
5-HT
with his behavior. In the last two years he has 5-HT
become increasingly restless and moody, inter-
rupts other children in the classroom, and of- 5-HT Serotonin transport protein
ten runs into the street without looking out for
cars first. The psychiatrist prescribes a medica-
tion that works through which of the following
mechanisms? 5-HT receptor

(A) Increases release of norepinephrine


(B) Inhibits acetylcholine activity
(C) Inhibits reuptake of serotonin Reproduced, with permission, from USMLERx.com.

High-Yield Systems
(D) Stimulates dopamine receptors
(E) Stimulates serotonin receptors
(A) Bipolar disorder
17. A 15-year-old girl is brought to the emergency (B) Delerium tremens
department by her mother after experiencing (C) Dissociative identity disorder
a first-time seizure. The thin-appearing girl (D) Obsessive-compulsive disorder
has a heart rate of 55/min, signs suggestive of (E) Schizophrenia
dehydration, and fine, velvety hair covering
her arms and legs. The physician calculates 19. An 18-year-old woman complains of weakness,
her body mass index to be 16.4 kg/m². When fatigue, decreased appetite, and insomnia over
the patient’s mother leaves the room for a the past month. She is no longer interested in
moment, the patient admits to the physician her favorite activities, and has been unable to
that she has been feeling depressed recently concentrate in school. She also reports feel-
and that for the past week she has been self- ing guilty about not hanging out with her
medicating with normal daily doses of one of friends even though they ask her out almost
her friend’s antidepressant medications. What every weekend. As part of her treatment plan,
antidepressant is the patient most likely taking? her physician prescribes a medication. On

Psychiatry
a follow-up visit, she reports that her mood
(A) Amitriptyline
has improved, but she now feels that her face
(B) Bupropion
flushes more frequently and she is more sensi-
(C) Fluoxetine
tive to the hot weather outside. She is also wor-
(D) Mirtazapine
ried that at times she feels like her heart is rac-
(E) Selegiline
ing. On further questioning, she admits to some
constipation. Which of the following drugs was
18. The image depicts a biochemical pathway oc-
most likely prescribed for this patient?
curring in the nervous system. An “X” marks
the effect of a certain class of medications on (A) Amitriptyline
this pathway. For which condition is this class (B) Clonazepam
of medications an effective first-line treatment? (C) Lithium
(D) Sertraline
(E) Venlafaxine
368 Section II: Organ Systems  •  Questions

20. A 20-year-old woman is brought to the emer- (A) Activating an enzyme responsible for the
gency department by her roommate because termination of a drug’s inactivation
she was “walking funny,” had difficulty breath- (B) Amplifying the effect of an endogenous
ing, and slurred her speech. She was recently neurotransmitter by inhibiting its break-
diagnosed and given medication for panic dis- down
order. Her blood pressure is 110/75 mm Hg, (C) Displacement of the drug from its binding
pulse is 58/min, and respiratory rate is 8/min. site
She is afebrile. Her mucous membranes are (D) Inhibiting the formation of a toxic metabo-
moist and pupil size is normal. Serum labora- lite
tory studies are negative for evidence of etha- (E) Inhibiting the storage of a neurotransmitter
nol, organophosphate, or opioid ingestion. The
agent that would be used to reverse the effects
High-Yield Systems

of the patient’s anxiety medication works by


which of the following mechanisms?
Psychiatry
Chapter 14: Psychiatry  •  Answers 369

An s w e r s

1. The correct answer is C. This patient is ex- Answer A is incorrect. Buspirone is a partial
hibiting symptoms of central nervous system agonist at the 5-HT1S receptor that is com-
(CNS) depression that cannot be explained monly used as an alternative to benzodiaz-
by his blood alcohol level alone (this level of epines in the treatment of generalized anxiety
CNS depression would typically be seen in a disorder. Although its onset of action is slower
nonchronic drinker at blood alcohol levels of than that of benzodiazepines, it does not po-
250-300 mg/dL). It is reasonable, based on his tentiate the CNS depression of alcohol, and
psychiatric diagnoses, to think he may have in- has little potential for abuse and addiction.
gested a benzodiazepine along with the alco-
Answer B is incorrect. Carbamazepine is an
hol, resulting in a synergistic effect. To reverse

High-Yield Systems
anticonvulsant medication that also can be
the effect of the benzodiazepine, the drug of
used as a mood stabilizer in bipolar mood dis-
choice is flumazenil, a competitive GABA an-
order.
tagonist.
Answer D is incorrect. Propranolol is a non-
Answer A is incorrect. Benztropine is a cen-
specific b-blocker. It is useful in the treatment
trally acting anticholinergic agent that acts as
of panic disorder and simple phobia.
an acetylcholine receptor antagonist. It is used
to treat parkinsonism as well as extrapyramidal Answer E is incorrect. Trazodone is a het-
and dystonic reactions. It is not used to treat erocyclic antidepressant with sedative quali-
benzodiazepine overdose. ties that is used in the treatment of depression
complicated by insomnia. It works by inhibit-
Answer B is incorrect. Flucytosine is a potent
ing serotonin reuptake, but also acts as a par-
antifungal.
tial serotonin agonist. Male patients should be
Answer D is incorrect. Naloxone is used to re- warned of its potential to cause priapism.
verse opioid overdose.
3. The correct answer is A. This patient is show-
Answer E is incorrect. Naltrexone, an opioid
ing signs of alcohol withdrawal, manifested by
receptor antagonist, is used to treat opiate ad-
tachycardia, fever, nausea, vomiting, tremors,

Psychiatry
diction.
and hypertension, and is at risk for delirium
tremens. Delirium tremens is an extreme and
2. The correct answer is C. This patient meets
life-threatening form of withdrawal character-
the criteria for diagnosis of posttraumatic stress
ized by perceptual disturbances and confu-
disorder (PTSD). He has experienced an event
sion. Intravenous benzodiazepines, such as
that involved actual death of or threatened
chlordiazepoxide, are indicated in the treat-
death to self or others; the traumatic event is
ment of both mild withdrawal and delirium
persistently re-experienced through nightmares
tremens. Used early, they can prevent progres-
and flashbacks; he suffers from insomnia; and
sion to withdrawal-induced seizures, psychosis,
he has an exaggerated startle response. Other
and coma. Chlordiazepoxide is a long-acting
symptoms of PTSD include difficulty concen-
benzodiazepine that works via stimulation of
trating, hypervigilance, and dissociative symp-
GABA receptors. Other drugs in the same class
toms. In PTSD, symptoms are present for lon-
include lorazepam, oxazepam, and diazepam,
ger than one month, whereas in acute stress
each of which could be used in this scenario.
disorder, symptoms last between two days and
one month. Selective serotonin reuptake in- Answer B is incorrect. Disulfiram inhibits ac-
hibitors such as fluoxetine are first-line medi- etaldehyde dehydrogenase, which causes ac-
cations for the treatment of PTSD. Side effects cumulation of acetaldehyde with ingestion of
include nausea, headache, anxiety, agitation, alcohol. This buildup of alcohol byproducts
insomnia, and sexual dysfunction. leads to extremely unpleasant adverse effects,
370 Section II: Organ Systems  •  Answers

including flushing, headache, diaphoresis, ethylene glycol poisoning. There is no evi-


nausea, and vomiting. This drug is given to al- dence that this patient ingested methanol or
coholics to help them maintain sobriety. ethylene glycol.
Answer C is incorrect. Haloperidol is a typi- Answer D is incorrect. N-acetylcysteine is
cal antipsychotic. It can be used in patients used in cases of acetaminophen poisoning.
withdrawing from alcohol who suffer psychotic It also has an indication for relieving mucus
symptoms such as hallucinations. thickening in cystic fibrosis patients.
Answer D is incorrect. Methadone is a potent, Answer F is incorrect. Naltrexone, like nalox-
long-acting opioid agonist used in the treat- one, is an opioid receptor antagonist; however,
ment of opioid addiction. This patient is an it is not indicated for reversal of acute opioid
alcoholic and does not require a methadone overdose. Naltrexone is more commonly used
High-Yield Systems

taper. to treat alcohol and opioid dependence.


Answer E is incorrect. Naltrexone is an opi- Answer G is incorrect. Phenobarbital is a
oid antagonist used to help maintain opioid long-acting barbiturate useful in patients with
sobriety. It also is used to help prevent alcohol seizure disorders.
relapses in alcohol dependence. This patient,
however, requires acute care, not maintenance 5. The correct answer is B. Clozapine is an
treatment. atypical antipsychotic used to treat schizophre-
nia that is refractory to traditional therapy. It is
4. The correct answer is E. This patient has signs considered atypical because it blocks serotonin
indicating opioid overdose: she is comatose receptors, in addition to the dopamine block-
with miosis and bradycardia. Naloxone, an ade common to all typical antipsychotics. This
opioid antagonist given intravenously, should dual action may be useful in the treatment of
quickly reverse the effects of the overdose. the positive and negative symptoms of schizo-
phrenia. Perhaps the most dangerous adverse
Answer A is incorrect. Chlordiazepoxide is a
effect of clozapine is bone marrow suppres-
long-acting benzodiazepine used in the man-
sion, specifically agranulocytosis. This neces-
agement of alcohol withdrawal. Though pa-
sitates frequent monitoring of the WBC count
tients with alcohol intoxication can become
Psychiatry

for all patients who are started on this drug. A


unresponsive, alcohol tends to cause pupil-
sudden increase in infections or bouts of ill-
lary dilation, not constriction as seen with this
ness in a patient on clozapine should raise
patient. Additionally, there is no mention of
concern about the development of agranulocy-
the smell of alcohol on the patient’s breath or
tosis. If laboratory tests indicate this is the case,
clothing, which can be a clue to alcohol intox-
the drug must be discontinued immediately
ication in an unresponsive patient.
and the patient should be monitored carefully.
Answer B is incorrect. Flumazenil is an an-
Answer A is incorrect. Chlorpromazine, a tra-
tagonist at benzodiazepine receptors and is
ditional antipsychotic, has a adverse-effect pro-
used to reverse benzodiazepine intoxication.
file similar to that of haloperidol. Agranulocy-
Though benzodiazepines can cause pupillary
tosis can occur with its use, but occurs much
changes, respiratory depression along with hy-
more commonly as an adverse effect of cloza­
potension are more likely to be noted in a pa-
pine.
tient with overdose.
Answer C is incorrect. Haloperidol is a tradi-
Answer C is incorrect. Fomepizole is an in-
tional antipsychotic that acts by blocking do-
hibitor of alcohol dehydrogenase, and is used
pamine receptors and is not associated with
to prevent the conversion of ethylene glycol
agranulocytosis. It is best known for causing
and methanol to the toxic substances oxalic
extrapyramidal adverse effects.
acid and formic acid, respectively. Thus it is
mainly used as an antidote for methanol or
Chapter 14: Psychiatry  •  Answers 371

Answer D is incorrect. Risperidone, another trast, patients with schizoid personality disorder
atypical antipsychotic agent, has a mechanism have intact reality testing.
of action similar to that of clozapine. It does
Answer E is incorrect. Schizophreniform dis-
not, however, produce the adverse effect of
order is identical to schizophrenia except that
agranulocytosis. Significant adverse effects of
symptoms last for at least one month but less
risperidone include QT-interval prolongation
than six months. Patients with schizophreni-
and metabolic aberrations.
form disorder have a better prognosis than do
Answer E is incorrect. Thioridazine is a tradi- most patients with schizophrenia, and may re-
tional antipsychotic that has an adverse-effect turn to their baseline mental functioning.
profile similar to that of haloperidol and chlor-
Answer F is incorrect. Schizotypal and schiz-
promazine. Although agranulocytosis is pos-
oid personality disorder are rather similar, but
sible, it is much less frequent than with cloza­

High-Yield Systems
the former is distinguished in that these pa-
pine.
tients tend to be more similar to schizophren-
ics. Patients with schizotypal disorder are
6. The correct answer is C. This man has schiz-
strikingly odd, with peculiar notions, ideas of
oid personality disorder, marked by a lifelong
reference, illusions, magical thinking, and de-
pattern of social withdrawal. Patients with this
realization.
disorder experience discomfort with human
interaction, so they avoid close relationships,
7. The correct answer is A. The patient has clas-
and engage in solitary activities. These patients
sic signs of schizophrenia and was likely given
often are viewed as eccentric, isolated, lonely,
haloperidol, a typical antipsychotic agent
and emotionally cold. Unlike those with other
that acts by blocking dopamine receptors.
cluster A personality disorders, which are
Haloperidol has a high affinity for the D2-
schizotypal and paranoid, those with schizoid
dopaminergic receptor. The patient is expe-
personality disorder are not more likely to have
riencing an acute dystonic reaction soon after
relatives with schizophrenia. Men are twice as
receiving the medication. The painful muscle
likely as women to be affected.
spasm of the neck is known as torticollis. This
Answer A is incorrect. Avoidant patients are acute extrapyramidal adverse effect is the result
like schizoid patients in their pervasive pattern of unopposed cholinergic activity in the CNS

Psychiatry
of social inhibition. However, they do desire following blockade of dopaminergic transmis-
companionship; an intense fear of rejection sion. The treatment for this adverse effect is
leads to avoiding any situation where there is a initiation of an anticholinergic agent such as
perceived risk of rejection. Think avoidant per- benztropine.
sonality disorder in patients inhibited by feel-
Answer B is incorrect. Dantrolene is effective
ings of inadequacy and social ineptness to the
in the treatment of neuroleptic malignant syn-
extent they will participate socially only when
drome. Dantrolene acts by preventing the re-
they are certain to be liked.
lease of calcium from the endoplasmic reticu-
Answer B is incorrect. Patients with paranoid lum.
personality disorder tend to be more socially
Answer C is incorrect. Diazepam can be used
engaged than those with schizoid personal-
as an hypnotic, a sedative, an anticonvulsant,
ity disorder, even though they have a lifelong
and a muscle relaxant. As a muscle relaxant,
history of suspiciousness and mistrust of other
diazepam is used to treat chorea, an involun-
people. Examples include recurrent suspicion
tary abnormal movement disorder or dyskine-
of a sexual partner’s fidelity or blaming others
sia that is a hallmark of Huntington disease.
for their problems.
Answer D is incorrect. Fluphenazine, like hal-
Answer D is incorrect. Patients with schizo-
operidol, can induce potent D2-dopaminergic
phrenia exhibit a formal thought disorder with
receptor blockade. It is a high-potency typical
hallucinations, or delusional thinking. In con-
372 Section II: Organ Systems  •  Answers

antipsychotic that is sometimes used as an al- commonly with use of older, typical antipsy-
ternative to haloperidol for patients suffering chotic medications such as fluphenazine and
from schizophrenia or bipolar disorder. Admin- haloperidol.
istration of fluphenazine will likely exacerbate
Answer A is incorrect. Clozapine, an atypical
this patient’s symptoms rather than alleviate
antipsychotic that modulates both serotonin-
them.
ergic and dopaminergic neurons in the CNS,
Answer E is incorrect. Prochlorperazine is a has a relatively low risk of inducing tardive dys-
typical antipsychotic agent with potent antido- kinesia. The most concerning adverse effect of
paminergic effects. It can also be used to treat clozapine is agranulocytosis, which can be fa-
nausea because of its weak anticholinergic and tal if left untreated.
antihistaminic effects. In this case it would be
Answer C is incorrect. Lithium is a mood sta-
of no benefit for the patient, and could make
High-Yield Systems

bilizer that is used primarily to treat episodes


his symptoms worse.
of mania in patients with bipolar disorder. Ad-
verse effects of lithium include nephrogenic
8. The correct answer is C. The patient’s symp-
diabetes insipidus, nausea, anorexia, and mild
toms are caused by cocaine. Cocaine pre-
diarrhea.
vents the reuptake of norepinephrine, dopa-
mine, and serotonin by presynaptic transporter Answer D is incorrect. Selegiline is a mono-
pumps in the central and peripheral nervous amine oxidase B inhibitor that is used to treat
systems. Parkinson disease by decreasing the breakdown
of dopamine. It has no role in the treatment
Answer A is incorrect. Phencyclidine causes
of schizophrenia. Selegiline has the opposite
aggressive and impulsive behavior, nystagmus,
effect of antipsychotics; it blocks the effects of
and tachycardia. It acts as an NMDA receptor
dopamine. Adverse effects of selegiline include
antagonist.
gastrointestinal (GI) upset, nausea, heartburn,
Answer B is incorrect. Barbiturates cause and dry mouth.
res­
piratory depression and act by increasing
Answer E is incorrect. Ziprasidone, like clo-
GABA activity by increasing the duration of
zapine, is an atypical antipsychotic and has a
chloride channel opening.
lower incidence of tardive dyskinesia when
Psychiatry

Answer D is incorrect. Bretylium and guan­ compared to typical antipsychotics. More often
ethidine prevent the fusion of presynaptic ves- than other atypical antipsychotics, it has been
icles with the presynaptic membrane, resulting associated with QT prolongation and the risk
in an inhibition of the release of norepineph- of malignant ventricular arrhythmias.
rine into the synapse.
10. The correct answer is A. Olanzapine is an
Answer E is incorrect. Cocaine works on nor-
atypical antipsychotic that blocks both sero-
adrenergic neurons; it does not work on cho-
tonin and dopamine receptors. Drugs in this
linergic neurons.
class are noted for their ability to treat both
9. The correct answer is B. This patient is dis- positive symptoms of schizophrenia (ie, hal-
playing signs of tardive dyskinesia, a compli- lucinations and delusions) and negative symp-
cation of long-term antipsychotic use thought toms (ie, blunted affect and social withdrawal).
to be the result of increased dopamine recep- With the onset of negative symptoms, addition
tor synthesis in response to long-term receptor of an atypical antipsychotic such as olanzap-
blockade by antipsychotics. Abnormal move- ine, can effectively treat both positive and neg-
ments such as tongue-thrusting and jaw devia- ative symptoms of this disorder.
tions, as seen in this patient, are the result of Answer B is incorrect. Haloperidol, another
relative dopamine excess affecting motor path- typical agent, would be less effective at mitigat-
ways. This complication is encountered more
Chapter 14: Psychiatry  •  Answers 373

ing the patient’s negative symptoms compared antidepressant, lithium prescribed as mono-
to olanzapine. therapy for depression is not recommended.
Answer C is incorrect. Lithium is a mood Answer B is incorrect. Nortriptyline is a tricy-
stabilizer that is used to treat the acute manic clic antidepressant associated with the “3 Cs:”
phases of bipolar disorder. It is not used in the Convulsions, Coma, and Cardiotoxicity (con-
treatment of schizophrenia and thus would duction defects and arrhythmias). Tricyclic
have no effect on this patient’s negative symp- antidepressants primarily have anticholinergic
toms. adverse effects as well, including dry mouth,
mydriasis, constipation, and urinary retention.
Answer D is incorrect. Fluphenazine is a typi-
cal antipsychotic that blocks only dopamine Answer C is incorrect. Phenelzine is an MAO
receptors. Agents in this class are more effec- inhibitor. There is no evidence that two MAO

High-Yield Systems
tive at mitigating positive symptoms of schizo- inhibitors lead to serotonin syndrome when
phrenia, but are less effective at relieving the taken together. Adverse effects of phenelzine
negative symptoms such as flattened affect and include postural hypotension, headache, dry
catatonia. mouth, sexual dysfunction, weight gain, and
sleep disturbances.
Answer E is incorrect. Phenelzine, a mono-
amine oxidase (MAO) inhibitor, is used to Answer E is incorrect. Trazodone is a hetero-
treat depression in patients who are unrespon- cyclic associated with sedation, nausea, pria-
sive to tricyclic antidepressants or who experi- pism, and postural hypotension.
ence concomitant anxiety. Such agents are not
used to treat schizophrenia. 12. The correct answer is B. Over the course of
two visits, the patient has exhibited psychotic
11. The correct answer is D. This patient has and residual symptoms characteristic of schizo-
likely been taking a MAO inhibitor for the phrenia and related disorders. A diagnosis of
past seven years since he was told he had to schizophrenia, however, requires active phase
avoid certain foods. Sertraline is a selective (“positive”) symptoms, and may include “nega-
serotonin reuptake inhibitor (SSRI) that can tive” ones as well, over a period of >6 months.
lead to serotonin syndrome when taken in con- In this case, the patient’s symptoms have lasted

Psychiatry
junction with MAO inhibitors. Serotonin syn- <4 months, and were potentially incited by a
drome is the result of excess serotonin in the traumatic event and its repercussions. If the
nervous system and is characterized by mental symptoms had lasted <1 month, a diagnosis
status changes, autonomic changes (eg, fever, of brief psychotic disorder would be accurate;
diaphoresis, tachycardia), and neuromuscular in such a diagnosis, most patients make a full
changes (eg, tremor or rigidity). The treatment recovery. In this patient, symptoms with a du-
of serotonin syndrome consists of prompt dis- ration of >1 month but <6 months yield a di-
continuation of the implicated agent(s) and agnosis of schizophreniform disorder. Negative
supportive care including intravenous fluids, symptoms, as seen here, worsen the prognosis
benzodiazepines for control of delirium, cool- of a patient with schizophreniform disorder.
ing measure for hyperthermia, and neuromus-
Answer A is incorrect. There is no evidence
cular blockers such as dantrolene for hyper-
that either the patient or his mother is actively
thermia, muscle rigidity, and the prevention of
seeking the attention of medical personnel, or
rhabdomyolysis.
that the symptoms experienced were falsified
Answer A is incorrect. Lithium is typically for secondary gain of tangible items such as
used to treat bipolar disorder. Its use has been food, shelter, or money, as would be the case
associated with tremor, hypothyroidism, and in malingering.
nephrogenic diabetes insipidus. While lithium
Answer C is incorrect. Explicit in the diagno-
is considered an effective adjunctive therapy
sis of schizophrenia is the presence of “posi-
for depression in combination with a second
374 Section II: Organ Systems  •  Answers

tive” and (but not always) “negative” symptoms Answer C is incorrect. Phenylalanine is a pre-
of >6 months’ duration. Many patients with a cursor of tyrosine, dopamine, norepinephrine,
prior diagnosis of schizophreniform disorder and epinephrine.
eventually receive a diagnosis of schizophrenia.
Answer E is incorrect. Tyrosine is a precursor
Answer D is incorrect. The diagnosis of a of dopamine, norepinephrine, and epineph-
schizoaffective disorder requires the symp- rine.
toms of schizophrenia (often both “positive”
and “negative” symptoms) as well as those of a 14. The correct answer is C. Depression is often
mood disorder (ie, depression, mania). These associated with disrupted sleep. Specifically,
patients typically have less cognitive impair- sleep studies performed in patients with de-
ment than those with strict psychotic disorders. pression reveal increased time spent in REM
sleep, decreased REM latency, and decreased
High-Yield Systems

Answer E is incorrect. Delusions are fixed,


delta waves, which are characteristic of stages
false beliefs or ideas by a patient that are not
3 and 4 sleep. Patients with depression often
shared by other individuals. Delusional dis-
experience decreased daytime energy and
order refers to a pathologic state whereby
motivation to complete tasks; these are com-
construct(s) of delusions impair one’s social
monly misdiagnosed as side effects of poor
and/or cognitive functioning. Shared delusions
sleep rather than warning signs of depression.
are those transmitted from one person to an-
Fluoxetine, an SSRI, is an appropriate first-line
other in a parent-to-child or spouse-to-spouse
agent for patients with depression. This pro-
relationship. There is no evidence presented
vider should also consider referral to a psychi-
that the patient’s mother shares the false beliefs
atrist for management of this patient’s depres-
of her son.
sion.
13. The correct answer is D. The patient has pel- Answer A is incorrect. Avoidance of caffeine
lagra due to niacin (vitamin B3) deficiency. is particularly helpful in patients with insom-
Niacin is found in unrefined and enriched nia, defined as difficulty falling asleep or stay-
grains, cereal, milk, and lean meats. Niacin ing asleep three times per week for at least one
is required for adequate cellular function and month. Although avoidance of caffeine may
metabolism as an essential component of help this patient with her sleep disturbances,
Psychiatry

nicotinamide adenine dinucleotide and nico- her specific sleep patterns indicate concern for
tinamide adenine dinucleotide phosphate. depression. Thus, further evaluation and treat-
Because cellular functions in multiple organs ment with an SSRI should be considered.
and tissues are impacted by niacin deficiency,
Answer B is incorrect. Continuous positive
there is a systemic clinical expression of pel­
airway pressure would be helpful in a patient
lagra involving the skin, GI tract, and CNS.
suffering from sleep apnea, which occurs
The symptoms of pellagra progress through the
when a patient briefly stops breathing during
3 D’s: Dermatitis, Diarrhea, and Dementia. If
the night and awakens from sleep. Patients
untreated, it can result in the fourth D: Death.
with severe sleep apnea might awaken hun-
Pellagra is Italian for thickened skin, and it is
dreds of times per night. This patient’s poly-
usually seen in sun-exposed areas of the body.
somnographic tracing is not consistent with
Since niacin is derived from tryptophan, a de-
sleep apnea.
crease in tryptophan absorption or an increase
in tryptophan metabolism can produce similar Answer D is incorrect. Methylphenidate is
symptoms. used to treat narcolepsy, or sudden sleep at-
tacks during the day despite normal nighttime
Answer A is incorrect. Arginine is a precursor
sleep.
of creatine, urea, and nitric oxide.
Answer E is incorrect. Normal sleep patterns
Answer B is incorrect. Histidine is a precursor
in young adults include 25% of total sleep
of histamine.
Chapter 14: Psychiatry  •  Answers 375

time spent in REM sleep and 25% of total sion, anxiety, and obsessive-compulsive disor-
sleep time is spent in delta wave (stages 3 and der.
4) sleep.
Answer D is incorrect. Drugs that stimulate
dopamine receptors are used in the treatment
15. The correct answer is C. This patient is expe-
of Parkinson disease.
riencing hallucinations, delusions, and dilated
pupils, but very few observable behavioral Answer E is incorrect. Buspirone is a sero-
changes. These are symptoms consistent with tonin receptor agonist that is used in the treat-
lysergic acid diethylamide (LSD) abuse. LSD ment of anxiety.
is a hallucinogenic drug that can cause marked
anxiety, depression, nausea, weakness, and par- 17. The correct answer is B. This patient has
esthesias. physical signs consistent with anorexia nervosa,
most notably a low body mass index, bradycar-

High-Yield Systems
Answer A is incorrect. Alcohol abuse is char-
dia, evidence of hypotension, fine body hair
acterized by a general disinhibition, slurred
(called lanugo), and concomitant depression.
speech, and ataxia. It does not usually cause
Anorexia nervosa is a serious condition that re-
patients to hallucinate or become delusional.
quires intensive mental health care, as well as
Benzodiazepines can be used to prevent de-
close medical monitoring of weight, electrolyte
lirium tremens and other signs of alcohol with-
levels, and hydration status. The mainstay of
drawal.
therapy is a combination of cognitive behav-
Answer B is incorrect. Cocaine can cause ioral therapy and SSRIs. Use of the antidepres-
many of the symptoms this patient is experi- sant bupropion is contraindicated in patients
encing. However, patients with recent cocaine with anorexia nervosa because it increases the
use are usually hypertensive and tachycardic risk of seizure in this population.
because of its stimulant effects.
Answer A is incorrect. Amitriptyline is a tri-
Answer D is incorrect. Marijuana can cause cyclic antidepressant (TCA). TCAs, although
many of the symptoms this patient is experi- effective, are not first-line therapy in the man-
encing. However, patients with recent mari- agement of anorexia nervosa, given the poten-
juana use usually have an increase in appetite tial for cardiac adverse effects in anorexic pa-

Psychiatry
and dry mouth as well. tients already suffering from bradycardia and
electrolyte abnormalities. TCAs are not known
Answer E is incorrect. Phencyclidine is a hal-
to increase the risk of seizure in anorexic pa-
lucinogenic drug that is often associated with
tients.
belligerence and acting out impulsively.
Answer C is incorrect. Fluoxetine is an SSRI
16. The correct answer is A. The boy exhibits the most commonly used as an antidepressant. It
characteristic emotional lability and impulsiv- has also been used to treat anorexia nervosa,
ity seen in patients with attention deficit/hyper- although with questionable efficacy. SSRIs are
activity disorder (ADHD). Methylphenidate is not known to increase the risk of seizure in an-
a first-line treatment for ADHD. It works simi- orexic patients.
larly to amphetamines by increasing the pre-
Answer D is incorrect. Mirtazapine in an
synaptic release of norepinephrine.
atypical antidepressant that induces weight
Answer B is incorrect. Antimuscarinic drugs gain, which may be beneficial in patients with
like benztropine can be used in conjunction weight control issues, although this has not yet
with typical antipsychotics in the treatment been studied rigorously. Mirtazapine is not
of schizophrenia to alleviate extrapyramidal known to increase the risk of seizure in an-
symptoms. orexic patients.
Answer C is incorrect. Selective serotonin Answer E is incorrect. Selegiline is a MAO in-
reuptake inhibitors can be used to treat depres- hibitor most commonly used as an antidepres-
376 Section II: Organ Systems  •  Answers

sant; it is not typically used to manage anorexia Of note, MAO inhibitors, another class of anti-
nervosa. MAO inhibitors are not known to in- depressants, do not have anticholinergic prop-
crease the risk of seizure in anorexic patients. erties, but can cause adverse effects similar to
those of anticholinergic medications, includ-
18. The correct answer is D. SSRIs block the ing dry mouth and urinary retention. MAO in-
reuptake of serotonin (5-hydroxytryptamine [5- hibitors often are associated with tyramine cri-
HT]) by the serotonin transport protein (STP) ses on the USMLE exam, especially when the
in presynaptic neurons; the result is an effec- patient described has consumed tannin-rich
tive increase in serotonin within the synaptic foods, such as red wines and aged cheeses.
space. SSRIs act at the “X” in the image by
Answer B is incorrect. Clonazepam is a ben-
inhibiting the binding of 5-HT to STP. SSRIs
zodiazepine sometimes prescribed as an anxio-
have demonstrated efficacy for numerous med-
lytic at the initiation of anti-depressant therapy.
High-Yield Systems

ical and psychiatric conditions, most notably


The most commonly reported adverse effects
depression, anxiety, obsessive-compulsive dis-
are those associated with CNS depression,
order, and eating disorders.
such as sedation or respiratory depression at
Answer A is incorrect. SSRIs are not first-line higher doses. Dependence and rebound anxi-
treatment for bipolar disorder; a mood stabiliz- ety can result from benzodiazepine abuse.
ing agent (eg, lithium or valproic acid) would
Answer C is incorrect. Lithium is a mood sta-
be the treatment of choice.
bilizer used to treat bipolar affective disorder.
Answer B is incorrect. SSRIs are not first-line It indirectly inhibits the reuptake of serotonin
treatment for delirium tremens; a long-acting and norepinephrine by inhibiting the phos-
benzodiazepine (eg, chlordiazepoxide) would phatidylinositol second messenger system.
be the treatment of choice. Adverse effects include CNS depression, diz-
Answer C is incorrect. SSRIs are not first-line ziness, nephrogenic diabetes insipidus, acne,
treatment for multiple personality disorder; an edema, and hypothyroidism, as well as many
antipsychotic (eg, haloperidol or risperidone) others.
would be the treatment of choice. Answer D is incorrect. Sertraline and other
Answer E is incorrect. SSRIs are not first-line SSRIs are associated with adverse effects re-
Psychiatry

treatment for schizophrenia; an antipsychotic lated to CNS stimulation such as headache,


(eg, haloperidol or risperidone) would be the anxiety, tremor, insomnia, anorexia, nausea,
treatment of choice. and vomiting. Weight gain and sexual dysfunc-
tion are also frequently reported with SSRI
19. The correct answer is A. This patient is being use.
treated for depression. Amitriptyline, a tricyclic Answer E is incorrect. Venlafaxine is a sero-
anti-depressant, is as effective as the selective tonin/norepinephrine reuptake inhibitor. It
serotonin reuptake inhibitors, but often is not has adverse effects similar to those of selective
prescribed as a first-line agent because of its serotonin reuptake inhibitors, plus additional
many adverse effects. These include sedation, adverse effects due to the norepinephrine,
a-blocking effects, and, most commonly, anti- such as dizziness and diaphoresis. Venlafaxine
cholinergic effects such as dry mouth, blurry is also known to cause hypertension.
vision, tachycardia, urinary retention, consti-
pation, confusion, and dry, hot skin. These 20. The correct answer is C. This patient has evi-
adverse effects can be remembered with the dence of benzodiazepine intoxication. This
following: red as a beet (flushing), dry as a is the most likely scenario given her recent
bone (anhidrosis), hot as a hare (overheating diagnosis and treatment for panic disorder, in
secondary to anhidrosis), blind as a bat (blurry addition to the exclusion of other causes with
vision), mad as a hatter (hallucinations or de- similar presentations. Benzodiazepines are
lirium), and full as a flask (urinary retention). relatively safe in overdose; however, shorter-
Chapter 14: Psychiatry  •  Answers 377

acting benzodiazepines such as temazepam, Answer B is incorrect. Physostigmine is an


triazolam, and alprazolam pose a greater risk indirect-acting cholinomimetic that inhibits
for morbidity and mortality. Competitive the action of acetylcholinesterase, thereby am-
antagonists work by displacing a drug from plifying the effect of endogenous acetylcholine.
its binding site. Flumazenil is a competi- It is indicated in cases of anticholinergic (but
tive antagonist that can be used in the case of not tricyclic) poisoning, which would present
benzodiazepine overdose, and naloxone is a with the classic picture described by the mne-
competitive antagonist that is used to reverse monic “red as a beet, blind as a bat, mad as a
symptoms of opiate overdose. When using flu- hatter, dry as a bone, and hot as a hare.” One
mazenil, be aware that rapid reversal of benzo- would expect fever, flushing, delirium, dry mu-
diazepine overdose may lead to rebound sei- cous membranes, and miosis on physical exam.
zure activity. In clinical practice flumazenil is
Answer D is incorrect. Ethanol is indicated

High-Yield Systems
rarely used except in children.
in cases of toxic alcohol ingestion (eg, metha-
Answer A is incorrect. Pralidoxime, a cholin- nol or ethylene glycol). Toxic metabolites are
esterase regenerator, is indicated in cases of or- formed when alcohol dehydrogenase metabo-
ganophosphate poisoning. Organophosphates lizes methanol or ethylene glycol. Ethanol
such as parathion and malathion are indirect- works by inhibiting the formation of these
acting cholinomimetics that inhibit acetylcho- harmful substances by competing for binding
linesterase by forming a very stable bond with sites on alcohol dehydrogenase.
it. This results in general cholinergic CNS
Answer E is incorrect. Reserpine inhibits the
stimulation (incontinence, bronchoconstric-
storage of norepinephrine in adrenergic nerve
tion, miosis, and bradycardia). Pralidoxime has
terminals, thereby depleting the neuron of its
a greater affinity for binding to organophos-
stores. It has been classified as a postganglionic
phates than acetylcholinesterase. As such, it is
sympathetic nerve terminal blocker, and is
thought of as an organophosphate “chemical
rarely used as an antihypertensive medication.
antagonist.”

Psychiatry
This page intentionally left blank
Chapter 15

Renal

379
380 Section II: Organ Systems  •  Questions

Q u e st i o n s

1. A patient presents with the chief complaint of (E) Urine glucose begins to be detectable at
an unrelenting headache of several months’ 375 mg/dL
duration. Her vital signs are significant for a
blood pressure of 200/100 mm Hg. Abdominal 3. A 17-year-old boy with insulin-dependent dia-
ultrasound confirms the cause of her hyperten- betes is brought to the emergency department
sion. Decreased perfusion of which of the fol- unconscious with a fruity odor on his breath.
lowing structures results in the renin release Upon his admission, insulin is administered
that is responsible for this patient’s condition? and serum potassium levels are monitored
closely. Potassium balance is essential for the
(A) Adrenal medulla
normal function of excitable tissues such as
High-Yield Systems

(B) Afferent arteriole


muscles and nerves. Which of the following
(C) Distal convoluted tubule
clinical situations could also put a patient at
(D) Loop of Henle
risk for hypokalemia?
(E) Zona glomerulosa
(A) High-K+ diet
2. The following graph was obtained by collect- (B) Hypoaldosteronism
ing and assaying urine from a healthy volun- (C) Metabolic acidosis
teer given varying amounts of glucose by intra- (D) Overuse of amiloride
venous infusion. Which statement concerning (E) Overuse of hydrochlorothiazide
the graph is most accurate?
4. A 66-year-old woman with acute renal insuffi-
ciency and unremarkable medical history un-
dergoes a renal biopsy that reveals glomerular
capillary subendothelial deposits. Which of
Urine glucose (mg/dL)

the following symptoms often occurs in pa-


tients with this disease?
(A) Achalasia
(B) Aortic aneurysm
Renal

(C) Esophageal reflux


(D) Malar rash
(E) Morbilliform rash

0 200 400 5. Nonsteroidal anti-inflammatory drugs (NSAIDs)


Serum glucose (mg/dL) are associated with the development of acute re-
nal failure in patients living in states of effective
Reproduced, with permission, from USMLERx.com. volume depletion, such as heart failure, cirrhosis,
and true volume depletion. What is the mecha-
nism by which NSAIDs are believed to mediate
(A) Glomerular filtration of glucose is satu-
this harmful effect on the kidney?
rated at 375 mg/dL
(B) Proximal convoluted tubule glucose reab- (A) Increased prostaglandin synthesis con-
sorption is saturated at 375 mg/dL stricts preglomerular vessels
(C) Proximal convoluted tubule glucose secre- (B) Inhibition of prostaglandin synthesis de-
tion begins at 200 mg/dL creases glomerular capillary permeability
(D) Proximal convoluted tubule glucose secre-
tion begins at 375 mg/dL
Chapter 15: Renal  •  Questions 381

(C) Inhibition of prostaglandin synthesis de-


creases preglomerular resistance
(D) Inhibition of prostaglandin synthesis in-
creases preglomerular resistance
(E) The drug has direct toxic effects on the re-
nal tubules

6. Immune complex-mediated glomerular disease


can be classified according to the site of com-
plex formation. The site also partly dictates
whether the disease would give rise to either a
nephritic or a nephrotic clinical presentation.

High-Yield Systems
Where does the immune complex deposition
form in membranoproliferative glomerulone-
phritis?
(A) Between the glomerular basement mem- Courtesy of Dr. Per-Lennart Westesson, University of Roches-
brane and visceral epithelium ter Medical Center.
(B) Between the glomerular basement mem-
brane and the podocytes
(C) Between the glomerular basement mem- (A) Decreased antithrombin III levels
brane and the visceral epithelial cells (B) Decreased factor II levels
(D) Between the endothelium and the mesan- (C) Decreased fibrinogen levels
gium (D) Increased protein C levels
(E) Between the visceral epithelial cells and (E) Increased protein S levels
the parietal epithelium
8. A 28-year-old woman with a history of asthma
7. A 58-year-old man with nephrotic syndrome is presents to the emergency department with a
brought by ambulance to the emergency de- temperature of 38.2°C (100.8°F) and com-
partment with altered mental status. His wife plains of shaking chills and pain on her right
reports that this morning he had difficulty side, which she locates by pointing to the area
above her right iliac crest. She notes that she

Renal
moving the right side of his body, and that she
couldn’t arouse him from an afternoon nap. has never had this occur before. During the ex-
On physical examination the patient is ob- amination, there is tenderness to percussion at
tunded and has absent right-sided movement. the junction of the lower ribs and the thoracic
His International Normalized Ratio (INR) is = vertebrae. Urinalysis reveals WBC casts. What
0.5. CT angiogram of the brain is shown in the is the organism most likely causing her condi-
image. What is the most likely etiology of this tion?
patient’s symptoms? (A)
Escherichia coli
(B)
Klebsiella pneumoniae
(C)
Proteus mirabilis
(D)
Staphylococcus saprophyticus
(E)
Ureaplasma urealyticum
382 Section II: Organ Systems  •  Questions

9. A 64-year-old man presents to his primary care (A) Glomeruli


physician for a routine physical. He is in good (B) Interstitium of the kidney
health, smokes half a pack of cigarettes daily, (C) Kidney tubules
and has no other known medical problems. (D) Spleen
However, he was diagnosed with hypertension (E) Urethra
in the past. The physician notes a blood pres-
sure of 160/100 mm Hg and 1+ pitting edema 12. A 68-year-old woman with a history of diabe-
to his knees. The physician also notes bilateral tes and hypertension, who is recovering from
bruits when auscultating the abdomen. Which a total hip replacement, is given ketorolac, a
of the following is contraindicated in this pa- nonsteroidal anti-inflammatory drug, for the
tient’s care? management of pain. Twenty-four hours later,
her urine production decreases, and her se-
(A) Angioplasty
High-Yield Systems

rum blood urea nitrogen and creatinine levels


(B) Angiotensin-converting enzyme inhibitor
rise to 44 mg/dL and 3.1 mg/dL, respectively.
(C) Diuretics
What is the effect of this medication on glo-
(D) Smoking cessation
merular filtration rate, renal plasma flow, and
(E) Surgical management
filtration fraction?
10. A 2-month-old boy is brought to the physi- (A) Decreased renal plasma flow, decreased
cian after his parents palpated an abdominal glomerular filtration rate, no change in fil-
mass while bathing him. They deny any fevers, tration fraction
chills, decreased urine output, or hematuria. (B) Decreased renal plasma flow, increased
On physical examination a large mass is pal- glomerular filtration rate, increased filtra-
pated in the midline. Abdominal ultrasound tion fraction
shows pelvic kidneys fused at their lower poles. (C) Increased renal plasma flow, decreased
Which structure prevents this abnormal kidney glomerular filtration rate, increased filtra-
from occupying its appropriate position? tion fraction
(D) No change in renal plasma flow, decreased
(A) Aorta
glomerular filtration rate, decreased filtra-
(B) Celiac trunk
tion fraction
(C) Inferior mesenteric artery
(E) No change in renal plasma flow, increased
(D) Inferior vena cava
Renal

glomerular filtration rate, increased filtra-


(E) Superior mesenteric artery
tion fraction
11. A 23-year-old man is beginning chemotherapy
13. A 56-year-old man with a 60 pack-year smok-
for leukemia when he develops severe inter-
ing history and normal fluid intake presents to
mittent left flank pain that soon migrates to the
his physician with two months of fatigue and
pelvis. Three days later, the patient’s creatinine
weakness accompanied by cough and mild
level rises and he is diagnosed with acute re-
dyspnea. The patient’s vital signs are normal,
nal failure. His fractional excretion of sodium
but a lower left lobe mass is noted on X-ray of
(FENa) is >4% with a urine osmolality of <350
the chest. Biopsy leads to the diagnosis of small
mOsm/kg. Blood and urine cultures are nega-
cell carcinoma. Results of laboratory tests are
tive for bacteria and eosinophilia. An abdomi-
shown below.
nal radiograph fails to locate any pathology.
Which of the following is the most likely loca- Plasma sodium: 125 mEq/L
tion of the lesion causing this patient’s renal Plasma potassium: 3.9 mEq/L
failure? Plasma carbon dioxide: 24 mEq/L
Plasma osmolality: 253 mOsm/L
Urine sodium: 48 mEq/L
Urine osmolality: 280 mOsm/L
Chapter 15: Renal  •  Questions 383

The hormone most likely responsible for this 15. A 50-year-old man with a history of large-bowel
patient’s abnormal laboratory values has which obstruction is diagnosed with colon cancer and
of the following direct effects? undergoes resection of his colon. At check-up,
he feels well except that over the past three
(A) Activation of G-protein coupled receptors
weeks he has had significant swelling in his
in the adrenal cortex elevates cAMP levels
legs. Physical examination reveals 2+ pitting
and leads to increased production and se-
edema and a blood pressure of 155/94 mm
cretion of corticosteroids
Hg. Urinalysis shows 4+ protein with no RBCs
(B) Activation of G-protein coupled receptors
or casts. Which of the following would most
in the hypothalamus results in elevated
likely be present on a kidney biopsy from this
cAMP levels and inhibition of hypotha-
patient?
lamic-induced thirst mechanism
(C) Activation of V2 receptors leads to an in- (A) A spike-and-dome pattern of deposition on

High-Yield Systems
crease in total peripheral resistance; activa- electron microscopy
tion of V1 receptors results in the concen- (B) A tram-track pattern on light microscopy
tration of urine (C) Lumpy subepithelial deposits on electron
(D) Activation of V2 receptors results in the in- microscopy
sertion of aquaporins into the collecting (D) Nonlinear mesangial staining with IgA im-
duct; activation of V1 receptors leads to a munofluorescence
decrease in total peripheral resistance (E) “Splintering” of the lamina densa
(E) Activation of V2 receptors results in the in-
sertion of aquaporins into the renal collect- 16. A 5-year-old girl is brought to the pediatrician
ing duct; activation of V1 receptors leads because of generalized edema. Laboratory test-
to an increase in total peripheral resistance ing reveals proteinuria of 5 gm/day, and podo-
(F) Cleavage of angiotensinogen to angioten- cyte foot process effacement is seen on elec-
sin I leads to an increase in both aldos­ tron microscopy of a kidney biopsy. Disruption
terone levels and total peripheral resis- of which of the following components of the
tance filtration barrier is contributing to this patient’s
proteinuria?
14. A hospitalized patient is given methicillin for
(A) Brush border
a suspected staphylococcal infection around a
(B) Endothelial cell

Renal
central line insertion site. Approximately one
(C) Heparan sulfate
week after beginning treatment, the patient de-
(D) Integrin
velops signs of acute renal failure, including an
(E) Type IV collagen
increased serum creatinine level. A urine sam-
ple shows microscopic hematuria and WBCs,
but no bacteria. Methicillin is withdrawn and
the patient’s renal symptoms soon improve.
Administration of which of the following drugs
would most likely also cause this condition?
(A) Cyclophosphamide
(B) Diphenhydramine
(C) Hydrochlorothiazide
(D) Isoniazid
(E) Lithium
384 Section II: Organ Systems  •  Questions

17. A 67-year-old woman with osteoporosis is given 19. A 28-year-old woman who received no prena-
a diuretic to treat her hypertension. This par- tal care gives birth at 37 weeks’ gestation. The
ticular diuretic has the adverse effect of limit- fetus is stillborn and has a number of anoma-
ing calcium excretion by the kidney. Referring lies including a flattened face, large and low-
to the image, where along the nephron does set ears, and clubbed feet. What other condi-
this drug act? tion would result in an amount of amniotic
fluid similar to that found in this situation?
E (A) Anencephaly
B
A (B) Chronic uteroplacental insufficiency
(C) Duodenal atresia
(D) Maternal diabetes
(E) Trisomy 18
High-Yield Systems

20. A 6-month-old girl is brought to the emer-


F
gency department by her mother after ingest-
ing a bottle of unmarked pills from the family
medicine cabinet. On examination the child
D appears agitated and tachypneic. Urgent blood
chemistry measurements are taken and reveal
C the following:
Na+: 143 mEq/L
Reproduced, with permission, from USMLERx.com. K+: 4.0 mEq/L
Cl-: 104 mEq/L
HCO3-: 19 mEq/L
(A) A pH: 7.28
(B) B Which of the following medications did this
(C) C patient most likely ingest?
(D) D
(E) E (A) Aldosterone antagonist
(F) F (B) Antacid
Renal

(C) Iron supplement


18. A 22-year-old college student is brought to the (D) Loop diuretic
emergency department after he began com- (E) Opiate analgesic
plaining of ants crawling over his body and his
friends noted increasing agitation and threat- 21. A 16-year-old boy comes to the physician be-
ening gestures. On physical examination he is cause of a one year history of intermittent,
febrile, restless, and tachycardic, and his pupils painless hematuria without dysuria or in-
are markedly dilated. His friends report he has creased frequency of micturition. He notes
been studying “at all hours” for his upcoming that the hematuria becomes more prominent
final examinations. Appropriate treatment in- when he is also suffering from a respiratory in-
cludes which of the following? fection. Which of the following is most likely
to be found if the boy is diagnosed with IgA ne-
(A) Acidifying urine to increase renal clear- phropathy (Berger disease)?
ance
(B) Alkalinizing urine to increase renal clear- (A) Increased antistreptolysin O titer
ance (B) Lumpy-bumpy electron-dense deposits
(C) Treating with flumazenil (C) Mesangial deposits
(D) Treating with naloxone (D) Proteinuria exceeding 3.5 gm/24 h
(E) Treating with water to dilute drug effects (E) Subepithelial deposits
Chapter 15: Renal  •  Questions 385

22. A 34-year-old woman comes to the hospital


Total
to deliver a full-term infant. Labor is compli- Heart
peripheral Vasopressin
Choice Renin
cated by an amniotic fluid embolism, and rate
resistance
subsequent blood tests show the presence of
A
fibrin split products. The next day the patient
abruptly develops anuria, gross hematuria, and B
flank pain accompanied by rapidly increasing C
blood urea nitrogen and creatinine levels and
a new cardiac friction rub. The patient’s ul- D

trasound demonstrates hypodensities within E


the renal cortex. Which of the following is the
most appropriate treatment? Reproduced, with permission, from USMLERx.com.

High-Yield Systems
(A) Aggressive fluid support
(B) Biopsy to evaluate for malignancy (A) A
(C) Broad-spectrum antibiotics (B) B
(D) Dialysis (C) C
(E) No treatment is necessary at this time (D) D
(E) E
23. A 40-year-old man presents with hematuria
and sharp, sudden, sporadic pain in his lower 25. A 72-year-old retired man recently diagnosed
back. His blood pressure is normal, and his with heart disease arrives at the emergency de-
physical examination is significant for flank partment complaining of sudden-onset abdom-
pain that comes in waves. A plain film of the inal pain, diarrhea, and vomiting, with mild
abdomen and pelvis shows focal, marked den- pain and swelling throughout his face, lips,
sities bilaterally in the mid-abdomen. A follow- and mouth. The patient denies any pruritus.
up noncontrast CT scan demonstrates several On questioning he says his primary care phy-
large stones in the ureters, bilaterally. Which sician recently put him on a regimen of drugs
of the following is the most likely cause of this to control his high blood pressure and choles-
man’s symptoms? terol. He also mentions that he sometimes gets
(A) A full rectum, and bilateral calcified com- a rash after taking antibiotics and usually gets

Renal
mon iliac arteries “hives” every spring. The physician suspects
(B) A large stool passing through the small that these symptoms are related to one of the
bowel patient’s medications. The accumulation of
(C) Hyperparathyroidism what substance is most likely causing the pa-
(D) Hyperuricemia tient’s symptoms?
(E) Staphylococcal infection of the bladder (A) Bradykinin
(B) Histamine
24. A man is chopping wood outside when he ac-
(C) Prostacyclin
cidentally chops into his arm, causing it to
(D) Prostaglandin E2
bleed profusely. A friend drives him to the hos-
(E) Serotonin
pital, 30 minutes away. Although pressure is
applied to the wound, there is continued blood
loss. Upon arrival to the emergency depart-
ment, he appears lethargic and pale. Which
of the following set of vital signs would most
likely be seen in this patient as he is being
transported to the hospital?
386 Section II: Organ Systems  •  Questions

26. A 56-year-old woman who has been taking 28. A patient suffering from an upper respiratory
cefoxitin for treatment of Klebsiella pneumo- infection presents to his physician with com-
nia is found to still have Klebsiella organisms plaints of body aches, urinary frequency, and
in her blood one week after beginning treat- “strange-colored urine.” His previous medical
ment. Another drug is added to the patient’s history is significant only for recently begin-
regimen. Two days later, the following labora- ning treatment with a statin drug for high cho-
tory values are obtained: lesterol. His serum creatinine level is 2.0 mg/
dL. A histologic section of the patient’s kidney
Na+: 141 mEq/L
is shown in the image. Which of the following
K+: 4.3 mEq/L
is the most likely diagnosis?
Cl-: 102 mEq/L
HCO3-: 24 mEq/L
Blood urea nitrogen: 65 mg/dL
High-Yield Systems

Creatinine: 4.4 mEq/L


Which of the following medications was most
likely added to this patient’s regimen?
(A) Azithromycin
(B) Aztreonam
(C) Clindamycin
(D) Piperacillin
(E) Tobramycin

27. A 2-year-old boy is brought to the emergency


department (ED) with complaints of fever,
chills, and flank pain. His immunizations
are up to date and his mother states that this Reproduced, with permission, from USMLERx.com.
is the second time he has been to the ED be-
cause of these symptoms. His temperature is
(A) Acute tubular necrosis
39.1°C (102.2°F) and physical examination is
(B) Amyloidosis
unremarkable except for costovertebral angle
(C) Analgesic nephropathy
Renal

tenderness on the right. A complete blood cell


(D) Focal segmental glomerulosclerosis
count shows leukocytosis, and urinalysis dem-
(E) Pyelonephritis
onstrates the presence of WBCs and RBCs in
the urine. What is the most likely mechanism
29. A 40-year-old woman presents to the emer-
of this patient’s recurrent complaints?
gency department after five days of profuse
(A) Immunoglobulin deficiency vomiting. She has a history of rheumatoid ar-
(B) Nephroblastoma thritis, which is treated with celecoxib. She
(C) Poststreptococcal glomerulonephritis complains of joint pain at present. Which of
(D) Vesicoureteral reflux the reasons below describes why celecoxib
would be contraindicated in this patient at pre-
sentation?
Chapter 15: Renal  •  Questions 387

(A) Because of its effects on platelet function cessful. Immunofluorescence shows deposition
(B) Because of its effects on the arterioles of of IgM in the affected segments. Which of the
the kidney following is most likely to be seen on biopsy?
(C) Because of its effects on the gastrointesti-
(A) Extensive glomerular crescent formation
nal mucosa
(B) IgA deposition in the glomerular mesan-
(D) Because of its effects on the macula densa
gium
(E) Because of its effects on the production of
(C) Immune complex deposition in subepithe-
inflammatory cytokines
lial space
(D) Large focal hyaline deposits
30. A 57-year-old man with a 15-year history of
(E) No deposits on immunofluorescence,
type 2 diabetes mellitus and hypertension who
some retraction of epithelial foot processes
is on insulin therapy presents with complaints
on electron microscopy

High-Yield Systems
of fatigue and swollen feet. Physical examina-
tion reveals 3+ pitting edema bilaterally in his
32. A large body of epidemiologic research has
feet as well as loss of pain and temperature sen-
shown that a diet high in sodium can con-
sation in all of his toes. Vital signs are within
tribute to hypertension, resulting in left ven-
normal limits except for a blood pressure of
tricular hypertrophy. The Intersalt Epidemiol-
150/95 mm Hg. Blood tests show an LDL level
ogy Study relates increased sodium intake to
of 154 mg/dL, blood glucose level of 213 mg/
higher blood pressures across diverse popula-
dL, and a hemoglobin A1c level of 9.2%. A ba-
tions. Also, rigorous prospective clinical trials
sic metabolic panel shows a blood urea nitro-
demonstrate that lowering sodium intake can
gen level of 67 mg/dL and creatinine level of
lower arterial pressure in normotensive and hy-
2.6 mg/dL. Urinalysis shows 4+ protein with
pertensive individuals. Which of the following
no casts. Which of the following antihyperten-
is a mechanism of how the kidney responds to
sive medications may have helped prevent this
high sodium intake?
patient’s current condition?
(A) Increase in atrial natriuretic peptide and
(A) A selective b-blocker
dilation of glomerular afferent arterioles
(B) An antagonist of cardiac and vascular cal-
(B) Increased release of renin due to chemore-
cium channels
ceptor recognition of increased plasma os-
(C) An inhibitor of a thick ascending limb
molarity

Renal
transport protein
(C) Increased secretion of potassium into the
(D) An inhibitor of the sodium-chloride sym-
urine
porter in the distal convoluted tubule
(D) Mechanoreceptor detection of constriction
(E) Angiotensin receptor blocker
of glomerular afferent arterioles
(E) Mechanoreceptor detection of dilation of
31. A 27-year-old HIV-positive man presents to the
glomerular afferent arterioles
emergency department with swollen legs. He
states that his urine has been very foamy. Uri-
nalysis shows massive proteinuria. Treatment
with corticosteroids is initiated but is not suc-
388 Section II: Organ Systems  •  Questions

33. A 72-year-old man presents to his physician brane prevent albumin from being freely fil-
complaining of pain in his lower abdomen, tered into the urine?
increased difficulty urinating, and decreased
(A) A combination of small pore size and neg-
urine output for the past couple days. The phy-
atively charged pore-forming molecules
sician notes an enlarged prostate on digital rec-
prevents albumin filtration
tal examination. Serum creatinine level is 2.5
(B) A combination of small pore size and posi-
mg/dL. Renal ultrasound is performed, and
tively charged pore-forming molecules pre-
the image shows what is visualized bilaterally.
vents albumin filtration
Which of the following would most likely be
(C) Albumin is freely filtered across the base-
expected on urinalysis?
ment membrane but is readily reabsorbed
along the nephron
(D) The positive charge of proteoglycans in the
High-Yield Systems

basement membrane repels albumin


(E) The small size of the glomerular basement
membrane pores excludes albumin mole-
cules

35. A 54-year-old homeless woman is found un-


conscious under a bridge. On admission to the
emergency department, her laboratory tests
show:
Sodium: 137 mEq/L
Potassium: 3.3 mEq/L
Chloride: 112 mEq/L
Bicarbonate: 15 mEq/L
Arterial blood gas on room air: pH 7.28
Partial pressure of carbon dioxide: 28 mm Hg
Partial pressure of oxygen: 90 mm Hg
Which of the following most likely caused her
Renal

Reproduced, with permission, from Tanagho EA, McAninch acidosis?


JW. Smith’s General Urology, 17th ed. New York: McGraw-
Hill, 2008: Figure 6-12A.
(A) An aspirin overdose
(B) Diabetic ketoacidosis
(C) Severe diarrhea
(A) Blood urea nitrogen:creatinine ratio <15 (D) Severe underperfusion of her peripheral
(B) Epithelial casts muscles
(C) Fractional excretion of sodium <1% (E) Uremia
(D) Urine Na+ <10 mmol/L
(E) Urine osmolality <350 mmol/kg 36. A patient with hepatocellular carcinoma de-
velops severe ascites such that 3-5 L of fluid
34. Nephrotic syndrome is characterized by se- must be drained from her peritoneal cavity ev-
vere proteinuria, a decreased serum albumin ery three days. This procedure may have detri-
level, and edema. This results from damage mental effects on kidney function that neces-
to the glomerular capillary wall. In particular, sitates monitoring of glomerular filtration rate.
the glomerular basement membrane is essen- Laboratory values are as follows:
tial for maintaining serum oncotic pressure. In Creatinine clearance: 120 mL/min
non-pathologic states, which of the following Glomerular capillary hydrostatic pressure: 40
properties of the glomerular basement mem- mm Hg
Chapter 15: Renal  •  Questions 389

Plasma inulin: 1.5 mg/mL (A) Alport syndrome


Urinary inulin: 50 mg/mL (B) Membranous glomerulonephritis
(C) Minimal change disease
Which of the following is her urine flow rate?
(D) Poststreptococcal glomerulonephritis
(A) 1.6 mL/min (E) Systemic lupus erythematosus nephritis
(B) 3.6 mL/min
(C) 4.5 mL/min 39. A 36-year-old man presents for his annual flu
(D) 36 mL/h shot. He has been treated for hypertension
(E) 450 mL/day with an angiotensin-converting enzyme in-
hibitor since age 30 years. He denies smok-
37. Mountain climbers sometimes take acetazol- ing and alcohol use. Family history is notable
amide to help the body rapidly acclimatize to for his father requiring dialysis at age 50. An
higher altitudes. Which of the following is the

High-Yield Systems
older brother recently underwent unilateral
mechanism by which acetazolamide does this? nephrectomy to decompress intra-abdominal
(A) It causes a mixed metabolic acidosis and organs. On examination, the patient appears
respiratory alkalosis barrel-chested and has a sitting blood pressure
(B) It causes metabolic acidosis of 135/90 mm Hg. His likely genetic renal pa-
(C) It causes metabolic alkalosis thology is associated with an increased inci-
(D) It causes respiratory acidosis dence of which of the following?
(E) It causes respiratory alkalosis (A) Cerebral aneurysm
(B) Horseshoe kidney
38. A longtime patient returns to visit her rheu- (C) Macular cherry-red spots
matologist with complaints of headaches (D) Potter syndrome
and blood in her urine. Her blood pressure is (E) Pott disease
152/88 mm Hg. Further testing reveals protein
in the urine as well as RBC casts. The rheuma- 40. A 48-year-old man is hospitalized after a mo-
tologist orders a renal biopsy, which is shown tor vehicle accident. He is hypotensive and
in the image. The pathology seen in the image is given several units of packed RBCs by
is most likely caused by which of the following transfusion. He is kept in the intensive care
processes? unit for monitoring. On the patient’s second

Renal
day in the hospital, his blood urea nitrogen
(BUN) and creatinine levels begin to rise and
he develops pitting edema to his knees. His
BUN:creatinine ratio is 12:1. A subsequent uri-
nalysis shows numerous muddy brown epithe-
lial and granular casts. Which of the following
is another common cause of this man’s condi-
tion?
(A) Ascending urinary tract infection
(B) Crush injury
(C) Diabetes mellitus
(D) Nonsteroidal anti-inflammatory drug toxic-
ity
(E) Septic shock
Reproduced, with permission, from USMLERx.com.
390 Section II: Organ Systems  •  Questions

41. Nephrolithiases, or kidney stones, can obstruct


the flow of urine and cause pain. Stones are
more likely to cause an obstruction at a junc-
tion or location where the tract is constricted.
Which of the following describes a location
where stones are most likely to cause an ob-
struction?
(A) In the bladder
(B) In the penile urethra
(C) In the prostatic urethra
Reproduced, with permission, from USMLERx.com.
(D) In the renal calyx
(E) In the ureter over the iliac vessels
High-Yield Systems

(A) Acute pyelonephritis


42. A 53-year-old woman is undergoing renal func- (B) Chronic reflux-associated pyelonephritis
tion testing to evaluate proteinuria detected by (C) Drug-induced interstitial nephritis
her primary care physician at a routine visit. (D) Hypertension
The patient’s glomerular filtration rate (GFR) (E) Minimal change disease
is initially estimated at 100 mL/min by inulin
clearance. Subsequently, as part of a research 44. A 52-year-old postmenopausal woman sees her
study, the patient’s GFR is determined using physician because she is worried about osteo-
a novel marker (compound X) and is found porosis. Her physician decides that he should
to be 125 mL/min. Which mechanism most investigate her calcium reabsorption in the
likely explains the difference noted between thick ascending limb of Henle. Which of the
these two GFR estimates? following interventions would most effectively
(A) Compound X is not freely filtered at the increase her calcium reabsorption in the thick
glomerulus ascending limb of the loop of Henle?
(B) Compound X is reabsorbed in the de- (A) Administration of exogenous parathyroid
scending limb of the loop of Henle hormone
(C) Compound X is reabsorbed in the proxi- (B) Discontinuation of her loop diuretic
mal convoluted tubule (C) Discontinuation of her thiazide diuretic
Renal

(D) Compound X is secreted in the proximal (D) Increasing serum calcium by administra-
convoluted tubule tion of a vitamin supplement
(E) Inulin is reabsorbed in the proximal con- (E) Maintenance of negative charge in the lu-
voluted tubule men

43. The kidneys of a 65-year-old patient with long- 45. A 67-year-old man with a history of mild hyper-
standing diabetes mellitus are examined at au- tension has a severe heart attack while walking
topsy and one kidney is shown in the image. to his car. When he arrives at the emergency
In the later stage of his disease, this patient fre- department he is pale, cold, and diaphoretic.
quently required a Foley catheter to adequately On physical examination he is tachycardic
drain his bladder. What condition most likely and hypotensive, and his ECG shows ST-
caused the anomalous appearance of his kid- segment elevations. He is treated with mor-
neys? phine, oxygen, and aspirin and is sent to the
cardiac catheterization laboratory. The next
Chapter 15: Renal  •  Questions 391

day he has low urine output, his blood urea partment with hemoptysis. He has never had
nitrogen level is 35 mg/dL, his creatinine level an episode like this before. He does recall hav-
is 1.3 mg/dL, and his blood pressure is 85/55 ing a cold approximately three weeks earlier
mm Hg. Urinalysis reveals few hyaline casts. and has been experiencing increased fatigue
Which of the following is the most likely cause and breathlessness since then, but he denies
of his low urine input? any current symptoms of rhinorrhea, sore
throat, nausea, vomiting, or diarrhea. X-ray of
(A) A blockage of the ureters or urethra
the chest shows diffuse opacities in both lower
(B) Acute tubular necrosis
lung fields, and urinalysis reveals RBCs in
(C) Interstitial nephritis
the urine. Which of the following is the most
(D) Low urine output is normal in times of
likely diagnosis?
stress
(E) Poor perfusion of the kidneys (A) Acute poststreptococcal glomerulonephri-

High-Yield Systems
tis
46. A 13-year-old boy is brought to the emergency (B) Alport syndrome
department with periorbital edema, hyperten- (C) Berger disease
sion, and tea-colored urine. His parents say (D) Goodpasture syndrome
that he had a sore throat about three weeks (E) Minimal change disease
ago. Urinalysis shows RBCs with casts. A posi-
tive antistreptolysin O titer and decreased 49. A 63-year-old man is seen by his doctor after
levels of complement are also noted. What measuring his blood pressure at home and
findings would be expected in this patient’s finding it to be 168/100 mm Hg. The patient
glomeruli? is concerned because he has had high blood
pressure for the past 12 months that has not
(A) Granular subendothelial deposits
improved with dietary changes, exercise, or
(B) Linear subendothelial pattern
medication. A physical examination is unre-
(C) Mesangial deposits
markable except for a thrill heard when aus-
(D) Subepithelial humps
cultating the abdomen just to the left of the
47. A 55-year-old man admitted to the hospital be- midline. What is the most likely cause of the
cause of complications of alcoholic cirrhosis patient’s hypertension?
and concurrent hepatitis C infection progres- (A) Decreased levels of ADH

Renal
sively develops an increase in serum blood (B) Decreased levels of angiotensin II
urea nitrogen and creatinine levels, followed (C) Elevated levels of aldosterone
closely with urine output of <300 mL/day. (D) Excessive production of cortisol
Urinalysis reveals a sodium concentration of (E) Increased levels of angiotensin-converting
5 mEq/L and the presence of benign urinary enzyme
sediment. No proteinuria or hematuria is pres-
ent. Imaging of this patient’s kidneys is likely to 50. A newborn with hypercalciuria and hypokale-
reveal which of the following? mic alkalosis is diagnosed with neonatal Bart-
ter’s syndrome, a rare inherited dysfunction
(A) Enlarged cystic kidneys
of the thick ascending limb of the nephron.
(B) Enlarged kidneys with severe hydrone-
Which diuretic may mimic these symptoms by
phrosis
blocking a cotransporter found in this part of
(C) Horseshoe kidneys
the nephron?
(D) Normal kidneys with “flea-bitten” in-
farcted appearance (A) Acetazolamide
(E) Normal size and shape (B) Furosemide
(C) Hydrochlorothiazide
48. A 22-year-old man with no significant past (D) Mannitol
medical history presents to the emergency de- (E) Triamterene
392 Section II: Organ Systems  •  Answers

An s w e r s

1. The correct answer is B. The juxtaglomeru- not for sensing perfusion or secreting renin.
lar (JG) cells in the afferent arteriole and the Recall that the descending loop of Henle is
macula densa in the distal convoluted tubule permeable to water, while the ascending loop
together make up the JG apparatus, which is is not.
responsible for controlling renal blood flow via
Answer E is incorrect. The zona glomeru-
renin release. This patient’s condition is con-
losa is the outermost layer of the adrenal cor-
firmed by abdominal ultrasound, suggesting
tex and is responsible for the production of
that renal artery stenosis is the cause of hyper-
aldos­terone. Decreased perfusion of the zona
tension. In renal artery stenosis, the blood flow
glomerulosa would result in atrophy and de-
to the kidney is impeded. This low pressure is
High-Yield Systems

creased production of aldosterone, which


detected by JG cells in the afferent arteriole,
would lead to a decrease in blood pressure. Re-
which then secrete renin to raise blood pres-
call that aldosterone is responsible for sodium
sure and renal perfusion through the renin-
retention in the collecting duct.
angiotensin-aldosterone axis.
Answer A is incorrect. The adrenal medulla is 2. The correct answer is B. Glucose is freely
responsible for the release of catecholamines filtered at the glomerulus and, in healthy in-
and is composed of chromaffin cells. Whereas dividuals, is completely reabsorbed in the
catecholamines can increase blood pressure, proximal convoluted tubule (PCT). However,
decreased perfusion to the adrenal medulla as the filtered load of glucose increases, the re-
would result in a decrease in catecholamine absorption ability of the nephron is eventually
production. This would result in a lower blood saturated, and glucose begins to be excreted
pressure. Recall that the two catecholamines of in the urine. Due to significant variability in
interest are epinephrine and norepinephrine, nephron physiology, certain nephrons are satu-
of which epinephrine is the principal hormone rated at lower filtered loads than others. Thus,
produced in the adrenal medulla. Lastly, it between 200 and 375 mg/dL glucose begins to
should be noted that a pheochromocytoma is a appear in the urine, but not all nephrons are
tumor of the adrenal medulla and would result saturated. Above 375 mg/dL, all nephrons’
in paroxysms of hypertension. However, this transport maxima are reached, and any addi-
Renal

patient complains of an “unrelenting” head- tional glucose is excreted in the urine.


ache, which is not associated with a pheochro- Answer A is incorrect. Glomerular filtration is
mocytoma. never saturated because it does not depend on
Answer C is incorrect. The distal convoluted carrier-mediated transport. Furthermore, even
tubule contains the macula densa cells that if glucose were being incompletely filtered,
sense low sodium flow inside the nephron, an- this would tend to decrease (not increase)
other sign that the kidney is not well perfused. urine glucose levels.
The macula densa cells are in intimate contact Answer C is incorrect. To whatever extent pos-
with the JG cells, and although the macula sible, the kidney attempts to fully reabsorb glu-
densa cells help in sensing low perfusion, it is cose in the proximal tubule. Thus, it is never
the JG cells that both sense perfusion and se- secreted in the PCT.
crete renin. Furthermore, it is the afferent ar-
terioles that most directly sense a decrease in Answer D is incorrect. To whatever extent pos-
perfusion, as these structures receive blood di- sible, the kidney attempts to fully reabsorb glu-
rectly from the renal artery. cose in the proximal tubule. Thus, it is never
secreted in the PCT.
Answer D is incorrect. The loop of Henle is
primarily responsible for concentrating urine,
Chapter 15: Renal  •  Answers 393

Answer E is incorrect. The graph shows that gias), and hematologic (anemia, various cyto-
the urine glucose level begins to rise at a se- penias) systems. Treatment generally consists
rum glucose level of 200 mg/dL and is there- of immunosuppressive drugs such as predni-
fore detectable well before 375 mg/dL. sone.
Answer A is incorrect. Achalasia is not particu-
3. The correct answer is E. Thiazide diuretics
larly associated with lupus. It can be associated
inhibit Na+-Cl- reabsorption in the early dis-
with Chagas disease, as a result of infection
tal tubule. Increased distal Na+ delivery to the
with Trypanosoma cruzi.
principal cells of the late distal tubule causes
an increase in Na+ reabsorption paired with Answer B is incorrect. Aortic aneurysms are
an increase in K+ secretion to maintain elec- not particularly associated with lupus. They
troneutrality. Prolonged thiazide diuresis can can be associated with Marfan syndrome.

High-Yield Systems
cause hypokalemia.
Answer C is incorrect. Esophageal reflux is
Answer A is incorrect. A high-K+ diet will not particularly associated with lupus.
cause increased K+ secretion as the filtered
Answer E is incorrect. Morbilliform rashes are
load of K+ is increased.
more commonly associated with viral infec-
Answer B is incorrect. Aldosterone leads to in- tions.
creased synthesis of luminal Na+ channels and
basolateral Na+/K+-ATPase in the principal 5. The correct answer is D. Nonsteroidal anti-
cells of the collecting duct. Decreased Na+/ inflammatory drugs (NSAIDs) are cyclooxy-
K+-ATPase activity results in increased intracel- genase inhibitors. Because cyclooxygenase
lular K+ levels. catalyzes the initial step in the metabolism of
arachidonic acid, NSAIDs inhibit prostaglan-
Answer C is incorrect. In metabolic acidosis,
din and thromboxane production. In healthy
excess protons enter the principal cells of the
people, prostaglandins (which are vasodila-
late distal tubule and collecting duct. Proton
tors) are secreted at a low basal rate and have
entry forces the exit of K+ ions to maintain
little effect on renal perfusion. In patients with
electroneutrality. With decreased intracellular
states of effective volume depletion such as
K+ concentration, the driving force for K+ se-
heart failure and cirrhosis, there is increased
cretion is lost and hyperkalemia results.
secretion of the vasoconstrictors angiotensin

Renal
Answer D is incorrect. Amiloride falls into the II and norepinephrine. In these cases, there is
class of potassium-sparing diuretics. It acts on increased secretion of prostaglandins (mostly
the Na+ channels of the collecting duct to pre- prostacyclin and prostaglandin E2) to main-
vent Na+ reabsorption and consequently, K+ tain renal blood flow by counteracting the va-
secrection. The potassium-sparing diuretics are soconstrictive effects of these hormones at pre-
used often in combination with a thiazide or glomerular vessels (eg, the afferent arteriole).
a loop diuretic to prevent the excessive K+ loss When prostaglandin synthesis is inhibited by
that can occur with these other agents. NSAIDs in these patients, preglomerular re-
sistance increases and renal blood flow (and
4. The correct answer is D. Subendothelial de- thus glomerular filtration rate) is reduced. This
posits in the glomerular capillaries are indica- ischemia can lead to acute renal failure.
tive of lupus nephritis, a complication of sys-
Answer A is incorrect. NSAIDs act to inhibit,
temic lupus erythematosus (SLE) that largely
not increase, prostaglandin synthesis.
dictates the patient’s prognosis. SLE classically
presents with a malar “butterfly” rash on expo- Answer B is incorrect. Prostaglandins act as
sure to sunlight. Various manifestations of SLE vasodilators and do not have an effect on glo-
can affect different aspects of the body, such merular capillary permeability.
as the cardiovascular (pericarditis), respiratory
(pleural effusion), musculoskeletal (arthral-
394 Section II: Organ Systems  •  Answers

Answer C is incorrect. Inhibiting the vasodi- percoagulable state. Patients with nephrotic
latory effects of prostaglandins acts to increase syndrome are at increased risk for thromboem-
preglomerular resistance, causing renal ische­ bolic events due to renal losses of antithrom-
mia. bin III, protein C, and protein S, all of which
normally function as anticoagulants. Patients
Answer E is incorrect. Although NSAIDs can
with nephrotic syndrome also commonly have
also induce acute renal failure by causing an
other factors contributing to a hypercoagulable
acute interstitial nephritis or minimal change
state, including hemoconcentration, increased
disease, they have not been associated with di-
fibrinogen, and thrombocytosis. Mental sta-
rect toxic effects on the tubules or acute tubu-
tus or neurologic changes in patients with
lar necrosis.
nephrotic syndrome should be taken very se-
6. The correct answer is D. Membranoprolifera- riously. An INR of <1 indicates that a patient
High-Yield Systems

tive glomerulonephritis (MPGN) can present is prone to clot formation, whereas an INR >1
as a nephrotic syndrome (more common) or indicates that a patient is at increased risk of
as an acute nephritic syndrome. In MPGN, hemorrhage.
deposits are found in the subendothelial and Answer B is incorrect. Patients with decreased
mesangial space. The afferent and efferent factor II levels have an increased risk of hemor-
vessels are lined by fenestrated endothelium, rhagic events; nephrotic syndrome is not asso-
which is then partially lined by basement ciated with decreased factor II levels.
membrane. Beyond the basement membrane
Answer C is incorrect. Patients with nephrotic
lie the visceral epithelial cells and podocytes.
syndrome have increased fibrinogen levels,
The glomerular basement membrane does not
which predispose them to thromboembolic
completely surround the capillary, especially
events.
the endothelium adjoining the mesangial area.
As such, circulating immune complexes can Answer D is incorrect. Patients with nephrotic
deposit in the mesangium and subendothelial syndrome have decreased protein C levels,
space without having to cross the negatively which predispose them to thromboembolic
charged glomerular basement membrane. events.
Answer A is incorrect. Immune complex de- Answer E is incorrect. Patients with nephrotic
posits do not form in the subepithelial space syndrome have decreased protein S levels,
Renal

between the basement membrane and the vis- which predispose them to thromboembolic
ceral epithelium. events.
Answer B is incorrect. This describes the sub-
8. The correct answer is A. Pyelonephritis is an
epithelial space. Examples include membra-
infection of the kidneys caused by an ascend-
nous nephropathy.
ing infection from the lower urinary tract, most
Answer C is incorrect. This describes the sub- often caused by E coli from the periurethral/
epithelial space. The foot processes of podo- perianal area. The classic symptoms are fever,
cytes and the visceral epithelial cells together chills, flank pain, and costovertebral angle
entangle the side of the glomerular basement tenderness, all of which are demonstrated by
membrane opposite from the endothelium. this patient. Casts indicate that the origin of
the WBCs is the kidney, which confirms the
Answer E is incorrect. This describes Bow-
clinical suspicion of pyelonephritis. E coli is
man space and is the site of filtration. This
the major cause of pyelonephritis in uncom-
space is contiguous to the collecting duct.
plicated cases, accounting for 82% of cases in
7. The correct answer is A. This patient is suf- women and 73% of cases in men.
fering from a massive left middle cerebral Answer B is incorrect. K pneumoniae ac-
artery stroke that occurred secondary to a hy- counts for approximately 3% of cases in
Chapter 15: Renal  •  Answers 395

women and 6% of cases in men. It more com- intravascular stents in the renal artery, thereby
monly causes community-acquired pneu- restoring blood flow to the kidney. This form
monia or secondary infections in patients of therapy is the primary treatment for renal
with chronic obstructive pulmonary disease stenosis in symptomatic patients. Patency rates
(COPD). However, it can cause emphysema- after angioplasty are strongly dependent on the
tous urinary tract infections, especially in dia- size of the vessel treated and the quality of in-
betics. flow and outflow through the vessel.
Answer C is incorrect. P mirabilis is an un- Answer C is incorrect. In patients with hy-
common cause of pyelonephritis. It is a gram- pertension caused by bilateral renal artery ste-
negative bacterium that can be diagnosed nosis, both kidneys will be underperfused, so
because of its swarming motility and positive both will retain sodium and water by activating
urease activity. It is a more common cause of the renin-angiotensin/aldosterone system. Di-

High-Yield Systems
infected nephrolithiasis. uretics can counteract this effect and control
blood pressure; therefore they are appropriate
Answer D is incorrect. S saprophyticus is
in this clinical scenario.
a causative agent of pyelonephritis in ap-
proximately 3% of cases. More commonly, it Answer D is incorrect. Smoking is a risk factor
causes localized cystitis. Studies have found for development of atherosclerotic plaques that
this pathogen in 5%-20% of cases of cystitis. may occlude vessels such as the renal arteries.
Localized cystitis usually is not accompanied Quitting may lower the rate of atherosclerotic
by systemic symptoms such as fever. However, build-up. This recommendation is appropriate
cystitis may be present in conjunction with the for this patient.
pyelonephritis, as it is an ascending infection.
Answer E is incorrect. Surgery is another ther-
Symptoms of cystitis include urinary frequency
apeutic option for renal stenosis. It is indicated
and dysuria. Casts would not be seen if the in-
particularly when angioplasty cannot be per-
fection were localized to the urinary bladder.
formed, as in completely occluded renal ves-
Answer E is incorrect. U urealyticum can sels.
cause recurrent episodes of pyelonephritis, but
is uncommon in uncomplicated episodes such 10. The correct answer is C. The existence of a
as in this patient. single kidney that has not migrated from the

Renal
pelvis suggests a horseshoe kidney. A horse-
9. The correct answer is B. This patient is most shoe kidney forms when the inferior poles of
likely suffering from bilateral renal artery ste- two kidneys fuse during development. As the
nosis, indicated on physical exam by renal kidneys rise from the pelvis, they encounter
bruits. Stenosis of the renal arteries leads to a the inferior mesenteric artery and cannot rise
decrease in perfusion of the kidney. The re- to the normal level in the abdomen. These
sult is a drop in intraglomerular pressure and patients are typically asymptomatic if they
glomerular filtration rate (GFR). The under- have no other abnormalities, but they do have
perfused kidneys respond by upregulating the increased risks of obstruction, infection, and
renin-angiotensin-aldosterone system. Angio- stones.
tensin-converting enzyme (ACE) inhibitors
Answer A is incorrect. The aorta would not
prevent the vasoconstrictive effect of angio-
obstruct the path of a rising horseshoe kidney
tensin II on the efferent arterioles. Dilation
during development.
of the efferent arterioles eliminates the body’s
attempt to increase effective GFR. ACE in- Answer B is incorrect. The celiac trunk leaves
hibitors therefore should be avoided in patients the aorta at a level above the location of nor-
with renal stenosis. mally developed kidneys, and thus cannot be
responsible for the low location of a horseshoe
Answer A is incorrect. Angioplasty is a mini-
kidney.
mally invasive procedure that involves placing
396 Section II: Organ Systems  •  Answers

Answer D is incorrect. The inferior vena cava failure is normally >2% with an osmolality that
would not obstruct the path of a rising horse- is <350 mOsm/kg (similar to postrenal). How-
shoe kidney during development. ever, the presentation of severe intermittent
pelvic pain in the context of leukemia therapy
Answer E is incorrect. The superior mesen-
is more likely to be caused by a kidney stone.
teric artery leaves the aorta at the level where
normally developing kidneys are located, and Answer D is incorrect. The spleen can be in-
thus it cannot be responsible for the low level volved in leukemia, but the presence of acute
of a horseshoe kidney. renal failure in this case makes a urethral ob-
struction more likely.
11. The correct answer is E. The history of be-
ing started on chemotherapy for leukemia is 12. The correct answer is A. Renal prostaglan-
strongly suggestive of tumor lysis syndrome. din synthesis produces a vasodilatory effect on
High-Yield Systems

Tumor lysis syndrome occurs when leukemic the afferent arterioles, whereas angiotensin
cells die and release potassium, phosphate, II vaso­constricts primarily efferent arterioles.
and uric acid. Uric acid stones are radiolucent, NSAIDs, such as ketorolac, inhibit renal pros-
so they may not appear on x-ray films. It is taglandin synthesis. This results in a reduced
likely that this patient’s presentation has been ability to dilate the afferent arterioles. The pa-
caused by a kidney stone that has passed into tient may have underlying renal disease as a
the left ureter and now into the urethra, caus- result of her hypertension and diabetes. With-
ing postrenal failure. The FENa >4% tells you out adequate afferent arteriolar vasodilation,
that the obstruction is starting to damage the the GFR and renal plasma flow (RPF) are
nephrons as they are not conserving sodium. reduced, leading to acute renal failure. The
blood urea nitrogen (BUN):creatinine (Cr) ra-
Answer A is incorrect. Kidney failure as a re-
tio in the question stem is consistent with an
sult of glomerular dysfunction presents with
intrinsic cause of renal failure. There is no
a prerenal azotemia. There is an effective de-
change in the filtration fraction because both
crease in glomerular filtration rate, and sodium
RPF and GFR decrease proportionately.
and water are retained by the kidney. The frac-
tional excretion of sodium in prerenal failure is Answer B is incorrect. Constriction of the ef-
normally less than 1% with an osmolality that ferent arteriole would serve to decrease RPF
is >350 mOsm/kg. and increase GFR. NSAIDs cause constriction
Renal

of the afferent arteriole, not the efferent arte-


Answer B is incorrect. Acute interstitial dis-
riole. Both filtration fraction and GFR are in-
ease of the kidney is commonly caused by an
creased and RPF is decreased.
allergic reaction to medicine (eg, penicillin,
NSAIDs) or infection. In an acute setting, it Answer C is incorrect. Increased RPF, de-
presents with an intrinsic renal picture as is creased GFR, and increased filtration frac-
seen in this patient. In the setting of an infec- tion (FF) comprise an invalid combination,
tion, urine cultures are usually positive; in the because FF = GFR/RPF. Therefore, if GFR
setting of an allergic reaction, eosinophilia is decreases and RPF increases, FF also should
common. decrease.
Answer C is incorrect. Kidney failure as a re- Answer D is incorrect. No change in RPF, de-
sult of tubular dysfunction presents with an crease in GFR, and decrease in FF occur with
intrinsic renal picture. This is most commonly increased plasma protein concentration or
due to acute tubular necrosis or ischemia/tox- with constriction of the ureter, such as with ei-
ins. Patchy necrosis leads to debris obstruct- ther acquired or congenital urethral strictures
ing the tubules and fluid backflow, leading to or with prostate hypertrophy, a common condi-
a drop in glomerular filtration rate. The frac- tion in men over 60 years.
tional excretion of sodium in intrinsic renal
Chapter 15: Renal  •  Answers 397

Answer E is incorrect. No change in RPF, in- tors are coupled to the contraction of vascular
creased GFR, and increased filtration fraction smooth muscle.
occur with decreased plasma protein concen-
Answer D is incorrect. Activation of V1 recep-
tration.
tors leads to an increase in total peripheral re-
sistance.
13. The correct answer is E. In healthy people, os-
moreceptors in the wall of the third ventricle Answer F is incorrect. Renin is secreted by
sense increased body fluid osmolarity and trig- smooth muscle cells in the afferent arteriole
ger the release of ADH from the posterior pi- and acts to cleave angiotensinogen to angio-
tuitary. ADH exerts its main effects on the V2 tensin I. This activates the renin-angiotensin-
receptors located in the principal cells of the aldosterone axis, leading to increased salt and
late distal tubule and collecting duct, where water retention. A patient with persistent acti-

High-Yield Systems
a Gs protein-coupled mechanism directs the vation of this axis would present primarily with
insertion of aquaporin water channels into hypertension and edema with relatively low
the luminal wall. These channels are perme- urine sodium levels.
able only to water and result in a reabsorption
of water, concentration of urine, and dilution 14. The correct answer is C. This patient has
of body fluids. Activation of the V1 receptor drug-induced acute tubulointerstitial nephritis,
found in the vascular smooth muscles results which manifests histologically as edema and
in activation of Gq protein second-messenger inflammation of the renal tubules and intersti-
cascade and contraction of vascular smooth tium with sparing of the glomeruli. Tubuloin-
muscle, leading to an increase in total periph- terstitial nephritis can be caused by infections
eral resistance. In patients with the syndrome and autoimmune phenomena, but is associ-
of inappropriate ADH secretion (SIADH), ated most commonly with drug toxicity. The
which can be caused by central nervous system typical picture is that of acute renal failure
(CNS) disturbances (eg, stroke, hemorrhage, after drug administration. Patients classically
infection), small cell lung carcinoma, intracra- present with the triad of low-grade fever, rash,
nial neoplasms, and occasionally by pancreatic and arthralgias, although some studies indicate
tumors, the unregulated release of ADH leads <10% of patients report all three symptoms.
to the persistent excretion of concentrated Other symptoms include those associated with
urine high in sodium. This causes hyponatre- acute renal failure, such as oliguria, malaise,

Renal
mia and decreased serum osmolality without anorexia, and vomiting. Common findings on
potassium or acid-base disturbances. urinalysis are sterile pyuria, microscopic he-
maturia, and eosinophiluria. Drugs that have
Answer A is incorrect. ACTH is secreted by
been associated with tubulointerstitial nephri-
the anterior pituitary in response to the pres-
tis include sulfonamides (including thiazide
ence of corticotropin releasing hormone pro-
diuretics and most loop diuretics), methicillin,
duced in the hypothalamus. It can also be se-
ciprofloxacin, cephalosporins, allopurinol, pro-
creted by pituitary tumors or small cell lung
ton pump inhibitors, rifampin, cimetidine, and
carcinomas, but would present with Cushing
nonsteroidal anti-inflammatory agents. With-
syndrome (hypertension, weight gain, buffalo
drawal of the causative agent is often the best
hump, truncal obesity, striae, hyperglycemia,
treatment, but it may take months for a patient
and osteoporosis) rather than hyponatremia.
to fully recover renal function. Hydrochloro-
Answer B is incorrect. Neuronal signals from thiazide is a thiazide diuretic and a first-line
the osmoreceptors of the third ventricle stimu- antihypertensive drug that is especially useful
late the production of ADH as well as stimu- in the elderly and African-American popula-
late the sensation of thirst. tions. Adverse effects of hydrochlorothiazide
include electrolyte disturbances such as hypo-
Answer C is incorrect. V2 receptors are cou-
kalemia and hypercalcemia.
pled to the insertion of aquaporins; V1 recep-
398 Section II: Organ Systems  •  Answers

Answer A is incorrect. Cyclophosphamide is lar pattern of IgG and complement along the
an alkylating agent that cross-links DNA and basement membrane. Membranous glomeru-
prevents DNA synthesis and cell division. It is lonephritis is the most common cause of adult-
used as an anti-neoplastic agent or as an im- onset nephrotic syndrome. Patients with this
munosuppressant in transplant recipients and disease normally present with a nephrotic pic-
patients with autoimmune disease. Common ture of generalized edema due to massive loss
adverse effects include alopecia, myelosuppres- of albumin and other proteins.
sion, nausea and vomiting, and hemorrhagic
Answer B is incorrect. This is a finding of
cystitis. Cyclophosphamide also can cause
membranoproliferative glomerulonephritis, an
renal tubular necrosis, but it is not associated
uncommon autoimmune renal disorder that
with tubulointerstitial nephritis.
normally affects young individuals (8-30 years
Answer B is incorrect. Diphenhydramine is a of age). The diagnosis is based on a histologic
High-Yield Systems

first-generation H1-antagonist used to treat al- presentation that includes mesangial prolif-
lergic reactions, motion sickness, and dystonic eration and a tram-track appearance on light
reactions. It may cause sedation, anticholiner- microscopy. As this patient is 50 years old, this
gic effects, and anti-a-adrenergic effects. It is diagnosis is less likely.
not associated with acute tubulointerstitial ne-
Answer C is incorrect. This is a description
phritis.
of the findings in acute poststreptococcal glo-
Answer D is incorrect. Isoniazid decreases the merulonephritis, an autoimmune disease most
synthesis of mycolic acids and is used to treat frequently seen in children. It normally pre­
tuberculosis (TB). It is associated with neuro- sents a few weeks after a streptococcal infec-
toxicity, hepatotoxicity, a lupus-like syndrome, tion (throat or skin) with peripheral and peri-
and hemolysis in patients with glucose-6-­ orbital edema, dark, tea-colored urine, and
phosphate dehydrogenase deficiency. Isoniazid proteinuria. These symptoms are caused by
is not associated with acute tubulointerstitial circulating anti-streptococcal antibody-antigen
nephritis. complexes that deposit in the glomerular base-
ment membrane, leading to complement acti-
Answer E is incorrect. Lithium is a mood sta-
vation and glomerular damage. As the patient
bilizer used to treat bipolar affective disorder.
has been otherwise healthy and is 50 years old,
It indirectly inhibits the reuptake of serotonin
this diagnosis is unlikely.
Renal

and norepinephrine by inhibiting the phos-


phatidylinositol second messenger system. Answer D is incorrect. This is the main find-
Adverse effects include CNS depression, diz- ing in IgA nephropathy (Berger disease). This
ziness, nephrogenic diabetes insipidus, acne, disease presents within several days of an
edema, and hyperthyroidism, as well as many infection (as opposed to poststreptococcal glo-
others. Lithium has been associated with merulonephritis, which presents weeks after
chronic tubulointerstitial nephritis, which infection) with a nephritic picture due to IgA
presents after years of chronic lithium therapy. deposition in the mesangium. It is the most
Proteinuria, hypertension, and anemia may be common global nephropathy, but it is a mild
seen in these patients, along with other signs of disease with minimal clinical significance. As
chronic lithium poisoning such as thyroid ab- this patient has not had a recent infection, this
normalities or ECG changes. diagnosis is unlikely.
Answer E is incorrect. This is a finding of Al-
15. The correct answer is A. This vignette de-
port syndrome, a heterogeneous (although
scribes a nephrotic syndrome. Spike-and-dome
most commonly X-linked) genetic disorder
deposits are found only in membranous glo-
with either absent or mutated collagen IV,
merulonephritis. Membranous glomerulo-
which leads to a nephritic renal disease and
nephritis is an immune complex-mediated
to sensorineural hearing loss and ocular disor-
disease. Immunofluorescence shows a granu-
ders. This patient has no other complaints, and
Chapter 15: Renal  •  Answers 399

the edema is a fairly recent finding, making known as Alport syndrome. This syndrome is
this diagnosis less likely. characterized by lens displacement, cataracts,
and nerve deafness and is associated with he-
16. The correct answer is C. The glomerular fil- maturia.
tration barrier is composed of endothelial cells,
glomerular basement membrane, and epithe- 17. The correct answer is E. The only diuret-
lial podocytes. It is responsible for the filtra- ics that specifically limit calcium loss are the
tion of plasma according to size, shape, and thiazides. They act in the early distal tubule,
net charge. The distances between the podo- which is marked as region E in the image.
cyte foot processes (filtration slits), the pores
Answer A is incorrect. There are no diuretics
of the glomerular basement membrane, and
that act at the glomerulus.
the fenestrations between the endothelial cells
Answer B is incorrect. Carbonic anhydrase in-

High-Yield Systems
limit the size and shape of the filtrate. The
negatively charged heparan sulfate coating the hibitors, which act in the proximal convoluted
filtration barrier prevents negatively charged tubule, do not affect calcium excretion.
molecules, such as albumin, from being fil- Answer C is incorrect. Osmotic diuretics act
tered into the urine. This patient has minimal in the loop of Henle (as well as the proximal
change disease manifested by nephrotic syn- convoluted tubule and collecting duct), but
drome, in which the negatively charged hepa- they do not affect ion channels.
ran sulfate is lost, thereby allowing plasma pro-
tein to be lost in the urine. Answer D is incorrect. Loop diuretics, which
encourage calcium excretion, act in the thick
Answer A is incorrect. A brush border is char- ascending limb.
acteristic of the proximal tubules and refers to
the thickened appearance of the apical surface Answer F is incorrect. Potassium-sparing di-
of these tubules due to the presence of micro- uretics and ADH antagonists such as lithium
villi covered by a dense glycocalyx. It is not and demeclocycline act along the collecting
part of the glomerulus and is not affected in tubule, although neither class affects calcium
minimal change disease. excretion.
Answer B is incorrect. The endothelial cell, 18. The correct answer is A. This patient is suf-
as previously mentioned, makes up part of the fering an acute overdose of amphetamine. He

Renal
glomerular filtration barrier. However, it is in- should be treated with ammonium chloride to
tact in minimal change disease. acidify his urine and increase renal clearance
Answer D is incorrect. Integrins are trans- of the weak base. This phenomenon, called
membrane proteins that serve as cell adhesion ion trapping, occurs because increasing the
molecules, allowing cells to adhere to the un- ratio of ionized to non-ionized drug species in
derlying extracellular matrix. In leukocyte ad- the renal tubule allows more of the drug to be
hesion deficiency type 1, a deficiency in b-2 retained in the urine and excreted. Weak bases
integrin results in an inability on the part of in acidic environments have high ratios of ion-
leukocytes to adhere to the endothelium for ized species, which are water-soluble and do
transmigration into the tissue, resulting in re- not cross membranes. When urine is acidified,
current infections. Integrins are not involved the levels of ionized amphetamine are high,
in the glomerular filtration barrier and play no and therefore more of the drug is trapped in
role in the etiology of minimal change disease. the renal tubule.
Answer E is incorrect. Type IV collagen is the Answer B is incorrect. Alkalinization with bi-
collagen component of the basement mem- carbonate is used to increase renal clearance
brane and is not compromised in nephrotic of weak acids such as phenobarbital, metho-
syndrome. When mutated, however, it gives trexate, tricyclic antidepressants, and aspirin.
rise to a form of basement membranopathy
400 Section II: Organ Systems  •  Answers

Answer C is incorrect. Flumazenil is a treat- Answer D is incorrect. Maternal diabetes has


ment for acute benzodiazepine overdose. been associated with polyhydramnios. The
pathophysiology behind this phenomenon is
Answer D is incorrect. Naloxone is a treat-
that increased serum glucose in the mother is
ment for opioid overdose.
transmitted to the fetus. Just as in any other di-
Answer E is incorrect. Treatment with water abetic patient, increased blood glucose acts as
would have no effect on this patient’s acute in- an osmotic diuretic, causing an increased pro-
toxication. duction of urine. Any cause of increased urina-
tion in the fetus will lead to polyhydramnios.
19. The correct answer is B. The neonate de-
scribed above is suffering from Potter syn- Answer E is incorrect. Trisomy 18, or Edwards
drome, a constellation of abnormalities in- syndrome, also can present with facial defor-
cluding flattened facies, large and low-set ears, mities such as low-set ears and limb deformi-
High-Yield Systems

and limb deformities. The hallmark of this ties such as clenched hands. It is, however, not
syndrome, however, is that these fetuses have associated with a flattened facies. Instead, the
bilateral renal agenesis, which is incompatible pathognomonic features of this syndrome are
with life. As a result of bilateral renal agenesis, micrognathia and a prominent occiput, along
the fetus cannot urinate. This results in oligo- with the aforementioned anomalies in the
hydramnios. Another very common cause of ears and limbs. Edwards syndrome is the most
oligohydramnios is chronic uteroplacental common trisomy resulting in live birth after
insufficiency (UPI). In cases of UPI, oligohy- Down syndrome, and has not been associated
dramnios occurs, because the fetus does not with oligohydramnios in utero.
receive the proper nutrients or blood volume
20. The correct answer is C. This patient is suf-
to maintain an adequate glomerular filtration
fering from a severe metabolic acidosis, as
rate. As a result, the fetus does not make urine
indicated by low serum pH, a low plasma bi-
and oligohydramnios develops.
carbonate level, and compensatory hyperventi-
Answer A is incorrect. Anencephaly is a com- lation manifested as tachypnea. Next, note that
mon neural tube defect. It is defined as partial the anion gap is elevated at 20 mmol/L (nor-
or complete absence of the fetal brain or cra- mal: 3-11 mmol/L). Of the choices, only iron
nial vault. This condition is lethal, and most toxicity (the I in the mnemonic MUDPILES)
Renal

fetuses are stillborn. It can be diagnosed using is associated with elevated anion gap metabolic
ultrasound to identify the neural tube defect. acidosis.
Polyhydramnios also is commonly found in an-
Answer A is incorrect. Aldosterone normally
encephaly, because the fetus is unable to swal-
functions to increase sodium reabsorption, po-
low amniotic fluid.
tassium excretion, and proton excretion in the
Answer C is incorrect. Duodenal atresia distal nephron. Thus, aldosterone antagonists
would lead to polyhydramnios. Polyhydram- cause diminished proton excretion and meta-
nios refers to excessive amounts of amniotic bolic acidosis. However, unlike this case, the
fluid. Conditions that cause polyhydramnios acidosis would be expected to occur in the set-
include fetal malformations that impair swal- ting of a normal anion gap.
lowing such as esophageal atresia and anen-
Answer B is incorrect. Most simple antacids
cephaly. Polyhydramnios can be diagnosed
are alkaline and operate by neutralizing the
with ultrasound imaging, but determining the
low pH of the stomach. Thus, overdose would
etiology may require additional clinical work-
be expected to cause a metabolic alkalosis, not
up. This condition is not associated with bilat-
an acidosis.
eral renal agenesis, which would prevent the
fetus from urinating and result in oligohydram- Answer D is incorrect. Loop diuretic toxicity
nios. is associated with metabolic alkalosis, not aci-
dosis.
Chapter 15: Renal  •  Answers 401

Answer E is incorrect. Ingestion of opiates Answer D is incorrect. IgA nephropathy usu-


results in reduced respiratory drive and subse- ally presents with a nephritic picture, which
quent hypoventilation. In this situation, a res­ does not involve the massive proteinuria that is
piratory acidosis would have been expected, seen in nephrotic syndromes.
and tachypnea would not be observed.
Answer E is incorrect. IgA deposition in
Berger disease is primarily in the mesangium
21. The correct answer is C. IgA nephropathy
and not the subepithelium.
(Berger disease) occurs within several days of
an infection, as opposed to poststreptococcal
22. The correct answer is D. This patient has dif-
glomerulonephritis, which presents weeks af-
fuse cortical necrosis: generalized infarctions
terward. Classically, patients present with a
of the cortices of both kidneys, which is a com-
nephritic picture due to IgA deposition in the
mon complication of disseminated intravascu-

High-Yield Systems
mesangium. Berger disease is the most com-
lar coagulation (DIC). DIC commonly occurs
mon global nephropathy and is generally a
after a complication of pregnancy such as am-
mild disease. IgA nephropathy usually pre­sents
niotic fluid embolus and placental abruption,
in children with recurrent hematuria that is
and affected patients develop the abrupt on-
of minimal clinical significance. Nonlinear
set of the triad of anuria, gross hematuria, and
mesangial deposits of IgA are evident on im-
flank pain. The diagnosis can usually be es-
munofluorescence. Treatment is with ACE in-
tablished by ultrasonography, which will dem-
hibitors and corticosteroids. Patients with IgA
onstrate hypodense areas in the renal cortex.
nephropathy have a risk of disease recurrence.
Although many patients can be sustained on
Answer A is incorrect. Increased antistreptoly- dialysis, only 20%-40% have partial recovery of
sin O (ASO) titers are associated with acute kidney function. Indications for acute dialysis
poststreptococcal glomerulonephritis rather in DIC include (1) Acidosis refractory to bi-
than IgA nephropathy. The classic findings in- carbonate, (2) severe Electrolyte abnormalities
clude RBCs and casts in the urine (causing the refractory to medical intervention (especially
tea-colored appearance), elevated ASO titers, high potassium levels), (3) Intoxication with
decreased complement levels, and “lumpy- some drugs, (4) volume Overload refractory to
bumpy” electron-dense deposits in the glo­ diuretics, and (5) Uremic symptoms (eg, car-
merulus. diac friction rub and altered mental status),

Renal
making up the mnemonic AEIOU. The fact
Answer B is incorrect. Acute poststreptococcal
that this patient has a new-onset pericardial
glomerulonephritis is an autoimmune disease
friction rub indicates uremia and makes dialy-
most frequently seen in children. It normally
sis imperative.
presents a few weeks after a streptococcal in-
fection with a nephritic picture of peripheral Answer A is incorrect. Aggressive fluid sup-
and periorbital edema, dark urine, and protein- port is not beneficial for kidney recovery after
uria. These symptoms are caused by circulat- the development of diffuse cortical necrosis.
ing antistreptococcal antibody-antigen com- The kidney has been severely damaged by the
plexes that deposit in the glomerular basement microthrombi of DIC. Aggressive fluid resusci-
membrane, leading to complement activation tation is contraindicated due to (1) the lack of
and glomerular damage. The classic findings hypotension, and (2) the renal failure. Fluids
are RBCs and casts in the urine (which cause will only cause volume overload if they cannot
the characteristic tea-colored urine), a positive be excreted.
ASO titer, decreased levels of complement,
Answer B is incorrect. While renal malig-
and “lumpy-bumpy” electron-dense deposits in
nancy can cause hematuria, it is less likely to
the subepithelium of the glomerulus. Recov-
cause renal failure. In this case a biopsy to look
ery is spontaneous and treatment is supportive.
for renal malignancy is not necessary because
the cause of the patient’s symptoms is already
402 Section II: Organ Systems  •  Answers

known. The first treatment should be dialysis of nucleic acid turnover, which is heightened
to counteract renal failure and allow any re- in the setting of cell destruction.
maining renal tissue to recover.
Answer E is incorrect. Urinary tract infection
Answer C is incorrect. Broad-spectrum anti- with urease-positive microorganisms such as
biotics are indicated in cases of shock due to Staphylococcus saprophyticus can form large
sepsis. This patient has DIC caused by an am- struvite calculi that are radiopaque, but would
niotic embolus, and since there is no infection, not backflow into the ureters.
antibiotics will not be beneficial. Additionally,
as the patient is experiencing renal failure, any 24. The correct answer is E. The man is going
antibiotic that is renally metabolized should into hypovolemic shock due to hemorrhage.
be renally dosed to account for the patient’s The body’s normal response to hypovolemia in
creatinine clearance rate. the short term results from a decreased blood
High-Yield Systems

volume that is sensed by baroreceptors, which


Answer E is incorrect. This patient is in severe
leads to an increased sympathetic output. The
acute renal failure. Failure to treat will result
response is an increased heart rate and vaso-
in death.
constriction to increase cardiac output so that
the body can continue to perfuse vital organs.
23. The correct answer is C. Calcium stones
The kidney senses decreased volume and in-
are the most common cause of kidney stones
creases renin production, which will lead to
(80%-85%). Therefore, states that lead to in-
increased angiotensin II and aldosterone lev-
creased calcium (such as hyperparathyroidism,
els. In addition, the body will increase levels of
or other destructive bone diseases) can lead to
vasopressin, or ADH, which will conserve so-
their formation. The stones are made of cal-
dium and water by facilitating water reabsorp-
cium oxalate or calcium phosphate, and are
tion in the distal collecting duct.
radiopaque; thus large, numerous stones would
likely be seen on a plain film as well as non- Answer A is incorrect. The heart rate will
contrast CT. Other risk factors are increased be increased in hypovolemic shock, not de-
vitamin D and milk-alkali syndrome. Re- creased.
member, parathyroid hormone (PTH) brings
Answer B is incorrect. The peripheral resis-
calcium from bones into the bloodstream.
tance will be increased in response to hypovo-
Calcitonin “tones down” the bloodstream
Renal

lemia in order to increase effective cardiac out-


(“channels”) of calcium and puts it on bone.
put to the vital organs.
Answer A is incorrect. Calcified arteries are
Answer C is incorrect. In a normal patient, re-
unlikely to be mistaken for ureters, and a full
nin is secreted in response to low blood pres-
rectum would not cause peristaltic pain.
sure, so it should increase not decrease.
Answer B is incorrect. A large stool is unlikely
Answer D is incorrect. Vasopressin levels will
to cause flank pain, and pain would not come
not be decreased in hypovolemic shock.
in the waves of pain this patient is experienc-
ing. 25. The correct answer is A. The patient’s symp-
Answer D is incorrect. Normally hyperurice- toms are of angioedema, a well-known adverse
mia leads to kidney stones that are radiolucent effect of ACE inhibitors. These drugs are gen-
and therefore not seen on X-ray. They would, erally prescribed to patients with hyperten-
however, be seen on noncontrast CT. These sion. Angioedema is caused by the secondary
stones are often seen in the setting of diseases activity of ACE on the degradation of kinins,
with increased cell proliferation and turnover, including, most commonly, bradykinin. The
such as leukemia and myeloproliferative disor- blockage of ACE, and the commensurate ac-
ders. Remember that uric acid is a metabolite cumulation of high levels of bradykinin, ac-
count for increased vessel permeability and
Chapter 15: Renal  •  Answers 403

subsequent edema in the face, lips, mouth, When aminoglycosides are combined with
and subglottic tissues, typically without pruri- cephalosporins such as cefoxitin, the nephro-
tus. Intestinal edema accounts for the patient’s toxic effects are greatly increased. Renal failure
gastrointestinal symptoms of diarrhea, vomit- is usually reversible when the drugs are discon-
ing, and abdominal pain. Older patients with tinued.
a prior history of drug-induced hypersensitivity
Answer A is incorrect. Azithromycin is a mac-
and environmental allergies are particularly
rolide that is not an appropriate treatment for
susceptible to ACE inhibitor-induced angio-
Klebsiella pneumoniae infection. It is used
edema.
mostly against infection by gram-positive or-
Answer B is incorrect. Histamine is a biogenic ganisms. Although it can be associated with
amine that has a variety of functions, includ- allergic hepatitis and thrombophlebitis, it does
ing inflammation, smooth muscle and vascular not cause nephrotoxicity.

High-Yield Systems
dilatation, and neurotransmission. Histamine
Answer B is incorrect. Aztreonam shows
released by mast cells is primarily responsible
strong activity against gram-negative organ-
for the hypersensitivity reaction, although in
isms, and it is highly resistant to b-lactamase
this case histamine levels are not affected by
degradation. It is not associated with nephro-
the use of ACE inhibitors.
toxicity.
Answer C is incorrect. Prostacyclin (or pros-
Answer C is incorrect. Clindamycin use is
taglandin I2) is an arachidonic acid derivative,
typically limited to anaerobic abscesses. It is
produced by the vascular endothelial cells
known to cause pseudomembranous colitis
from PGH2. Its major function is to prevent
in up to 20% of patients, but it does not cause
platelet aggregation during coagulation. Pros-
nephrotoxicity.
tacyclin is also a potent vasodilator, although
its metabolism is not affected by ACE, and Answer D is incorrect. Piperacillin, although
thus prostacyclin levels are not affected by the effective against Klebsiella, is associated with
use of ACE inhibitors. hypersensitivity reactions in about 10% of pa-
tients. It is not associated with nephrotoxicity.
Answer D is incorrect. Prostaglandin E2
(PGE2) is an arachidonic acid derivative that 27. The correct answer is D. This patient pre­
controls smooth muscle contraction, dilata- sents with pyelonephritis, which is character-

Renal
tion, and constriction of blood vessels, as well ized by costovertebral angle tenderness, fever,
as the modulation of inflammation. It is also and chills. Symptoms of lower urinary tract
implicated in the induction of fever. However, infection (UTI) may also be present, such as
PGE2 has no relationship to ACE inhibitors dysuria, increased frequency of urination, and
and the kinin-related effects of ACE. urgency. The onset of symptoms of pyelone-
Answer E is incorrect. Serotonin is primarily phritis often occurs approximately one week
a neurotransmitter produced by the CNS and after the onset of a lower UTI. In children, re-
certain peripheral neurons (enteric neurons). current UTIs suggest an anatomic abnormality
It is not involved in vascular dilatation and in- and warrant further investigation. Lower UTIs
flammation, and has no relationship to ACE may ascend to the kidneys through incompe-
inhibitors. tent ureterovesical sphincters, leading to pyelo-
nephritis, dilatation of the ureters, and renal
26. The correct answer is E. Tobramycin is an pelves, potentially causing renal scarring. Thus
aminoglycoside, and like other drugs in this urologic repair is often recommended to pre-
family (eg, gentamicin, streptomycin) it can vent renal damage in children with vesicoure-
cause nephrotoxicity. This is the result of acute teral reflux.
tubular necrosis, and leads to a reduction in
Answer A is incorrect. Immunoglobulin defi-
the glomerular filtration rate and a rise in the
ciency, such as Bruton X-linked agammaglob-
serum creatinine level, as seen in this patient.
404 Section II: Organ Systems  •  Answers

ulinemia, is associated with recurrent bacterial Answer D is incorrect. Focal segmental glo-
infections that in this age group typically affect merulosclerosis is associated with a number
the lungs, ears, skin, and sinuses. of conditions, and it is often presents with ne-
phrotic syndrome. It is recognized histologi-
Answer B is incorrect. Nephroblastoma, or
cally by the sclerosis of parts of a minority of
Wilms tumor, is the most common solid renal
the glomeruli.
tumor in children and usually presents with a
palpable abdominal mass. Vesicoureteral re- Answer E is incorrect. Pyelonephritis is usu-
flux is more prevalent than nephroblastoma. ally caused by an ascending infection. It is
characterized by tubular necrosis, as seen here,
Answer C is incorrect. Poststreptococcal
but it is seen on a background of inflamma-
glomerulonephritis typically presents one-
tion, with numerous neutrophils present in the
two weeks after an infection with Group A
tubule lumen.
High-Yield Systems

b-hemolytic streptococci. The vignette does


not provide any evidence of a recent strepto-
29. The correct answer is B. When the amount of
coccal infection.
fluids in the body contracts, the body attempts
to compensate by releasing angiotensin II, a
28. The correct answer is A. This patient is suf-
potent vasoconstrictor. In order to protect the
fering from acute tubular necrosis (ATN) as a
kidney from losing its perfusion due to this
result of rhabdomyolysis brought on by statin
vaso­constriction, the kidney simultaneously
use. Myoglobin released by the muscles is
releases prostaglandins at both the afferent and
toxic to the kidney tubule cells, especially
efferent arterioles, where they act as vasodila-
those of the proximal tubule. In this image, all
tors. By inhibiting cyclooxygenase (COX)-1
of the tubules are necrotic with sloughed pink
and/or COX-2 enzymes, the pathway that
epithelial cells and debris and loss of nuclear
produces the prostaglandins that keep the kid-
detail. It is commonly associated with oligu-
neys perfused becomes blocked, leading to de-
ria, but ATN actually presents with increased
creased blood flow to the kidneys and resulting
urine output in 50% of patients. Rhabdomy-
in a prerenal cause of renal failure. Celecoxib
olysis is an adverse effect of statins that is more
is a selective COX-2 inhibitor that affects the
likely with higher doses of statins. In addition,
arterioles of the kidney and can cause renal
statins are metabolized by cytochrome P450
failure in dehydrated patients.
enzymes. If some of this patient’s cold medi-
Renal

cations inhibited the P450 enzymes, then the Answer A is incorrect. Inhibition of the
risk of rhabdomyolysis would be increased COX-2 enzyme will lead to a decrease in the
through increasing statin levels above normal. production of prostacyclin, causing a possible
increase in platelet aggregation as they con-
Answer B is incorrect. Amyloidosis has a num-
tinue to produce thromboxane, which is pro-
ber of causes, but is characterized histologi-
duced mainly by the COX-1 pathway. How-
cally by deposits of cotton candy-like material
ever, this effect is not the reason why COX-2
in the blood vessel walls and the glomeruli.
inhibitors are contraindicated in the context of
Answer C is incorrect. Analgesic nephropathy volume contraction.
is a type of tubulointerstitial nephritis that is
Answer C is incorrect. Celecoxib is a selec-
associated with the long-term use of NSAIDs.
tive COX-2 inhibitor that is effective because it
The histologic appearance shows interstitial fi-
spares the gastric mucosa the damaging effects
brosis and inflammation. Although this patient
of COX-1 inhibition.
has been taking over-the-counter cold rem-
edies (which often contain NSAIDs), it usually Answer D is incorrect. The macula densa is a
takes many years of high-level exposure to de- specialized portion of the thick ascending limb
velop this condition. adjacent to the hilus of the glomerulus. The
cells of the macula densa sense changes in so-
dium and chloride concentrations as well as a
Chapter 15: Renal  •  Answers 405

drop in blood pressure, and they secrete renin However, it does not offer renal protective ef-
in response. This patient’s vomiting will acti- fects in diabetic patients.
vate the renin-angiotensin pathway due to fluid
Answer C is incorrect. Furosemide is a loop
and electrolyte loss, but the celecoxib will have
diuretic that acts by inhibiting sodium reab-
little impact on the macula densa itself.
sorption in the loop of Henle. It is used in the
Answer E is incorrect. Inhibition of the treatment of volume overload in patients with
COX-2 enzyme will lead to a decrease in the CHF as well as in drug-resistant hypertension.
production of prostaglandins, thromboxane, It does not offer diabetics any renal protective
and prostacyclins, which affect platelet func- effects.
tion and small vessel diameter. This effect is
Answer D is incorrect. Hydrochlorothiazide
not contraindicated in the context of volume
is a thiazide diuretic that works by inhibiting
contraction.

High-Yield Systems
sodium reabsorption in the distal convoluted
tubule. It is used often as first-line treatment of
30. The correct answer is E. This patient has ne-
hypertension. However, it offers no renal pro-
phropathy, as evidenced by the lower-extremity
tective effects in diabetic patients or others.
swelling (as a result of hypoalbuminemia), hy-
perlipidemia, and proteinuria. In this case the
31. The correct answer is D. Focal hyaline de-
most likely explanation is diabetic nephropa-
posits are characteristic of focal segmental
thy. This is caused by constant vasoconstric-
glomerulosclerosis (FSGS). FSGS presents as
tion of the efferent arteriole of the nephron by
nephrotic syndrome (massive proteinuria, hy-
angiotensin II. Initially this serves to preserve
poalbuminemia, generalized edema, hyperlip-
glomerular filtration rate and kidney function
idemia, and lipiduria) and often occurs in pa-
in the face of hypertension, but over time, hy-
tients with HIV infection.
perfiltration injury to the glomerulus, in ad-
dition to hyperglycemic injury (this patient Answer A is incorrect. Rapidly progressive
clearly has suboptimal glucose control) to the glomerulonephritis is characterized by the
glomerulus by non-enzymatic glycosylation, presence of crescents in most glomeruli. The
led to the development of nephritic disease. description of this patient indicates that many
One of the main therapies aimed at prevention glomeruli are spared, making this syndrome
of diabetic nephropathy is the use of an ACE unlikely.

Renal
inhibitor or angiotensin receptor blocker such Answer B is incorrect. IgA nephropathy (Ber­
as losartan to prevent hyperfiltration injury to ger disease) is characterized by the presence of
nephrons and to treat hypertension. prominent IgA, not IgM, deposits in the mes­
Answer A is incorrect. Metoprolol is a specific angial region of the glomerulus. As it causes
b1-receptor blocking agent used to treat hyper- the nephritic syndrome, mild proteinuria usu-
tension and congestive heart failure (CHF). ally is present, and hematuria usually is found.
Blockade of b1-receptors leads to the inhibi- Answer C is incorrect. Although membranous
tion of sympathetic stimulation of heart rate, glomerulonephritis is the most common cause
then to a slower heart rate and decreased car- of nephrotic syndrome, it is characterized by
diac muscle contractility. It does not have any diffuse thickening of capillary walls and depos-
renal protective effects. its of IgG in the glomerular basement mem-
Answer B is incorrect. Amlodipine is a cal- brane. Thus it is an unlikely the cause of this
cium channel blocker that is used often as first- patient’s symptoms.
line antihypertensive therapy. Inhibition of Answer E is incorrect. Minimal change dis-
calcium channels leads to decreased ability of ease is the most common cause of the ne-
vascular smooth muscle to contract, leading to phrotic syndrome in children. It usually re-
less vasoconstriction and lower blood pressure. sponds well to corticosteroids, however, and
appears normal on light microscopy. Retrac-
406 Section II: Organ Systems  •  Answers

tion of foot processes is consistently seen on prerenal (due to lack of perfusion of the kid-
electron microscopy. ney), intrinsic renal (due to acute tubular ne-
crosis from ischemia or toxins), and postrenal
32. The correct answer is E. Increased sodium (due to obstruction of outflow). Each type of
intake leads to volume expansion and in- renal failure has distinct characteristics, allow-
creased stretch in mechanoreceptors located ing differentiation by urinalysis. In postrenal
in the afferent arteriole. The mechanorecep- failure, the kidneys are unable to effectively
tor response to increased plasma volume is concentrate the urine, so the urine osmolality
decreased renin secretion, which causes vaso- would be <350 mmol/kg.
dilation of glomerular afferent arterioles. This
Answer A is incorrect. A BUN:Cr ratio <15 is
increases the GFR while also decreasing so-
seen with intrinsic renal failure because renal
dium reabsorption in the proximal tubule.
damage causes decreased BUN reabsorption.
High-Yield Systems

Answer A is incorrect. Atrial natriuretic pep- In postrenal failure, reduced flow causes in-
tide is secreted by the atria in response to in- creased BUN reabsorption without increased
creased extracellular fluid volume and causes Cr reabsorption, therefore the BUN:Cr ratio is
dilation of the glomerular afferent arterioles. >15.
However, this is a physiologic response by cells
Answer B is incorrect. Epithelial casts are
in the atria of the heart to high sodium intake
seen in acute tubular necrosis, a cause of in-
and volume expansion, not a response intrinsic
trinsic renal failure.
to the kidney.
Answer C is incorrect. The fractional excre-
Answer B is incorrect. Increased delivery of
tion of sodium (FeNa) is a measure of the
sodium and water to the macula densa would
amount of sodium in the urine compared
lead to a decrease in renin release from the
to the amount of sodium in the plasma, and
juxtaglomerular apparatus, as renin would oth-
can be calculated by [(UNa × PCr) / (PNa ×
erwise lead to aldosterone release and further
UCr)] × 100. A FeNa <1% would be expected
sodium retention.
in prerenal failure, where the kidney is work-
Answer C is incorrect. Increased plasma so- ing to reabsorb as much sodium as possible to
dium and water leads to increased sodium increase plasma volume and thereby improve
chloride delivery to the macula densa, leading perfusion. In postrenal failure, the kidney is
Renal

to the suppression (not the increase) of renin unable to effectively reabsorb sodium, and
release by the juxtaglomerular apparatus. De- therefore the FeNa would commonly be >4%.
creased renin levels lead to decreased aldos­
Answer D is incorrect. A urine sodium level
terone, which would result in increased potas-
of <10 mmol/L would be expected in prerenal
sium reuptake, not potassium wasting.
failure, where the kidney is working to reab-
Answer D is incorrect. This is the opposite sorb as much sodium as possible to increase
response to the correct answer. High sodium plasma volume and thereby improve perfu-
intake leads to volume expansion, dilation sion. In postrenal failure, the kidney is unable
of afferent arterioles, and increased stretch in to effectively reabsorb sodium, and therefore
baroreceptors, which leads to decreased renin the urine sodium level would commonly be
release. Constriction of the afferent arteriole >40 mmol/L.
would cause a decrease in the GFR.
34. The correct answer is A. The glomerular base-
33. The correct answer is E. This clinical presen- ment membrane is composed of endothelial
tation is classic for benign prostatic hyperplasia fenestrae with filtration slits lined with anionic
obstructing the urethra and causing bilateral glycoproteins on the lamina rara interna and
hydronephrosis (seen on ultrasound as dilation externa. The small diameter of the filtration
of the collecting ducts), thereby leading to re- slits partially blocks albumin filtration physi-
nal failure. Renal failure can be divided into cally, but the anionic charge of the barrier pro-
Chapter 15: Renal  •  Answers 407

vides the largest obstacle to filtration by elec- Answer A is incorrect. Salicylate overdose
trostatically repelling the negatively charged causes an anion gap acidosis (ingested salicylic
albumin molecules. acid is the unmeasured anion).
Answer B is incorrect. This choice is incorrect Answer B is incorrect. Diabetic ketoacidosis
because the filtration slits are lined with nega- (DKA) causes a severe anion gap acidosis (the
tively charged anionic glycoproteins and are unmeasured anions in this case are ketoacids).
not positively charged. Positive charges would A helpful way to remember the causes of anion
attract albumin and conglomerate, thereby im- gap acidosis is the mnemonic MUDPILES:
peding further filtration. Methanol, Uremia, Diabetic ketoacidosis, Par-
aldehyde or Phenformin, Iron tablets or Isoni-
Answer C is incorrect. Albumin is neither
azid, Lactic acidosis, Ethylene glycol, and Sal­
freely filtered by the glomerulus nor reab-
icylates. In this case the patient has no anion

High-Yield Systems
sorbed along the nephron.
gap so DKA would not be correct.
Answer D is incorrect. Basement membrane
Answer D is incorrect. Underperfusion causes
proteoglycans are negatively charged, as is al-
anion gap acidosis (the anion in this case is lac-
bumin.
tic acid). This patient has a non-anion gap aci-
Answer E is incorrect. The size selectivity of dosis.
the endothelial filtration slits provides an ob-
Answer E is incorrect. Uremia indicates renal
stacle to albumin filtration, but size selectivity
failure. The inability of the kidney to excrete
alone does not account for the complete ab-
organic acids leads to an anion gap acidosis.
sence of albumin filtration in non-pathologic
Renal failure also causes hyperkalemia, be-
states.
cause the kidney is unable to excrete potas-
35. The correct answer is C. To understand this sium.
metabolic abnormality, first look at the pH
36. The correct answer is B. Inulin is freely fil-
and then the bicarbonate (HCO3-) and partial
tered across the glomerular capillary wall
pressure of carbon dioxide (PCO2). Her pH is
and is neither reabsorbed nor secreted. It is
7.28, which indicates a form of acidosis. Meta-
therefore used to calculate the GFR, other-
bolic acidosis is the presence of low pH with
wise known as clearance of inulin. Creatinine
low plasma HCO3-, in this case 15 mEq/L

Renal
clearance can also be used as a physiologic
(normal = 22-26 mEq/L); she is suffering from
approximation of GFR. GFR is calculated as:
metabolic acidosis. Her lungs are blowing off
urinary concentration of inulin × urinary flow
more carbon dioxide in order to raise the pH.
rate/plasma concentration of inulin. Using
The causes of metabolic acidosis are events
this equation, urine flow rate = GFR × plasma
that either increase acid levels (eg, diabetic
concentration of inulin/urinary inulin concen-
ketoacidosis, uremia, hypovolemic shock) or
tration = 120 mL/min × 1.5 mg/mL / 50 mg/
decrease the amount of base present (eg, diar-
mL = 3.6 mL/min. Glomerular capillary hy-
rhea, kidney failure). Metabolic acidosis can
drostatic pressure, listed in the table of labora-
be subdivided further into non-anion gap and
tory values, is a distracter that is not used in the
anion gap metabolic acidosis. A normal anion
equation.
gap is 8-12. This patient’s anion gap is 10, so
there is no anion gap acidosis. If the primary Answer A is incorrect. This answer underesti-
cause of acidosis is a loss of HCO3-, there will mates the patient’s urine flow rate, which is 3.6
be an increase in chloride and the anion gap mL/min.
will be normal, as is the case with severe diar-
Answer C is incorrect. This answer overesti-
rhea. Of the answers listed, only diarrhea can
mates the patient’s urine flow rate, which is 3.6
cause a non-anion gap acidosis.
mL/min.
408 Section II: Organ Systems  •  Answers

Answer D is incorrect. This answer translates SLE eventually develop nephritis. Circulat-
into a value of 0.6 mL/min, which underesti- ing immune complexes in the serum deposit
mates the patient’s urine flow rate, which is 3.6 in the glomerulus and activate complement,
mL/min. leading to leukocyte infiltration and mem-
brane damage. Signs and symptoms can in-
Answer E is incorrect. This answer translates
clude proteinuria, hematuria, hypertension,
into a value of 0.3 mL/min, which underesti-
and RBC casts in the urine sediment. Biopsy
mates the patient’s urine flow rate, which is 3.6
will show a characteristic “wire-loop” appear-
mL/min.
ance along the glomerular membrane.
37. The correct answer is B. Acetazolamide can Answer A is incorrect. Alport syndrome is an
be taken prophylactically or to alleviate symp- inherited connective tissue disorder that is
toms of acute mountain sickness. It acts by diagnosed by the findings of hematuria, senso-
High-Yield Systems

inhibiting carbonic anhydrase, which is impor- rineural deafness, and anterior bulging of the
tant for bicarbonate reabsorption in the proxi- ocular lenses. Biopsy and visualization with
mal tubule of the kidney. This leads to alkalin- electron microscopy would show a thickened
ization of the urine and a metabolic acidosis. basement membrane, with a split and distorted
The drop in the plasma pH results in an in- lamina densa.
creased breathing drive and higher oxygen lev-
Answer B is incorrect. Membranous glomeru-
els in the body, helping to reverse the effects of
lonephritis is a nephrotic syndrome that pre­
hypoxemia. Acetazolamide should be avoided
sents with heavy proteinuria, but not typically
in patients with an allergy to sulfa drugs.
obvious hematuria. Other symptoms include
Answer A is incorrect. Acetazolamide does not edema, hyperlipidemia, and hypertension. On
cause a mixed metabolic acidosis and respira- biopsy light microscopy shows a uniform thick-
tory alkalosis. This combination can be seen ening of the basement membrane and electron
with aspirin toxicity, but respiratory alkalosis is microscopy shows subepithelial deposits.
not an effect of acetazolamide.
Answer C is incorrect. Minimal change dis-
Answer C is incorrect. Acetazolamide does ease is a nephropathy that occurs most often
not cause metabolic alkalosis. Loop diuretics in the pediatric population, but it can also
and thiazides both cause metabolic alkalosis. present in adults. The clinical presentation in-
Renal

Furosemide is the opposite of acetazolamide cludes proteinuria, marked edema (especially


in that it lowers urine pH and raises blood pH. periorbitally), hyperlipidemia, and hypoalbu-
Answer D is incorrect. Acetazolamide does minemia. Urinary sediment is usually acel-
not cause respiratory acidosis. Respiratory aci- lular. On biopsy light microscopy reveals no
dosis results from impaired respiration and can obvious abnormality, but electron microscopy
be seen with COPD, sedatives/opiates, pneu- shows effacement of the foot processes along
mothorax, acute respiratory distress syndrome, the epithelium.
etc. Answer D is incorrect. Poststreptococcal glo-
Answer E is incorrect. Respiratory alkalosis re- merulonephritis is typically seen a few weeks
sults from respiratory carbon dioxide output in after a streptococcal infection causing phar-
excess of normal for the metabolic production yngitis or impetigo, and would present with
of carbon dioxide by tissue. This can be seen similar symptoms of hematuria, RBC casts,
with hyperventilation in a patient with anxi- hypertension, and headaches. However, his-
ety or pain, salicylate toxicity, or CNS lesions, tologic section of a renal biopsy would show
among other factors. hypercellularity of mesangial cells as well as
subepithelial deposits of immunoglobulins and
38. The correct answer is E. This patient has ne- complement, giving a “humpy” appearance,
phritis due to SLE. Up to 60% of adults with rather than the wire-loop appearance seen in
this patient’s biopsy specimen.
Chapter 15: Renal  •  Answers 409

39. The correct answer is A. Autosomal domi- into the bloodstream. Myoglobin is nephro-
nant polycystic kidney disease (ADPKD) is as- toxic and causes acute tubular necrosis. Toxins
sociated with berry aneurysms in the circle of such as chemotherapeutic agents and amino-
Willis and with cyst formation not only in the glycoside antibiotics also may cause this condi-
kidneys but in the liver, pancreas, and spleen. tion.
Hypertension is a common early finding, with
Answer A is incorrect. An ascending urinary
an average age at onset of 30 years. Later in
tract infection, most commonly with a gram-
the course, kidneys can reach sizes triple their
negative rod such as Escherichia coli, causes
normal volumes, often resulting in disabling
pyelonephritis. Pyelonephritis generally af-
symptoms due to pressure on intra-abdominal
fects the renal cortex, with relative sparing of
organs. ADPKD does not present at a young
the vessels and glomeruli. Patients present with
age. Autosomal recessive polycystic kidney
fever, costovertebral angle tenderness, nausea,

High-Yield Systems
(ARPKD) disease is diagnosed most often in
and vomiting. On urinalysis, these patients of-
pediatric patients; it is associated less often
ten present with WBC casts, which differ from
with cyst formation in multiple visceral organs
the muddy brown epithelial casts seen in acute
and with the formation of berry aneurysms.
tubular necrosis.
Answer B is incorrect. Horseshoe kidney is as-
Answer C is incorrect. When causing kidney
sociated with many renal diseases, but it does
injury, diabetes generally manifests as renal
not involve the extra-renal findings described
papillary necrosis. This disease manifests as
in the question.
gross hematuria and proteinuria caused by
Answer C is incorrect. Children with inborn sloughing of the renal papillae. Diabetes is not
errors of metabolism can present with oph- associated with muddy brown casts on urinaly-
thalmologic findings such as cherry-red spots sis. The hallmark of diabetic nephropathy is
in the macula. An example is Niemann-Pick the presence of acellular, eosinophilic depos-
disease, a lysosomal storage disease in which its, known as Kimmelstiel-Wilson nodules, on
a sphingomyelinase deficiency results in ac- renal biopsy.
cumulation of sphingomyelin. The disease is
Answer D is incorrect. NSAID toxicity leads
characterized by varying degrees of splenomeg-
to acute interstitial nephritis, not acute tubu-
aly and neurologic deficits.
lar necrosis. This syndrome is associated with

Renal
Answer D is incorrect. Potter syndrome is a pyuria (typically eosinophils) and azotemia oc-
condition involving renal agenesis, and it is not curring one-two weeks after administration of
associated with any cystic changes. the medication. It is also associated with fever,
rash, hematuria, and costovertebral angle ten-
Answer E is incorrect. Pott disease is a TB in-
derness. It is not typically associated with gran-
fection of the vertebrae.
ular casts on urinalysis.
40. The correct answer is B. This man’s urinaly- Answer E is incorrect. Septic shock results in
sis indicates he is suffering from acute tubular diffuse cortical necrosis of the kidneys, possi-
necrosis secondary to ischemia of the epithe- bly as a result of a combination of vasospasm
lial cells of the renal tubules. Granular casts on and DIC caused by the release of the bacterial
urinalysis are pathognomonic for acute tubu- endotoxin into the bloodstream. Patients with
lar necrosis. Due to their high metabolic rate, this syndrome generally present with abrupt
these particular renal cells are particularly sen- onset of oliguria or anuria accompanied by
sitive to a drop in blood pressure such as that hematuria, flank pain, and hypotension. Dif-
experienced in hemorrhagic shock. Another fuse cortical necrosis of the kidneys is gener-
common cause of acute tubular necrosis is a ally a much more severe disease than acute
crush injury, in which the patient undergoes tubular necrosis, and can be diagnosed by ul-
rhabdomyolysis, or muscle death. When the trasonography or CT scanning, which reveals
muscle breaks down, myoglobin is released hypoechoic or hypodense areas in the renal
410 Section II: Organ Systems  •  Answers

cortex. Renal cortical necrosis has also been as- underestimate, not overestimate, the inulin-
sociated with obstetric catastrophe, such as pla- calculated GFR.
cental abruption.
Answer E is incorrect. Inulin is freely filtered
and neither reabsorbed nor secreted in the
41. The correct answer is E. Renal stones typi-
nephron. It is these characteristics that allow
cally obstruct constricted areas of the urinary
its use as an accurate measure of GFR.
tract.
Answer A is incorrect. Stones may develop in 43. The correct answer is B. Long-time diabetes
the bladder, but these do not typically cause in this patient most likely caused a neurogenic
obstruction. bladder. Overfilling of the bladder could have
caused vesicoureteral reflux to the kidneys.
Answer B is incorrect. While passing stones
Renal damage results when a bacterial urinary
through the penile urethra can be painful, it
High-Yield Systems

tract infection is superimposed on the reflux.


does not typically cause an obstruction.
Chronic pyelonephritis results in a kidney that
Answer C is incorrect. The prostatic urethra grossly shows blunting and thickened dilation
is not a typical location for renal stones to ob- of the calyces and uneven scarring. The con-
struct flow. dition may happen unilaterally if due to some
Answer D is incorrect. While staghorn calculi congenital anatomic abnormality. In the image
may develop in the renal pelvis or calyces, they above, there is a staghorn calculus filling caly-
do not typically produce obstructive symptoms ces of kidney with chronic pyelonephritis; the
until they have grown to a considerable size. cortex and medulla is atrophic and the calyces
are dilated. Staghorn calculi (struvite stones)
42. The correct answer is D. Inulin clearance can be seen in association with chronic infec-
is an accurate estimate of GFR because it is tion and are classically associated with urease
freely filtered and neither reabsorbed nor se- producing organisms such as proteus species.
creted in the nephron. Because it overesti- The classic diagnostic clue for a struvite stone
mates true GFR, compound X must therefore is alkaline urine.
undergo net secretion in the nephron. That Answer A is incorrect. While diabetic patients
is, more compound X is excreted in the urine are susceptible to acute pyelonephritis from in-
than is filtered at the glomerulus. Indeed, it is creased instrumentation and neurogenic blad-
Renal

precisely this mechanism that is responsible for der, the kidney shown in the image is not char-
the characteristic overestimation of GFR by acteristic of acute pyelonephritis. In the acute
measuring creatinine clearance. condition, the kidney surface is frequently
Answer A is incorrect. If compound X were studded with microabscesses, indicating a re-
not freely filtered at the glomerulus, its clear- cent infection.
ance would underestimate GFR. Answer C is incorrect. There is nothing in
Answer B is incorrect. The descending limb this patient’s history to indicate acute tubu-
of the loop of Henle is poorly permeable to sol- lointerstitial nephritis. Acute tubulointerstitial
ute and instead allows passive efflux of water nephritis is often caused by hypersensitivity
from the filtrate. Furthermore, compound X to a drug (eg, methicillin, rifampin, thiazide
cannot undergo net reabsorption in the neph- diuretics, and NSAIDs). It is characterized by
ron. fever, rash, eosinophilia, and renal anomalies
(hematuria, increased serum creatinine, and
Answer C is incorrect. The proximal convo- oliguria).
luted tubule is responsible for most ultrafiltrate
reabsorption. However, if compound X un- Answer D is incorrect. While hypertension
derwent net reabsorption, its clearance would may be a sign of the chronic renal failure be-
ing caused by chronic pyelonephritis, hyper-
Chapter 15: Renal  •  Answers 411

tension alone would not cause the pathology 45. The correct answer is E. This patient has ex-
shown in the image. perienced cardiogenic shock, as evidenced by
his hypotension (systolic blood pressure <90
Answer E is incorrect. Minimal change dis-
mm Hg) after a myocardial infarction. This has
ease is a glomerular disease most frequently
led to decreased renal perfusion, which has, in
seen in pediatric patients. It is a cause of the
turn, led to prerenal failure. Prerenal failure
nephrotic syndrome in a healthy patient. Most
is defined by a BUN:Cr ratio of >20:1. When
patients respond to steroid therapy although a
the glomerular filtration rate drops, there is an
small fraction are at risk for chronic renal fail-
increase in sodium and water reabsorption in
ure years later.
the proximal tubule. This leads to an increase
44. The correct answer is B. Calcium is reab- in tubular urea concentration, which favors
sorbed in three areas along the nephron: the increased reabsorption of urea. This will raise

High-Yield Systems
proximal tubule, the thick ascending limb of the BUN (remember, this is urea in the blood)
the loop of Henle, and the distal tubule. In and, therefore, the BUN:Cr ratio will rise.
the proximal tubule, calcium reabsorption is Answer A is incorrect. Blockage of the ureters
coupled to sodium reabsorption. In the distal is a postrenal cause of renal failure. It would
tubule, calcium reabsorption is controlled by present with pain on urination and is unlikely
PTH. In the thick ascending limb, however, in the setting of cardiogenic shock.
calcium reabsorption is dependent on the
Answer B is incorrect. This patient’s BUN:Cr
function of the Na+-K+-2Cl- co-transporter.
ratio of >20:1 indicates that this is a prerenal
Calcium reabsorption occurs paracellularly,
process, not an intrarenal one. An acute tubu-
driven by the electrochemical gradient created
lar necrosis would present with renal failure in
by this transporter. The cell’s ATPase surface
the setting of a BUN:Cr ratio of 10-15:1 with
creates an electro-negative environment inside
muddy brown granular and epithelial cell
the cell by transferring three cations out (Na+)
casts, as well as free renal tubular epithelial
for every two cations in (K+). Because loop di-
cells in the urine.
uretics (such as furosemide) inhibit this trans-
porter, discontinuation of the patient’s loop Answer C is incorrect. Although interstitial
diuretic would increase functionality of this nephritis can occur from the use of NSAIDs,
channel and thus increase calcium resorption it usually occurs only after sustained chronic

Renal
in this segment. use. Signs and symptoms include rash, fever,
eosinophilia, eosinophiluria, and elevated IgE
Answer A is incorrect. PTH controls calcium
levels.
reabsorption in the distal tubule.
Answer D is incorrect. Urine output is con-
Answer C is incorrect. Thiazide diuretics in-
trolled mainly by two factors: the hydration
hibit the sodium chloride symporter in the dis-
state of the body and the level of kidney func-
tal convoluted tubules.
tion. Therefore, low urine output is seen only
Answer D is incorrect. Calcium delivery plays in the setting of dehydration or kidney dys-
a role in driving calcium reabsorption, but function. Stress by itself will not cause low
the transluminal gradient established by the urine output unless it is coupled with dehydra-
sodium-potassium-chloride co-transporter is tion or an acute renal disease process.
more important. Moreover, calcium’s effect
would not be specific for the ascending limb of 46. The correct answer is D. Acute poststrepto-
the loop of Henle. coccal glomerulonephritis is an autoimmune
disease most frequently seen in children. Un-
Answer E is incorrect. The triple transporter
der light microscopy, the glomeruli appear en-
maintains a lumen positive charge; positive
larged and hypercellular, with neutrophils and
ions are reabsorbed to maintain electroneutral-
subepithelial immune complex depositions
ity.
described as “lumpy-bumpy.” Under electron
412 Section II: Organ Systems  •  Answers

microscopy, the large irregular deposits are ob- implicate a hepatorenal (prerenal) cause of re-
served in the subepithelium of the glomerulus. nal failure.
This condition normally presents a few weeks
Answer C is incorrect. Horseshoe kidney is
after a streptococcal infection with peripheral
a pediatric abnormality that is generally as-
and periorbital edema, dark urine, and protein-
ymptomatic if not associated with other ab-
uria. These symptoms are caused by circulat-
normalities. Horseshoe kidneys are caused by
ing antistreptococcal antibody-antigen com-
the embryonic blastemas (embryonic kidneys)
plexes that deposit in the glomerular basement
partially fusing at their inferior poles and be-
membrane, leading to complement activation
coming trapped against the inferior mesenteric
and glomerular damage. The classic findings
artery during their migration. In children it is
are RBCs and casts in the urine (which cause
sometimes associated with Wilms tumor.
the characteristic tea-colored urine), a posi-
High-Yield Systems

tive ASO titer, and decreased levels of comple- Answer D is incorrect. A flea-bitten appear-
ment. ance in kidneys is usually seen in patients with
malignant hypertension.
Answer A is incorrect. Granular subendothe-
lial deposits are usually seen in SLE. 48. The correct answer is D. Goodpasture syn-
Answer B is incorrect. Linear subendothelial drome is a rare autoimmune disorder in which
patterns are seen in vasculitides such as Good- antibodies directed against glomerular base-
pasture syndrome. ment membrane (GBM) antigen cause a
rapidly progressive glomerulonephritis. The
Answer C is incorrect. Mesangial deposits are
antibodies can also deposit in the alveolar
usually seen in IgA nephropathy.
basement membranes, resulting in hemoptysis.
It is most often seen in young men. This con-
47. The correct answer is E. The patient de-
dition can be diagnosed by the detection of the
scribed in this vignette has hepatorenal syn-
anti-GBM antibodies in the blood, in combi-
drome, which is a progressive functional renal
nation with a kidney biopsy that demonstrates
failure caused by a reduction in the glomeru-
a linear staining pattern on immunofluores-
lar filtration rate due to declining hepatic func-
cence. Because this condition may rapidly lead
tion. It is characterized by splanchnic vasodila-
to a compromised airway or declining kidney
tion and concomitant vasoconstriction in the
function, immediate airway protection, plas-
Renal

renal vascular beds due to the production of


mapheresis (to remove the autoantibodies from
mediators and to the activation of the renin-
circulation), and treatment with corticosteroids
angiotensin system. The combination of these
and cyclophosphamide is indicated.
two factors causes a prerenal type azotemia
that develops, most commonly, without severe Answer A is incorrect. Poststreptococcal glo-
oliguria. One of the features of hepatorenal merulonephritis typically affects 2-14 year
syndrome is that kidney anatomy is completely olds who have recently suffered from either
unaffected, and thus visualization of the kid- impetigo or pharyngitis caused by particular
neys by most modalities would reveal normal M types of streptococci (nephritogenic strains).
size and shape. Poststreptococcal glomerulonephritis is an
immune-mediated disease that develops two-
Answer A is incorrect. Enlarged cystic kidneys
six weeks after skin infection and one-three
can be a feature of polycystic kidney disease,
weeks after streptococcal pharyngitis. The clas-
but usually do not present with prerenal azo-
sic presentation is an acute nephritic picture
temia.
with hematuria, pyuria, RBC casts, edema, hy-
Answer B is incorrect. Enlarged kidneys with pertension, and oliguric renal failure. This pa-
hydronephrosis can be caused by obstruction tient’s age and gender in combination with his
from stones in the renal calyx or ureters. How- symptoms of lung hemorrhage and hematuria
ever, the other clinical findings in this patient
Chapter 15: Renal  •  Answers 413

make Goodpasture syndrome the most likely 49. The correct answer is C. The patient most
diagnosis. likely has hypertension as a result of the uni-
lateral stenosis of a renal artery. Renal artery
Answer B is incorrect. Alport disease is an in-
stenosis is usually caused by atherosclerosis or
herited disorder caused by a mutation in col-
fibromuscular dysplasia, and the affected kid-
lagen type IV, which is an important structural
ney usually becomes atrophic. The decreased
component in the basement membranes of
perfusion of the kidney causes juxtaglomerular
the eyes, ears, and kidneys. Patients with this
cells to release renin, which cleaves angioten-
syndrome usually present in childhood with a
sinogen into angiotensin I. ACE in alveolar
combination of ocular defects, deafness, and
capillaries converts angiotensin I to angioten-
hematuria. Given this patient’s age and clini-
sin II, which stimulates the secretion of aldos­
cal presentation, Goodpasture syndrome is a
terone and which is itself a vasoconstrictor.
more likely diagnosis than Alport syndrome.

High-Yield Systems
Increased levels of aldosterone lead to sodium
Answer C is incorrect. Berger disease, other- and water retention. All of these things com-
wise known as IgA nephropathy, is the most bined lead to hypertension.
common cause of glomerulonephritis world-
Answer A is incorrect. Renal artery stenosis
wide. It is caused by the deposition of IgA an-
would lead to an increased level of angioten-
tibodies in the mesangium of the kidney and
sin II, which in turn would cause increased re-
is often triggered by a recent upper respiratory
lease of ADH from the posterior pituitary.
infection. Classically patients with Berger dis-
ease present with new onset frank hematuria, Answer B is incorrect. Renal artery stenosis
usually within days of an upper respiratory would lead to increased levels of angiotensin
tract infection. Although the hematuria may II.
persist for several days, renal function usually
Answer D is incorrect. Cushing syndrome,
remains normal, and the long-term prognosis
which can be of pituitary or adrenal origin, is
is relatively favorable. Given the patient’s lung
marked by the excessive production of corti-
involvement, Goodpasture syndrome is a more
sol, which can cause hypertension. The thrill
likely diagnosis than Berger disease.
heard in the patient, however, is indicative of
Answer E is incorrect. In the pediatric popu- renal artery stenosis.
lation, nephrotic syndrome (characterized by
Answer E is incorrect. Renal artery stenosis

Renal
proteinuria, hypoalbuminemia, hyperlipid-
would not have an effect on the levels of ACE
emia, and edema) is most commonly caused
in the lungs; however, more angiotensin I
by minimal change disease (MCD). MCD is
would be available and therefore there would
definitively diagnosed on renal biopsy, which
also be an increase in angiotensin II.
reveals effacement of the foot process support-
ing the epithelial podocytes with weakening of 50. The correct answer is B. The only diuretics
slit-pore membranes on electron microscopy, acting along the thick ascending limb of the
but normal findings on light and immunofluo- nephron are loop agents (eg, furosemide, torse-
rescent microscopy. The pathophysiology of mide, ethacrynic acid). These drugs work by
this lesion is uncertain, although most agree inhibiting the sodium-potassium-chloride co-
there is a circulating cytokine that alters capil- transporter. It is worth noting that ethacrynic
lary charge and podocyte integrity. Treatment acid is unique because it is not a sulfonamide
with steroids is indicated based on clinical sus- and therefore can be used in individuals with
picion of MCD; renal biopsy is not necessary an allergy to sulfa drugs.
before treatment is initiated. Given this pa-
tient’s age and clinical presentation, Goodpas- Answer A is incorrect. Acetazolamide, which
ture syndrome is a more likely diagnosis than acts in the proximal convoluted tubule, blocks
MCD. resorption of sodium bicarbonate.
414 Section II: Organ Systems  •  Answers

Answer C is incorrect. Hydrochlorothiazide Answer E is incorrect. Triamterene, a potas-


inhibits sodium chloride resorption in the ­sium-sparing diuretic, blocks sodium channels
early distal tubule. in the cortical collecting tubule.
Answer D is incorrect. Osmotic agents act in
three places along the nephron: the proximal
convoluted tubule, the thin descending limb,
and the collecting tubule. They do not, how-
ever, act on any of the ion pumps.
High-Yield Systems
Renal
Chapter 16

Reproductive

415
416 Section II: Organ Systems  •  Questions

Q u e st i o n s

1. A 53-year-old woman calls her primary care (A) One-two glasses of wine per week
physician because she wants to begin hormone (B) Age of 42 years
replacement therapy (HRT). She has read that (C) Family history of breast cancer
estrogen replacement therapy is supposed to (D) Never having had a mammogram
ease the transition to a postmenopausal state, (E) Occasional irregular periods
and that HRT can be cardioprotective, de-
crease cancer risk, and promote healthy bones. 3. A 29-year-old woman presents to the gynecolo-
Which of the following is an indication for this gist complaining of a “fishy”-smelling vaginal
woman to start HRT? odor noticeable during intercourse. Pelvic ex-
amination reveals a homogenous gray-white
High-Yield Systems

(A) To decrease risk of breast cancer


discharge, and a saline wet mount of vaginal
(B) To decrease risk of deep venous thrombo-
epithelial cells is obtained (see image). What
sis and stroke
is the morphology of the organism that is caus-
(C) To decrease risk of myocardial infarction
ing this patient’s symptoms?
(D) To decrease risk of osteoporosis
(E) To treat vasomotor symptoms

2. A 42-year-old woman presents to her gynecolo-


gist for a check-up. She states that this is her
first gynecologic appointment in five years.
As such, her physician conducts a full history
and physical examination. Her medical his-
tory is significant for hyperlipidemia treated
with simvastatin, and her only surgery was an
appendectomy at age 21. Her family history is
significant for an aunt who had breast cancer
diagnosed at age 36, and a cousin with breast
Reproductive

cancer diagnosed at age 29. On social history,


she reports being happily married, never hav-
ing had any children, drinking one-two glasses Courtesy of Dr. M. Rein, Centers for Disease Control and
of wine every evening, and smoking about Prevention.
five cigarettes a week. She reports occasional
irregular periods, and has never had a mam-
(A) Budding yeast and/or hyphae
mogram. On physical examination, the physi-
(B) Gram-negative diplococci
cian palpates a discrete, 1.5-×-2-cm mass in the
(C) Gram-positive rod
upper outer quadrant of the left breast. There
(D) Obligate intracellular parasite
are no palpable lymph nodes in the left axilla.
(E) Pleomorphic, gram-variable rod
The patient is concerned, and asks about the
chances the mass could represent breast can- 4. A 24-year-old woman presents with severe
cer. Which of the following features of this pa- right lower quadrant pain. She has a history of
tient’s history most increases her risk of breast multiple sexual partners but has been married
cancer? and faithful to her husband for the last three
years. A urine b-human chorionic gonadotro-
Chapter 16: Reproductive  •  Questions 417

pin test is positive, but no gestational sac is vi-


sualized on transvaginal ultrasound imaging of
the uterus. During the examination, her pain
increases and she is becoming tachycardic.
Which of the following best characterizes the
organism most likely predisposed this woman
to her current clinical picture?
(A) It is a gram-negative bacillus
(B) It is a gram-variable bacillus
(C) It is a spirochete
(D) It is a teardrop-shaped trophozoite
(E) It is an obligate intracellular organism

High-Yield Systems
5. A 29-year-old woman in her third trimester of Reproduced, with permission, from USMLERx.com.
pregnancy is brought by her husband to the
emergency department with contractions. The
husband informs the nurse that “her physi- (A) Enlargement during pregnancy
cian said something about being at risk for (B) Formation of chocolate cysts
seizures.” Her blood pressure is 195/123 mm (C) Malignant transformation
Hg and she is afebrile. Laboratory tests show a (D) Pelvic inflammatory disease
platelet count of 5000/mm³, a lactate dehydro- (E) Postmenopausal metastasis
genase level of 699 U/L, aspartate aminotrans-
ferase of 89 U/L, and alanine aminotransferase 7. An endocrine research laboratory is investi-
of 90 U/L. Urinalysis reveals proteinuria. The gating the regulation of the hypothalamic-
patient is given a medication and taken to the pituitary axis through pharmacologic means
labor and delivery room. Which of the follow- in the brains of chimpanzees. They find that
ing is associated with the drug this patient most administering extremely high doses of chlor-
likely received? promazine causes parkinsonism, amenorrhea,
lack of positive reward trainability, and de-

Reproductive
(A) Alteration of sleep cycles creased cognitive ability test scores in female
(B) Decreased tendon reflexes chimpanzees. These findings allow them to
(C) Epidermal erythema and sloughing identify specific neural tracts in the brain that
(D) Extensor plantar reflex are interrupted by chlorpromazine. Interrup-
(E) Tetany of facial nerve when tapped tion of what neural tract is responsible for the
amenorrhea observed in the studied chimpan-
6. A 29-year-old African-American woman with
zees?
menometrorrhagia undergoes ultrasound of
the pelvis, which reveals uterine masses. Bi- (A) Mesocortical tract
opsy shows a whorled pattern of smooth mus- (B) Mesolimbic tract
cle. A gross specimen from a patient with simi- (C) Nigrostriatal tract
lar pathology is shown below. These masses are (D) Tuberoinfundibular tract
most commonly associated with which of the
following conditions?
418 Section II: Organ Systems  •  Questions

8. A 24-year-old man presents for his annual (C) Luteinizing hormone


physical and is noted to have a nontender right (D) Progesterone
testicular nodule. After an initial ultrasound, (E) Prolactin
he is sent for CT imaging of his abdomen
and pelvis, which shows enlarged para-aortic 11. A 57-year-old woman is scheduled for elective
lymph nodes that are greater on the right side. hysterectomy. Severing which of the following
What testicular pathology does this patient structures during surgery would most severely
most likely have? disrupt blood flow to the ipsilateral ovary?
(A) Epididymitis (A) Cardinal ligament
(B) Leydig cell tumor (B) Fallopian tube
(C) Seminoma (C) Round ligament
(D) Testicular torsion (D) Suspensory ligament
High-Yield Systems

(E) Ureter
9. An 8-year-old boy presents to the clinic with
a complaint of a runny nose and difficulty 12. A 68-year-old retired furniture mover comes to
breathing. His mother says the boy has had the doctor for the first time in years because of
recurrent respiratory infections, often with constant backaches that radiate down his legs.
a productive cough. The child’s symptoms He also reports fatigue, a 9.1-kg (20-lb) weight
are tolerable in the morning, but seem to be- loss over the past few months, and trouble uri-
come progressively worse as the day continues. nating, including decreased force of stream,
Chest auscultation reveals crackles and wheez- hesitancy, and intermittency. He smokes two
ing, but the physician is unable to auscultate packs of cigarettes per day and drinks at least
a normal S1 and S2 heart sound. Radiographic three beers per day, and has done so for the
examination reveals pulmonary hyperinfla- past 15 years. Laboratory tests show:
tion, bronchiectasis, and a complete left/right Hemoglobin: 10.1 g/dL
reversal of his circulatory system. What other Hematocrit: 28%
complication(s) is this patient likely to have? WBC count: 8500/mm³
(A) Flushing and diarrhea Platelet count: 240,000/mm³
Reproductive

(B) Infertility Na+: 135 mEq/L


(C) Panacinar emphysema K+: 4.2 mEq/L
(D) Pulsus paradoxus HCO3-: 24 mEq/L
(E) Simple partial seizures Cl-: 101 mEq/L
Blood urea nitrogen: 18 mg/dL
10. A young couple is trying to conceive their first Creatinine: 1.3 mg/dL
child. The woman speaks to her mother, who
A radionucleotide bone scan was also per-
tells her that she should take her temperature
formed; results are shown in the image. What
to determine when she ovulates. The action
is the most important risk factor for this pa-
of which of the following hormones is respon-
tient’s most likely current condition?
sible for this change in body temperature?
(A) Estrogen
(B) Human chorionic gonadotropin
Chapter 16: Reproductive  •  Questions 419

14. Following a normal pregnancy and labor, a


baby is born to a young couple. The baby cries
immediately and has Apgar scores of 9 at both
one and five minutes. While examining the
baby, the pediatrician present at delivery notes
that the baby has ambiguous genitalia and la-
bial fusion. The baby is also hypotensive and
hypovolemic. Genotyping reveals a karyotype
of 46,XX. Which of the following is the most
likely cause of the patient’s physical findings
and symptoms?
(A) 11b-Hydroxylase deficiency

High-Yield Systems
(B) 17a-Hydroxylase deficiency
(C) 21a-Hydroxylase deficiency
(D) 5a-Reductase deficiency
Reproduced, with permission, from USMLERx.com. (E) Mutation in the androgen receptor gene

15. A 50-year-old postmenopausal G5P5 woman


(A) Advanced age sees her gynecologist for a yearly well-woman
(B) Alcohol use check-up. Her body mass index is 34 kg/m².
(C) History of physical labor Results of a bimanual examination and a
(D) Lack of regular primary care breast examination are normal. The patient
(E) Smoking history mentions that her grandmother died of endo-
metrial cancer, and claims that she will stop
13. An asymptomatic 30-year-old woman presents drinking and smoking if it will save her from
to her physician’s office for a routine health the agony her grandmother experienced. The
maintenance examination. Papanicolaou smear physician reviews the patient’s medical history
indicates high-grade dysplasia. What is the with her and discusses her risk factors for endo-
mechanism by which the responsible agent metrial cancer. Which of the following inter-

Reproductive
causes disease? ventions would have best decreased the risk for
developing endometrial cancer in this patient?
(A) Arrest of blood vessel growth
(B) Continuation of cell cycle after DNA dam- (A) Avoidance of alcohol and cigarette use
age (B) Delaying sexual activity
(C) Inhibition of DNA replication (C) Diet and exercise
(D) Inhibition of mitogenic signal transduction (D) Nulliparity
(E) Inhibition of RNA-dependent DNA po- (E) Use of a combined progesterone-estrogen
lymerization birth control product
420 Section II: Organ Systems  •  Questions

16. A 64-year-old man visits his physician for an quent nighttime awakenings because she wor-
annual examination. He complains of a recent ries about the baby. She feels like she “can’t
9.1-kg (20-lb) unintentional weight loss and handle a child right now.” She sometimes feels
pain in his back and pelvis. Otherwise, he has angry at the baby and finds herself bursting
no complaints. His examination is notable for into tears “for no apparent reason.” She used
point tenderness along his spine and pelvis. He to enjoy reading, even up to the baby’s birth,
also had firm prostate nodules palpated on dig- but finds no joy in this presently. She assures
ital rectal examination. Which of the following the physician that she is taking good care of
laboratory values would be expected in this pa- the baby, but has recently asked her mother to
tient? come stay with her for assistance. A review of
her medical record reveals that she had an un-
Prostate Alkaline complicated spontaneous vaginal delivery of a
Choice specific Calcium
High-Yield Systems

phosphatase
antigen healthy baby boy weighing 7 lb 7 oz (3.37 kg)
A at 39 3/7 weeks’ gestation. There was moderate
B
loss of blood during the delivery. Her hemo-
globin level following the delivery was 12.0 g/
C
dL. What is the most likely diagnosis?
D
(A) Iron deficiency anemia
E
(B) Postpartum blues
Reproduced, with permission, from USMLERx.com.
(C) Postpartum major depression
(D) Postpartum psychosis
(E) Sheehan syndrome
(A) A
(B) B 19. A 28-year-old woman presents to her physician
(C) C with concerns that she is unable to produce
(D) D breast milk, despite having given birth approxi-
(E) E mately one month ago. On further question-
ing she indicates she has been exceptionally
17. A 63-year-old man with a history of a myocar- thirsty lately, and describes feelings of fatigue
Reproductive

dial infarction, chronic stable angina, hyper- and cold intolerance. Physical examination re-
tension, and diabetes presents to his physician veals no abnormalities except a scarcity of ax-
with a complaint of erectile dysfunction. He illary hair. Laboratory tests reveal a serum so-
asks about the use of a medication he saw on a dium level of 150 mEq/L and urinalysis reveals
TV commercial. He is currently taking meto- a urine osmolality of 220 mOsmol/kg. Which
prolol, nitroglycerin, lisinopril, and metformin. of the following most likely increased the pa-
His physician informs him that it is unsafe for tient’s risk of developing this condition?
him to take medication for erectile dysfunc- (A) Abnormalities of the placenta
tion. What is the mechanism of the medica- (B) Alcohol intake during pregnancy
tion that is contraindicated in this situation? (C) Endometriosis
(A) Blocks sodium channel (D) Gestational diabetes
(B) Decreases the level of cGMP (E) Incorrect use of tampons
(C) Decreases the level of nitric oxide (F) Multiple sexual partners
(D) Increases the level of cGMP (G) Pelvic inflammatory disease
(E) Increases the level of nitric oxide
20. A 46,XY infant is born with a nonsense muta-
18. A patient presents two weeks after giving birth tion in the SRY gene. The product of the SRY
to her first child. For the past two weeks she gene is most directly responsible for which of
admits to feeling “hopeless” and “helpless.” the following normal processes in the sexual
She has not slept well and attributes this to fre- development of male fetuses?
Chapter 16: Reproductive  •  Questions 421

(A) Development of internal male genitalia (C) Intracellular elementary body


from the mesonephric duct (D) Multiplication of reticulate bodies
(B) Development of testes from indifferent go- (E) Reticulate body
nads
(C) Development of the genital tubercle into 23. A 30-year-old woman finds a lump in her right
the glans penis breast during a monthly self-examination. Di-
(D) Development of the urogenital sinus into agnostic mammography reveals a 2-cm mass
the prostate gland with uneven borders and spiculated calcifica-
(E) Involution of paramesonephric ducts tions; this finding is suspicious for malignancy.
A biopsy is performed. The report identifies
21. A woman in labor continues to be dilated ductal carcinoma in situ, and a lumpectomy
2 cm after two hours in labor. She is given a is performed. The pathology report further

High-Yield Systems
synthetic version of a hormone to help dilate states that the malignant tissue is positive for
her cervix. Where is the endogenous version of human epidermal growth factor 2/neu recep-
this hormone stored within the body? tor, but negative for estrogen and progesterone
receptors. Which of the following therapies is
(A) Adrenal cortex
specifically targeted to treat this patient’s breast
(B) Anterior pituitary gland
cancer?
(C) Hypothalamus
(D) Mammary glands (A) Bevacizumab
(E) Posterior pituitary gland (B) Cyclophosphamide
(C) Doxorubicin
22. A biotechnology firm is developing a new (D) Tamoxifen
small protein drug designed to prevent the (E) Trastuzumab
spread of a sexually transmitted infection.
Scientists want to block the infectious step of 24. A 19-year-old mother of two rushes to her ob-
the bacteria’s reproductive cycle. A Giemsa- stetrician’s office at 35 weeks’ gestation be-
stained smear of a patient’s urethral discharge cause of cramping abdominal pain and mild
shows cytoplasmic inclusions similar to those vaginal bleeding. Both cramps and bleeding
shown in the image. During which stage of started about two hours ago. Pain is constant,

Reproductive
this pathogen’s life cycle is it most infectious? with intermittent episodes of severe cramping.
Bleeding has been sufficient to soak through
four to five pads within the past few hours. She
denies a history of trauma or leakage of clear
fluid. She does admit to using cocaine three
times within the past month, most recently last
night. She is afebrile, blood pressure is 155/95
mm Hg, and heart rate is 100/min. Bi-manual
examination reveals tenderness over the fun-
dus and a nondilated cervix. Ultrasound re-
veals a normally implanted placenta with no
visible abnormalities. Fetal heart rate is in the
170-180/min range. Which of the following is
the most likely diagnosis?

Courtesy of Dr. E. Arum and Dr. N. Jacobs, Centers for (A) Abruptio placentae
Disease Control and Prevention. (B) Concealed abruption
(C) Labor
(D) Placenta accreta
(A) Cytoplasmic inclusion body (E) Placenta previa
(B) Extracellular elementary body
422 Section II: Organ Systems  •  Questions

25. A 14-year-old boy is brought to the clinic by 28. A 70-year-old woman presents to her primary
his parents who are concerned because he has care physician for a check-up after undergo-
not yet begun puberty. Laboratory results indi- ing repair of a broken hip due to a minor fall.
cate hypogonadism secondary to failure of the Which of the following is the most likely hor-
hypothalamic-pituitary-gonadal axis. Which of monal profile of this woman?
the following are possible adverse effects of the
treatment for this patient’s condition? Follicle- Gonadotropin-
Choice Estrogen Luteinizing
stimulating releasing
(A) Anemia hormone
hormone hormone
(B) Decreased serum LDL cholesterol levels A
(C) Increased spermatogenesis B
(D) Leukocytosis C
(E) Premature closing of the epiphyseal plates D
High-Yield Systems

E
26. A 33-year-old G1P0 woman with no previous
prenatal care visits a gynecologist for a pre- Reproduced, with permission, from USMLERx.com.
natal triple screen. The triple screen reveals
an a-fetoprotein level 0.5 times the mean, a
(A) A
b-human chorionic gonadotropin level twice
(B) B
the mean, and an estriol level 0.75 times the
(C) C
mean. From which birth defect will the baby
(D) D
most likely suffer?
(E) E
(A) Aortic coarctation
(B) Endocardial cushion defect 29. A 23-year-old athletically built woman comes
(C) High-pitched, cat-like cry to the physician complaining of multiple red,
(D) Prominent occiput ring-like lesions on her body. A careful his-
(E) Tuft of hair at the small of the back tory and physical reveals the woman has tinea
corporis acquired while working on poorly
27. Vasectomies can be done in an outpatient set- cleaned yoga mats at a local gym. The physi-
Reproductive

ting through a small incision in the scrotum. A cian prescribes a medicine to clear her ery-
vasectomy involves bilateral excision of a seg- thematous lesions. After 15 days of treatment
ment of the ductus deferens between the exit the lesions she returns to the office. While
from the epididymis and the entrance to the the lesions are clearing, she has noticed that
pelvis. After the ductus deferens is cut, sperma- patches of her skin have become darker than
tozoa can no longer travel into the urethra and normal. Which of the following drugs did this
the sperm degenerate in the epididymis and patient most likely receive?
ductus deferens. When excising the ductus
(A) Amphotericin B
deferens segments, the surgeon takes care not
(B) Fluconazole
to injure which anatomic structure that crosses
(C) Flucytosine
directly posterior to the ductus as it courses
(D) Itraconazole
from scrotum to urethra?
(E) Ketoconazole
(A) Efferent ductules
(B) Spermatic cord 30. A 26-year-old man presents to the urologist be-
(C) Sympathetic nerve fibers cause he and his wife have failed to conceive
(D) Testicular artery for 14 months. His total and free testosterone
(E) Ureter levels are normal, and initial semen analysis
shows significantly decreased volume and no
Chapter 16: Reproductive  •  Questions 423

detectable sperm. Testicular fine-needle bi- 33. Pathologic examination of bilateral ovarian
opsy demonstrates normal sperm motility and masses reveals round, mucin-secreting cells as
normal sperm morphology. His medical his- seen in the image. Which of the following is
tory is significant for hypercholesterolemia and most likely to be seen on physical examination
a surgical bilateral inguinal hernia repair at 8 in this case?
years of age. He also admits to smoking mari-
juana four-five times/week during college, and
to drinking three-four alcoholic beverages/day
until recently. Which of the following aspects
of his work-up is the most likely cause of the
patient’s infertility?
(A) Four year history of heavy marijuana use

High-Yield Systems
(B) Decreased semen volume
(C) History of drinking three-four alcoholic
beverages/day
(D) Hypercholesterolemia
(E) Surgical inguinal hernia repair

31. After 11 days in the intensive care unit follow-


ing an automobile accident, an 18-year-old
woman develops increased urinary urgency, Reproduced, with permission, from USMLERx.com.
burning during urination, and lower abdomi-
nal pain. When her urine is cultured, a red
(A) Galactorrhea
pigment is observed in the Petri dish. What is
(B) Hematochezia
the most likely organism causing these symp-
(C) Palpable gallbladder
toms?
(D) Pearly papules on the face
(A)
Escherichia coli (E) Supraclavicular lymphadenopathy
(B)
Klebsiella pneumoniae

Reproductive
(C)
Proteus mirabilis 34. A previously healthy 25-year-old man comes to
(D)
Serratia marcescens the physician because of a tingling sensation
(E)
Staphylococcus saprophyticus on his penis for the past day. Physical exami-
nation reveals a slightly raised, erythematous
32. A 25-year-old woman comes to her physician area on the head of the penis. He has not been
complaining of pain with intercourse and dys- sexually active for over a year. When his lesion
menorrhea that worsens one-two days before is scraped with a microscope slide and Giemsa
her period starts. Ultrasonography reveals uni- dye applied, multinucleated giant cells can be
lateral adnexal masses, and laparoscopy shows seen. Which of the following mechanisms is
reddish-brown cysts on the ovaries. This pa- involved in the pathogenesis of the infective
tient is at increased risk for developing which agent?
of the following conditions?
(A) Direct invasion into the bloodstream
(A) Carcinoma (B) Exotoxin production
(B) Infertility (C) Invasion of CD4+ cells
(C) Intracranial hemorrhage (D) Migration to the peripheral nervous system
(D) Masculinization by anterograde axonal transport
(E) Obesity (E) Migration to the peripheral nervous system
by retrograde axonal transport
424 Section II: Organ Systems  •  Questions

35. A 15-year-old who has been healthy and meet- (A) Auer rods
ing normal developmental milestones presents (B) Call-Exner bodies
with a painless, homogenous testicular mass. (C) Lewy bodies
Following removal, the family is informed that (D) Mallory bodies
the mass is a common germ cell tumor that is (E) Multinucleated giant cell
highly sensitive to radiotherapy. They are told
their son has a good prognosis. Which of the 38. A 32-year-old pregnant woman presents to the
following pathologies was most likely seen? emergency department with vaginal bleeding.
She reports that her last menstrual period was
(A) A glandular/papillary morphologic pattern
24 weeks ago. She denies any pain or recent
(B) Cowdry type A nuclear inclusions
trauma. Transabdominal ultrasound confirms
(C) Large cells in lobules, with watery cyto-
the presence of a gestational sac and an intra-
plasm
High-Yield Systems

uterine fetal heartbeat. Which of the following


(D) Reinke crystals
in this patient’s past medical or surgical history
(E) Structures resembling primitive glomeruli
would have increased her risk for this compli-
cation?
36. A 32-year-old woman presents to her gynecolo-
gist with oligomenorrhea. On examination she (A) History of endometriosis
is found to have enlarged, nontender ovaries (B) History of pelvic inflammatory disease
bilaterally. The patient’s body mass index is 31 (C) Prior cesarean section delivery
kg/m². What other symptom or finding is this (D) Prior ectopic pregnancy
patient likely to have? (E) Use of assisted reproductive technologies
(A) Decreased estrogen levels
39. A woman who has had four previous spontane-
(B) Epicanthal folds
ous, first-trimester pregnancy losses is found to
(C) Hirsutism
have a uterus with the structure illustrated in
(D) Increased bleeding time
the image. What is the cause of this variation?
(E) Numerous spontaneous abortions

37. A 35-year-old man comes to the physician


Reproductive

complaining of painful genital vesicles. On


further questioning, he admits to unprotected
sex with multiple partners. To confirm the di-
agnosis, the physician performs a specific test.
Results are shown in the image. Which of the
following is the pathognomonic finding on this
patient’s test?

Reproduced, with permission, from USMLERx.com.

(A) Complete failure of fusion of the


parameson­ephric (müllerian) ducts
(B) Failure of resorption of the midline uterine
septum
(C) Maternal diethylstilbestrol exposure
(D) Partial failure of fusion of the parameso-
nephric (müllerian) ducts
Courtesy of Dr. Joe Miller, Centers for Disease Control and
Prevention.
Chapter 16: Reproductive  •  Questions 425

40. A 1-year-old infant is noted by his parents to air, and her heart rate is 110 bpm. Urinalysis
have a gradually enlarging scrotal mass. There shows no protein, leukocytes, or bacteria, with
is no history of trauma. On physical examina- few RBCs. Pelvic examination reveals dark-red
tion no scrotal skin lesions are noted, and the blood in the vaginal vault and a hypertonic
mass is not painful on palpation. However, the uterus. Fetal heart tones indicate fetal distress.
scrotum is enlarged, boggy, and soft bilaterally. Which of the following is the most appropriate
Results of a transillumination test are positive. course of action?
Where is the site of fluid accumulation in this
(A) Administer magnesium
patient?
(B) Await spontaneous vaginal delivery
(A) Body of the testis (C) Do an abdominal FAST (focused assess-
(B) Epididymis ment with sonography in trauma) to assess
(C) Inguinal lymphatics for free blood in the peritoneum

High-Yield Systems
(D) Pampiniform plexus (D) Perform cesarean section
(E) Tunica vaginalis (E) Perform urgent CT of the chest to assess
for amniotic fluid embolism
41. A 24-year-old bartender returning from Mexico
presents to the clinic because of a painful pe- 43. A normal woman is on day 19 of her menstrual
nile lesion that appeared about one week af- cycle, as measured from the first day of her
ter having unprotected sex with a new female most recent period. Which of the following
partner. On examination the ulcer is 1.5 cm accurately describes the changes in progeste­
in diameter with an erythematous base and rone and estrogen levels occurring at this time,
clearly demarcated borders. The base of the ul- as well as the feedback of these two hormones
cer is covered with a yellow purulent exudate on luteinizing hormone (LH) and follicle-
and bleeds when scraped. Gram stain of the stimulating hormone (FSH) release at this
exudates shows gram-negative rods in chains stage of the menstrual cycle?
in a school-of-fish appearance. Otherwise, ex-
amination is notable only for tender inguinal
Feedback on
lymphadenopathy. Which of the following is Choice Progesterone Estrogen LH and
the most appropriate treatment at this time?

Reproductive
FSH
A negative
(A) A 10-day course of oral acyclovir
(B) Highly active retroviral therapy until the B no change negative

lesion resolves C no change positive


(C) Nystatin powder to the lesion daily for at D negative
least one week E negative
(D) One high dose of intramuscular peni­cillin G
(E) One high dose of oral azithromycin Reproduced, with permission, from USMLERx.com.

42. A 32-year-old woman with a history of hy-


pertension and cocaine abuse presents at 32 (A) A
weeks’ gestation because of vaginal bleeding (B) B
and painful abdominal cramps. She denies (C) C
trauma. Her blood pressure is 105/60 mm (D) D
Hg, blood oxygen saturation is 97% on room (E) E
426 Section II: Organ Systems  •  Questions

44. A 65-year-old man visits his physician because (A) This condition has no malignant potential
of increasingly difficult urination. He has (B) This condition is associated with a family
trouble initiating a stream and experiences history of BRCA1
postvoid dribbling. He wakes from sleep three (C) This condition is associated with purulent
times per night to urinate. His baseline cre- nipple discharge
atinine level was 1.0 mg/dL, and it is now 1.5 (D) This condition is most common in women
mg/dL. Which treatment is most feasible to under 25 years old
immediately improve this patient’s creatinine (E) This patient likely had breast trauma
level?
47. A 24-year-old woman with a history of diabetes
(A) Administration of fluid boluses
presents to her gynecologist, because she and
(B) Dialysis
her husband have been trying to conceive for
(C) Placement of a Foley catheter
High-Yield Systems

more than a year without success. Laboratory


(D) Prostatectomy
samples were drawn and her ovarian ultra-
sound image is shown. Which of the following
45. A 15 year-old girl presents to the clinic because
laboratory results is expected?
of primary amenorrhea and recent masculin-
ization of her genitalia. Her mother reports (A) Decreased estrogen
that the patient was born phenotypically fe- (B) Decreased testosterone
male with clitoromegaly. On initial laboratory (C) Excess follicle-stimulating hormone
testing as an infant, the patient had a testoster- (D) Excess luteinizing hormone
one level of 482 ng/dL (normal: 437-707 ng/ (E) Excess progesterone
dL), an estrogen level of 12 pg/mL (normal:
10-60 pg/mL) and a luteinizing hormone level 48. A 23-year-old woman presents to the emer-
of 8 U/L (normal: 7-24 U/L). On physical ex- gency department because of vaginal bleeding.
amination at this visit, testes are found to be She says that she is in her ninth week of preg-
present. Which of the following disorders does nancy according to her last menstrual period.
this patient most likely have? Laboratory studies show a b-human chorionic
gonadotropin level of 103,000 mIU/L. The
(A) 5a-reductase deficiency
sample shown in the image is retrieved from
Reproductive

(B) Complete androgen insensitivity


the patient’s uterus. There are no recognizable
(C) Double Y syndrome
fetal parts. Which of the following describes
(D) Female pseudohermaphroditism
the most likely genotype and parental source
(E) True hermaphroditism
of DNA in this mass?
46. A 35-year-old woman presents to her physician
with complaints of monthly, recurring bilateral
breast pain in the upper outer quadrants that
radiates to the arms and shoulders, as well as
discrete lumps in the upper outer quadrants of
her breasts. She reports that her symptoms are
most prominent prior to each menstrual cycle.
The patient adds that she has one daughter
and chose to bottle-feed. A mammogram is
performed and shows clusters of calcifications
in both breasts. She underwent core needle
biopsy and the pathology demonstrated non-
proliferative lesions. Which of the following is
true regarding the patient’s diagnosis?
Courtesy of Armed Forces Institute of Pathology.
Chapter 16: Reproductive  •  Questions 427

(A) 46,XX; maternal 50. A 38-year-old oncology patient comes to the


(B) 46,XX; paternal physician complaining of vaginal burning and
(C) 46,XX; maternal and paternal itching. On physical examination a whitish,
(D) 69,XXX; maternal and paternal curd-like vaginal discharge and inflammation
(E) 69,XXY; maternal and paternal of the walls of the vagina and vulva are ob-
served. Which of the following is another man-
49. After fertilization of the ovum, implanta- ifestation of infection with this organism?
tion occurs in the endometrium. At this time
(A) Chronic lung disease resembling tubercu-
the developing placenta begins to produce a
losis
hormone necessary for embryonic viability.
(B) Esophagitis
Which of the following best describes this hor-
(C) Lesions in lung cavities
mone’s action?
(D) Meningoencephalitis

High-Yield Systems
(A) Increases the production of milk by the (E) Migrating synovitis
mammary glands
(B) Increases the threshold for uterine contrac-
tion
(C) Initiates parturition at the end of preg-
nancy
(D) Stimulates the corpus luteum to produce
estriol and progesterone
(E) Stimulates the placenta to produce estriol
and progesterone

Reproductive
428 Section II: Organ Systems  •  Answers

An s w e r s

1. The correct answer is E. Menopause occurs Answer C is incorrect. The risk of MI is in-
when a woman has no menstrual cycles for creased by 29% in women receiving HRT. It
one year. The two-eight years leading up to this was initially thought that estrogens would have
time are called peri-menopause. During this a beneficial effect on the patient’s lipid profile,
time, hormones fluctuate tremendously, even- and thus decrease the risk of MI, but it seems
tually leading to a decrease in estrogen, an in- that the thrombotic risks outweigh the benefi-
crease in follicle-stimulating hormone (FSH), cial lipid profile effects. Cardiovascular risk is
an increase in luteinizing hormone (LH), and due to the fact that hepatic estrogen receptor
an increase in gonadotropin-releasing hor- agonist activity upregulates protein synthesis,
mone (GnRH). Associated symptoms include including clotting factors. As such, HRT is not
High-Yield Systems

Hot flashes, Atrophy of the Vagina, Osteopo- indicated as a clinically cardioprotective treat-
rosis, and Coronary artery disease (remem- ment.
ber, menopause causes HAVOC). All of these
Answer D is incorrect. Although HRT does
hormone fluctuations can result in vasomotor
have a beneficial effect in warding off bone
symptoms, which are more commonly known
demineralization, this is not currently an indi-
as “hot flashes.” Combinations of estrogen
cation by itself for HRT in a postmenopausal
and progestin are used as HRT to decrease hot
woman.
flashes, vaginal dryness, and mood swings in
postmenopausal women. Unfortunately, HRT 2. The correct answer is C. You do not need to
has recently been associated with an increased memorize all the risk factors for breast cancer
risk of breast cancer, stroke, myocardial infarc- to answer this question. One thing that should
tion (MI) in the first year after starting therapy, immediately jump out in this patient’s history
and deep venous thrombosis that leads to pul- is her strong family history of breast cancer
monary embolism. at a young age (<40 years). This should raise
Answer A is incorrect. The risk of breast can- red flags in your mind for a genetic predispo-
cer is increased by 26% in women receiving sition to breast cancer, such as the BRCA1 or
Reproductive

HRT. Breast cancer has been shown to be an BRCA2 gene.


estrogen-dependent disease. Women who have Answer A is incorrect. While consumption of
never been exposed to estrogen, through a lack alcohol does increase one’s risk for breast can-
of ovarian function and absence of hormone cer, this is not a significant amount, and is not
treatment, do not develop breast cancer. more important than this patient’s family his-
Answer B is incorrect. The risk of deep ve- tory of breast cancer.
nous thrombosis and pulmonary embolism is Answer B is incorrect. While breast cancer
doubled in women receiving HRT. It was ini- increases in incidence with age, this does not
tially thought that estrogen’s beneficial effects raise this patient’s risk more than her family
on the patient’s lipid profile would decrease history.
thromboembolic risk, but it appears that the
paradoxical thrombotic risks outweigh the ben- Answer D is incorrect. Having never had a
efits to the lipid profile. Increased cardiovas- mammogram does increase the chance this pa-
cular risk is due to hepatic estrogen receptor tient has an undetected cancer; however, her
agonist activity upregulating protein synthesis, significant family history is a bigger risk factor.
including clotting factors. The risk of stroke is Answer E is incorrect. Occasional irregu-
increased by 41% in women receiving HRT. As lar periods may mean this patient has begun
such, HRT is not indicated to clinically reduce menopause; however, it does not significantly
risk of stroke. increase her risk for breast cancer, and cer-
Chapter 16: Reproductive  •  Answers 429

tainly is not more important than her family lopian tube, the growing fetus will eventually
history. rupture the organ, leading to life-threatening
intra-abdominal bleeding or it will die and
3. The correct answer is E. The term pleomor- spontaneously abort. One risk factor for the
phic, gram-variable rod is used to describe development of ectopic pregnancy is previous
Gardnerella vaginalis, which causes vaginosis pelvic inflammatory disease (PID) due to its as-
that is characterized by a gray-white vaginal sociated damage to fallopian tubes. Scarring of
discharge with a fishy odor. Clue cells on sa- the lining of the tubes renders them unable to
line wet mount are diagnostic and appear as propel the fertilized ovum toward the uterus.
vaginal epithelial cells covered with bacteria. Organisms associated with PID are Chla-
Inflammatory cells often are present as well. mydia trachomatis, an obligate intracellular
Answer A is incorrect. Budding yeast and/or bacterium, and Neisseria gonorrhoeae, a gram-

High-Yield Systems
hyphae are used to describe Candida albicans, negative diplococcus.
a fungus that causes “yeast infection,” a vul- Answer A is incorrect. Escherichia coli, a
vovaginitis that presents with vulvar pruritus, gram-negative bacillus, is a common cause of
dysuria, and a thick, adherent “cottage cheese- urinary tract infections in women, but is not
like” discharge. The organism is visualized on associated with PID or ectopic pregnancy.
wet mount after addition of potassium hydrox-
Answer B is incorrect. Gardnerella vaginalis,
ide.
a bacillus that has variable Gram staining, is a
Answer B is incorrect. The term gram- cause of bacterial vaginosis, which is not asso-
negative diplococci is used to describe Neis- ciated with an increased risk of PID or ectopic
seria gonorrhoeae, which can cause urethritis, pregnancy.
cervicitis, and pelvic inflammatory disease.
Answer C is incorrect. Treponema pallidum, a
Presenting symptoms often include pain and
spirochete, causes the sexually transmitted dis-
mucopurulent vaginal discharge. These organ-
ease syphilis, which is not associated with PID
isms often are found within surrounding neu-
or ectopic pregnancy.
trophils.
Answer D is incorrect. Trichomonas vaginalis
Answer C is incorrect. The term gram-positive

Reproductive
is a teardrop-shaped trophozoite that is spread
rod is used to describe Lactobacillus, which
through sexual contact. In females, it colonizes
comprises part of the normal vaginal flora in
the vagina and produces a greenish, watery,
adults.
and foul-smelling vaginal discharge and pruri-
Answer D is incorrect. The term obligate in- tus.
tracellular parasite is used to describe Chla-
mydia trachomatis, which can cause urethritis, 5. The correct answer is B. This patient has pre-
cervicitis, and pelvic inflammatory disease. eclampsia, which is characterized by hyperten-
Presenting symptoms often include pelvic pain sion and proteinuria. If seizures are present,
with mucopurulent vaginal discharge, and in- the diagnosis of eclampsia is made. Preeclamp-
clusion bodies within epithelial cells can be sia generally occurs during the second or third
seen on Giemsa stain or fluorescent antibody trimester, and common symptoms include
smear. headache, blurred vision, abdominal pain,
edema of face and extremities, altered menta-
4. The correct answer is E. The woman is pre- tion, and hyperreflexia. Thrombocytopenia
senting with classic signs of ectopic pregnancy. and hyperuricemia may also occur. It is associ-
Once the b-human chorionic gonadotropin ated with HELLP (Hemolysis, Elevated Liver
(b-hCG) is above 1500, and intrauterine preg- function tests, Low Platelets) syndrome, which
nancy should be visible on transvaginal ultra- is associated with higher morbidity and mor-
sound imaging. If it is not, ectopic pregnancy tality for both mother and fetus. The primary
must be suspected. If embedded in the fal- treatment for preeclampsia is delivery of the
430 Section II: Organ Systems  •  Answers

fetus. Patients may be managed expectantly women and present with multiple masses.
with bed rest and frequent monitoring of blood These tumors are benign and can be associ-
pressures if remote from term and no evidence ated with dysmenorrhea (menstrual pain),
of severe disease. Intravenous (IV) magnesium menorrhagia/menometrorrhagia (heavy pro-
is administered to prevent eclamptic seizures. longed bleeding), infertility, and abnormal pel-
Alternative medications include diazepam and vic exams including palpable masses extend-
phenytoin, but these are second-line agents. ing from the uterus. Because they are estrogen
Administration of magnesium may cause de- sensitive, they tend to increase in size during
pressed tendon reflexes. If toxic levels are menses or pregnancy and decrease in size af-
reached, then respiratory paralysis or cardiac ter menopause. Treatment of fibroids is solely
arrest can occur. dependent on the severity of symptoms and the
desire of the woman to preserve or not preserve
Answer A is incorrect. Alteration of sleep cy-
High-Yield Systems

fertility. Myomectomy can be performed in a


cles is seen in some patients taking phenobar-
woman wishing to preserve fertility, whereas
bital. Another potential yet significant adverse
hysterectomy is used in women with severe
effect of this medication is dependence. Nei-
symptoms not wishing to preserve fertility.
ther has been demonstrated in patients given
Medical therapy can be tried with monother-
magnesium sulfate.
apy or combinations of GnRH analogs (leu­
Answer C is incorrect. The development of prolide), combined oral contraceptives, proges-
erythematous papules and plaques, progressing tins, and/or GnRH antagonists (centrorelix).
to epidermal necrosis and sloughing, is called
Answer B is incorrect. Chocolate cysts and
Stevens-Johnson syndrome (SJS). It can occur
“powder burns” are most often associated with
following exposure to a number of medica-
endometriosis (nonneoplastic ectopic endo-
tions, including the anticonvulsants carbam­
metrial tissue outside the uterus). Endometri-
azepine, phenytoin, and lamotrigine. It is
osis is often associated with severe menstrual-
generally preceded by malaise and fever, and
related pain and infertility, but this patient’s
symptoms begin after two weeks. However,
masses are confined to the uterus, and the bi-
magnesium sulfate is administered for seizure
opsy shows smooth muscle proliferation rather
prophylaxis in preeclampsia, and no associa-
than the glandular and stromal proliferation at
Reproductive

tion has been found between magnesium sul-


ectopic sites that would be expected with en-
fate and SJS.
dometriosis.
Answer D is incorrect. An extensor plantar re-
Answer C is incorrect. Leiomyomas are be-
flex, also known as Babinski’s sign, can occur
nign tumors that are very rarely associated with
from a number of conditions, including hy-
malignant transformation. Malignant leiomyo-
pomagnesemia. On the contrary, this patient
sarcomas most typically arise de novo with ar-
would have hypermagnesemia from adminis-
eas of necrosis and hemorrhage, not from leio-
tration of magnesium sulfate, resulting in de-
myomas.
creased tendon reflexes.
Answer D is incorrect. Leiomyomas are
Answer E is incorrect. In patients with hypo-
not the cause of PID, but rather occur in-
calcemia, tapping the facial nerve at the angle
dependently of the development of PID.
of the jaw results in ipsilateral contraction of
PID is the end result of a polymicrobial in-
the facial muscles. This is known as Chvostek
fection of the genital tract, most commonly
sign. A patient with preeclampsia, however,
caused by untreated Neisseria gonorrhoeae
would be treated with magnesium. Clinical
and Chlamydia trachomatis. PID commonly
modulation of calcium levels is not indicated.
presents with odiferous vaginal discharge,
6. The correct answer is A. Leiomyomas, or fi- pain with sexual intercourse, urinary symp-
broids, are common smooth muscle tumors toms, and dysmenorrhea as well as a his-
that are most often seen in African-American tory of multiple unprotected sexual encoun-
Chapter 16: Reproductive  •  Answers 431

ters. However, PID can also be asymptomatic. tract causes a decrease in positive reward train-
Treatment of PID is aimed at treating the most ability and a decrease in the positive symptoms
likely microbial pathogen(s). PID does not of schizophrenia such as hallucinations, delu-
cause proliferation of the smooth muscle cells sions, and frank psychosis. Both typical and
leading to benign tumors. atypical antipsychotics act as D2-receptor an-
tagonists in this tract to curb the positive symp-
Answer E is incorrect. Leiomyomas, or fi-
toms of schizophrenia.
broids, are estrogen-sensitive benign tumors
and thus frequently regress in size after meno- Answer C is incorrect. The nigrostriatal tract
pause. Also, they are not malignant nor do is a part of the basal ganglia that is involved
they metastasize. A metastatic mass in a post- in the production of movement. D2-receptor
menopausal woman should warrant further antagonism in this area by typical antipsychot-
evaluation of a neoplastic process, and should ics produces Parkinson-like movements and

High-Yield Systems
lessen the suspicion of uterine fibroids. tardive dyskinesia. The atypical antipsychotics
tend to have less D2-receptor blockade in this
7. The correct answer is D. Chlorpromazine is a tract and thus cause less movement-related ad-
typical antipsychotic agent that works as a D2- verse effects compared to the typical antipsy-
receptor antagonist in all of the four major do- chotics. Interestingly, atypical antipsychotics
pamine tracts listed as choices. In the normal work more effectively antagonizing 5-HT re-
hypothalamic-adrenal axis, all hormones are ceptors than D2-receptors in this tract.
regulated by increasing or decreasing releasing
hormones except prolactin. Prolactin secretion 8. The correct answer is C. Testicular cancer is
is regulated by a negative feedback system that most often diagnosed in men who are between
is controlled by the dopaminergic neurons. 15 and 35 years old, and seminomas are the
An inverse relationship is seen, where an in- most common type, accounting for approxi-
crease in dopamine causes a decrease in pro- mately 40% of testicular cancers. The lym-
lactin secretion and vice versa. Knowing that phatic spread of testicular cancers is often seen
the typical antipsychotics act as D2-receptor in the paraaortic chain of lymph nodes.
antagonists in the tuberoinfundibular tract
Answer A is incorrect. Epididymitis is an in-
causes hyperprolactinemia is the first step to

Reproductive
flammation of the epididymis, which is poste-
answering this question. Combining this with
rior to the testis. This condition would be pain-
the fact that hyperprolactinemia causes abnor-
ful, and it is unlikely to have grossly evident
mal menstruation and/or amenorrhea leads us
lymphadenopathy in the para-aortic chain.
to choose the tuberoinfundibular tract as the
interrupted tract leading to prolactin disinhibi- Answer B is incorrect. A Leydig cell tumor is
tion by D2-receptor antagonism. a form of testicular cancer, but it is much less
common than the seminoma.
Answer A is incorrect. The mesocortical tract
is thought to be related to cognition, and D2- Answer D is incorrect. Testicular torsion is a
receptor antagonism by typical antipsychotics urologic emergency, and it would present with
in this tract are thought to produce and/or ex- acute, high-intensity pain as a result of ische­
acerbate the negative symptoms of schizophre- mia.
nia such as decreased cognition, avolition, and
alogia. This explains why atypical antipsychot- 9. The correct answer is B. Kartagener syndrome
ics are preferred as they more effectively tar- is an inherited disease that results in immo-
get dopamine receptors in this tract, resulting tile cilia caused by a dynein arm defect. The
in fewer of the negative symptoms commonly clinical symptoms that result from this defect
seen in schizophrenia. include bronchiectasis, recurrent sinusitis, and
male and female infertility. Situs inversus oc-
Answer B is incorrect. D2-receptor antago- curs in approximately 50% of those with the
nism by chlorpromazine in the mesolimbic
432 Section II: Organ Systems  •  Answers

syndrome and usually has no serious adverse responsible for the production of breast milk,
health consequences. and high levels of prolactin cause anovulation.
Answer A is incorrect. Flushing and diarrhea
11. The correct answer is D. The suspensory liga-
are symptoms of a carcinoid tumor. Additional
ments (also known as the infundibulopelvic
symptoms include wheezing and salivation.
ligaments) contain the ovarian arteries and
The peak incidence is between ages 50 and 70
veins, which are responsible for the direct
years.
blood supply to the ovaries. The ovaries also
Answer C is incorrect. Panacinar emphysema receive collateral flow from the uterine arteries
is a symptom of a1-antitrypsin deficiency. a1- that travel in the cardinal (transverse cervical)
Antitrypsin deficiency may also present with ligament at the base of the broad ligament.
liver disease due to accumulation of variant a1-
Answer A is incorrect. The cardinal (trans-
High-Yield Systems

antitrypsin molecules within the hepatocytes.


verse cervical) ligament carries descending
Answer D is incorrect. Pulsus paradoxus is branches of the uterine artery. Although the
an exaggerated fall in systemic blood pressure uterine arteries provide collateral blood flow
during inspiration as felt in a peripheral pulse. to the ovaries, severing the cardinal ligament
This condition may be caused by asthma and, should not significantly decrease blood flow to
in this case, is not the most likely answer. the ovary if the ovarian arteries remain intact.
Answer E is incorrect. Simple partial sei- Answer B is incorrect. The fallopian tubes
zures in children can be caused by genetic, carry the ova from the ovary to the uterus dur-
infectious, traumatic, congenital or metabolic ing ovulation. Severing this structure would
causes. They are not associated with Karta- disrupt normal fertilization but would not sig-
gener syndrome. nificantly affect blood flow to the ovary.
Answer C is incorrect. The round ligament
10. The correct answer is D. Progesterone is pro-
runs inferior to the ovary before attaching to
duced by the corpus luteum shortly after ovu-
the uterus. It contains no important structures
lation. One of its locations of action is the
and is not a source of blood for the ovary.
hypothalamic thermoregulatory center, lead-
Reproductive

ing to a slightly elevated basal body tempera- Answer E is incorrect. The ureters run directly
ture (up to 1°F). Therefore it is possible to as- inferior to the uterine arteries before feeding
sess when ovulation has occurred by checking into the bladder. Remember: “Water under the
one’s basal body temperature on a daily basis. bridge.” Severing this structure would not af-
fect ovarian blood flow.
Answer A is incorrect. Estrogen does have
some effect at the hypothalamic thermoregula-
12. The correct answer is A. Advancing age is the
tory center during menopause, contributing to
most important risk factor for prostate carci-
“hot flashes,” but it does not affect body tem-
noma. It is the most common cancer in adult
perature during the normal menstrual cycle.
males and the second most common cause of
Answer B is incorrect. Human chorionic go- death due to cancer in adult males. This con-
nadotropin has no effect on body temperature dition is more common in African-American
following ovulation. It is present during preg- men and is usually asymptomatic until ad-
nancy and is the hormone detected during a vanced. Advancing age, family history, race,
pregnancy test. and smoking are all risk factors. The screening
test for prostate cancer is prostate-specific an-
Answer C is incorrect. LH surge causes ovula-
tigen level (PSA) and subsequent biopsy. Pros-
tion to occur but has no physiologic effects on
tate cancer can invade locally or spread via the
body temperature.
lymphatics or bloodstream to bone, lung, and
Answer E is incorrect. Prolactin has no effect liver. The bone scan in this patient shows dif-
on body temperature following ovulation. It is fuse bony metastases.
Chapter 16: Reproductive  •  Answers 433

Answer B is incorrect. There is no evidence controlled divisions and is an area of emerging


that links prostate cancer and alcohol use. cancer therapy research.
Answer C is incorrect. This patient has a his-
14. The correct answer is C. This female baby
tory of physical labor and may have sought
has masculinization of her external genitalia
medical care for back pain earlier in life.
due to congenital adrenal hyperplasia (CAH).
Given his current symptoms of weight loss, fa-
CAH is caused by deficiencies in enzymes
tigue, and trouble urinating, he most likely has
required for adrenocortical steroid synthesis,
prostate cancer that has metastasized to bone,
such as 21-hydroxylase and 11b-hydroxylase.
not simply a back problem.
21-Hydroxylase deficiency results in an in-
Answer D is incorrect. Lack of primary care ability to synthesize aldosterone or cortisol,
can be a risk factor for prostate cancer, but in resulting in shuttling of the intermediates to

High-Yield Systems
this patient it is not the most important risk generate androgens, leading to an elevation
factor. of androgen levels and masculinization. Ad-
ditionally, the lack of aldosterone leads to salt
Answer E is incorrect. Smoking is a risk factor
wasting, which can present with hypovolemia
for prostate carcinoma, but it is not the most
and hypotension. Blood tests will reveal hyper-
important risk factor.
kalemia. Treatment includes IV saline and ste-
13. The correct answer is B. The causative agent roid hormone replacement.
of cervical dysplasia and cancer is the human Answer A is incorrect. 11b-Hydroxylase defi-
papillomavirus. Its critical E6 and E7 gene ciency is a cause of CAH, but it does not re-
products downregulate p53 and pRb, respec- sult in hypotension or hypovolemia. Instead,
tively, allowing the cell to cycle out of control it can result in hypertension because the de-
despite any damage to cellular DNA. ficient enzyme allows an accumulation of an
Answer A is incorrect. Blood vessel growth is aldosterone precursor (11-deoxycorticosterone)
an important component of many, if not all, that acts as a mineralocorticoid to cause salt
tumors. However, the arrest of blood vessel retention and hypervolemia. This precursor is
growth is not known to be inhibited directly by not able to be formed with 21-hydroxylase de-
ficiency. 11b-Hydroxylase deficiency typically

Reproductive
human papillomavirus.
presents with masculinization (due to shuttling
Answer C is incorrect. DNA replication (as by of the intermediates to generate androgens)
DNA polymerase) is not inhibited by human and hypertension.
papillomavirus. Rather, the downstream effect
of its oncogenes is an upregulation of DNA Answer B is incorrect. 17a-Hydroxylase de-
replication. ficiency results in a phenotypically female
newborn who will not undergo sexual matura-
Answer D is incorrect. Mitogenic signal trans- tion later in life. This enzyme has a role in the
duction is commonly increased by oncogenes, conversion of progesterone and progenolone to
such as myc, src, or bcr-abl. Inhibition of these precursors that will go on to form cortisol, tes-
proteins would not cause tumorigenesis. tosterone, and estrogen. The intermediates that
Answer E is incorrect. RNA-dependent DNA build up will produce an excessive amount of
polymerization is an essential component of aldosterone, resulting in hypertension and hy-
telomerase, which has been implicated in the pokalemia. This deficiency, and the resulting
limitless replicative potential of tumor cells. lack of testosterone and estrogen, result in a
Telomerase elongates the DNA caps at the end phenotypically female 46,XY baby with no in-
of each chromosome, preventing the gradual ternal reproductive organs, or a phenotypically
erosion of genetic material that ultimately female 46,XX baby who will not undergo nor-
forces the cell to cease dividing. Inhibiting mal pubertal development.
this process would stop the cancer cells’ un-
434 Section II: Organ Systems  •  Answers

Answer D is incorrect. 5a-Reductase defi- tions, but tobacco use is actually protective
ciency occurs in 46,XY newborns that have against endometrial cancer to a certain extent.
phenotypically female external genitalia (al-
Answer B is incorrect. Early sexual activity has
though clitoromegaly may be present) and
little relation to endometrial cancer but is a
male internal reproductive organs. Normally,
major risk factor for cervical cancer. Other risk
5a-reductase converts testosterone to dihy-
factors for cervical cancer include multiple sex
drotestosterone (DHT). DHT is essential in
partners, human papillomavirus infection, co-
the development of the male external genita-
infection with other sexually transmitted dis-
lia, and a lack of DHT results in feminization
eases, smoking, and low socioeconomic status.
of the penis and scrotum with normal internal
male reproductive organs. At puberty these pa- Answer D is incorrect. Nulliparity (not mul-
tients may suddenly experience virilization of tiparity) is a risk factor for endometrial can-
High-Yield Systems

the external organs due to the increase in tes- cer. Multiparity protects against endometrial
tosterone. cancer, because it gives the endometrium a
“resting period” in which it is not actively pro-
Answer E is incorrect. Complete androgen
liferating through the menstrual cycle. By the
insensitivity is a result of a mutation in the an-
same logic, menopause occurring at or after 53
drogen receptor gene. These 46,XY patients
years of age would put the patient at increased
develop testes, which are usually undescended
risk because of an increased amount of endo-
or found in the labia majora, and female ex-
metrial active proliferation.
ternal genitalia and vagina with no internal
reproductive organs. They typically present as Answer E is incorrect. Although proges-
normal-appearing girls, with normal breast de- terone has a protective effect, estrogen-
velopment and body habitus, who consult their progesterone synthetic birth control products
physician when they do not begin menstrua- have not shown any benefit in reducing the in-
tion. These patients typically have decreased cidence of endometrial cancer.
or absent axillary and pubic hair and are taller
than average. 16. The correct answer is D. The firm prostate
nodules and weight loss suggest prostate can-
15. The correct answer is C. Obesity (especially cer. The prostate-specific antigen level is typi-
Reproductive

when 22.7 kg [50 lb] or more overweight) in- cally elevated in prostate cancer. Calcium
creases risk for the development of endome- levels should be low because calcium is being
trial cancer five to ten-fold. The increased risk used to build new bone in the areas of metas-
in obese patients is from increased aromatiza- tases. The alkaline phosphatase level should
tion in peripheral tissues, resulting in higher be increased because this is a marker of bone
levels of circulating estrogen. Endometrial tis- formation.
sue is estrogen sensitive, and higher levels of Answer A is incorrect. Prostate-specific anti-
circulating estrogen lead to increased glandu- gen should be elevated in prostate cancer, and
lar proliferation and increased risk for dysplas- the alkaline phosphatase level should be high.
tic transformation. Because weight is one of
the modifiable risk factors for endometrial can- Answer B is incorrect. Prostate-specific anti-
cer, lowering the body mass index in patients gen should be elevated in prostate cancer.
at risk for developing endometrial cancer is a Answer C is incorrect. In prostate cancer, be-
good method of primary prevention. cause bone is being made, the alkaline phos-
Answer A is incorrect. Alcoholism has little phatase level should increase.
relation to endometrial cancer, but is strongly Answer E is incorrect. The calcium level
associated with chronic pancreatitis, pancre- should be low in prostate cancer because it
atic adenocarcinoma, and cirrhosis of the liver. is being used to make new bone. The alka-
Smoking is also a risk factor for these condi-
Chapter 16: Reproductive  •  Answers 435

line phosphatase level should be elevated as a Answer E is incorrect. PDE-5 inhibitors such
marker of bone formation. as sildenafil prevent the degradation of cGMP,
thus enhancing the effect of NO. However,
17. The correct answer is D. Combining an erec- NO levels are not increased by sildenafil.
tile dysfunction medication such as sildenafil
with a nitrate can lead to severe hypotension. 18. The correct answer is C. Postpartum major
Formation and maintenance of an erection depression (PMD) occurs in 10% of women
requires both nitric oxide (NO) and cGMP. after the birth of a child, and begins anywhere
These mediate increased arterial blood flow from 24 hours to several months after delivery.
and pressure in the corpora cavernosa and Symptoms of postpartum depression (PPD)
corpus spongiosum such that the penile ve- are identical to the Diagnostic and Statistical
nous outflow becomes obstructed. NO acts as Manual of Mental Disorders, Fourth Edition,

High-Yield Systems
a local neurotransmitter that relaxes trabeculae Text Revision (DSM-IV-TR) symptoms for
within the corpora cavernosa and allows maxi- major depression, which can be recalled us-
mal engorgement. It also activates guanylyl ing the mnemonic SIGECAPS: Sleep distur-
cyclase to promote the formation of cGMP, bances, loss of Interest, Guilt, loss of Energy,
which is necessary for vasodilation. Sildenafil difficulty with Concentration, loss of Appetite,
and other emergency department medications Psychomotor agitation or retardation, and Sui-
belong to a class called phosphodiesterase-5 cidal ideation. PMD can be difficult to diag-
(PDE-5) inhibitors. PDE-5 metabolizes cGMP nose because many symptoms can be confused
and prevents NO-induced vasodilation. Silde- with normal sequelae of labor and delivery.
nafil and other PDE-5 inhibitors prevent this
Answer A is incorrect. Hemorrhage may cause
metabolism, thereby increasing the level of
iron deficiency anemia, which may present
cGMP and permitting the maintenance of an
with a picture similar to that of major depres-
erection. Nitrates that are used intermittently
sion. Psychiatric diseases normally require that
for angina are contraindicated within 24 hours
medical conditions with overlapping symptom-
of the ingestion of sildenafil because the two
atology have been excluded before the diagno-
drugs act by a similar mechanism. Nitrates are
sis is made. The patient’s prominent psychi-
metabolized to NO and nitrosothiols, which
atric symptoms and normal hemoglobin rule

Reproductive
activate guanylyl cyclase to produce cGMP
out iron deficiency anemia as a cause of her
and cause vasodilation. When combined, se-
illness.
vere hypotension can ensue because blood
pressure may drop by as much as 50/25 mm Answer B is incorrect. Twenty-five to 85% of
Hg. women experience minor mood fluctuations
after delivery. These “postpartum blues” tend
Answer A is incorrect. Sildenafil is a drug that
to peak around day five postpartum and resolve
inhibits PDE-5, thus causing increased levels
by day 10. Persistence of symptoms beyond
of cGMP; this drug has no action on sodium
postpartum day 10 warrants further investiga-
channels.
tion. Postpartum blues is a risk factor for the
Answer B is incorrect. PDE-5 metabolizes development of PMD.
cGMP and prevents NO-induced vasodilation.
Answer D is incorrect. Less than 0.5% of
Sildenafil and other PDE-5 inhibitors prevent
child-bearing women experience postpartum
this metabolism, thereby increasing (not de-
psychosis, usually within one month of deliv-
creasing) the level of cGMP.
ery. It is characterized by a manic-like episode
Answer C is incorrect. Sildenafil and other of agitation, expansive or irritable mood, no
PDE-5 inhibitors do not directly affect the sleep for several nights, and avoidance of the
level of NO; rather, they enhance the effect of infant. Delusions and hallucinations are pres-
NO by increasing levels of cGMP. ent and often involve the baby (eg, command
hallucinations to harm the infant).
436 Section II: Organ Systems  •  Answers

Answer E is incorrect. The most common ectopic endometrial tissue outside the uterus.
presentation of postpartum hypopituitarism It generally presents in women 20-40 years
(Sheehan syndrome) is failure to lactate, and old, but its pathogenesis is poorly understood.
it is caused by severe puerperal hemorrhage. Although its clinical presentation varies, endo-
Eventually the patient experiences the symp- metriosis can present with pelvic pain associ-
toms of hypothyroidism (fatigue, constipation, ated with the menstrual cycle, dysmenorrhea,
and non-resumption of menses), and hypo­ and dyspareunia, or it can be asymptomatic.
adrenalism (hyponatremia and hyperkalemia Endometriosis is a risk factor for ectopic preg-
due to decreased aldosterone, and loss of pubic nancy and infertility, and many women who
and axillary hair because androgens in women have endometriosis first present with problems
are produced in the adrenal cortex). getting pregnant. It is not associated with Shee-
han syndrome.
High-Yield Systems

19. The correct answer is A. This patient is pre-


Answer D is incorrect. Gestational diabe-
senting with Sheehan syndrome or postpartum
tes is a form of diabetes that is present during
pituitary necrosis, caused by hemorrhage dur-
pregnancy and is often transient, although
ing delivery. Risk factors include pregnancy
overt nongestational diabetes may later de-
with multiples (twins or triplets) and abnor-
velop. Gestational diabetes is associated with
malities of the placenta. Peripartum hemor-
increased fetal birth weight, increased fetal
rhage predisposes the already enlarged pitu-
mortality, and increased incidence of neonatal
itary to ischemia, leading to necrosis of parts
respiratory distress syndrome. In addition, in-
of the anterior and/or posterior pituitary. The
creased fetal insulin levels created in response
most common clinical feature of Sheehan
to maternal glucose levels can cause a hypogly-
syndrome is an inability to lactate, caused by
cemic crisis after birth, when the maternal sup-
damage to the anterior pituitary and decreased
ply of glucose is no longer present. Gestational
prolactin production. Other symptoms include
diabetes is not a known risk factor for Sheehan
those associated with hypothyroidism (as seen
syndrome and would not cause this patient to
by the patient’s cold intolerance), along with
present with the symptoms seen.
central diabetes insipidus (DI) caused by de-
creased production of ADH. DI presents with Answer E is incorrect. Incorrect use of tam-
Reproductive

polyuria and dilute urine in the presence of pons is associated with toxic shock syndrome
elevated serum sodium levels. Decreased FSH (TSS), originally associated with the use of
and LH levels often lead to amenorrhea and highly absorbent tampons left in the vagina
scant pubic and axillary hair growth. Treat- for long periods of time. TSS is caused by an
ment involves lifelong hormone replacement exotoxin produced by Staphylococcus aureus,
therapy of all deficient hormones, along with which grows on the tampon. This toxin is a
estrogen and progesterone supplementation. superantigen that allows nonspecific binding
of MHC class II with T-lymphocyte receptors,
Answer B is incorrect. Alcohol intake dur-
resulting in polyclonal T-lymphocyte activa-
ing pregnancy is not associated with any of
tion and systemic symptoms. TSS generally
the symptoms seen in this patient. Instead, it
presents acutely with fever, hypotension, and
is associated with growth and developmental
desquamation of the palms and soles, nausea,
defects in the offspring, such as microcephaly,
vomiting, and diarrhea.
facial dysmorphism, and malformations of the
brain, cardiovascular system, and genitourinary Answer F is incorrect. Having multiple sexual
system. Fetal alcohol syndrome is the leading partners is associated with an increased risk of
cause of mental retardation and is easily pre- various sexually transmitted diseases (STDs),
ventable by maternal abstinence from alcohol including HIV and human papillomavirus,
during pregnancy. the virus associated with cervical cancer. STDs
present with various symptoms depending
Answer C is incorrect. Endometriosis is a
common condition characterized by growth of
Chapter 16: Reproductive  •  Answers 437

on the underlying infectious agent, but none Answer C is incorrect. Development of exter-
would cause the symptoms seen in this patient. nal male genitalia is controlled most directly
by DHT. Insufficient levels of DHT results in
Answer G is incorrect. PID is most commonly
male pseudohermaphroditism, where the in-
caused by infection with Chlamydia trachoma-
dividual has internal male genitalia but are fe-
tis or Neisseria gonorrhea, the latter presenting
male externally. One cause is a deficiency in
more acutely. Symptoms include fever, cervi-
5a-reductase, the enzyme responsible for pro-
cal motion tenderness, lower abdominal pain,
ducing DHT from testosterone. Other causes
and painful intercourse, although PID is often
leading to a male pseudohermaphrodite in-
asymptomatic. Treatment of the underlying
clude partial androgen insensitivity and defects
cause is important because PID is a risk factor
in testosterone production.
for ectopic pregnancy and infertility. It is not
associated with Sheehan syndrome. Answer D is incorrect. Development of the

High-Yield Systems
urogenital sinus into the prostate gland is con-
20. The correct answer is B. The SRY (sex-­ trolled most directly by DHT. In the presence
determining region on Y) gene on the Y chro- of estrogen and absence of DHT, the urogeni-
mosome encodes a transcription factor called tal sinus develops into the urethral and para-
testis-determining factor. This transcription urethral glands (Skene glands) instead.
factor binds DNA, inducing a pattern of gene
Answer E is incorrect. The involution of
expression that ultimately results in develop-
paramesonephric (müllerian) ducts is con-
ment of the testes from indifferent gonads.
trolled by the presence of müllerian-inhibiting
The testes secrete both testosterone and mül-
factor, produced by Sertoli cells. In the female
lerian-inhibiting factor, triggering a sequence
they develop to form the fallopian tubes, the
of changes that give rise to a male phenotype.
uterus, and the upper portion of the vagina.
Without a functional SRY gene (as one might
see in the setting of a nonsense mutation), this
21. The correct answer is E. Oxytocin is a poly-
process is derailed, and the fetus will progress
peptide hormone that is responsible for the
along the default pathway of sexual develop-
dilation of the cervix and contraction of the
ment (that of a female). The mesonephric
uterus during labor, as well as the let down of
(Wolffian) ducts will regress spontaneously,

Reproductive
milk during breastfeeding. When labor is ar-
and the paramesonephric ducts will develop
rested, a synthetic analog may be applied topi-
as they normally do in females. Such individu-
cally onto the cervix to facilitate dilation. En-
als will be born with a female phenotype, and
dogenously, oxytocin is produced in the cell
they may not have any symptoms until pu-
bodies of hypothalamic neurons in the para-
berty, at which point they often present with
ventricular nucleus. After synthesis, it is stored
primary amenorrhea. These women may have
in terminal swellings of these neurons in the
other abnormalities including underdevelop-
posterior pituitary, known as herring bodies.
ment of both breasts and the internal female
reproductive organs. Mutations in the SRY Answer A is incorrect. The adrenal cortex me-
gene lead to XY females with gonadal dysgen- diates the production and release of hormones
esis, also known as Swyer syndrome. related to the stress response as well as sexual
development. It is divided into three layers:
Answer A is incorrect. The mesonephric
(from superficial to deep) zona glomerulosa,
(Wolffian) duct develops into the internal male
which produces mineralocorticoids such as
genitalia that connect the testes to the prostate.
aldosterone; zona fasciculata, which produces
Exposure to testosterone, produced by Ley-
glucocorticoids such as cortisol; and zona re-
dig cells, during embryogenesis is critical to
ticularis, which produces androgens. The adre-
proper development.
nal cortex is not responsible for the storage of
oxytocin.
438 Section II: Organ Systems  •  Answers

Answer B is incorrect. Whereas secretion of Answer E is incorrect. The reticulate body is


hormones from the posterior pituitary is con- solely intracellular. This is the stage in which
trolled by neurons in the hypothalamus, the the bacteria replicate within the cell.
anterior pituitary is a glandular secretory organ
that receives releasing/inhibiting factors via a 23. The correct answer is E. Trastuzumab is a
capillary plexus that connects it with the hypo- monoclonal antibody against human epider-
thalamus. Hormones released by the anterior mal growth factor receptor 2 (HER2) that can
pituitary include ACTH, thyroid-stimulating kill breast cancer cells over-expressing HER2.
hormone, FSH, LH, growth hormone, and This patient’s breast cancer is positive for
prolactin. HER2/neu receptor, thus she is likely to have
a positive response to the chemotherapeutic
Answer C is incorrect. Oxytocin is produced,
agent trastuzumab.
not stored, in the cell bodies in the paraven-
High-Yield Systems

tricular nucleus of the hypothalamus. Answer A is incorrect. Bevacizumab, like all


drugs ending in “mab,” is a monoclonal anti-
Answer D is incorrect. Oxytocin acts at recep-
body. This drug binds to vascular endothelial
tors in the mammary glands to induce milk let
growth factor, a cytokine frequently produced
down in breastfeeding mothers. Stimulation of
by cancer cells to promote angiogenesis. It was
the nipple will trigger the release of oxytocin
approved by the Food and Drug Administra-
from the posterior pituitary. Oxytocin is not
tion in 2004 to treat metastatic colon cancer
stored in the mammary glands themselves.
and non-small cell lung cancer. It also has
been used off-label by ophthalmologists to
22. The correct answer is B. Chlamydia tracho-
slow the progression of macular degeneration
matis infection causes urethritis, cervicitis,
and diabetic retinopathy, diseases that occur
and PID in women, as well as conjunctivitis
through abnormal proliferation of blood ves-
and Reiter syndrome, although it is frequently
sels in the eye. It is not indicated for the treat-
asymptomatic. Cytoplasmic inclusions can
ment of breast cancer.
be seen on Giemsa- or fluorescent antibody-
stained urethral or cervical smear, but diagno- Answer B is incorrect. Cyclophosphamide is
sis can also be made from a urine sample using an alkylating agent that cross-links DNA, lead-
Reproductive

nucleic acid amplification techniques. Treat- ing to apoptosis. Cyclophosphamide is com-


ment of Chlamydia infection requires a course monly used to treat non-Hodgkin lymphoma
of either doxycycline or erythromycin. The and breast/ovarian cancer, but is not specifi-
extracellular elementary body is the infectious cally targeted to a HER2/neu receptor-positive
form of Chlamydia as it can attach to host cells tumor.
and enter them.
Answer C is incorrect. Doxorubicin is an anti-
Answer A is incorrect. The cytoplasmic inclu- tumor antibiotic commonly used to treat breast
sions are collections of elementary bodies in cancer. It acts by intercalation into DNA, in-
the host cell before their release. Because they hibiting topoisomerase II and resulting in
are intracellular, they are not infectious. DNA damage and cell death. Doxorubicin is
associated with congestive heart failure. Doxo-
Answer C is incorrect. The intracellular el-
rubicin might be helpful for this patient, but
ementary body does not have access to host
is not specifically targeted to an HER2/neu
cells, and therefore is not infectious.
receptor-positive tumor.
Answer D is incorrect. The multiplication of
Answer D is incorrect. Tamoxifen as a che-
reticulate bodies is an important step in the re-
motherapeutic agent is useful only in estrogen
production of Chlamydia. Because this process
receptor-positive breast cancer. It is a selective
is intracellular, however, it is not an infectious
estrogen receptor modulator with estrogen re-
step in the life cycle of this bacterium.
ceptor antagonist effects whose mechanism of
action relies on binding, and thereby blocking,
Chapter 16: Reproductive  •  Answers 439

estrogen receptors to impede the production common setting for accreta involves a placenta
of estrogen-responsive genes. Tamoxifen is not previa after a prior cesarean delivery.
useful in a breast cancer that is negative for es-
Answer E is incorrect. Placenta previa occurs
trogen receptor.
when the placenta overlies the internal cervi-
cal os. The distinction between placenta pre-
24. The correct answer is A. Abruptio placentae
via and abruptio placentae is classically made
typically presents as bleeding along with uter-
based on the presence (abruptio placentae) or
ine contractions and pain due to premature
absence (placenta previa) of pain.
separation of the placenta from the uterus, de-
spite its implantation in a normal location. It
25. The correct answer is E. Androgenic steroids
is caused most often by a rupture of defective
are used to treat hypogonadism either due to
maternal vessels in the decidua basalis. This
failure of the hypothalamic-pituitary-gonadal

High-Yield Systems
patient is at high risk for abruptio placentae
axis (secondary hypogonadism) or due to Ley-
because of her known hypertension and co-
dig cell dysfunction (primary hypogonadism).
caine use. Other risk factors include short um-
Patients should be warned that androgens
bilical cord, trauma, prior abruption, cigarette
cause premature closing of the epiphyseal
smoking, uterine fibroids, advanced age, sud-
plates by promoting calcium deposition in the
den uterine decompression, preterm prema-
bones.
ture rupture of the membranes, and a bleed-
ing diathesis. Whereas ultrasound can reveal Answer A is incorrect. Androgenic steroids can
abruptio placentae, it is not very sensitive and cause polycythemia rather than anemia. This
will be positive in only 25% of cases of abrup- adverse affect is another risk factor for prema-
tion confirmed at delivery. ture coronary artery disease and thrombosis.
Answer B is incorrect. Concealed abruption Answer B is incorrect. Some androgenic ste-
refers to abruptio placentae that occurs near roids increase, rather than decrease, LDL cho-
the center of the placenta. By definition, there lesterol levels. The lipid profile disturbance in-
is no external bleeding because the blood creases the possibility of atherosclerotic change
forms a hematoma that is hidden behind the and raises the risk of early coronary artery dis-
placenta. Thus this case cannot represent a ease.

Reproductive
concealed abruption. Answer C is incorrect. Excess androgens can
Answer C is incorrect. Labor is defined as cause decreased spermatogenesis by down-
regular uterine contractions that result in cer- regulating GnRH. Decreased GnRH causes
vical change. This patient does not describe decreased release of LH and FSH, which are
contractions at regular intervals, but rather a necessary for spermatogenesis.
constant pain with intermittent cramping. In Answer D is incorrect. A serious adverse reac-
addition, her cervix remains closed, so she has tion to some androgens is not leukocytosis, but
not started labor. rather leukopenia because of decreased mar-
Answer D is incorrect. Placenta accreta re- row production or decreased WBC survival.
fers to the abnormally strong adherence of the
placenta to the uterine wall. This happens be- 26. The correct answer is B. A decreased
cause the placental villi attach directly to the a-fetoprotein (AFP) level indicates that this
myometrium as a result of a defect in the de- fetus may develop Down syndrome due to
cidua basalis layer. Placenta accreta manifests trisomy of chromosome 21. (Remember the
as incomplete separation of the placenta after phrase: “AFP goes down in Down syndrome.”)
delivery and can result in severe postpartum Individuals with Down syndrome have excessive
hemorrhage. Risk factors include placenta skin at the nape of the neck, upslanting palpe-
previa, prior cesarean delivery, and prior in- bral fissures, and epicanthic folds, among other
trauterine manipulation or surgery. The most characteristic features. In addition, it is associ-
440 Section II: Organ Systems  •  Answers

ated with multiple congenital anomalies in- lation. The efferent ductules are located infe-
cluding ventricular septal defects (VSDs), en- rior, not posterior, to the ductus.
docardial cushion defects, and omphalocele.
Answer B is incorrect. The spermatic cord
Cardiac disease is an important cause of death
contains the ductus deferens, testicular artery,
in these patients.
pampiniform plexus, and lymphatic vessels. It
Answer A is incorrect. Coarctation of the does not cross posterior to ductus deferens.
aorta is a characteristic defect in Turner syn-
Answer C is incorrect. Sympathetic nerve fi-
drome, caused by a 45,XO genotype. Addi-
bers are constituents of the spermatic cord that
tional defects include ovarian dysgenesis and
run with the testicular arteries. They run paral-
horseshoe kidney. Turner syndrome is diag-
lel to the ductus deferens but do not cross pos-
nosed on karyotyping, rather than by the triple
teriorly.
screen.
High-Yield Systems

Answer D is incorrect. The testicular artery


Answer C is incorrect. Deletion of 5p causes
arises from the aorta and supplies the testis
cri-du-chat syndrome. These patients demon-
and epididymis. It runs parallel to, but does
strate hypotonia, downslanting of the lateral
not cross, the ductus deferens to enter the sper-
portion of the palpebral fissures, and micro-
matic cord. The testicular artery crosses the
cephaly.
ureter.
Answer D is incorrect. A prominent occiput
is a sign of Edwards syndrome, due to trisomy 28. The correct answer is C. This patient most
18. This syndrome can affect nearly any organ likely suffers from osteoporosis, or weakened
system, with hypertonia, micrognathia, and bones, as a complication of menopause. Es-
VSDs being common signs. More than 90% of trogen regulates bone resorption, maintaining
these children do not survive past 12 months, proper bone mass in women. At menopause,
and those who do are severely mentally re- estrogen production ceases due to a decreased
tarded. number of ovarian follicles. The reduction in
estrogen results in increased bone resorption.
Answer E is incorrect. This choice describes
Along with low estrogen levels, postmeno-
spina bifida occulta, a specific type of neural
pausal women have high levels of LH, FSH,
Reproductive

tube defect. Neural tube defects are charac-


and GnRH due to the lack of negative feed-
terized by an increased AFP level in the triple
back of estrogen on the anterior pituitary gland
screen and are dramatically reduced in fre-
and hypothalamus.
quency by maternal intake of folate during
pregnancy. Answer A is incorrect. While estrogen levels
are indeed low in postmenopausal women,
27. The correct answer is E. The ureters are levels of LH and FSH are high due to the lack
muscular ducts with narrow lumina that carry of negative feedback on the anterior pituitary.
urine from the kidneys to the urinary bladder. This profile suggests that the problem is in the
These retroperitoneal structures cross the ex- anterior pituitary, which is receiving GnRH
ternal iliac artery just beyond the bifurcation stimulation but is unable to produce enough
of the common iliac artery and pass under the LH and FSH to stimulate the ovaries. Meno-
ductus deferens and testicular vessels (“water pause, however, is a primary dysfunction of
under the bridge”). estrogen production in the ovaries, with a nor-
mally functioning anterior pituitary gland.
Answer A is incorrect. Efferent ductules
transport sperm from the rete testis to the epi- Answer B is incorrect. While estrogen levels
didymis, where they are stored. The tail of the are indeed low in postmenopausal women,
epididymis is continuous with the ductus def- LH, FSH, and GnRH levels are high due to
erens, the next sperm destination during ejacu- the lack of negative feedback on the anterior
pituitary and hypothalamus. This profile sug-
Chapter 16: Reproductive  •  Answers 441

gests a problem in the hypothalamus, result- crine side effects. It has good penetration into
ing in low GnRH, which leads to suboptimal the cerebrospinal fluid and is used to treat
stimulation of the anterior pituitary causing Cryptococcus neoformans. Adverse effects in-
low FSH and LH. The low FSH and LH lev- clude nausea and vomiting.
els cause decreased stimulation of the ovaries,
Answer C is incorrect. Flucytosine is an anti-
resulting in low estrogen levels. Menopause,
fungal drug used solely in combination with
however, is a primary dysfunction of estrogen
amphotericin B to treat systemic Cryptococcus
production in the ovaries, with a normally
neoformans and systemic Candida. Adverse ef-
functioning anterior pituitary gland.
fects include pancytopenia, elevated liver en-
Answer D is incorrect. Menopause is a pri- zyme levels, nausea, and vomiting.
mary dysfunction of estrogen production in the
Answer D is incorrect. Itraconazole is an anti-
ovaries. Thus estrogen levels are low in post-

High-Yield Systems
fungal that lacks the endocrine effects of keto-
menopausal women.
conazole. It is used to treat blastomycosis and
Answer E is incorrect. Menopause is a pri- AIDS-associated histoplasmosis. Adverse effects
mary dysfunction of estrogen production in the include nausea and vomiting, as well as rash in
ovaries. Thus estrogen levels are low in post- immunocompromised patients.
menopausal women.
30. The correct answer is E. The history of ingui-
29. The correct answer is E. Ketoconazole is an nal hernia repair strongly suggests an obstruc-
antifungal drug used to treat tinea corporis that tive abnormality of the vas deferens, leading
acts by blocking the formation of fungal mem- to disordered sperm transport. The vas defer-
brane sterols. It also has an endocrine effect ens can be ligated accidentally during hernia
because it blocks the enzyme desmolase/CYP- repair, or scar tissue can make passage of the
450scc, which is necessary for adrenal produc- sperm through the vas deferens impossible. In
tion of testosterone and cortisol from choles- the case of vas deferens obstruction, the semen
terol. Free cortisol is responsible for feedback volume will be low with decreased or absent
inhibition of the POMC gene, which codes sperm. In these cases testicular biopsy can con-
for synthesis of ACTH, lipotropin, melanocyte- firm normal sperm production, and sperm can

Reproductive
stimulating hormone, and some endogenous be collected for intracytoplasmic sperm injec-
endorphins. Without desmolase activity to tion with in vitro fertilization.
create cortisol, this feedback inhibition is re-
Answer A is incorrect. Like excessive alcohol
moved and thus the POMC gene products are
intake, heavy marijuana smoking has been as-
freely transcribed. Excessive melanocyte-stim-
sociated with decreased sperm production in
ulating hormone can cause increased integu-
men. In this particular case, however, sperm
mentary pigmentation such as in patients with
production appears to be normal. Sperm trans-
Addison disease. Other endocrine effects in-
port abnormalities appear to be causing this
clude decreased libido, impotence, and gyne­
man’s infertility.
comastia in men.
Answer B is incorrect. Although vas deferens
Answer A is incorrect. Amphotericin B is an
obstruction usually causes semen volumes to
antifungal drug used to treat systemic mycoses.
decrease, it is not the decreased semen volume
It acts by disrupting fungal wall synthesis by
itself that has produced this patient’s primary
binding to ergosterol (a component of the cell
infertility but the obstructive azoospermia
wall). Adverse effects include fever and chills,
caused by his prior inguinal hernia repair.
decreased creatinine clearance, hypotension,
and anemia. Answer C is incorrect. Alcoholic beverage in-
gestion that exceeds two drinks/day has been
Answer B is incorrect. Fluconazole is an an-
associated with decreased sperm production in
tifungal drug with the same mechanism of
men. In this particular case, however, sperm
action as ketoconazole, but without the endo-
442 Section II: Organ Systems  •  Answers

production appears to be normal. It is the im- endometriosis, a condition that results in non-
pairment in sperm transport that appears to be neoplastic endometrial glands/stroma being
causing this man’s infertility. abnormally located outside the endometrial
cavity. The ovary and the pelvic peritoneum
Answer D is incorrect. Hypercholesterolemia
are the most common sites. The cysts are
never has been shown to be associated with
formed during cyclic bleeding from the tissue,
dysfunctional sperm transportation.
mimicking menstruation. Severe menstrual-
31. The correct answer is D. Serratia is a com- related pain (dysmenorrhea), pain during sex-
mon cause of nosocomial urinary tract infec- ual intercourse (dyspareunia), and infertility
tions, along with E coli, Proteus, Klebsiella are possible complications of endometriosis.
and Pseudomonas. Some strains produce a red Answer A is incorrect. Common cancers in
pigment. Nosocomial infections generally are the reproductive tract include endometrial
High-Yield Systems

more resistant than community acquired infec- carcinoma, which can arise from endometrial
tions. Serratia is a gram-negative, facultatively hyperplasia caused by excess estrogen stimula-
anaerobic bacillus. tion, and cervical carcinoma, associated with
Answer A is incorrect. Escherichia coli is the human papillomavirus infection. Endometrio-
number-one cause of urinary tract infections sis does not progress to cancer.
in ambulatory patients, accounting for about Answer C is incorrect. Autosomal dominant
80% of patients without catheters, urologic ab- (adult) polycystic kidney disease (ADPKD) is
normalities, or calculi. E coli is not associated characterized by bilaterally enlarged kidneys
with red pigmentation. with multiple cysts. Cysts can also involve
Answer B is incorrect. Klebsiella species are the liver, pancreas, heart, and brain. This dis-
common etiologic agents of urinary tract infec- ease manifests as renal abnormalities that pro-
tion that predispose to stone formation through gresses to end stage renal disease. In patients
the production of extracellular slime and with ADPKD there is usually a family history,
polysaccharide, and are frequently found in bilateral involvement, and constant pain that
patients with calculi. Urease-positive bacteria does not wax and wane in conjunction with
such as Klebsiella and Proteus species can form menstrual cycles. Another manifestation of
Reproductive

ammonium magnesium phosphate (struvite) ADPKD is the emergence of intracranial berry


stones that can be a nidus for future recurrent aneurysms that can rupture and produce intra-
infections. Klebsiella is not associated with red cranial hemorrhage. Endometriosis is not asso-
pigmentation. ciated with ADPKD.

Answer C is incorrect. Proteus mirabilis pre- Answer D is incorrect. Polycystic ovarian syn-
disposes to stone formation because of its drome (PCOS or Stein-Leventhal syndrome)
urease production, and is found more fre- has many causes. The main characteristics of
quently in patients with calculi. Proteus is a PCOS are irregular or anovulation and hyper-
gram-negative, facultatively anaerobic, urease- androgenism. Features seen in PCOS include
positive bacillus. Proteus is not associated with ovarian cysts, amenorrhea, infertility, obesity,
red pigmentation. and hirsutism caused by excess LH and andro-
gens. In some women this is associated with in-
Answer E is incorrect. Staphylococcus sap- sulin resistance and hyperinsulinemia, which
rophyticus is the second most common cause increases androgen production in the ovarian
of urinary tract infection in young ambulatory theca cells and, secondarily, LH production.
women (10-15%). S saprophyticus is coagulase The insulin resistance also leads to hypergly-
negative and resistance to novobiocin. cemia and suppresses hepatic steroid hormone
binding globulin (SHBG) synthesis. The de-
32. The correct answer is B. A blood-filled cyst crease in SHBG along with the increase in
(a so-called chocolate cyst) is characteristic of androgen production leads to a vicious cycle
Chapter 16: Reproductive  •  Answers 443

of amenorrhea and infertility. Endometrio- Answer D is incorrect. Basal cell carcinoma


sis is not associated with masculinization and (BCC) often presents as “pearly papules” on
PCOS. sun-exposed areas, such as the face and arms.
Papules are not associated with stomach or
Answer E is incorrect. Endometriosis is not as-
ovarian cancers. BCC is locally invasive but al-
sociated with obesity.
most never metastasizes.
33. The correct answer is E. Krukenberg tumors
34. The correct answer is E. The history is suspi-
are stomach cancer metastases to the ovaries
cious for the prodrome of an HSV outbreak.
that are described as mucin-secreting “signet-
The pathology procedure in the vignette de-
ring” cells. Stomach cancer is often adenocar-
scribes a Tzanck stain, which was positive for
cinoma that can spread aggressively to lymph
multinucleated giant cells, suggesting an active
nodes and the liver. A classic sign of metastatic

High-Yield Systems
HSV infection. Herpes simplex virus (HSV) is
stomach cancer is involvement of the left su-
an enveloped virus with a double-stranded lin-
praclavicular lymph node, called Virchow
ear DNA genome. After the primary outbreak,
node. Involvement is on the left side because
HSV travels in a retrograde fashion along the
the thoracic duct drains all structures on the
axon via microtubular-dependent transport to
left in the thoracic cavity and all structures be-
the neuronal cell body and remains latent in
low the diaphragm on both sides.
dorsal root ganglia. Over time, the virus can be
Answer A is incorrect. Galactorrhea is leakage reactivated, causing recurrent outbreaks that
from the breasts that is not associated with nor- are usually less severe in symptoms, duration,
mal lactation but is associated with elevated and viral shedding than the primary infection.
prolactin levels secondary to prolactinomas
Answer A is incorrect. HSV can disseminate
in the anterior pituitary. Prolactin stimulates
and cause systemic illness; however, it is very
breast development and milk production while
unlikely in immunocompetent individuals.
also inhibiting ovulation and spermatogenesis
by inhibiting the release of GnRH and subse- Answer B is incorrect. HSV does not produce
quently suppressing LH and FSH. Galactor- an exotoxin. Exotoxins are important in the
rhea is not associated with stomach or ovarian pathogenesis of some bacteria, such as Cam-

Reproductive
cancers. Prolactinomas rarely metastasize. pylobacter jejuni, some Staphylococcus infec-
tions, and some group A streptococcal infec-
Answer B is incorrect. Hematochezia is bright
tions.
red, bloody stool and is often an early sign of
colorectal carcinoma. Risk factors for colorec- Answer C is incorrect. HSV does not infect
tal carcinoma include villous adenomas, CD4+ cells. This is characteristic of HIV,
inflammatory bowel disease, low-fiber diet, which can initially present with an acute ill-
familial adenomatous polyposis, hereditary ness with symptoms that resemble infectious
nonpolyposis colorectal cancer, and a positive mononucleosis. This is usually followed by
family history. Hematochezia is not associated complete resolution of symptoms followed
with stomach or ovarian cancers. Colorectal by an asymptomatic carrier state, which can
carcinomas usually metastasize to the liver. last many years before infection with AIDS-
defining pathogens occurs.
Answer C is incorrect. A palpable gallbladder
(Courvoisier sign) is associated with pancre- Answer D is incorrect. After the primary out-
atic duct obstruction secondary to pancreatic break of genital herpes, HSV travels via retro-
adeno­carcinoma. Other signs and symptoms of grade, not anterograde, transport along neuro-
pancreatic cancer include abdominal pain ra- nal axons to the cell bodies of the peripheral
diating to the subscapular area, weight loss, an- nerves, where it remains latent until reactiva-
orexia, and migratory thrombophlebitis (Trous- tion.
seau syndrome). A palpable gallbladder is not
associated with stomach or ovarian cancers.
444 Section II: Organ Systems  •  Answers

35. The correct answer is C. This boy likely has 37. The correct answer is E. A Tzanck test is a
a seminoma. Seminomas are the most com- smear of an opened skin lesion to detect mul-
mon testicular neoplasm in young men (ages tinucleated giant cells and assay for the HSV.
15-35), and they present without pain as in this The Tzanck test can detect HSV-1 and -2 and
case. Histologically, seminomas are character- the varicella-zoster virus. Epidermal cells in
ized by large cells in lobules, a watery cyto- the vesicles sometimes develop eosinophilic
plasm, and a “fried egg” appearance. intranuclear viral inclusions, or several cells
may fuse to produce giant cells (multinucleate
Answer A is incorrect. This describes an em-
polykaryons), changes that are demonstrated
bryonal carcinoma. These lesions are typically
by the diagnostic Tzanck test based on micro-
painful.
scopic evaluation of the vesicle fluid. The ves-
Answer B is incorrect. Cowdry type A nuclear icles usually clear within three-four weeks, but
High-Yield Systems

inclusions are associated with cytomegalovirus the virus travels along the regional nerves and
infection rather than a testicular neoplasm. becomes dormant in the local ganglia.
Answer D is incorrect. Reinke crystals are Answer A is incorrect. Auer rods are rod-
commonly seen with Leydig cell tumors. shaped bodies in myeloid cells. They are found
These lesions usually produce androgens, re- in acute myelogenous leukemia (AML). They
sulting in gynecomastia in men or precocious represent abnormal azurophilic granules and
puberty in boys. Additionally, they are a non- are particularly numerous in AML associated
germ cell tumor. with the t(15;17) translocation.
Answer E is incorrect. Yolk sac tumors have Answer B is incorrect. Call-Exner bodies are
structures resembling primitive glomeruli. spaces between granulosa cells in ovarian fol-
They are called Schiller-Duval bodies. licles and in granulosa cell tumors.

36. The correct answer is C. This patient has Answer C is incorrect. Lewy bodies are abnor-
polycystic ovarian syndrome. It consists of a mal aggregates of protein that develop inside
constellation of findings, including enlarged nerve cells. On microscopy of affected brain
polycystic ovaries, anovulation, oligomenor- tissue, these spherical masses displace other
cell components. The main disease associated
Reproductive

rhea or amenorrhea, obesity, and hirsutism.


with the presence of Lewy bodies is Parkinson
Answer A is incorrect. Patients with polycystic disease. Lewy bodies also are present in pa-
ovarian syndrome are anovulatory, so they have tients with dementia
elevated estrogen and testosterone levels.
Answer D is incorrect. Mallory bodies are eo-
Answer B is incorrect. Epicanthal folds are as- sinophilic intracytoplasmic inclusions found in
sociated with several congenital genetic abnor- hepatic cells and seen in a variety of diseases,
malities, including Down syndrome, but they including alcoholic liver disease.
are not associated with polycystic ovarian syn-
drome. 38. The correct answer is C. This patient has pla-
Answer D is incorrect. Polycystic ovarian syn- centa previa, or the attachment of the placenta
drome is not associated with increased bleed- to the lower uterine segment, often occluding
ing time. the internal cervical os. Placenta previa typi-
cally presents with painless vaginal bleeding
Answer E is incorrect. Ovulatory dysfunction after 20 weeks’ gestation. Risk factors include
in patients with polycystic ovarian syndrome history of a prior cesarean section delivery, in-
typically results in infertility and the need to creased number of prior pregnancies, and his-
induce ovulation with agents such as clomi- tory of prior curettage (scraping of the uterine
phene in order to conceive. lining to remove fetal and placental tissue) for
spontaneous or elective abortion.
Chapter 16: Reproductive  •  Answers 445

Answer A is incorrect. A history of endome- vention of spontaneous abortions in pregnant


triosis may lead to increased risk of infertility women. However, it was later found to be a
and epithelial ovarian cancer, but it does not teratogen, as it causes a variety of structural
increase a patient’s risk for placenta previa. changes of the cervix of female fetuses in utero
and predisposes them to adenocarcinoma of
Answer B is incorrect. History of pelvic in-
the vagina as young women. Bicornuate uterus
flammatory disease leads to increased risk of
is not associated with DES exposure.
ectopic pregnancy, but does not increase a pa-
tient’s risk for placenta previa.
40. The correct answer is E. A positive transil-
Answer D is incorrect. History of a prior ecto- lumination test suggests that this child has a
pic pregnancy leads to increased risk of future cystic mass, and a testicular hydrocele is one
ectopic pregnancies, but does not increase a of the most common causes of painless scrotal

High-Yield Systems
patient’s risk for placenta previa. enlargement in newborns. It is composed of a
collection of serous fluid between the parietal
Answer E is incorrect. Use of assisted repro-
and visceral layers of the tunica vaginalis. Most
ductive technologies leads to increased risk for
cases are idiopathic.
multiple gestations, but does not increase a pa-
tient’s risk for placenta previa. Answer A is incorrect. Fluid here would sug-
gest orchitis, which is testicular inflammation
39. The correct answer is D. Although genetically typically caused by viral (especially mumps
different from the moment of conception, and rubella) or bacterial infection. Pain, ten-
male and female differentiation does not begin derness, and erythema of the overlying skin
until around the eighth week of development, would typically be seen.
when the gonads secrete hormones that influ-
Answer B is incorrect. Fluid here would sug-
ence the development of either the parameso-
gest epididymitis, which is inflammation of the
nephric ducts in the female or the mesoneph-
epididymis. Typical causes of this infection are
ric ducts in the male. In the absence of the
Chlamydia, Neisseria gonorrhoeae, and Esch-
testis-determining factor located on the Y chro-
erichia coli. Pain is usually present, along with
mosome, the mesonephric ducts begin to de-
urinary frequency, dysuria, and urethral dis-
generate and form a matrix for the developing

Reproductive
charge.
paramesonephric ducts. The paramesonephric
ducts fuse at their inferior margin, forming the Answer C is incorrect. This is typically seen in
single lumen of the uterovaginal canal. Failure elephantiasis as a complication of a parasitic fi-
of complete fusion of these ducts can result in larial infection, which blocks lymphatic drain-
a bicornuate uterus, as shown in the image. age and can cause severe swelling.
This anatomic variation is associated with re-
Answer D is incorrect. This is the location of a
current pregnancy loss.
varicocele, which is a collection of dilated and
Answer A is incorrect. Complete failure of fu- tortuous veins in the pampiniform plexus sur-
sion of the paramesonephric (müllerian) ducts rounding the spermatic cord in the scrotum. It
leads to uterus didelphys, in which there are typically has a “bag of worms” texture on physi-
two separate uterine cavities that each have a cal examination.
cervix and vagina.
41. The correct answer is E. This patient has
Answer B is incorrect. Failure of resorption of
chancroid, a painful genital ulcer caused by
the midline uterine septum results in a septate
Haemophilus ducreyi. Chancroid is relatively
or arcuate uterus.
uncommon in the United States, but is more
Answer C is incorrect. Diethylstilbestrol common in sub-Saharan Africa and Latin
(DES) is a synthetic nonsteroidal estrogenic America. Chancroid presents with a pain-
compound given to women in the mid-1900s ful exudative ulcer with lymphadenitis, and
for a variety of indications, including the pre-
446 Section II: Organ Systems  •  Answers

the most appropriate treatment for this gram- of membranes. Complications include prema-
negative organism is single-dose azithromycin. ture delivery, hyrdrops fetalis, uterine tetany,
and hypvolemic shock. In contrast to placenta
Answer A is incorrect. A 10-day course of oral
previa, placental abruption is a clinical diagno-
acyclovir would be appropriate treatment for
sis, because ultrasound is not sensitive for ab-
genital herpes viral infection. Herpes lesions
ruption inasmuch as the clot has the same tex-
are also painful, but are commonly multiple,
ture as the placenta and is easily missed. In any
and microscopic examination (Tzanck test)
case, this patient is hemodynamically unstable
shows multinucleated giant cells with viral in-
and there is fetal distress, so an emergency ce-
clusion bodies.
sarean section is indicated regardless of the un-
Answer B is incorrect. Highly active antiret- derlying diagnosis.
roviral therapy (HAART) is appropriate treat-
Answer A is incorrect. Pre-eclampsia usually
High-Yield Systems

ment for HIV infection in the absence of ful-


presents during the third trimester of preg-
minant concomitant opportunistic infections.
nancy with hypertension, proteinuria, and
Although chancroid and HIV are often found
edema. It can also be associated with HELLP
in the same patients, chancroid also frequently
syndrome (Hemolysis, Elevated Liver enzyme
occurs in immunocompetent hosts. This par-
levels, and Low Platelet count). Treatment
ticular patient shows no evidence of immuno-
would be administration of magnesium and
deficiency, and HAART will not be effective
delivery of the baby. This patient does not
for his penile lesion. However, he is clearly at
meet all the criteria for pre-eclampsia. A few
high risk for HIV infection, and his HIV status
RBCs in the urine would be normal in a pa-
should be further discussed.
tient with vaginal bleeding.
Answer C is incorrect. Nystatin powder is ef-
Answer B is incorrect. Spontaneous delivery is
fective treatment for candidal intertrigo in
preferable to cesarean section, as it has fewer
moist areas such as inguinal folds. Although
sequelae for future pregnancies. However, in a
this patient has inguinal lymphadenopathy, he
setting where the life of both the mother and
has no reported skin changes in this area con-
fetus are threatened, cesarean section’s benefits
sistent with candidal intertrigo.
outweigh its risks and should be undertaken
Reproductive

Answer D is incorrect. Intramuscular peni- immediately.


cillin G is appropriate treatment for syphilis.
Answer C is incorrect. FAST assesses for free
However, genital ulcers in syphilis are typically
fluid in the pericardium and abdominal cav-
painless, and Gram stain is unrevealing.
ity. It is a tool used to look for internal bleed-
42. The correct answer is D. In a patient with an- ing as may be the case in a ruptured ectopic
tepartum hemorrhage (hemorrhage after 20 pregnancy. However, ectopic pregnancies usu-
weeks’ gestation), the two diagnoses that must ally become symptomatic within the first few
be considered first are placental abruption (the weeks of pregnancy and are hence unlikely
cause in 30% of cases of antepartum hemor- in this patient. FAST will most likely produce
rhage) and placenta previa (the cause in 20% negative results in this patient, as the bleeding
of cases). Abruptio placentae is premature is intrauterine, not extrauterine.
separation of the placenta, most commonly Answer E is incorrect. Amniotic fluid em-
during the third trimester after week 30 of ges- bolism is also referred to as anaphylactoid
tation. It results in painful vaginal bleeding, syndrome of pregnancy and is characterized
uterine contractions, and possible fetal death by hypoxia, respiratory distress, cardiogenic
secondary to uteroplacental insufficiency. Risk shock, and disseminated intravascular coagula-
factors for placental abruption include hyper- tion. It occurs with tearing of maternal vessels
tension, cocaine use, pelvic trauma, uteropla- and can result in maternal respiratory distress.
cental insufficiency, submucosal fibroids, ciga- Unless the clinician is quickly observant, it is
rette smoking, and preterm premature rupture usually diagnosed on autopsy. Amniotic fluid
Chapter 16: Reproductive  •  Answers 447

embolisms are very rare and occur in about is responsible for secretion of the hormone
4/100,000 deliveries. Treatment is usually sup- later in the cycle.
portive, and includes emergent cesarean sec-
Answer C is incorrect. This combination
tion to reduce maternal oxygen demand and
predominates in the days prior to ovulation.
prevent more amniotic fluid from entering ma-
During this time, the now mature and sizable
ternal circulation
ovarian follicle secretes a burst of estrogen that
stimulates LH (and FSH) release from the an-
43. The correct answer is E. This woman is in the
terior pituitary; thus the feedback mechanism
secretory (also called luteal) phase of her men-
of estrogen on the anterior pituitary is positive,
strual cycle, which occurs after ovulation (ap-
a reversal of the negative feedback relationship
proximately day 14 of a typical menstrual cy-
that occurred during the earlier parts of the fol-
cle) through the end of the cycle. Progesterone
licular phase. Progesterone levels remain low

High-Yield Systems
increases after ovulation (as it is produced by
and stable, due to the lack of a corpus luteum,
the corpus luteum) and usually peaks around
which is responsible for the secretion of the
day 21-22. Progesterone is responsible for the
hormone later in the cycle.
increased glandular production of glycogen
and the differentiation and maintenance of the Answer D is incorrect. This combination of
endometrium. The estrogen level is high just hormone changes is seen immediately fol-
prior to ovulation (it induces the LH peak), lowing ovulation, when estrogen levels are
but falls dramatically around the time of ovu- decreasing (prior to increasing again during
lation (when the follicle becomes the corpus the luteal phase) and progesterone levels are
luteum). After ovulation the estrogen levels increasing (since the hormone is secreted by
begin to rise again, returning to a relative peak the newly formed corpus luteum). FSH and
during the luteal phase (around day 21). The LH levels are decreasing at this time due to the
high levels of estrogen and progesterone act in negative feedback exerted by estrogen and pro-
negative feedback during the luteal phase, in- gesterone on their release.
hibiting the release of FSH and LH. Thus dur-
ing the secretory phase, progesterone and es- 44. The correct answer is C. This patient has
trogen levels are high and FSH and LH levels symptoms consistent with benign prostatic

Reproductive
are low due to negative feedback. hyperplasia (BPH), including difficulty initiat-
ing a stream, post-void dribbling, and frequent
Answer A is incorrect. This combination of
nighttime urination. In this case, the large
hormone changes is seen during the end of the
prostate encasing the prostatic urethra caused
menstrual cycle, when the corpus luteum de-
urinary obstruction, leading to a decline in re-
generates. Decreasing progesterone levels initi-
nal function reflected by an increase in creati-
ate the menstrual phase in the endometrium,
nine level. On ultrasound one may be able to
while low levels of estrogen disinhibit FSH
see dilation of the urinary collection system. A
release (ie, a negative feedback mechanism),
well-placed Foley catheter definitively relieves
which is required for the recruitment of fol-
the obstruction.
licles to begin a new cycle.
Answer A is incorrect. Giving fluids would
Answer B is incorrect. This combination is
improve renal function if the patient were hy-
characteristic of the follicular phase of the
povolemic. The patient’s symptoms point to
menstrual cycle, during which estrogen in-
an obstructive reason for his rise in creatinine
creases slowly due to the maturation of the
level rather than to a prerenal cause.
follicle(s) under the influence of FSH. Estro-
gen inhibits further FSH release, ultimately al- Answer B is incorrect. Dialysis would be ap-
lowing a single follicle to develop fully. During propriate if the patient were in renal failure.
this time, progesterone levels are low and sta- However, his creatinine level is not severely
ble due to the lack of a corpus luteum, which elevated, and he is still making urine. Placing
448 Section II: Organ Systems  •  Answers

a Foley catheter directly addresses the cause of lignant potential and are the most common
the patient’s declining renal function. breast mass in women from 25 years old to
menopause.
Answer D is incorrect. While BPH represents
an overgrowth of cells, it is distinctly not a ma- Answer B is incorrect. Fibrocystic changes are
lignant process. Therefore removal of the pros- not associated with any familial breast cancer
tate is not indicated, as it might be for some genes like BRCA1 and BRCA2.
cases of prostate cancer.
Answer C is incorrect. Purulent nipple dis-
charge is associated with breast-feeding and
45. The correct answer is A. 5a-reductase con-
lactation, and it is often due to Staphylococcus
verts testosterone to dihydrotestosterone.
aureus. Since this woman is not lactating or
Dihydrotestosterone is required for the de-
breast-feeding, her diagnosis cannot be acute
velopment of the penis and scrotum during
High-Yield Systems

mastitis.
embryogenesis. An infant with 5a-reductase
deficiency is phenotypically female, with nor- Answer D is incorrect. This patient has fi-
mal levels of testosterone, estrogen, and LH. brocystic change, which is the most common
However, the infant will have bilateral testes breast mass in women <50 years old. Fibroad-
and a normal male internal urogenital tract. enoma usually presents with a small, mobile,
The dramatic increase in testosterone levels rubbery mass with sharp edges, which can
during puberty causes the external genita- change size with a woman’s menstrual cycle.
lia to be masculinized. Because the patient is
Answer E is incorrect. Breast trauma can lead
genotypically male with male internal genital
to traumatic fat necrosis, which can produce a
organs, the phenotypically female patient will
painful indurated mass with possible skin re-
fail to menstruate and may present to the phy-
traction, simulating cancer. Given this patient
sician complaining of primary amenorrhea.
has no history of breast trauma and this condi-
Answer B is incorrect. Complete androgen in- tion only occurs sporadically, it is most likely
sensitivity, or testicular feminization syndrome, not traumatic fat necrosis.
is caused by a defect in the gene that encodes
the androgen receptor. These XY individuals 47. The correct answer is D. This patient has
Reproductive

are phenotypically female, but have bilateral PCOS, which is characterized by excessive
testes that are undescended or located in the androgen production (hirsutism and severe
labia majora. Levels of testosterone, estrogen, acne in this patient), menstrual irregulari-
and LH are high. ties, polycystic ovaries, and obesity. The exact
pathogenesis of the disease is often debated,
Answer C is incorrect. Males with the geno-
but excess LH is thought to play a role by
type XYY are phenotypically normal males.
stimulating ovarian theca cells to increase an-
Answer D is incorrect. Female pseudoher- drogen production and secretion. These excess
maphroditism occurs when ovaries are present androgens can be converted into testosterone
and the external genitalia are virilized or am- by most peripheral tissues, leading to hirsut-
biguous. This condition is usually due to expo- ism, acne, or male pattern alopecia. Also,
sure to androgens early in gestation. consistently high levels of LH can lead to an-
ovulation due to down-regulation of the LH
Answer E is incorrect. True hermaphroditism
receptors at the ovaries. On ultrasound imag-
occurs only when both ovarian and testicular
ing, the common appearance is that of mul-
tissues are present in the same patient; this is
tiple ovarian “cysts”; these are actually imma-
rare in humans.
ture follicles with arrested development due to
46. The correct answer is A. This patient’s condi- ovarian dysfunction. They usually are located
tion is consistent with fibrocystic changes of along the periphery, appearing as a “string of
the breast. Fibrocystic changes have no ma- pearls” on imaging
Chapter 16: Reproductive  •  Answers 449

Answer A is incorrect. The majority of pa- Answer E is incorrect. Maternally and pa-
tients with PCOS have elevated or normal es- ternally derived 69,XXY describes one pos-
trone and estradiol levels. sible DNA make-up of a partial mole. A partial
mole contains more than two sets of chromo-
Answer B is incorrect. Serum androgen levels,
somes that usually consist of both paternal and
including testosterone, androstenedione, and
maternal sets, resulting in triploidy or tetra-
dehydroepiandrosterone sulfate, are increased
ploidy. Partial moles may present with a simi-
in the majority of patients with PCOS.
lar grapelike mass and are associated with fetal
Answer C is incorrect. FSH levels may be nor- parts.
mal or low in PCOS, contributing to the ele-
vated LH:FSH ratio seen in many patients. 49. The correct answer is D. b-hCG is produced
by the placenta immediately after implanta-
Answer E is incorrect. Elevated LH levels

High-Yield Systems
tion in the endometrium of the uterus. b-hCG
and frequent anovulation leads to decreased
acts on the corpus luteum to rescue it from
ovarian progesterone secretion. Patients with
regression. It stimulates the corpus luteum to
PCOS at an increased risk for endometrial hy-
produce estriol and progesterone to maintain
perplasia and carcinoma due to the unopposed
the pregnancy until the placenta takes over
estrogen stimulation of the endometrium.
this role in the second and third trimesters. b-
48. The correct answer is B. A hydatidiform mole hCG levels peak at week nine of gestation and
is a noninvasive tumor caused by aberrant fer- then begin to decline until reaching a steady
tilization, leading to cystic swelling of chori- state around week 25.
onic villi and proliferation of the trophoblast. Answer A is incorrect. Prolactin, not b-hCG,
It results in a mass that can look like a “cluster is responsible for stimulating production of
of grapes,” as seen in the image. Hydatidiform milk by the mammary glands.
moles can be complete or partial. The geno-
Answer B is incorrect. Progesterone, not b-
type of a complete mole is usually 46,XX, com-
hCG, raises the threshold for uterine contrac-
pletely consisting of paternal DNA. It results
tion. This helps prevent spontaneous abortion
when two sperm fertilize an empty egg. There
of the fetus.
is no associated fetus despite the elevated levels

Reproductive
of b-hCG. Answer C is incorrect. b-hCG does not initi-
ate parturition at the end of pregnancy. The
Answer A is incorrect. Maternally derived
actual initiating event is unknown.
46,XX would not cause a hydatidiform mole.
Moles are derived from “empty” ova that are Answer E is incorrect. Estrogen and proges-
then fertilized by sperm. terone are produced by the corpus luteum in
the first trimester and by the placenta in the
Answer C is incorrect. 46,XX describes the
second and third trimesters. The developing
genotype of a normal fetus, receiving one set of
placenta starts producing b-hCG after implan-
chromosomes from each parent.
tation, which stimulates the corpus luteum to
Answer D is incorrect. Paternally derived produce estrogen and progesterone until the
69,XXX describes another possible DNA placenta takes over.
make-up of a partial mole. A partial mole con-
tains more than two sets of chromosomes that 50. The correct answer is B. This patient has
usually consist of two paternal and one mater- symptoms of Candida albicans vulvovaginitis.
nal set of chromosomes, resulting in triploidy Infection with this organism also can cause
or tetraploidy. Partial moles may present with thrush, usually in immunocompromised pa-
a similar grapelike mass but are also associated tients. Candidal esophagitis is common in pa-
with fetal parts. tients with HIV infection. C albicans also can
cause disseminated disease of any organ.
450 Section II: Organ Systems  •  Answers

Answer A is incorrect. Chronic lung disease Answer D is incorrect. In immunocompro-


resembling tuberculosis (TB) is suggestive of mised hosts, several fungi cause meningoen-
infection with Histoplasma organisms, among cephalitis, most notably Cryptococcus neo-
others. Histoplasmosis typically does not pre­ formans. Cryptococcus is a non-dimorphic,
sent symptomatically, although some patients heavily encapsulated yeast found in soil and
experience a flu-like illness with fever, cough, pigeon droppings. In an immunocompetent
headaches, and myalgias. In addition to caus- host, infection often is asymptomatic.
ing lung disease resembling TB, histoplasmosis
Answer E is incorrect. Migrating synovitis is a
can disseminate, affecting the liver, spleen, ad-
feature of disseminated gonococcal infection
renal glands, mucosal surfaces, and meninges.
with Neisseria gonorrhoeae. N gonorrhoeae is a
Histoplasmosis occurs most commonly in the
gram-negative diplococcus that causes urethri-
Mississippi and Ohio River valleys.
tis in both sexes and pelvic inflammatory dis-
High-Yield Systems

Answer C is incorrect. Lesions in lung cavi- ease in women. It is treated with ceftriaxone.
ties (“fungus balls”) are typical of Aspergillus
fumigatus infection. In the lungs A fumigatus
also can cause a patchy pneumonia or pulmo-
nary nodules. A fumigatus is a non-dimorphic
mold with septate hyphae that branch at a V-
shaped 45-degree angle.
Reproductive
Chapter 17

Respiratory

451
452 Section II: Organ Systems  •  Questions

Q u e st i o n s

1. A 25-year-old woman with a history of asthma diffuse, bilateral interstitial infiltrates. Labo-
is brought to the emergency department by ratory studies are remarkable only for an el-
emergency medical services (EMS) after in- evated lactate dehydrogenase level. Which of
gesting a full bottle of theophylline in a suicide the following is the best choice for prophylaxis
attempt. At presentation, she is having a tonic- against this infection in a patient with a sulfa
clonic seizure. Her blood pressure is 80/40 allergy?
mmHg, respiratory rate is 30 breaths/minute,
(A) Aerosolized pentamidine
and her heart rate is 160/minute. The EMS
(B) Ciprofloxacin
personnel report that she has been seizing for
(C) Fluconazole
at least 15 minutes. What is the mechanism of
High-Yield Systems

(D) Terbinafine
action of the most appropriate drug to counter-
(E) Trimethoprim-sulfamethoxazole
act her intoxication?
(A) Decreasing intracellular cAMP 4. A previously healthy 41-year-old man misses
(B) Increasing intracellular cAMP through several days of work as a result of a viral illness
b2-adrenergic receptors with symptoms including fever, headache, and
(C) Increasing intracellular cAMP through fatigue. He also experiences a nonproductive
nonselective adrenergic receptors cough and a sore throat. By the third day, his
(D) Inhibiting the Na+-K+-2Cl- co-transporter symptoms begin to subside and he is able to
(E) Inhibiting the Na+-K+-ATPase pump return to work. The next week, however, he
experiences a rapid relapse. His cough returns,
2. A 28-year-old smoker presents to the emer- but now it is a productive cough with muco-
gency department because of sudden onset of purulent sputum. He also begins to experience
chest pain and dyspnea while at rest. His heart pleuritic chest pain. On visiting a physician,
rate is 115/min, respiratory rate is 24/min, and x-ray of the chest is ordered and is shown in
blood pressure is 140/80 mm Hg in both arms. the image. Which of the following describes
Lung examination shows decreased breath the structure of the viral genome that most
Respiratory

sounds and decreased fremitus on the right likely caused his initial illness?
with hyperresonance to percussion. Which of
the following would most likely be seen on this
patient’s x-ray of the chest?
(A) A widened mediastinum
(B) Barrel chest and flattened diaphragm
(C) Consolidation in the right lower lobe
(D) Contralateral deviation of the trachea
(E) Tracheal deviation to the ipsilateral side;
elevated diaphragm on the right side

3. A 35-year-old HIV-positive woman with a


CD4+ count of 175/bL (normal = 500-1500/
bL) presents to the clinic with a two-week his-
tory of fever, nonproductive cough, and pro-
gressive dyspnea. She has a history of sulfa
allergy. Physical examination reveals diffuse Reproduced, with permission, from Fauci AS, et al, eds. Harri­
son’s Principles of Internal Medicine, 17th edition. New York:
crackles and rhonchi. X-ray of the chest shows
McGraw-Hill, 2008; Figure 128-1.
Chapter 17: Respiratory  •  Questions 453

(A) Enveloped, double-stranded DNA (C) Gram stain


(B) Enveloped, nonsegmented, single-stranded (D) India ink stain
RNA (E) Serum polymerase chain reaction
(C) Enveloped, segmented, single-stranded
RNA 7. A 2-year-old boy presents to the emergency de-
(D) Non-enveloped, nonsegmented, double- partment because of sore throat, fever, hoarse-
stranded RNA ness, and stridor. The physician suspects a
(E) Non-enveloped, nonsegmented, single- diagnosis of croup, but wishes to exclude epi-
stranded RNA glottitis. Compared with croup, which of the
following is characteristic of epiglottitis?
5. A newborn baby is cyanotic and is having great
(A) Epiglottitis is associated with inflammation
difficulty breathing. The baby is tachypneic
of the larynx and subglottic trachea

High-Yield Systems
and does not improve with time. At autopsy
(B) Epiglottitis is associated with rhinorrhea
a few days later, the lungs are wet and heavy
and conjunctivitis
with areas of atelectasis alternating with oc-
(C) Epiglottitis often leads to respiratory dis-
casional dilated alveoli or alveolar ducts.
tress
Intra-­
alveolar hyaline membranes consisting
(D) Symptom onset is gradual in epiglottitis
of fibrin and cellular debris are also present.
(E) The barking cough of epiglottitis becomes
A patent ductus arteriosus and intraventricu-
inspiratory stridor
lar brain hemorrhage are also seen at autopsy.
(F) Throat swab in epiglottitis would reveal
Which of the following is the likely etiology of
parainfluenza virus
the baby’s condition?
(A) Deficiency of hepatic glucuronyl transfer- 8. A 25-year-old man presents with new-onset
ase hemoptysis for the past 12 hours. The patient,
(B) Dipalmitoyl phosphatidylcholine defi- who recently immigrated to the United States
ciency from Vietnam, has had fever and night sweats
(C) Full-term uncomplicated pregnancy on a daily basis for the past four years. He has
(D) Lecithin:sphingomyelin ratio in amniotic no other complaints and no past medical his-
fluid >1.5 tory. The patient is diagnosed with active tu-

Respiratory
(E) Maternal steroid abuse prior to delivery berculosis and sent home on an antimycobac-
terial regimen. One month later, the patient
6. A 25-year-old medical student presents to the returns with new complaints of joint pain,
clinic with a nonproductive cough, low-grade photosensitivity, and a facial rash. Liver func-
fever, and malaise of three weeks’ duration. A tion tests are found to be elevated. Which of
few friends in his study group have been feel- the following is the mechanism of action of the
ing the same way. Sputum cultures are nega- drug with the adverse effects described above?
tive. The patient denies exposure to farm ani-
(A) Disrupts the cell membrane’s osmotic
mals, travel, or HIV. The physician treats for
properties
an atypical pneumonia. Which of the follow-
(B) Inhibits arabinosyl transferases
ing methods could help identify the organism
(C) Inhibits DNA-dependent RNA polymerase
responsible for this most likely causative patho-
encoded by the rpo gene
gen?
(D) Inhibits folic acid synthesis
(A) Acid-fast stain (E) Inhibits synthesis of mycolic acids
(B) Cold agglutinin testing
454 Section II: Organ Systems  •  Questions

9. A 26-year-old man presents to the emergency healthy. His vaccinations are up-to-date, and
department with respiratory difficulty. Starting he takes no medications. On chest x-ray, which
at age of 15, he began to have periodic short- portion of the lung is most likely to appear ab-
ness of breath. Serial pulmonary function tests normal?
revealed gradually increasing total capacity
(A) Left lower lobe
and residual volume. His renal function labo-
(B) Left upper lobe
ratory results were normal, but his aspartate
(C) Lingula
aminotransferase and alanine aminotransferase
(D) Right lower lobe
levels were significantly elevated. He began re-
(E) Right upper lobe
quiring home oxygen earlier this year, but his
condition continued to worsen. He died during 11. A 57-year-old man presents to his primary care
this admission, and an autopsy was conducted. physician for a routine wellness check. He
High-Yield Systems

Examination of his lungs showed the pathol- denies any complaints. Social history is sig-
ogy seen in the image. This patient’s illness nificant for a 50-pack-year smoking history. On
was most likely caused by which of the follow- physical examination, his vital signs are within
ing? normal limits except for his blood pressure,
which is 170/95 mm Hg. On his previous visit,
his blood pressure was 155/90 mm Hg. Which
of the following antihypertensive agents is rela-
tively contraindicated in this patient?
(A) Acebutolol
(B) Atenolol
(C) Esmolol
(D) Metoprolol
(E) Nadolol

12. A physiologist divides the lung into three


zones, with the apex being zone 1, the mid-
dle region being zone 2, and the base being
Respiratory

Reproduced, with permission, from USMLERx.com. zone 3. In an experiment the physiologist ap-
plies a small amount of positive pressure ven-
tilation while studying blood flow in different
(A) Decreased levels of a1-antitrypsin lung zones. Which of the following will be
(B) Decreased levels of elastase noted in this experiment assuming the subject
(C) Increased levels of anti-neutrophil cyto- is standing?
plasmic autoantibodies
(A) Blood flow in zone 3 will be driven by the
(D) Increased levels of copper
difference between alveolar and venous
(E) Increased levels of iron
pressures
10. A 3-year-old boy is brought to the hospital with (B) Blood flow will be reduced in zone 1
acute shortness of breath. He was playing with (C) Blood flow will increase in zone 2
marbles in the playground when his mother (D) Regional differences between blood flow
noticed him cough and become acutely short will not be as great as differences in venti-
of breath. Her attempt to dislodge the object lation
on site was unsuccessful, and he was brought (E) Whether supine or standing, blood flow
to the hospital. Prior to this incident he was will remain uneven throughout the lung
zones
Chapter 17: Respiratory  •  Questions 455

13. A 60-year-old white man comes to his physi- (A) Pseudostratified columnar
cian because of a productive cough of a few (B) Simple squamous
months’ duration. The patient reports having (C) Stratified columnar
three of these episodes over the past two years, (D) Stratified squamous
with each episode lasting approximately four (E) Transitional
months. Histological examination of the lung
reveals hypertrophy of mucus-secreting glands 15. The oxygen-hemoglobin dissociation curve
in the bronchioles, with a Reid index >50%. represents the percent saturation of hemoglo-
What other changes are likely to be noted on bin with oxygen as a function of the partial
biopsy? pressure of oxygen in the blood. This curve
is sigmoidal in shape due to the change in af-
(A) Alveolar fluid and hyaline membranes
finity of heme groups for additional oxygen
(B) An infiltration of eosinophils and CD4+

High-Yield Systems
molecules. Which of the following in an adult
and TH2 lymphocytes
would cause a shift in the curve so that it re-
(C) An infiltration of monocytes and CD8+
sembles that of a neonate?
lymphocytes
(D) Necrotizing vasculitis and granulomatous (A) Increased 2,3-diphosphoglycerate
eosinophilic tissue infiltration (B) Increased partial pressure of carbon diox-
(E) Noncaseating granulomas ide in the blood
(C) Increased pH
14. A 65-year-old man with an 80-pack-year history (D) Increased temperature
of smoking presents to his physician because of (E) Intense exercise
a cough and increasing dyspnea over the past
six weeks. X-ray of the chest shows a 2-cm mass 16. A 50-year-old woman complains of dark-
in the left lower lobe of the lung, which is bi- colored urine and says she has not been feel-
opsied, revealing squamous cell carcinoma. A ing well for the past two-three weeks; she has
sample of non-neoplastic tissue from the lung generalized malaise and a nagging cough that
biopsy is shown in the image. Which of the occasionally is productive of blood-tinged
following types of epithelium not normally sputum. However, she noticed changes in
present in the lung lines the bronchus shown her urine for the first time today. Physical ex-

Respiratory
in this image? amination reveals an ill-appearing middle-aged
woman with a blood pressure of 180/110 mm
Hg. Diminished air entry in the lungs bilater-
ally, and an ulcerated lesion of the mucosa of
the right naris, are noted. There is no history
of asthma or allergies. Urinalysis is grossly posi-
tive for blood, and serum chemistry panel re-
veals a creatinine level of 1.7 mg/dL. What ad-
ditional finding would confirm the most likely
diagnosis?
(A) Eosinophilia on WBC differential
(B) IgA deposition in glomerular mesangium
(C) Linear IgG deposition in the kidney
(D) Positive for cytoplasmic anti-neutrophilic
cytoplasmic antibodies
(E) Positive for hepatitis B
Courtesy of Wikipedia.
456 Section II: Organ Systems  •  Questions

17. A 35-year-old African-American woman pre­ 19. A patient undergoing lung transplantation be-
sents to the clinic complaining of fatigue, cause of pulmonary fibrosis had his pressure-
dry cough, and dyspnea. X-ray of the chest is volume curves monitored throughout the
shown in the image. Lung parenchymal biopsy operation. The following events took place
reveals a noncaseating granuloma. Which of in the operating room: (1) his right lung was
the following cutaneous manifestations is asso- resected, (2) the new right lung was trans-
ciated with this condition? planted, and (3) positive end-expiratory pres-
sure (PEEP) was added to prevent pulmonary
edema. Which pressure-volume loop most
likely represents the patient’s pulmonary func-
tion when the patient was being ventilated on
one lung prior to the new lung being trans-
High-Yield Systems

planted?

D
A

600

Volume (mL)
C
B

Reproduced, with permission, from USMLERx.com.

(A) Erythema infectiosum 0 20


Pulmonary artery wedge pressure (cm H2O)
(B) Erythema migrans
(C) Erythema multiforme Reproduced, with permission, from USMLERx.com.
(D) Erythema nodosum
(E) Erythema toxicum
Respiratory

(A) Loop A
18. An 18-year-old man comes to the physician (B) Loop B
complaining of a runny nose, sneezing, and (C) Loop C
difficulty breathing for the past two days. He (D) Loop D
says that when he goes outside or is away from
his house, his symptoms improve. On physical 20. A 5-year-old girl visiting from Mexico is
examination his turbinates are boggy and viola- brought to the emergency department by her
ceous. This type of reaction is most similar to aunt because of a sore throat and general mal-
which of the following? aise for the past three days. Physical examina-
tion reveals temperature of 38°C (100.4°F)
(A) Anaphylaxis
and a grayish-white membrane on the pharynx
(B) Contact dermatitis
that bleeds on attempted dislodgement. Which
(C) Goodpasture syndrome
of the following is the most appropriate culture
(D) Graft-versus-host disease
media for diagnosing this patient’s infection?
(E) Post-streptococcal glomerulonephritis
(A) Bordet-Gengou agar
(B) Chocolate agar with factors V and X
(C) Sabouraud agar
(D) Tellurite agar
(E) Thayer-Martin agar
Chapter 17: Respiratory  •  Questions 457

21. A 57-year-old man presents to the emergency (D)


Pneumocystis jiroveci
department (ED) with fever, night sweats, and (E)
Toxoplasma gondii
a productive cough with occasional hemopty-
sis. He is started empirically on several medi- 23. A pregnant woman who suffers from hyperten-
cations for his underlying disease. At follow-up sion and thrombocytopenia and has elevated
several months later, he reports difficulty read- liver function tests suddenly has a seizure.
ing the paper in the morning and has been Suspecting eclampsia, the obstetrics team per-
found to wear unusual color combinations at forms an emergency cesarean section. The
work. Which of the following is the most likely neonate is delivered at 30 weeks and is found
cause of this patient’s new symptoms? to have bradycardia. The child also appears to
have labored breathing. He is rushed to the
(A) Ethambutol toxicity
neonatal intensive care unit, where he is intu-
(B) Isoniazid toxicity

High-Yield Systems
bated and treated with steroids. Which of the
(C) Pyrazinamide toxicity
following is a characteristic of the alveoli of
(D) Rifampin toxicity
this neonate’s lungs?
(E) Tuberculous eye infection
(A) An increased pressure is needed to col-
22. A 36-year-old woman with a history of leuke- lapse the lungs
mia receives a bone marrow transplant. Two (B) Increased surface tension
and a half weeks later, she experiences fever, (C) The lungs have increased compliance
cough, and dyspnea. Bronchoalveolar lavage (D) The lungs lack a substance produced
reveals large cells with prominent intranuclear mainly by type I pneumocytes
inclusions, as shown in the image. What is the (E) The lungs produce a substance with a
most likely cause of this patient’s infection? lecithin:sphingomyelin ratio >1.5

24. A 12-year-old boy is found unconscious in his


bedroom by his parents and is taken to the
emergency department. On arrival the pa-
tient’s skin is pale and lacks turgor, and there is
a sweet scent on his breath. His parents report

Respiratory
constant urination and weight loss in the two
weeks prior to presentation. Laboratory tests
show a glucose level of 610 mg/dL, sodium
of 130 mEq/L, bicarbonate of 9 mEq/L, and
chloride of 95 mEq/L. Which of the following
would most likely be associated with this pa-
tient’s condition at presentation?
(A) Calcium oxalate crystals in the urine
(B) Decreased anion gap
Reproduced, with permission, from USMLERx.com. (C) Decreased blood partial pressure of carbon
dioxide
(D) Elevated blood partial pressure of carbon
(A) Candida albicans
dioxide
(B) Cytomegalovirus
(E) Hypokalemia
(C) Mycobacterium tuberculosis
458 Section II: Organ Systems  •  Questions

25. A medical student is asked to perform a car- 26. A 46-year-old woman presents to the emer-
diovascular examination on a patient. After 10 gency department because of a one-week his-
minutes of auscultation with no success, the tory of worsening nausea and lethargy. While
medical student gives up and asks his resident she is waiting to see the doctor she experiences
for help. The resident puts up the patient’s an- a seizure. Her past medical history is signifi-
teroposterior chest radiograph (see image) and cant for tuberculosis. Laboratory values show:
begins to explain the molecular etiology of this
Serum Na+: 109 mEq/L
patient’s condition. What is one respiratory
Serum osmolality: 255 mOsm/kg
complication that this patient is at increased
Urine osmolality: 850 mOsm/kg
risk of developing as a result of his condition?
Hematocrit: 27%
Which of the following drugs is also known to
High-Yield Systems

cause the underlying disorder with which this


patient presents?
(A) Cyclophosphamide
(B) Demeclocycline
(C) Hydrochlorothiazide
(D) Indomethacin
(E) Lithium

27. A 17-year-old girl involved in a car accident


presents to the emergency department with
penetrating chest trauma to her left side. She
is having difficulty breathing and has an oxy-
gen saturation of 86%. After x-ray of the chest
Reproduced, with permission, from Knoop KJ, et al. The Atlas is performed, a chest tube is placed, and her
of Emergency Medicine, 3rd ed. New York: McGraw-Hill, oxygen saturation improves. Which of the fol-
2010; Fig. 7.31.
lowing is responsible for her difficulty breath-
ing upon presentation?
Respiratory

(A) Acute respiratory distress syndrome (A) Her intrapleural pressure is equal to atmo-
(B) Bronchiectasis sphere pressure during inspiration
(C) Carcinoma of the lung (B) Her intrapleural pressure is less than atmo-
(D) Exercise-induced asthma spheric pressure during inspiration
(E) Idiopathic pulmonary fibrosis (C) Pain from the trauma has made it difficult
to breathe
(D) Pressure within the pericardial space is in-
creased relative to the pleural space
(E) The elastic force of the chest wall is pull-
ing it inward
Chapter 17: Respiratory  •  Questions 459

28. A 74-year-old retired shipyard laborer with a (A) Autoimmune attack of lung parenchyma
45-pack-year smoking history and previous (B) Idiopathic (unknown) etiology
work in sandblasting and fiberglass operations (C) Living in a polluted city for years
presents with increasing shortness of breath (D) Long-term complication of steroid abuse
and peripheral edema. On physical exami- (E) Reactivation of a contained primary dis-
nation he is a thin, cyanotic man in moder- ease
ate pulmonary distress. His chest shows an (F) Working in a coal mine for 40 years
increased anteroposterior diameter, and the (G) Working in a shipyard for 40 years
breath sounds are faint with a prolonged expi-
ration. The liver edge is 3 cm below the right 30. A 158.8-kg (350-lb) man with a body mass
costal margin. There is no digital clubbing, index of 40 kg/m² comes to the physician
but marked peripheral edema is present. Arte- complaining of frequent fatigue, shortness of

High-Yield Systems
rial blood gas analysis reveals a partial oxygen breath, general sleepiness, and an inability to
pressure of 43 mm Hg, a partial carbon diox- concentrate. Physical examination shows an
ide pressure of 22 mm Hg, and a pH of 7.51. extremely obese, tired-looking man with hyper-
Which set of laboratory parameters is most tension and an elongated uvula. Which of the
likely to be found? following metabolic findings is most likely?
(A) Decreased serum glucose
Choice
Total (B) Increased HDL cholesterol
FEV1 FVC FEV1:FVC lung
capacity (C) Increased renal H+ reabsorption
A
(D) Increased renal HCO3- reabsorption
B (E) Increased renal HCO3- secretion
C normal

D normal normal 31. A 68-year-old man who smokes and is alco-


E holic abruptly develops high fever, shakes, a
severe headache, and abdominal and mus-
Reproduced, with permission, from USMLERx.com. cle pain. He initially has a dry, insignificant
cough, but over the next few days he develops
marked shortness of breath requiring assisted

Respiratory
(A) A ventilation. X-ray of the chest demonstrates ho-
(B) B mogenous radiographic shadowing involving
(C) C both the lungs extensively. Culture of bron-
(D) D choalveolar lavage fluid on buffered charcoal
(E) E yeast extract demonstrates a coccobacillary
pathogen. What is the most likely causative or-
29. A 74-year-old patient presents with increased
ganism?
shortness of breath. A sputum sample reveals
golden-brown beaded fibers, which result (A)
Legionella pneumophila
from iron- and protein-coated fibers. On CT (B)
Listeria monocytogenes
scan, dense fibrocalcific plaques of the parietal (C)
Spirillum minus
pleura are seen. A particular pneumoconio- (D)
Staphylococcus aureus
sis is suspected. Which of the following is the (E)
Streptococcus pneumoniae
likely etiology of the patient’s condition?
460 Section II: Organ Systems  •  Questions

32. A 62-year-old woman underwent bilateral knee (B) Inhibition of phosphodiesterase resulting
replacement and was discharged without com- in increased cAMP levels
plications on postoperative day two. Nine days (C) Inhibition of the degranulation of mast
after surgery she develops severe respiratory cells
distress and dies suddenly in the emergency (D) Inhibition of the synthesis of cytokines
department. Postmortem examination of her (E) Stimulation of adenylyl cyclase resulting in
pulmonary artery reveals the pathology seen increased cAMP levels
in the image. What medical condition could
predispose to a similar pathology as observed in 34. In an attempt to better understand the patho-
this patient? physiology of obesity hypoventilation syn-
drome, a medical student is reviewing the ways
in which the body exerts control of respiration.
High-Yield Systems

Which of the following is true with regard to


regulation of respiration?
(A) Breathing is centrally regulated via the hy-
pothalamus and amygdala
(B) Central chemoreceptors affect breathing
by directly detecting blood levels of hydro-
gen ions
(C) Peripheral chemoreceptors stimulate
breathing when the partial pressure of oxy-
gen dips below 60 mm Hg
(D) Stretch, irritant, and J receptors all func-
tion outside the lung to regulate breathing
(E) The aortic and carotid bodies are consid-
Reproduced, with permission, from USMLERx.com. ered central chemoreceptors

35. A 56-year-old man presents with fatigue, fever,


(A) Aspiration
weight loss, and hemoptysis of five weeks’ du-
(B) Factor V deficiency
Respiratory

ration. Radiography and CT of the chest reveal


(C) Factor VIII deficiency
a centrally located mass. Results of a lung bi-
(D) Protein C deficiency
opsy are shown in the image. Laboratory tests
(E) Thrombocytopenia
show:
33. A 55-year-old man with a 30-pack-year smok- Sodium: 130 mEq/L
ing history presents to his physician because of Potassium: 3.9 mEq/L
a 3-month history of productive cough. He is Chloride: 101 mEq/L
diagnosed with chronic obstructive pulmonary Bicarbonate: 24 mEq/L
disease after x-ray of the chest demonstrates hy- Calcium: 9.8 mg/dL
perinflated lungs and a flattened diaphragm. WBC count: 11,600/mm³
The physician prescribes inhaled steroids, a β2- Hemoglobin: 12 g/dL
agonist, and ipratropium bromide. Ipratropium Hematocrit: 38.1%
bromide will produce bronchodilation through Platelet count: 420,000/mm³
which of the following mechanisms? Blood urea nitrogen: 8 mg/dL
Creatinine: 0.8 mg/dL
(A) Blockade of acetylcholine at muscarinic
Glucose: 108 mg/dL
receptors
What is the reasoning behind the best manage-
ment for this patient?
Chapter 17: Respiratory  •  Questions 461

37. Public health investigators are looking into a


series of illnesses that have occurred in a small
community. Many patients presented with
acute-onset hyperpyrexia and a particularly se-
vere pneumonia. Gram staining of their spu-
tum cultures reveals neutrophils and very few
organisms. Which of the following organisms
is most likely to have caused this outbreak?
(A)
Bordetella pertussis
(B)
Haemophilus influenzae type B
(C)
Legionella pneumophila
(D)
Mycobacterium tuberculosis

High-Yield Systems
(E)
Streptococcus pneumoniae
Reproduced, with permission, from USMLERx.com.
38. A 26-year-old recent immigrant from Mexico
presents to the emergency department with a
(A) Surgery carries a risk of provoking para- three-week history of fevers accompanied by
neoplastic syndromes on removal of the night sweats and chills, weight loss of 2.3 kg
mass (5 lb), and cough that is often productive of
(B) Surgery has not been shown to improve blood-tinged sputum. Bronchoalveolar lavage
survival is performed and an acid-fast stain of the sam-
(C) Surgery is palliative but not curative ple reveals the organism shown in the image.
(D) Surgery often is curative in every lung can- Which of the following should be included in
cer without identifiable metastases this patient’s therapy to prevent a common tox-
(E) This lesion is likely to regress in seven icity of treatment?
years

36. A 61-year-old man is frustrated because he


is no longer able to walk up a flight of stairs

Respiratory
without stopping to catch his breath. He has
also been plagued by a dry cough for the past
six months. He has not visited his primary
care physician because he is not a smoker and
does not believe that he could have a serious
pulmonary condition. He ignores his symp-
toms for another eight months, during which
time they continue to worsen. He finally visits
his physician at the urging of his wife. While
shaking hands, his physician notices that the
patient has clubbing of the fingers. A clini- Courtesy of Dr. George P. Kubica, Centers for Disease Con-
cal work-up and medical history fail to find a trol and Prevention.
cause for this restrictive lung disease. What is
the definitive therapy for this patient’s most
(A) Cobalamin
likely condition?
(B) Pyridoxine
(A) Albuterol (C) Vitamin B1
(B) Azathioprine (D) Vitamin C
(C) Cyclosporine (E) Vitamin E
(D) Lung transplantation
(E) Steroids
462 Section II: Organ Systems  •  Questions

39. A mother brings her 10-year-old son with fever, 6.0


cough, and difficulty breathing to the emer- Volume
gency department. Approximately two days A
(L)
ago she noted the development of a rash on E G
2.7
his face that spread downward over his body. B
2.2
Physical examination reveals a toxic-appearing
C
child with a temperature of 40°C (104°F), F
1.2
rapid pulse, and rapid respiratory rate. The
D
physician notes the appearance of a reddish-
0
brown blotchy rash throughout the child’s
body. In his mouth he has small red spots with Reproduced, with permission, from USMLERx.com.
blue-white centers. Chest examination reveals
High-Yield Systems

clear breath sounds with poor inspiratory ef-


fort. CT of the chest shows diffuse interstitial (A) A
involvement. Which of the following would (B) B
the physician most likely see in this child’s spu- (C) C
tum? (D) D
(E) E
(A) Acid-fast bacilli
(F) F
(B) Cells with nuclei surrounded by halo and
(G) G
clear cytoplasm
(C) Cowdry-type inclusions in cells 42. A 63-year-old smoker visits his primary care
(D) Gram-negative coccobacilli and polymor- physician because of recent weight gain and
phonuclear leukocytes worsening coughs. On physical examination
(E) Gram-positive diplococci and polymorpho- the physician notes that the patient’s extremi-
nuclear leukocytes ties are thinner than before, while his waist is
(F) Multinucleated giant cells increased in girth. The patient also has a pad
of adipose tissue at the base of his neck and
40. A premature child is born in respiratory dis-
purple striae on his abdomen. The physician
tress and is emergently intubated. Synthetic
decides to run some blood tests and obtain an
Respiratory

pulmonary surfactant is administered with no


x-ray of the chest, which shows a central le-
improvement in pulmonary function. After
sion. Which of the following is the most likely
conducting a detailed physical examination,
diagnosis?
the pediatrician believes that this child’s con-
dition is related to a failure of the embryonic (A) Adenocarcinoma
pleuroperitoneal folds to form and close. What (B) Bronchial carcinoid
physical examination finding would support (C) Metastatic disease affecting the lung
the most likely diagnosis in this child? (D) Small cell carcinoma
(E) Squamous cell carcinoma embolism
(A) Bowel sounds in the left lower lung zone
(B) Continuous cardiac murmur 43. A 37-year-old man is brought to the emergency
(C) Marked splenomegaly department after being stabbed superior to his
(D) Midline deviation of the trachea right nipple with a knife. His blood pressure is
100/60 mm Hg, heart rate is 126/min, respira-
41. Which of the following letter choices repre-
tory rate is 26/min, and oxygen saturation is
sents the residual volume?
90% on 100% oxygen facemask. The wound
Chapter 17: Respiratory  •  Questions 463

is bubbling, and the skin immediately around (A) Etanercept


the wound is moving in and out with respira- (B) Methotrexate
tions. Which of the following will most likely (C) Nonsteroidal anti-inflamatory drugs
be found on the right side during x-ray of this (D) Risedronate
patient’s chest? (E) Sulfasalazine
(A) Hemothorax
45. A 56-year-old man presents to the emergency
(B) Ninth rib fracture
department because of cough, dyspnea, and
(C) Pleural effusion
hemoptysis. X-ray of the chest shows dilation
(D) Pneumothorax
of his airways, which is confirmed by bron-
(E) Upper lobe consolidation
choscopy. Which of the following conditions is
most likely responsible for the dilation of this
44. A 54-year-old woman complains about a per-

High-Yield Systems
patient’s airways?
sistent cough she has had for the past three
months. The cough has been bothering her (A) Adult respiratory distress syndrome
a lot and making her anxious. She thinks the (B) Asthma
anxiety is why she has lost some weight re- (C) Atelectasis
cently. She also blames the anxiety whenever (D) Bronchiectasis
she wakes up in the middle of the night and (E) Churg-Strauss syndrome
finds herself drenched in sweat. Further his-
tory reveals she has rheumatoid arthritis (RA), 46. A 15-year-old boy with a history of severe
but her joint pains and swellings are well con- asthma presents to the emergency department
trolled by medications her rheumatologist has in obvious respiratory distress. After multiple
prescribed her. Following a physical examina- nebulizer treatments and doses of intravenous
tion, the physician orders an x-ray of the chest corticosteroids, he develops nausea, vomiting,
(see image). Based on the results, the physi- and weakness. Studies reveal a potassium level
cian immediately prescribes an antibiotic regi- of 2.6 mEq/L and U waves on ECG. Which of
men and asks her to discontinue one of the the following medications most likely would
drugs used to treat her RA. Which of the fol- have elicited these symptoms?
lowing drugs increased her risk of developing (A) Albuterol

Respiratory
the disease shown on the radiograph? (B) Cromolyn
(C) Ipratropium
(D) Theophylline
(E) Zileuton

Courtesy of the Centers for Disease Control and Prevention.


464 Section II: Organ Systems  •  Questions

47. A 32-year-old African-American woman pre­ (B) Gram-positive rods in chains with a pro-
sents to her physician complaining of a cough tein capsule
for the past two months and increased short- (C) Gram-negative pleomorphic coccobacilli
ness of breath over the past year. After com- requiring cysteine for growth
pleting a full physical examination, her physi- (D) Gram-positive weakly acid-fast rods form-
cian orders an x-ray of the chest, which shows ing long branching filaments
enlarged hilar nodes bilaterally as well as lung (E) Poorly stained gram-negative rods that
nodules. Results of a lung biopsy are shown in stain best on silver stain and require iron
the image. Which of the following treatments and cysteine.
would be most appropriate for this patient?
49. A 64-year-old man with sepsis has an increased
peripheral metabolic rate and hypercapnia.
High-Yield Systems

With regard to carbon dioxide transport in this


patient, which of the following physiologic
processes is taking place?
(A) Bicarbonate travels in the RBCs until it
reaches the lung, where it gets exhaled as
carbon dioxide
(B) Carbaminohemoglobin (carbon dioxide
bound to hemoglobin) becomes the pri-
mary transport carrier of the additional car-
bon dioxide
(C) More chloride enters the RBCs peripher-
ally to compensate for increased carbonic
Reproduced, with permission, from USMLERx.com.
anhydrase activity
(D) The acidic environment of the lungs shifts
the hemoglobin-oxygen curve to the right
(A) Cisplatin and causes the release of carbon dioxide
(B) Cyclophosphamide from hemoglobin
Respiratory

(C) Dexamethasone (E) The additional dissolved carbon dioxide


(D) Hydroxychloroquine becomes converted to bicarbonate, which
(E) Rifampin binds to hemoglobin

48. A 32-year-old man returns from an in-depth 50. A 65-year-old man with a 110-pack-year his-
tour of a sheep and goat farm. Five days later tory of smoking presents to his primary care
he develops fever, malaise, a dry cough, and physician because of shortness of breath, dys-
pressure in his chest. These symptoms re- pnea on exertion, and cough for three months’
solve after a few days. He then develops high duration. X-ray of the chest reveals flattened
fever, severe shortness of breath, chest pain, diaphragms bilaterally. The doctor orders pul-
cyanosis, and diaphoresis and is rushed to the monary function tests to evaluate the patient.
emergency department, where work-up reveals Which of the following pulmonary function
hemorrhagic mediastinitis, bloody pleural effu- test results would most likely be found in this
sions, and mediastinal widening on x-ray of the patient?
chest. Within a few hours the patient develops (A) Decreased FEV1:FVC
septic shock and dies. Which of the following (B) Decreased functional residual capacity
characterizes the most likely causative organ- (C) Decreased total lung capacity
ism? (D) Increased FEV1
(A) Gram-negative pleomorphic aerobic coc- (E) Increased FEV1:FVC
cobacilli
Chapter 17: Respiratory  •  Answers 465

An s w e r s

1. The correct answer is A. The drug theophyl- is treated by inserting a chest tube to remove
line is a phosphodiesterase inhibitor that leads the air from the pleural space.
to the decreased hydrolysis of cAMP to ad-
Answer A is incorrect. Aortic dissection, not a
enosine monophosphate. An overdose of the-
pneumothorax, would show up on x-ray of the
ophylline will therefore result in an elevated
chest as a widened mediastinum. Aortic dissec-
intracellular level of cAMP. b-blockers such as
tion can occur in trauma, or a dissecting aor-
metoprolol may therefore be given to reduce
tic aneurysm can occur in connective tissue
cAMP levels through inactivation of adenyl-
diseases such as Marfan disease. The physical
ate cyclase. A cardioselective b-blocker must
findings are not consistent with aortic dissec-
be used in cases of asthma to avoid inducing

High-Yield Systems
tion: first, the chest would not be hyperreso-
bronchial hyperreactivity.
nant to percussion and second, the blood pres-
Answer B is incorrect. b-agonists such as al­ sure would not be equal in both arms.
buterol would potentiate the effects of theoph-
Answer B is incorrect. Barrel chests and flat-
ylline by activating adenylate cyclase and in-
tened diaphragms are seen in patients with
creasing the conversion of ATP to cAMP.
obstructive lung diseases such as chronic em-
Answer C is incorrect. Epinephrine would physema. Although emphysema is caused
potentiate the effects of theophylline by acti- by smoking, this patient does not have a long
vating adenylate cyclase and increasing cAMP enough smoking history. It is possible that this
formation. man could have emphysema due to an a1-­
antitrypsin deficiency, but in that case it would
Answer D is incorrect. Furosemide is a di-
not present this acutely.
uretic unrelated to theophylline overdose.
It works by inhibiting the Na+-K+-2Cl- co- Answer C is incorrect. Consolidation of the
transport system of the thick ascending loop of right lower lobe on x-ray of the chest usually
Henle. means pneumonia. The patient would present
with fever, productive cough, and a high WBC
Answer E is incorrect. Digoxin inhibits the

Respiratory
count. Physical exam would show decreased
Na+-K+-ATPase pump. It is used to increase
resonance on the affected side and increased
myocardial contractility in patients with con-
fremitus.
gestive heart failure. It is unrelated to theoph-
ylline overdose. Answer D is incorrect. In tension pneumotho-
rax, a flap-like pleural tear allows air to enter
2. The correct answer is E. This patient is most into the pleural cavity, but prevents its exit. It
likely suffering from a spontaneous pneumo- can be caused by penetrating trauma to the
thorax. Caused by the rupture of a small apical chest resulting in increased pleural cavity pres-
bleb on the surface of the lung, spontaneous sure. Clinical findings include sudden onset
pneumothoraces typically present in tall young of severe dyspnea, tympanitic percussion, and
men. The patient usually has sudden pain and absent breath sounds. There is tracheal devia-
dyspnea. Examination will show decreased tion and mediastinal structure deviation to the
breath sounds and hyperresonance on the af- contralateral side. If tension pneumothorax oc-
fected side. X-ray of the chest shows overexpan- curs on the left side, there would be compres-
sion of the rib cage and an elevated hemidia- sion of venous return to the heart. Treatment
phragm on the affected side. This paradoxical of tension pneumothorax is emergent needle
abdominal motion occurs because of the nega- decompression into the pleural cavity to re-
tive intrathoracic pressure that causes the fa- lieve the pressure.
tigued diaphragm to be pulled into the thorax
on the right side. Spontaneous pneumothorax
466 Section II: Organ Systems  •  Answers

3. The correct answer is A. This vignette sug- therefore, is, “Which viral illness predisposes
gests a patient with Pneumocystis jiroveci to subsequent bacterial pneumonia in an oth-
pneumonia based on the HIV status, physi- erwise healthy individual?” The classic an-
cal examination and x-ray findings, and the swer is influenza. The influenza virus is an
elevated lactate dehydrogenase level. Pro- enveloped, single-stranded RNA virus with a
phylactic therapy for P jiroveci pneumonia segmented genome that permits reassortment
is indicated for an HIV-positive patient with of the genes encoding the hemagglutinin and
a CD4+ T-lymphocyte count <200/μL. The neuraminidase proteins, resulting in the phe-
standard prophylactic therapy for P jiroveci nomenon of antigenic shift. Complications of
pneumonia is trimethoprim-sulfamethoxazole influenza include both viral pneumonia (due
(TMP-SMX). This combination, however, to a spreading of the illness into the lower res­
is contraindicated for patients with a sulfa al- piratory tract) and bacterial pneumonia. The
High-Yield Systems

lergy, because sulfamethoxazole is a sulfa drug. latter is thought to be due largely to the fact
In these cases, the best alternative treatment is that influenza damages the epithelium of the
aerosolized pentamidine upper respiratory tract, compromising its abil-
ity to keep the lower respiratory tract sterile.
Answer B is incorrect. Ciprofloxacin is a fluo-
Streptococcus pneumoniae, Staphylococcus
roquinolone agent used to treat urinary tract
aureus, and Haemophilus influenzae are the
or gastrointestinal (GI) infections caused by
organisms most commonly seen in bacterial
gram-negative organisms. Adverse effects in-
pneumonia secondary to influenza. This chest
clude GI upset and tendinitis.
x-ray shows a consolidation in the right lower
Answer C is incorrect. Fluconazole is an anti- lobe along with a para-pneumonic effusion,
fungal agent used to treat systemic infections. highly suspicious for bacterial pneumonia.
It is also indicated for treatment of hypercorti-
Answer A is incorrect. The patient’s initial
solism found in ACTH-secreting tumors and
symptoms might be seen in a person with
polycystic ovarian syndrome. It inhibits hor-
acute infectious mononucleosis, consistent
mone synthesis.
with an infection with Epstein-Barr virus,
Answer D is incorrect. Terbinafine is an an- which is an enveloped, double-stranded DNA
tifungal agent that blocks ergosterol synthesis virus. However, the symptoms of mononucle-
Respiratory

by inhibiting squalene epoxidase. It is used to osis typically last longer than three days, and
treat dermatophytoses. bacterial pneumonia is not a common compli-
Answer E is incorrect. TMP-SMX is the ini- cation.
tial choice for prophylactic treatment of P jir­ Answer B is incorrect. The patient’s initial
oveci pneumonia unless patients are unable to symptoms are consistent with an upper re-
tolerate its harsh adverse effects, or if they have spiratory infection, which could be caused
a sulfa allergy. The most common adverse ef- by coronavirus, which is an enveloped, non-­
fects are fever, rash, and bone marrow suppres- segmented, single-stranded RNA virus. Infec-
sion. tion with a coronavirus would not be expected
to lead to bacterial pneumonia in a healthy in-
4. The correct answer is C. From the history, dividual.
it appears that this man initially experienced
nonspecific viral symptoms, but there is not Answer D is incorrect. Non-enveloped, non-
enough information to determine which virus segmented, double-stranded RNA viruses in-
he has. What is clear, however, is that his ini- clude reovirus and rotavirus, neither of which
tial symptoms are distinct from what he expe- is a cause of respiratory illness in adults.
riences on relapse. The radiograph shows that Answer E is incorrect. Another cause of upper
he has lobar pneumonia (lower right lobe), respiratory infections in adults is rhinoviruses,
which can be caused by any number of bacte- which are non-enveloped, non-segmented,
rial species. The question that must be asked, single-stranded RNA viruses. Rhinovirus infec-
Chapter 17: Respiratory  •  Answers 467

tions are typically mild and uncomplicated in Answer C is incorrect. Gram stains are used to
healthy individuals, and a secondary bacterial visualize gram-positive or gram-negative bacte-
pneumonia would be atypical. ria. Mycoplasma pneumoniae is not visible on
Gram stain.
5. The correct answer is B. Dipalmitoyl phos-
Answer D is incorrect. India ink stain can be
phatidylcholine is a primary component of
used to visualize mucoid encapsulated yeasts
surfactant, which is deficient in neonatal res­
such as Cryptococcus. This diagnostic tool is
piratory distress syndrome (NRDS). Chronic
not useful in identifying Mycoplasma species
hypoxemia to the fetus can result in congenital
or viruses.
abnormalities such as a patent ductus arterio-
sus and intraventricular brain hemorrhage. Answer E is incorrect. Serum polymerase
chain reaction (PCR) can be used to diagnose
Answer A is incorrect. Deficiency of hepatic

High-Yield Systems
viral illnesses such as HIV. PCR testing of the
glucuronyl transferase occurs in all newborns,
throat and sputum can be used to detect Myco­
because the enzyme is not found at adult lev-
plasma, but this is not commonly performed.
els in neonates. This leads to physiologic jaun-
Cold agglutinin testing is preferred over PCR
dice, which has nothing to do with NRDS, but
for the detection of Mycoplasma.
could lead to kernicterus.
Answer C is incorrect. Predisposing factors for 7. The correct answer is C. Epiglottitis is a medi-
NRDS include prematurity, maternal diabetes cal emergency, and 90% of patients require
mellitus (DM), and birth by cesarean section. surgery to reestablish an airway. At presenta-
tion patients with epiglottitis can have little
Answer D is incorrect. In NRDS, the
or no respiratory compromise, but this can
lecithin:sphingomyelin ratio in the amniotic
progress to life-threatening respiratory distress
fluid is usually <1.5.
within a matter of hours.
Answer E is incorrect. Steroids are given to
Answer A is incorrect. Epiglottitis on x-ray
mothers who will deliver prematurely to try to
film of the neck reveals a “thumbs up” sign (ie,
prevent NRDS. Intratracheal administration of
“thumbprint” on radiograph), which correlates
artificial surfactant to the newborn can also be
with an inflamed epiglottis. Inflammation of
performed.

Respiratory
the larynx and sublgottic trachea is not associ-
6. The correct answer is B. In patients who ated with epiglottitis.
present with insidious onset of dry cough, Answer B is incorrect. Patients with epiglotti-
low-grade fever, headache, myalgias, nausea, tis do not have the symptoms or physical find-
or emesis, an atypical pneumonia should be ings of conjunctivitis or rhinorrhea. These
considered. Atypical pneumonias are mostly findings are more typical of croup. Epiglottitis
caused by Mycoplasma or viruses. Mycoplasma has additional symptoms of drooling and la-
cannot be cultured and is detected by the cold bored breathing.
agglutinin test, which measures the aggluti-
Answer D is incorrect. In general, the onset of
nation of immunoglobulins when they are
symptoms is abrupt with epiglottitis and grad-
cooled. X-ray of the chest is often more impres-
ual with croup.
sive than physical examination findings, and is
characterized by a patchy interstitial pattern. Answer E is incorrect. A typical barking cough
Treatment consists of antibiotic therapy with a is seen with croup, which may eventually lead
macrolide, usually azithromycin, for five days. to inspiratory stridor. Epiglottitis typically pre­
sents with stridor and hoarseness.
Answer A is incorrect. The acid-fast stain is
used to diagnose mycobacterial illness, specifi- Answer F is incorrect. Most of the time, a throat
cally Mycobacterium tuberculosis. swab in epiglottitis will reveal Haemophilus
468 Section II: Organ Systems  •  Answers

influenzae, not parainfluenza. Parainfluenza is Answer C is incorrect. Increased levels of anti-


more often seen in croup. neutrophil cytoplasmic autoantibodies are as-
sociated with certain small-vessel vasculitic
8. The correct answer is E. Of the antimycobac- syndromes, including Wegener granulomato-
terial drugs, only isoniazid (INH) produces the sis. Wegener granulomatosis is characterized
lupus-like syndrome described above. INH by granulomatous inflammation of various
decreases synthesis of mycolic acids. Further- organs resulting in acute renal failure, pulmo-
more, hepatotoxicity is common to many an- nary disease, and other manifestations. We-
tituberculosis drugs (rifampin, pyrazinamide, gener granulomatosis is not associated with
and INH). hepatic disease.
Answer A is incorrect. Disruption of the cell Answer D is incorrect. Excess copper is a sign
membrane’s osmotic properties describes the of Wilson disease, an autosomal-recessive de-
High-Yield Systems

mechanism of action of polymyxins. Polymyx- fect that impairs the transport of copper from
ins are not part of the treatment for TB. the liver into bile for excretion. The subse-
Answer B is incorrect. Ethambutol, not iso- quent accumulation of copper in the liver
niazid, inhibits the arabinosyl transferase-­ causes cirrhosis and leakage of copper into
mediated synthesis of arabinogalactin for the blood, where it damages other organs re-
mycobacterial cell walls. The side effects of sulting in neurologic, hematologic, and renal
ethambutol include dose-dependent visual dis- disease. Wilson disease is also characterized by
turbances, decreased visual acuity, red-green decreased serum ceruloplasmin due to a defect
color blindness, optic neuritis, and retinal in the incorporation of copper into ceruloplas-
damage. min. Wilson disease is not associated with lung
pathology.
Answer C is incorrect. The mechanism of ac-
tion here is that of rifampin, another antimyco- Answer E is incorrect. Excess iron is a sign of
bacterial drug. hereditary hemochromatosis, an autosomal-
recessive disorder characterized by excessive
Answer D is incorrect. Inhibition of folic acid dietary iron absorption due to the impaired
synthesis is the mechanism of action of dap- regulation of iron stores (HFE gene mutation).
sone.
Respiratory

While iron overload can cause cirrhosis in ad-


dition to DM and cardiomyopathy, it is not
9. The correct answer is A. The image shows a associated with lung disease. Diagnostically,
gross fixed lung with focal intrapulmonary hemochromatosis is associated with increased
hemorrhage as a result of pulmonary emphy- levels of transferrin saturation and serum fer-
sema. The combination of early-onset em- ritin.
physema and hepatic cirrhosis suggests ho-
mozygous a1-antitrypsin (AAT) deficiency, an 10. The correct answer is D. The right main bron-
uncommon autosomal-recessive disorder. AAT chus is more vertical and wider than the left,
is normally secreted by the liver and acts to and aspirated particles are more likely to lodge
neutralize elastase activity in the lung. In pa- at the junction of the right inferior and right
tients with AAT deficiency, increased elastase middle bronchi. Because of this, aspiration
activity leads to a loss of elastic fibers and in- pneumonia contracted when an individual is
creased lung compliance, resulting in panaci- in an upright position is most common in the
nar emphysema. Furthermore, the accumu- right lower and middle lobes. On x-ray, the
lation of dysfunctional AAT in hepatocytes right lower lobe may appear collapsed as a re-
results in liver damage and cirrhosis. sult of foreign object aspiration.
Answer B is incorrect. It is the increased lev- Answer A is incorrect. The left main bron-
els of elastase activity resulting from AAT defi- chus is narrower and less vertical than the right
ciency that causes the disease. main bronchus. The right main bronchus is
Chapter 17: Respiratory  •  Answers 469

more vertical and wider than the left, and as- Answer B is incorrect. Atenolol is cardioselec-
pirated particles are more likely to lodge at the tive and can be used in patients with asthma or
junction of the right inferior and right middle other obstructive lung diseases.
bronchi.
Answer C is incorrect. Esmolol is cardioselec-
Answer B is incorrect. The left main bron- tive and can be used in patients with asthma or
chus is narrower and less vertical than the right other obstructive lung diseases. However, it has
main bronchus. The right main bronchus is a very short half-life (about nine minutes), and
more vertical and wider than the left, and as- would not be used for long-term outpatient
pirated particles are more likely to lodge at the management.
junction of the right inferior and right middle
Answer D is incorrect. Metoprolol is cardio­
bronchi.
selective and can be used in patients with

High-Yield Systems
Answer C is incorrect. The lingula is in the asthma or other obstructive lung diseases
left lung, and the left main bronchus is nar-
rower and less vertical than the right main 12. The correct answer is B. In zone 1, alveo-
bronchus. The right main bronchus is more lar pressure is greater than arterial pressure.
vertical and wider than the left, and aspirated There is risk of blood flow obstruction in
particles are more likely to lodge at the junc- zone 1 where ventilation/perfusion (V/Q) is
tion of the right inferior and right middle bron- high (wasted ventilation). In positive pressure
chi. ventilation alveolar pressure increases, which
also increases V/Q and compresses the capil-
Answer E is incorrect. When a person is su-
laries, limiting blood flow.
pine, aspiration pneumonia may affect the up-
per lobes and posterior segments of the lungs, Answer A is incorrect. Normally in zone 3,
since they become the gravity-dependent re- arterial pressure is greatest and alveolar pres-
gions when a person lies flat. sure is weakest. Thus the alveolar pressure is
too weak to impact blood flow, and the differ-
11. The correct answer is E. Nonselective b- ence between arterial and venous pressure de-
blockers are contraindicated in patients with termines blood flow. This will still be the case
lung disease because they can cause broncho- with a small amount of positive pressure venti-

Respiratory
constriction by blocking b2-receptors responsi- lation.
ble for relaxation of bronchial smooth muscle.
Answer C is incorrect. Normally in zone 2, ar-
Nadolol is a nonselective b-blocker and should
terial pressure is greater than alveolar pressure,
not be used in a patient with lung disease.
which is greater than venous pressure.
Other nonselective b-blockers include pro-
pranolol, timolol, and pindolol. Acebutolol, at- Answer D is incorrect. Regional differences
enolol, esmolol, metoprolol, and betaxolol are between blood flow are greater than differ-
cardioselective b1-blockers that should be fa- ences in ventilation, due to the effects of
vored in patients with lung/airway disease. Al- gravity. Positive pressure ventilation will not
though the stem does not specifically state that change this.
this patient has lung disease, smoking causes Answer E is incorrect. Blood flow changes de-
airway hyperreactivity and bronchoconstric- pending on whether a person is lying down or
tion. Adding a nonselective b-blocker could ex- standing up. When supine, blood flow is dis-
aggerate these adverse effects of smoking. tributed evenly throughout the lung because
Answer A is incorrect. Acebutolol is cardio­ the effects of gravity are not present.
selective and can be used in patients with
asthma or other obstructive lung diseases. 13. The correct answer is C. A diagnosis of
chronic bronchitis can be made based on the
patient’s symptoms and biopsy results. Along
with the hypertrophy of mucus-secreting
470 Section II: Organ Systems  •  Answers

glands and goblet cells, one typically sees an squamous epithelium is characterized by a
inflammatory infiltrate with a lymphocytic pre- single sheet of flattened cells lying on a basal
dominance, squamous cell metaplasia, and fi- lamina. It does not play a role in this case.
brosis.
Answer C is incorrect. Stratified columnar
Answer A is incorrect. Alveolar fluid and hya- epithelium is found in only a few places in the
line membranes are characteristic of adult res­ body, namely, the conjunctivae of the eye and
piratory distress syndrome. regions of the male urethra. It is composed of
a low polyhedral to cuboidal deeper layer in
Answer B is incorrect. Although these cells
contact with the basal lamina along with a su-
can be found in chronic bronchitis, they are
perficial layer of columnar cells.
much more typical of asthma.
Answer E is incorrect. The bladder is lined
Answer D is incorrect. Necrotizing vasculitis
High-Yield Systems

by transitional epithelium, not the lung. Tran-


and granulomatous eosinophilic tissue infil-
sitional epithelium is characterized by several
tration is characteristic of Churg-Strauss syn-
layers of cuboidal cells, with the surface layer
drome.
being large and dome-shaped.
Answer E is incorrect. Noncaseating granu-
lomas are typical of pulmonary sarcoid, not 15. The correct answer is C. Neonates have high
chronic bronchitis. concentrations of fetal hemoglobin in their
blood. Fetal hemoglobin has a higher affinity
14. The correct answer is D. In smokers, pseu- for oxygen than adult hemoglobin (to allow fe-
dostratified ciliated columnar epithelium lin- tuses to extract oxygen from mother’s blood),
ing the bronchi can undergo metaplasia and therefore fetal oxygen-hemoglobin dissociation
transform into stratified squamous epithelium. curves are left-shifted. Other physiologic con-
Stratified epithelium is defined as epithelial ditions that cause left shifts (higher hemoglo-
membrane composed of more than one cell bin affinity for oxygen) in the curve include
layer. Stratified squamous epithelium is classi- increased pH (or reduced hydrogen ion con-
fied by the flattened shape of the cells in the centration), decreased temperature, decreased
surface layer. Examples of tissues with strati- 2,3-diphosphoglycerate (2,3-DPG) levels, and
fied squamous epithelium include the skin, decreased arterial carbon dioxide pressure.
Respiratory

mouth, anus, vagina, and esophagus. While it Conversely, during exercise, temperature,
is wrongly believed that stratified epithelium is 2,3-DPG levels, and partial pressure of carbon
the result of the need for additional protection dioxide increase as a result of increased metab-
from the noxious smoke, the metaplasia actu- olism in the skeletal muscle. These would de-
ally results from genetic mutation related to crease hemoglobin affinity for oxygen, facilitat-
the developing cancer. ing unloading of oxygen to the tissue of highest
Answer A is incorrect. Pseudostratified colum- metabolic activity, and hence there would be
nar epithelium is the normal respiratory epi- a right shift of the oxygen-hemoglobin dissocia-
thelium on the right that is undergoing meta- tion curve.
plasia. This type of epithelium only appears Answer A is incorrect. An increase in 2,3-
stratified; however, all cells are in contact with DPG binds to deoxygenated hemoglobin in
basal lamina and only some cells reach the sur- RBCs, allosterically upregulating the ability
face of epithelium. of RBCs to release oxygen. Because it reduces
Answer B is incorrect. Simple squamous epi- RBC affinity for oxygen, increased 2,3-DPG
thelium lines alveoli, the loops of Henle, and right-shifts the oxygen-hemoglobin dissociation
blood vessels. Simple epithelium indicates that curve.
the epithelial membrane is composed of a sin- Answer B is incorrect. An increase in the par-
gle layer of cells, which helps when diffusion tial pressure of carbon dioxide in the blood
is important. Under the microscope, simple right-shift the curve, allowing increased deliv-
Chapter 17: Respiratory  •  Answers 471

ery of oxygen to tissues because of decreased immune response. Like Wegener granuloma­
affinity for oxygen by RBCs. tosis, Goodpasture syndrome is associated
with hemorrhagic pneumonitis and glomeru-
Answer D is incorrect. Increased temperature
lonephritis. However, neither lesions of the
is an indicator of metabolic activity (and there-
nares and sinuses, nor positive c-ANCA titers,
fore increased oxygen demand). At high tem-
are characteristic of this disease. Moreover,
peratures, the curve shifts to the right, indicat-
Goodpasture syndrome more commonly af-
ing a decreased affinity for oxygen by RBCs.
fects young adults.
Answer E is incorrect. The acidosis that re-
Answer E is incorrect. Polyarteritis nodosa is
sults from the production of lactate during in-
an immune complex inflammation occurring
tense exercise causes a shift in the curve to the
in medium-sized vessels. It is associated with
right, reducing the affinity of RBCs for oxygen,
hepatitis B virus in 30% of patients, is associ-

High-Yield Systems
thereby allowing them to deliver more oxygen
ated with lesions of various ages, and can oc-
to tissues.
cur in almost any organ. However, it rarely
16. The correct answer is D. This patient’s con- involves the lung. Patients typically have fever,
stellation of symptoms is most consistent with weight loss, malaise, abdominal pain, melena,
Wegener granulomatosis, with the triad of fo- and hypertension, as well as cutaneous erup-
cal necrotizing vasculitis, necrotizing granu- tions, neurologic dysfunction, and hematuria.
lomas of the upper and/or lower airways, and
17. The correct answer is D. The patient has signs
necrotizing glomerulonephritis. Most patients
and symptoms of sarcoidosis, with classic race,
have positive titers for anti-neutrophil cytoplas-
pathology (noncaseating granuloma), and x-ray
mic antibodies with a cytosolic staining pattern
of the chest revealing prominent bilateral hi-
(c-ANCA). The disease is caused by systemic
lar lymphadenopathy, which is present in
granulomatous inflammation, particularly of
>90% of patients with sarcoidosis. Erythema
small- and medium-sized arteries such as those
nodosum, an inflammatory panniculitis, is
supplying the kidneys and lungs. If not treated
the most common cutaneous manifestation
with immunomodulating drugs, focal glomer-
of sarcoi­dosis, and frequently presents as bilat-
ulonephritis can progress to a crescentic form,
eral tender red bumps on the shins. Additional
with ensuing renal failure.

Respiratory
features of sarcoidosis include hypercalcemia
Answer A is incorrect. Eosinophilia is associ- due to increased activation of vitamin D by ac-
ated with Churg-Strauss syndrome, also known tivated macrophages.
as allergic granulomatous angiitis. Patients of-
Answer A is incorrect. Erythema infectiosum,
ten have asthma and/or allergies. Involvement
or fifth disease, is a common childhood viral
can include the lungs, heart, skin, kidneys, and
infection caused by erythrovirus or parvovi-
nerves. The lack of a history of asthma or aller-
rus B19. It is often associated with bright red
gies argues against Churg-Strauss syndrome.
cheeks in the early stages. It is not seen in sar-
Answer B is incorrect. IgA nephropathy (such coidosis.
as Berger disease), characterized by deposition
Answer B is incorrect. Erythema migrans is a
of IgA in glomerular mesangium, is a highly
characteristic rash that is often seen in patients
variable entity, ranging from asymptomatic
with early-stage Lyme disease. It often presents
hematuria to rapidly progressive glomerulone-
as a “bull’s-eye” lesion. It is not seen in sarcoi­
phritis. IgA nephropathy is limited to the kid-
dosis.
neys.
Answer C is incorrect. Erythema multiforme
Answer C is incorrect. Goodpasture syn-
is a skin condition caused by inflammation of
drome is associated with anti-basement mem-
the microvasculature and mucous membranes.
brane IgG antibodies that recognize an epitope
It is most often precipitated by herpes sim-
on collagen IV. It is a type II hypersensitivity
plex virus infection, mycoplasma infection, or
472 Section II: Organ Systems  •  Answers

adverse drug reactions. It is not seen in sarcoi­ hypersensitivity reaction in which antigen-­
dosis. antibody complexes activate complement. The
complex then causes the release of lysosomal
Answer E is incorrect. Erythema toxicum is
enzymes from neutrophils. Other type III reac-
a harmless rash that appears in approximately
tions include serum sickness, the Arthus-type
half of all newborns. It is not seen in sarcoi­
reaction, and systemic lupus erythematosus
dosis.
(SLE).
18. The correct answer is A. This patient’s symp-
19. The correct answer is B. Loop B is notable for
toms are characteristic of allergic rhinitis,
the decreased volume and increased pressure
likely due to an indoor allergen. This response
of respiration compared to loops A, C, and D.
is an example of a type I hypersensitivity reac-
This loop was recorded while the patient was
tion in which an allergen cross-links antigen-
High-Yield Systems

breathing with only one lung. The decreased


specific IgE on the surface of mast cells and
volume compared to the other loops should
basophils. Anaphylaxis occurs via the same
make you think of a lung that had a large de-
mechanism. Subsequently, the mast cells and
crease in area available for oxygen exchange.
basophils release vasoactive amines such as his-
The lower baseline pressures compared to loop
tamine. Because antibodies are preformed in
D show that PEEP was not being used when
this type of hypersensitivity, the reaction devel-
this tracing was made.
ops quite rapidly.
Answer A is incorrect. Loop A shows the pa-
Answer B is incorrect. Contact dermatitis
tient’s pulmonary function after the com-
is an example of type IV hypersensitivity or
pleted transplant. There is improved volume
delayed-type hypersensitivity. In this reaction,
and compliance (change in volume/change
sensitized T lymphocytes release lymphokines
in pressure) of the pulmonary system over the
in response to antigen. The clinical response
baseline loop C.
occurs several days after antigen exposure.
Other type IV reactions include Guillain-Barré Answer C is incorrect. Loop C is the patient’s
syndrome, graft-versus-host disease, and a posi- baseline prior to transplant. The total lung vol-
tive purified protein derivative (PPD), or tuber- ume and pressures would be expected to be
culin, skin test. higher in a patient after total lung resection.
Respiratory

These diseased lungs have a poorer compli-


Answer C is incorrect. Goodpasture syn­drome
ance than the transplanted lungs and baseline
is an example of a type II hypersensitiv­ity re-
pressures show that PEEP was not being used
action in which antibodies bind directly to
when these tracings were made.
antigen, leading to complement-mediated cell
lysis. Other type II reactions include hemolytic Answer D is incorrect. Loop D reflects the
anemia, idiopathic thrombocytopenia purpura, patient’s condition after transplant and main-
and bullous pemphigoid. tenance on PEEP, as can be seen by the right
shift of the curve on expiration (at low lung
Answer D is incorrect. Graft-versus-host dis-
volumes) compared to the other three curves.
ease is an example of type IV hypersensitiv-
The application of PEEP increases the volume
ity or delayed-type hypersensitivity. In this
of the alveoli and helps open collapsed alveoli,
reaction, sensitized T lymphocytes release
increasing the portion of ventilated lung and
lymphokines in response to antigen. The clini-
improving the V/Q ratio.
cal response occurs several days after antigen
exposure. Other type IV reactions include
20. The correct answer is D. This patient likely
Guillain-Barré syndrome, contact dermatitis,
has diphtheria, an infection caused by the
and a positive PPD, or tuberculin, skin test.
gram-positive rod Corynebacterium diphthe­
Answer E is incorrect. Poststreptococcal glo- riae. Diphtheria classically presents with a
merulonephritis is an example of a type III grayish-white pseudomembrane on the phar-
Chapter 17: Respiratory  •  Answers 473

ynx or tonsils; this pseudomembrane should ditional coverage of pyrazinamide during the
not be disrupted in order to avoid increased first two months.
absorption of the lethal exotoxin. Fever is usu-
Answer B is incorrect. Although rifampin is
ally mild or absent. It is seen very rarely in vac-
considered the best antituberculous agent, iso-
cinated populations but is endemic to certain
niazid is used for prophylaxis in asymptomatic
parts of the world. Culture of C diphtheriae
patients with a positive PPD test. A six-month
requires tellurite agar (Loeffler medium) to
course of isoniazid prevents activation of latent
prevent growth of normal upper respiratory
TB in 90% of patients for at least 20 years. Iso-
tract flora. Colonies will become gray to black
niazid blocks mycolic acid cell-wall synthesis
within 24 hours.
and is bactericidal for rapidly multiplying or-
Answer A is incorrect. Bordet-Gengou agar is ganisms. Major adverse effects include hepato-
used to culture Bordetella pertussis. Pertussis toxicity and peripheral neuropathy, but many

High-Yield Systems
presents with paroxysmal coughing spells and other adverse effects occur, such as lupus-like
whooping sounds on inspiration. syndrome and optic atrophy.
Answer B is incorrect. Chocolate agar is used Answer C is incorrect. Like isoniazid, the
to grow Haemophilus influenzae. Encapsulated spectrum of action of pyrazinamide is limited
strains of H influenzae cause invasive diseases to Mycobacterium tuberculosis. The site of ac-
such as septicemia, meningitis, cellulitis, sep- tivity for pyrazinamide is thought to be a fatty
tic arthritis, epiglottitis, and pneumonia. Non- acid synthase gene. The major adverse effect of
encapsulated strains are likely to cause otitis pyrazinamide therapy is hepatotoxicity, but it is
media, conjunctivitis, bronchitis, and sinusitis. rare at recommended dosages. Another major
adverse effect is hyperuricemia and subsequent
Answer C is incorrect. Sabouraud agar is used
gout.
to grow fungi.
Answer D is incorrect. Rifampin is the most
Answer E is incorrect. Thayer-Martin agar is
potent antituberculous agent available. Rif­
a chocolate agar plate, which has VCN antibi-
ampin blocks DNA-dependent RNA poly-
otics (vancomycin, colistin, and nystatin) that
merase, preventing RNA synthesis. Although
suppress the growth of endogenous flora while
it is a better agent than isoniazid for prevent-
supporting Neisseria gonorrhoeae growth. This

Respiratory
ing active TB infection, it has a significant risk
patient does not have symptoms of gonorrhea.
of liver toxicity that outweighs its benefits as a
21. The correct answer is A. Ethambutol is ac- preventive medicine.
tive against Mycobacterium tuberculosis, and it Answer E is incorrect. Mycobacterium tuber­
is among the first-line agents used to treat tu- culosis can have extrapulmonary manifesta-
berculosis (TB) infection (others are isoniazid, tions, but the eye is not commonly involved.
rifampin, and pyrazinamide). Ethambutol’s Miliary TB infection can affect the eye and
mechanism of action appears to be the inhibi- cause chorioretinitis, uveitis, and conjuncti-
tion of polymerization of cell-wall precursors. vitis, but these manifestations are rare. Color
Although the drug generally is well tolerated, blindness would not be associated with such
its most common adverse effects involve ocu- an infection.
lar toxicity such as loss of visual acuity and red-
green color blindness, which usually appears 22. The correct answer is B. Cytomegalovirus
several months after the initiation of treatment. (CMV) infection is a common complication
Ethambutol usually is used in an anti-TB in immunocompromised patients following
regimen with rifampin for patients who either bone-marrow transplantation. Histopathology
cannot tolerate isoniazid or are infected with shows large cells with intranuclear inclusions
isoniazid-resistant M tuberculosis. For chil- (so-called “owl’s eyes”) typical of CMV infec-
dren, most literature supports a regimen of six tion.
months with isoniazid and rifampin, with ad-
474 Section II: Organ Systems  •  Answers

Answer A is incorrect. Histopathology would anion gap (>10 mEq) as calculated by the fol-
show budding cells with pseudohyphae. lowing formula: Anion gap = Na+ - [HCO3- +
Cl-]. This leads to respiratory compensation
Answer C is incorrect. Histopathology would
by deep respiration (Kussmaul’s respiration),
show a granulomatous reaction, with the pos-
resulting in a decrease in blood partial pres-
sible presence of giant cells.
sure of carbon dioxide. The large anion gap is
Answer D is incorrect. Histopathology would due to the overproduction of ketones in the ab-
show acellular, foamy material. sence of insulin production.
Answer E is incorrect. Histopathology would Answer A is incorrect. Calcium oxalate crys-
show cysts with bradyzoites. tals may be seen in ethylene glycol poisoning,
which can be another cause of metabolic aci-
23. The correct answer is B. This child is suffer- dosis with an increased anion gap.
High-Yield Systems

ing from NRDS, which is the result of un-


derproduction of surfactant by the immature Answer B is incorrect. This case of metabolic
neonatal lungs. Surfactant is produced most acidosis involves an increased anion gap. Nor-
abundantly after week 35 of gestation. This mal-gap acidosis is caused by renal loss of bi-
substance, produced by type II pneumocytes, carbonate due to tubular dysfunction (caused
contains mainly phospholipids and apopro- by drug toxicity, SLE) or GI loss of bicarbonate
teins. It coats alveoli and small airways and through vomiting or diarrhea.
serves to reduce surface tension over the air- Answer D is incorrect. In metabolic acido-
water interface. sis, respiratory compensation occurs and re-
Answer A is incorrect. According to LaPlace’s duces blood partial pressure of carbon dioxide
law, increasing the surface tension increases through deep respiration. Elevated blood par-
the pressure required to keep the alveoli open. tial pressure of carbon dioxide is seen in pa-
This results in less pressure required to col- tients with metabolic alkalosis with respiratory
lapse the alveoli. compensation or respiratory acidosis.

Answer C is incorrect. Surfactant decreases Answer E is incorrect. Hyperkalemia, not hy-


surface tension, which increases compliance. pokalemia, is associated with diabetic ketoaci-
dosis.
Respiratory

This is because, per LaPlace’s law, decreasing


the surface tension increases the collapsing
25. The correct answer is B. The reason the over-
pressure, and compliance is inversely related
whelmed medical student could not hear any
to the collapsing pressure. Thus, in a child
heart sounds on the left is because this patient
lacking surfactant, the compliance of the al-
has complete situs inversus. As can be seen
veoli will be decreased.
from the anteroposterior chest radiograph, the
Answer D is incorrect. This child’s lungs are cardiac outline is on the right, as is the gastric
lacking surfactant. Surfactant is produced bubble. This condition is usually not harm-
mainly by type II pneumocytes. ful if the reversal of viscera is complete, but
Answer E is incorrect. In NRDS, the lungs is quite debilitating if the reversal is limited
are immature and thus do not produce mature to the heart. Often, situs inversus is associated
surfactant. A hallmark of surfactant production with Kartagener syndrome, a condition caused
is a lecithin:sphingomyelin ratio >1.5. Because by an autosomal recessive defect in the molec-
this child has immature lungs and is not pro- ular motor protein dynein. This genetic defect
ducing surfactant, this ratio is likely to be <1.5. results in immotile cilia, impairing a number
of important processes. The important findings
24. The correct answer is C. This patient most are: male infertility due to immotile sperm,
likely has type 1 DM and a resulting ketoaci- female infertility due to immotile cilia in the
dosis. He has a metabolic acidosis with a large Fallopian tube, recurrent sinusitis due to a
failure in bacteria and particle clearance, and
Chapter 17: Respiratory  •  Answers 475

bronchiectasis due to a nonfunctional muco- known cause of SIADH. HCTZ would be un-
ciliary elevator. likely to account for the degree of hyponatre-
mia (as well as the other laboratory abnormali-
Answer A is incorrect. Kartagener syndrome
ties) seen in this patient. Like indomethacin,
is not associated with acute respiratory distress
HCTZ can be used to treat nephrogenic DI.
syndrome.
Answer D is incorrect. Indomethacin has not
Answer C is incorrect. Kartagener syndrome
been associated with SIADH. It is, however,
is not associated with an increased incidence
sometimes used to treat nephrogenic DI. Pa-
of lung cancer.
tients with nephrogenic DI typically present
Answer D is incorrect. Kartagener syndrome with serum hyperosmolality, hypernatremia,
is not associated with exercise-induced asthma. and urine hypoosmolality.
Answer E is incorrect. Kartagener syndrome is

High-Yield Systems
Answer E is incorrect. Lithium toxicity has
not associated with idiopathic pulmonary fibro- been shown to cause nephrogenic DI rather
sis. than SIADH. In nephrogenic DI the kidneys
are unable to absorb water appropriately in re-
26. The correct answer is A. The patient is suffer- sponse to ADH. Patients present with produc-
ing from the syndrome of inappropriate ADH tion of large quantities of dilute urine, serum
secretion (SIADH), a condition in which ex- hyperosmolality, and hypernatremia. They
cessive ADH is secreted independently of se- report both polydipsia and polyuria. Medical
rum osmolality; this can be seen in a variety treatment of nephrogenic DI may consist of
of pulmonary diseases (including TB) as well hydrochlorothiazide or indomethacin.
as central nervous system (CNS) disorders that
enhance ADH release (eg, stroke, hemorrhage, 27. The correct answer is A. The patient’s pen-
infection, and trauma) and certain carcinomas etrating chest wound opened her intrapleural
(most commonly small cell lung carcinoma). space to the atmosphere. Therefore, as she at-
SIADH can also be an adverse effect of some tempts to inhale, her thoracic cavity expands
drugs, notably high-dose intravenous cyclo- but air enters through the wound, equalizing
phosphamide. Other drugs shown to cause the pressure; this prevents the normal expan-
SIADH include carbamazepine, vincristine, sion of the lungs. If air is not able to escape

Respiratory
vinblastine, cisplatin, amitriptyline, amio- through the wound duration exhalation, this is
darone, and monoamine oxidase inhibitors. called a tension pneumothorax, in which the
Excessive ADH secretion can lead to nausea, quantity of free air in the thoracic cavity in-
lethargy, seizures, and even coma. The pa- creases after each breath.
tient’s laboratory values are typical of someone
Answer B is incorrect. Intrapleural pressure
with SIADH, showing hyponatremia, serum
should be less than atmospheric pressure dur-
hypo-osmolality, urine hyperosmolarity, and
ing inspiration, allowing air entry. The prob-
decreased hematocrit secondary to dilution.
lem with this patient is that air is entering
Answer B is incorrect. Like lithium, demeclo- through a penetrating wound, rather than only
cycline has been associated with nephrogenic into the lungs.
diabetes insipidus (DI). Demeclocycline is
Answer C is incorrect. Pain with inspiration
not known to cause SIADH, the underlying
is a frequent complication of traumatic in-
disorder that accounts for this patient’s presen-
jury. While it may decrease the tidal volume,
tation. In fact, demeclocycline is sometimes
it would also increase breathing frequency,
used to treat SIADH.
resulting in an oxygen saturation closer to nor-
Answer C is incorrect. Hydrochlorothiazide mal. Additionally, this would not be corrected
(HCTZ) is a diuretic that may sometimes re- by chest tube placement.
sult in hyponatremia. However, this patient is
presenting with SIADH, and HCTZ is not a
476 Section II: Organ Systems  •  Answers

Answer D is incorrect. Increased pressure in 29. The correct answer is G. Working in a ship-
the pericardial space may result in a condition yard is associated with asbestos exposure.
called cardiac tamponade, during which the Chronic inhalation of asbestos fibers can result
heart cannot properly dilate. This may occur in asbestosis, which is marked histologically
when blood or other fluid enters the pericar- by ferruginous bodies that stain positively with
dial space. This can result in decreased blood Prussian blue. Asbestosis, unlike most other
pressure, increased jugular venous distention, pneumoconioses, results in marked predispo-
and distant heart sounds (called Beck’s triad). sition to bronchogenic carcinoma and to ma-
None of these clinical signs was noted in this lignant mesothelioma. Smoking and asbestos
patient; furthermore, a chest tube would not exposure together greatly increase one’s risk of
relieve the symptoms in cardiac tamponade. developing bronchogenic carcinoma.
Answer E is incorrect. The elastic properties Answer A is incorrect. Asbestosis is not related
High-Yield Systems

of the chest wall would tend to spring out, but to an autoimmune phenomenon.
the negative intrapleural pressure normally
Answer B is incorrect. The cause of asbesto-
created during inspiration opposes this ten-
sis is the inhalation of asbestos fibers into the
dency. After the penetrating injury equalized
lungs. Idiopathic restrictive lung diseases in-
the pressure of the intrapleural space and the
clude sarcoidosis and idiopathic pulmonary fi-
atmosphere, the chest wall will spring out, not
brosis.
pull inward.
Answer C is incorrect. Living in an urban area
28. The correct answer is B. Increased antero- for years can cause anthracosis, which is a re-
posterior diameter and prolonged expiration sult of inhalation of carbon dust. It is charac-
suggests the patient is suffering from chronic terized histologically by carbon-carrying mac-
obstructive pulmonary disease (COPD). Re- rophages and results in irregular black patches
duction of FEV1, forced vital capacity (FVC), visible on gross inspection. Anthracosis is
and the FEV1:FVC ratio are the hallmark of harmless.
airway obstruction. Total lung capacity (TLC)
Answer D is incorrect. Ferruginous bodies and
is increased due to lung hyperinflation, sec-
ivory-white pleural plaques are not long-term
ondary to expiratory flow limitation.
sequelae of steroid abuse.
Respiratory

Answer A is incorrect. These changes are not


Answer E is incorrect. TB has Ghon com-
consistent with either an obstructive or restric-
plexes in primary infection. Cavitary lesions
tive pattern. TLC will be increased in COPD.
are present in secondary reactivation.
Answer C is incorrect. In restrictive lung con-
Answer F is incorrect. Working in a coal mine
ditions, there is a decrease in the lung’s abil-
can cause “coal workers’ pneumoconiosis”
ity to expand. The FVC is decreased more
or silicosis. It involves inhalation of coal dust,
than the decrease in FEV1; therefore, the
which contains both carbon and silica. It is
FEV1:FVC ratio is normal or high. TLC is
marked histologically by macrophages con-
decreased because there is a loss of lung tissue
taining coal dust particles located around the
elasticity.
bronchioles.
Answer D is incorrect. These changes are not
consistent with either an obstructive or restric- 30. The correct answer is D. This man is likely
tive pattern. suffering from obstructive sleep apnea (OSA)
secondary to extreme obesity (Pickwickian syn-
Answer E is incorrect. These changes are not
drome). During the night he has intermittent
consistent with either an obstructive or restric-
cessation of airflow at the nose and mouth.
tive pattern.
During this progressive asphyxia, he has a brief
arousal, restores airway patency, and returns
to sleep. This patient’s obesity and elongated
Chapter 17: Respiratory  •  Answers 477

uvula are very good indicators of OSA, as are (DVT) due to venous stasis. Virchow’s triad de-
his daytime sleepiness, inability to concentrate, scribes the three biggest risk factors for throm-
and hypertension. Periodic, recurrent asphyxia boembolism: (1) stasis, (2) hypercoagulability,
has the effect of causing a respiratory acidosis and (3) endothelial damage. To prevent DVTs
that, when present chronically, is compensated in postoperative, bed-bound patients, heparin
for by renal retention of HCO3-. is often started in the hospital and transitioned
to warfarin on an outpatient basis. Once in the
Answer A is incorrect. If anything, this pa-
lungs, DVTs can cause chest pain, shortness
tient’s glucose is likely elevated.
of breath, pulmonary hypertension, right heart
Answer B is incorrect. This patient likely has a strain/failure, or death. Her physical exam
decreased HDL level. findings are consistent with right heart strain
Answer C is incorrect. Increased reabsorption and are related to the obstruction of the pul-

High-Yield Systems
of H+ would worsen acidosis. monary artery, effectively forcing the right side
of heart to work against increased resistance
Answer E is incorrect. Renal secretion of (leading to delayed, pronounced pulmonic
HCO3- would worsen acidosis. valve closure and a split S2). Protein C defi-
ciency can cause a hereditary prothrombotic
31. The correct answer is A. The patient has a se- disorder because protein C is normally respon-
vere, potentially fatal, pneumonia with promi- sible for inactivating factors Va and VIIIa in
nent systemic symptoms. Culture on buffered the coagulation cascade.
charcoal yeast extract is the specific clue that
the organism is Legionella pneumophila. The Answer A is incorrect. The pathology shown
disease is respiratory legionellosis, also known indicates a vascular problem. It does not show
as legionnaire’s disease, so named because the signs of aspiration. Aspiration pneumonia is
disease was first described when it occurred in certainly a concern in elderly patients postop-
epidemic form after an American Legion con- eratively. Furthermore, aspiration pneumonia
vention at a Philadelphia hotel. Patients tend would likely lead to hospitalization first, rather
to be older and may have risk factors, includ- than sudden death caused by a pulmonary em-
ing cigarette use, alcoholism, diabetes, chronic bolus.
illness, or immunosuppressive therapy.

Respiratory
Answer B is incorrect. Factor V deficiency
Answer B is incorrect. Listeria monocytogenes does not increase the risk of coagulopathy.
causes listeriosis and is not a notable cause of However, factor V Leiden, a mutation in factor
pneumonia. V that confers resistance to activated protein
C, does result in a hypercoagulable state. Es-
Answer C is incorrect. Spirillum minus is sentially this mutation results in excessive fac-
a cause of rat-bite fever and is not a notable tor V activity. Do not confuse factor V Leiden
cause of pneumonia. with factor V deficiency.
Answer D is incorrect. Staphylococcus aureus Answer C is incorrect. Factor VIII deficiency
can cause pneumonia but is easily cultured on is a bleeding disorder. Congenital factor VIII
routine media. deficiency occurs in hemophilia A and is
Answer E is incorrect. Streptococcus pneu­ X-linked recessive. Acquired factor VIII defi-
moniae can cause pneumonia but is easily cul- ciency may occur in autoimmune conditions
tured on routine media. (inhibitors to clotting factors) or in liver disease
(defective production of factors). Factor VIII
32. The correct answer is D. This patient most deficiency is not associated with an increased
likely died from a massive pulmonary embo- risk of deep venous thrombosis or pulmonary
lism; the image shows an embolus in a pulmo- embolism.
nary artery. Immobilization postoperatively in- Answer E is incorrect. Thrombocytopenia,
creases a patient’s risk of deep vein thrombosis characterized by a decreased platelet count
478 Section II: Organ Systems  •  Answers

and increased bleeding time, would unlikely breathing. Stretch receptors are located in the
be the cause of a hypercoaguable state leading smooth muscle; irritant receptors are located
to a pulmonary embolus. in the airway epithelial cells; and the J (juxta-
capillary) receptors are located in the alveolar
33. The correct answer is A. Ipratropium bromide walls close to the capillaries.
is a muscarinic antagonist used to treat COPD
Answer E is incorrect. The aortic and carotid
and asthma. It competitively blocks muscarinic
bodies are not central chemoreceptors. They
receptors, preventing acetylcholine-mediated
are the peripheral chemoreceptors that are
bronchoconstriction. It is administered directly
able to respond to decreased partial pressure of
to the airway and is minimally absorbed, lead-
oxygen in arterial blood.
ing to few adverse events. At high doses, how-
ever, atropine-like toxicity may occur.
35. The correct answer is B. The patient has hy-
High-Yield Systems

Answer B is incorrect. Methylxanthines such ponatremia, which can be attributed to the


as theophylline act by inhibiting phosphodies- SIADH secretion from a presenting para-
terase, resulting in increased cAMP levels. neoplastic phenomenon (which is more com-
mon in small-cell than non-small-cell lung
Answer C is incorrect. Cromolyn acts by in-
cancers). Lung cancer management initially
hibiting the degranulation of mast cells.
involves distinguishing small-cell from non-
Answer D is incorrect. Corticosteroids such as small-cell carcinomas. Early small-cell car-
beclomethasone and prednisone act by inhibit- cinoma management entails chemotherapy;
ing the synthesis of cytokines. early non-small-cell carcinomas can be treated
Answer E is incorrect. b-Agonists such as al­ via surgical resection. Survival time for those
buterol and salmeterol act by stimulating ade- with untreated small-cell carcinoma is 6-17
nyl cyclase, resulting in increased cAMP levels. weeks. With chemotherapy, median survival
increases to 18 months.
34. The correct answer is C. Peripheral, not cen- Answer A is incorrect. Surgery does not carry
tral chemoreceptors stimulate breathing in re- a risk of provoking para-neoplastic syndromes
sponse to oxygen levels <60 mm Hg. in small-cell carcinoma of the lung. In pheo-
chromocytoma, however, manipulation of the
Respiratory

Answer A is incorrect. Central control is lo-


cated in the brain stem and cerebral cortex. tumor during surgical resection is known to
The most important structures include the stimulate catecholamine release.
medullary respiratory center located in the re- Answer C is incorrect. Surgical resection has
ticular formation, the apneustic center in the not been shown to improve morbidity or mor-
lower pons, the pneumotaxic center in the up- tality for small-cell lung cancer, and is thus
per pons, and the cerebral cortex. contra-indicated because of increased and un-
Answer B is incorrect. Central chemorecep- necessary morbidity. Resection may be more
tors do affect breathing with varying levels of effective for non-small-cell lung carcinoma.
hydrogen in the blood, but they do not directly Some patients with very advanced GI tumors
detect levels of hydrogen in the blood. The hy- benefit from surgical resection to relieve ob-
drogen in blood cannot cross the blood-brain structive symptoms.
barrier. Thus, carbon dioxide in the blood Answer D is incorrect. Surgical resection is
crosses the cerebrospinal fluid and combines more effective for non-small-lung carcinoma.
with water to form hydrogen and bicarbonate. It leads to increased morbidity and no im-
The resulting hydrogen acts directly on the re- provement in survival in patients with small-
ceptors. cell carcinoma.
Answer D is incorrect. Stretch, irritant, and Answer E is incorrect. This explanation would
J receptors function within the lung to regulate suggest that no therapy is the best course of
Chapter 17: Respiratory  •  Answers 479

action, which is false. There are few masses for causes of community-acquired pneumonia but
which nothing is done; an example is the cuta- is identified as the cause in only 3% of cases.
neous hemangioma seen in pediatric patients. The organism is present only in water sources
(eg, air conditioning systems, whirlpools, mist
36. The correct answer is D. Interstitial (or idio- sprayers) and causes infection when aerosol-
pathic) pulmonary fibrosis (IPF) is a chronic, ized water droplets are inhaled. Transmission
progressive fibrotic disorder of the lower res­ is not by person-to-person contact. Typically,
piratory tract that affects older adults. It is more severe illness is seen in patients who are
characterized by the abnormal proliferation >50 years of age, those who smoke, and those
of mesenchymal cells, disruption of collagen whose Gram stain shows neutrophils and very
structures, and impaired gas exchange. The few organisms, as in this case. Treatment is
exact pathogenesis of IPF is still unknown. If with erythromycin, because L pneumophila

High-Yield Systems
not treated, IPF often results in death within produces a b lactamase that renders it resistant
five years. Lung transplantation is currently to penicillin derivatives.
the only “cure” for this disease. On x-ray of
Answer A is incorrect. Bordetella pertussis
the chest, IPF usually is seen best in the lower
is a gram-negative rod that causes whooping
parts of both lungs as white lines in a netlike
cough, characterized by paroxysms of cough-
pattern.
ing followed by a loud inspiration, or “whoop.”
Answer A is incorrect. Albuterol is a b2- B pertussis infection does not typically cause
adrenergic agonist used in treating patients with pneumonia.
asthma. It is not a definitive therapy for IPF.
Answer B is incorrect. Haemophilus influen­
Answer B is incorrect. Azathioprine may be zae type B is a gram-negative rod commonly
useful in the treatment of IPF, but a patient’s associated with acute epiglottitis or meningi-
pulmonary status will still deteriorate despite tis. It is an exclusively human pathogen that
its use. is transmitted by aerosolized droplets or direct
Answer C is incorrect. Cyclosporine is an im- contact with secretions. The H influenzae type
munosuppressive drug most commonly used B vaccine has rendered these infections far less
after transplantation. By inhibiting T-lympho- common, making it an unlikely agent in this

Respiratory
cytes from producing interleukin-2 and other scenario.
lymphokines, cyclosporine is able to reduce Answer D is incorrect. Mycobacterium tu­
inflammation. Although it can be useful in the berculosis is an acid-fast mycobacterium that
treatment of IPF, a patient’s pulmonary status causes TB. The first exposure to M tubercu­
often continues to deteriorate. losis, or primary TB, is usually asymptomatic.
Answer E is incorrect. Steroids may be useful Secondary, or reactivation, TB occurs after the
in the treatment of idiopathic pulmonary fibro- bacteria have been dormant for some time and
sis, but the patient’s pulmonary status generally re-emerge as a result of temporary weakening
continues to deteriorate despite their use. The of the immune system. This phase of disease
only cure for the disease is lung transplanta- is typically a chronic process characterized by
tion. low-grade fever, night sweats, weight loss, and a
productive cough. This chronic picture differ-
37. The correct answer is C. Legionella pneu­ entiates TB from the acute picture of Legion-
mophila is an aerobic, gram-negative rod that naires disease.
causes Legionnaire’s disease, a condition in Answer E is incorrect. Streptococcus pneu­
which patients develop acute, severe pneumo- moniae presents acutely and is a significant
nia and a high fever. Other signs and symp- cause of bacterial pneumonia in adults. Un-
toms include hyponatremia (which is unique like L pneumophila, however, sputum culture
to this pneumonia) and CNS changes. Le- in S pneumoniae infection would reveal sig-
gionnaire’s disease is one of the most common nificant growth of gram-positive diplococci.
480 Section II: Organ Systems  •  Answers

S pneumoniae is transmitted by person-to-­ Answer E is incorrect. There is no role for the


person contact. administration of vitamin E in the treatment
of TB. Vitamin E is a potent lipid-soluble anti-
38. The correct answer is B. The patient is suf- oxidant that protects the cell membranes from
fering from TB, with the causative organism lipid peroxide.
(Mycobacterium tuberculosis) seen as red on
acid-fast stain. Symptoms include fever, night 39. The correct answer is F. This child has mea-
sweats, chills, cough, and weight loss. His treat- sles complicated by pneumonia. Pneumonia
ment regimen will include isoniazid and other complicates approximately 4% of measles cases
antimycobacterial agents. Isoniazid inhibits in the United States and as many as 50% of
mycolic acid synthesis in the mycobacterial cases abroad. Clinically, this child has a high
cell wall. Because of rapid development of re- fever, Koplik’s spots, maculopapular rash, and
High-Yield Systems

sistance, isoniazid should never be used alone CT of the chest showing diffuse interstitial in-
to treat active TB. Isoniazid depletes pyridox- volvement. Measles-infected respiratory cells
ine (vitamin B6), which is required for the will fuse and form multinucleated giant cells,
production of dopamine, epinephrine, norepi- which can be detected in sputum samples.
nephrine, and monoamine neurotransmitters. Measles is a member of the Paramyxoviri-
Hence, one of the adverse effects of isoniazid dae family, a group of negative-sense, single-
therapy is peripheral neuropathy, which can stranded RNA viruses. In immunocompro-
be prevented by co-administration of vitamin mised hosts, measles pneumonia may evolve to
B6. giant cell pneumonia, which is often fatal.
Answer A is incorrect. There is no role for the Answer A is incorrect. Acid-fast bacilli would
administration of cobalamin (vitamin B12) in be expected in the sputum of a child infected
the treatment of TB. Vitamin B12 is used to with mycobacteria such as Mycobacterium tu­
treat patients with deficiency who are showing berculosis.
neurologic symptoms and macrocytic anemia.
Answer B is incorrect. Cells with nuclei sur-
Vitamin B12 is a coenzyme that facilitates ar-
rounded by a halo and clear cytoplasm are
rangement of a hydrogen atom between two
koilocytes and would be found in cells infected
adjacent atoms and methyl group transfer be-
with human papillomavirus. This child has
Respiratory

tween two molecules. Deficiency is seen in


measles, which will form multinucleated giant
chronic alcoholics, pure vegans, and those
cells.
with pernicious anemia.
Answer C is incorrect. Cowdry-type inclusions
Answer C is incorrect. There is no role for
in cells are suggestive of infection with CMV.
the administration of vitamin B1 (thiamine) in
Although CMV can cause pneumonia, it does
the treatment of TB. Thiamine combines with
so more commonly in immunocompromised
ATP to form thiamine pyrophosphate, a cofac-
hosts.
tor for oxidative decarboxylation of a-ketoacids
and branched-chain amino acid dehydro- Answer D is incorrect. Gram-negative cocco-
genase; it is also a cofactor for transketolase bacilli and polymorphonuclear leukocytes are
in the hexose monophosphate shunt. Thia- commonly associated with Haemophilus influ­
mine deficiency leads to beriberi, Wernicke-­ enzae, which is a common cause of pneumo-
Korsakoff syndrome, and lactic acidosis. Vi- nia in children. However, this child is likely
tamin B1 is used to treat alcoholics to prevent infected with measles.
Wernicke-Korsakoff syndrome. Answer E is incorrect. Gram-positive dip-
Answer D is incorrect. There is no role for the lococci and polymorphonuclear leukocytes
administration of vitamin C in the treatment of are often seen in pneumococcal pneumonia,
TB. Vitamin C is used to treat scurvy. which would more likely present with a dense
Chapter 17: Respiratory  •  Answers 481

lobar pneumonia. This child has other signs Answer A is incorrect. Choice A represents
and symptoms characteristic of measles. the inspiratory reserve volume, which is the
volume that can be inspired after inspiration of
40. The correct answer is A. This child suf- the tidal volume.
fers from a congenital diaphragmatic hernia
Answer B is incorrect. Choice B represents
caused by the failure of the diaphragm to prop-
the tidal volume, which is the volume inspired
erly form and close. The presence of bowel
or expired with each normal breath.
sounds in a lung zone indicates that abdomi-
nal contents have herniated past the boundary Answer C is incorrect. Choice C represents
of the diaphragm into the thorax. The devel- the expiratory reserve volume, which is the vol-
oping diaphragm is derived from the Septum ume that can be expired after the expiration of
transversum, Pleuroperitoneal folds, Body the tidal volume.

High-Yield Systems
wall, and Dorsal mesentery of the esophagus.
Answer E is incorrect. Choice E represents
These four components can be remembered
the inspiratory capacity, which is the sum of
by the mnemonic “Several Parts Build the Di-
tidal volume and inspiratory reserve volume.
aphragm.”
Answer F is incorrect. Choice F represents
Answer B is incorrect. A continuous cardiac
the functional reserve capacity. It is the sum of
murmur (ie, present during both systole and
the expiratory reserve volume and the residual
diastole) could be the consequence of a patent
volume, and it is the volume that remains in
ductus arteriosus, but is not related to the pleu-
the lungs after a tidal volume is expired.
roperitoneal folds and is unlikely to cause the
presentation in this patient. Answer G is incorrect. Choice G represents
vital capacity, which is the sum of tidal vol-
Answer C is incorrect. Marked splenomegaly
ume, inspiratory reserve volume, and expira-
in children has many etiologies, but is un-
tory reserve volume. Vital capacity (also called
likely to be consistent with the features of this
FVC) is the volume of air that can be forcibly
vignette. Causes of splenomegaly include
expired after a maximal inspiration.
congenital infections and metabolic genetic
disorders. Congenital infections include the 42. The correct answer is D. This patient is show-
ToRCHeS infections, which include Toxo-

Respiratory
ing signs of Cushing syndrome with a buffalo
plasmosis, Rubella, Cytomegalovirus, Herpes- hump and purple striae. Cushing syndrome
virus/HIV, and Syphilis. These infections often is caused by an excess of cortisol either be-
cause hepatosplenomegaly, jaundice, mental cause of a pituitary adenoma producing excess
retardation, and intrauterine growth retarda- ACTH, an adrenal adenoma producing ex-
tion. Lysosomal storage diseases such as Gau- cess cortisol, or ectopic ACTH production by
cher disease, Niemann-Pick disease, Hunter a neoplasm. This man’s smoking history and
syndrome, and Hurler syndrome also have lung nodule shown on chest radiography point
symptoms of hepatosplenomegaly. to lung cancer. Taken together with ectopic
Answer D is incorrect. Midline deviation of production of ACTH, this patient has para-
the trachea is commonly associated with pneu- neoplastic syndrome, with ectopic production
mothorax or space-occupying lesions of the of ACTH by the malignant lung mass. Of the
cervical region. different histological classifications of lung
cancer listed above, small cell carcinoma is
41. The correct answer is D. Choice D represents the most likely in the case for several reasons:
the residual volume, which is the volume that Squamous cell and small cell carcinomas are
remains in the lungs after a maximal expira- most closely linked to smoking history (>98%
tion. The residual volume increases dramati- are associated with smoking) and both pre­
cally in emphysema. sent as central lesions such as that shown on
the x-ray film. Additionally, tumors producing
482 Section II: Organ Systems  •  Answers

ACTH or ADH are usually small cell carcino- x-ray of the chest as a collapsed lung with a
mas. mediastinum shifted away from the collapsed
lung. With pneumothorax, the patient should
Answer A is incorrect. Adenocarcinoma is the
be assessed for signs and symptoms of hemody-
most common lung cancer found in women
namic compromise. This patient, for example,
and nonsmokers (although 75% are found in
is hypotensive, tachycardic, and tachypnic,
smokers). Adenocarcinomas are usually pe-
and therefore requires urgent management.
ripherally located, and are less likely to cause
para-neoplastic conditions such as Cushing Answer A is incorrect. It is possible that this
syndrome. patient has a hemothorax, but this vignette de-
scribes a pneumothorax injury. A hemothorax
Answer B is incorrect. Bronchial carcinoid is
is characterized by blood in the thoracic cavity.
a rare neuroendocrine lung tumor that is not
High-Yield Systems

linked to smoking. These tumors cause cough, Answer B is incorrect. The stab wound is
hemoptysis, and an increase the number of res­ above the nipple, which is about the level of
piratory infections. Some of them are capable the fourth and fifth ribs, superior to the ninth
of producing serotonin and causes the classic and tenth ribs. It is possible that the man has
“carcinoid syndrome” characterized by epi- also sustained injury to his lower ribs, but this
sodic attacks of diarrhea, flushing, and cyano- would not be related to the knife injury and is
sis. not described in this vignette. Of note is the
risk that a fractured lower rib (11th or 12th)
Answer C is incorrect. Many cancers cause
may puncture the kidney, leading to retroperi-
metastases to the lung. These typically present
toneal bleeding.
as multiple discrete nodules found in all lobes.
The picture shown here is that of a single sol- Answer C is incorrect. A pleural effusion is
itary lesion, which is more likely to be a pri- seen on radiographs as a fluid collection in the
mary lung cancer. dependent portions of the thorax. Pleural effu-
sions can occur in heart failure, pneumonia,
Answer E is incorrect. Squamous cell carci-
or iatrogenic fluid overload (eg, improper fluid
noma accounts for 25%-40% of lung cancers
management of a hospitalized patient).
and is closely linked to smoking. Like small
cell carcinoma, squamous cell carcinoma also Answer E is incorrect. Right upper lobe con-
Respiratory

arises centrally and is associated with para- solidation would be consistent with right upper
neoplastic syndromes. However, while small lobe pneumonia, which is not described in this
cell carcinomas are responsible for Cushing vignette. One would expect to see a history of
syndrome and the SIADH secretion, squamous fever and other signs of infection, which is not
cell carcinomas usually cause hypercalcemia the case here. Also, radiographs would show an
by producing parathyroid hormone-related uninterrupted opacity.
peptide.
44. The correct answer is A. This patient is likely
43. The correct answer is D. This question re- suffering from a TB infection that was reacti-
quires knowledge of both the anatomy and the vated by her use of etanercept. The x-ray of the
physiology of the sucking chest wound, as de- chest shows a dense cavitary apical lung lesion
scribed in this patient. A penetrating wound that is highly indicative of a reactivated TB
to the chest can puncture the pleura, making infection. Etanercept is a fusion protein that
an opening for air to be sucked into the pleu- contains two identical tumor necrosis factor
ral space. With inspiration, the diaphragm (TNF)-receptor monomers fused to a human
descends, lowering the intrapleural pressure. IgG Fc domain. Therefore, it acts as a TNF
If there is a communication directly between antagonist. In TB infections, TNF (secreted
the pleural space and the outside world, air is by activated macrophages) recruits monocytes
sucked into this negative-pressure space and to form the epithelioid granulomas required to
collapses the lung. Pneumothorax is seen on contain the mycobacteria. When TNF is ef-
Chapter 17: Respiratory  •  Answers 483

fectively removed from the infection site (by Answer B is incorrect. Asthma is a condition
drugs or other forms of immunosuppression), associated with airway constriction, marked by
patients face an increased risk of reactivation wheezing. It does not present with hemoptysis
with caseation and cavitary lesions. or airway dilation
Answer B is incorrect. Methotrexate is an anti- Answer C is incorrect. Atelectasis is alveolar
inflamatory agent used in the treatment of RA. collapse and is not associated with airway dila-
It inhibits dihydrofolate reductase and blocks tion.
thymine synthesis. It is not an inhibitor of tu-
Answer E is incorrect. Churg-Strauss syn-
mor necrosis factor. The dosage of methotrex-
drome is a multisystem disease that commonly
ate used in the treatment of RA does not in-
affects the lung. It is characterized by eosino-
duce myelosuppression, although higher doses
philia and vasculitis, not airway dilation.
can produce this complication.

High-Yield Systems
Answer C is incorrect. Nonsteroidal anti- 46. The correct answer is A. b-agonists such as
inflammatory drugs (NSAIDs) do not impair albuterol may cause potassium to shift into
immunity, although they do impair platelet cells, resulting in hypokalemia. This may lead
function. These drugs can help decrease in- to ECG abnormalities due to destabilization of
flammation, but do not slow the progression of cardiac cell membranes, the classic examples
RA. It is quite unlikely that a patient with RA of which are U waves. Short-acting b-agonists
who complains of well-controlled joint pain such as albuterol are used in the treatment of
would be relying on NSAIDs alone. acute asthma exacerbations because of their
relaxing effects on bronchial smooth muscle.
Answer D is incorrect. Risedronate is a
Long-acting β-agonists such as salmeterol are
bisphosphonate used in the prevention and
used for prophylaxis of bronchospasm.
treatment of osteoporosis. Bisphosphonates are
not used in the treatment of RA, and do not Answer B is incorrect. Cromolyn inhibits
have immunosuppressive effects. antigen-induced bronchospasm by inhibiting
mediator release from bronchial mast cells,
Answer E is incorrect. Although the exact
and suppressing chemotaxis of neutrophils, eo-
mechanism of action of sulfasalazine is not
sinophils, and monocytes. It is used as a pro-
known, it is believed to suppress the activity

Respiratory
phylactic agent in mild to moderate asthma.
of natural killer cells and impair lymphocyte
Cromolyn is generally well tolerated, and ad-
transformation, which would not directly allow
verse effects are generally minor, including
mycobacteria to overcome immune surveil-
bronchospasm, cough, wheezing, angioedema,
lance and reactivate.
headache, and nausea.
45. The correct answer is D. Bronchiectasis can Answer C is incorrect. Ipratropium is an anti-
be caused by a chronic necrotizing infection muscarinic agent that is used for both asthma
of the bronchi leading to dilated airways. In ad- and COPD. Common adverse effects include
dition to bronchopulmonary infections, bron- cough, nausea, and dizziness. It is not known
chiectasis can be caused by bronchial obstruc- to cause hypokalemia.
tions or congenital abnormalities (bronchial
Answer D is incorrect. Theophylline most
cysts, tracheobronchial fistulas). Bronchiecta-
likely causes bronchodilation by increas-
sis is a common cause of hemoptysis and also
ing levels of cAMP. It does this by inhibiting
frequently presents with cough and dyspnea.
phosphodiesterase, an enzyme that hydrolyses
Answer A is incorrect. Adult respiratory dis- cAMP to AMP. Theophylline has a narrow
tress syndrome causes diffuse alveolar damage therapeutic window and may cause cardiotox-
that leads to increased alveolar capillary per- icity (and neurotoxicity) but does not result in
meability. It does not cause airway dilation. hypokalemia.
484 Section II: Organ Systems  •  Answers

Answer E is incorrect. Zileuton is an asthma Answer D is incorrect. SLE can be associated


medication that blocks the production of leu- with pleuritis, but it is not associated with non-
kotrienes. Serious adverse reactions include caseating granulomas. Initial treatment of non-
hepatotoxicity and neutropenia. It is not life-threatening SLE includes analgesics and
known to cause hypokalemia. antimalarials such as hydroxychloroquine. As
this patient has sarcoidosis, not lupus, hydroxy-
47. The correct answer is C. This image shows chloroquine would not be an appropriate treat-
noncaseating granulomas involving lung sep- ment for her.
tae. Noncaseating granulomas are character-
Answer E is incorrect. TB is characterized
istic of sarcoidosis. Sarcoidosis is a multiorgan
by caseating granulomas, which can be rec-
inflammatory disorder of unknown etiology. It
ognized by the necrotic, cheese-like center in
is thought to be immune mediated. The lung
the granuloma. Bacteria within the granuloma
High-Yield Systems

is the most frequently involved organ, but


may not be destroyed, but may rather be dor-
other commonly affected organs are lymph
mant only to be later reactivated. This is more
nodes, skin, eyes, kidneys, the heart, and the
likely if the patient is immunosuppressed at
CNS. Findings that might be expected in a pa-
any time. TB is treated with a multidrug regi-
tient with sarcoidosis include γ Globulinemia,
men consisting of isoniazid, rifampin, pyra-
Rheumatoid arthritis, elevated Angiotensin-
zinamide, and ethambutol for two months fol-
converting enzyme levels, Interstitial fibrosis,
lowed by a four-month course of isoniazid and
and Noncaseating granulomas (remember the
rifampin.
mnemonic GRAIN). Initial treatment of sar-
coidosis includes a short course of glucocorti-
48. The correct answer is B. This describes Bacil­
coids such as dexamethasone if the patient is
lus anthracis, which can cause cutaneous an-
symptomatic. For chronic disease, glucocorti-
thrax, inhalation anthrax, and GI anthrax. This
coids may be continued or alternative agents
patient had inhalation anthrax (also known as
such as methotrexate may be used.
“wool-sorter’s disease”), which usually has two
Answer A is incorrect. Small cell lung cancer phases: the initial phase characterized by mal-
is recognized by numerous small blue neoplas- aise, dry cough, and chest pressure that resolve
tic cells on histologic exam. The primary treat- in a few days; and the second phase in which
Respiratory

ment of small cell lung cancer is operative, but patients suddenly develop acute respiratory dis-
in patients with stages IB and II disease, adju- tress and hypoxemia followed by hemorrhagic
vant therapy with cisplatin has shown a trend mediastinitis and bloody pleural effusions. A
toward improved survival. The image shows classic radiologic finding is mediastinal widen-
noncaseating granulomas, which are charac- ing. If a patient is not rapidly treated with peni-
teristic of sarcoidosis and would not be treated cillin, doxycycline, ciprofloxacin, or levofloxa-
with chemotherapy. cin, systemic infection can cause septic shock
(due to exotoxins produced by the bacteria)
Answer B is incorrect. Goodpasture syndrome
and death within 24 hours. Spores from sheep
is caused by anti-basement membrane anti-
or goat skin are the primary mode of trans-
bodies, which can be demonstrated on im-
mission in this kind of anthrax. Interestingly,
munofluorescence. It is not associated with
B anthracis is the only medically relevant bac-
noncaseating granulomas. Initial treatment of
teria with a protein capsule.
Goodpasture syndrome is a five-day course of
methylprednisolone followed by a long taper Answer A is incorrect. This describes Brucella.
and maintenance. However, if the disease is Brucella is transmitted from cattle to humans
particularly severe, immunosuppressive agents who have contact with infected animal meat,
such as cyclophosphamide or azathioprine milk products, or aborted animal placentas.
may be started. The pathogen penetrates multiple organs, in-
cluding the lungs, skin, conjunctiva, and GI
tract. Patients with brucellosis have systemic
Chapter 17: Respiratory  •  Answers 485

symptoms such as fever (undulant fever that is carbonate. Carbaminohemoglobin accounts


worse in the evening), chills, loss of appetite, for only about 5%.
and lymphadenopathy. Brucellosis is rarely fa-
Answer D is incorrect. The lungs do not have
tal, and its symptoms can last from months to
an acidic environment; the peripheral tissues
years.
have an acidic environment. The oxygenation
Answer C is incorrect. This describes Fran­ of hemoglobin in the lungs promotes the disso-
cisella tularensis, which causes tularemia, ciation of hydrogen ions from deoxyhemoglo-
characterized by abrupt onset of fever, chills, bin and the equilibrium is shifted toward the
malaise, and fatigue. Six clinical forms of tu- production of carbon dioxide from carbonic
laremia exist: ulceroglandular, glandular, ocu- acid.
loglandular, oropharyngeal, pneumonic, and
Answer E is incorrect. While most of the car-
typhoidal (septicemic). Pulmonic tularemia is

High-Yield Systems
bon dioxide is converted to bicarbonate, bicar-
very similar to inhalational anthrax; however,
bonate does not bind to hemoglobin; hydrogen
hemorrhagic mediastinitis is not seen in tu-
ions bind hemoglobin. Dissolved carbon di-
laremia, and death does not occur within 24
oxide that remains in the plasma accounts for
hours. Tularemia is also associated with rabbit,
about 5% of transport.
tick, or deerfly contact.
Answer D is incorrect. This describes Nocar­ 50. The correct answer is A. The long smoking
dia asteroides, which causes pulmonary infec- history and presence of flattened diaphragms
tions primarily in immunocompromised indi- on radiographs are suggestive of COPD. The
viduals. hallmark of obstructive lung disease is a de-
creased FEV1:FVC ratio (to <80%). FEV1
Answer E is incorrect. This describes Legion­
is reduced in patients with COPD because
ella pneumophila. Legionella is a cause of
of obstruction. In restrictive lung diseases,
severe pneumonia, particularly in cigarette
FEV1:FVC is normal or increased.
smokers and immunocompromised individu-
als. It is associated with environmental water Answer B is incorrect. Functional residual ca-
sources. It does not cause mediastinitis or hem- pacity (FRC) is increased in COPD because
orrhagic pleural effusions. the patient is unable to expire air fully, result-

Respiratory
ing in air trapping in the lungs. In restrictive
49. The correct answer is C. In the tissues, more lung disease, FRC is decreased because the
carbon dioxide is being produced because of lung has restricted expansion so all lung vol-
the increased metabolic rate. The additional umes are decreased.
carbon dioxide enters the RBC and is com-
Answer C is incorrect. The total lung capacity
bined with water by carbonic anhydrase to
in patients with COPD increases. It decreases
form H2CO3, which then dissociates into hy-
in patients with restrictive lung disease.
drogen and bicarbonate. The hydrogen ions
are buffered by deoxyhemoglobin, while the Answer D is incorrect. COPD has a character-
bicarbonate diffuses out of the RBCs in ex- istically decreased FEV1.
change for chloride ions. This is called the Answer E is incorrect. Emphysema is an ob-
chloride shift. structive lung disease, which has a characteris-
Answer A is incorrect. Bicarbonate travels in tically decreased FEV1:FVC ratio. Restrictive
the plasma, not the RBCs. When it gets to the lung diseases can have increased FEV1:FVC
lung, it enters the RBCs, transforms back to ratios because FVC is reduced along with total
water and carbon dioxide, and the latter is ex- lung volume.
haled.
Answer B is incorrect. The primary transport
of carbon dioxide in the blood (90%) is via bi-
This page intentionally left blank
SECTION III

Full-Length
Examinations

 Test Block 1

 Test Block 2

 Test Block 3

 Test Block 4

 Test Block 5

 Test Block 6

 Test Block 7

487
This page intentionally left blank
Test Block 1

489
490 Section III: Full-Length Examinations  •  Questions

Q u e st i o n s

1. An investigator is attempting to create new amination findings. What is the most likely ge-
treatments for patients in shock. Her premise netic makeup of this child?
is that increasing oxygen delivery and availabil-
(A) Trisomy 18
ity in tissue should reduce some of the tissue
(B) Trisomy 21
damage seen in most forms of shock. From a
(C) XO
basic physiologic standpoint, which of the fol-
(D) XXY
lowing steps would increase peripheral oxygen
availability? (E) XYY

(A) Administering high doses of norepineph- 4. A 66-year-old man is seen in the emergency
rine department after an acute episode of severe
(B) Administering hydroxyurea hip pain that caused him to fall while walking
(C) Inhibiting synthesis of 2,3-bisphosphoglyc- at home. X-ray shows the presence of a patho-
erate receptors logical fracture of the pelvis. On questioning,
(D) Lowering the patient’s core body tempera- the man reports that in recent months he has
ture felt very fatigued and has been bothered by
(E) Raising levels of partial pressure of arterial constant dull pain in his hips, back, and head.
carbon dioxide A radiograph of his skull is shown in the im-
age. Laboratory studies show a serum calcium
2. A 32-year-old woman with pheochromocytoma of 13.5 mg/dL and alkaline phosphatase of 60
is being treated with phenoxybenzamine. After U/L. Which of the following is an additional
surgical excision of the tumor, the patient has finding that would be likely in this patient?
an episode of hypotension requiring 30 seconds
of cardiopulmonary resuscitation and subse-
quent treatment in the intensive care unit. The
Full-Length Exams

attending physician asks his intern what physi-


ologic responses he would expect to see if the
patient had been given epinephrine during re-
suscitation. What would have been observed
following administration of epinephrine?
(A) Decrease in blood pressure
(B) Decrease in heart rate
(C) Increase in blood pressure
(D) Increase in respiratory rate
(E) No changes in vital signs

3. On physical examination, a 5-year-old girl has


hypertension in the upper extremities. She also Reproduced, with permission, from USMLERx.com.
has weak pedal and popliteal pulses. Her pe-
diatrician is concerned and orders an echocar-
diogram that confirms the diagnosis. Based on (A) Blue sclerae
this diagnosis and on the patient’s characteris- (B) Facial muscle contraction on tapping the
Test Block 1

tic physical features, the physician performs a facial nerve


karyotype analysis that reveals a genetic abnor- (C) Hard prostatic nodule on digital rectal ex-
mality in the patient. The parents are told that amination
the abnormalities observed in the karyotype (D) Palpable parathyroid nodule
analysis can explain the abnormal physical ex- (E) Proteinuria
Test Block 1  •  Questions 491

5. A 33-year-old man presents to his physician years ago. The physician suspects that he has
because of recurrent headaches. Although his an immunodeficiency that results from abnor-
symptoms are consistent with tension head- mal NADPH oxidase activity. Which of the
aches and are alleviated by ibuprofen, he has following would confirm the most likely diag-
been conducting research on the Internet nosis?
and is concerned that his headaches may be
(A) Large lysosome vesicles in neutrophils on
caused by a subarachnoid hemorrhage. He in-
light microscopy
sists on undergoing CT of the head. Which of
(B) Low levels of IgM but elevated levels of
the following is the most appropriate next step?
IgE and IgA
(A) Address the patient’s concerns about a sub- (C) Low levels of IgM, IgE, and IgA
arachnoid hemorrhage (D) Negative nitroblue tetrazolium dye reduc-
(B) Compromise with the patient and send tion test
him for a less costly x-ray of the head (E) Positive nitroblue tetrazolium dye reduc-
(C) Obtain a CT scan to alleviate the patient’s tion test
fears
(D) Send the patient for a neurology consult 8. A 79-year-old man presents to his family phy-
(E) Tell him he does not have a subarachnoid sician for an annual physical examination. He
hemorrhage denies chest pain, dyspnea, fevers, or chills. He
(F) Tell the patient ibuprofen will cure a sub- suffered a myocardial infarction five years ago.
arachnoid hemorrhage He is taking aspirin, lisinopril, and metoprolol.
His ECG is shown in the image. He is admit-
6. A 21-year-old woman with no family or per- ted to the hospital for therapy consisting of
sonal history of breast cancer presents to the rate control and anticoagulation. After success-
clinic with a small, firm mass in the lower in- ful in-patient treatment, he is discharged with
ner quadrant of her right breast. Palpation of new medicines. Which of the following is the
the mass reveals the mass is firm, nontender, mechanism of the anticoagulant used in the
and mobile, with no overlying skin changes long-term therapy of this patient’s condition?

Full-Length Exams
and no nipple discharge. There is no asso-
ciated palpable axillary or supraclavicular
lymphadenopathy. A urine pregnancy test is
negative. Which of the following would most
likely be found on histological examination of
this mass?
(A) Branching fibrovascular core extending
from a dilated duct
(B) Fibrosing stroma around normal glandular
tissue
(C) Large cells with clear “halos”
(D) Parallel arrays of small, monomorphic Reproduced, with permission, from USMLERx.com.
cells with scant cytoplasm
(E) Sheets of pleomorphic cells infiltrating ad-
jacent stroma (A) Activation of antithrombin III
(B) Antibody to tumor necrosis factor-a
7. A 16-year-old boy comes to the physician be- (C) Inhibition of γ-carboxylation of vitamin K-
Test Block 1

cause of recurrent sinusitis that progressed to a dependent clotting factors


pulmonary infection with left-sided chest pain (D) Inhibition of cyclooxygenase
and fever. One year ago he had an episode of (E) Inhibition of the ADP pathway of throm-
Staphylococcus aureus pneumonia, and he had bus formation
repeated oral Candida infections until a few
492 Section III: Full-Length Examinations  •  Questions

9. A 22-year-old woman presents to her family


physician because of increasing fatigue and be-
cause she looks “pale” despite spending many
hours outside on her sailboat. She also states
that her urine looks “cola-colored” when she
first goes to the bathroom in the morning. The
patient feels well otherwise. Blood analysis
shows a low platelet count, a low RBC count,
and a low WBC count. The patient’s RBCs are
mixed with acidified normal serum and com-
pared to normal RBCs at room temperature
and at 37°C (98.6°F); both temperatures cause
the patient’s, but not the normal, RBCs to Courtesy of Dr. George P. Kubica, Centers for Disease Con-
lyse. Which of the following best describes the trol and Prevention.
pathophysiology behind this patient’s disorder?
(A) Decreased respiratory rate while sleeping (A) Cycloserine
→ respiratory acidosis → complement-­ (B) Isoniazid and rifampin
mediated lysis of RBCs (C) Isoniazid and vitamin B6
(B) Decreased respiratory rate while sleeping (D) Rifampin
→ respiratory acidosis → osmotic lysis of (E) Rifampin and vitamin B6
RBCs
(C) Decreased respiratory rate while sleeping 11. A 4-year-old boy has a sublingual mass. 99mTc
→ respiratory alkalosis → complement- pertechnetate scanning shows significant up-
mediated lysis of RBCs take in this region. Which of the following is
(D) Decreased respiratory rate while sleeping the embryologic explanation for this mass?
→ respiratory alkalosis → osmotic lysis of
RBCs (A) The thymus has developed ectopically
Full-Length Exams

(E) Increased respiratory rate while sleeping (B) The thymus has hypertrophied
→ respiratory acidosis → complement-­ (C) The thyroid has failed to migrate caudally
mediated lysis of RBCs (D) The thyroid has migrated too far rostrally
(F) Increased respiratory rate while sleeping (E) The third and fourth branchial (pharyn-
→ respiratory alkalosis → osmotic lysis of geal) arches have hypertrophied
RBCs
12. A 27-year-old healthy man presents because he
10. An acid-fact stain of a sputum sample taken and his wife have been repeatedly unsuccess-
from a 49-year-old woman who recently im- ful in conceiving a child. His wife has been
migrated to the United States from Mexico is tested and determined to be fertile. Upon ques-
shown in the image. Which of the following tioning, the patient denies coronary or lipid ab-
should be administered to people who have normalities but admits to having multiple sinus
come in contact with this patient and who infections and a chronic productive cough.
have a positive tuberculin test but negative Further analysis of his semen shows a normal
findings on x-ray of the chest? number of sperm. Which of the following is
the most likely etiology for the patient’s infer-
tility?
Test Block 1
Test Block 1  •  Questions 493

(A) Age-related increase in estradiol with possi-


ble prostate dihydrotestosterone sensitiza-
tion
(B) Autosomal recessive dysfunction of a chlo-
ride ion channel
(C) Failure of testicles to descend into the
scrotum
(D) Familial disease causing early atherosclero-
sis leading to erectile dysfunction
(E) Lack of dynein ATPase arms in microtu-
bules of cilia

13. An anxious young woman presents to the


emergency department because of acute-onset
Reproduced, with permission, from USMLERx.com.
severe abdominal pain. She consumed eight
or nine alcoholic drinks earlier in the evening.
She also admits to using diuretics to “lose wa- (A) Eye movement
ter weight.” Physical examination reveals peri- (B) Facial movement
umbilical tenderness to palpation. Her stool (C) Hearing
is guaiac negative. Arterial blood gas analysis (D) Mastication
reveals a pH of 7.55, a bicarbonate level of 21 (E) Tongue movements
mEq/L, and a partial pressure of carbon diox-
ide of 25 mm Hg. Her sodium level is within 15. A 65-year-old woman with a history of anxiety
normal limits. Which of the following is the presents to her doctor with renewed anxiety.
most likely cause of her acid-base disturbance? She had been prescribed barbiturates in the
(A) An accumulation of unmeasured anions as past, but has heard that benzodiazepines are
a result of hepatic metabolism of alcohol now more commonly prescribed. She wants

Full-Length Exams
(B) Electrolyte imbalance due to diuretic use to know how the medications are different.
(C) Hyperventilation secondary to pain and Which of the following statements accurately
anxiety contrasts the features of benzodiazepines with
(D) Hypoventilation due to the respiratory de- barbiturates?
pression caused by alcohol ingestion (A) Benzodiazepines are less safe for the treat-
(E) Vomiting due to alcohol toxicity ment of anxiety
(B) Benzodiazepines have a higher incidence
14. Below is an artist’s rendition of the anatomy of of respiratory depression
the base of the brain. What is a major function (C) Benzodiazepines have a shorter half-life
of the cranial nerve indicated by the arrow? (D) Benzodiazepines increase the duration of
the chloride channel opening
(E) Benzodiazepines increase the frequency of
the chloride channel opening
Test Block 1
494 Section III: Full-Length Examinations  •  Questions

16. A 3-week-old girl is brought to the emergency pholipid antibody titer. Which of the following
department by her mother, who says her would, if present, predispose a patient to the
daughter has suddenly developed a large, tense condition causing symptoms in this woman?
bulge on the top of her head and a fever. The
(A) Cholecystitis
mother also notes that the patient has become
(B) Chronic obstructive pulmonary disease
more irritable and has not been feeding well.
(C) Polycythemia vera
Although her mother received limited prena-
(D) Primary biliary cirrhosis
tal care, the child was born at 38 weeks’ gesta-
(E) Renal failure
tion via spontaneous vaginal delivery without
complications. The patient undergoes lumbar 18. A 42-year-old man with no medical history vis-
puncture, and a Gram stain of cerebrospinal its a doctor after he develops sudden, unilateral
fluid is shown in the image. Which of the fol- left-arm weakness after an argument with his
lowing best characterizes the disease-causing wife. He states that he packed up a few of his
agent in this patient? things and walked out, but when he got to the
garage, he could not open the car door. Physi-
cal examination findings are normal except
for 0/5 strength in his left arm and 2+ deep-
tendon reflexes. Sensation is intact. CT scan of
the head is negative for acute bleeding. What
is the most likely cause of his sudden arm
weakness?
(A) Conversion disorder
(B) Factitious disorder
(C) Hypochondriasis
(D) Malingering
(E) Stroke
Full-Length Exams

19. A 4-year-old boy with a history of mental retar-


Courtesy of the Centers for Disease Control and Prevention.
dation and seizures is brought to the physician
with a three-month history of worsening short-
(A) Gram-negative bacilli, lactose fermenter ness of breath. During physical examination,
(B) Gram-negative coccobacilli, grows on the physician notices numerous acne-like pap-
chocolate agar with factors V and X ules on the patient’s face. Echocardiography
(C) Gram-positive bacilli, facultative intracel- shows significant left ventricular outflow ob-
lular struction. Which of the following is the most
(D) Gram-positive cocci, a-hemolytic, opto- likely diagnosis for this patient’s heart condi-
chin-sensitive, bile-soluble tion?
(E) Gram-positive cocci, b-hemolytic, bacitra- (A) Dilated cardiomyopathy
cin-resistant (B) Lipoma
(C) Myxoma
17. A 34-year-old woman at 26 weeks’ gestation (D) Rhabdomyoma
presents with severe abdominal pain, jaundice, (E) Transposition of the great vessels
ascites, and mental status changes. She has a
history of multiple spontaneous abortions.
Test Block 1

20. A 26-year-old man presents with left eye pain


Ultrasonography reveals a blockage in the he- and intermittent double vision. When at rest,
patic venous connection to the inferior vena his left eye is deviated downwards and laterally,
cava and absence of any waveform in the he- as pictured in the image. Upward gaze and ad-
patic veins. She has a positive serum antiphos- duction are limited in the affected eye; how-
Test Block 1  •  Questions 495

ever, abduction appears intact. Also, the left lid reveal a gram-positive, catalase-negative organ-
droops, and the left pupil is dilated and unre- ism. Which of the following culture conditions
sponsive to light. Which of the following cra- would aid in identifying the most likely single
nial nerves is most likely to have been injured? causative organism?
(A) Absence of colonies in the presence of
penicillin
(B) Growth in 6.5% sodium chloride
(C) Growth of colonies in the presence of op-
tochin
(D) Growth on chocolate agar
(E) Soluble in bile

23. A researcher is designing an in vitro experi-


mental system to study the kinetics of GLUT
Reproduced, with permission, from USMLERx.com.
4-mediated glucose transport into mammalian
cells of insulin-dependent organs. The system
(A) Abducens nerve will measure radiolabeled glucose levels in cell
(B) Oculomotor nerve culture media both before and at intervals after
(C) Optic nerve the addition of insulin. Which of the following
(D) Trigeminal nerve cell types is the best choice for use in this ex-
(E) Trochlear nerve perimental system?
(A) Adipocytes
21. A 61-year-old man with a medical history of (B) Cortical neurons
cancer presents with a two-week history of (C) Erythrocytes
constant and severe headaches that are most (D) Hepatocytes
severe when he wakes up in the morning. He (E) Pancreatic b cells
also notes changes in vision associated with the

Full-Length Exams
headaches. Physical examination shows a heal- 24. Myasthenia gravis is an autoimmune disorder
ing ecchymotic lesion on the right forearm, that affects approximately 3 in 100,000 people.
papilledema in the left eye, a right-sided pro- Individuals with mysasthenia gravis classically
nator drift, and weakness of the right arm. The present with complaints of muscle weakness
diagnosis of an intracranial hemorrhage is con- and fatigue secondary to the formation of au-
firmed with CT of the head. Which of the fol- toantibodies directed against the acetylcholine
lowing cancers is most likely to have resulted receptors at neuromuscular junctions. The
in this patient’s presentation? most accurate method of diagnosis involves the
(A) Angiosarcoma detection of these autoantibodies. On average,
(B) Basal cell carcinoma this test is approximately 80% sensitive and
(C) Colorectal carcinoma 90% specific. If an individual has a positive test
(D) Melanoma for autoantibodies against the acetylcholine re-
(E) Prostate cancer ceptor, what is the approximate posttest proba-
bility of having this disease, assuming a pretest
22. A 24-year-old man is brought to the emer- probability of 50%?
gency department by his mother because of a (A) 80%
change in mental status. Physical examination
Test Block 1

(B) 85%
reveals a febrile, dysarthric patient with retinal (C) 89%
hemorrhages and numerous crusted puncture (D) 95%
marks on his left forearm. Auscultation at the (E) 99%
apex reveals a new murmur. Blood cultures
496 Section III: Full-Length Examinations  •  Questions

25. A 55-year-old alcoholic woman presents to (A) Collagen


the emergency department because of bloody (B) Fibrillin
emesis. She woke up that morning cough- (C) Hemoglobin
ing and retching, and later vomited; she had (D) Macrophage colony-stimulating factor
a few more episodes of emesis throughout the (E) Nitric oxide
day, with the most recent episode contain-
ing blood-tinged vomitus. The patient denies 27. An 82-year-old woman with chronic obstruc-
any history of bloody emesis, gastroesophageal tive pulmonary disease is hospitalized for an
reflux, or ascites. She denies trouble swallow- acute exacerbation and is placed on ventila-
ing or recent weight loss. She does complain tory support. During the course of her hospi-
of some epigastric pain with inspiration. Physi- talization, she becomes febrile with an associ-
cal examination is largely benign except for ated elevation in her WBC count. X-ray of the
some tenderness in the epigastric area. She chest shows a new infiltrate in the right lower
continues to periodically vomit bright-red con- lobe. Sputum studies of the infiltrate show that
tent; she undergoes an endoscopic evaluation. it is the gram-negative bacillus Acinetobacter
Which of the following is a finding consistent baumannii. She is treated with intravenous
with this patient’s clinical presentation? (IV) antibiotics. Several hours after starting IV
antibiotics, the patient has a seizure that lasts
(A) Esophageal mass
45 seconds. A subsequent lumbar puncture is
(B) Esophageal stricture
negative. Which of the following antibiotics
(C) Gallbladder inflammation and perichole-
would most likely cause this effect?
cystic fluid accumulation
(D) Partial-thickness tears near gastroesopha- (A) Gentamicin
geal junction (B) Imipenem
(E) Transmural esophageal tear (C) Levofloxacin
(D) Linezolid
26. A 30-year-old man is involved in a motorcycle (E) Rifampin
accident and is pronounced dead on arrival at
Full-Length Exams

the emergency department. A cross-section of 28. A 23-year old woman presents to the physi-
his coronary arteries at autopsy is shown in the cian with a chief complaint of an excruciating,
image. Under normal circumstances, which sharp pain in her right upper quadrant and
substance is produced in the innermost cells fever. Complete physical examination is per-
of the layer (ie, the layer closest to the blood) formed. Pelvic exam reveals bilateral adnexal
where these plaques develop? and cervical motion tenderness and purulent
discharge. Other laboratory studies reveal:
WBC count: 14,200/mm³
Amylase: 28 U/L
Aspartate aminotransferase: 19 U/L
Alanine aminotransferase: 17 U/L
Alkaline phosphatase: 23 U/L
Hepatitis A IgG antibody: positive
Which of the following is the most likely diag-
nosis?
(A) Acute cholecystitis
Test Block 1

(B) Chlamydial cervicitis


(C) Fitz-Hugh-Curtis syndrome
(D) Gonococcal urethritis
(E) Hepatitis A
Reproduced, with permission, from USMLERx.com. (F) Tubo-ovarian abscess
Test Block 1  •  Questions 497

29. A 60-year-old woman with a 25-year history of (A) Adult-onset asthma


type 2 diabetes mellitus presents with pruritus, (B) Amiodarone
diffuse bone pain, and proximal muscle weak- (C) Diltiazem
ness. Laboratory studies show a serum calcium (D) Sotalol
level of 6.5 mg/dL, serum phosphate of 6.0 (E) Tobacco
mg/dL, serum creatinine of 2.7 mg/dL, and in-
tact parathyroid hormone of 300 pg/mL (nor- 31. While working in a rural village in central
mal: 10-65 pg/mL). The laboratory findings in Mexico, a volunteer physician encounters a
this patient are most likely due to which of the 7-year-old girl who presents with a 1-week his-
following conditions? tory of jaundice. The patient’s mother reports
that, for the past few weeks, the girl has not
(A) Parathyroid adenoma
eaten well and that she has often felt nause-
(B) Parathyroid insufficiency
ated and has vomited after the few meals that
(C) Renal failure
she has eaten. A few days ago, the girl’s urine
(D) Underlying malignancy
darkened and her stool became pale. On phys-
(E) Vitamin D intoxication
ical examination, the physician notes a fever
of 38.5°C (101.3°F), hepatomegaly, jaundice,
30. A 73-year-old man diagnosed with atrial fibril-
and icterus. A liver enzyme panel reveals an
lation has been treated pharmacologically for
alanine aminotransferase level of 10,103 IU/
the past 10 years. He presents to his primary
ml and an aspartate aminotransferase level
care physician complaining of shortness of
of 8030 IU/ml. The patient and her mother
breath and gasping. Pulmonary function tests
deny any illicit drug use or sexual contacts or
show that the forced expiratory volume in
abuse of the patient. The mother also reports
1 second (FEV1) and the forced vital capac-
that 1 month ago, two of the girl’s playmates
ity (FVC) are each <70% of predicted values,
had similar symptoms. Which of the follow-
with an FEV1:FVC ratio of 81%. The flow-
ing pathogens shares the route of transmission
volume curve is shown in the image. Which
as the pathogen most likely causing this girl’s
of the following is the most likely cause of this
symptoms?

Full-Length Exams
patient’s clinical findings?
(A) Flavivirus
(B) Hepatitis B virus
(C) Poliovirus
5 (D) Rabies virus
(E) Varicella-zoster virus
4
FEF25–75
Volume, L

3
FVC
FEV1
2

0
0 1 2 3 4 5 6

Reproduced, with permission, from USMLERx.com.


Test Block 1
498 Section III: Full-Length Examinations  •  Questions

32. A 35-year-old man is brought to the emergency 34. A 3-year-old boy comes to the physician be-
department by ambulance after having a tonic- cause of fever and erythema in his conjunc-
clonic seizure at work. The patient reports that tivae, oral mucosa, palms, and soles for the
he has always been healthy and has never had past week. Physical examination is significant
a seizure before. On further questioning, the for fever, enlarged cervical lymph nodes, and
patient reports that he has been having inter- edema of the hands and feet. Although the
mittent bloody stools for the past four months. precise cause of the patient’s disease is un-
CT of the head reveals an irregular 3-cm × known, it is speculated that autoantibodies
4-cm mass extending from the right to the left may play a role. Based on the known structure
hemisphere. CT of the abdomen shows mul- that is primarily affected in the patient’s dis-
tiple polypoid masses in the sigmoid colon. ease, what autoantibodies are suspected to be
Which of the following is the most likely diag- associated with this condition?
nosis?
(A) Anti-endothelial cell antibodies
(A) Familial adenomatous polyposis (B) Anti-IgG antibodies
(B) Gardner syndrome (C) Anticentromere antibodies
(C) Hereditary nonpolyposis colorectal carci- (D) Antihistone antibodies
noma (E) Antinuclear antibodies
(D) Tuberous sclerosis
(E) Turcot syndrome 35. A 6-year-old boy arrives at the emergency de-
partment breathing rapidly and complaining
33. A 24-year-old previously healthy woman in the of tinnitus and nausea. His parents reported he
final stages of labor suddenly becomes short of had swallowed half a bottle of aspirin acciden-
breath. She becomes hypotensive and begins tally. The emergency department physician
to lose large volumes of blood from her vagina. decides to administer a medication that alters
Moments later, small ecchymoses appear on the pH of the boy’s urine to improve excretion
her legs and near her intravenous site. Which of the drug. How does altering the pH of the
of the following is this woman’s most likely co- urine improve the excretion of aspirin?
Full-Length Exams

agulation profile?
(A) Acidification of urine increases the glo-
merular filtration of salicylate molecules
Choice Platelet Bleeding Prothrombin
Partial
thromboplastin
(B) Acidification of urine ionizes salicylate
count time time time molecules, trapping them in the proximal
A normal normal normal
tubule
B normal normal normal
(C) Acidification of urine neutralizes ionized
C normal normal normal salicylate molecules in the proximal tu-
D normal normal bule
E normal normal (D) Alkalinization of urine increases the glo-
F merular filtration of salicylate molecules
(E) Alkalinization of urine ionizes salicylate
Reproduced, with permission, from USMLERx.com. molecules, trapping them in the proximal
tubule
(F) Alkalinization of urine neutralizes ionized
(A) A salicylate molecules in the proximal tu-
(B) B bule
(C) C
Test Block 1

(D) D
(E) E
(F) F
Test Block 1  •  Questions 499

36. A 45-year-old man visited his primary care does this disease affect the pressures governing
physician one month ago because of chest the flow of fluid across the glomeruli?
pain that he had experienced four times in the
(A) Bowman space hydrostatic pressure will be
past four months. The onset of the pain is sud-
decreased
den and radiates to his left jaw. He has experi-
(B) Bowman space hydrostatic pressure will be
enced this pain while watching television but
increased
has never felt it during exercise. During last
(C) Bowman space oncotic pressure will be de-
month’s visit, the physician prescribed sublin-
creased
gual nitroglycerin, and the patient reports that
(D) Glomerular capillary hydrostatic pressure
this has shortened the duration of his episodes.
will be increased
Last month’s ECG is shown in the image.
(E) Glomerular capillary oncotic pressure will
Which of the following is the most likely cause
be decreased
of this patient’s chest pain?
38. A 62-year-old woman who has been on dialy-
R sis for end-stage renal failure for six months
comes to the physician’s office. She is accom-
panied by her 45-year-old daughter, who is
P T
present because of their close relationship.
During the visit, her daughter hears about the
need for transplantation for the first time, and
Systole Diastole is visibly surprised, but declares that they will
Reproduced, with permission, from USMLERx.com. do “anything it takes.” After the physician and
the women discuss the possible options, the
patient turns to her daughter and asks her to be
(A) Myocardial infarction the donor. For which of the following reasons
(B) Pericarditis should consent be obtained from the daughter
(C) Prinzmetal’s angina at a different time?

Full-Length Exams
(D) Stable angina (A) More of the family should be involved in
(E) Unstable angina the discussion
(B) The daughter cannot have a full under-
37. A 7-year-old boy presents to the physician with
standing of the procedure
acute-onset edema and facial swelling. Dip-
(C) The daughter is not free from coercion
stick urinalysis reveals 4+ proteinuria. Renal
(D) The emergent nature of the patient’s clini-
biopsy shows no appreciable changes under
cal symptoms requires immediate action
light and fluorescence microscopy, but elec-
(E) The patient is not competent to make
tron microscopy demonstrates glomerular epi-
health decisions due to her condition
thelial cell foot process effacement. A diagno-
sis of minimal change disease is made. How
Test Block 1
500 Section III: Full-Length Examinations  •  Questions

39. A 67-year-old obese man complains of swell- (C) 100


ing in his hands, feet, and face. Physical ex- (D) 270
amination is significant for 3+ edema in his (E) 630
lower extremities and 1+ edema in his hands
and around his eyes. Urine dipstick reveals 3+ 41. A 35-year-old man presents to the physician
protein and no blood. Renal biopsy is shown in with a two-month history of non-bloody, non-
the image. In addition to the underlying cause mucoid, non-oily watery diarrhea. He has a dia-
of his renal disease, which of the following stolic murmur that gets louder with inspiration
comorbidities is most likely present in this pa- and is best heard over the left lower sternal
tient? border. His face is warm and appears to be en-
gorged with blood for several minutes during
the examination. His laboratory studies show
the following:
Vanillylmandelic acid: 5 mg/day (normal 0-7
mg/day)
Metanephrine, urine: 250 μg/g of creatinine
(normal 0-300 μg/g)
Homovanillic acid, urine: 14 mg/day (normal
0-15 mg/day)
5-HIAA: 28 mg/day (normal 0-9 mg/day)
Gastrointestinal endoscopy is most likely to
show a lesion located near which of the follow-
ing?
(A) Gastroesophageal junction
Reproduced, with permission, from USMLERx.com (B) Ligament of Treitz
(C) Pancreaticoduodenal junction
Full-Length Exams

(D) Rectosigmoid junction


(A) Arthritis
(E) Splenic flexure
(B) Bone pain
(C) Coronary artery disease 42. A 46-year-old man comes to his physician af-
(D) Hearing loss ter visiting his optometrist. While being fitted
(E) Hemoptysis for new glasses, the patient was found to have
(F) Pharyngitis persistent constriction of his right pupil. Af-
ter a thorough history and physical examina-
40. A town with 1000 citizens has a 10% preva-
tion, x-ray of the chest was ordered, and results
lence of disease X. A screening test for disease
are shown in the image. In which ganglion is
X just came out, with a sensitivity of 80% and a
there evidence of decreased sympathetic syn-
specificity of 70%. How many people with dis-
apse activity?
ease X will be missed by this screening test?
(A) 20
(B) 80
Test Block 1
Test Block 1  •  Questions 501

16 mEq/L, and a serum anion gap of 22 (nor-


mal: 7-16). Intravenous fluids and insulin are
administered. Measurement and management
of which of the following electrolytes are most
critical in this patient?
(A) Bicarbonate
(B) Calcium
(C) Chloride
(D) Potassium
(E) Sodium

45. A 51-year-old man presents to the emergency


department (ED) 30 minutes after experienc-
Reproduced, with permission, from Hanley ME and Welsh ing difficulty speaking and moving the left side
CH. Current Diagnosis & Treatment in Pulmonary Medicine, of his body. The patient is alert and oriented
New York: McGraw-Hill, 2003; Fig. 3-8. to person and place. His wife states that he has
a history of benign prostatic hypertrophy and
high blood pressure. According to the wife, the
(A) Inferior cervical ganglion patient has never experienced symptoms like
(B) Sphenopalatine ganglion this before and has never had surgery of any
(C) Superior cervical ganglion type. The ED physicians determine that the
(D) Superior mesenteric ganglion patient is hemodynamically stable and an ini-
(E) T4 dorsal root ganglion tial CT scan is normal. Which of the following
is the best next step in management?
43. A 38-year-old white woman presents to the
physician with a two-week history of ach- (A) Echocardiogram
ing pain in her left calf that is made worse by (B) Hemicraniectomy
dorsiflexion of her foot. On physical examina- (C) Heparin

Full-Length Exams
tion, her left calf is found to be erythematous, (D) Insulin
warm, and swollen. Which of the following (E) Tissue plasminogen activator
measures should she take to decrease similar
problems in the future? 46. A 3-year-old girl presents to the pediatrician
for her annual well child visit. She is develop-
(A) Begin taking a bile acid resin ing well and her mother has no concerns at
(B) Begin taking a statin this time. Her temperature is 38°C (100.4°F).
(C) Begin taking low-dose oral contraceptives Physical examination reveals an enlarged, ten-
(D) Exercise 30 minutes three times per week der, erythematous left axillary lymph node.
(E) Quit smoking Close inspection of the skin reveals a series of
(F) Reduce alcohol consumption to one or small linear scratches on the left forearm, with
two glasses of red wine per week a nearby erythematous papule. Which of the
following organisms is the most likely cause of
44. A 10-year-old boy is brought to the emergency these findings?
department after his parents noted that he was
acting confused and lethargic following several (A)
Bartonella henselae
bouts of nausea and vomiting. Upon arrival the (B)
Borrelia burgdorferi
Test Block 1

patient is tachycardic and is breathing deeply (C)


Eikenella corrodens
and slowly. Laboratory studies are remarkable (D)
Francisella tularensis
for a serum pH of 7.21, a serum glucose level (E)
Pasteurella multocida
of 700 mg/dL, a serum bicarbonate level of
502 Section III: Full-Length Examinations  •  Questions

47. A 35-year-old man presents to his primary 48. A 12-year-old boy who recently emigrated from
care physician with a chief complaint of pal- Nigeria presents with a six-month history of in-
pitations and occasional chest pain. Further termittent fever, fatigue, and night sweats. He
questioning reveals a recent history of weight has a large mass on his right mandible. Biopsy
loss, diarrhea, and heat intolerance. Labora- of the mass shows an interspersed pattern of
tory evaluation shows anti-thyroid-stimulating macrophages with sheets of lymphoblasts. The
hormone (TSH) receptor antibodies in the pathogen associated with this patient’s presen-
patient’s serum. Which of the following best tation also is associated with which of the fol-
describes this patient’s TSH and thyroid hor- lowing conditions?
mone levels relative to normal baseline values?
(A) Cervical adenocarcinoma
(B) Gastric adenocarcinoma
Thyroid-
Total Free
(C) Hepatocellular carcinoma
Choice stimulating
hormone
thyroxine thyroxine (D) Heterophile-negative mononucleosis
(E) Nasopharyngeal carcinoma
A

Reproduced, with permission, from USMLERx.com.

(A) A
(B) B
(C) C
(D) D
Full-Length Exams

(E) E
Test Block 1
Test Block 1  •  Answers 503

An s w e r s

1. The correct answer is E. This question is ask- Answer D is incorrect. Lowering a patient’s
ing about the basic physiology behind the temperature shifts the oxygen-hemoglobin
oxygen-hemoglobin dissociation curve, which curve to the left reducing oxygen availability.
shows how much oxygen is bound to hemo-
globin at a given partial pressure of oxygen 2. The correct answer is A. Phenoxybenzamine
under normal conditions. Under abnormal is a nonselective a-antagonist that will block
conditions the curve can shift to the left, indi- both a1- and a2-receptors. In this patient,
cating increased affinity for oxygen, or shift to the administration of high-dose epinephrine
the right, indicating decreased affinity (lower (which is both an a- and a b-agonist) would
percentage of bound oxygen at a given partial result in unopposed b1- (increased heart rate,
pressure of oxygen). Having a decreased affin- increased contractility) and b2- (vasodilation,
ity means that it may take higher partial pres- bronchodilation) agonist effects because the
sures to bind four oxygen molecules to one a-effects of epinephrine are blocked by prior
molecule of hemoglobin, but in the periphery phenoxybenzamine administration. The net
hemoglobin is quicker to release oxygen mole- effect will be b-agonist effects, including an
cules. Therefore shifting the curve to the right increase in heart rate and a decrease in blood
would increase peripheral oxygen availability. pressure.
pH has a direct relationship with oxygen affin- Answer B is incorrect. Unopposed b-agonist
ity. Therefore raising partial pressure of arterial effects will cause an increase in heart rate be-
carbon dioxide, which translates into decreas- cause b1-agonists result in increased heart rate.
ing pH, in principle ought to increase periph-
eral oxygen availability. Answer C is incorrect. Unopposed b-agonist
effects will cause a decrease in blood pressure
Answer A is incorrect. Norepinephrine is because b2-agonists result in vasodilation.
clinically used in shock to maintain a pa-
Answer D is incorrect. Unopposed b-agonist

Full-Length Exams
tient’s blood pressure by inducing peripheral
vasoconstriction. It maintains blood flow to effects do not have a significant effect on respi-
the brain at the expense of the some organs ratory rate.
and peripheral tissue. At high doses, periph- Answer E is incorrect. Unopposed b-agonist
eral vaso­constriction is so severe that cyanotic effects will cause changes in both blood pres-
fingers may be seen. Therefore while norepi- sure and heart rate.
nephrine has no direct effect on hemoglobin,
it decreases peripheral oxygen availability. 3. The correct answer is C. The upper-extremity
Answer B is incorrect. Hydroxyurea is an anti- hypertension and weak pedal and popliteal
tumor agent that is also used in sickle cell dis- pulses suggest coarctation of the aorta. Coarc-
ease. It actually raises fetal hemoglobin levels. tation of the aorta typically is a discrete nar-
Higher fetal hemoglobin levels raise oxygen rowing of the thoracic aorta just distal to the
affinity which decreases peripheral availability. left subclavian artery. The major clinical find-
ing in patients with coarctation of the aorta is
Answer C is incorrect. 2,3-Bisphosphoglyc- a difference in systolic blood pressure between
erate (BPG) has a inverse relationship with the upper and lower extremities. Coarcta-
oxygen affinity. It is a by-product of glycolysis. tion of the aorta is sometimes associated with
Test Block 1

High levels of 2,3-BPG decrease oxygen affin- Turner syndrome. Other common features
ity. Therefore inhibiting its synthesis will in- of Turner syndrome include short stature, a
crease oxygen affinity and make hemoglobin webbed neck, streaked ovaries, and primary
less likely to release oxygen at a given partial amenorrhea. The genetic makeup of a person
pressure of oxygen in the periphery. with Turner syndrome is XO.
504 Section III: Full-Length Examinations  •  Answers

Answer A is incorrect. Trisomy 18 is Ed- teins, with the light chains readily excreted.
wards syndrome. These children have severe Excessive serum levels of immunoglobulin can
mental retardation, “rocker bottom” feet, and lead to nephropathy or amyloidosis in these pa-
clenched hands (ie, flexion of fingers). Al- tients.
though patients with Edwards syndrome do
Answer A is incorrect. Osteogenesis imper-
have congenital heart disease, it generally is
fecta results from a defect in the synthesis
not associated with coarctation of the aorta.
of collagen I and leaves patients susceptible
Answer B is incorrect. Trisomy 21 also is to pathologic fractures. In addition to brittle
known as Down syndrome. These patients bones, insufficient collagen I can lead to ab-
have mental retardation, prominent epicanthal normal dentition, conductive hearing loss,
folds, and congenital heart disease (most often and a bluish hue in the sclera. However, this
atrial septal defects). They typically do not suf- disease does not first present in late adulthood,
fer from coarctation of the aorta. and would not be associated with lytic bone le-
sions.
Answer D is incorrect. XXY is the karyotype
in Klinefelter syndrome, which is not associ- Answer B is incorrect. This is a description
ated with coarctation of the aorta. Klinefelter of Chvostek sign, which is a manifestation of
syndrome manifests in phenotypic males as tes- neuromuscular hyperexcitability. This would
ticular atrophy, androgenous body shape, long be expected in a person with symptomatic hy-
extremities, and gynecomastia. pocalcemia, not in a patient with hypercalce-
mia.
Answer E is incorrect. XYY is the karyotype in
“double Y” males. The overwhelming majority Answer C is incorrect. Although adenocarci-
of XYY males have normal phenotype and are noma of the prostate commonly metastasizes
unaware of their chromosomal status. Never- to bone, it is an unlikely cause of this patient’s
theless, tall stature, acne, learning difficulties, symptoms. Bone metastases from the prostate
and a tendency toward aggressive behavior are tend to produce dense and sclerotic osteoblas-
all associations drawn on the USMLE. The as- tic bone lesions, rather than the lytic lesions
Full-Length Exams

sociation between XYY and antisocial behav- seen on the radiograph. Moreover, one would
ior is somewhat contentious if not apocryphal expect an elevated alkaline phosphatase level
(prior studies suggesting a propensity toward in the setting of osteoblastic metastases.
violent criminal behavior have not been sub-
Answer D is incorrect. Parathyroid adenomas
stantiated). In any case, this karyotype is not as-
are a common cause of hyperparathyroidism
sociated with coarctation of the aorta.
and consequent hypercalcemia. However, al-
kaline phosphatase levels should be markedly
4. The correct answer is E. This patient is suffer-
elevated in hyperparathyroidism. Moreover,
ing from multiple myeloma. The x-ray shows
parathyroid neoplasms are rarely malignant,
multiple lytic “punched out” lesions in the
and widespread skeletal metastases would not
skull, which is a classic finding for this plasma
be expected in a patient with a parathyroid ad-
cell malignancy. Other areas of the skeleton
enoma.
commonly infiltrated by myeloma cells in-
clude the vertebrae, ribs, and pelvis, explaining
5. The correct answer is A. It is appropriate to
the patient’s fracture in the absence of overt
discuss the patient’s concerns. Explaining the
trauma. The lytic bone lesions are responsible
thought process behind your diagnosis and ed-
for this patient’s hypercalcemia in the setting
ucating about the disease process may alleviate
Test Block 1

of normal alkaline phosphatase levels. Uri-


the patient’s fears, and make him more com-
nary protein (termed Bence-Jones proteins in
fortable with your proposed course of action. A
this setting) is a common finding in multiple
discussion of the utility as well as the risks of
myeloma because the neoplastic plasma cells
CT of the head may help the patient under-
secrete an abundance of immunoglobulin pro-
stand your perspective.
Test Block 1  •  Answers 505

Answer B is incorrect. This action violates Answer A is incorrect. Intraductal papillomas


the principle of non-maleficence because, are benign solitary lesions that line the lactifer-
although less risky than a CT scan, x-ray files ous ducts. These masses present in premeno-
are associated with a radiation dose and are un- pausal women with serosanguinous (serous
likely to be of any benefit. fluid and/or blood) and unilateral nipple dis-
charge. Usually the mass undergoes cytology
Answer C is incorrect. It is inappropriate to
to rule out invasive papillary carcinoma due
send this patient for CT of the head if a sub-
to the bloody discharge associated with both.
arachnoid hemorrhage is not being enter-
These benign masses rarely undergo malignant
tained in the differential diagnosis. Unfortu-
transformation and are treated with ice packs,
nately, many tests are performed upon patient
the cessation of breast-feeding, and tight-fitting
request despite the associated risks, simply be-
support bras. If the mass does not subside in
cause it is less time consuming to order the test
a couple weeks, then excision is warranted to
than to explain why it is unwarranted. Order-
avoid abscess formation.
ing CT of the head in this case would violate
the principle of non-maleficence because the Answer C is incorrect. Paget disease of the
risks outweigh the benefits. breast is a rare manifestation of breast cancer
that presents unilaterally with eczematous skin
Answer D is incorrect. It is important to be
findings associated with underlying ductal
compassionate by attempting to understand
carcinomas. The eczematous nipple is due to
and alleviate patients’ concerns rather than to
the tumor cells disrupting the tight junctions.
pass the responsibility on to someone else.
Paget cells (cells of the underlying ductal car-
Answer E is incorrect. Simply stating that he cinoma) are large cells with halo-like clearings.
does not have a subarachnoid hemorrhage fails
Answer D is incorrect. Infiltrating lobular
to address the patient’s concerns and may ul-
carcinomas are identified as irregular masses
timately lead to increased patient anxiety. It’s
on palpation or serendipitously on mammog-
also unlikely to change the patient’s desire for
raphy. These cells are found in clusters or in
a CT scan.
a linear formation; the histological hallmark

Full-Length Exams
Answer F is incorrect. It is never appropriate is a pattern of monomorphic infiltrating cells
to lie to a patient. usually only one cell wide. These masses are
more common in postmenopausal women and
6. The correct answer is B. Fibroadenomas present like invasive ductal carcinoma, so they
are the most common benign breast tumors must be differentiated with excisional biopsy
and usually occur in young women 20-35 for cytology.
years old. They present as small, firm, mobile
masses. They are not associated with malig- Answer E is incorrect. Invasive ductal car-
nancy progression. On histology, fibrosing in- cinoma (nonspecific type) is the most com-
terlobular stroma is seen around normal duct mon breast mass in older women. It usually
and gland structures. Fibroadenomas are fre- becomes clinically apparent when there are
quently single, well-circumscribed, rubbery, associated overlying skin changes such as pig-
and painless masses. They are hormone re- mented dimpling (due to the carcinoma im-
sponsive during the menstrual cycle and often pinging on the suspensory ligaments [also
become hyalinized and can calcify, mimicking referred to as peau d’orange skin]) and/or uni-
breast carcinomas on mammography. A stable lateral bloody nipple discharge. On palpation,
fibroadenoma in a young woman is usually fol- they are usually firm, irregular, fixed masses;
Test Block 1

lowed by ultrasonography; however, cytology on mammography, they appear as multiple


is indicated if there is any doubt about malig- small calcifications.
nancy or if mass is growing in size.
7. The correct answer is D. This patient has
chronic granulomatous disease (CGD), which
506 Section III: Full-Length Examinations  •  Answers

usually presents with an increased susceptibil- lymphocytes. Severe combined immunodefi-


ity to opportunistic bacterial and fungal in- ciency is a defect in early differentiation of im-
fections. It results from defective neutrophil mune cells. Patients usually have a low num-
phagocytosis due to a lack of NADPH oxidase ber of T lymphocytes and a normal number of
(or similar enzyme) activity. A decrease in free B lymphocytes.
radical production because of this enzyme de-
Answer E is incorrect. A positive nitroblue
ficiency renders host neutrophils sensitive to
tetrazolium dye reduction test is present in
catalase-producing organisms (notably Staphy-
patients with normal neutrophils, which are
lococcus aureus, Candida albicans, Aspergillus
able to reduce the nitroblue tetrazolium. This
flavus, Escherichia coli, and Pseudomonas aeru-
causes a dark blue precipitate in their cyto-
ginosa), because catalase breaks down free rad-
plasm.
icals. To confirm the diagnosis, nitroblue tetra-
zolium (a yellow liquid) is added to a sample 8. The correct answer is C. This patient suf-
of the patient’s blood. In patients with normal fers from atrial fibrillation, as demonstrated
levels of NADPH oxidase, there is a positive by the absence of P waves on ECG, which
result as the neutrophils reduce the nitroblue places him at higher risks for mural throm-
tetrazolium and a dark blue granular substance bosis and embolic complications. Warfarin is
precipitates in their cytoplasm. Patients with the anticoagulant of choice. Warfarin inhib-
CGD have a negative result: the cytoplasm of its γ-carboxylation of vitamin K-dependent
their neutrophils remains colorless despite the clotting factors II, VII, IX, and X and proteins
addition of nitroblue tetrazolium. C and S, and is used for chronic anticoagula-
Answer A is incorrect. Large lysosome vesicles tion. It is taken orally and has a long half-life.
in neutrophils are characteristic of Chédiak- Measurement of the degree of anticoagulation
Higashi disease, an autosomal recessive condi- must be followed by measurement of the Inter-
tion that presents with recurrent streptococcal national Normalized Ratio (INR).
and staphylococcal infections. A defect in lyso- Answer A is incorrect. Heparin works by cata-
somal emptying of phagocytic cells due to mi- lyzing the activation of antithrombin III, de-
Full-Length Exams

crotubular dysfunction is the underlying cause creasing the level of available thrombin, and
of the disease. inhibiting factor Xa. Heparin has a short half-
Answer B is incorrect. Low IgM levels with life and can be monitored by using the partial
elevated IgE and IgA levels are characteris- thromboplastin time (PTT). Heparin is used
tic of Wiskott-Aldrich syndrome, an X-linked commonly in the in-patient setting to treat
disorder resulting in a cytoskeletal defect that thrombosis or to anticoagulate symptomatic
affects immune cells and reduces the body’s patients before bridging them to warfarin; how-
ability to mount an IgM response to bacte- ever, it is not used in the long-term manage-
ria. Recurrent pyogenic infections, eczema, ment of atrial fibrillation.
and thrombocytopenia are the typical triad of Answer B is incorrect. Infliximab is a mono-
symptoms. Wiskott-Aldrich syndrome does not clonal antibody to tumor necrosis factor-a,
present with any specific enzyme abnormality. thereby interfering with inflammation. It is
Answer C is incorrect. Low levels of IgM, IgE, used to treat many chronic inflammatory/
and IgA are characteristic of Bruton agamma- rheumatologic conditions including refractory
globulinemia and of the severe combined im- Crohn disease, rheumatoid arthritis, and anky-
munodeficiency syndromes. In both instances, losing spondylitis, and to promote fistula heal-
Test Block 1

levels of all immunoglobulin isotypes are de- ing.


creased. Bruton agammaglobulinemia is an Answer D is incorrect. Aspirin works by ir-
X-linked defect in a tyrosine kinase involved reversibly inhibiting cyclooxygenase, thereby
in B lymphocyte development. Patients have preventing the conversion of arachidonic acid
normal T lymphocytes and a low number of B to prostaglandins. The four effects of aspirin
Test Block 1  •  Answers 507

are antiplatelet, antipyretic, analgesic, and Answer C is incorrect. Decreased respiratory


anti-inflammatory. Aspirin is a powerful anti- rate (hypoventilation) causes excess carbon
platelet agent; however, it is not used routinely dioxide in the blood, leading to production of
in the management of atrial fibrillation unless acid. A respiratory alkalosis would result from
the patient is low risk based on the 2006 Amer- hyperventilation.
ican Heart Association guidelines.
Answer D is incorrect. The mechanism of
Answer E is incorrect. Clopidogrel acts by RBC damage in PNH is through the comple-
inhibiting platelet aggregation by irreversibly ment cascade. Other hemolytic anemias are
inhibiting the ADP pathway involved in the due to osmotic lysis of RBCs, including heredi-
binding of fibrinogen to the platelet surface. tary spherocytosis.
It is used in the setting of acute coronary syn-
Answer E is incorrect. Respiratory rate does
drome and stenting.
not increase while sleeping; it decreases.
9. The correct answer is A. This patient presents Answer F is incorrect. Respiratory rate does
with signs and symptoms of anemia. The dark not increase while sleeping; it decreases.
urine is due to the presence of hemoglobin,
which occurs only in the setting of intravas- 10. The correct answer is C. The image re-
cular hemolysis. Urine color change points to veals Mycobacterium tuberculosis infection.
paroxysmal nocturnal hemoglobinuria (PNH), Although rifampin is considered the best
a rare form of hemolytic anemia. We would anti-­
tuberculous agent, isoniazid is used for
expect laboratory abnormalities, including a prophylaxis in asymptomatic patients with a
normocytic anemia, elevated unconjugated positive PPD. A six-month course of isoniazid
bilirubin and LDH, and low haptoglobin. prevents active tuberculosis (TB) in 90% of pa-
PNH classically presents as the triad of hemo- tients for at least 20 years. Isoniazid blocks my-
lytic anemia, pancytopenia, and thrombosis. colic acid cell wall synthesis and is bactericidal
In PNH, a defective protein known as glyco- for rapidly multiplying organisms. Vitamin B6
sylphosphatidylinisotol (GPI) anchor (encoded is given with isoniazid to prevent neurotoxicity,

Full-Length Exams
by the PIG-A gene) is present on the RBC an adverse effect of isoniazid therapy.
membrane. Normally, GPI attaches proteins Answer A is incorrect. Cycloserine is a broad-
to the surface of RBCs and prevents the at- spectrum antibiotic active against Mycobac-
tachment of complement to the membrane terium tuberculosis. Adverse effects related to
and subsequent lysis. Because complement cycloserine include peripheral neuropathy,
preferentially attaches to the RBC membrane psychosis, and seizures. One of the reasons
at acidic pH, PNH can be diagnosed by mix- cycloserine is so effective is that it spreads
ing the patient’s RBCs and control RBCs with throughout the body, including the cerebrospi-
an acidic solution and observing for increased nal fluid.
hemolysis of the patient’s RBCs. This pH-­
dependent adhesion explains why RBC lysis Answer B is incorrect. Combined isoniazid
happens selectively overnight in those with and rifampin therapy is effective for treating
PNH: While sleeping, the decrease in respira- Mycobacterium tuberculosis. However, the risk
tory rate leads to retained carbon dioxide and of liver toxicity with rifampin outweighs its
thus a slight acidification of the blood. benefits in patients without active infection.
Answer B is incorrect. The mechanism of Answer D is incorrect. Rifampin is the most
RBC damage in PNH is through the comple- potent antituberculous agent available. Rif­
Test Block 1

ment cascade. Other hemolytic anemias are ampin blocks DNA-dependent RNA polym­
due to osmotic lysis of RBCs, including heredi- erase, preventing RNA synthesis. Although it
tary spherocytosis. is a better agent than isoniazid for preventing
active TB infection, it has a significant risk of
508 Section III: Full-Length Examinations  •  Answers

liver toxicity that outweighs its benefits in this 12. The correct answer is E. This patient has
population. Kartagener syndrome, which is caused by a
lack of dynein arms in microtubules in cilia,
Answer E is incorrect. Vitamin B6 is given
rendering them immotile. It results in infertil-
with isoniazid to prevent neurotoxicity, an ad-
ity due to immotile sperm, as well as recurrent
verse effect of isoniazid therapy. The other ma-
sinusitis due to deficient removal of bacteria
jor adverse effect of isoniazid is hepatotoxicity.
and other infectious particles. It is also associ-
Rifampin does not cause peripheral neuropa-
ated with situs inversus, in which the major or-
thy, and thus coadministration of vitamin B6 is
gans are reversed or mirrored from their origi-
unnecessary.
nal locations.
11. The correct answer is C. The uptake of 99mTc Answer A is incorrect. Benign prostatic hyper-
pertechnetate (which is captured by thyroid tis- trophy could cause impairment of ejaculation
sue just as iodine is) in this mass and its sub- by not allowing semen to be expelled from the
lingual position strongly suggest that it is com- body. Because the patient is without an en-
posed of ectopic thyroid tissue. Normally the larged prostate and is only 27 years old, this di-
thyroid diverticulum develops from the floor of agnosis is highly unlikely.
the primitive pharynx and then descends into
Answer B is incorrect. Cystic fibrosis does
the neck. The presence of thyroid tissue at-
cause infertility, but usually because of bilat-
tached to the tongue implies that it has failed
eral absence of the vas deferens, which would
to migrate caudally. The tongue is the most
lead to lack of sperm in semen.
common site of ectopic thyroid tissue.
Answer C is incorrect. Undescended testicles
Answer A is incorrect. The thymus is located
are associated with infertility and an increased
in the anterior mediastinum, deep to the ster-
risk of testicular cancer. It is usually found at a
num. Thymic tissue would not be found in ei-
very young age and resolves by itself or is sur-
ther the oropharynx or the neck. Furthermore,
gically corrected before serious complications
the uptake of 99mTc pertechnetate suggests
occur.
that this mass is composed of ectopic thyroid
Full-Length Exams

tissue. Answer D is incorrect. Familial hypercho-


lesterolemia can cause atherosclerosis of the
Answer B is incorrect. The thymus is not nor-
vessels of the male genitalia, causing erectile
mally found in the neck; it is instead located
dysfunction. Without a history of erectile dys-
in the anterior mediastinum. Thymic hy-
function or elevated lipid levels, this diagnosis
pertrophy would not explain the location of
is highly unlikely.
this mass. Furthermore, the uptake of 99mTc
pertechnetate suggests that this mass is com-
13. The correct answer is C. According to her lab
posed of thyroid tissue.
data, this patient has an acute respiratory alka-
Answer D is incorrect. The thyroid does not losis. Respiratory alkalosis is caused by a loss
migrate rostrally during development. Instead, of carbon dioxide, which is compensated for
it develops near the tongue and migrates cau- by increased renal excretion of bicarbonate.
dally (descends) to its normal position in the The key to this question is to recognize that
lower neck. respiratory alkalosis can be caused only by an
increase in ventilation, which can be caused
Answer E is incorrect. The third and fourth
by low oxygen (in high altitudes) or by sym-
branchial (pharyngeal) arches form the poste-
pathetic stimulation such as anxiety, panic at-
Test Block 1

rior third of the tongue. However, the 99mTc


tack, or pain. This patient is described as anx-
pertechnetate uptake in this mass indicates
ious and presents with severe abdominal pain,
that it is composed of thyroid, and not lingual,
which is most likely the result of acute alcohol-
tissue.
induced pancreatitis. Both the anxiety and the
pain could be causing her to hyperventilate.
Test Block 1  •  Answers 509

Answer A is incorrect. An increase in anions Answer C is incorrect. Hearing is a function


would be consistent with anion-gap metabolic of the vestibulocochlear nerve (cranial nerve
acidosis. Metabolic acidosis is indicated by VIII) rather than the trigeminal nerve.
the presence of a low pH with a low plasma
Answer E is incorrect. Tongue movements are
bicarbonate, a low carbon dioxide, and an in-
controlled by the hypoglossal nerve (cranial
creased anion gap. The anion gap, measured
nerve XII) rather than the trigeminal nerve.
by a formula involving sodium, chloride, and
bicarbonate {[Na+] – ([Cl-] + [HCO3-])}, is 15. The correct answer is E. Benzodiazepines and
normally between 10 and 16 mEq/L. barbiturates differ in their effect on the chlo-
Answer B is incorrect. Diuretic use can cause ride channel adjacent to the γ-aminobutyric
metabolic alkalosis by volume contraction. acid receptor. Benzodiazepines increase the
This causes the kidney to compensate by re- frequency of the channel’s opening, while bar-
absorbing sodium and excreting hydrogen biturates increase the duration the channel is
ions. A metabolic alkalosis would present with open.
elevated pH, elevated carbon dioxide, and el- Answer A is incorrect. Benzodiazepines have
evated bicarbonate. largely replaced barbiturates in the treatment
Answer D is incorrect. Hypoventilation causes of anxiety because of their more favorable
a reduction in pH due to carbon dioxide reten- after-effect profile. They are less likely than
tion. The excess retained carbon dioxide leads barbiturates to lead to respiratory depression
to a respiratory acidosis. The compensatory and central cardiac depression.
mechanism for respiratory acidosis is an in- Answer B is incorrect. Barbiturates are far
crease in bicarbonate retention by the kidneys more likely than benzodiazepines to cause
to normalize the pH. central respiratory and cardiac depression and
Answer E is incorrect. Vomiting causes a met- are significantly less safe.
abolic alkalosis secondary to the loss of acid Answer C is incorrect. Half-life depends on
and chloride from the stomach. If this were the the pharmacokinetics of the particular medica-

Full-Length Exams
cause, this patient’s lab results would show a tion and not its class. Some half-lives may be
high pH, a high bicarbonate, and (with respira- longer, some shorter.
tory compensation) a high carbon dioxide. The
causes of metabolic alkalosis include vomiting, Answer D is incorrect. Barbiturates, not ben-
diuretic therapy, and chloride restriction. The zodiazepines, increase the duration of chloride
compensation for metabolic alkalosis is hy- channel opening.
poventilation.
16. The correct answer is E. This patient’s pre-
14. The correct answer is D. The arrow points to sentation is highly suspicious for bacterial
the trigeminal nerve (cranial nerve V) in the meningitis. In the neonate, clinical symptoms
image. The major functions of the trigeminal are nonspecific (fever, irritability, lethargy,
nerve include mastication and facial sensation. poor feeding) when compared with those in
older children or adults (nuchal rigidity). The
Answer A is incorrect. Eye movements are most common cause of neonatal meningitis is
controlled by the oculomotor, trochlear, and group B Streptococcus, specifically Streptococ-
abducens nerves (cranial nerves III, IV, and cus agalactiae, a b-hemolytic, gram-positive
VI, respectively), rather than the trigeminal coccus found in chains. In contrast with its
nerve.
Test Block 1

group A counterparts, S agalactiae is bacitra-


Answer B is incorrect. Facial movement is a cin resistant. Infection with this organism oc-
function of the facial nerve (cranial nerve VII) curs during vaginal delivery, as 40% of women
rather than the trigeminal nerve. are asymptomatic carriers of this bacterium
in the gastrointestinal (GI) tract and vagina.
510 Section III: Full-Length Examinations  •  Answers

Escherichia coli and Listeria monocytogenes are tive use, and intra-abdominal neoplasms (he-
the second and third most common causes of patocellular carcinoma, renal cell carcinoma).
meningitis in infants <3 months old. Thus this patient has two conditions (preg-
nancy and antiphospholipid antibody syn-
Answer A is incorrect. Escherichia coli is a
drome) that predispose her to developing BCS.
gram-negative, lactose-fermenting bacillus. It is
the second leading cause of neonatal meningi- Answer A is incorrect. BCS leads to blockage
tis. of hepatic venous outflow, congestion, and
ultimately portal hypertension. However, cho-
Answer B is incorrect. Haemophilus influ-
lecystitis is inflammation of the gallbladder,
enzae type B (Hib) is a small gram-negative
usually due to blockage of the cystic duct by a
encapsulated coccobacillus that was once a
gallstone. This condition does not predispose
major cause of serious bacterial infections, in-
to BCS.
cluding meningitis, sepsis, and epiglottitis. It
requires factors V and X to successfully culture Answer B is incorrect. Chronic obstructive
on chocolate agar. However, the incidence of pulmonary disease (COPD) does not increase
H influenzae meningitis has decreased sig- the risk of developing BCS.
nificantly in the last 10-15 years as a result of
Answer D is incorrect. Primary biliary cir-
the widespread administration of the Hib vac-
rhosis is an intrahepatic autoimmune disease
cine. Nevertheless, Hib still causes 5% of cases
that leads to granulomatous destruction of bile
of meningitis in children ages 6 months to 6
ducts, ultimately causing cirrhosis due to bili-
years.
ary obstruction. However, BCS is a syndrome
Answer C is incorrect. Listeria monocytogenes of venous, not biliary, flow obstruction. Both
is the third most common cause of neonatal can lead to liver failure, but primary biliary
meningitis (2%). It is a gram-positive bacillus. cirrhosis would not cause the flow obstruction
seen in the hepatic veins in this patient. Pri-
Answer D is incorrect. Streptococcus pneu-
mary biliary cirrhosis does not increase the risk
moniae is a gram-positive coccus that is found
of developing BCS.
in chains, but it is not a significant cause of
Full-Length Exams

meningitis in neonates. However, in children Answer E is incorrect. Renal failure does not
ages 6 months to 6 years, it is the leading cause increase the risk of developing BCS.
of meningitis.
18. The correct answer is A. A conversion disor-
17. The correct answer is C. Budd-Chiari syn- der mimics dysfunction in the voluntary mo-
drome (BCS) is a nearly complete obstruction tor or sensory system. Common presentations
to blood flow by an acute clot in the hepatic include pseudo seizures, vocal cord dysfunc-
veins or in the inferior vena cava. This sudden tion, blindness, tunnel vision, deafness, and a
event is followed by the onset of hepatomegaly, variety of paresthesias and paralyses. On care-
pain, ascites, and jaundice. The patient has ful clinical examination and with the aid of
antiphospholipid antibody syndrome (posi- laboratory investigations, these symptoms lack
tive antiphospholipid antibody titer, seizures, physiologic explanation. A clinical example is
and multiple abortions), a coagulation disor- the presence of normal deep-tendon reflexes
der that is often associated with systemic lupus and normal sensation in a person with a “para-
erythematosus (SLE) and is a risk factor for de- lyzed” arm. Patients with conversion disorder
veloping BCS. The ultrasound result further involuntarily have loss of function, usually in
supports the diagnosis by suggesting hepatic response to an unconscious conflict, and seek
Test Block 1

venous occlusion. Predisposing conditions for secondary gain in the form of assumption of
BCS are hematologic disorders (polycythemia the sick role.
vera, essential thrombocytosis), thrombotic di­
Answer B is incorrect. Patients with factitious
atheses (antiphospholipid antibody syndrome,
disorder voluntarily have a loss of function or
factor V Leiden), pregnancy, oral contracep-
Test Block 1  •  Answers 511

voluntarily do things to themselves to create Answer B is incorrect. Lipomas, like rhabdo-


illness or injury, usually in response to an un- myomas, are capable of obstruction. These can
conscious conflict, and seek secondary gain in create ball-valve obstructions and are most of-
the form of assumption of the sick role. ten located in the left ventricle, right atrium,
or atrial septum. However, in conjunction with
Answer C is incorrect. Hypochondriasis is the
the history suggesting tuberous sclerosis, lip­
preoccupation with fears of having a particular
oma is a less likely diagnosis.
disease or diagnosis based on one’s misinterpre-
tation of symptoms, despite clear evidence to Answer C is incorrect. Myxomas, like rhabdo-
the contrary. It lasts >6 months. myomas, are capable of obstruction. However,
these are seen in adults and are often located
Answer D is incorrect. Patients with malinger-
in the atria.
ing voluntarily have loss of function, usually
consciously in response to known situations, Answer E is incorrect. Transposition of the
and seek secondary gain in the form of tangi- great vessels is a situation in which the pul-
ble gain (monetary, housing, or avoidance of monary trunk arises from the left ventricle and
responsibilities). the aorta arises from the right ventricle. This
arrangement is incompatible with life, and a
Answer E is incorrect. The fact that the pa-
compensatory anomaly such as a patent ductus
tient has no medical history, normal reflexes,
arteriosus is necessary.
and normal results of physical examination,
other than the left-arm weakness, indicates he
20. The correct answer is B. This patient has ocu-
is unlikely to have had a stroke. The events
lomotor palsy from disruption of cranial nerve
surrounding the situation make it more likely
(CN) III. This nerve innervates the medial rec-
that he has conversion disorder.
tus, the superior rectus, the inferior rectus, and
the inferior oblique muscles of the eye. When
19. The correct answer is D. Tuberous sclerosis
damaged, the extraocular muscles not inner-
is a genetic condition (autosomal dominant)
vated by CN III dominate (ie, the superior
characterized by nodular proliferation of mul-
oblique, which depresses and intorts the eye,

Full-Length Exams
tinucleated atypical astrocytes. These form tu-
and the lateral rectus, which abducts the eye).
bers, which are found throughout the cerebral
Hence the eye appears “down and out.” CN
cortex and periventricular areas. The classic
III also innervates the levator palpebrae supe-
triad, which is manifest in only the most severe
rioris, which causes ptosis. The pupil is dilated
of cases, consists of seizures, mental retarda-
because of involvement of the parasympathetic
tion, and facial angiofibromas (also known as
fibers that run on the outside of the oculomo-
adenoma sebaceum). Half of patients with tu-
tor nerve and can be compressed by structures
berous sclerosis develop rhabdomyomas, pri-
such as tumors and aneurysms. If it is a pupil-
mary tumors of cardiac muscle that, although
sparing third nerve palsy, then infarction of the
benign, may compromise cardiac function,
nerve is commonly the cause.
especially of the atrioventricular valves. Tuber-
ous sclerosis is also notable for a link to angio- Answer A is incorrect. The abducens nerve
myolipomas of the kidney. (cranial nerve VI) innervates the lateral rectus,
which abducts the eye. Abduction is intact in
Answer A is incorrect. Dilated cardiomy-
this patients affected eye.
opathy is often idiopathic. It involves four-
chamber hypertrophy and dilation, and even- Answer C is incorrect. The optic nerve does
tually heart failure. This condition is not not innervate any of the extraocular muscles
Test Block 1

associated with tuberous sclerosis. Note that and therefore cannot account for the findings
hypertrophic cardiomyopathy also causes ven- above.
tricular outflow obstruction and is often re-
Answer D is incorrect. The trigeminal nerve
sponsible for sudden death in young athletes.
does not innervate any of the extraocular mus-
512 Section III: Full-Length Examinations  •  Answers

cles and therefore cannot account for the find- the vertebra. Thus it should be considered in
ings above. an elderly man with localized back pain that is
most severe in the supine position.
Answer E is incorrect. The trochlear nerve in-
nervates the superior oblique muscle, which
22. The correct answer is B. This patient likely
depresses and intorts the eye. In trochlear
has bacterial endocarditis as a result of intra-
nerve palsy, the affected eye is elevated and
venous (IV) drug use, and the dysarthria and
extorted, and the patient often tilts their head
retinal hemorrhages are suggestive of a left-
away from the affected eye to compensate.
sided valvular lesion showering septic emboli
These abnormalities were not found here.
to the brain and retinal arteries. The blood
culture points to a Streptococcus species; these
21. The correct answer is D. Intracranial metasta-
are gram positive and catalase negative. To
ses represent nearly half of all brain tumors, yet
further characterize Streptococcus, evaluate
only 15% of tumors metastasize to the brain.
the degree of hemolysis of the plate medium.
Intracranial hemorrhages are a recognized but
a-Hemolytic streptococci demonstrate partial
relatively uncommon complication of brain
hemolysis, whereas b-hemolytic streptococci
tumors and can result in intraparenchymal,
show complete hemolysis. γ-Hemolytic or-
subarachnoid, subdural, and epidural hema-
ganisms show no hemolysis and include En-
tomas. Focal neurologic signs are frequently
terococcus species. The only organisms of this
evident and are due to pressure exerted on the
group known to cause endocarditis are the
brain parenchyma. Because melanoma is a rel-
viridans group streptococci and Enterococcus
atively frequent source of metastatic lesions to
faecalis. Viridans streptococcus is a hemolytic,
the brain (although less common than breast
optochin resistant, and bile soluble. E faeca-
or lung carcinoma) and demonstrates a ten-
lis is γ hemolytic, optochin resistant, and bile
dency to hemorrhage, melanoma is the correct
soluble, but unlike the Streptococcus species,
answer in this case.
it is able to grow on 6.5% sodium chloride so-
Answer A is incorrect. Angiosarcomas are ma- lution. Therefore, to differentiate between the
lignant endothelial neoplasms that resemble two, 6.5% sodium chloride would be the best
Full-Length Exams

hemangiomas. Although these tumors may culture condition to isolate a single organism.
bleed, angiosarcomas rarely metastasize, and
Answer A is incorrect. The absence of colo-
only a few case reports exist of hemorrhage of
nies in the presence of penicillin suggests that
cerebral metastasis from angiosarcoma.
the organism is sensitive to penicillin. Trepo-
Answer B is incorrect. Some cancers rarely nema pallidum and any number of strepto-
metastasize to the brain; these include carci- cocci are sensitive to this antibiotic; therefore,
nomas of the oropharynx, esophagus, and pros- this would not be best culture condition for
tate, as well as nonmelanoma skin cancers. isolating a single type of organism.
Answer C is incorrect. Colorectal carcinoma Answer C is incorrect. Both the viridans group
does metastasize to the brain (though less fre- streptococci and E faecalis are optochin re-
quently than melanoma) but does not typi- sistant. Therefore, use of optochin would not
cally result in intracranial hemorrhage. Since help you differentiate between the two strains.
colorectal carcinoma is less likely than mela-
Answer D is incorrect. Haemophilus influen-
noma to result in brain metastases and is not
zae is cultured on chocolate agar, along with
as likely to hemorrhage, melanoma is a better
factors V (nicotinamide adenine dinucleotide)
answer.
Test Block 1

and X (hematin) for growth. However, this or-


Answer E is incorrect. Carcinoma of the pros- ganism is not a common cause of bacterial en-
tate almost never results in metastatic brain docarditis; rather, it causes epiglottitis, menin-
disease. Prostate cancer can cause malignant gitis, otitis media, and pneumonia.
spinal cord compression by metastasizing to
Test Block 1  •  Answers 513

Answer E is incorrect. Streptococcus pneu- probability becomes the prevalence). If we set


moniae is a gram-positive bacterium that is bile the number of those with the disease as 10,
soluble, but it is not a common cause of bacte- then TP = 8 and FP = 1, given the sensitivity
rial endocarditis. of 80% and specificity of 90%. Therefore, the
PPV would be calculated as 8 / (8 + 1) = 89%,
23. The correct answer is A. Adipocytes are the or about 90%. The same answer can also be
cells that comprise adipose (fat) tissue. GLUT obtained by converting the pretest probability
4-mediated glucose transport occurs in only to an odds ratio (1:1) and multiplying it by the
two tissue types: adipose tissue and skeletal test’s positive likelihood ratio (LR+), which
muscle. In the fasting state, when insulin lev- can be calculated using the formula LR+ =
els are low, there is decreased intake of glu- sensitivity / (1 - specificity) = 0.80 / (1 - 0.90)
cose into adipose tissue and skeletal muscle, = 8. Therefore, the posttest odds of having the
enabling glucose to be utilized by more per- disease is 8:1 or 8/9 = 89% once the figure is
tinent organs. In this context, decreased glu- converted back into a probability.
cose intake into fat and muscle cells will pro-
Answer A is incorrect. This value is too low to
mote mobilization of stored precursors such
be the correct answer.
as amino acids and free fatty acids. This is the
only choice among those listed that could be Answer B is incorrect. This value is too low to
used in the hypothetical experimental system be the correct answer.
described.
Answer D is incorrect. This value is too high
Answer B is incorrect. Cortical neurons are to be the correct answer.
derived from the brain, where glucose trans-
Answer E is incorrect. This value is too high
port occurs independent of insulin stimula-
to be the correct answer.
tion. Thus these cells could not be used in this
hypothetical system. Brain and RBCs take up 25. The correct answer is E. This patient’s clini-
glucose via GLUT 1 transport. cal presentation, history of alcoholism, and
Answer C is incorrect. RBCs take up glucose hemodynamic instability suggest a rapid loss

Full-Length Exams
independent of insulin levels using GLUT 1 of blood related to an upper GI bleed. Boer-
transport. haave perforation is a transmural perforation
that normally presents with the Mackler triad:
Answer D is incorrect. Insulin has no effect
vomiting, lower thoracic pain, and subcutane-
on glucose uptake in hepatocytes, so this cell
ous emphysema. Boerhaave perforation has
type could not be used in this hypothetical sys-
a high mortality rate and a rate of progression
tem.
much more rapid than seen in the patient in
Answer E is incorrect. Pancreatic b cells ex- this vignette. These patients need to be treated
press GLUT 2 transporters, which serve as glu- with emergent surgical repair; the single great-
cose sensors. These cells do not express GLUT est factor impacting survival is diagnosis and
4 transporters and would not be appropriate for treatment within 24 hours.
use in this hypothetical system.
Answer A is incorrect. A patient with an
esophageal mass usually would have a more
24. The correct answer is C. The positive predic-
chronic evolution of symptoms that includes
tive value (PPV) of the test can be calculated
progressive swallowing difficulties and a his-
with the following formula, where TP is true-
tory of hematemesis. The onset of this patient’s
positive results and FP is false-positive results:
Test Block 1

symptoms is acute, without a history of dyspha-


TP / (TP + FP). Given the pretest probability
gia or upper GI bleeding, making esophageal
of 50%, we need to set up a hypothetical 2 × 2
mass an unlikely cause in this scenario.
table in which the number of subjects with the
disease is equal to the number not having the Answer B is incorrect. Esophageal stricture
disease (or to be said differently, the pretest usually does not present with hematemesis; it
514 Section III: Full-Length Examinations  •  Answers

most often presents with complaints of solid- Answer C is incorrect. Hemoglobin is a mol-
food dysphagia. ecule found in RBCs and is not produced by
any cells in the arterial wall.
Answer C is incorrect. Acute cholecystitis
would present typically with epigastric or right Answer D is incorrect. Macrophage colony-
upper quadrant pain that is worse with inspi- stimulating factor is made by macrophages,
ration (Murphy sign). However, patients with which are not part of the typical cell archi-
acute cholecystitis typically are hemodynami- tecture of the arterial wall. Under pathologic
cally stable, unlike this patient. In addition, conditions, macrophages invade the intima
the history of hematemesis is not consistent and facilitate the formation of atherosclerotic
with acute cholecystitis, which typically pre­ plaques.
sents with right upper quadrant pain, nausea,
non-bloody vomiting, and fever that may be ex- 27. The correct answer is B. Imipenem is a broad-
acerbated by consumption of fatty foods. spectrum, b-lactamase-resistant antibiotic of
the carbapenem class. These drugs are struc-
Answer D is incorrect. This patient’s history
turally similar to b-lactam antibiotics but are
of alcohol abuse and acute onset of bright-red
b-lactamase-resistant and are administered
emesis after an episode of retching is consis-
with cilastatin to decrease renal metabolism.
tent with a Mallory-Weiss tear. These nonpen-
Imipenem can be used to treat gram-positive
etrating mucosal tears frequently are found at
and gram-negative infections and it is first-
the gastroesophageal junction. A sudden in-
line therapy in the treatment of Acinetobacter
crease in transabdominal pressure as seen in
and Enterobacter species infection. Imipenem
vomiting and retching is believed to be the
is not useful in treating methicillin-resistant
pathophysiology. Alcoholism is a predisposing
Staphylococcus aureus, Enterococcus faecium,
risk factor because of the violent vomiting that
or Staphylococcus epidermidis. Adverse effects
may follow an alcohol binge. Frequently the
include GI distress, thrombophlebitis, skin
bleeding is self-limited; therefore the hemody-
rashes, and seizures.
namic instability in this case most likely is due
to esophageal rupture. Answer A is incorrect. Gentamicin is an ami-
Full-Length Exams

noglycoside antibiotic used to treat gram-nega-


26. The correct answer is E. The plaque shown tive bacilli that works by inhibiting protein syn-
in the image is an atheroma. These lesions of thesis. Adverse effects include nephrotoxicity,
extracellular lipid develop within the intima of ototoxicity, and neuromuscular blockade. Gen-
the arterial wall. The intima lines the luminal tamicin is effective in treating Acinetobacter
side of the artery; it is the most “intimate” with species infection, but it is not as efficacious as
the blood. The innermost layer of cells of the imipenem, which is the first-line choice. It is
intima is therefore the endothelial cells. In a not associated with seizures.
nonpathologic state, endothelial cells prevent
Answer C is incorrect. Levofloxacin is a fluo-
plaque formation by releasing antithrombotic
roquinolone antibiotic that acts by inhibiting
factors such as prostacyclin and nitric oxide.
DNA gyrase. It is used to treat gram-negative
Answer A is incorrect. Collagen is produced rods and some gram-positive organisms and
by the smooth muscle cells in the media of the therefore is good for upper respiratory infec-
arterial wall. These cells also produce elastin tions and urinary tract infections. Fluoroquin­
and proteoglycans that are the other two im- olones are generally well tolerated but are as-
portant components of the vascular extracellu- sociated with anaphylaxsis, GI discomfort,
Test Block 1

lar matrix of arterial walls. headache, and phototoxicity. They are also
associated with damaged cartilage in animal
Answer B is incorrect. Fibrillin is a compo-
models, thus are contraindicated in children.
nent of elastin, which is made by the smooth
muscle cells of the media. Answer D is incorrect. Linezolid is used for
community acquired pneumonias and acts by
Test Block 1  •  Answers 515

binding to the bacterial 50S ribosomal RNA The cervix will be erythematous on examina-
subunit and blocking protein synthesis. Ad- tion.
verse effects include GI upset, headache, and
Answer D is incorrect. Gonococcal infection
myelosuppression. It does not cause seizures.
is often asymptomatic in women. A patient
Answer E is incorrect. Rifampin is used with urethritis will present with a mucopuru-
to treat TB and acts by inhibiting DNA- lent urethral discharge, urinary frequency, dys-
dependent RNA polymerase. Important ad- uria, and urgency. The clinical picture may be
verse effects include a transient rise in hepatic identical to cystitis.
aminotransferase levels (which usually return
Answer E is incorrect. Hepatitis A (HAV)
to normal without discontinuation of rifampin)
causes an acute viral hepatitis after a four-week
and hepatitis, which can occur directly or due
incubation period. The disease is self-limited
to rifampin potentiating the hepatic toxicity of
and presents with symptoms such as jaundice
other drugs. Rifampin also causes discoloration
and right upper quadrant pain, along with
of body fluids. It does not cause seizures.
highly elevated transaminases. The IgM anti-
body is useful to diagnose the disease during its
28. The correct answer is C. Perihepatitis, or Fitz-
presentation. However, the IgG antibody is el-
Hugh-Curtis syndrome, is seen in up to 25%
evated indefinitely after infection and indicates
of women with pelvic inflammatory disease
only previous exposure.
(PID). PID is an ascending, polymicrobial in-
fection of the female genital tract that can in- Answer F is incorrect. A tubo-ovarian abscess
volve the endometrium, uterine tube, and/or is a complication of PID. It is often polymicro-
peritoneal cavity. Chlamydia trachomatis and bial, with a predominance of anaerobes. The
Neisseria gonorrhoeae cause up to half of the clinical presentation will involve abdominal
cases of PID. Perihepatitis is a complication pain and bilateral or unilateral adnexal masses.
that usually presents with right upper quadrant
pain associated with symptoms of PID (the 29. The correct answer is C. This patient has sec-
classic triad is fever, abdominal pain, and vagi- ondary hyperparathyroidism due to chronic

Full-Length Exams
nal discharge). On laparoscopy, “violin-string” renal insufficiency, or renal osteodystrophy.
adhesions will be present in the peritoneal cav- Chronic renal failure is the most common
ity. cause of secondary hyperparathyroidism. There
are numerous etiologies of chronic renal fail-
Answer A is incorrect. Acute cholecystitis
ure; nephropathy secondary to diabetes melli-
presents with constant, severe pain and tender-
tus (DM) is the most common in the United
ness in the right upper quadrant (RUQ) or epi-
States. In renal osteodystrophy, nephron dam-
gastrium, nausea and vomiting, and fever and
age leads to impaired calcium reabsorption,
leukocytosis. The acute attack often follows a
impaired phosphate excretion, and impaired
large, fatty meal. Murphy sign on physical ex-
activation of vitamin D in the kidney (vita-
amination will show a cessation of inspiration
min D normally increases the absorption of
on simultaneous palpation of the RUQ. Liver
calcium in the intestines). This results in de-
function tests will be elevated.
creased serum calcium and increased serum
Answer B is incorrect. Chlamydia trachoma- phosphate levels. The resulting hypocalcemia
tis infection is often asymptomatic in women. stimulates secretion of parathyroid hormone
The clinical silence may eventually lead to (PTH; secondary hyperparathyroidism), caus-
PID via ascending infection. However, symp- ing increased bone turnover, which further
Test Block 1

tomatic chlamydial cervicitis presents with a contributes to the hyperphosphatemia and


mucopurulent cervical discharge that may or leads to an increased serum alkaline phos-
may not have an associated vaginal discharge. phatase level. The elevated creatinine level in
Patients may complain of postcoital bleeding. this patient is a clue that her kidney function
is abnormal. Diffuse bone pain and proximal
516 Section III: Full-Length Examinations  •  Answers

muscle weakness are typical symptoms of sec- Answer C is incorrect. Diltiazem is an antiar-
ondary hyperparathyroidism caused by the rhythmic that is sometimes used in IV form to
hypocalcemia and increased bone resorption. treat atrial fibrillation. It infrequently causes
Pruritus is occasionally seen in renal insuffi- hypotension or bradyarrhythmias, but is not
ciency due to deposition of excess calcium. known to cause pulmonary fibrosis.
Answer A is incorrect. Parathyroid adenoma Answer D is incorrect. Sotalol works by both
would cause primary hyperparathyroidism with nonselectively antagonizing b-receptors and
increased secretion of PTH, resulting in hyper- by prolonging action potentials. It is used to
calcemia and hypophosphatemia rather than treat ventricular and supraventricular arrhyth-
hypocalcemia and hyperphosphatemia. mias in children and life-threatening ventricu-
lar arrhythmias in adults. Sotalol can some-
Answer B is incorrect. Parathyroid insuffi-
times cause torsades des pointes when taken at
ciency would result in hypocalcemia, with
higher doses. However, it does not cause pul-
typical symptoms of tetany and increased neu-
monary fibrosis.
romuscular excitability and decreased serum
intact PTH levels. Additionally, it cannot ac- Answer E is incorrect. Tobacco is a known risk
count for the hyperphosphatemia presented in factor for COPD. COPD presents with a re-
this case. duced FEV1:FVC ratio, typically <80%. There
is also a convex inward sloping of the flow-
Answer D is incorrect. Malignancy usually re-
volume curve on pulmonary function testing.
sults in hypercalcemia due either to lytic me-
tastases to bone (with increased serum alkaline
31. The correct answer is C. The patient is pre-
phosphatase activity and hyperphosphatemia)
senting with the classic signs and symptoms
or to production of PTH-related peptide (with
of HAV. Hepatitis often presents with a pro-
hypophosphatemia).
drome of flulike symptoms that is followed by
Answer E is incorrect. Vitamin D intoxication a possible icteric phase during which patients
results in hypercalcemia and hyperphosphate- present with jaundice, icterus, hepatomegaly,
mia with decreased serum intact PTH levels, pruritus, arthralgias, and rashes. Aspartate
Full-Length Exams

and thus would be inconsistent with the lab aminotransferase (AST) and alanine amino-
values presented in the vignette. However, vi- transferase (ALT) levels can exceed 10,000 IU/
tamin D intoxication may indeed present with ml, with ALT > AST levels (as opposed to al-
clinical findings similar to those stated above, coholic hepatitis, in which AST levels > ALT
including pruritus, weakness, and renal dys- levels). The next thing to do is to determine
function. which hepatitis virus this is. The patient’s de-
mographics and contact history as well as the
30. The correct answer is B. This clinical picture lack of any sexual or illicit drug abuse suggest
is highly suggestive of restrictive lung disease; the diagnosis of HAV. Of the viral hepatitides,
although the FEV1:FVC ratio is near normal the hepatitis A and E viruses are transmitted
levels, both the FEV1 and FVC are markedly via the fecal-oral route. HAV is not common
reduced. Amiodarone is an antiarrhythmic in the United States, but it is very common in
agent that is known to cause pulmonary fibro- Africa, Asia, and South America. Acquisition
sis, a restrictive lung disease. Additional ad- in childhood is the norm, and most adults in
verse effects include interstitial pneumonitis, those areas show seropositivity for HAV. Fac-
photosensitivity, thyroid disorders, and GI dis- tors that predispose this population to infection
turbances. include overcrowding, poor sanitation, and
Test Block 1

Answer A is incorrect. Asthma is a cause of lack of a clean water source. Of the answer op-
COPD. Asthma-induced abnormalities on pul- tions, the only virus that also has a fecal-oral
monary function testing would not fit a restric- route of transmission is poliovirus, an RNA en-
tive pathology profile. terovirus that causes acute flaccid paralysis in
<1% of cases.
Test Block 1  •  Answers 517

Answer A is incorrect. Flavivirus, which is the 33. The correct answer is F. This woman is suf-
causative agent of yellow fever, is one of the fering from disseminated intravascular co-
arboviruses, and it is transmitted by the Aedes agulation (DIC), most likely secondary to an
mosquito. amniotic fluid embolism that traveled to her
pulmonary circulation. Common causes of
Answer B is incorrect. Hepatitis B virus
DIC are gram-negative sepsis, malignancy,
(HBV) is transmitted sexually and parenterally;
pancreatitis, trauma, transfusion reactions, and
this includes both maternal-fetal and blood-
obstetric complications. During DIC, there is
borne routes.
a massive activation of the coagulation cascade
Answer D is incorrect. Rabies virus is trans- that results in thrombus formation through-
mitted via the bite of an infected animal, out the microvasculature. This results in rapid
which in the United States would most likely consumption of both platelets and coagula-
be a skunk or a bat. tion factors. Concurrent with this consumptive
Answer E is incorrect. Varicella-zoster vi- coagulopathy is activation of the fibrinolytic
rus, which causes chickenpox and shingles, is system. Ultimately, complications in DIC re-
transmitted by respiratory secretions. sult from thrombosis and bleeding and may
include massive blood loss and organ failure.
32. The correct answer is E. This patient has Tur- Laboratory findings include a decreased plate-
cot syndrome, an autosomal dominant disease. let count, elevated bleeding time, elevated pro-
All familial polyposis syndromes, with the ex- thrombin time (PT), and elevated PTT.
ception of Peutz-Jeghers syndrome, predispose Answer A is incorrect. An isolated increase in
to colorectal cancer. Turcot syndrome is asso- PTT indicates a problem with the intrinsic co-
ciated with two separate dominant mutations. agulation cascade. This can be caused by he-
The first is a mutation of the APC gene lead- mophilia A or B, both of which are X-linked
ing to polyposis and medulloblastoma, and the disorders that result in deficiencies in factor
second is associated with the hMLH1 DNA VIII and IX, respectively.
mismatch repair gene leading to polyposis and
Answer B is incorrect. An isolated increase in

Full-Length Exams
glioblastoma multiforme.
PT indicates a problem with the extrinsic co-
Answer A is incorrect. Familial adenoma- agulation cascade. This can be caused by treat-
tous polyposis is associated with hundreds of ment with warfarin.
colorectal polyps, and nearly all affected pa-
tients will develop colorectal cancer. Answer C is incorrect. An isolated increase in
bleeding time indicates a functional defect in
Answer B is incorrect. Gardner syndrome is platelets. This can be caused by vascular bleed-
characterized by colorectal polyposis and os- ing and platelet defects found in diseases such
teomas or other bone and soft tissue tumors. as Glanzmann thrombasthenia.
Answer C is incorrect. Hereditary nonpolyp­ Answer D is incorrect. An increase in both
osis colorectal carcinoma is associated with bleeding time and PTTs indicates both plate-
dozens of colorectal polyps, and a majority of let dysfunction and a problem with the intrin-
affected patients will develop colorectal cancer. sic coagulation cascade. This can be caused by
Answer D is incorrect. Tuberous sclerosis is von Willebrand disease, an inherited bleeding
an autosomal dominant condition character- disorder that affects both platelet function and
ized by mental retardation, seizures, tuberous factor VIII availability.
Test Block 1

central nervous system tumors, angiomyolipo- Answer E is incorrect. Decreased platelet


mas of the kidneys, leptomeningeal tumors, count and elevated bleeding time indicates
and skin lesions such as ash-leaf spots and thrombocytopenia. This can be caused by
shagreen patches. aplastic anemia, leukemia, immune thrombo-
cytopenic purpura, thrombotic thrombocyto-
518 Section III: Full-Length Examinations  •  Answers

penic purpura/hemolytic-uremic syndrome, ecule. Thus neutral molecules diffusing into


and splenic sequestration. the tubule will become ionized. Once in the
charged state, molecules cannot diffuse back
34. The correct answer is A. To answer this ques- across tubular epithelial membranes to the
tion, one must know that Kawasaki syndrome bloodstream. Thus the clearance of aspirin is
(also referred to as mucocutaneous lymph increased greatly when urine pH is alkalinized.
node syndrome) is an arteritis that primarily af-
Answer A is incorrect. Acidification of urine
fects medium- and small-sized arteries. Hence,
has no effect on the glomerular filtration rate
it makes sense that there is evidence suggest-
(GFR). GFR is affected by the difference in
ing the formation of anti-endothelial cell (and
pressures across the glomerulus and glomeru-
anti-smooth muscle cell) autoantibodies in
lar permeability.
patients with this disease. The clinical mani-
festations of this disease include fever for more Answer B is incorrect. Acidification of the
than five days, cervical lymphadenopathy, a urine would lower the pH and shift the equi-
skin rash (which often has desquamation, or librium toward the protonated neutral form of
shedding of the skin), and erythema of the aspirin. These non-ionized molecules could
conjunctivae, oral mucosa, palms, and soles. then move back into the bloodstream, and
Eighty percent of patients are under the age of clearance of aspirin would be decreased.
four years. Twenty percent of patients develop
Answer C is incorrect. Acidification of the
cardiovascular disease, including coronary ar-
urine would lower the pH and shift the equi-
tery vasculitis and coronary artery aneurysm.
librium toward the protonated neutral form of
Answer B is incorrect. Anti-IgG (rheumatoid aspirin, but these molecules can diffuse across
factor) is not particularly associated with Kawa- cell membranes back into the bloodstream and
saki syndrome. Elevated levels of serum rheu- would not be excreted.
matoid factor are present in 80% of patients
Answer D is incorrect. Alkalinization of urine
with rheumatoid arthritis.
has no effect on the GFR. GFR is affected by
Answer C is incorrect. Anticentromere an- the difference in pressures across the glomeru-
Full-Length Exams

tibodies, which are found in 90% of patients lus and glomerular permeability.
with the CREST variant of scleroderma, are
Answer F is incorrect. Alkalinization of urine
not particularly associated with Kawasaki syn-
promotes ionization of aspirin in the urine; the
drome.
concentration of non-ionized molecules of as-
Answer D is incorrect. Antihistone antibod- pirin in the tubule would decrease as the urine
ies, which are found in over 95% of patients is alkalinized.
with drug-induced lupus erythematosus, are
not particularly associated with Kawasaki syn- 36. The correct answer is C. This patient has
drome. classic symptoms of cardiac ischemia: chest
pain with sudden onset that radiates to his left
Answer E is incorrect. Antinuclear antibodies,
shoulder or jaw and is relieved by sublingual
which are present in over 95% of patients with
nitroglycerin. However, the patient is young,
SLE, are not particularly associated with Kawa-
and the pain is not prompted by activity but
saki syndrome.
occurs at rest. Additionally, his ECG is nor-
mal, showing no evidence of infarct or ische­
35. The correct answer is E. Because aspirin is
mia. As a result, he probably suffers from coro-
a weak acid with an acid dissociation con-
Test Block 1

nary vasospasm, also known as Prinzmetal’s


stant (pKa) near 3.5, it can interconvert be-
(variant) angina.
tween neutral and negatively charged forms
depending on the pH. Increasing the pH of Answer A is incorrect. The absence of evi-
tubular fluid shifts the equilibrium toward dence for an infarct on ECG makes myocar-
the non-protonated charged state of the mol-
Test Block 1  •  Answers 519

dial infarctions an unlikely etiology of his chest could be increased with constriction of the ef-
pain. ferent arteriole, for example.
Answer B is incorrect. Pericarditis can cause
38. The correct answer is C. Informed consent
sudden onset of chest pain without exertion,
requires that the patient understand the risks,
but the pain would be not relieved with nitro-
benefits, and alternatives to treatment. Addi-
glycerin. Typically, an ECG would also show
tionally, following discussion of pertinent in-
diffuse ST-segment elevations.
formation, the patient must agree to care in a
Answer D is incorrect. Although the patient’s setting free from coercion. In this setting, the
clinical symptoms are of cardiac ischemia, they patient of concern is the daughter. It is not
are not induced by a specific amount of exer- clear that this setting is free from coercion.
cise, which is the classic definition of stable Before allowing the daughter to consent, she
angina. should be addressed alone.
Answer E is incorrect. Because the patient’s Answer A is incorrect. While it is ideal to in-
symptoms have not increased in frequency, volve as much of the family as possible, the
changed in intensity, and are not prompted most important people involved (the patient
by a light amount of exercise or strain, it is un- and her potential donor) are present. Involving
likely that they are due to unstable angina. the rest of the family ensures a good support
network should the transplant proceed.
37. The correct answer is E. Minimal change dis-
Answer B is incorrect. There is not enough
ease results in nephrotic syndrome, which is
information in the question to determine
manifested primarily in the loss of significant
whether the patient has sufficient understand-
protein in the urine. As a result of this protein
ing. Although this is required for informed
loss the plasma protein concentration will de-
consent, there is a better answer.
crease, thus decreasing the oncotic pressure
in the glomerular capillary. According to the Answer D is incorrect. Transplantation is
Starling equation (glomerular filtration rate rarely an emergent procedure. Ample time

Full-Length Exams
= Kf [(PGC - PBS) - (pGC - pBS)]), this change should be given to both the donor and the re-
will lead to a higher glomerular filtration rate cipient to prepare for the operation.
by decreasing the oncotic forces that normally
Answer E is incorrect. There is no informa-
oppose ultrafiltration.
tion indicating that this patient is not compe-
Answer A is incorrect. Tubular hydrostatic tent to make decisions. Simply being ill does
pressures are not affected by nephrotic syn- not suggest that she is incompetent.
drome. The Bowman space hydrostatic pres-
sure generally does not decrease. 39. The correct answer is C. This patient most
likely has diabetic nephropathy caused by
Answer B is incorrect. Tubular hydrostatic
long-standing and often poorly managed DM.
pressures are not affected by nephrotic syn-
The patient’s presentation is typical for ne-
drome. The Bowman space hydrostatic pres-
phrotic syndrome, with massive proteinuria
sure could be increased in a patient with an
and peripheral and periorbital edema. Blood
obstruction to urine flow.
tests would likely have shown hypoalbumin-
Answer C is incorrect. Bowman space oncotic emia and hyperlipidemia, which are also as-
pressure will increase, not decrease, as protein sociated with nephrotic syndrome. The im-
is filtered into Bowman space and thus in- age shows changes typically associated with
Test Block 1

creases the protein concentration there. diabetic nephropathy, including basement


membrane thickening and presence of hya-
Answer D is incorrect. Hydrostatic pressures
line deposits in the periphery of the glomeru-
are not affected in minimal change disease.
lus (known as Kimmelstiel-Wilson nodular
The glomerular capillary hydrostatic pressure
lesions). The increased glucose levels in diabe-
520 Section III: Full-Length Examinations  •  Answers

tes can lead to vascular damage, and diabetes Answer E is incorrect. Goodpasture disease
is strongly associated with coronary artery dis- results in a rapidly progressive glomerulone-
ease. In addition to management of lipids and phritis, with proteinuria and hematuria, and al-
blood glucose, angiotensin-converting enzyme veolar hemorrhage causing shortness of breath
inhibitors and/or angiotensin II receptor block- and hemoptysis. It is caused by antibodies di-
ers are beneficial in the treatment of diabetic rected against the glomerular basement mem-
nephropathy. brane. Renal biopsy with immunofluorescence
would show linear deposition of IgG along the
Answer A is incorrect. Although renal disease
glomerular membrane. Kimmelstiel-Wilson
associated with SLE may have similar present-
nodular lesions, as seen in the image, would
ing symptoms (proteinuria, peripheral and
not be observed.
periorbital edema, and hypoalbuminemia),
the image shows Kimmelstiel-Wilson nodular Answer F is incorrect. Acute poststreptococcal
lesions, which are characteristic of diabetic glomerulonephritis is associated with recent
nephropathy. With SLE, there are five differ- streptococcal infection, which would present
ent patterns of renal involvement. In the mem- with a history of pharyngitis, low-grade fever,
branous glomerulonephritis pattern, biopsy swollen lymph nodes, and tonsillar exudates. It
reveals wire-loop lesions with subepithelial is seen most often in children, and although it
deposits. In addition to arthritis, symptoms of shares with diabetic nephropathy the present-
SLE include fatigue, malar rash, photosensitiv- ing symptoms of peripheral and periorbital
ity, pleuritis, pericarditis, and many more. edema and proteinuria, it usually also presents
with either gross or microscopic hematuria,
Answer B is incorrect. Although renal disease
which is not found in this patient. Addition-
associated with amyloidosis has similar pre-
ally, renal biopsy of a patient with acute post-
senting symptoms (proteinuria, peripheral and
streptococcal glomerulonephritis would typi-
periorbital edema, and hypoalbuminemia),
cally show a “lumpy bumpy” appearance on
the image shows Kimmelstiel-Wilson nodular
light microscopy, with neutrophilic infiltrate
lesions, which are characteristic of diabetic
and subepithelial deposits.
nephropathy. Renal biopsy viewed under im-
Full-Length Exams

munofluorescence with Congo red stain re-


40. The correct answer is A. Sensitivity = true-
veals apple-green birefringence in patients
positives / (true-positives + false-negatives).
with amyloidosis. One excellent example of
False-negatives signify the people with disease
amyloidosis is amyloid from immunoglobulin
X who will be missed by the screening test. In
light chains, which is produced by cancerous
this case, 100 people have the disease, and 80%
plasma cells in multiple myeloma. In addition
will be diagnosed correctly (80 people are the
to bone pain, signs and symptoms of multiple
true-positive fraction). Rearranging the equa-
myeloma include renal failure, elevated cal-
tion yields: false-negatives = (true-positives /
cium, anemia, and increased vulnerability to
sensitivity) - true-positives or (80 / .8) - 80 =
infection.
20. Thus 20 people with disease X will not be
Answer D is incorrect. Alport syndrome is an diagnosed with this screening test (ie, they will
inherited glomerular disease caused by a muta- be false-negatives).
tion in type IV collagen. It can be inherited in
Answer B is incorrect. The figure 80 is the
an X-linked or an autosomal recessive manner.
number of people who will have a correct posi-
It typically presents with recurrent episodes of
tive screening test result (ie, true-positives). If
gross hematuria during childhood. It is associ-
100 people have the disease (10% prevalence)
Test Block 1

ated with sensorineural hearing loss and ocu-


and the test is 80% sensitive, then 80 people
lar disorders. Renal biopsy would show a split
will be correctly diagnosed (true-positives).
basement membrane, not the Kimmelstiel-
Wilson nodular lesions shown in the image. Answer C is incorrect. The figure 100 is the
number of people in the town with disease X
Test Block 1  •  Answers 521

(ie, the prevalence of disease X). This is calcu- endocrine origin. An elevated urinary 5-HIAA
lated: 1000 × 0.10 = 100. level is diagnostic of carcinoid syndrome.
Answer D is incorrect. The figure 270 is the Answer A is incorrect. The gastroesophageal
number of people who will have an incor- junction is affected by gastroesophageal reflux
rect positive screening test result (ie, false- disease, not carcinoid tumors.
positives). One way of calculating this is that
Answer C is incorrect. The pancreaticoduo-
there are 900 people without the disease (with
denal junction is the site where pancreatic en-
a 10% incidence in 1000 people 100 will have
docrine and exocrine secretions empty into the
the disease and 900 will not). If the specificity
small bowel to aid in digestion. It is part of the
is 70% (the percentage of true-negative test re-
foregut-derived intestine, and it is a rare site for
sults in people without the disease) then there
carcinoid tumors.
will be 630 people who are correctly negative
(true-negatives). This means that there are Answer D is incorrect. The rectosigmoid junc-
900 - 630 = 270 people without the disease tion is not a common location for carcinoid tu-
that will test positive (false-positives). mors.
Answer E is incorrect. The figure 630 is the Answer E is incorrect. The splenic flexure is a
number of people who will have a correct neg- watershed area that is susceptible to ischemic
ative screening test result (ie, true-negatives). If damage if cardiac output becomes low. It is
100 people have the disease (10% prevalence) not, however, a common site for carcinoid tu-
then there are 900 left that don’t have the dis- mors.
ease: 1000 - 100 = 900. Of these 900 people
the test is 70% specific (meaning it’s the per- 42. The correct answer is C. The plain film re-
centage of true-negatives detected by the test veals a lung tumor located in the superior
in a population without the disease): 900 × sulcus and lung apex. Cancers in this loca-
0.70 = 630. tion, which are also termed Pancoast tumors,
frequently strangulate and/or damage the sym-
41. The correct answer is B. This patient presents pathetic fibers that ascend to synapse in the

Full-Length Exams
with chronic diarrhea, intermittent facial flush- superior sympathetic ganglion. This results
ing, and a murmur consistent with tricuspid in Horner syndrome: unilateral ptosis, miosis,
stenosis, a triad of findings classic for carcinoid and anhidrosis.
syndrome. One-third of carcinoid tumors of Answer A is incorrect. The sympathetic fibers
the GI tract occur in the midgut-derived small that are damaged in Horner syndrome and
bowel, which begins at the ligament of Treitz cause the symptoms this man is experiencing
and ends at the mid-transverse colon. While synapse in the superior cervical ganglion and
adenocarcinoma is the most common type of not the inferior cervical ganglion.
small bowel tumor, carcinoid tumors are most
likely to occur in the small bowel. Carcinoid Answer B is incorrect. The sympathetic fibers
tumors of the small intestine secrete serotonin, that are damaged in Horner syndrome and
which is usually metabolized by the liver and cause the symptoms this man is experiencing
doesn’t cause the symptoms of the carcinoid synapse in the superior cervical ganglion and
syndrome. However, when metastases to the not the sphenopalatine ganglion.
liver are present, the bioactive amines can Answer D is incorrect. The sympathetic fibers
no longer be metabolized and enter the sys- that are damaged in Horner syndrome and
temic circulation causing diarrhea, abdominal
Test Block 1

cause the symptoms this man is experiencing


cramps, GI bleeding, malabsorption, flushing, synapse in the superior cervical ganglion and
bronchospasm, and right heart valvular disease not the superior mesenteric ganglion.
from serotonin-mediated fibroelastosis. Elec-
tron microscopy reveals “salt and pepper” gran- Answer E is incorrect. The sympathetic fibers
ulation of cells, consistent with their neuro- that are damaged in Horner syndrome and
522 Section III: Full-Length Examinations  •  Answers

cause the symptoms this man is experiencing 44. The correct answer is D. The pH and elec-
synapse in the superior cervical ganglion and trolyte profiles for this patient describe an el-
not the T4 dorsal root ganglion. Furthermore, evated anion gap metabolic acidosis (low pH,
the sympathetic impulses that are blocked low bicarbonate, high anion gap). Given the
arise from C8-T2, levels rostral to T4. signs and symptoms described, the most likely
cause of acidosis in this patient is diabetic ke-
43. The correct answer is E. This patient presents toacidosis (DKA), which is often the present-
with deep venous thrombosis (DVT). Ery- ing syndrome in type 1 DM. Typical signs and
thematous, warm, and tender unilateral calf symptoms of DKA include polyuria, polydip-
swelling is classic for DVT. Risk factors for sia, fatigue, vomiting, abdominal pain, abnor-
DVT and subsequent pulmonary thromboem- mal breathing (tachypnea early in the course,
bolism include Virchow’s triad, which consists Kussmaul breathing with slow, deep breaths
of stasis (eg, immobility, obesity, congestive later), drowsiness, lethargy, and coma. The
heart failure), endothelial injury (eg, trauma, initial management of DKA requires aggres-
surgery, previous DVT), and hypercoagulable sive fluid resuscitation and correction of hyper-
state (eg, pregnancy, oral contraceptive use, glycemia with insulin. Insulin stimulates the
coagulation disorders, malignancies, smok- shift of potassium from the extracellular com-
ing). This patient also has a positive Homans partment to the intracellular compartment,
sign (calf pain on forced dorsiflexion), which causing a decrease in serum potassium levels.
further supports the diagnosis. Not only should In addition, the rise in serum pH (as a result
this patient be anticoagulated with heparin or of correcting the ketoacidosis with insulin)
warfarin upon presentation, but she should will cause hydrogen ions to come out of the
quit smoking to decrease her clotting tenden- cells, which occurs in exchange for positively
cies. charged potassium ions that move intracel-
Answer A is incorrect. Bile acid resins such as lularly, leading to further hypokalemia. Thus
cholestyramine and colestipol decrease serum patients with an apparent low or normal serum
triglycerides and cholesterol, which may indi- potassium level before administering insulin
are at risk of having a potentially life-threat-
Full-Length Exams

rectly, although not directly, improve vascular


health. eningly low total body potassium level, which
can cause cardiac conduction abnormalities
Answer B is incorrect. Statins decrease LDL and death. Hence, after the administration of
cholesterol but do not affect the rate of deep insulin, judicious monitoring and administra-
venous thrombosis formation. tion of potassium is the most important next
Answer C is incorrect. Oral contraceptives are step in the treatment of DKA.
associated with hypercoagulable state, so they Answer A is incorrect. Bicarbonate does not
would make deep venous thrombosis more undergo insulin-mediated transcellular shifts
likely. as does potassium. Bicarbonate levels often
Answer D is incorrect. Moderate exercise has normalize with the correction of hyperglyce-
been linked to improved cardiovascular health mia and fluid administration, which promotes
and a decreased incidence of acute coro- the diuresis of serum ketoacids. Bicarbonate
nary syndromes, although it is not specifically should be administered only in severe cases of
linked to deep venous thrombosis. Stasis, how- acidosis, such as a pH <6.9.
ever, can make deep venous thrombosis more Answer B is incorrect. Calcium does not un-
likely. dergo insulin-mediated transcellular shifts as
Test Block 1

Answer F is incorrect. Modest alcohol con- does potassium; hence, serum levels of cal-
sumption has been associated with improved cium do not fluctuate to the same extent with
cardiovascular health, although no specific link DKA and insulin administration. Serum cal-
to deep venous thrombosis has been proven. cium levels are usually not a major concern in
patients with DKA.
Test Block 1  •  Answers 523

Answer C is incorrect. Chloride does not un- nificantly increased risk of thromboembolism.
dergo insulin-mediated transcellular shifts as Thrombolysis/anticoagulation, however, helps
does potassium; hence, serum levels of chlo- restore perfusion to the brain and therefore
ride do not fluctuate to the same extent with should be done first.
DKA and insulin administration. Appropri-
Answer B is incorrect. Some strokes, particu-
ate fluid resuscitation is generally sufficient to
larly those involving the middle cerebral ar-
manage serum chloride levels in patients who
tery, are associated with significant parenchy-
may be dehydrated.
mal edema and subsequent mass effect, with
Answer E is incorrect. Sodium does not un- possible sequelae including herniation and
dergo insulin-mediated transcellular shifts as death. Hemicraniectomy is a rather novel ther-
does potassium. However, correction of the hy- apeutic intervention that involves temporarily
perglycemia will lead to a decrease in plasma removing half of the skull over the edematous
osmolality, which will cause water to move area with the goal of relieving pressure and
into the cells. This movement of water will reducing the chance of herniation. This inter-
lead to an increase in serum sodium concen- vention is not widely used and is not consid-
tration. However, unlike with potassium, this ered the standard of care in the management
change in serum sodium level is usually not of acute stroke.
potentially life-threatening. Fluid resuscitation
Answer C is incorrect. Heparin therapy is
is generally sufficient to manage serum sodium
the next step in management for patients
levels.
who have contraindications to thrombolytic
therapy. Such contraindications include past
45. The correct answer is E. The history and
history of hemorrhagic stroke, active internal
physical examination suggest a possible cere-
bleed, history of surgery within the past three
brovascular accident. Before any therapeutic
weeks, and any form of coagulopathy. As with
intervention is done, an emergent CT scan
tissue plasminogen activator, it is imperative
must be performed to rule out hemorrhage. In
to rule out the presence of intracranial hemor-
this case, we are told that the CT was normal,
rhage with a CT scan before initiating therapy.

Full-Length Exams
meaning that no hemorrhage was seen. Given
the high clinical suspicion for stroke, your at- Answer D is incorrect. Hyperglycemia wors-
tention should focus on the likely possibility of ens functional outcomes in cases of ischemic
an ischemic etiology. It is important to remem- stroke. It has been hypothesized that hypergly-
ber, however, that an ischemic infarct will of- cemia may increase local tissue acidosis and
ten not be visible on the initial scan, especially blood-brain barrier permeability. While glu-
if the scan is done within a few hours of symp- cose control with insulin would help minimize
tom onset. Tissue plasminogen activator (tPA), the harmful effects of hyperglycemia, it is not
a thrombolytic agent, is the best next step in the next step in management.
management given that the patient does not
have any obvious contraindications to throm- 46. The correct answer is A. This patient pre­
bolytic therapy. Treatment with tPA has been sents with a primary inoculation lesion, re-
shown to be very effective in the management gional lymphadenopathy, and low-grade fever
of acute ischemic stroke, especially if admin- characteristic of cat scratch disease, which is
istered within three hours of symptom onset. caused by Bartonella henselae. Typically (in
This form of treatment does, however, carry a 60% of cases) infection occurs when a child is
risk of hemorrhage. scratched or bitten by a bacteremic young cat.
Test Block 1

Answer A is incorrect. Once a hemorrhagic Answer B is incorrect. The spirochete Borrelia


stroke has been ruled out by CT scan, the pos- burgdorferi is the cause of Lyme disease. The
sibility of a cardioembolic source should be spirochete is carried by the Ixodes tick, which
investigated with an echocardiogram. Patients is most common in the northeastern United
with a history of atrial fibrillation are at sig- States. It initially presents with an expanding
524 Section III: Full-Length Examinations  •  Answers

ring-shaped lesion at the site of the tick bite total T4 level should be elevated rather than
known as erythema migrans. low.
Answer C is incorrect. Eikenella corrodens is a Answer C is incorrect. Graves disease is char-
gram-negative organism that is part of the nor- acterized by a low TSH level due to the circu-
mal flora of the mouth and nasopharynx. It is lating thyroid-stimulating immunoglobulins,
associated with infections resulting from hu- which elevate T3 and T4 levels and, via nega-
man bites. tive feedback, downregulate the level of TSH.
In this answer choice, the level of TSH is el-
Answer D is incorrect. Francisella tularensis is
evated, which would lead to elevated, not di-
the cause of tularemia. This disease is carried
minished, levels of T3 and T4.
by wild rabbits and ticks in the southeastern
United States. It often presents with lymphade- Answer E is incorrect. Total and free T4 lev-
nopathy and an ulcer at the site of entry as well els are expected to be low in the setting of
as with fever. low TSH levels. However, in Graves disease,
stimulation of TSH receptors on the thyroid
Answer E is incorrect. Pasteurella multocida is
follicular cells by anti-TSH receptor antibodies
caused by cat bites and dog bites. This infec-
stimulates the secretion of thyroid hormones
tion causes a rapid inflammation (often within
and results in increased total and free T4 lev-
hours) and is accompanied by purulent drain-
els in the setting of normal or even low TSH
age.
levels. The resulting negative feedback loop
47. The correct answer is D. The vignette de- to the anterior pituitary leads to reduced TSH
scribes a classic history of an autoimmune hy- levels.
perthyroidism, Graves disease. In this disorder,
48. The correct answer is E. Endemic African
thyroid follicular cells are stimulated to synthe-
Burkitt lymphoma frequently manifests as
size and secrete thyroid hormone by anti-TSH
masses involving the mandible (and less often
receptor antibodies, leading to increased levels
the kidneys, ovaries, and adrenals). Affected
of thyroxine (T4) and triiodothyronine (T3)
lymph nodes show a classic “starry-sky” pattern
Full-Length Exams

in the blood, which results in negative feed-


of neoplastic lymphoblasts and non-neoplastic
back on the anterior pituitary and suppression
“tingible body” macrophages. The neoplastic
of TSH secretion. Thus, both free T4 and to-
cells typically show a translocation of the c-myc
tal T4, which includes free T4 and T4 bound
gene on chromosome 8 onto the immunoglob-
to proteins in the blood (eg, albumin and
ulin heavy-chain gene on chromosome 14. It is
thyroxine-binding globulin) will be increased,
more common in African children and is asso-
while blood TSH levels will be low relative to
ciated with the Epstein-Barr virus (EBV). EBV
the normal baseline.
also is associated with heterophile-positive
Answer A is incorrect. An elevated TSH level mononucleosis, oral hairy leukoplakia in pa-
is not characteristic of Graves disease, and el- tients infected with HIV, lymphoproliferative
evated T4 levels should result in a lower TSH disorders in immunocompromised hosts (in-
level due to negative feedback on the anterior cluding Hodgkin and non-Hodgkin lympho-
pituitary. mas), and nasopharyngeal carcinoma. Interest-
Answer B is incorrect. An elevated TSH level ingly, nonendemic Burkitt lymphoma presents
is not characteristic of Graves disease, and an as an abdominal mass and is associated with
elevated free T4 level should result in a lower HIV, not EBV.
Test Block 1

TSH level due to negative feedback on the an- Answer A is incorrect. Cervical adenocarci-
terior pituitary. Furthermore, thyroid hormone noma is associated with human papillomavi-
binding to proteins in the blood should not be rus.
decreased but instead should be increased in
Answer B is incorrect. Gastric adenocarci-
the setting of increased free T4. Therefore, the
noma may be associated with Helicobacter py-
Test Block 1  •  Answers 525

lori, but not EBV. Gastric carcinomas, 90% of cirrhosis (secondary to alcohol, or crypto-
which are adenocarcinomas, are more com- genic), hemochromatosis, aflatoxin ingestion,
mon in developing countries than in industri- and a1-antitrypsin deficiency.
alized countries; they rarely occur before age
Answer D is incorrect. Heterophile-negative
40 years; and incidence peaks in the seventh
mononucleosis is associated with cytomegalo-
decade with a male:female ratio of 2:1. Gas-
virus, acute HIV, toxoplasmosis, and human
tric carcinoma also is associated with atrophic
herpesvirus types 6 and 7. Although hetero-
gastritis, post-gastrectomy states, achlorhydria,
phile-negative mononucleosis may present
pernicious anemia, Ménétrier’s disease, and
with similar symptoms of fever and lymphade-
adenomatous polyps.
nopathy without pharyngitis (as in this case),
Answer C is incorrect. Hepatocellular car- these infectious agents typically would not re-
cinoma is associated with chronic HBV and sult in the large mass on the right mandible.
HCV infections. Other noninfectious associa- Heterophile-positive mononucleosis is associ-
tions with hepatocellular carcinoma include ated with EBV.

Full-Length Exams
Test Block 1
This page intentionally left blank
Test Block 2

527
528 Section III: Full-Length Examinations  •  Questions

Q u e st i o n s

1. An oncologist recently discovered that certain 3. A 9-year-old girl is brought to the emergency
cancerous cells secrete a protein named ca- department with an arrhythmia that started
1panc. Using this protein, he developed a se- while she was sitting in class. Since birth she
rum test to detect this type of cancer. He per- has had a disorder that predisposes her to ar-
formed the blood test on 1000 patients. One rhythmias, but because her parents were not
hundred of these patients had the cancer, and present, the hospital staff was unable to deter-
the test came back positive for 60 of them, mine her medical history. The staff attempts to
while for the remaining 40 patients the test was treat the patient with an atrioventricular nodal
negative. Nine hundred of the patients did not block antiarrhythmic, which does not cure the
have the cancer; however, the test was positive problem and instead makes it worse. What
for 100 of them. In the remaining 800, the test congenital pathophysiology could this girl
came back negative. Which of the following have had?
numbers represents how well the test identified
(A) Atrioventricular accessory tract
those who had the cancer?
(B) Atrioventricular nodal re-entry
(A) 10.0% (C) Tetralogy of Fallot
(B) 37.5% (D) Ventricular hypertrophy
(C) 60.0% (E) Ventricular tachycardia
(D) 88.8%
(E) 90.0% 4. A 57-year-old African-American man com-
(F) 95.2% plains of lethargy, weakness, and confusion.
His wife says that he also has been complain-
2. A 33-year-old pregnant woman presents to ing of increasing pain in his bones, particu-
her physician with tendonitis of the left shoul- larly in his hips, lower back, and legs. Levels
der. The patient has taken nonsteroidal anti- of protein and calcium are elevated. Results of
Full-Length Exams

inflammatory drugs (NSAIDs) in the past, digital rectal examination and prostate-specific
but developed gastric ulcers associated with antigen testing are normal. X-ray of his skull is
long-term NSAID use. A medication is avail- shown in the image. Which of the following
able that will reduce the incidence of NSAID- laboratory test results are diagnostic?
associated gastric ulcers, but it is contraindi-
cated in pregnancy. Which of the following is
an adverse event associated with this medica-
tion that makes it absolutely contraindicated
during pregnancy?
(A) Induction of labor
(B) Intrauterine growth retardation
(C) Neural tube defects
(D) Placenta previa
(E) Premature fetal closure of ductus arteri­
osus
Test Block 2

Reproduced, with permission, from USMLERx.com.


Test Block 2  •  Questions 529

(A) Bone mineral density T score >2.5 Alkaline


(B) Elevated serum parathyroid hormone Choice Calcium Phosphorus
phosphatase
(C) Fasting glucose >126 mg/dL
(D) Monoclonal M spike on protein electro- A
phoresis
B normal normal normal
(E) Positive HLA-B27 haplotype
(F) Prostate specific antigen of 4 ng/mL C normal normal

5. A 14-year-old high school freshman presents to D


her family doctor for a sports physical. She has
not played organized sports in the past but is
Reproduced, with permission, from USMLERx.com.
in good physical shape. She mentions that she
experienced severe leg cramps after trying out
for the soccer team last week. The night after (A) A
the tryouts, she noticed that her urine had a (B) B
reddish tinge. She has no other medical com- (C) C
plaints. Her physician orders an ischemic fore- (D) D
arm exercise test, which reveals no increase
in venous lactate. Which of the following en- 7. A 45-year-old man comes to the physician with
zymes is most likely deficient in this patient? a three-day history of a temperature of 39°C
(A) a-1,6-Glucosidase (102.2°F). He also complains of headache,
(B) Cystathionine synthase neck stiffness, and a maculopapular rash on
(C) Glucose-6-phosphatase his trunk. A diagnosis of meningitis is made,
(D) Glycogen phosphorylase and a smear and culture of his cerebrospinal
(E) Lysosomal a-1,4-glucosidase fluid reveal a gram-negative diplococcus as the
causative agent. Which of the following symp-
6. A 62-year-old woman visits her family doctor toms can develop as a severe complication of

Full-Length Exams
with complaints of chronic lower back pain. this infection?
She has also suffered from severe rheuma- (A) Acute renal failure and thrombocytopenia
toid arthritis of the knees for the past 10 years, with hemolytic anemia
which has significantly limited her physical (B) Fever, migratory polyarthritis, and carditis
activity despite appropriate medical treatment. (C) Fever, new murmur, small erythematous
Her mother died at age 58 years from a mas- lesions on the palms, and splinter hemor-
sive pulmonary embolus following hip replace- rhages on the nail bed
ment surgery after falling. Which of the fol- (D) Shock, widespread purpura, disseminated
lowing sets of serum laboratory findings best intravascular coagulation, and adrenal in-
describes her condition? sufficiency
(E) Symmetric ascending muscle weakness be-
ginning in the distal lower extremities
Test Block 2
530 Section III: Full-Length Examinations  •  Questions

8. A 57-year-old man with a history of arthritis 10. The retinoblastoma gene on chromosome 13,
treated with a cyclooxygenase (COX)-2 in- which encodes the retinoblastoma tumor sup-
hibitor presents to the emergency department pressor protein, has served as a paradigm for
because of sudden shortness of breath and the study of several other tumor suppressor
chest pain radiating to his jaw and left arm. genes. Survivors of hereditary retinoblastoma
Which of the following proposals best explains have increased risk for the development of ad-
why the patient may have been better off us- ditional neoplasms. Which of the following is
ing aspirin for his arthritis instead of selective the most likely non-ocular tumor to occur in a
COX-2 inhibitors? (PGI2 = prostaglandin I2, survivor of hereditary retinoblastoma?
TxA2 = thromboxane A2)
(A) Esophageal adenocarcinoma
(B) Femoral osteosarcoma
(C) Medullary carcinoma of the thyroid
Choice PGI2 TxA2 (D) Renal cell carcinoma
(E) Serous cystadenoma of the ovary
COX-2 inhibitor --
A
Aspirin (F) Squamous cell carcinoma of the lung
--
(G) Transitional cell carcinoma
COX-2 inhibitor
B
Aspirin
11. An obese, 56-year-old African-American man
C COX-2 inhibitor
Aspirin
with a 25-pack-year history of smoking experi-
ences chest pain associated with myocardial
D
COX-2 inhibitor -- infarction. The pain radiates to the man’s left
Aspirin
shoulder and down his left arm. What is the
COX-2 inhibitor -- reason for referred pain to this region?
E
Aspirin

(A) Common lymphatic drainage pathways of


Reproduced, with permission, from USMLERx.com. the molecular mediators of inflammation
and pain
(B) Proximity of sensory nerve fiber tracts in
Full-Length Exams

(A) A the anterior horn of the spinal cord


(B) B (C) Proximity of sensory nerve fiber tracts in
(C) C the posterior horn of the spinal cord
(D) D (D) Shared parasympathetic pathways
(E) E (E) Shared sympathetic pathways

9. A 30-year-old woman with systemic lupus ery- 12. A 57-year-old man who is HIV-positive pre­
thematosus treated with high-dose prednisone sents to his physician with headache, nausea
comes to her physician complaining of easy and vomiting, and a change in mental status.
fatigability. Blood studies reveal a hemoglobin No nuchal rigidity is noted. A lumbar punc-
level of 10 g/dL, a low serum iron level, ele- ture is performed and shows a high opening
vated ferritin level, and low total iron-binding pressure. A preparation of his bronchoalveolar
capacity. Normocytic RBCs are seen on blood lavage fluid with India ink stain is shown in
smear. Which of the following is the most ap- the image. Intravenous treatment is started for
propriate treatment for this patient’s anemia? the acute condition. Which of the following
adverse effects might occur with this patient’s
(A) Erythropoietin
initial treatment?
Test Block 2

(B) Ferrous sulfate


(C) Folate
(D) Parenteral vitamin B12
(E) Phlebotomy
Test Block 2  •  Questions 531

(C) Metformin
(D) Orlistat
(E) Rosiglitazone

14. A woman with a 2-year-old son comes to her


physician because she has been unable to
conceive a second child for more than a year.
The woman is currently breastfeeding her son.
Which of the following best explains the physi-
ologic mechanism currently preventing her
from getting pregnant?
(A) Prolactin inhibits secretion of estrogen
Courtesy of Dr. Leanor Haley, Centers for Disease Control from the ovaries
and Prevention. (B) Prolactin inhibits secretion of follicle-
stimulating hormone from the anterior pi-
tuitary gland
(A) Arrhythmia (C) Prolactin inhibits secretion of gonadotro-
(B) Bone marrow suppression pin-releasing hormone from the hypothala-
(C) Flushing mus
(D) Gynecomastia (D) Prolactin inhibits secretion of luteinizing
(E) Nausea and vomiting hormone from the anterior pituitary gland
(E) Prolactin inhibits secretion of progesterone
13. A 56-year-old obese man with unknown medi- from the corpus luteum
cal history presents to the emergency depart-
ment. He was found by his wife on the floor 15. A neonate has apparent difficulty with feeding,
of the bathroom, conscious but disoriented, including repeated episodes of choking and
vomiting, and clutching his abdomen. She is coughing with attempted breastfeeding and

Full-Length Exams
unsure of the name of his one oral medication, bottle-feeding. The child, when not feeding,
but knows he takes it regularly, twice daily. He has a normal temperature, heart rate, respira-
was recently admitted to another hospital for a tory rate, and blood pressure. On physical ex-
cholecystectomy. On examination, the patient amination the lungs are clear and the heart has
is breathing heavily and quickly, but other vi- no adventitial sounds. Facies are normal. What
tal signs are normal. Laboratory test results in- is the embryologic abnormality that has caused
clude: this child’s feeding difficulties?
Na+: 142 mEq/L (A) Failure of the first and second branchial
K+: 4.0 mEq/L arches to fuse
Cl-: 105 mEq/L (B) Failure of the maxillary processes to fuse
HCO3-: 19 mEq/L (C) Failure of the medial nasal processes to
Blood urea nitrogen: 20 mg/dL fuse
Creatinine: 1.6 mg/dL (D) Failure of the palatine processes to fuse
Glucose: 256 mg/dL (E) Failure of the second and third branchial
What drug is the patient most likely taking? arches to fuse

(A) Glyburide
Test Block 2

(B) Insulin
532 Section III: Full-Length Examinations  •  Questions

16. A woman strikes her head in a car crash and is


admitted to the hospital. On admission, she re-
ports a medical history of hypertension, hyper-
cholesterolemia, diabetes type II, fibromyalgia,
and breast cancer for which she underwent a
lumpectomy and chemotherapy. Her current
medications include simvastatin, metformin,
furosemide, hydrocodone, and tamoxifen.
During the interview, she complains of severe
thirst and frequent urination, and her urine
bag is nearly full after only two hours. Which
medication should the patient be given imme-
diately?
(A) Demeclocycline Reproduced, with permission, from USMLERx.com.
(B) Desmopressin
(C) Furosemide
(D) Insulin (A)
c-myc; t(8,14)
(E) Mannitol (B)
bcl-2; t(14;18)
(C)
erb B2; t(11;14)
17. A 46-year-old man with a history of obstruc- (D)
p53; t(8:14)
tive sleep apnea, hypertension, and type 2 dia- (E)
ras; t(11:14)
betes mellitus is referred to a sleep laboratory
for evaluation. On arrival he is hooked up to 19. A 15-year-old girl presents to her pediatrician
an electroencephalograph (EEG) and be- for her annual physical examination. She re-
gins to fall asleep a few minutes later. During ports that she has been in good health but is
the night the patient has multiple apneic epi- somewhat concerned because, unlike all of
sodes preceded by very loud snoring spells. At her friends, she has not yet started to menstru-
Full-Length Exams

3 o’clock in the morning the patient has beta ate or develop breasts. On physical examina-
waves on his EEG. This patient is currently in tion, she is 150 cm (59 in) tall and is obese
what stage of sleep? with a short neck. Her genitalia and breasts
are Tanner stage I. Laboratory studies show el-
(A) Awake evated serum follicle-stimulating hormone and
(B) Stage 1 luteinizing hormone levels and a low estradiol
(C) Stage 2 level. Which of the following findings is most
(D) Stage 3 likely to be seen in a patient with this condi-
(E) Stage 4 tion?
18. A 51-year-old man visits his physician for a rou- (A) Blood pressure in upper extremity greater
tine physical examination. During the exami- than that in lower extremity
nation, the physician notices cervical, axillary, (B) Carotid bruits
inguinal, and femoral lymphadenopathy. The (C) III/IV late systolic murmur with midsys-
patient reports the size of the nodes has waxed tolic click
and waned over the past several months. He (D) Mediastinal widening on x-ray of the chest
never came in to get them checked because (E) Pulsus parvus et tardus
they did not cause him any pain. A lymph
Test Block 2

node biopsy is performed (see image). Which 20. A 53-year-old man with a long-standing his-
of the following oncogenes and translocations tory of allergic rhinitis and asthma presents
is associated with this type of lymphoma? with uveitis, mild hearing loss, numbness and
tingling in his right hand, and diffuse joint
pain for the past 10 days. Physical examina-
Test Block 2  •  Questions 533

tion shows weak to absent left knee patellar 22. A 32-year-old female dialysis patient visits her
reflexes (right knee reflex strong and intact). general internist for a health maintenance
Laboratory studies show a markedly elevated visit. She subsequently has a dual-energy x-ray
eosinophil count. A diagnosis is made, and absorption examination, which demonstrates
the patient is treated with cyclophosphamide. significant osteoporosis. What is the most likely
Further laboratory studies show elevated se- etiology of this patient’s osteoporosis?
rum levels of the most common autoantibody
(A) 1,25-Dihydroxycholecalciferol excess
associated with this condition. What structure
(B) Chronic metabolic alkalosis
is primarily targeted by the autoantibodies that
(C) Hypercalcemia
are most likely elevated in this patient’s serum?
(D) Hypophosphatemia
(A) Acetylcholine receptors (E) Secondary hyperparathyroidism
(B) Neutrophils
(C) Oligodendrocytes 23. A 62-year-old man arrives at his doctor’s of-
(D) RBCs fice complaining of recent onset dull pain in
(E) Thyroid-stimulating hormone receptors his left flank region. He is a retired steel plant
worker with a long history of excessive smok-
21. A 48-year-old woman is brought to the emer- ing, hypertension, and obesity. He does not
gency department after having her first gen- recall any history of similar illness in his fam-
eralized seizure. When questioned, she com- ily. On physical examination a firm, homoge-
plains of chronic early morning headaches neous, nontender movable mass is palpated
that have increased in severity over the past six deep in the left umbilical region near the
months. A neurological examination shows a lower pole of the kidney. Laboratory tests show
slight loss of her temporal visual field on the hypercalcemia, hypophosphatemia, and mod-
right. Imaging studies reveal the presence of a erate polycythemia. Urinalysis reveals the pres-
tumor compressing the optic nerve. Biopsy re- ence of hematuria. Which of the following is
sults are shown in the image. Which of the fol- the most likely diagnosis?
lowing is the most likely diagnosis?
(A) Adult polycystic kidney disease

Full-Length Exams
(B) Angiomyolipoma
(C) Pheochromocytoma
(D) Renal cell carcinoma
(E) Wilms tumor

24. A 3-year-old boy is brought to his pediatrician’s


office because his parents noticed worsening
protrusion of his abdomen. After thorough
imaging studies and histopathologic confirma-
tion of Wilms tumor, the physician prescribes
a medication that acts by causing arrest of the
cell cycle during metaphase. However, the
parents are concerned about the use of that
medication because of what they have read
Courtesy of Wikipedia.
about its adverse effects. Which adverse effect
are the parents most likely concerned about?
Test Block 2

(A) Hemangioblastoma (A) Cardiotoxicity


(B) Meningioma (B) Hemorrhagic cystitis
(C) Oligodendroglioma (C) Hyperglycemia
(D) Papillary thyroid carcinoma (D) Pulmonary fibrosis
(E) Pituitary adenoma (E) Neurotoxicity
534 Section III: Full-Length Examinations  •  Questions

25. A 7-year-old girl has numerous vesicles on her ings are shown in the image. What laboratory
face, particularly around her mouth after fall- changes can be expected with this patient’s
ing and scraping her face on the ground. Over condition?
a few days the vesicles turn into pustules and
crust over, becoming flaky and light yellow in
color. Which of the following statements about
the organism most likely responsible for this
girl’s infection is correct?
(A) Endotoxin is present in the outer mem-
brane of this organism
(B) Sabouraud agar is required to culture this
bacterium
(C) The bacterium is b-hemolytic and resistant
to bacitracin
(D) The bacterium is b-hemolytic and sensi-
tive to bacitracin
(E) The bacterium is a facultative intracellular
organism
Reproduced, with permission, from USMLERx.com.
26. An 18-year-old man presents complaining of
an inability to keep his balance. A complete (A) Decreased serum ferritin
neurologic examination is conducted, and fun- (B) Increased iron-binding capacity
duscopy reveals retinal hemangioblastomas. (C) Increased transferrin saturation
He reports that his mother has a history of a (D) No change in plasma iron
“brain tumor,” the specifics of which he is un- (E) No change in serum ferritin
aware. Given the classic evolution of this pa-
tient’s disease, for which of the following does 28. A 25-year-old woman presents to her family
Full-Length Exams

this patient require close monitoring? physician for a routine check-up. Physical ex-
(A) Astrocytoma amination reveals a mildly overweight woman,
(B) Colon cancer but is otherwise unremarkable. A fasting lipid
(C) Depression panel, however, shows an LDL cholesterol
(D) Pheochromocytoma level of 310 mg/dL, HDL cholesterol level of
(E) Renal mass 42 mg/dL, triglyceride level of 150 mg/dL, and
total cholesterol level of 382 mg/dL. Because
27. A 45-year-old white man presents to his doc- a diagnosis of familial hypercholesterolemia is
tor complaining of weakness, lethargy, and suspected, the doctor initiates treatment of her
decreased libido over the past few months. condition. Soon after starting treatment, how-
He also notes that his skin seems darker than ever, she presents with myalgias. Laboratory
usual. The patient denies any past medical values show elevated levels of aspartate ami-
conditions and has a family history of diabe- notransferase, alanine aminotransferase, and
tes mellitus. The patient’s father died from a creatinine kinase. Which of the following de-
“long-standing liver problem.” On physical ex- scribes the mechanism of the intervention that
amination the patient appears to have skin pig- was initiated to treat this patient’s hypercholes-
mentation, which is most evident on his face terolemia?
Test Block 2

and arms. The heart and lung examination is (A) Inhibition of 3-hydroxy-3-methylglutaryl
normal. On abdominal examination, the liver coenzyme A reductase enzyme
edge is palpable two finger-widths below the (B) Inhibition of bile-acid reuptake in the in-
costochondral angle. The patient is referred testine
to a surgeon who performs a liver biopsy; find-
Test Block 2  •  Questions 535

(C) Inhibition of cholesterol uptake by the in- 31. A 78-year-old man comes to the physician
testinal brush border for evaluation after falling five times in two
(D) Reduced transfer of cholesterol from HDL months. An x-ray skeletal survey reveals no
to LDL and delayed HDL clearance fractures, but the patient admits to worsen-
(E) Selective removal of LDL molecules from ing urinary incontinence over the previous
the blood via immunoadsorption columns four months. His wife states that his memory
and concentration have deteriorated recently.
The patient’s vital signs are normal, and his
29. Informed consent is the legal demonstration of physical examination is notable for a wide-
a patient’s understanding of the risks, benefits, based gait with short steps. A Mini-Mental
and outcomes of treatments and alternatives. State Examination results in a score of 26/30.
As such, the patient is taking responsibility for His funduscopic examination is normal, and
making decisions in the medical process, but is his neurologic examination is notable for
authorizing the physician to provide the treat- slight bradykinesia without tremor. Labora-
ment. While informed consent is an important tory tests, including serum vitamin B12, folate,
ethical principle, there are exceptions to the and thyroid-stimulating hormone, are normal.
requirement of obtaining it. Which of the fol- What is the most likely etiology of this patient’s
lowing circumstances is a valid exception to recent decline?
the requirement of obtaining informed con-
(A) Alzheimer disease
sent?
(B) Hypothyroidism
(A) A competent patient’s son wishes to waive (C) Multi-infarct dementia
his father’s rights (D) Normal pressure hydrocephalus
(B) A paralyzed patient cannot speak but can (E) Parkinson disease
nod his head for consent
(C) A patient is mentally disabled but legally 32. A 31-year-old woman with newly diagnosed
competent tuberculosis is begun on a standard treatment
(D) A physician believes that informing the pa- regimen. During a follow-up appointment, it is

Full-Length Exams
tient will be detrimental to the patient’s noted as she walks into the room that her gait
health is markedly unsteady. Physical examination is
(E) A stable patient is in the emergency de- notable for decreased sensation over the upper
partment for treatment of a femur fracture and lower extremities. Which of the following
drugs is most likely causing this patient’s symp-
toms?
30. A 45-year-old man presents to the clinic be-
(A) Ethambutol
cause of severe back pain, muscle weakness,
(B) Isoniazid
and fatigue that gradually started two months
(C) Levofloxacin
ago. Physical examination reveals slightly
(D) Pyrazinamide
darkened skin and a systolic blood pressure in
(E) Rifampin
the 90s. Upon further questioning the patient
reveals he stopped taking a medication about
three months ago. Which of the following is
the most likely medication?
(A) Dehydroepiandrosterone
(B) Fluconazole
Test Block 2

(C) Hydrocortisone
(D) Metformin
536 Section III: Full-Length Examinations  •  Questions

33. A 19-year-old man who recently emigrated


from Mexico comes to the emergency depart-
ment because of blood in his sputum. The
patient mentions he has had weight loss and
night sweats. On examination, the patient has
a fever and bronchial breath sounds with crepi-
tant rales. Laboratory tests show lymphocytosis
and an increased erythrocyte sedimentation
rate. X-ray of the chest shows a calcified lung
lesion and hilar lymphadenopathy. Of the fol-
lowing, which is the stain used to identify the
most likely infectious organism?
(A) Congo red
(B) Giemsa Reproduced, with permission, from USMLERx.com.
(C) India ink
(D) Periodic acid-Schiff
(E) Ziehl-Neelsen (A) IgA
(B) IgD
34. A 65-year-old postmenopausal woman presents (C) IgE
with progressive constipation and frequent, ex- (D) IgG
cessive urination. She reports a 40-pack-year (E) IgM
history of smoking. On physical examination,
respiratory findings prompt an x-ray film of the 36. A patient presents to the emergency depart-
chest, in which a concerning circular lesion ment with a severe headache, palpitations, and
is found overlying the right hilum. Laboratory elevated blood pressure. He is found to have
testing demonstrates a decreased phosphorus elevated urinary vanillylmandelic acid levels.
level. Which of the following is the most likely He is diagnosed with a pheochromocytoma
Full-Length Exams

cause of this patient’s symptoms? with predominantly elevated norepinephrine


levels. Which of the following agents will an-
(A) Central bronchogenic carcinoma tagonize both the vascular and cardiac actions
(B) Cervical sympathetic chain compression of norepinephrine?
(C) Chronic silica exposure
(D) Congenital chloride channel dysfunction (A) Atenolol
(E) Dynein arm defect in cilia (B) Doxazosin
(F) Ectopic ADH production (C) Esmolol
(G) Solitary parathyroid adenoma (D) Isoproterenol
(E) Labetalol
35. The image demonstrates a specialized epi-
thelium that overlies a type of peripheral 37. A physician is caring for a hospitalized 31-year-
lymphoid tissue. It is thought that most dis- old man with long-standing, poorly controlled
seminated Mycobacterium avium infections in type 1 diabetes mellitus. He is blind and has
patients with AIDS are acquired by the bacte- peripheral neuropathy with sensory loss in
ria penetrating through this tissue-type of the both feet, and his most recent hemoglobin A1c
immune system. What is the main class of an- level was 13.9%. He recently presented with
tibodies associated with this lymphoid tissue? altered mental status, polyuria, and polydip-
Test Block 2
Test Block 2  •  Questions 537

sia. At that time, his serum glucose level was 800 people for the study. The physician places
475 mg/dL, arterial blood pH was 6.96, and his 100 patients with the worst and most frequent
anion gap was 27. Since then his acidosis has migraines in the medication group, as he
resolved with appropriate treatment, and fin- thinks that they are in most need of the drug’s
gerstick blood glucose levels have normalized. benefit. Which of the following best explains
However, he has persistent nasal discharge; why the drug may not perform up to expecta-
paranasal sinus tenderness; and new onset tions?
of periorbital edema, proptosis, facial numb-
(A) Differences in group size
ness, and obtundation. Fungal stain of fluid
(B) Late-look bias
obtained from urgent surgical sinus drainage
(C) Recall bias
would most likely reveal which of the follow-
(D) Sampling bias
ing?
(E) Selection bias
(A) 45-degree angle branching, septate hyphae
with rare fruiting bodies 40. A 62-year-old man with a known diagnosis of
(B) 5- to 10-μm yeasts with wide capsular halo benign prostatic hyperplasia is seen at his an-
on India ink stain nual physical and found to have a prostate-
(C) Broad-based budding dimorphic fungi specific antigen level of 11.2 ng/mL, up from
(D) Irregular, broad, nonseptate hyphae with 6.4 ng/mL the previous year. Ultrasound-
90-degree branching guided transrectal prostate biopsies are per-
(E) Pseudohyphae with budding yeasts formed. If the biopsies are positive for prostate
cancer, what zone of the prostatic tissue is
38. A 64-year-old woman presents to her primary most likely involved by the cancer cells?
care physician with fatigue, weakness, and a
(A) Neurovascular bundle
weight loss of 4.5 kg (10 lb) in the past four
(B) Peripheral
months. Also, her vision has deteriorated over
(C) Periurethral
that time, and she has had several severe nose-
(D) Seminal vesicle
bleeds. Physical examination demonstrates
(E) Transition

Full-Length Exams
hepatosplenomegaly, and laboratory tests show
an increased total protein level. Serum pro- 41. A patient presents to the emergency depart-
tein electrophoresis reveals a large spike in the ment complaining of chills, cough, and mal-
gamma region. A skeletal survey is negative. aise. His temperature is 38.6°C (101.5°F). An
Which of the following is the most likely diag- x-ray of his chest demonstrates consolidation of
nosis? the right middle lobe. A complete blood count
(A) Chronic lymphocytic leukemia reveals leukocytosis with a left shift on differ-
(B) Diabetes mellitus ential. Based on the results of Gram staining
(C) Monoclonal gammopathy of undeter- and sputum culture, the patient is treated with
mined significance cefazolin. Gram staining and sputum culture
(D) Multiple myeloma most likely revealed which of the following?
(E) Waldenström macroglobulinemia (A) Anaerobic, encapsulated, gram-negative
rods without endotoxin
39. A pharmaceutical company has created a new
(B) Gram-positive cocci in clusters
drug that, when taken daily, is thought to be
(C) Gram-positive rods with long branching
highly effective at preventing the onset of mi-
filaments
graines. The company would like to market
Test Block 2

(D) Gram-positive, aerobic, spore-forming rods


the drug and is conducting a study to look at
(E) Oxidase-positive, non-lactose-fermenting,
its benefits and possible risks. In coordination
gram-negative rods
with a physician at a local hospital, it enrolls
538 Section III: Full-Length Examinations  •  Questions

42. A 20-year-old woman presents to the physician 45. A 36-year-old man who completed a marathon
because of a history of bloody diarrhea and ab- six hours earlier presents to the emergency de-
dominal pain. She states that she has not trav- partment with severe muscle pain and swelling
eled recently or changed her eating habits. A and complaints of red urine. Laboratory tests
stool culture is negative for an infectious cause show a creatine kinase level of 6800 U/L but
of diarrhea. Flexible sigmoidoscopy shows nu- no RBCs or WBCs on urinalysis. Which of the
merous lesions in the descending colon inter- following symptoms would most likely also be
rupted by normal-appearing mucosa. Which present?
of the following features would most likely be
(A) Arrhythmia
present on a tissue biopsy of the affected re-
(B) Hepatomegaly
gion?
(C) Pain in a dermatomal distribution
(A) Cells with loss of mucin and hyperchro- (D) Pain on urination
matic nuclei (E) Shuffling gait
(B) Crypt abscesses
(C) Hyperplastic goblet cells 46. A 43-year-old woman visits her obstetrician for
(D) Noncaseating granulomas her scheduled check-up at 20 weeks’ gestation.
(E) Ulcerated mucosa only Her obstetrician recommends that she have
a series of screening tests. One of these tests
43. A 38-year-old man from rural Guatemala dies shows an elevated serum a-fetoprotein level.
of heart failure. Notable autopsy findings in- Which of the following is most likely respon-
clude dilated large bowel. Microscopy of a sible for this result?
blood sample taken in the emergency depart-
(A) Bilateral renal agenesis
ment before the man’s death shows flagellated
(B) Failure of the ductus arteriosus to close
parasites. Which of the following parasites did
(C) Inadequate folic acid intake
this man most likely harbor?
(D) Inheritance of a recessive disorder
(A)
Cryptosporidium species (E) Nondisjunction occurring during meiosis
(B)
Entamoeba histolytica
Full-Length Exams

(C)
Giardia lamblia 47. The father of a 7-year-old boy is contacted by
(D)
Toxoplasma gondii his child’s schoolteacher because she is con-
(E)
Trypanosoma cruzi cerned about his inattentiveness during class.
The teacher states that the boy appears to be
44. A 27-year-old man presents to the emergency daydreaming multiple times each day, during
department with a cough productive of blood- which time he blinks his eyes repeatedly. She
tinged sputum. He also complains that in the reports that the boy’s daydreaming episodes are
past couple weeks he has noticed increased brief and he is able to refocus shortly following
fatigue and some blood in his urine. A renal the daydream. What is the most appropriate
biopsy is performed that, upon on immuno- therapy for the child’s underlying condition?
fluorescence staining, shows a linear pattern
(A) Carbamazepine
of IgG deposition along the basement mem-
(B) Clonazepam
brane. Which of the following is most likely
(C) Ethosuximide
responsible for this patient’s disease?
(D) Gabapentin
(A) Anti-type III collagen antibodies (E) Methylphenidate
(B) Anti-type IV collagen antibodies (F) Tiagabine
Test Block 2

(C) Antineutrophil cytoplasmic antibodies


(D) Immune complexes
(E) T lymphocytes
Test Block 2  •  Questions 539

48. A 68-year-old man with a six-month history of


back pain and fatigue presents to the emer-
gency department because of severe low back
pain. MRI shows multiple lytic bone lesions
scattered throughout the man’s body, and
laboratory results show a serum M spike and
light chain proteinuria. Bone marrow biopsy
demonstrates an excessive number of the cells
shown in the image. Which of the following is
most likely to be present in these cells?

Reproduced, with permission, from USMLERx.com.

(A) Abundant mitochondria


(B) Abundant rough endoplasmic reticulum
(C) Abundant secondary lysosomes
(D) Abundant smooth endoplasmic reticulum
(E) Abundant Nissl bodies

Full-Length Exams
Test Block 2
540 Section III: Full-Length Examinations  •  Answers

An s w e r s

1. The correct answer is C. It is important to un- Answer F is incorrect. The 95.2% figure is the
derstand that the question is asking for the sen- negative predictive value, or the probability
sitivity, the proportion of people who have the that the person with a negative test really does
disease and test positive out of all the people not have the cancer. It is calculated as TN /
who have the disease. It is calculated by TP / (TN + FN) (where TN means true-negative
(TP + FN), where TP means true-positive and and FN means false-negative), or 800 / (800 +
FN means false-negative. The true-positives 40) = 95.2%.
in the vignette represent those with the can-
cer who correctly tested positive with this new 2. The correct answer is A. Misoprostol is a pros-
test (n = 60). False-negatives are those with the taglandin E1 analog. One of its mechanisms of
cancer who tested negative with the new test action consists of increased production and se-
(n = 40); thus, 60 / (60 + 40) = 60%. Screen- cretion of gastric mucous barrier. This charac-
ing tests theoretically would aim to identify teristic makes the drug useful in concomitant
all those with the disease, and therefore high chronic NSAID use to decrease the incidence
sensitivities are desired. In this case 60% rep- of NSAID-induced peptic ulcers. Misoprostol,
resents a low number, and the ca-1panc blood however, is contraindicated in women of child-
test would not be a good screening test for the bearing potential due to its abortifacient prop-
cancer. erties. It can also be used for medical termina-
tion of pregnancy of <49 days (in conjunction
Answer A is incorrect. The 10.0% figure repre-
with mifepristone) and off-label for ripening
sents the prevalence of the disease, calculated
and labor induction. Another common adverse
as total cancer/total people (100/1,000).
effect of misoprostol is diarrhea.
Answer B is incorrect. The 37.5% figure is
Answer B is incorrect. Intrauterine growth
the positive predictive value, or the probabil-
restriction (IUGR) can result from poor ma-
ity that someone with a positive test (ca-1panc)
Full-Length Exams

ternal nutrition, hypertension, infections, con-


truly does have the cancer. The predictive val-
genital anomalies, and smoking. Misoprostol
ues vary with how prevalent a disease is in the
has not been associated with IUGR.
population. It is calculated as TP / (TP + FP)
(where TP means true-positive and FP means Answer C is incorrect. The chance of neural
false-positive) or 60 / (60 + 100) = 37.5%. tube defects is increased in women who do not
take prenatal folate supplements. Misoprostol
Answer D is incorrect. The 88.8% figure rep-
has not been associated with neural tube de-
resents the specificity of the blood test, which
fects.
measures the proportion of the people who
don’t have the disease and test negative out of Answer D is incorrect. Placenta previa results
all the people who don’t have the disease. It is from abnormal placental implantation where
important to correctly detect those without the placenta covers the cervical os. Placenta previa
disease in order to prevent them from undergo- is a common cause of painless third-trimester
ing unnecessary treatment or studies that could bleeding. Some of the risk factors for this phe-
be painful or harmful to the patient. It is cal- nomenon are prior cesarean section, multi-
culated as TN / (TN + FP) (where TN means parity, advanced maternal age, multiple gesta-
true-negative and FP means false-positive), or tions, and prior placenta previa. Misoprostol
does not appear to increase the risk of placenta
Test Block 2

800 / (800 + 100) = 88.8%.


previa.
Answer E is incorrect. The 90.0% figure sim-
ply represents the percentage of people with- Answer E is incorrect. Misoprostol keeps the
out the cancer, 900/1000. ductus arteriosus open, not closed. NSAIDs
Test Block 2  •  Answers 541

are commonly used for closure of patent duc- strate a monoclonal M spike, as opposed to a
tus arteriosus. normal polyclonal distribution.
Answer A is incorrect. The classic findings
3. The correct answer is A. The patient likely
in osteoporosis are a significant loss of bone
has an abnormal atrial re-entrant tract that by-
mass and vertebral fractures. Unlike mul-
passes the atrioventricular (AV) node and goes
tiple myeloma, osteoporosis does not produce
straight from the atrium to the ventricle. That
“punched-out” lesions on radiographs. In addi-
is why the AV nodal block medication would
tion, osteoporosis does not cause mental status
not work and instead provided an unopposed
changes, increased total protein levels, or poly-
pathway for the abnormal tract. The syndrome
uria. Diagnostic testing may include measure-
resulting from the presence of an accessory
ment of bone mineral density by dual-energy
tract is Wolff-Parkinson-White.
x-ray absorptiometry. In this technique the en-
Answer B is incorrect. An abnormal tract ergy absorbed by dual x-ray beams is used esti-
through the AV node likely would be ablated if mate the surface area and density of mineral-
the patient were given an antiarrhythmic that ized tissue.
blocked the AV node.
Answer B is incorrect. Parathyroid adenoma
Answer C is incorrect. Tetralogy of Fallot usu- is the most common cause of primary hyper-
ally does not lead to arrhythmias. parathyroidism, a disorder characterized by hy-
Answer D is incorrect. Hypertrophic car- percalcemia, kidney stones, and bone lesions,
diomyopathy indeed may lead to ventricu- all of which result from increased circulating
lar fibrillation and sudden death, but a more parathyroid hormone (PTH) levels. However,
common presentation would be a previously the serum protein level typically is normal.
healthy adolescent who suddenly collapses Treatment includes surgical removal of the
under extreme exertion (eg, during a sports adenoma and biopsy of the remaining parathy-
game). roid glands to rule out parathyroid hyperplasia.

Answer E is incorrect. Ventricular tachycardia Answer C is incorrect. Diabetes mellitus

Full-Length Exams
is not a congenital heart defect. Rather, it is (DM) can present with several of the symp-
usually the result of other heart disorders, such toms seen in multiple myeloma, including
as acute myocardial infarction (MI), scar from mental status changes, polyuria, and noctu-
an old infarct, long-QT syndrome, or electro- ria. However, the lytic skull lesions, hypercal-
lyte abnormalities. cemia, and increased total protein level make
multiple myeloma a far more likely diagnosis.
4. The correct answer is D. This patient’s pre- Answer E is incorrect. Ankylosing spondylitis
sentation is consistent with multiple myeloma. is a degenerative inflammation of the spine
The primary pathophysiology is a neoplastic and sacroiliac joints, resulting in a stiff spine.
proliferation of mature plasma cells produc- Radiographs typically show forward curvature
ing abnormal immunoglobulins, most com- of the spine and fusion of the lumbar verte-
monly IgG. Characteristic features of multiple brae, often referred to as a “bamboo spine.”
myeloma include destructive “punched-out” Though it is highly associated with HLA-B27
bone lesions (as demonstrated on the radio- haploptype, this finding is not diagnostic. Pa-
graph), hypercalcemia (causing the lethargy, tients with HLA-B27 haplotypes are more
weakness, and confusion in this case), and re- likely to have aortic regurgitation secondary to
nal insufficiency (often causing polyuria and aortitis, or may develop blindness secondary to
Test Block 2

nocturia). Excess immunoglobulins are re- uveitis.


sponsible for the increased total protein levels.
Pathologic fractures also are seen commonly. Answer F is incorrect. Features of prostate
Serum protein electrophoresis would demon- cancer include a palpable mass on digital rec-
tal examination, dysuria, nocturia, and, if me-
542 Section III: Full-Length Examinations  •  Answers

tastasis has occurred, back pain due to spinal glycogen in the liver, hepatomegaly, and in-
lesions. Many of these symptoms overlap with creased blood lactate. These findings are in-
those seen in multiple myeloma; however, the consistent with the symptoms observed in this
increased total serum protein level and hyper- patient.
calcemia seen in this patient are more typical
Answer E is incorrect. Lysosomal a-1,4-glu-
of multiple myeloma. Screening for prostate
cosidase is the defective enzyme in Pompe dis-
cancer includes digital rectal examination
ease, another glycogen storage disorder. The
and testing for prostate-specific antigen (PSA)
findings in Pompe disease typically manifest
yearly after age 50 years. PSA is sensitive but
in early childhood and include respiratory dif-
not specific for prostate cancer. For example,
ficulties (due to diaphragmatic weakness), car-
PSA between 4-10 ng/mL may be indicative of
diomegaly, and progressive loss of muscle tone
either early cancer or benign prostatic hyper-
leading to early death.
plasia (BPH). In this patient, PSA was in the
normal range, which means <4 ng/mL. 6. The correct answer is B. Osteoporosis is a
metabolic bone disease characterized by de-
5. The correct answer is D. This patient suffers
creased bone mass. It can be caused by im-
from McArdle disease, a glycogen storage disor-
paired synthesis or increased resorption of
der in which glycogen phosphorylase is deficient
bone matrix protein. It is clinically associated
in muscle. The enzyme is responsible for lib-
with a postmenopausal state, physical inactiv-
erating individual units of glucose-1-phosphate
ity, hypercortisolism, hyperthyroidism, and cal-
from branches of a glycogen molecule. Onset
cium deficiency. This patient has several risks
of the disease typically occurs in adolescence or
factors for osteoporosis: She is postmenopausal
early adulthood and is characterized by muscle
and has a family history of osteoporosis, as evi-
cramping, rapid fatigue, and poor endurance
denced by her mother, who fell at age 58 and
during exertion. Severe myoglobinuria is also
needed hip replacement surgery. The pulmo-
observed in some patients.
nary embolism was most likely provoked by
Answer A is incorrect. a-1,6-Glucosidase is the surgery. Her history of severe rheumatoid
Full-Length Exams

the enzyme responsible for the debranching arthritis predisposes her to hypercortisolism
of glycogen. It is implicated in Cori disease, as a result of treatment with corticosteroids.
which is a mild form of Von Gierke disease Her lower back pain suggests the possibility
with normal blood lactate levels. It is not im- of a vertebral compression fracture. Lab tests
plicated in McArdle disease as it wouldn’t in osteoporosis reveal normal serum calcium,
cause the muscle cramping. normal serum phosphorus, and normal or
decreased alkaline phosphatase levels. Treat-
Answer B is incorrect. Homocystinuria is an
ments for osteoporosis include weight-bearing
inborn error of metabolism caused by a de-
exercise, calcium supplementation, hormone
fect in cystathionine synthase, the enzyme that
replacement therapy, and bisphosphonates.
converts homocysteine to cystathionine. In ad-
dition to Marfan-like features, these patients Answer A is incorrect. These findings describe
are at increased risk for a variety of cardiovas- hypoparathyroidism, which may occur congen-
cular derangements due to increased athero- itally (DiGeorge syndrome) or after thyroidec-
sclerosis, including premature vascular disease tomy. Symptoms are related to hypocalcemia:
and early death. tetany, depression, dementia, and seizures.
ECG will show a prolonged QT interval,
Answer C is incorrect. Glucose-6-phosphatase
which predisposes the patient to developing
Test Block 2

is the enzyme responsible for converting glu-


torsades de pointes. This patient does not have
cose-6-phosphate to glucose. It is a component
symptoms of hypocalcemia.
of gluconeogenesis. A deficiency of this en-
zyme causes Von Gierke disease, characterized Answer C is incorrect. These lab values are
by a severe fasting hypoglycemia, increased seen in Paget disease, also known as osteitis
Test Block 2  •  Answers 543

deformans. This is a disease of abnormal bone ascending paralysis. It is thought to follow a va-
architecture due to haphazard osteoblastic and riety of infectious diseases, such as cytomega-
osteoclastic activity. Symptoms include bony lovirus, Epstein-Barr virus, HIV, and gastroen-
pain, increased risk for bony fractures, hearing teritis caused by Campylobacter jejuni.
loss, and headaches. Patients are at increased
risk for osteosarcoma as well as high-output 8. The correct answer is E. PGI2 inhibits plate-
cardiac failure from multiple arteriovenous let aggregation and therefore is an antithrom-
shunts. This patient does not have symptoms botic agent. On the other hand, TxA2 increases
of Paget disease. platelet aggregation and is a prothrombotic
agent. COX-2 inhibitors selectively decrease
Answer D is incorrect. These findings de-
PGI2, leaving the action of TxA2 unopposed.
scribe hyperparathyroidism. PTH stimulates
This could well result in increased cerebro-
bone resorption, calcium reabsorption, and
vascular and cardiovascular events due to the
phosphorus excretion. Symptoms are related
tonic, unopposed prothrombotic action of
to hypercalcemia: osteopenia, kidney stones,
TxA2. This patient is most likely suffering from
polyuria, constipation, abdominal pain, de-
an acute thrombus in his coronary artery caus-
pression, and psychosis. ECG will show a
ing a MI.
shortened QT interval. This patient does not
have symptoms of hypercalcemia. Answer A is incorrect. COX-2 inhibitors do
not increase PGI2. They are thought to spare
7. The correct answer is D. The causative agent the gastric mucosa because they selectively
in this scenario, based on the Gram stain, is block the synthesis of other prostaglandins, not
Neisseria meningitidis. Waterhouse-Friderichsen because they increase the production of PGI2.
syndrome is a possible complication of menin-
Answer B is incorrect. COX-2 inhibitors do
gococcemia. In this disorder, bilateral hemor-
not increase TxA2. COX-2 inhibitors in gen-
rhage into the adrenal gland causes adrenal
eral do not increase PGI2 and TxA2, which are
insufficiency. This results in hypotension,
downstream products.
tachycardia, a rapidly enlarging petechial skin

Full-Length Exams
lesion, disseminated intravascular coagulation, Answer C is incorrect. COX-2 inhibitors and
and coma. aspirin do not have the same actions.
Answer A is incorrect. Hemolytic-uremic syn- Answer D is incorrect. COX-2 inhibitors do
drome (HUS) is characterized by acute renal decrease PGI2, but aspirin does not increase
failure and thrombocytopenia with hemolytic PGI2 and TxA2. Aspirin is a nonselective COX
anemia. HUS can be a complication of infec- inhibitor that decreases both PGI2 and TxA2.
tion caused by E coli O157:H7 and not Neis-
seria meningitidis. 9. The correct answer is A. This patient’s history
and presentation are consistent with anemia
Answer B is incorrect. Rheumatic fever is of chronic disease (ACD) in the setting of sys-
characterized by fever, migratory polyarthritis, temic lupus erythematosus (SLE). Hepcidin,
and carditis. It may follow group A streptococ- an acute-phase reactant that is elevated in
cal pharyngitis. SLE, impairs the transfer of iron from macro-
Answer C is incorrect. Fever, a new murmur, phages to erythroid precursors. Hence ACD
Janeway lesions, and nail-bed hemorrhages are presents with a low serum iron level, low to-
all signs of bacterial endocarditis. Acute endo- tal iron-binding capacity, an elevated ferritin
carditis is caused by Staphylococcus aureus and level, and normochromic/normocytic RBCs.
Test Block 2

subacute infection can be caused by Strepto- ACD resolves if the underlying condition is
coccus viridans. corrected, but in the absence of a successful
primary treatment, erythropoietin can be effec-
Answer E is incorrect. Guillain-Barré syn-
tive in treating the anemia.
drome is characterized by rapidly progressing
544 Section III: Full-Length Examinations  •  Answers

Answer B is incorrect. Iron therapy is not in- ria, a palpable mass, and flank pain. It is associ-
dicated in the treatment of ACD. In ACD the ated with von Hippel-Lindau syndrome.
problem is iron utilization, not iron deficiency.
Answer E is incorrect. A serous cystadenoma
Answer C is incorrect. Folate supplementa- of the ovary is a benign tumor of columnar epi-
tion is appropriate therapy in macrocytic ane- thelium and is not associated with hereditary
mia caused by folate deficiency. retinoblastoma. It occurs in 20% of ovarian tu-
mors and is frequently bilateral and lined with
Answer D is incorrect. Parenteral vitamin
fallopian-tube-like epithelium. Risk factors in-
B12 is appropriate for treating pernicious ane-
clude the BRCA1 gene and hereditary nonpol-
mia. In pernicious anemia, because of a lack
yposis colorectal cancer.
of intrinsic factor, vitamin B12 is not absorbed.
Macrocytic RBCs are seen on blood smear. Answer F is incorrect. Squamous cell car-
cinoma of the lung is a tumor of squamous
Answer E is incorrect. Phlebotomy is appro-
surface epithelium and is not associated with
priate for treating significant iron overload, as
hereditary retinoblastoma. Its characteristics
seen in patients with chronic transfusion ther-
include a hilar mass arising from the bron-
apy and hemochromatosis.
chus, cavitation, and PTH-related protein. His-
10. The correct answer is B. Hereditary retino- tologically, keratin “pearls” and intercellular
blastoma survivors are at increased risk for soft bridges are seen. A major risk factor is smok-
tissue sarcomas, osteosarcomas, melanomas, ing.
and several types of brain cancer. Osteosar- Answer G is incorrect. Transitional cell carci-
coma, the most common malignant primary noma is a tumor of transitional surface epithe-
bone tumor, most frequently originates in the lium in the bladder and is not associated with
distal femur, proximal tibia, or proximal hu- hereditary retinoblastoma. Painless hematuria
merus. Other risk factors for osteosarcoma in- is suggestive of this diagnosis. It is associated
clude Paget disease of bone, bone infarcts, and with problems of your “Pee SAC”: Phenac-
radiation. Codman triangle or sunburst pattern etin, Smoking, Aniline dyes, and Cyclophos-
Full-Length Exams

is seen on x-ray. phamide.


Answer A is incorrect. Esophageal adenocar-
11. The correct answer is C. Afferent pain fibers
cinoma is a tumor of glandular epithelium
of the heart enter the posterior horn of the
and is not associated with hereditary retino-
spinal cord at the same level as the brachial
blastoma. Risk factors for esophageal cancer
plexus, leading to pain perceived as being lo-
include ABCDEFG: Alcohol/Achalasia,
cated in the neck and shoulder region.
Barrett Esophagus, Cigarettes, Diverticuli,
Esophageal web (Plummer-Vinson)/ Esopha- Answer A is incorrect. Lymphatic drainage
gitis, and Familial and Gastroesophageal re- does occur in the left upper quadrant, but it
flux disease. plays no role in the model of referred myocar-
dial pain.
Answer C is incorrect. Medullary carcinoma
of the thyroid is a tumor of thyroid solid glan- Answer B is incorrect. Sensory neurons have
dular epithelium and is not associated with their origin in the dorsal root ganglion and
hereditary retinoblastoma. It forms from para- send their axons to the posterior horn of the
follicular C cells, and produces calcitonin and spinal cord instead of the anterior horn, where
sheets of cells in an amyloid stroma. It is asso- efferent neurons arise.
Test Block 2

ciated with MEN 2A and 2B.


Answer D is incorrect. The heart and the neck
Answer D is incorrect. Renal cell carcinoma and shoulder region do not share similar para-
(RCC) is a tumor of renal solid glandular epi- sympathetic innervation patterns.
thelium and is not associated with hereditary
retinoblastoma. RCC manifests with hematu-
Test Block 2  •  Answers 545

Answer E is incorrect. The heart and the neck infections. It is often used in conjunction with
and shoulder region do not share similar sym- amphotericin B to treat cryptococcal menin-
pathetic innervation patterns. gitis, but is not first-line treatment as a single
agent. Flucytosine inhibits DNA synthesis be-
12. The correct answer is A. Patients with AIDS cause it is converted to fluorouracil in vivo.
are susceptible to a variety of infections that
are unusual in the immunocompetent popu- 13. The correct answer is C. Given the patient’s
lation. Among diseases that cause fever and body habitus and glucose level, a diagnosis
headache in these patients are Cryptococcus, of diabetes is extremely likely. The patient is
toxoplasmosis, and central nervous system presenting after recent surgery with symptoms
(CDS) lymphoma. An encapsulated yeast that consistent with lactic acidosis. His low bicar-
stains with India ink is a pathognomonic de- bonate is concerning for an acidotic state. Cal-
scription of Cryptococcus neoformans, which culating the anion gap (Na - Cl - HCO3- =
is a yeast found in pigeon droppings. Infec- anion gap) gives a result of 18. The physiologic
tion occurs when patients inhale fungus par- range of the anion gap in healthy adults is
ticles, which can lead to pneumonia. Initial 8-12 mEq/L. High-anion-gap metabolic acido-
treatment of C neoformans is intravenous (IV) sis can be caused by multiple conditions and
amphotericin B, followed by fluconazole once can be remembered by the mnemonic MUD-
the patient’s condition is stable. Amphotericin PILES: Methanol, Uremia, Diabetic ketoaci-
toxicity can cause fever and chills, hypoten- dosis, Paraldehyde, Infection or isoniazid, Lac-
sion, nephrotoxicity, and arrhythmias. The ar- tic acidosis, Ethylene glycol, and Salicylates.
rhythmias are due to QT prolongation, which Metformin is a biguanide that suppresses he-
is exacerbated by changes in potassium and patic glucose production, decreases intestinal
magnesium levels. absorption of glucose, and improves insulin
sensitivity. It is known to increase the risk of
Answer B is incorrect. Bone marrow suppres-
lactic acidosis, particularly in those with renal
sion is seen with a number of drugs, including
impairment (the patient in this case has an el-
flucytosine.
evated creatinine of 1.6 mg/dL), as well as in

Full-Length Exams
Answer C is incorrect. Flushing can be the postoperative period if it is stopped for an
caused by caspofungin, an antifungal medi- extended period.
cation used to treat aspergillosis infection.
Answer A is incorrect. Glyburide is a sulfo-
Caspofungin inhibits synthesis of the b(1,3)-
nylurea that increases pancreatic secretion of
D-glucan component of the fungal cell wall.
insulin by depolarizing the b-cell membranes.
Other adverse effects include gastrointestinal
Its major adverse effect is hypoglycemia, which
(GI) upset.
could present as loss of consciousness, seizure,
Answer D is incorrect. Gynecomastia is an or altered mental status. However, the patient’s
adverse effect of fluconazole treatment. The lab values show that he is actually hyperglyce-
-azole antifungals inhibit ergosterol synthesis. mic, making glyburide less likely.
They are used to treat systemic mycoses but
Answer B is incorrect. Insulin as a pharma-
are less effective than amphotericin B and are
cologic preparation is not administered orally,
adjunct therapies in acute cases. In patients
but rather is subcutaneously injected. It binds
with AIDS, the initial amphotericin course
the insulin receptor to increase hepatic glyco-
should be followed by maintenance fluco­
gen production from glucose and promote pro-
nazole therapy daily for life. Other adverse ef-
tein synthesis in muscle. A major adverse effect
Test Block 2

fects include liver dysfunction and fever.


of insulin treatment is hypoglycemia, weight
Answer E is incorrect. Nausea and vomiting gain, and injection-site lipodystrophy, but it is
(along with diarrhea and bone marrow sup- not associated with lactic acidosis.
pression) are toxicities associated with flucy-
tosine, which is used to treat systemic fungal
546 Section III: Full-Length Examinations  •  Answers

Answer D is incorrect. Orlistat alters fat me- lation, thus preventing the formation of the
tabolism by decreasing pancreatic lipase activ- corpus luteum.
ity. It is often used for long-term weight man-
agement, and causes GI adverse effects, such 15. The correct answer is D. This child has nor-
as loose and fatty stools (steatorrhea), dimin- mal facies and is suffering from an isolated
ished absorption of vitamins A, D, K, and E, cleft palate. This orofacial defect makes it dif-
and headache. It has not been associated with ficult to create the suction needed for proper
lactic acidosis. feeding. It results in choking and coughing, as
well as aspiration and poor weight gain in af-
Answer E is incorrect. Rosiglitazone is a thia-
fected children. It is the result of the failure of
zolidinedione that increases tissue sensitivity/
the fusion of the lateral palatine processes, the
target cell response to insulin. A major adverse
medial palatine processes, and/or the nasal sep-
effect of this medication is weight gain and
tum. Surgical correction is usually attempted
edema, and it has also recently been associated
between 9 and 12 months of age.
with increased cardiovascular risks. However, it
is not known to cause lactic acidosis. Answer A is incorrect. The first and second
branchial arches do not fuse.
14. The correct answer is C. Lactation is main-
Answer B is incorrect. Failure of the maxillary
tained by prolactin secretion from the ante-
processes to fuse results in cleft lip. Cleft lip
rior pituitary. Prolactin prevents ovulation
and cleft palate often occur together.
by inhibiting the secretion of gonadotropin-
releasing hormone (GnRH) from the hypo- Answer C is incorrect. Failure of the medial
thalamus. This decrease in GnRH leads to nasal processes to fuse results in cleft lip.
decreased secretion of luteinizing hormone Answer E is incorrect. The second and third
(LH) and follicle-stimulating hormone (FSH) branchial arches do not fuse.
from the anterior pituitary gland, which pre-
vent ovulation. This woman would likely have 16. The correct answer is B. Desmopressin is
a better chance of becoming pregnant if she 1-deamino-8-D-arginine vasopressin (dDAVP),
Full-Length Exams

were to stop breastfeeding her son. an analog of ADH. This woman has central di-
Answer A is incorrect. Prolactin does not di- abetes insipidus caused by trauma to the poste-
rectly inhibit the secretion of estrogen from rior pituitary. This inhibits secretion of ADH.
the ovaries. There is a decrease in estrogen se- Of the options given, repleting her ADH is the
cretion due to decreased FSH levels, which is most appropriate.
due to prolactin’s direct inhibition of GnRH Answer A is incorrect. Demeclocycline is
secretion from the hypothalamus. used to treat the syndrome of inappropriate
Answer B is incorrect. Prolactin does not di- ADH secretion. This compound acts to inhibit
rectly inhibit the secretion of FSH from the ADH action and would exacerbate her condi-
anterior pituitary gland. There is a secondary tion.
decrease in FSH secretion due to a decrease in Answer C is incorrect. Furosemide is a loop
GnRH release from the hypothalamus. diuretic and is likely to exacerbate her condi-
Answer D is incorrect. Prolactin does not di- tion.
rectly inhibit the secretion of LH from the Answer D is incorrect. Insulin is an inappro-
anterior pituitary gland. There is a secondary priate treatment. Central diabetes insipidus
decrease in LH secretion due to a decrease in shares only the symptoms of polydipsia and
Test Block 2

GnRH release from the hypothalamus. polyuria with DM. The treatments and causes
Answer E is incorrect. Prolactin does not af- are completely different.
fect the secretion of progesterone from the Answer E is incorrect. Mannitol is an osmotic
corpus luteum. It does, however, prevent ovu- diuretic that would exacerbate her condition.
Test Block 2  •  Answers 547

17. The correct answer is A. Beta waves are seen locus and the IgH locus. On histopathology,
on EEG when the patient is awake and ac- you would expect to see a “starry sky” pattern
tively concentrating. Alpha waves are seen on with benign macrophages containing clear cy-
EEG when the patient is awake and relaxed toplasm distributed throughout a dense field of
with eyes closed. intermediate-sized neoplastic lymphoid cells.
Answer B is incorrect. Stage 1 sleep is the Answer C is incorrect. Erb B2 is an oncogene
transition from the awake state to sleep and present in some breast cancers. The t(11;14)
is characterized by theta waves in the range translocation is involved in mantle cell lym-
of 4-7 cycles per second on EEG. Stage 1 ac- phomas. Tumor cells of mantle cell lymphoma
counts for 5% of total sleep. are similar to the normal mantle zone B lym-
phocytes, and neoplastic cells surround germi-
Answer C is incorrect. Stage 2 sleep, known as
nal centers, which may be small and atrophic.
true physiologic sleep, is defined by sleep spin-
Erb B2 is not related to the t(11;14) transloca-
dles and K-complexes on EEG. Stage 2 sleep
tion.
accounts for 45% of total sleep.
Answer D is incorrect. p53 is not an onco-
Answer D is incorrect. Stage 3, or deep sleep,
gene, but rather a tumor suppressor gene that
is characterized by delta waves on EEG. Delta
is downregulated in many cancers. It is not
waves are slow waves that are <3 Hz. Stage 3
paired with t(8:14).
accounts for 12% of total sleep.
Answer E is incorrect. ras is the oncogene im-
Answer E is incorrect. Stage 4 sleep is identi-
plicated in many colon cancers. The t(11;14)
cal to stage 3 sleep, and they are now charac-
translocation juxtaposes the cyclin D1 and IgH
terized together as stage N3. Stage 4 sleep ac-
loci, and is involved in mantle cell lymphoma.
counts for 13% of total sleep.
Tumor cells of mantle cell lymphoma are simi-
18. The correct answer is B. The image shows lar to the normal mantle zone B lymphocytes
nodular collections of lymphoma cells in a and neoplastic cells surround germinal cen-
lymph node consistent with follicular lym- ters, which may be small and atrophic. ras is

Full-Length Exams
phoma (also known as small-cleaved-cell lym- not related to the t(11;14) translocation.
phoma). Follicular lymphoma is one of the
19. The correct answer is A. This patient has fea-
non-Hodgkin lymphomas. This type of lym-
tures of Turner syndrome (XO), a chromo-
phoma is characterized by numerous irregu-
somal disorder occurring in 1:3000 births that
larly sized follicles; the neoplastic cells appear
is associated with short stature, primary amen-
similar to normal germinal center B lympho-
orrhea, and webbing of the neck. Laboratory
cytes. The neoplastic cells express bcl-2; nor-
values are consistent with the premature ovar-
mal cells do not. Bcl-2, an antiapoptotic gene,
ian failure of Turner syndrome. Coarctation
is overexpressed due to a translocation be-
of the aorta is a narrowing of the aorta found
tween the IgH locus on chromosome 14 and
in 3-10% of people with Turner syndrome.
the bcl-2 locus on chromosome 18. When you
The finding of upper extremity blood pressure
think of follicular lymphoma, you should think
greater than lower extremity blood pressure
B lymphocytes, bcl-2, and t(14;18). It typically
would suggest that this patient has coarcta-
occurs in middle-aged adults, has an indolent
tion of the aorta. The narrowing of the aorta
course, and is incurable. Patients with small-
is usually distal to the three great vessels com-
cleaved-cell lymphoma often present with
ing off the aortic arch (brachiocephalic trunk,
complaints of waxing and waning painless
Test Block 2

left common carotid, and left subclavian),


lymphadenopathy.
increasing the pressure above this narrowing
Answer A is incorrect. c-myc and t(8;14) are and obstructing flow to the vessels distal to the
involved in Burkitt lymphoma. The t(8;14) narrowing. Coarctation in Turner syndrome is
translocation juxtaposes the c-myc oncogene usually the infantile (preductal) type, with the
548 Section III: Full-Length Examinations  •  Answers

narrowing located between the subclavian ar- uveitis, conductive hearing loss, and muscle/
tery and the ductus arteriosus. joint pain. An eosinophilic gastroenteritis may
precede the onset of the other symptoms.
Answer B is incorrect. Carotid bruits may be
heard when carotid arteries are narrowed and Answer A is incorrect. Autoantibodies to ace-
obstructed due to atherosclerosis. This condi- tylcholine receptors are not particularly associ-
tion is not associated with Turner syndrome. ated with Churg-Strauss syndrome. Myasthe-
nia gravis is characterized by an autoimmune
Answer C is incorrect. Aortic coarctation may
attack on the acetylcholine receptors of the
produce a systolic murmur, but a late systolic
neuromuscular junction between motor neu-
murmur with a mid-systolic click is character-
rons and skeletal muscle fibers.
istic of mitral valve prolapse. This condition,
while associated with other genetic disorders Answer C is incorrect. Autoantibodies to oli-
such as Marfan syndrome, is not associated godendrocytes are not particularly associated
with Turner syndrome. with Churg-Strauss syndrome. There is evi-
dence suggesting that multiple sclerosis may
Answer D is incorrect. Mediastinal widening
be partially caused by autoimmune antibody
on x-ray of the chest can be evidence of an
attack on CNS myelin-secreting oligodendro-
aortic dissection, caused by a longitudinal tear
cytes.
within the aortic wall. This life-threatening
condition is associated with genetically ac- Answer D is incorrect. Autoantibodies to
quired connective tissue disorders such as RBCs, which may be found in certain cases
Marfan syndrome, but is not associated with of immune hemolytic anemia, are not particu-
Turner syndrome. larly associated with Churg-Strauss syndrome.
Answer E is incorrect. Slowly rising and weak Answer E is incorrect. Autoantibodies to
carotid pulses are associated with aortic steno- thyroid-stimulating hormone (TSH) receptors
sis. A subset of patients with Turner syndrome are not particularly associated with Churg-
can have aortic valve disease, including a bi- Strauss syndrome. Graves disease is a disorder
cuspid valve that can calcify over time. How- resulting from IgG-type autoantibodies to the
Full-Length Exams

ever, in a young patient with Turner syndrome TSH receptor.


aortic coarctation is more likely.
21. The correct answer is B. The whorled pattern
20. The correct answer is B. This patient has of cell growth surrounding lamellated areas of
Churg-Strauss syndrome (also known as allergic dystrophic calcification represents psammoma
granulomatosis and angiitis), which is one of a bodies. This pathologic finding is most com-
trio of diseases (Wegener granulomatosis and monly seen in meningiomas, papillary thyroid
microscopic polyangiitis being the others) that tumors, and certain ovarian tumors. Given this
are commonly referred to as the ANCA (anti- patient’s complaints of a seizure, headache,
neutrophil cytoplasmic antibody)-associated vas- and visual-field deficit, meningioma is the
culitides (ie, diseases causing inflammation of most likely answer.
blood or lymphatic vessels). Fifty to seventy per-
Answer A is incorrect. Hemangiomas are
cent of patients with Churg-Strauss syndrome
more common in children and are often lo-
have elevated levels of ANCA, usually the peri-
cated in the cerebellum. The cells tend to be
nuclear pattern of staining type (P-ANCA).
“foamy,” and the tumors display a high degree
Pulmonary vasculature involvement is com-
of vascularity. Psammoma bodies would not be
mon and patients often have preexisting
Test Block 2

expected in tumors of this type.


asthma and allergic rhinitis. They also present
with markedly elevated eosinophil counts and Answer C is incorrect. Pathology specimens
mononeuritis multiplex (simultaneous deficits of oligodendrogliomas reveal “fried egg” cells,
of two or several peripheral nerves in different not psammoma bodies. These fried egg cells
areas of the body). Other symptoms include have round nuclei amidst a halo of clear cyto-
Test Block 2  •  Answers 549

plasm. Oligodendrogliomas are benign brain cholecalciferol excess does not result in osteo-
tumors found in the cerebral hemispheres. porosis.
They tend to have areas of calcification that
Answer B is incorrect. Patients with renal fail-
can be detected radiographically.
ure have a chronic metabolic acidosis due to
Answer D is incorrect. While psammoma decreased renal handling of acid anions. Meta-
bodies can be found in papillary thyroid car- bolic alkalosis does not result in osteoporosis.
cinoma, this diagnosis does not fit with the
Answer C is incorrect. Patients with renal fail-
patient’s history and clinical presentation.
ure have hypocalcemia as a result of decreased
Papillary thyroid carcinoma is the most com-
intestinal absorption of calcium and increased
mon cancer of the thyroid. It is associated with
calcium phosphate deposition in tissues. Hy-
psammoma bodies and “orphan Annie” nuclei.
percalcemia is not associated with renal failure
Compared to other cancers of the thyroid, this
and does not result in osteoporosis.
variant has the best prognosis.
Answer D is incorrect. Patients with renal fail-
Answer E is incorrect. Psammoma bodies are
ure have hyperphosphatemia due to decreased
not associated with pituitary adenomas. Fur-
renal excretion of phosphorous. Hypophospha-
thermore, the visual field deficit that would
temia does not result in osteoporosis.
result from a pituitary adenoma is bilateral
hemianopsia, due to pressure on the optic chi- 23. The correct answer is D. This patient most
asm. This patient only had loss of her temporal likely suffers from RCC. RCC is character-
visual field on the right. Additionally, some evi- ized by the triad of flank pain, hematuria, and
dence of hypopituitarism or other endocrine abdominal mass, although <10% of patients
abnormality would be likely by the time the have all three symptoms. It occurs most com-
adenoma was large enough to cause seizures. monly in men and is associated with risk fac-
tors such as obesity, hypertension, smoking,
22. The correct answer is E. Patients with signifi-
and environmental toxin exposure (this patient
cant renal disease are at particularly high risk
was a worker in a steel plant). Importantly, the
for developing skeletal complications, gen-

Full-Length Exams
major distinguishing feature of RCC is its as-
erally known as renal osteodystrophy. Renal
sociation with paraneoplastic syndromes due
failure produces numerous downstream con-
to the ectopic production of hormones such
sequences that affect bone health, including
as parathyroid hormone-related protein (hyper-
increased phosphate retention (resulting in
calcemia, hypophosphatemia), erythropoietin
calcium phosphate deposition leading to hypo-
(polycythemia), and ACTH. RCC is also asso-
calcemia and secondary hyperparathyroidism),
ciated with hereditary conditions such as Von
decreased renal conversion of 25-hydroxchole-
Hippel-Lindau (VHL) syndrome. In VHL-re-
calciferol to 1,25-dihydroxycholecalciferol (re-
lated RCC, patients usually present with bilat-
sulting in decreased intestinal calcium absorp-
eral tumors. RCC has a relatively poor progno-
tion and decreased suppression of parathyroid
sis because most tumors are asymptomatic until
hormone production), and chronic metabolic
they have undergone metastasis.
acidosis (resulting in increased bone reabsorp-
tion). The resulting secondary hyperparathy- Answer A is incorrect. Adult polycystic kidney
roidism increases osteoclast activity and the re- disease (APKD) is usually an autosomal domi-
absorption of bone. nant genetic disease that causes rapid cystic
enlargement of the kidneys, leading to renal
Answer A is incorrect. Patients with renal fail-
failure. It usually manifests with hypertension,
Test Block 2

ure have decreased levels of 1,25-dihydroxy-


pain, and hematuria. Patients with APKD do
cholecalciferol because of decreased renal
have occasional palpable masses, but renal
conversion of 25-hydroxycholecalciferol to
masses are usually bilateral.
1,25-dihydroxycholecalciferol. 1,25-Dihydroxy-
550 Section III: Full-Length Examinations  •  Answers

Answer B is incorrect. Angiomyolipomas are antibiotics such as doxorubicin and daunoru-


relatively rare, benign vascular tumors that bicin. It is thought that these medications can
occur in the kidney. Renal angiomyolipomas, cause toxic levels of free radicals to build up in
however, have a more insidious course of pro- the myocardium, leading to muscle damage,
gression than RCC, are more common in decreased left ventricular ejection fraction,
women, and do not cause paraneoplastic syn- and symptoms of congestive heart failure.
dromes. Nevertheless, the triad of flank pain,
Answer B is incorrect. Hemorrhagic cystitis is
hematuria, and palpable abdominal mass can
an adverse reaction seen with the use of cyclo-
also occur with angiomyolipomas.
phosphamide, an alkylating agent used to treat
Answer C is incorrect. Pheochromocytoma solid tumors and lymphoma. It is also used as
is a rare tumor derived of catecholamine- an immunosuppressant to treat severe rheuma-
producing chromaffin cells of the adrenal tologic disorders.
glands. Major clinical findings usually result
Answer C is incorrect. Hyperglycemia is a
from the ectopic production of catechol-
common adverse effect of prednisone and
amines, including episodic headache, palpita-
other steroid drugs. Vincristine, however, does
tions, and sweating with severe hypertension.
not cause hyperglycemia. Prednisone is part of
It can also be associated with café-au-lait spots
the MOPP regimen. Its mechanism of action
and neurofibromas. These clinical features
is unclear, but is thought to relate to triggering
predominate in the presentation of the pheo-
apoptosis.
chromocytoma-affected patient, rather than
the triad of flank pain, hematuria, and pal- Answer D is incorrect. Pulmonary fibrosis is
pable mass and the electrolyte abnormalities seen with several chemotherapeutic agents
seen in this patient. including busulfan, which is used to treat
chronic myelogenous leukemia, and bleomy-
Answer E is incorrect. Wilms tumor is a com-
cin, which is used to treat testicular cancer and
mon pediatric malignancy and is not found in
lymphoma.
adult patients. It can be sporadic or familial.
In almost all cases, Wilms tumor is caused by
Full-Length Exams

25. The correct answer is D. This girl has impe-


mutations in the WT1 gene. In familial Wilms tigo, most likely caused by Streptococcus pyo-
tumor the disease can be associated with nu- genes, a gram-positive group A b-hemolytic
merous congenital abnormalities, including organism that is bacitracin sensitive. This in-
single, horseshoe, or ectopic kidney; hypospa- fection is characterized by an eruption of ves-
dias; cryptorchidism; and aniridia. icles on the face. These vesicles later turn into
pustules with a characteristic honey-colored
24. The correct answer is E. The patient has a
crust. A distinctly bullous form of impetigo is
Wilms tumor, a childhood nephroblastoma
caused by Staphylococcus aureus infection.
that can be treated with the MOPP regimen,
which includes the plant alkaloid vincristine Answer A is incorrect. Endotoxin is a charac-
(Oncovin). Vincristine halts cell division by in- teristic of gram-negative bacteria and Listeria,
hibiting microtubule polymerization, thus pre- but not S pyogenes, which is the most likely
venting formation of the mitotic spindle and causative organism in this case.
causing metaphase arrest. It has the adverse ef- Answer B is incorrect. Sabouraud agar is re-
fect of neurotoxicity as manifested by areflexia, quired to culture fungi, not S pyogenes.
peripheral neuritis, muscle weakness, and par-
alytic ileus. The MOPP regimen refers to the Answer C is incorrect. Streptococcus agalac-
Test Block 2

chemotherapeutic agents Mechlorethamine, tiae is a group B b-hemolytic organism that is


Oncovin, Procarbazine, and Prednisone. bacitracin resistant. However, it is not a com-
mon cause of impetigo.
Answer A is incorrect. Cardiotoxicity is most
often associated with the use of anthracycline
Test Block 2  •  Answers 551

Answer E is incorrect. Mycobacterium, Bru- Answer D is incorrect. Pheochromocytomas,


cella, Francisella, Listeria, Yersinia, Legionella, “the 10% tumor,” occur in the medulla of
and Salmonella are facultative intracellular the adrenal glands, and can appear as a con-
organisms, but S pyogenes is not. None of the sequence of a mutation in the VHL gene, the
other bacteria are common causes of impetigo. same gene responsible for VHL disease. How-
ever, pheochromocytomas are rarer sequelae
26. The correct answer is E. Patients with VHL than renal carcinomas, the more specific an-
disease, which is autosomal dominant, have swer to the question.
hemangioblastomas, or cavernous hemangio-
mas of the retina, cerebellum, and medulla. 27. The correct answer is C. This patient has
The cerebellar manifestations are suggested clinical findings and a biopsy consistent with
by the difficulty keeping his balance, and the hereditary hemochromatosis. The image above
retinal hemangioblastoma. Patients with von shows iron deposition in the liver parenchyma.
Hippel-Lindau disease are at increased risk of Hereditary hemochromatosis is associated with
developing renal cell carcinoma. Only 10% of a defect in a gene that encodes the protein
patients develop the so-called “classic triad” HFE. This gene is near the MHC locus on
of flank pain, palpable flank mass, and hema- chromosome 6. As a result of this defect, there
turia, but this is the classic presentation of a is an increase in the efficiency of dietary iron
renal cell carcinoma that many VHL disease absorption. This, coupled with the inability to
patients go on to develop. VHL disease is asso- excrete iron, leads to an increase in hepatic
ciated with the deletion of the VHL gene on iron storage. Increased iron storage causes the
chromosome 3. liver to secrete more ferritin and serum trans-
ferrin becomes increasingly saturated with
Answer A is incorrect. Astrocytomas, CNS tu-
iron. Transferrin saturation is calculated as se-
mors that arise in the cranial vault, are not as-
rum iron/total iron binding capacity (TIBC).
sociated with VHL disease, but with tuberous
TIBC is calculated as 1.4 times the serum
sclerosis. The penetrance of tuberous sclerosis
transferrin level.
is incomplete, and its symptoms are variable.

Full-Length Exams
Among its symptoms is a retinal hamartoma. Answer A is incorrect. Serum ferritin is usu-
The funduscopy could be interpreted as a reti- ally increased in hereditary hemochromatosis
nal hamartoma, but none of the other charac- secondary to increasing iron loads and the in-
teristics of tuberous sclerosis are present. These ability to excrete iron.
would include facial lesions, hypopigmented
Answer B is incorrect. The iron-binding ca-
spots on the skin, and seizures.
pacity remains about the same in hereditary
Answer B is incorrect. The deletion of the hemochromatosis.
tumor-suppressing VHL gene on chromosome
Answer D is incorrect. Plasma iron levels in-
3p, which causes VHL disease, does not affect
crease in hereditary hemochromatosis.
the appearance of colon cancer. This is more
often a sequela of familial adenomatous pol- Answer E is incorrect. Serum ferritin is usu-
yposis. The deletion of the APC gene on chro- ally increased in hereditary hemochromatosis
mosome 5 results in familial adenomatous pol- secondary to increasing iron loads and the in-
yposis, also an autosomal-dominant disease. ability to excrete iron.
Answer C is incorrect. Depression is a co- 28. The correct answer is A. Dietary modification
morbidity of many diseases, and particularly, (drastically limiting saturated and trans fats and
Test Block 2

among the autosomal-dominant disorders, of cholesterol), weight loss, and aerobic exercise
Huntington disease. But, where VHL disease is are the first-line treatment options for any pa-
concerned, renal cell carcinoma is a better and tient with elevated cholesterol levels, and al-
more specific answer than depression. though these lifestyle modifications should be
attempted by this patient, they likely will have
552 Section III: Full-Length Examinations  •  Answers

only minimal effect in a patient with familial 29. The correct answer is D. The therapeutic priv-
hypercholesterolemia. Statin medications are ilege is a rare case of an appropriate exception
3-hydroxy-3-methylglutaryl coenzyme A reduc- to informed consent. The principle is that in-
tase inhibitors. By blocking the rate-limiting forming the patient will be detrimental to his
step in cholesterol synthesis, they can increase or her health. In general a physician should
hepatic synthesis of LDL receptors, thereby consult another physician not involved in the
lowering serum LDL levels. High-dose therapy patient’s care, a psychiatrist, and/or an eth-
with a statin such as atorvastatin, or combined ics committee when invoking this principle.
therapy with one of the fibrate drugs, is first- It does not refer to withholding information
line treatment for patients with familial hyper- a physician believes will make a patient less
cholesterolemia. The toxicity of statin medica- likely to have a procedure performed.
tions includes myositis, which is causing this
Answer A is incorrect. Only a patient can
patient’s symptoms, and elevated creatinine ki-
waive his or her own right to informed con-
nase levels. Rarely, patients can develop rhab-
sent. If a patient is incapacitated such that he
domyolysis with renal failure. Elevated liver
or she no longer retains competency, then a
enzyme levels also can be observed with statin
proxy (such as the son) may be designated, but
treatment, although this usually is reversible.
this is not the case here.
Answer B is incorrect. Bile acid sequestrants
Answer B is incorrect. This patient has met
such as cholestyramine or colesevelam inhibit
the four basic requirements for valid informed
reuptake of bile acids in the intestine, reduc-
consent: mental capacity, disclosure, under-
ing total body LDL. Such medications can
standing, and voluntariness. Consent does not
cause elevated liver enzymes, but are more
have to be given verbally or in writing.
commonly associated with GI bloating and di-
arrhea. Answer C is incorrect. Any competent adult
can make informed consent.
Answer C is incorrect. Ezetimibe inhibits cho-
lesterol uptake by the intestinal brush border Answer E is incorrect. Informed consent is
by blocking specific transporters, with no ef- assumed in an emergency situation in which
Full-Length Exams

fect on the absorption of fat-soluble vitamins or reasonable persons would want treatment. In
minerals. Although adjunctive use with a statin this case the patient is stable and therefore ca-
does augment lipid-lowering effects, combined pable of giving informed consent.
therapy does not reduce cardiovascular events.
30. The correct answer is C. This patient most
Answer D is incorrect. Although niacin (vita- likely has Addison disease, which is charac-
min B3) can lower LDL, its primary effect is an terized by insufficient production of adrenal
increase in HDL levels. Niacin can be added hormones including cortisol, androgen, and al-
to the regimen as a third drug (with a fibrate) dosterone. Its signs and symptoms include hy-
or can be used in patients who are refractory perpigmentation, low blood pressure, muscle
to statin treatment. Its use often is limited by weakness, and salt cravings, among others. The
tolerability (it causes flushing in the majority condition is usually treated with oral hydrocor-
of patients), although new formulations offer tisone. However, sudden discontinuation of hy-
reduced adverse effects. drocortisone can lead to severe back pain.
Answer E is incorrect. LDL apheresis is a Answer A is incorrect. Dehydroepiandros-
method for selectively removing LDL mol- terone is an androgen replacement therapy
ecules from the blood using immunoadsorp-
Test Block 2

sometimes used to treat Addison disease. Dis-


tion columns. This process takes at least three continuation of this therapy usually would not
hours and is done every one-two weeks. It is result in the signs and symptoms found in this
very expensive and not readily available. patient.
Test Block 2  •  Answers 553

Answer B is incorrect. Fluconazole is an an- of prior cerebrovascular disease and stroke. In


tifungal agent. Its discontinuation would not this case, the decline has been steadily progres-
result in the signs and symptoms found in this sive in a patient with no history of vascular dis-
patient. ease.
Answer D is incorrect. Metformin sensitizes Answer E is incorrect. Parkinson disease clas-
the body’s response to insulin and is used to sically presents with bradykinesia, masklike fa-
treat type 2 diabetes, not Addison disease. cies, shuffling gait, tremor, and rigidity. This
patient has mild bradykinesia and no rigidity or
31. The correct answer is D. This patient has a tremor, so this diagnosis is a less likely possibil-
potentially reversible case of dementia: normal ity.
pressure hydrocephalus (NPH), with the clas-
sic triad of incontinence, gait difficulty, and 32. The correct answer is B. This patient pre­
mental decline (“wet, wobbly, and wacky”). sents to her appointment exhibiting ataxia and
Patients with NPH often demonstrate mild paresthesias, signs of vitamin B6 (pyridoxine)
bradykinesia and their gait has been described deficiency in a patient taking isoniazid. Pyri-
as “magnetic” because their feet seemingly doxine is the precursor to the coenzyme pyri-
cling to the floor. The score of 26/30 on the doxal phosphate, and isoniazid, which is used
Mini-Mental State Examination (MMSE) to treat tuberculosis (TB), inhibits pyridoxine.
indicates only that some mild abnormality is Pyridoxine plays a role in neurotransmitter
present. Regardless, the patient should un- production and the conversion of tryptophan
dergo magnetic resonance imaging to rule out to niacin (vitamin B3). Patients taking iso-
a mass lesion that could cause similar symp- niazid without supplemental vitamin B6 can
toms. The pathophysiology of NPH is not well develop neuropathy as well as symptoms of
understood, but it is thought that neurons are depression, irritability, confusion, and convul-
stretched secondary to ventricular dilation sions. Cheilosis (cracks or sores on the lips),
caused by excessive cerebrospinal fluid pro- glossitis, and stomatitis can also be seen. Vi-
duction, decreased absorption, or both. It is tamin B6 is routinely administered to TB pa-

Full-Length Exams
imperative to identify these patients because tients taking isoniazid; deficiency rarely occurs
timely intervention with a ventriculoperitoneal in well-nourished adults.
shunt can reverse the dementia and decline.
Answer A is incorrect. Ethambutol can be
Answer A is incorrect. Alzheimer disease can used in combination therapy to treat TB, but it
present with some of the symptoms in this causes optic neuropathy (change in visual acu-
case. However, significant physical impair- ity or red-green color blindness). Ethambutol
ment tends to occur later in the Alzheimer is not recommended in children, since visual
disease process and would thus correlate with acuity and changes in color perception are dif-
a much lower score on the MMSE. The time ficult to assess in them.
course and the relatively rapid progression in
Answer C is incorrect. Levofloxacin, a respi-
symptoms are not consistent with this diagno-
ratory fluoroquinolone, is used as second-line
sis.
combination therapy for the treatment of TB.
Answer B is incorrect. Hypothyroidism, an- Potential toxicities include tendinitis or tendon
other potential cause of reversible dementia rupture in adults, and cartilage damage in chil-
in the elderly, should be ruled out early in the dren.
work-up. This patient’s TSH level is normal,
Answer D is incorrect. Pyrazinamide can be
Test Block 2

indicating euthyroidism.
used in combination therapy to treat TB, and
Answer C is incorrect. Multi-infarct dementia it causes hepatotoxicity.
is the most common cause of cognitive decline
Answer E is incorrect. Rifampin is a first-line
with a stepwise drop in function in the setting
therapy used in the treatment of TB, but it
554 Section III: Full-Length Examinations  •  Answers

does not cause peripheral neuropathy. Adverse 34. The correct answer is A. The patient has
effects include GI (nausea, vomiting, and di- symptoms of hypercalcemia in combination
arrhea), CNS (headache and fever), dermato- with a history of smoking and a “coin” lesion
logic (rash, itching, and flushing), and hemato- in the lung, very suspicious of a lung tumor
logic (thrombocytopenia and acute hemolytic that produces parathyroid hormone-related
anemia). Patients should also be informed that peptide. Squamous cell carcinoma is a cen-
rifampin causes red-orange discoloration of trally located bronchogenic carcinoma.
body fluids (eg, sweat, saliva, and tears).
Answer B is incorrect. Horner syndrome is
characterized by ptosis, miosis, and anhidrosis.
33. The correct answer is E. This patient most
It is a complication of lung cancer at the apex,
likely has a Mycobacterium tuberculosis in-
referred to as Pancoast tumor.
fection. M tuberculosis is an aerobic, gram-
positive, acid-fast bacillus, and the Ziehl- Answer C is incorrect. Chronic silica expo-
Neelsen stain is used to reveal acid-fast sure is associated with increased TB suscepti-
bacteria. Characteristics favoring a diagnosis of bility. TB usually presents with chronic cough,
TB include pulmonary symptoms, immigrant hemoptysis, fevers, chills, and weight loss. This
status, night sweats, weight loss, and chest x- patient does not demonstrate the classic signs
ray findings. Primary TB is known to result in and symptoms of silicosis.
Ghon complexes, which show up as calcifica-
Answer D is incorrect. Cystic fibrosis causes
tions on x-ray imaging. Ghon complexes are
respiratory, reproductive, and GI symptoms.
a combination of parenchymal lesions and in-
Although not completely impossible, it would
volved hilar and/or mediastinal lymph nodes.
be highly unlikely for a 65-year-old patient to
The lesions are calcified because of the case-
be afflicted with cystic fibrosis.
ating granuloma formation. Secondary TB pre­
sents with cavitary lesions and is seen more in Answer E is incorrect. Kartagener syndrome
immunocompromised patients. Other patholo- is associated with sinusitis, bronchiectasis,
gies that can present with hilar/mediastinal and infertility. Always think of Kartagener syn-
nodes are lymphoma and sarcoidosis, making drome if situs inversus is suspected on physical
Full-Length Exams

the Gram stain important in diagnosis. exam.


Answer A is incorrect. Congo red is used to vi- Answer F is incorrect. Ectopic ADH produc-
sualize amyloid, showing apple-green birefrin- tion, as often observed in small cell carcinoma,
gence in polarized light. It is used to visualize would cause water retention (oliguria) and fa-
amyloidosis associated with multiple myeloma, tigue.
TB, rheumatoid arthritis, and chronic condi- Answer G is incorrect. Solitary parathyroid
tions. adenoma can present with hypercalcemia and
Answer B is incorrect. Giemsa stain is used low phosphorus levels. However, the patient’s
to reveal Borrelia, Plasmodium, trypanosomes, smoking history and new findings on chest im-
and Chlamydia organisms. aging cannot be ignored.
Answer C is incorrect. India ink is the stain of 35. The correct answer is A. The image de-
choice for Cryptococcus neoformans. Mucicar- picts the epithelium that lies above the Peyer
mine can also be used to stain the thick poly- patches, found within the ileum. This epithe-
saccharide capsule red. lium superficial to Peyer patches has several
Answer D is incorrect. Periodic acid-Schiff microfold cells (known as M cells), specialized
Test Block 2

stains glycogen and mucopolysaccharides. It cells that function to endocytose and phago-
is used to diagnose Whipple disease, caused by cytose particles in the lumen of the gut. Thus,
Tropheryma whipplei. they serve as immune surveillance in the intes-
tines. In adults, B lymphocytes predominate
in Peyer patches and secrete IgA, the main
Test Block 2  •  Answers 555

antibody present within the mucosal lining of Answer D is incorrect. Isoproterenol is a non-
the gut. IgA is synthesized by plasma cells that selective agonist at both b1 and b2 receptors. It
reside within the lamina propria. Of note is would enhance the cardiac effects of norepi-
the fact that several gut pathogens express viru- nephrine via its actions on b1 receptors, while
lence factors, known as IgA proteases, which its action on a1 receptors (vasodilation) would
cleave and therefore deactivate the dimeric antagonize the vasoconstrictive properties that
IgA antibodies. Mycobacterium avium infec- norepinephrine would exert via a1 receptors.
tion is common in patients with AIDS. Eighty
percent to 90% acquire the infection by oral 37. The correct answer is D. A feared infec-
ingestion and subsequent penetration of the tious complication seen in patients with long-
bacteria through Peyer patches of the ileum. standing diabetic ketoacidosis is invasive rhi-
nocerebral mucormycosis. As in this case, this
Answer B is incorrect. IgD is found only on
infection leads to persistent sinusitis with inevi-
the surface of B lymphocytes; its function is not
table invasion into adjacent neural structures
known.
such as the trigeminal nerve and the frontal
Answer C is incorrect. IgE orchestrates the lobe. Rhizopus organisms thrive in serum con-
type I hypersensitivity response. Cross-linking taining high glucose levels and low pH. Other
of two IgE molecules on the surface of mast conditions predisposing patients to this aggres-
cells by antigen results in mast cell degranula- sive infection include iron overload/chelator
tion and allergic reaction. treatment, AIDS, immunosuppression due to
prolonged steroid use, and hematologic malig-
Answer D is incorrect. IgG is the main anti-
nancies. Under the microscope, Mucor species
body produced during a secondary immune re-
appear as irregular, broad, nonseptate hyphae
sponse and also the most abundant.
with 90-degree branching. Both Mucor and Rhi-
Answer E is incorrect. While IgM can be zopus species can cause this condition.
found within the gut lumen, IgA predominates.
Answer A is incorrect. Aspergillus species
36. The correct answer is E. Norepinephrine ex- appear microscopically as 45-degree angle

Full-Length Exams
erts its agonist effect at a1, a2, and b1 recep- branching, septate hyphae with rare fruiting
tors. Its vascular effects are due to its action at bodies. Invasive Aspergillus infection occurs
a (predominantly a1) receptors, and its cardiac mainly after prolonged profound immunosup-
effects are due to its action on b receptors. La- pression (as in patients with AIDS or cancer or
betalol is a nonselective antagonist at a and b in individuals with chronic granulomatous dis-
receptors, and therefore would prevent the ac- ease) and typically leads to bronchopulmonary
tion of norepinephrine at both sites. aspergillosis with cavitary lesions in the lung.

Answer A is incorrect. Atenolol is a b1- Answer B is incorrect. Cryptococcus spe-


selective antagonist and would only mitigate cies appear as 5- to 10-μm yeasts with a wide
the cardiac effects of norepinephrine. capsular halo. These infections represent an
important opportunistic infection in patients
Answer B is incorrect. Doxazosin is a selective with AIDS, causing cryptococcal meningitis.
a1-antagonist that is used to treat hypertension Cryptococcal infections are extremely rare
and urinary retention in the setting of BPH. in patients with normal CD4+ T-lymphocyte
It would be effective at blocking norepineph- counts.
rine’s effect on the vasculature, but would not
antagonize its action on the heart. Answer C is incorrect. Blastomycetes species
Test Block 2

appear as broad-based, budding, dimorphic


Answer C is incorrect. Esmolol is a short- fungi. Blastomycosis is mainly a pulmonary
acting b1-selective antagonist, and therefore infection, endemic to states east of the Missis-
would only control the cardiac effects of nor- sippi River and Central America. Infection of
epinephrine. the lung leads to polygranulomatous infection
556 Section III: Full-Length Examinations  •  Answers

with frequent hematogenous dissemination. is similar to the condition described in the


These fungal species are cultured on Sab- correct answer in that it, too, has a mono-
ouraud agar. clonal spike. An important difference is that
MGUS is asymptomatic as a result of a lower
Answer E is incorrect. Candidal species ap-
level of protein. Some patients may experi-
pear microscopically as pseudohyphae with
ence mild polyneuropathy, but they will not
budding yeasts. Candidal infections are more
have the bone pain, renal failure, and anemia
common in patients with poorly controlled
of multiple myeloma or the hyperviscosity of
diabetes, but they rarely cause invasive rhino-
Waldenström macroglobulinemia. Nonethe-
cerebral infections as described in this patient.
less, MGUS may be a pre-malignant lesion
Rather, candidal infection may cause vulvo-
that can progress to multiple myeloma.
vaginosis, chronic mucocutaneous infections,
or disseminated candidiasis in advanced cases. Answer D is incorrect. Multiple myeloma is
similar to the condition described in the cor-
38. The correct answer is E. The disease that is rect answer, but it also involves abnormal
described is Waldenström macroglobulinemia, plasma cells overproducing immunoglobulin,
which is characterized by weakness, weight seen as a monoclonal M spike (critical for di-
loss, a monoclonal M spike on serum protein agnosis). Multiple myeloma typically presents
electrophoresis (seen as a large spike in the with a collection of characteristic symptoms,
gamma region), and a hyperviscosity syndrome which include lytic bone lesions causing bone
(manifesting as nosebleeds, headaches, and vi- pain and hypercalcemia, renal insufficiency
sion disturbances). Hyperviscosity is caused by and azotemia, increased susceptibility to infec-
the large amount of IgM protein in the blood tion, and anemia. Additionally, one may find
that is produced by a B-cell neoplasm. These Bence Jones protein (immunoglobulin light
large proteins interfere with microvascular and chains) in the urine and a rouleaux formation
cellular processes, causing blood vessel dam- of RBCs on peripheral blood smear.
age, which results in headaches due to im-
paired cranial blood flow and in disturbances 39. The correct answer is E. Selection bias is be-
Full-Length Exams

in vision due to poor ocular blood flow. Addi- ing displayed in this scenario. The physician
tionally, circulating IgM proteins can bind to is selecting his more serious cases for the treat-
clotting factors and inhibit them, causing in- ment group (ie, those who are in most need of
creased bleeding. the benefit). The placebo group contains pa-
tients who are healthier, less symptomatic, and
Answer A is incorrect. Chronic lymphocytic
more likely to have a better outcome. There-
leukemia (CLL) typically presents with lym­
fore, when it comes time for collecting data,
phadenopathy, hepatosplenomegaly, a warm-
the drug’s beneficial effect compared to pla-
antibody autoimmune hemolytic anemia, and
cebo may be blunted.
smudge cells in the peripheral blood. The hy-
perviscosity syndrome is not present in CLL. Answer A is incorrect. Studies can still be
valid if there are differences in group size.
Answer B is incorrect. Diabetes presents with
There is no evidence that there is a difference
nocturia, polyuria, and polydipsia. Blood tests
in group size in this scenario.
would demonstrate increased glucose. Super-
ficial resemblances between the hyperviscos- Answer B is incorrect. Late-look bias occurs
ity syndromes and diabetic retinopathy, and when information or results are gathered at an
diabetic kidney disease with the renal insuffi- inappropriate time. Late-look bias is not dis-
Test Block 2

ciency of multiple myeloma, may be mislead- played in this scenario.


ing. However, bleeding complications due to
Answer C is incorrect. Recall bias occurs
diabetes alone would be rare.
when knowledge of the presence of a disorder
Answer C is incorrect. Monoclonal gammop- alters the way a subject remembers his or her
athy of undetermined significance (MGUS)
Test Block 2  •  Answers 557

history. For example, patients may over- or surgical wound infections as well as with en-
underestimate their consumption of a certain docarditis, skin infections, and toxic shock syn-
drug upon learning of its detrimental effect on drome.
the body. Recall bias is not displayed in this
Answer A is incorrect. Bacteroides fragilis is an
scenario.
anaerobic bacterium that is found in a variety
Answer D is incorrect. Sampling bias occurs of infections but is especially common in ab-
when those in the trial are not truly representa- dominal infections. It can lead to peritonitis or
tive of the general population. Therefore, the intraperitoneal abscesses. Treatment of anaero-
results (both positive and negative) of the study bic infection involves use of metronidazole or
cannot be truly applied to the general popula- clindamycin. Abscesses may require surgical
tion. There is no evidence of sampling bias in drainage. B fragilis is one of the few gram-
this scenario. negative bacteria that do not contain lipid A in
its outer cell membrane; thus, it has no endo-
40. The correct answer is B. Prostate cancer toxin.
is present in the peripheral zone in 70% of
Answer C is incorrect. Actinomyces species
cases. The peripheral zone has a different em-
can be identified as gram-positive rods with
bryologic derivation than the transition zone,
branching filaments. Actinomyces infection
which is the most common site of BPH.
usually involves the cervical or facial region
Answer A is incorrect. The neurovascular and is associated with abscesses, sinus tract in-
bundle is a periprostatic structure. It may be fections, and fistulas. It is easily confused with
involved in prostate cancer that spreads to ad- other diseases of the head and neck, such as
jacent structures beyond the prostate capsule. malignancy. Antibiotic treatment of Actinomy-
Answer C is incorrect. The periurethral zone ces species infections is with penicillin G or
is synonymous with the transition zone, which with tetracyclines in the case of penicillin al-
is the most common site for BPH. Periurethral lergy.
involvement causes voiding problems, and this Answer D is incorrect. Bacillus anthracis is

Full-Length Exams
is a presenting symptom in many patients with a gram-positive, aerobic, spore-forming, rod-
periuretheral involvement. shaped bacterium. Infection can be of either
Answer D is incorrect. The seminal vesicles the cutaneous or the inhalational form. The
are organs adjacent to the prostate, but they cutaneous form is transmitted by contact with
are not made of prostatic tissue. Invasion of spores and is manifested by cutaneous ulcer-
prostate cancer into the seminal vesicles would ation and eschar formation. Treatment for cu-
increase the cancer stage. taneous infection is with penicillin, erythromy-
cin, or ciprofloxacin. The inhalational variety,
Answer E is incorrect. The transition zone is also called woolsorters’ disease, presents with
the most common area for BPH, but not for nonspecific symptoms of fever and malaise but
prostate cancer. progresses to respiratory failure. Treatment of
the inhalational infection is with IV penicillin,
41. The correct answer is B. This patient presents although most patients will die despite treat-
with a lobar pneumonia caused by Staphylo- ment.
coccus aureus. Cefazolin is a first-generation
cephalosporin that has excellent coverage Answer E is incorrect. Pseudomonas aeru-
of gram-positive cocci. It does not cover ginosa is an oxidase-positive, non-lactose-
fermenting, gram-negative, rod-shaped bac-
Test Block 2

methicillin-resistant S aureus or vancomycin-


resistant enterococcus. It is commonly used in terium that causes many different types of
surgical prophylaxis and is a first-line treatment infection, including pneumonia, swimmer’s ear,
of S aureus. S aureus infection is associated with urinary tract infection, and hot-tub folliculitis.
indwelling catheters, respiratory infections, and It is an aerobic gram-negative rod that produces
pyocyanin, a blue-green pigment. P aeruginosa
558 Section III: Full-Length Examinations  •  Answers

infection can be treated with many agents, in- normal-appearing tissue, as described in this
cluding anti-pseudomonal penicillins, cipro- vignette. Due to the transmural quality of the
floxacin, and anti-pseudomonal cephalospo- inflammation, fistulas may develop more fre-
rins such as fourth-generation cephalosporins quently in Crohn disease than in ulcerative
like cefipime. First-generation cephalosporins colitis.
(such as cefazolin) are not effective against
Pseudomonas species. 43. The correct answer is E. Trypanosoma cruzi
infection can cause aganglionic megacolon
42. The correct answer is D. Inflammatory bowel and Chagas disease, a condition in which the
disease typically presents during late adoles- heart is enlarged and flaccid. T cruzi is trans-
cence to early adulthood with symptoms of mitted via the reduviid bug. Microscopic ex-
abdominal pain and frequent bouts of diar- amination reveals flagellated trypomastigotes
rhea. Types of Inflammatory bowel disease are in the blood and non-flagellated amastigotes
differentiated and diagnosed on the basis of in cardiac muscle. T cruzi infection is treated
their clinical picture, their appearance on en- with nifurtimox. The fact that this man was
doscopy and biopsy, and the exclusion of other from Central America is a second clue to his
intestinal infectious etiologies. In this patient, illness; epidemiologically, T cruzi infections
the areas of normal-appearing mucosa should are most common among the poor in rural
immediately point to the diagnosis of Crohn Central and South America.
disease as opposed to ulcerative colitis. Ulcer­
Answer A is incorrect. Cryptosporidium in-
ative colitis is characterized by continuous
fection presents with severe diarrhea in HIV-
mucosal inflammation that is limited to the co-
positive patients and mild watery diarrhea in
lon and always involves the rectum. Crypt ab-
HIV-negative patients. Cryptosporidium is
scesses and ulceration of the mucosa are clas-
transmitted via cysts in water (fecal-oral trans-
sically seen on biopsy. Crohn disease, however,
mission). Microscopically, acid-fast-staining
shows transmural inflammation interspersed
cysts are found. Unfortunately, there is no
with normal mucosa (“skip” lesions), as seen in
treatment available for Cryptosporidium infec-
this patient. Crohn disease can affect any part
Full-Length Exams

tion; however, in healthy patients, cryptospo-


of the GI tract but usually spares the rectum.
ridiosis is self-resolving.
Noncaseating granulomas may be found in
Crohn disease but are not found in ulcerative Answer B is incorrect. Entamoeba histolytica
colitis. is acquired by ingestion of viable cysts from
fecally contaminated water, food, or hands
Answer A is incorrect. Cells with loss of mu-
(fecal-oral transmission). Infection presents
cin and hyperchromatic nuclei are present in
with bloody diarrhea, abdominal cramps with
colon cancer, which is more commonly associ-
tenesmus (a feeling of incomplete defeca-
ated with a long history of ulcerative colitis.
tion), and pus in the stool. It also can cause
Answer B is incorrect. Crypt abscesses are typ- right-upper-quadrant pain and liver abscesses.
ically seen in ulcerative colitis. On microscopy, one observes amebas with in-
gested RBCs. Treatment includes metronida-
Answer C is incorrect. Hyperplasia of goblet
zole and iodoquinol.
cells is the central feature of hyperplastic pol-
yps, the most common type of non-neoplastic Answer C is incorrect. Giardia lamblia in-
polyp. Although usually asymptomatic, they fection presents with bloating, flatulence,
may cause bleeding, abdominal pain, and, foul-smelling diarrhea, and light-colored fatty
Test Block 2

rarely, obstruction. stools. G lamblia is transmitted via cysts in wa-


ter (fecal-oral transmission). On microscopy,
Answer E is incorrect. Ulceration limited
one observes teardrop-shaped trophozoites
to the mucosa is a feature of ulcerative coli-
with a ventral sucking disc or cysts. Metronida-
tis. In Crohn disease the inflammation is of-
zole is used to treat G lamblia infection.
ten transmural and interspersed with areas of
Test Block 2  •  Answers 559

Answer D is incorrect. Toxoplasma gondii in- had Wegener granulomatosis, one would ex-
fection presents with brain abscesses in HIV- pect to see a specific pattern of symptoms in-
positive patients and with birth defects if infec- volving the sinuses, lungs, and kidneys.
tion occurs during pregnancy. Toxoplasmosis
Answer D is incorrect. Immune complex de-
is one of the ToRCHeS organisms (Toxoplas-
position causes damage to the glomerulus in
mosis, Rubella, Cytomegalovirus, Herpesvirus/
many diseases such as poststreptococcal glo-
HIV, Syphilis). T gondii is transmitted via cysts
merulonephritis and SLE. The cause can
in raw meat or cat feces. The definitive stage
be idiopathic, due to an antigenic stimulus,
(sexual stage) occurs in cats. The diagnosis
or due to a systemic immune complex disor-
most often is made serologically. Sulfadiazine
der. On immunofluorescence one would see
and pyrimethamine are used to treat toxoplas-
lumpy or granular deposition of immune com-
mosis.
plexes in the glomerulus.
44. The correct answer is B. A young man pre- Answer E is incorrect. The patient’s presen-
senting with hemoptysis should raise a high tation is characteristic of Goodpasture syn-
index of suspicion for Goodpasture syndrome. drome, which is caused by antibodies specific
This diagnosis is supported by his fatigue and to type IV collagen. Immune-related injury to
hematuria (although typically renal symp- the glomerulus can be separated into three cat-
toms follow pulmonary symptoms by weeks to egories: immune-complex glomerulonephritis,
months). As the disease progresses, one would anti-GBM, and pauci-immune glomerulone-
expect a nephritic picture with hematuria, phritis (no antibodies, complement, or im-
hypertension, and oliguria. The diagnosis of mune deposition). It has been proposed that
Goodpasture syndrome is confirmed by the pauci-immune glomerulonephritis is mediated
renal biopsy, which on immunofluorescence by T lymphocytes, which release cytokines and
staining shows a linear pattern of IgG deposi- thereby recruit inflammatory cells.
tion along the basement membrane. These
anti-glomerular basement membrane (GBM) 45. The correct answer is A. The patient has expe-
antibodies are specific to the a3 chain of type rienced rhabdomyolysis secondary to extreme

Full-Length Exams
IV collagen, and cause injury to both the glo- muscle strain. Rhabdomyolysis causes the re-
merular and alveolar basement membranes. lease of muscle cell contents into the blood-
stream, leading to an elevated creatine kinase
Answer A is incorrect. Goodpasture syndrome
level and myoglobinuria (red urine character-
is caused by anti-GBM antibodies specific
ized by a urine dipstick test that is positive for
to the a3 chain of type IV collagen. Type III
blood but shows no RBCs on urinalysis). Re-
collagen is found in skin, blood vessels, and
lease of intracellular potassium may lead to the
other organs and is not affected by anti-GBM
development of significant arrhythmias and
antibodies. The most common pathology in-
possibly death.
volving type III collagen is Ehlers-Danlos syn-
drome, a connective tissue disorder in which Answer B is incorrect. Hepatomegaly is a non-
patients bleed very easily and have hyperelastic specific sign of many medical conditions but is
skin. not a typical consequence of rhabdomyolysis.
Answer C is incorrect. Antineutrophil cy- Answer C is incorrect. Pain in a dermatomal
toplasmic antibodies (ANCA) are found in distribution is characteristic of shingles and is
certain pauci-immune glomerulonephritides unrelated to rhabdomyolysis.
such as Wegener granulomatosis. This could
Test Block 2

Answer D is incorrect. Pain on urination


account for a nephritic picture, but immuno- would be a symptom of a urinary tract infec-
fluorescence would show an absence of any tion. Because this patient does not have WBCs
immue deposition. Furthermore, if the patient in his urine, he most likely does not have a uri-
nary tract infection.
560 Section III: Full-Length Examinations  •  Answers

Answer E is incorrect. A shuffling gait may be Answer D is incorrect. It is possible that this
seen in Parkinson disease and is unrelated to fetus inherited a recessive disorder such as cys-
rhabdomyolysis. tic fibrosis, phenylketonuria, or sickle cell ane-
mia. However, these diseases are usually tested
46. The correct answer is C. An elevated a- for in patients with a family history using DNA
fetoprotein (AFP) level in amniotic fluid and studies of fetal cells collected from amniotic
maternal serum may indicate neural tube de- fluid, not by measuring AFP levels.
fects such as spina bifida, meningocele, and
Answer E is incorrect. Nondisjunction during
meningomyelocele. These defects are caused
meiosis is the usual cause of trisomy 21, the ge-
by the failure of the caudal portion of the neu-
netic abnormality in Down syndrome. Trisomy
ral tube to close. The AFP level is elevated
21 is more common in women >35 years old,
because AFP is leaked into the amniotic fluid
so this patient’s age puts her at increased risk
from the neural tube defect. Children with
of having a baby with Down syndrome. How-
these defects suffer from a varying degree of
ever, unlike neural tube defects, Down syn-
symptoms that usually include motor and sen-
drome causes a decrease in AFP levels and an
sory defects in the lower extremities and dys-
increase in b-human chorionic gonadotropin
function of bowel and bladder control. Folate
levels.
deficiency during the first four weeks of preg-
nancy has been implicated in causing neural
47. The correct answer is C. The boy has a history
tube defects. Drugs that increase the risk of
consistent with absence seizures. On clinical
neural tube defect include valproate and car-
examination, typical absence seizures appear
bamazepine.
as brief staring spells with no warning or post-
Answer A is incorrect. Bilateral renal agenesis ictal phase. Children are not responsive during
(Potter syndrome) is caused by disruption in the seizure and are amnestic of what happened
the interaction between the ureteric bud and during the attack. In fact, patients are gener-
the metanephrogenic tissue. Because the fetus ally unaware that a seizure has occurred. Clas-
does not produce urine (which is a component sically, a regular and symmetric 3-Hz spike is
Full-Length Exams

of amniotic fluid), there is a smaller volume of found on electroencephalography. Ethosuxi-


amniotic fluid than normal. This is described mide is the primary treatment option in cases
by the term oligohydramnios. The smaller of absence (petit mal) seizures.
amount of protective fluid results in pulmo-
Answer A is incorrect. Carbamazepine has
nary hypoplasia, fetal compression with altered
been associated with the exacerbation of ab-
facies, and positioning defects of hands and
sence seizures.
feet. Bilateral renal agenesis is not compatible
with life. Oligohydramnios, not increased AFP Answer B is incorrect. Clonazepam and the
levels, would be noted in this case. ketogenic or medium-chain triglyceride diet
have been attempted to reduce seizure fre-
Answer B is incorrect. The ductus arteriosus
quency. These adjunctive therapies, however,
is a connection between the pulmonary artery
have limited efficacy.
and the aorta that allows oxygenated blood from
the placenta to bypass the fetal lungs and enter Answer D is incorrect. Gabapentin has been
the systemic circulation. This pathway should associated with the exacerbation of absence
be open during gestation and is not an abnor- seizures.
mality. At birth, as the infant takes a breath, an Answer E is incorrect. The teacher’s concerns
increase in oxygen content causes a decrease in
Test Block 2

regarding the boy are quite common in the


prostaglandins, resulting in closure of the con- case of absence seizures. Often such concerns
nection. If the ductus arteriosus remains patent will be incorrectly attributed to inattentiveness
after birth, the baby can be given indomethacin
to help stimulate the vessel to close.
Test Block 2  •  Answers 561

and may even lead to a misdiagnosis of atten- might also manifest anemia, increased suscep-
tion deficit/hyperactivity disorder (ADHD). tibility to infection, or clotting abnormalities
Methylphenidate, a CNS stimulant, is the cor- secondary to the reduced production of nor-
nerstone of therapy in ADHD. mal blood components.
Answer F is incorrect. Tiagabine has been as- Answer A is incorrect. The primary func-
sociated with the exacerbation of absence sei- tion of mitochondria is the synthesis of ATP.
zures. They are abundant in cells that require a large
amount of energy, such as myocytes.
48. The correct answer is B. This patient has
Answer C is incorrect. Secondary lysosomes
multiple myeloma, a neoplastic proliferation
are formed when a primary lysosome, with its
of plasma cells in the bone marrow that often
hydrolytic enzymes, fuses with materials for
leads to lytic bone lesions and pathological
degradation. Secondary lysosomes have sub-
fractures. The plasma cell is a differentiated
strates at different stages of digestion. Cells
B lymphocyte that can produce and secrete
such as macrophages, which are responsible
large amounts of antibody specific to a par-
for phagocytosis of cell debris, may contain
ticular antigen. Rough endoplasmic reticulum
multiple secondary lysosomes.
is the site of synthesis of secretory proteins;
thus antibody-secreting plasma cells are rich Answer D is incorrect. Smooth endoplasmic
in RER. Normally there is a polyclonal distri- reticulum (SER) is the site of steroid synthesis
bution of immunoglobulins of different iso- and detoxification of drugs and poisons; thus,
types and antigen specificities in the serum. In cells like hepatocytes and steroid-producing
multiple myeloma, however, the majority of cells of the adrenal cortex are rich in SER.
plasma cells are producing immunoglobulin
Answer E is incorrect. Areas of rough endo-
of one isotype and antigen specifically, which
plasmic reticulum in neurons are called Nissl
can be detected as an M spike by serum pro-
bodies. This is the site of enzyme and peptide
tein electrophoresis. The free light chains (ei-
neurotransmitter synthesis.
ther kappa or lambda) can often be detected

Full-Length Exams
in the urine; this is referred to as Bence-Jones
proteinuria. Patients with multiple myeloma

Test Block 2
This page intentionally left blank
Test Block 3

563
564 Section III: Full-Length Examinations  •  Questions

Q u e st i o n s

1. An 18-month-old child is brought to the phy- 3. A 41-year-old man is admitted to the hospital
sician by her distraught parents because of a for progressive obtundation. On admission the
sore throat, difficulty breathing, and a barking patient’s serum sodium level is 114 mEq/L.
cough for the past day. On physical examina- Treatment is initiated, and seven hours later
tion, the toddler is found to have some respi- the patient’s serum sodium level is 134 mEq/L.
ratory stridor and a runny nose but is not in Over the next four days the patient’s condition
acute distress. The rest of the examination is worsens with the development of dysarthria,
unremarkable. Which of the following is the dysphagia, and paraparesis. What pathologic
most appropriate treatment for this patient? process is most likely responsible for this pa-
tient’s new symptoms?
(A) Amantadine
(B) Bronchoalveolar lavage (A) Cerebral edema
(C) Emergency department admittance (B) Diffuse axonal injury
(D) Penicillin (C) Intracerebral hemorrhage
(E) Supportive therapy (D) Osmotic demyelination
(E) Uncal herniation
2. A 32-year-old woman presents to her family
doctor complaining of fatigue, myalgia, and 4. A 74-year-old man with chronic renal failure
anorexia for nearly one week. Physical exami- has had repeated pathological fractures. Labo-
nation reveals cervical lymphadenopathy and ratory analysis reveals:
the rash seen in the image. If this illness is left
Serum calcium: 6.3 mg/dL
untreated, which of the following symptoms or
Serum phosphate: 4.7 mg/dL
conditions is most likely to occur next?
Parathyroid hormone: 750 pg/mL
Which of the following most likely contributes
to the pathogenesis of this man’s bone disease?
Full-Length Exams

(A) Decreased 1,25-dihydroxycholecalciferol


production
(B) Decreased 25-hydroxyvitamin D produc-
tion
(C) Decreased calcium intake
(D) Decreased vitamin D2 intake
(E) Decreased vitamin D3 intake

5. An epidemic of a diarrhea has broken out in


a city hospital. Colonoscopy of one of the af-
fected patients reveals colonic inflamma-
tion with exudates and necrosis of the muco-
Courtesy of Dr. James Gathany. sal surface. Assays for toxin A and toxin B are
positive. Which of the following is the micro-
biology laboratory likely to isolate from the af-
(A) Aortic aneurysm
fected patients?
(B) Argyll Robertson pupil
Test Block 3

(C) Autoimmune polyarthritis (A) A gram-negative facultative intracellular


(D) Facial nerve palsy organism
(E) Opportunistic infection with Pneumocystis (B) A gram-negative lactose fermenter
jiroveci (C) A gram-negative lactose nonfermenter
Test Block 3  •  Questions 565

(D) A gram-positive aerobe 7. A 68-year-old woman presents to the emer-


(E) A gram-positive anaerobe gency department with altered mental status.
Her temperature is 38.8°C (101.8°F), heart
6. A 9-year-old boy is brought to the emergency rate is 116/min, respiratory rate is 23/min,
department with a two-day history of abdomi- and blood pressure is 132/87 mm Hg. Arterial
nal pain, vomiting, and a rash. His mother re- blood gas shows a pH of 7.28, partial pressure
ports that he had a runny nose and mild cough of carbon dioxide of 15 mm Hg, and a bicar-
about a week ago. On examination there is bonate level of 7 mEq/L. Which of the follow-
diffuse abdominal tenderness and a rash over ing is the most accurate description of the pa-
the arms and the legs. His complete blood cell tient’s acid-base status?
count is within normal limits and urinalysis
(A) Metabolic acidosis
shows 12 RBCs/hpf, 2 WBCs/hpf, no protein,
(B) Metabolic acidosis with respiratory alkalo-
and no glucose. An image of the rash is shown
sis
in the image. What is the most likely etiology
(C) Metabolic acidosis with respiratory com-
of this patient’s symptoms?
pensation
(D) Metabolic alkalosis with respiratory acido-
sis
(E) Respiratory alkalosis

8. A patient is found to have hypertension, hema-


turia, and oliguria. A renal biopsy specimen is
obtained and reveals a focal proliferative glo-
merulonephritis, characterized by linear stain-
ing of the basement membrane on immuno-
fluorescence for IgG. He is also found to have
lung involvement. What pulmonary condition
would present with similar respiratory func-

Full-Length Exams
tion?
(A) Asthma
Courtesy of Wikipedia. (B) Emphysema
(C) Kartagener syndrome
(D) Pneumothorax
(A) Deficiency of von Willebrand factor- (E) Sarcoidosis
cleaving metalloproteinases
(B) IgA antibody deposition in the mesangium
(C) IgA immune complexes deposited in small
vessels
(D) IgG antibodies against platelets
(E) IgG antibodies deposited in the glomerular
basement membrane
Test Block 3
566 Section III: Full-Length Examinations  •  Questions

9. Referring to the image, where A = afferent ar- (A) Atropine


teriole and B = efferent arteriole, which of the (B) Flumazenil
following most accurately reflects the actions (C) Fluoxetine
of angiotensin II and prostaglandins in a dehy- (D) Labetalol
drated patient? (E) Naloxone
(F) Physostigmine
Glomerulus Bowman’s capsule 11. A 2-year-old child has no red reflex in the right
eye. He is subsequently found to have an eye
tumor that is caused by dysfunction of a spe-
B cific cell-cycle regulatory gene product. What
is the normal function of this gene product in
Capillary Renal a quiescent cell?
hydrostatic
pressure tube (A) Inhibits apoptosis
A (B) Prevents cell-cycle progression past the
G1/S checkpoint
(C) Prevents cell-cycle progression past the
G2/M checkpoint
(D) Promotes DNA damage repair
Reproduced, with permission, from USMLERx.com. (E) Promotes histone acetylation

12. Acute allograft rejection is mediated by cyto-


(A) Angiotensin II vasoconstricts A; prostaglan- toxic T-lymphocytes that recognize and are
dins vasodilate B; there is an overall de- activated by the major histocompatibility com-
crease in GFR and increase in RPF plex proteins expressed by the donated organ.
(B) Angiotensin II vasoconstricts A; prostaglan- A depleting monoclonal antibody to which of
dins vasodilate B; there is an overall in- the following cell surface molecules would be
Full-Length Exams

crease in GFR and decrease in RPF most useful in reducing this immune-mediated
(C) Angiotensin II vasoconstricts B; prostaglan- graft rejection?
dins vasodilate A; there is an overall in-
(A) CD3
crease in GFR and decrease in RPF
(B) CD4
(D) Angiotensin II vasoconstricts B; prostaglan-
(C) CD14
dins vasodilate A; there is an overall de-
(D) CD16
crease in GFR and increase in RPF
(E) CD19
(E) Angiotensin II vasodilates B; prostaglan-
dins vasoconstrict A; there is an overall de-
13. A 67-year-old former landscaper is referred to
crease in GFR and increase in RPF
the dermatologist for a lesion on his right fore-
arm. The lesion is a flesh-colored pearly pap-
10. A 24-year-old man presents to the emergency
ule approximately 1.5 cm in diameter with a
department with hypertension, tachycardia,
central telangiectasia. A biopsy is taken, and
fever, diaphoresis, mydriasis, and severe agita-
the results are shown in the image. Which of
tion. When asked, his mother states that her
the following is the most likely diagnosis?
son and his friends “probably used some drugs
they got in the neighborhood.” Which of the
Test Block 3

following agents is the most appropriate ther-


apy for this patient?
Test Block 3  •  Questions 567

(A) A blastocyst implanted in the ampulla


(B) A blastocyst implanted in the posterior su-
perior uterine wall
(C) A fecalith obstructing the appendiceal lu-
men
(D) Ectopic endometrial tissue implanted on
the ovary
(E) Two spermatozoa fertilizing a single ovum

15. A 64-year-old man with a history of hyper-


tension, coronary artery disease, and type 2
diabetes presents to his physician because he
“has trouble seeing.” Visual field testing re-
veals a defect in the left half of the visual field
Reproduced, with permission, from USMLERx.com.
for both eyes, with sparing of central acuity.
Which of the following is the most likely cause
(A) Actinic keratosis of the patient’s symptoms?
(B) Basal cell carcinoma (A) Complete infarction of the optic chiasm
(C) Dermatitis herpetiformis (B) Infarction of the lower division of the mid-
(D) Melanoma dle cerebral artery
(E) Seborrheic keratosis (C) Left retinal artery occlusion with sparing of
(F) Squamous cell carcinoma the vessels supplying the macula
(D) Right posterior cerebral artery infarction
14. A 28-year-old woman with a past medical his- with sparing of Meyer’s loop
tory significant for pelvic inflammatory dis- (E) Right posterior cerebral artery infarction
ease presents to the emergency department with sparing of the occipital pole
with right lower quadrant abdominal pain.

Full-Length Exams
The pain began two hours ago, has been con- 16. A 60-year-old woman is receiving chemother-
sistently localized to the right lower quadrant apy for breast cancer. She presents to her on-
without migration, and has been associated cologist complaining of fatigue and dyspnea
with nausea and vomiting. Although her peri- on exertion. Physical examination reveals an
ods are usually regular, her last menstruation elevated jugular venous pressure, crackles, and
was approximately six weeks ago. On examina- 4+ pitting edema bilaterally. X-ray of the chest
tion, she is found to be afebrile with a blood shows an enlarged cardiac silhouette. Her on-
pressure of 90/60 mm Hg, a pulse of 110/min, cologist believes her chemotherapeutic agent
and a respiratory rate of 26/min. Abdominal is responsible for these complaints. What is the
examination shows localized tenderness with mechanism of the chemotherapeutic agent she
guarding in the right lower quadrant. Pelvic is most likely receiving?
examination is deferred due to excessive pain,
(A) DNA intercalation
but vaginal bleeding is noted. Laboratory stud-
(B) Inhibition of dihydrofolate reductase
ies show a hematocrit of 29.8% and an el-
(C) Inhibition of microtubule formation
evated human chorionic gonadotropin (hCG)
(D) Inhibition of purine synthesis
level. Which of the following is the most likely
(E) Selective estrogen receptor modulator
etiology of this patient’s illness?
Test Block 3
568 Section III: Full-Length Examinations  •  Questions

17. A 22-year-old woman comes to your office Glucose: 96 mg/dL


complaining of vaginal itching and burn- Serum osmolality: 250 mOsm/kg
ing. She says she feels as if she “has the flu”
Which of the following is the most likely cause
and has had intermittent fevers and muscle
of this patient’s condition?
aches over the past few days. Vaginal examina-
tion reveals the lesion seen in the image, and (A) Excessive fluid intake
treatment is started. The patient subsequently (B) Glioblastoma multiforme
develops elevated levels of blood urea nitro- (C) Metastatic breast cancer
gen and creatinine in addition to a tremor and (D) Small cell lung cancer
mental status changes. Which of the following (E) Squamous cell lung cancer
agents did this patient most likely receive?
19. A 33-year-old man from upstate New York
comes to his physician because of flu-like
symptoms after a camping trip one week
ago. He also notes a troubling rash on his leg
that has moved slowly from his ankle to his
midthigh over the past several days. The phy-
sician diagnoses Lyme disease and prescribes
tetracycline. The patient recently lost his pre-
scription benefits through his health insurance
plan, so he uses an old bottle of tetracycline
from his medicine cabinet at home. One week
later the man presents to the emergency de-
partment with signs of dehydration and a cre-
atinine level of 3.6, up from his baseline of 0.6.
Which of the following renal conditions is this
Reproduced, with permission, from USMLERx.com. man most likely experiencing?
Full-Length Exams

(A) Acute tubular necrosis


(A) Acyclovir (B) Glomerulonephritis
(B) Fluconazole (C) Kidney stones
(C) Metronidazole (D) Renal papillary necrosis
(D) Penicillin (E) Renal tubular dysfunction
(E) Ribavirin
20. A 21-year-old woman presents to her male fam-
18. A 60-year-old woman with chronic obstruc- ily practitioner complaining of sleep depriva-
tive pulmonary disease (COPD) is brought tion and severe depression. When asked about
into the emergency department after having a her sleeping habits in greater depth, she re-
witnessed tonic-clonic seizure lasting two min- ports sleeping for seven-nine hours per night,
utes. She is currently unresponsive. Her family but states that it is “just not right.” In addi-
states that she has been increasingly confused tion, she later claims to not have any problems
over the past two days or so, and has no prior with her personal life and is happy and excited
history of seizures. Physical examination re- about her recent promotion. Throughout the
veals no abnormalities, and the patient’s vital course of the visit, the patient becomes pro-
signs are all within normal limits. Laboratory gressively more animated and begins making
Test Block 3

tests show: aggressive sexual advances toward the physi-


cian and the staff. She reports that her mood
Na+: 123 mEq/L has been good, but not overly elevated, expan-
K+: 3.8 mEq/L sive, or elated. Which of the following is the
Cl-: 100 mEq/L most likely diagnosis?
HCO3-: 24 mEq/L
Test Block 3  •  Questions 569

(A) Bipolar disorder type I (E) Converts ganglioside M2 to ganglioside


(B) Histrionic personality disorder M3
(C) Obsessive-compulsive personality disorder
(D) Paranoid personality disorder 23. The following graph is a depiction of the Star-
(E) Schizotypal personality disorder ling curve showing the relationship between
cardiac output and ventricular end diastolic
21. A 37-year-old man complains of an unsteady volume in a patient with congestive heart fail-
gait when he walks. He has a history of drug ure. If this patient is treated with a positive
abuse, alcohol abuse, and numerous sex part- inotropic agent, the Starling curve would do
ners. A myelin stain of the spinal cord is shown which of the following?
in the image. Which of the following organ-
isms is responsible for this spinal cord lesion?
4.0

Cardiac output
3.0

2.0

1.0
0 2 4 6 8 10 12
End-diastolic volume

Reproduced, with permission, from USMLERx.com.

Courtesy of Dr. Susan Lindsley, Centers for Disease Control (A) Flatten out
and Prevention.

Full-Length Exams
(B) Shift down and left
(C) Shift down and right
(A) Herpes simplex virus (D) Shift right only
(B) HIV (E) Shift up and left
(C) Mycobacterium tuberculosis (F) Shift up and right
(D) Poliovirus
24. A 16-year-old girl visits her family physician
(E) Treponema pallidum
with complaints of amenorrhea. Although she
22. A 2-year-old boy presents to his pediatrician falls 20% below the minimum body weight
with hepatosplenomegaly, failure to thrive, and expected for her height, she doesn’t think she
progressive central nervous system deteriora- is skinny enough. After breaking up with her
tion. Liver biopsy shows that hepatocytes and boyfriend a year ago, she dropped out of cheer-
Kupffer cells have a foamy, vacuolated appear- leading and has been struggling with school,
ance. The pediatrician suspects that the boy although she used to be an honors student.
will die by age three. Which of the following She admits to crying spells, feeling guilty, and
is the function of the metabolic enzyme defi- thoughts of suicide. Which of the following
cient in this patient? medications is contraindicated in this patient?
Test Block 3

(A) Converts ceramide trihexoside to lactosyl (A) Bupropion


cerebroside (B) Buspirone
(B) Converts galactocerebroside to cerebroside (C) Haloperidol
(C) Converts sphingomyelin to cerebroside (D) Phenelzine
(D) Converts glucocerebroside to cerebroside (E) Sertraline
570 Section III: Full-Length Examinations  •  Questions

25. A patient with adult T-lymphocyte leukemia


receives a bone marrow transplant from an un-
related donor. Despite an immunosuppressive
p55
post-transplant treatment regimen, over the
course of several weeks the patient develops a
severe cutaneous rash and intractable diarrhea.
Blood tests were normal except for alanine
aminotransferase (1032 U/L), aspartate ami- p24
notransferase (829 U/L), lactate dehydrogen­
ase (634 U/L), and alkaline phosphatase (446 gag
U/L). Which of the following is the most likely p18
etiology of the patient’s current symptoms?
(A) Acute graft rejection
(B) Graft-versus-host disease mediated by allo- 1 2 3
reactive donor T-lymphocytes
Reproduced, with permission, from USMLERx.com.
(C) Graft-versus-host disease mediated by allo-
reactive recipient T-lymphocytes
(D) Hyperacute graft rejection (A) Begin this patient on AZT therapy
(E) Recurrence of leukemia (B) Profile this patient’s T cells
(C) Repeat Western blot because of contami-
26. An infectious disease clinician sends a serum nation
sample to a clinical laboratory technician (D) Repeat Western blot because of failed posi-
for HIV diagnostics. To confirm a positive tive control
enzyme-linked immunosorbent assay result, (E) The patient does not have HIV
the technician performs a Western blot, assay-
ing for the presence of antibodies to three dif- 27. Vinca alkaloids such as vincristine are che-
ferent HIV proteins in the patient’s serum. The motherapeutic agents that are used for the
Full-Length Exams

image shows the results: tube 1 is the control treatment of choriocarcinoma as a part of the
for positives, tube 2 is the control for negatives, mechlorethamine/Oncovin/procarbazine/pred-
and tube 3 is the patient’s sample. Which of nisone regimen. Vincristine’s mechanism of
the following is the most appropriate next step action makes the drug specific for which phase
for the clinician? of the cell cycle?
(A) G0 phase
(B) G1 phase
(C) G2 phase
(D) M phase
(E) S phase
Test Block 3
Test Block 3  •  Questions 571

28. A 29-year-old reports to her obstetrician’s of- 30. A 3-year-old boy is brought to the pediatrician
fice three months after a difficult delivery. She because of decreased vision and pain in his
describes constant fatigue and recent weight right eye. Past medical history is significant for
loss. Her menses have not returned. She has the diagnosis of glaucoma shortly after birth
been bottle-feeding, because breastfeeding “did that has been refractory to standard medical
not work.” An endocrine abnormality is sus- therapies. Focused physical examination re-
pected. Which of the following laboratory test veals iris hamartomas. Which of the following
abnormalities would support the most likely additional signs is most likely on physical ex-
diagnosis? (Note that normal values are as fol- amination?
lows: total serum thyroxine, 5-12 μg/dL; serum
(A) Bilateral acoustic neuromas
thyroid-stimulating hormone (TSH), 0.5-5.0
(B) Bilateral renal cell carcinomas
μU/mL; serum ACTH, 9-52 pg/mL.)
(C) Cystic medial necrosis of the aorta
(A) Total serum thyroxine, 0.1 μg/dL; serum (D) Leptomeningeal angioma
TSH, 0.15 U/mL; serum ACTH, 10 pg/ (E) Scoliosis
mL
(B) Total serum thyroxine, 0.1 μg/dL; serum 31. A 10-year-old boy is brought to the emergency
TSH, 10 U/mL; serum ACTH, 25 pg/mL department after falling from his bicycle. He
(C) Total serum thyroxine, 0.1 μg/dL; serum presents with a large, painfully swollen knee;
TSH, 10 U/mL; serum prolactin, 200 ng/ aspiration shows gross hemarthrosis. On fur-
mL ther questioning, the patient’s parents say that
(D) Total serum thyroxine, 1.0 μg/dL; serum he bruises easily and that he had an episode
TSH, 4 U/mL; serum ACTH, 250 pg/mL of prolonged bleeding after losing a tooth one
(E) Total serum thyroxine, 2.5 μg/dL; serum month ago. His maternal uncle had similar
TSH, 0.15 U/ml; serum ACTH, 25 pg/mL bleeding difficulties. After further testing, the
patient is diagnosed with an X-linked recessive
29. A neonate who is born with a cleft palate and disorder. Which of the following laboratory test
abnormal facies becomes cyanotic and hypoxic results corresponds to the patient’s disorder?

Full-Length Exams
soon after birth. On physical examination, the
neonatologist hears a crescendo-decrescendo Partial
Platelet Bleeding Prothrombin thromboplastin
murmur with a harsh systolic ejection. Further Choice
count time time time
investigation shows tetralogy of Fallot. Exami- A normal normal normal
nation of this patient’s serum is likely to reveal B normal normal normal
which of the following? C normal normal

(A) High IgM and normal T cell number D normal normal

(B) Hypogammaglobulinemia and normal T E

cell number
(C) Hypogammaglobulinemia and reduced T Reproduced, with permission, from USMLERx.com.
cell number
(D) Low IgM and normal T cell number
(A) A
(E) Reduced T cell number alone
(B) B
(C) C
(D) D
(E) E
Test Block 3
572 Section III: Full-Length Examinations  •  Questions

32. An obese 32-year-old man presents at the (A) Cimetidine


clinic complaining of increasing difficulty (B) Magnesium hydroxide
catching his breath. He also reports that he has (C) Misoprostol
been wheezing and has a productive cough. (D) Omeprazole
He was told that his lungs are lacking a par- (E) Sucralfate
ticular enzyme due to genetic mutations. Rel-
evant social history reveals that he has been 35. A 43-year-old man with a history of hypercal-
smoking cigarettes for the past 15 years. Fam- cemia and bitemporal hemianopsia presents
ily history is significant for a father with sig- to the emergency department with muscle
nificant lung disease. X-ray of the chest shows weakness, lethargy, and watery diarrhea. He re-
basilar hyperlucency that is localized solely to ports brief episodes of complete paralysis in his
the lung bases and not the apices. This patient lower extremities. The pH of the patient’s na-
is also at increased risk for which other condi- sogastric suction fluid is increased. An abdom-
tion? inal mass is noted on CT scan. The patient’s
family history is positive for numerous endo-
(A) Cor pulmonale
crine organ abnormalities. Which of the fol-
(B) Liver cirrhosis
lowing is the most likely cause of this patient’s
(C) Pseudomonal pneumonia
symptoms?
(D) Renal cysts
(A) Carcinoid tumor
33. A 32-year-old woman presents to the emer- (B) Gastrinoma
gency department with mental status changes, (C) Insulinoma
severe weakness, and multiple petechiae evolv- (D) Pheochromocytoma
ing over the past three weeks. Physical exami- (E) VIPoma
nation is also notable for pale conjunctiva.
Temperature is 101.7°F (38.7°C). Laboratory 36. Patients with Paget disease of the bone usually
studies show a severe anemia, thrombocyto- have serum calcium levels between 8.4 and
penia, and leukocytosis. A peripheral blood 10.5 mg/dL and phosphate levels between 2.7
Full-Length Exams

smear is shown in the image. Which of the and 4.0 mg/dL. During the second phase of
following chromosomal translocations is most the disease, which of the following laboratory
likely involved in this disorder? values is most likely to be seen?
(A) t(8;14)
(B) t(9;22)
Alkaline
(C) t(11;22) Choice phosphatase
Phosphate
(D) t(11;14) (mg/dL)
(U/L)
(E) t(15;17)
A 20 3.7
34. A 17-year-old girl who is six weeks pregnant
presents to the emergency department be- B 50 5.2
cause of abdominal pain and vaginal bleeding.
C 100 5.4
Ultrasound imaging shows no fetal heartbeat
and incomplete fetal development. The deci- D 400 3.5
sion is made to terminate the pregnancy. On
questioning, the patient reports that she has E 1500 5.2
been taking high doses of her mother’s ulcer
Test Block 3

medication for her own heartburn. Which of Reproduced, with permission, from USMLERx.com.
the following medications did this patient most
likely take?
Test Block 3  •  Questions 573

(A) A (A) Acidification of colonic contents, causing


(B) B ammonium trapping
(C) C (B) Decreasing substrate for ammonia-
(D) D producing reactions
(E) E (C) Eliminating colonic flora and decreasing
their subsequent ammonia production
37. A 40-year-old nulligravid woman visits her gy- (D) Facilitating the binding of GABA to the
necologist because of menstrual irregularities. GABA receptor
She also recently started producing breast milk (E) Inhibiting synthesis of tumor necrosis
even though she is not nursing. A pregnancy factor-a
test is negative. Which of the following recent
medical diagnoses is most likely related to the 39. A 39-year-old man who is HIV-positive pre­
patient’s symptoms? sents to the physician with fever, cough, and
difficulty breathing. Physical examination
(A) Menopause
shows that the patient is breathing abnormally
(B) Parkinson disease
fast, and the tips of his fingers have a slight
(C) Schizophrenia
bluish tinge. X-ray of the chest reveals diffuse
(D) Turner syndrome
interstitial pneumonia with a ground-glass ap-
(E) Type 2 diabetes mellitus
pearance. Which of the following therapies
38. A 37-year-old man with end-stage liver disease should be used to treat this patient?
secondary to hepatitis C presents to the emer- (A) Fluconazole or ketoconazole
gency department confused and lethargic. He (B) Itraconazole or potassium iodide
has ascites, spider angiomata, and asterixis. (C) Sulfadiazine and pyrimethamine
Bowel sounds are normal. Laboratory studies (D) Topical miconazole or selenium sulfide
show the following results: (E) Trimethoprim-sulfamethoxazole
Aspartate aminotransferase: 46 U/L
40. A 17-year-old girl with type 1 diabetes mellitus
Alanine aminotransferase: 55 U/L

Full-Length Exams
gives birth to a baby boy at 37 weeks’ gestation
Alkaline phosphatase: 100 U/L
by cesarean section. At one minute after birth,
Bilirubin, total serum: 1.4 mg/dL
the boy is pink with blue fingers and toes. His
Prothrombin time: 38 seconds
heart rate is 80/min, and he does not respond
Albumin: 2.0 g/dL (normal: 3.4-5.4 g/dL)
to noxious stimuli. Muscle tone is absent, and
Which of the following is the mechanism of his cry is weak. What is this child’s one-minute
action of the most appropriate acute treatment APGAR score?
of this patient’s condition?
(A) 1
(B) 3
(C) 5
(D) 7
(E) 9
Test Block 3
574 Section III: Full-Length Examinations  •  Questions

41. A 60-year-old Scandinavian woman presents a show large pleural-based mass in the right
to her doctor with a two-month history of pro- lower lung. A thoracoscopic-guided biopsy
gressive fatigue. She also reports tingling and demonstrates malignant cells in the mesothe-
numbness in her lower extremities and feeling lium. What other microscopic findings are
“wobbly” lately. She has no significant medical likely to be found?
history. Physical examination reveals a pulse of
(A) Granulomas with central caseous necrosis
101/min and decreased light touch and vibra-
and Langhans’ giant cells
tion sense on her lower extremities. Laboratory
(B) Histiocytic cells with cytoplasmic inclu-
studies show a hemoglobin level of 9 g/dL and
sions resembling tennis rackets
a mean corpuscular volume of 110 fL. Periph-
(C) Intra-arterial thrombus with RBC extrava-
eral blood smear is shown in the image. The
sation
etiology of this patient’s anemia results from
(D) Marked intra-alveolar fibrin and cellular
which of the following?
debris
(E) Thick-walled spherules containing endo-
spores with surrounding inflammatory
cells
(F) Yellow-brown, rod-shaped bodies with
clubbed ends that stain positively with
Prussian blue

43. A 27-year-old man enters the emergency de-


partment in an agitated state. He complains
of severe abdominal pain, but soon becomes
paranoid and combative, requiring five-point
restraint. His vital signs show elevated blood
pressure and tachycardia. When a straight
catheter is inserted, reddish urine flows into
Full-Length Exams

the Foley bag. The urine is negative for RBCs,


Reproduced, with permission, from USMLERx.com. and a toxicity screen result is negative. The
doctor suspects a porphyria. Laboratory tests
for urine porphobilinogen are positive. Which
(A) Abnormal neural crest cell migration
of the following enzyme deficiencies is most
(B) Antibodies against parietal cells
likely responsible for this patient’s disorder?
(C) Bacterial overgrowth of colon
(D) Diet deficient in leafy vegetables (A) Aminolevulinate dehydratase
(E) Embolus to the superior mesenteric artery (B) Aminolevulinate synthase
(C) Ferrochelatase
42. An 83-year-old retired construction worker (D) Heme oxygenase
comes to his physician because his chronic (E) Porphobilinogen deaminase
cough has gotten worse over the past year. (F) Uroporphyrinogen decarboxylase
He also notes some moderate facial swelling (G) Uroporphyrinogen III cosynthase
and bilateral arm edema. He has a 50-pack-
year smoking history. X-ray of the chest shows
Test Block 3
Test Block 3  •  Questions 575

44. A 4-year-old girl presents to her pediatrician, (A) Diffuse capillary and basement membrane
who obtains the peripheral blood smear shown thickening
in the image. The loss of function of what (B) Enlarged hypercellular glomeruli with
body part will cause her to become symptom- neutrophils
atic? (C) Nodular glomerulosclerosis with thick-
ened basement membranes
(D) Segmental sclerosis with hyalinosis
(E) Wire-loop appearance with subendothelial
basement membrane deposits

46. A 15-month-old boy sustained scald burns


of his legs and feet in a bathtub. His mother,
who brought him into the emergency depart-
ment, says he was being lowered into the water
with his knees bent when he screamed in pain,
and was immediately removed from the bath-
tub. Physical examination reveals deep partial-
thickness to full-thickness burns bilaterally on
his anterior shins and ankles, the dorsa of his
feet, and the plantar surfaces of the toes and
Reproduced, with permission, from USMLERx.com. distal feet. What is/are the next step(s) in the
management of this child?
(A) Obtain collateral history information from
(A) Bone marrow
other family members, neighbors, or
(B) Kidney
friends
(C) Lining of the small intestine
(B) Order imaging of head, chest, and extremi-
(D) RBCs
ties
(E) Spleen

Full-Length Exams
(C) Provide appropriate treatment for the
45. A 59-year-old man with a history of obesity, wounds and admit to the hospital without
myocardial infarction, and foot ulcers com- informing the authorities
plains to his primary care physician about (D) Provide appropriate treatment for the
numbness and tingling in his lower extremi- wounds and call authorities immediately
ties. Hemoglobin A1c level is 10%. Which of (E) Provide appropriate treatment for the
the following describes the glomerular pathol- wounds, complete a full physical examina-
ogy most likely seen on light microscopy of this tion, and release the patient back to guard-
patient’s kidneys? ian

Test Block 3
576 Section III: Full-Length Examinations  •  Questions

47. A 45-year-old man goes to his primary care


physician complaining of recent headaches.
They rarely persist for more than an hour, but
are sporadic and often accompanied by sudden
sweating and palpitations. “Doc,” he says, “it
feels like my heart is racing.” Acetaminophen
provides minimal relief from the pain. He is
afebrile and his blood pressure is 128/70 mm
Hg. His physical examination is unremarkable
with the exception of moist palms and pale
skin. A urinalysis is notable for elevated meta-
nephrines. This patient’s most likely diagnosis
is often associated with which of the following
conditions?
(A) Hashimoto thyroiditis
(B) Insulinoma
(C) Parathyroid tumor
(D) Prolactinoma
(E) Zollinger-Ellison syndrome Courtesy of Dr. Per-Lennart Westesson, University of Roches-
ter Medical Center.
48. A 39-year-old woman presents to her primary
care physician with sporadic shooting pains
across the left side of her face. She has no his- (A) Acetaminophen
tory of migraine headaches. On further ques- (B) Corticosteroids
tioning she says that three years ago she expe- (C) Heparin
rienced several weeks of tingling in her right (D) Interferon
lower extremity, and as recently as last year, (E) Sumatriptan
Full-Length Exams

she was unable to drive her car because of bi-


lateral arm weakness. The physician proceeds
with a full neurologic work-up, including MRI
of the head (see image). Given the patient’s
likely diagnosis, what is the best long-term
therapy?
Test Block 3
Test Block 3  •  Answers 577

An s w e r s

1. The correct answer is E. The most likely di- Answer A is incorrect. Aortic aneurysms form
agnosis is croup, which is most commonly in approximately 10% of patients with tertiary
caused by parainfluenza virus. Croup is an in- syphilis. The cardiac involvement typical of
fection of the upper airway causing narrowing disseminated Lyme disease is atrioventricular
that leads to inspiratory wheezing and a bark- nodal block, myocarditis, or left ventricular
ing cough. Most cases of croup require only function.
supportive therapy as treatment. Severe cases
Answer B is incorrect. Argyll Robertson pupil
may require supplemental oxygen, corticoste-
is characteristic of tertiary syphilis. As a result
roids, and epinephrine. While parainfluenza is
of a midbrain lesion, the pupil constricts dur-
the most common agent responsible for croup,
ing accommodation but not in response to
it can also be caused by influenza, respiratory
light.
syncytial virus, and measles.
Answer C is incorrect. Autoimmune polyar-
Answer A is incorrect. Amantadine is an an-
thritis does occur in untreated Lyme disease.
tiviral that has a narrow spectrum and is used
However, this phenomenon occurs months to
to treat influenza type A. While influenza can
years after the initial presentation of the dis-
cause croup, it is not the most common cause
ease (late stage).
of this disease.
Answer E is incorrect. Opportunistic infection
Answer B is incorrect. Bronchoalveolar lavage
with Pneumocystis jiroveci is found in immu-
is used to sample the lower respiratory tract in
nocompromised patients, such as those in the
severe pneumonia, in the diagnosis of a lung
later stages of HIV infection.
tumor, and in the assessment of fibrosing al-
veolitis, among other indications. 3. The correct answer is D. Osmotic demyeli­
Answer C is incorrect. Admission to the emer- nation, also known as central pontine myeli­
nolysis, can result from overaggressive treat-

Full-Length Exams
gency department may be called for if the
child is in acute respiratory distress. ment of hyponatremia. As hyponatremia
develops, the brain prevents cerebral edema
Answer D is incorrect. Penicillin can be used
by gradually reducing its own osmolarity, thus
to treat streptococcal pharyngitis, which pre­
reducing the osmotic gradient that would oth-
sents with red, swollen tonsils and pharynx and
erwise force water intracellularly. The brain
a high fever.
can gradually replace these lost osmoles as the
serum osmolarity is corrected, but correction
2. The correct answer is D. This patient presents
of the serum sodium level at a rate faster than
in the early, localized stage (stage 1) of Lyme
about 1 mEq/L/hr outpaces the brain’s ability
disease, caused by infection with the spiro-
to compensate, resulting in neuronal shrink-
chete Borrelia burgdorferi. This organism is
age and death. The clinical manifestations oc-
carried by several species of the Ixodes tick and
cur several days later and include dysarthria,
is common in the northeastern United States.
dysphagia, and flaccid quadriparesis that can
This first stage is characterized by a flu-like
become spastic and may progress to a “locked-
illness and the erythema migrans rash, which
in” syndrome, in which the patient retains full
classically spreads over time and develops a
awareness but can move only the extraocular
central clearing. The second stage (early dis-
muscles.
Test Block 3

seminated disease) targets four organ systems;


skin, central nervous system, heart, and joints. Answer A is incorrect. Cerebral edema occurs
Bilateral cranial nerve VII palsy (Bell palsy) is with acute hyponatremia as water flows freely
a common early effect of B burgdorferi infec- across both the blood-brain barrier and cell
tion. membranes and into brain cells to compen-
578 Section III: Full-Length Examinations  •  Answers

sate for the drop in serum sodium. However, Answer C is incorrect. Calcium intake should
cerebral edema does not typically accompany be increased in this patient to above-normal
overly aggressive treatment of hyponatremia levels. Decreased intake, however, is not the
with hypertonic saline, but rather the opposite, pathogenesis of his disease.
as cell shrinkage and death occur as a result of
Answer D is incorrect. Ergocalciferol, vitamin
water leaving the cells.
D2, is found in plants. Patients with renal os-
Answer B is incorrect. Diffuse axonal injury teodystrophy can have normal or even elevated
occurs in the setting of central nervous system vitamin D intake.
trauma or angular acceleration or both and re-
Answer E is incorrect. Cholecalciferol, or vita-
sults in disruption of the axon at the nodes of
min D3, is found in meats. It is also produced
Ranvier. Diffuse axonal injury does not occur
in humans in the dermis by ultraviolet light
with electrolyte abnormalities.
exposure. Patients with renal osteodystrophy
Answer C is incorrect. Intracerebral hemor- can have normal or even elevated vitamin D
rhage can occur as a result of hypertension, intake.
arteriovenous malformations, anticoagulation,
thrombolysis, or amyloid angiopathy; however, 5. The correct answer is E. The description of
it does not occur as a result of hyponatremia or colonic inflammation with exudates and ne-
the associated treatment. crosis of the mucosal surface describes the
pseudomembranous colitis of Clostridium dif-
Answer E is incorrect. Uncal herniation can
ficile, of which there have been several out-
result only from focal processes within the cra-
breaks. C difficile is a gram-positive anaerobe
nial vault, such as intracranial hemorrhage,
spore-former that produces toxin A (which
but does not occur with diffuse processes asso-
causes diarrhea) and toxin B (which is cy-
ciated with electrolyte abnormalities.
totoxic). Strains that produce an increased
amount of these toxins have led to increased
4. The correct answer is A. This man has renal
morbidity and even mortality associated with
osteodystrophy, a common complication of
C difficile colitis.
Full-Length Exams

chronic renal insufficiency. In these patients,


decreased conversion of 25-hydroxyvitamin Answer A is incorrect. Salmonella is an exam-
D to the active 1,25-dihydroxycholecalciferol ple of a gram-negative facultative intracellu-
in kidney cells leads to decreased calcium ab- lar organism and could produce diarrhea, but
sorption and thus a low serum calcium level. only C difficile produces toxin A and toxin B.
Secretion of parathyroid hormone increases
Answer B is incorrect. Gram-negative lactose
to counteract the low calcium levels by in-
fermenters that can cause diarrhea include
creasing bone resorption. The hyperparathy-
Escherichia coli, but not C difficile.
roid state also decreases kidney reabsorption
of phosphate. Renal osteodystrophy is treated Answer C is incorrect. Gram-negative lactose
with calcium, phosphate binders, and calcitriol nonfermenters that can cause diarrhea include
(synthetic vitamin D3) supplementation. Shigella and Salmonella, but not C difficile.
Answer B is incorrect. Active vitamin D is Answer D is incorrect. C difficile is an obli-
1,25-dihydroxycholecalciferol (1,25-OH-D) gate anaerobe; the other gram-positive organ-
produced from two sequential hydroxylation isms are aerobic or facultative anaerobes.
reactions of vitamin D. The intermediate is
25-hydroxyvitamin D (25-OH-D), which is the 6. The correct answer is C. Henoch-Schönlein
Test Block 3

storage form of vitamin D. The production of purpura (HSP) is a systemic vasculitis caused
active 1,25-OH-D would be reduced in this by the deposition of IgA immune complexes.
patient, but production of 25-OH-D would be It often follows a respiratory infection, and
unaffected. is characterized by the triad of purpura, ab-
dominal pain, and glomerulonephritis. Some
Test Block 3  •  Answers 579

children also present with arthritis in major Answer B is incorrect. This patient is acidotic,
joints. The rash of HSP is usually described as with a pH below the normal range of 7.35-
palpable purpura on the buttocks and legs. It 7.45. Her bicarbonate level is low, so this is a
is the most common small-vessel vasculitis in metabolic acidosis. Next we look to see if she
children, and rarely affects adults. The disease has appropriate respiratory compensation us-
is self- limiting and treatment is supportive. An ing Winter’s formula: partial pressure of car-
older adult presenting with the same symp- bon dioxide = 1.5 (HCO3-) + 8 (±2). The
tomatology is more likely to have a vasculitis expected partial pressure of carbon dioxide
associated with antineutrophil cytoplasmic an- would therefore be 16.5 mm Hg at the lowest,
tibodies. but the patient’s level is 15 mm Hg, telling us
that there is a simultaneous respiratory alkalo-
Answer A is incorrect. The metalloproteinase
sis. This picture of metabolic acidosis with res­
ADAMTS-13 normally cleaves von Willebrand
piratory alkalosis is seen with severe salicylate
factor (vWF) multidimers, which then enter
intoxication, which would explain the patient’s
the circulation and rest on the surface of en-
altered mental status.
dothelial cells. In thrombotic thrombocytope-
nic purpura (TTP), deficiency of ADAMTS-13 Answer C is incorrect. If the patient were ex-
leads to large vWF multidimers in the circu- periencing metabolic acidosis with respiratory
lation, where they bind platelets causing the compensation, given the bicarbonate level of 7
formation of platelet thrombi. TTP is charac- mEq/L, we would expect to see a partial pres-
terized by fever, thrombocytopenia, hemolytic sure of carbon dioxide of 16.5 mm Hg. How-
anemia, and renal and neurologic impairment. ever, the patient’s partial pressure of carbon
dioxide is 15 mm Hg, suggesting that there is a
Answer B is incorrect. IgA antibody deposi-
simultaneous respiratory alkalosis that is more
tion in the mesangium is the characteristic
than compensatory.
pattern of nephropathy associated with Berger
disease, which presents with hematuria and Answer D is incorrect. This patient is acidotic,
low-grade proteinuria during or a few days after and her bicarbonate level is low, so we know
an infection. this is a metabolic acidosis, not a respiratory

Full-Length Exams
acidosis or metabolic alkalosis.
Answer D is incorrect. IgG antibodies against
platelets is the pathophysiology of idiopathic Answer E is incorrect. The patient is experi-
thrombocytopenic purpura, which is charac- encing a respiratory alkalosis, but there is also
terized by thrombocytopenia leading to mu- a simultaneous metabolic acidosis.
cosal or skin bleeding, purpura or petechiae,
and epistaxis. In children it has an acute onset 8. The correct answer is E. The report of the
after a viral infection, whereas in adults it has kidney biopsy mentions a characteristic pat-
a gradual onset and often follows a viral infec- tern of linear antibody deposition, consistent
tion or the administration of a new drug (eg, with anti-GBM disease. Anti-GBM disease, or
sulfa drugs). Goodpasture syndrome, occurs when circulat-
ing auto-antibodies bind to type IV collagen
Answer E is incorrect. IgG antibodies depos-
in basement membranes in the lung and kid-
ited in the glomerular basement membrane is
ney, leading to nephritic kidney disease and
the etiology of Goodpasture syndrome, which
a restrictive lung disease. Sarcoidosis is char-
is a type II hypersensitivity reaction and pre­sents
acterized by immune-mediated noncaseating
with glomerulonephritis and pneumonitis.
granulomas and also produces restrictive lung
Test Block 3

pathology. Restrictive lung diseases are clas-


7. The correct answer is A. The patient is experi-
sified as pathologies that result in decreased
encing a metabolic acidosis, but there is also a
lung volumes with a relative preservation of
simultaneous respiratory alkalosis.
the FEV1/FVC ratio. In contrast, obstructive
lung diseases are characterized by expanded
580 Section III: Full-Length Examinations  •  Answers

lung volumes with a decrease in the FEV1/ Answer B is incorrect. Angiotensin II would
FVC ratio. cause vasoconstriction of B, while the prosta-
glandins would cause vasodilation of A. The
Answer A is incorrect. Asthma is a character-
overall result would be an increase in GFR
istic obstructive respiratory disease caused by
and a decrease in RPF.
airway hyperreactivity. Goodpasture syndrome
produces a restrictive lung disease. Answer D is incorrect. Angiotensin II would
cause vasoconstriction of B, while the prosta-
Answer B is incorrect. Emphysema is an ob-
glandins would cause vasodilation of A. The
structive respiratory disease caused by alveo-
overall result would be an increase in GFR
lar destruction and airway collapse, whereas
and a decrease in RPF.
Goodpasture syndrome produces a restrictive
lung disease. Answer E is incorrect. Angiotensin II would
cause vasoconstriction of B, while the prosta-
Answer C is incorrect. Kartagener syndrome
glandins would cause vasodilation of A. The
leads to bronchiectasis, a disease with obstruc-
overall result would be an increase in GFR
tive pathology due to immotile cilia and im-
and a decrease in RPF.
paired mucociliary clearance of particles from
the lung. In contrast, Goodpasture syndrome
10. The correct answer is D. The patient de-
produces a restrictive lung disease.
scribed above is probably under the influence
Answer D is incorrect. Pneumothorax results of a central nervous system (CNS) stimulant
in a decrease of total lung capacity. It is a re- such as methamphetamine. Labetalol is a non-
strictive pattern seen because of restriction of selective a- and b-antagonist that blocks many
lung expansion. However, there is not specifi- of the dangerous peripheral effects of CNS
cally destruction of lung parenchyma. stimulants, such as hypertension and cardiac
stimulation. Other appropriate medications
9. The correct answer is C. During hemor- that could be administered under these condi-
rhage, blood loss leads to an increase in the tions would be neuroleptic agents (to control
renin-angiotensin-aldosterone (RAA) system. the agitation and psychotic symptoms) and di-
Full-Length Exams

Angiotensin II has the effect of preferentially azepam (to control possible seizures).
constricting the efferent arteriole. Renal pros-
Answer A is incorrect. Atropine is a musca-
taglandins are produced in response to in-
rinic antagonist that would be appropriate
creased sympathetic activity and act to pref-
therapy for overdose of an acetylcholinesterase
erentially vasodilate afferent arterioles. The
inhibitor. A patient presenting with acetylcho-
overall result is an increase in GFR and a
linesterase inhibitor overdose would have mi-
decrease in RPF. It is notable that the reason
otic pupils and bradycardia.
angiotensin converting enzyme inhibitors are
protective for the kidney is that they prevent Answer B is incorrect. Flumazenil is a ben-
vasoconstriction of the efferent arterioles and zodiazepine receptor antagonist. It is used in
thereby prevent a decrease in RPF. The rea- cases of benzodiazepine overdose. The clini-
son that drugs blocking the formation of pros- cal features of acute benzodiazepine intoxica-
taglandins (ie, nonsteroidal anti-inflammatory tion include slurred speech, lack of coordina-
drugs) are damaging to the kidney is that they tion, unsteady gait, and impaired attention or
block vasodilation at the afferent arterioles and memory. A severe overdose may lead to stupor
thereby cause a decrease in both GFR and RPF. or coma.
Test Block 3

Answer A is incorrect. Angiotensin II would Answer C is incorrect. Fluoxetine is a selec-


cause vasoconstriction of B, while the prosta- tive serotonin reuptake inhibitor. It would not
glandins would cause vasodilation of A. The be helpful in a case of CNS stimulant over-
overall result would be an increase in GFR dose.
and a decrease in RPF.
Test Block 3  •  Answers 581

Answer E is incorrect. Naloxone is an opioid- to cause cancer. In inherited cases, the first
receptor antagonist that would be appropriate hit is carried in the germline. A second hit to
therapy for an opiate overdose such as with any retinoblast will result in cancer, making it
heroin or morphine. A patient who presents more likely that multiple tumors will occur. In
with opioid overdose would appear sleepy, le- sporadic cases, both hits have to occur in the
thargic, or comatose, depending on the degree same retinoblast. This is a rare event, therefore
of overdose. Pupils would be miotic, not myd- tumors are typically solitary and more often oc-
riatic. Blood pressure and heart rate are typi- cur later in life.
cally decreased, and respiration would be de-
Answer A is incorrect. p53, not Rb, plays an
pressed.
important regulatory role in apoptosis.
Answer F is incorrect. Physostigmine is an
Answer C is incorrect. The G2/M check-
acetylcholinesterase inhibitor that might be
point is another important cell-cycle regula-
used for an antimuscarinic drug overdose,
tory checkpoint. It provides another oppor-
such as with atropine, scopolamine, or Jimson
tunity to prevent the cell from undergoing
weed. An antimuscarinic overdose can look
mitosis should the environment be inappropri-
similar to a CNS stimulant overdose, but has
ate for cell replication or there is DNA dam-
one important exception. The hyperthermia
age. Some important regulatory proteins at
seen with an antimuscarinic overdose is ac-
the G2/M checkpoint include the CHK1 and
companied by hot and dry skin (due to block-
CHK2 kinases through interactions with G2-
ade of cholinergic receptors present on sweat
specific cyclin A and CDK2.
glands); however, stimulant overdose is associ-
ated with profuse sweating. Tachycardia, hy- Answer D is incorrect. The Rb protein does
pertension, hyperthermia, and mydriasis are not promote DNA damage repair. This func-
common to both. tion is often associated with p53, which is a
protein that like Rb, promotes cell-cycle arrest
11. The correct answer is B. This child most in the presence of DNA damage (or other fac-
likely has retinoblastoma, a rapidly progressive tors that do not constitute a favorable environ-

Full-Length Exams
neoplastic growth in the retina. Retinoblas- ment for successful cell replication). In addi-
toma may present in one eye, as in this patient, tion, it also initiates the apoptotic cascade in
or bilaterally, as in approximately 30% of cases. the presence of overwhelming DNA damage
The clinical vignette does not allude to any that cannot be repaired by the cell.
family history, in which case the retinoblas-
Answer E is incorrect. The Rb protein does
toma is called sporadic, in contrast with the fa-
not promote histone acetylation. In fact, the
milial form, which is associated with a family
Rb-E2F complex promotes histone deacety-
history. In either case, the disease is believed
lation on chromatin.
to arise from a loss-of-function mutation in
the RB1 gene. RB1 is a tumor suppressor gene 12. The correct answer is A. Anti-CD3 antibod-
that normally binds the E2F transcription fac- ies that bind to CD3 and trigger destruction of
tor complex in quiescent cells, which prevents T-lymphocytes (via phagocytes or complement-
the cell from progressing through the G1/S mediated lysis) would be most useful in this
checkpoint. Under appropriate conditions for scenario, as CD8+ T-lymphocytes are the
cell replication, E2F is released when the Rb main effectors mediating acute allograft rejec-
protein becomes phosphorylated by cyclins D tion. Note that monoclonal antibodies may be
and E, and their associated cyclin-dependant triggering, depleting, or blocking, and there-
Test Block 3

kinases (CDK 2, 4, and 6). Loss of function of fore it is absolutely necessary to characterize
the RB1 gene is associated with osteosarcoma which of these effector functions they elicit,
and retinoblastoma. According to the two-hit as those three scenarios would have three very
hypothesis, both copies of the RB1 gene in a different therapeutic applications.
single retinoblast must be mutated in order
582 Section III: Full-Length Examinations  •  Answers

Answer B is incorrect. Cytotoxic T-lympho- face. It is locally invasive but nonmetastatic.


cytes express CD8, while helper T-lympho- Histology is characterized by keratin pearls.
cytes express CD4. While targeting CD4
T-lymphocytes may be partially effective, it 14. The correct answer is A. This patient is pre-
would be more useful to deplete all T-lympho- senting with a ruptured ectopic pregnancy,
cytes with an anti-CD3 monoclonal antibody. which occurs when a blastocyst implants in
an inappropriate location, most commonly
Answer C is incorrect. CD14 is a common
the ampulla of the uterine tube. This typically
macrophage cell surface marker.
presents as described in the question stem and
Answer D is incorrect. CD16 is a common constitutes a medical emergency. The most
natural killer-cell surface marker. common risk factors are pelvic inflammatory
disease, prior appendicitis or endometriosis,
Answer E is incorrect. CD19 is a common
and previous abdominal surgery.
B-lymphocyte surface marker.
Answer B is incorrect. This describes appro-
13. The correct answer is B. The patient has a priate implantation of a blastocyst in a normal
likely diagnosis of basal cell carcinoma. It is pregnancy and is therefore not directly associ-
characterized by pink or flesh-colored pearly ated with pathology.
papules found in sun-exposed areas; the pap-
Answer C is incorrect. This describes the
ules are locally invasive but usually non-
likely etiology of acute appendicitis. This will
metastatic. Histology shows islands of tumor
present with right lower quadrant (RLQ) ab-
within mucinous dermis, as show in the im-
dominal pain but typically begins with diffuse
age. There is a purely basaloid population with
periumbilical pain that later migrates to the
minimal stromal response. Areas of palisading
RLQ. This condition is not associated with
nuclei, or small fusiform cells with little cy-
prior PID, a missed period, or elevated hCG
toplasm and hyperchromic dense nuclei, are
level, and will typically produce a fever.
characteristic of the disease.
Answer D is incorrect. This describes endo-
Answer A is incorrect. Actinic keratosis is a
Full-Length Exams

metriosis. Although this can be associated with


premalignant lesion characterized by small,
irregular bleeding and abdominal/pelvic pain,
rough erythematous or brownish papules. It is
it does not typically result in missed periods,
commonly found in sun-exposed areas and is a
shock-like signs, or elevated hCG level.
precursor to squamous cell carcinoma.
Answer E is incorrect. When two sperm fer-
Answer C is incorrect. Dermatitis herpetifor-
tilize a single ovum, a partial hydatidiform
mis is a dermatologic condition associated with
mole is formed. Like a ruptured ectopic preg-
celiac disease that is characterized by pruritic
nancy, this will produce vaginal bleeding and
papules and vesicles.
an elevated hCG level but will not cause acute
Answer D is incorrect. Melanoma commonly shock-like signs and is not associated with prior
presents with a dysplastic nevus that has un- PID. It will also cause a rapid increase in uter-
dergone malignant transformation. It is com- ine size.
monly found in sun-exposed areas. However,
histology would show tumor cells with large 15. The correct answer is E. The right posterior
nuclei located directly below the epidermis. cerebral artery (PCA) supplies the right occipi-
tal lobe, which is responsible for perceiving
Answer E is incorrect. Seborrheic keratosis is
the left lateral visual field in both eyes. The
Test Block 3

a benign, flat, pigmented squamous prolifera-


occipital pole is the extreme posterior end of
tion with keratin cysts.
the occipital lobe, which houses the fibers that
Answer F is incorrect. Squamous cell carci- originate from the macula. The macula is re-
noma commonly appears on the hands and sponsible for central vision. Thus PCA infarcts
that spare the occipital pole cause hemianopia
Test Block 3  •  Answers 583

(in this case, left homonymous hemianopia) Answer B is incorrect. Methotrexate inhibits
with sparing of central vision. dihydrofolate reductase in the S phase of the
cell cycle, causing decreased synthesis of pu-
Answer A is incorrect. A lesion in the optic
rines. Adverse effects include gastrointestinal
chiasm would cause bitemporal hemianopia,
(GI) irritation, mucositis, bone marrow sup-
not left homonymous hemianopia. The most
pression, and renal failure. Methotrexate is
common cause of optic chiasm lesions is a pi-
also used as an anti-inflammatory agent and to
tuitary adenoma.
induce abortion. It is not associated with car-
Answer B is incorrect. Infarction of the lower diotoxicity.
division of the middle cerebral artery would
Answer C is incorrect. Vincristine and vin-
cause defects of the temporal lobe, including
blastine inhibit microtubule formation and
the lower optic radiations (Meyer’s loop). Such
prevent assembly of the mitotic spindle. Ad-
lesions would cause contralateral superior
verse effects include neurotoxicity, fever, vom-
quad­rantanopia, or “pie in the sky” defects.
iting, and (with vinblastine) severe bone mar-
Answer C is incorrect. A left retinal artery oc- row supression. These agents are not associated
clusion with sparing of the vessels supplying with cardiotoxicity.
the macula could cause a hemianopia of the
Answer D is incorrect. 6-Mercaptopurine acts
left eye, with macular sparing, but “left ho­
as a false metabolite and blocks purine synthe-
monymous hemianopia” means that the left
sis. Adverse effects include myelosuppression,
visual field of both eyes is defective.
GI effects, and liver toxicity. 6-Mercaptopurine
Answer D is incorrect. Meyer’s loop refers to is not associated with cardiotoxicity.
the inferior division of optic radiations as they
Answer E is incorrect. Tamoxifen is a selective
pass through the temporal lobe. Meyer’s loop
estrogen receptor modulator used as an adju-
lesions are caused by middle cerebral artery
vant or preventative treatment for breast can-
infarcts. All PCA lesions should spare Meyer’s
cer. It acts as a partial agonist at the estrogen
loop. This answer choice does not support the
receptor. Adverse effects include endometrial
macular sparing seen in our patient, a phe-

Full-Length Exams
hyperplasia and carcinoma, increased bone
nomenon caused by sparing of the occipital
density, hypercoagulability, hot flashes, night
pole.
sweats, and vaginal discharge. Tamoxifen is not
16. The correct answer is A. This patient is most associated with cardiotoxicity.
likely taking doxorubicin, which is associated
17. The correct answer is A. The image shows a
with cardiotoxicity. Doxorubicin and daunoru-
genital herpes lesion, which is caused by the
bicin are DNA intercalators that act by binding
herpes simplex virus (HSV). The treatment for
to DNA and disrupting nucleic acid synthesis.
genital herpes is acyclovir, usually given orally.
The risk of heart failure is related to the cur-
Acyclovir is activated by viral thymidine kinase
rent dose and cumulative dose administered
to form a complex that inhibits viral DNA
to the patient. Patients can present in acute
polymerase. Intravenous (IV) acyclovir can
heart failure, with ECG changes, arrhythmias,
cause phlebitis, headache, nausea, neurotoxic-
pericarditis, or myocardial infarction. Chronic
ity, and renal toxicity. Neurotoxicity can mani-
use of these agents can lead to congestive heart
fest as lethargy, confusion, tremor, delirium,
failure, as seen in this patient, particularly a
or seizures. Kidney injury can take the form
dilated cardiomyopathy. Other adverse effects
of a crystalluria that can lead to obstructive
of doxorubicin include bone marrow suppres-
Test Block 3

nephropathy or interstitial nephritis. Lesions


sion, local skin irritation, and red urine. Other
of HSV infection consist of vesicles, pustules,
cardiotoxic chemotherapeutic agents include
and ulcers on an erythematous base. Patients
fluorouracil, busulfan, cisplatin, mitoxantrone,
complain of pain, itching, dysuria, and vaginal
and paclitaxel.
or urethral discharge. A primary episode is usu-
584 Section III: Full-Length Examinations  •  Answers

ally accompanied by fever, malaise, and my- Answer E is incorrect. Ribavirin is an antiviral
algias. Subsequent episodes resolve faster and agent used to treat chronic hepatitis C (HCV)
have fewer systemic symptoms. or respiratory syncytial virus infections. The
oral form is associated with hemolytic anemia
Answer B is incorrect. Fluconazole is an an-
and headache, and the aerosolized form is as-
tifungal agent that inhibits fungal steroid syn-
sociated with bronchospasm, rash, and con-
thesis. It is used to treat candidiasis and cryp-
junctival irritation.
tococcal meningitis and as prophylaxis against
fungal infections in immunocompromised pa-
18. The correct answer is D. This patient is pre-
tients. It can cause GI upset, alopecia, elevated
senting with acute mental status changes and
liver enzyme levels, and occasionally neurotox-
seizures secondary to severe hyponatremia. A
icity. Vulvovaginal candidiasis (a yeast infec-
hyponatremia of this severity in the presence
tion) is associated with erythematous labia with
of decreased serum osmolality is highly sug-
shallow ulcerations, and tiny papules (“satellite
gestive of the syndrome of inappropriate ADH
lesions”) beyond the main area of erythema.
secretion (SIADH). Given this patient’s his-
Patients complain of pruritus, dysuria, and dys-
tory of COPD, which is almost always second-
pareunia. The discharge classically associated
ary to an extensive history of smoking, and the
with vaginal candidiasis is thick and white.
presence of SIADH, this patient is most likely
Simple vulvovaginal candidiasis is not accom-
presenting with a paraneoplastic syndrome sec-
panied by systemic symptoms such as fever and
ondary to small cell lung cancer. This cancer
muscle aches.
has a high association with smoking, and is
Answer C is incorrect. Metronidazole is used known to present in particular with SIADH. A
to treat infections caused by protozoa and hint for the future: a sodium ion level in the
anaerobic bacteria, including Trichomonas 120s is almost always suggestive of SIADH.
vaginalis and bacterial vaginosis. It works by
Answer A is incorrect. While it is certainly
forming DNA-damaging metabolites in the
possible to become hyponatremic from exces-
bacterial cell. Adverse effects of metronidazole
sive fluid intake (polydipsia), one would not
include a disulfiram-like reaction with alco-
Full-Length Exams

expect to see a hyponatremia of this severity


hol ingestion, GI distress, headache, a metal-
purely from drinking too much fluid. In ad-
lic taste, and, rarely, neurotoxicity. Infection
dition, a patient who is purely drinking too
with T vaginalis presents with a malodorous,
much should have relatively appropriate com-
frothy green discharge, vaginal pruritus, and
pensatory mechanisms, such as the ability to
erythema. Bacterial vaginosis, which is usu-
suppress ADH secretion. This would lead to
ally due to Gardnerella vaginalis infection,
water loss from the kidneys to compensate, and
presents with perivaginal inflammation and
thus serum osmolality should not decrease.
irritation, dysuria, dyspareunia, and a grayish
This patient clearly has a pathologically low se-
discharge with a “fishy” odor. Neither illness is
rum osmolality, essentially ruling out polydip-
usually accompanied by systemic symptoms or
sia as the cause of this patient’s hyponatremia.
demarcated vaginal lesions as seen in the im-
age. Answer B is incorrect. A new-onset seizure in
an older adult always raises concern for a pri-
Answer D is incorrect. Penicillin is the treat-
mary brain process, especially tumor. While
ment of choice for syphilis. It is a b-lactam
primary brain tumors can certainly present
antibiotic and inhibits bacterial cell-wall syn-
with mental status changes or seizures, and
thesis. Adverse effects include hypersensitivity
can be associated with SIADH, this patient’s
Test Block 3

reactions and hemolytic anemia. A syphilitic


lung pathology and probable smoking history
chancre, indicative of primary syphilis, is usu-
place her at a much higher risk of lung cancer
ally a solitary, painless, 1- to 2-cm papule with
than for primary brain cancer. So while both
an indurated base. Primary infection is not ac-
are certainly possible, small cell lung cancer is
companied by systemic symptoms.
much more likely in this patient.
Test Block 3  •  Answers 585

Answer C is incorrect. Mental status changes phritis, is not associated with either Fanconi
and seizures always raise concern for pathol- syndrome or tetracycline. Renal papillary ne-
ogy occurring in the brain, in particular space- crosis can be caused by an overdose of analge-
occupying lesions such as tumors. Metastatic sics such as aspirin, phenacetin, and acetamin-
tumors in the brain are much more common ophen. Necrosis results from a combination of
than primary brain tumors. While this patient’s decreased blood flow to the kidney, consump-
age and smoking history place her at increased tion of antioxidants, and subsequent oxidative
risk for breast cancer, the paraneoplastic syn- damage.
dromes associated with breast cancer do not
include SIADH. This patient’s hyponatremia 20. The correct answer is B. Histrionic personal-
in the setting of COPD make small cell lung ity disorder is one of the cluster B personality
cancer far more likely. disorders (the “wild” group) that presents early
in adulthood. It is best characterized as a pat-
Answer E is incorrect. While squamous cell
tern of excessive emotionality and attention
lung cancer is clearly associated with smoking,
seeking, and it is often accompanied by so-
the paraneoplastic syndrome it presents with is
matoform disorders (somatization is a process
usually the secretion of a parathyroid-like pep-
by which an individual uses his or her body or
tide, which causes signs and symptoms relating
symptoms for a range of psychologic purposes
to the resultant hypercalcemia, such as fatigue,
and gains). An especially important character-
depression, muscle weakness, abdominal pain,
istic to remember about histrionic personal-
nausea, and constipation.
ity disorder is the often overtly sexual nature
of those affected. Relationships with physi-
19. The correct answer is E. Degraded tetracy-
cians in particular are affected by the patient’s
cline is associated with Fanconi syndrome, a
attention-seeking behavior, as is evident in this
disorder of proximal tubule function that re-
case by the patient reporting both sleep depri-
sults in severe loss of protein, glucose, and es-
vation and depression without objective signs
sential minerals (especially calcium and mag-
of either.
nesium). Tetracycline’s primary degradation

Full-Length Exams
product is anhydro-4-epitetracycline, which is Answer A is incorrect. Type 1 bipolar disor-
toxic and accumulates in the proximal tubules der is characterized by manic episodes (peri-
to cause Fanconi syndrome. Patients present ods of elevated or irritable mood that must last
with symptoms of polydipsia, polyuria, and de- at least one week) as well as depressive syn-
hydration due to excess loss of water and sol- dromes and mixed syndromes. Sleep of seven-
utes in their urine. nine hours per night is not usually consistent
with a manic syndrome.
Answer A is incorrect. Acute tubular necrosis
is typically associated with hypoperfusion and Answer C is incorrect. Obsessive-compulsive
is not associated with either Fanconi syndrome personality disorder is characterized by an ex-
or tetracycline. cessive preoccupation with control, order, and
perfectionism. This patient presents none of
Answer B is incorrect. Glomerulonephritis is
these characteristics.
most often caused by immune complex deposi-
tion and is not associated with either Fanconi Answer D is incorrect. Paranoid personality
syndrome or tetracycline. disorder is characterized by the inherent belief
that the world is a dangerous and threatening
Answer C is incorrect. Kidney stones, com-
place. Upon meeting these individuals they
monly caused by hypercalciuria or infection,
Test Block 3

often project strength and capability, and their


are not associated with either Fanconi syn-
distrust and suspiciousness of everyone is evi-
drome or tetracycline.
dent. These individuals tend to believe in vari-
Answer D is incorrect. Renal papillary necro- ous conspiracy theories.
sis, often caused by diabetes or acute pyelone-
586 Section III: Full-Length Examinations  •  Answers

Answer E is incorrect. Schizotypal personality directly target the anterior horns of the spinal
disorder is characterized by interpersonal awk- cord. Infection is prevented by vaccination.
wardness, odd beliefs or magical thinking, and
an eccentric appearance. 22. The correct answer is C. Sphingomyelinase
converts sphingomyelin to cerebroside. Defi-
21. The correct answer is E. This patient is ex- ciency of sphingomyelinase in Niemann-Pick
periencing neurosyphilis caused by Trepo- disease causes accumulation of sphingomyelin
nema pallidum. Doxycycline can be used in and cholesterol in parenchymal and reticulo-
the treatment of both syphilis and Chlamydia endothelial cells.
infection. When the dorsal columns are pro-
Answer A is incorrect. a-galactosidase A con-
gressively demyelinated and the posterior
verts ceramide trihexoside to lactosyl cerebro-
nerve roots are sclerosed, the condition is re-
side. This enzyme is deficient in Fabry disease.
ferred to as tabes (Latin for “shriveled”) dor-
salis. These patients have decreased reflexes, Answer B is incorrect. b-galactosidase con-
decreased pain sensation, and decreased pro- verts galactocerebroside to cerebroside. This
prioception. In this situation the patient’s chief enzyme is deficient in Krabbe disease.
complaint was ataxia associated with walking, Answer D is incorrect. b-glucocerebrosidase
which is caused by his lack of proprioception. converts glucocerebroside to cerebroside. This
The Romberg test is useful in this situation, enzyme is deficient in Gaucher disease.
because it can reveal a loss of proprioception
when visual input is removed. Generally, the Answer E is incorrect. Hexosaminidase A con-
pathophysiology of syphilis is based on oblit- verts ganglioside M2 to ganglioside M3. This
erative endarteritis; however, the precise cause enzyme is deficient in Tay-Sachs disease.
of tabes dorsalis is not fully understood. In the
spinal cord the disease is limited to the dorsal 23. The correct answer is E. A positive inotropic
columns. agent would increase the contractility of the
heart, causing both stroke volume and cardiac
Answer A is incorrect. HSV type 1 can cause output to increase at any given end-diastolic
Full-Length Exams

viral encephalitis. However, the swelling and volume. Therefore the curve would shift up
inflammation occur in the brain, not the spi- and to the left.
nal cord. Infection is treated with acyclovir.
Answer A is incorrect. The Starling curve
Answer B is incorrect. HIV can cause a my- does not change its shape in response to ino-
elopathy (usually in untreated patients with tropic agents; rather, it shifts to the left and up-
low T-lymphocyte counts) that can manifest ward.
as a painful or painless syndrome. This disease
mimics the neuropathy associated with co- Answer B is incorrect. A shift down and to the
balamin deficiency. Infection is treated with a left indicates a very low end-diastolic volume
cocktail of three anti-retroviral drugs. and cardiac output, which would occur in in-
stances of decreased blood volume such as in
Answer C is incorrect. Tuberculosis (TB) can severe hemorrhage.
invade the spinal column and cause bone de-
struction, it can compress various regions of Answer C is incorrect. A shift of the Starling
the spinal cord, or it can do both. This often curve down and to the right would indicate an
is called Pott disease, and the first symptom increasingly failing heart. A positive inotropic
usually is pain rather than ataxia. TB can be agent would affect the heart in the opposite
manner and ameliorate the effects of heart fail-
Test Block 3

treated with long-term courses of the antibiot-


ics isoniazid and rifampin. ure.

Answer D is incorrect. Poliovirus has a spe- Answer D is incorrect. The Starling curve
cific trophism to motor neurons, and its lesions shifts left, not right, as end-diastolic volume de-
creases.
Test Block 3  •  Answers 587

Answer F is incorrect. A shift up and to the Answer D is incorrect. Phenelzine is a mono-


right would indicate that there is an extremely amine oxidase (MAO) inhibitor. Seizure is
high cardiac output along with a high end- not a contraindication to its use. Tyramine cri-
diastolic volume. This would occur in the sis, a potentially lethal condition, may occur
presence of an extremely high blood volume, when patients taking MAO inhibitors ingest
not an increase in contractility. tyramine-rich foods (eg, certain cheeses and
wine). However, it is not contraindicated in pa-
24. The correct answer is A. Anorexia nervosa is tients who have seizures.
an eating disorder characterized by excessive
Answer E is incorrect. Sertraline is selective
dieting, excessive exercising, and body image
serotonin reuptake inhibitor. It is not contra-
disturbances. It is most common in adoles-
indicated in patients with seizures. Concomi-
cents and young adults. Physiologic conse-
tant use of a MAO inhibitor may precipitate
quences of the condition include severe weight
a dangerous complication known as serotonin
loss, amenorrhea, lanugo (downy body hair on
syndrome.
the trunk), melanosis coli (a blackened area
on the colon as a result of laxative abuse), an
25. The correct answer is B. Graft-versus-host dis-
increased risk of osteoporosis, mild anemia,
ease (GVHD) is an unwanted side effect of
leukopenia, and electrolyte disturbances. This
bone marrow transplantation whereby donor
patient’s amenorrhea and very low body weight
T-lymphocytes recognize the recipient as for-
indicate that she may be suffering from an-
eign and mount an immune response. The or-
orexia. She also has symptoms of depression:
gans most often affected are the gut, skin, and
sadness, anhedonia, feelings of guilt, and sui-
liver. Human leukocyte antigen matching of
cidal ideation. Although she may benefit from
the donor and recipient can help reduce the se-
therapy and an antidepressant medication,
verity of GVHD, but the disease may still occur
her electrolyte disturbances put her at risk for
due to a minor histocompatibility mismatch.
seizures, and contraindicate the use of bupro-
pion, which is an atypical antidepressant. Bu- Answer A is incorrect. Acute graft rejection is
propion functions as a norepinephrine and a potential side effect of solid organ transplant

Full-Length Exams
dopamine reuptake inhibitor, and unlike other and is mediated by the recipient’s cytotoxic
selective serotonin reuptake inhibitors, it does T-lymphocytes. The recipient of a bone mar-
not carry sexual adverse effects. row transplant undergoes myeloablative ther-
apy before transplant, and therefore it is not ex-
Answer B is incorrect. Buspirone is a sero-
pected that the patient would have significant
tonin 5-hydroxytryptamine1A-receptor partial
numbers of T-lymphocytes.
agonist used to treat depression and general-
ized anxiety disorder. This drug does not cause Answer C is incorrect. GVHD is mediated by
addiction or sedation, and does not interact donor T-lymphocytes; recipient T-lymphocytes
with alcohol. Seizure is not a contraindication are ablated before transplant.
against its use. Answer D is incorrect. Hyperacute graft re-
Answer C is incorrect. Haloperidol is a high- jection is a potential side effect of solid organ
potency, typical anti-psychotic drug. It is a do- transplant and is mediated by preformed re-
pamine receptor antagonist used for the treat- cipient antibodies. It occurs within minutes to
ment of schizophrenia. It is not indicated for hours post-transplant.
the treatment of depression. It is associated Answer E is incorrect. The clinical scenario
with a relatively high incidence of extrapyrami-
Test Block 3

described is more suggestive of GVHD than


dal symptoms, such as dystonia and ultimately recurrence of leukemia.
tardive dyskinesia. However, seizure is not a
contraindication against its use. 26. The correct answer is B. Enzyme-linked im-
munosorbent assay has a high false-positive
588 Section III: Full-Length Examinations  •  Answers

rate, and its result must be confirmed by West- Answer C is incorrect. Podophyllotoxins and
ern blot. The Western blot is a highly specific bleomycin are specific to the G2 phase of the
test with a very low false-positive rate, such that cell cycle.
a positive reaction for two of the three com-
Answer E is incorrect. Antimetabolites and
monly tested HIV antigens indicates disease.
podophyllotoxins are S-phase specific.
Positive and negative controls are commonly
used in the same batch to ensure that the assay 28. The correct answer is A. This patient is exhib-
was successful. The most appropriate next step iting classic symptoms of Sheehan syndrome,
is to establish a risk profile for this patient by or postpartum hypopituitarism. Although
obtaining the CD4+ count. Sheehan syndrome is thought to result from
Answer A is incorrect. Once a CD4+ count infarction of the pituitary gland from severe
is established and viral load obtained, this pa- bleeding and hypotension during delivery,
tient should begin appropriate prophylactic most patients do not experience hypotension
treatment for opportunistic infections such or severe blood loss in delivery. Patients ex-
as Pneumocystis jiroveci pneumonia. For anti- hibit signs of global hypopituitarism and often
retroviral treatment, a combination of highly present complaining of fatigue, anorexia, poor
active anti-retroviral therapy (also known as lactation, and loss of pubic and axillary hair.
HAART) is the appropriate approach, not AZT Treatment includes replacement of all defi-
monotherapy. cient hormones.
Answer C is incorrect. There is no reason to Answer B is incorrect. These lab values sug-
believe contamination has occurred. In fact, if gest primary hypothyroidism. Patients with
the positive control antibodies were mistakenly hypothyroidism can present with weight gain,
loaded in the patient’s sample lane, one would cold intolerance, weakness, myxedema and
expect that there would be three bands present fatigue. Primary hypothyroidism can result
in that lane. from iodine deficiency, surgical removal of
the thyroid gland, pharmacologic thyroid ab-
Answer D is incorrect. Both the positive and
lation, or autoimmune attack, as in Hashi-
Full-Length Exams

negative controls worked well, as three bands


moto thyroiditis.
are clearly present in the positive control lane,
and no bands are evident in the negative con- Answer C is incorrect. Primary hypothyroid-
trol lane. ism can present with these lab values. In-
creased thyrotropin-releasing hormone can
Answer E is incorrect. The Western blot is
cause elevated prolactin levels as well as el-
confirmatory for HIV.
evated levels of thyroid-stimulating hormone
(TSH). Patients can, therefore, present with
27. The correct answer is D. Vincristine and
galactorrhea in addition to normal symptoms
other vinca alkaloids block the polymerization
of hypothyroidism, such as weight gain, cold
of microtubules, thereby preventing the forma-
intolerance, weakness, myxedema, and fatigue.
tion of a mitotic spindle. The mitotic spindle
is necessary for mitosis; thus, vincristine is spe- Answer D is incorrect. In this set of lab val-
cific to the M phase. ues, serum ACTH is elevated while thyroxine
and TSH are normal, indicating Cushing syn-
Answer A is incorrect. Vincristine prevents
drome. Cushing can be caused by a primary
the formation of the mitotic spindle through
pituitary adenoma, an adrenal neoplasm, ec­
the blockage of microtubule polymerization.
topic ACTH production, or exogenous ad-
Test Block 3

Therefore, it is specific to the M phase.


ministration of corticosteroids. Patients with
Answer B is incorrect. Vincristine prevents Cushing syndrome present with hypertension,
the formation of the mitotic spindle through weight gain, moon facies, increased truncal
the blockage of microtubule polymerization. obesity, hyperglycemia, amenorrhea, immune
Therefore, it is specific to the M phase.
Test Block 3  •  Answers 589

suppression, and skin changes, such as skin Answer D is incorrect. Low IgM levels and
thinning and abdominal striae. normal T cell numbers are typical of Wiskott-
Aldrich syndrome, an X-linked defect associ-
Answer E is incorrect. In this set of lab val-
ated with elevated IgA levels, elevated IgE lev-
ues, thyroxine is elevated, while TSH is de-
els, normal IgG levels, and low IgM levels. It
creased. This represents the hormone levels
involves a defect in the body’s ability to mount
present in primary hyperthyroidism, such as
an IgM response to bacteria. Patients have a
Graves disease, which results from stimulation
normal number of T cells, but their T cells re-
of the thyroid gland by autoimmune antibod-
spond ineffectively to antigens. Recurrent pyo-
ies. Symptoms of hyperthyroidism include heat
genic infections, eczema, and thrombocytope-
intolerance, hyperactivity, weight loss, heart
nia are the typical symptoms. Wiscott-Aldrich
palpitations, diarrhea, increased reflexes, and
syndrome does not present with any specific
exophthalmia.
enzyme abnormality.
29. The correct answer is E. This patient has thy-
30. The correct answer is E. This patient demon-
mic aplasia (DiGeorge syndrome), in which
strates several characteristics classic for neuro-
the third and fourth pharyngeal pouches, and
fibromatosis type 1 (also known as von Reck-
thus the thymus and parathyroid glands, fail to
linghausen disease). Potential findings include
develop. This disease often presents with con-
café au lait spots, two or more neurofibromas,
genital defects such as cardiac abnormalities,
optic glioma, iris hamartomas (Lisch nodules),
cleft palate, and abnormal facies. Patients suf-
a positive family history (autosomal dominant
fer frequent viral and fungal infections because
inheritance), and a distinctive bony lesion
of T-cell deficiency.
such as sphenoid dysplasia or scoliosis. Patients
Answer A is incorrect. High IgM levels and with this disease generally demonstrate 95% of
normal T cell number are suggestive of hyper- the criteria by age 8 years and all of the criteria
IgM syndrome, in which B cells are unable by age 20. These patients are also at increased
to class switch because of a defect in helper risk for tumors. The gene is located on the
T cells. Patients have normal numbers of T long arm of chromosome 17.

Full-Length Exams
cells and high IgM levels; levels of IgA, IgE,
Answer A is incorrect. Bilateral acoustic neu-
and IgG are low.
romas are characteristic of neurofibromatosis
Answer B is incorrect. Hypogammaglobu- type 2. It is much less common than type 1
linemia with normal T cell number is char- and typically manifests with multiple central
acteristic of Bruton agammaglobulinemia, an nervous system tumors. The NF2 gene is lo-
X-linked defect in a tyrosine kinase that is nec- cated on chromosome 22.
essary for B cell maturation. After six months
Answer B is incorrect. Bilateral renal cell car-
of age, when the levels of maternal antibodies
cinoma occurs in von Hippel-Lindau disease,
have declined, patients with the disease tend to
an autosomal dominant disease that is charac-
present with recurrent bacterial infections.
terized by hemangioblastomas of the retina,
Answer C is incorrect. Hypogammaglobu- cerebellum, and medulla. About half of pa-
linemia with reduced T cell numbers is sug- tients develop bilateral renal cell carcinomas.
gestive of severe combined immunodeficiency The disease is associated with the deletion of
(SCID). SCID presents with recurrent viral, the VHL gene located on chromosome 3,
bacterial, fungal, and protozoal infections due which is a tumor suppressor gene.
to a total lack of cellular immunity second-
Test Block 3

Answer C is incorrect. Cystic medial necrosis


ary to a stem cell deficit in the bone marrow.
of the aorta leading to aortic insufficiency and
SCID does not present with the congenital de-
dissecting aortic aneurysm is associated with
fects described in the vignette.
Marfan syndrome, a connective tissue disorder
590 Section III: Full-Length Examinations  •  Answers

caused by the autosomal dominant inheritance In this disorder, widespread intravascular coag-
of a defective fibrillin gene. ulation consumes platelets and clotting factors,
resulting in lab findings indicative of a defi-
Answer D is incorrect. Leptomeningeal angi-
ciency in all elements of the clotting machin-
oma is associated with Sturge-Weber syndrome
ery. In DIC one would also see an increase in
(SWS), which is a rare congenital vascular dis-
fibrin split products and D-dimers.
order of unknown etiology affecting capillary-
sized blood vessels. Its characteristic features
32. The correct answer is B. In the lung, a1-anti-
include angiomas and a facial port-wine stain.
trypsin deficiency predisposes to chronic ob-
Only a small portion of patients with port-wine
structive pulmonary disease, specifically pan-
stains at birth have SWS.
acinar emphysema. Additionally, misfolded
gene products of a1-antitrypsin can be depos-
31. The correct answer is A. Hemophilia (types A
ited in the hepatocellular endoplasmic reticu-
and B) is an X-linked recessive disorder, with
lum. Therefore, patients with a1-antitrypsin
affected male individuals inheriting a defective
deficiency are at increased risk for developing
copy of the X chromosome from heterozygous
end-stage liver disease like cirrhosis.
(asymptomatic) mothers. It is caused by a de-
ficiency in factor VIII (hemophilia A) or fac- Answer A is incorrect. Cor pulmonale can
tor IX (hemophilia B) of the clotting cascade. present with dyspnea and is the result of dys-
Platelet number and bleeding time are normal function of the right ventricle caused by pul-
because there is no deficiency of platelet func- monary hypertension in diseases affecting the
tion. Prothrombin time (PT) measures activity lung or its vasculature. Patients with a1-anti-
of factors VII, X, V, prothrombin, and fibrino- trypsin deficiency are not known to have in-
gen, thus it is also normal in hemophilia. Par- creased risk of cor pulmonale.
tial thromboplastin time (PTT) measures activ-
Answer C is incorrect. Pseudomonas aerugi-
ity of factors VIII, IX, XI, and XII in addition to
nosa is commonly associated with nosocomial
factors X, V, prothrombin and fibrinogen. PTT
infections through contaminated ventilators
is therefore elevated in the case of factor VIII
or bronchoscopes. In the community, immu-
Full-Length Exams

or IX deficiency.
nocompromised patients (eg, HIV-positive
Answer B is incorrect. This profile describes and transplant patients) are most susceptible
qualitative platelet defects such as Bernard- to Pseudomonas infection. a1-antitrypsin de-
Soulier disease and Glanzmann thrombas- ficiency is not known to increase the risk of
thenia. Since there is no clotting factor defi- pseudomonal pneumonia.
ciency, PT and PPT are normal.
Answer D is incorrect. Renal cysts are associ-
Answer C is incorrect. This profile describes ated with inheritable renal conditions such as
von Willebrand disease, an autosomal domi- autosomal-dominant polycystic kidney disease,
nant disease. Von Willebrand factor promotes tuberous sclerosis, and Von Hippel-Lindau
platelet adhesion to damaged endopthelium, syndrome. a1-antitrypsin is not present in the
therefore its deficiency prolongs bleeding time. kidney and thus is not associated with develop-
It also serves as a carrier for factor VIII, so PTT ment of renal cysts.
is also prolonged in this disorder.
33. The correct answer is E. The disease de-
Answer D is incorrect. This profile describes
scribed in this patient is acute myelogenous
thrombocytopenia. Since there is no clotting
leukemia (AML). This condition is character-
factor deficiency, PT and PPT time are nor-
Test Block 3

ized by acute onset of myelosuppression and


mal. Since platelets are low, bleeding time is
the presence of increased myeloblasts in the
prolonged.
peripheral smear and bone marrow. One sub-
Answer E is incorrect. This profile describes type of AML is acute promyelocytic leukemia
disseminated intravascular coagulation (DIC). with abnormal presence of t(15;17), which en-
Test Block 3  •  Answers 591

codes for a fusion protein of the retinoic acid Answer A is incorrect. Cimetidine is an H2-­
receptor with the promyelocytic leukemia antagonist and is associated with headache,
gene. Auer rods, as shown in the image, are of- confusion, gynecomastia, thrombocytopenia,
ten present in this condition. and inhibition of the cytochrome P450 system.
It is not considered a teratogen and can be
Answer A is incorrect. In general, translo-
taken safely by pregnant women.
cations involving chromosome 14 occur in
B-cell lymphomas, as the locus for immuno- Answer B is incorrect. Magnesium hydroxide
globulin production is on chromosome 14. is an antacid that is not absorbed and does not
Translocation t(8:14) is associated with Burkitt exhibit any systemic adverse effects. It is asso-
lymphoma and induces overproduction of the ciated with diarrhea, but it is not considered a
c-myc oncogene. teratogen and can thus be taken safely by preg-
nant women.
Answer B is incorrect. Translocation t(9;22),
also known as the Philadelphia chromosome, Answer D is incorrect. Omeprazole is an ir-
which encodes the Bcr-Abl fusion protein, is reversible proton pump inhibitor. It is used to
found in more than 90% of cases of chronic treat hypersecretory states, recurrent ulcers,
myelogenous leukemia (CML). If this translo- gastroesophageal reflux disease, and stress-
cation is found in an acute leukemia, it is as- related gastritis. It is associated with headache
sociated with a poor prognosis. The Bcr-Abl and GI disturbances. It is not considered a ter­
fusion protein is a constitutively active tyrosine atogen and can be taken safely by pregnant
kinase that drives the cells to express a cancer- women.
ous phenotype.
Answer E is incorrect. Sucralfate is a polysac-
Answer C is incorrect. Translocation t(11;22), charide combined with aluminum hydroxide.
found in Ewing sarcoma of bone, results in It binds to ulcers, providing a physical protec-
production of the EWS transcription factor, tive barrier. It has been known to cause consti-
which induces the overexpression of various pation and nausea, but it is not considered a
oncogenes such as bcl-1. teratogen and can be taken safely by pregnant

Full-Length Exams
women.
Answer D is incorrect. Translocation t(11;14)
is associated with mantle cell lymphoma, a
35. The correct answer is E. This patient has
type of lymphoma with a very poor prognosis.
clinical evidence of multiple endocrine neo-
The translocation produces increased activity
plasia (MEN) type I, which can cause tumors
of cyclin D1, which causes rapid progression
in the “3 P’s”: the Pituitary gland, the Para-
of the cell cycle.
thyroid gland, and the Pancreas. The MEN I
syndrome follows an autosomal dominant
34. The correct answer is C. Misoprostol is a
pattern of inheritance, thus this patient’s fam-
prostaglandin E1 analog that can be used to
ily history of multiple endocrine organ abnor-
prevent ulcers produced by nonsteroidal anti-
malities further supports this diagnosis. In this
inflammatory drug use. It is also used as a
patient, parathyroid involvement is suggested
medical abortifacient in many countries, par-
by hypercalcemia; and a pituitary adenoma is
ticularly Latin American countries, and is
most likely causing his bitemporal hemianop-
therefore strictly contraindicated in pregnant
sia. This patient has signs and symptoms con-
women. Prostaglandins E1 (misoprostol) and
sistent with elevated levels of vasoactive intesti-
E2 have been successfully used to induce labor
nal peptide (VIP). VIP acts on the gut mucosa
by activating the dissolution of collagen bun-
Test Block 3

to promote Na+ secretion, causing a secretory


dles, increasing the submucosal water content
diarrhea. VIP also stimulates K+ secretion in
of the cervix and potentiating effects of endog-
the colon, causing hypokalemia, which can
enous oxytocin. As an antacid, misoprostol acts
lead to the muscle weakness, tetany, and even
on parietal cells to inhibit acid secretion and
periodic paralysis seen in this patient. Finally,
stimulate bicarbonate and mucus production.
592 Section III: Full-Length Examinations  •  Answers

VIP inhibits gastric acid secretion, leading to ing the period of haphazard bone formation,
hypochlorhydria, which can be tested by an bone-­specific alkaline phosphatase levels are
elevated pH on nasogastric suction fluid. The elevated. Rarely, the total alkaline phosphatase
majority of VIPomas arise within the pancreas level is normal while the bone-specific alkaline
and are one type of pancreatic tumor seen in phosphatase level is elevated. The level of el-
MEN I. evation of alkaline phosphatase rarely exceeds
10 times the upper limit of normal. The serum
Answer A is incorrect. Carcinoid tumors are
phosphate level stays normal, while the cal-
malignant neuroendocrine tumors that tend to
cium level may be normal or slightly elevated.
arise in the GI tract. These tumors may secrete
substances such as serotonin and, when they Answer A is incorrect. A value of 20 U/L is
metastasize, they may cause carcinoid syn- an abnormally low level of alkaline phospha-
drome (symptoms include bronchoconstric- tase. It would not be found in a patient who
tion, cutaneous flushing, diarrhea, and right- was forming large amounts of bone. The phos-
sided valvular heart disease). Carcinoid tumors phate level is within normal limits.
are not known to be associated with any of the
Answer B is incorrect. A value of 50 U/L is
MEN syndromes.
within the normal range of alkaline phospha-
Answer B is incorrect. Gastrinomas are non-b tase, and it is not likely to be found during the
islet cell tumors that commonly arise from the osteoblastic phase of the disease. The phos-
pancreas and secrete gastrin, leading to hyper- phate level is elevated; Paget disease is associ-
secretion of hydrochloric acid. Although gas- ated with a normal level of phosphate.
trinomas do cause diarrhea and are associated
Answer C is incorrect. A value of 100 U/L is
with MEN I, the pH of the nasogastric suction
within the normal range of alkaline phospha-
fluid would be decreased, not increased, in a
tase. It is not likely to be found during the os-
patient with a gastrinoma.
teoblastic phase of the disease. Furthermore,
Answer C is incorrect. Insulinomas are islet the phosphate level in this choice is elevated;
cell tumors that secrete insulin. These tumors Paget disease is associated with a normal level
Full-Length Exams

are associated with Whipple triad: hypoglyce- of phosphate.


mia, symptoms of hypoglycemia that include
Answer E is incorrect. A value of 1500 U/L is
mental status changes, and relief of symptoms
>10 times the normal level of alkaline phos-
upon glucose administration.
phatase, and it is not likely to be found, even
Answer D is incorrect. Pheochromocytoma in patients with highly active Paget disease.
is a tumor arising from the chromaffin cells Highly elevated alkaline phosphatase levels are
of the adrenal medulla. It is associated with typically found in patients with involvement
MEN types II and III (but not MEN type I), of the skull and at least one other site. In addi-
and its symptoms include episodic palpita- tion, this phosphate level is elevated, which is
tions, hypertension, and other adrenergic not typically seen in Paget disease.
symptoms.
37. The correct answer is C. The patient most
36. The correct answer is D. Paget disease of the likely has hyperprolactinemia. Increased secre-
bone is characterized by three stages: an initial tion of prolactin explains the lactation in the
osteolytic lesion involving marked bone resorp- absence of recent pregnancy and breastfeeding.
tion, a period of disorganized bone formation, Also, because prolactin inhibits gonadotropin-
and a final sclerotic or burned-out phase. The releasing hormone synthesis and release, the
Test Block 3

primary abnormality is the overproduction and patient is experiencing menstrual irregulari-


overactivity of osteoclasts, which are derived ties. Hyperprolactinemia commonly is caused
from the bone marrow. Alkaline phosphatase by tumors or anti-psychotic drugs. Anti-psy-
is a marker of bone formation, whereas hy- chotic agents are dopamine antagonists and
droxyproline signifies bone resorption. Dur- lead to increased secretion of prolactin (dopa-
Test Block 3  •  Answers 593

mine inhibits prolactin release). The patient exam findings suggestive of cirrhosis (jaundice,
has most likely been diagnosed with a psychiat- spider angiomata, ascites), the patient has de-
ric disorder such as schizophrenia that requires creased liver synthetic function (elevated PT,
dopamine suppression. low albumin). This patient also manifests signs
of hepatic encephalopathy, including asterixis
Answer A is incorrect. Premature menopause
(flapping tremor), confusion, and lethargy.
is associated with loss of ovarian function be-
The etiology of hepatic encephalopathy is not
fore the age of 40 years. The cause usually is
entirely understood, but it is thought that am-
idiopathic. Lab tests will reveal high levels of
monia acts as a toxin to the central nervous
follicle-stimulating hormone and luteinizing
system when it is not converted into urea by
hormone, with low levels of estrogen. How-
the cirrhotic liver. Lactulose, when digested
ever, with premature menopause we would not
by colonic bacteria, acidifies the colonic con-
expect to see high levels of prolactin and lacta-
tents. This acidification then converts ammonia
tion.
into a nonabsorbable protonated form. It also
Answer B is incorrect. Parkinson disease is a changes the bowel flora so that fewer ammonia-
movement disorder characterized by four car- forming organisms are present.
dinal signs: resting tremor, rigidity, bradykine-
Answer B is incorrect. Although a restricted
sia, and postural instability. Because the dis-
protein diet should be standard in all patients
order stems from a lack of dopamine (which
with end-stage liver disease, it will not acutely
functions in the basal ganglia to stimulate the
decrease ammonia concentrations.
motor cortex), one type of medication used
to treat Parkinson disease is the dopamine Answer C is incorrect. Although neomycin
agonist. However, dopamine downregulates can be used as an adjunct to decrease ammo-
prolactin. Thus, we would not expect to see nia production by gut flora, it is not first-line
effects of hyperprolactinemia if the patient therapy. Neomycin works by destroying the gut
were taking a medication to treat Parkinson flora that normally produce ammonia. Neomy-
disease. cin is more appropriate for prophylactic treat-
ment of hepatic encephalopathy.

Full-Length Exams
Answer D is incorrect. Turner syndrome is a
chromosomal disorder (45,XO). It frequently is Answer D is incorrect. Lorazepam, as well as
diagnosed via amniocentesis during pregnancy. other benzodiazepines that are metabolically
Typical physical features include a webbed cleared by the liver, should not be used and
neck, short stature, low-set ears, low hairline, can actually worsen the encephalopathy.
and lymphedema. Females with Turner syn-
Answer E is incorrect. Pentoxifylline is a
drome may be infertile and may have primary
methylated xanthine derivative that acts as a
amenorrhea, unlike this patient, who has men-
competitive, nonselective phosphodiesterase
strual irregularities (secondary amenorrhea).
inhibitor. Ultimately it decreased tumor necro-
Answer E is incorrect. Drugs for treating sis factor-a and leukotriene synthesis, and thus
type 2 diabetes include metformin, sulfonyl- reduces inflammation. It is useful in treating
ureas, glitazones, and a-glucosidase inhibitors. patients with advanced cirrhosis, as it can de-
None of these drugs would produce hyperpro- crease complications such as hepatic encepha-
lactinemia. Notably, sulfonylureas can cause lopathy and GI hemorrhage.
hypoglycemia. Metformin rarely causes lactic
acidosis. Glitazones may cause weight gain 39. The correct answer is E. Pneumocystis jiroveci
and hepatotoxicity. a-glucosidase inhibitors (formerly carinii), like most fungal infections,
Test Block 3

may cause GI disturbances. does not present with symptoms in the im-
munocompetent host. In children or patients
38. The correct answer is A. This patient has de- afflicted with HIV, cancer, or inherited im-
veloped cirrhosis of the liver from chronic mune deficiencies, P jiroveci can present with
HCV infection. In addition to the physical pneumonia. Symptoms begin suddenly. The
594 Section III: Full-Length Examinations  •  Answers

patient develops a fever and begins to cough score. Values of 7 or higher indicate survival
and breathe abnormally fast. Often the pa- is highly likely; values of 4 or lower indicate
tient’s lips, fingernails, and skin turn blue or greater mortality risk. This child scores one
gray because the patient has difficulty drawing point each for Appearance, Pulse, and Respi-
in air. On chest x-ray, the diffuse interstitial ration; he scores 0 points for Grimace (ie, no
pneumonia gives a ground-glass appearance. response to noxious stimuli) and Activity (ab-
P jiroveci infection is treated with trimethoprim- sence of muscle tone). Thus, his one-minute
sulfamethoxazole. APGAR score is 3.
Answer A is incorrect. Fluconazole or keto- Answer A is incorrect. This child has a higher
conazole is used for the treatment of local blas- APGAR score than 1.
tomycosis infections, and amphotericin B is
Answer C is incorrect. This APGAR score is
used for the treatment of systemic infections.
too high for this child.
Blastomycosis can present with flu-like symp-
toms, fevers, chills, productive cough, myalgia, Answer D is incorrect. This APGAR score is
arthralgia, and pleuritic chest pain. Some pa- too high for this child.
tients will fail to recover from an acute infec- Answer E is incorrect. This APGAR score is
tion and develop chronic pulmonary infection too high for this child.
or widespread disseminated infection.
Answer B is incorrect. Itraconazole or potas- 41. The correct answer is B. This patient has neu-
sium iodide is used for the treatment of Spo- rologic symptoms consistent with vitamin B12
rothrix schenckii infection. S schenckii is the (cobalamin) deficiency caused by demyelin-
cause of sporotrichosis. When S schenckii is ation of the dorsal columns, spinocerebellar
introduced into the skin, usually by a thorn tract, and lateral corticospinal tract. Pernicious
prick, it causes a local pustule or ulcer with anemia is a vitamin B12 deficiency associated
nodules along draining lymphatics (ascending with chronic atrophic gastritis. Autoantibod-
lymphangitis). S schenckii is a dimorphic fun- ies are directed against gastric parietal cells,
gus that has cigar-shaped budding yeast visible leading to an intrinsic factor deficiency. With-
Full-Length Exams

in pus. out intrinsic factor, vitamin B12 cannot be ab-


sorbed in the ileum. Patients may present with
Answer C is incorrect. Sulfadiazine and py- fatigue, dyspnea, and tachycardia. A peripheral
rimethamine are used to treat toxoplasmosis. blood smear will show macrocytic RBCs with
Toxoplasma gondii infection presents with hypersegmented polymorphonuclear leuko-
brain abscesses in HIV-positive patients and cytes, consistent with megaloblastic anemia (as
with birth defects. T gondii is transmitted via seen in the image). It is imperative to check
cysts in raw meat or cat feces. The definitive folate and vitamin B12 levels before beginning
stage (sexual stage) occurs in cats. Microscopi- treatment with vitamin B12 injections.
cally, acid-fast staining cysts are found.
Answer A is incorrect. Abnormal neural crest
Answer D is incorrect. Topical miconazole or cell migration leads to Hirschsprung disease,
selenium sulfide are treatments for Malassezia which is a congenital aganglionic motility dis-
furfur. M furfur is the cause of tinea versicolor. order affecting the large bowel. Patients pre­
Symptoms of this infection include hypopig- sent with obstructive symptoms such as con-
mented skin lesions that occur in hot and hu- stipation, abdominal distention, and bilious
mid conditions. emesis.
Test Block 3

40. The correct answer is B. The APGAR scoring Answer C is incorrect. The colon is not the
system evaluates newborns’ general health as site of vitamin B12 absorption, and bacterial
a measure of likely survival in the immediate overgrowth there, such as with Clostridium dif-
period surrounding birth. Five areas are evalu- ficile, will produce symptoms such as diarrhea,
ated on a scale of 0-2 to produce a 10-point flatulence, and weight loss.
Test Block 3  •  Answers 595

Answer D is incorrect. Green leafy vegetables ratory distress and no history of stasis, trauma,
contain folate, not vitamin B12. Folate is an es- or deep venous thrombosis. In this regard, the
sential cofactor in nucleic acid synthesis, and histologic finding of a pulmonary embolus is
its deficiency commonly leads to megaloblas- unlikely.
tic anemia as seen in the image. Therapy with
Answer D is incorrect. Marked intra-alveolar
folate should not be started until vitamin B12
fibrin and cellular debris is a histologic feature
deficiency is ruled out. However, folate defi-
of acute respiratory distress syndrome (ARDS).
ciency does not explain the neurologic symp-
There is nothing in this patient’s presentation
toms experienced by this patient
to suggest severe hypoxia or an etiology for
Answer E is incorrect. An embolus to the su- ARDS. Causes of ARDS are varied and in-
perior mesenteric artery can lead to an acute clude shock, sepsis, trauma, uremia, and acute
bowel infarction, a life-threatening problem. pancreatitis.
Patients typically present with abdominal pain,
Answer E is incorrect. This answer is a histo-
bloody stools, fever, and peritoneal signs. Ane-
logic description of coccidoidomycosis. The
mia in a patient with acute blood loss is typi-
patient has no history of an influenza-like ill-
cally a normocytic anemia (normal mean cor-
ness, arthralgias, or erythema nodosum (red,
puscular volume).
tender nodules on extensor surfaces). The pa-
tient has no known history of exposure and no
42. The correct answer is F. This answer choice
demonstrated positive skin test.
is a description of a ferruginous body, which
is consistent with asbestosis. The patient’s ca-
43. The correct answer is E. Acute intermittent
reer as a construction worker makes asbestos
porphyria (AIP) and porphyria cutanea tarda
exposure likely. Asbestosis results in a marked
(PCT) are the two most common porphyrias
predisposition to bronchogenic carcinoma,
seen clinically. AIP results from a defect in the
and specifically increases the risk of malignant
enzyme porphobilinogen (PBG) deaminase,
mesothelioma of the pleura or peritoneum.
also called uroporphyrinogen (URO) I syn-
Cigarette exposure, as in this patient, further
thase. PBG deaminase is the third enzyme in

Full-Length Exams
increases the risk of lung cancer.
the heme synthetic pathway, and its absence
Answer A is incorrect. This answer is a charac- leads to an aberrant accumulation of aminolev-
teristic description of the microscopic appear- ulinate (ALA) and PBG. In contrast, PCT is
ance of pulmonary TB. While the patient has a defect in the enzyme URO decarboxylase,
a history of a progressive cough, other nonspe- the fifth step in the heme pathway, which
cific symptoms of TB are not present, includ- causes an accumulation of URO but not ALA
ing symptoms of fever, hemoptysis, pleural ef- or PBG. In this case the patient presents with
fusions, night sweats, or generalized wasting. symptoms most consistent with AIP: neuro-
visceral symptoms (most commonly abdomi-
Answer B is incorrect. This answer is a histo-
nal pain, muscle weakness, and psychiatric
logic description of intracellular Birbeck gran-
manifestations such as anxiety, paranoia, and
ules, a feature of eosinophilic granuloma. This
depression) and high PBG levels in urine. In
interstitial lung disease is caused by a localized
severe cases the PBG can cause the urine to
proliferation of histiocytic cells that are closely
appear the color of port wine.
related to the Langerhans’ cells of the skin. An
eosinophilic granuloma does not share features Answer A is incorrect. ALA dehydratase, the
or a common etiology with bronchogenic car- enzyme responsible for condensing ALA into
Test Block 3

cinoma. PBG, is absent in ALA dehydratase porphyria.


Whereas this porphyria clinically resembles
Answer C is incorrect. This answer is a his-
AIP, deficiency of ALA dehydratase is ex-
tologic description of a pulmonary embolus.
tremely rare, making AIP a more likely diagno-
While many cancers produce a hypercoagula-
sis in this patient.
ble state, this patient has no symptoms of respi-
596 Section III: Full-Length Examinations  •  Answers

Answer B is incorrect. ALA synthase defi- Answer C is incorrect. One important func-
ciency is associated with X-linked sideroblas- tion of the small intestine is to absorb nutri-
tic anemia. ALA synthase is the first and rate- ents. If the image showed hypochromic, mi-
limiting step in heme synthesis. crocytic RBCs without WBC abnormalities, it
would be suggestive of iron-deficiency anemia.
Answer C is incorrect. Ferrochelatase incor-
Conversely, a macrocytic anemia would sug-
porates iron into protoheme, the last step of
gest folate or vitamin B12 deficiency. These
heme biosynthesis. Deficiency of ferrochela-
findings could be due to intestinal dysfunction.
tase results in erythropoietic porphyria, a disor-
der that usually begins with marked photosen- Answer D is incorrect. If this blood smear
sitivity in childhood. showed RBCs infected with parasites, we
could diagnose malaria. Diagnosis is made us-
Answer D is incorrect. Heme oxygenase cata-
ing both thin and thick blood smears because
lyzes the oxidation of heme to biliverdin.
the former reveals the species of Plasmodium,
Answer F is incorrect. PCT is seen frequently and the latter quantifies the percentage of
in the clinical setting. PCT results from a defi- RBCs that are infected (parasite density). One
ciency in URO decarboxylase. This deficiency important function of the RBC is to carry
may be inherited or acquired, the latter due to hemoglobin-bound oxygen to tissues. The loss
the inactivation of URO decarboxylase by iron, of functioning RBCs can lead to symptomatic
alcohol, estrogens, and infection with HCV anemia in patients with malaria.
or HIV. In contrast to AIP, patients with PCT
Answer E is incorrect. Loss of function of
present with photosensitivity, chronic, blister-
the spleen would show Howell-Jolly bodies
ing lesions on sun-exposed skin, and an ab-
(clusters of DNA) in RBCs on a blood smear.
sence of neuropsychiatric signs.
A functioning spleen usually removes these
Answer G is incorrect. URO III cosynthase protein aggregates. The spleen’s function is
deficiency can result in congenital erythropoi- similar to that of lymph nodes, the major dif-
etic porphyria, and is associated with hemo- ference being that the spleen is the major site
lytic anemia and photosensitive cutaneous le- of immune responses to blood-borne antigens,
Full-Length Exams

sions. whereas lymph nodes respond to antigens in


the lymph. The spleen is also important in the
44. The correct answer is A. This blood smear defense against encapsulated organisms. An
shows abundant lymphoblasts typical of acute example of loss of splenic function is seen in
lymphocytic leukemia (ALL). Lymphoblasts children with sickle cell disease who are at risk
can be distinguished from normal mature lym- for sepsis, meningitis, and pneumonia from
phocytes by their fine, homogenous chroma- encapsulated bacteria such as pneumococcus
tin, irregular nuclear borders, and scant cyto- and Haemophilus influenzae.
plasm. ALL is a neoplasm that originates from
a single B- or T-lymphocyte progenitor cell. 45. The correct answer is C. This patient has a
Blast cells proliferate and accumulate in the history consistent with uncontrolled type 2
marrow, crowding out other blood cell lines diabetes mellitus. Long-term hyperglycemia in
and resulting in suppression of hematopoiesis. these patients, reflected by the increased he-
Eventually, patients develop symptomatic ane- moglobin A1c, may result in diabetic nephrop-
mia, thrombocytopenia, and neutropenia. athy. The pathogenesis of diabetic nephropa-
Answer B is incorrect. If this child suffered thy involves non-enzymatic glycosylation of
the glomerular and tubule basement mem-
Test Block 3

from chronic kidney disease, she may become


anemic due to decreased erythropoietin secre- branes, thereby increasing permeability to pro-
tion from the kidneys. Her blood smear would teins; hence, microalbuminuria is an early sign
not correlate with that in the image. of diabetic nephropathy. Nonenzymatic glyco-
sylation is a ubiquitous process in poorly con-
trolled diabetes, and this patient’s neuropathic
Test Block 3  •  Answers 597

lower extremity pain likely is due to nonenzy- Answer B is incorrect. Imaging may be part
matic glycosylation of nerve fibers. Glomerular of the work-up of a child in cases of suspected
hypertrophy also occurs due to cytokine re- abuse, but in this situation the child’s scald
lease. On light microscopy, early changes show burns should be treated initially, prior to ob-
diffuse mesangial expansion in the glomeruli, taining imaging to look for further evidence of
whereas more advanced diabetic nephropathy abuse (eg, old, healed fractures). In addition,
(as might be seen in this patient) demonstrates the authorities need to be contacted.
nodular glomerulosclerosis (Kimmelstiel-
Answer C is incorrect. This answer choice
Wilson nodules). Nodular glomerulosclerosis
outlines appropriate, but incomplete, actions.
is characterized by increased cellularity and
The child needs to be treated for his wounds,
mesangial matrix deposition, as well as hyaline
and a full physical examination should be con-
masses and thickening of the lamina densa.
ducted to look for other signs of child abuse.
Diabetic nephropathy can present with either
In cases of suspected child abuse such as this
a nephrotic or a nephritic syndrome, although
one, the appropriate authorities must be con-
nephrotic is more common.
tacted. This answer is incorrect because it does
Answer A is incorrect. Diffuse capillary and not reflect the need to contact the appropriate
basement membrane thickening is associated personnel.
with membranous glomerulonephritis.
Answer E is incorrect. Given the inconsisten-
Answer B is incorrect. Enlarged hypercellular cies between the patient’s wounds and the his-
glomeruli with neutrophils can be found in tory provided, further investigation by authori-
acute poststreptococcal glomerulonephritis. ties is necessary before the child is released
back to his mother.
Answer D is incorrect. Segmental sclerosis
with hyalinosis is seen in focal segmental glo-
47. The correct answer is C. Intermittent head-
merulosclerosis.
aches, sweating, and palpitations in an other-
Answer E is incorrect. Glomeruli demonstrat- wise healthy man are suggestive of a pheochro-
ing a wire-loop appearance with subendothe- mocytoma, a catecholamine-secreting tumor

Full-Length Exams
lial basement membrane deposits are seen in most commonly found in the adrenal glands.
lupus nephropathy. Episodes are limited in duration, but blood
pressure during these events can reach dan-
46. The correct answer is D. The pattern and lo- gerously high levels. High urinary catechol-
cation of the child’s wounds are consistent amines, metanephrine, and vanillylmandelic
with the mother’s description. However, burns acid confirm the diagnosis. Pheochromocyto-
of the depth described in the question stem mas are associated with parathyroid tumors as
could only be caused by a much longer dura- part of the MEN type II syndrome, which also
tion of contact with hot water than the mother includes medullary carcinomas of the thyroid.
indicates. The physical findings suggest this
Answer A is incorrect. Hashimoto thyroiditis
child has been forcibly held in deeper, much
is an autoimmune disorder resulting in hypo-
hotter water, which suggests child abuse. Sus-
thyroidism. Antimicrosomal and antithyro-
pected child abuse requires further investiga-
globulin antibodies are present and diagnostic.
tion by authorities once immediate attention
During the initial phase of glandular injury, a
to wounds is provided.
transient state of hyperthyroidism may result
Answer A is incorrect. Collateral information from cellular rupture. Hashimoto thyroiditis is
Test Block 3

may be important in the investigation of the not associated with pheochromocytomas.


potential abuse of this child, but it should be
Answer B is incorrect. Insulinomas are associ-
obtained only after treating the child’s wounds
ated with parathyroid and pituitary tumors as
and contacting appropriate authorities.
part of the MEN type I complex. Insulinomas
are insulin-secreting tumors of the pancreas
598 Section III: Full-Length Examinations  •  Answers

that produce symptoms due to hypoglycemia. physical examination, laboratory tests such as
They are not associated with pheochromocyto- cerebrospinal fluid oligoclonal banding, and
mas. imaging findings is used to diagnose MS. In-
terferon beta-1a is indicated for the long-term
Answer D is incorrect. Prolacintomas are as-
treatment of patients with relapsing forms of
sociated with pancreatic tumors (Zollinger-
the disease to slow the accumulation of physi-
Ellison syndrome, insulinomas, and VIPomas)
cal disability and decrease the frequency of
and parathyroid tumors as part of the MEN
clinical exacerbations. Patients with MS in
type I syndrome. Prolactinomas cause exces-
whom interferon’s efficacy has been demon-
sive secretion of prolactin, resulting in second-
strated include patients who have experienced
ary amenorrhea in women and galactorrhea.
a first clinical episode and have MRI features
Prolactinomas are not associated with pheo-
consistent with MS.
chromocytomas.
Answer A is incorrect. The patient presents
Answer E is incorrect. Zollinger-Ellison syn-
with trigeminal neuralgia in the setting of sev-
drome is associated with parathyroid and pitu-
eral other past neurologic complaints. While
itary tumors as part of the MEN type I com-
trigeminal neuralgia is characterized by unilat-
plex. Zollinger-Ellison syndrome is caused by
eral shooting facial pains, it is important to dis-
a gastrin-secreting tumor (gastrinoma), result-
tinguish this pain from that of a headache (eg,
ing in recurrent upper GI ulcers that are re-
migraine, cluster, or tension). Acetaminophen
sistant to medical treatment. Zollinger-Ellison
may be considered in the initial treatment of
syndrome is not associated with pheochromo-
tension headache.
cytomas.
Answer B is incorrect. Short courses of IV cor-
48. The correct answer is D. Multiple sclerosis ticosteroids are commonly used to treat acute
(MS) is a chronic inflammatory demyelinat- MS flares associated with neurologic deficits.
ing disease of unknown etiology. It typically However, interferon or glatiramer, not cortico-
has a relapsing-remitting course and is most steroids, are appropriate for the long-term treat-
commonly seen in female patients with peak ment of MS.
Full-Length Exams

age of onset between 20 and 40 years. MS usu-


Answer C is incorrect. Heparin would be an
ally presents with weakness and/or numbness
appropriate therapeutic intervention in the
in one or more extremities. Another common
case of an ischemic stroke. Given that the pa-
presentation is visual loss secondary to optic
tient’s neurologic signs and symptoms are sep-
neuritis and unilateral shooting facial pain
arated both by time and space, a stroke is not
secondary to trigeminal neuralgia. MRI is the
likely. Moreover, the patient is quite young
most sensitive radiographic technique for im-
and an ischemic stroke in such a young patient
aging MS, with sensitivity of nearly 85%. Clas-
would be exceedingly rare.
sic findings on MRI are periventricular white
matter lesions known as “Dawson’s fingers,” Answer E is incorrect. Unilateral facial pain
as seen in the image. These lesions appear as is more characteristic of trigeminal neuralgia
“finger like” projections around the ventricles (a common feature of MS) than of migraine
and are easiest to see with a sagittal image. headache. Triptans are used as initial treat-
Commonly, foci identified on MRI imaging ment in the case of cluster and migraine head-
are clinically silent. A combination of history, ache.
Test Block 3
Test Block 4

599
600 Section III: Full-Length Examinations  •  Questions

Q u e st i o n s

1. A 6-year-old boy was brought to a pediatrician ager position. On physical examination, the
two weeks ago because his teacher noticed he physician finds fine hair growth on her face
had begun “blanking out” in the classroom, and extremities, with body mass index of 16.5
staring into space, failing to respond to his kg/m². Which of the following autoantibodies
name, and occasionally drooling. The pediatri- is responsible for this woman’s condition?
cian prescribed a drug for these episodes one
(A) Antimicrosomal antibodies
week ago. The boy’s parents bring him to the
(B) Thyrotropin receptor inhibitory antibodies
emergency department because he has wors-
(C) Thyrotropin receptor stimulatory antibod-
ening gastrointestinal upset, pain on swallow-
ies
ing, and blistering around his nose and mouth.
(D) Thyrotropin-releasing hormone receptor
Which drug most likely caused this boy’s
inhibitory antibodies
symptoms?
(E) Thyrotropin-releasing hormone receptor
(A) Carbamazepine stimulatory antibodies
(B) Ethosuximide
(C) Lamotrigine 4. A 59-year-old woman was recently admitted
(D) Phenytoin to the hospital because of oral ulcers and dif-
(E) Valproic acid fuse, crusted, erythematous plaques on her
torso and upper arms (see image). She has
2. A 30-year-old man presents with a seven- tested positive for anti-epithelial cell antibod-
month history of fatigue, weight loss, depres- ies. Which of the following is the most likely
sion, and abdominal pain worsened by eat- diagnosis?
ing. He recently experienced two episodes of
kidney stones, which he has never had before,
and reports decreased libido over the past cou-
Full-Length Exams

ple years. He was adopted at an early age, has


no children, and is unsure of his family history.
What is the most likely mode of inheritance of
this man’s condition?
(A) Autosomal dominant
(B) Autosomal recessive
(C) Mitochondrial inheritance
(D) X-linked dominant
(E) X-linked recessive

3.
A 29-year-old woman presents to the
obstetrician-gynecologist with complaints of
amenorrhea for the past two months. She Reproduced, with permission, from USMLERx.com.
notes that she is sexually active with her boy-
friend of six months, and they do not use any
(A) A disease associated with a type II hyper-
form of contraception. She is worried about
sensitivity reaction to type IV collagen
pregnancy, despite several negative home preg-
(B) A disease associated with a type IV hyper-
Test Block 4

nancy tests a few days ago. She also complains


sensitivity reaction to poison ivy
about feeling increasingly anxious, “hot all the
(C) A disease associated with anti-scl-70 auto­
time,” and weight loss, but she attributes these
antibodies
symptoms to increased stress at work, where
she was recently promoted to a project man-
Test Block 4  •  Questions 601

(D) A disease associated with linear deposits of 7. A peripheral T-helper lymphocyte engages
IgG in the epidermal basement membrane peptide-bound class II major histocompat-
(E) A disease associated with the HLA-B27 his- ibility complex molecules on the surface of an
tocompatibility complex allele antigen-presenting cell (APC). No other con-
tact is made between cell surface molecules
5. A 40-year-old woman develops bloody mucoid present on the T lymphocyte and the APC.
diarrhea with abdominal cramping after eat- Which of the following can be concluded
ing fresh fruit in a small Mexican village. Four about this peripheral T lymphocyte?
months later, she still has intermittent abdomi-
(A) The T lymphocyte will be activated and
nal pain. A CT scan of the abdomen shows
fully able to perform effector functions
evidence of a necrotic liver lesion. Her labora-
(B) The T lymphocyte will be activated but
tory studies show the following results:
unable to perform effector functions
Aspartate aminotransferase (AST), serum: 18 (C) The T lymphocyte will cause the APC to
U/L undergo apoptosis
Alanine aminotransferase (ALT), serum: 20 (D) The T lymphocyte will undergo anergy
U/L (E) The T lymphocyte will undergo clonal ex-
Alkaline phosphatase: 300 U/L pansion
Bilirubin, total serum: 1.1 mg/dL
Bilirubin, direct: 0.3 mg/dL 8. A 27-year-old man is brought to the emergency
department after he was found shuffling un-
Which of the following is the best treatment
steadily around a busy intersection for several
for this patient?
hours. The patient is unreliable in providing
(A) A course of ampicillin, gentamicin, and his medical history. His vital signs are within
clindamycin normal limits. On physical examination his
(B) A course of metronidazole liver edge is palpable 6 cm below the cos-
(C) Core needle biopsy of the hepatic lesion tal margin and truncal spider angiomata are
(D) Surgical excision of the hepatic lesion noted. Ophthalmologic examination reveals

Full-Length Exams
(E) Ultrasound-guided aspiration of the he- dark rings around the iris. His abdomen is soft
patic lesion and non-tender. Which of the following is the
most likely diagnosis?
6. A 46-year-old man comes to the clinic with a
cough that is occasionally productive of blood, (A) Drug intoxication
diffuse muscle and joint pain in the upper ex- (B) HIV infection
tremities, and blood in his urine for the past (C) Parkinson disease
several days. On further questioning the pa- (D) Schizophrenia
tient reveals that he has had chronic sinusitis (E) Wilson disease
for the past several years. Laboratory studies
9. A 22-year-old man with metastatic testicular
show a markedly elevated erythrocyte sedi-
carcinoma is undergoing treatment with high
mentation rate, and staining for antibodies to
doses of cisplatin and experiences severe nau-
cytoplasmic antigens of neutrophils is positive.
sea after each treatment. What class of drug
Which of the following is the most likely find-
could be added to his chemotherapeutic regi-
ing on kidney biopsy?
men to reduce his nausea?
(A) A split basement membrane
(A) 5-Hydroxytryptamine3 antagonist
(B) Normal glomeruli with foot process efface-
Test Block 4

(B) Anticholinergic
ment
(C) Benzodiazepine
(C) Segmental necrotizing glomerulonephritis
(D) Glucocorticoid receptor agonist
(D) Smooth linear staining on immunofluores-
(E) Histamine receptor antagonist
cence
(E) Wire-loop lesion with subepithelial deposits
602 Section III: Full-Length Examinations  •  Questions

10. An elderly woman with a history of atrial fi- (C) Demyelination of white matter tracts in
brillation is admitted to the hospital because the spinal cord
of congestive heart failure. Laboratory studies (D) Hyperreflexia and bilateral positive Babin-
show: ski reflexes
(E) Slower conduction velocity of action po-
Na+: 140 mEq/L
tentials in nerves myelinated by Schwann
K+: 4 mEq/L
cells
HCO3-: 25 mEq/L
Cl-: 104 mEq/L
12. A 58-year-old East Asian man with recently
Glucose: 100 mg/dL
diagnosed type 2 diabetes mellitus presents
Blood urea nitrogen: 18 mg/dL
to the emergency department because of the
Creatinine: 2.1 mg/dL
acute onset of flushing, tachycardia, nausea,
A complete blood count is within normal lim- and hyperventilation. The drug he recently
its. She develops the arrhythmia shown in the started taking to treat his diabetes works by
image. Which of the following antiarrhythmic stimulating the release of endogenous insulin.
agents is associated with the arrhythmia shown Which of the following drugs did the patient
in the image? recently start taking?
(A) Acarbose
(B) Glargine
(C) Metformin
(D) Tolbutamide
(E) Troglitazone

13. A 4-year-old boy is brought to his pediatri-


cian’s office by his mother because he has
been “acting out” for the past six months. The
Reproduced, with permission, from USMLERx.com.
child’s past medical history is unremarkable,
Full-Length Exams

and he has reached all of his developmental


(A) Adenosine milestones on time. The pediatrician inquires
(B) Esmolol about the parent’s disciplinary methods. The
(C) Lidocaine mother says that she and her husband “reason”
(D) Propranolol with the child when he misbehaves. According
(E) Quinidine to Piaget’s theory of cognitive development,
(F) Verapamil this 4-year-old patient is able to process infor-
mation in which of the following ways?
11. A patient presents two weeks after a severe up- (A) He can deal with abstract ideas
per respiratory infection complaining of in- (B) He can formulate and test hypotheses
creasing weakness in his legs. Strength is 2/5 in (C) He has some limited logical thought pro-
his bilateral lower extremities, 4/5 in his bilat- cesses
eral upper extremities. Sensation in the upper (D) He possesses an awareness of conservation
and lower extremities is intact. Which of the (E) He understands symbolic representations
following is a consequence of this disease pro- of the world
cess?
14. A 27-year-old woman is brought to the emer-
Test Block 4

(A) A descending paralysis beginning in the


proximal parts of the lower extremities gency department with an intense headache,
(B) Demyelination of the basal ganglia left-sided weakness, and blurred vision that
began after she was ejected from her car in a
Test Block 4  •  Questions 603

motor vehicle accident. Paramedics report (A) a-Galactosidase A


that she was ambulatory and cooperative at (B) b-Galactosidase
the scene of the crash but was unable to recall (C) Arylsulfatase A
the events leading up to the accident. While in (D) Hexosaminidase A
the ambulance, the patient had an episode of (E) Sphingomyelinase
projectile vomiting and has displayed steadily
deteriorating mental status. CT of the head 17. A 65-year-old African-American man pre­sents
shows extra-axial fluid collection on the right with increasing difficulty swallowing over the
side and a temporal bone fracture on the same past three months. Upper gastrointestinal en-
side. Injury to which of the following is the doscopy reveals an annular mass with ulcer-
most likely cause of the patient’s presentation? ation in the lower third of his esophageal wall,
similar to the gross specimen depicted in the
(A) Inferior cerebral veins
image. Histologic examination reveals in-
(B) Middle meningeal artery
creased mitosis, dysplasia, and nuclear atypia
(C) Posterior ethmoidal artery
of glandular columnar epithelial cells. Which
(D) Sigmoid sinus
of the following is a risk factor for this variant
(E) Superior sagittal sinus
of esophageal cancer, but not the other com-
mon morphologic variant of esophageal cancer
15. An infant with severe jaundice that is not cor-
commonly found in the upper to middle third
rected by phototherapy is in danger of develop-
of the esophagus?
ing kernicterus. This can occur in infants with
Crigler-Najjar syndrome, a genetic disorder in
which there is a near-complete deficiency of
glucuronyl transferase. Which of the follow-
ing laboratory findings would be expected in
blood tests in an infant with Crigler-Najjar syn-
drome?
(A) Decreased hematocrit

Full-Length Exams
(B) Decreased indirect bilirubin
(C) Increased direct bilirubin
(D) Increased indirect bilirubin
(E) Increased reticulocyte count

16. A 3-year-old girl is brought to her pediatrician Reproduced, with permission, from USMLERx.com.
because of a progressive loss of motor function
and a decline in her cognitive abilities. On
physical examination, it is noted that the pa- (A) Achalasia
tient has decreased deep tendon reflexes, trun- (B) African-American decent
cal ataxia, and a decreased attention span in (C) Male gender
comparison to the child’s last visit six months (D) Obesity
ago. The physician knows that her pathology (E) Tobacco use
is due to an abnormal accumulation of cere-
broside sulfate in her brain, peripheral nerves,
kidney, and liver. A deficiency of which of the
following enzymes leads to this condition?
Test Block 4
604 Section III: Full-Length Examinations  •  Questions

18. A 27-year-old man presents to the physician (A) Do not treat, because this patient’s labora-
with fever, malaise, cough, and wheezing. tory tests are most consistent with previous
During the physical examination, the patient hepatitis B vaccination
intermittently coughs up brownish-colored (B) Lamivudine and interferon-a
mucus. The rest of the physical examination is (C) Lamivudine and interferon-b
unremarkable. Mucus cultures are taken and (D) Ribavirin and lamivudine
the results are pending. An x-ray shows a mass (E) Ribavirin and pegylated interferon-a
within a cavity in his left lung. If the causative
agent is a fungus, which of the following will 21. A 40-year-old woman presents to her physi-
most likely be seen on microscopic observation cian with complaints of blood in her urine
of the mucus? and decreased urine output for the past week.
She had a sore throat three weeks ago, which
(A) A cyst with a thick cell wall and intracystic
has since resolved. Her temperature is 37°C
bodies
(98.6°F), heart rate is 70/min, and blood pres-
(B) A heavily encapsulated yeast that is not di-
sure is 147/93 mm Hg. Physical examina-
morphic
tion reveals bilateral pedal edema to the mid-
(C) A mold with irregular nonseptate hyphae
calf. In addition to several serologic tests, the
branching at wide angles (more than 90
patient undergoes renal biopsy (see image).
degrees)
Which of the following is the most accurate di-
(D) A mold with septate hyphae that branch at
agnosis?
a V-shaped 45-degree angle
(E) Cigar-shaped budding yeast visible in pus

19. A 65-year-old obese man with a 60-pack-year


smoking history presents with partial loss of vi-
sion after suffering a stroke. Physical examina-
tion reveals a bilateral defect in the upper left
visual quadrants. Where is this patient’s lesion
Full-Length Exams

most likely located?


(A) Dorsal optic radiation in the left parietal
lobe
(B) Dorsal optic radiation in the right parietal
lobe
(C) Left optic tract
(D) Meyer’s loop in the left temporal lobe
(E) Meyer’s loop in the right temporal lobe Reproduced, with permission, from USMLERx.com.
(F) Right optic tract

20. A 44-year-old woman with end-stage renal dis- (A) Diabetic nephropathy
ease and on hemodialysis presents to the physi- (B) Goodpasture syndrome
cian with abdominal pain. Laboratory tests are (C) IgA nephropathy
positive for hepatitis B surface antigen (HB- (D) Lupus nephritis
sAg), anti-hepatitis B core antibody (HBcAb), (E) Poststreptococcal glomerulonephritis
and hepatitis B early antigen (HBeAg), but (F) Rapidly progressive glomerulonephritis
negative for anti-hepatitis B surface and anti- (G) Renal amyloidosis
Test Block 4

hepatitis B early antibodies (anti-HBsAb and


anti-HBeAb, respectively). Which of the fol-
lowing is the appropriate treatment for this pa-
tient’s disease?
Test Block 4  •  Questions 605

22. A 38-year-old woman at 24 weeks’ gestation


undergoes prenatal ultrasonography. Results Normal 5′ TTCGUTCCGACT 3′

show a male fetus with bilateral hydrouretero-


Mutant 5′ TTCAUTCCGACT 3′
nephrosis. Soon after birth, laboratory values
show a moderately elevated creatinine. A void-
ing cystourethrogram reveals posterior urethral Reproduced, with permission, from USMLERx.com.
valves in the prostatic urethra. The etiology
of posterior urethral valves remains unclear,
but failure of regression of the developmental (A) Capping
structure that comprises the prostatic urethra (B) Hybridization
has been postulated. Anomalous development (C) Polyadenylation
of what structure may give rise to posterior ure- (D) Splicing
thral valves? (E) Transcription

(A) Genital tubercle 25. A 20-year-old college student presents com-


(B) Mesonephric duct plaining of severe fatigue and lethargy. She
(C) Paramesonephric duct experienced a recent mononucleosis infection
(D) Urogenital folds that has resolved. Physical examination shows
(E) Urogenital sinus scleral icterus, cervical lymphadenopathy,
and splenomegaly. The tips of her fingers are
23. An 8-year-old boy is brought to his pediatri- purple. Laboratory testing shows a decreased
cian for a school physical. His parents report hemoglobin level, an appropriate reticulocyte
that he is in good health. They are concerned, count, and a positive heterophile test. Which
however, because occasionally they find dark of the following is the most likely diagnosis?
brown urine in the toilet when their son forgets
to flush. Urinalysis is ordered; while his urine (A) Aplastic anemia
initially appears normal, it turns dark after (B) Disseminated intravascular coagulation
standing. Which of the following enzymes is (C) IgG-mediated (warm) hemolytic anemia

Full-Length Exams
defective in this child? (D) IgM-mediated (cold) hemolytic anemia
(E) Paroxysmal nocturnal hemoglobinuria
(A) Branched-chain a-ketoacid dehydrogenase
(B) Homogentisic acid oxidase 26. A new growth factor, XGEF, has been found
(C) Phenylalanine hydroxylase to be upregulated in breast carcinomas. This
(D) Phytanic acid oxidase growth factor appears to activate a seven-
(E) Tyrosinase transmembrane-domain cell surface receptor
on endothelial cells. The binding of the recep-
24. A 2-year-old child has required frequent trans- tor results in the increase of calcium in the cell
fusions throughout his life because of anemia. cytosol. By what mechanism does XGEF exert
A peripheral blood smear demonstrates micro- its effect on endothelial cells?
cytic, hypochromic red cells, with target cells
and anisopoikilocytosis. After further genetic (A) Activation of Gi
testing, the child’s spleen is removed to lessen (B) Activation of Gq
the need for transfusions. When his genetic (C) Activation of Gs
code is sequenced, the mutation in the image (D) Upregulation of cGMP
is seen at the 5’ end of one of his introns. This
change is most likely to affect which of the fol-
Test Block 4

lowing processes?
606 Section III: Full-Length Examinations  •  Questions

27. A 71-year-old Russian man comes to the phy- thyroid. A thyroid biopsy is significant for a
sician complaining of a four-month history lymphocytic and plasma cell infiltrate with
of fatigue, low-grade fevers, night sweats, and occasional germinal center formation. The
cough. He became extremely worried yes- follicles contain little colloid, and the follicu-
terday when he noticed blood in his sputum. lar epithelial lining shows enlarged epithelial
On physical examination, he is extremely cells containing acidophilic cytoplasm. Which
thin, with enlarged, nontender left-sided cer- of the following antibodies is most likely to be
vical lymph nodes. His x-ray film of the chest found in this patient?
is shown. The physician prescribes medication
(A) Anti-smooth muscle
not only for the patient, but also for those who
(B) Anti-U1 RNP
may have been in close contact with the pa-
(C) Anticentromere
tient. Which of the following is an important
(D) Antimicrosomal
adverse effect of the first-line medication for
(E) Antimitochondrial
the prevention of this disease?
29. A couple brings their 3-year-old daughter to
her doctor because she has not behaved nor-
mally in the past month. The physician notes
that the girl is quiet and less expressive than
she was at her well-child visit several months
ago. She was previously able to name four col-
ors, but she now stutters and recalls only one.
Furthermore, she responds to questions with
one- or two-word sentences, which is unchar-
acteristic of her usually articulate personality.
What other symptom might be expected in this
patient?
(A) Hand wringing
Full-Length Exams

(B) Hearing loss


Courtesy of the Centers for Disease Control and Prevention. (C) Hyperactivity
(D) Psychosis
(E) Repetitive behaviors
(A) A disulfiram-like reaction with alcohol (F) Tics
(B) Aplastic anemia
(C) Hepatotoxicity 30. The peripheral blood smear shown in this im-
(D) Inhibition of bone growth in children age is from a patient who has taken phenytoin
(E) Nephrotoxicity for five years for a seizure disorder. The patient
has a well-balanced diet and is taking no other
28. A 47-year-old woman presents to her physician medications. Which of the following condi-
complaining of weight gain, fatigue, and leth- tions most likely caused the abnormality seen
argy. Her appetite has decreased recently and in this image?
she has been constipated. Physical examina-
tion reveals an enlarged, symmetric, and firm
Test Block 4
Test Block 4  •  Questions 607

32. Tertiary syphilis may lead to cardiovascular


complications, resulting in aortic aneurysm
and death. Patients with this pathology may
have a cardiac abnormality that causes a high-
pitched diastolic murmur. This murmur is
caused by a valvular defect similar to the type
of defect found in which condition?
(A) Chronic rheumatic heart disease
(B) Congenital bicuspid aortic valve
(C) Congenital pulmonary stenosis
(D) Marfan syndrome

33. A 48-year-old woman with amenorrhea visits


Reproduced, with permission, from USMLERx.com. her doctor. Her last menstrual period was 12
months ago. Results of physical and pelvic ex-
aminations are normal. Blood test levels show:
(A) Decreased b-hemoglobin synthesis
Estrogen: 35 pg/mL (normal: 150-200 pg/mL)
(B) Folic acid deficiency
Follicle-stimulating hormone: 60 IU/L (nor-
(C) Iron deficiency
mal: <12 IU/L)
(D) Vitamin B12 deficiency
Luteinizing hormone: 40 IU/L (normal:
(E) Vitamin B6 deficiency
<12 IU/L)
31. A 28-year-old woman begins taking a drug to What is the most likely underlying physiologic
treat her schizophrenia. The patient misses her mechanism explaining the hormonal changes
subsequent psychiatric appointments and is seen in this woman?
lost to follow-up. Two weeks later she is admit-
(A) Decrease in negative feedback
ted to the hospital with fever, dyspnea, and a
(B) Decrease in positive feedback

Full-Length Exams
productive cough. Cultures are consistent with
(C) Down-regulation of nuclear receptors
Streptococcus pneumoniae. Laboratory tests
(D) Increase in negative feedback
show a WBC count of 3000/mm³ and an ab-
(E) Increase in positive feedback
solute neutrophil count of 1000/mm³ (normal:
(F) Up-regulation of nuclear receptors
<1500/mm³). The physician suspects a drug
interaction when he sees that the patient is not
34. A 3-week-old infant presents with failure to
taking other medication and is HIV negative.
thrive, poor feeding, and lethargy. A physi-
What is the prescribed drug’s main mecha-
cal examination reveals an enlarged liver and
nism of action?
jaundice. Laboratory analysis reveals an el-
(A) D1- and D2-receptor agonist evated blood galactitol level and increased
(B) D1- and D2-receptor antagonist urinary reducing substance. Which of the fol-
(C) GABA receptor agonist lowing could correctly describe the levels of
(D) GABA receptor antagonist intermediates of galactose metabolism in this
(E) N-methyl-D-aspartate receptor antagonist patient?
(A) Decreased galactose
(B) Decreased uridine diphosphoglucose
Test Block 4

(C) Elevated glucose-1-phosphate


(D) Increased galactose-1-phosphate
(E) Increased glycogen
608 Section III: Full-Length Examinations  •  Questions

35. A 25-year-old white woman of Ashkenazi Jew- be depressed. The patient states that his father,
ish descent presents to the physician with a who died at age 50 years, had a similar condi-
three-week history of lower abdominal cramps tion as a young man. Which of the following
and intermittent bloody stools two times per is the most likely location of this man’s brain
day. She has not been febrile and reports no lesion?
sick contacts, unusual food exposures, or travel
(A) Amygdala
history. She has an aunt with similar symp-
(B) Caudate nucleus
toms. Gross pathologic findings from a patient
(C) Lateral corticospinal tracts
with similar symptoms are shown in the image.
(D) Mammillary bodies
Her laboratory studies show the following re-
(E) Nucleus basalis of Meynert
sults:
(F) Substantia nigra
Hemoglobin: 13.0 g/dL
Hematocrit: 39% 37. To assess the efficacy of a new bronchodila-
WBC count: 6000/mm³ tor drug X, the peak expiratory flow rate of a
Platelet count: 200,000/mm³ population of asthmatics is measured during
Erythrocyte sedimentation rate: 35 mm/h an asthma attack under three conditions: af-
ter no intervention, after the administration of
Which of the following is the most appropriate
albuterol, or after the administration of drug
treatment for this patient’s condition?
X. The graph shows the mean peak flow and
95% confidence intervals for each condition.
Which is the best statement regarding the re-
sults of the study?

600
Mean peak expiratory flow (L/min)

500
Full-Length Exams

400

300

200

100
Reproduced, with permission, from USMLERx.com.
0 No intervention Albuterol Drug X

(A) Ciprofloxacin
(B) Emergent surgery Reproduced, with permission, from USMLERx.com.
(C) Infliximab
(D) Loperamide
(A) There is likely a significant difference be-
(E) Oral steroids
tween the mean peak expiratory flow rate
(F) Sulfasalazine
of albuterol versus drug X
(B) There is likely a significant difference be-
36. A 32-year-old man presents with progressive
Test Block 4

tween the mean peak expiratory flow rate


dementia and sudden, jerky, purposeless move-
of albuterol versus no intervention
ments. On evaluation the patient is noted to
(C) There is likely a significant difference be-
tween the mean peak expiratory flow rate
of drug X versus no intervention
Test Block 4  •  Questions 609

(D) There is not likely a significant difference birth were not complicated, and there is an
between the mean peak expiratory flow unremarkable family history and healthy sib-
rate of albuterol versus drug X because the lings, this child has had developmental delay
ratio of these two means is close to 1 since birth. On examination the child has nor-
(E) There is not likely a significant difference mal vital signs, coarse facial features, diffuse
between the means of any of the condi- joint stiffness, claw hand deformities, and ky-
tions because the confidence intervals are phoscoliosis. Muscle biopsy reveals numerous
too wide intracytoplasmic inclusions in cells. What is
the pathophysiology of this disease?
38. A 51-year-old man with HIV infection presents
(A) Congenital herpes infection
to the clinic with a four-month history of in-
(B) Congenital rubella infection
creasing cognitive decline characterized by in-
(C) Failure to cross-link collagen fibrils
creasing apathy and mental slowing. His medi-
(D) Improper protein trafficking
cal history is significant for several infections
(E) Mutations in the structural protein fibrillin
with Pneumocystis jiroveci (formerly carinii)
pneumonia, and a recent CD4+ cell count
41. A 35-year-old woman presents with a fever of
was 112/mm³. Physical examination is notable
38°C (100.8°F), night sweats, fatigue, and a
for impaired saccadic eye movements, diffuse
weight loss of 4.5 kg (10 lb) over the past six
hyperreflexia, frontal release signs, and dysdi-
months. CT scan demonstrates mediasti-
adochokinesia. Lumbar puncture reveals a to-
nal lymphadenopathy in multiple contigu-
tal protein level of 72 mg/dL and an elevated
ous nodes. A photomicrograph of the biopsy
IgG level. MRI of the brain shows global cere-
specimen of the nodes is shown in the image.
bral atrophy with multiple ill-defined areas of
Which of the following is the most likely diag-
signal hyperintensity on T2-weighted images.
nosis?
Which of the following is the most likely cause
of this patient’s cognitive decline?
(A) Central nervous system lymphoma

Full-Length Exams
(B) Cytomegalovirus encephalitis
(C) Disseminated Mycobacterium avium com-
plex infection
(D) HIV-associated dementia
(E) Toxoplasmosis

39. A 62-year-old man presents to his physician


complaining of a milky discharge from his
nipples. The physician determines that the pa-
tient is suffering from a prolactinoma. What is
the embryologic derivative of the region of the
pituitary gland responsible for prolactin secre-
tion? Reproduced, with permission, from USMLERx.com.
(A) Endoderm
(B) Mesoderm
(A) Acute myelogenous leukemia
(C) Neural crest cells
(B) Burkitt lymphoma
(D) Neuroectoderm
Test Block 4

(C) Lymphocyte-depletion Hodgkin disease


(E) Oral ectoderm
(D) Mixed-cellularity Hodgkin disease
(E) Nodular-sclerosing Hodgkin disease
40. An 8-week-old boy is brought to his pediatri-
cian because his mother notes abnormal limb
movements. Although both pregnancy and
610 Section III: Full-Length Examinations  •  Questions

42. A 22-year-old man presents to the emergency 44. A 63-year-old man with a history of malignant
department complaining of itching over his hypertension presents to the emergency de-
palms and soles and recent onset of diarrhea. partment with severe chest pain radiating to
The patient received an unrelated donor bone his back. An ECG is performed immediately
marrow transplant six weeks ago. He denies but does not reveal the typical patterns of a
eating different foods and has no allergies. On myocardial infarction. Unfortunately, the pa-
physical examination, a maculopapular rash is tient dies before further tests can be initiated.
present over his neck, shoulders, palms, and On autopsy, the pathologist will most likely de-
soles. Laboratory studies show: tect abnormalities in which area?
WBC count: 7000/mm³ (A) Aorta
Hemoglobin: 10.2 g/dL (B) Esophagus
Hematocrit: 32% (C) Left coronary artery
Platelet count: 125,000/mm³ (D) Left ventricular wall of the heart
Aspartate aminotransferase: 432 U/L (E) Pericardium
Alanine aminotransferase: 356 U/L
Alkaline phosphatase: 400 U/L 45. An obese 35-year-old woman presents to her
physician with a six-month history of amenor-
Which of the following is the most likely diag-
rhea. She is concerned particularly because
nosis?
she and her husband have been trying for
(A) ABO incompatibility more than one year to conceive, with no suc-
(B) Acute graft-versus-host disease cess. Results of physical examination are nor-
(C) Chronic graft-versus-host disease mal, except the patient has noticed darkly
(D) Graft rejection pigmented periareolar hair since she stopped
(E) Hyperacute rejection menstruating. Test results show:
Blood pressure is 128/88 mm Hg
43. After three days of flu-like symptoms, a patient
Luteinizing hormone level: 300 mIU/mL
is feeling unsteady on her feet and dizzy when
Follicle-stimulating hormone level: 5 mIU/mL
Full-Length Exams

she attempts to stand up. During her illness


Thyroid-stimulating hormone level: 0.7 μU/mL
she has eaten very little and has had frequent
emesis. Blood tests reveal an arterial pH of 7.5 Results of a urine pregnancy test are negative.
and a partial pressure of carbon dioxide of 53 An ultrasound shows enlarged ovaries bilater-
mm Hg. What is the most likely etiology of ally. Which of the following additional findings
this acid-base disturbance? is associated most commonly with her condi-
tion?
(A) Consumption of antacids
(B) Decreased hydrogen excretion in the distal (A) Anosmia
tubule (B) Galactorrhea
(C) Increased bicarbonate reabsorption by the (C) Hyperglycemia
proximal tubule (D) Polycystic kidneys
(D) Production of ADH (E) Weak pulses in the lower extremities de-
(E) Volume depletion and increased hydrogen posits
excretion in the distal tubule
Test Block 4
Test Block 4  •  Questions 611

46. A 4-year-old girl is brought to the emergency


department by ambulance late at night. Her
caretaker reports that the child has been un-
able to bear weight on her left leg and that
she fell down the stairs this morning. She was
brought to the emergency department two
other times in the past year, each time with
trauma after falling down at home. Her ex-
amination is mostly unremarkable, but there
are several bruises at varying stages of healing.
X-ray of the leg shows a spiral fracture of her
left femur. Which of the following is the ap-
propriate course of action? Reproduced, with permission, from USMLERx.com.

(A) Ask about family history of brittle bones


(B) Consult ophthalmology department for ur- (A) Appendix epididymis
gent retinal examination (B) Appendix testes
(C) Contact child protective services (C) Bulbourethral glands
(D) Request a private meeting with the family (D) Paradidymis
(E) Talk to the child about being more careful (E) Skene glands
at home
48. A 35-year-old woman presents to her physician
47. The image shows one anatomic variation that at the first sign of painful genital ulcerations.
can occur from abnormal development of this She requests a treatment to shorten the course
paired, embryonic, mesodermally derived or- of her illness. Which of the following medica-
gan. In the presence of müllerian inhibiting tions is her physician most likely to prescribe
factor, the embryonic organ giving rise to the as first-line treatment?
structure below will develop into which of the
(A) Acyclovir

Full-Length Exams
following?
(B) Didanosine
(C) Foscarnet
(D) Ganciclovir
(E) Zanamivir

Test Block 4
612 Section III: Full-Length Examinations  •  Answers

An s w e r s

1. The correct answer is B. Based on the de- Answer E is incorrect. Valproic acid also is
scription of this boy’s “episodes,” he is suffer- indicated for absence seizures, and adverse ef-
ing from absence seizures. Ethosuximide is fects include GI distress and a rare, but fatal,
the only drug that is both used to treat absence hepatotoxicity. It acts by elevating concentra-
seizures and associated with Stevens-Johnson tions of GABA in the brain. Valproic acid is
syndrome. Ethosuximide is an antiepileptic not associated with Stevens-Johnson syndrome.
indicated for absence seizures that works by
blocking T-type calcium channels. A rare but 2. The correct answer is A. Multiple endocrine
severe adverse effect is drug-induced Stevens- neoplasia type I (MEN type I) is an autosomal
Johnson syndrome. This is characterized by dominant disorder that typically involves tu-
extensive shedding of the epidermis, blistering mors of the Parathyroid glands, anterior Pitu-
of the nasal, oral, and genital mucosa as well itary, and Pancreatic islet cells (the three Ps).
as the conjunctivae. This results in complaints The patient’s complaints of fatigue, weight
of severe pain on swallowing and dehydration. loss, depression, and kidney stones are related
Erythema, palpable purpura, and epidermal to hypercalcemia secondary to increased para-
necrolysis also may ensue. Ethosuximide also thyroid hormone (PTH) secretion from a tu-
causes gastrointestinal (GI) disturbances, fa- mor of the parathyroid gland. MEN type I
tigue, dizziness, and, in rare cases, blood dys- can also involve the anterior pituitary, which
crasias. Stevens-Johnson syndrome is treated in this case manifests as a prolactinoma lead-
by stopping the offending agent, managing ing to decreased libido and infertility. Pan-
fluid balance, preventing secondary infections creatic neoplasias of MEN type I include in-
of the skin and possibly administering cortico- sulinomas, VIPomas, and Zollinger-Ellison
steroids, although this last measure is contro- syndrome, as in this patient, which involves
versial. increased gastrin secretion that may manifest
with multiple peptic ulcers producing abdomi-
Full-Length Exams

Answer A is incorrect. Carbamazepine is


nal pain that is worse after eating food.
used to treat partial and tonic-clonic seizures,
but not absence seizures. It acts by increas- Answer B is incorrect. MEN type I is inher-
ing sodium channel inactivation. Carbamaze- ited in an autosomal dominant fashion, not au-
pine can cause diplopia, induction of the cy- tosomal recessive. Important diseases to know
tochrome P450 system, blood dyscrasias, liver that are inherited in an autosomal recessive
toxicity, and Stevens-Johnson syndrome. fashion include cystic fibrosis, phenylketo­
nuria, thalassemias, sickle cell anemias, glyco-
Answer C is incorrect. Lamotrigine is used to
gen storage diseases, infant polycystic kidney
treat partial and tonic-clonic seizures, but not
disease, and hemochromatosis.
absence seizures. It acts by blocking voltage-
sensitive sodium channels. Lamotrigine can Answer C is incorrect. MEN type I is inher-
cause GI upset, dizziness, diplopia, amnesia, ited in an autosomal dominant fashion. Indi-
and Stevens-Johnson syndrome. viduals inherit all of their mitochondrial DNA
from their mother because unlike the sperm,
Answer D is incorrect. Phenytoin is used to
the ovum is large enough to house a multitude
treat partial and tonic-clonic seizures, but not
of copies of mitochondrial DNA. Thus, mito-
absence seizures. It acts by increasing sodium
chondrial diseases are passed only from mother
Test Block 4

channel inactivation. Phenytoin toxicity causes


to offspring.
nystagmus, diplopia, ataxia, gingival hyperpla-
sia, and hirsutism. Like ethosuximide, it is as- Answer D is incorrect. MEN type I is in-
sociated with Stevens-Johnson syndrome. herited in an autosomal dominant fashion.
X-linked dominant inheritance is rare; some
Test Block 4  •  Answers 613

examples include hypophosphatemic rickets heals without scarring. Oral mucosal lesions
and adrenomyeloneuropathy. are found in 10%-40% of patients with bullous
pemphigoid. The disease is characterized by
Answer E is incorrect. MEN type I is in-
IgG autoantibodies against the epidermal base-
herited in an autosomal dominant fashion.
ment membrane, and deposits of antibody or
X-linked recessive diseases include glucose-
complement are seen on electron microscopy.
6-phosphate dehydrogenase deficiency, Du­
chenne muscular dystrophy, hemophilia A and Answer A is incorrect. A type II hypersensitiv-
B, Fabry disease, and Hunter syndrome. ity reaction against collagen type IV is char-
acteristic of Goodpasture syndrome, which
3. The correct answer is C. Graves disease is classically presents with hemoptysis and hema-
caused by autoantibodies to the thyrotropin turia, not skin lesions.
(thyroid-stimulating hormone) receptor. The
Answer B is incorrect. Contact dermatitis is
autoantibodies activate the receptor, stimu-
a type IV hypersensitivity reaction that occurs
lating thyroid hormone synthesis and secre-
following exposure to allergens such as poi-
tion and growth of the thyroid gland. Hence,
son ivy and nickel. These can form blisters in
Graves disease most commonly presents with
the epidermis similar to the ones seen in the
the signs and symptoms of hyperthyroidism, in-
photo, but tend to be localized only to the area
cluding heat intolerance, sweating, weight loss,
that came into contact with the allergen. Fur-
tremor, anxiety, weakness, and diarrhea as well
thermore, patients would not test positive for
as exophthalmos and pretibial myxedema.
anti-epithelial antibodies.
Answer A is incorrect. Antimicrosomal anti-
Answer C is incorrect. Systemic scleroderma
bodies are present in >90% of patients with
is associated with anti-scl-70 autoantibodies.
Hashimoto thyroiditis and in 50%-80% patients
Patients with this disease present with thick-
with silent thyroiditis.
ened, indurated skin.
Answer B is incorrect. Thyrotropin (thyroid-
Answer E is incorrect. Several autoimmune
stimulating hormone) receptor inhibitory au-
diseases are associated with the HLA-B27 al-

Full-Length Exams
toantibodies lead to Hashimoto thyroiditis and
lele. These include Crohn disease and the
thus signs and symptoms consistent with hypo-
spondyloarthropathies, but not pemphigus vul-
thyroidism.
garis.
Answer D is incorrect. Graves disease does
not involve thyrotropin-releasing hormone re- 5. The correct answer is B. This patient has am-
ceptor inhibitory antibodies, but rather is a re- ebiasis due to infection with Entamoeba histo-
sult of stimulatory autoantibodies to the thyrot­ lytica. She has classic features of amebiasis, in-
ropin (thyroid-stimulating hormone) receptor. cluding bloody mucoid diarrhea, recent travel
to a developing country, and hepatic abscess.
Answer E is incorrect. Graves disease does
Amebic hepatic abscesses may show liver func-
not involve thyrotropin-releasing hormone re-
tion tests consistent with an infiltrative pattern
ceptor stimulatory antibodies, but rather is a re-
of liver injury (ie, increased alkaline phospha-
sult of stimulatory autoantibodies to the thyro-
tase with near-normal AST, ALT, and biliru-
tropin (thyroid-stimulating hormone) receptor.
bin). The treatment of hepatic abscess due to
4. The correct answer is D. This patient has bul- E histolytica infection consists of a course of
lous pemphigoid, a subepidermal blistering metronidazole, which usually leads to com-
plete resolution of pathology. Metronidazole
Test Block 4

disease that is most commonly seen in the el-


derly. Patients with bullous pemphigoid have is an antibiotic used for anaerobic infections
skin vesicles that are filled with a clear fluid such as Giardia, Entamoeba, and Tricho-
and may rupture, but not as easily as the ves- monas. Metronidazole is well tolerated but
icles of pemphigus vulgaris; usually the skin can cause adverse effects, including abdominal
614 Section III: Full-Length Examinations  •  Answers

discomfort, nausea, and a disulfiram-like effect maturia, but the hallmarks are ocular disorders
when combined with alcohol. and nerve deafness due to defective synthesis
of collagen type IV. Furthermore, C-ANCA is
Answer A is incorrect. Ampicillin, gentami-
not associated with Alport syndrome.
cin, and clindamycin are used for broad cov-
erage of GI infections with gram-negative rods. Answer B is incorrect. This finding on kidney
Gram-negative rod infections typically present biopsy is consistent with minimal change dis-
with GI symptoms in the absence of liver le- ease, which occurs most commonly in chil-
sions. dren and is associated with edema rather than
blood in the urine.
Answer C is incorrect. Core needle biopsy is a
technique that is rarely used in the workup of Answer D is incorrect. This finding on kidney
liver lesions. biopsy is consistent with Goodpasture syn-
drome. Goodpasture syndrome is a type II hy-
Answer D is incorrect. Surgical excision of he-
persensitivity reaction against collagen type IV,
patic lesions is the standard treatment for Echi-
causing the smooth linear staining on immu-
nococcus infection causing a hydatid cyst in
nofluorescence. While this disease does com-
the liver. Hydatidosis is often caused by eating
monly present with concurrent hemoptysis
contaminated meat. It presents as a granulom­
and hematuria, Goodpasture is not associated
atous liver lesion, not a necrotic liver lesion.
with C-ANCA nor is it characterized by ar-
Hydatidosis lesions are walled off and poten-
thralgias or sinusitis.
tially anaphylactic if they burst; thus, careful
surgical removal is first-line treatment. Answer E is incorrect. This finding on kidney
biopsy is associated with systemic lupus erythe-
Answer E is incorrect. Aspiration is the first-
matosus (SLE), specifically a membranous glo-
line technique for hepatic cysts and lesions
merulonephritis (Type V) pattern. While SLE
suspicious for cancer or other unknown pathol-
can present with varied symptoms, antinuclear
ogy. An infectious etiology is more likely than
antibodies and anti-double-stranded DNA anti-
cancer because the patient has diarrheal symp-
bodies are usually present and not C-ANCA, as
toms, a recent history of foreign travel, and
Full-Length Exams

in this patient.
crampy abdominal pain.
7. The correct answer is D. This is a phenome-
6. The correct answer is C. This patient has We-
non known as peripheral tolerance. It is an im-
gener granulomatosis, a disease characterized
portant factor because deletion of self-reactive
by necrotizing, granulomatous vasculitis affect-
T lymphocytes within the thymus (“central
ing several organs, most notably the upper res­
tolerance”) is not completely efficient at re-
piratory tract, lung, and kidney. The elevated
moving all self-reactive T lymphocytes. Thus,
erythrocyte sedimentation rate (ESR) also
one mechanism of peripheral tolerance is that
points toward a rheumatologic cause. Further-
of anergy: When a T lymphocyte receives the
more, elevated serum cytoplasmic antineutro-
first signal (peptide-major histocompatibility
phil cytoplasmic antibody (C-ANCA) is found
complex) but no second signal (costimulation,
in 90% of patients with Wegener granulom­
such as CD28-B7 ligation), that T lymphocyte
atosis and is highly specific for the disease.
undergoes a reprogramming known as anergy,
Blood in the urine indicates renal involve-
wherein it is subsequently made refractory to
ment, and Wegener granulomatosis is associ-
any future stimulation. Note that an anergic
ated with a segmental necrotizing glomerulo-
T lymphocyte cannot be activated later even if
nephritis, sometimes with crescents, on kidney
Test Block 4

costimulation is present.
biopsy.
Answer A is incorrect. For a T lymphocyte to
Answer A is incorrect. This finding on kid-
become activated and fully able to perform its
ney biopsy is consistent with Alport syndrome.
effector functions, it must receive a second or
Patients with Alport syndrome may have he­
costimulatory signal. Without a costimulatory
Test Block 4  •  Answers 615

signal, the T lymphocyte cannot be activated phadenopathy. The patient does not have any
and instead becomes anergic. of these symptoms.
Answer B is incorrect. A T lymphocyte that Answer C is incorrect. Parkinson disease tends
becomes anergic does not undergo activation. to affect older adults, with a mean age of onset
around 60 years. Given that Wilson disease is
Answer C is incorrect. Causing cells to un-
characterized partly by basal ganglia degenera-
dergo apoptosis is a function of an activated
tion, both diseases may present with a rather
T-cytotoxic lympocyte.
unsteady, shuffling gait. A more convincing
Answer E is incorrect. The T lymphocyte will scenario for Parkinson disease would involve
not clonally expand. Clonal expansion is more an older man with bradykinesia, masked facies,
typical of B lymphocytes and requires a costim- and resting tremor.
ulatory signal to first activate the lymphocyte.
Answer D is incorrect. Schizophrenia is the
8. The correct answer is E. The patient pre­ classic psychotic disorder with an onset that
sents with several classic features of Wilson closely correlates with this patient’s age (age of
disease. Wilson disease, a genetic condition onset: males, 15-25 years; females 20-30 years).
caused by an absolute decrease in the body’s In order to fulfill the diagnosis of schizophre-
production of a copper-binding protein called nia, the patient must exhibit positive (eg,
ceruloplasmin, is characterized by basal gan- hallucinations, delusions, and disorganized
glia degeneration (producing parkinsonian-like speech and behavior) and negative (eg, poverty
symptomatology), elevated plasma copper lev- of speech and avolition) symptoms. None of
els, Kaiser-Fleischer rings (corneal copper de- these symptoms is present in this patient.
posits), micronodular cirrhosis of the liver, and
9. The correct answer is A. The most effective
neuropsychiatric symptoms.
class of drug for the treatment of chemotherapy-
Answer A is incorrect. While the strange na- induced nausea is the serotonin 5-hydroxytryp-
ture in which the patient was initially found tamine3 antagonist, which includes ondanse-
in the street and his unwillingness to provide tron, dolasetron, and granisetron. These drugs

Full-Length Exams
his medical history puts drug intoxication on can cause mild fatigue, headache, constipa-
the differential, the presence of other signs and tion, urinary retention, and dizziness.
symptoms (eg, corneal deposits, angiomata,
Answer B is incorrect. The anticholinergic
and hepatomegaly) supports the diagnosis of
class of anti-emetics includes scopolamine.
a metabolic disorder. In addition, pupillary
These drugs are used for motion sickness, not
changes (ie, dilation and constriction) and an
for the treatment of chemotherapy-induced
abnormal respiratory rate would likely be pres-
nausea.
ent in the case of drug intoxication.
Answer C is incorrect. Benzodiazepines in-
Answer B is incorrect. The patient’s age puts
clude lorazepam and diazepam. They may be
him in the highest risk group for HIV infec-
effective in treatment of the anticipatory nau-
tion. Given the patient’s lack of cooperation
sea associated with chemotherapy, but they are
and the physician’s lack of knowledge regard-
not as effective as the 5-hydroxytryptamine3 an-
ing his sexual activity, HIV should always be
tagonists.
considered in a young patient presenting with
altered mental status. However, the variety of Answer D is incorrect. Glucocorticoids (such
other clinical findings supports a different diag- as dexamethasone) can suppress nausea, but
Test Block 4

nosis. In addition, most patients newly infected are not routinely used for this indication be-
with HIV present with a mononucleosis-like cause of their myriad other effects in the body
syndrome characterized by low-grade fever, (which may or may not be desirable).
myalgias, generalized rash, fatigue, and lym­
Answer E is incorrect. Histamine receptor an-
tagonists include diphenhydramine and pro-
616 Section III: Full-Length Examinations  •  Answers

methazine. They are used for acid reflux and 11. The correct answer is E. Guillain-Barré syn-
allergies. Promethazine also is prescribed as an drome (GBS), also known as acute inflamma-
anti-emetic, but usually in the postoperative tory demyelinating polyradiculopathy (radicu-
setting to combat post-narcotic nausea, or for lopathy means a disease of the spinal nerve
motion sickness. roots and spinal nerves), is characterized mor-
phologically by segmental demyelination of
10. The correct answer is E. The rhythm strip areas of peripheral nerves; inflammation of
shows torsades de pointes, a potentially fa- the axons themselves may also occur. Because
tal rapid ventricular rhythm. The rate is vari- myelin (produced by Schwann cells in the pe-
able between 250 and 350/min and is usually ripheral nervous system) acts physiologically
transient. Torsades de pointes can be caused as a nerve insulator to increase conduction ve-
by a low potassium level (this patient’s potas- locity, GBS causes slower conduction velocity
sium level is within normal limits), potassium of action potentials through peripheral nerves
channel blockers such as sotalol, congenital and spinal nerve roots. GBS is often associated
abnormalities such as long QT syndrome, and, with viral infection, so this patient’s recent up-
in this case, quinidine. Quinidine is notable per respiratory infection is suspicious for the
because induced torsades de pointes occurs at syndrome. With proper respiratory support, the
therapeutic or even subtherapeutic levels. In mortality rate has fallen from 25% in the past
addition, quinidine induces cinchonism, a syn- to 2%-5% in 2001. Most deaths are due to pa-
drome that includes headaches and tinnitus. ralysis, autonomic instability, or cardiac arrest.
Answer A is incorrect. Adenosine is not known Answer A is incorrect. The clinical hall-
to cause torsades de pointes. Adenosine can mark of GBS is an ascending motor paraly-
cause flushing, dyspnea, and chest pain, as sis, which begins in the distal extremities and
well as sinus bradycardia and ventricular ec- spreads proximally. The most common causes
topy following paroxysmal supraventricular of death from this disease are respiratory paral-
tachycardia conversion. ysis, complications of tracheostomy, and auto-
Answer B is incorrect. Esmolol, a b-blocker, nomic instability.
Full-Length Exams

is not known to cause torsades de pointes. Be- Answer B is incorrect. GBS primarily causes
cause esmolol is extremely short acting (15-20 demyelination of peripheral nerves, not demy-
minutes), there are few major adverse effects if elination of the basal ganglia.
used as indicated.
Answer C is incorrect. GBS primarily causes
Answer C is incorrect. Lidocaine, a sodium demyelination of peripheral nerves, not demy-
channel blocker, is not known to cause tor­ elination of tracts in the spinal cord.
sades des pointes. The possible adverse effects
Answer D is incorrect. Hyperreflexia and a
of lidocaine are third-degree atrioventricular
positive Babinski reflex are hallmarks of an up-
(AV) heart block, altered AV node conduction,
per motor neuron lesion, which occurs in the
and depressed sinoatrial node automaticity.
central nervous system (CNS). GBS primarily
Answer D is incorrect. Propranolol, a affects peripheral nerves.
b-blocker, is not known to cause torsades de
pointes. Propranolol can cause bradycardia 12. The correct answer is D. Tolbutamide is a
and hypotension. first-generation sulfonylurea that acts by stim-
ulating the closing of potassium channels ex-
Answer F is incorrect. Verapamil, a calcium
pressed in the cell membrane of pancreatic
Test Block 4

channel blocker, is not known to cause tor­


acinar b cells. It causes cellular depolarization
sades des pointes. Verapamil can cause tran-
and then calcium influx, which in turn trig-
sient hypotension if incorrectly administered.
gers insulin release. An adverse effect of the
This can be countered by pretreatment with
first-generation sulfonylureas is a disulfiram-
calcium.
like reaction, including flushing, tachycardia,
Test Block 4  •  Answers 617

nausea, and hyperventilation. Tolbutamide is Answer A is incorrect. Dealing with abstract


ineffective in conditions in which there is an ideas occurs in the formal operational stage
absolute deficiency of insulin, such as type 1 around age 12 years into adulthood. Once
diabetes. children reach this stage, they are able to rea-
son logically and abstractly.
Answer A is incorrect. Acarbose is an
a-glucosidase inhibitor that acts at the intesti- Answer B is incorrect. Formulation and test-
nal brush border to decrease the absorption of ing of hypotheses is a skill achieved in the
starches and other polysaccharides. This agent formal operational stage, around age 12 years
would be effective in maintaining glycemic though adulthood. Children can play with pos-
control in someone with type 2 diabetes. sibilities at this stage and can see the potential
strengths and weaknesses of different ideas.
Answer B is incorrect. Glargine is a long-acting
synthetic insulin that provides a continuous Answer C is incorrect. Children in the con-
baseline level of insulin in the blood. This agent crete stage, around ages 7-11 years, are capable
would be appropriate for use in a patient with of limited logical thought processes. They can
type 1 diabetes, where it can be used in com- see the relationships between thoughts and
bination with a short- or intermediate-acting ideas and can classify them in a rational man-
insulin to cover the glycemic loads associated ner.
with meals and snacks. This agent would also
Answer D is incorrect. Awareness of conserva-
be suitable for a patient with type 2 diabetes,
tion is a skill from the concrete stage of cogni-
where it can be used in combination with sul-
tive development that occurs during ages 7-11
fonylurea.
years.
Answer C is incorrect. Metformin is an oral
hypoglycemic agent that is thought to decrease 14. The correct answer is B. This patient’s pre-
gluconeogenesis and increase glycolysis, re- sentation is consistent with intracranial injury
sulting in decreased blood glucose levels. This sustained from the significant force caused by
agent would be effective in helping to main- the ejection from her car. The patient was able

Full-Length Exams
tain glycemic control in someone with type 2 to talk with the police at the scene of the ac-
diabetes. cident but was unable to recall how the acci-
dent occurred, which likely demonstrates an
Answer E is incorrect. Troglitazone is a thia-
initial loss of consciousness followed by normal
zolidinone that acts to sensitize peripheral
mentation and subsequent deterioration of
tissues to insulin and increase target cell re-
consciousness. While different types of intra-
sponse; it does not act on pancreatic acinar
cranial bleeds can occur with trauma, this “lu-
cells. This agent would be effective in helping
cid interval” is classically seen with an epidural
to maintain glycemic control in someone with
hematoma. An overlying skull fracture is also
type 2 diabetes.
most consistent with an epidural hematoma
with rupture of the middle meningeal artery.
13. The correct answer is E. According to Piaget,
An urgent neurosurgical consult is indicated
children from the ages of two-seven years are
for immediate evacuation of the expanding he-
in the preoperational stage of cognitive devel-
matoma, which can lead to herniation.
opment. According to Piaget, preoperatory
thought includes any procedure for mentally Answer A is incorrect. Injury to the inferior
acting on objects. At this age the procedures cerebral veins would result in subarachnoid
are typically logically inadequate. The parents bleeding. Bleeding from the inferior cerebral
Test Block 4

in this vignette have an unreasonable expecta- veins does not result from a fracture of the
tion of their 4-year-old child’s abilities to rea- temporal bone and is unlikely to cause the
son and think logically. Children in the pre- rapid deterioration evident in this case because
operational stage are not capable of sustained it has a slower rate of bleeding. However, sub-
logical thought. arachnoid bleeding is frequently seen in the
618 Section III: Full-Length Examinations  •  Answers

setting of trauma and could be an associated anemia is not the cause of Crigler-Najjar syn-
finding with the epidural hematoma. drome, nor is it associated with that syndrome.
Answer C is incorrect. The posterior eth-
16. The correct answer is C. Arylsulfatase A con-
moidal artery supplies the anterior superior
verts sulfatide to galactocerebroside. This en-
nose and nasal septum with blood; its tearing
zyme is deficient in patients with metachro-
would not result in an epidural hematoma sec-
matic leukodystrophy, an autosomal recessive
ondary to a temporal bone fracture.
lysosomal storage disease in which patients
Answer D is incorrect. Injury to the sigmoid cannot degrade sulfatides, leading to accumu-
sinus would result in a subarachnoid hemor- lation of cerebroside sulfate in both neuronal
rhage. Thus such a finding is inconsistent with and non-neuronal tissues. There is abnormal
an injury to the temporal bone, the lucid in- myelination with widespread loss of myelina-
terval in this patient’s history, and the CT find- tion in the CNS and peripheral nerves, leading
ings. to the clinical signs. Metachromatic granules
can be seen on histologic examination.
Answer E is incorrect. Injury to the superior
sagittal sinus would also result in a subarach- Answer A is incorrect. a-galactosidase A
noid hemorrhage, but this is a far less likely converts ceramide trihexoside to lactosyl cere-
cause of this patient’s deterioration than an broside. This enzyme is deficient in Fabry dis-
epidural hematoma. ease.
Answer B is incorrect. b-galactosidase con-
15. The correct answer is D. In Crigler-Najjar
verts galactocerebroside to cerebroside. This
syndrome, the absence of glucuronyl transfer-
enzyme is deficient in Krabbe disease.
ase results in an inability to conjugate biliru-
bin, leading to an unconjugated hyperbilirubi- Answer D is incorrect. Hexosaminidase A con-
nemia (high indirect bilirubin). The jaundice verts ganglioside M2 to ganglioside M3. This
will become more severe as bilirubin accu- enzyme is deficient in Tay-Sachs disease.
mulates, and at high levels will result in brain
Answer E is incorrect. Sphingomyelinase con-
Full-Length Exams

damage. Two entities have been identified:


verts sphingomyelin to cerebroside. This en-
type 1 (autosomal recessive) and type 2 (auto-
zyme is deficient in Niemann-Pick disease.
somal dominant). A partial glucuronyl transfer-
ase deficiency is found in Gilbert syndrome. 17. The correct answer is D. The specimen from
Answer A is incorrect. While the patient may this patient indicates adenocarcinoma, which
have abnormalities in hematocrit, they would typically develops in the lower third of the
not be due to a glucuronyl transferase defi- esophagus near the gastric cardia. Chronic
ciency. gastroesophageal reflux disease leads to Barrett
esophagus, metaplasia of the squamous epithe-
Answer B is incorrect. If an enzyme for conju-
lium of the distal esophagus to columnar epi-
gation is lacking, unconjugated (indirect) bili-
thelium as a result of chronic acid exposure.
rubin will increase, not decrease.
Obesity increases the risk of esophageal ad-
Answer C is incorrect. Because the enzyme enocarcinoma by exacerbating reflux esopha-
missing is used for conjugating bilirubin, di- gitis. Other risk factors include known Barrett
rect bilirubin will decrease, not increase. esophagus, tobacco use, and prior radiation
therapy.
Answer E is incorrect. A hemolytic anemia
Test Block 4

would cause an increased reticulocyte count Answer A is incorrect. Achalasia is caused by


and also increase bilirubin level, which would esophageal hypomotility and defective relax-
result in jaundice. While this would further ation of the lower esophageal sphincter, lead-
complicate the infant’s condition, hemolytic ing to dysphagia, chest pain, and regurgitation
of food. It is associated with squamous cell car-
Test Block 4  •  Answers 619

cinoma of the esophagus, most likely as a re- films will normally show a ground-glass ap-
sult of chronic irritation from food abnormally pearance, not cavitations.
retained in the esophagus.
Answer B is incorrect. Cryptococcus neofor-
Answer B is incorrect. The other common mans infection often does not present with
variant of esophageal cancer is squamous cell any symptoms in an immunocompetent host.
carcinoma. It presents similarly to adenocar- However, in an immunocompromised individ-
cinoma with dysphagia, odynophagia, and ual, it can present with meningoencephalitis.
weight loss, but is differentiated on the basis of Cryptococcus is a heavily encapsulated yeast
histology and location of focus in the esopha- that is not dimorphic. The fungus is found in
gus (the upper to middle third). Whereas ad- soil and pigeon droppings.
enocarcinoma of the esophagus is more com-
Answer C is incorrect. Mucor species are
mon in white men, squamous cell carcinoma
molds with irregular nonseptate hyphae
is typically found in African-American men.
branching at wide angles (more than 90 de-
The higher incidence is most likely due to
grees). Symptoms of Mucor pulmonary infec-
poverty and increased rates of alcohol and to-
tion include an allergic reaction and infarction
bacco use.
of distal tissue due to fungal proliferation in
Answer C is incorrect. Both types of esopha- the walls of blood vessels. The disease is typi-
geal cancer are more common in male pa- cally seen in patients with diabetic ketoacidosis
tients. Squamous cell carcinoma of the and other causes of diminished immune func-
esophagus is four times more common, and tion (eg, hematologic malignancies). Surgery
adenocarcinoma seven times more common, and high doses of amphotericin B are used to
in men than in women. treat pulmonary mucormycosis.
Answer E is incorrect. Tobacco use is associ- Answer E is incorrect. Sporothrix schenckii is
ated with both morphologic variants of esopha- the cause of sporotrichosis. When S schenckii
geal cancer. is introduced into the skin, usually by a thorn
prick, it causes a local pustule or ulcer with

Full-Length Exams
18. The correct answer is D. This patient’s his- nodules along draining lymphatics (ascend-
tory is suggestive of infection with Aspergillus ing lymphangitis). S schenckii is a dimorphic
fumigatus, a ubiquitous fungus that can cause fungus that has cigar-shaped budding yeast vis-
lung cavity aspergillomas (“fungus balls”), usu- ible in pus. Itraconazole or potassium iodide is
ally in patients with preexisting cavities in their used for the treatment of S schenckii infection.
lungs. Treatment for these lesions is by surgi-
cal removal, if they are causing significant he- 19. The correct answer is E. Meyer’s loop is lo-
moptysis; otherwise itraconazole is commonly cated in the temporal lobe. It carries fibers
given. A fumigatus can also cause bronchopul- from the inferior retina that are responsible
monary aspergillosis and invasive aspergillosis, for the contralateral upper quadrant of vision.
the latter almost exclusively in patients who The fibers first pass through the ipsilateral op-
are immunocompromised. A fumigatus is a tic tract. Damage to Meyer’s loop on the right
non-dimorphic mold with septate hyphae that temporal lobe would result in bilateral left up-
branch at a V-shaped 45-degree angle. per quadrant anopsia.
Answer A is incorrect. Pneumocystis jiroveci Answer A is incorrect. The dorsal optic ra-
recently was identified as a fungus and typi- diation carries fibers from the superior retina
cally infects HIV-positive patients and other that are responsible for the contralateral lower
Test Block 4

immunocompromised individuals. Symptoms quadrant of vision. Damage to the left dor-


include fever, malaise, dyspnea, and a nonpro- sal optic radiation causes bilateral right lower
ductive cough. Microscopy shows cup-shaped quadrant anopsia.
cysts with intracystic bodies. However, chest
620 Section III: Full-Length Examinations  •  Answers

Answer B is incorrect. The dorsal optic ra- bination with interferon-a2 is used in the treat-
diation carries fibers from the superior retina ment of hepatitis B.
that are responsible for the contralateral lower
Answer E is incorrect. Ribavirin in combina-
quadrant of vision. Damage to the right dorsal
tion with pegylated interferon-a is used in the
optic radiation causes bilateral left lower quad-
treatment of hepatitis C.
rant anopsia.
Answer C is incorrect. The optic tract carries 21. The correct answer is F. The image shows a
fibers from the temporal retina of the ipsilat- crescent of epithelial cells and fibrin as well as
eral eye and from the nasal retina of the con- a hypercellular glomerulus, which together are
tralateral eye. Damage to the left optic tract characteristic of rapidly progressive glomerulo-
would cause right homonymous hemianopia. nephritis (RPGN). Furthermore, the clinical
scenario demonstrates a nephritic syndrome,
Answer D is incorrect. Damage to Meyer’s
with hematuria, oliguria, and hypertension,
loop in the left temporal lobe would result in
which one would expect with RPGN. In about
bilateral right upper quadrant anopsia.
50% of cases, RPGN is caused by deposition of
Answer F is incorrect. The optic tract carries immune complexes, but it can also be caused
fibers from the temporal retina of the ipsilat- by pauci-immune glomerulonephritis and
eral eye and from the nasal retina of the con- anti-glomerular basement membrane (GBM)
tralateral eye. Damage to the right optic tract disease (called Goodpasture syndrome when
would cause left homonymous hemianopia. associated with pulmonary hemorrhage).
Answer A is incorrect. The most common
20. The correct answer is B. This patient recently
cause of end-stage renal disease in the United
acquired acute hepatitis B virus (HBV) infec-
States is diabetic nephropathy, which causes
tion. Abdominal pain, positive HBsAg, and
a nephrotic syndrome. A renal biopsy would
presence of IgM anti-HBcAb are consistent
show Kimmelstiel-Wilson lesions, which are
with the diagnosis. Up to 15 weeks after ex-
nodules of mesangial matrix.
posure, patients with HBV infection are posi-
Full-Length Exams

tive for HBV DNA, HBsAg, anti-HBcAb, and Answer B is incorrect. Patients with Good-
HBeAg, but they are negative for anti-HBeAb pasture syndrome present with pulmonary
and anti-HBsAb. The correct combination hemorrhage and glomerulonephritis. The
therapy for hepatitis B is lamivudine and syndrome is caused by anti-GBM antibodies.
interferon-a. Lamivudine is a cytidine analog On immunofluorescence, renal biopsy would
that inhibits reverse transcription and there- show a linear pattern tracing the basement
fore blocks hepatitis replication. It also is used membrane of the glomeruli. Anti-GBM an-
as an HIV medication. Interferon-a is thought tibodies are implicated in the pathogenesis of
to block viral replication and thereby inhibit type 1 RPGN.
hepatitis replication.
Answer C is incorrect. IgA nephropathy, or
Answer A is incorrect. The presence of Berger disease, often presents in children as
HBeAg in the patient’s serum indicates an ac- hematuria following infection. Renal biopsy
tive HBV infection. HBeAg levels often are shows mesangial expansion due to deposition
used as a marker of infectivity of the patient. A of IgA.
patient previously vaccinated for HBV would
Answer D is incorrect. Lupus nephritis is the
have only anti-HBsAb and no other markers.
most serious symptom of SLE and often deter-
Test Block 4

Answer C is incorrect. Interferon-b is not used mines the prognosis of the disease. Additional
in the treatment of hepatitis. symptoms of SLE included malar rash, discoid
rash, photosensitivity, oral ulcers, arthritis, se-
Answer D is incorrect. Ribavirin in combina-
rositis, neurologic disorders, and immunologic
tion with pegylated interferon-a2 is used in the
disorders. Lupus nephritis causes a nephrotic
treatment of hepatitis C. Lamivudine in com-
Test Block 4  •  Answers 621

syndrome with proteinuria, hypoalbuminemia, the urogenital sinus, which normally gives rise
edema, and hyperlipidemia; many patients to the prostatic urethra.
also develop hypertension. Renal biopsy is im-
Answer D is incorrect. The urogenital folds
portant to determine treatment, and histologic
make up the ventral shaft of the penis and pe-
findings are classified in five patterns, includ-
nile urethra in the male. In contrast, posterior
ing mesangial and subendothelial deposits
urethral valves may be due to failure of regres-
(called “wire-loop” lesions).
sion of the urogenital sinus, which normally
Answer E is incorrect. Poststreptococcal glo- gives rise to the prostatic urethra.
merulonephritis is a common cause of ne-
phritic syndrome that occurs about 10 days 23. The correct answer is B. This patient suffers
after pharyngitis. On light microscopy one from the autosomal recessive disorder alkap-
would see diffuse proliferative glomerulone- tonuria, a deficiency in homogentisic acid oxi-
phritis without crescents. Electron microscopy dase. This enzyme is responsible for degrada-
would show subepithelial humps. Most cases tion of tyrosine. As a result of the deficiency,
are subclinical, and usually patients recover on there is an accumulation of alkapton bodies
their own, but some go on to develop RPGN, (homogentisic acid) in urine and cartilage.
like the patient in this vignette. The lack of homogentisic oxidase blocks the
metabolism of phenylalanine-tyrosine at the
Answer G is incorrect. Renal amyloidosis is
level of homogentisic acid. The homogentisic
associated with chronic inflammatory diseases
acid accumulates and a large amount is ex-
and causes nephrotic syndrome. Glomerular
creted, imparting a black color to the urine if
amyloid deposits can be seen on renal biopsy
allowed to stand and undergo oxidation. Dark
by staining with Congo red and examining the
spots can also be observed in the eyes of some
specimen under polarized light.
patients. Affected patients are usually asymp-
tomatic in childhood other than the change
22. The correct answer is E. The urogenital sinus
in urine color upon standing. In adulthood,
in the male gives rise to the bladder, prostate,
the build-up of pigment in cartilage and its
prostatic and membranous parts of the urethra

Full-Length Exams
calcification can cause arthritic changes. By
and bulbourethral glands.
an unknown mechanism, the pigment causes
Answer A is incorrect. The genital tubercle the cartilage to lose its resiliency and become
gives rise to the glans penis and corpus spon- brittle and fibrillated. The arthropathy devel-
giosum in the male, or the glans clitoris in the ops slowly and usually does not manifest until
female. In contrast, posterior urethral valves the patient is >30 years old. Although it is not
may be due to failure of regression of the uro- life-threatening, it may be severely disabling.
genital sinus, which normally gives rise to the
Answer A is incorrect. A deficiency of
prostatic urethra.
branched-chain a-ketoacid dehydrogenase
Answer B is incorrect. The mesonephric would result in maple syrup urine disease, an
(wolffian) duct develops into the seminal vesi- inability to break down branched-chain amino
cles, epididymis, ejaculatory duct, and vas def- acids.
erens. In contrast, posterior urethral valves may
Answer C is incorrect. Phenylketonuria is
be due to failure of regression of the urogenital
caused by a deficiency in phenylalanine hy-
sinus, which normally gives rise to the pros-
droxylase.
tatic urethra.
Answer D is incorrect. Refsum disease is a
Test Block 4

Answer C is incorrect. The paramesonephric


deficiency of phytanic acid oxidase and is
(müllerian) duct develops into the fallopian
characterized by an inability to break down
tube, uterus, and superior third of the vagina
branched-chain fatty acids.
in the female. In contrast, posterior urethral
valves may be due to failure of regression of
622 Section III: Full-Length Examinations  •  Answers

Answer E is incorrect. Tyrosinase is the en- glutination of RBCs in the periphery can lead
zyme responsible for the synthesis of tyrosine. to gray/purple discoloration of the fingers, as
This deficiency results in albinism because pa- seen in this patient.
tients are unable to make melanin, a derivative
Answer A is incorrect. Aplastic anemia results
of tyrosine.
in pancytopenia, malaise, and severe infection.
This disorder is a hypoproliferative anemia in
24. The correct answer is D. Introns are noncod-
which an appropriate reticulocytosis, as seen in
ing regions of RNA that are spliced out of ma-
this patient, is absent.
ture mRNA. Almost all introns begin and end
with 5’-GU------AG-3’. A mutation in one of Answer B is incorrect. Acute disseminated
those nucleotides affects splicing. This type of intravascular coagulation (DIC) results in
mutation is one of those found in the thalas- bleeding and shock. Chronic DIC results in
semia picture, as described in this patient. thrombosis and clotting. DIC typically is not
associated with mononucleosis, but instead
Answer A is incorrect. Capping of the mRNA
with gram-negative sepsis, acute myelogenous
occurs at the 5’ end as it is being transcribed. A
leukemia (AML), and obstetric complications.
mutation at the 5’ end of an intron would not
affect capping. Answer C is incorrect. Coombs test for IgG-
mediated (warm) hemolytic anemia is posi-
Answer B is incorrect. Hybridization is a pro-
tive for IgG that preferentially binds RBCs at
cess in which single-stranded DNA base-pairs
warmer temperatures. Warm hemolytic ane-
with a complementary sequence. A mutation
mia is associated with autoimmune disease,
at the 5’ end of an intron would not affect hy-
lymphoproliferative disorders, and drug abuse.
bridization
Answer E is incorrect. Paroxysmal nocturnal he-
Answer C is incorrect. A poly (A) tail is added
moglobinuria is caused by a defect in the RBC’s
to the 3’ end of heterogeneous nuclear RNA
protective mechanism against complement-
(hnRNA, the initial RNA transcript) after tran-
mediated lysis. Loss of the PIGA gene product,
scription. Poly(A) polymerase uses ATP as a
required for the surface anchoring of decay-
Full-Length Exams

precursor for adding adenosine one molecule


accelerating factor, results in episodic acute in-
at a time. A mutation at the 5’ end of an intron
travascular hemolysis and thrombosis.
would not affect polyadenylation.
Answer E is incorrect. Transcription of the 26. The correct answer is B. Seven-transmembrane-
DNA into RNA would not be affected by this domain receptors on cell surfaces are G-coupled
mutation. receptors. Many drugs bind to G-protein-coupled
receptors. Activation of Gq leads to activation
25. The correct answer is D. This patient’s pre- of phosphatidylcholine, which breaks down
sentation suggests an IgM-mediated (cold) phosphatidylinositol bisphosphate into ino-
hemolytic anemia, an anemia often encoun- sitol 1,4,5-triphosphate (IP3) and diacylglyc-
tered after a recent Epstein-Barr virus (EBV) erol (DAG). IP3 activates IP3 receptors in the
infection. Infectious mononucleosis (which endoplasmic reticulum, leading to release of
is caused by EBV) is associated with IgM an- calcium from intracellular stores, while DAG
tibodies directed at the i (lower case) antigen activates protein kinase C.
found on RBCs. In contrast, Mycoplasma
Answer A is incorrect. Activation of Gi leads
pneumoniae infection typically yields a hemo-
to downregulation of adenylyl cyclase and sub-
lytic anemia in which IgM antibodies are di-
Test Block 4

sequent decrease in cAMP and downregula-


rected at the I (upper case) antigen. Following
tion of protein kinase A. Activation of Gi is not
either infection, IgM binds both complement
linked to a rise in intracellular calcium.
and RBCs at lower temperatures (ie, as found
in the extremities), but falls off the RBCs as Answer C is incorrect. Activation of Gs leads
the cells return to the central circulation. Ag- to upregulation of adenylyl cyclase and an in-
Test Block 4  •  Answers 623

crease in cAMP, which then would lead to ac- 28. The correct answer is D. The patient is suf-
tivation of protein kinase A. Activation of Gs is fering from Hashimoto thyroiditis, an auto-
not related to a rise in intracellular calcium. immune disorder that is a common cause
of hypothyroidism. Other classic signs and
Answer D is incorrect. Upregulation of cGMP
symptoms of hypothyroidism include cold in-
does not lead to an increase in intracellular
tolerance, hypoactivity, weakness, diminished
calcium.
reflexes, dry and cool skin, and coarse hair.
27. The correct answer is C. The symptoms of Laboratory studies would reveal increased
fever, fatigue, night sweats, lymphadenopathy, thyroid-stimulating hormone (TSH) levels
and hemoptysis are consistent with the diagno- (the most sensitive laboratory test for primary
sis of tuberculosis (TB). The causative agent hypothyroidism) and decreased total triiodo-
of TB is Mycobacterium tuberculosis. Primary thyronine and thyroxine levels. In patients
TB infections are only rarely symptomatic in with Hashimoto thyroiditis, the thyroid gland
patients with normal immune function due to is usually enlarged and firm, while histology
rapid containment by resident alveolar macro- reveals a significant lymphocyte and plasma
phages and infiltrating monocytes and lympho- cell infiltrate with germinal center formation,
cytes. Symptomatic primary infection is mostly colloid-sparse follicles, and Hörthle cells (the
seen in the elderly, children, and immuno- acidophilic cells described in the vignette).
compromised individuals. Primary TB re- Patients with Hashimoto thyroiditis frequently
sembles an acute bacterial pneumonia, is typi- have a personal or family history of autoim-
cally located in the lower and middle lobes, mune disease, and there is an increased inci-
and rarely causes cavitation. Secondary TB dence of in the disease among individuals with
often localizes to the apex/upper lobes of the HLA-DR5 and -B5 haplotypes. Antimicrosomal
lungs, as shown in the x-ray film. A caseating antibodies, also called antithyroid peroxidase
granuloma is formed in which necrotic tissue antibodies, are associated with Hashimoto thy-
and bacteria are surrounded by macrophages roiditis. Antithyroglobulin antibodies may also
and giant cells. TB is initially treated with iso- be seen.

Full-Length Exams
niazid, rifampin, ethambutol, pyrazinamide, Answer A is incorrect. Anti-smooth muscle an-
and streptomycin, while isoniazid can be used tibodies may be seen in the setting of autoim-
alone to prevent TB or to treat suspected cases mune hepatitis.
of latent TB. The adverse effects of isoniazid
Answer B is incorrect. Anti-U1 RNP antibod-
include neurotoxicity and hepatotoxicity.
ies are associated with mixed connective tissue
Answer A is incorrect. This is an adverse effect disease.
of metronidazole, which is not used to treat or
Answer C is incorrect. Anticentromere anti-
prevent TB.
bodies are associated with CREST syndrome.
Answer B is incorrect. Aplastic anemia is not
Answer E is incorrect. Antimitochondrial an-
commonly associated with isoniazid.
tibodies are associated with primary biliary cir-
Answer D is incorrect. This effect has been as- rhosis.
sociated with tetracyclines, which are not used
as first-line treatment or for prevention of TB. 29. The correct answer is A. This is a typical clini-
cal picture of Rett syndrome, a rare pervasive
Answer E is incorrect. Nephrotoxicity, ototox-
developmental disorder nearly always affect-
icity, and teratogenic effects are associated with
ing girls 4 years old or younger. The hallmark
Test Block 4

aminoglycosides, which are often used to treat


features include decelerating social, cognitive,
gram-negative infections, not to prevent or
and verbal development that slowly progress to
treat TB.
degeneration in these areas. Children become
mentally retarded, expressionless, and nonver-
bal over the course of several years. Character-
624 Section III: Full-Length Examinations  •  Answers

istic hand-wringing movements begin at about children will suffer from encephalopathy and
the same time as the developmental decline. seizures.
Answer B is incorrect. Although language and
31. The correct answer is B. This woman is suf-
social degeneration occurs in patients with
fering from bacterial pneumonia, but despite
Rett syndrome, gross hearing is unaffected.
her infection, her WBC count and absolute
Answer C is incorrect. Hyperactivity is not as- neutrophil count are low. The most likely ex-
sociated with Rett syndrome; in fact, children planation for her laboratory results is agranu-
with Rett syndrome are typically withdrawn, locytosis, an adverse effect of clozapine use.
apathetic, and solemn. Clozapine antagonizes D1- and D2-receptor.
Patients taking clozapine must have regular
Answer D is incorrect. Psychosis is not a fea-
blood panel testing to monitor for abnormali-
ture of Rett syndrome.
ties.
Answer E is incorrect. Repetitive and compul-
Answer A is incorrect. Bromocriptine, per-
sive behaviors such as self-injury and purpose-
golide, pramipexole, and ropinirole are dopa-
less movements characterize autism and As-
mine receptor agonists working mainly at D1
perger’s disorder. They are not associated with
to D3 receptors in the brain. They are used as
Rett syndrome.
adjunctive treatment to levodopa in Parkinson
Answer F is incorrect. Tics are characteristic disease. Dopamine receptor agonists are not as-
of Tourette syndrome, which typically begins sociated with agranulocytosis, but neurologic
in childhood. Loss of social and cognitive adverse effects include dyskinesias, hallucina-
function is not associated with this disorder. tions, and confusion.

30. The correct answer is B. The image shows Answer C is incorrect. GABA receptor ago-
hypersegmented neutrophils, which are com- nists, such as benzodiazepines, ethanol, or bar-
monly seen in megaloblastic anemia. Causes biturates, inhibit the CNS, causing relaxation
of megaloblastic anemia are folate and vitamin and sedation. They are not associated with
agranulocytosis or other hematologic abnor-
Full-Length Exams

B12 deficiency. By an unknown mechanism,


phenytoin blocks absorption of folate and in- malities.
creases utilization of folate by the body, lead- Answer D is incorrect. The GABA receptor
ing to folic acid deficiency. antagonist flumazenil is used as an antidote to
Answer A is incorrect. b-thalassemias are he- benzodiazepine overdose. Flumazenil compet-
reditary disorders of hemoglobin that result in itively binds at the site on the GABA receptor
decreased b-chain synthesis. This disorder re- where benzodiazepines usually bind, thus re-
sults in a microcytic anemia. versing their sedating effects. It does not antag-
onize the CNS effects of other GABA agonists.
Answer C is incorrect. Iron deficiency results Flumazenil does not have any hematologic ad-
in a microcytic anemia with small, hypochro- verse effects.
mic RBCs on peripheral blood smear.
Answer E is incorrect. N-methyl-D-aspartate
Answer D is incorrect. While vitamin B12 (NMDA) receptors open in response to bind-
deficiency can cause a megaloblastic anemia, ing glutamate, allowing an influx of cations.
phenytoin is not a cause of vitamin B12 defi- NMDA receptors play an important role in
ciency. learning and memory. NMDA receptor an-
Answer E is incorrect. Vitamin B6 (pyridox- tagonists are most often used as anesthetics for
Test Block 4

ine) deficiency can cause dermatitis, glossitis, animals and are rarely used in humans. More
stomatitis, a microcytic anemia, and peripheral commonly, humans use NMDA receptor an-
neuropathy in adolescents and adults. Younger tagonists as recreational drugs because of their
hallucinogenic effects; examples include keta­
mine and phencyclidine. NMDA receptor an-
Test Block 4  •  Answers 625

tagonists have a number of neurologic and psy- Answer B is incorrect. Positive feedback is a
chiatric adverse effects, but they do not have rare regulatory mechanism. One example of its
hematologic adverse effects. use is when estrogen has positive feedback on
the pituitary, causing the estrogen-dependent
32. The correct answer is D. Treponema pallidum, estrogen surge right before the LH surge of
the bacterium that causes syphilis, produces ovulation. Although a decrease in estrogen
an endarteritis obliterans of the vasa vasorum, may lead to less estrogen to cause this normal
which supplies blood to the arch of the aorta. positive feedback mechanism, this is not the
This can lead to ischemia of the tissue, weak- most likely explanation of menopause.
ening and dilation of the aorta, and subsequent
Answer C is incorrect. There would be no
aortic regurgitation. The murmur described is
down-regulation of nuclear receptors in meno-
generally associated with this type of pathol-
pause.
ogy. Marfan syndrome, an autosomal domi-
nant genetic disorder resulting in defects in the Answer D is incorrect. There is a decrease,
fibrillin-1 protein, is also associated with aortic not an increase, in the negative feedback of es-
regurgitation, due both to instrinsic valvular trogen on the pituitary that leads to increases
degeneration and to dilation of the aortic root. in LH and FSH.
Answer A is incorrect. Chronic rheumatic Answer E is incorrect. There is no increase in
heart disease can lead to various types of val- positive feedback as would be seen during ovu-
vular damage, most commonly mitral stenosis. lation of a normal non-menopausal cycle.
Although it can lead to aortic valvular dam-
Answer F is incorrect. Another mechanism of
age, it would almost always be accompanied by
increasing the sensitivity of hormonal changes
symptoms of mitral stenosis as well.
is in up-regulating and down-regulating hor-
Answer B is incorrect. Congenital bicuspid mone receptors. Although in theory this might
aortic valves generally lead to calcification of be expected as menopause ensues and there is
the valves and aortic stenosis, not aortic re- less estrogen, this is not the best answer to ex-
gurgitation. Aortic stenosis is associated with a plain the increases in FSH and LH.

Full-Length Exams
crescendo-decrescendo systolic ejection mur-
mur that follows an ejection click. 34. The correct answer is D. Elevated galacti-
tol levels is the cause of clinical symptoms in
Answer C is incorrect. Congenital pulmonary
patients with galactosemia. Galactose is con-
stenosis is not associated with aortic insuffi-
verted in two steps to glucose-1-phosphate.
ciency. It is associated with right ventricular
The first step is catalyzed by the enzyme ga-
hypertrophy, increased jugular venous pres-
lactokinase, which phosphorylates galactose
sure, and poor oxygenation of blood.
to galactose-1-phosphate. The second step is
catalyzed by galactose-1-phosphate uridyl trans-
33. The correct answer is A. This patient presents
ferase (G1PUR), which converts galactose-
with amenorrhea due to menopause, defined
1-phosphate to glucose-1-phosphate. In the
by 12 months of amenorrhea that results from
absence of G1PUR, upstream intermediates in
ovarian follicular depletion with consequent
galactose metabolism accumulate, including
decrease in estrogen. Because of a lack of a
galactose-1-phosphate and galactitol. A defi-
maturing follicle, there is decreased estrogen
ciency in enzymes involved in other aspects of
secretion. Normally, estrogen feeds back to the
galactose metabolism leads to a much milder
pituitary to decrease gonadotropin-releasing
presentation (ie, only infantile cataracts). Al-
Test Block 4

hormone, follicle-stimulating hormone (FSH),


though treatment is not available, prevention of
and luteinizing hormone (LH). Without estro-
disease progression involves excluding galactose-
gen there is a decrease in the negative feed-
containing foods from the diet, including breast
back. Thus FSH and LH levels increase with-
milk and lactose-containing formulas.
out estrogen’s suppression.
626 Section III: Full-Length Examinations  •  Answers

Answer A is incorrect. Galactose is upstream should be reserved for patients who have failed
from G1PUR and ts levels would be elevated. medical therapy.
Answer B is incorrect. Classic galactosemia is Answer C is incorrect. Infliximab, a monoclo-
caused by a deficiency of this uridyltransferase nal antibody against tumor necrosis factor a, is
and would theoretically lead to a build up of used for the treatment of severe ulcerative coli-
galactose-1-phosphate, uridine diphosphoglu- tis following failure of more conservative thera-
cose, and galactose, and decreased glucose- pies.
1-phosphate.
Answer D is incorrect. Loperamide, an antidi-
Answer C is incorrect. Glucose-1-phosphate is arrheal, should be used only in mild ulcerative
a downstream product of G1PUR, and would colitis. While it may be useful for treatment of
be decreased in galactosemia. mild symptoms, it is not first-line therapy. Cau-
tion for development of fulminant colitis and/
Answer E is incorrect. Glucose-1-phosphate
or toxic megacolon should preclude the use of
is an intermediate in glycogen pathways. How-
antidiarrheals in patients with severe disease.
ever, galactosemia does not affect glycogen lev-
els. Answer E is incorrect. Oral steroids are used
for treatment of moderate-severity ulcerative
35. The correct answer is F. Based on the pre- colitis that is refractory to first-line therapy.
senting symptoms of abdominal pain, bloody This patient is suffering from mild disease and
stools, Ashkenazi Jewish ancestry, and family may benefit from the addition of oral steroids if
history, this patient is suffering from inflam- sulfasalazine therapy fails.
matory bowel disease (IBD), most likely ul-
cerative colitis. The image shows classical 36. The correct answer is B. Huntington disease is
diffuse mucosal inflammation with pseudo- inherited in an autosomal dominant manner. It
polyps. Pseudopolyps are areas where mucosa is caused by the expansion of CAG repeats on
has been eroded away such that only islands chromosome 4, which is associated with the
of intact mucosa remain; given their polypoid progressive degeneration of the caudate nu-
Full-Length Exams

shape, these islands are referred to as pseudo- cleus and subsequent loss of GABAergic neu-
polyps. However, they are actually the only rons. A primary function of the caudate is to
“normal” parts of mucosa left, and the pathol- modulate motor action plans arising from the
ogy is the ulcerative lesions all around these frontal cortex. Patients typically present in the
pseudopolyps. Infectious etiologies of diarrhea third or fourth decades of life with symptoms of
are less likely given the patient’s absence of fe- chorea, depression, and dementia.
ver, left shift, sick contacts, and foreign travel.
Answer A is incorrect. Klüver-Bucy syndrome,
Increased ESR is also consistent with diagno-
clinically manifested by hyperorality, hypersex-
sis of IBD. Sulfasalazine is first-line therapy
uality, and disinhibited behavior, is associated
for ulcerative colitis. It is metabolized to
with bilateral obliteration of the amygdala.
5-aminosalicylic acid in the digestive tract and
decreases inflammation locally. Adverse effects Answer C is incorrect. Amyotrophic lateral
include renal insufficiency and increased risk sclerosis, more commonly known as Lou Geh-
of bleeding. rig’s disease, is associated with degeneration of
the lateral corticospinal tracts.
Answer A is incorrect. Ciprofloxacin may be
useful in the treatment of ulcerative colitis Answer D is incorrect. Wernicke encepha-
complicated by strictures and infections of the lopathy, which is most commonly seen in mal-
Test Block 4

GI tract; however, it is not first-line treatment nourished alcoholics, is associated with atro-
for inflammatory bowel disease. phy of the mammillary bodies.
Answer B is incorrect. Surgery, specifically
colectomy in the case of ulcerative colitis,
Test Block 4  •  Answers 627

Answer E is incorrect. Alzheimer disease is relative risk versus the unexposed population.
marked by a decreased number of neurons in However, in this case, the CI refers to means
the nucleus basalis of Meynert. of the population, not to odds ratio or relative
risk, and these values cannot be determined
Answer F is incorrect. Parkinson disease is
simply by dividing the two means. Thus, it is
characterized histologically by neuronal deple-
unlikely that there is a significant difference
tion and depigmentation of cells in the sub-
between the means of albuterol and drug X be-
stantia nigra.
cause the CI of the means overlap.
37. The correct answer is B. Confidence inter- Answer E is incorrect. The CI of the mean
vals (CIs) specify a lower and upper limit for no intervention does not overlap the CI of
of variability around a specific probability the mean for albuterol. Thus, there is likely a
in which data points are likely to fall. Fre- significant difference between these means. Al-
quently, the 95% CI is used; this means that though the width of the CIs signifies increased
in 95% of cases, the true value for the popu- variability within the population, this does not
lation will fall within the given CI. Generally, preclude significant differences between the
if the 95% CIs do not overlap, the means of means.
the groups differ significantly. If the CI over-
lap considerably, it is less likely that there is 38. The correct answer is D. HIV-associated de-
a significant difference between the means of mentia (also known as AIDS dementia com-
the two groups. However, one must be careful plex) presents with memory loss, gait disorder,
with this assumption because if two CI come and spasticity. It represents the most common
very close but do not overlap, there may not direct CNS complication of HIV disease and
be significant difference at the p = 0.05 level. generally occurs later in the course of ill-
Conversely, a small overlap of CI does not ness. Early symptoms may be subtle and may
preclude the possibility of difference between include depressive symptoms and apathetic
the means, but the chance that this difference withdrawal; later symptoms include global de-
is significant at the p = 0.05 level is much less mentia and motor deficits. As the dementia

Full-Length Exams
likely. In this case, the CI of the mean for no progresses, patients experience difficulty with
intervention does not overlap the CI of the smooth limb movement, dysdiadochokinesia
mean for albuterol. Thus, there is likely a sig- (impairment in performing rapid, alternating
nificant difference between these means. movements), impaired saccadic eye move-
ments, hyperreflexia, and frontal release signs.
Answer A is incorrect. Because there is over-
Imaging studies are imperative to rule out mass
lap in the CI of the mean peak expiratory flow
lesions; 20%-40% of patients will demonstrate
rate with albuterol and drug X, it is unlikely
non-enhancing, poorly demarcated areas of in-
that there is a significant difference between
creased T2 signal intensity in the deep white
the means of these two conditions.
matter. The symptoms must be distinguished
Answer C is incorrect. Because there is over- from typical focal neurologic signs and symp-
lap in the CIs of the mean peak expiratory flow toms that may be evident in patients with mass
rate with albuterol and no intervention, it is lesions. Elevated levels of protein and IgG on
unlikely that there is a significant difference cerebrospinal fluid (CSF) examination are
between the means of these two conditions. present in approximately 45% and 80% of
Answer D is incorrect. When assessing odds cases, respectively.
ratio or relative risk, if the 95% CI includes Answer A is incorrect. CNS lymphoma typi-
Test Block 4

the value 1, the null hypothesis cannot be re- cally affects patients with AIDS whose CD4+
jected. This means that the data do not con- cell counts are <50/mm³ with one or more en-
firm a significant difference between the odds hancing lesions on MRI (50% multiple; 50%
of having a disease based on the odds ratio single). It can present with many signs and
or the risk of getting a disease based on the symptoms that overlap with HIV-associated
628 Section III: Full-Length Examinations  •  Answers

dementia, but it is less insidious and typically Answer B is incorrect. Neither the anterior
causes more focal signs earlier in the course of nor the posterior pituitary is derived from
the illness. CNS lymphoma can present with a meso­derm. The patient’s adenoma developed
positive polymerase chain reaction for Epstein- in the anterior pituitary, which is derived from
Barr virus within the CSF. oral ectoderm.
Answer B is incorrect. Cytomegalovirus Answer C is incorrect. Neither the anterior
(CMV) encephalitis can mimic HIV-associated nor the posterior pituitary is derived from neu-
dementia, but is usually more rapidly progres- ral crest cells. The patient’s adenoma devel-
sive and is typically concurrent with more oped in the anterior pituitary, which is derived
generalized CMV infections. MRI typically from oral ectoderm.
demonstrates enhancing periventricular white
Answer D is incorrect. Neuroectoderm gives
matter lesions in cortical and subependymal
rise to the posterior pituitary, but the patient’s
regions.
adenoma is a prolactin-secreting tumor that
Answer C is incorrect. Disseminated Myco- develops in the anterior pituitary. The anterior
bacterium avium complex infection is a late- pituitary is derived from oral ectoderm.
stage complication of AIDS and is associated
with CD4+ cell counts <50/mm³. It typically 40. The correct answer is D. I-cell disease is an
presents with constitutional signs and symp- autosomal recessive disorder that results from
toms that include fever, night sweats, lym­ improper intracellular trafficking. This im-
phadenopathy, hepatosplenomegaly, weight paired trafficking results from the failure to
loss, and pancytopenia. The symptoms are add a mannose-6-phosphate residue to pro-
more generalized and severe than those in teins that should be directed to lysosomes. On
HIV-associated dementia. a cellular level, this results in the presence of
numerous intracytoplasmic inclusions in cells
Answer E is incorrect. Space-occupying le-
of mesenchymal origin. These inclusions are
sions secondary to toxoplasmosis infection
membrane-bound vacuoles that are filled with
begin to occur with CD4+ cell counts <100/
fibrillogranular material, including a variety
Full-Length Exams

mm³ and typically appear as enhancing CNS


of lipids, mucopolysaccharides, and oligosac-
lesions (which may be multiple) on MRI, with
charides. Clinically this deficiency results in a
positive serologies. Treatment is typically with
select group of identifying features. Be on the
sulfadiazine and pyrimethamine, with imag-
lookout for coarse facial features in a baby that
ing studies repeated after a few weeks. If no re-
is developmentally delayed and has restricted
gression has occurred, the diagnosis should be
joint movement.
reconsidered, and CNS lymphoma should be
considered the most likely diagnosis. Answer A is incorrect. Congenital herpes in-
fection is characterized by acute CNS find-
39. The correct answer is E. The hormone pro- ings; keratoconjunctivitis, vesicles on the skin,
lactin is secreted by the anterior pituitary. In eyes, and mucous membranes.
contrast to the posterior pituitary, which is de-
Answer B is incorrect. Congenital rubella
rived from neuroectoderm and considered an
infection is a devastating disease. It is char-
extension of the brain, the anterior pituitary is
acterized by cataracts, glaucoma, pigmented
derived from oral ectoderm on the roof of the
retinopathy, cardiac malformations and deaf-
mouth.
ness. None of these symptoms is evident in this
Answer A is incorrect. Neither the anterior child.
Test Block 4

nor the posterior pituitary is derived from en-


Answer C is incorrect. Ehlers-Danlos syn-
doderm. The patient’s adenoma developed in
drome is not explained by this mechanism.
the anterior pituitary, which is derived from
This baby does not demonstrate any of the
oral ectoderm.
symptoms of this disease; furthermore, this dis-
Test Block 4  •  Answers 629

ease is not usually diagnosed until the end of subtype is more common in men and more
the first decade of life. likely to be diagnosed at a later stage. The
overall prognosis is good.
Answer E is incorrect. Fibrillin mutations ac-
count for Marfan syndrome, not a likely diag-
42. The correct answer is B. This patient is under-
nosis in this baby.
going acute graft-versus-host disease (GVHD),
which classically has a triad of dermatitis, hep-
41. The correct answer is E. As seen in the im-
atitis, and gastroenteritis. GVHD is a serious
age the presence of many lymphocytes and
complication of allogeneic blood or marrow
few Reed-Sternberg cells with collagen bands
transplantation, and is mediated by donor lym-
that circumscribe the lymphoid tissue into dis-
phocytes reacting against major or minor his-
crete nodules is consistent with the nodular-
tocompatibility antigens on recipient cells that
sclerosing subtype of Hodgkin disease. This
are recognized as foreign.
is the subtype with the best prognosis, and it
also is the most common. Nodular-sclerosing Answer A is incorrect. ABO incompatibility
Hodgkin lymphoma is more common in occurs when there is transfusion of incompati-
women. This histologic picture also resembles ble blood types. If patients are exposed to blood
the lymphocyte-predominance subtype, which that is incompatible with their own blood type,
is much less common but has an excellent they may undergo an immune-mediated hemo-
prognosis; it also is found in women. lytic anemia that could eventually lead to jaun-
dice. The most common example is Rh incom-
Answer A is incorrect. AML does not affect
patibility in pregnant patients.
the lymph nodes but, rather, produces abnor-
malities in the blood and bone marrow. This Answer C is incorrect. By definition, chronic
disease usually affects patients of middle age graft-versus-host disease occurs >100 days after
(35-50 years) and is characterized by the pres- transplant and can affect any organ system.
ence of numerous myeloid precursor cells with
Answer D is incorrect. Primary graft rejection
the presence of Auer’s rods.
occurs when neutrophil and platelet recovery

Full-Length Exams
Answer B is incorrect. Burkitt lymphoma is does not occur in the usual time frame ex-
a non-Hodgkin type of lymphoma that pre- pected after transplantation, and is mediated
dominantly is a B-lymphocyte lymphoma. It by the recipient immune system against alloan-
is associated with Epstein-Barr virus infections tigens expressed on donor stem cells. Patients
that can lead to activating mutations of c-myc in acute rejection do not present with derma-
caused by chromosomal translocation t(8;14). titis, hepatitis, and gastroenteritis as in graft-
Histologically Burkitt lymphoma is character- versus-host disease.
ized by sheets of lymphocytes with interspersed
Answer E is incorrect. Hyperacute rejection
macrophages; this is referred to commonly as a
is seen only in solid organ transplants. Hyper-
“starry sky” appearance.
acute graft-versus-host disease is an entity that
Answer C is incorrect. Any type of Hodgkin may occur within minutes of the time of en-
disease (HD) involving many Reed-Sternberg graftment.
cells and no or few lymphocytes describes the
rare lymphocyte-depletion subtype. This has 43. The correct answer is E. This patient has a
the worst prognosis of any type of HD and gen- metabolic alkalosis as her pH is >7.4 and par-
erally is present in elderly men with dissemi- tial pressure of carbon dioxide is > 40 mm Hg.
nated disease. This is secondary to her dehydration (which
Test Block 4

is apparent by her symptoms of orthostatic


Answer D is incorrect. The heterogeneous
hypotension). Her frequent emesis results in
mixture of many mononuclear cells, many
the loss of large quantities of protons from the
Reed-Sternberg cells, and many lymphocytes
body in the form of stomach acid. In addition,
is consistent with mixed-cellularity HD. This
rapid loss of bicarbonate-free fluids such as
630 Section III: Full-Length Examinations  •  Answers

stomach contents or urine can result in a net Answer B is incorrect. Gastrointestinal reflux
increase in plasma bicarbonate concentration; disease (GERD) can often mimic the symp-
this effect is termed “contraction alkalosis.” toms of MI and cause pathologic abnormali-
Finally, volume depletion leads to stimulation ties in the esophagus. GERD, however, rarely
of the renin-angiotensin-aldosterone system results in the sudden death of a patient.
(RAAS), causing (1) an angiotensin-mediated
Answer C is incorrect. An occlusion of the left
increase in hydrogen secretion via the sodium-
coronary artery is a very common cause of MI
hydrogen antiporter in the proximal tubule,
that is detectable on ECG.
and (2) an aldosterone-triggered influx of so-
dium (and water) and an efflux of potassium Answer D is incorrect. Acute thrombosis due
and protons in the distal tubule. The loss of to coronary artery atherosclerosis results in
protons and build-up of bicarbonate in this pa- myocyte necrosis of the ventricular wall. This
tient causes metabolic alkalosis. would be an MI with a typical and strongly de-
tectable pattern on ECG.
Answer A is incorrect. Consumption of antac-
ids can contribute to metabolic alkalosis but is Answer E is incorrect. Changes in the peri-
not the cause in this patient. cardium may evolve over time as a result of an
MI. Fibrinous pericarditis (friction rub) can
Answer B is incorrect. Dehydration causes an
occur three-five days after an MI. Dressler syn-
increase, not decrease, in hydrogen excretion
drome (an autoimmune phenomenon) may
in the distal tubule.
occur several weeks after an MI in a surviving
Answer C is incorrect. Total bicarbonate re- patient.
absorption in the setting of metabolic alkalosis
and volume depletion is likely to be reduced. 45. The correct answer is C. The condition de-
Acutely, volume depletion will result in a net scribed is polycystic ovarian syndrome (PCOS,
decrease in the filtered load of bicarbonate, de- or Stein-Leventhal syndrome). The main
spite an increase in bicarbonate concentration. characteristics of PCOS are oligoovulation or
In addition, increased plasma levels of bicar- anovulation and hyperandrogenism. Features
Full-Length Exams

bonate will impair the ability of the proximal seen in PCOS include ovarian cysts, amenor-
tubule cells to secrete acid necessary for bicar- rhea, infertility, obesity, and hirsutism caused
bonate reabsorption. Although angiotensin II by excess LH production and androgens. In
stimulation serves to partially counteract these some women PCOS is associated with insu-
effects, the proximal tubule is not the primary lin resistance and hyperinsulinemia, which
site of acid loss in this patient. increases androgen production in the ovar-
ian theca cells and, secondarily, LH produc-
Answer D is incorrect. ADH does not have
tion. Insulin resistance leads to hyperglycemia
direct effects on acid-base status because the
and also suppresses hepatic steroid hormone-
aquaporin channels it mobilizes to the cell
binding globulin (SHBG) synthesis. The de-
membrane are permeable only to water.
crease in SHBG along with the increase in
androgen production leads to a vicious cycle of
44. The correct answer is A. The patient suffered
amenorrhea and infertility.
a thoracic aortic dissection. Malignant hyper-
tension predisposes to this sudden tear along Answer A is incorrect. Patients with Kall-
the aorta. In fact, aortic dissections often pre­ mann syndrome present with hypogonado-
sent with a tearing pain along the patient’s trophic (decreased levels of gonadotropin-
back. Because the symptoms of aortic dissec- releasing hormone) hypogonadism (decreased
Test Block 4

tion are similar to those of a myocardial infarc- LH and FSH levels) along with anosmia (lack
tion (MI), and because dissections are often of smell).
not recognized on ECG, they are often either
Answer B is incorrect. Amenorrhea and ga-
misdiagnosed or not recognized at all.
lactorrhea are signs of hyperprolactinemia.
Test Block 4  •  Answers 631

Prolactin is controlled by dopamine secretion Answer B is incorrect. Retinal hemorrhage


from the hypothalamus and inhibits gonado- and detachment also are seen frequently in
tropin secretion. Patients with hyperprolac- cases of child abuse. If there is suspicion of
tinemias typically have pituitary microadeno- retinal damage, the child should then see an
mas that secrete prolactin. They present with ophthalmologist, but an urgent retinal exami-
visual field defects, nipple discharge bilaterally, nation is not necessary at this time.
and hypogonadism. This leads to decreased
Answer D is incorrect. Although confronting
levels of LH and FSH, oligomenorrhea or
the family and offering assistance may be well
amenorrhea, infertility, and galactorrhea. This
intended, it is not the appropriate course of ac-
would not explain the high level of LH, ele-
tion in this situation.
vated androgen levels, or enlarged ovaries seen
in this patient. Answer E is incorrect. The possibility of child
abuse should not be taken lightly. Normal or
Answer D is incorrect. Polycystic kidney dis-
even careless play is an insufficient explana-
ease (PKD) is an autosomal dominant disease.
tion for a serious injury such as a spiral frac-
Patients generally present in their 40s-50s with
ture of the femur.
abdominal discomfort, hematuria, urinary tract
infections, hypertension, and renal insuffi- 47. The correct answer is B. The paired, meso-
ciency. On gross pathology, the kidneys are en- dermally derived organs that give rise to a bi-
larged and the normal parenchyma is replaced cornuate uterus are the müllerian tubes, oth-
by dozens of cysts. Although PKD is associated erwise known as the paramesonephric ducts.
with berry aneurysms and cardiac anomalies, These paired structures also develop into the
this disease is not associated with polycystic fallopian tubes, and the upper, proximal por-
ovarian syndrome. tion of the vagina. In the presence of mül-
Answer E is incorrect. Weak pulses in the lerian inhibiting factor, which is synthesized
lower extremities may be a sign of coarctation and secreted by Sertoli’s cells (ie, in XY indi-
of the aorta, which is associated with Turner viduals), the müllerian tubes degenerate into a
syndrome. Turner syndrome is the most com- vestigial remnant called the appendix testes, or

Full-Length Exams
mon cause of primary amenorrhea, and pa- hydatid of Morgagni. These vestigial structures
tients present with short stature, webbed neck, are most often found on the upper pole of the
and infantile genitalia. Ovaries are replaced testes adjacent to the tunica vaginalis.
with fibrous streaks, and appear small and fi- Answer A is incorrect. The appendix epididy-
brotic on ultrasound, as opposed to enlarged mis is a small appendage on the head of the
and cystic. epididymis derived from the wolffian (meso-
nephric) duct, which is the genital duct that
46. The correct answer is C. The suspicion of
develops into the seminal vesicles, epididymis,
child abuse arises when the injury and the
ejaculatory duct, and vas deferens in the pres-
story of the injury do not match. Spiral frac-
ence of androgens.
tures generally do not occur with a simple fall
down the stairs. Another indication of abuse Answer C is incorrect. The bulbourethral
is multiple bruises at varying stages of healing glands (also known as Cowper glands) are
and this patient’s history of trauma occurring small exocrine glands found on the posterior
in the home. Suspicion of child abuse must be and lateral aspect of the membranous urethra
reported in all 50 states. at the base of the penis. They are not derived
from the müllerian ducts.
Test Block 4

Answer A is incorrect. Children with osteo-


genesis imperfecta can present with spiral frac- Answer D is incorrect. The paradidymis is an-
tures as a result of seemingly benign accidents. other structure derived from the wolffian duct.
In this case, however, the history and evidence It is composed of tubules located above the
strongly support child abuse. head of the epididymis.
632 Section III: Full-Length Examinations  •  Answers

Answer E is incorrect. Skene glands (also is used to treat CMV retinitis when ganciclovir
known as paraurethral glands) are small glands fails. It is also used to treat acyclovir-resistant
found on the anterior wall of the vagina. They herpes simplex virus infection.
are not derived from the müllerian ducts.
Answer D is incorrect. Ganciclovir is phos-
phorylated by viral kinase. It preferentially in-
48. The correct answer is A. Acyclovir is a guano-
hibits CMV DNA polymerase and is used to
sine analog used in the treatment of active her-
treat CMV retinitis, especially in immunocom-
pesvirus infections. Acyclovir causes premature
promised patients. It is not first-line treatment
DNA chain termination and limits the length
for herpes virus infection.
of a herpes outbreak, although it does not
change the frequency with which they occur. Answer E is incorrect. Zanamivir inhibits in-
fluenza neuraminidase, reducing the release
Answer B is incorrect. Didanosine is a nucleo-
of progeny virus. It is used in the treatment of
side reverse transcriptase inhibitor used in HIV
both influenza A and B virus infections.
therapy. It is not used in treating herpes virus
infection.
Answer C is incorrect. Foscarnet is a viral
DNA polymerase inhibitor that binds to the
pyrophosphate binding site of the enzyme. It
Full-Length Exams
Test Block 4
Test Block 5

633
634 Section III: Full-Length Examinations  •  Questions

Q u e st i o n s

1. A 47-year-old man with a history of hyperpara- 3. A 50-year-old man comes to the physician
thyroidism presents to his physician because of because of abdominal fullness, fatigue, and
a mass in his anterior neck. Laboratory stud- weight loss but denies any fever or night
ies show an elevated serum calcitonin level. sweats. Physical examination is significant
The patient reports that multiple family mem- for splenomegaly. Blood tests are positive for
bers have had “thyroid problems” in the past. pancytopenia and on bone marrow aspiration
Which of the following is the most likely his- predominant, large white cells demonstrate
tology of this patient’s neck mass? tartrate-resistant acid phosphatase activity. The
patient’s peripheral blood smear is shown in
(A) Atrophic follicles with lymphocyte infil-
the image. Which of the following disease pro-
trate and germinal centers
cesses is most likely causing this patient’s prob-
(B) Nests of hormone-secreting tumor cells in
lems?
an amyloid-filled stroma
(C) Papillary pattern with ground-glass nuclei
and psammoma bodies
(D) Sheets of undifferentiated, pleomorphic
cells
(E) Uniform follicles with sparse colloid and a
large cell lining

2. A 65-year-old cancer patient gives his best


friend durable power of attorney. As his con-
dition worsens, he goes into respiratory fail-
ure and is now on a ventilator in a coma. The
friend believes the patient would have wished
Full-Length Exams

for life support to be withdrawn. However, the


patient had previously made a living will stipu-
lating that all measures should be undertaken
to maintain his life. The patient’s son believes Reproduced, with permission, from USMLERx.com.
his father’s living will reflects his wishes and
wants extreme measures taken to maintain his
(A) Chronic lymphocytic leukemia
life. What action should the physician take?
(B) Follicular lymphoma
(A) Advise the friend to listen to the son’s (C) Hairy cell leukemia
wishes (D) Mantle cell lymphoma
(B) Appoint the son durable power of attorney (E) Nodular sclerosis Hodgkin lymphoma
because he is next of kin
(C) Keep the patient on life support in accor- 4. A 55-year-old white woman with type 2 diabe-
dance with the living will tes mellitus has the following lipid profile on
(D) Poll all family members present and follow routine laboratory testing:
the most supported course of action Total cholesterol: 280 mg/dL
(E) Respect the durable power of attorney and LDL cholesterol: 170 mg/dL
withdraw life support HDL cholesterol: 23 mg/dL
Test Block 5

Triglycerides: 320 mg/dL


Test Block 5  •  Questions 635

A treatment strategy is initiated. A few weeks


later, the patient develops hip pain and weak-
ness. Laboratory analysis reveals normal liver
markers and a creatine kinase of 3000 mg/dL.
Which of the following medication(s) places
this patient at the greatest risk of developing
these adverse effects?
(A) Ezetimibe alone
(B) Gemfibrozil alone
(C) Simvastatin and ezetimibe
(D) Simvastatin and gemfibrozil
(E) Simvastatin and niacin

5. A 42-year-old African-American woman with a Reproduced, with permission, from USMLERx.com.


history of sarcoidosis presents to a neurologist
with a sudden onset of a unilateral inability to
close the eye and decreased tearing. When she (A) Folate level
wrinkles her forehead or smiles, the affected (B) Homocysteine level
side of her face remains relaxed. The muscles (C) Methylmalonic acid level
involved in this condition are derived from (D) Serum B12 level
which of the following embryologic structures? (E) Urine vitamin B12 level
(A) First branchial (pharyngeal) arch 7. While working in a microbiology laboratory, a
(B) Second branchial (pharyngeal) arch researcher comes across an unlabeled cryotube
(C) Third branchial (pharyngeal) pouch in the –80° freezer. She deduces that it con-
(D) Fourth branchial (pharyngeal) pouch tains a strain of Escherichia coli and decides
(E) Thyroglossal duct to test whether this strain has an intact lactose

Full-Length Exams
(lac) operon. After growing the cells in media
6. A 43-year-old man presents to his physician
containing both glucose and lactose, she ob-
with fatigue. The patient says he is concerned
serves that the b-galactosidase, encoded by the
about his fatigue because he has a strong fam-
lac operon, is expressed. No protein products
ily history of cancer. He thinks that if his par-
are produced when the E coli is grown only
ents did not drink, smoke, and eat such poor
with glucose. Based on this observation, where
diets they would have lived longer lives. He
is the mutation most likely located?
says that because of all this, he never drinks al-
cohol or smokes tobacco. He has also followed (A) Cyclic adenosine monophosphate-receptor
a strict vegan diet for 10 years. He says that all protein
of his meals are high in leafy green vegetables. (B) Inducer-binding site
Laboratory tests show a hematocrit of 35% with (C) Promoter
a normal RBC distribution width. Results of a (D) Repressor
peripheral blood smear are shown in the im- (E) RNA polymerase
age. Which of the following laboratory tests
will most definitively determine the likely
cause of his abnormal blood smear?
Test Block 5
636 Section III: Full-Length Examinations  •  Questions

8. A mother brings her 2-month-old infant to (A) Chromosomal rearrangement


the emergency department because of leth- (B) Dominant negative effect
argy, failure to thrive, and a fever of 39.2°C (C) Gain of function
(102.6°F). Physical examination reveals in- (D) Loss of function
creased head circumference and prominent (E) Viral insertion
hepatosplenomegaly. Screening laboratory
tests reveal a profound anemia and leukopenia. 11. A 45-year-old woman with a long history of
Despite fluid resuscitation and initiation of menstrual irregularities and infertility presents
antibiotic therapy, the child dies. On autopsy, with complaints of worsening vision. A thor-
histologic analysis of the child’s bone marrow ough review of systems reveals the presence of
space reveals a marked infiltration of the med- constipation, cold intolerance, and increased
ullary canal space by primary bony spongiose pigmentation of skin. The physician performs
tissue. Which malfunctioning cells are the basic visual field testing and maps the visual
cause of this patient’s disease process? disturbance in the patient’s chart. Which of
the following visual field defects is most likely
(A) Hepatocytes
in this patient?
(B) Lymphoid progenitor cells
(C) Osteoblasts
(D) Osteoclasts Defect in visual field of
(E) Reticulocytes Left eye Right eye

9. A 7-year-old boy is brought to his pediatrician A


for evaluation of a rash. The boy’s mother re-
ports that the family moved to a new home in B
a forested area approximately one month prior
to the onset of the rash. On questioning, the C
boy reports that he last played in the woods
two days before the rash began. Which of the D
Full-Length Exams

following is characteristic of the cells that me-


diate this boy’s immune reaction? E
(A) Buffering of acid generated by tissue me-
tabolism Reproduced, with permission, from USMLERx.com.
(B) Cell surface immunoglobulin receptor
(C) Defense against parasitic disease
(D) Degranulation and release of histamine (A) A
(E) Interaction with antigen-MHC complex (B) B
(F) Nonspecific phagocytosis and presentation (C) C
of skin antigens (D) D
(G) Phagocytosis of opsonized cells (E) E

10. A woman whose mother had cancer in both 12. A 12-year-old boy is brought to the emergency
breasts develops breast cancer at age 26 years. department suffering from an acute asthma
The patient’s identical twin sister decides attack. The intern reaches for a drug used for
to undergo genetic testing to determine her the chronic prevention of asthma exacerba-
chances of developing breast cancer. What tions rather than a drug needed for this acute
Test Block 5

mechanism causes the genes that are most attack. The intern likely reached for which of
commonly tested for breast cancer to become the following medications?
tumorigenic? (A) Cromolyn
(B) Epinephrine
Test Block 5  •  Questions 637

(C) Ipratropium (A) A-positive donor and O-negative recipient


(D) Terbutaline (B) AB-negative donor and A-positive recipient
(E) Theophylline (C) AB-positive donor and B-negative recipient
(D) B-negative donor and AB-positive recipient
13. A 44-year-old woman presents with worsening (E) O-negative donor and A-positive recipient
fatigue for the past two months. She recently (F) O-positive donor and B-negative recipient
overcame her battle with alcoholism and says
her next goal is to improve her “horrible diet.” 15. A 10-year-old boy with an X-linked immu-
Physical examination is unremarkable. Labo- nodeficiency disease suffers from chronic re-
ratory studies are significant for a hemoglobin current gastrointestinal inflammation, which
level of 8 g/dL and a mean corpuscular vol- only moderately improves with cyclosporine
ume of 110 fL. A folic acid deficiency is sus- therapy. The child has had previous laboratory
pected. Which of the following is the most ac- evaluation that showed a negative reaction to
curate description of the function of folic acid? the nitroblue tetrazolium test. The patient’s fa-
ther wants to know how his son can have both
(A) Catalyzes γ-carboxylation of glutamic acid
an immunodeficiency disease and an autoim-
residues
mune disease. Deficiencies in which of the
(B) Hydroxylates prolyl and lysyl residues
following provide a logical link between this
(C) Increases intestinal calcium and phosphate
child’s immunodeficiency and his autoim-
absorption
mune gut inflammation?
(D) Makes up the constituents of the visual
pigments (A) Antibodies
(E) Transfers one-carbon intermediates (B) IgA
(C) IgM
14. ABO testing on a sample of B-positive blood (D) Lysosomes
and two unknown sample types (donor and (E) Neutrophils
recipient) is performed. Positive antibody-
antigen interaction results in RBC agglutina- 16. A 3-year-old girl presents to the emergency

Full-Length Exams
tion. Five experiments are conducted for each department with two weeks of abdominal
sample, as shown in the image. Which of the pain. The mother denies any nausea, vomit-
following pairs represents the blood types for ing, or fever in the child. The child says that
donor and recipient? her hands feel “funny,” and she apparently
has been stumbling, while walking, more fre-
Blood group
quently over the past few months. Serum labo-
B+ Donor Recipient ratory tests are normal. The child’s peripheral
blood smear reveals stippling of RBCs. Poison-
1. anti-A
ing with which of the following substances is
2. anti-B most likely?
3. A cells (A) Arsenic
(B) Copper
4. B cells
(C) Iron
5. anti-D
(D) Lead
(E) Mercury
Test Block 5

No agglutination Agglutination

Reproduced, with permission, from USMLERx.com.


638 Section III: Full-Length Examinations  •  Questions

17. A mother brings her 1-month-old infant to the (C) Huntingtin


pediatrician. She says the baby is crying more (D) Presenilin
than usual, is vomiting, and does not want to (E) Tau
eat. Physical examination reveals a bulging
fontanel. Lumbar puncture shows: 19. A 60-year-old man is found to have advanced
adenocarcinoma of the stomach. The tumor is
Opening pressure: 240 mm H2O (normal: 100-
centered at the pyloric zone just near the py-
200 mm H2O)
loric sphincter, on the lesser curvature. Which
WBC count: 1200/mm³
of the following signs and symptoms is most
Protein: 200 mg/dL
likely to be seen in this patient due to the mass
Glucose: 30 mg/dL
effect of the tumor?
Gram stain: gram-positive rods
(A) Anemia
Which of the following organisms is most
(B) Constipation
likely responsible for this infant’s presentation?
(C) Hoarseness
(A) Escherichia coli (D) Jaundice
(B) Herpes simplex virus (E) Periumbilical swelling
(C) Listeria monocytogenes
(D) Neisseria meningitides 20. Kabuki make-up syndrome (KMS, or Niikawa-
(E) Streptococcus agalactiae Kuroki syndrome) is a very rare genetic dis-
order of unknown cause that presents in
18. A 68-year-old man suffered from a resting neonates as mental retardation, unusual skin
tremor and postural instability during his last ridging in the hands, fingers, and toes, and
five years of life. The neuropathology shown in an everted lower eyelid. Estimates of world-
the image was observed during postmortem ex- wide prevalence range from 1:100,000 to
amination. Mutations in which protein are ge- 1:10,000,000. If a new genetic test were de-
netically linked to the disease from which this veloped that was positive in 99 neonates with
man suffered? KMS and negative in one neonate with KMS,
Full-Length Exams

and additional studies demonstrated that this


test had a consistently high sensitivity and
specificity, what can be expected for the posi-
tive predictive value of this test?
(A) It will be high because of a low prevalence
(B) It will be high because of high accuracy
(C) It will be low because of a bias
(D) It will be low because of a low prevalence
(E) It will be low because of low accuracy

21. A 35-year-old woman is brought to the emer-


gency department because of diffuse muscle
contractions. On examination she is unable to
open her mouth. Her husband reports that last
week she accidentally stuck her finger with a
Reproduced, with permission, from USMLERx.com. rusty nail. By which of the following mecha-
nisms does this organism cause the symptoms
Test Block 5

associated with this disease?


(A) a-Synuclein
(B) Dystrophin (A) The organism produces a cytotoxin that
damages colonic mucosa
Test Block 5  •  Questions 639

(B) The organism produces a heat-labile toxin (C) Cabergoline


that stimulates adenylate cyclase (D) Chlorpromazine
(C) The organism produces an exotoxin that (E) Clozapine
blocks glycine release at spinal synapses
(D) The toxin blocks release of acetylcholine 24. A 20-year-old woman is referred to an endo-
at spinal synapses crinologist for lack of menarche. Physical ex-
(E) The toxin produced is a superantigen that amination reveals normal breast tissue and ex-
binds to MHC II protein and T lympho- ternal genitals. However, the vagina ends in a
cyte receptors blind pouch. Laboratory studies show elevated
levels of testosterone, estrogen, and luteinizing
22. A 62-year-old man comes to the physician hormone. Which of the following is character-
complaining of a skin rash that is extremely istic of this patient’s condition?
painful. A picture of the rash is shown. A pa-
(A) Deficiency of 5a-reductase
thology specimen is obtained. Which of the
(B) Elevated 17-hydroxyprogesterone
following results would most likely be seen on
(C) Excessive early gestational androgenic ex-
biopsy?
posure
(D) Karyotype of 46,XX or 47,XXY
(E) Unresponsive testosterone receptors

25. A newborn initially is healthy but begins to


have bilious emesis and fails to pass meconium
during the first 48 hours of life. An emergent
abdominal barium study is performed during
this time and results are shown in the image.
What is the pathophysiology of this disease?

Full-Length Exams
Courtesy of Wikipedia.

(A) Auer bodies


(B) Cabot ring bodies
(C) Call-Exner bodies
(D) Cowdry A inclusion bodies
(E) Mallory bodies

23.
A 32-year-old woman with schizophrenia
presents to the physician with amenorrhea
and a milky discharge from both nipples. A Reproduced, with permission, from USMLERx.com.
pregnancy test is negative. Laboratory results
show an increased serum prolactin level, a de-
creased serum gonadotropin level, and a nor- (A) Failure of neural crest cell migration
mal thyroid stimulating hormone level. Which (B) Failure of neural tube closure
Test Block 5

of the following medications is the most likely (C) Herniation of mucosal tissue
cause of this patient’s presentation? (D) Remnant portion of the vitelline duct
(E) Volvulus
(A) Amantadine
(B) Bromocriptine
640 Section III: Full-Length Examinations  •  Questions

26. An 18-year-old woman is brought to the pe- (A) Ceftriaxone


diatrician by her mother because of changes (B) Cyclophosphamide
in her behavior. The mother states that her (C) Methotrexate
daughter has always been “a bit strange,” often (D) Probenecid
keeping to herself and reading books on witch- (E) Tamoxifen
craft and UFOs. Recently her daughter has be-
gun to wear only black clothing and plays the 29. A 28-year-old man presents to the primary care
lute for many hours each day while wearing clinic because his thinking has been “slow”
a cape and a witch’s hat. The patient says she recently, citing for example that he has had
feels fine and just wants to be left alone, but trouble remembering the names of his friends.
the physician notes her speech is rather vague The patient also mentions that he has been
and shows little affect. She continues to per- feeling depressed, and that he has recently lost
form well in school and has few friends, but is a significant amount of weight. The patient
otherwise healthy. What personality type best reports no history of familial illnesses. Physi-
describes this patient? cal examination is notable for purplish skin
lesions distributed across his torso. CT of the
(A) Antisocial
brain shows diffuse volume loss. Which of the
(B) Avoidant
following are most consistent with his current
(C) Borderline
presentation?
(D) Paranoid
(E) Schizotypal (A) Ataxia, urinary incontinence, and seizures
(B) High fever, rigidity of the neck, and encap-
27. A previously healthy 5-year-old boy is brought sulated yeasts in cerebrospinal fluid
to the pediatrician with a three-day history of (C) Involuntary, dance-like movements of the
sore throat, conjunctivitis, rhinitis, and cough. arms and gross cortical atrophy
His mother explains that more than 10 chil- (D) Paralysis and atrophy of muscles localized
dren in his class at school have similar symp- to the extremities
toms, particularly conjunctivitis. No cultures (E) Progressively decreased mental status and
Full-Length Exams

are ordered, and the mother is assured that her amyloid plaque formation in the brain
son’s illness will go away on its own. One week
later, the mother reports that her son is healthy 30. A 20-year-old mother is unsure of the paternity
and back at school. Which of the following is of her newborn son. To determine the father
the most likely causative agent in this child’s of her child, a genetic test based on DNA re-
illness? striction fragment length polymorphism was
performed. Blood was drawn from the four
(A) Adenovirus
men suspected to be the father (F1, F2, F3,
(B) Coxsackie A virus
F4) as well as from the mother (M) and the
(C) Cytomegalovirus
infant (C). DNA extracted from the samples
(D) Herpes simplex virus type 1
was amplified using polymerase chain reac-
(E) Rotavirus
tion and then treated with the restriction en-
zyme EcoRI. The resulting fragments were
28. A 40-year-old woman presents with progressive
separated with gel electrophoresis and a South-
fatigue and bilateral joint inflammation char-
ern blot analysis was performed. According to
acterized by pain, swelling, warmth, and morn-
the Southern blot shown in the image, who is
ing stiffness. The patient says that the symp-
most likely the father of the child?
toms began in her hands over one year ago but
Test Block 5

have now begun to affect her knees. Which of


the following agents would be most useful in
her treatment?
Test Block 5  •  Questions 641

F1 F2 F3 F4 M C 32. A 53-year-old woman presents with irregular


menstrual periods. She claims that she some-
times goes two-three months without a period.
On further questioning, she also complains
10 Kb of vaginal dryness and occasional hot flashes.
Blood tests reveal an estrogen level of 22 pg/mL
9 Kb (normal: 60-400 pg/mL), follicle-stimulating
hormone (FSH) level of 100 mIU/L (normal:
1-26 mIU/L), and luteinizing hormone level
6 Kb of 50 mIU/mL (normal: 1-12 mIU/L), without
surge. Which of the following is most likely
4 Kb the primary cause of this patient’s symptoms?
2 Kb (A) Decreased estrogen levels
(B) Decreased feedback on the anterior pitu-
itary
Reproduced, with permission, from USMLERx.com.
(C) Increased FSH levels
(D) Increased progesterone levels
(A) F1 (E) Increased testosterone levels
(B) F2
(C) F3 33. A young couple presents to a fertility clinic,
(D) F4 reporting that they have been attempting to
conceive a child for 16 months without any
31. A 2-week-old premature male infant is exam- success. The 25-year-old wife has undergone
ined in the neonatal intensive care unit, and hormonal analysis, and it has been determined
shows a wide pulse pressure and a holosys- that she menstruates normally and her follicles
tolic and holodiastolic murmur. On echocar- are viable. The 27-year-old husband is 188 cm
diography he has blood flow between the left (6’2”) tall and weighs 64.4 kg (142 lb). On

Full-Length Exams
pulmonary artery and the aorta. Which of the examination, he has small testes. Karyotype
following symptoms would the mother have analysis is performed, and reveals the presence
experienced during pregnancy to increase the of an extra sex chromosome. Which of the fol-
risk of having a child with this disorder? lowing laboratory results is most consistent
with this man’s condition?
(A) A firm, nonpainful, red lesion on the out-
side of her vagina followed several weeks (A) Decreased follicle-stimulating hormone
later by a maculopapular rash on her level
palms (B) Decreased thyroid-stimulating hormone
(B) Maculopapular rash spreading from face to level
body (C) Decreased gonadotropin-releasing hor-
(C) Mild fever, sore throat, body aches, mal- mone level
aise, and swollen glands (D) Increased luteinizing hormone level
(D) Prolonged, persistent paroxysmal cough (E) Increased testosterone level
(E) Vaginal itching and mucopurulent dis-
charge
Test Block 5
642 Section III: Full-Length Examinations  •  Questions

34. A G2P1 woman at 39 weeks’ gestation is persistently elevated blood pressure with a
rushed to the hospital by her husband because mean arterial pressure of 120 mm Hg, but she
she is in labor. She elects to have epidural an- remains asymptomatic. Her physician would
esthesia for the delivery. After injection of the like to begin an antihypertensive agent but the
anesthetic agent, the woman complains of pal- patient refuses, fully understanding the risks,
pitations and severe dizziness. ECG shows evi- benefits, and outcomes that would result with
dence of heart block. Upon investigation it is or without treatment. What ethical principle
discovered that the resident did not ensure that has priority in the treatment of this patient?
the epidural needle did not pierce a vessel.
(A) Beneficence
Which of the following anesthetic agents was
(B) Informed consent
most likely administered for the procedure?
(C) Justice
(A) Bupivacaine (D) Non-maleficence
(B) Fentanyl (E) Patient autonomy
(C) Halothane
(D) Ibuprofen 37. A 19-year-old woman with no significant past
(E) Morphine medical history presents to her primary care
physician for a sports physical. Her examina-
35. Comparing the oxygen-hemoglobin dissocia- tion is notable for a brachial artery pressure
tion curves in the image, which of the follow- of 160/110 mm Hg and a weak femoral pulse.
ing conditions represents curve B as compared Prompted by the weak pulse, her physician
to curve A? measures her blood pressure in the lower ex-
tremity and finds it to be 80/40 mm Hg. An
x-ray film of the chest shows rib notching. This
woman is presenting with a congenital condi-
B tion that places her at high risk for bacterial
endocarditis and which of the following other
Hemoglobin saturation (%)

A
conditions?
Full-Length Exams

(A) Acute lymphocytic leukemia


(B) Boot-shaped heart
(C) Cerebral hemorrhage
(D) Cor pulmonale
(E) Eisenmenger syndrome

38. A 21-year-old man presents to a new primary


PO2 (mm Hg) care physician for routine physical examina-
tion. He demonstrates hyperextensible skin
Reproduced, with permission, from USMLERx.com. and reports a history of finger and shoulder
dislocations, which he has reduced himself.
He reports a history of easy bruising. He has no
(A) A higher concentration of fetal hemoglo-
cardiac abnormalities. A defect in the synthe-
bin
sis of which molecule likely accounts for these
(B) An elevated concentration of adult hemo-
symptoms?
globin
(C) Decreased pH (A) Collagen type I
Test Block 5

(D) High altitude (B) Collagen type II


(E) Increased temperature (C) Collagen type III
(D) Collagen type IV
36. A 27-year-old woman is involuntarily commit- (E) Elastin
ted to a psychiatric ward. Her physician notes
Test Block 5  •  Questions 643

(F) Fibrillin (E) Tracheal deviation away from the affected


(G) Sphingomyelinase lung

39. A 56-year-old man who is undergoing che- 42. A 7-year-old girl with no significant medical
motherapy for colorectal carcinoma develops history presents with a five-month history of
profound nausea and vomiting four hours af- persistent weakness despite taking vitamins
ter receiving treatments. The drug most likely and supplements. Physical examination is
to relieve the patient’s symptoms functions by completely benign, with normal blood pres-
which of the following mechanisms? sure and no peripheral edema. Laboratory
studies show hyponatremia, hypokalemia,
(A) Acetylcholine antagonist
metabolic alkalosis, and an increased plasma
(B) Dopamine agonist
renin level. Renal biopsy reveals juxtaglomeru-
(C) Norepinephrine reuptake inhibitor
lar cell hyperplasia. Which of the following di-
(D) Serotonin agonist
uretics acts at the same location as the defect
(E) Serotonin antagonist
in the patient’s syndrome?
40. A 15-month-old boy is brought to the pediatri- (A) Acetazolamide
cian by his parents because they have noticed (B) Furosemide
that he has difficulty walking. On physical ex- (C) Hydrochlorothiazide
amination, the child exhibits a broad-based (D) Spironolactone
waddling gait. Laboratory studies show a serum (E) Triamterene
calcium level of 6.0 mg/dL, a serum phosphate
level of 2.0 mg/dL, and a serum alkaline phos- 43. A 47-year-old man presents with diarrhea, ab-
phatase activity of 85 U/L. Which of the fol- dominal pain, loss of appetite, weight loss, and
lowing is the most likely cause of this patient’s fatigue. A jejunal biopsy is obtained and is
condition? shown in the image. Which of the following is
the most likely diagnosis?
(A) Decreased bone mineral density
(B) Diminished hydroxylation of proline and

Full-Length Exams
lysine in collagen synthesis
(C) Diminished intestinal absorption of cal-
cium and phosphate
(D) Increased bone turnover
(E) Renal failure

41. A 24-year-old man who has smoked 1.5 packs


per day for the past six years comes to the local
emergency room complaining of sudden onset
of unilateral pleuritic chest pain and shortness
of breath. He is tall and thin. He has no per-
sonal or family history of medical illnesses and
denies trauma prior to the onset of his pain.
On examination, he is tachypneic and has de-
creased breath sounds at the right upper zone Reproduced, with permission, from USMLERx.com.
of the lung. Which of the following additional
physical findings is likely to be present?
Test Block 5

(A) Disaccharidase deficiency


(A) Bilateral chest expansion (B) Chronic pancreatitis
(B) Bronchial breath sounds (C) Infection by Tropheryma whipplei
(C) Hyperresonance on percussion (D) Tropical sprue
(D) Increased tactile fremitus (E) Celiac sprue
644 Section III: Full-Length Examinations  •  Questions

44. A 28-year-old woman comes to the physician’s (A) A lumbar puncture


office complaining of anxiety and a recent 4.5- (B) A transesophageal echocardiogram
kg (9.9-lb) weight loss. Her physical examina- (C) Coagulation studies
tion is significant for an inability to fully cover (D) Tissue pressure in the lower extremities
her eyes with her eyelids and swelling on the (E) Venous Doppler ultrasound imaging of the
anterior surface of both legs. The skin of her lower extremities
anterior legs appears dry and waxy and has sev-
eral diffuse, slightly pigmented papules. Labo- 46. A 42-year-old man comes to the physician
ratory studies show low levels of serum thyroid- complaining of abdominal pain for the past
stimulating hormone (TSH) and high levels three months. His pain is sharp, and is worse
of serum total thyroxine (T4) and serum free after meals and when lying down. He also
T4. The drug of choice for this disorder acts at mentions having black, tarry stools for the past
what step in thyroid hormone synthesis? month. Laboratory tests show:
(A) Active absorption of iodide from the blood Hemoglobin: 11.2 g/dL
(B) Oxidation of I- to I2 Hematocrit: 35%
(C) Production of TSH Mean corpuscular volume: 98 fL
(D) Proteolysis of colloid to release T4 and tri- WBCs: 7000/mm³
iodothyronine Platelets: 260,000/mm³
(E) Tyrosine iodination and coupling Endoscopy of the stomach reveals a small flat
lesion on the fundus; it has smooth borders
45. A 61-year-old alcoholic presents to the emer-
and is filled with exudate. Which of the follow-
gency department with disorientation, confu-
ing is a major risk factor for his condition?
sion, and an unsteady gait. Horizontal nystag-
mus, pulmonary râles, and edematous lower (A) Bacterial infection
extremities are noted on physical examina- (B) Celecoxib usage
tion. On questioning, the patient states that (C) Diabetes
he started drinking alcohol when he was a (D) Hypertension
Full-Length Exams

prisoner of war in Vietnam. His current alco- (E) Vegan diet


hol level is within the legal limit, a toxicology
screen is negative, and a stroke has been ruled
out by imaging. Which of the following ad-
ditional tests should be performed to estimate
the extent of his disease?
Test Block 5
Test Block 5  •  Questions 645

47. A patient with long-standing renal failure sec- 48. A 40-year-old woman presents with a persistent
ondary to focal segmental glomerulosclero- cough of two months’ duration. She reports a
sis undergoes parathyroid biopsy that shows 6.8-kg (15-lb) unintentional weight loss and
marked hyperplasia. On physical examination, general loss of appetite. She says she drinks
tapping over the cheek elicits facial muscle wine occasionally but has never smoked ciga-
spasm. Which of the following sets of labora- rettes. X-ray of the chest reveals a subpleural
tory values is most likely to be seen in this pa- mass. Which of the following is the most likely
tient? diagnosis?
(A) Adenocarcinoma of the lung
Parathyroid Serum Serum Alkaline (B) Carcinoid tumor of the lung
Choice
hormone Ca2+ phosphate phosphatase (C) Metastases to lung from a primary tumor
A from another tissue type
B (D) Small-cell carcinoma of the lung
C (E) Squamous cell carcinoma of the lung
D
E

Reproduced, with permission, from USMLERx.com.

(A) A
(B) B
(C) C
(D) D
(E) E

Full-Length Exams
Test Block 5
646 Section III: Full-Length Examinations  •  Answers

An s w e r s

1. The correct answer is B. This patient most therefore supersedes even a living will. The pa-
likely has multiple endocrine neoplasia tient, in good state of mind, believed that the
(MEN) type 2 (formerly known as type 2A), a friend would make decisions with which he
genetic disorder characterized by tumors of the would agree. It is always appropriate to facili-
parathyroid gland, medullary carcinoma of the tate a discussion between people involved in
thyroid, and tumors of the adrenal medulla. making end-of-life decisions, but it is unethical
All MEN syndromes follow an autosomal dom- to try to sway the friend by making the choice
inant mode of inheritance with incomplete for him or telling him to listen to the patient’s
penetrance. MEN 2 is thought to result from son or the living will.
an activating mutation in the proto-oncogene Answer A is incorrect. The durable power of
ret. MEN type 1 involves parathyroid hyperpla- attorney should always make decisions consis-
sia, pancreatic islet cell tumors, and pituitary tent with what he believes the patient would
adenomas, whereas MEN type 3 (formerly want. The physician should not advise the
known as type 2B) involves medullary thyroid friend to comply with the son’s wishes, but
carcinomas, mucosal neuromas, and pheo- rather remind them both as to the responsibili-
chromocytomas. Medullary carcinoma of the ties of the durable power of attorney.
thyroid is a calcitonin-secreting tumor of para-
follicular thyroid cells (“C cells”). Microscopi- Answer B is incorrect. Only the patient can
cally, the tumor consists of nests of tumor cells change his/her power of attorney. If the patient
in an amyloid-filled stroma. is non-communicative, the power of attorney
cannot be transferred. Many states acknowl-
Answer A is incorrect. Atrophic follicles with edge next of kin as durable power of attorney,
prominent germinal center formation and lym- unless it has been otherwise specifically as-
phocyte infiltrate are characteristic of Hashi- signed, as in this case.
moto thyroiditis.
Answer C is incorrect. The durable power of
Full-Length Exams

Answer C is incorrect. A papillary branching attorney is the ultimate decision maker and
pattern of epithelial cells with ground-glass their decision to withdraw life support will be
nuclei and psammoma bodies (laminated con- upheld despite the existence of a living will.
centric calcified spherules) is seen in papillary
carcinoma of the thyroid, the most common Answer D is incorrect. The durable power of
form of thyroid cancer and also the form with attorney was appointed as such to make deci-
the best prognosis. sions in accordance with the patient’s wishes.
Input from family members is both important
Answer D is incorrect. Sheets of undifferenti- and appropriate, but is not legally necessary.
ated pleomorphic cells are seen in anaplastic,
or undifferentiated, thyroid cancer. This form 3. The correct answer is C. A presentation of
of thyroid cancer is more common in older pa- fullness, fatigue, and weight loss along with
tients and has a very poor prognosis. splenomegaly immediately raises the clini-
Answer E is incorrect. Follicular carcinoma cian’s suspicion of a possible slowly progressing
of the thyroid can resemble normal thyroid hematopoietic neoplasm. Hairy cell leukemia
tissue. It is composed of relatively uniform fol- is caused by malignant B lymphocytes that
licles lined with cells that are typically larger commonly show varying numbers of projec-
Test Block 5

than those seen in a normal thyroid. Colloid is tions from cytoplasm, giving the cell a “hairy”
sparse. or “ruffled” appearance as seen in the image
above. It is four times more prevalent in men
2. The correct answer is E. The appointed du- than in women, and patients usually complain
rable power of attorney is truly durable and of abdominal fullness, fatigue, and weight loss
Test Block 5  •  Answers 647

but rarely of night sweats or fevers. Elevation sitis. In combination, statins and fibrates have
of tartrate-resistant acid phosphatase in the an additive risk of myositis.
B lymphocytes from bone marrow confirms
Answer A is incorrect. The adverse effects of
the diagnosis of hairy cell leukemia. Think
ezetimibe include diarrhea, abdominal dis-
“hair trap.”
comfort, and arthralgias.
Answer A is incorrect. Chronic lymphocytic
Answer B is incorrect. The adverse effects of
leukemia is also a B lymphocyte-derived neo-
fibrates include myositis and gastrointestinal
plasm whose presentation is very similar to that
(GI) discomfort. The combination of fibrates
of hairy cell leukemia. Its B lymphocytes, how-
with statin medications has an additive effect
ever, would not typically demonstrate an eleva-
that might more frequently result in myositis.
tion in tartrate-resistant acid phosphatase.
Answer C is incorrect. Simvastatin and ezeti-
Answer B is incorrect. Follicular lymphomas
mibe combined can cause elevation in liver
are the most common type of indolent non-
transaminases, so patients taking these drugs
Hodgkin lymphomas and are characterized by
together should have periodic liver function
an increase in number of normal-appearing
tests.
germinal centers, which are not described here.
Answer E is incorrect. Whereas statins can
Answer D is incorrect. Mantle cell lymphoma
cause myositis, the most common adverse ef-
is a B lymphocyte subtype of non-Hodgkin
fects of niacin are flushing and GI upset.
lymphoma that is characterized by small cells
with cleaved nuclei resembling the cells in 5. The correct answer is B. A Bell palsy is a le-
germinal centers. Although its clinical pre- sion of cranial nerve VII (facial nerve) and af-
sentation can be similar to that of hairy cell fects the muscles of facial expression. These
leukemia, it lacks the “hairy” appearance his- muscles are derived from the second branchial
tologically and does not show any increase in (pharyngeal) arch. Patients with sarcoidosis,
tartrate-resistant acid phosphatase. tumors, diabetes, AIDS, and Lyme disease are
Answer E is incorrect. All Hodgkin lym- at increased risk for Bell palsy, although most

Full-Length Exams
phoma variants are differentiated by the pres- cases are idiopathic. Important signs of Bell
ence of Reed-Sternberg cells and commonly palsy are ptosis and facial droop. The second
present clinically with night sweats, fevers, and arch also gives rise to the posterior belly of
weight loss. The nodular sclerosis variant is dis- the digastric, the stylohyoid, and the stapedius
tinguished by a nodular pattern separated by muscles.
areas of collagen banding and the presence of Answer A is incorrect. The first branchial arch
lacunar cells. develops into the muscles of mastication, the
mylohyoid, the anterior belly of the digastric,
4. The correct answer is D. Myositis and rhab-
the tensor veli palatini, and the tensor tym-
domyolysis are potential complications of
pani. These muscles are innervated by cranial
co-treatment with a statin drug and a fibric
nerve V and are not affected by Bell palsy.
acid derivative. Although this combination is
not explicitly contraindicated, the two drugs Answer C is incorrect. The third branchial
should be used together with caution. Statin pouch develops into the inferior parathyroid
drugs decrease LDL substantially but also pose glands and the thymus. It is implicated in Di-
a risk of myositis. Fibrates cause a decrease George syndrome but has no relation to Bell
in triglyceride levels and an increase in HDL palsy.
Test Block 5

cholesterol levels by promoting lipolysis and Answer D is incorrect. The fourth branchial
decreasing the secretion of triglycerides by the pouch develops into the superior parathyroid
liver, and are also associated with a risk of myo- glands. It is implicated in DiGeorge syndrome
but is not related to Bell palsy.
648 Section III: Full-Length Examinations  •  Answers

Answer E is incorrect. The thyroglossal duct at times because a large fraction of vitamin B12
connects the thyroid diverticulum to the fore- is bound to protein and therefore unavailable
gut in the embryo but is obliterated during de- for other metabolic processes.
velopment. Its only remnant in the adult is the
Answer E is incorrect. Urine vitamin B12 lev-
foramen cecum.
els are not normally measured.
6. The correct answer is B. This patient has
7. The correct answer is A. The lactose (lac)
a macrocytic anemia with hypersegmented
operon is an example of an inducible operon.
neutrophils, a condition most likely caused
Glucose is the preferred energy source for
by either vitamin B12 deficiency or folate defi-
the E coli bacterium and in the presence of
ciency. Increased homocysteine levels are in-
glucose, lac operon transcription is inhibited
dicative of folate deficiency or vitamin B12 de-
by a repressor protein that binds to the opera-
ficiency, but are not helpful in distinguishing
tor region of the lac operon and blocks RNA
between the two.
polymerase. In the absence of glucose and the
Answer A is incorrect. Any patient with a mac- presence of lactose, lactose binds the repressor
rocytic anemia and hypersegmented neutro- molecule and changes its shape so that it can
phils could have a folate deficiency. However, no longer bind to the operator, allowing tran-
this is highly unlikely in this patient who in- scription to occur. Furthermore, as glucose
gests a diet high in leafy green vegetables. levels decrease, cAMP levels rise and bind to
the cAMP receptor protein (CRP). The CRP-
Answer C is incorrect. This patient has a mac-
cAMP complex binds to the operon and pro-
rocytic anemia with hypersegmented neutro-
motes the binding of RNA polymerase to the
phils. Two causes of these findings on blood
promoter. When the cells are exposed to both
smear are vitamin B12 deficiency and folate
glucose and lactose, lactose does bind the re-
deficiency. A primary dietary deficiency can
pressor and release its repression. However, be-
often be seen in people who maintain a strict
cause the presence of glucose decreases levels
vegan diet for many years with no vitamin
of cAMP, the CRP-cAMP complex formation
supplements, since vitamin B12 is primarily ob-
Full-Length Exams

decreases consequently and RNA polymerase


tained from animal products. Foods rich in vi-
does not efficiently bind the promoter. Thus,
tamin B12 include eggs, milk, fish, poultry, and
under normal conditions, if the bacterium is
meats. Using the blood levels of vitamin B12
grown in the presence of both lactose and glu-
as a clue to deficiency, however, can be mis-
cose, the lac operon should be mostly inactive.
leading because a large fraction of this vitamin
In this vignette, glucose fails to suppress tran-
is bound to protein and therefore unavailable
scription of the lac operon. Therefore, the mu-
for other metabolic processes. Methylmalonic
tation is most likely located in the CRP.
acid is a product of methylmalonyl CoA. In
normal metabolism, methylmalonyl CoA is Answer B is incorrect. Lactose is the inducer
converted to succinyl CoA with the cofactor vi- of the lactose operon. Lactose binds tightly
tamin B12. If there is not enough vitamin B12 to the repressor so that the repressor can
present, methylmalonyl CoA is alternatively no longer bind to the operator and block the
converted into methylmalonic acid. There- RNA polymerase from transcribing the prod-
fore, vitamin B12 deficiencies can be diagnosed ucts of the lac operon. If the inducer-binding
based on high methylmalonic acid levels. site were mutated, the lactose operon would
not be expressed in the presence of lactose.
Answer D is incorrect. Although this patient
Test Block 5

has a macrocytic anemia with hypersegmented Answer C is incorrect. A mutation in the pro-
neutrophils, the serum vitamin B12 level is moter of the lactose operon is not consistent
not the best test to determine whether the pa- with the above observation. If the promoter
tient is deficient in folate or vitamin B12. The were mutated, the ability of RNA polymerase
blood levels of vitamin B12 can be misleading
Test Block 5  •  Answers 649

to bind to it would be altered regardless of the Answer E is incorrect. Patients with osteope-
presence of glucose or lactose. trosis may often present with anemia. How-
ever, this is not due to a defect in RBC matura-
Answer D is incorrect. In the absence of lac-
tion. It is due to the crowding effect of limited
tose, the lactose operon repressor binds to the
intramedullary space.
operator and halts transcription. In this case,
a mutation in the repressor protein would ei-
9. The correct answer is E. The patient is expe-
ther increase or decrease repressor binding to
riencing a type IV, or delayed-type, hypersen-
the operator, which would alter the amount of
sitivity reaction. These immune reactions re-
products produced. No such changes were ob-
quire prior exposure to the triggering antigen
served, which makes it unlikely that the muta-
and typically manifest 24-48 hours after new
tion is located in the repressor.
contact with a trigger. Type IV hypersensi-
Answer E is incorrect. If RNA polymerase tivity is mediated by the action of T lympho-
were mutated in these bacterial cells, all gene cytes, which secrete cytokines that, in turn,
expression would be affected. recruit additional inflammatory cells to the
site of contact. T-lymphocyte activation de-
8. The correct answer is D. The most likely di- pends on interaction between the cell surface
agnosis for this child is osteopetrosis. This rare T-lymphocyte receptor and an antigen-MHC
hereditary disorder occurs from a failure of the complex on an antigen-presenting cell.
resorption and remodeling of bone due to mal-
Answer A is incorrect. RBCs are partially re-
functioning osteoclasts. The skeleton becomes
sponsible for the transport and buffering of
diffusely sclerotic and dense as new bony ma-
acid generated by tissue metabolism. RBC
trix is laid into the medullary canal, replacing
carbonic anhydrase converts carbon dioxide to
the hematopoietic tissue. Patients compensate
carbonic acid, which rapidly dissociates into a
with extramedullary hematopoiesis, leading to
proton and a bicarbonate ion. The bicarbon-
hepatosplenomegaly. Despite the increased
ate ion is in turn exchanged across the RBC
density, the bone is brittle and predisposed to
membrane in a process termed the physiologic
fracture. There are two main types of the dis-

Full-Length Exams
chloride shift. None of these processes is in-
ease, characterized by their inheritance pat-
volved in delayed-type hypersensitivity.
terns. The autosomal-recessive form is more
malignant and is often fatal in utero or in the Answer B is incorrect. Cell surface immuno-
neonatal period. The autosomal-dominant globulin receptors are characteristic of B lym-
form is usually benign and may be discovered phocytes. Through their production of anti-
incidentally on x-ray. bodies, B lymphocytes are involved in types I,
II, and III hypersensitivity, but are unrelated to
Answer A is incorrect. Osteopetrosis presents
the pathophysiology of delayed-type hypersen-
with hepatomegaly due to the need for extra-
sitivity.
medullary hematopoiesis, not a defect of the
intrinsic cellular makeup of the liver. Answer C is incorrect. Eosinophils are espe-
cially important in the immune system’s re-
Answer B is incorrect. Despite finding leuko-
sponse to parasites and may also be associated
penia in osteopetrosis, the root cause is not a
with neoplastic processes. Eosinophils are not,
defect in the lymphoid progenitor cells. Their
however, involved in the physiology of delayed-
normal proliferation is prevented by the filling
type hypersensitivity.
of the intramedullary space with bony tissue.
Answer D is incorrect. Degranulation and his-
Test Block 5

Answer C is incorrect. In osteopetrosis, the


tamine release are characteristic of mast cells
function of the osteoblasts goes unchecked by
and basophils. Both are important in mediat-
malfunctioning osteoclasts. Though there is an
ing type I hypersensitivity reactions via their
excess of bony material, osteopetrosis is not a
cell surface IgE Fc receptors, but are not in-
neoplastic process.
volved in delayed-type hypersensitivity.
650 Section III: Full-Length Examinations  •  Answers

Answer F is incorrect. Nonspecific phagocy- disease, because either the body cannot pro-
tosis in the epidermal layer is characteristic of duce enough of the necessary protein product
Langerhans cells, a differentiated type of den- from just one functioning allele or the mu-
dritic cell. They are found in lymph nodes, tated allele produces an altered gene product
where they act as antigen-presenting cells. that is antagonistic to the wild-type allele. An
example of this is osteogenesis imperfecta,
Answer G is incorrect. Opsonization refers to
which caused by mutations in the COL1A1
the “marking” of foreign antigens either by an-
gene. The abnormal gene product from the
tibody binding or by complement binding. In
mutated allele incorporates itself into the col-
either case, certain phagocytic cells, predomi-
lagen matrix, weakening the structure.
nately macrophages, can subsequently inter-
nalize and destroy the antigens. Although mac- Answer C is incorrect. Oncogenes acquire
rophages may be recruited by cytokines to the gain-of-function mutations that lead to in-
site of a delayed-type hypersensitivity reaction, creased activity of the gene product, which
it is the T lymphocyte that mediates the reac- causes uninhibited cellular proliferation. This
tion. mutational event typically occurs in a single
allele of the oncogene and acts in a dominant
10. The correct answer is D. BRCA1 and BRCA2 fashion. An example of this is the c-Myc gene,
are tumor-suppressor genes whose protein which is implicated in Burkitt lymphoma.
products function in DNA repair. Frameshift
Answer E is incorrect. Viral insertion de-
or nonsense mutations commonly occur in
scribes the process in which a viral gene is in-
BRCA1 and BRCA2 and produce truncated
serted into the host cell and promotes malig-
protein products. Mutations in these genes
nant transformation. Human papilloma virus
result in a gene product that has less or no
inserts its genes into the cellular DNA. Several
function, and can lead to DNA instability and
of the viral sequences act as oncogenes that
subsequent gene rearrangements. Both alleles
promote tumor growth and may cause cervi-
of these tumor-suppressor genes must be inac-
cal and anal cancers. The BRCA1 and BRCA2
tivated to cause loss of function of these genes.
genes are tumor suppressor genes, and have
Full-Length Exams

Mutations to BRCA can increase the risk of


not been shown to be inactivated by viral inser-
many cancers aside from breast cancer.
tion.
Answer A is incorrect. The chromosomal al-
terations in human solid tumors are heteroge- 11. The correct answer is B. The patient has bi-
neous and complex, and allow for selection of temporal hemianopia secondary to a lesion
the loss of tumor suppressor genes on the in- compressing the optic chiasm. The history of
volved chromosome. However, in leukemias menstrual irregularities, cold intolerance, con-
and lymphomas, the chromosomal alterations stipation, and increased skin pigmentation sug-
are often simple translocations, in which the gests multiple endocrine disorders that can be
breakpoints of chromosomal arms occur at attributed to dysfunction of the anterior pitu-
the site of cellular oncogenes. For example, itary. Due to the close anatomic relationship of
the Philadelphia chromosome in chronic lym- the pituitary and the optic chiasm, expanding
phocytic leukemia is produced from a recipro- lesions of the pituitary can compress the optic
cal translocation involving the ABL oncogene chiasm, leading to visual loss in the temporal
(a tyrosine kinase on chromosome 9) being portions of the visual fields bilaterally. This is
placed in proximity to the BCR (breakpoint because only the nasal (or medial) retinal fi-
cluster region) on chromosome 22. The ex- bers for each eye cross in the optic chiasm,
Test Block 5

pression of the BCR-ABL gene product leads and the medial retinal fibers are responsible
to tumorigenic growth. for the temporal (or lateral) hemifields as de-
picted in the drawing above. Common lesions
Answer B is incorrect. Dominant negative ef-
leading to bitemporal hemianopia include pi-
fects occur when the loss of one allele leads to
Test Block 5  •  Answers 651

tuitary adenoma, meningioma, craniopharyn- 12. The correct answer is A. Cromolyn is given
gioma and hypothalamic glioma. by aerosol for the prophylactic treatment of
asthma. The mechanism of action involves sta-
Answer A is incorrect. This defect is mon-
bilization of mast cells and thus a decrease in
ocular visual loss caused by complete destruc-
the release of mediators (eg, leukotrienes and
tion of the ipsilateral retina or ipsilateral optic
histamine) responsible for bronchoconstric-
nerve. A lesion in this area would not account
tion. These drugs are insoluble and thus have
for this patient’s endocrine abnormalities.
only local effects. They are used only in the
Answer C is incorrect. This defect is contra- prophylaxis of acute asthma, not during an at-
lateral homonymous hemianopia, which can tack.
be caused by lesions of the contralateral optic
Answer B is incorrect. Epinephrine is an ac-
tract (in this case, the right optic tract), a lesion
ceptable treatment and is often the drug of
of the contralateral optic radiation, or lesions
choice for a severe acute asthma attack. This
diffusely damaging the contralateral primary
autonomic activator relaxes smooth muscle, fa-
visual cortex. In general, retrochiasmal lesions
cilitating breathing during an acute attack.
(those distal to the optic chiasm including the
optic tracts, lateral geniculate nucleus, optic Answer C is incorrect. Ipratropium is an ac-
radiations, or visual cortex) cause homony- ceptable treatment for an acute asthma attack.
mous visual field defects (meaning the same It works as a competitive blocker of muscarinic
regions of the fields for both eyes are involved). receptors, preventing bronchoconstriction.
Lesions in these areas would not account for
Answer D is incorrect. Terbutaline is an ac-
this patient’s endocrine abnormalities.
ceptable treatment for an acute asthma attack.
Answer D is incorrect. This defect is contralat- This drug relaxes bronchial smooth muscle.
eral superior quadrantanopia due to lesions in
Answer E is incorrect. Theophylline is an ac-
the contralateral Meyer’s loop (or inferior optic
ceptable treatment for an acute asthma attack.
radiations) through the temporal lobe. The in-
It likely causes bronchodilation by inhibiting
ferior optic radiations carry information from
phosphodiesterase, decreasing cAMP hydroly-

Full-Length Exams
the inferior retina or the superior visual field.
sis.
Inferior optic radiations terminate in the infe-
rior aspect of the primary visual cortex in the 13. The correct answer is E. Folic acid plays a key
occipital lobe. Therefore, lesions of the contra- role as a coenzyme for one-carbon transfer as
lateral inferior primary visual cortex could lead seen in methylation reactions and is essential
to contralateral superior quadrantanopia. How- for the biosynthesis of purines and the pyrimi-
ever, lesions in these areas would not account dine thymidine. Deficiency of the vitamin is
for this patient’s endocrine abnormalities. characterized by growth failure in children
Answer E is incorrect. This defect is contra- and macrocytic megaloblastic anemia. The
lateral inferior quadrantanopia due to lesions anemia is a result of diminished DNA syn-
in the contralateral superior optic radiations thesis in erythropoietic stem cells. Large cells
that pass under the parietal lobe. The superior are seen with mean cell volumes of 100-150
optic radiations carry information from the su- fL and reduced levels of hemoglobin. Folic
perior retina or the inferior visual field. Supe- acid is a water-soluble vitamin stored in small
rior optic radiations terminate in the superior amounts by the body; thus a continuous sup-
aspect of the primary visual cortex in the oc- ply is needed from foods such as green, leafy
cipital lobe. Therefore, lesions of the contralat- vegetables, lima beans, and whole-grain cere-
Test Block 5

eral superior primary visual cortex could lead als. Nutrient deficiency can commonly lead
to contralateral inferior quadrantanopia. How- to folate deficiency. The deficiency is usually
ever, lesions in these areas would not account seen in pregnant women and alcoholics, and
for this patient’s endocrine abnormalities. is the most common vitamin deficiency in
the United States. Folic acid supplementation
652 Section III: Full-Length Examinations  •  Answers

by pregnant women reduces the incidence of dryness of conjunctiva and cornea, leading to
neural tube defects. corneal ulceration and ultimately blindness.
Answer A is incorrect. Vitamin K serves as a
14. The correct answer is E. Because the donor’s
coenzyme in the γ-carboxylation of glutamic
RBCs do not agglutinate with anti-A or anti-B
acid residues in blood clotting proteins. A vi-
antibodies, the donor blood type must be O.
tamin K deficiency is rarely seen because ad-
The fact that donor sera agglutinates both A
equate amounts are generally produced by
cells and B cells confirms this, because type O
intestinal bacteria or easily obtained from the
serum contains both anti-A and anti-B antibod-
diet. Decreased bacterial production in the
ies. By similar reasoning, the recipient must
gut (as with antibiotics, for example) can lead
be blood type A. Anti-D agglutination implies
to hypoprothrombinemia and, subsequently,
that RBCs in a sample have the Rh antigen.
hemorrhage. Newborns have sterile intestines
Thus the donor is Rh negative, and the recipi-
and cannot initially synthesize vitamin K. Be-
ent is Rh positive. Hence the correct pair is
cause human milk fails to provide the ade-
an O-negative donor and A-positive recipient.
quate daily requirement of vitamin K, it is rec-
Donor and recipient blood should always be
ommended that all newborns receive a single
cross-matched, because pre-formed antibodies
dose of vitamin K as prophylaxis against hem-
in the recipient can lyse donor RBCs and vice
orrhagic diseases.
versa. This immunologic response can culmi-
Answer B is incorrect. Ascorbic acid (vita- nate in an acute hemolytic transfusion reac-
min C) acts as a coenzyme in hydroxylation tion with sequelae of shock, pyrexia, and both
of prolyl- and lysyl- residues of collagen, allow- chest and flank pain.
ing collagen fibers to crosslink and providing
Answer A is incorrect. Anti-A antibodies do
greater tensile strength to the assembled fiber.
not agglutinate donor RBCs. Thus the donor
A deficiency of ascorbic acid results in scurvy,
cannot be type A.
a disease characterized by sore, spongy gums,
loose teeth, fragile blood vessels, swollen Answer B is incorrect. Anti-A and anti-B anti-
joints, anemia, and poor wound healing. bodies do not agglutinate donor RBCs. Thus
Full-Length Exams

the donor cannot be type AB.


Answer C is incorrect. 1,25-Dihydroxychole-
calciferol, the active molecule of vitamin D, Answer C is incorrect. Anti-A and anti-B an-
produces its effect at the DNA level to produce tibodies do not agglutinate donor RBCs. Thus
proteins in intestinal cells that allow for greater the donor cannot be blood type AB. No ag-
calcium and phosphate absorption. Vitamin D glutination implies that the antigen is not pre­
deficiency causes a net demineralization of sent on the sample RBCs (hence this donor is
bone, resulting in rickets in children and os- type O). Both A and B RBCs are agglutinated
teomalacia in adults. Rickets is characterized in the donor serum, as expected, because anti-
by continuous formation of collagen matrix of A and anti-B antibodies are generated in pa-
bone but incomplete mineralization, resulting tients with blood type O.
in soft, pliable bones. In osteomalacia, demin- Answer D is incorrect. Anti-B antibodies do
eralization of preexisting bones increases their not agglutinate donor RBCs. Thus the donor
susceptibility to fracture. cannot be type B.
Answer D is incorrect. Vitamin A (retinol) is Answer F is incorrect. Donor RBCs do not
a component of the visual pigments of rod and agglutinate with either anti-A or anti-B anti-
cone cells. This fat-soluble vitamin plays an es-
Test Block 5

body. Thus the donor blood type must be O.


sential role in vision, growth, maintenance of Because anti-D agglutination is negative, the
epithelial cells, and reproduction. Night blind- donor must be Rh negative. The donor blood
ness is one of the earliest signs of vitamin A type is therefore O negative, not O positive.
deficiency as a result of a loss in the number
of visual cells. Further deficiency can lead to
Test Block 5  •  Answers 653

15. The correct answer is E. Chronic granu­ levels of only 10 μg/dL. Toxic effects include
lomatous disease (CGD) is a disorder in which abdominal pain, peripheral neuropathy, and
neutrophils are unable to completely eradicate basophilic RBC stippling. Additional findings
certain phagocytosed bacteria and fungi. As a might include lead lines along the gingival
result, the chronic immune response to these and cognitive impairment.
lingering pathogens leads to the development
Answer A is incorrect. Acute arsenic poisoning
of self-tissue damage. This answer provides a
results in immediate GI distress. Cardiac insta-
logical link between a correct description of
bility, respiratory distress, and death sometimes
CGD (a disorder of neutrophil production of
follow.
nicotinamide adenine dinucleotide phospha-
tase [NADPH] oxidase, which results in neu- Answer B is incorrect. Wilson disease results
trophils that are unable to completely eradi- from inadequate hepatic copper excretion and
cate certain phagocytosed bacteria and fungi) failure of copper to enter circulation as ceru-
and the development of autoimmune disease. loplasmin. It is characterized by asterixis, par-
This answer points to theories questioning why kinsonian symptoms, dementia, and hemolytic
50% of patients with CGD suffer a chronic gut anemia.
inflammation that is similar to Crohn disease. Answer C is incorrect. Iron poisoning is char-
Answer A is incorrect. Common variable im- acterized by vomiting, diarrhea, GI bleeding,
munodeficiency, the most common sympto­ cyanosis, and metabolic acidosis.
matic primary antibody deficiency syndrome, Answer E is incorrect. Mercury toxicity is
is a disorder characterized by differing degrees characterized by intention tremor, nephrotox-
of deficiency of antibody production, leading icity, and change in personality.
to recurrent sinopulmonary and GI infections.
Answer B is incorrect. This choice is not the 17. The correct answer is C. Increased irritabil-
correct immunologic deficiency of CGD. The ity, feeding difficulty, and other general non-
disorder characterized by a deficiency of IgA specific signs along with a bulging fontanel
antibodies is called IgA deficiency, the most characterize meningitis in infants. The cere-

Full-Length Exams
common primary immunodeficiency disease brospinal fluid (CSF) findings point toward a
in the Western hemisphere. bacterial meningitis (increased protein and
decreased glucose). In infants 0-3 months old,
Answer C is incorrect. Wiskott-Aldrich syn- the most common organisms causing meningi-
drome is an X-linked disorder that results in tis are Listeria monocytogenes, Escherichia coli,
the body being unable to mount an IgM re- and Group B streptococci. L monocytogenes is
sponse to capsular polysaccharides or bacteria. the only gram-positive rod in the group.
It is associated with low levels of IgM, high lev-
els of IgA, and normal levels of IgE. Answer A is incorrect. Escherichia coli can
cause bacterial meningitis in infants, but a
Answer D is incorrect. Chédiak-Higashi syn- Gram stain of CSF would show gram-negative
drome is an autosomal recessive disease affect- rods.
ing a lysosomal trafficking regulator gene; it
leads to defective microtubular function and Answer B is incorrect. Herpes simplex vi-
lysosomal emptying of phagocytic cells. This rus has also been shown to cause meningitis
disease is characterized by a partial oculocu- in infants. In viral meningitis, the CSF find-
taneous albinism, abnormally large granules ings would show far fewer leukocytes (11-500/
found in many different cell types, and recur- mm³), protein levels between 50 and 200 mg/
Test Block 5

rent pyogenic staphylococcal and/or strepto- dL, and normal glucose levels; a Gram stain
coccal infections. would not show any organisms.
Answer D is incorrect. Neisseria meningitidis
16. The correct answer is D. Toxicities associ- also causes bacterial meningitis but is com-
ated with lead poisoning begin at blood lead
654 Section III: Full-Length Examinations  •  Answers

monly seen in older children. It is also a gram- other neurodegenerative disorders including
negative diplococcus. Alzheimer disease, Pick disease, and progres-
sive supranuclear palsy present with abnormal
Answer E is incorrect. Streptococcus agalac-
cytoplasmic accumulations of tau protein.
tiae can cause bacterial meningitis in infants,
Lewy bodies are not associated with these neu-
but a Gram stain of CSF would show gram-
rodegenerative disorders.
positive cocci.
19. The correct answer is D. This question tests
18. The correct answer is A. This patient suffered
the concept of mass effect of tumors, as well as
from Parkinson disease (PD), a neurodegenera-
anatomy. These two topics are inseparable and
tive disorder of the substantia nigra and locus
are necessary to understanding the etiology of
ceruleus that is most often characterized by
some symptoms seen in the context of neo-
Tremor, Rigidity, Akinesia, and Postural in-
plasms. The adenocarcinoma impinges on the
stability (TRAP). The photomicrograph shows
omental foramen, which is formed partly by
several neurons from the substantial nigra
the hepatoduodenal ligament. This ligament
stained with haematoxylin and eosin at 500
contains the common bile duct along with
times magnification, at least two of which (ar-
the hepatic artery proper and the hepatic por-
rows) exhibit large Lewy bodies (eosinophilic
tal vein. Obstruction of the common bile duct
cytoplasmic inclusions that consist of a dense
would lead to cholestasis and subsequently
core surrounded by a halo of 10-nm wide ra-
conjugated hyperbilirubinemia.
diating fibrils). These aggregates are the major
pathological feature of PD, and are composed Answer A is incorrect. Anemia may be a sign
primarily of the protein a-synuclein. Some in- of GI bleeding (seen with stomach or colonic
herited forms of PD have been shown to be cancers) or that bone marrow is dysfunctional
caused by mutations in the a-synuclein gene. and/or being replaced with malignant cells. Al-
though anemia may be seen in this patient due
Answer B is incorrect. Dystrophin is the gene
to bleeding into the stomach, anemia is not a
disrupted in Duchenne muscular dystrophy.
direct result of mass effect of the tumor.
This disease is characterized by progressive
Full-Length Exams

muscle weakness and does not have any signifi- Answer B is incorrect. Constipation may be a
cant neuropathology. symptom of obstruction of the left colon.
Answer C is incorrect. Trinucleotide repeat Answer C is incorrect. Persistent hoarseness
expansions inside the huntingtin protein cause could be a manifestation of impingement of
Huntington disease (HD), a neurodegenerative the recurrent laryngeal nerve. This symptom
disease that primarily affects the caudate nu- may be seen with thyroid or lung cancer.
cleus and clinically presents with chorea and
Answer E is incorrect. Although the tumor
dementia. This patient’s clinical symptoms and
can metastasize to the periumbilical region to
pathology are not consistent with HD.
form a subcutaneous nodule, known as a Sis-
Answer D is incorrect. Mutations in the pre- ter Mary Joseph nodule, the direct mass effect
senilin gene have been linked to early forms of the tumor does not affect the periumbilical
of Alzheimer disease (AD), the most common region.
cause of dementia in the elderly. Neuropatho-
logically, AD presents with darkly staining neu- 20. The correct answer is D. As the disease preva-
rofibrillary tangles and extracellular amyloid lence decreases, the likelihood of a positive
plaques. Resting tremor and Lewy bodies are test being a true-positive decreases. In diseases
Test Block 5

not associated with AD. with very low prevalence, a positive test is
more likely a false-positive. In this case, if the
Answer E is incorrect. Mutations in tau cause
prevalence was 1:1,000,000, and the sensitivity
frontotemporal dementia with parkinsonism
and specificity are both 99%, then performing
linked to chromosome 17 (FTDP-17). Several
the test on 100,000,000 people will yield 99
Test Block 5  •  Answers 655

true-positives and 999,999 false-positives, for toms. While cranial nerves are often the first to
a positive predictive value of approximately be affected, the paralysis is flaccid rather than
0.00009. contractile. Furthermore, the method of trans-
mission is often through improperly canned
Answer A is incorrect. As disease prevalence
goods and honey (in babies) rather than rusty
decreases, the positive predictive value will de-
nails.
crease, not increase.
Answer E is incorrect. Staphylococcus aureus
Answer B is incorrect. Accuracy measures va-
causes disease by producing a toxin that acts
lidity and not reliability, but changes between
as a superantigen and binds to MHC II pro-
different tests instead of between prevalences.
tein and T lymphocyte receptors. This leads
Positive predictive value is a measure that
to toxic shock characterized by fever, rash, and
changes as prevalence changes
shock.
Answer C is incorrect. There is no reason to
suspect bias in this test. Bias would imply that 22. The correct answer is D. The image shows
the information given is inaccurate. grouped vesicular lesions following the distri-
bution of a dermatome unilaterally. This is the
Answer E is incorrect. Accuracy measures va-
classic appearance of herpes zoster. Cowdry A
lidity and not reliability, but changes between
inclusion bodies are seen in pathology prepara-
different tests instead of between prevalences.
tions of herpes zoster skin rashes. They are in-
Positive predictive value is a measure that
tranuclear eosinophilic inclusions surrounded
changes as prevalence changes
by a clear halo.
21. The correct answer is C. Strong muscle con- Answer A is incorrect. Auer bodies, or Auer
tractions and trismus (contraction of the jaw rods, are rod-shaped bodies in myeloid cells.
muscles) are symptoms of tetanus. Tetanus is They appear primarily in acute promyelocytic
caused by Clostridium tetani spores that are leukemia (M3) and are made of fused lyso-
found in soil. They usually enter the body somes. Care must be taken in treating these
through a puncture wound. The disease is patients because release of Auer rods may lead

Full-Length Exams
caused by the exotoxin produced by the bacte- to disseminated intravascular coagulation.
rium. The exotoxin blocks glycine and GABA
Answer B is incorrect. Cabot ring bodies are
release, preventing the inhibitory signal from
ring-shaped structures found in RBCs in severe
reaching motor neurons downstream, thus pre-
cases of megaloblastic anemia.
disposing motor neurons to tonic contraction,
or tetanus. Answer C is incorrect. Call-Exner bodies are
spaces between granulosa cells in ovarian fol-
Answer A is incorrect. Clostridium difficile
licles and in granulosa cell tumors.
causes pseudomembranous colitis by produc-
ing a cytotoxin (the A-B toxin) that kills entero- Answer E is incorrect. Mallory bodies, or al-
cytes and causes pseudomembranous colitis, coholic hyaline bodies, are accumulations of
not lockjaw. eosinophilic material in the cytoplasm of dam-
aged hepatic cells. They are commonly found
Answer B is incorrect. Escherichia coli pro-
in hepatocytes of patients with alcoholic hepa-
duces a heat-labile toxin that stimulates
titis whose livers would also show cirrhosis and
adenylate cyclase by adenosine diphosphate
fatty change.
ribosylation of G proteins, which then causes
watery diarrhea.
Test Block 5

23. The correct answer is D. Chlorpromazine


Answer D is incorrect. Clostridium botuli- is an antipsychotic agent used to treat schizo-
num causes botulism by producing a toxin that phrenia and psychosis. It acts by blocking do-
blocks the release of acetylcholine at spinal pamine D2 receptors (some have posited that
synapses and can cause anticholinergic symp- schizophrenia may result from aberrant do-
656 Section III: Full-Length Examinations  •  Answers

paminergic signal transduction). Because do- tients often present in infancy as a result of am-
pamine inhibits the secretion of prolactin, a biguous genitals and salt-wasting.
dopamine antagonist such as chlorpromazine
Answer C is incorrect. Excessive exposure
can cause hyperprolactinemia.
to androgenic steroids during early gestation
Answer A is incorrect. Amantadine is used leads to female pseudohermaphroditism (XX),
for influenza A prophylaxis. It causes release a condition in which ovaries are present, but
of dopamine from intact nerve terminals, and the external genitalia are virilized.
thus is sometimes used to treat Parkinson dis-
Answer D is incorrect. Very rare true her-
ease. A relative excess of dopamine would not
maphroditism is karyotype 46,XX or 47,XXY
account for this woman’s presentation, nor is
with ambiguous genitals.
there any suggestion that she might be taking
amantadine. 25. The correct answer is A. Hirschsprung dis-
Answer B is incorrect. Bromocriptine is a do- ease is characterized by a lack of Auerbach and
pamine agonist that decreases (rather than Meissner plexuses, both of which are compo-
increases) prolactin levels. It can actually be nents of the nerve plexus in the large intes-
used to treat hyperprolactinemia. tine. These plexuses are composed of cells that
have migrated from the neural crest. Auerbach
Answer C is incorrect. Cabergoline, like bro-
plexus is found between the outer circular and
mocriptine, is a dopamine agonist that de-
longitudinal muscle layers that are responsible
creases prolactin levels.
for motility. Meissner plexus is in the submu-
Answer E is incorrect. One of the atypical an- cosal layer. Failure of these plexuses to develop
tipsychotic agents, clozapine blocks both sero- leads to the characteristic megacolon (colonic
tonin and dopamine receptors. It tends to have dilation) seen on this radiograph.
fewer extrapyramidal adverse effects (of which
Answer B is incorrect. Failure of neural tube
hyperprolactinemia is one example) than typi-
closure during embryogenesis leads to neural
cal antipsychotics such as chlorpromazine.
tube defects such as anencephaly and spina bi-
Patients taking clozapine, however, must be
Full-Length Exams

fida, both of which would be obvious at birth.


screened regularly for the development of
Folic acid supplementation by women prior to
agranulocytosis. Although clozapine might
conception decreases the fetus’ risk of develop-
cause hyperprolactinemia, chlorpromazine is
ing neural tube defects, although folic acid’s
much more likely to do so.
mechanism of action is unknown.
24. The correct answer is E. This patient has tes- Answer C is incorrect. Herniation of mucosal
ticular feminization syndrome (androgen in- and submucosal tissue at the cricopharyngeus
sensitivity), which is due to unresponsiveness muscle (junction of the pharynx and esopha-
of the testosterone receptor protein to andro- gus) describes Zenker diverticulum. This is a
genic stimulation. false diverticulum (does not involve the mus-
cular layers) with presenting symptoms of hali-
Answer A is incorrect. A deficiency of 5a-
tosis (malodorous breath), dysphagia, and re-
reductase presents with ambiguous genitals
gurgitation of undigested food.
until puberty, when there is masculinization
of the genitals. Testosterone and estrogen Answer D is incorrect. A remnant portion
levels are normal. of the vitelline duct is describing Meckel di­
verticulum. It commonly is located in the
Answer B is incorrect. 17-Hydroxyprogester-
Test Block 5

antimesenteric border of the ileum, usually


one is elevated in congenital adrenal hyper-
within 60 cm of the ileocecal valve. Meckel
plasia (CAH) commonly as a result of a defi-
diverticulum can be remembered with the
ciency of 21a-hydroxylase, or 11a-hydroxylase
“rule of 2s”: 2 inches long; 2 feet from the il-
enzymes necessary for proper steroid synthesis.
eocecal valve; 2% of the population get it;
Although there are many variants of CAH, pa-
Test Block 5  •  Answers 657

commonly presents in the first 2 years of life; Answer D is incorrect. Paranoid patients have
and may have 2 types of epithelium (gastric a pervasive mistrust and suspiciousness of oth-
and/or pancreatic). Ectopic acid-secreting gas- ers without odd behavior.
tric mucosa found in Meckel diverticulum can
cause ulcers, which may bleed and result in 27. The correct answer is A. Adenovirus is a major
bloody stool. cause of epidemic keratoconjunctivitis (pink
eye). It is the fourth most common cause of
Answer E is incorrect. Volvulus occurs when
childhood respiratory tract infections, after
a portion of the bowel rotates around its mes-
respiratory syncytial virus, parainfluenza, and
entery, leading to bowel obstruction, ischemia,
rhinovirus. It is a naked, icosahedral, double-
and possible perforation with resultant perito-
stranded linear DNA that results in a self-
nitis. Whereas midgut volvulus can occur in
limited illness that requires no treatment.
infants with congenital intestinal malrotation
and in the cecum in young adults, volvulus is Answer B is incorrect. Coxsackie A virus
most common in redundant segments of the causes cold symptoms and rashes. It is also the
colon such as the sigmoid in elderly, debili- causative agent of herpangina and hand, foot,
tated patients. and mouth disease.
Answer C is incorrect. Cytomegalovirus
26. The correct answer is E. Schizotypal patients
(CMV) can reactivate and cause a variety of
are often described by others as odd. They dis-
illnesses in the immunocompromised but is
play magical thinking, idiosyncratic thought
usually asymptomatic in healthy individuals.
processes, and unusual beliefs or behaviors.
Schizotypal patients also manifest unusual Answer D is incorrect. Herpes simplex virus
perceptions, suspiciousness, constricted affect, type 1 causes gingivostomatitis, herpetic kerati-
lack of close relationships, and social anxiety. tis of the eye, and encephalitis.
Patients with schizotypal personality disorder Answer E is incorrect. Rotavirus is the most
are at a higher-than-average risk of develop- common cause of diarrhea in infants less than
ing schizophrenia. Under stress, people with 3 years old.

Full-Length Exams
schizotypal personality may decompensate and
have psychotic symptoms, but these are brief 28. The correct answer is C. This patient is pre-
and fragmentary. Excessive social anxiety is as- senting with signs suggestive of rheumatoid
sociated with paranoid fears. arthritis. Methotrexate, a folic acid analog
Answer A is incorrect. Antisocial patients have antimetabolite that inhibits dihydrofolate re-
a disregard for the rights of others, are often ductase, is often used as treatment due to re-
in trouble with the law, and show a lack of re- duction of adenosine-mediated inflammatory
morse for wrongful acts. changes. Methotrexate is used in a number of
neoplastic conditions, including breast carci-
Answer B is incorrect. Avoidant patients are noma, head and neck carcinoma, lung carci-
socially inhibited, often have feelings of in- noma, choriocarcinoma, acute lymphoblastic
adequacy, and are hypersensitive to criticism. leukemia, and non-Hodgkin lymphoma. Other
Although both avoidant and schizoid patients non-oncologic uses of methotrexate include
have minimal social contacts, avoidant patients ectopic pregnancy, psoriasis, and inflammatory
desire relationships but do not pursue them bowel disease.
out of fear of rejection, whereas schizoid pa-
tients genuinely prefer solitude. Answer A is incorrect. Ceftriaxone is a third-
Test Block 5

generation cephalosporin antibiotic that in-


Answer C is incorrect. Borderline patients hibits bacterial transpeptidase and cell wall
show instability in relationships, self-image, synthesis. It is most commonly used to treat se-
and emotions. They often manifest impulsive rious gram-negative infections, including men-
behaviors and parasuicidal behavior such as ingitis and gonorrhea. Although gonorrhea can
cutting themselves with a sharp object.
658 Section III: Full-Length Examinations  •  Answers

present with unilateral arthritis of the knee, slowed cognition, depressed mood, ataxia, sei-
this patient’s clinical presentation is more con- zures, and urinary and bowel incontinence
sistent with rheumatoid arthritis. Ceftriaxone later in the course of the disease. Symptoms
would therefore not be an effective treatment result from HIV-induced demyelination of
for this patient. Notably, tetracyclines can be neurons in the central nervous system. The pa-
used to inhibit the activity of metalloprotein- tient’s recent weight loss supports the diagnosis
ases involved in joint destruction by the rheu- of HIV infection, and purplish skin lesions are
matoid synovium, and are therefore effective consistent with Kaposi sarcoma, a cancer that
agents in patients with early rheumatoid arthri- predominantly affects patients with AIDS.
tis.
Answer B is incorrect. High fever, rigidity of
Answer B is incorrect. Cyclophosphamide is the neck, and encapsulated yeasts in CSF are
an alkylating agent that is useful in the treat- characteristic of meningitis caused by Cryp-
ment of non-Hodgkin lymphoma and breast tococcus neoformans. Although patients with
and ovarian carcinomas. It is also used as an AIDS are at higher risk for developing crypto-
immunosuppressant in systemic lupus eryth­ coccal meningitis, the patient’s current presen-
ematosus, multiple sclerosis, and autoimmune tation does not suggest this etiology.
hemolytic anemia. It is not generally used as a
Answer C is incorrect. Involuntary, dance-like
treatment for rheumatoid arthritis.
movements of the extremities is descriptive of
Answer D is incorrect. Probenecid is an or- the chorea of Huntington disease, an autoso-
ganic acid that is used most commonly for mal dominant disorder that involves dementia,
the treatment of chronic tophaceous gout or psychosis, and gross motor dysfunction, pro-
increasingly frequent gouty attacks. The drug gressing to death. CT scan or autopsy often re-
acts at the anionic transport sites in the renal veals gross cortical atrophy.
tubule to inhibit the reabsorption of uric acid,
Answer D is incorrect. Paralysis and atrophy
promoting its secretion. Gout normally pre­
of the extremities is characteristic of infection
sents with intermittent acute inflammatory
with poliovirus.
arthritis, most often at only one site. In more
Full-Length Exams

chronic disease, more joints become involved Answer E is incorrect. Progressively decreased
and the intervals between attacks become mental status and amyloid plaque formation
shorter. Advanced gout results in chronic ar- are characteristic of Alzheimer disease. Given
thropathy, characterized by persistent asym- the patient’s young age, absence of family his-
metric and asynchronous joint inflammation tory, and associated symptoms, there is no rea-
accompanied by uric acid deposits known as son to suspect Alzheimer disease.
tophi, and can occasionally resemble rheuma-
toid arthritis. Nevertheless, the progressive and 30. The correct answer is B. F2 is most likely the
steady nature of this patient’s disease strongly father of this child. Restriction site polymor-
suggests a diagnosis of rheumatoid arthritis phisms are characteristic sites on individual al-
rather than gout. Probenecid would therefore lele that are recognized by restriction enzymes.
be an ineffective pharmacologic therapy. These sites can be inherited along with the
chromosomes on which they reside. There-
Answer E is incorrect. Tamoxifen is an estro- fore, every fragment in the child’s Southern
gen receptor mixed agonist-antagonist that is blot should be found in either the father’s or
most useful against estrogen-sensitive breast the mother’s profile. The child could have re-
cancers. ceived the 6-kb fragment from the mother (M)
Test Block 5

and the 9-kb fragment from F2.


29. The correct answer is A. This patient’s clinical
description is consistent with AIDS, and his Answer A is incorrect. F1 is unlikely to be the
symptoms are consistent with AIDS complex father of this child. The child could have re-
dementia (ADC). Symptoms of ADC include ceived the 6-kb fragment from his mother (M),
Test Block 5  •  Answers 659

but he could not have received the 9-kb frag- Secondary syphilis presents with skin lesions
ment from either the mother or F1. on the palms or soles, condylomata lata, and
systemic symptoms of lymphadenopathy, fe-
Answer C is incorrect. F3 is unlikely to be the
ver, malaise, and weight loss. Syphilis is one of
father of this child. The child could have re-
the ToRCHeS infections. Congenital syphilis
ceived the 6-kb fragment from his mother (M),
classically manifests as Hutchinson’s triad of
but he could not have received the 9-kb frag-
notched central incisors, blindness, and deaf-
ment from either the mother or F3.
ness due to cranial nerve VIII injury. Maternal
Answer D is incorrect. F4 is unlikely to be the transmission most frequently occurs during the
father of this child. The child could have re- primary or secondary stage of syphilis.
ceived the 6-kb fragment from his mother (M),
Answer C is incorrect. These symptoms are
but he could not have received the 9-kb frag-
nonspecific and are seen commonly in infec-
ment from either the mother or F4.
tious mononucleosis. Although infectious
31. The correct answer is B. The child has a pat- mononucleosis is caused by Epstein-Barr virus,
ent ductus arteriosus (PDA), identified in this a similar picture can occur in some healthy
question by its role in fetal circulation. This individuals who are infected by CMV. CMV
causes exercise intolerance, a wide pulse pres- is one of the ToRCHeS organisms, but it typi-
sure, and a continuous machine-like murmur. cally causes mental retardation, microcephaly,
Indomethacin may be used to close PDA in and deafness; CMV does not commonly cause
the neonate, but older children will require PDA.
surgery or catheter placement. An important Answer D is incorrect. These are symptoms of
risk factor for PDA is congenital rubella. Ru- whooping cough, caused by Bordetella pertus-
bella is a mild, self-limited illness in adults, sis. Although a vaccine exists to prevent infec-
which manifests itself as a maculopapular rash tion by B pertussis, this is not one of the organ-
beginning in the face and spreading down the isms known to cause congenital disease and is
body. It is one of the ToRCHeS (Toxoplasmo- not a risk factor for PDA.
sis, Rubella, Cytomegalovirus, Herpesvirus/

Full-Length Exams
Answer E is incorrect. These symptoms are
HIV, and Syphilis) organisms that can cross
suggestive of a gonococcal infection. Gonor-
the placenta and cause congenital disease.
rhea is an important cause of pelvic inflamma-
There is no specific treatment for rubella, but
tory disease, infertility, and ectopic pregnan-
a vaccine exists that can prevent maternal in-
cies, but does not cross the placenta and is not
fection and thus significantly reduce the risk
associated with PDA.
of congenital PDA. The Centers for Disease
Control and Prevention warn that rubella vac-
32. The correct answer is A. Menopause is the
cine should not be given to a pregnant woman
cessation of estrogen production because of
because of a possible risk to the fetus. Admin-
a loss of ovarian sensitivity to gonadotropin
istering the rubella vaccine to a woman be-
stimulation caused by ovarian dysfunction and
fore she becomes pregnant, however, would
a decreased number of available ovarian fol-
lower the risk of congenital PDA in any fu-
licles. In the United States, the average age at
ture offspring. Most ToRCHeS infections pre­
onset of menopause is 51 years, with a normal
sent as mild or asymptomatic infections in the
range between 45-55 years. Remember, meno-
mother, but can cause a variety of congenital
pause causes HAVOC: Hot flashes, Atrophy
defects when transmitted to the fetus.
of the Vagina, Osteoporosis, and Coronary ar-
Test Block 5

Answer A is incorrect. A firm, non-painful, red tery disease (CAD). Estrogen is necessary for
genital lesion is suggestive of primary syphilis the maintenance and development of the va-
with the chancre at the site of treponemal en- gina and bone deposition, so a decrease leads
trance. This heals in three to six weeks regard- to vaginal atrophy and osteoporosis. There
less of whether or not the individual is treated. is no direct link between estrogen and heart
660 Section III: Full-Length Examinations  •  Answers

disease, but the incidence of CAD following father can contribute this extra chromosome,
menopause is two to three times higher than and the origin of the chromosome appears to
in premenopausal women, suggesting some make no difference. The patient’s karyotype is
protective effect of endogenous estrogen. Hot 47,XXY. In Klinefelter syndrome, the testicles
flashes are related to changes in the ability of are nonfunctional, and therefore testosterone
the hypothalamus to recognize body tempera- levels are decreased, resulting in a secondary
ture. Because estrogen replacement therapy increase in gonadotropin levels (FSH and LH)
alleviates these symptoms, there may be a role due to the lack of negative feedback. Patients
for estrogen in body temperature regulation. with Klinefelter syndrome tend also to have
The estrogen level in this patient is low (<100 the following signs: They are tall, long-limbed,
pg/ml), and levels of follicle-stimulating hor- and have a distribution pattern of fat and hair
mone (FSH) and luteinizing hormone (LH) more typical of women. They also tend to have
are elevated. FSH and LH levels are elevated gynecomastia, or enlargement of the breasts.
in menopause because of decreased feedback
Answer A is incorrect. Because of testicu-
inhibition from estrogen, but these levels are
lar dysfunction, FSH secondarily becomes
not the primary cause of menopause.
increased, not decreased, in Klinefelter syn-
Answer B is incorrect. Decreased negative drome.
feedback on the pituitary occurs secondary
Answer B is incorrect. Thyroid-stimulating
to decreased levels of estrogen. A primary de-
hormone (TSH), a measure of thyroid func-
crease in negative feedback would lead to in-
tion, has no clinical application in diagnosing
creased levels of FSH and LH and therefore
Klinefelter syndrome.
increased levels of estrogen. However, this pa-
tient shows symptoms of decreased estrogen. Answer C is incorrect. Gonadotropin-releasing
hormone will be increased secondary to the di-
Answer C is incorrect. Although increased
minished testosterone level.
FSH levels are found in menopause, this is
secondary to decreased estrogen levels, which Answer E is incorrect. Klinefelter syndrome
decrease negative feedback on the anterior pi- causes testicular atrophy. This results in de-
Full-Length Exams

tuitary. The increased FSH level itself is not creased, not increased, testosterone.
known to be the cause of menopausal symp-
toms. The symptoms are due to low levels of 34. The correct answer is A. Bupivacaine is an
estrogen. amide-based local anesthetic. It has greater
cardiotoxic effects than other drugs in this class
Answer D is incorrect. High levels of proges- and can thus produce arrhythmias and hypo-
terone may result in fatigue, depression, and tension if used intravenously.
vaginal dryness. However, this woman’s symp-
toms are more characteristic of menopause, Answer B is incorrect. Fentanyl is an opioid
which is due to low levels of estrogen. and thus its toxicity profile is similar to that of
morphine. Adverse effects include constipa-
Answer E is incorrect. Increased testosterone tion, nausea, vomiting, respiratory depression,
levels in a woman are classically seen in con- and coma.
genital adrenal hyperplasia or polycystic ovar-
ian syndrome and lead to masculine features Answer C is incorrect. Halothane is a gas-
such as hirsutism. Testosterone levels are typi- eous anesthetic used for both its analgesic and
cally low in menopause. muscle relaxant properties. It is hepatotoxic
and may cause malignant hyperthermia in ge-
Test Block 5

33. The correct answer is D. A common cause netically predisposed patients or when used in
of male infertility is Klinefelter syndrome. It conjunction with depolarizing paralytic agents
occurs in one out of about every 900 men. A such as succinylcholine. It also leads to a sig-
patient with this condition has an extra X chro- nificant decrease in cardiac output and blood
mosome, and either the patient’s mother or the pressure.
Test Block 5  •  Answers 661

Answer D is incorrect. Ibuprofen, a nonste- the treatment, are defining characteristics of


roidal anti-inflammatory drug (NSAID), may competence. In a competent adult, a physi-
cause gastric ulceration and renal failure due cian is obligated to respect patient autonomy
to interaction with kidney autoregulation. over the principles of non-maleficence and be-
neficence. Beneficence and non-maleficence
Answer E is incorrect. Morphine, an opioid,
may take precedence with minors, those who
may cause constipation, nausea, vomiting, res­
are physically or mentally impaired, and in
piratory depression, and coma.
emergency situations. Patients are admitted
35. The correct answer is A. Curve B represents involuntarily only for the treatment of psychi-
hemoglobin with greater affinity (lower par- atric conditions. Even then, unless they pose
tial pressure of oxygen at which the hemoglo- a risk to themselves or others, competent pa-
bin is 50% saturated) for oxygen compared to tients can refuse psychiatric treatment. There-
curve A. Of the answers provided, only increas- fore, the physician should respect the patient’s
ing the concentration of a2γ2 hemoglobin wishes and refrain from treating the hyperten-
(fetal hemoglobin, HbF) will shift the curve sion.
to the left. In adult hemoglobin (HbA), the Answer A is incorrect. Beneficence implies
two γ chains have been replaced with two b that an intervention is in a patient’s best inter-
chains. The oxygen affinity of HbF is higher est. All interventions have this characteristic
than that of HbA because of a lower affinity for in modern medicine but cannot be instituted
2,3-biphosphoglyceric acid (BPG) (from hav- without patient consent.
ing a neutral serine in the BPG binding site
Answer B is incorrect. Informed consent im-
compared to a positive histamine in HbA), en-
plies patient acknowledgment of the risks,
abling oxygen to be transferred from mother to
benefits, and alternatives of a procedure or
fetus. Continued production of HbF in adults
treatment, and also implies the patient’s auton-
is seen in patients with b thalassemia due to a
omous permission for the physician to inter-
decreased or absent production of b chain.
vene. Informed consent is generally obtained
Answer B is incorrect. Hemoglobin a2b2 is before a procedure, and is not applicable in

Full-Length Exams
HbA, the normal HbA. Elevated hemoglobin this case.
increases the total oxygen content carried in
Answer C is incorrect. The ethical principle
blood, but it does not affect the affinity of he-
of justice implies that the benefits and bur-
moglobin for oxygen.
dens placed on patients are distributed fairly
Answer C is incorrect. A decrease in pH, among patients. By having offered the patient
through the stabilization of deoxyhemoglobin an established treatment for hypertension, the
by hydrogen, results in a shift of the curve to physician has followed the principle of justice.
the right. This answer is incorrect, because patient au-
Answer D is incorrect. High altitude, through tonomy takes precedence to justice.
the increased concentration of 2,3-diphos- Answer D is incorrect. Non-maleficence is the
phoglycerate in RBCs, results in a shift of the principle that the physician will do no harm to
curve to the right. the patient, and therefore implies that risks of a
Answer E is incorrect. Increased temperature treatment are superseded by the benefits. Such
promotes the offloading of oxygen in tissues by a treatment can be offered by physicians but
shifting the curve to the right. cannot be instituted without patient consent.
Test Block 5

36. The correct answer is E. Patient autonomy 37. The correct answer is C. A blood pressure in
in a legally competent patient is paramount. the upper extremity significantly greater than
Understanding the risks and benefits of a treat- in the lower extremity, a weak to nonexistent
ment, as well as outcomes with and without femoral pulse, and rib notching on chest x-ray
are all consistent with postductal coarctation
662 Section III: Full-Length Examinations  •  Answers

of the aorta. This condition is associated with a Answer A is incorrect. Collagen type I is the
high risk of bacterial endocarditis and cerebral primary component of bone, skin, tendon,
hemorrhage. Postductal coarctation is caused dentin, fascia, cornea, and late wound repair.
by an abnormal constriction of the aorta distal The most common form of osteogenesis im-
to the ductus arteriosus during fetal develop- perfecta occurs as a result of an autosomal
ment. dominant defect in type I collagen. Osteogen-
esis imperfecta is characterized by multiple
Answer A is incorrect. Acute lymphocytic
fractures, blue sclerae, hearing loss, and dental
leukemia is associated with Down syndrome,
imperfections. Because of the multiple frac-
which is also associated with an increased risk
tures, this disorder often is confused with child
of an atrial septal defect. However, this is not
abuse.
associated with coarctation of the aorta.
Answer B is incorrect. Collagen type II is the
Answer B is incorrect. A boot-shaped heart re-
primary component of cartilage (including
fers to the cardiac silhouette produced in cases
hyaline), vitreous body, and the nucleus pulpo-
of isolated right ventricular hypertrophy, clas-
sus. Abnormalities in type II collagen result in
sically seen in tetralogy of Fallot. While aortic
a fatal form of osteogenesis imperfecta in utero
coarctation may theoretically lead to right ven-
or in the neonatal period.
tricular hypertrophy, it will do so only after the
left ventricle has hypertrophied and thus will Answer D is incorrect. Collagen type IV is
not produce the boot-shaped silhouette. the primary component of basement mem-
branes or the basal lamina of the kidney, ears,
Answer D is incorrect. Cor pulmonale is de-
and eyes. Alport syndrome, characterized by
fined as heart failure secondary to lung disease.
progressive hereditary nephritis and deafness,
If lung disease produces pulmonary hyperten-
is identified most commonly as an X-linked re-
sion, it will lead to right-sided failure. Since
cessive disorder.
aortic coarctation is not lung disease, it cannot
be associated with cor pulmonale. Answer E is incorrect. Elastin is a stretchy pro-
tein within the lungs, large arteries, elastic liga-
Answer E is incorrect. Eisenmenger syn-
Full-Length Exams

ments, vocal cords, and ligamenta flava that is


drome is the secondary development of cya-
broken down by elastase. a1-antitrypsin defi-
nosis in conditions that produce a left-to-right
ciency results in excess elastase activity, which
shunt, such as ventricular septal defects. The
is a major cause of emphysema.
increased blood flow in the pulmonary circu-
lation leads to pulmonary hypertension, which Answer F is incorrect. Marfan syndrome re-
raises the pressure on the right side of the sults from a defect in fibrillin, the major com-
heart, eventually reversing the shunt. Because ponent of microfibrils found in the extracellu-
blood is now shunted right to left, avoiding the lar matrix. These patients display bilateral lens
pulmonary circulation, cyanosis develops. Aor- subluxation or dislocation, distinctive skeletal
tic coarctation does not produce a left-to-right abnormalities, and aortic aneurysms (dilation
shunt and thus does not lead to Eisenmenger of the aortic ring resulting in aortic incompe-
syndrome. tence), as well as incompetent mitral and tri-
cuspid valves.
38. The correct answer is C. Ehlers-Danlos syn-
Answer G is incorrect. Sphingomyelinase de-
drome is characterized by defects in the syn-
fects characterize Niemann-Pick disease. This
thesis or structure of collagen type III. Accord-
disease is characterized by progressive neuro-
ingly, patients with this disorder have collagen
Test Block 5

degeneration, hepatosplenomegaly and a char-


that lacks tensile strength, and they demon-
acteristic cherry-red spot on the macula.
strate hyperextensible skin and hypermobile
joints. Because of a defect in connective tissue,
39. The correct answer is E. The first-line therapy
patients with this disorder are more susceptible
for treating severe nausea and vomiting due to
to berry aneurysms.
Test Block 5  •  Answers 663

chemotherapy is ondansetron. It is the stron- then is further hydroxylated to 1,25-dihydroxy-


gest available antiemetic, surpassing more cholecalciferol in the kidney.
common agents, such as metoclopramide, in
Answer A is incorrect. A decrease in bone
its ability to decrease symptoms. The mecha-
mineral density is typical of osteoporosis.
nism of action of ondansetron is blockade of
Patients with osteoporosis are usually older
serotonin 5-HT3 receptors. Adverse effects of
women, unlike the patient in this vignette.
ondansetron are headache and constipation.
Patients with osteoporosis tend to suffer from
Answer A is incorrect. Anticholinergic drugs fractures including vertebral compression frac-
include atropine, benztropine, scopolamine, tures and hip fractures. Patients with osteopo-
and ipratropium. These drugs decrease para- rosis will have normal serum levels of phos-
sympathetic activity by blocking muscarinic phate, calcium, and alkaline phosphatase.
receptors. Scopolamine is commonly used
Answer B is incorrect. A deficiency in the hy-
to treat motion sickness but would not be the
droxylation of proline and lysine in collagen
first-line therapy for chemotherapy-induced
synthesis is typically a result of ascorbic acid,
nausea and vomiting.
or vitamin C, deficiency. This usually results
Answer B is incorrect. Dopamine agonists in- in the clinical syndrome known as scurvy,
clude bromocriptine, L-dopa, pramipexole, which is not consistent with the clinical sce-
and amantadine. These drugs are mainly used nario described. Patients with scurvy may have
for the treatment of Parkinson disease and are bleeding gums and unhealed wounds. This vi-
not indicated for nausea and vomiting. tamin deficiency is usually the result of insuf-
ficient dietary intake, the so-called “tea and
Answer C is incorrect. Tricyclic antidepres-
toast” diet.
sants (TCAs) such as imipramine and amitrip-
tyline act by inhibiting presynaptic reuptake of Answer D is incorrect. An increase in bone
norepinephrine, thus augmenting their effect turnover is typical of Paget disease of bone.
on postsynaptic receptors. TCAs are primarily This disorder is marked by excessive bone re-
used to treat depression. Their adverse effects sorption followed by excessive bone formation.

Full-Length Exams
include sedation, a-blocking effects, and anti- It results in disorganized bone formation that
cholinergic properties. is more likely to result in fracture than nor-
mal bone and may result in deafness through
Answer D is incorrect. Serotonin agonists in-
restructuring of the bony surroundings of the
clude selective serotonin reuptake inhibitors
ear. Furthermore, patients are susceptible to
such as paroxetine and sertraline, which are
the formation of sarcomas. In Paget disease
antidepressants.
of bone the serum alkaline phosphatase level
40. The correct answer is C. The vignette de- is highly elevated. The patient in this vignette
scribes a patient with rickets, the clinical syn- does not meet the clinical manifestations of
drome that results from vitamin D deficiency. Paget disease.
Hallmarks of this condition include a broad- Answer E is incorrect. Although a patient with
based waddling gait, bending of long weight- renal failure would also exhibit a deficiency
bearing bones on radiographs, and hypocal- of vitamin D (because it is synthesized in the
cemia with low to normal serum phosphate kidneys), one would expect to see an elevated
levels and elevated serum alkaline phosphatase phosphate level due to decreased excretion.
activity. Vitamin D functions in its active form, One would also expect to see a low calcium
1,25-dihydroxycholecalciferol, to increase in- level due to diminished vitamin D (and hence,
Test Block 5

testinal absorption of calcium and phosphate. diminished intestinal absorption of calcium).


A deficiency in this nutrient will result in a de- Lastly, one would also expect to see elevated
ficiency in intestinal calcium and phosphate parathyroid hormone (PTH) levels as the body
absorption. Note that 25-dihydroxycholecal- attempts to increase serum calcium.
ciferol is synthesized initially in the liver and
664 Section III: Full-Length Examinations  •  Answers

41. The correct answer is C. Dyspnea and uni- Answer A is incorrect. Acetazolamide acts at
lateral pleuritic chest pain in a young male the proximal convoluted tubule, inhibiting
smoker are highly suggestive of spontaneous carbonic anhydrase. It causes a reduction in to-
pneumothorax. Risk factors include male sex, tal-body bicarbonate. Acetazolamide can result
smoking, and a tall, thin stature. Because air is in type 2 renal tubular acidosis, but not Bartter
filling up the space previously occupied by the syndrome.
lung, there will be hyperresonance on percus-
Answer C is incorrect. Hydrochlorothiazide
sion on the side with the lesion.
is a thiazide diuretic that acts early in the dis-
Answer A is incorrect. Primary pneumothorax tal tubule. Metabolic derangements seen with
presents with unilateral chest expansion, in- this drug include hypokalemic metabolic aci-
dicating that one side is not being filled with dosis, hyponatremia, hyperglycemia, hyperlip-
air during inspiration. It may or may not be idemia, hyperuricemia, and hypercalcemia.
accompanied by tracheal deviation away from
Answer D is incorrect. Spironolactone is a
the affected lung.
potassium-sparing diuretic that acts as a com-
Answer B is incorrect. Lobar pneumonia may petitive aldosterone receptor antagonist in
have bronchial breath sounds over the lesion, the cortical collecting tubule. Adverse effects
whereas pneumothorax will have decreased include hyperkalemia and antiandrogenic ef-
breath sounds over the lesion. fects, such as gynecomastia.
Answer D is incorrect. Lobar pneumonia Answer E is incorrect. Triamterene is a
would present with increased tactile fremitus, potassium-sparing diuretic that blocks sodium
whereas pneumothorax will have absent tactile channels in the cortical collecting tubule.
fremitus.
43. The correct answer is C. This image shows
Answer E is incorrect. Tension pneumotho-
numerous vacuolated macrophages crowding
rax will have tracheal deviation away from the
the lamina propria. On periodic acid-Schiff
side with the lesion. In spontaneous pneumo-
(PAS) stain, vacuoles of the macrophages
thorax, there may or may not be any tracheal
Full-Length Exams

within the lamina propria react intensely and


deviation. If it is present, a contralateral shift of
appear purple, indicating the presence of gly-
the mediastinum will be seen on chest x-ray.
coprotein. PAS-positive macrophages are seen
in Whipple disease, which is caused by infec-
42. The correct answer is B. This is a classic de-
tion with Tropheryma whippelii. Whipple dis-
scription of a patient with Bartter syndrome, a
ease most commonly affects older white men.
defect in the ion channels of the thick ascend-
Although this disease can affect any part of the
ing loop of Henle. Clinically, patients have
body (including the heart, lungs, brain, joints,
low-to-normal blood pressure, with short stat-
and eyes), the most common presenting fea-
ure and failure to thrive, and various lab ab-
ture of Whipple disease is diarrhea due to mal-
normalities, including hypokalemia, metabolic
absorption.
alkalosis, and hyperaldosteronism. Depend-
ing on the extent of the metabolic derange- Answer A is incorrect. The most common di-
ments, patients may also have nonspecific saccharidase deficiency is lactase deficiency,
symptoms such as nausea, vomiting, diarrhea, which results in lactose intolerance. In patients
and muscle cramps. If the electrolyte abnor- with lactose intolerance, drinking milk can
malities become severe, patients are at risk for cause abdominal pain, diarrhea, and increased
seizures. Bartter syndrome has three variants, flatulence. PAS-positive macrophages, how-
Test Block 5

each affecting the sodium-potassium-chloride ever, are not seen in lactase deficiency, and the
pump and each resulting in a different defec- villi appear normal.
tive channel in the thick ascending limb of the
Answer B is incorrect. Chronic pancreatitis is
loop of Henle, the site at which furosemide, a
characterized by malabsorption of protein, fat,
loop diuretic, acts.
Test Block 5  •  Answers 665

and vitamins A, D, E, and K; thus steatorrhea Answer B is incorrect. Iodide must be oxi-
is a common sign of chronic pancreatitis. PAS- dized to iodine before it can be attached to
positive macrophages, however, are not seen in thyroglobulin. This step is not affected by pro-
this condition. pylthiouracil.
Answer D is incorrect. Similar to celiac sprue, Answer C is incorrect. TSH regulates many
tropical sprue is characterized by autoanti- steps in the synthesis or thyroid hormone and
bodies to gluten (gliadin) in wheat and other is released from the anterior pituitary gland.
grains. Although its cause is unknown, it is Due to negative feedback, one would expect
likely due to infection. PAS-positive macro- to see a low serum TSH level in this patient.
phages are not seen in sprue, however. Thus, blocking this step would not help a pa-
tient with Graves disease.
Answer E is incorrect. Histologic examina-
tion in celiac sprue reveals villus flattening Answer D is incorrect. Colloid is the stored
and lymphocytic infiltrate. Antigliadin, anti- form of thyroid hormone and when stimulated
endomysial, and anti-tissue transglutaminase by TSH, it is cleaved by proteases to release
antibodies are present, resulting in malabsorp- thyroxine and T3 This step can be inhibited by
tion. PAS-positive macrophages are not seen in iodide salts but is not commonly used as a first-
sprue. line treatment.

44. The correct answer is E. This patient has 45. The correct answer is B. This patient suffers
Graves disease, an autoimmune disorder re- from Wernicke-Korsakoff syndrome. Alcoholic
sulting from the presence of elevated levels of patients often have reduced intake of calories
thyroid-stimulating immunoglobulin (TSI), an other than from alcohol and become deficient
IgG that binds to and stimulates the TSH re- in various nutrients, among them vitamin B1
ceptor of the thyroid gland. This causes an in- (thiamine). Thiamine is a needed cofactor in
crease in the production and release of thyroid some of the key reactions in the Krebs cycle.
hormone. The presence of TSI is relatively Without it, cells are in a low-energy state that
specific for Graves disease. While the exact will eventually damage those cells with high-

Full-Length Exams
trigger for this autoimmune response is un- energy requirements (neurons and myocar-
known, Graves disease is associated with the dium). Wernicke encephalopathy is an acute
HLA-B8 subtype. The three classic findings as- syndrome that is reversible with thiamine ad-
sociated with Graves disease are hyperthyroid- ministration; it is characterized by mental sta-
ism, ophthalmopathy (exophthalmos), and der- tus changes (disorientation, confusion, inatten-
mopathy/pretibial myxedema (ie, nonpitting tion), ophthalmoplegia, ataxia, and nystagmus.
edema on the anterior surface of both legs, Korsakoff dementia is a chronic syndrome and
with overlying skin that is dry and waxy and is irreversible; it is characterized by symptoms
may have several diffuse, slightly pigmented of Wernicke encephalopathy, selective antero-
papules). Propylthiouracil and methimazole grade and retrograde amnesia, and confabula-
are used to treat hyperthyroidism. They work tions (note that the patient is too young to have
by blocking tyrosine iodination (also known as been in World War II). Additionally, periph-
organification) and coupling. Propylthiouracil eral neuropathy (dry beriberi) or dilated car-
also decreases peripheral conversion of thyro- diomyopathy (wet beriberi) may develop, as in
nine to triiodothyronine (T3). b-blockers are this patient. A transesophageal echocardiogram
also used to control the cardiac abnormalities allows for assessment of the extent of the car-
of thyrotoxicosis. diomyopathy.
Test Block 5

Answer A is incorrect. Active absorption of io- Answer A is incorrect. Lumbar puncture may
dide is known as “trapping” and is the first step be helpful in the case of meningitis. How-
in thyroid hormone synthesis. However, it is ever, nothing in the patient’s history points to
not affected by propylthiouracil.
666 Section III: Full-Length Examinations  •  Answers

meningitis (headache, stiff neck). Imaging is Answer B is incorrect. Although the use of
needed to assess the extent of cerebral atrophy. NSAIDs is a risk factor associated with PUD,
celecoxib spares cyclooxygenase isoform 1
Answer C is incorrect. Coagulation studies
found in gastric mucosa. Thus celecoxib use is
will most likely reveal prolonged times in this
not a risk factor for PUD.
patient (ie, the patient is anticoagulated). This
is commonly seen in patients with damaged Answer C is incorrect. Diabetes is not a risk
livers. Although coagulation studies allow ap- factor for PUD.
preciation of the amount of liver dysfunction,
Answer D is incorrect. Hypertension is not as-
they do not measure the extent of the patient’s
sociated with PUD.
primary problem, which is the nutritional
deficit of thiamine and its associated conse- Answer E is incorrect. In the past, PUD was
quences. attributed to many factors, including stress,
spicy foods, chewing gum, and poor eating
Answer D is incorrect. Tissue pressures are a
habits. While some spicy foods might exacer-
measure of the severity of compartment syn-
bate the symptoms of PUD, poor eating habits
drome. Although compartment syndrome
do not cause PUD. A vegan diet might lead to
pre­sents with edematous tissues, it is not asso-
megaloblastic anemia from a lack of vitamin
ciated with neurologic symptoms such as con-
B12, but this patient’s anemia is likely due to
fusion and confabulation. Furthermore, the
chronic blood loss.
question stem does not describe any possible
historical clues for compartment syndrome to 47. The correct answer is C. The patient is suf-
develop (such as recent trauma). fering from hyperparathyroidism secondary to
Answer E is incorrect. Venous Doppler ultra- renal disease, also known as renal osteodystro-
sonography is used to diagnose deep venous phy. As the glomerular filtration rate decreases,
thrombosis (DVT). Although a patient’s legs excretion of phosphate also decreases, leading
may become edematous and pulmonary râles to hyperphosphatemia. Additionally, the dis-
(in the case of embolization to the lung) may eased kidney no longer can produce adequate
Full-Length Exams

develop, DVT does not explain the neurologic amounts of 1,25(OH)2D3, which decreases
status of this patient. The patient is too old for the amount of calcium being absorbed from
an inherited clotting disorder to appear (usu- the gut, resulting in hypocalcemia. The hy-
ally occurs before age 40), and no predispos- perphosphatemia amplifies the fall in serum
ing factors are listed in the question stem (eg, calcium and independently increases PTH se-
recent surgery, cancer, oral contraceptives). In cretion. The fall of serum calcium, low levels
fact, many alcoholics are somewhat anticoagu- of 1,25(OH)2D3 and the hyperphosphatemia
lated, as a damaged liver produces fewer clot- combine to result in hyperparathyroidism. The
ting factors. hypocalcemia in this patient is demonstrated
as a positive Chvostek sign, in which tapping
46. The correct answer is A. This patient has pep- on the hyperexcitable facial nerve causes tem-
tic ulcer disease (PUD), which presents as a porary facial muscle spasms. The increase in
burning epigastric pain worsened by eating. PTH also activates osteoclasts, leading to bone
Bleeding from the ulcer may lead to black, resorption and increased levels of alkaline
tarry stools and anemia. The gold standard for phosphatase.
diagnosis is endoscopy. Benign lesions are flat
Answer A is incorrect. Patients with renal dis-
and have smooth borders, unlike malignant le-
ease have decreased serum calcium levels sec-
Test Block 5

sions, which may protrude into the lumen and


ondary to impaired activation of vitamin D to
have irregular borders. H pylori infection is a
1,25(OH)2D3. Lack of biologically active vita-
major cause of PUD; >70% of patients with
min D results in decreased absorption of cal-
PUD have concurrent H pylori infection.
cium from the GI tract. Thus these lab values
would not be consistent with this patient.
Test Block 5  •  Answers 667

Answer B is incorrect. This set of lab values is tamine release). They are of neuroendocrine
characteristic of a patient with primary hyper- origin and have no link with smoking.
parathyroidism. The major difference between
Answer C is incorrect. Although metastases
the lab results for a patient with primary ver-
from other organs arise more commonly than
sus secondary hyperparathyroidism is the re-
primary lung tumors, they are most often mul-
lationship between PTH and serum calcium.
tifocal, not a single peripheral nodule. Metas-
Patients with primary hyperparathyroidism will
tases to lung principally arise from primary tu-
have a resultant increase in serum calcium lev-
mors of the breast, colon, and kidney.
els, while those with secondary hyperparathy-
roidism have a decreased serum calcium level, Answer D is incorrect. Small cell carcinoma
which then leads to the increase in PTH. of the lung is an undifferentiated tumor usu-
ally present in a central location. There is a
Answer D is incorrect. Increased levels of
strong association with smoking and it is more
PTH in patients with secondary hyperparathy-
often seen in men. Small cell carcinoma me-
roidism leads to osteoclast activation. This of
tastasizes early and may secrete hormones
course increases bone resorption and alkaline
such as ADH and ACTH. Small cell lung
phosphatase.
cancer is rarely resectable and is most often
Answer E is incorrect. Patients with renal dis- treated with combination chemotherapy and
ease have increased, not decreased, levels of radiation.
phosphate.
Answer E is incorrect. Squamous cell carci-
noma of the lung often presents as a centrally
48. The correct answer is A. Critical points to
located hilar mass. This lung cancer subtype
consider in this question are first, that the pa-
is more common in men, and it exhibits the
tient is a nonsmoker; second, that the lesion is
strongest link to smoking.
localized peripherally (subpleural mass); and
third, that the patient is a woman. About 95%
of all lung cancers can be classified into one
of two categories: small cell lung cancers make

Full-Length Exams
up 13%, and non-small cell lung cancers (con-
sisting of adenocarcinoma, squamous cell car-
cinomas, and other histologic types) make up
the rest. Adenocarcinoma is the most prevalent
type of lung cancer, representing 38% of all
diagnosed cases. Relative to squamous cell car-
cinoma (the second most prevalent subtype),
adenocarcinoma is more often seen in non-
smokers than in smokers (62% vs 18%), and
more often seen in women than in men irre-
spective of smoking status. Treatment interven-
tions differ by type. Because adenocarcinomas
are typically peripherally located (75%), they
often are more amenable to surgical removal
than other tumor types, although the success
of treatment depends more on the stage of the
tumor than the type. Adenocarcinoma often,
Test Block 5

but not always, presents as multiple masses.


Answer B is incorrect. Carcinoid tumors are
found in major bronchi and may cause carci-
noid syndrome (flushing due to excessive his-
This page intentionally left blank
Test Block 6

669
670 Section III: Full-Length Examinations  •  Questions

Q u e st i o n s

1. A 45-year-old man presents to his physician vere upper respiratory infection that did not
complaining of a five-month history of oc- respond to antibiotics. The patient is not dia-
casional burning mid-epigastric pain that betic, hypertensive, or dyslipidemic, and all of
improves when he eats food. He denies any his family members are healthy. Which of the
history of recent travel or excessive use of non- following is the most likely cause of this pa-
steroidal anti-inflammatory drugs. The physi- tient’s symptoms?
cian begins a course of pharmacologic therapy
(A) Adenovirus
to improve the patient’s symptoms. Blockade
(B) Brugia malayi
of which of the following receptors would
(C) Coxsackie B virus
improve the patient’s symptoms most signifi-
(D) Haemophilus influenzae type B
cantly?
(E) Streptococcus pneumoniae
(A) Cholecystokinin B (F) Wuchereria bancrofti
(B) Histamine2
(C) Muscarinic3 4. A primigravida at 30 weeks’ gestation has been
(D) Norepinephrine diagnosed with diabetes mellitus. She is rushed
(E) Secretin to delivery because of signs of preeclampsia.
After a cesarean section delivery, the newborn
2. A 27-year-old woman with a history of re- baby is noted to have signs of cyanosis, tachy-
current neurologic complaints (including pnea, and dyspnea. Which of the following is
temporary loss of vision, migrating areas of a characteristic of the substance that is insuf-
weakness and sensory deficits, and bladder in- ficient in the baby?
continence) presents to her internist’s office
(A) It consists primarily of sphingomyelin
with a new neurologic complaint. She says
(B) It decreases compliance of small alveoli
her husband has noticed that her eyes have re-
(C) It disrupts liquid intermolecular forces in
Full-Length Exams

cently been “moving out of sync” with one an-


alveoli
other. She admits that at times she has double
(D) It increases the surface tension of alveoli
vision. A lesion of the right medial longitudi-
(E) It is produced by type I pneumocytes in al-
nal fasciculus would result in which combina-
veoli
tion of eye movements?
(F) It lines medium-sized bronchi, not alveoli
(A) Normal conjugate movement with at-
tempted lateral gaze 5. A 30-year-old African-American man presents
(B) Palsy of the left medial rectus with at- to his primary care physician complaining of
tempted left lateral gaze dizziness and lethargy. About a week ago he
(C) Palsy of the left medial rectus with at- completed a 14-day course of antibiotics pre-
tempted right lateral gaze scribed by his doctor for bacterial sinusitis. He
(D) Palsy of the right medial rectus with at- is normally in good health, eats a diet full of
tempted left lateral gaze fruits and vegetables, and exercises daily. He
(E) Palsy of the right medial rectus with at- denies any changes in his diet and has not
tempted right lateral gaze experienced any gastrointestinal distress, al-
though he admits to tea-colored urine over the
3. A 22-year-old man comes to the physician past three-four days. On physical examination
Test Block 6

complaining of difficulty walking up the stairs he appears pale and uncomfortable, with ic-
because he “can’t catch his breath.” Physical teric sclerae. His blood pressure is 100/70 mm
examination reveals moderate pitting edema in Hg, pulse is 110/min, and respirations are 19/
his lower extremities and a new S3 gallop. The min. Laboratory studies show:
patient’s history includes a recently treated se-
Test Block 6  •  Questions 671

RBC count: 5300/mm³


WBC count: 9,500/mm³
Hemoglobin: 8 g/dL
Hematocrit: 24%
Mean corpuscular volume: 90 fL
Which of the following is the most likely cause
of this patient’s anemia?
(A) a-Thalassemia
(B) Blood donation
(C) Folic acid deficiency
(D) Glucose-6-phosphate dehydrogenase defi-
ciency
(E) Intrinsic factor deficiency
Reproduced, with permission, from USMLERx.com.
(F) Iron deficiency
(G) Sickle cell disease
(A) Karyotypic analysis demonstrating t(8;14)
6. A 27-year-old man dies of acute respiratory
(B) Karyotypic analysis demonstrating t(9;22)
distress syndrome one day after presenting to
(C) Lymphocytes positive for interleukin-21 on
the hospital with shortness of breath and a fe-
flow cytometry
ver of 38°C (100.4°F). On the second hospital
(D) Lymphocytes positive for terminal deoxy-
day, he developed extreme pulmonary edema
nucleotidyl transferase on flow cytometry
and hypotension before he died. His family
(E) Physical examination that reveals bilateral
says that he had recently gone hiking and cav-
orchitis
ing in an area heavily populated with rodents.
Which of the following is the most likely cause 8. Intravenous administration of drug X to an
of death in this patient? anesthetized animal produces an increase in

Full-Length Exams
(A) Dengue virus its systolic blood pressure. After administration
(B) Ebola virus of drug Y, readministration of drug X produces
(C) Hantavirus a decrease in the animal’s systolic blood pres-
(D) Marburg virus sure. Which of the following pairs of drugs
(E) Rhabdovirus could produce this sequence of events?
(A) Drug X = acetylcholine; Drug Y = neostig-
7. A 33-year-old refugee from Sudan presents to
mine
the emergency department with a history of
(B) Drug X = epinephrine; Drug Y = phentol-
increasing fever and fatigue over the past six
amine
months. A large mass on his left mandible is
(C) Drug X = isoproterenol; Drug Y = atropine
observed. Biopsy of the mass is shown in the
(D) Drug X = norepinephrine; Drug Y = pro-
image. Which of the following results supports
pranolol
the diagnosis?
(E) Drug X = phenylephrine; Drug Y = hexa-
methonium
Test Block 6
672 Section III: Full-Length Examinations  •  Questions

9. An 82-year-old retired banker comes to the blood under the fingernails. Blood tests reveal
physician because of a three-week history of an increased prothrombin time and partial
progressively worsening nonlocalizing, non- thromboplastin time. Which of the following
radicular low back pain that is not relieved by describes the function of the enzymes or cofac-
sitting or sleeping. He describes urinary hesi- tors most likely to be involved?
tancy and a long history of benign prostatic
(A) Activation of antithrombin III
hyperplasia that has recently worsened. He has
(B) Carboxylation of glutamic acid residues
been afebrile throughout this time. Physical
(C) Cross-linking of fibrin polymers
examination is notable for perianal hyperesthe-
(D) Hydrolyzation of fibrinogen
sia with slightly decreased rectal tone, a uni-
(E) Hydroxylation of proline residues
formly enlarged prostate, and diminished an-
kle jerk reflexes bilaterally. No pain is elicited 12. The inhaled anesthetic drugs A, B, C, and D
with straight leg raises in the supine position. have the properties indicated in the chart.
X-ray films of the thoracic and lumbosacral Which of the following statements best de-
spine show moderate osteoarthritis. Laboratory scribes the properties of these drugs?
studies are unremarkable. Which of the fol-
lowing is the most likely cause of the patient’s
symptoms? Minimum
Blood
alveolar
(A) Conus medullaris tumor concentration
solubility
(B) Guillain-Barré syndrome
(C) L4-5 disc herniation Drug A 7.3 0.5
(D) Spinal epidural abscess
(E) Vertebral osteomyelitis Drug B 3.1 1.0

10. An 8-year-old girl presents to the clinic for a Drug C 2.0 0.1
check-up required by her new school. She is
a recent immigrant to the United States and Drug D 4.4 0.15
Full-Length Exams

did not have access to regular medical care


growing up. Her mother reports that when her Reproduced, with permission, from USMLERx.com.
daughter was an infant she had swollen hands
and feet as well as “something wrong with
the sides of her neck.” The patient’s left arm (A) Drug A is more potent than drug D
is smaller than the right. When height and (B) Drug B is less soluble in blood than
weight data are plotted on a growth curve, she drug C
is short for her age. What congenital cardio- (C) Drug C will have both the highest potency
vascular anomaly does this patient most likely and the most rapid induction
have? (D) Drug D will induce anesthesia more rap-
idly than drug C
(A) Postductal coarctation of the aorta
(E) Patients treated with drug B will recover
(B) Preductal coarctation of the aorta
more quickly from anesthesia than drug A
(C) Subclavian steal syndrome
(D) Tetralogy of Fallot 13. A 46-year-old man presents with a three-week
(E) Transposition of the great vessels history of burning substernal and epigastric
pain that improves after meals. Medical his-
11. A 6-year-old boy with recurrent otitis media
Test Block 6

tory is significant for hypertension, which is


was treated with ceftriaxone by his pediatri-
controlled with exercise and diet. Medications
cian. One week after beginning treatment, his
include a daily vitamin supplement. He has
father notices he has been bruising easily on
no other complaints. Social history reveals that
his legs and bleeding heavily when he brushes
the patient was born in Cambodia, and lived
his teeth. Examination reveals streaks of
Test Block 6  •  Questions 673

there until five years ago when he moved to (B) Inhibiting serotonin reuptake and acting as
the United States. A urease breath test per- a partial serotonin agonist
formed in the office is positive. What is the (C) Inhibiting the presynaptic enzyme respon-
recommended treatment for the patient? sible for the breakdown of dopamine, nor-
epinephrine, and serotonin
(A) Ampicillin, clindamycin, and bismuth
(D) Inhibiting the reuptake mechanism re-
(B) Bismuth, metronidazole, tetracycline, and
sponsible for terminating the action of
omeprazole
both norepinephrine and serotonin
(C) Imipenem and cilastatin
(E) Inhibiting the reuptake mechanism re-
(D) Ranitidine, omeprazole, and bismuth
sponsible for terminating the action of
(E) Vancomycin, gentamicin, and aztreonam
sero­tonin only
14. A 34-year-old woman is brought to the emer-
16. A 13-year-old boy is brought to the physician
gency department in mid-August after collaps-
because of swelling and pain in his right leg.
ing during a picnic at a local park. She is hypo-
He says he first noticed these symptoms about
tensive, tachycardic, and diaphoretic with cold
three months ago, but the pain has gotten
extremities. Her oxygen saturation is 85% on
much worse over the past few weeks. An x-ray
room air. Physical examination reveals she is
film of the leg shows a large lytic lesion with
using accessory muscles to breathe. Which of
an onion-skin appearance located midway
the following is involved in the production of
along the femur. Malignancy is suspected, and
this patient’s symptoms?
a karyotype of the biopsied bone tumor cells is
(A) C3a ordered. Which of the following chromosomal
(B) Granulocyte macrophage colony- translocations would most likely be found in
stimulating factor the karyotype of this patient?
(C) IgA
(A) t(8;14)
(D) Interleukin-3
(B) t(9;22)
(E) Membrane attack complex
(C) t(11;14)

Full-Length Exams
15. A 25-year-old woman presents to her primary (D) t(11;22)
care physician because she has been feeling fa- (E) t(21;22)
tigued. She states the fatigue has been getting (F) t(15;17)
progressively worse for the past three months
17. A 19-year-old woman is brought to the emer-
and is not relieved by sleep, although she has
gency department after losing a substantial
been averaging 12-16 hours of sleep per night.
amount of blood in a motor vehicle collision.
She also complains of difficulty concentrating
The patient requires large amounts of intrave-
at work, poor appetite, a 4.5-kg (10-lb) weight
nous fluids, blood replacement, and medica-
loss, and reduced interest in socializing with
tions to maintain her blood pressure. On the
her friends. Physical examination is normal.
third day of hospitalization she becomes se-
Laboratory tests reveal a negative Monospot
verely acidotic and has a tense, tender abdo-
test, normal liver function, and normal thy-
men. Which of the following is most likely the
roid function. Serum iron, ferritin, total iron-
cause of her abdominal pain?
binding capacity, vitamin B12, and folate are
all within normal limits. The patient is diag- (A) Cecum
nosed with a major depressive episode and (B) Hepatic flexure, large bowel
given a prescription for an antidepressant. Nor- (C) Ileum
Test Block 6

triptyline acts by which of the following mech- (D) Jejunum


anisms? (E) Splenic flexure, large bowel
(F) Stomach
(A) Depleting the adrenergic nerve terminal of
norepinephrine
674 Section III: Full-Length Examinations  •  Questions

18. The image shows a mutation that can cause 20. A lanky 13-year-old boy presents to the emer-
hemoglobinopathy. This is an example of gency department complaining that he “can’t
which type of mutation? see.” He has many bruises. He says he is
“double-jointed,” and to demonstrate, touches
his long thumb to his thin wrist. The ophthal-
mRNA transcript of normal gene UCU UCA CGU mologic examination reveals that both lenses
have been dislocated upward and outward.
mRNA transcript of mutation UCU UAA The boy’s chest is sunken and he has a heart
murmur. Which of the following is most likely
Reproduced, with permission, from USMLERx.com.
deficient in this patient?
(A) Glycosylation of the pro-a chain
(B) Hydroxylation of preprocollagen
(A) Frameshift mutation (C) Normal synthesis of type I collagen
(B) Insertion mutation (D) Synthesis of type III collagen
(C) Missense mutation (E) Transcription of fibrillin
(D) Nonsense mutation
(E) Silent mutation 21. A 56-year-old heart transplant recipient pre­
sents to the emergency department with short-
19. A 35-year-old woman from Arizona comes to ness of breath and dyspnea at rest. His cardiol-
the physician with an x-ray film of her chest ogist immediately observes that the patient has
taken during a routine health insurance ex- jugular venous distention and lower extremity
amination. The x-ray film shows bilateral hilar edema. However, he eventually lapses into de-
adenopathy. The patient is completely asymp- compensated heart failure and dies. The ap-
tomatic. Sarcoidosis is suspected. She was told pearance of his lung at autopsy is shown in the
that her serum levels of angiotensin-converting image. Compared to normal lung tissue, what
enzyme (ACE) were elevated. The sensitivity cells are more prominent in this patient’s lung?
and specificity of using ACE levels to test for
the disease in question are 80% and 50%, re-
Full-Length Exams

spectively. Assuming that sarcoidosis is highly


prevalent among residents in the southwestern
United States, how would the patient’s place
of residence affect the positive predictive value
(PPV) and negative predictive value (NPV) of
the test?
(A) In areas of higher prevalence, both the
PPV and the NPV are higher
(B) In areas of higher prevalence, both the
PPV and the NPV are lower
(C) In areas of higher prevalence, the PPV is
higher and the NPV is lower
(D) In areas of higher prevalence, the PPV is
lower and the NPV remains the same Reproduced, with permission, from USMLERx.com.
(E) Regardless of the prevalence, both the
PPV and the NPV are unchanged
Test Block 6

(A) Fluid-filled dendritic cells


(B) Hemosiderin-laden macrophages
(C) T-cells
(D) Type I pneumocytes
(E) Type II pneumocytes
Test Block 6  •  Questions 675

22. A 64-year-old man with a history of type 2 24. A 47-year-old man complains of episodic dia-
diabetes mellitus comes to his primary care phoresis, palpitations, and headache. These
physician complaining of sweating, tremors, spells have been occurring more frequently
palpitations, and memory impairment since and are lasting longer. Physical examination
starting an oral medication to treat his diabe- reveals a patient in obvious distress with facial
tes. His doctor decides to change the patient’s pallor, sweating, tachycardia, and severe hy-
oral medication. Which of the following would pertension. Urinary and plasma catecholamine
be the best choice of medication with which to and metanephrine levels are increased. MRI
replace his current regimen? of the abdomen reveals a mass. Which of the
following is the embryologic origin of the cells
(A) Glargine
that comprise this mass?
(B) Glipizide
(C) Glyburide (A) Endoderm
(D) Pioglitazone (B) Mesoderm
(E) Ultralente insulin (C) Neural crest
(D) Neuroectoderm
23. A 6-year-old boy is brought to the pediatrician (E) Surface ectoderm
by his mother, who is very concerned about
his health. Physical examination shows an ex- 25. A 30-year-old man presents to the emergency
tensive erythematous reticular skin rash on the department complaining of shortness of
face trunk, and extremities (see image), along breath, dizziness, nausea, and vomiting. He
with swelling around his wrists that causes him also says that his heart feels “like it is jump-
pain on movement at the joint. He is in no ing out of my chest.” Two days ago, he passed
acute distress, but his mother is very anxious. a burning house and stopped to help the resi-
Which of the following is the next best step in dents evacuate. He reports inhaling a signifi-
treatment for this illness? cant amount of smoke, but declined medi-
cal assistance at the scene because he had no
symptoms. The patient reports feeling very fa-

Full-Length Exams
tigued the day prior to presentation and stayed
in bed for most of the day. On physical exami-
nation, his pulse is 90/min, blood pressure is
100/60 mm Hg, and respiratory rate is 30/min
with deep, gasping respirations. The rest of the
examination is unremarkable with the excep-
tion of bright red vessels in both of his eyes
and a smell of bitter almonds on his breath.
What is the mechanism of action of the toxic
agent that resulted in this patient’s symptoms?
(A) Decreased blood oxygen-carrying capacity
Courtesy of the Centers for Disease Control and Prevention.
(B) Direct tissue injury
(C) Inhibition of cellular respiration
(D) Inhibition of cholinesterase
(A) No treatment necessary; it is a self-limiting (E) Ribosome inhibition
disease
(B) Prescribe a corticosteroid cream
Test Block 6

(C) Prescribe an oral corticosteroid


(D) Prescribe the appropriate antibiotic cream
(E) Prescribe the appropriate oral antibiotic
676 Section III: Full-Length Examinations  •  Questions

26. The human leukocyte antigen complex is a (C) Ovarian hyperstimulation


4-megabase region on chromosome 6 that is (D) Psychosis
densely packed with expressed genes that lead (E) Weight gain
to proteins critical for immunologic specific-
ity and thus autoimmune diseases. Pedigree is 28. An intoxicated man is found unresponsive in
shown in the image: Note that the two DR al- the woods and is brought to the emergency
leles possessed by the proband’s grandparents department. He is found to have animal bite
are shown above their pedigree symbols. What marks on his lower left extremity. He is unable
combination of alleles that could be inherited to explain their existence. Which of the follow-
by the proband would confer the greatest risk ing is the most likely recommended, most ap-
to the patient of contracting type 1diabetes? propriate next course of action?
(A) Administer human immune globulin im-
DR5/DR7 DR4/DR8 DR8/DR8 DR3/DR7 mediately, and follow-up with injections of
killed virus vaccine only if the patient de-
velops symptoms
(B) Administer human immune globulin im-
mediately, and give a series of five injec-
tions of killed virus vaccine
(C) Administer human immune globulin in a
series of five doses
(D) Administer killed virus vaccine immedi-
ately, and follow-up with injections of hu-
man virus vaccine only if the patient devel-
ops symptoms
(E) Administer killed virus vaccine immedi-
ately, and give a series of five injections of
Reproduced, with permission, from USMLERx.com. human immune globulin
Full-Length Exams

(F) Do nothing
(A) DR3/DR3
29. An 82-year-old woman presents to the emer-
(B) DR3/DR4
gency department with a three-week history
(C) DR5/DR3
of fever, weight loss, and malaise in the setting
(D) DR7/DR3
of hip and shoulder girdle pain that is most
(E) DR8/DR8
severe in the morning. She also reports a one-
week history of headaches and left-sided jaw
27. A 26-year-old woman and her husband visit the
pain that occurs at every meal. The patient’s
clinic, because they have been trying to con-
temperature is 38.2°C (100.8°F), her pulse is
ceive for the past 14 months without success.
104/min, and her blood pressure is 140/80 mm
An infertility work-up of the husband shows
Hg. Laboratory studies show a hemoglobin
viable, healthy sperm capable of fertilization.
level of 11.8 g/dL, a WBC count of 11,900/
After structural causes are ruled out in the
mm3, and an erythrocyte sedimentation rate of
woman, the physician and the couple decide
121 mm/h. Physical examination is unremark-
to attempt in vitro fertilization. The physician
able except for moderate synovitis of the ankles
utilizes a common oral medication to induce
and wrist. Which of the following procedures
ovulation for egg collection and assessment.
Test Block 6

is most likely to be diagnostic in this patient?


Which of the following is a common adverse
effect of this medication? (A) Arthrocentesis
(B) Mesenteric angiogram
(A) Esophagitis
(B) Nausea and vomiting
Test Block 6  •  Questions 677

(C) Temporal artery biopsy (A) Aspergillus


(D) Testing for anti-double-stranded DNA and (B) Cryptococcus
antinuclear antibody levels (C) Echovirus
(E) Testing for rheumatoid factor and anti- (D) Haemophilus
cytidine cyclic phosphate levels (E) Neisseria
(F) Toxoplasma
30. A 45-year-old man presents to his primary care (G) Treponema
physician with a blood pressure of 160/90 mm
Hg that has failed to drop substantially after 32. Oxygen unloading is increased when hemoglo-
initiation of lifestyle changes. The patient is bin passes through active tissues. Which of the
subsequently placed on a low dose of hydro- following is one reason for this phenomenon?
chlorothiazide, which lowers his blood pres-
(A) Decreased temperature in the peripheral
sure to 128/86 mm Hg. Which of the follow-
tissues shifts the hemoglobin dissociation
ing accurately represents the site of action of
curve to the left
hydrochlorothiazide?
(B) Decreased temperature in the peripheral
(A) Chloride-binding sites on the cytoplasmic tissues shifts the hemoglobin dissociation
surface of the early distal tubule curve to the right
(B) Chloride-binding sites on the luminal sur- (C) Increased CO2 and H+ in the tissues shift
face of the early distal tubule the hemoglobin dissociation curve to the
(C) Chloride-binding sites on the luminal sur- left
face of the thick ascending limb of the (D) Increased CO2 and increased diphospho-
loop of Henle glycerate shift the hemoglobin dissociation
(D) Sodium binding sites on the basolateral curve to the left
membrane of the thick ascending limb of (E) Increased CO2 and H+ in the tissues shift
the loop of Henle the hemoglobin dissociation curve to the
(E) Sodium-binding sites on the luminal sur- right
face of the early distal tubule

Full-Length Exams
33. A 59-year-old man presents to the emergency
31. The micrograph shown in the image was ob- department after waking up in the middle of
tained from cerebrospinal fluid that demon- the night with a very severe headache. When
strated lymphocytosis, decreased glucose, and asked about the intensity of pain, the patient
increased protein. Which of the following or- exclaims, “I feel like my head is going to ex-
ganisms is most likely responsible for the pa- plode.” Emergent CT of the head demon-
tient’s symptoms? strates blood tracking down the sulci and fol-
lowing the contours of the pia. What is the
most likely underlying pathophysiologic mech-
anism for this patient’s condition?
(A) Arteriovenous malformation
(B) Atherosclerosis
(C) Cerebral contusion
(D) Hypertension
(E) Rupture of arterial aneurysm
Test Block 6

Courtesy of Dr. Leanor Haley, Centers for Disease Control


and Prevention.
678 Section III: Full-Length Examinations  •  Questions

34. A 39-year-old man was seen by a psychiatrist (A) Blockade of ergosterol synthesis
after reports that he had been locking himself (B) Blockade of the pathway that utilizes pteri-
in his apartment because “the devil is trying to dine and PABA in nucleotide formation
put thoughts into my head.” After three weeks (C) Cell wall synthesis
of pharmacologic treatment, he begins to ex- (D) Inhibition of the 30S ribosomal subunit
perience muscular rigidity, decreased perspira- (E) Inhibition of the 50S ribosomal subunit
tion, hyperthermia, and signs of autonomic in-
stability. Which of the following drugs should 37. A 30-year-old male drug user with a history of
be administered to the patient immediately? methylphenyltetrahydropyridine (MPTP) ex-
posure presents to the clinic with a tremor at
(A) Dantrolene
rest, cogwheel rigidity, and postural instability.
(B) Diazepam
Direct innervation from the site of the lesion
(C) Flumazenil
to which nuclei has been damaged?
(D) Haloperidol
(E) Phenobarbital (A) Caudate and putamen (striatum)
(B) Globus pallidus externus
35. The mother of a 3-year-old boy is referred to (C) Globus pallidus internus
genetic counseling after her son is diagnosed (D) Lateral geniculate nucleus
with an enzyme deficiency. Recently, the (E) Subthalamic nucleus
mother noticed that her son has an abnormal
facial appearance as well as pearly papular skin 38. A 25-year-old construction worker presents to
lesions over the scapulae and on the lateral up- his primary care physician complaining of ab-
per arms and thighs, however, his corneas are dominal pain and constipation. The pain is dif-
clear bilaterally. She has also noticed that her fuse, and is neither better nor worse following
son is hyperactive compared to other children a meal. He also reports fatigue and difficulty
of the same age. This patient carries a diagno- concentrating. His vital signs are within nor-
sis of which of the following syndromes? mal limits. His examination is remarkable only
for darkened, painless gingival lesions, and a
(A) Hunter syndrome
Full-Length Exams

non-distended but tender abdomen. Which of


(B) Hurler syndrome
the following is the most appropriate medical
(C) Morquio syndrome
treatment for this patient?
(D) Sanfilippo syndrome
(E) Sly syndrome (A) Deferoxamine
(B) Dimercaprol
36. A 31-year-old man comes to the physician (C) N-acetylcysteine
with a five-day history of shortness of breath. (D) Naloxone
The patient says that he also has had a non- (E) Protamine sulfate
productive cough in the same time period. (F) Thiosulfate
X-ray film of the chest reveals bilateral diffuse
infiltrates, and laboratory results show a WBC 39. A 9-year-old boy with type 1 diabetes melli-
count of 2500/mm³. An enzyme-linked im- tus is brought to the emergency department
munosorbent assay is positive for HIV infec- because he has become delirious in the past
tion, and meth­enamine silver stain shows the hour. Earlier this afternoon, after his usual
causative organism. The patient is started on dose of insulin, the patient began complain-
sulfamethoxazole-trimethoprim. Which of the ing of abdominal pain and shortly thereafter
following is the mechanism of action of the an- he vomited. On further questioning his father
Test Block 6

tibiotic used to treat this patient’s infection? says the boy has had a cough and fever for the
past couple days. Blood is drawn for laboratory
Test Block 6  •  Questions 679

studies, and the patient is immediately started (A) Cytomegalovirus


on fluids with added potassium. Which of the (B) Epstein-Barr virus
following sets of laboratory parameters is most (C) Herpes simplex virus
likely to be seen in this patient? (D) HIV
(E) Rubella
(F) Syphilis
(G) Toxoplasmosis
Choice pH Sodium Potassium Bicarbonate

A 42. A 67-year-old man with a history of prostate


B cancer presents for follow-up after surgical
C management. He has been having back pain
D that has awakened him at night for the past two
E months, and that responds poorly to ibuprofen.
On x-ray, a significant lesion is found in the L2
Reproduced, with permission, from USMLERx.com. vertebral body. A bone biopsy is obtained and
reveals poorly differentiated cells with some
resemblance to prostatic cells. The overexpres-
(A) A sion of what factor could allow the neoplastic
(B) B cells to metastasize?
(C) C
(D) D (A) γ-Glutamyl transpeptidase
(E) E (B) CD44
(C) Collagenase
40. A 32-year-old man presents with a three-month (D) E-cadherin
history of arthralgias, weight loss, diarrhea with (E) Keratin
fatty stools, and abdominal pain. After careful
observation and testing, his physician obtains a 43. A 41-year-old man comes to the physician
biopsy of the lamina propria of the small intes- complaining of crampy, bloating abdominal

Full-Length Exams
tine, which shows periodic acid-Schiff-positive discomfort. He also reports changes in his
material, particularly in macrophages. What is bowel habits and recently noticed dark stool.
the cause of this man’s symptoms? His father, sister, and uncle died of colorectal
cancer. At the physician’s office, a fecal occult
(A) Celiac sprue blood test is positive. Colonoscopy reveals 8-10
(B) Crohn disease small flat polyps in the proximal colon. Which
(C) MALT lymphoma of the following is the most likely cause of
(D) Ulcerative colitis these findings?
(E) Whipple disease
(A) A defect in mismatch repair
41. A pregnant woman comes to the physician for (B) A defect in nucleotide excision repair
a check-up before the beginning of her third (C) A translocation between chromosomes 15
trimester. It is learned that she has been ex- and 17
posed to an infectious disease. Fortunately, the (D) Exposure to benzo(a)pyrene
infectious disease caused no morbidity to the (E) The bcr-abl hybrid gene
fetus, and the resulting pregnancy is uncompli-
cated. The woman later gives birth to a healthy
child. To which of the following pathogens was
Test Block 6

the woman most likely exposed?


680 Section III: Full-Length Examinations  •  Questions

44. A 75-year-old man comes to the physician imately two-three weeks. Upon presentation at
because he recently began experiencing sei- the campus health clinic, a blood sample tests
zures. CT of the head shows an irregular, ring- positive for heterophil antibodies and she is
enhancing lesion with central necrosis in his diagnosed with symptomatic primary Epstein-
right cerebral hemisphere. Results of biopsy Barr virus (EBV) infection. Which of the fol-
are shown in the image. Which of the follow- lowing innate immune cell types plays a direct
ing is the most likely diagnosis? and important role in controlling the early
stages of the systemic response to EBV infec-
tion?
(A) Eosinophils
(B) Mast cells
(C) Megakaryocytes
(D) Microglia
(E) Natural killer cells
(F) Plasma cells
(G) Regulatory T lymphocytes

46. A 4-month-old girl who was born full-term


presents to her pediatrician with an upper res­
piratory infection. Her mother notes that this is
the fifth time her daughter has had an upper
respiratory infection since birth. Her past med-
Reproduced, with permission, from USMLERx.com. ical history is significant for seizures shortly
after birth. In addition to pulmonary findings,
the physical examination is notable for oropha-
(A) Glioblastoma multiforme
ryngeal candidiasis that the patient’s mother
(B) Medulloblastoma
says has been occurring regularly. This child is
(C) Meningioma
Full-Length Exams

presenting with a syndrome that is due to ab-


(D) Neurilemmoma
errant development of which of the following
(E) Oligodendroglioma
embryonic structures?
45. A 19-year-old college student developed sore (A) First and second branchial arches
throat, palatal petechiae, splenomegaly, fever, (B) First and second branchial pouches
and generalized lymphadenopathy after she (C) Fourth and sixth branchial arches
began dating her first serious boyfriend. The (D) Second and third branchial clefts
symptoms were self-limiting and lasted approx- (E) Third and fourth branchial pouches
Test Block 6
Test Block 6  •  Questions 681

47. A 91-year-old man is brought to the emergency 48. A 67-year-old man presents to the emergency
department by his daughter after being found department with diaphoresis and crushing
unresponsive in his home. The patient’s oral chest pain that radiates down his left arm.
temperature is 39.8°C (103.6°F), blood pres- ECG shows ST-segment elevations and in-
sure is 80/50 mm Hg, heart rate is 120/min, verted T waves in leads II, III, and aVF. His
and respiratory rate is 26/min. Initial laboratory troponin I level is high. He is taken to the
tests reveal leukocytosis and elevated lactate cardiac catheterization unit, where he is diag-
levels. The patient’s blood pressure remains nosed with an obstructive myocardial infarc-
at 80/50 mm Hg despite adequate hydration. tion due to occlusion of the right coronary ar-
What is the expected hemodynamic pattern in tery. Assuming this man’s coronary vasculature
this patient in terms of peripheral vascular re- is right dominant, which of the following areas
sistance, cardiac output, and pulmonary capil- is most likely to be spared after this event?
lary wedge pressure?
(A) Anterior interventricular septum
(B) Atrioventricular node
Peripheral Pulmonary (C) Posterior interventricular septum
Cardiac capillary
Choice vascular
resistance output wedge pressure (D) Right ventricle
(E) Sinoatrial node
A

Reproduced, with permission, from USMLERx.com.

(A) A

Full-Length Exams
(B) B
(C) C
(D) D
(E) E

Test Block 6
682 Section III: Full-Length Examinations  •  Answers

An s w e r s

1. The correct answer is B. This patient’s symp- and the presence of fatty acids and bile in the
toms are consistent with those of a duodenal duodenum. Secretin increases pancreatic and
ulcer. A duodenal ulcer can be caused by hy- biliary bicarbonate secretion, and decreases
persecretion of stomach acid, Helicobacter gastric acid secretion. Blockade of secretin re-
pylori infection, or the use of nonsteroidal ceptors potentially could increase the amount
anti-inflammatory drugs. Initial treatment of a of acid produced as a result of loss of feedback
duodenal ulcer involves a trial of a histamine2 inhibition.
(H2)-blocker such as cimetidine or a proton
pump inhibitor such as omeprazole. Activa- 2. The correct answer is D. Medial longitudinal
tion of H2-receptors leads to increased gastric fasciculus (MLF) syndrome (also known as
acid production. By inhibiting the H2-recep- internuclear ophthalmoplegia) is character-
tor, gastric acid secretion is decreased, allow- istic of multiple sclerosis (MS), from which
ing the ulcer to heal. If a biopsy of the ulcer this patient likely suffers. Normally, the right
is positive for H pylori, appropriate treatment MLF connects the left nucleus of cranial nerve
involves “triple therapy,” commonly involving (CN) VI with the right subnucleus of CN III.
cla­rithromycin 500 mg twice a day, amoxicillin Consequently, with left lateral gaze, the left
1 g twice a day, and a proton pump inhibitor nucleus of CN VI sends a signal via the right
twice a day for 10-14 days. MLF to the right subnucleus of CN III, which
stimulates contraction of the right medial rec-
Answer A is incorrect. Gastrin is a gastroin-
tus, preserving conjugate gaze. In MS, demy-
testinal (GI) hormone secreted by the G cells
elination of the right MLF would prevent nor-
of the stomach in response to small peptides
mal signal transmission, leading to a palsy of
and amino acids in the gut, stomach disten-
the right medial rectus with attempted left lat-
tion, and vagal stimulation. Gastrin causes an
eral gaze (horizontal nystagmus of the left eye
increase in gastric acid secretion, and whereas
will also likely be observed).
Full-Length Exams

blockade of gastrin receptors (cholecystoki-


nin B) would decrease gastric acid production Answer A is incorrect. Patients with MS who
slightly, alternative receptors would continue have lesions affecting either one or both of the
to stimulate gastric acid secretion. medial longitudinal fasciculus tracts will have
disconjugate gaze with attempted lateral gaze.
Answer C is incorrect. Acetylcholine is re-
leased by cholinergic neurons in the enteric Answer B is incorrect. Left lateral gaze re-
nervous system, resulting in contraction of quires contraction of the left lateral rectus, not
smooth muscle, relaxation of sphincters, in- the left medial rectus. To assess the function of
crease in gastric secretion, and increase in pan- the left medial rectus, the examiner should in-
creatic secretion. Blockade of the muscarinic3- struct the patient to attempt right lateral gaze.
receptor would not lead to a significant Answer C is incorrect. Palsy of the left medial
decrease in acid production, which is neces- rectus with attempted right lateral gaze would
sary for this patient. result from a lesion affecting the left medial
Answer D is incorrect. Norepinephrine is longitudinal fasciculus.
released from adrenergic neurons in the en- Answer E is incorrect. Right lateral gaze re-
teric nervous system, resulting in relaxation of quires contraction of the right lateral rectus,
Test Block 6

smooth muscle, contraction of sphincters, and not the right medial rectus. To assess the func-
increase in salivary secretion. tion of the right medial rectus, the examiner
Answer E is incorrect. Secretin is a GI hor- should instruct the patient to attempt left lat-
mone secreted by the S cells of the duodenum eral gaze.
in response to acidification of the duodenum
Test Block 6  •  Answers 683

3. The correct answer is C. Based on the physi- terium, which would therefore respond to an-
cal exam findings, the patient is likely expe- tibiotics.
riencing congestive heart failure. Given his
Answer F is incorrect. Wuchereria bancrofti is
young age and lack of risk factors, a myocardi-
a filarial worm that produces symptoms similar
tis should be high on the differential. Half of
to those of Brugia malayi. However, of all of
all viral cases of myocarditis are caused by cox-
the filariae, W bancrofti causes the overwhelm-
sackie B. Coxsackie B, an icosahedral member
ing majority of genital lymphatic involvement.
of the Picornaviridae family, can cause a vari-
ety of illnesses, including meningitis, respira- 4. The correct answer is C. This neonate is suf-
tory infections, and epidemic pleurodynia. fering from neonatal respiratory distress syn-
However, it is important to note that while drome (NRDS) caused by a lack of surfactant.
coxsackievirus is the most common cause, Surfactant is formed relatively late in fetal
there are many different agents that can cause life (begins at 28 weeks and is produced most
myocarditis, including other viruses (eg, ad- abundantly at 34 weeks). It reduces surface
enovirus, HIV), bacteria (eg, Haemophilus in- tension by disrupting the intermolecular forces
fluenzae, Mycoplasma pneumoniae, streptococ- (hydrogen bonds) between molecules of wa-
cal infections), and protozoa (Toxoplasma and ter. This prevents small alveoli from collapsing
Trypanosoma cruzi). and increases compliance. Synthesis of surfac-
Answer A is incorrect. Adenoidal-pharyngeal- tant is decreased by insulin. Thus, maternal
conjunctival viruses, also known as adeno- diabetes is also a risk factor for RDS in infants,
viruses, can cause acute upper respiratory because fetal hyperglycemia stimulates an in-
diseases, conjunctivitis, gastroenteritis, and creased release of insulin. Cortisol increases
hemorrhagic cystitis. While in rare cases it can synthesis of surfactant; therefore, women who
be associated with infections of the heart, it is have to deliver their babies prematurely typi-
not the most common cause of myocarditis. cally receive glucocorticoids to help prevent
RDS in their infants.
Answer B is incorrect. Brugia malayi is a fila­
rial worm that infects the lymphatics and sub- Answer A is incorrect. Surfactant consists pri-

Full-Length Exams
cutaneous tissues. It can cause lymphadenopa- marily of dipalmitoyl phosphatidylcholine (lec-
thy, lymphatic destruction, and subsequent ithin), the main lipid component, not sphingo-
edema. Infection with this worm would not, myelin.
however, explain the S3 gallop. Answer B is incorrect. Surfactant prevents
Answer D is incorrect. Haemophilus influ- small alveoli from collapsing by decreasing
enzae type B is a rod-shaped gram-negative alveolar surface tension. The compliance in
bacterium that is responsible for respiratory small alveoli is increased (not decreased), al-
infections and meningitis in childhood. It lowing the lung to inflate more easily and de-
is currently recommended that all children creasing the work of breathing.
receive the Hib vaccine, which is effective Answer D is incorrect. Surfactant is a lipo-
against this strain of bacteria. While it can be protein consisting of both a hydrophobic and
associated with infections of the heart, it is (1) hydrophilic region. It reduces alveolar surface
not the most common cause of myocarditis, tension by adsorbing to the air-water inter-
and (2) a bacterium, which would therefore re- face—the hydrophilic portion interfering with
spond to antibiotics. the hydrogen bonds between molecules of wa-
Answer E is incorrect. Streptococcus pneu- ter and the hydrophilic portion facing the air.
Test Block 6

moniae is a gram-positive diplococcus and Decreasing surface tension prevents small al-
the most common cause of lobar pneumonia veoli from collapsing.
worldwide. While it can be associated with
infections of the heart, it is (1) not the most
common cause of myocarditis, and (2) a bac-
684 Section III: Full-Length Examinations  •  Answers

Answer E is incorrect. Type II pneumocytes viduals with poor nutrition, neither of which is
secrete pulmonary surfactant and serve as pre- present in this patient.
cursors to both type I and type II pneumocytes.
Answer E is incorrect. Intrinsic factor (IF) de-
Type I pneumocytes make up 97% of the alve-
ficiency causes pernicious anemia. IF is syn-
olar surface and are thin, squamous cells opti-
thesized in the stomach, and a deficiency re-
mal for gas diffusion.
sults from destruction of those cells. However,
Answer F is incorrect. Surfactant lines alveoli, there is no reason to believe that this patient
not larger airways. has suffered from any autoimmune attack on
gastric cells. Pernicious anemia also causes a
5. The correct answer is D. Glucose-6-phosphate megaloblastic anemia, not seen here.
dehydrogenase (G6PD) deficiency is an
Answer F is incorrect. Iron deficiency anemia
X-linked recessive disorder that is particularly
is one of the most common causes of anemia
common in African-American males. Individu-
but is not likely in this patient given the pre-
als of Mediterranean heritage are also affected.
sentation. It is typically observed in menstruat-
G6PD is the first enzyme in the pentose phos-
ing women.
phate pathway, and it reduces oxidized nico-
tinamide adenine dinucelotide phosphate to Answer G is incorrect. Sickle cell disease is
reduced nicotinamide adenine dinucelotide the result of a point mutation in the b-globin
phosphate (NADPH) as it oxidizes glucose- chain of hemoglobin. Symptoms typically
6-phosphate. NADPH is a necessary cofactor present in childhood as severe joint pain with
for the enzyme glutathione reductase, which exertion and do not resemble the presentation
generates reduced glutathione. Reduced glu- of this patient.
tathione, in turn, can reduce peroxides into
less harmful substances such as water and alco- 6. The correct answer is C. Hantavirus pulmo-
hol. A deficiency of G6PD makes erythrocytes nary syndrome is a rare viral cause of acute
more susceptible to oxidative stress, resulting respiratory distress syndrome (ARDS). Hanta-
in a hemolytic anemia that can be triggered by virus is a Bunyavirus that has been found in
Full-Length Exams

antibiotics such as sulfa drugs or bacterial in- rodents throughout the United States. It is
fections. Heinz bodies are a hallmark of G6PD thought to be transmitted via rodent droppings
deficiency; they represent accumulations of and saliva.
denatured hemoglobin in affected red blood Answer A is incorrect. Dengue virus is an Ae-
cells. des mosquito-transmitted virus that causes a
Answer A is incorrect. a-thalassemia is caused hemorrhagic fever. It is found in tropical re-
by a defect in one or more of the four a-globin gions of Asia and has spread to South and Cen-
chains of hemoglobin. Symptoms range in se- tral America. Patients present with rash and
verity from silent carriers who have no detect- bleeding from mucous membranes.
able changes on a complete blood count to in Answer B is incorrect. Ebola virus and Mar-
utero fatalities. However, these are typically burg virus are members of the Filovirus ge-
constant conditions that are not precipitated by nus, which cause hemorrhagic fever. Both
infection or medications. are found only in central and southern Af-
Answer B is incorrect. Blood donation would rica. They have an animal reservoir that has
not be expected to cause anemia in a healthy not been found. Treatment is supportive, and
individual, and there is no reason to believe symptoms include massive hemorrhage from
Test Block 6

that this patient has recently donated blood. the mucous membranes accompanied by high
fevers.
Answer C is incorrect. Folic acid deficiency
can result in a megaloblastic anemia. This type Answer D is incorrect. Ebola virus and Mar-
of deficiency is often seen in alcoholics or indi- burg virus are members of the Filovirus ge-
nus, which cause hemorrhagic fever. Both
Test Block 6  •  Answers 685

are found only in central and southern Af- onset bone marrow suppression resulting in
rica. They have an animal reservoir that has anemia, neutropenia, and thrombocytopenia.
not been found. Treatment is supportive, and This condition is frequently diagnosed in chil-
symptoms include massive hemorrhage from dren. Lymphoma cells typically are positive for
the mucous membranes accompanied by high immature B-cell marker terminal deoxynucleo-
fevers. tidyl transferase.
Answer E is incorrect. Rhabdovirus is the Answer E is incorrect. The symptoms of
causative agent of rabies. It is possible to be- mumps virus include a prodromal stage of fe-
come infected with rabies from a rodent; how- ver, malaise, and anorexia followed by tender
ever, the incubation period is much longer swelling of the parotid glands that typically re-
(weeks to a year), and the later stages of the dis- solves within one week. Although this patient
ease are classically acute encephalitis. has a lesion on his mandible that is suggestive
of parotitis, his symptoms of fatigue and mal-
7. The correct answer is A. This is a case of aise have lasted for six months, which is longer
Burkitt lymphoma. Burkitt lymphoma is than the typical clinical course of mumps. Two
associated with Epstein-Barr virus infection, clinically significant complications of mumps
resulting in a t(8;14) translocation. This fre- virus include orchitis and aseptic meningitis.
quently involves the mandible as well as the Furthermore the histologic characterization of
kidneys, ovaries, and adrenals. Histology typi- the tumor makes mumps virus less likely.
cally demonstrates a characteristic “starry-sky”
pattern; this pattern is produced by macro- 8. The correct answer is B. Epinephrine is an
phages, which ingest tumor cells, producing agonist at a1, a2, b1, and b2 receptors; phentol-
light spots against a background of highly mi- amine is an antagonist at a1 and a2 receptors.
totic basophilic lymphoma cells. Burkitt lym- Therefore, after the administration of phentol-
phoma is among the most common cancers in amine, epinephrine administration stimulates
Africa. only b receptors, which results in decreased
blood pressure. This is called epinephrine
Answer B is incorrect. Chronic myelogenous

Full-Length Exams
reversal because epinephrine originally in-
leukemia (CML) is characterized by a mark-
creased blood pressure and then produced the
edly elevated WBC count, fatigue, night
opposite effect after phentolamine administra-
sweats, low-grade fever, abdominal fullness
tion.
secondary to splenomegaly, and sternal tender-
ness. CML is associated with the Philadelphia Answer A is incorrect. Acetylcholine stimu-
chromosome, t(9;22). lates the non-innervated muscarinic (M3) re-
ceptors that are located on endothelial cells of
Answer C is incorrect. The most common
the vasculature. Stimulation of these receptors
presentation of Hodgkin lymphoma includes
releases endothelial-derived relaxing factor (ni-
constitutional symptoms including night
tric oxide), which produces a relaxation of the
sweats, fever, or weight loss, and mediastinal
neighboring smooth muscle cells, leading to a
lymphadenopathy. Biopsy of the nodes demon-
decrease in blood pressure. Neostigmine, an
strates the characteristic Reed-Sternberg cells
acetylcholinesterase inhibitor, would simply
with reactive lymphocytes, which are large and
prolong the action of acetylcholine at its recep-
bi-nucleate with a prominent nucleolus, giving
tors and would thus indirectly cause a decrease
the cell its “owl-eye” appearance. B cells asso-
in blood pressure.
ciated with Hodgkin lymphoma may express
Test Block 6

the T-cell marker interleukin-21. Answer C is incorrect. Isoproterenol, a non-


specific b-agonist, decreases blood pressure by
Answer D is incorrect. Acute lymphoblastic
stimulation of b2 receptors in the vasculature.
lymphoma typically presents with circulating
Epinephrine, norepinephrine, and phenyleph-
lymphoid blasts (leukemic phase) and a me-
rine all increase blood pressure, so the remain-
diastinal mass. The symptoms include rapid-
686 Section III: Full-Length Examinations  •  Answers

ing answers must be eliminated by examining Answer C is incorrect. The physical examina-
the effects of drug Y on drug X. tion largely excludes the possibility of disc her-
niation as a cause of the patient’s symptoms be-
Answer D is incorrect. Norepinephrine is an
cause of absent positive straight leg raises and
agonist at a1, a2, and b1 receptors; propranolol
pain that is neither aggravated by activity nor
is a nonselective b-antagonist. After administra-
relieved by resting. Furthermore, the perianal
tion of propranolol, norepinephrine can stim-
region is innervated by sacral nerve roots that
ulate only a receptors, which will still cause
would not be impinged by a disc herniation at
vaso­constriction (primarily via a1 stimulation
the L4-5 level.
in the vasculature) and therefore increase
blood pressure. Answer D is incorrect. The triad for spinal
epidural abscess includes back pain, fever, and
Answer E is incorrect. Phenylephrine is an
progressive weakness. This patient does not
a1 agonist, and hexamethonium is a nicotinic
have the second two symptoms, making ab-
ganglionic blocker. Hexamethonium admin-
scess a less likely possibility.
istration will eliminate the baroreceptor re-
sponse after the second phenylephrine admin- Answer E is incorrect. Vertebral osteomyelitis
istration by blocking the peripheral ganglia. would most likely present in a patient that was
However, phenylephrine will still reach the a1 suffering constitutional signs and symptoms,
receptors on the vasculature to produce an in- as opposed to the relatively focal findings pre-
crease in blood pressure. sented in this case. Characteristically, a patient
with osteomyelitis would have night sweats
9. The correct answer is A. This patient is suf- and fever. The patient would have symptoms
fering from a cord compression syndrome due that would point to the source of the hematog-
to a conus medullaris tumor. These tumors enous dissemination of the infectious agent to
are relatively uncommon and can be very dif- the spine (eg, urinary tract infection).
ficult to diagnose because they are difficult to
differentiate from tumors of the cauda equina. 10. The correct answer is B. Turner syndrome is
Patients can manifest symptoms of one or both a chromosomal abnormality that results from
Full-Length Exams

syndromes. Night and rest pain is an immedi- the loss of all or part of one of the two X chro-
ate red flag for metastatic disease, multiple my- mosomes (the absence of one set of genes from
eloma, or spinal infections. The x-ray films and the short arm of one X chromosome) in a fe-
normal laboratory values help to make meta- male fetus. This patient has the characteristic
static disease or myeloma less likely. The ex- short stature and webbed neck often seen in
amination is consistent with conus medullaris Turner patients. Other clinical manifestations
syndrome because of the relatively rapid, bilat- of Turner syndrome include a flat, shield-like
eral onset of moderate back pain with a mini- chest (with widely spaced nipples), wide carry-
mal radicular component and preserved ankle ing angle at the elbows, and congenital lymph-
reflexes. These patients tend to have perianal edema of hands and feet in neonates. Patients
numbness and urinary retention with an atonic with Turner syndrome have a higher risk of
rectal sphincter, as opposed to the saddle an- cardiac abnormalities than their peers, the
esthesia more typically found in cauda equina most common being coarctation of the aorta.
syndrome. This patient’s presentation warrants Classically the coarctation is preductal, and
empiric steroids and emergent MRI. can result in a small left arm due to compro-
mised blood flow if the left subclavian artery is
Answer B is incorrect. Guillain-Barré syn-
involved.
Test Block 6

drome is a symmetrical ascending weakness


that commonly follows diarrheal or flu-like ill- Answer A is incorrect. A postductal coarcta-
ness in young adults. It is not associated with tion is not commonly found in patients with
focal neurological pain and is much less com- Turner syndrome. Furthermore, we would not
mon in elderly patients. expect a small left arm in a patient with a post-
Test Block 6  •  Answers 687

ductal coarctation because the left subclavian factor XIIIa, which cross-links fibrin polymers
artery must be intact. and solidifies the clot.
Answer C is incorrect. Subclavian steal syn- Answer E is incorrect. The hydroxylation of
drome refers to retrograde flow in a vertebral proline and lysine residues requires the coen-
artery due to a stenosed subclavian artery as- zyme ascorbic acid, or vitamin C. Deficiency
sociated with neurologic symptoms. This is of vitamin C results in scurvy, a defective con-
not more common in patients with Turner syn- nective tissue syndrome that results in swollen
drome than in their peers. gums, bruising, anemia, and poor wound heal-
ing. Vitamin C is a water-soluble vitamin; oral
Answer D is incorrect. Patients with Turner
antibiotic use does not lead to its deficiency.
syndrome do not have a higher incidence of te-
tralogy of Fallot than their peers.
12. The correct answer is C. The potency of in-
Answer E is incorrect. Patients with Turner haled anesthetics is quantified as the mini-
syndrome do not have a higher incidence of mum alveolar concentration (MAC). This is
transposition of the great vessels than their the concentration of inhaled gas that is needed
peers. to eliminate movement in 50% of patients who
are challenged by surgical incision. For po-
11. The correct answer is B. Vitamin K-depen- tent anesthetics, the MAC will be numerically
dent γ-glutamyl carboxylase converts glutamic small, meaning that it is inversely proportional
acid residues to γ-carboxyglutamic acid (GLA) to the anesthetic’s potency. Drug C, which
residues on clotting factors II, VII, IX, and has the smallest MAC value, is thus the most
X, as well as proteins C and S. GLA residues potent. The blood solubility of an anesthetic
function to bind calcium, which is required is the physical property that determines both
to activate the factors. Vitamin K deficiency is the speed of induction and time to recovery.
rare because it is synthesized by gut flora. This Drugs with low blood solubility, such as ni-
patient’s deficiency is probably secondary to trous oxide, will rapidly induce anesthesia, and
elimination of these organisms by oral antibi- patients will recover quickly. In contrast, an

Full-Length Exams
otics. This deficiency is also seen in neonates anesthetic gas with high blood solubility, such
who have sterile guts and a relatively new liver. as halothane, will have a longer time to induc-
Easy bruising, melena, hematuria, and splinter tion and a slower time to recovery. Therefore,
hemorrhages can be seen with vitamin K defi- drug C, which has the lowest blood solubility,
ciency. will have the most rapid induction.
Answer A is incorrect. Activation of anti- Answer A is incorrect. The MAC for a drug is
thrombin III is the function of heparin, which inversely related to its potency. Drug A has a
inhibits serine proteases of the clotting cas- larger MAC than drug D and will be less po-
cade. tent than drug D.
Answer C is incorrect. The cross-linking of Answer B is incorrect. Drug B, with a solubil-
fibrin polymers is the function of factor XIIIa, ity of 1.0 (100% soluble in blood), will dissolve
which is activated by thrombin in the clotting completely in the bloodstream, whereas drug
cascade. This cross-linking functions to solidify C, with its solubility value of 0.1, will be only
the clot after the convergence of the intrin- 10% dissolved in blood. Drug B is more solu-
sic and extrinsic clotting cascades. Thrombin ble than Drug C.
functions both to convert fibrinogen to fibrin
Answer D is incorrect. An anesthetic with
Test Block 6

and to activate factor XIIIa.


greater blood solubility will have a slower in-
Answer D is incorrect. The hydrolyzation of duction time. Drug D, which is more soluble
fibrinogen to form fibrin is the function of ac- in blood than drug C, will therefore induce
tive thrombin, a serine protease in the clotting anesthesia less rapidly than drug C.
cascade. Thrombin also functions to activate
688 Section III: Full-Length Examinations  •  Answers

Answer E is incorrect. Recovery time from hibitor of the renal enzyme dihydropeptidase-I
anesthesia is based on the blood solubility of that otherwise inactivates imipenem in the re-
a gas. Gases with high blood solubility result nal tubules. Given its broad activity, imipenem
in slower recovery when compared with drugs is used clinically to treat infections caused by
that have lower blood solubility. Drug B has gram-positive cocci, gram-negative rods, and
the highest blood solubility of all those that are anaerobes. It is also the drug of choice for
listed and thus will have the slowest recovery treatment of Enterobacter infection. It is not
time of all. used in the treatment of H pylori.
Answer D is incorrect. Ranitidine is a hista-
13. The correct answer is B. This patient most
mine2 blocker that decreases acid production
likely suffers from peptic ulcer disease caused
and is used in the treatment of gastroesopha-
by Helicobacter pylori, a bacterium that causes
geal reflux disease, peptic ulcers, and gastri-
70% of gastric ulcers and up to 90% of duode-
tis. Omeprazole is a proton pump inhibitor
nal ulcers. Those from certain ethnic groups
that decreases acid production and is used in
often have a higher rate of H pylori infection,
the treatment of peptic ulcer disease, gastritis,
especially if they grew up in areas with poor
esophageal reflux, and Zollinger-Ellison syn-
sanitation. Peptic ulcer disease typically pre­
drome. Bismuth binds to the base of a gastric
sents with burning substernal epigastric pain.
ulcer to provide physical protection to the gas-
In the case of gastric ulcers, the pain increases
tric mucosa; it also allows bicarbonate secre-
after meals; for duodenal ulcers, the pain im-
tion in order to re-establish the pH gradient in
proves after eating. Infection with H pylori
the mucous layer and stimulate ulcer healing.
can be diagnosed with a urease breath test
None of these medications effectively eradi-
(H pylori contains the enzyme urease that
cates H pylori.
cleaves urea to ammonia), or with biopsy that,
on silver staining, demonstrates a spiral-shaped Answer E is incorrect. Vancomycin is an
flagellated organism adherent to the mucosal antimicrobial that acts by inhibiting cell
epithelium. Treatment of H pylori involves wall mucopeptide formation by binding the
therapy consisting of bismuth, metronidazole, D-alanyl-D-alanine portion of cell wall precur-
Full-Length Exams

omeprazole, and either tetracycline or amoxi- sors. It is used for serious, gram-positive multi-
cillin; or metronidazole in combination with drug-resistant organisms, including Staphylo-
omeprazole and clarithromycin. coccus aureus and Clostridium difficile. It is not
used in the treatment of H pylori. Gentamicin
Answer A is incorrect. Ampicillin is an amino-
belongs to the antimicrobial family of amino-
penicillin antimicrobial with a wider spectrum
glycosides, which are bactericidal and act by
than penicillin. It is penicillinase-sensitive but
inhibiting the formation of the initiation com-
is often combined with clavulanic acid to en-
plex and causing misreading of mRNA. They
hance its spectrum. It is used to treat certain
require oxygen for uptake and are therefore
gram-positive and gram-negative infections, in-
ineffective against anaerobes. They are used to
cluding Haemophilus influenzae, Escherichia
treat severe gram-negative rod infections, but
coli, Listeria, Proteus, Salmonella, entero-
are not used in the treatment of H pylori. Az-
cocci, and Shigella, but is not effective against
treonam is a monobactam antimicrobial that
H pylori. Clindamycin is a bacteriostatic anti-
is resistant to b lactamases. It acts by inhibit-
microbial that acts by blocking peptide bond
ing cell wall synthesis and is used clinically to
formation at the 50S ribosomal subunit. It is
treat gram-negative-rod infections. However, it
used to treat anaerobic infections, including
is not used in the treatment of H pylori.
Test Block 6

Bacteroides fragilis and Clostridium perfringens.


It is not used in the treatment of H pylori.
14. The correct answer is A. The clinical picture
Answer C is incorrect. Imipenem is a broad- of hypotension (cold, clammy extremities,
spectrum, b-lactamase-resistant carbapenem. tachycardia, and hypotensive syncope) and res­
It is always administered with cilastatin, an in- piratory compromise (hypoxia and use of ac-
Test Block 6  •  Answers 689

cessory muscles to breathe) indicates that this causes a drop in blood pressure as you would
woman is likely in anaphylactic shock. Given expect. Depletion of these neurochemicals can
the setting of a picnic, the venom from a bee also cause depression.
sting may have triggered the reaction. Ana-
Answer B is incorrect. Trazodone, a heterocy-
phylactic shock may be mediated by binding
clic or second-generation antidepressant, works
of C3a and C5a to IgE, causing histamine
by inhibiting serotonin reuptake, and acting
release. C3 is converted to C3a and C3b by
as a partial agonist at serotonin receptors. It is
one of two C3 convertases. Similarly, C5 is
useful for the treatment of depression with in-
converted to C5a and C5b by one of two C5
somnia. Male patients should be warned of its
convertases. Both C3a and C5a circulate in
potential to cause priapism.
the bloodstream and trigger anaphylaxis. Thus
profuse release of C3a and C5a could lead to Answer C is incorrect. Monoamine oxidase
anaphylactic shock. (MAO) catabolizes monoamines such as do-
pamine, norepinephrine, and serotonin. Phen-
Answer B is incorrect. Granulocyte macro-
elzine, tranylcypromine, and selegiline are all
phage colony-stimulating factor triggers the
examples of MAO inhibitors that increase the
growth and differentiation of granulocytes,
levels of these neurotransmitters in the brain.
monocytes, and megakaryocytes.
Selegiline is an MAO-B inhibitor also used to
Answer C is incorrect. IgA is most abundant treat parkinsonism. These drugs can cause hy-
at mucous membranes and is responsible for pertensive crises when taken with substances
defense against the attachment of bacteria and high in tyramine such as red wine and cheese.
viruses. It has no role in anaphylaxis. When taken alone they can cause dose-related
orthostatic hypotension.
Answer D is incorrect. Interleukin-3 is se-
creted by activated T lymphocytes and func- Answer E is incorrect. Drugs such as fluox-
tions similar to granulocyte macrophage etine, sertraline, and citalopram are selective
colony-stimulating factor. serotonin reuptake inhibitors (SSRIs), and
work by selectively blocking the reuptake of
Answer E is incorrect. When C3 cleavage re-

Full-Length Exams
serotonin from the synaptic cleft. Adverse ef-
sults in activation of C5, C6, C7, C8, and C9,
fects of SSRIs include nausea, headache, anxi-
these components form the membrane attack
ety, insomnia, and sexual dysfunction (delayed
complex (MAC), which physically inserts into
ejaculation and anorgasmia). This last adverse
the membranes of target cells or bacteria and
effect is taken advantage of in the treatment of
lyses them. Although important for the elimi-
premature ejaculation. Their adverse effects
nation of pathogens, the MAC is not involved
can mimic the akathisia (subjective sense of
in triggering anaphylaxis.
restlessness) often seen with antipsychotics. A
15. The correct answer is D. Nortriptyline is a fatal serotonin syndrome can result from com-
tricyclic antidepressant that inhibits the reup- bining SSRIs with MAO inhibitors such as
take of both norepinephrine and serotonin phenelzine.
from the synaptic cleft of central nervous sys-
16. The correct answer is D. This is a case of Ew-
tem (CNS) neurons. Serious adverse effects
ing sarcoma, which can present with symp-
include arrhythmia, a prolonged QT interval,
toms similar to those of an infection. The
and seizures (remember the 3 Cs: Coma, Con-
x-ray illustrates the classic location and ap-
vulsions, and Cardiotoxicity). Anticholinergic
pearance of the sarcoma. Ewing sarcoma is a
adverse effects are common (eg, dry mouth
Test Block 6

small blue-cell tumor of childhood that most


and urinary retention).
commonly arises in the medullary cavity in
Answer A is incorrect. Reserpine depletes pre- the diaphyses of long bones and the pelvis. It
synaptic vesicles in the nerve terminal of their is found most often in boys <15 years of age.
catecholamine and serotonin stores, which
690 Section III: Full-Length Examinations  •  Answers

Patients typically present with localized pain Answer D is incorrect. The jejunum is sup-
over the site of the tumor. The periosteal reac- plies by the superior mesenteric artery; it is not
tion results in reactive bone deposition in an a watershed area.
onion-skin manner. The tumor cells carry the
Answer F is incorrect. The stomach is richly
t(11;22) translocation 85% of the time, and
supplied by the celiac trunk; it is not a water-
this would be seen on karyotype. Positive stain-
shed area.
ing for glycogen with periodic acid-Schiff is
also seen. In all cases, there is a fusion of the 18. The correct answer is D. A nonsense mutation
EWS gene on 22q12 to a transcription factor. occurs when a single base substitution in DNA
Answer A is incorrect. t(8;14) is most com- (in this case, cytosine to adenosine) results in a
monly associated with Burkitt lymphoma. chain termination codon.
Answer B is incorrect. t(9;22), the Philadel- Answer A is incorrect. A frameshift mutation
phia chromosome, is most commonly associ- involves a deletion or insertion that is not an
ated with chronic myeloid leukemia. exact multiple of three base pairs and therefore
changes the reading frame of the gene down-
Answer C is incorrect. t(11;14) is associated
stream of the mutation.
with mantle cell lymphoma.
Answer B is incorrect. An insertion muta-
Answer E is incorrect. Although the translo-
tion is a chromosomal abnormality in which
cation t(21;22) is seen in 5%-10% of cases of
a DNA segment from one chromosome is in-
Ewing sarcomas, it is not as common as the
serted into a non-homologous chromosome,
t(11;22) translocation.
maintaining the appropriate reading frame.
Answer F is incorrect. The translocation
Answer C is incorrect. A missense mutation
t(15;17) is associated with the M3 type of acute
is a mutation that changes a codon specific for
myeloid leukemia. This type is treatable by all-
one amino acid to specify another amino acid.
trans retinoic acid.
Answer E is incorrect. A silent mutation pro-
Full-Length Exams

17. The correct answer is E. Given the patient’s duces a mutant gene that has no detectable
large blood loss and development of acido- phenotypic effect. The mutation is usually a
sis, ischemic bowel is the likely culprit. The point mutation, often in the third position of
splenic flexure of the large bowel is most vul- the codon.
nerable to ischemia from hypoperfusion be-
cause it lies at the junction of two vascular ter- 19. The correct answer is C. Sarcoidosis is a sys-
ritories, the superior and inferior mesenteric temic granulomatous disease of unknown ori-
arteries. (This watershed phenomenon also oc- gin that particularly involves the lungs. The
curs in the brain in stroke.) pathologic hallmark is the presence of nonca-
seating granulomas. Bilateral hilar adenopathy
Answer A is incorrect. The cecum is supplied
can be the presenting sign in asymptomatic pa-
by the superior mesenteric artery; it is not a
tients with sarcoidosis. Sarcoidosis is associated
watershed area.
with increased levels of angiotensin-converting
Answer B is incorrect. The hepatic flexure enzyme (ACE), but testing serum ACE levels
of the large bowel is supplied by the superior does not provide a definitive diagnosis of sar-
mesenteric artery; unlike the splenic flexure, it coidosis because of the relatively low specific-
is not in a border zone between two vascular ity and sensitivity of the test. Nevertheless, for
Test Block 6

territories. any given test, the PPV (ie, the probability that
a positive test result is truly positive for the dis-
Answer C is incorrect. The ileum is supplied
ease one is looking for) increases and the NPV
by the superior mesenteric artery; it is not in a
(ie, the probability that a negative test result is
watershed area.
a truly negative for the disease one is looking
Test Block 6  •  Answers 691

for) decreases as the prevalence of the disease and remains un-cross-linked and weak. Scurvy
increases. In the United States, sarcoidosis manifests as hemorrhages, widening of the
used to be strongly associated with residence epiphyseal cartilage into bone, gingival swell-
in the southeastern United States; however, ing, and impaired wound healing.
recent studies have suggested that the disease
Answer C is incorrect. Abnormalities in the
may be common in other areas as well.
synthesis of type I collagen result in osteogene-
Answer A is incorrect. For any given test, as sis imperfecta. This typically presents as an in-
the prevalence of the disease increases, the fant or child with multiple fractures at a young
PPV increases and the NPV decreases. age.
Answer B is incorrect. For any given test, as Answer D is incorrect. EDS is clinically and
the prevalence of the disease increases, the genetically heterogenous. Each type affects
PPV increases and the NPV decreases. collagen synthesis and structure. While Mar-
fan syndrome can result in hyperextensible
Answer D is incorrect. For any given test, as
joints, EDS is more noted for its resulting in
the prevalence of the disease increases, the
hyperextensible skin.
PPV increases and the NPV decreases.
Answer E is incorrect. For any given test, as 21. The correct answer is B. The patient experi-
the prevalence of the disease increases, the enced progressive left ventricular failure that
PPV increases and the NPV decreases. eventually led to his decompensated heart fail-
ure. A fall in his forward cardiac output led to
20. The correct answer is E. Marfan syndrome increased pulmonary venous congestion, pul-
and Ehlers-Danlos syndrome (EDS) both pre­ monary venous distention, and transudation
sent with joint hyperextensibility. One form of of fluid into the lung. This backs up into the
EDS presents with vascular changes producing right heart, leading to symptoms of right heart
berry aneurysms that could result in a heart failure (edema, distended jugular vein). The
murmur. However, lens subluxation or dislo- image shows intra-alveolar fluid, engorged cap-
cation (ectopia lentis) points strongly toward illaries, and hemosiderin-laden macrophages

Full-Length Exams
Marfan syndrome. Marfan develops as a result (also called “heart failure” cells), the hallmarks
of an inherited autosomal-dominant defect in of pulmonary edema.
fibrillin. Fibrillin, the scaffolding for elastic
Answer A is incorrect. Dendritic cells are not
fibers, is found abundantly in the aorta, liga-
prominent in lung tissue, and are not known
ments, and the ciliary zonules of the lens.
to be affected in the lung secondary to pul-
Answer A is incorrect. In the endoplasmic re- monary edema. Dendritic cells are antigen-
ticulum, collagen synthesis is first translated, presenting cells, located mainly in the skin
then hydroxylated, then glycosylated. Glyco- and the inner lining of nose, lungs, and GI
sylation is the attachment of carbohydrates to tract. They interact with T- and B-lymphocytes
the newly bound hydroxy groups. The failure in lymphoid tissues.
of glycosylation results in osteogenesis imper-
Answer C is incorrect. Although inflamma-
fecta, type I of which manifests clinically as
tion from the pulmonary edema can cause
brittle bones, blue sclerae (not lens disloca-
an increase in the number of T-lymphocytes,
tions), hearing deficits, and dental imperfec-
hemosiderin-laden macrophages are much
tions.
more characteristic.
Answer B is incorrect. Scurvy is caused by a
Test Block 6

Answer D is incorrect. Type I pneumocytes


deficiency of vitamin C. Vitamin C is needed
comprise 97% of the alveolar surfaces and line
to complete the hydroxylation of preprocol-
the alveoli. Their numbers are not increased in
lagen in the endoplasmic reticulum. Without
patients with pulmonary edema. Type I pneu-
vitamin C, preprocollagen cannot achieve a
mocytes are thin cells that line the alveolar
stable helical formation for its hydroxylation,
692 Section III: Full-Length Examinations  •  Answers

surface of the lung. They are responsible for in intracellular calcium activates phospho­
gas exchange. lipase C, cleaving PIP2 into IP3 and DAG.
IP3 causes the sarcoplasmic reticulum to spill
Answer E is incorrect. Type II pneumocytes
calcium intracellularly, which stimulating the
produce pulmonary surfactant, which de-
release of insulin. This insulin release can be
creases the alveolar surface tension. They also
responsible for the symptoms of hypoglycemia
serve as precursors to type I pneumocytes and
seen in this patient.
other type II pneumocytes. However, they are
not significantly increased in number in pul- Answer E is incorrect. Ultralente insulin is
monary edema. a long-acting insulin that has its peak effects
around 16-28 hours after dosing and lasts
22. The correct answer is D. This patient exhib- around 18-24 hours. Like any other exogenous
its symptoms of hypoglycemia secondary to his insulin (especially long-acting insulins), ultra-
current medication. Exogenous insulin and lente insulin may cause hypoglycemia in el-
the sulfonylureas are effective treatments for derly patients who skip meals, thus leaving in-
type 2 diabetes, but they can cause hypoglyce- sulin relatively unopposed.
mia. The orally active antihyperglycemic drugs
(metformin, rosiglitazone, and pioglitazone) 23. The correct answer is A. This description is
are classified as such because they won’t cause classic for fifth disease, which is caused by par-
hypoglycemia in a euglycemic person or a pa- vovirus B19. It is a pediatric illness common
tient with type 2 diabetes mellitus, but instead in children 3-12 years old. The rash is called
combat hyperglycemia via their mechanism erythema infectiosum and develops after fever
of action. Pioglitazone is the only agent listed has resolved as a bright, blanchable erythema
that doesn’t have the potential for the hypogly- on the cheeks (“slapped cheeks”) with peri-
cemic adverse effects described in the vignette. oral pallor. A more diffuse rash appears on the
trunk and extremities and may wax and wane
Answer A is incorrect. Glargine is a long-
with temperature changes over three weeks.
acting synthetic insulin that mimics the endog-
However, the disease itself is self-limiting, re-
enous basal insulin levels. Like any other exog-
Full-Length Exams

quiring no treatment.
enous insulin (especially long-acting insulins),
glargine may cause hypoglycemia in elderly Answer B is incorrect. The rash seen in fifth
patients who skip meals, thus leaving insulin disease will dissipate on its own after a few
relatively unopposed. weeks and will not respond to corticosteroid
cream.
Answer B is incorrect. Glipizide, a second-
generation sulfonylurea, acts by blocking Answer C is incorrect. Although the rash of
ATP-sensitive potassium channels, which de- erythema infectiosum is a centrally distrib-
polarizes pancreatic b islet cells, causing the uted maculopapular eruption (like the rashes
voltage-gated calcium channels to open. A rise that accompany rickettsial illnesses, drug-in-
in intracellular calcium activates phospho­ duced eruptions, and Still’s diseases), it is self-
lipase C, cleaving PIP2 into IP3 and DAG. limiting, requiring no treatment.
IP3 causes the sarcoplasmic reticulum to spill
Answer D is incorrect. Erythema infectiosum
calcium intracellularly, which stimulating the
is caused by infection with parvovirus B19 and
release of insulin. This insulin release can be
will not respond to antibiotic treatment.
responsible for the symptoms of hypoglycemia
seen in this patient. Answer E is incorrect. Erythema infectiosum
Test Block 6

is caused by infection with parvovirus B19 and


Answer C is incorrect. Glyburide, a second-
will not respond to antibiotic treatment.
generation sulfonylurea, acts by blocking
ATP-sensitive potassium channels, which de- 24. The correct answer is C. The patient has a
polarizes pancreatic b islet cells, causing the pheochromocytoma, a tumor of the adrenal
voltage-gated calcium channels to open. A rise
Test Block 6  •  Answers 693

medulla. Pheochromocytomas can secrete cat- lar respiration. Tissues with the highest oxygen
echolamines (norepinephrine, epinephrine, demands, such as the heart, brain, and liver,
and dopamine), resulting in episodes charac- are most significantly affected because cyanide
terized by headaches, diaphoresis, palpitations, prevents oxygen from binding to cytochrome
and severe hypertension. Urinary catechol- oxidase and serving as the final electron ac-
amine metabolites and plasma catecholamine ceptor in the chain. On physical examination
levels are elevated in these patients. Most of the retinal arteries and veins are bright red due
these tumors are benign, unilateral, and lo- to absent tissue oxygen extraction. Addition-
cated in the adrenal gland. Pheochromocyto- ally, in some patients there is a smell of bitter
mas arise from the chromaffin cells of the ad- almonds on the breath. These findings are not
renal medulla, which are derived from neural present with carbon monoxide inhalation, the
crest cells. other common toxin associated with smoke
inhalation. Treatment includes induction of
Answer A is incorrect. The chromaffin cells
methemoglobinemia with a nitrite, then ad-
of the adrenal medulla are not of endodermal
ministration of a sulfate.
origin. Adult endoderm derivatives include the
gut epithelium and its derivatives. Answer A is incorrect. Carbon monoxide is a
colorless, odorless, tasteless, nonirritating gas
Answer B is incorrect. The adrenal cortex,
produced from the incomplete combustion
which secretes aldosterone, glucocorticoids,
of any carbon-containing material. Common
and sex hormones, is derived from the meso-
sources include smoke inhalation in fires (as
derm. The adrenal cortex does not secrete
in this patient), automobile exhaust fumes,
catecholamines, which are responsible for the
and poorly vented charcoal or gas stoves.
hyperadrenergic episodes seen in patients with
Carbon monoxide binds to hemoglobin 250
pheochromocytomas.
times more strongly than oxygen, resulting in
Answer D is incorrect. The chromaffin cells reduced oxyhemoglobin saturation and de-
of the adrenal medulla are not of neuroecto- creased blood oxygen-carrying capacity, as well
dermal origin. Adult neuroectoderm deriva- as impairing oxygen delivery at the tissues. The

Full-Length Exams
tives include the neurohypophysis, CNS neu- majority of patients complain of headache, diz-
rons, oligodendrocytes, astrocytes, ependymal ziness, and nausea; prolonged exposure results
cells, and the pineal gland. in impaired thinking, syncope, coma, convul-
Answer E is incorrect. The chromaffin cells of sions, and death. While patients with carbon
the adrenal medulla are not derived from sur- monoxide poisoning may have bright red ve-
face ectoderm. Surface ectoderm gives rise to nous blood, as seen in this patient, they do
the adenohypophysis, lens of the eye, epider- not have the smell of bitter almonds on their
mis, and the epithelial linings of the skin, ear, breath. Treatment is with 100% oxygen, either
eye, and nose. by rebreather or through an endotracheal tube;
in severe cases a hyperbaric chamber may be
25. The correct answer is C. Cyanide is a very used.
toxic compound that can be formed in the Answer B is incorrect. Substances such as
high-temperature combustion of many materi- ammonia or chlorine cause direct injury to
als, such as polyurethane, acrylonitrile, nylon, exposed tissues. Symptoms include shortness
wool, and cotton, thus making cyanide poi- of breath, severe throat pain, vomiting, and he-
soning common in the setting of smoke inha- moptysis. The ocular symptoms in this patient
lation. The most common cause of cyanide are not characteristic of this type of poisoning.
Test Block 6

poisoning in industrialized countries is house-


hold fires. Cyanide modifies the iron within Answer D is incorrect. Sarin gas irreversibly
cytochrome oxidase (cytochrome aa3) in the inhibits acetylcholinesterase, resulting in an
mitochondria, thereby abnormally interrupting overload of acetylcholine at synapses. As a re-
the electron transport chain and halting cellu- sult, the body experiences a parasympathetic
694 Section III: Full-Length Examinations  •  Answers

overload and flaccid paralysis. Symptoms of as it acts to induce ovulation for egg collection.
sarin poisoning depend on the degree of expo- It successfully treats women with anovulatory
sure and the form of the toxin; they resemble cycles but is not effective in women whose
some of the symptoms this patient is experienc- infertility is secondary to a pituitary abnormal-
ing. However, the circumstances of his injury ity. It induces ovulation by acting as a selec-
point to an adverse effect of smoke inhalation tive estrogen receptor antagonist in the hypo-
as opposed to poisoning with a biological war- thalamus and anterior pituitary, shielding the
fare agent. negative feedback of estrogen and disinhibit-
ing gonadotropin-releasing hormone (GnRH)
Answer E is incorrect. Ricin’s toxic effects are
release. GnRH stimulates the anterior pitu-
the result of its ability to inhibit protein synthe-
itary to release follicle-stimulating hormone
sis by stripping ribosomes of their purine bases.
and luteinizing hormone, which can in turn
It is one of the most potent cytotoxins known,
stimulate the ovary. A common adverse effect
with a single ricin molecule able to affect
of clomiphene-induced ovulation is hyper-
1500 ribosomes. Inhalation of ricin is charac-
stimulation of the ovaries and overrecruitment
terized by a delay of clinical features for up to
of follicles, producing a greater incidence of
six hours, followed by fever, itchy eyes, cough,
multiple births when compared with the gen-
congestion, chest tightness, dyspnea, and nau-
eral population. Clomiphene treatment can
sea. Ricin is derived from the same plant that
also cause menopausal symptoms because of
produces castor oil, and processing of castor
the induced hypoestrogenic state. Clomiphene
plants has been associated with occupational
also acts as an estrogen agonist in the liver,
disease.
stimulating protein synthesis; this increases
26. The correct answer is B. Human leukocyte clotting factors and thus increases the risk of
antigens (HLAs) DR4 and DR3 molecules are deep venous thrombosis. Other adverse effects
known to confer greater-than-average suscepti- of clomiphene include ovarian enlargement,
bility to type 1 diabetes. In fact, epidemiologic stillbirths, and temporary scintillating scotoma.
studies suggest that a carrier of both HLA-DR4 Answer A is incorrect. Esophagitis is a com-
Full-Length Exams

and HLA-DR3 is 50 times more susceptible to mon adverse effect in postmenopausal women
type 1 diabetes than a non-carrier. taking bisphosphonates such as alendronate
Answer A is incorrect. In would not be pos- and ibandronate to protect their bones against
sible for the patient to inherit two HLA-DR3 osteoporosis. To prevent esophagitis, patients
alleles, as only one grandparent has it. taking bisphosphonates are directed to take the
drug with water and to remain upright until af-
Answer C is incorrect. HLA-DR5 is associated ter the first meal of the day.
with pernicious anemia and Hashimoto thy-
roiditis. Answer B is incorrect. Nausea and vomiting
are common adverse effects of estrogen ther-
Answer D is incorrect. Although HLA-DR3 is apy, not anti-estrogen therapy. Estrogen ther-
associated with type 1 diabetes, HLA-DR7 is apy can be used as postmenopausal hormone
not and is associated with nephrotic syndrome. therapy (to treat the symptoms of menopause
Answer E is incorrect. HLA-DR8 is weakly as- and protect against osteoporosis) and for hypo-
sociated with primary biliary cirrhosis. gonadism.
Answer D is incorrect. Psychosis, nasal con-
27. The correct answer is C. Clomiphene is a se- gestion, orthostasis, nausea/vomiting, and
Test Block 6

lective estrogen receptor modulator, meaning headache may occur with cabergoline in the
that it acts as an estrogen receptor agonist in treatment of infertility due to hyperprolacti-
some tissues and as an estrogen receptor antag- noma. Cabergoline is an ergoline derivative
onist in others. Clomiphene treatment is one with potent inhibitory effects on prolactin se-
of the initial steps during in vitro fertilization, cretion by dopamine receptor agonist activity.
Test Block 6  •  Answers 695

Answer E is incorrect. Weight gain and fluid evidence is provided by the new-onset jaw
retention are adverse effects of gonadotropins, claudication and constitutional symptoms that
which can be used to increase ovarian follicu- usually present in patients with temporal arte-
lar maturation. ritis.
Answer A is incorrect. Analysis of joint fluid
28. The correct answer is B. This man may have
would be neither diagnostic nor possible in
been bitten by a rabid animal. The best pro-
this patient because she is only currently suf-
phylaxis for rabies infection is immediate ad-
fering from synovitis of her wrists and ankles.
ministration of human rabies immune globu-
lin to provide passive immunity, followed with Answer B is incorrect. Mesenteric angiog-
a series of five injections of killed rabies virus raphy is the primary imaging modality used
vaccinations to develop active immunity. The to determine the presence of aneurysms and
idea is to provide immunity while the virus is vessel narrowing in patients with polyarteritis
still in the incubation period. nodosa (PAN). PAN is a necrotizing vasculi-
tis that typically presents with constitutional
Answer A is incorrect. If the patient develops
signs as well as with myalgias, arthralgias, and
symptoms, the disease will have progressed to
fatigue. This patient’s girdle stiffness and jaw
an incurable stage.
claudication are not consistent with this diag-
Answer C is incorrect. The administration of nosis.
human rabies immune globulin is not the best
Answer D is incorrect. Testing for anti-double-
course of action, because it does not provide
stranded DNA and antinuclear antibody levels
long-term active immunity.
would be appropriate to diagnose lupus, which
Answer D is incorrect. If the patient develops is uncommon in patients this age and does not
symptoms, the disease will have progressed to involve jaw claudication.
an incurable stage.
Answer E is incorrect. Testing for rheumatoid
Answer E is incorrect. Providing the killed factor and anti-cytidine cyclic phosphate levels
rabies virus vaccine first and following with a would be appropriate to diagnose rheumatoid

Full-Length Exams
series of human rabies immune globulin does arthritis, which may produce symmetrical and
not give immediate passive immunity and is proximal joint symptoms. However, rheuma-
not the standard of care. toid arthritis does not cause jaw claudication
Answer F is incorrect. Although rabies is not and does not usually present for the first time
curable after symptoms develop, it is possible in someone this elderly.
to provide immunity to a patient who has been
30. The correct answer is B. Thiazides work
exposed before the virus replicates enough to
through binding to the chloride site of the
cause disease. Therefore doing nothing is not
sodium-chloride cotransporter on the luminal
the appropriate response.
surface of the early distal tubule and inhibiting
29. The correct answer is C. This patient has sodium-chloride reabsorption. Other thiazides
symptoms consistent with polymyalgia rheu- include chlorothiazide, chlorthalidone, and
matica (PMR) and giant cell arteritis (tempo- metolazone.
ral arteritis). She requires immediate steroids Answer A is incorrect. Most diuretic agents,
for treatment and subsequent temporal artery including thiazides, loop diuretics, and most
biopsy to confirm the diagnosis. PMR occurs potassium-sparing diuretics, act at the lumi-
Test Block 6

in 50% of patients with temporal arteritis and nal surface by inhibiting transporters. Ex-
involves symmetrical aching of the proximal ceptions are carbonic anhydrase inhibitors,
muscles and girdle stiffness. The elevated which inhibit a cytoplasmic enzyme, and the
erythrocyte sedimentation rate indicates a gen- potassium-sparing diuretic spironolactone,
eralized inflammatory process, and additional which inhibits steroid receptor function.
696 Section III: Full-Length Examinations  •  Answers

Answer C is incorrect. Loop diuretics such Answer D is incorrect. Haemophilus is a gram-


as furosemide, bumetanide, torsemide, and negative coccobacillus that can be found in
ethacrynic acid bind to the chloride-binding CSF if it is the cause of meningitis.
site of the sodium-potassium-chloride sym-
Answer E is incorrect. Neisseria meningitis
porter of the thick ascending limb of the loop
can be demonstrated by finding gram-negative
of Henle. Thiazides do not act at this site.
diplococci in bacterial culture of the CSF.
Answer D is incorrect. Aldosterone acts to
Answer F is incorrect. Toxoplasma is most
increase the number of sodium-potassium ex-
commonly a cause of encephalitis in immuno-
change channels in the basolateral membrane
compromised patients. It can, however, cause
at several sites, but especially in the collecting
meningitis. If this were the case, Toxoplasma
duct, effectively increasing sodium reabsorp-
tachyzoites would be seen in CSF.
tion and potassium excretion. Digitalis agents
can act to inhibit the action of the sodium- Answer G is incorrect. Treponema, the spiro-
potassium exchange pump, but are used as in- chete that causes syphilis, can also cause sub-
otropes, not diuretics. Thiazides do not act on acute meningitis, but this answer choice is in-
basolateral sodium channels or pumps. consistent with the image shown.
Answer E is incorrect. Thiazides do bind to 32. The correct answer is E. Active tissues pro-
the luminal surface of the distal convoluted tu- duce acid, CO2­, higher temperatures, and di-
bule, but to the chloride-binding sites. phosphoglycerate (DPG). Each of these shifts
the hemoglobin dissociation curve to the
31. The correct answer is B. This is a case of cryp-
right, facilitating oxygen unloading. A shift of
tococcal meningitis, which can be diagnosed if
the curve to the right means that at the same
encapsulated yeast forms are seen on India ink
partial pressure of oxygen, the percent satu-
stain of cerebrospinal fluid (CSF). Cryptococ-
ration of hemoglobin is lower. A shift to the
cus is the most common opportunistic cause
left means that at the same partial pressure of
of meningitis that presents in a subacute man-
oxygen, the percent saturation of hemoglobin
ner. Increased opening pressure on lumbar tap
Full-Length Exams

is higher, or hemoglobin affinity for oxygen is


is present in most patients with cryptococcal
higher. A shift to the left occurs with low CO2,
meningitis. The CSF findings are typical of
low DPG, low temperature, and alkaline pH.
fungal or mycobacterial meningitis; the differ-
A useful mnemonic is CADET, face right!:
entiating factor between bacterial and fungal/
CO2, Acid (or low pH), DPG, Exercise, and
mycobacterial meningitis is the predominant
Temperature shift the curve to the RIGHT.
cell type found in the CSF. Bacterial menin-
gitis results in increased polymorphonuclear Answer A is incorrect. Temperatures are in-
leukocytes, and fungal/tubercular meningitis creased, not decreased, in active tissues and
results in lymphocytosis. Intravenous (IV) drug serve to shift the hemoglobin dissociation
users are at increased risk of acquiring HIV curve to the right to facilitate oxygen unload-
and developing opportunistic infections such ing.
as cryptococcal meningitis. Answer B is incorrect. Temperatures are in-
Answer A is incorrect. Aspergillus is a filamen- creased, not decreased, in active tissues and
tous fungus, so hyphae would be seen on stain- serve to shift the hemoglobin dissociation
ing of fungal culture from CSF. curve to the right to facilitate oxygen unload-
ing.
Answer C is incorrect. Echovirus is also a
Test Block 6

cause of meningitis but is not diagnosed by Answer C is incorrect. Increased CO2 and H+
staining of CSF. It is the most common cause are present in active tissues and serve to shift
of aseptic meningitis. the hemoglobin dissociation curve to the right
to facilitate oxygen unloading. A shift to the
Test Block 6  •  Answers 697

left would produce higher oxygen binding and is usually intraparenchymal, but it can extend
would oppose oxygen unloading. to the intraventricular or subarachnoid space
at times.
Answer D is incorrect. Increased CO2 and
increased diphosphoglycerate are present in Answer B is incorrect. Atherosclerosis is asso-
active tissues and serve to shift the hemoglo- ciated with ischemic strokes, not SAH.
bin dissociation curve to the right to facilitate
Answer C is incorrect. Traumatic SAH is most
oxygen unloading. A shift to the left would pro-
often caused by bleeding into the CSF from
duce higher oxygen binding and would oppose
damaged blood vessels associated with cerebral
oxygen unloading.
contusion. While traumatic SAH is more com-
mon than non-traumatic (spontaneous) SAH,
33. The correct answer is E. The patient is likely
there is no evidence of trauma in the case
suffering from a subarachnoid hemorrhage
above. Indeed, the patient’s pain began during
(SAH). Subarachnoid hemorrhages begin
sleep, a relatively common finding in cases of
abruptly, occurring at night in 30% of cases,
nontraumatic aneurysmal SAH.
and are classically described as the “worst
headache of my life.” The headache is uni- Answer D is incorrect. While not the most
lateral in approximately one-third of patients. common cause of SAH, hypertensive hemor-
The onset of the headache may or may not be rhage is the most common cause of nontrau-
associated with a brief loss of consciousness, matic intraparenchymal hemorrhage. The
seizure, nausea, vomiting, focal neurologic pathogenesis of hypertensive hemorrhage
deficit, or stiff neck. There are usually no im- is unknown, but is believed to be related to
portant focal neurologic signs at presentation chronic pathologic effects of hypertension on
unless bleeding occurs into the brain and CSF the small penetrating blood vessels.
at the same time (meningocerebral hemor-
rhage). Rupture of arterial aneurysms repre- 34. The correct answer is A. This patient is likely
sents the chief cause of SAH. Upon rupture, suffering from neuroleptic malignant syn-
aneurysms release blood directly into the CSF drome, a severe and potentially life-threatening

Full-Length Exams
under arterial pressure. The blood spreads extrapyramidal adverse effect of antipsychotic
quickly within the CSF, rapidly increasing in- agents. Classic symptoms of this syndrome
tracranial pressure. Death or deep coma en- include hyperpyrexia, autonomic instability,
sues if the bleeding continues. The bleeding and severe muscle rigidity. Treatment requires
usually lasts only a few seconds, but rebleeding immediate discontinuation of all neurolep-
is common. Risk factors for intracranial aneu- tics, supportive care, and the administration
rysm include atherosclerotic disease, arteriove- of dantrolene. Dantrolene uncouples muscle
nous malformations (AVMs), adult polycystic excitation-contraction coupling by binding
kidney disease, and connective tissue disease. to the ryanodine receptor, and preventing ac-
When present, risk factors for aneurysmal rup- cumulation of intracellular calcium that is
ture include hypertension, smoking, alcohol, needed to sustain contraction.
and situations causing sudden elevations in Answer B is incorrect. Diazepam, a benzodi-
blood pressure. Other less common causes of azepine, is not indicated for the treatment of
SAH, including vascular malformations, bleed- neuroleptic malignant syndrome. It is first-line
ing diatheses, trauma, amyloid angiopathy, and treatment for status epilepticus and is used in
illicit drug use, feature bleeding that is less most alcohol withdrawal protocols.
abrupt and may continue over a longer period.
Test Block 6

Answer C is incorrect. Flumazenil, a competi-


Answer A is incorrect. AVM is the cause of tive antagonist at the γ-aminobutyric acid re-
SAH in approximately 4%-5% of cases. AVMs ceptor, is used to treat an overdose of benzodi-
are congenital anomalies in which there are azepines but is not indicated for the treatment
abnormal direct connections between arteries of neuroleptic malignant syndrome.
and veins. Hemorrhage secondary to an AVM
698 Section III: Full-Length Examinations  •  Answers

Answer D is incorrect. Haloperidol, a neuro- Answer E is incorrect. Patients with Sly syn-
leptic agent, would worsen the symptoms of drome have a defect in the beta-glucuronidase
neuroleptic malignant syndrome and therefore enzyme and are generally diagnosed as tod-
should not be given. dlers. The disorder is autosomal recessive, and
the presentation can resemble that of Hurler
Answer E is incorrect. Phenobarbital, a barbi-
syndrome. Mental retardation is not a signifi-
turate, would not be helpful in a patient who is
cant component of Sly syndrome, although
experiencing neuroleptic malignant syndrome.
various musculoskeletal abnormalities are
It is used as a third-line agent for status epilep-
common.
ticus when first- and second-line agents fail.
36. The correct answer is B. The patient de-
35. The correct answer is A. Hunter syndrome is
scribed is infected with Pneumocystis jiroveci
an X-linked disorder that is caused by a defi-
pneumonia, which is commonly associated
ciency of iduronate sulfatase. Although Hunter
with AIDS and immunosuppression. It is diag-
syndrome and Hurler syndrome are similar,
nosed with methenamine silver stain of lung
Hunter syndrome is notable for the absence of
biopsy tissue. P jiroveci pneumonia is treated
corneal clouding, which is present in Hurler
primarily with sulfamethoxazole-trimethoprim,
syndrome.
but it can be treated with pentamidine or
Answer B is incorrect. Hurler syndrome is a dapsone. Both sulfamethoxazole and trimeth­
severe disorder with a broad spectrum of clini- oprim inhibit the folate-synthesis pathway.
cal findings. It is generally diagnosed within In the folate synthesis pathway, pteridine and
the first year of life and is characterized by a PABA are incorporated into folic acid, which
variety of musculoskeletal abnormalities, cor- is important in the formation of nucleic acids
neal clouding, hepatosplenomegaly, and se- and certain amino acids.
vere mental retardation.
Answer A is incorrect. Many antifungal agents
Answer C is incorrect. Morquio syndrome inhibit ergosterol synthesis, including fluco­n-
is typically diagnosed around the age of one azole and terbinafine. Although Pneumocystis
Full-Length Exams

year and is characterized primarily by short jiroveci is a fungus, antifungals that block er-
stature and joint laxity. Other musculoskeletal gosterol synthesis are not effective in the treat-
abnormalities are also associated with this au- ment of this infection.
tosomally transmitted disorder. Some patients
Answer C is incorrect. Cell wall synthe-
demonstrate hepatosplenomegaly, mild cor-
sis is blocked by many antibiotics, including
neal clouding, and valvular heart disease.
penicillins, cephalosporins, and vancomycin.
Answer D is incorrect. There are multiple en- Trimethoprim-sulfamethoxazole is the drug
zyme deficiencies associated with Sanfilippo of choice for treating P jiroveci infection, and
syndrome, but this class of disorders is primar- does not act on the cell wall synthesis pathway.
ily distinguished by the CNS symptoms in
Answer D is incorrect. Inhibition of the small
these patients. Some of the physical abnormal-
ribosomal subunit (30S) is the mechanism of
ities seen in the other mucopolysaccharidoses
action for many antibiotics, including amino-
are also observed in Sanfilippo patients, but
glycosides and tetracyclines. None of these an-
the hallmarks of this disease are developmental
tibiotics is used for Pneumocystis jiroveci pneu-
delay and behavioral problems such as aggres-
monia.
sive tendencies and hyperactivity that manifest
in early childhood. Sleep disorders are also Answer E is incorrect. Inhibition of the larger
Test Block 6

common in these patients, and the physical ribosomal subunit (50S) is the mechanism of
findings typically develop after the behavioral action of chloramphenicol, erythromycin,
and sleep pattern abnormalities. clindamycin, and linezolid. None of these is
used to treat Pneumocystis jiroveci pneumonia.
Test Block 6  •  Answers 699

37. The correct answer is E. The subthalamic sophilic stippling of erythrocytes. Dimercaprol
nucleus is innervated by the globus pallidus and ethylenediamine tetraacetic acid (EDTA)
externus in the indirect pathway of the basal are first-line medical treatments for lead poi-
ganglia. It is not directly innervated by the sub- soning. Succimer is the preferable treatment
stantia nigra. for lead poisoning in children.
Answer A is incorrect. This patient has the Answer A is incorrect. Deferoxamine is the
classic signs of Parkinson disease (PD). Acute treatment for iron toxicity. High levels of iron
PD in young patients has been associated with produce cellular damage through the forma-
drug users who have had exposure to MPTP, tion of free radicals and lipid peroxidation.
a contaminant in some illicit street drugs that Early symptoms include abdominal pain, di-
causes damage to the substantia nigra and in- arrhea, and GI bleeding. Later complications
duces early-onset PD. The site of the lesion is include cardiovascular dysfunction and liver
the substantia nigra, which sends direct pro- failure.
jections to the striatum. The striatum is com-
Answer C is incorrect. N-acetylcysteine is the
posed of the caudate and putamen, which
treatment for acetaminophen toxicity, which is
are involved in both direct and indirect mo-
often asymptomatic initially. Patients will occa-
tor pathways of the basal ganglia. The direct
sionally present with nausea, vomiting, sweat-
pathway, promoted by dopamine release from
ing, and lethargy. With continued exposure,
the substantia nigra, facilitates movement,
signs of nephrotoxicity (oliguria and electrolyte
and the indirect pathway, inhibited by dopa-
abnormalities) develop.
mine release from the Sn, inhibits movement.
When the substantia nigra is damaged in PD, Answer D is incorrect. Naloxone is the treat-
dopamine is no longer released in adequate ment for opioid overdose. Patients with opioid
amounts, so the direct pathway is inhibited toxicity most commonly present with respira-
and the indirect pathway is unchecked, lead- tory depression, depressed mental status, and
ing to the paucity of movement characteristic constricted (“pinpoint”) pupils.
of PD. Answer E is incorrect. Protamine sulfate is the

Full-Length Exams
Answer B is incorrect. The globus pallidus treatment for heparin overdose.
externus is innervated by the striatum in the Answer F is incorrect. Thiosulfate is one of
indirect pathway of the basal ganglia. It is not the first-line treatments for cyanide poisoning.
directly innervated by the substantia nigra. Symptoms of cyanide poisoning stem primar-
Answer C is incorrect. The globus pallidus in- ily from neurologic (headache, confusion, and
ternus is a downstream nucleus in both direct seizure) and cardiovascular (tachycardia and
and indirect pathways of the basal ganglia. It is hypertension) dysfunction.
not directly innervated by the substantia nigra.
39. The correct answer is B. The patient has
Answer D is incorrect. The lateral geniculate type 1 diabetes and is suffering from diabetic
nucleus is a thalamic nuclei involved in visual ketoacidosis (DKA); his recent illness has
processing. It is not part of the basal ganglia prompted an increased need for insulin. In-
motor pathway and is not innervated by the creased ketone production causes an anion gap
Sn. metabolic acidosis, characterized by decreased
pH and bicarbonate levels. The low insulin
38. The correct answer is B. This patient presents
and hyperosmolarity cause an increase in se-
with signs and symptoms of lead poisoning.
Test Block 6

rum potassium levels. Although the potassium


Construction workers, especially those exposed
levels are high initially, they fall rapidly with
to industrial paints (found on bridges), are at
fluid administration and insulin therapy, thus
risk for lead toxicity that may manifest with
supplementary IV potassium is needed. Hy-
wrist and foot drop, in addition to the symp-
perglycemia and the resulting fluid shift out of
toms described above. Blood smears reveal ba-
700 Section III: Full-Length Examinations  •  Answers

the cells causes a dilution of the serum sodium dense, monotonous, lymphoid infiltrate in
level. the lamina propria and pale-staining marginal
zone B cells surrounding the epithelium are
Answer A is incorrect. DKA causes initially
apparent.
high potassium levels.
Answer D is incorrect. Ulcerative colitis is
Answer C is incorrect. DKA causes low so-
a type of irritable bowel disease that typically
dium and initially high potassium levels.
begins at the rectum (spares the anus) and
Answer D is incorrect. DKA causes low so- spreads continuously up the colon. It can pre­
dium and bicarbonate levels. sent with bloody diarrhea, abdominal pain,
Answer E is incorrect. DKA causes low pH and extra-intestinal symptoms such as arthri-
and sodium levels and initially high potassium tis. Major findings of intestinal biopsy include
levels. crypt abscesses and chronic changes including
branching of the crypts, atrophy of glands, and
40. The correct answer is E. The cause of these loss of mucin in goblet cells.
four common symptoms (arthralgias, weight
loss, diarrhea, and abdominal pain) is Troph- 41. The correct answer is B. Epstein-Barr virus
eryma whippelii, which can exist throughout (EBV), the virus that causes infectious mono-
the intestinal tract, lymphoreticular system, nucleosis, is a rare cause of congenital de-
and CNS as a result of exposure to soil mi- fects. The other answer choices make up the
crobes. This is a gram-positive, non-acid-fast, ToRCHeS diseases, a collection of serious in-
periodic acid-Schiff (PAS)-positive bacillus fections of pregnancy that are associated with
with a recognizable trilaminar plasma mem- morbidity and mortality of the fetus and new-
brane. Biopsy of the lamina propria shows born. ToRCHeS stands for Toxoplasmosis,
accumulation of macrophages with brightly Rubella, Cytomegalovirus, Herpes/HIV, and
stained PAS-positive intracellular material. Syphilis.

Answer A is incorrect. Celiac sprue is an au- Answer A is incorrect. Congenital cytomega-


lovirus (CMV) can result in hepatosplenomeg-
Full-Length Exams

toimmune disorder of the small intestine. Pa-


tients with the disease make autoantibodies to aly, jaundice, and brain calcifications.
the gluten in wheat and other grains (gliadin). Answer C is incorrect. Herpes simplex virus
Symptoms include chronic diarrhea, abdomi- (HSV) can result in a variety of congenital de-
nal pain, and malabsorption. Biopsy of the fects, spontaneous abortion, and neonatal en-
small intestine in celiac sprue shows flattened cephalitis.
villi, decreased brush border enzymes, and
Answer D is incorrect. Congenital HIV results
lymphocytic infiltration.
in neonatal AIDS.
Answer B is incorrect. Crohn disease is a
Answer E is incorrect. Congenital rubella in-
type of inflammatory bowel disease that has
fection can result in deafness, patent ductus
intestinal complications such as chronic di-
arteriosus, pulmonary artery stenosis, cataracts,
arrhea, malabsorption, and abdominal pain,
and microcephaly.
as well as extra-intestinal symptoms such as
rashes (erythema nodosum), arthritis, and Answer F is incorrect. Congenital syphilis can
uveitis. Major findings of intestinal biopsy in result in cranial nerve VIII deafness, mulberry
Crohn disease include focal ulcerations as well molars, saber shins, saddle nose, and Hutchin-
as acute and chronic inflammation. son’s teeth.
Test Block 6

Answer C is incorrect. Gastric MALT lym- Answer G is incorrect. Congenital toxoplas-


phoma (mucosa-associated lymphoid tissue mosis infection can result in mental retarda-
tumor) is frequently associated with chronic tion and chorioretinitis.
infection with Helicobacter pylori. On biopsy,
Test Block 6  •  Answers 701

42. The correct answer is C. Collagen is a ma- cancers of the colorectum and endometrium,
jor component of the basement membrane. but cancers may also occur in the stomach,
Neoplastic cells bound to the basement mem- ovary, pancreas, ureter and renal pelvis, biliary
brane can potentially escape if they can break tract, small bowel, and brain.
through it into the underlying tissue. Aberrant
Answer B is incorrect. A defect in nucleotide
expression of collagenase and hydrolase can
excision repair results in xeroderma pigmento-
confer that ability, leading to an invasive neo-
sum, a disease characterized by extreme sensi-
plasm. Metastasis occurs when these cells es-
tivity to sunlight, skin damage, and a predispo-
cape the confines of the basement membrane,
sition to malignancies such as melanoma.
find their way to the lymph or bloodstream,
and subsequently adhere to tissues in another Answer C is incorrect. t(15;17) is character-
part of the body. istic of acute promyelocytic leukemia. This
translocation causes the polymorphonuclear
Answer A is incorrect. γ-glutamyl transpep-
leukocytes and RAR a genes to fuse, arresting
tidase (GGT) participates in the transfer of
the development of myeloid lineage cells at
amino acids across the cellular membrane and
the promyelocyte stage.
in the metabolism of glutathione. High con-
centrations are found in the liver, bile ducts, Answer D is incorrect. Benzo(a)pyrene is a
and kidney. A test for serum GGT is used to carcinogen found in cigarette smoke. This
detect diseases of the liver, bile ducts, and kid- carcinogen binds to DNA and forms bulky ad-
ney and to differentiate liver or bile duct (hep- ducts with guanine residues. Bulky lesions are
atobiliary) disorders from bone disease. repaired by nucleotide excision.
Answer B is incorrect. CD44 is an adhesion Answer E is incorrect. The bcr-abl hybrid
molecule used by T lymphocytes to migrate to gene is the result of a translocation between
selective sites in lymphoid tissue. Although im- chromosomes 9 and 22, t(9,22). Chronic my-
portant in metastasis, it is not a factor in base- eloid leukemia is associated with t(9,22), or the
ment membrane invasion. Philadelphia chromosome.

Full-Length Exams
Answer D is incorrect. E-cadherins keep the 44. The correct answer is A. Glioblastoma mul-
epithelial cells together and play a role in re- tiforme (GBM) is the most common primary
laying signals between the cells. In several epi- intracranial neoplasm and is typically seen
thelial tumors, including adenocarcinoma of in older patients. The characteristic features
the colon and breast, E-cadherin expression is shown in the image include a central area of
decreased, not increased. necrosis surrounded by a hypercellular zone
Answer E is incorrect. Keratin is a marker of called pseudopallisading necrosis. Patients
epithelial differentiation and is not a factor in with GBM have a very poor prognosis regard-
metastasis. less of management, which typically consists of
chemotherapy, radiotherapy, and/or surgery.
43. The correct answer is A. This patient has
Answer B is incorrect. Medulloblastomas are
hereditary nonpolyposis colorectal cancer
tumors that preferentially affect children in
(HNPCC), which can be caused by an in-
the first decade of life. The cells have a char-
herited mutation in one of the five DNA mis-
acteristic blue appearance and crowd together
match repair genes. The mismatch repair
to form perivascular rosettes or pseudorosettes.
mechanism replaces segments of DNA that
They arise from the cerebellum and com-
include mismatched bases. Without this proof-
Test Block 6

monly block the fourth ventricle, resulting in


reading function, errors can accumulate in
obstructive (noncommunicating) hydrocepha-
crucial areas, such as inactivating mutations
lus.
in cancer suppressor genes or activating mu-
tations in proto-oncogenes. HNPCC is associ- Answer C is incorrect. Meningiomas are the
ated predominantly with an increased risk for second most common primary intracranial tu-
702 Section III: Full-Length Examinations  •  Answers

mor in adults and are typically benign, slow- to CD21 on B lymphocytes, which become
growing, and resectable. They commonly infected. In addition to the NK cell-mediated
affect people in the fourth and fifth decades response, CD8+ T lymphocytes mediate the
of life but can affect younger adults as well. main cellular immune response to this infec-
There is a female predominance and an asso- tion.
ciation with neurofibromatosis type 2. Menin-
Answer A is incorrect. Eosinophils are impor-
giomas arise from arachnoidal cells of the me-
tant effector cells in host defense against para-
ninges and classically exhibit calcified whorls
sites.
called psammoma bodies on histologic exami-
nation. Answer B is incorrect. Mast cells control the
early inflammatory response by release of po-
Answer D is incorrect. Neurilemmomas are
tent vasoactive granules.
benign tumors arising from Schwann cells as-
sociated with cranial nerve VIII (appropriately, Answer C is incorrect. Megakaryocytes are
they are also called acoustic schwannomas). resident bone marrow cells that give rise to
They are the third most common primary in- platelets.
tracranial tumor in adults and, if bilateral, are Answer D is incorrect. Microglia are tissue
associated with neurofibromatosis type 2. The macrophages located within the CNS and do
two typical patterns seen histologically are ei- not play as direct a role in the control of sys-
ther compact palisading nuclei (Antoni A) or temic viral infections as natural killer cells do.
loose arrangement of cells (Antoni B).
Answer F is incorrect. Plasma cells are anti-
Answer E is incorrect. Oligodendrogliomas body-producing B lymphocytes and an impor-
are slow-growing tumors that arise from oligo- tant component of the adaptive immune sys-
dendrocytes. They are the fourth most com- tem.
mon primary intracranial tumors. Oligoden-
drogliomas typically have the appearance of Answer G is incorrect. Regulatory T lympho-
fried eggs with interspersed capillaries that ap- cytes are components of the adaptive immune
pear like chicken wire. response that suppress effector T-lymphocyte
Full-Length Exams

functions in both an antigen-specific and


45. The correct answer is E. Natural killer (NK) antigen-nonspecific manner.
cells are a component of the innate immune
system. These cells have a battery of germline- 46. The correct answer is E. The child is present-
encoded activating and inhibitory receptors ing with DiGeorge syndrome, which is due to
that can detect and distinguish virally infected abnormal development of the third and fourth
cells from uninfected cells. For example, vi- branchial (pharyngeal) pouches. This leads
rally infected cells often express less major to hypoplasia of the thymus and parathyroid
histocompatibility complex class I on their sur- glands. Without a properly functioning thy-
face, and this absence is detected by the NK mus, T lymphocyte maturation fails, resulting
cell. Detection of a virally infected cell signals in impaired cell-mediated immunity. Thus, pa-
the NK cell to release cytotoxic granules onto tients with DiGeorge syndrome often present
the infected cell; thus they play a direct and with recurrent viral and fungal infections, as in
important role in controlling the early stages this patient. Without adequate production of
of systemic response to viral infection. NK parathyroid hormone, these patients are often
cells express CD16 and CD56 among other hypocalcemic, leading to tetany and seizures.
markers. It should be noted that individuals DiGeorge syndrome can be summarized by
Test Block 6

with defective NK cell function are particu- the mnemonic CATCH-22: Cardiac defects,
larly susceptible to herpes virus infection. This Abnormal facies, Thymic hypoplasia, Cleft
suggests that NK cells play an important, non- palate, and Hypocalcemia due to a microdele-
redundant physiologic function in the control tion on chromosome 22.
of this family of viral infections. EBV binds
Test Block 6  •  Answers 703

Answer A is incorrect. The first and second put (CO), and decreased pulmonary capillary
branchial arches play no role in DiGeorge wedge pressure (PCWP).
syndrome. For first-arch derivatives, think
Answer A is incorrect. Neurogenic shock fol-
“M”: Mandible, Malleus, spheno Mandibular
lows CNS trauma. It is characterized by loss of
ligament; muscles of Mastication (teMporalis,
vasomotor tone, decreased CO, and decreased
Masseter, Medial and lateral pterygoids). The
PCWP.
first arch is associated with cranial nerve V. For
second-arch derivatives, think “S”: Stapes, Sty- Answer C is incorrect. Hypovolemic shock
loid process, Stylohyoid ligament, muscles of is a less likely diagnosis because this patient’s
facial expression, Stapedius, Stylohyoid. Cra- blood pressure does not respond to fluid ad-
nial nerve VII is associated with the second ministration and he has an elevated WBC
arch. count. This form of shock is characterized by
increased SVR, decreased CO, and decreased
Answer B is incorrect. The first branchial
PCWP.
pouch arises in the pharynx and extends later-
ally and cephalad to contact the first branchial Answer D is incorrect. Cardiac shock is the re-
cleft, forming the eustachian tube. The second sult of decreased pump function. As such, it is
branchial pouch originates in the oropharynx characterized by increased SVR in an attempt
and contributes to the middle ear and tonsils. to maintain blood pressure, decreased CO,
They are not involved in DiGeorge syndrome. and increased PCWP.
Answer C is incorrect. The fourth and sixth Answer E is incorrect. This pattern of param-
pharyngeal arches do not play a role in Di- eters is not seen in the setting of any form of
George syndrome. The fourth arch is respon- shock.
sible for muscles of the soft palate (but not the
tensor veli palatini, a first arch derivative), the 48. The correct answer is A. The right coronary
muscles of the pharynx (except the stylophar­ artery (RCA) arises from the aortic sinus of the
yngeus), the cricothyroid, and the aortic arch. ascending aorta and runs along the right side
Fourth-arch muscles are innervated by the su- of the pulmonary trunk in the coronary groove.

Full-Length Exams
perior laryngeal branch of cranial nerve X. The It gives off a sinoatrial (SA) nodal branch, the
sixth arch produces the muscles of the larynx acute marginal artery, and an atrioventricular
(except for the cricothyroid) as well as the pul- (AV) nodal branch. About 80% of people have
monary arteries. These muscles are innervated “right-dominant” circulation, meaning the
by the recurrent laryngeal branch of cranial RCA also gives off the posterior descending ar-
nerve X. tery, which supplies the posterior and inferior
ventricles and the posterior one-third of the
Answer D is incorrect. The second through interventricular septum. An acute occlusion
fourth branchial clefts form temporary sinuses of the RCA commonly manifests on ECG as
but are obliterated before maturation. Thus, ST-segment elevations in leads II, III, and aVF.
they have no derivatives in the adult. These leads are called the “inferior leads” be-
cause they represent the inferior surface of the
47. The correct answer is B. This pattern of he-
heart. ST-segment elevations in these leads
modynamic parameters is evident in septic
suggest an occlusion of the RCA because the
shock. This patient’s vital signs on presentation
inferior surface is most commonly supplied by
to the emergency department are consistent
this vessel. The left anterior descending (LAD)
with a diagnosis of sepsis. The lack of blood
artery branches from the left main coronary
Test Block 6

pressure rise on fluid administration advances


artery as it approaches the AV junction. It de-
this patient’s diagnosis to septic shock. Endo-
scends toward the apex on the anterior wall of
toxin, and the cytokines released in response,
the heart between the right and left ventricles
will cause vasodilation (decreased systemic vas-
and supplies the anterior wall of both ven-
cular resistance [SVR]), increased cardiac out-
tricles and the anterior two-thirds of the inter-
704 Section III: Full-Length Examinations  •  Answers

ventricular septum, including the AV bundle. Answer D is incorrect. The right ventricle re-
These areas of myocardium supplied by the ceives most of its blood supply from the acute
LAD would be less affected by an occlusion of marginal and posterior descending arteries,
the RCA. both of which are branches of the RCA in
right-dominant individuals. The LAD, which
Answer B is incorrect. In 80% of people the
runs in the interventricular groove, also pro-
AV node is supplied by AV nodal branches
vides some blood flow to portions of the right
that come off the RCA, making it unlikely to
ventricle. However, the primary source of
be spared after an RCA occlusion. Note that
blood supply is from the RCA, so the right ven-
in 20% of people the AV node is supplied by
tricle is less likely to be spared when the RCA
branches of the left main coronary artery.
becomes occluded.
Answer C is incorrect. In right-dominant in-
Answer E is incorrect. In 60% of people the
dividuals, the RCA courses around to the pos-
RCA gives off SA nodal branches near its ori-
terior wall of the heart and gives off the pos-
gin that supply the SA node. Thus this area is
terior descending artery. This vessel supplies
less likely to be spared in an acute occlusion of
the posterior regions of both the left and right
the RCA. However, it is important to note that
ventricles, as well as the posterior one-third of
in the remaining 40% of people, the SA node
the interventricular septum. Thus the posterior
is supplied by branches from the circumflex
septum is less likely to be spared in an occlu-
artery, which comes off the left main coronary
sion of the RCA. If this patient’s coronary ves-
artery.
sels were left-dominant (as occurs in 20% of
people), the posterior descending artery would
branch off the left circumflex artery and the
posterior septum would not be affected by an
RCA occlusion.
Full-Length Exams
Test Block 6
Test Block 7

705
706 Section III: Full-Length Examinations  •  Questions

Q u e st i o n s

1. A 23-year-old G1P0 woman is 39 weeks’ preg-


nant. For three days she experiences dysuria,
polyuria, and fever, which she attributes to a
recent urinary tract infection. Instead of con-
sulting with her doctor, the woman decides to
use the remaining pills from her old prescrip-
tion to treat her symptoms. Three weeks later
she gives birth to a mildly jaundiced boy who
is otherwise healthy. Five days later, however,
she brings the infant to the emergency depart-
ment stating that the baby has become fussy,
refuses feeding, and wails at a high pitch. He
soon becomes extremely lethargic and stops
Courtesy of Dr. Libero Ajello, Centers for Disease Control
producing urine. Which of the following medi- and Prevention.
cations did the mother most likely use during
the last weeks of her pregnancy to treat what
she believed to be a recurrence of her urinary (A)
Blastomyces species
tract infection? (B)
Coccidioides species
(A) Amoxicillin (C)
Malassezia furfur
(B) Ampicillin (D)
Pneumocystis jiroveci
(C) Nitrofurantoin (E)
Sporothrix schenckii
(D) Ofloxacin
(E) Trimethoprim-sulfamethoxazole 3. A 9-year-old boy with no vaccine history pre­
sents with an erythematous maculopapular
rash that erupted about five days after the on-
Full-Length Exams

2. A 42-year-old woman presents with red papular


lesions along her arm, some of which have ul- set of cough, conjunctivitis, coryza, high fever,
cerated. She reports that the lesions appeared and white spots on the buccal mucosa. The
one week ago, a day after she worked in her patient’s immune system is activated to com-
garden. Initially there were only a couple of le- bat the infecting virus. Which of the following
sions on her right forearm. Since then, more types of cells are the most capable of providing
lesions have appeared along her arm approach- the two signals necessary for T lymphocyte ac-
ing her axilla. The physician cultures material tivation?
from one of the lesions at 25°C, which grows (A) Activated B lymphocytes, other T lympho-
the organism shown in the image. This patient cytes, and natural killer cells
is most likely infected with which of the fol- (B) Activated CD8+ T lymphocytes, B lym-
lowing fungi? phocytes, and dendritic cells
(C) Activated macrophages, epithelial cells,
and B lymphocytes
Test Block 7
Test Block 7  •  Questions 707

(D) Activated macrophages, Langerhans cells, rupts and states, “I can’t stand Dr. Smith. He
and B lymphocytes is a lousy surgeon and he doesn’t know how to
(E) Activated macrophages, Langerhans cells, treat people. Next time you see him, tell him
and natural killer cells he has made his patients extremely angry and
should learn how to become a better doctor.”
4. A 54-year-old white man presents for a routine What would be the most suitable response to
preoperative assessment. Physical examina- this patient?
tion reveals that the man is unable to close his
(A) “Do you think I have treated you appropri-
left eye completely or to seal the corner of his
ately?”
mouth on the left side. ECG reveals new atrio-
(B) “I will bring up the issues with Dr. Smith
ventricular heart block. The patient denies any
in our next meeting.”
recent changes in his health, but notes that
(C) “Dr. Smith is an excellent doctor. He does
he recently traveled to upstate New York for a
a great job with all of my patients.”
camping trip. The patient is given a common
(D) “I understand you are upset, but I suggest
medication to treat the underlying cause of his
you speak directly to Dr. Smith regarding
symptoms. What is a major adverse effect of
your concerns.”
this treatment in adults?
(A) Acute cholestatic hepatitis 7. A young woman comes to the her primary
(B) Discoloration of teeth care physician complaining of eight weeks of
(C) Nephrotoxicity fevers, chills, night sweats, fatigue, and weight
(D) Ototoxicity loss. About three weeks ago she noticed a large
(E) Photosensitivity bump around her clavicle. It is surgically re-
moved and biopsied (see image). The type
5. A healthy 29-year-old, gravida 2 para 0 woman of cell shown in the image is a key to making
who is 36 weeks’ pregnant presents to the which of the following diagnoses?
emergency department experiencing contrac-
tions that are five minutes apart. She delivers

Full-Length Exams
an apparently healthy baby boy. On initial
survey, the physician notes that the baby has
good color and is crying loudly, but seems un-
able to move either of his legs very well and
is not startled by loud noises. Radiography re-
veals healing fractures bilaterally in the femo-
ral shaft and new fractures in the right tibia.
The woman denies any significant abdominal
trauma over the course of the pregnancy. The
baby’s condition is caused by which of the fol-
lowing?
(A) A mutation in COL43A, resulting in de-
fective formation of type IV collagen
(B) A mutation in fibrillin-I, leading to re- Reproduced, with permission, from USMLERx.com.
duced integrity of connective tissues
(C) A substitution of glycine by a bulky amino
(A) Hodgkin lymphoma
acid in type I collagen
Test Block 7

(B) Infectious mononucleosis


(D) Maternal vitamin D deficiency
(C) Leukemia
(D) Multiple myeloma
6. A medical oncologist is seeing a patient who
(E) Non-Hodgkin lymphoma
has been diagnosed with breast cancer. As he
begins to take the patient’s history, she inter-
708 Section III: Full-Length Examinations  •  Questions

8. A 35-year-old woman comes to the emergency 11. A 60-year-old man who is being treated for
department after returning home from a trip multiple myeloma develops osteonecrosis of
to Mexico with a one-week history of intrac- his maxilla. His presentation is significant for
table vomiting. Laboratory tests show a serum a molar tooth extraction site that has healed
bicarbonate level of 50.9 mmol/L, pH of 7.61, poorly four months after surgery. Localized
serum sodium of 125 mEq/L, potassium of swelling, exposed bone, erythema, and a pu-
1.8 mEq/L, and chloride of 55 mEq/L. She is rulent discharge are noted on intraoral exami-
treated aggressively with fluid, electrolytes, and nation. Which of the following drug classes is
a diuretic. Which of the following is the most most likely associated with this lesion?
appropriate diuretic to treat this patient?
(A) Angiotensin converting enzyme inhibitors
(A) Acetazolamide (B) Bisphosphonates
(B) Ethacrynic acid (C) Cephalosporins
(C) Furosemide (D) Osmotic diuretics
(D) Hydrochlorothiazide (E) Vinca alkaloids
(E) Mannitol
12. A 12-year-old girl presents to her pediatri-
9. A 54-year-old woman with breast cancer in cian because of recent onset of bruising. Her
her right breast recently underwent surgical mother states that she has been healthy other
lumpectomy with axillary lymph node dissec- than a recent infection that she developed on
tion. Several weeks into her subsequent ra- a trip to Guatemala, for which she received an-
diation treatment, she presents with a swollen tibiotics while there. Physical examination is
right arm and fingers, right facial edema that is remarkable for non-blanching purple and yel-
most pronounced around the orbit, and short- low bruises on her legs. Complete blood cell
ness of breath. X-ray of the chest reveals some count reveals pancytopenia. Bone marrow bi-
accumulation of fluid in the right pleural cav- opsy shows hypercellularity with fatty infiltra-
ity. Which of the following is the most likely tion. Which of the following is the mechanism
cause of these findings? of action of the medication that most likely
Full-Length Exams

caused the findings in this patient?


(A) Deep venous thrombosis of the right ce-
phalic vein (A) Binding to the 50S ribosomal subunit
(B) Disruption of the right lymphatic duct (B) Binding to the 30S ribosomal subunit
(C) Disruption of the thoracic duct (C) Competitive inhibitor of dihydropteroate
(D) Metastatic disease to the humerus synthetase
(E) Normal adverse effect of radiation (D) Inhibition of DNA gyrase
(E) Inhibition of formation of peptidoglycan
10. A 36-year-old man with a history of depression cross-links in the cell wall
is laid off from work. That evening he goes to
the gym and plays a particularly vigorous game 13. A 50-year-old white woman presents to her
of basketball. Which defense mechanism is physician with pain that she has had for sev-
this man employing? eral years. She describes the pain as constant,
located in her neck, lower back, and hips, and
(A) Displacement
as interfering with her day-to-day activities.
(B) Projection
Over the years she has had extensive work-ups,
(C) Reaction formation
including laboratory testing and multiple mo-
(D) Sublimation
dalities of imaging, none of which led to an
Test Block 7

(E) Suppression
Test Block 7  •  Questions 709

explanation for her symptoms. She has had 15. A 62-year-old woman with stage IV ovarian
no history of trauma, but does report feeling cancer presents with shortness of breath and
“down” occasionally. Which treatment would chest pain. Diagnostic imaging indicates a
be most appropriate for her at this time? fluid collection within the right pleural space.
The physician prepares the patient for thera-
(A) A 14-day course of oral doxycycline
peutic thoracentesis, and she inserts the needle
(B) Behavioral therapy
in the midaxillary line on the right side, in the
(C) Group therapy
lower margin of the ninth intercostal space.
(D) Low-dose fentanyl patch
Which structure(s) does the physician aim to
(E) No treatment; her symptoms will resolve
avoid by inserting the needle at this point?
spontaneously with time
(A) Ninth intercostal nerve, artery, and vein
14. The image shows the gross pathology of a con- (B) Parietal pleura
genital abnormality that results from incom- (C) Phrenic nerve
plete obliteration of the omphalomesenteric (D) Right pericardiophrenic artery and vein
(vitelline) duct. From which of the following is (E) Tenth intercostal nerve, artery, and vein
the pictured tissue derived? (F) Visceral pleura

16. A 3-month-old girl is brought to her pediatri-


cian because she has a high fever and cough.
The patient is started on an antibiotic regi-
men. Over the next 24 hours, her condition
deteriorates and she is taken to the emergency
department. Further testing reveals a defi-
ciency in the protein LFA-1. Which of the fol-
lowing best explains the etiology of the immu-
nodeficiency syndrome from which the patient
most likely suffers?

Full-Length Exams
(A) There is a defect in the development of T
and B lymphocytes
(B) There is a defect in an adhesion protein
necessary for neutrophil migration
Reproduced, with permission, from F. Charles Brunicardi, et
(C) There is a defect in the development of
al. Schwartz’s Principles of Surgery, Ninth Edition. New York:
the third and fourth pharyngeal pouches
McGraw-Hill, 2010; Fig. 28-23.
(D) There is a defect in the emptying of phago-
cytic cells
(A) Dorsal mesentery (E) There is a deficiency of NADPH oxidase
(B) Foregut in phagocytic cells
(C) Hindgut
(D) Mesonephric duct
(E) Midgut
(F) Neural crest cells
Test Block 7
710 Section III: Full-Length Examinations  •  Questions

17. The statistical distribution of two studies is 19. A 47-year-old woman visits her physician with
shown below. The mean is equal to the me- complaints of having to use rewetting eye
dian and the mode in the first curve (labeled drops throughout the day. Physical examina-
A). Which of the following correctly describes tion and review of systems are unremarkable
the mean, median, and mode in the second except for a low-grade fever and morning joint
curve (labeled B)? pain that has progressively worsened over the
past four years. Which of the following addi-
tional conditions is this patient most at risk for
A developing?
(A) Dactylitis
(B) Dental caries
(C) Hyperglycemia
(D) Jaundice
(E) Septic joints

B 20. A biopsy of a lymph node from a 3-year-old


who developed paralytic poliomyelitis follow-
ing administration of a live attenuated polio
vaccine is performed, revealing nodal archi-
tecture that lacks germinal centers (shown
below). This patient would likely respond to
Reproduced, with permission, from USMLERx.com. treatment with which of the following thera-
pies?
(A) Mean < median < mode
(B) Mean < mode < median
(C) Median < mean < mode
(D) Median < mode < mean
Full-Length Exams

(E) Mode < mean < median


(F) Mode < median < mean

18. A 12-year-old boy with moderate mental retar-


dation comes to the physician because of pain-
ful swollen joints. During the examination, the
physician notices that the boy makes several
uncontrolled spastic muscle movements. Past
medical history includes a diagnosis of mus-
cular hypotonia at five months of age. At age
three, the patient was referred to a pediatric Courtesy of Wikipedia.
dentist for severe repetitive biting of his lip and
tongue. Which of the following is the most
likely cause of these findings? (A) Cyclosporine
(B) Regular intramuscular γ-globulin injec-
(A) A deficiency of adenosine deaminase tions
(B) A deficiency of b-glucocerebroside (C) Sargramostim
Test Block 7

(C) A mutation of an enzyme in the de novo (D) Tacrolimus


biosynthetic pathway (E) Thrombopoietin
(D) Absence of hypoxanthine guanine phos-
phoribosyltransferase
(E) An excision repair enzyme deficiency
Test Block 7  •  Questions 711

21. A study is designed to compare an experimen- 23. A 71-year-old man presents to his physician
tal drug to a standard medication used in the because of a six-week history of progressive
treatment of psoriasis. One hundred patients dyspnea, wheezing, and coughing. He has hy-
are enrolled and randomized in the study. The pertension and a 60-pack-year history of smok-
study runs for 12 months, during which 30 par- ing. The patient has difficulty speaking and is
ticipants drop out of the treatment group and breathing using his accessory respiratory mus-
10 participants drop out of the control group. cles. Neck vein distention and tongue swelling
At the conclusion of the study, researchers de- are evident. Which of the following is most
cide to only analyze data from those who re- likely diagnosis?
mained in the study. The results show that the
(A) Gastric adenocarcinoma
experimental drug is better than the standard
(B) Hodgkin lymphoma
medication at treating psoriasis. The results of
(C) Hypertrophic cardiomyopathy
this study may be invalid because of what ma-
(D) Idiopathic pulmonary fibrosis
jor issue?
(E) Squamous cell carcinoma
(A) Procedural bias
(B) Recall bias 24. A town with 1,000 citizens has a 10% preva-
(C) Sampling bias lence of disease X. A screening test for disease
(D) Selection bias X was just developed, with a sensitivity of 80%
(E) Small number of patients in the study and a specificity of 70%. How many people
without disease X will be falsely diagnosed pos-
22. A 20-year-old man is an avid runner and sports- itive by this screening test?
man. In his early 20s, however, he begins to
(A) 20
smoke cigarettes, initially smoking one pack a
(B) 80
day, escalating eventually to two-three packs
(C) 100
per day. As he ages, his lung function gradu-
(D) 270
ally declines, and at age 60, he notices that he
(E) 630
becomes tired easily and cannot climb a flight

Full-Length Exams
of stairs without having to catch his breath. He 25. A 54-year-old woman has had longstanding
goes to see his primary care physician, who rheumatoid arthritis. Her rheumatologist re-
finds that his resting oxygen saturation is 90%. cently started her on methotrexate, a competi-
The physician performs in-office spirometry tive inhibitor of dihydrofolate reductase. What
and makes a diagnosis of emphysema. Which effect does methotrexate have on dihydrofolate
of the following is responsible for this patient’s reductase (DHFR)?
decreased oxygen saturation?
(A) Methotrexate acts on DHFR by decreasing
(A) Decreased ability of hemoglobin to bind its Michaelis-Menten constant
oxygen (B) Methotrexate acts on DHFR by increasing
(B) Decreased diffusion of oxygen across alve- its Michaelis-Menten constant
oli (C) Methotrexate does not affect
(C) Decreased hemoglobin content of the DHFR’s Michaelis-Menten constant
blood (D) The maximum reaction rate is decreased
(D) Decreased perfusion to alveoli (E) The maximum reaction rate is increased
(E) Decreased solubility of oxygen in the
blood
Test Block 7
712 Section III: Full-Length Examinations  •  Questions

26. An 80-year-old man is brought to the emer- (C) Hyaline casts


gency department by his daughter after suf- (D) RBC casts
fering a seizure. For months he has taken (E) WBC casts
exceptionally short steps, unable to raise his
legs. He is incontinent of urine and does not 28. A 66-year-old right-handed man with a history
“know when to go.” His recent memory is also of atrial fibrillation is brought to the emer-
impaired, and he denies having fallen despite gency department by his daughter, who reports
evidence to the contrary. There is no history that he began “talking funny” that morning.
of stroke or intracranial mass lesions. On neu- The man is unable to speak or write. However,
rologic examination cranial nerves are intact, he is able to respond to both verbal and written
muscle strength is normal, and sensorium is commands such as “close your eyes,” and is
normal. The patient sways during Romberg’s visibly frustrated by his unsuccessful attempts
test with eyes open or closed. A CT scan of to speak. In addition, the man has mild right-
the head is shown in the image. Cerebrospi- sided spastic paralysis of his face and arm. This
nal fluid pressure is normal. What is the most man has a lesion that includes which of the
likely etiology of the dilated ventricles? following brain regions?
(A) Left inferior frontal gyrus
(B) Left postcentral gyrus (parietal lobe)
(C) Left superior temporal gyrus
(D) Right inferior frontal gyrus
(E) Right postcentral gyrus (parietal lobe)
(F) Right superior temporal gyrus

29. A 50-year-old Chinese immigrant presents to


his primary care physician complaining of fe-
ver, night sweats, and blood-tinged sputum
for the past three weeks. X-ray of the chest is
Full-Length Exams

remarkable for calcification in the right upper


lobe. Which of the following adverse effects is
commonly associated with a drug used in the
treatment of this man’s disease?
Reproduced, with permission, from USMLERx.com.
(A) Blue and yellow color blindness
(B) Myalgias
(A) Choroid plexus papilloma (C) Numbness and tingling
(B) Communicating hydrocephalus (D) Ophthalmoplegia
(C) Hydranencephaly (E) Renal failure
(D) Hypervitaminosis A
(E) Noncommunicating hydrocephalus 30. A 76-year-old woman presents to the emer-
gency department complaining of blurry vision
27. Parents bring their 8-year-old boy to the emer- and a headache that started five hours ago. She
gency department because his face appears has a long history of hypertension and poorly-
swollen. The child recently completed a controlled asthma. She is afebrile with a heart
course of amoxicillin for pharyngitis. On fur- rate of 75/min and blood pressure of 210/140
ther testing, he has a creatinine level of 1.4 mm Hg. Fundoscopic examination shows
Test Block 7

mg/dL. What additional finding would we swelling of the optic disks. Which of the fol-
likely see based on this patient’s history and lowing medications should be administered
symptoms? immediately?
(A) Epithelial cell casts
(B) Fatty casts
Test Block 7  •  Questions 713

(A) Captopril (D) Apolipoprotein E


(B) Hydrochlorothiazide (E) LDL cholesterol receptors
(C) Labetalol (F) Leptin receptors
(D) Losartan (G) Lipoprotein lipase
(E) Sodium nitroprusside
32. A 15-year-old boy is brought to his pediatrician
31. A 35-year-old man presents to his primary because he feels short of breath and has to stop
care physician complaining of episodes of and walk after brief runs. He underwent car-
chest pain during exercise. Approximately one diac surgery when he was an infant for a con-
month ago he began running two miles on a genital heart defect. An inspiratory chest x-ray
treadmill three times a week. For the past two shows an intact right hemidiaphragm that is
weeks he has experienced a severe, throbbing much higher than the left one. This boy’s exer-
pain in his chest and felt dizzy after running tional dyspnea is probably caused by which of
for about 20 minutes, but claims that the pain the following conditions?
stops immediately when he stops running. He
(A) Damaged left phrenic nerve
is concerned because his younger brother, who
(B) Damaged right phrenic nerve
is 32 years old, recently had a mild heart at-
(C) Eisenmenger syndrome
tack. In addition, both his father and his uncle
(D) Left diaphragmatic hernia
died from complications after having heart at-
(E) Right diaphragmatic hernia
tacks when they were 45 years old. The patient
neither smokes nor drinks alcohol. Physical 33. Some fats and proteins are both produced by
examination reveals the lesions shown in the the body and consumed in the diet. Many
image; lesions are also present on his arms and other nutritional components, however, can-
knees. His blood pressure is 130/80 mm Hg, not be synthesized by the body and must be
pulse is 85/min, and respiratory rate is 12/min. obtained in food. Which of the following sub-
Laboratory tests show a serum cholesterol level stances can a healthy adult synthesize?
of 350 mg/dL, a serum LDL cholesterol level
of 275 mg/dL, and a serum triglyceride level of (A) The fatty acid linoleic acid

Full-Length Exams
130 mg/dL. This patient’s lipid disorder is char- (B) The glucogenic amino acid histidine
acterized by a deficiency in which of the fol- (C) The ketogenic amino acid leucine
lowing? (D) The micronutrient folic acid
(E) The micronutrient vitamin K

34. A 52-year-old man presents with nocturia,


dysuria, and crippling back pain. X-ray of the
spine reveals lytic lesions in the lower spine.
Laboratory studies are notable for a hemoglo-
bin level of 10.2 mg/dL, calcium level of 13.1
mg/dL, and increased total protein. Which of
the following is the most likely explanation
of the increased total protein observed in this
man?
(A) Inability to clear chylomicron particles
Reproduced, with permission, from USMLERx.com. from the blood
Test Block 7

(B) Increased production of albumin


(C) Increased production of clotting factors
(A) 3-Hydroxy-3-methylglutaryl coenzyme A and acute-phase reactants
reductase (D) Increased production of IgG molecules
(B) 7a-Hydroxylase (E) Increased production of IgM molecules
(C) Apolipoprotein C-II
714 Section III: Full-Length Examinations  •  Questions

35. A 33-year-old HIV-positive man is taken to the (A) Anaplastic carcinoma


emergency department by his roommate. Ini- (B) Follicular carcinoma of the thyroid
tially the patient complained of generalized (C) Medullary carcinoma of the thyroid
weakness and vision difficulties. Later, the pa- (D) Thyroid lymphoma
tient lost his peripheral vision and became un- (E) Papillary carcinoma
able to talk or walk. Lumbar puncture reveals
unremarkable cerebrospinal fluid. MRI of the 37. Oogenesis results in the production of one
brain reveals multiple, non-contrast-enhancing haploid ovum and three polar bodies in a pro-
lesions in the white matter, primarily in the cess of meiotic cell division. Female game-
parietal and occipital lobes. What is the most togenesis involves two arrest phases that are
likely diagnosis? released by ovulation and fertilization, respec-
tively. What stage of the cell cycle do oocytes
(A) Cryptococcal meningitis
remain in from birth to ovulation?
(B) Guillain–Barré syndrome
(C) Herpes simplex meningitis (A) Anaphase I
(D) Progressive multifocal leukoencephalopa- (B) Anaphase II
thy (C) Metaphase I
(E) Syringomyelia (D) Metaphase II
(E) Prophase I
36. During a routine visit to his primary care phy- (F) Prophase II
sician, a 55-year-old man with no significant
medical history is found to have a singular 38. In a discussion with his psychiatrist, a univer-
thyroid nodule. Work-up suggests the nodule sity student reveals an obsession with a fellow
to be neoplastic. A section of resected tissue is student. He also notes he intends to purchase
shown in the image. What type of thyroid ma- a gun but does not clearly state why. Whom
lignancy has this patient developed? must the physician be sure is contacted?
(A) Law enforcement authorities only
(B) Law enforcement authorities and the fel-
Full-Length Exams

low student
(C) No one; the physician is bound by confi-
dentiality
(D) The fellow student only
(E) The patient’s parents

39. A 28-year-old woman presents to the clinic


concerned that she has begun growing hair on
her upper lip and around her nipples. Physi-
cal examination reveals a 167 cm (5′6″) tall,
85.5-kg (188-lb) woman with coarse facial hair
on her upper lip, chin, shoulders, nipples, and
back. She says she has always been overweight,
Courtesy of Dr. Edwin P. Ewing Jr, Centers for Disease Con-
trol and Prevention.
but has gained 9.1 kg (20 lb) in the past three
months. Physical examination reveals hyper-
Test Block 7
Test Block 7  •  Questions 715

pigmented, velvety patches of skin on the nape (A) Deletion mutation


of her neck and around her axillae. When (B) Frameshift mutation
questioned, she admits she never has been sex- (C) Insertion mutation
ually active, and she always has had irregular, (D) Missense mutation
spotty menstrual periods, often missing them (E) Nonsense mutation
completely. Which laboratory finding is most (F) Silent mutation
closely associated with this patient’s condition? (G) Transition
(A) Hypercalcemia
41. A 4-year-old girl is transferred to the pediatric
(B) Hyperinsulinemia
intensive care unit because of suspected bacte-
(C) Hypermagnesemia
rial sepsis secondary to a bout of Clostridium
(D) Hyperuricemia
difficile enterocolitis. A course of antibiotic
therapy is begun via infusion. Ten minutes
40. To ascertain the specific genetic defect in pa-
later, the child begins crying inconsolably and
tients with cystic fibrosis, scientists obtained
scratching her face. Confluent, erythematous
buccal smears from several patients and iso-
patches without exfoliation appear on her face,
lated DNA from these cells. The DNA was
neck, and upper body, and she develops tachy-
amplified by polymerase chain reaction and
cardia and fever. Treatment is administered
then sequenced. The region of the sequenc-
and the rash clears within three hours. This
ing gel where the normal gene differs from the
patient’s reaction was most likely caused by ad-
mutated gene is shown in the image. Which of
ministration of which of the following agents?
the following types of DNA mutations caused
this disease? (A) Amoxicillin
(B) Chloramphenicol
(C) Clindamycin
Normal Mutant
G A T C G A T C (D) Metronidazole
16 (E) Vancomycin
15

Full-Length Exams
14 42. A 25-year-old man is witnessed by his family to
13 have a seizure of sudden onset. The seizure in-
12
11 volved movement of all four of his extremities
10 and was associated with loss of consciousness.
9 When emergency personnel arrived at the
8 home 35 minutes later, the patient continued
7
6 to show signs of tonic-clonic activity without
5 return to full consciousness. Which of the fol-
4 lowing agents is an effective medication to be
3 given at this point?
2
1 (A) Carbamazepine
G A T C G A T C
(B) Diazepam
(C) Ethosuximide
Reproduced, with permission, from USMLERx.com. (D) Gabapentin
(E) Phenobarbital
Test Block 7
716 Section III: Full-Length Examinations  •  Questions

43. A 45-year-old man comes to the physician (C) Decreased prothrombin time
because of a fever of 39.6º C (103.2º F) that (D) Increased activated partial thromboplastin
developed suddenly the previous night. On time
physical examination, the physician notes a (E) Increased International Normalized Ratio
new-onset murmur along with white spots on (F) Increased prothrombin time
the retina. The appearance of his nailbeds
is shown in the image. Which agent, if used 45. A 43-year-old woman in a psychiatry ward is
alone, is most likely to be used to treat this pa- admitted from a medical floor after attempting
tient prior to the results of blood culture? suicide by overdosing on extra strength Tyle-
nol. She states that she recently broke up (for
the fifth time) with her boyfriend, who disap-
proves of her cocaine and marijuana use. She
also states that she and her ex-boyfriend fight
constantly, and that she has made “hundreds”
of suicide attempts and has been hospitalized
many times. Physical examination reveals
reddened conjunctivae, an eroded nasal sep-
tum, and numerous small, uniformly sized
punched-out skin lesions on the upper and
lower extremities. Which of the following de-
fense mechanisms is associated with this pa-
tient’s condition?

Reproduced, with permission, from USMLERx.com.


(A) Denial
(B) Humor
(C) Reaction formation
(A) Ceftriaxone (D) Splitting
(B) Ciprofloxacin (E) Sublimation
Full-Length Exams

(C) Erythromycin
(D) Gentamicin 46. A 20-year-old man is brought to the emergency
(E) Vancomycin department by the police after being picked
up in the streets for acting violently towards
44. A 5-year-old boy is in the intensive care unit others. He was found in an agitated state, act-
because of an intraventricular hemorrhage ing belligerently. On physical examination
sustained in a car accident. A blood sample is his temperature is 38.5°C (101.3°), heart rate
drawn from an intravenous line that has been is 115/min, blood pressure is 140/95 mm Hg,
kept patent by intermittent flushing with hep- and he has vertical and horizontal nystagmus.
arin. For accurate coagulation studies to be Which of the following treatments should this
obtained, the first 5 mL of blood drawn from patient receive?
such lines must be discarded prior to blood (A) Benzodiazepines
collection. Which of the following laboratory (B) Flumazenil
results is most likely in this patient if the first 5 (C) N-acetylcysteine
mL of blood are not discarded? (D) Naloxone
(A) Decreased activated partial thromboplastin (E) Sodium bicarbonate
time
Test Block 7

(B) Decreased International Normalized Ratio


Test Block 7  •  Questions 717

47. A 70-year-old woman presents with fevers, (A) Aspergillosis


chills, and a nonproductive cough. She has (B) Brucellosis
also been experiencing chest pain, and diar- (C) Legionnaire’s disease
rhea. Gram stain and routine culture of in- (D) Mycoplasma pneumonia
duced sputum is unrevealing. A silver stain (E) Pneumocystis jiroveci pneumonia
of an induced sputum specimen is shown in
the image. Which of the following is the most 48. A 64-year-old woman presents to the physician
likely diagnosis? because of new onset postmenopausal bleed-
ing. Ultrasonography reveals a small mass in
the left adnexa, along with a thickened endo-
metrial stripe. A biopsy of her left ovary reveals
the presence of Call-Exner bodies. Which of
the following is the most likely diagnosis?
(A) Endometrioid tumor
(B) Granulosa cell tumor
(C) Krukenberg tumor
(D) Serous cystadenocarcinoma
(E) Teratoma

Courtesy of Dr. William Cherry, Centers for Disease Control


and Prevention.

Full-Length Exams
Test Block 7
718 Section III: Full-Length Examinations  •  Answers

An s w e r s

1. The correct answer is E. Trimethoprim- Answer A is incorrect. Blastomycosis can


sulfamethoxazole (TMP-SMX) is one of the present with flu-like symptoms: fevers, chills,
most commonly used treatments for simple productive cough, myalgia, arthralgia, and
urinary tract infections, most of which are pleuritic chest pain. It is most common in
caused by Escherichia coli. Sulfamethoxazole, the upper Mississippi and Ohio River basins
as is the case with all sulfonamides, binds to as well as around the Great Lakes. Diagnosis
and will displace unconjugated bilirubin from is made by use of potassium hydroxide prep to
albumin. In a newborn this can lead to kernic- reveal big, broad, budding organisms in spu-
terus, also known as bilirubin encephalopathy. tum or tissues. Some patients will fail to re-
The condition results from bilirubin deposi- cover from an acute infection and progress to
tion and accumulation in the brain because of develop chronic pulmonary infection or wide-
an incompletely formed blood-brain barrier. spread disseminated infection. Fluconazole or
The basal ganglia are particularly susceptible ketoconazole is used for the treatment of local
to injury. Early symptoms include lethargy, blastomycosis, and amphotericin B is used for
poor feeding, and an absent Moro reflex. In- the treatment of systemic infections.
fants who survive can develop seizures, mental Answer B is incorrect. Coccidioidomycosis is
retardation, deafness, choreoathetoid move- the second most common fungal infection en-
ments, and decreased upward eye movements. countered in the United States and is usually
Answer A is incorrect. Amoxicillin is not asso- contracted in the Southwest. There are several
ciated with kernicterus. cutaneous signs including erythema nodosum
(“desert bumps”), erythema multiforme, and
Answer B is incorrect. Ampicillin is not associ-
toxic erythema. Severe forms of the infection
ated with kernicterus.
can present with blood-tinged sputum, loss
Answer C is incorrect. Nitrofurantoin, com- of appetite, weight loss, a painful red rash on
Full-Length Exams

monly used to treat urinary tract infections, is the legs, and change in mental status. Triazole
not associated with kernicterus. antifungals are first-line drugs for most cases
Answer D is incorrect. Ofloxacin is not associ- of coccidioidomycosis, but amphotericin B is
ated with kernicterus. used in severe cases.
Answer C is incorrect. Malassezia furfur in-
2. The correct answer is E. When Sporothrix fection is the cause of tinea versicolor. Symp-
schenckii is introduced via inoculation of soil toms of this infection include hypo-pigmented
through the skin, usually by a thorn prick, a skin lesions that occur in hot and humid con-
papule develops days to weeks later at the site ditions. M furfur has a “spaghetti and meat-
of inoculation. The primary lesion can ulcer- ball” appearance on slides because of the
ate or stay nodular, and similar lesions can short curved hyphae and yeast clusters, and is
be found along lymphatic channels (ascend- treated with topical miconazole or selenium
ing lymphangitis). In an immunocompetent sulfide.
host, pain is generally mild and there are no
systemic symptoms. S schenckii is a dimor- Answer D is incorrect. Pneumocystis jiroveci
phic fungus that produces septate hyphae and (formerly carinii) infection, like most fungal
conidia when grown at 25°C (as shown in the infections, does not present with any symp-
Test Block 7

image) and cigar-shaped yeast when grown at toms in the immunocompetent host. In chil-
37°C. Itraconazole or potassium iodide is used dren or patients afflicted with AIDS, cancer,
for the treatment of sporotrichosis. or inherited immune deficiencies, P jiroveci
can present with pneumonia. Symptoms begin
suddenly in this form of pneumonia. The pa-
Test Block 7  •  Answers 719

tient develops a fever and begins to cough and Answer C is incorrect. Nephrotoxicity is an
breathe abnormally fast. The diffuse interstitial adverse effect associated with aminoglycosides
pneumonia gives a ground-glass appearance on and vancomycin.
x-ray of the chest. P jiroveci infection is treated
Answer D is incorrect. Ototoxicity is an ad-
with TMP-SMX.
verse effect of aminoglycosides, vancomycin,
and high doses of aspirin.
3. The correct answer is D. T-lymphocyte activa-
tion requires two signals: (1) T-lymphocyte re-
5. The correct answer is C. This child has osteo-
ceptor recognition of major histocompatibility
genesis imperfecta (OI), an inherited defect
complex/peptide, and (2) CD28-B7 interac-
in collagen type I. The substitution of glycine
tion. Only professional antigen-presenting cells
by a bulky amino acid interferes with the for-
(APCs) are capable of delivering both of these
mation of the triple helix. The spectrum of
signals. The three types of professional APCs
severity observed with OI ranges from prepu-
are the dendritic cells, macrophages, and
bertal fractures with mild deformity and nor-
B lymphocytes. Langerhans cells are a type of
mal stature, to frequent childhood fractures, to
dendritic cells present within the skin.
even more severe in utero fractures that result
Answer A is incorrect. T lymphocytes and in fetal death (Note that gravida 2 para 0 in-
natural killer cells are not professional antigen- dicates a prior unsuccessful pregnancy). The
presenting cells. most common form of the disease is an auto-
somal dominant mutation in the genes coding
Answer B is incorrect. T lymphocytes are not
for collagen type I. Patients typically experi-
professional antigen-presenting cells.
ence multiple fractures with minimal trauma,
Answer C is incorrect. Epithelial cells are not the first of which may occur during birth; blue
professional antigen-presenting cells. sclera; conductive hearing loss due to deformi-
Answer E is incorrect. Natural killer cells are ties of the middle ear bones; and dental imper-
not professional antigen-presenting cells. fections.
Answer A is incorrect. Alport syndrome is an-

Full-Length Exams
4. The correct answer is E. This patient presents other connective tissue disease that may pre­
with stage 2 Lyme disease, which is marked by sent with hearing loss. However, it is not usu-
Bell palsy and evidence of a new heart block. ally associated with in utero fractures. This
Bell palsy results from a lower motor neuron disease is caused by a mutation in collagen
lesion of the facial nerve, and manifests as an type IV, which is found in the basement mem-
ipsilateral facial paralysis with the inability to brane of various organs. A defective basement
close the eye or seal the corner of the mouth membrane tends to cause problems in the kid-
on the affected side. The patient most likely ney, inner eye, and ear. Patients with Alport
acquired Lyme disease from an Ixodes tick syndrome typically have a history of glomeru-
bite during his recent camping trip to upstate lar nephritis and sensorineural hearing loss.
New York. The most common antibiotic used
to treat Lyme disease is doxycycline. Photosen- Answer B is incorrect. Marfan syndrome in-
sitivity, gastrointestinal (GI) distress, and the volves an inherited defect in fibrillin, a glyco-
discoloration of teeth in children are common protein that forms a sheath around elastin to
adverse effects of doxycycline. ensure its proper function. The condition is
characterized by ocular defect, arachnodactyly,
Answer A is incorrect. Acute cholestatic hepa- and a predisposition to aortic dissection due to
Test Block 7

titis is an adverse effect of macrolides. a weakened aortic wall.


Answer B is incorrect. Discoloration of the Answer D is incorrect. Vitamin D is necessary
teeth from doxycycline occurs among children for ossification of the cartilaginous framework.
who are <8 years old rather than in adults. Reduced availability of vitamin D leads to soft,
bowed long bones. Clinically, this disease is re-
720 Section III: Full-Length Examinations  •  Answers

ferred to as “rickets” in children and “osteoma- Answer C is incorrect. Leukemias are an ab-
lacia” in adults. normal proliferation of cells arising from the
bone marrow and are characterized by their
6. The correct answer is D. The appropriate lymphoid or hematopoietic lineage as well as
response when a patient expresses concerns by the level of differentiation seen. They are
about another physician is to have the patient not characterized by the presence of Reed-
address those concerns with that physician di- Sternberg cells.
rectly. If the problem is with a member of your
Answer D is incorrect. Multiple myeloma is a
office staff, tell the patient you will speak to
blood dyscrasia characterized by a hyperprolif-
that individual.
eration of plasma cells, which are recognized
Answer A is incorrect. Although it is impor- by their off-center, clock-face nuclei. The
tant to obtain feedback from patients, directly RBCs can also take on a rouleaux formation
after a patient has expressed concerns about resembling a stack of poker chips. Multiple
another physician is not the correct time to do myeloma usually affects older individuals and
so. typically involves bone (lytic lesions). It is as-
Answer B is incorrect. It is not appropriate to sociated with prominent serum and urinary
mediate between your patient and another of protein abnormalities (hyperglobulinemia and
their physicians, particularly when the issues Bence Jones protein).
are not directly related to concerns regarding Answer E is incorrect. Non-Hodgkin lym-
the patient’s treatment regimen. phoma is also a lymphoproliferative neoplasm
Answer C is incorrect. While you should that arises in the lymph nodes but is not char-
avoid getting drawn into mediating between acterized by the presence of Reed-Sternberg
your patient and other physicians, it is impor- cells.
tant not to dismiss a patient’s concern without
8. The correct answer is A. This patient has two
acknowledging it.
primary disturbances. She is dehydrated due
to excessive vomiting and she has a metabolic
Full-Length Exams

7. The correct answer is A. This binucleated


cell with prominent nucleoli and surrounded alkalosis from loss of HCl and hypokalemia.
by lymphocytes is a classic picture of a Reed- Acetazolamide inhibits the enzyme carbonic
Sternberg (RS) cell. Hodgkin lymphoma is a anhydrase, which is important in the reabsorp-
lymphoproliferative neoplasm that typically tion of sodium, bicarbonate, and chloride in
affects young adults, especially men, and pre­ the proximal tubule. Since it promotes the loss
sents with fever, night sweats, and weight loss. of HCO3- in the urine, it will create a meta-
The presence of RS cells is essential in the di- bolic acidosis that will help balance the pa-
agnosis of any of the Hodgkin lymphoma vari- tient’s metabolic alkalosis.
ants. In addition, increasing numbers of RS Answer B is incorrect. Ethacrynic acid is a
cells correlate with a progressively poorer prog- phenoxyacetic acid derivative that essentially
nosis. has the same action as furosemide. It is used
Answer B is incorrect. Infectious mono- in patients who are likely to be allergic to furo­
nucleosis is caused by Epstein-Barr virus and semide, and it is contraindicated in this patient
demonstrates characteristic atypical (reactive) because it will worsen her metabolic alkalosis.
lymphocytes on a blood smear. It is not char- Answer C is incorrect. Furosemide is a sulfon-
acterized by the presence of Reed-Sternberg amide loop diuretic that also causes a meta-
Test Block 7

cells. Clinical characteristics include promi- bolic alkalosis. Furosemide is a strong diuretic
nent sore throat, fever, fatigue, generalized that inhibits the Na+/K+/2Cl- cotransporter of
lymphadenopathy, and often hepatospleno- the thick ascending loop of Henle and, like the
megaly. The spleen is susceptible to traumatic thiazides, is contraindicated in this patient be-
rupture. cause it will worsen her metabolic alkalosis.
Test Block 7  •  Answers 721

Answer D is incorrect. Hydrochlorothiazide explain this patient’s symptoms of swelling and


inhibits sodium chloride reabsorption in the edema in the face and arm.
early distal tubule. It is contraindicated in this
Answer E is incorrect. Radiation has many ad-
patient because it leads to hypokalemic meta-
verse effects, including systemic effects, such
bolic alkalosis, which would only worsen her
as decreased WBC count, as well as local ef-
acid-base balance.
fects, such as skin irritation. Local radiation
Answer E is incorrect. Mannitol is an osmotic to the right breast, however, does not explain
diuretic that will only remove fluids and will this patient’s edema of the arm and face. These
not be beneficial in correcting the patient’s symptoms should prompt a more thorough
metabolic alkalosis. work-up.

9. The correct answer is B. Most of the lymph 10. The correct answer is D. Sublimation occurs
in the body is drained via the thoracic duct. It when one replaces an unacceptable impulse
passes through the diaphragm with the aorta with a course of action that is similar to the
and azygous vein posteriorly at the level of impulse but does not conflict with one’s value
T12. The right chest, back, arm, neck, and system. This method is considered one of the
head, however, are drained via the right lym- four mature defense mechanisms, along with
phatic duct, which empties into the angle altruism, humor, and suppression.
between the internal jugular and subclavian
Answer A is incorrect. Displacement is an im-
veins. With symptoms of swelling in the right
mature defense mechanism in which avoided
upper quadrant of the body, one must consider
ideas and feelings are transferred to a neu-
disruption of this structure, especially in a pa-
tral person or object. An example of this is a
tient with a history of surgery and/or radiation
woman who is angry at her boss and instead
in the right breast and axilla.
yells at her sister on the phone.
Answer A is incorrect. Symptoms of deep ve-
Answer B is incorrect. Projection is an im-
nous thrombosis (DVT) include swelling, red-
mature defense mechanism in which an un-
ness, and tenderness of the areas distal to the

Full-Length Exams
acceptable internal impulse is attributed to an
thrombosis. DVTs can develop in the axillary
external source. An example of this is a man
or subclavian veins, most commonly as a com-
who is attracted to another woman and accuses
plication of venous catheters in these sites, and
his wife of cheating on him.
may present with unilateral swelling. Patients
with cancer have an increased risk of DVTs, Answer C is incorrect. Reaction formation is
since malignancies cause a prothrombotic an immature defense mechanism in which a
state. A DVT in the cephalic vein, however, warded-off idea or feeling is replaced by an un-
does not explain the facial edema. consciously derived emphasis on its opposite.
An example of this is a patient with libidinous
Answer C is incorrect. The thoracic duct
thoughts entering a monastery.
drains all of the lymph in the body except that
from the right upper quadrant, which includes Answer E is incorrect. Suppression is another
the right arm and right side of the face. There- mature defense mechanism in which un-
fore, a lesion of the thoracic duct could not ex- wanted feelings are voluntarily (unlike other
plain the edema in this patient. defenses) withheld from conscious awareness.
An example of this is a patient with pancre-
Answer D is incorrect. Metastatic disease to
atic cancer who decides that he will only think
bone is common in advanced breast cancer,
Test Block 7

about his illness 15 minutes per day.


and most metastatic disease occurs in the cen-
tral skeleton (vertebrae, pelvis, ribs, upper legs, 11. The correct answer is B. Osteonecrosis of
and upper arms). Bone metastases may cause the jaw (avascular necrosis of the jaw) has
pain and pathologic fractures, but do not best been associated with dental extraction, local
722 Section III: Full-Length Examinations  •  Answers

infection, pathologic fracture of the jaw, and Answer B is incorrect. Gentamicin binds to
chronic bisphosphonate therapy. Bisphospho- the 30S subunit and is associated with nephro-
nates, such as alendronate and risedronate, toxicity and ototoxicity with prolonged use.
are used to treat metastatic bone diseases and
Answer C is incorrect. Erythromycin is asso-
osteoporosis. Multiple myeloma causes bone
ciated with cholestatic hepatitis, eosinophilia,
destruction due to increased osteoclast activity.
and skin rashes. Azithromycin, a related mac-
Bisphosphonates decrease pain and fractures
rolide, is better tolerated and results in fewer
by reducing the number and activity of osteo-
adverse events. The sulfonamides act by inhib-
clasts and inhibiting bone resorption. GI toxic-
iting dihydropteroate synthetase, an enzyme
ity and hypocalcemia have also been reported
involved in folate synthesis. They can cause se-
in patients taking bisphosphonates.
rious allergic reactions, urinary tract disorders,
Answer A is incorrect. Angiotensin converting and porphyria. Although the sulfonamides can
enzyme (ACE) inhibitors, such as enalapril, cause aplastic anemia, it occurs far less fre-
inhibit angiotensin-converting enzyme and quently than with chloramphenicol.
are utilized in the treatment of hypertension.
Answer D is incorrect. Ciprofloxacin, an in-
ACE inhibitor toxicity can include cough, an-
hibitor of DNA gyrase, is associated with su-
gioedema, proteinuria, taste changes, rash, and
perinfections, skin rashes, headache, and dizzi-
hyperkalemia. Osteonecrosis is not associated
ness. The more unique complications include
with use of ACE inhibitors.
tendonitis and tendon rupture in adults, and
Answer C is incorrect. Cephalosporins are cartilage malformation in children and in the
b-lactam antibiotics used to treat infections developing fetus.
caused by gram-positive and gram-negative
Answer E is incorrect. Clindamycin is classi-
bacteria. Use of cephalosporins does not cause
cally associated with intestinal colonization
osteonecrosis.
by Clostridium difficile. This leads to pseudo-
Answer D is incorrect. Osmotic diuretics such membranous colitis, in which patients usually
as mannitol are used to treat shock and drug present with cramping, watery diarrhea, and a
Full-Length Exams

overdose. Osmotic diuretics are not associated low-grade fever. The penicillin drugs act by in-
with osteonecrosis. hibiting peptidoglycan cross-linking. The most
serious adverse effect is the possibility of an
Answer E is incorrect. The vinca alkaloids,
anaphylactic reaction.
such as vincristine and vinblastine, are mi-
crotubule inhibitors used in the treatment of
13. The correct answer is B. This patient is pre-
testicular carcinoma and Hodgkin and non-
senting with pain disorder. This is character-
Hodgkin lymphomas. Vinca alkaloids are not
ized by pain symptoms that are inconsistent
associated with osteonecrosis.
with physiological processes. There is typically
a close temporal relationship with psychologi-
12. The correct answer is A. Despite its potent
cal factors, and such disorders are seen more
bacteriostatic activity, chloramphenicol is
commonly in females than males. Peak onset
now used rarely because of its toxicities (gray
is at age 40-50, and it may be associated with
baby syndrome and aplastic anemia). Aplastic
depression. Treatment typically includes reha-
anemia with chloramphenicol use is a dose-
bilitation, such as behavioral therapy, physi-
dependent adverse event that can occur after
cal therapy, and psychotherapy. Analgesia is
only a short course of therapy and can be fatal.
usually not helpful. Tricyclic antidepressants
Because it is inexpensive, chloramphenicol is
Test Block 7

(TCAs) and venlafaxine may be therapeutic.


often used in resource-limited settings over-
seas. Chloramphenicol acts by binding to and Answer A is incorrect. Doxycycline is the stan-
inhibiting the 50S ribosomal subunit. dard treatment for Lyme disease. Lyme dis-
ease in advanced stages can present with joint
pain and fatigue. However, the patient has had
Test Block 7  •  Answers 723

extensive laboratory and clinical assessments, of life, and may have 2 types of epithelium
with no indication of an infectious process. (gastric and pancreatic). Occasionally acid se-
creted from the gastric mucosa in a Meckel
Answer C is incorrect. Group therapy is help-
diverticulum may cause local ulceration and
ful for many psychiatric conditions, including
bleeding.
hypochondriasis. In this disorder there is pre-
occupation with or fear of having a serious dis- Answer A is incorrect. The dorsal mesentery
ease despite medical reassurance. This leads gives rise to, among other things, the spleen.
to significant distress and impairment, and of- Meckel diverticulum, however, arises from the
ten involves a history of prior physical disease. midgut.
Men and women are equally affected, and
Answer B is incorrect. The foregut gives rise
the onset typically occurs in adulthood. Hy-
to the GI tract from the esophagus through the
pochondriasis is managed with group therapy
upper duodenum, as well as the liver, gallblad-
and regularly scheduled appointments with
der, and pancreas. These structures are proxi-
the patient’s primary caregiver. This patient
mal to the region that would be affected by a
has not described a preoccupation about her
Meckel diverticulum.
illness, so it is unlikely that she is suffering
from hypochondriasis. Answer C is incorrect. The hindgut gives
rise to the GI tract from the distal third of the
Answer D is incorrect. Analgesia, including
transverse colon to the upper anal canal. This
opioid treatment, is generally not indicated for
is distal to the region where a Meckel divertic-
pain disorder. Physical therapy, TCAs and ven-
ulum would appear.
lafaxine are more appropriate therapy.
Answer D is incorrect. The mesonephric
Answer E is incorrect. This answer is incor-
(Wolffian) duct gives rise to male internal re-
rect because there are standard treatments for
productive organs: seminal vesicles, epididy-
pain disorder. Moreover, pain disorder may not
mis, ejaculatory duct, and ductus deferens. It is
resolve on its own once it has been persistent
not related to Meckel diverticulum.
for several years. In contrast, conversion disor-

Full-Length Exams
der often resolves spontaneously. Patients with Answer F is incorrect. Neural crest cells
conversion disorder present with symptoms or give rise to many structures in the body, in-
deficits of voluntary or sensory function (eg, cluding the intestinal ganglia affected in
blindness, seizure, or paralysis). These symp- Hirschsprung disease. However, this image
toms often occur in close temporal relation- shows a Meckel diverticulum, a midgut mal-
ship to stress or intense emotion. Conversion formation that does not arise directly from neu-
disorder is more common in young females, ral crest cells.
less-educated people, and those from lower so-
cioeconomic classes. 15. The correct answer is A. This patient presents
with a pleural effusion, the accumulation of
14. The correct answer is E. The image shows a excess fluid in the pleural space. Pleural effu-
Meckel diverticulum, the result of the persis- sions can have a number of etiologies, includ-
tence of a portion of the vitelline duct. This ing pneumonia, congestive heart failure, and
manifests as a blind pouch that protrudes from cancer. Therapeutic thoracentesis can be per-
the ileum. The ileum is derived from the mid- formed to relieve symptoms and improve res­
gut, a portion of the primitive gut tube that piratory function. The intercostal vein, artery,
gives rise to the intestinal tract from the dis- and nerve run in the intercostal groove on the
Test Block 7

tal duodenum to the proximal two-thirds of inferior surface of each rib. When thoracente-
the transverse colon. Meckel diverticulum is sis is performed, the needle is always inserted
characterized by the “rule of 2s”: it is 2 inches at the most inferior aspect of an intercostal
long, 2 feet from the ileocecal valve, occurs in space to avoid these structures running along
2% of the population, presents in first 2 years the superior aspect of the space.
724 Section III: Full-Length Examinations  •  Answers

Answer B is incorrect. The parietal pleura is immunodeficiency syndromes (SCIDs), which


the outer layer of the pleura and is attached to can have many causes. Their typical presenta-
the chest wall. When performing a therapeutic tion includes recurrent bacterial, viral, proto-
thoracentesis, it is necessary to pierce the pari- zoal, and fungal infections. SCIDs are not as-
etal pleura in order to access the pleural space. sociated with a deficiency of LFA-1.
Answer C is incorrect. The phrenic nerve is Answer C is incorrect. A defect in the devel-
found deep in the thorax, running along the opment of the third and fourth pharyngeal
mediastinum and pericardium; it is too deep to pouches is the cause of thymic aplasia (com-
be injured by thoracentesis. monly in DiGeorge syndrome), in which the
thymus and parathyroid glands fail to develop.
Answer D is incorrect. The pericardiophrenic
The disease often presents with congenital
vessels travel with the phrenic nerve along the
defects such as cardiac abnormalities, cleft
mediastinum and pericardium. These vessels
palate, and abnormal facies, but it is not asso-
are too deep to be injured by this procedure.
ciated with delayed umbilical separation. Thy-
Answer E is incorrect. The needle here is in- mic aplasia also can present with tetany due to
serted above the tenth rib, in the ninth inter- hypocalcemia.
costal space. The tenth intercostal vessels and
Answer D is incorrect. A defect in the empty-
nerve run below the tenth rib, in the tenth in-
ing of phagocytic cells, due to microtubular
tercostal space.
dysfunction, is the cause of Chédiak-Higashi
Answer F is incorrect. The visceral pleura is disease, an autosomal recessive condition
the inner layer of pleura that covers the lungs that presents with recurrent streptococcal and
and adjoining structures in the thorax. It is im- staphylococcal infections. It is not associated
portant to avoid piercing the visceral pleura with a deficiency of LFA-1.
because of its association with the lung. Avoid-
Answer E is incorrect. A deficiency of
ance of this structure, however, is not strongly
NADPH oxidase is characteristic of chronic
dependent on inserting the needle at the infe-
granulomatous disease, which presents with
rior aspect of the intercostal space. This is in-
Full-Length Exams

an increased susceptibility to infections by


stead best aided by asking patients to hold their
microbes that produce their own catalase (eg,
breath.
Staphylococcus). A negative nitroblue tetrazo-
16. The correct answer is B. This patient most lium dye reduction test confirms the diagnosis
likely has leukocyte adhesion deficiency of chronic granulomatous disease.
(LAD) syndrome, which is caused by a defi-
17. The correct answer is A. The first curve, with
ciency of the b2-integrin subunit and subse-
mean = median = mode, represents a normal
quent defects in several proteins, including
Gaussian distribution. The second curve rep-
LFA-1. LFA-1 is an adhesion protein on the
resents a negative skew. The mean is equal to
surface of neutrophils. The disease usually
the center of the graph. The mode is equal to
presents with marked leukocytosis and local-
the most common result. This is represented at
ized bacterial infections that are difficult to de-
the top of the curve. The median is the middle
tect until they have progressed to an extensive,
value if the value were ordered sequentially. It
life-threatening level. Because neutrophils
turns out that during either a positive skew or
are unable to adhere to the endothelium and
a negative skew, the median is in between the
transmigrate into tissues, infections in patients
mean and the mode. Therefore, mean < me-
with LAD syndrome act similarly to those ob-
Test Block 7

dian < mode.


served in neutropenic patients.
Answer B is incorrect. In Gaussian distribu-
Answer A is incorrect. A defect in the develop-
tions, the median is always between the mode
ment and differentiation of T and B lympho-
and the mean.
cytes is characteristic of the severe combined
Test Block 7  •  Answers 725

Answer C is incorrect. In Gaussian distribu- would result in deficiencies in nucleotides


tions, the median is always between the mode needed for DNA synthesis. Symptoms may
and the mean. resemble conditions in which dietary deficien-
cies impede de novo nucleotide synthesis, such
Answer D is incorrect. In Gaussian distribu-
as megaloblastic anemia due to folic acid and/
tions, the median is always between the mode
or vitamin B12 deficiency.
and the mean.
Answer E is incorrect. An excision repair en-
Answer E is incorrect. In Gaussian distribu-
zyme deficiency would result in xeroderma
tions, the median is always between the mode
pigmentosum, which is marked by dry and hy-
and the mean.
perpigmented skin that is extremely sensitive
Answer F is incorrect. This would be the case to exposure to ultraviolet radiation. Therefore,
in a positively skewed data distribution, rather individuals with this disease are at increased
than a negative skew. risk for severe sunburns and skin cancer.

18. The correct answer is D. Lesch-Nyhan syn- 19. The correct answer is B. The patient’s com-
drome is an X-linked recessive disorder caused plaints of dry eyes (xerophthalmia) and associ-
by a deficiency in the production of hypoxan- ated joint pain are consistent with the clinical
thine guanine phosphoribosyltransferase that presentation of Sjögren syndrome; this condi-
leads to the overproduction of purine and the tion can also present with dry mouth (xerosto-
accumulation of uric acid. This rare biochemi- mia). Sjögren syndrome results from autoim-
cal disorder is characterized clinically by hy- mune destruction of the lacrimal and salivary
peruricemia, excessive production of uric acid, glands. It can occur as an isolated disorder or
and certain characteristic neurologic features, in association with another autoimmune dis-
including self-mutilation, choreoathetosis, ease (secondary form). Among associated disor-
spasticity, and mental retardation. ders, rheumatoid arthritis is the most common,
Answer A is incorrect. A deficiency of adeno­ explaining the joint pain and fever in this pa-
sine deaminase would result in SCID, which tient. Because patients with Sjögren syndrome

Full-Length Exams
prevents development of both the humoral have decreased salivary secretions, the defense
and cell-mediated immune systems. Therefore against pathogenic bacteria that cause dental
individuals with SCID are faced with recur- caries is compromised.
rent devastating bacterial, viral, and fungal in- Answer A is incorrect. Dactylitis (inflamma-
fections. tion of an entire finger or toe) is a common
Answer B is incorrect. A deficiency of b-glu- condition associated with psoriatic arthritis.
cocerebroside would result in Gaucher dis- Psoriatic arthritis is a member of the seronega-
ease. There are several types of Gaucher dis- tive (rheumatoid factor negative) spondylo-
ease based on the type of mutation, but most arthropathies and is strongly associated with
forms are marked by lipid-laden macrophages HLA-B27 and with male gender. Without a
(termed Gaucher cells) that invade the bone long history of multiple plaques over the skin
marrow and cortex, leading to bone infarction, or immunosuppressive treatment, psoriatic ar-
vertebral collapse, and anemia and thrombocy- thritis is very unlikely.
topenia. Answer C is incorrect. Hyperglycemia is most
Answer C is incorrect. This boy’s findings can commonly associated with insulin resistance
best be explained by Lesch-Nyhan syndrome, and/or absence of insulin production associated
Test Block 7

which is caused by a deficiency in the produc- with diabetes mellitus. It is not a common phe-
tion of hypoxanthine guanine phosphoribo- nomenon in Sjögren syndrome, although there
syltransferase, not a mutation of an enzyme is a suggested association between Sjögren syn-
in the de novo biosynthetic pathway, which drome and type 1 diabetes, both of which are
autoimmune diseases.
726 Section III: Full-Length Examinations  •  Answers

Answer D is incorrect. Jaundice is usually as- ference in outcome between the group that
sociated with pathologic processes involving dropped out and the group that continued
liver or hemolytic anemias. These patholo- treatment. It can also occur with nonrandom
gies are relatively uncommon in Sjögren syn- assignment of participants to study groups,
drome. such as when the subject chooses whether to
enter a drug group or a placebo group rather
Answer E is incorrect. Septic joints are usu-
than being randomly assigned, or when the
ally a consequence of an infectious process,
investigator purposely chooses to put a sub-
whereas the arthritis in Sjögren syndrome is
ject in a drug or placebo group. In this experi-
an inflammatory arthritis. Septic joints are usu-
ment, for example, one possible result of selec-
ally acute onset monoarticular arthritides that
tion bias is that people reacting poorly to the
produce warm and tender joints on physical
experimental drug may choose to drop out.
exam.
Thus, when we account for these patients, we
20. The correct answer is B. This patient’s his- may see that the experimental drug actually
tologic section demonstrates a lymph node has worse effects on psoriasis, causing patients
lacking germinal centers. Activation of the to drop out of the study. Researchers need to
B-lymphocyte response occurs in the follicu- account for dropout data in order to detect the
lar zone in the outer cortex of the lymph node. actual difference between the two treatments.
Proliferating B cells form clusters, termed ger- Answer A is incorrect. Procedural bias is the
minal centers, where somatic hypermutation same as recall bias.
and affinity maturation take place. Lack of ger-
Answer B is incorrect. In recall bias, knowl-
minal centers and the clinical scenario of dis-
edge of the presence of disorders alters the way
ease following vaccination with live pathogen
subjects remember their histories. It is most
suggest a B-lymphocyte immunodeficiency.
common in retrospective studies when patients
Thus, the most helpful treatment would be ad-
are asked to recall information. For example,
ministration of immunoglobulins.
those who develop a cold are more likely to
Answer A is incorrect. Cyclosporine is an im- identify their exposure than those who do not.
Full-Length Exams

munosuppressant that increases susceptibility They may recall being sneezed on, for exam-
to infection and is contraindicated in this pa- ple, while those who do not develop a cold do
tient. not.
Answer C is incorrect. Granulocyte-macro- Answer C is incorrect. Sampling bias de-
phage colony-stimulating factor (GM-CSF) is scribes when volunteer subjects in a study are
used to speed recovery of bone marrow granu- not representative of the population being
locytes and monocytes. studied, and as a consequence, the results of
Answer D is incorrect. Like cyclosporine, ta- the study may not be applicable to the entire
crolimus is an immunosuppressive agent and is population. This is also referred to as external
not the appropriate treatment for a patient suf- validity.
fering from immunodeficiency. Answer E is incorrect. Although larger sample
Answer E is incorrect. Used clinically to in- sizes increase the power of a study, a study can
crease platelet counts, thrombopoietin would be valid even with a relatively small number of
not be a useful treatment option for this pa- subjects depending on the magnitude of dif-
tient. ferences in outcomes between the two groups
and the variance of outcome measures among
Test Block 7

21. The correct answer is D. Selection bias can individuals within groups.
occur when the researcher decides to not use
data obtained from patients who have dropped 22. The correct answer is B. This patient is suf-
out of the study, especially if there is a dif- fering from emphysema, an obstructive lung
disease. Smoking leads to increased neutro-
Test Block 7  •  Answers 727

phil elastase activity in alveoli in the lungs, available oxygen to bind to hemoglobin and
and over time this results in destruction of the low oxygen saturation.
alveolar walls. This destruction results in de-
Answer E is incorrect. The solubility of oxy-
creased diffusion of oxygen into the alveolar
gen in the blood is dependent on the chemical
walls, resulting in improper oxygenation of
properties of blood as a liquid and oxygen as a
the blood running through the capillaries that
component of the air. Emphysema does not af-
supply these alveoli. This results in the oxy-
fect these basic chemical properties.
genation of the pulmonary circulation chang-
ing from perfusion-limited (as it is normally) to 23. The correct answer is E. Lung cancer is the
diffusion-limited. The oxygen-carrying capac- leading malignant cause of superior vena cava
ity of the blood is now limited by the rate at syndrome (SVCS), as seen in this patient.
which oxygen can diffuse into the capillaries. Compression of the superior vena cava leads
Since there is now pathologic destruction of al- to dilation of the venous collateral circulation
veolar walls, there is not enough surface area and to head and neck edema. Although other
for oxygen to properly diffuse across. malignancies can lead to SVCS, the patient’s
Answer A is incorrect. Decreased ability of he- age and history of smoking make squamous
moglobin to bind oxygen would have to be the cell carcinoma of the lung the most likely eti-
result of a defect in the hemoglobin molecule ology. Squamous cell lung carcinoma and
itself, which would most likely have been of small-cell lung carcinoma demonstrate strong
congenital origin, or carbon monoxide poison- associations with smoking of 90% and 99%, re-
ing. This man clearly is not affected with car- spectively. Of note, the most common type of
bon monoxide poisoning, and has no history lung cancer in nonsmokers is adenocarcinoma
of any sort of problem with oxygen-carrying ca- of the lung.
pacity of his blood. Emphysema is a problem Answer A is incorrect. Most cases of SVCS are
of ventilation-perfusion mismatch, not a defect attributed to mediastinal tumors or lympho-
in hemoglobin. mas, not to GI cancers such as gastric adeno-
Answer C is incorrect. The hemoglobin con- carcinoma.

Full-Length Exams
tent of the blood will never affect a patient’s Answer B is incorrect. Lymphoma, particu-
oxygen saturation levels. Saturation levels are larly Hodgkin lymphoma, is the leading cause
always a reflection of oxygen’s ability to get of SVCS in young adults. However, the age
into the bloodstream, not the amount of he- and medical history of this patient make this
moglobin in the blood, since hemoglobin (as- diagnosis unlikely.
suming it is normal, which it is in this case)
always has the same binding capacity and affin- Answer C is incorrect. Hypertrophic cardio-
ity for oxygen. However, patients with chronic myopathy (HCM) can present as sudden death
diseases such as emphysema, who have chroni- in young athletes as a result of expansion of
cally low oxygen saturation levels, typically the interventricular septum and obstruction of
have blood with higher levels of hemoglobin the outflow tract. Although dyspnea can be a
to attempt to compensate for the inability of presenting symptom of HCM, this patient has
blood to properly oxygenate in the lungs. additional clinical signs suggestive of SVCS,
and his smoking history and age make it more
Answer D is incorrect. In emphysema, blood likely that lung cancer is the etiology.
oxygenation in the pulmonary circulation is
diffusion-limited. However, there is no impair- Answer D is incorrect. Idiopathic pulmonary
Test Block 7

ment of perfusion to the alveoli. Rather, it is fibrosis (IPF) is a form of interstitial lung dis-
the decreased surface area of oxygenation over ease that causes progressive dyspnea on exer-
all alveoli that results in inadequate diffusion tion and, in some cases, a nonproductive dry
of oxygen into the bloodstream, leading to less cough. Common chest radiographic findings
are increased lung markings at the bases and
728 Section III: Full-Length Examinations  •  Answers

a honeycombing pattern. However, the head Answer A is incorrect. The Michaelis-Menten


and neck edema, in addition to the smoking constant of an enzyme is increased by a com-
history, in this patient suggest SVCS secondary petitive inhibitor.
to lung cancer rather than IPF.
Answer C is incorrect. The Michaelis-Menten
constant of an enzyme is increased by a com-
24. The correct answer is D. The question is ask-
petitive inhibitor.
ing for the number of false-positives. Specific-
ity = true-negatives/(true-negatives + false- Answer D is incorrect. The maximum reac-
positives). False-positive signifies the number tion rate of an enzyme is unchanged by a com-
of people without disease X who will be petitive inhibitor.
falsely diagnosed by the screening test. In this
Answer E is incorrect. The Vmax is the maxi-
case, 900 people do not have the disease, rep-
mum rate or velocity in which substrate mol-
resented by true-negatives + false-positives.
ecules are converted to product per unit time.
Using a specificity of 70%, the number of
At high substrate concentrations, the reaction
true-negatives is 630, while the number of
rate levels off, reflecting the saturation of all
false-positives is 270. Thus, 270 people with-
available binding sites with substrate. At a suf-
out disease X will be falsely diagnosed with this
ficiently high substrate concentration, the reac-
screening test (ie, they will be false-positives).
tion velocity reaches the Vmax observed in the
Answer A is incorrect. The figure 20 is the absence of inhibitor. Thus, the Vmax of an en-
number of people with the disease who will zyme is unchanged by a competitive inhibitor,
have an incorrect negative screening test result as an increase in substrate outcompetes this
(ie, false-negatives). type of inhibitor.
Answer B is incorrect. The figure 80 is the
26. The correct answer is B. This patient has
number of people who will have a correct posi-
normal-pressure hydrocephalus (NPH), which
tive screening test result (ie, true-positives).
is a condition of the elderly characterized by
Answer C is incorrect. The figure 100 is the chronically dilated ventricles. Patients with
Full-Length Exams

number of people in the town with disease X NPH typically present with the following clini-
(ie, the prevalence of disease X). cal triad: urinary incontinence, gait difficul-
ties, and mental decline (remember the mne-
Answer E is incorrect. The figure 630 is the
monic: “wet, wobbly, and wacky”). Although
number of people who will have a correct neg-
the exact mechanism is not known, NPH is be-
ative screening test result (ie, true-negatives).
lieved to be a form of communicating hydro-
25. The correct answer is B. The Michaelis- cephalus in which cerebrospinal fluid (CSF)
Menten constant (Km) of an enzyme such as reabsorption is impaired at the arachnoid villi.
DHFR reflects the enzyme’s affinity for a par- Answer A is incorrect. Choroid plexus papil-
ticular substrate [S], such as methotrexate, loma is a rare cause of hydrocephalus from
in an inverse fashion. The Km of an enzyme excess CSF production. If our patient had this
is the concentration of substrate required to lesion, we would expect to see an enhancing
achieve a reaction velocity equal to half of the mass in the lateral ventricle. A choroid plexus
maximum reaction rate (Vmax). A competi- papillomas usually presents in patients under
tive inhibitor binds reversibly to the same site age 10, so it is unlikely in this case.
that the substrate would normally occupy and
Answer C is incorrect. Hydranencephaly is
thus competes with the substrate for that site.
Test Block 7

the absence of cerebral hemispheres, which


Therefore, in the presence of a competitive
have been replaced by fluid-filled sacs lined by
inhibitor, the concentration of methotrexate
leptomeninges. The skull and its brain cavities
required to achieve half of Vmax will be in-
are normal. This patient clearly has cerebral
creased.
hemispheres.
Test Block 7  •  Answers 729

Answer D is incorrect. Excessive ingestion of dividuals during dehydration or vigorous exer-


vitamin A can increase secretion of CSF and/ cise.
or increase permeability of the blood-brain
Answer E is incorrect. WBC casts are indica-
barrier, leading to hydrocephalus. However,
tive of inflammation or infection. Thus WBC
there is no reason to believe in this case that
casts strongly suggest pyelonephritis. They also
the patient is taking (or being given) excess vi-
may be seen in inflammatory states such as
tamin A.
acute allergic interstitial nephritis, nephrotic
Answer E is incorrect. Non-communicating syndrome, or poststreptococcal acute glomeru-
hydrocephalus is caused by obstruction of CSF lonephritis.
flow within the ventricular system. Obstruction
can be secondary to tumors, intraparenchymal 28. The correct answer is A. This patient has a le-
hemorrhage, other masses, and congenital sion affecting Broca area (Brodmann area 44)
malformations. This patients CT was negative in the inferior frontal gyrus, causing an expres-
for these abnormalities. sive aphasia characterized by decreased flu-
ency of spontaneous speech with intact com-
27. The correct answer is D. This patient is suffer- prehension. Broca’s aphasia is often caused by
ing from poststreptococcal glomerulonephritis an infarction of the areas supplied by the su-
(PSG), and RBC casts are pathognomonic of perior division of the middle cerebral artery,
glomerulonephritis. Symptoms in PSG usually which often includes both Broca area and the
develop one-three weeks after acute infection primary motor cortex, leading to an expressive
with specific nephritogenic strains of group aphasia and right-sided spastic paralysis. The
A b-hemolytic streptococcus. The incidence majority of people (about 95% of right-handed
of glomerulonephritis is approximately 5%- and 70% of left-handed) have language lateral-
10% in persons with pharyngitis and 25% in ized to the left hemisphere.
those with skin infections during an epidemic
Answer B is incorrect. The primary somato-
of infection caused by group A b-hemolytic
sensory cortex corresponding to the right side
streptococcus. Some other examples of glo-
of the body is located in the left postcentral

Full-Length Exams
merulonephritis include diffuse proliferative
gyrus. A lesion of this area would cause right-
glomerulonephritis and rapidly progressive
sided sensory defects, which are not relevant to
crescentic glomerulonephritis.
this case as this patient presents with aphasia
Answer A is incorrect. The epithelial cell cast and motor defects.
is formed by inclusion or adhesion of desqua-
Answer C is incorrect. Wernicke area (Brod-
mated epithelial cells of the tubule lining.
mann area 22) is located in the posterior two-
These casts can be seen in acute tubular ne-
thirds of the superior temporal gyrus in the
crosis and toxic ingestion (eg, mercury, diethyl-
dominant hemisphere. Assuming that this pa-
ene glycol, or salicylate).
tient has a left-dominant hemisphere for lan-
Answer B is incorrect. Fatty casts are prod- guage, a lesion of this area would lead to a re-
ucts of breakdown of lipid-rich epithelial cells. ceptive aphasia with impaired comprehension
They resemble hyaline casts with fat globule, and meaningless, empty speech (Wernicke
yellowish-tan in color. These casts are found aphasia). Such a lesion could be caused by an
in various disorders, including nephrotic syn- infarct of the inferior division of the left middle
drome, and diabetic or lupus nephropathy. cerebral artery. This patient, however, has an
expressive aphasia and thus the inferior frontal
Answer C is incorrect. Hyaline casts are the
Test Block 7

gyrus (Broca area), not the superior temporal


most common type of cast. They are formed
gyrus (Wernicke area), is likely to be involved.
from the tubular epithelial cells of individual
nephrons. Contributing factors include low Answer D is incorrect. About 30%-40% of left-
urine flow, concentrated urine, or an acidic handed individuals and <5% of right-handed
environment. They may be seen in normal in- individuals will have a right-dominant hemi-
730 Section III: Full-Length Examinations  •  Answers

sphere for language. However, this patient’s Answer D is incorrect. Optic neuritis, not
right-sided paralysis and right-handedness are ophthalmoplegia, is associated with ethambu-
indications that the lesion is in the left, not tol. Optic neuritis is inflammation of the optic
right, hemisphere. nerve, which can cause abrupt partial or com-
plete loss of vision.
Answer E is incorrect. The primary somato-
sensory cortex corresponding to the left side Answer E is incorrect. Renal failure is not a
of the body is located in the right postcentral common adverse effect of any of the four drugs
gyrus (parietal lobe). A lesion to the right post- listed. Hepatotoxicity is associated with INH,
central gyrus would cause left-sided sensory rifampin, and pyrazinamide.
deficits, but not aphasia or motor defects as in
this patient. 30. The correct answer is E. This patient is hav-
ing a hypertensive emergency, defined as a
Answer F is incorrect. In right hemisphere-
systolic blood pressure >210 mm Hg and/or a
dominant individuals, a lesion of the right su-
diastolic blood pressure >120 mm Hg. In addi-
perior temporal gyrus would lead to a receptive
tion, she is exhibiting the signs and symptoms
aphasia, but this patient is most likely left-
of end-organ involvement: blurry vision and
hemisphere dominant for language and has
headache, as well as papilledema. Sodium ni-
an expressive, not a receptive, aphasia. Thus,
troprusside, a first-line medication for hyper-
the inferior frontal gyrus, and not the superior
tensive emergencies, acts by direct vasodilation
temporal gyrus, is likely to be involved.
of both arteries and veins. Adverse effects of
nitroprusside include reflex tachycardia as well
29. The correct answer is C. This patient’s symp-
as cyanide toxicity, especially in patients with
toms of fever, night sweats, and hemoptysis
liver disease.
are consistent with tuberculosis (TB). The
calcification in the right upper lobe is a Ghon Answer A is incorrect. Captopril, an ACE in-
complex, which is the TB granuloma (Ghon hibitor, is an extremely effective medication in
focus), with surrounding lobar and perihilar controlling chronic hypertension. However, it
lymph node involvement. TB is caused by My- is not useful in acute hypertensive emergen-
Full-Length Exams

cobacterium tuberculosis, which can be visual- cies because its mechanism of action is too
ized with an acid-fast stain. The current treat- slow.
ment for active TB is a multidrug regimen,
Answer B is incorrect. Hydrochlorothiazide
including rifampin, isoniazid (INH), pyrazin-
is an effective diuretic medication used for the
amide, and ethambutol. Of the adverse effects
treatment of chronic essential hypertension.
listed, only numbness and tingling (peripheral
Because of its mechanism of action, it does
neuropathy) are commonly associated with
not cause a reduction in blood pressure rapid
INH and ethambutol. INH-associated periph-
enough to be useful in an emergent situation
eral neuropathy may be prevented or reversed
such as the one described here.
with vitamin B6 supplementation.
Answer C is incorrect. b-blockers such as the
Answer A is incorrect. Blue and yellow color
nonselective agent labetalol are also first-line
blindness is not an adverse effect associated
agents in managing hypertensive emergencies,
with any of the anti-tuberculosis drugs. Red
although their mechanism is not completely
and green color blindness is associated with
understood. With first-time emergent use, the
ethambutol. Other adverse effects of ethambu-
antihypertensive action of labetalol is thought
tol include optic neuritis, peripheral neuropa-
to result from a decrease in cardiac output.
Test Block 7

thy, arthralgia, and vertical nystagmus.


However, this patient’s history of asthma makes
Answer B is incorrect. Arthralgias, not myal- labetalol a suboptimal choice, as adrenergic
gias, are associated with ethambutol and pyra- blockade in the airways could precipitate bron-
zinamide. chospasm.
Test Block 7  •  Answers 731

Answer D is incorrect. As an angiotensin re- researchers to a better understanding of the


ceptor blocker, losartan is a useful drug with role of leptin in the proper uptake of HDL
which to control chronic hypertension. It has cholesterol.
no role in the acute management of hyperten-
Answer G is incorrect. A deficiency in lipo-
sive emergency because of its slow onset of ef-
protein lipase would result in hyperchylomi-
fect.
cronemia but would not produce an elevated
cholesterol level. Lipoprotein lipase is required
31. The correct answer is E. This is a case of fa-
for the hydrolysis of triglycerides in chylomi-
milial hypercholesterolemia (FH), a genetic
crons and VLDL cholesterol. Genetic defi-
deficiency of LDL cholesterol receptors. As a
ciency of lipoprotein lipase results in impaired
result of this condition, serum LDL and total
lipolysis and profound elevations in plasma
cholesterol levels are grossly elevated, although
chylomicron levels. Although these patients
levels of other lipids may be within normal
also have elevations in plasma VLDL choles-
limits. Patients with FH often have a family
terol levels, chylomicronemia predominates.
history significant for myocardial infarctions
before the age of 40 years. In addition, xan-
32. The correct answer is B. An x-ray film of the
thomas (cholesterol deposits) are sometimes
chest indicates that the boy has a paralyzed
seen within the eyelids (shown) as well as the
right hemidiaphragm. It is possible for the
skin of the upper and lower extremities (not
phrenic nerve to become damaged during
shown).
heart surgery, since it runs along the pericar-
Answer A is incorrect. A deficiency in dium. It is not unusual for a patient to remain
3-hydroxy-3-methylglutaryl coenzyme A reduc- asymptomatic until starting to run long dis-
tase (HMG CoA reductase) would result in ex- tances.
tremely low levels of cholesterol, since this is
Answer A is incorrect. When a patient holds
the rate-limiting enzyme in cholesterol synthe-
his or her breath during a chest x-ray, a con-
sis. Statin drugs block HMG CoA reductase
tracted diaphragm will move downward, and a
in order to lower LDL cholesterol levels in pa-
paralyzed diaphragm will paradoxically move

Full-Length Exams
tients with hypercholesterolemia.
upward because of the negative pressure gen-
Answer B is incorrect. 7a-hydroxylase is the erated on the left side of the thorax pulling the
rate-limiting enzyme in bile acid synthesis. A mediastinal structures towards the left.
defect in this enzyme would not result in hy-
Answer C is incorrect. Eisenmenger syn-
percholesterolemia.
drome is the cyanosis and symptoms that occur
Answer C is incorrect. Apolipoprotein C-II when a prior left-to-right shunt reverses and be-
is a cofactor for lipoprotein lipase. A defect in comes a right-to-left shunt. Cyanosis does not
apolipoprotein C-II would thus result in hyper- occur in a left-to-right shunt because oxygen-
chylomicronemia due to the inability to acti- ated arterial blood simply reenters the pulmo-
vate lipoprotein lipase. nary circulation. After years of arterial blood
overloading the right side of the heart, how-
Answer D is incorrect. A defect in apolipopro-
ever, pulmonary pressures can increase above
tein E would result in hyperchylomicronemia
systemic pressures and the shunt reverses.
and elevated levels of triglycerides and VLDL
When this occurs, deoxygenated blood enters
cholesterol. However, serum triglycerides are
systemic circulation and cyanosis results. Al-
within normal limits in this patient, making an
though the question states that this patient had
apolipoprotein E defect less likely.
Test Block 7

a cardiac defect repaired in infancy, there is no


Answer F is incorrect. Defective leptin recep- evidence of cyanosis, which is the hallmark of
tors would result in high serum levels of HDL Eisenmenger syndrome.
cholesterol, not LDL cholesterol. The recent
Answer D is incorrect. An x-ray film of a left
discovery of the scavenger receptor BI has led
diaphragmatic hernia would also reveal ab-
732 Section III: Full-Length Examinations  •  Answers

dominal viscera in the thoracic cavity, which is 34. The correct answer is D. This question de-
not reported in this scenario. scribes the most common symptoms of mul-
tiple myeloma. The increased total protein
Answer E is incorrect. An x-ray film of a right
could be studied further by ordering a serum
diaphragmatic hernia would also reveal ab-
protein electrophoresis, which would show
dominal viscera in the thoracic cavity, which is
increased gamma-globulin fraction. Mul-
not reported in this scenario.
tiple myeloma is a neoplastic proliferation of
33. The correct answer is B. Histidine is an im- plasma cells, which produce immunoglobu-
portant and “essential” glucogenic amino acid lins. The molecule most commonly produced
in times of intense anabolic states such as in- by the plasma cells is IgG.
fancy, growth spurts, and recovery from in- Answer A is incorrect. The inability to clear
fection; however, it is usually categorized as chylomicron molecules from the blood is the
a nonessential amino acid because it can be primary pathologic process in type I family
synthesized by the healthy adult. The eight es- dyslipidemias. However, these typically pre­
sential amino acids are leucine, lysine, isoleu- sent at a young age with increased lipids, not
cine, phenylalanine, tryptophan, methionine, increased total protein.
threonine, and valine; these must be obtained
Answer B is incorrect. The production of al-
exogenously from the diet.
bumin typically is normal in multiple my-
Answer A is incorrect. The essential fatty acids eloma.
are linoleic and linolenic acid. They cannot be
Answer C is incorrect. There can be increased
synthesized by healthy adults but are ubiqui-
clotting factors and acute-phase reactants in
tous in natural diets.
a variety of conditions, but these are not the
Answer C is incorrect. Leucine and lysine main cause of increased total protein in mul-
are essential ketogenic amino acids; they can- tiple myeloma.
not be synthesized by the healthy adult. Glu-
Answer E is incorrect. IgM molecules are pro-
cogenic amino acids can be converted into
duced in Waldenström macroglobulinemia,
Full-Length Exams

glucose through gluconeogenesis. Ketogenic


a condition related to multiple myeloma.
amino acids can only be converted to ketone
Waldenström typically presents with hyper-
bodies (not glucose) through ketogenesis. Both
viscosity syndrome, adenopathy, and hepato-
of these processes occur in the liver.
splenomegaly. Hypercalcemia, lytic lesions,
Answer D is incorrect. Folic acid is produced and renal insufficiency are much less common
by symbiotic bacteria from the precursor in this condition.
p-aminobenzoic acid. This production is in-
hibited by sulfa antibiotics. Folic acid cannot 35. The correct answer is D. This patient most
be produced through cellular metabolism. Fo- likely now has AIDS and progressive multifo-
lic acid is also obtained from leafy vegetables cal leukoencephalopathy (PML), an opportu-
and cereal. Supplementation is recommended nistic infection that affects the central nervous
for pregnant women to prevent congenital system (CNS). PML is the result of the reac-
neural tube defects. tivation of latent JC papovavirus, usually after
the patient’s CD4 count falls to less than 200/
Answer E is incorrect. The healthy adult re-
mm³. Approximately 75% of all humans have
lies on microflora in the gut to synthesize vi-
been exposed to the JC virus. PML is a fatal
tamin K. This vitamin is also found in green,
CNS disease that causes demyelination of the
Test Block 7

leafy vegetables. Patients taking warfarin or


white matter. Disease progression is subacute,
broad-spectrum antibiotics should be advised
and it is initially marked by visual field deficits,
to maintain a stable intake of these vegetables
mental status changes, and weakness. The dis-
from day to day, in order to prevent bleeding
ease progresses to blindness, dementia, coma,
complications.
and death, typically within six months. CSF
Test Block 7  •  Answers 733

analysis is usually unremarkable, although the ally can be cured by resection of the primary
fact that PML does not enhance on MRI with tumor. Diagnosis is often made by fine-needle
contrast is a key feature. biopsy. The histologic finding in the image
is a psammoma body (a structure with lami-
Answer A is incorrect. Cryptococcal meningi-
nated, concentric, calcified spherules), which
tis is classically associated with the acute onset
is found in approximately 50% of papillary
of a severe headache, fever, nuchal rigidity,
adenocarcinomas of the thyroid. Psammoma
a change in mental status, focal neurologic
bodies are also found in serous papillary cyst-
signs, and high intracranial pressures with pap-
adenocarcinomas of the ovary, meningiomas,
illedema. The clinical presentation described
and malignant mesotheliomas.
is more consistent with progressive multifocal
leukoencephalopathy than cryptococcal men­ Answer A is incorrect. Anaplastic carcinoma
inigitis. is a poorly differentiated and aggressive type
of thyroid cancer with a poor prognosis. Psam-
Answer B is incorrect. Guillain-Barré syn-
moma bodies are not seen.
drome (GBS) generally causes symmetric
weakness that begins in the distal extremities Answer B is incorrect. Follicular carcinomas
and that ascends to affect the proximal ex- of the thyroid are less common but more ma-
tremities and the trunk. It is associated with lignant than papillary carcinomas of the thy-
recent infections of the upper respiratory and roid. Unlike papillary carcinoma, in which
GI tracts. The classic CSF finding is albu- hematogenous metastasis is rare, follicular car-
minocytologic dissociation, which involves in- cinomas commonly metastasize via the blood
creased protein without increased WBCs. GBS to lungs or bones. Histologically, follicular car-
is caused by immune reaction against epitopes cinomas tend to form acini or follicles lined
in Schwann cell surface membrane as a result with cells that are larger than those found in
of molecular mimicry. The most commonly normal thyroid. Psammoma bodies are not
described precipitant of GBS is Campylobacter found in follicular carcinoma.
jejuni. GBS is also seen in patients with HIV
Answer C is incorrect. Medullary carcinoma
infection.

Full-Length Exams
of the thyroid arises from the C cells of the
Answer C is incorrect. Herpes simplex virus thyroid, which produce calcitonin. It is a rare
(HSV) may cause meningitis and encephalitis cancer of the thyroid and can be associated
and is usually characterized by a necrotic tem- with multiple endocrine neoplasia types II and
poral lobe lesion. Classic symptoms include fe- III. Histologic examination of a medullary car-
ver, headache, altered mental status, olfactory cinoma would reveal sheets of tumor cells in
hallucinations, seizures, and vomiting. CSF an amyloid stroma. Psammoma bodies are not
analysis typically reveals an increased number found in these tumors.
of lymphocytes and an elevated protein level.
Answer D is incorrect. Thyroid lymphomas
The clinical course is usually acute or sub-
often arise in the setting of Hashimoto, but
acute.
would not have a psammoma body.
Answer E is incorrect. Syringomyelia, or the
enlargement of the central canal of the spinal 37. The correct answer is E. Primary oocytes en-
cord, causes bilateral sensory deficits in the up- ter meiosis I during fetal life. At birth, all oo-
per extremities in a cape-like distribution. It is cytes are arrested in prophase of meiosis I, and
generally a congenital condition, and it is not remain that way just prior to ovulation of the
associated with HIV. graafian follicle. After ovulation, the oocyte
Test Block 7

progresses through meiosis I and is arrested in


36. The correct answer is E. The patient has pap- metaphase II until after fertilization occurs, at
illary adenocarcinoma of the thyroid, which which point the egg will complete the second
accounts for approximately 80% of thyroid meiotic division, followed by mitotic growth of
carcinomas. Papillary thyroid carcinoma usu- the embryo.
734 Section III: Full-Length Examinations  •  Answers

Answer A is incorrect. Prior to ovulation, oo- Answer E is incorrect. Notifying the patient’s
cytes are arrested at prophase I, not anaphase I. parents is unnecessary and would be an unwar-
ranted breach in patient confidentiality.
Answer B is incorrect. Prior to ovulation, oo-
cytes are arrested at prophase I, not anaphase
39. The correct answer is B. This patient has the
II.
clinical stigmata of polycystic ovary syndrome
Answer C is incorrect. Prior to ovulation, oo- (PCOS). PCOS results from hormone de-
cytes are arrested at prophase I, not metaphase rangements (luteinizing hormone hypersecre-
I. tion is the hallmark of PCOS) that manifest as
obesity, hirsutism, oligomenorrhea or amen-
Answer D is incorrect. Prior to ovulation, oo-
orrhea, and acanthosis nigricans (the velvety
cytes are arrested at prophase I, not metaphase
hyperpigmentation described). PCOS often is
II. Once ovulation occurs the oocyte re-enters
associated with insulin resistance, hyperglyce-
meiosis and progresses to metaphase II. The
mia, and hyperlipidemia. Diagnosis is made
oocyte then remains in metaphase II until fer-
by ultrasound of the ovaries, which will reveal
tilization.
>10 follicles per ovary as well as bilateral ovar-
Answer F is incorrect. Prior to ovulation, oo- ian enlargement. Oral contraceptive pills of-
cytes are arrested at prophase I, not prophase ten are used to reduce the levels of circulating
II. androgens that result in the hirsutism, and to
help regulate ovulation. Another commonly
38. The correct answer is B. Patients entrust used treatment is clomiphene for women who
their physicians with highly personal infor- desire pregnancy.
mation, and a physician who discloses such
information without consent can be held li- Answer A is incorrect. Although acanthosis
able. However, there are exceptions to the le- nigricans sometimes is seen in occult visceral
gal protection of confidentiality, such as the malignancies, which are associated with hyper-
need to report certain infectious diseases or to calcemia, this patient does not exhibit any of
warn third parties known to be at risk of harm. the clinical signs or symptoms of hypercalce-
Full-Length Exams

The information presented in this question is mia (“stones, bones, groans, and moans”).
based on Tarasoff v. Regents of the University of Answer C is incorrect. Hypermagnesemia fre-
California (1976), in which a student was mur- quently is associated with hospitalized patients
dered by a patient who implied his intentions in renal failure and not with PCOS. Magne-
to his psychiatrist. The Supreme Court ruled sium levels rise as the renal failure worsens
in a rehearing that “confidentiality ends with because the only regulatory method of magne-
public peril” and that third parties must be in- sium is through renal excretion. Magnesium
formed in such cases. levels of 2-4 mEq/L are associated with nausea,
Answer A is incorrect. The physician must vomiting, and lightheadedness; higher levels
also make sure the potential victim is notified. are associated with depressed consciousness,
respiratory depression, and cardiac arrest.
Answer C is incorrect. This choice is perhaps
the most seductive but is also the most frankly Answer D is incorrect. Hyperuricemia is not
inappropriate. A physician has a legal obliga- associated with PCOS, but rather with gout.
tion to protect the public from “peril” accord- Patients with gout typically have a history of
ing to the Supreme Court of the United States, extremely painful monoarticular arthritis, hy-
regardless of the breach of confidentiality re- peruricemia, and subcortical bone cysts (to-
Test Block 7

quired to do so. phi). Definitive diagnosis is made using polar-


ized microscopy that demonstrates negatively
Answer D is incorrect. The physician must birefringent (yellow and parallel) monosodium
also make sure law enforcement officials are urate crystals from the aspirated joint fluid.
notified.
Test Block 7  •  Answers 735

This woman’s history is not consistent with induced erythroderma because of the rapid
gout or an acute gouty arthritis attack. onset after administration of the drug and lack
of skin exfoliation. This reaction bears the
40. The correct answer is A. This is a deletion unfortunate name “red man syndrome.” The
mutation. Bases 7-9 of the normal gene have histamine release can be prevented with and
been deleted in the mutant gene, resulting treated by administration of diphenhydramine,
in the subsequent loss of one amino acid. Be- a first-generation H1- receptor antagonist. Di-
cause three nucleotides were deleted, there phenhydramine is a member of the ethanol-
is no change in the reading frame. The most amine family, and as such can cause marked
common cystic fibrosis mutation, ΔF508, does sedation. These agents are also used as anti-
in fact yield gene product three nucleotides emetics.
shorter than the normal gene product.
Answer A is incorrect. Amoxicillin would not
Answer B is incorrect. A frameshift mutation cause a rash such as that described. However,
is an insertion or deletion of nucleotides that it is known to produce a generalized rash in
results in a misreading of all codons down- patients who are infected with the Epstein-Barr
stream. Deletions or insertions in multiples of virus.
three do not cause a shift in the reading frame.
Answer B is incorrect. Chloramphenicol is
Answer C is incorrect. An insertion mutation most often used to treat meningitis. Adverse
is an addition of one or more nucleotides to effects include aplastic anemia and gray baby
the DNA. syndrome.
Answer D is incorrect. A missense mutation Answer C is incorrect. Clindamycin is used
occurs when a point mutation causes one to treat anaerobic infections. Its most com-
amino acid in a protein to be replaced by a dif- mon adverse effect is pseudomembranous coli-
ferent amino acid. tis. Antibiotic-induced enterocolitis is often
Answer E is incorrect. A nonsense mutation treated with vancomycin.
occurs when a point mutation results in an Answer D is incorrect. Metronidazole is used

Full-Length Exams
early stop codon. This type of mutation causes to treat infection with anaerobic organisms
a truncated protein. and protozoa. It can cause a disulfiram-like
Answer F is incorrect. A silent mutation oc- reaction when taken with alcohol. However,
curs when a point mutation does not change it does not typically cause the hypersensitivity
the amino acid sequence of the protein. The reaction seen in this patient.
point mutation is often in the third position of
42. The correct answer is B. A person is said to be
the codon.
in status epilepticus when seizure activity has
Answer G is incorrect. A transition is a mu- continued for more than 30 minutes without
tation in which a nucleotide is replaced by regaining consciousness between episodes.
another nucleotide of the same type (ie, pu- The drug of choice for the treatment of status
rine for purine, or pyrimidine for pyrimidine). epilepticus is the benzodiazepine diazepam,
Pu­rine-for-pyrimidine and pyrimidine-for-purine due to its short duration of action. Benzodiaz-
substitutions are called transversions. epines act by increasing the opening frequency
of the chloride channel associated with the
41. The correct answer is E. Rapid administration γ-aminobutyric acid (GABA) receptor, which
of vancomycin can cause an anaphylactoid re- inhibits further neuronal firing. When admin-
Test Block 7

action mediated by IgE that leads to histamine istered intravenously, diazepam’s onset of ac-
release, causing redness of the face, neck, tion is virtually immediate. Of note, diazepam
upper body, back, and arms as well as tachy- is often more readily available in the pre-hospi-
cardia, hypotension, and nausea. The rash tal setting, as the drug does not require refrig-
produced is distinct from other types of drug- eration and can be administered either intrave-
736 Section III: Full-Length Examinations  •  Answers

nously or per rectum. Generally, in a hospital Answer A is incorrect. Ceftriaxone is not used
setting lorazepam is the initial drug of choice to treat Staphylococcus aureus endocarditis.
for antiseizure therapy.
Answer B is incorrect. Ciprofloxacin is not
Answer A is incorrect. Carbamazepine is ef- used to treat Staphylococcus aureus endocardi-
fective for the treatment of partial and gener- tis.
alized tonic-clonic seizures; however, it is not
Answer C is incorrect. Erythromycin is not
used for the treatment of status epilepticus.
used to treat Staphylococcus aureus endocardi-
Carbamazepine blocks repetitive activation of
tis.
sodium channels.
Answer D is incorrect. Gentamicin may be
Answer C is incorrect. Ethosuximide is pri-
used in combination with vancomycin to treat
marily used in the treatment of absence sei-
Staphylococcus aureus endocarditis, but not
zures and is not effective in the treatment of
alone.
status epilepticus. Ethosuximide blocks low-
threshold T-type calcium channels. 44. The correct answer is D. Heparin is used ther-
Answer D is incorrect. Gabapentin, an ana- apeutically for thromboembolism prophylaxis
log of the neurotransmitter GABA, is used for and treatment. Heparin also can be flushed
the treatment of simple or complex partial sei- through an intravenous line (IV) that has been
zures, but is not used to treat status epilepticus. disconnected temporarily from its source to
Gabapentin blocks H-current modulators. prevent clogging and obstruction of the tubing;
such an IV may be referred to as “hep-locked.”
Answer E is incorrect. Phenobarbital is used
Heparin primarily acts on the intrinsic path-
as a second-line agent to treat simple partial
way of coagulation, accelerating the action of
and generalized tonic-clonic seizures, and can
anti-thrombin III, which inactivates thrombin
also be used for febrile seizures in children.
and factors IXa, Xa, XIa, and XIIa. Therefore,
Phenobarbital is a very effective anticonvul-
intentionally or contaminated heparinized
sant, but is not first-line treatment for status
blood will show a delayed partial thromboplas-
epilepticus because it requires a longer time to
Full-Length Exams

tin time (PTT).


administer, and is associated with a higher in-
cidence of respiratory depression compared to Answer A is incorrect. Heparin primarily acts
benzodiazepines. Phenobarbital increases the on the intrinsic pathway of coagulation, accel-
duration of chloride channel opening. erating the action of anti-thrombin III, which
inactivates thrombin and factors IXa, Xa, XIa,
43. The correct answer is E. Staphylococcus au- and XIIa. Therefore, heparin prolongs, rather
reus causes acute bacterial endocarditis, which than shortens, the PTT.
presents with an acute onset of high fever,
Answer B is incorrect. The International Nor-
other flu-like symptoms, new-onset murmur,
malized Ratio (INR) is a standardized measure
petechiae, Roth’s spots (white spots on the
of PT adjusted for the particular assay type and
retina formed by microemboli), Janeway le-
the machine used to measure the PT. INR is
sions (painless macules on the palms or soles),
increased if PT is increased. Because heparin
Osler’s nodes (small, painful nodules on the
exerts little effect on the extrinsic pathway, the
pads of the fingers or toes), and nail bed (sub-
INR may not be changed significantly by the
ungual) hemorrhages. The bacteria can attack
presence of heparin.
healthy valves and result in vegetations that are
much larger than those of subacute bacterial Answer C is incorrect. Heparin facilitates in-
Test Block 7

endocarditis. Vancomycin is typically used to activation of intrinsic and common pathway


treat S aureus endocarditis, and provides cov- factors, measured by PTT, more than extrinsic
erage in the case of methicillin-resistant S au- factors, which are measured by PT. Thus PT
reus. is less affected than the PTT in clotting studies
of heparin-containing blood samples. In suf-
Test Block 7  •  Answers 737

ficiently high doses, heparin prolongs, rather Answer C is incorrect. Reaction formation has
than shortens, the PT. been called unconscious hypocrisy. The per-
son adopts opposite attitudes to unconscious
Answer E is incorrect. The INR is a standard-
emotions. A burnt out doctor who dislikes
ized measure of PT adjusted for the particular
medicine may spend his time encouraging
assay type and the machine used to measure
medical students in their professional develop-
the PT. INR is increased if PT is increased.
ment.
Because heparin exerts little effect on the ex-
trinsic pathway, the INR may not be changed Answer E is incorrect. Sublimation is a ma-
significantly by the presence of heparin. ture defense mechanism that involves the ex-
pression of uncomfortable emotions (anger) in
Answer F is incorrect. Heparin facilitates in-
a socially useful way, such as becoming an ad-
activation of factor Xa and thrombin, both
vocate for social justice. In reaction formation,
of which are part of the common pathway of
the person unconsciously replaces the uncom-
coagulation. Thus prothrombin time (PT)
fortable emotion with actions that are opposite
may be prolonged in blood samples contain-
his/her true emotion.
ing heparin. However, the PT prolongation is
minimal and may even be normal compared
46. The correct answer is A. This patient is pre-
to prolongation of the PTT. PTT is influenced
senting with phencyclidine (PCP) intoxica-
by intrinsic pathway factors, which makes it
tion. Patients often display assaultive behavior,
the preferred marker of heparin anticoagulant
belligerence, psychosis, violence, impulsive-
activity.
ness, psychomotor agitation, fever, tachycar-
dia, vertical and horizontal nystagmus, hyper-
45. The correct answer is D. Splitting is a defense
tension, impaired judgment, ataxia, seizures,
mechanism associated with borderline person-
and delirium. The most dangerous aspect of
ality disorder (BPD). Patients with BPD often
PCP use is the reckless behavior it precipitates.
engage in self-destructive behaviors such as
The treatment is typically benzodiazepines
cutting or burning themselves with cigarettes.
or haloperidol for severe symptoms. Reassur-
They have extremely labile moods and un-

Full-Length Exams
ance is also important in calming the patient.
stable relationships. Splitting involves the ei-
ther/or categorization of things as all good or Answer B is incorrect. Flumazenil is the an-
all bad. The patient might state that the male tidote to benzodiazepine overdose. Benzodiaz-
resident is the only doctor who has ever un- epines have an effect on the body quite similar
derstood her, and that all the other doctors are to alcohol, and overdose can cause amnesia,
ignorant. Or she might view the nursing staff ataxia, somnolence, and mild respiratory de-
as wonderful and the medical staff as abomi- pression. Flumazenil reverses the effects of
nable. benzodiazepines by competitive inhibition
at the benzodiazepine-binding site on the
Answer A is incorrect. Denial involves the
GABAA receptor.
refusal to accept a reality that a person deems
unsettling. For instance, a person may write Answer C is incorrect. N-acetylcysteine is
off persistent negative job performance reviews used as an antidote to acetaminophen over-
rather than accepting that his work abilities are dose. Symptoms of acetaminophen overdose
deficient. include abdominal pain, nausea, vomiting,
convulsions, diarrhea, jaundice, and occasion-
Answer B is incorrect. Humor is a mature
ally convulsions and coma. Excessive use can
defense mechanism, which involves express-
Test Block 7

damage multiple organs, especially the liver


ing uncomfortable emotions in a comfortable
and kidney. In both organs, toxicity is not from
way (ie, as funny): an overweight patient might
the drug itself, but from one of its metabolites,
make jokes about “fat people.”
N-acetyl-p-benzoquinoneimine (NAPQI).
In the liver, the cytochrome P450 enzymes
738 Section III: Full-Length Examinations  •  Answers

CYP2E1 and CYP3A4 are primarily respon- disease is confirmed by laboratory tests that de-
sible for the conversion of acetaminophen to tect the presence of the bacterium Legionella
NAPQI. In the kidney, cyclooxygenases are pneumophila. The bacterium Gram stains
the principal route by which acetaminophen is poorly, so silver stain is used to visualize the
converted to NAPQI. Overdose leads to the ac- rods. Transmission does not occur from person
cumulation of NAPQI, which undergoes conju- to person; rather, it occurs by aerosol transmis-
gation with glutathione. Conjugation depletes sion from an environmental water source such
glutathione, a natural antioxidant. This, in com- as an air conditioner.are most often found on
bination with direct cellular injury by NAPQI, the upper pole of the testes adjacent to the tu-
leads to cell damage and death. Acetaminophen nica vaginalis.
overdose is one of the most common causes
Answer A is incorrect. Aspergillosis is caused
of acute liver failure. If untreated, overdose
by the fungus Aspergillus. Symptoms include
can lead to liver failure and death within days.
fever and a productive cough. Invasive infec-
Treatment is aimed at removing acetamino-
tion often occurs in an immunocompromised
phen from the body and replacing glutathione
host. Aspergilloma (fungus ball) often oc-
stores. Activated charcoal can be used to de-
curs in a TB cavity. Silver stain of Aspergillus
crease absorption of paracetamol if the patient
would show septate hyphae that branch into a
presents for treatment soon after the overdose.
V shape.
While the antidote, N-acetyl­cysteine, acts as a
precursor for glutathione and helps the body re- Answer B is incorrect. Brucellosis is caused
generate enough to prevent damage to the liver, by bacteria of the genus Brucella. Humans be-
a liver transplant is often required if damage to come infected by coming in contact with ani-
the liver becomes severe. mals or animal products that are contaminated
with these bacteria. Brucellosis can cause a
Answer D is incorrect. Naloxone is a remedy
range of symptoms that may include fever,
for opioid intoxication (heroin). Patients with
sweats, headaches, back pains, and physical
opioid intoxication present with euphoria that
weakness. Severe infections of the CNS or of
often progresses to apathy, CNS depression,
the endocardium may occur.
Full-Length Exams

constipation, papillary constriction, and respi-


ratory depression that can be life-threatening Answer D is incorrect. Mycoplasma pneumo-
in overdose. Naloxone/naltrexone block opioid nia is a common, mild pneumonia that usually
receptors and reverse the depressant effects. affects people <40 years old. Its symptoms are
very similar to those of Legionnaire’s disease,
Answer E is incorrect. Sodium bicarbonate is
with the exception of diarrhea. Additionally,
used to treat TCA overdose, and it helps narrow
silver stain would not be used to diagnose My-
the QRS complex, which is widened in this tox-
coplasma pneumoniae infection. Instead, se-
icity. Remember the 3 Cs: Coma, Convulsions,
rology, cold agglutinins, or polymerase chain
and Cardiac toxicity, for TCA overdose. Diaze-
reaction testing for mycoplasma would be in
pam or lorazepam should be administered if the
order.
patient is seizing, and there should be cardiac
monitoring for arrhythmias. Answer E is incorrect. Pneumocystis jiroveci
(formerly carinii) pneumonia presents with
47. The correct answer is C. Legionnaire’s dis- fever, shortness of breath, and nonproductive
ease, which is also known as legionellosis, is a cough. It targets mainly immunocompromised
form of pneumonia. Most infections with this patients (eg, those with AIDS), and it is identi-
disease occur in elderly people. Symptoms fied by immunofluorescent staining of sputum
Test Block 7

include fever, chills, and a nonproductive or lavage fluid. Silver stain is also used, but
cough. Other symptoms include muscle aches, would show cysts, not the rod-shaped bacteria
headache, malaise, fatigue, shortness of breath, seen in this image.
chest pain, diarrhea, and ataxia. Legionnaire’s
Test Block 7  •  Answers 739

48. The correct answer is B. The classic histologic Answer A is incorrect. An endometrioid tu-
finding in a granulosa cell tumor of the ovary is mor, as the name suggests, histologically re-
the presence of Call-Exner bodies, which are sembles endometrium. It does not have Call-
follicles with granulosa cells haphazardly ar- Exner bodies on histologic examination, and
ranged around a space containing eosinophilic because it does not secrete estrogen, it would
secretions. The clinical presentation also sup- not present with vaginal bleeding and a thick-
ports this diagnosis. About two-thirds of ovar- ened endometrial stripe.
ian granulosa cell tumors occur in postmeno-
Answer C is incorrect. Krukenberg tumors
pausal women (average age at menopause in
are tumors that are metastatic to the ovaries
the United States is 51 years). Granulosa cell
from the GI system, most commonly the stom-
tumors are usually estrogen-secreting, but oc-
ach. The classic histologic finding is a mucin-
casionally produce androgens leading to viril-
secreting signet-ring cell, not a Call-Exner
ization. Estrogen-secreting tumors in children
body.
and adolescents can cause precocious puberty.
In adults they are often associated with endo- Answer D is incorrect. Call-Exner bodies
metrial hyperplasia, cystic breast disease, and would not be found in serous cystadenocarci-
endometrial carcinoma. Up to 15% of patients noma. Instead, one would expect to see a tu-
with estrogen-secreting tumors develop an en- mor lined with epithelium resembling that
dometrial carcinoma. Evidence of increased of the fallopian tube and psammoma bodies
endometrial thickness (revealing hyperpla- (concentric rings of calcification). These types
sia or carcinoma) is found in this patient as a of tumors are very common and account for
thickened endometrial stripe on ultrasonogra- >50% of ovarian carcinomas. Because they
phy. The endometrial stripe is an area of the do not secrete estrogen, they do not classically
endometrium with differing echogenicity, present with vaginal bleeding and would not
allowing the thickness to be measured. This cause a thickened endometrial stripe.
increased risk of endometrial hyperplasia Answer E is incorrect. A teratoma contains tis-
and carcinoma is due to the unopposed es- sue derived from at least two different embry-
trogen secretion by the tumor, which stimu-

Full-Length Exams
onic layers. For example, thyroid tissue, neural
lates growth of the endometrium. This abnor- tissue, muscle tissue, bone, and even teeth may
mal endometrial growth leads to subsequent be present. Immature teratomas are more ag-
sloughing off, resulting in abnormal vaginal gressive and are always malignant, while ma-
bleeding, a common presentation of endome- ture teratomas are well differentiated and be-
trial abnormalities. nign. One would not expect to see Call-Exner
bodies or vaginal bleeding with a teratoma.

Test Block 7
This page intentionally left blank
Common Laboratory Values

* = Included in the Biochemical Profile (SMA-12)

Blood, Plasma, Serum Reference Range SI Reference Intervals

*Alanine aminotransferase (ALT, GPT at 30°C) 8–20 U/L 8­–20 U/L


Amylase, serum 25–125 U/L 25–125 U/L
*Aspartate aminotransferase (AST, GOT at 8–20 U/L 8–20 U/L
30°C)
Bilirubin, serum (adult)
Total // Direct 0.1–1.0 mg/dL // 0.0–0.3 mg/dL 2–17 µmol/L // 0–5 µmol/L
*Calcium, serum (Total) 8.4–10.2 mg/dL 2.1–2.8 mmol/L
*Cholesterol, serum 140–250 mg/dL 3.6–6.5 mmol/L
*Creatinine, serum (Total) 0.6–1.2 mg/dL 53–106 µmol/L
Electrolytes, serum
Sodium 135–147 mEq/L 135–147 mmol/L
Chloride 95–105 mEq/L 95–105 mmol/L
* Potassium 3.5–5.0 mEq/L 3.5–5.0 mmol/L
Bicarbonate 22–28 mEq/L 22–28 mmol/L
Gases, arterial blood (room air)
PO 2 75–105 mmHg 10.0–14.0 kPa
PCO2 33–44 mmHg 4.4–5.9 kPa
pH 7.35–7.45 [H+] 36–44 nmol/L
*Glucose, serum Fasting: 70–110 mg/dL 3.8–6.1 mmol/L
2-h postprandial: < 120 mg/dL < 6.6 mmol/L
Growth hormone - arginine stimulation Fasting: < 5 ng/mL < 5 µg/L
provocative stimuli: > 7 ng/mL > 7 µg/L
Osmolality, serum 275–295 mOsm/kg 275–295 mOsm/kg
*Phosphatase (alkaline), serum (p-NPP at 30°C) 20–70 U/L 20–70 U/L
*Phosphorus (inorganic), serum 3.0–4.5 mg/dL 1.0–1.5 mmol/L
*Proteins, serum
Total (recumbent) 6.0–7.8 g/dL 60–78 g/L
Albumin 3.5–5.5 g/dL 35–55 g/L
Globulins 2.3–3.5 g/dL 23–35 g/L
*Urea nitrogen, serum (BUN) 7–18 mg/dL 1.2–3.0 mmol urea/L
*Uric acid, serum 3.0–8.2 mg/dL 0.18–0.48 mmol/L
(continues)

741
742 Appendix: Common Laboratory Values
Appendix

Cerebrospinal Fluid

Glucose 40–70 mg/dL 2.2–3.9 mmol/L


Hematologic

Erythrocyte count Male: 4.3–5.9 million/mm3 4.3–5.9 × 1012/L


Female: 3.5–5.5 million/mm3 3.5–5.5 × 1012/L
Hematocrit Male: 41–53% 0.41–0.53
Female: 36–46% 0.36–0.46
Hemoglobin, blood Male: 13.5–17.5 g/dL 2.09–2.71 mmol/L
Female: 12.0–16.0 g/dL 1.86–2.48 mmol/L
Hemoglobin, plasma 1–4 mg/dL 0.16–0.62 µmol/L
Common Laboratory Values

Leukocyte count and differential


  Leukocyte count 4500–11,000/mm3 4.5–11.0 × 109/L
  Segmented neutrophils 54–62% 0.54–0.62
  Band forms 3–5% 0.03–0.05
  Eosinophils 1–3% 0.01–0.03
  Basophils 0–0.75% 0–0.0075
  Lymphocytes 25–33% 0.25–0.33
  Monocytes 3–7% 0.03–0.07
Mean corpuscular hemoglobin 25.4–34.6 pg/cell 0.39–0.54 fmol/cell
Platelet count 150,000–400,000/mm3 150–400 × 109/L
Prothrombin time 11–15 seconds 11–15 seconds
Reticulocyte count 0.5–1.5% of red cells 0.005–0.015
Sedimentation rate, erythrocyte Male: 0–15 mm/h 0–15 mm/h
(Westergren) Female: 0–20 mm/h 0–20 mm/h
Proteins, total < 150 mg/24 h < 0.15 g/24 h
Index

A Adenosine, 166, 183, 616 Aldosterone, 351–352, 393, 400,


deaminase (ADA) 696
Abciximab, 282, 303 deficiency of, 43–44 Alkaline phosphatase, 592
Abdominal adhesions, 239, 257
inhibition of, 35 Alkaptonuria, 18, 30–31, 44, 50,
Abetalipoproteinemia, 116, 289
Adenovirus, 320, 640, 657, 683 605, 621
ABO incompatibility, 629
Adenylate cyclase, 232 Allergic rhinitis, 456, 472
Abruptio placentae, 421, 425,
Adipocytes, 495, 513 Allograft rejection, 566, 581–582
439, 446
Adrenal cortex, 437, 693 Allopurinol, 303, 309, 321, 326
Acanthocytes, 289
Adrenal gland, 206, 226 All-trans retinoic acid, 143, 149,
Acanthosis nigricans, 284, 734
Adrenal insufficiency, 198, 212 291, 690
Acarbose, 617
Adrenal medulla, 392, 693 α-fetoprotein (AFP), elevated,
Acebutolol, 469
Adrenocortical adenoma, 44 538, 560
Acetaminophen, 357, 598
Adrenocorticotropic hormone Alport’s syndrome, 134, 324–325,
Acetazolamide, 389, 408, 413,
(ACTH), 215–216, 221, 398–399, 408, 413, 520,
664, 708, 720
Acetylcholine, 250, 682, 685 225, 230–231, 351–352, 614, 662, 719
N-Acetylcysteine, 155, 370, 699, 397, 588–589 Aluminum hydroxide, 263
737–738 ectopic production of, 231, Alzheimer disease, 344, 349, 356,
Achalasia, 235, 250, 618–619 481–482 357, 553, 627, 654, 658
Achondroplasia, 313, 329 Aedes mosquito, 292 Amantadine, 86, 577, 656
Acinetobacter baumannii, 496, Afferent arteriole, 380, 392 Amebiasis, 601, 613–614
514 Aflatoxin B, 139 Aminocaproic acid, 155, 279, 298
Acne vulgaris, 324 Agranulocytosis, 10 Aminolevulinate dehydratase,
Acromegaly, 202, 218 AIDS, 116, 128, 137, 142, 148, 595
Actinic keratosis, 284, 582 200, 214, 358, 536, 554– Aminolevulinate synthase, 596
Actinomyces, 557 555, 640, 658, 714, 732– Amiodarone, 154, 497, 516
israelii, 100, 316, 334 733 Amitriptyline, 362, 367, 375, 376
Acute intermittent porphyria Albinism, 26, 46 Amlodipine, 405
(AIP), 273, 286–287, 574, Albright’s hereditary osteodystro- Ammonia poisoning, 693
595 phy, type Ia, 221 Amniotic fluid embolism, 446–
Acute tubular necrosis (ATN), Albuterol, 192, 463, 465, 478, 447
386, 389, 404, 406, 409, 479, 483 Amoxicillin, 79, 95, 735
585 Alcohol Amphetamine
Acyclovir, 83, 86, 146, 155, 191, abuse, 375 intoxication, 11
446, 568, 583–584, 611, intoxication, 11 overdose, 384, 399
632 withdrawal, 364, 369–370 Amphotericin B, 214, 334, 441
Addison disease, 203, 217, 218, Aldolase B, deficiency of, 28, 49– Ampicillin, 688
221, 535, 552 50 Amyloidosis, 107, 119, 178, 301,
Adenoid cystic carcinoma (ACC), Aldose B, 38 404, 520, 621
265–266 Aldose reductase, 35 renal, 621

743
Amyotrophic lateral sclerosis Ankylosing spondylitis, 242, 264, Appendix epididymis, 631
(ALS), 341, 351, 626 317, 325, 332, 336, 541 Arachnoid granulations, 124, 131
Analgesic nephropathy, 404 Anopheles mosquito, 292 Arcuate fasciculus, 354
Anaphylactic shock, 94 Anorectal cancer, 129, 138 Arginine, 374
Anaphylaxis, 103, 110–111, 268, Anorexia nervosa, 347, 361–362, Argyll Robertson pupil, 577
456, 472 367, 375, 569, 587 Arsenic poisoning, 653
Androgen insensitivity, 434, 448, Anorgasmia, 10 Arteriolosclerosis, hyaline, 212
639, 656 Anterior cerebral artery (ACA), Arteriovenous malformation
Androgenic steroids, 422, 439 204, 223, 340, 342, 348, (AVM), 697
Anemia, 654 352 Arthritis
aplastic, 132, 298, 622, 722 Anterior cruciate ligament acute gouty, 208, 229
of chronic disease, 277, 295, (ACL), 313, 330 Lyme, 325, 577
300, 530, 543 Anterior inferior cerebellar artery psoriatic, 325, 334, 725
Cooley, 305 (AICA), 204, 223 reactive, 311, 325
hemolytic, 133, 297, 300, 492, Anthracosis, 133, 476 rheumatoid, 105, 109–110,
507 Anthrax, inhalation, 464, 484 264, 314, 317, 321, 329,
autoimmune, 295 Anticholinergics, 615, 663 331, 334, 336, 337, 640,
IgG-mediated, 622 Anticipation, 135 657, 695
IgM-mediated, 605, 622 Antidepressants, tricyclic, 663, juvenile (JRA), 105, 115
iron deficiency, 113, 251, 275, 722–723 Arthus reaction, 336
288, 291, 295, 297, 435, Antidiuretic hormone (ADH), Arylsulfatase A, 603, 618
684 201, 215, 224, 297, 351, Asbestos exposure, 130, 139, 294,
macrocytic, 27, 48, 112, 361, 357–358, 382–393, 397 459, 476
635, 648 ectopic production, 554 Asbestosis, 125, 133, 459, 476,
megaloblastic, 60, 112, 200, Antigenic drift, 87 574, 595
215, 259, 295, 300, 606– Antigenic shift, 74, 87 Ascaris lumbricoides, 252
607, 624 Anti-Jo-1 antibody, 318, 337 Ascites, 235, 249–250
microcytic, 61, 113, 289, 291, Antineutrophil cytoplasmic anti- Ascorbic acid (vitamin C), 226,
300, 361 bodies (ANCA), 336, 559 480, 652, 687, 691
normocytic, 361 Antinuclear antibodies, 336 deficiency, 61, 652, 663, 687,
pernicious, 22, 37, 132, 200, Antipsychotics, 217 691
215, 259, 291, 544, 574, Antisocial personality disorder, Aspartate aminotransferase
594 657 (AST), 189
sideroblastic, 295–296 Antistreptolysin O, 391, 401, Aspergillosis, 103, 191, 738
Anencephaly, 67, 400, 656 411–412 Aspergillus, 109, 139, 555, 696,
Angelman syndrome, 127, 135 α1-Antitrypsin deficiency, 20, 32– 738
Angina, 171, 192–193 33, 137–138, 432, 468, flavus, 506
Prinzmetal’s, 499, 518 572, 590 fumigatus, 450, 604, 619
unstable, 519 Aortic aneurysm, 577 Aspirin, 153, 263, 498, 506–507,
Angioedema, 385, 402–403 Aortic arches, 169, 189 518
Angiomyolipoma, 550 Aortic dissection, 172, 194, 465, overdose, 498, 518
Angioplasty, 395 548, 610, 630 Asterixis, 250
Angiosarcoma, 512 Aortic stenosis, 97, 170, 190 Asthma, 23, 39, 463, 470, 483,
Angiotensin II, 413, 566, 580 Aortitis, syphilitic, 166, 182 516, 580, 636–637, 651
receptor blockers, 168, 186, Apgar score, 573, 594 Astrocytes, 353
387, 405 Apolipoprotein C-II, 731 Astrocytomas, 41, 551
Angiotensin-converting enzyme Apolipoprotein E deficiency, 40, Ataxia-telangiectasia, 108, 113–
(ACE) inhibitors, 150, 72 114, 118, 120
187, 382, 395, 722 Appendicitis, 240, 248, 254, 260, Atelectasis, 483
Anion gap acidosis, 407 582 Atenolol, 469, 555

744
Atheroma, 496, 514 Beck’s triad, 187, 476 transitional cell carcinoma of,
Atherosclerosis, 149, 180–181, Bedwetting (enuresis), 229 284
182, 697 Behavioral therapy, 708–709, Bladder exstrophy, 63
Atherosclerotic plaque, 24, 40 722 Blastomyces, 555–556, 718
ATP depletion, 33 Bell palsy, 328, 343, 354–355, dermatitidis, 72, 83
Atrial fibrillation, 491, 506 577, 635, 647, 707, 719 Blastomycosis, 718
chronic, 145, 153 Beneficence, 661 Bleeding time, 286, 498, 517,
Atrial septal defect (ASD), 66, Benign prostatic hypertrophy 571, 590
188 (BPH), 144, 151, 206, Bleomycin, 151, 277, 296, 299
Atrioventricular (AV) block, 170, 227, 426, 447–448, 508, Boerhaave syndrome, 248
190–191 537, 557 Bohr effect, 42
Mobitz type I (Wenckebach), Benzo(a)pyrene, 701 Bone, metastatic disease to, 721
170, 190–191, 194 Benzodiazepines, 493, 509, 615, Borderline personality disorder,
Mobitz type II, 172, 194 716, 737 657, 716, 737
Atropine, 174, 580 intoxication, 368, 376–377 Bordet-Gengou medium, 75, 88,
Attention deficit/hyperactivity dis- Benztropine, 365, 369, 371, 375 94, 473
order (ADHD), 367, 375 Berger disease, 394, 398, 401, Bordetella pertussis, 75, 88, 93,
Auer rods, 289, 444, 591, 629, 405, 413, 471, 579, 620 94, 473, 479, 659
655 Beriberi, 61 Borrelia burgdorferi, 84, 86, 89,
Autoantibodies, 318, 337 Bernard-Soulier disease, 288, 118, 292, 325, 332, 523–
Autonomy, patient, 642, 661 304, 590 524, 577
Avoidant personality disorder, Berry aneurysms, 24, 41, 182, IgA antibodies to, 336–337
371, 657 310, 324, 348, 442 Bowel
Axillary nerve injury, 359 β1 agonists, 39, 503 ischemic, 673, 690
Azathioprine, 479, 484 β-blockers, 142, 148, 150, 192, obstruction, 254
Azithromycin, 403, 425, 446, 722 195, 465 Bowing fracture, 325
Aztreonam, 403 β1 blockers, 39, 186 Bowman space, 499, 519
nonselective, 454, 469 “Boxer’s” fracture, 318, 337
B β2 agonists, 23, 39 Bradykinin, 402–403
Bevacizumab, 438 Brain natriuretic peptide, 189
Babesia microti, 89, 292, 333 Bicarbonate, 146, 154–155, 215, Branchial arches, 57, 67
Babesiosis, 168–169, 188, 292 256, 399, 464, 485, 522, Branchial cleft, 67
Babinski’s sign, 430 630, 738 cyst of, 59
Bacillus Biceps muscle, 333 Branchial pouches, 67–68
anthracis, 94, 261, 333, 464, Bicuspid aortic valves, congeni- BRCA gene, 650
484, 557 tal, 65 Breast
cereus, 76, 91, 267 Bile acid sequestrants, 152, 552 cancer, 276, 289, 291, 428,
Bacterial vaginosis, 96 Biliary atresia, extrahepatic, 58, 567, 583
Bacteroides fragilis, 557 68 genetic testing for, 636, 650
Barbiturates, 372, 493, 509 Bipolar disorder, 376 infiltrating lobular carci-
Barrett esophagus, 264–265, 269, II, 4–5, 10 noma, 505
273, 285 Bismuth, 263, 285, 673, 688 invasive ductal carcinoma,
Bartonella henselae, 501, 523 2,3-Bisphosphoglycerate (BPG), 505
Bartter syndrome, 391, 413, 643, 24, 42, 503 Paget disease, 505
664 Bisphosphonates, 305, 708, 721– risk of, 416, 428–429
Basal cell carcinoma, 272, 284, 722 fibroadenomas of, 491, 505
443, 566–567, 582 Bite cells, 294 fibrocystic changes of, 426, 448
bcr-abl hybrid gene, 701 Bitemporal hemianopia, 54, 59 intraductal papillomas of, 505
Becker muscular dystrophy Bladder Bretylium, 183, 372
(BMD), 12 adenocarcinoma of, 56, 63 Broca’s aphasia, 712, 729

745
Bromocriptine, 357, 656 Captopril, 13, 177, 730 posterior (PCA), 204, 223, 567,
Bronchiectasis, 463, 483 interaction with sildenafil, 13 582–583
Bronchitis, chronic, 455, 469– Carbamazepine, 10, 212, 369, Cerebral hemorrhage, 642, 661–
470 560, 612, 736 662
Bronchoalveolar lavage, 577 Carbidopa, 357 Cerebrospinal fluid (CSF), ob-
Brucella, 484–485, 738 Carbohydrate, 30 struction of, 131
Brucellosis, 484–485, 738 Carbon monoxide poisoning, 693 Cerebrovascular accident (CVA),
Brugia malayi, 683 Carcinoid syndrome, 199, 213, 279, 298
Brugada syndrome, 84 482, 500, 521 Cervical cancer, 126, 134, 419,
Brush border, 399 Carcinoid tumor, 432, 592, 667 433
Bruton agammaglobulinemia, Cardiac arrhythmias, 172, 194 Chagas disease, 84, 178, 262,
106, 109, 116, 506, 589 Cardiac catheterization, 185 292, 393, 538, 558
Buckle fracture, 325 Cardiac glycosides, 173, 195 Chancroid, 72, 83, 425, 445–446
Budd-Chiari syndrome, 235, Cardiac murmurs, 190 Charcoal yeast extract, 88
249–250, 494, 510 Cardiac shock, 703 Charcot-Marie-Tooth disease,
Buerger disease, 114, 335 Cardiac tamponade, 168, 187, 351
Bulbourethral (Cowper) glands, 476 Charcot’s triad, 95, 257
631 Cardinal ligament, 432 Chédiak-Higashi disease, 506,
Bullous pemphigoid, 600, 613 Cardiogenic shock, 390–391, 653, 724
Bupivacaine, 642, 660 411 Chemoreceptor trigger zone
Bupropion, 347, 361–362, 367, Cardiomyopathy (CTZ), 352
375, 569, 587 dilated (DCM), 177–178, 511 Chest injury, 458, 475–476
Burkitt lymphoma, 87, 97, 274, hypertrophic, 84, 541, 727 Child abuse, 323, 575, 597, 611,
287, 298, 502, 524, 547, obstructive (HOCM), 164, 631
591, 629, 671, 685, 690 177 Chlamydia, 445
Burr cells, 289 Carotid artery, interior, 342, 352 pneumoniae, 85
Buspirone, 213, 369, 375, 587 Carotid sinus baroreceptor, 171, trachomatis, 92, 96, 421, 429,
Busulfan, 296, 303, 550 193 430, 437, 438, 515
Caspofungin, 191, 214, 545 Chloral hydrate, 9
C Cat scratch disease, 501, 523 Chloramphenicol, 86, 95, 150,
Cataplexy, 4, 9 708, 722, 735
C3a, 673, 688–689 Caudate nucleus, 354 Chlordiazepoxide, 364, 369, 370
Cabergoline, 656, 694 Cefazolin, 537, 557 Chloride, 523
Cabot ring bodies, 655 Ceftriaxone, 72, 83, 93, 95, 657– poisoning, 693
Cadmium exposure, 139 658 Chloroquine, 262
Calcaneofibular ligament, 322 Celecoxib, 386–387, 404–405, Chlorpromazine, 370, 417, 431,
Calcitonin, 200, 214, 321 666 639, 655–656
Calcium, 522, 578 Celiac sprue, 106, 116, 240, 257, Chocolate agar, 88, 94, 473
increased intracellular, 20, 33 259, 265, 334, 665, 700 Chocolate cysts, 423, 430, 442
Calcium carbonate, 242, 262– Central nervous system lym- Cholangitis, primary sclerosing,
263, 285 phoma, 85, 627–628 240, 257, 259, 265
Calcium channels, voltage-gated, Central pontine myelinolysis, Cholecalciferol (vitamin D3),
163, 175 564, 577 578
Call-Exner bodies, 444, 655, 717, Cephalosporins, 722 Cholecystitis, 79, 95, 514, 515
739 Cerebellum, 352 Cholecystokinin (CCK), 234,
Campylobacter, 92 Cerebral artery 248, 261, 285, 682
jejuni, 91, 98, 239, 258, 260, anterior (ACA), 204, 223, 340, Choledocholithiasis, 238–239,
308, 320, 357, 543, 733 342, 348, 352 256–257
Candida, 96, 137, 441, 556 middle (MCA), 204, 223, 348, Cholelithiasis, 248
albicans, 427, 429, 449, 506 583 Cholera, 236, 253

746
Cholesterol Cluster headache, 346, 360 Congo red stain, 554
LDL, 40 Coagulation studies, 666 Conjunctivitis, 311, 325
VLDL, 40 Coarctation of the aorta, 179, Conn syndrome, 202, 203, 217–
Chondrosarcoma, 319 440, 490, 503–504, 548, 218, 219, 221, 229
Choriocarcinoma, 280, 299, 300 631 Continuous positive airway pres-
Choroid plexis papilloma, 728 Cobalamin (vitamin B12), 480, sure, 374
Chromaffin cells, 226, 693 544 Conus medullaris tumor, 672,
Chronic granulomatous disease deficiency, 48, 60, 104, 112, 686
(CGD), 42, 109, 119, 120, 258–259, 275, 278, 290– Conversion disorder, 494, 510,
491, 505–506, 637, 653, 291, 295, 296, 297–298, 723
724 574, 594, 648 Cooley anemia, 305
Chronic obstructive pulmonary Cocaine, 11, 192, 194, 365, 372 Coproporphyria, hereditary, 34
disease (COPD), 459, intoxication, 11 Cor pulmonale, 590, 662
464, 476, 485, 516, 568, Coccidioides, 718 Cord compression syndrome,
584 immitis, 83, 89 672, 686
Churg-Strauss syndrome, 108, posadasii, 89 Coronary artery, 170, 190–191
120, 470, 471, 483, 532– Coccidioidomycosis, 75, 89, 595, atherosclerosis, 630
533, 548 718 right (RCA), 681, 703–704
Chvostek sign, 259, 430, 504, 666 Cohort study, 7, 14 Coronary artery disease, 500,
Cimetidine, 263–264, 591 retrospective, 8, 15 519–520
Ciprofloxacin, 242, 262–263, Colchicine, 326 Coronavirus, 98, 466
466, 626, 722 Collagen, 514, 701 Corpus callosum, 354
Circle of Willis, 204, 223, 340, synthesis, 691 Corticosteroids, 234–235, 249,
348 type I, 662 259, 326, 332
Cirrhosis, 572, 573, 590, 593 type II, 662 chronic use of, 332
micronodular, 124, 127, 132, type IV, 399, 662 Cortisol, 222
135–136 Collagenase, 321 Corynebacterium diphtheriae, 86,
primary biliary, 259, 510 Colles’ fracture, 337 88, 93, 94, 472–473
Cisplatin, 152, 484 Colorectal carcinoma, 300, 512 Countertransference, 10
Citrate, 18, 30 hereditary nonpolyposis Courvoisier sign, 46, 443
Cleft lip, 57, 68 (HNPCC), 517, 679, 701 Cowdry type A nuclear inclu-
Cleft palate, 120, 531, 546, 571, Common peroneal nerve, 315, sions, 444, 639, 655
589 332 Cowper (bulbourethral) glands,
Clindamycin, 86, 97, 192, 251, Common variable immunodefi- 631
403, 722, 735 ciency, 653 Coxsackievirus, 98, 164, 177–
Clomiphene, 676, 694 Compartment syndrome, 666 178, 657, 670, 683
Clomipramine, 212 Complement, 40, 115, 119 Cranial nerves, 312, 327, 509
Clonazepam, 376, 560 Compound X, 390, 410 III, 494–495, 509, 511, 682
Clonidine, 176 Condylomata acuminata, 129, IV, 509
Clopidogrel, 303, 507 138 V, 509, 647
Clostridium Confidence interval (CI), 608– V2, 327
botulinum, 75, 89, 93, 99, 655 609, 627 V3, 327
difficile, 93, 95, 97, 251–252, Confidentiality, breach of, 714, VI, 509, 511, 682
293, 564, 578, 594, 655 734 VII, 509, 635, 647
enterocolitis, 715, 735 Congenital adrenal hyperplasia VIII, 509
superinfection, 251–252 (CAH), 203, 204, 218– X, 327–328
perfringens, 89, 91, 348 219, 222, 656 XII, 312, 328, 509
tetani, 348, 638–639, 655 Congestive heart failure (CHF), Craniopharyngioma, 54, 59, 286
Clozapine, 10, 150, 364, 370, 149, 165, 169, 179, 188, C-reactive protein, 189
372, 607, 624, 656 670, 673 Creatine kinase-MB fraction, 189

747
CREST syndrome, 104, 113, Death, leading causes of, 7, 15 type 1, 204, 206, 222–223, 225,
121, 268, 321, 329 Deep venous thrombosis (DVT), 226, 678–679, 699–700,
Creutzfeldt-Jakob disease, 359 273, 285–286, 428, 460, 725
Crigler-Najjar syndrome, 39, 68, 477, 501, 522, 666, 721 type 2, 199, 209, 212, 231–232,
245, 249, 268, 603, 618 risk of, in women receiving 575, 593, 596–597, 602,
Crohn disease, 235, 238, 251, HRT, 428 616–617, 675, 692
252, 255, 265, 346–347, Defense mechanisms, 8, 15, 708, Diabetic ketoacidosis (DKA),
361, 538, 558, 613, 700 716, 721, 737 361, 407, 457, 474, 501,
Cromolyn, 478, 483, 636–637, Deferoxamine, 146, 155, 699 522–523, 678–679, 699–
651 Dehydroepiandrosterone, 552 700
Croup, 564, 577 Delerium tremens, 369, 376 Diabetic nephropathy, 143, 150
Cryptococcus, 467, 555, 677, 696 Deltoid muscle, 326–327 Diazepam, 357, 371, 697, 715,
neoformans, 83, 137, 358, 441, Delusional disorder, 8 735–736
450, 530, 545, 554, 619, Demeclocycline, 208, 229, 475, overdose, 147, 155
658 546 Didanosine, 148, 632
Cryptosporidium, 96, 558 Dementia, 149, 359, 553 Diethylstilbestrol (DES), 445
Cushing syndrome, 208, 218, 221, AIDS complex, 640, 658 Diffuse axonal injury, 578
230–231, 297, 299, 413, Alzheimer, 344, 349, 356, 357, Diffuse cortical necrosis, 385,
462, 481–482, 588–589 553 401
Cushing ulcer, 237, 254 frontotemporal, 654 DiGeorge syndrome, 55, 61, 68,
Cyanide poisoning, 675, 693, 699 HIV-associated, 609, 627–628 103, 109, 112, 114, 116,
Cyclooxgenase (COX)-2 inhibi- with Lewy bodies, 344, 356, 118, 119, 208, 229–230,
tors, 530, 543 359 542, 571, 589, 647, 680,
Cyclophilin, 293 multi-infarct, 553 702–703, 724
Cyclophosphamide, 51, 150, 152, Dendritic cells, 102, 110, 691 Digitalis, 192
398, 438, 458, 475, 484, Dengue virus, 684 Digoxin, 147, 157, 174, 185–186,
658 Denial, 8, 15, 737 195, 465
Cycloserine, 507 Depressed mood, 4, 10 toxicity, 185–186
Cyclosporine, 276, 293, 294, 309, Depression, 367, 376, 551, 708, Dihydrotestosterone, 437, 448
321, 479, 726 721 1,25-Dihydroxycholecalciferol,
Cystadenocarcinoma, 739 Dermatitis, contact, 472, 613 549, 578, 652, 663
Cystinuria, 50 Dermatitis herpetiformis, 582 1,25-Dihydroxyvitamin D, 43
Cystic fibrosis (CF), 33, 38, 65, Dermatomyositis, 318, 337 Diltiazem, 177, 179, 187, 516
138, 255, 329–330, 508, Dermis, 317, 335 Dimercaprol, 155, 678, 699
554 Dermoepidermal junction, 317, Dipalmitoyl phosphatidylcholine,
Cystic medial necrosis, 182 335 453, 467, 683
Cysticercosis, 74–75, 88 Dermoid cyst, 59 Diphenhydramine, 398, 615–616
Cystitis, hemorrhagic, 550 Desmolase, 225 Diphenoxylate, 253
Cytarabine, 149 Desmopressin, 229, 532, 546 Diphtheria, 85–86, 456, 472–473
Cytomegalovirus (CMV), 91, 128, Desmosomes, 310, 323 Disc herniation, 686
137, 457, 473, 657, 700 Developmental milestones, 8, 15, Displacement, 15, 721
congenital, 90 602, 617 Disseminated intravascular coag-
encephalitis, 85, 628 Dexamethasone, 464, 484, 615 ulation (DIC), 94, 133,
Diabetes insipidus (DI) 207, 229, 281, 282, 289,
central, 345, 357–358 302, 304, 385, 401–402,
D
nephrogenic, 199, 208, 212, 498, 517, 590, 622
Dactinomycin, 152, 296 229, 297, 357–358 Dissociative identity disorder,
Dantrolene, 371, 678, 697 Diabetes mellitus (DM), 541, 376
Dapsone, 262, 327, 468, 698 556 Distal convoluted tubule, 392
Daunorubicin, 149, 550, 583 gestational, 400, 436 Disulfiram, 364, 369–370

748
Diuretics, 395 Eczema, 324 Eosinophils, 702
loop, 390, 391, 399, 400, 411, Edema, of extremities, 173, 196 Ependymoma, 60
413, 695–696 Edinger-Westphal nucleus, 354 Epicondylitis, lateral (tennis
osmotic, 399, 722 Edwards syndrome (trisomy 18), elbow), 315, 333
potassium-sparing, 393, 399, 56–57, 65, 400, 440, 504 Epidermis, 317, 335
695 Efferent ductules, 440 Epidermolysis bullosa, 324
thiazide, 380, 384, 393, 399, Efficacy, drug, 156 Epididymitis, 431, 445
411, 695–696 Ehlers-Danlos syndrome, 49, 323, Epidural hemorrhage, 348
Diverticulitis, 244, 266–267 559, 628–629, 662, 691 Epiglottitis, 103, 111, 453, 467
Diverticulosis, 244, 266 Ehrlichia chaffeensis, 89–90 Epinephrine, 465, 651, 671, 685
DNA binding, 30 Ehrlichiosis, 292 Epispadias, 60
Dobutamine, 211 Eikenella corrodens, 524 Epoetin, 297
Donepezil, 345, 357 Eisenmenger syndrome, 66, 130, Epstein-Barr virus, 87, 97, 136,
Dopamine agonists, 663 139–140, 641, 662, 731 284, 287, 466, 629, 659,
Dose-response curve, 147, 156 Elastase, 50–51 679, 685, 700, 720
Down syndrome (trisomy 21), 65, Elastin, 662 Erb palsy, 346, 359–360
67, 238, 255, 422, 439– Emancipation of minors, 6, 13 Erectile dysfunction, 6, 13, 420,
440, 504, 560, 662 Emphysema, 20, 32–33, 454, 435
Doxazosin, 555 465, 468, 485, 580, 711, Ergocalciferol (vitamin D2), 578
Doxorubicin, 296, 438, 550, 583 726–727 Erythema infectiosum (fifth
Doxycycline, 86, 95, 586, 719, panacinar, 432 disease), 471, 675, 692
722–723 Enalapril, 225 Erythema migrans, 471
D2-receptor antagonism, 431 Encainide, 156, 183, 185 Erythema multiforme, 471–472
Drepanocytes, 294 Enchondroma, 319 Erythema nodosum, 310, 324,
Dressler syndrome, 172, 193 Encopresis, 5, 10 456, 471
Dubin-Johnson syndrome, 268 Endocardial cushion defect, 422, Erythema toxicum, 472
Duchenne muscular dystrophy 439–440 Erythroblastosis fetalis, 102–103,
(DMD), 12, 27, 47, 322, Endocardium, fibrosis of, 179 110
654 Endocarditis Erythromycin, 86, 276, 293–294,
Duodenal atresia, 64, 67, 400 bacterial, 495, 512, 543 722
Duodenal hematoma, 247–248 acute (ABE), 168, 186, 258, Erythropoietin, 530, 543
Duodenal ulcer, 241, 255, 261– 543, 716, 736 Escherichia coli, 84, 91, 98, 109,
262, 670, 682 subacute, 170, 191–192, 328, 348, 381, 394, 409,
Dura mater, 349 258, 543 429, 442, 445, 506, 510,
Durable power of attorney, 634, Endometrial cancer, 419, 434, 442 635, 648, 653, 655, 718
646 Endometrioid tumor, 739 enterotoxigenic, 91
Dysautonomia, familial, 224 Endometriosis, 436, 582 O157:H7, 92, 258, 543
Dyslipidemia, type I, 23, 24, 40, Endothelial cells, 399 Esmolol, 469, 555, 616
41 Entamoeba histolytica, 96, 558, Esophagitis, 694
Dystrophin, 47, 654 601, 613–614 Esophagus
Enteritis, viral, 116 Barrett, 264–265
Enterococcus, 99 cancer of, 254
E
faecalis, 186, 512 adenocarcinoma, 269, 544,
Ebola virus, 684–685 faecium, 22, 37 603, 618
E-cadherins, 701 Enterocolitis, 254 squamous cell carcinoma,
Echothiophate, 152 Enterovirus, 164, 177–178 265, 619
Echovirus, 696 Enuresis (bedwetting), 229 laceration of, 243, 264
Ectopic pregnancy, 248, 299– Enzyme-linked immunosorbent muscle function of, 244, 266
300, 416–417, 429 assay (ELISA), 32, 36, perforated, 248, 496, 513
ruptured, 567, 582 570, 587–588 Boerhaave, 496, 513

749
Esophagus (Continued) Femoral nerve, 332 buckle, 325
resection, 148 Fentanyl, 660 Colles’, 337
stricture of, 248, 265, 513–514 Ferrochelatase, 596 compression, vertebral, 337
varices of, 250, 254–255, 265 Fetal alcohol syndrome, 64 greenstick, 325
17β-Estradiol, 205, 225 Fibrates, 144, 152 hamate, 337
Estrogen, 225, 425, 432, 440, Fibrillin, 514, 662, 691 humeral, 310, 323, 346, 359
447, 449, 625, 641, 660, Fibrinogen, 687 phalanges, 337
694 increased levels of, 394 scaphoid, 337
Etanercept, 463, 482–483 Fifth disease (erythema infectio- spiral, 311, 325, 611, 631
Ethacrynic acid, 720 sum), 471, 675, 692 Fragile X syndrome, 29, 41–42,
Ethambutol, 457, 468, 473, 553, Finasteride, 206–207, 227 51, 65, 135
712, 730 Fitz-Hugh-Curtis syndrome, 496, Francisella tularensis, 485, 524
toxicity, 457, 473 515 Fresh frozen plasma (FFP), 95
Ethanol, 377 Flavivirus, 517 Friedreich’s ataxia, 26, 44–45
Ethics, 707, 720 Flecainide, 185 Fructose intolerance, 28, 49
Ethosuximide, 538, 560, 600, Flexor carpi ulnaris, 333, 360 hereditary, 38, 50
612, 736 Flexor digitorum superficialis, Furosemide, 322, 391, 405, 413,
Etoposide, 51 360 465, 546, 643, 664, 720
Ewing sarcoma, 293, 314, 319, Fluconazole, 93, 100, 334, 441,
330, 591, 673, 689–690 466, 545, 553, 584, 594, G
Exotoxin A, 89 698
Extensor carpi radialis muscle, Flucytosine, 214, 369, 441, 545 G-protein coupled receptors, 20,
315, 333 Fludarabine, 149 33–34, 605, 622–623
Extensor carpi ulnaris muscle, Flumazenil, 364, 369, 370, 377, GABA receptor agonists, 624
333 400, 580, 624, 697, 737 Gabapentin, 560, 736
External validity, 726 Fluoxetine, 13, 212, 362, 364, Galactokinase, deficiency of, 31
Ezetimibe, 265, 552, 647 369, 375, 580 Galactorrhea, 201, 217, 443
Fluphenazine, 213, 371–372, 373 Galactose-1-phosphate uridyl-
F Flutamide, 151, 227 transferase, 35, 38
Foam cells, 180–181 Galactosemia, 607, 625–626
Fab fragment, 119 Focused assessment with sonogra- α-Galactosidase A, 618
Fabry disease, 38, 49, 224, 586, phy in trauma (FAST), β-Galactosidase, 618
618 446 Gallbladder
Factitious disorder, 373, 510–511 Folate, 595 adenocarcinoma of, 236, 252
Factor II, decreased levels of, 394 deficiency, 48, 60, 104, 112, palpable, 443
Factor V deficiency, 477 295, 361, 648 Gallstones, 234, 236, 244, 248,
Factor V Leiden, 164, 178, 477 maternal, 64–65 252, 257, 265
Factor VIII, 304 Folic acid, 213, 297, 637, 651– Gametogenesis, female, 714,
deficiency (hemophilia A), 652, 732 733–734
178, 288, 298, 304, 477 deficiency, 684 γ-Glutamyl transpeptidase
Factor XIIIa, 687 Follicle-stimulating hormone (GGT), 701
Falciform ligament, 66 (FSH), 225, 352, 425, 447, Ganciclovir, 86, 632
Fallopian tubes, 432 546, 625, 641, 660 Gap junctions, 92, 323
Famciclovir, 86 Follicular lymphoma, 276, 292, Gardner syndrome, 517
Familial adenomatous polyposis 532, 547 Gardnerella vaginalis, 416, 429
(FAP), 239, 257, 517 Fomepizole, 370 Gastric cancer, 237, 253, 269,
Fanconi syndrome, 568, 585 Foscarnet, 83, 632 279, 298–299, 524–525,
Fatty acid, 30, 732 Fractures 638, 654
Fecal incontinence, 5, 10 bowing, 325 Gastrin, 215, 248, 261, 263, 272,
Femoral cutaneous nerve, 335 “boxer’s,” 318, 337 284–285, 682

750
Gastrinoma, 592 pauci-immune, 559 Granulosa cell tumor, 717, 739
Gastroduodenal artery, 208, 230 poststreptococcal, 391, 398, Granulomatous disease, chronic,
Gastroenteritis, 242, 262 401, 404, 408, 411–412, 42, 109, 119, 120
Gastroesophageal reflux disease 472, 559, 597, 621, 712, Graves disease, 198, 204, 210,
(GERD), 237, 246, 254, 729 221, 502, 524, 589, 600,
263, 269, 630 Glomerulosclerosis 613, 644, 665
Gastroparesis, 142, 148 focal segmental, 387, 404, 405 hyperthyroidism induced by,
Gastroschisis, 65 nodular (Kimmelstiel-Wilson), 200, 213
Gaucher disease, 586, 725 199, 212 Greenstick fracture, 325
Gaussian distribution, 710, 724– Glucagon, 155, 192, 223 Group therapy, 723
725 Glucagonoma, 205, 224 Growth hormone (GH), 218
Gel electrophoresis, 36 Glucocorticoids, 203, 219, 357, Guanethidine, 372
Gemfibrozil, 167, 183, 244, 265, 615 Guanylate cyclase, 232
634–635, 647 Glucokinase, 23, 39–40 Guillain-Barré syndome, 81, 92,
Genital tubercle, 621 Glucose, 39, 380, 392–393 98, 239, 258, 308, 320,
Genitofemoral nerve, 335 Glucose-6-phosphatase defi- 344, 357, 543, 602, 616,
Gentamicin, 95, 251, 514, 688, ciency, 542 686, 733
722, 736 Glucose-6-phosphate dehydro­ Gynecomastia, 545
German measles (rubella), 86, genase (G6PD) defi-
90, 700 ciency, 25, 29, 42, 50, 297, H
congenital, 628 670–671, 684
Giant-cell (temporal) arteritis, α-Glucoside inhibitors, 218 Haemophilus, 696
114, 178–179, 335, 676– α-1,4-Glucosidase deficiency, 44, aphrophilus, 186
677, 695 542 ducreyi, 83, 445
Giant cell tumor, 330 α-1,6-Glucosidase deficiency, influenzae, 84, 88, 89, 94, 111,
Giardia lamblia, 241, 262, 558 542 466, 473, 479, 480, 512
Giemsa stain, 554 GLUT 1 transporters, 42, 513 type B, 111, 510, 683
Gilbert syndrome, 39, 68, 234– GLUT 2 transporters, 42, 513 Hageman factor, 94
235, 249, 268 GLUT 4 transporters, 42, 495, Half-life, 145, 153
Glanzmann thrombocytopenia, 513 Haloperidol, 201, 217, 370, 371,
288, 590 Glyburide, 206, 225, 545, 692 372–373, 587, 698
Glargine, 617, 692 Goiter, 198, 201, 210, 216 Halothane, 660
Gleevec (imatinib), 255, 291– Golgi apparatus, 26, 45–46 Hansen disease, 327
292, 293, 294, 303 Gonadotropin-releasing hor- Hantavirus, 671, 684
Glioblastoma multiforme mone, 225, 546 Hashimoto thyroiditis, 22, 37,
(GBM), 124, 131, 286, Goodpasture syndrome, 412–413, 201, 202, 216, 217, 221–
680, 701 471, 472, 484, 520, 538, 222, 321, 597, 606, 613,
Glipizide, 692 559, 565, 579–580, 613, 623, 646
Glitazones, 218 614, 620 Headache
Globus pallidus, 343, 354 Gout, 309, 311, 321, 325–326, cluster, 346, 360
externus, 678, 699 329, 334, 658 medication overuse, 360
Glomerular basement membrane Graft rejection migraine, 360
(GBM), thickening of, acute, 587 recurrent, 491, 504–505
134 hyperacute, 587 temporomandibular joint dys-
Glomerulonephritis, 117, 212, Graft-versus-host disease function syndrome, 360
405, 585, 604, 620 (GVHD), 112, 117, 472, tension, 360
membranoproliferative, 117, 570, 587, 610, 629 Heart block, third-degree, 84
381, 394, 398 Granulocyte macrophage colony- Heart failure, 674, 691
membranous, 383, 398, 405, stimulating factor, 689, Heavy-chain disease, 301
408, 597, 614 726 Heberden’s nodes, 313, 329

751
Heinz bodies, 288, 297 Hernia Hormone replacement therapy
Helicobacter pylori, 90, 91, 248, diaphragmatic, 55, 62, 319– (HRT), 416, 428
255, 257, 261–262, 524– 320 Horner syndrome, 500–501, 521–
525, 666, 682, 688, 700 congenital, 55, 62, 462, 522, 554
Hemagglutinin, 87–88 481 Horseshoe kidney, 382, 395–396,
Hemangioblastoma, 60 femoral, 320 412
Hematochezia, 443 hiatal, 320 Howell-Jolly bodies, 274, 288,
Hematopoiesis, 69 inguinal, 317, 335, 423, 441 294, 596
Heme oxygenase, 596 direct, 308, 319 β-Human chorionic gonadotro-
Hemianopia indirect, 320 pin (β-hCG), 234, 248,
bitemporal, 583, 636, 650–651 Herpes zoster, 146, 155, 191, 416–417, 427, 429, 432,
homonymous, 567, 582–583, 639, 655 449
620 Herpesvirus, 98, 137, 191, 287, Human immunodeficiency virus
contralateral, 651 320, 358, 423, 424, 443, (HIV), 87, 92, 105, 115–
Hemicraniectomy, 523 444, 568, 583–584, 611, 116, 129, 138, 345, 358,
Hemidesmosomes, 92, 323–324 632, 653, 700 586, 615
Hemochromatosis, hereditary, congenital, 90, 628 associated dementia, 609, 627–
35, 128–129, 137, 468, HSV type 1, 586, 657 628
534, 551 Heteroplasmy, 135 congenital, 700
Hemoglobin, 24–25, 42 Hexamethonium, 152 Human leukocyte antigen
Hemolysis, 39 Hexokinase, 35, 40 (HLA), 204, 222–223, 264
Hemolytic-uremic syndrome Hexosaminidase A, 618 B27, 317, 336
(HUS), 77, 92, 258, 543 Highly active antiretroviral ther- DR3, 676, 694
Hemophilia apy (HAART), 446 DR4, 676, 694
A (factor VIII deficiency), 178, Hirschsprung disease, 238, 255, DR5, 694
288, 298, 304, 323, 571, 594, 639, 656 DR7, 694
590 Histamine, 223, 403 DR8, 694
B, 571, 590 Histamine1 agonists, 39 Human papillomavirus (HPV),
Hemothrorax, 482 Histamine2 74, 87, 129, 138, 287, 320,
Henoch-Schönlein purpura agonists, 39 419, 433
(HSP), 565, 578–579 blockers, 670, 682 Human T-cell lymphotrophic vi-
Hepadnavirus, 320 Histidine, 374, 713, 732 rus, 87
Heparan sulfate, 383, 399 Histoplasma capsulatum, 83 Humeral fracture, 310, 323
Heparin, 153, 304, 506, 598, 687, Histoplasmosis, 89, 450 Humor, 737
716, 736–737 Histrionic personality disorder, Hunter disease, 43
Hepatic encephalopathy, 250 568–569, 585 Hunter syndrome, 49, 678, 698
Hepatitis HMG CoA reductase inhibitor Huntington disease, 65, 135, 340,
A virus, 91, 97, 497, 517 (statin), 19, 31–32, 35, 349, 356, 608, 626, 654,
alcoholic, 132, 136 152, 731 658
B virus, 77, 87, 93, 97, 234, Hodgkin lymphoma, 105, 115, Hurler syndrome, 698
247, 517, 604, 620 145, 152–153, 264, 276, Hutchinson’s triad, 659
C virus, 76, 87, 91, 97, 240, 277, 279, 294, 296, 298, Hybridization, 622
249, 259–260, 620 299, 685, 707, 720, 727 Hydatidiform mole, 280, 299,
E virus, 91–92 nodular-sclerosing, 609, 629 426–427, 449
Hepatocellular carcinoma Homocysteine, 290–291, 635, partial, 582
(HCC), 243, 264, 280, 648 Hydatidosis, 614
287, 300, 525 Homocystinuria, 24, 41, 542 Hydralazine, 162, 174, 327
risk factors for, 284 Homogentisic acid oxidase Hydranencephaly, 728
Hepatorenal syndrome, 391, 412 defect in, 605, 621 Hydrocephalus, 124, 131
Hermaphroditism, true, 448, 656 deficiency of, 30–31, 44 communicating, 124, 131

752
ex vacuo, 131 Hypersensitivity reactions, 107, IgA nephropathy. See Berger dis-
normal-pressure (NPH), 359, 111, 117, 119, 268, 317, ease
535, 553, 712, 728 336, 456, 472, 613, 636, Imatinib (Gleevec), 255, 291–
noncommunicating, 131, 729 649–650 292, 293, 294, 303
Hydrochloric acid, 200, 215 Hypertension, 172, 193–194, Imipenem, 496, 514, 688
Hydrochlorothiazide, 145, 154, 410–411, 413 Immune thrombocytopenic pur-
174, 187, 225, 358, 380, hemorrhage due to, 697 pura (ITP), 277, 290, 294
383, 393, 397, 414, 475, malignant, 412 Immunodeficiency, severe com-
664, 677, 695–696, 721, Hypertensive emergency, 163, bined, 102, 109
730 177, 712–713, 730 Immunoglobulins, 260
Hydrocortisone, 535, 552 Hyperthyroidism, 18, 30, 142, A, 89, 113–114, 115, 118, 134,
Hydroxychloroquine, 484 148–149, 200, 211, 213, 394, 398, 401, 405, 413,
7α-Hydroxylase, 731 297, 502, 524, 589, 665 471, 536, 554–555, 579,
11β-Hydroxylase, 218 Graves disease-induced, 200, 689
deficiency, 433 213 deficiency, 107, 113–114,
17α-Hydroxylase, 204, 218, 222 Hypertriglyceridemia, type V, 40 117–118, 119, 120, 653
deficiency, 204, 222, 433 Hypertrophic obstructive cardio- protease, 89
21α-Hydroxylase, 203, 218 myopathy (HOCM), 164, D, 115, 555
deficiency, 419, 433, 656 177 deficiency, 403–404
5-Hydroxytryptamine3 antagonist, Hyperuricemia, 734–735 E, 115, 118, 119, 134–135, 555
601, 615 Hypervitaminosis A, 729 G, 105, 115, 118, 134, 555,
Hydroxyurea, 290, 294, 303, 503 Hypoalbuminemia, 242, 262 579, 726
Hydroxyzine, 9 Hypocalcemia, 55, 61, 118, 504 M, 115, 118, 134, 555, 589
Hyperaldosteronism, primary, Hypochondriasis, 511 deficiency, 119
202, 203, 217–218, 219, Hypodermis, 317, 335 Impetigo, 534, 550
229 Hypogammaglobulinemia, 589 Incontinence, fecal, 5, 10
Hyperammonemia, 22, 38 Hypoglycemia, 27, 47, 206, 225 India ink stain, 554
Hyperbilirubinemia, 47, 68 ethanol-induced, 27, 47 Indomethacin, 475
Hypercalcemia, 220, 278, 298, Hypogonadism, 422, 439 Inferior mesenteric artery (IMA),
516, 734 Hyponatremia, 460–461, 478, 166, 181–182, 208, 230
Hypercholesterolemia, 442 564, 568, 577–578, 584 Inferior vena cava (IVC) com-
familial, 508, 534–535, 551– Hypoparathyroidism, 61, 220, pression, 165, 180, 219
552, 713, 731 542 Inflammatory bowel disease. See
Hypercortisolism. See Cushing Hypophyseal portal system, 205, Crohn disease; Ulcerative
syndrome 223–224 colitis
Hyperglycemia, 523, 550, 610, Hypospadias, 54, 60 Infliximab, 238, 252, 255, 506,
630, 725 Hypothalamic-pituitary axis, 417, 626
Hyper-IgM syndrome, 104, 109, 431 Influenza virus, 74, 87–88, 452–
113, 118, 589 Hypothyroidism, 216–217, 222, 453, 466
Hyperinsulinemia, 714–715, 734 297, 328, 588 Informed consent, obtaining, 7,
Hyperkalemia, 162, 174–175, Hypovolemic shock, 385, 402, 14–15, 499, 519, 661
407, 474 703 exception to, 535, 552
Hyperlipidemia, 165, 180 Infraspinatus muscle, 326, 327
Hypermagnesemia, 734 Insulin, 215, 218, 223, 226, 358,
I
Hyperparathyroidism, 25, 43, 545, 546, 675, 692
215, 218, 220, 497, 515, Ibuprofen, 151, 661 ultralente, 692
543, 645, 666, 667 I-cell disease, 26, 45–46, 609, Insulin-like growth factor (IGF),
Hyperpigmentation of skin, 250 628 35–36, 226
Hyperprolactinemia, 573, 592– Idiopathic thrombocytopenic Insulinoma, 224, 592, 597–598
593, 630–631 purpura, 132, 579 Integrins, 399

753
Interferon J clear cell carcinoma, 295
α, 303 polycystic, 24, 41, 63, 412, 631
γ, 40, 94, 110, 113 Janeway lesions, 170, 186, 191, adult (APKD), 127, 135,
pegylated, 240, 259–260 736 295, 389, 409, 442, 549
Jaundice, 26–27, 46–47, 638,
Interleukin transitional cell carcinoma
654, 725
2 (IL-2), 135 (TCC), 295
obstructive, 23, 38–39
3 (IL-3), 689 Kidney stones, 128, 136, 203,
physiologic, 68
6 (IL-6), 40–41 220, 385, 402, 585. See
JC virus, 73, 85, 345, 358
7 (IL-7), 110 also Nephrolithiasis
Job syndrome, 114, 119, 120
12 (IL-12), 113 Kimmelstiel-Wilson nodules,
Jugular foramen, 350
Intermediate junctions, 92 143, 150, 199, 212, 409,
Juvenile rheumatoid arthritis
Internal capsule, 354 519–520, 597, 620
(JRA), 105, 115
International Normalized Ratio Klebsiella pneumoniae, 386, 394–
(INR), 286, 736–737 395, 403, 442
Internuclear ophthalmoplegia K Klinefelter syndrome, 504, 641,
(INO), 342, 353 660
Kabuki make-up syndrome, 638,
Intracerebral hemorrhage, 578 Klüver-Bucy syndrome, 353, 626
654–655
Intracranial injury, 602–603, Krabbe disease, 586, 618
Kallmann syndrome, 630
617–618 Krause end bulb, 213
Kaposi sarcoma, 287
Intraparenchymal hemorrhage, Kartagener syndrome, 22, 37, 51, Krukenberg tumor, 254, 269,
349 418, 431–432, 458, 474– 423, 443, 739
Intrauterine growth restriction 475, 492–493, 508, 554, Kugelberg-Welander disease
(IUGR), 528, 540 580 (SMA, type III), 12
Intrinsic factor, 285, 684 Kasai’s procedure, 68 Kupffer cells, 110
Introns, 602, 622 Kawasaki disease, 114, 170, 179,
Intussusception, 61, 248 191, 335–336 L
Inulin, 388–389, 407, 410 Kawasaki syndrome, 498, 518
Iodide, 211, 665 Keratin, 701 L4 nerve root, 332
Iodine, 226–227 Keratosis Labetalol, 177, 536, 555, 566,
deficiency, 222 actinic, 284, 582 580, 730
radioactive, 149, 214 seborrheic, 582 Labor, 439
Ipratropium bromide, 460, 478, α-Keto-acid dehydrogenase induction of, 528, 540
483, 651 branched-chain, 621 β-Lactam antibiotics, 37
Irinotecan, 149–150 deficiency of, 31 Lactase deficiency, 50, 116, 664
Iron Ketoacidosis, 204, 222–223 Lactate dehydrogenase, 189,
deficiency, 258 Ketoconazole, 151, 200, 214, 291
anemia, 113, 251, 275, 288, 227, 334, 422, 441, 594 Lactic acidosis, 541, 545–546
291, 295, 297–298, 684 α-Ketoglutarate, 30 Lactobacillus, 429
overdose, 146, 155 Kidney. See also Kidney disease; Lactose operon, 635, 648–649
Isocarboxazide, 213 Renal topics Lamivudine, 148, 260, 620
Isoniazid, 294, 312, 327, 397, ectopic, 63 Lamotrigine, 612
453, 468, 480, 492, 507, failure, 396 Langerhans cells, 102, 110
535, 553, 712, 730 acute, 385, 401–402 LaPlace’s law, 474
Isoproterenol, 176, 555, 685–686 chronic, 144, 150–151 Late-look bias, 556
Isovolumetric contraction, 171, horseshoe, 382, 395–396, 409, Lateral collateral ligament, 330
193 412 Lateral medullary syndrome, 344,
Isovolumetric relaxation, 171, 193 response to high sodium 356
Itraconazole, 441, 594 intake, 387, 406 Latissimus dorsi, 360
Ixodes tick, 292, 523–524, 577, Kidney disease Lead poisoning, 34, 275, 289–
719 acute interstitial, 396 290, 637, 653, 678, 699

754
Leber hereditary optic neurop­ Lipoprotein lipase, 731 Lyme arthritis, 325
athy, 302 Listeria, 89, 550 Lyme disease, 72, 84, 86, 118,
Lecithinase, 89 monocytogenes, 93, 340, 348, 168–169, 187–188, 292,
Left ventricular failure, 193 477, 510, 638, 653 523–524, 564, 577, 707,
Left ventricular hypertrophy, 97, Listeriosis, 348 719, 722–723
167, 184 Lithium, 10, 199, 212, 372, 373, Lymphoma, 328
Legionnaire’s disease, 459, 461, 376, 397 acute lymphoblastic, 685
477, 479, 717, 738 toxicity, 475 Burkitt, 80, 97, 274, 287, 298,
Legionella pneumophila, 84, 88, Liver, 226 502, 524, 547, 591, 629,
459, 461, 477, 479, 485, carcinoma of, 243, 264, 280, 671, 685, 690
738 287, 300 central nervous system, 85,
Leiomyomas, 417, 430–431 disease, chronic, 247 627–628
Leishmania donovani, 96, 276, Locus heterozygosity, 135 follicular, 276, 292, 532, 547,
292, 333 Loeffler medium, 78, 88, 94, 473 647
Lentigo, 284 Loop of Henle, 390, 392, 411, Hodgkin, 105, 114–115, 145,
Lesch-Nyhan syndrome, 41, 710, 664 152–153, 264, 276, 277,
725 Loperamide, 253, 626 279, 294, 296, 298, 299,
Leucine, 732 Lorazepam, 593 685, 707, 720, 727
Leucovorin, 297–298 Losartan, 326, 731 nodular-sclerosing, 609,
Leukemia Lou Gehrig’s disease. See Amyo- 629
acute lymphocytic (ALL), 293, trophic lateral sclerosis mantle cell, 547, 591, 690
575, 596, 662 (ALS) mucosa-associated lymphoid
acute myelogenous (AML), Lovastatin, 13 tissue (MALT), 76, 90
274–275, 289, 444, 572, Löwenstein-Jensen agar, 94 non-Hodgkin, 276, 292, 341,
590–591, 629, 690 Lumbar puncture, 665–666 352, 647, 720
acute promyelocytic (APL), Lung. See also Pulmonary topics Lysergic acid diethylamide (LSD)
143, 149, 293, 701 cancer, 269, 272, 285, 289 intoxication, 11, 366,
adult T-lymphocyte, 287 adenocarcinoma, 48, 645, 375
chronic lymphocytic (CLL), 667 Lysine, 732
556, 647 bronchogenic carcinoma, Lysosomes, 46
chronic myelogenous (CML), 536, 554
281, 303, 591, 685 non-small-cell carcinoma, M
hairy cell, 634, 646–647 460, 478
Leukocyte adhesion deficiency Pancoast tumor, 272, 285, Macrolides, 252
(LAD) syndrome, 114, 500–501, 521, 554 Macrophage colony-stimulating
709, 724 small cell carcinoma, 278, factor, 514
Leuprolide, 151, 303 296–297, 462, 481–482, Macula adherens, 310, 323
Levodopa/carbidopa, 357 484, 568, 584, 667 Macular cherry-red spots, 409
Levofloxacin, 514, 553 squamous cell carcinoma, Magnesium, 227, 591
Levothyroxine, 149, 198, 211, 455, 470, 482, 536, 544, Major histocompatibility com-
214, 232 554, 585, 667, 711, 727 plex II, 119
Lewy bodies, 344, 356, 359, 444 regional differences in, 454, Malaria, 86, 89, 133, 188, 292
Leydig cell tumor, 431, 444 469 Malassezia furfur, 594, 718
Lhermitte sign, 350 transplantation, 456, 472, 479 Malate, 30
Lidocaine, 183, 616 Lupus nephritis, 117, 380, 393, Malingering, 511
Ligamentum venosum, 66 620–621 Mallory bodies, 132, 136, 138,
Linezolid, 86, 514–515 Luteinizing hormone (LH), 225, 444, 655
Lipoma, 511 351, 425, 432, 447–448, Mallory-Weiss syndrome, 243,
Lipopolysaccharide, 89 546, 625, 641, 660 264, 514
lipid A, 93 excess, 447–448 Malpractice, proof of, 5, 12

755
Mandibular nerve, 350 Meningomyelocele, 54, 60, 350 Michaelis-Menten constant val-
Mannitol, 546, 721 Meniscus ues, 20, 33–34, 40, 711,
Mantle cell lymphoma, 547, 591, lateral, 313, 330 728
690 medial, 330 Miconazole, 594
Maprotiline, 213 Menometrorrhagia, 417, 430 Midazolam, 155
Marburg virus, 684–685 Menopause, 416, 428, 440–441, Middle cerebral artery (MCA),
Marfan’s syndrome, 41, 135, 322, 607, 625, 641, 659–660 204, 223, 348, 583
324, 393, 589–590, 629, premature, 593 Midgut herniation, 64
662, 674, 691, 719 Menstrual cycle, 425, 447 Mifepristone, 154
Marijuana, 375, 441 6-Mercaptopurine, 583 Migraine headache, 360
Mast cells, 702 Mercury toxicity, 653 Minimal change disease, 133,
McArdle disease, 542 Merkel nerve endings, 213 383, 399, 405–406, 408,
Measles (rubeola), 73, 85, 462, Mesonephric (wolffian) duct, 60, 411, 413, 499, 519, 614
480–481 420–421, 621, 631, 723 Minimum alveolar concentration
Mebendazole, 192 Mesothelioma, 130, 139 (MAC), 672, 687–688
Meckel diverticulum, 55, 61, 63, Metabolic acidosis, 195, 256, Miosis, 361
656–657, 709, 723 388, 393, 400, 407, 457, Mirtazapine, 362, 375
Medial lemniscus, 354 474, 501, 508, 522–523, Misoprostol, 154, 263, 264, 528,
Medial longitudinal fasciculus 565, 579 540, 572, 591
(MLF) syndrome, 670, anion gap, 501, 522–523 Mitral regurgitation, 164, 177,
682 chronic, 549 190
Median nerve injury, 359 Metabolic alkalosis, 238, 256, Mitral stenosis, 166, 181, 184,
Medulloblastoma, 59–60, 129, 508, 610, 629–630, 708, 190
138–139, 273, 286, 306, 720 Mitral valve prolapse, 97
701 Metachromatic leukodystrophy, Mobitz type I (Wenckebach) AV
Megakaryocytes, 702 49, 603, 618 block, 170, 190–191
Meissner corpuscles, 213 Metformin, 13, 202, 218, 358, Mobitz type II AV block, 172,
Melanocytes, 353 541, 545, 553, 617 194
Melanoma, 300, 319, 495, 512, Methadone, 370 Modafinil, 4, 9
582 Methemoglobinemia, 28, 48– Molluscum contagiosum, 308–
Melarsoprol, 262 49 309, 320
MELAS, 224 Methicillin-resistant Staphylococ- Monoamine oxidase (MAO) in-
Membrane attack complex cus aureus (MRSA), 236, hibitors, 689
(MAC), 689 251–252 Monoclonal gammopathy of un-
Meningioma, 131–132, 286, 306, Methimazole, 211 determined significance
342, 353, 533, 548, 701– Methotrexate, 51, 146, 152–153, (MGUS), 282, 301, 304–
702 154, 292, 297–298, 483, 305, 556
Meningitis, 207, 228–229, 494, 583, 640, 657, 711, 728 Mononucleosis, 466, 525, 659,
509–510, 529, 543, 658 Methylmalonic acid, 275, 290 720. See also Epstein-Barr
cryptococcal, 677, 696, 733 Methylphenidate, 207, 228, 374, virus
herpes simplex, 733 375, 560–561 heterophile-negative, 525
neonatal, 82, 100, 340, 348, Metoclopramide, 142, 148, 262 Morphine, 661
494, 509–510 Metoprolol, 174, 176, 177, 187, Morquio syndrome, 698
viral, 74, 86–87 225, 405, 465, 469 Mosaicism, 135
Meningocele, 350 Metronidazole, 100, 150, 153, Moxifloxacin, 100
Meningococcemia, 207, 228– 192, 241, 251–252, 262, mRNA, capping of, 622
229 558, 584, 673, 688, 735 Mucoepidermoid carcinoma,
Meningoencephalocele, 350 Mevalonic acid, 19, 31–32 266
Meningohydroencephalocele, Mexiletine, 156 Mucopolysaccaridosis, 329
340, 350 Meyer loop, 583, 604, 619–620 Mucor, 619

756
Mucormycosis, rhinocerebral, tuberculosis, 94, 99, 100, 118, Nephroblastoma, 404
536–537, 555 467, 473, 479, 480, 492, Nephrolithiasis, 203, 220, 390,
Mucosa-associated lymphoid tis- 507, 536, 554, 606, 623, 410. See also Kidney
sue (MALT) lymphoma, 730 stones
76, 90 Mycoplasma, 467 Nephrotic syndrome, 37, 126,
gastric, 700 pneumoniae, 84, 85, 467, 738 133–134, 262, 381, 383,
Müllerian (paramesonephric) Myelofibrosis, 133 388, 394, 398, 406–407,
ducts, 424, 445, 611, 621, Myocardial infarction (MI), 163, 413, 499, 500, 519, 620
631 167, 168, 170, 172, 176, steroid-responsive, 37
Multiple endocrine neoplasia 183–184, 190–191, 194, Neural crest cells, 723
(MEN) 518–519, 530, 544–545 Neural tube defects, 299, 538,
type 1 (Wermer syndrome), obstructive, 681, 703–704 540, 560, 656
214, 224, 261–262, 306, risk of, in women receiving Neuraminidase, 88
572, 591–592, 600, 612– HRT, 428 Neurilemmoma, 702
613, 646 Myoglobin, 189 Neuroblastoma, 44, 198, 211–
type 2A, 214, 306, 634, 646 Myopathies, idiopathic inflam- 212, 295, 300, 328
type 2B, 200, 214–215, 306, matory (IIMs), 4, 9 Neurocysticercosis, 74–75, 88
646 Myxoma, 511 Neurofibromas, 306
Multiple myeloma, 126, 134, Neurofibromatosis
278, 298, 301, 490, 504, N type 1, 138, 302, 571, 589
528, 539, 541, 556, 561, type 2, 135, 302, 589
Nadolol, 454, 469
708, 713, 720, 722, 732 Neurogenic shock, 703
Naegleria fowleri, 96
Multiple personality disorder. See Neuroleptic malignant syn-
Naloxone, 156, 364, 369, 370,
Dissociative identity disor- drome, 678, 697–698
377, 400, 581, 699, 738
der Naltrexone, 369, 370 Nevirapine, 142, 148
Multiple sclerosis (MS), 37, 350, Narcolepsy, 4, 9 Niacin (vitamin B3), 552, 647
576, 598, 682 Nasopharyngeal carcinoma, 128, deficiency, 26, 45, 61, 366,
Mumps, 685 136, 502, 524 374
Murphy sign, 514, 515 Natural killer cells, 41, 116, 680, Nicotinamide adenine dinucleo-
Muscular dystrophy, Duchenne 702 tide reductase (NADH),
(DMD), 12, 27, 47, 322, Negative predictive value, 540, 28, 47, 48
654 674, 690–691 Nicotine intoxication, 11
Mutation Neisseria, 696 Niemann-Pick disease, 569, 586,
deletion, 715, 735 gonorrhoeae, 88, 89, 92, 94, 96, 618, 662
frameshift, 690, 735 99, 332–333, 429, 430– Nifedipine, 163, 177, 187
insertion, 690, 735 431, 437, 445, 450, 515 Nifurtimox, 262, 292
missense, 690, 735 meningitidis, 84, 529, 543, Nigrostriatal tract, 431
nonsense, 674, 690, 735 653–654 Nikolsky sign, 335
silent, 690, 735 Neomycin, 593 Nitrates, 171, 192
transition, 735 Neonatal respiratory distress syn- Nitroglycerin
Myasthenia gravis, 108, 120, 312, drome (NRDS), 29, 50–51 interaction with sildenafil, 13
328 Neonatal sepsis, 82, 100 withdrawal, 142–143, 149
Mycobacterium Neostigmine, 343, 355–356 Nitroprusside, 712–713, 730
avium, 137, 536, 554–555, Nephritis Nitrosamines, 279, 284, 298–
628 due to systemic lupus erythe- 299
avium-intracellulare (MAC), matosus, 389, 408 Nizatidine, 252
100 lupus, 117, 380, 393 NMDA receptor antagonists,
kansasii, 100 tubulointerstitial, drug-­induced, 624–625
leprae, 327 383, 397, 404, 410 Nocardia, 100
marinum, 100 analgesic nephropathy, 404 asteroides, 485

757
Non-Hodgkin lymphoma, 276, Osler’s nodes, 170, 186, 191, 736 Paracoccidioides brasiliensis, 83–
292, 341, 352, 647, 720 Osmotic demyelination, 564, 577 84
Non-maleficence, 661 Osteoarthritis, 313, 329 Paradidymis, 631
Nonsteroidal anti-inflammatory Osteochondroma, 319, 330 Paramesonephric (müllerian)
drugs (NSAIDs), 150–151, Osteogenesis imperfecta, 41, 309, ducts, 424, 445, 611, 621,
185, 255, 261, 326, 380– 322, 324, 329, 504, 631, 631
381, 382, 393–394, 396, 662, 707, 719 Paranoid personality disorder,
404, 483 Osteomalacia, 45, 719–720 371, 585, 657
Norepinephrine, 176, 223, 503, Osteomyelitis, 81, 99, 312, 315, Parathyroid
682, 686 328, 329, 330, 332–333 adenoma, 504, 516, 529, 541,
Normal-pressure hydrocephalus vertebral, 686 554
(NPH), 359, 535, 553, Osteonecrosis, 708, 721–722 chief cells, 201, 215
712, 728 Osteopetrosis, 329, 636, 649 hormone (PTH), 43, 209, 215–
Northern blot, 32 Osteoporosis, 336, 529, 541, 542, 216, 228, 229, 231, 321,
Nortriptylene, 373, 673, 689 652, 663 328, 411, 541, 543
Norwalk virus, 98 Osteosarcoma, 278, 297, 308, elevated, 209, 231
Nystatin, 95, 446 319, 330–331 related peptide (PTHrP),
Ovarian cancer, 299, 544 231
O Oxaloacetate, 30, 39 hypoplasia, 208, 229–230
Oxygen-hemoglobin dissociation insufficiency, 516
Obesity hypoventilation syn- curve, 455, 470–471, 490, oxyphil cells, 215
drome, 460, 478 503, 642, 661, 677, 696– Paraurethral (Skene) glands,
Obsessive-compulsive disorder, 697 632
367, 375, 376 Oxytocin, 351, 421, 437–438 Parkinson disease, 343, 344, 349,
Obsessive-compulsive personality 353, 354, 356, 361, 375,
disorder, 585 P 553, 593, 615, 627, 638,
Obturator nerve, 260, 332 654, 699
Octreotide, 241, 261 p53 gene, mutation in, 21, 33, 36 Parotid gland, adenoma of, 244,
Oculomotor palsy, 494–495, 511 Pacinian corpuscle, 199, 212–213 265, 281, 303
Odds ratio, calculating, 14, 16 Paclitaxel, 153 Paroxysmal nocturnal hemoglo-
Olanzapine, 365, 372 Paget disease binuria (PNH), 492, 507,
Oligodendroglioma, 131, 286, of bone, 542–543, 572–573, 622
306, 353, 548–549, 702 592, 663 Partial thromboplastin time
Omeprazole, 148, 242, 252–253, of breast, 505 (PTT), 273, 285–286, 498,
261, 263, 273, 285, 591, Pain disorder, 708–709, 722 517, 571, 590, 736–737
673, 688 Pancoast tumor, 272, 285, 500– Parvovirus B19, 692
Omphalocele, 56, 64 501, 521, 554 Pasteurella multocida, 524
Ondansetron, 148, 253, 643, Pancreas, 208, 226, 230 Patau syndrome (trisomy 13), 65
662–663 adenocarcinoma of, 26–27, 46, Patellar ligament, 330
Opioid 225, 246, 253–254, 269– Patent ductus arteriosus (PDA),
intoxication, 361 270, 443 66, 90, 166, 169, 175, 182,
overdose, 364, 370, 401 annular, 56, 64 189, 540–541, 560, 641,
Organophosphate poisoning, 144, islet cells of, 226 659
151–152 Pancreatitis, 324 Pellagra, 26, 45, 61
Orlistat, 228, 546 acute, 242, 248, 263 Pelvic inflammatory disease
Ornithine transcarbamylase chronic, 241, 261, 664–665 (PID), 96, 430–431, 437,
(OTC), 38 Pancuronium, 152 515, 659
deficiency of, 44 Pancytopenia, 708, 722 Pemphigus vulgaris, 310, 317,
Orotate phosphoribosyltransfer- Papillary muscle rupture, 195 323, 335
ase, deficiency of, 31 Pappenheimer bodies, 294 Penicillamine, 155

758
Penicillin, 93, 95, 150, 191, 577 Phosphodiesterase-5 (PDE-5) in- Polyangiitis, microscopic, 104,
G, 316, 334, 446 hibitors, 435 114, 120
Pentamidine, 192, 452, 466, 698 Phrenic nerve, 331 Polyarteritis nodosa (PAN), 114,
Pentose-phosphate pathway, 43 damage to, 713, 731 317, 335, 471, 695
Pentoxyfylline, 593 Physostigmine, 156, 343, 355– Polycystic ovarian syndrome
Pepsin, 249 356, 377, 581 (PCOS), 424, 426, 442–
Peptic ulcer, 61–62, 644, 666, Pick disease, 345, 358 443, 444, 448–449, 610,
672–673, 688 Pickwickian syndrome, 476 630, 714–715, 734
Perchlorate, 149 Pioglitazone, 675, 692 Polycythemia vera, 235, 290, 494,
Pericardial friction rub, 172, 193 Piperacillin, 403 510
Pericarditis, 80–81, 96, 98, 167, Pituitary, 351–352, 609, 628 Polyhydramnios, 67, 400
184–185, 519 adenoma, 59, 218, 347, 362 Polymerase chain reaction
constrictive, 179–180 Placenta accreta, 439 (PCR), 32, 467
fibrinous, 630 Placenta previa, 424, 425, 439, Polymyalgia rheumatica (PMR),
Perihepatitis, 496, 515 444–445, 446, 540 676–677, 695
Periodic acid-Schiff stain, 554 Plague, 188 Polymyositis, 4, 9, 318, 337
Personality disorders Plasma cells, 226, 702 Polymyxins, 150, 153–154
antisocial, 657 Plasmodium, 89, 292 Polyomavirus, 320
avoidant, 371, 657 falciparum, 73, 86, 96 POMC gene, 441
borderline, 657, 716, 737 knowlesi, 86 Pompe disease, 23, 38, 39, 44,
Cluster B, 16 malariae, 86 542
Cluster C, 16 ovale, 86 Pons, 352
histrionic, 568–569, 585 vivax, 86 Population attributable risk
obsessive-compulsive, 585 Pleural effusion, 482, 709, 723 (PAR), 14
paranoid, 371, 585, 657 Plummer disease, 222 Porphobilinogen deaminase, defi-
schizoid, 16, 365, 371 Pneumoconiosis, coal workers’, ciency of, 25, 43
schizotypal, 371, 586, 640, 476 Porphyria, acute intermittent
657 Pneumocystis jiroveci, 137, 214, (AIP), 25, 34, 43
Pertussis (whooping cough), 75, 452, 466, 570, 573, 577, Porphyria cutanea tarda, 20–21,
78, 86, 88, 93 588, 593–594, 619, 678, 34
Phencyclidine (PCP), 5, 11, 372, 698, 718–719, 738 Portal hypertension, 235, 237,
375, 716, 737 Pneumocytes 250, 254–255
Phenelzine, 373, 587 type I, 690–691 Positive end-expiratory pressure
Phenobarbital, 212, 249, 370, type II, 691 (PEEP), 456, 472
417, 430, 698, 736 Pneumonia, 118, 462, 465, 466, Positive predictive value (PPV), 6,
Phenoxybenzamine, 490, 503 480–481, 537, 557, 573, 12–13, 495, 513, 540, 655,
Phentolamine, 176–177 593–594, 607, 624, 664, 674, 690–691
Phenylalanine, 374 678, 698, 738 Posterior cerebral artery (PCA),
Phenylephrine, 176–177, 192, aspiration, 454, 468–469, 477 204, 223, 567, 582–583
686 atypical, 72, 84, 453, 467 Posterior cruciate ligament, 330
Phenylketonuria (PKU), 26, 45, respiratory legionellosis, 459, Posterior inferior cerebellar artery
621 461, 477, 479, 717, 738 (PICA), 204, 223, 344,
Phenytoin, 153, 612 Pneumothorax, 462–463, 482, 356
Pheochromocytoma, 209, 218, 580, 664 syndrome, 344, 356
221, 226, 229, 232, 490, spontaneous, 452, 465, 643, Postpartum blues, 435
503, 536, 550, 551, 555, 664 Postpartum major depression
576, 592, 597, 675, 692– tension, 465, 664 (PMD), 420, 435
693 Poliomyelitis, 351, 710, 726 Postpartum psychosis, 435
Philadelphia chromosome, 293, Poliovirus, 315, 332, 586 Posttraumatic stress disorder
303, 591, 690 Polyadenylation, 622 (PTSD), 364, 369

759
Potassium iodide, 594 Prostate-specific antigen, 298, Pyelonephritis, 381, 386, 390,
Potency, drug, 156 420, 434, 537, 542, 557 394, 403, 404, 409, 410–
Potter syndrome, 57, 67, 384, Protamine sulfate, 153, 155, 298, 411
400, 409, 560 699 Pyloric stenosis, hypertrophic, 64
Pott’s disease, 99, 409 Protein, 30 Pyrazinamide, 473, 553, 730
Prader-Willi syndrome, 135 Protein C, 178, 394 Pyridostigmine, 152
Pralidoxime, 144, 151–152, 377 Protein S, 394 Pyridoxine (vitamin B6), 461,
Pravastatin, 265 Proteus mirabilis, 91, 395, 442 480, 492, 507–508
Prednisone, 150, 232, 279, 299 Prothrombin time (PT), 286, deficiency, 113, 535, 553, 624
Preeclampsia, 417, 429–430, 446 498, 517, 571, 590, 736– Pyrimethamine, 262, 594, 628
Pregnancy 737 Pyruvate, 39
alcohol intake during, 436 Proton pump inhibitors, 250
ectopic, 248, 299–300, 416– Proximal convoluted tubule, 392 Q
417, 429 Psammoma bodies, 198, 210,
ruptured, 567, 582 342, 353, 533, 548–549, Quadrantanopia, 651
Pressure-volume (P-V) loop, 173, 646, 714, 733 Quetiapine, 213
195 Pseudogout, 321, 329, 334 Quinidine, 164–165, 179, 602,
Prevalence, 5, 11, 14 Pseudohermaphroditism, female, 616
Probenecid, 658 448, 656 Quinine, 292
Procainamide, 147, 156, 185 Pseudohypoparathyroidism, 221
Procedural bias, 726 Pseudomembranous colitis, 80, R
Prochlorperazine maleate, 9, 97, 265, 564, 578, 655
341, 352, 372 Pseudomonas, 94, 95 Rabies virus, 517, 676, 695
Progesterone, 418, 425, 432, 447, aeruginosa, 84, 89, 91, 93, 99, Radial nerve injury, 346, 359
449, 660 328, 332, 506, 557–558, Radiation, adverse effects of, 721
Progressive multifocal leukoen- 590 Ranitidine, 262, 285, 688
cephalopathy (PML), 73, Psoriasis, 37, 324 Rationalization, 15
85, 345, 358, 714, 732–733 Psoriatic arthritis, 316, 325, 334, Raynaud phenomenon, 104, 113
Projection, 15, 721 725 Reaction formation, 721, 737
Prolactin, 351, 432, 449, 531, Psychosexual development, Recall bias, 556–557, 726
546, 609, 628, 630–631 Freud’s theories of, 5, 5α-Reductase
Prolactinoma, 598, 609, 628 10–11 deficiency of, 218, 426, 434,
Promethazine, 615–616 genital stage, 10–11 437, 448, 656
Pronator teres muscle, 333 latency stage, 11 inhibition of, 144, 151
Propafenone, 156 oral stage, 11 Reduviid bug, 292
Propionibacterium acnes, 324 phallic stage, 11 Reed-Sternberg cells, 105, 114,
Propranolol, 13, 171, 176, 179, Pudendal nerve, 335 294, 298, 609, 629, 647,
192, 214, 369, 616 Pulmonary edema, 674, 691–692 707, 720
overdose, 171, 192 Pulmonary embolism, 460, 477, Refsum disease, 621
Propylthiouracil (PTU), 142, 595 Reinke crystals, 444
148–149, 200, 211, 213, risk of, in women receiving Relative risk, calculating, 7, 8, 14,
232, 644, 665 HRT, 428 15–16
Prostaglandins, 393–394, 566, 580 Pulmonary fibrosis, 497, 516, 550 Renal artery stenosis, 382, 391,
E2, 403 interstitial/idiopathic (IPF), 395, 413
I2 (prostacyclin), 403, 530, 543 461, 479, 727–728 Renal calculus, 128, 136
Prostate cancer, 300, 308, 319, Pulmonary hypoplasia, 62 Renal cell carcinoma, 44, 289,
418–419, 420, 432–433, Pulmonary stenosis, 175, 189 319, 533, 544, 549, 589
434–435, 512, 537, 541– congenital, 625 Renal failure, 388, 406, 533, 549,
542, 557 Pulsus paradoxus, 187 663
bone metastases from, 504, 512 Putamen, 354 acute, 385, 401–402

760
chronic, 144, 150–151, 497, Ribosomes, 46 Schistocytes, 289, 297
515–516 Ricin poisoning, 693 Schistosoma
intrinsic, 406 Rickets, 207, 227, 314, 331, 643, haematobium, 252
postrenal, 406 652, 663, 719–720 mansoni, 254
prerenal, 406 Rickettsia rickettsii, 86, 95, 99, Schistosomiasis, 88, 237, 254
Renal cortical necrosis, 409 188 Schizoaffective disorder, 374
Renal insufficiency, chronic, 564, Riedel thyroiditis, 210 Schizoid personality disorder, 16,
578 Rifampin, 145, 153, 154, 327, 365, 371
Renal osteodystrophy, 564, 578, 473, 484, 507–508, 515, Schizophrenia, 16, 201, 217, 364,
645, 666 553–554 365, 370, 371, 372, 373–
Renal papillary necrosis, 409, 585 Riluzole, 351 374, 376, 573, 592–593,
Renal transplant rejection, 102, Risedronate, 483 615, 639, 655–656
103, 109, 111–112 Risperidone, 371 Schizophreniform disorder, 16,
Renin, 216, 226, 397 Rituximab, 292 366, 371, 373
Reovirus, 98, 466 RNA, 18, 30 Schizotypal personality disorder,
Repression, 15 Rocky Mountain spotted fever, 371, 586, 640, 657
Reserpine, 377, 689 78–79, 86, 95, 99, 188 Schwannoma, 283, 305–306
Residual volume, 462, 481 Roseola, 86 Sciatic nerve, 260
Respiratory acidosis, 408, 508 Rosiglitazone, 546 Scleroderma, 104, 113, 244, 266,
Respiratory alkalosis, 256, 408, Rotavirus, 92, 98, 116, 466, 657 268, 613
493, 508 Roth’s spots, 170, 186, 191, 736 CREST variant, 331
Respiratory distress syndrome, Rotor syndrome, 268 Scoliosis, adolescent idiopathic,
595, 684 Rough endoplasmic reticumum 7, 14
adult, 470, 483 (RER), 46 Scurvy, 61, 663, 687, 691
neonatal (NRDS), 453, 457, Round ligament, 432 Seborrheic keratosis, 582
467, 474, 670, 683 Rubella (German measles), 86, Secretin, 261, 285, 682
Retinoblastoma, 566, 581 90 Seizures, absence, 538, 560–561,
hereditary, 530, 544 congenital, 628, 700 600, 612
Retinol (vitamin A) deficiency, Rubeola (measles), 73, 85, 462, Selection bias, 537, 556, 711, 726
45, 239, 258, 652 480–481 Selective serotonin reuptake in-
excessive ingestion of, 729 Rubivirus, 75, 90 hibitors (SSRIs), 10, 367,
Rett syndrome, 606, 623–624 Ruffini corpuscles, 213 375, 376, 689
Rhabdomyolysis, 538, 559–560, Selegiline, 228, 303, 357, 362,
634–635, 647 S 372, 376
Rhabdomyoma, 494, 511 Selenium sulfide, 594
Rhabdovirus, 685 S2 heart sound, 189 Seminoma, 424, 444
Rheumatic fever, 80, 96, 166, S4 heart sound, 188–189 Sensitivity, 520–521, 528, 540,
178, 181, 258, 261, 543 Sabouraud’s agar, 473, 550 711, 728
Rheumatic heart disease, Salmonella, 99, 260, 312 Septic shock, 409–410, 681, 703
chronic, 625 typhi, 260 Sequencing, 19, 32, 36
Rheumatoid arthritis, 102, 109– Sampling bias, 557, 726 Serine kinases, 232
110, 264, 277, 295, 314, Sanfilippo syndrome, 698 Serotonin, 403
317, 321, 329, 331, 334, Saquinavir, 148 agonists, 663
336, 337, 640, 657, 695 Sarcoidosis, 107, 118, 456, 464, Serratia marcescens, 423, 442
juvenile (JRA), 105, 115 470, 471, 484, 565, 579– Sertraline, 366, 373, 376, 587
Rheumatoid factor, 337 580, 635, 647, 674, 690– Severe combined immunodefi-
Rhinitis, allergic, 456, 472 691 ciency (SCID), 102, 109,
Rhinovirus, 466–467 Sarcomere, 316, 333–334 589, 724, 725
Rhizopus, 555 Sarin poisoning, 693–694 Sexually transmitted diseases
Ribavirin, 83, 260, 584, 620 Schilling test, 200, 215, 291 (STDs), 436–437

761
Shared delusional disorder, 374 Specificity, 521, 540, 711, 728 Streptococcus, 99, 512
Sheehan syndrome, 420, 436– Spermatic cord, 440 agalactiae, 509, 550, 654
437, 571, 588 Spermatogenesis, 56, 63 bovis, 186
Shigella, 91, 92, 98, 241, 260 Spherocytes, 288–289 group B, 100, 348, 509
Shingles, 73, 86 Spherocytosis, hereditary, 50, 68, pneumoniae, 84–85, 89, 91,
Shock, 490, 503 278, 288–289, 297 98–99, 466, 477, 479–480,
anaphylactic, 94 Sphingomyelinase, 618, 662 510, 513, 683
cardiac, 703 Spider angiomata, 250 pyogenes, 93, 261, 328, 550
cardiogenic, 390–391, 411 Spina bifida occulta, 440 sanguis, 191
hypovolemic, 385, 402, 703 Spinal epidural abscess, 686 viridans, 186, 543
neurogenic, 703 Spinal muscular atrophy (SMA), Streptokinase, 153
septic, 409–410, 681, 703 6, 12 Stroke, 279, 298, 381, 394, 501,
Sialic duct stone, 266 Spiral fracture, 311, 325, 611, 523
Sicca syndrome, 330 631 Sturge-Weber syndrome, 590
Sickle cell anemia, 19, 31, 125, Spirillum minus, 477 Subacute combined degenera-
133, 255–256, 274, 288, Spironolactone, 162, 175, 227, tion, 346–347, 361
290, 332–333, 684 229, 664 Subarachnoid hemorrhage
Sideroblasts, ringed, 288 Spleen, 226, 481, 596 (SAH), 677, 697
Sigmoid colon, 166, 181–182 Splitting, 716, 737 Subclavian steal syndrome, 687
Sildenafil, 6, 13, 151, 420, 435 Sporothrix schenckii, 90, 594, Subdural hemorrhage, 348
Silicosis, 133, 139 619, 706, 718 Subscapularis muscle, 312, 326
Simvastatin, 265, 634–635, 647 Squamous cell carcinoma of skin, Sublimation, 708, 721, 737
Sipple’s syndrome, 306 284, 582 Succinyl-CoA, 30
Situs inversus, 458, 474–475 SRY gene, 420–421, 437 Sucralfate, 253, 285, 591
Sjögren syndrome, 121, 313, 329, Staphylococcal scalded skin syn- Sulfadiazine, 262, 594, 628
710, 725–726 drome (SSSS), 76–77, 92 Sulfasalazine, 236, 252, 483, 608,
Sjögren-Larsson syndrome, 330 Staphylococcus 626
Skene (paraurethral) glands, aureus, 91, 93, 95, 98, 99, 109, Sulfonamide, 334
632 111, 186, 245, 258, 267, Sulfonylureas, 218
Sleep 268, 328, 436, 466, 477, Sumatriptan, 598
apnea, 374 506, 537, 543, 550, 557, Superior gluteal nerve, 260
obstructive (OSA), 459, 476– 655, 716, 736 Superior vena cava syndrome
477, 532, 547 methicillin-resistant (SVCS), 711, 727
cycles, alteration of, 430 (MRSA), 236, 251–252 Suppression, 721
disrupted, 366, 374 epidermidis, 99 Supraspinatus muscle, 311, 326,
normal patterns of, 374–375 saprophyticus, 395, 402, 442 327
paralysis, 4, 9 Starling curve, 569, 586–587 Surfactant, 683–684
stage 1, 547 Statins, 19, 31–32, 162, 174 deficiency in, 29, 50–51
stage 2, 547 Statistical distribution, 710, 724– Suspensory ligaments, 418, 432
stage 3, 547 725 Syndrome of inappropriate anti­
stage 4, 547 Status epilepticus, 715, 735– diuretic hormone secre-
Sly syndrome, 698 736 tion (SIADH), 201, 208,
Smooth endoplasmic reticumum Steatosis, 132, 136 215, 229, 397, 458, 475,
(SER), 46 Steroid hormones, 28, 49 568, 584–585
Sodium, 523 Stevens-Johnson syndrome, 10, Syphilis, 81, 99, 584, 607, 625,
Somatostatin, 241, 249, 261, 285 430, 612 659
Somatotrophs, 226 Stomach, adenocarcinoma of, congenital, 90, 700
Sotalol, 183, 516 237, 253, 269, 279, 298– secondary, 81, 99
Southern blot, 32, 36–37, 640– 299, 443, 638, 654 tertiary, 99, 607, 625
641, 658–659 Stratum corneum, 317, 335 Syringomyelia, 733

762
Systemic lupus erythematosus Tetracycline, 86, 251, 673, 688 Hürthle cell, 211
(SLE), 106, 117, 118, 121, degraded, 568, 585 medullary, 211
259, 282, 303–304, 309, Tetralogy of Fallot, 38, 55, 62, 66, papillary, 210, 222, 549, 646,
320–321, 325, 331, 380, 163, 175, 541, 662 714, 733
393, 484, 510, 520, 530, Thalassemia development, 228
543, 559, 614 α-, 34–35, 305, 684 ectopic tissue, 492, 508
nephritis due to, 389, 408 major, 305 follicular cells, 215
minor, 305 lymphoma, 733
T β-, 21, 34, 283, 305, 624 Thyroid hormone, 30, 205, 223,
minor, 305 665
T lymphocytes, 601, 614–615, major, 280, 300 Thyroiditis
701 Thayer-Martin medium, 88, 94, Hashimoto, 22, 37, 201, 202,
activation, 706, 719 99, 473 216, 217, 221–222, 321,
regulatory, 702 Theophylline, 452, 465, 478, 597, 606, 613, 623, 646
Tacrolimus, 726 483, 651 Riedel, 210
Takayasu’s arteritis, 114, 164, 178, Thiamine (vitamin B1), 480, 665 Tiagabine, 561
336 deficiency, 48, 60, 61, 352– Tibial nerve, 332
Talofibular ligament, anterior, 353, 360 Tibiocalcaneal ligament, 322
309, 322 Thiazides, 150 Tibiotalar ligament, 322
Talonavicular ligament, 322 Thiocyanate, 149 Ticlopidine, 303
Tamoxifen, 154, 438–439, 583, Thioridazine, 371 Tight junctions, 92
658 Thiosulfate, 699 Tinea corporis, 422, 441
Tanner stages, 7, 14 Thoracic duct, 708, 721 Tissue plasminogen activator
Tardive dyskinesia, 372, 375 Thoracic nerve, long, 314, 331, (tPA), 298, 523
Target cells, 289 359 Tobramycin, 386, 403
Tay-Sachs disease, 28, 49, 256, injury, 359 Tocainide, 156
586, 618 Thoracentesis, 709, 723 Tolbutamide, 602, 616–617
Teardrop cells, 289 Threonine kinases, 232 ToRCHeS infection, 65, 90, 659,
Temporal (giant-cell) arteritis, Thrombocytopenia, 477–478, 700
114, 178–179, 335, 676– 590 Torsades de pointes, 164–165,
677, 695 Thrombocytosis, essential, 275, 179, 602, 616
Temporomandibular joint dys- 290 Tourette syndrome, 624
function syndrome, 360 Thrombopoietin, 726 Toxic shock syndrome (TSS),
Tennis elbow (epicondylitis, lat- Thrombotic thrombocytopenic 245, 268–269, 436
eral), 315, 333 purpura (TTP), 133, 290, Toxoplasma, 262, 696
Teratoma, 739 304, 579 gondii, 559, 594
Terbinafine, 466 Thromboxane A2, 182, 530, 543 Toxoplasmosis, 85, 358, 594, 628
Terbutaline, 651 Thymic aplasia. See DiGeorge congenital, 90, 700
Teres minor muscle, 326, 327 syndrome Tracheoesophageal fistula, 64, 67
Testicles, undescended, 508 Thymidine dimers, mutation of, Transference, 4, 9–10
Testicular artery, 440 18, 30 Transitional cell carcinoma, 44,
Testicular cancer, 281, 301–302, Thyroglossal duct, 648 544
418, 431 cyst, 54, 59, 207, 228 Transposition of the great arter-
Testicular feminization syn- Thyroid, 59 ies, 66, 511
drome, 434, 448, 639, C cells, 215, 226 Tranylcypromine, 228
656 cancer, 198, 210 Trapezius, 360
Testicular hydrocele, 425, 445 anaplastic, 210, 733 Trastuzumab, 255, 276, 291, 421,
Testicular torsion, 431 carcinoma, medullary, 283, 438
Testosterone, 205, 225, 227, 660 306, 544, 733 Trazodone, 199, 213, 369, 373,
Tetanus, 638–639, 655 follicular, 210–211, 646, 733 689

763
Treponema, 696 Turcot syndrome, 498, 517 Varicella-zoster virus (VZV), 73,
pallidum, 99, 429, 512, 569, Turner syndrome, 46, 256, 440, 86, 517
586, 607, 625 490, 503, 532, 547–548, Varicocele, 445
Triamterene, 225–226, 414, 664 593, 631, 672, 686–687 Vasoactive intestinal pep-
Trichinella spiralis, 333 Typhoid fever, 260 tide (VIP), 249, 261, 591–
Trichomoniasis, 79, 95 Typhus, epidemic, 188 592
Trichomonas vaginalis, 95, 429, Tyrosine, 374 Vasopressin, 223
584 Tzanck preparation, 99 Venereal Disease Research Labo-
Tricuspid regurgitation, 165, 181 Tzanck test, 424, 444, 446 ratory (VDRL) test, 99
Tricyclic antidepressants, 10 Venlafaxine, 376
Trigeminal nerve, 493, 509
U Ventricular ejection, 171, 193
divisions, 341, 350, 355
Ventricular filling, 171, 193
Trigeminal neuralgia, 598
Ulcerative colitis, 240, 242, 251, Ventricular rupture, 193, 195
Trimethoprim-sulfamethoxazole,
252–253, 257, 259, 264, Ventricular septal defect (VSD),
214, 251, 327, 466, 573,
558, 608, 626, 700 57, 65–66, 163, 175, 188
593–594, 698, 706, 718
Trisomy 13 (Patau syndrome), 65 Ulnar nerve injury, 359 Ventricular tachycardia, 541
Trisomy 18 (Edwards syndrome), Umbilical ligaments, 66–67 Verapamil, 174, 179, 187, 616
56–57, 65, 400, 440, 504 Umbilical vein, 57 Vertebral compression fracture,
Trisomy 21 (Down syndrome), Uncal herniation, 578 336
65, 67, 238, 255, 422, Urachus, patent, 63 Vibrio
439–440, 504, 560, 662 Urea cycle, disorders of, 39 cholerae, 78, 91, 92, 93–95, 98,
Trochlear nerve, 512 Ureaplasma urealyticum, 395 236, 253
Troglitazone, 617 Uremia, 407 parahaemolyticus, 267
Tropheryma whippelii, 116, 217, Ureters, 422, 432, 440 vulnificus, 267
554, 664, 679, 700 Urethritis, 77, 92, 515 Vinblastine, 276–277, 294, 296,
Tropic hormones, 224 gonococcal, 515 583, 722
Tropical sprue, 665 Urinary tract infection, 423, 442, Vincristine, 29, 51, 299, 570,
Troponin, 163, 169, 176, 189 706, 718 583, 588, 722
Trousseau syndrome, 225 ascending, 409 VIPoma, 225, 612
Truncus arteriosus, 66 Urobilinogens, 46–47 Virchow’s triad, 477, 522
Trypanosoma Urogenital folds, 621 Viscera, ruptured, 254
brucei gambiense, 262 Urogenital sinus, 605, 621 Vitamins
brucei rhodesiense, 262 Uroporphyrinogen I synthase, A (retinol)
cruzi, 84, 262, 292, 538, 558 deficiency, 45, 239, 258, 652
595
Tryptophan, 374
Uroporphyrinogen III cosynthase, excessive ingestion of, 729
Tuberculoma, cortical, 85
596 B1 (thiamine), 480, 665
Tuberculosis (TB), 118, 145, 153,
Ursodeoxycholic acid, 249 deficiency, 48, 60, 61, 346,
252, 312, 327, 332, 461,
Uteroplacental insufficiency 352–353, 360
463, 480, 482–483, 492,
(UPI), 384, 400 B3 (niacin), 552, 647
507, 536, 554, 586, 595,
606, 623, 712, 730 deficiency, 26, 45, 61, 366,
miliary, 332 V 374
Tuberous sclerosis, 41, 281, 302, B6 (pyridoxine), 461, 480, 492,
494, 511, 517 Vagotomy, 148 507–508, 535, 553
Tubo-ovarian abscess, 515 Vagus nerve, 355 deficiency, 113, 624
Tularemia, 485 Valproic acid, 612 B12 (cobalamin), 480, 544
Tumor lysis syndrome, 382, 396 Vancomycin, 22, 37, 83, 93, 95, deficiency, 48, 60, 104, 112,
Tumor necrosis factor 100, 110–111, 251–252, 258–259, 275, 278, 290–
α, 110, 134, 216, 255, 260 688, 715, 716, 719, 735, 291, 295, 296, 297, 346–
β, 110 736 347, 361, 574, 594, 648

764
C (ascorbic acid), 226, 480, Waterhouse-Friderichsen syn- Wolff-Parkinson-White (WPW)
652, 687, 691 drome, 221, 229, 543 syndrome, 162, 174, 528,
deficiency, 61, 652, 663, Wegener granulomatosis, 104, 541
687, 691 114, 121, 179, 455, 468, Wolffian (mesonephric) duct, 60,
D 471, 559, 601, 614 420–421, 621, 631, 723
deficiency, 45, 227, 314, Weil-Felix reaction assay, 99 Wool-sorter’s disease, 464, 484
331, 652, 663, 719–720 Wenckebach (Mobitz type I) AV Wuchereria bancrofti, 683
intoxication, 228, 516 block, 170, 190–191
supplementation, impact of Werdnig-Hoffmann disease X
on IIM symptoms, 4, 9 (SMA, type I), 12
Xanthochromia, 340, 348–349
D2 (ergocalciferol), 578 Wermer syndrome (MEN type Xeroderma pigmentosum, 18, 30,
D3 (cholecalciferol), 578 1), 214, 224, 306, 572, 41, 302, 701, 725
E, 227, 480 591–592, 600, 612–613,
deficiency, 45 646
Y
K, 155, 298, 652, 732 Wernicke aphasia, 729
deficiency, 45, 288, 302– Wernicke encephalopathy, 346, Yersinia pestis, 188, 316, 333
303, 652, 672, 687 360, 626, 665 Yohimbine, 227
Volvulus, 62, 234, 247, 657 Wernicke-Korsakoff syndrome, Yolk sac tumors, 444
Von Gierke disease, 542 26, 27, 48, 60, 342, 352–
Von Hippel-Lindau disease, 534, 353, 644, 665 Z
551, 589 Western blot, 32, 570, 587–588
Whipple disease, 116, 217, 554, Zanamivir, 632
Von Hippel-Lindau syndrome,
Zenker diverticulum, 656
302, 544, 549, 590 643, 664, 679, 700
Zidovudine (AZT), 148
Von Willebrand disease (vWD), Whipple triad, 592
Ziehl-Neelsen stain, 82, 100,
274, 288, 304, 590 Whooping cough (pertussis), 75,
536, 554
Von Willebrand factor, 178 78, 86, 88, 93
Zileuton, 484
Wilms tumor, 25, 44, 211–212, Zinc, 227
277, 294–295, 404, 533, Ziprasidone, 372
W
550 Zollinger-Ellison syndrome, 261–
Waldenström macroglobulin- Wilson disease, 35, 138, 245, 259, 262, 272, 284–285, 598,
emia, 280, 300–301, 537, 267, 349, 356, 359, 468, 612
556, 732 601, 615, 653 Zolpidem, 9
Wallenberg syndrome, 344, 356 Wiskott-Aldrich syndrome, 106– Zona adherens, 324
Warfarin, 145, 153, 321, 506 107, 117–118, 119, 506, Zona glomerulosa, 392
Warthin tumor, 266 589, 653 Zona occludens, 324

765
This page intentionally left blank
about the Senior editors

Tao Le, MD, MHS James A. Feinstein, MD

Tao Le, MD, MHS Tao has been a well-recognized figure in medical education for the past 20 years. As senior editor, he
led the expansion of First Aid into a global educational series. In addition, he is the founder and edi-
tor-in-chief of the USMLERx online test bank series as well as a co-founder of the Underground Clini-
cal Vignettes series. As a medical student, he was editor-in-chief of the University of California, San
Francisco Synapse, a university newspaper with a weekly circulation of 9000. Tao earned his medical
degree from the University of California, San Francisco in 1996 and completed his residency training
in internal medicine at Yale University and fellowship training at Johns Hopkins University in allergy
and immunology. In addition, he earned a Master of Health Science degree at the Johns Hopkins
Bloomberg School of Public Health. At Yale, he was a regular guest lecturer on the USMLE review
courses and an adviser to the Yale School of Medicine curriculum committee. Tao subsequently went
on to co-found Medsn and served as its chief medical officer. He is currently pursuing research in
asthma education at the University of Louisville.
James A. Feinstein, MD James is a practicing pediatrician and pediatric health services researcher. He attends in the Special
Care Clinic at The Children’s Hospital Colorado, where he provides care to children with complex
medical issues and special healthcare needs. He is also a research fellow at the University of
Colorado School of Medicine, where he is obtaining a graduate degree in biostatistics. His research is
focused on improving the outpatient care delivered to children with complex, chronic illnesses. He
attended Dartmouth College, the University of Pennsylvania School of Medicine, and completed
residency at the Children’s Hospital of Philadelphia. Outside of medicine, James writes (www.
shortwhitecoat.com) and he absolutely loves to run up (and then down) various Rocky mountains.

about the editors


Mark W. Ball, MD Mark hails from Harlan, Kentucky. He is an alumnus of Centre College, where he majored in biochem-
istry and molecular biology, the Johns Hopkins School of Medicine, and the Halsted General Surgery
Internship at Johns Hopkins Hospital. He is currently a urological surgery resident at the Brady Urolog-
ical Institute at Johns Hopkins Hospital. Mark’s academic interests include the detection and treat-
ment of malignancies of the bladder, prostate, and kidney. In his free time, he enjoys spending time
with his wife, cooking, running, and exploring the outdoors.
Annie Dude, MD Annie hails from Milwaukee, Wisconsin, but has been proud to call Chicago home for almost ten
years now. Annie graduated from the University of Chicago Pritzker School of Medicine in June and
then began residency training in obstetrics and gynecology at Duke University Medical Center. She
enjoys spending time with her husband, six-month-old daughter, dog, family, and friends, as well as
hiking, running, and traveling.
Rebecca L. Hoffman, MD Rebecca is currently a resident in the general surgery program at the Hospital of the University of
Pennsylvania. She grew up in a small town in central Pennsylvania and then attended Haverford Col-
lege, majoring in chemistry. Before starting medical school at the University of Pennsylvania, she
spent a year doing clinical research in the Division of Orthopaedic Surgery at The Children’s Hospital
of Philadelphia. When she isn’t taking care of patients or operating, Rebecca loves to write, edit,
teach, travel, and spend time in the outdoors.
Mark Robert Jensen Mark is a fourth-year student at the University of Rochester School of Medicine. He plays ice hockey
in his spare time and does his best to stay out of the penalty box. Off the ice, he tutors inner city stu-
dents in math and science and likes to travel when he can get away. He has always loved writing and
has published articles on bone sclerosis, dry eye disorder, the utility of CT in evaluation of the left
ventricular assist device, and the development of radical Islam in London. He plans to follow his fa-
ther and grandfather’s footsteps into anesthesia, and to hopefully, someday, publish a book of his
own.
Kimberly Kallianos Originally from Atlanta, Georgia, Kim attended the University of North Carolina at Chapel Hill where
she earned a Bachelor of Science degree in biology. She is currently a fifth-year student at Harvard
Medical School and will begin her radiology residency at the University of California, San Francisco in
2012.
Cesar Raudel Padilla Cesar was raised in the San Francisco Bay area and is a first-generation Mexican American. He
dropped out of high school but managed to get a scholarship to a community college and ultimately
graduated from the University of San Francisco with a degree in biological sciences. He plans to be-
come an anesthesiologist and specialize in critical care. Cesar hopes to serve the Latino community in
California as well as in Guadalajara, Mexico, where he has spent his summers since childhood. He
has helped the University of Rochester School of Medicine reach out to the local Latino community
by coordinating mock interviews with Spanish-speaking standardized patients. Cesar is an avid fútbol
fan and enjoys spending time with his wife and daughter Elena.
Lauren Rothkopf, MD Lauren attended Johns Hopkins University, earning a Bachelor of Arts degree in English with a minor
in psychology. She graduated from Temple University School of Medicine in 2007 and completed her
internship year in internal medicine at Beth Israel Deaconess Medical Center in Boston. She is cur-
rently pursuing a Master’s degree in public health while raising her 1-year-old son.
James Yeh, MD James is a clinical fellow in medicine at Harvard Medical School and a resident physician at the Cam-
bridge Hospital/Cambridge Health Alliance. He is a graduate of Boston University School of Medicine,
where he received the Henry J. Bakst Award in community medicine and was an Albert Schweitzer
Fellow. He completed his undergraduate and graduate degrees at the University of California, Berke-
ley and Harvard University. He has extensive basic science and clinical research background and has
received multiple grants and awards. James has wide-ranging experiences in tutoring and teaching,
and he has been working with First Aid and USMLERx since 2009. In his spare time, he enjoys travel-
ing around the world, exploring new places and museums, cooking/eating, playing guitar, riding his
bike, and photography.

about the Authors


Kirsten Austad
Kirsten graduated from the University of Wisconsin-Madison with degrees in English literature and medical microbiology & immunology. She is currently taking a year off after com-
pleting two years at Harvard Medical School to be a fellow at the Edmond J. Safra Center for Ethics at Harvard University. Her work looks at how to improve physician-pharmaceutical
industry interactions, and she will soon start her third year in Cambridge Hospital’s Integrated Clerkship. Kirsten plans to remain involved in medical education as well as practice
community-based primary care with underserved populations in the U.S. or abroad and work to link health care to poverty reduction and community empowerment.
Eike Blohm
Eike grew up in Germany and came to the U.S. to study medicine. He worked as a paramedic and ICU tech before starting his studies at Johns Hopkins. Now at the end of his third
year in medical school, Eike is starting the application process for a residency seat in emergency medicine. In his free time, he is an avid marathoner, competing in five marathons
this year alone.
Benjamin Caplan, MD
Ben is a family medicine resident at Boston Medical Center. After earning his undergraduate degree in psychology at Williams College, Ben worked as a staff researcher at the
UCLA Brain Mapping Center, where he authored several neuroscience publications and helped launch the international Human Brain Project. Apart from the medical world, he is a
concert cellist of 25 years, a world traveler, and a performance magician of 18 years.
Po-Hao Chen
Po-Hao is completing his medical training at Harvard Medical School and is a joint-degree student at Harvard Business School to learn more about health policy and hospital ad-
ministration. After graduation, he would like to enter a radiology residency program while continuing to participate in medical education and research. Ultimately, Po-Hao wishes
to hold a leadership position at a hospital as well as be involved in medical innovation to develop ways physicians can organize increasing amounts of patient information more
effectively.
Lauren de Leon, MD
Lauren is a recent graduate of the Alpert Medical School of Brown University. She is finishing her internship year in internal medicine and loving it. She enjoys traveling, cooking,
and searching for the perfect dumpling. In terms of her medical career, Lauren is debating between gastroenterology and palliative/end of life care.
Philip Eye
Philip grew up in Baldwin, New York. Four years after being accepted into Boston University’s seven-year medical program, he graduated with a Bachelor of Science degree in eco-
nomics and medical sciences. Phil is currently a Second Lieutenant in the United States Army and a medical student at Boston University School of Medicine. He plans to pursue a
residency in neurology upon graduation in 2012. In his spare time, Phil both studies and teaches kung fu.
Jim Griffin, MD
Jim is originally from Waynesboro, Georgia, and attended the University of Georgia, where he majored in biochemistry and molecular biology. After graduating from Johns Hopkins
School of Medicine in May 2011, he began a general surgery internship at the Johns Hopkins Hospital and plans to pursue a career in surgical oncology.
John Hegde
John was born in Philadelphia and grew up in Carmel, Indiana. He graduated from Indiana University and is currently a third-year student at Harvard Medical School. He is still un-
decided about which medical specialty to pursue, although he is considering a career in either radiation oncology or radiology. John enjoys reading, watching movies, traveling,
and cheering on all Philly sports teams.
Emily Heikamp
Emily is currently working in the MD/PhD program at Johns Hopkins. She is originally from New Orleans, Louisiana, and she attended Duke University as an undergraduate. Before
coming to Hopkins, Emily earned a Master’s degree in medical oncology from Oxford University. Her clinical and research interests include tumor immunology and autoimmunity.
Thomas Robert Hickey, MD
Originally from central New York, Tom recently graduated from Harvard Medical School and is currently pursuing an anesthesia residency at Brigham and Women’s Hospital.
Henry R. Kramer, MD
Henry attended Cornell University, where he studied biology and psychology. He earned his medical degree from the Johns Hopkins University School of Medicine. He is now a
resident in internal medicine at the Massachusetts General Hospital. Henry enjoys running with his dog, cooking, wine, craft beer, and bourbon.
Thomas Lardaro
Thomas is originally from Peace Dale, Rhode Island. After graduating summa cum laude from the University of Connecticut, he worked for two years researching molecular neuro-
biology at Harvard Medical School. Thomas has since enrolled at the Johns Hopkins University School of Medicine, where he has taken an additional year of study as part of a
funded clinical research training program. He will graduate with MD/MPH degrees in the spring of 2012. In his free time, Thomas enjoys playing soccer, traveling, and being out-
doors.
Katherine Latimer
Katherine is a third-year medical student at Johns Hopkins University School of Medicine. She is originally from Bethesda, Maryland, and earned her Bachelor of Science degree
from Georgetown University. She plans to pursue a career in obstetrics and gynecology.
Joseph Liao
Joey is from sunny Ventura, California. He earned a degree in biology from MIT and has been living in New England ever since. He is currently a third-year student at Boston Uni-
versity School of Medicine, and plans to go into radiology. As a second-year, Joey formed a rock band with two of his college buddies, proving that there is indeed time during
medical school to have fun. He also enjoys photography.
Jerry Loo
Jerry did his undergraduate studies in microbiology, immunology & molecular genetics at UCLA. He is currently finishing his fourth year at the USC Keck School of Medicine and
will be continuing his training there as a radiology resident in 2012. In his free time, Jerry enjoys playing piano, computer programming, basketball, and the occasional death
match on PlayStation 3.
Aya Michaels, MD
After graduating from Columbia University in 2007 with a Bachelor of Arts degree in biochemistry, Aya entered Harvard Medical School. She graduated in May 2011 and is excited
to start her training as a radiologist at Brigham and Women’s Hospital in Boston.
Somala Muhammed, MD
Somala recently graduated from Harvard Medical School and is excited to return to her hometown of Houston for a general surgery residency at Baylor College of Medicine. She
hopes to specialize in surgical oncology or pediatric surgery.
Behrouz Namdari, MD
Behrouz earned his medical degree from Chicago Medical School and is currently a resident in psychiatry at Duke University. In his nonmedical time, he is often found on the golf
course or basketball court.
Tashera Perry, MD
Tashera hails from Westfield, New Jersey. She studied biology at Mary Baldwin College and completed a two-year postbaccalaureate Intramural Research Training fellowship at the
National Heart, Lung, and Blood Institute prior to attending the University of Chicago Pritzker School of Medicine. She began residency training in obstetrics and gynecology at the
University of Illinois at Chicago College of Medicine in June 2011. Aside from medicine, Tashera’s interests include art history, mountain biking, and bikram yoga.
Christopher Roxbury
Christopher is currently a third-year medical student at Johns Hopkins University. He hails from Bridgewater, New Jersey, and attended Johns Hopkins University as an undergradu-
ate, majoring in molecular and cellular biology and Spanish. After completing medical school, he plans to pursue a career in otolaryngology—head and neck surgery.
Neepa Shah
Neepa is from Toronto, Canada and is a third-year medical student at Boston University. She plans to pursue a career in ophthalmology, with a focus on academics and teaching.
Her hobbies include traveling, baking, and Indian classical dance.
Bethany Strong
Bethany is originally from Oklahoma City, Oklahoma. She completed her undergraduate education at Spelman College, where she studied biology and biochemistry. As a partici-
pant in the five-year MD/PhD program at Harvard Medical School, she spent an additional year of training completing clinical rotations at McCord Hospital in Durban, South Africa
and conducting research at the Centers for Disease Control and Prevention in Atlanta. She plans to complete a residency program in general surgery, focusing her career on global
health and medical student education.
Seenu Susarla, MD, DMD, MPH
Seenu is from Vestal, New York, and graduated with a degree in chemistry from Princeton University. After graduating college, Seenu decided that being a degree-collector would
be an interesting career. As such, he earned his Master of Public Health, Doctor of Dental Medicine, and Doctor of Medicine degrees from Harvard University. He is currently a resi-
dent in the combined general surgery/oral & maxillofacial surgery residency program at Massachusetts General Hospital.
Jeffrey Tosoian
Jeff is originally from Farmington, Michigan, and attended the University of Michigan in Ann Arbor, where he majored in cellular and molecular biology. Jeff graduated from the
Johns Hopkins Bloomberg School of Public Health in May 2011 with a Master of Public Health degree in biostatistics and epidemiology. After completing medical school in 2012,
Jeff plans to pursue a career in urological surgery with a focus in genitourinary cancers.
Jackson Vane, MD
Jackson is from Whittier, California. He attended California State University, Los Angeles to earn his undergraduate degree in biology in 2005. He then ventured out of the sunny
confines of Southern California to Chicago, where he attended Rosalind Franklin University of Medicine and Science and earned his Master of Science degree in applied physiology
in 2006. He then attended Chicago Medical School and graduated in 2010. He is currently at the University of California, Irvine doing his residency in pediatrics. In his free time,
Jackson enjoys wandering aimlessly in his new surroundings, riding his bike, playing poker, trying all sorts of new restaurants, and catching up on the many movies he has missed
over the years.
Daniel J. Verdini, MD
After graduating Boston College with degrees in biology and economics, Dan worked as an investment analyst before deciding to embark on a career in medicine. After graduating
from Ross University, Dan spent a year at Massachusetts General Hospital as a research fellow in cardiac imaging and will be starting a radiology residency at the University of
Texas in San Antonio after completing a year in preliminary medicine at the University of Nevada School of Medicine in Reno.
Marc E. Walker
Marc is an MD/MBA student at Harvard Medical School and Harvard Business School. On the business side, Marc is interested in health care delivery systems development and in-
novation and entrepreneurship in medicine and surgery. He will be dedicating the upcoming year to the pursuit of his research interests in plastic surgery and plans to graduate in
May 2012 with the intent of entering a surgical residency thereafter.

You might also like